40% found this document useful (5 votes)
16K views

Pradeep Physics Class 12 Vol 2 2023-2024

Uploaded by

ABHIK THE GHOUL
Copyright
© © All Rights Reserved
We take content rights seriously. If you suspect this is your content, claim it here.
Available Formats
Download as PDF, TXT or read online on Scribd
40% found this document useful (5 votes)
16K views

Pradeep Physics Class 12 Vol 2 2023-2024

Uploaded by

ABHIK THE GHOUL
Copyright
© © All Rights Reserved
We take content rights seriously. If you suspect this is your content, claim it here.
Available Formats
Download as PDF, TXT or read online on Scribd
You are on page 1/ 913

P^unA-ceji^

FUNDAMENTAI

w
PHYSICS
Flo
Vol.n

eee
strictly according to new syllabus & examination pattern of CBSE, ISCE and state boards of Punjab, Haryana,

Fr
H.P., J & K, Odisha, Karnataka, Kerala, Nagaland, Manipur, Rajasthan, Jharkhand, Bihar, U.P.,
Uttarakhand, M.P., Chhatisgarh, Assam, Gujarat, West Bengal, A.P. etc.

for
ur
for class XII of+2 stage.
ks
Yo
K.L. GOGIA
oo

Dr. K.L GOMBER M.Sc.


M.SC: M. Phil. {Gold Medalist), Ph.D.
eB

Formerly Head of Physics Department Formerly Head of Physics Department


D.A.V. College, AMBALA CITY
S.A. Jain College, AMBALA CITY
Ex. Member, Board of Studies in
Formerly Post Doctoral Research Associate
Physics & Science Faculty
ur

State University of New York, Buffalo


NEW YORK, U.S.A.
K.U., KURUKSHETRA
ad
Yo
d
Re
in
F

2023-2024

Pradeep Publications
JALANDHAR (INDIA)
Contents
Unit - 5 [Electromagnetic Waves]

ww
Pages
Chapters
8/1 — 8/80
8. Electromagnetic Waves
8/1

Flo
8.1. Introduction
8/1

e
8.2. Displacement Current

ree
8/3
8.3. Continuity of Current

Fr
8/4
8.4. Consequences of Displacement Current or Prediction of Electromagnetic Wave

rr F
8/5
8.5.
uurr
Maxwell’s Equations and Lorentz Force 8/6
8.6. History of Electromagnetic Waves
8.7. Transverse Nature of Electromagnetic Waves sfo 8/7
8/8
kks
8.8. Velocity of Electromagnetic Waves 8/10
Yo
oooo

8.9. Production of Electromagnetic Waves


8/11
8.10. Important Characteristics and Properties of the Electromagnetic Waves
eB

8/13
8.11. Hertz Experiment 8/14
8.12. Electromagnetic Spectrum 8/15
ur

8.13. Main Parts of Electromagnetic Spectrum and Their Uses 8/18


ad

8.14. Microwave Oven


YYo

8/18
Solved Examples 8/24
Revision At A Glance
dd

8/25
Re
inn

Multiple Choice Questions 8/27


Assertion-Reason Type Questions
F

8/31
Conceptual Problems 8/34
Very Short Answer Questions [1 Mark]
8/36
Short Answer Questions [2 Marks]
8/41
Short Answer Questions [3 Marks]
8/41
Long Answer Questions [5 or more Marks]
8/41
Case-Based Very Short/Short Questions 8/43
Case-Based MCQs and Assertion-Reason Questions 8/45
Problems For Practice
8/49
NCERT Exercises with Solutions
8/52
NCERTExemplar Problems with Answers, Hints & Solutions 8/64
Competition Focus [NEET/JEE Special]
Unit-6 [Optics]
9. Ray Optics and Optical Instruments 9/1—9/206
9.1. Introduction
9/1
9.2. Reflection ofLight 9/1
9.3. Spherical Mirrors
9/3
9.4. The New Cartesian Sign Conventions 9/3
9.5. Principal Focus, Focal Length and Radius of Curvature of a Spherical Miirror 9/4
9.6. Relation between/and/? 9/5
9.7. Mirror Formula for Concave Mirror 9/6
9.8. Mirror Formula for Convex Mirror 9/7

w
9.9. Linear Magnification ofa Spherical Mirror 9/9
9.10.
Relation between the Speeds of Object and Image formed by a Spherical Mirror 9/10
9.11. Practical Applications of Spherical Mirrors

Flo
9/11
9.12. Basics of Refraction

e
9/11

ree
9.13.
Refraction ofLight Through a Rectangular Glass Slab 9/14

FFr
9.14. Real Depth andApparentDepthofaTank 9/15
9.15. Refraction Effects at Sun Rise and Sun Set 9/17

for
ur
9.16. Refraction of Light Through a Compound Plate 9/17
9.17. Total Internal Reflection ofLight 9/18
kkss
9.18. Experimental Demonstration ofTotal Internal Reflection 9/20
Yo
9.19. Some Applications ofTotal Internal Reflection
oo

9/20
9.20. Spherical Refracting Surfaces
eB

9/22
9.21. Refraction from Rarer to Denser Medium at a Convex Spherical Refracting Surface 9/23
9.22. Refraction from Rarer to Denser Medium at a Concave Spherical Refracting Surface 9/25
ur

9.23. Refraction from Denser to Rarer Medium at a Spherical Refracting Surface


ad

9/26
YYo

9.24. Lenses
9/28
9.25. Lens Maker’s Formula for Convex Lens 9/30
9.26. Lens Maker’s Formula for Concave Lens
d
Re

9/31
in

9.27. Derivation ofLens-Maker’s Formula from AB-INITIO 9/33


F

9.28. Principal Focal Lengths of a Lens 9/36


9.29. Lens Formula
9/37
9.30. Linear Magnification Produced by a Lens 9/39
9.31. Power of a Lens
9/40
9.32. Combination ofthin Lenses in Contact 9/41
9.33. Focal Length of Equivalent Lens 9/41
9.34. Combination ofa Lens and a Mirror 9/43
9.35. Displacement Method to find Position of Images (Conjugate Points): 9/44
Newton’s Relation
9.36. Prism
9/45
9.37. Refraction Through a Prism 9/45
9.38. Thin Prism 9/47
9/47
9.39. Dispersion of Light
9/48
9.40. Angular Dispersion
9/49
9.41. Dispersive Power
9/50
9.42. Scattering of Light
9.43. Raman Effect 9/51

9.44. Rainbow 9/51


9/52
9.45. Optical Instruments
9/52
*9.46. The Eye
*9.47. Defects of Vision 9/53
9/56
9.48. Simple Microscope or Magnifying Glass

ww
9/57
9.49. Compound Microscope
9/60
9.50. Astronomical Telescope
9/64
9.51. Reflecting Type Telescope (Cassegrainian Telescope)

Floo
9/65
Solved Examples
9/89

ree
Revision At A Glance
9/94
Multiple Choice Questions

rFee
9/96
Assertion-Reason Type Questions
9/101

F
Conceptual Problems

oor r
rur
[1 Mark] 9/107
Very Short Answer Questions s ff
[2 Marks] 9/111
Short Answer Questions
[i Marks] 9/115
Short Answer Questions
k
YYoou
9/117
[5 or more Marks]
ookos

Long Answer Questions


9/119
Case-Based Very Short/Short Questions
BBo

9/122
Case-Based MCQs andAssertion-Reason Questions
re

Problems For Practice 9/124

NCERTExercises with Solutions 9/147


ouur
ad

9/160
NCERT Exemplar Problems with Answers, Hints & Solutions
Yo

9/173
Competition Focus [NEET/JEE Special]
dY
Re

10/1—10/108
idn

10. Wave Optics


FFin

10.1. Introduction 10/1

10.2. WaveFront 10/2


10/3
10.3. Huygens Principle
10.4. Reflection on the Basis ofWave Theory 10/3
10/4
10.5. Refraction on the Basis ofWave Theory
10/5
10.6. Wavelength and Frequency
10/6
10.7. Behaviour of a Prism, a Lens, and a Spherical Mirror Towards Plane Wavefront
10/7
10.8. Superposition Principle
10/8
10.9. Coherent and Incoherent Addition of Light Waves
10.10. Coherent Sources 10/9
10/10
10.11. Interference of Light
* Not included in CBSE syllabus. These articles have been given only for the preparation ofcompetitive examinations.
10.12. Young’s Double Slit Experiment 10/10
10.13. Conditions for Constructive and Destructive Interference 10/11
10.14. Interference and Energy Conservation 10/13
10.15. Expression for Fringe Width in Interference or Theory of Interference of Light 10/14
10.16. Conditions for Sustained Interference of Light 10/17
10.17. Fringe Shift 10/17
10.18. Diffraction of Light 10/18
10.19. Diffraction of Light at a Single Slit 10/19
10.20. Difference between Interference and Diffraction of Light 10/23
10.21. Fresnel Distance 10/23
10.22. Concept of Resolving Power 10/24
10.23. Resolving Power of Microscope

w
10/25
10.24. Resolving Power of Telescope 10/25
*10.25. Doppler’s Effect in Light 10/26

Flo
* 10.26. Doppler’s Effect in Light is Symmetrical, but the same Effect in 10/28
Sound is Asymmetrical

ee
* 10.27. Applications of Doppler’s Effect in Light 10/28

Fr
10.28. Polarization of Light 10/28
10.29. Experimental Demonstration of Polarization of Light and 10/30

for
ur
Transverse Character of Light Waves
10.30. Nicol Prism 10/30
ks
10.31. Detection of Polarized Light 10/31
Yo
10.32. LawofMalus 10/31
oo

10.33. Polarization by Scattering 10/33


eB

10.34. Polarization ofLight by Reflection 10/33


10.35. Brewster’s Law 10/34
r

10.36. Polaroids 10/35


ou
ad

Solved Examples 10/36


YY

Revision At A Glance 10/45


Multiple Choice Questions 10/48
nd
Re

Assertion-Reason Type Questions 10/50


Conceptual Problems
Fi

10/53
Very Short Answer Questions [7 Mark] 10/57
Short Answer Questions [2 Marks] 10/60
Short Answer Questions [3 Marks] 10/63
Long Answer Questions [5 or more Marks] 10/64
Case-Based Very Short/Short Questions 10/65
Case-Based MCQs andAssertion-Reason Questions 10/67
Problems For Practice 10/69
NCERTExercises with Solutions 10/80
NCERTExemplar Problems with Answers, Hints & Solutions 10/82
Competition Focus [NEET/JEE Special] 10/93

* Not included in CBSE syllabus. These articles have been given only for the preparation of competitive examinations.
Unit - 7 [Dual Nature of Radiation and Matter]
11. Dual Nature of Radiation and Matter 11/1—11/106
11.1. Introduction 11/1

11.2. Free Electrons in Metals 11/2

11.3. Electron Emission 11/2

11.4. Photoelectric Effect 11/3

11/3
11.5. Hertz, Hallwachs and Lenard’s Observations on Photo Electric Effect
11/4
11.6. Experimental Study of Photoelectric Effect
11/7
11.7. Einstein’s Photoelectric Equation; Energy Quantum of Radiations
11.8. Relation between Cut OffPotential, Frequency of the Incident Photon 11/8

w
and Threshold Frequency/Wavelength
11.9. Determination of Planck’s Constant and Work Function of a Metal 11/8

Flo
11/9
11.10. Important Graphs Related to Photoelectric Effect

e
reee
11/11
11.11. Particle Nature of Light (The photon)

FFr
11/12
11.12. Failure ofWave Theory ofLight to Explain the Photoelectric Effect
11.13. Photoelectric Cell 11/13

11/14

for
ur
11.14. Dual Nature of Radiation
11/14
11.15. De-Broglie Dualistic Hypothesis
kkss
11/16
11.16. De-Broglie Wavelength of an Electron
Yo
11/17
11.17. Experimental Demonstration of Wave Nature of Electron
oo

11/18
11.18. Probability Interpretation of Matter Waves
eB

11/19
Solved Examples
Revision At A Glance 11/25
ur

11/27
Multiple Choice Questions
ad
YYo

11/30
Assertion-Reason Type Questions
11/34
Conceptual Problems
d

[I Mark] 11/39
Veiy Short Answer Questions
Re
in

[2 Marks] 11/44
Short Answer Questions
F

[3 Marks] 11/52
Short Answer Questions
11/55
Long Answer Questions [5 or more Marks]
11/55
Case-Based Very Short/Short Questions
11/58
Case-Based MCQs and Assertion-Reason Questions
Problems For Practice 11/60

NCERT Exercises with Solutions 11/68

11/70
NCERT Exemplar Problems with Answers, Hints & Solutions
11/80
Competition Focus [NEET/JEE Special]
Unit - 8 [Atoms and Nuclei]
12. Atoms 12/1—12/72
12.1. Introduction 12/1
12.2. Thomson’sModelofAtom 12/1

12.3. Rutherford’s a-Ray Scattering Experiment: Discovery ofAtomic Nucleus 12/1

12.4. Distance of Closest Approach (Size ofNucleus) 12/3


12.5. Impact Parameter 12/4
12.6. Rutherford’s Atom Model 12/6
12.7. Electron Orbits 12/6
12.8. Atomic Spectra 12/6
12.9. Spectral Series ofHydrogen 12/7

w
12.10. Limitations of Rutherford Atom Model 12/8
12.11. Bohr Model ofHydrogenAtom 12/9
12.12. Bohr’s Explanation of Spectral Series ofHydrogenAtom

Flo
12/14
12.13. Energy Level Diagram 12/15

ee
12.14. De Broglie’s Explanation of Bohr’s Second Postulate of Quantization 12/17
12.15. LimitationsofBohr’sTheory

Fr
12/18
12.16. Hydrogen Like Atoms 12/18
12.17. Excitation and Ionisation Potentials 12/19

for
ur
12.18. X-ray Spectra: Continuous and Characteristic 12/20
Solved Examples 12/21
ks
Revision At A Glance 12/26
Yo
Multiple Choice Questions
oo

12/28
Assertion-Reason Type Questions 12/30
eB

Conceptual Problems 12/34


Very Short Answer Questions [/ Mark] 12/35
ur

Short Answer Questions [2 Marks] \im


ad

Short Answer Questions [3 Marks] 12/39


Yo

Long Answer Questions [5 or more Marks] 12/39


Case-Based Very Short/Short Questions 12/39
nd
Re

Case-Based MCQs andAssertion-Reason Questions 12/41


Problems For Practice 12/43
Fi

NCERT Exercises with Solutions 12/47


NCERT Exemplar Problems with Answers, Hints & Solutions 12/49
Competition Focus [NEET/JEE Special] 12/60

13. Nuclei 13/1—13/96


13.1. Introduction 13/1
13.2. AtomicMasses 13/1
13.3. Composition of Nucleus 13/2
13.4. Nuclear Size 13/4
13.5. Nuclear Density 13/4
13.6. Isotopes 13/5
13.7. Isobars 13/5
13/5
13.8. Isotones
13/6
13.9. Mass-Energy Relation
13/6
13.10. Nuclear Binding Energy
13/8
13.11. Packing Fraction
13/9
13.12. Nuclear Forces
13/10
13.13. Nature of Nuclear Forces
13/10
13.14. Nuclear Stability
13/11
13.15. Radioactivity
13/12
13.16. Three Types of Radiations
13/14
13.17. Laws of Radioactive Disintegration
13/15
13.18. Disintegration Constant
13/15
13.19. Half Life of Radioactive Element

w
13/17
13.20. Average Life or Mean Life of Radioactive Element
13/19
13.21. Activity ofa Radioactive Substance
13/20

Flo
13.22. Alpha, Beta and Gamma Decay
13/20

e
13.23. Alpha Decay

ree
13/22
13.24. Beta Decay

FFr
13/23
13.25. GammaDecay
13/24
13.26. Nuclear Reaction
urr
13/25

for
13.27. Nuclear Energy: Energy from the Nucleus
13/25
13.28. Nuclear Fission
13/26
kkss
13.29. Nuclear Chain Reaction
13/27
Yo
13.30. Thermal Neutrons
ooo

13/28
13.31. Nuclear Reactor
13/29
eB

13.32. Nuclear Fusion


13/30
13.33. Distinction between Nuclear Fission and Nuclear Fusion
13/31
13.34. Controlled Thermonuclear Fusion
ur

13/31
ad

13.35. Nuclear Holocaust


YYo

13/31
13.36. India’s Atomic Energy Programme
13/32
Solved Examples
13/38
d

Revision At A Glance
Re

13/40
in

Multiple Choice Questions


13/43
F

Assertion-Reason 'type Questions


13/47
Conceptual Problems
13/50
Very Short Answer Questions [} Mark]
13/53
ShortAnswer Questions [2 Marks]
13/56
ShortAnswer Questions [3 Marks]
13/56
Long Answer Questions [5 or more Marks]
13/57
Case-Based Very Short/Short Questions
13/59
Case-Based MCQs andAssertion-Reason Questions
13/60
Problems For Practice
13/68
NCERT Exercises with Solutions
13/71
NCERTExemplar Problems with Answers, Hints &. Solutions
13/80
Competition Focus [NEET/JEE Special]
Unit - 9 [Electronic Devices]

14. Semiconductor Electronics Materials, Devices and Simple Circuits 14/1—14/160


14.1. Introduction
14/1
14.2. Classification of Solids on the Basis of their Conductivity 14/1
14.3. Energy Bands of Solids (Band Theory of Solids) 14/2
14.4. Difference between Valence Band, Conduction Band and Energy Band Gap 14/4
14.5. Distinction between Conductors (or Metals), Semi-conductor s and Insulators 14/4
on the Basis of their Energy Bands
14.6. Intrinsic Semi-conductors 14/6

ww
14.7. Formation of Holes in Semi-Conductors 14/7
14.8. Electrical Conduction in Semi-Conductors 14/8
14.9. Doping 14/9

Flo
14.10. Extrinsic Semi-conductors 14/9

e
14.11. Distinction between Intrinsic and Extrinsic Semi-conductors 14/12

ree
14.12. Distinction between «-Type Semi-conductors andp-Type Semi-conductors 14/12

Fr
14.13. Electrical Conductivity of Extrinsic Semi-conductors

rF
14/13
14.14. Effect ofTemperature on the Mobility and Conductivity of Electrons and Holes
uurr 14/15

for
14.15. P-NJunction
14/15
14.16. Biasing oftheP-NJunction 14/17
s
kks
14.17. Characteristics of a P-N Junction Diode 14/19
Yo
oooo

14.18. Dynamic Resistance or A.C. Resistance of the Junction Diode 14/21


14.19. P-N Junction Diode as a Rectifier 14/21
eB

14.20. Special Purpose P-N Junction Diodes 14/25


*14.21. Junction Transistor 14/30
ur

* 14.22. Transistor Action or Working ofJunction Transistor 14/31


ad
YYo

*14.23. Modes of Study ofJunction Transistors 14/33


* 14.24. Common Base Transistor Characteristics 14/33
*14.25. Common Emitter TransistorCharacteristics
dd

14/34
Re

*14.26. Transistor as a Switch


in

14/37
*14.27. ConceptofanAmplifier
F

14/39
* 14.28. Transistor as Common Base Amplifier 14/39
* 14.29. Transistor as Common Emitter Amplifier 14/42
*14.30. Feed BackAmplifierand Transistor Oscillator 14/46
*14.31. Transistor as an Oscillator 14/46
*14.32. Advantages of Semi-conductor Devices 14/47
* 14.33. Disadvantages of Semi-conductor Devices 14/48
*14.34. Decimal Number System 14/48
*14.35. Binary Number System 14/48
* 14.36. Conversion of Decimal Number System into Binary Number System 14/48

* Not included in CBSE syllabus. These articles have been given onlyfor the preparation of competitive examinations.
14/49
♦14.37. Conversion of Binary Number System into Decimal Number System
14/50
*14.38. Addition or Subtraction of Binary Numbers
14/51
* 14.39. Analogue Signal and Digital Signals
14/51
*14.40. Logic Gate
*14.41. BasicGates 14/51
14/52
* 14.42. Some Useful Laws of Boolean Algebra
*14.43. OR Gate 14/53

*14.44. AND Gate 14/54

*14.45. NOTGate 14/55

*14.46. Combination of Gates 14/57


14/58
*14.47. Integrated Circuit
14/60
* 14.48. Advantages of Integrated Circuits over Conventional Electronic Circuits

ww
14/60
* 14.49. Limitations of Integrated Circuits over Conventional Circuits
14/60
*14.50. Uses of Integrated Circuits
14/61
Solved Examples

FF loo
Revision At A Glance 14/70
14/75

ree
Multiple Choice Questions
14/78
Assertion-Reason Type Questions
14/81

rFee
Conceptual Problems
14/88
Very Short Answer Questions [/ Mark]

oor rF
rur
14/93
Short Answer Questions [2 Marks] s ff 14/100
Short Answer Questions [3 Marks]
14/103
Long Answer Questions [5 or more Marks]
k
14/104
YYoou
Case-Based Very Short/Short Questions
ookos

14/107
Case-Based MCQs andAssertion-Reason Questions
BBo

Problems For Practice 14/108


re

14/118
NCERT Exercises with Solutions
14/119
NCERTExemplar Problems with Answers, Hints <& Solutions
ouur
ad

14/132
Competition Focus [NEET/JEE Special]
Yo

Unit -10 [Communication Systems]


Yd
Re

*15. Communication 15/1—15/60


idn

15/1
15.1. Introduction
FFin

15/1
15.2. Elements of Communication System
15/2
15.3. Basic Terminology Used in Electronic Communication Systems
15/3
15.4. Message Signals and Their Band Widths
15/5
15.5. Transmission Medium or Communication Channel
15/5
15.6. Band Width ofTransmissionMedium
15/6
15.7. Antenna
15/7
15.8. Modulation and its Necessity
15/8
15.9. Types of Modulation
15/9
15.10. Amplitude Modulation
* Not included in CBSE syllabus. These articles have been given only for the preparation ofcompetitive examinations.
● Not included in CBSE syllabus. This chapter has been given only for the preparation of competitive examinations.
15.11. Frequency Modulation 15/12
15.12. Comparison ofAmlitude Modulation and Frequency Modulation 15/14
15.13. Production ofAmplitude Modulated Wave 15/14
15.14. Demodulation: Detection ofAmplitude Modulated Wave 15/16
15.15. Additional Information 15/17
15.16. Space Communication: (Propagation of Electromagnetic Waves) 15/18
15.17. Radio Waves 15/19
15.18. Earth’s Atmosphere 15/19
15.19. Ground Wave or Surface Wave Propagation 15/21
15.20. Sky Wave Propagation or Ionospheric Propagation 15/22
15.21. Space or Tropospheric Wave Propagation 15/24
15.22. TV Signal Propagation: Relation between Coverage Distance and 15/24
Height ofTransmitting Antenna

w
15.23. Microwave Link or Line of Sight Propagation 15/25
15.24. Satellite Communication 15/26

Flo
15.25. Merits and Demerits of Satellite Communication 15/27

e
15,26. Remote Sensing 15/28

rree
15.27. Major Milestones in the History of Communication 15/29

r FF
15.28. Internet-BasicIdeas 15/29
15.29. Mobile Telephony - Basic Ideas 15/32
uurr
15.30. Global Positioning System (GPS) - Basic Ideas
Solved Examples for 15/35
15/37
kss
Revision At A Glance 15/42
ooook
Yo
Competition Focus [NEET/JEE Special] 15/45
eB

Log Tables (i)-(vui)


urr
ad
Yo
dY
Re
innd
Fi
Re
F
Fi ad
nid
nY
Ydoo
Yo uurY
C3

CHAPTER 8.
reeBB

WAVES
oo
uor
kos
offk

ELECTROMAGNETIC WAVES
oro
uFFll
roow sFr
e
reF
ELECTROMAGNETIC

w re
e
8

ww
ELECTROMAGNETIC WAVES

Flo
e
8.1. INTRODUCTION

ree
Oersted, experimentally predicted that the electric current produces a magnetic field. Faraday troni his
experiments on electromagnetic induction found that a changing magnetic field produces an electric field.

Fr
rF
The question arises ; is the reverse true ? Can a changing electric field produce a magnetic field ? This answer
uurr
was given by Clerk Maxwell, staling that the magnetic field is not only produced by electric current but also

for
by a time varying electric field. Maxwell concluded his idea on the basis of his experiment, when he was
finding the magnetic field at a point outside and inside a capacitor connected to a time-varying current, using
s
Ampere’s circuital law. He found that the results were not consistent. Maxwell pointed out that Ampere's
kks
circuital law becomes consistent only if along with conduction cun eni, there is an additional current called by
Yo
oooo

him, the displacement current.


Maxwell formulated a set of equations involving electric and magnetic fields and their sources ; the
eB

charge and current. These equations are known as Maxwell’s equations. These equations along with Lorentz
force formula explain mathematically all the basic laws of electromagnetism.
Maxwell through his equations came out with a most important prediction, i.e., the existence of
ur

with a speed very close to the speed of light (3 x 10


ad

electromagnetic waves, which can propagate in space


YYo

ms -●). This led Maxwell to conclude that light is an electromagnetic wave. In 1885 Hertz demonstrated
Marconi used these waves in establishing a
experimentally, the existence of electromagnetic waves. Later on
communication between two locations at a distance of about 50 km, without using the wires between the two
dd
Re

locations. This led to the discovery of wireless communication and Marconi was considered as afather of
in

wireless communication Now-a-days the wireless communication has revolutionised our life.
F

In this unit, we shall study the production, properties and uses of electromagnetic waves.

8.2. DISPLACEMENT CURRENT


According to Ampere circuital law,
the line integral of magnetic field B around any closed path is equal to times the total
current threading the closed path, i.e.,

^ B.d { ...(1)

where / is the net current threading the surface bounded by a closed path C. Maxwell in 1864 argued that the
relation (1) is logically inconsistent. He explained the same with the help of following observations.
8/1
8/2
T^'utdec^ ^ Fundamental Physics (XII)
Consider a parallel plate capacitor having plates P and Q connected to a battery through a tapping
key K. When key K is pressed, the conduction current flows through the connecting wires. The capacitor
starts storing charge. As the charge on the capacitor grows, the conduction cuixent in the wires decreases.
When the capacitor is fully charged, the conduction current stops flowing in the wires. During charging of
capacitor, there is no conduction current between the plates of capacitor. During charging, let at an instant, I
be the conduction current in the wires. This current will produce magnetic field around the wires which can
be detected by using a compass needle.
Let us find the magnetic field at point R which is at a perpendicular distance r from connecting wire, in
a region outside the parallel plate capacitor. For this we consider a plane circular loop Cj, of radius r, whose
centre lies on the wire and its plane is perpendicular to the direction of current carrying wire
(Fig. 8.!(«)). The magnitude of the magnetic field is same at all points on the loop and is acting tangentially
along the circumference of the loop. If B is the magnitude of magnetic field at R, then using Ampere’s
circuital law, for loop Cj, we have

ww
^0^
c
^ B.dl = (^ BdlcosQ^ -BlKr = \iQl or B =
2nr
...(2)
1

Flo
Now. we consider a different surface, /.e., a
FIGURE 8.1
tiffin box shaped surface without lid with its circular p Q - ^ Q

e
R
rim, which has the same boundary as that of loop
+

rere
1 A
+

Cj. The box does not touch to the connecting wire I I


+
+

r FF
>
and plate P of capacitor. The flat circular bottom S \J
+
+
+
+

of the tiffin box lies in between the capacitor plates. Cl + CT +


s
uurr
'Fig. 8.1(/?). No conduction current is passing
through the tiffin box surface 5, therefore / = 0.
applying Ampere’s circuital law to loop C, of this
On B
for K B K
kss
tiffin box surface, we have
e o
ooook
Yo
() B.d!
= B2ii:r = pQx0 = 0 or 5 = 0 ...(3)
eBB

From (2) and (3), we note that there is a magnetic field at R calculated through one way and no magnetic field
at /?, calculated through aiiother way. Since this confiadiction arises from the use of Ampere’s circuital law, hence
urr

Ampere’s circuital law is logically inconsistent. Maxwell argued that the above inconsistency of Ampere’s
ad

circuital law must be due to something missing. The missing term must be such that one gets the same magnetic
Yo

field with whatsoever surface is used. This missing term must be related with a changing electric field which passes
dY

tlirough the surface 5 between the plates of capacitor, during chaiging.


Re

If at the given instant of time, q is the charge on the plate of capacitor and A is the plate area of capacitor,
innd

the magnitude of the electric field between the plates of capacitor is


Fi

E = -^

This field is perpendicular to surface S. It has the same magnitude over the area A of the capacitor plates
and becomes zero outside the capacitor.

The electric flux through the surface S is, ^^ = E.A = EA cos 0° = — lxA = -^ ...(4)
dq Gq A ^0
If —
dt is● the rate of change of charge with time on the plate of the capacitor, then
d^ E d ( q _Lf^ or dq
dt dt G Gr.0 dt
dt~^^ dt
dq
Here,
— - current through surface S corresponding to changing electric field = /y^, called Maxwell’s
displacement current. Thus,
8/3
ELECTROMAGNETIC WAVES

displacement current is that current which comes into play in the region in which the electric
field and hence the electric fiux is changing with time.
...(5)
This displacement current is given by di

This displacement current is the missing term in Ampere’s circuital law. Maxwell pointed out that for
consistency of Ampere’s circuital law, there must be displacement current If^ (= along with
conduction current I in the closed loop as (/ + Ip) has the property of continuity, although individually they
may not be continuous.
Maxwell modified Ampere’s circuital law in order to make the same logically consistent. He stated
Ampere’s circuital law to the form.
...(6)
0
dt

w
This is called Ampere Maxwell’s Law.
Important Properties of Displacement Current
1. Displacement current comes into play in a region or space where the electric field or electric flux is

Flo
changing with time. ● .. . u i .●
2. Displacement current does not exist when there is a steady condition in a region or where the electiic

ee
field is not changing with time.

Fr
3. Displacement current does not flow through the conducting wires like conduction current, i.c'.,
displacement current in conducting wire is zero. ■ u #■ u r

4. The magnitude of displacement cuirent in a region or space is equal to Gq times the rate ot cnange ot
for
ur
d(t) E
electric flux in that region or space, i.e., / D "^0 dt
ks
5. The displacement current incombination with conduction current satisfies the property of continuity.
Yo
6. The displacement current produces magnetic field in a region, like conduction current flowing in
oo

wires produces in and around the wires.


eB

8.3. CONTINUITY OF CURRENT


r

Maxwell’s modification of Ampere’s circuital law gives that ^ B.dl = Po


ou
ad

where I ^ , called displacement current, / is the conduction current and is the electric flux
Y

D ~^0 dt
across the loop C.
nd

conduction current and displacement current (i.e., / + Ip) has the important property
Re

of the
ThesuniL...
of continuity along any closed path although individually they may not be contmuoas.
Fi

To prove it, consider a parallel plate capacitor having plates P FIGURE 8.2

and Q, being charged with battery B. During the time, charging is P Q

taking place, let at an instant, I be the conduction current flowing A A


through the wires. Let Cj and C2 be the two loops, which have exactly 1
the same boundary as that of the plates of capacitor. Cj is little towards >■

left and C2 is a little towards irght of the plate P of parallel plate


capacitor, Fig. 8.2. \J \J \J
Due to battery B, let the conduction current I be flowing through Cl C2
the lead wires at any instant, but there is no conduction current across B K
the capacitor gap, as no charge is transported across this gap. I-

d^E = 0
For/oop Cj, there is no electric flux, i.e., (|)£ = 0and dt

d^ E
/ + /^ =/+G = / + Gq (0) = / ...(7)
D 0
dt
8/4
'<x. Fundamental Physics (XII) PMmw
For loop C2, there is no conduction current, i.e., 1 = 0

d(S?E
dt ...(8)

At the given instant if q is the magnitude of charge on the plates of the capacitor of area A, then electric
field E in the gap between the two plates of this capacitor is given by

A A
0 ^0 0

d
Electric flux. = fa = A =^
A
0 ^0

d f

ww
Thus from (8), we have
dt 6
— 1=-^
dt
=/ ...(9)
0

From (7) and (9), we conclude that the sum (/ + Ijy) has the same value on the left and right side of plate

Flo
P of the parallel plate capacitor. Hence (/ + Z^,) has the property of continuity although individually they may
not be continuous.

e
eree
It is to be noted that in any general medium, both conduction current and displacement current will be
present, giving rise to the total current. In a conducting medium, conduction current dominates over

FFr
displacement cun-ent, whereas in an insulating medium, displacement current dominates over the conduction

oorr
uur r
current. It is important to note that the displacement current has the same physical effects as the conduction
current. For example, during the charging of capacitor, if we measure the magnetic field at a point S
sf
between the two plates of capacitor, it is found to be the
sk
Yoo
same as that just outside the capacitor at point R. The electric FIGURE 8.3
oook

X
-) R + s
field E and magnetic field B between the plates of
eBB

'x

cap^itor at the point 5 are shown in Fig. 8.3(a). The direction


^ X X
'
+ -^- > I

of E is from pc^itive plate to the negative plate whereas


I +
X
I
1
uurr

+ —►- V

the direction of B at 5 is perpendicular to the plane of the \x S X


' 'B '
ad

+ X

B**-
paper. The cross-sectional view of the electric and + —►- X /
Yo

magnetic
X
X

fields are shown in Fig. 8.3(/?), where electric field is shown


dY

by crosses and the magnetic field is shown by the tangents O O


Re

to a circle in the same plane.


innd

8.4. CONSEQUFNCES OF DISPLACEMENT CURRENT


FFi

OR PREOIC nON OF ELECTROMAGNETIC WAVE


discovery ot displacement current is of great importance as it has established a symmetry between
the hiws of electricity and magnetism. Faraday’s law of electromagnetic induction states that the magnitude
of the emf induced m a coil is equal to the rate of change of magnetic flux linked with it. Since, the emf
between two points A and B is the measure of maximum workdone in taking a unit charge from point A to B,
therefore, the existence ol an emf shows the existence of an electric field. It is due to this fact, Faradax
concluded that a changing magnetic ifeld with time gives rise to an electric field.
The Maxwell s concept that a changing electric field with time gives rise to displacement cuirent which
also produce.s a magnetic field similar to that of
conduction current. It is infact, a symmetrical counterpart of
the haradays concept, which led Maxwell to conclude that the displacement current is also a source of
magnetic field. It means the time varying electric and magnetic fields give rise to each other. From these
concepts, Maxwell concluded the existence of electromagnetic wave in a region where electric and
magnetic fields were changing with time.

f
8/5
ELECTROMAGNETIC WAVES

Sample Problem How wouid you establish an intantaneous displacement current of 1*0 A in
the space between the parallel plates of 1 pF capacitor.

Sol. = ^0 dt
dt ^ dt\d

A dV CdV dV /D 1-0
7 —X — = or
- -
= 10^’ Vs"^
or D
d dt dt dt C 10"^

Thus an instantaneous displacement current of 1-0 A can be set up by changing the potential difference
across the parallel plates of capacitor at the rate of 10^ Vs"‘.
8.5. MAXWELL'S EQUATIONS AND LORENTZ FORCE

w
In the absence of any dielectric or magnetic material, the four Maxwell’s equations are given below:
(/) cE E. d7 - ql&Q. This equation is Gauss’s Law in electrostatics. This gives the value of total

Flo
S

S. This equation gives an electric field due to disaete

ee
electric flux in terms of the charge enclosed by surface
charge or due to certain charge distribution and tells that the electric lines of force start from some positive

Fr
charge and end at some other negative charge, i.e., the electric lines offorce do not form continuous closed
path. This law predicts that isolated charge exists.

for
ur
. —> ^

(iO cE B.d7=0. This equation is Gauss’s Law in magnetostatics, where <p B.ds represents the
s ^ ^
ks
surface integral of S over a closed surface (i.e., the magnetic flux over the surface S). This law shows that
Yo
the number of magnetic lines of force entering a closed surface is equal to number of magnetic lines of force
oo

leaving it. It means, there is no starting or end point of magnetic lines of force. Thus, the magnetic lines offorce
eB

always form closed paths. If it were not so, then the isolated magnetic monopoles would have existed. Thus,
the Maxwell’s equation («) confirms the absence of magnetic monopole.
ur

{Hi) () E.dl = - — f ~B d7. This equation is Faraday’s law of electromagnetic induction where,
ad

dt h
Yo

B. d s represents the surface integral of B over a surface bounded by a closed curve over which the line
nd
Re

integral for electric field is taken. This law gives a relation between electric field and a changing magnetic
Fi

flux. This law shows that the line integral of electric field around any closed path (i.e. the emf } is equal to
the time rate of change of magnetic flux through the surface bounded by the closed path.
(iv) i B. fr/ = P(j / + IIq Eq “ 1 ^ equation is generalised form of Ampere’s law as Modified
by Maxwell and is also known as Ampere-Maxwell law. This law shows that the line integral of the magnetic
field around any closed path is related to the conduction current (/) and displacement current
through that path. This law predicts that the magnetic field can be produced by a
d
'd E .d7
conduction current as well as by displacement current.

(v) Lorentz Force, F = q{E+ vxB)


The above five equations give a complete description of all electromagnetic interactions.

t
8/6
‘P’u^dee^'4^ Fundamental Physics (XII) VOL.II

Importance of Maxwell’s equations


On the basis of these equations. Maxwell predicted the following :
(/) An accelerated charge is the source of electromagnetic waves.
(ii) The electromagnetic waves can propagate through the space with the speed of light (=3x10^ m/s).
(///) The electromagnetic wave is of transverse nature.
(iv) Light itself is an electromagnetic wave as it is transverse in nature and it moves in space with a speed
= 3x10^ m/s.

8.6. HISTORY OF ELECTROMAGNETIC WAVES


Faraday (rom his experimental study of electromagnetic induction concluded that a magnetic field
changing with time at a region produces an electric ifeld in that region. Maxwell in 1865 from his theoretical
study pointed out “there is a great symmetry in nature”, i.e., an electric field changing with time in a

ww
region produces a magnetic field there. It means a change in either field (electric or magnetic) with time
produces the other field. This idea led Maxwell to conclude that the variation in electric and magnetic field
vectors perpendicular to each other leads to the production of electromagnetic disturbances in space. These
disturbances have the properties of wave and can travel in space even without any material medium. These

Flo
waves are called electromagnetic waves.

e
According to Maxwell,

rere
r FF
the electromagnetic waves are those waves in which there are sinusoidal variation of electric
and magnetic field vectors at irght angles to each other as well as at irght angles to the direction
of wave propagation.
uurr
foor
Both these fields vary with time and space and have the same frequency. In Fig. 8.4., the electric field
kss
Yoo
vector {E) and magnetic field vector (B) are vibrating along T and Z directions and propagation of electro
ooook

magnetic wave is shown in X-direction.


eBB

Maxwell also found that the electromagnetic wave should travel in free space (or vacuum) with a speed
given by
FIGURE 8.4
uurr

...(10)
y4
ad

ENVELOPE OF ELECTRIC
Yo

INTENSITY VECTOR
where Pq and Eq are permeability and E
dY

E
permittivity of the free space respectively.
/■ L
Re

We know, = 4 it x 10“'^ Wb A“'


innd

nr

eo = 8-85x m“2 o
Fi

Putting these values in (10), we have X

c= 3.00 X 10^ ms
-1
B B
Z
The velocity of electromagnetic wave in a E
ENVELOPE OF MAGNETIC
1 INDUCTION VECTOR
medium is given by V-

where p, e are the absolute permeability and absolute permittivity of the medium. We also know that
p - Po p^ and e = e 0 G ^ where p^, e ^ are the relative permeability and relative permittivity of the medium.
1 c
Therefore, V = I
.. ^ _

V^O^O _
Maxwell also concluded that electromagnetic wave is transverse in nature and light is
electromagnetic wave.

t
ELECTROMAGNETIC WAVES 8/7

Examples of electromagnetic waves iire radio waves, microwaves, infrared rays, light waves, ultraviolet
rays, X-rays and y-rays.
In 1888. Hertz demonstrated experimentally the production of electromagnetic waves using a spark
oscillator and then detecting them successfully. The wavelength of electromagnetic waves produced with the
help of Hertz experiment was 6 m. In recognition to the successful demonstration for the production of
electromagnetic waves by Hertz, the SI unit of frequency was named as hertz (Hz).
After 7 years in 1895, an Indian Physicist Jagdish Cliander Bose was able to produce electromagnetic
waves of wavelength 5 mm to 25 mm. But his experiment was confined to laboratory only.
In 1896. Guylielmo Marconi discovered that if one of the spark gap terminals of Hertz oscillator is
connected to an antenna and the other terminal is eaithed, the electromagnetic waves produced can go up to
several kilometer. These waves could be detected at a large distance by making use of antenna-earth system
as detector. Using these airangements, in 1899, Marconi was the first to establish a wireless communication

ww
across the English channel, a distance of about 50 km.

8.7. TRANSVERSE NATURE OF ELECTROMAGNETIC WAVES

Flo
Consider a plane electromagnetic wave travelling in the FIGURE 8.5

X-direction and an elementary rectangular parallelepiped

e
6
B
OABCDEFG placed with its edges parallel to the three axes,

rere
Fig. 8.5. The wave front of electromagnetic wave is in the YZ

r FF
F C

plane and ABCD is a portion of it at time t. The values of electric


and magnetic fields will be zero to the right of the face ABCD,
uurr
but to the left of this face the values of the two fields depend on
X and t but not on y and z, since we arc considering a plane
foor
kss
'A
0 X
wave.
Yoo
ooook

To establish transverse nature of electromagnetic waves, E D


eBB

—^ >

we will establish that E and B are perpendicular to the Z

direction of wave propagation.


If the rectangular parallelopiped does not enclose any charge, the total electric flux across it must be
uurr
ad
Yo

zero, according to Gauss’s law, i.e.. n E.d s =0


dY

^ f -»
+ E .d s + E.cls =0
1
Re

E.ds E.ds 4- E.ds


innd

or E.ds + DCFE OADC


ABCD OCFE BGFC OEDA
/ V
Fi

...(11)

Since E does not depend on y and z. therefore, the contribution to the total electric flux coming from
the faces normal to y and z axes will cancel out in pairs i.e.

E.ds + E.ds = 0 ...(12)


BGFC OEDA

—>

and
E.ds +
1 E.ds = 0 ...(13)
OABG
\DCFE
From equations (11), (12) and (13). we have
...(14)
—»
E.ds + E.ds =0

ABCD OGFE

f
8/8
Fundamental Physics (XII) VOL.III

If and £/ are the values of A:-component of electric field on faces ABCD and OGFE respectively and
S be the area of each of these faces, then

E.ds =E S and E.ds =-E 'S


X X
ABCD OGFE

The negative sign arises due to the fact that the area vector at the face OGFE is in oppositedirection to
that of face ABCD.

From Eq. (14), we have s - e; s = o or (£^. - Ef) 5 = 0


This relation is true only if either, E^ - EJ = 0, Le.. E^ = £/ or E^. = E^ = 0
Since 5^0, the possibility, E^ = £;/ predicts that the field is static. But a static field cannot propagate a
wave of finite wavelength. Therefore, E^ = E^' - 0, i.e., no component of electric field is parallel to the
direction of wave propagation. It means, the electric field is perpendicular to the direction ofpropagation

ww
of electromagnetic wave.
A similar argument holds true for the magnetic field. We thus conclude that in the propagation of
electromagnetic waves, the electric and magnetic fields are transverse i.e. perpendicular to the direction of

Flo
wave propagation. Hence electromagnetic waves are transverse in nature.

e
ree
8.8. VELOCITY OF ELECTROMAGNETIC WAVES

Fr
Consider a plane electromagnetic wave propagating along positive direction of X-axis in space with

rF
speed c. Since in electromagnetic wave, the electric and magnetic fields are transverse to the direction of
uurr
for
wave
propagation, therefore, the electric and magnetic fields are in Y-Z plane. Let the electric field E be
acting along Y-axis and magnetic field B along Z-axis.
s
kks
At any instant, the electric and magnetic fields varying sinusoidally with .r and t can be represented bv
Yo
oooo

the equations
E = Ey = Ef^ sin (o (r - x/c) ...(15)
eB

B = B^ = Bq sin (0 (/ - x/c) ...(16)


Here Eq and Bq are the amplitudes of electric and magnetic
ur

fields along T-axis and 2-axis respectively. Consider a rectangular


ad

path PQRS in X-Y plane as shown in Fig. 8.6.


YYo

The line integral of E over the closed path PQRS will be


dd

Q R p
Re

J) E.dl = E.dl + E.dl + E.dl + E.dl


in

PQRS p Q R
F

= 0+E / + () + £: (-/)
(-r?)

= Er,0 I sin (0 r - -sino) t—-


J c
/J
...(17)

Magnetic flux linked with surface surrounded by rectangular path PQRS will be
^2
5.. Ic
'/ .^9" I — cos CO ( / X~
\
‘^‘5 = B{x)ldx= B,J
0
sin CO —
COS CO
-V, CO c
-h (● /J

c/({) ]j _Bq Ic ^2 X,1 ^ X


-cosin (0 r —— +(0sin CO t- 1
dt CO c
= ~Bq Ic Sin CO / —^ -smeo t —-
/-J c
/J

...(18)

« 4
ELECTROMAGNETIC WAVES 8/9

Using Faraday’s law of electromagnetic induction, we have y FIGURE 8.7

'B
j) E,dl = - dt
E
4E

Putting the values from (17) and (18), we get


Eq = cB 0 ...(19)
Since E and B are in phase, we can write
E=c Bat any point in space.
Consider a rectangular path PUTQ in the X-Z plane as
shown in Fig. 8.7. The line integral of B over the closed path

ooww
PUTQ, we have

§ BM = \ B.dl +\ B.dl +\B.dl +jB.dl = B


(●«])
/ + 0-B
U2>
l +O
PUTQ P U T Q

e
ree
f X ^ r ^ ^

rFl = Bn/ sin© t—- -sin© t—- ...(20)

Fre
0
c

rrF
The electric flux linked with the surface surrounded by rectangular path PUTQ is
ouur
(j>g=
^2

E(x)ldx =EqI Jsin© t- ^ j sffoo (


- dx = --E^l
0
©
-cos© t
f
X ^
+COS©
( X
t—-
V
okks
●*^1
Yo
ooo

V
d^_ ^2'
BB

●^2 ●^1
or = -cEqI sin© t—^ -sm© t =cEnl
0
sin© t- — -sin© t—^ ...(21)
c
dt \ c J c ;j c
rr e

In space, there is no conduction current. According to Ampere Maxwell law in space


ouu
ad
YY

^ B.dl =^10^0 dt
PUTQ
dd
Re
iinn

Putting values from (20) and (21), we get


1
F

5o = Hoeo^^^o = l^oeoc(c 5q) or l = Poeo‘^ or ...(22)

Which is the speed of electromagnetic waves in vacuum.


-1
For vacuum, Po = 4 tc x 10“^ T mA
1 1
and = 9x10^ Nm2 C"2 N"' m“2 C2
or
^0= 47tx9 xlO^
47ce
0

1
Putting the values in (22), we get c = I = 3 X 10^ m/s
^471x10"”^ xl/(471x9x10^)
which is exactly the speed of light in vacuum.
This shows that light is an electromagnetic wave.

A
8/10
Fundamental Physics (XII)CHHD
1 r:
in in Memory
1. The speed (c') of electromagnetic wave in a dielectric of dielectric constant K is given by
1 c

V?
2. For an electromagnetic waves, the magnitude of the propagation vector (?) is,
27U 0)
it =

oww
X c
3. The speed of electromagnetic wave through a medium of permeability |i and permittivity e is,
1 1 c
V =

and€=€oe^_

e
where are the relative permeability and relative permittivity of medium.

re
FFrllo
reF
Sample Problem
The magnetic field in a plane electromagnetic wave is given by

e
By = 2 X IQ sin (O'S x 10^ j: + 1-5 x 10** /) T. (a) What is the wavelength and frequency of the wave ?
uoru
(b) Write an expression for the electric field. (NCERT Solved Example)

osFr
Sol. (ri) Comparing the given equation with the equation of magnetic field varying sinusoidally with x
and t as

B D ●
y=B^sm(a t + — =
^
fkfor 271
i +—
X ^
= Br. sin
■ 2ti 271X
\

= Br.0 sin
2 7t .r 2 7t r'
okso
J
t +
0
T T c X T
Y
Yo
2k 2n 2k
X =
oo

— = 0-5xl(^ m = 1-26 X 10-2 m = I-5xl0‘*


BB

we gel, or and
X 0-5 xlO^ T

l-5xl0'*
Y

2 7rv- I-5x 10*'


r ree

or or V - = 2-39x10**^ Hz
271
ouu

= 2 X lO-"^ T, Eq = cBq = {2x 10*^) X 2 X 10-^ = 60 Vm"*


ad
Ydo

In electromagnetic wave, the electric field is vaiying sinusoidally with x and /, perpendicular to the
direction of magnetic field as well as the direction of propagation of electromagnetic wave. So electric field
nidn

2tcx 2Kt
is varying along z-direction. Thus, £, = sin = 60 sin (0-5 x 10^ x + 1-5 x lO" t) V/m
Re

T
F
Fi

^ PRODUCTION OF ELECTROMAGNETIC WAVES


We know that an electric charge at rest has electric field in the region around it, but no magnetic field.
A moving charge produces both the electric and magnetic fields. If a charge is moving with a constant velocity
ii.e. if current is not changing with time), the electric and magnetic fields will not change with time, hence no
electromagnetic wave can be produced. But if the charge is moving with a non-zero acceleration {i.e., charge
is accelerated) both the magnetic field and electric field will change with space and time, it then produces
electromagnetic wave. This shows that an accelerated charge emits electromagnetic waves.

We have already explained in unit 4, chapter 7 that L—C circuit is an oscillatory circuit where the charge
is oscillating across the capacitor plates. An oscillating charge in L-C circuit has a non-zero acceleration,
hence it emits electromagnetic waves which have the same frequency as that of the oscillating charge.
In an atom, an electron while orbiting around the nucleus in a stable orbit, although accelerating, does
not emit electromagnetic waves. Electromagnetic waves are emitted only when it falls from higher energy
orbit to lower energy orbit.
Electromagnetic waves {i.e. X-rays) are also produced when fast moving electrons are suddenly stopped
on the metal target of high atomic number.

4
ELECTROMAGNETIC WAVES 8/11

8.10. IMPORTANT CHARACTERISTICS AND PROPERTIES OFTHE ELECTROMAGNETIC WAV!


1. The electromagnetic waves tire produced by accelerated or oscillating charge.
2. These waves do not require any material medium for propagation.
3. These waves travel n free space with a speed 3 x 10® ms {i.e. speed of light) given by the relation
A

-1

c = \/^l^ 0 ●

4. In electromagnetic waves the sinusoidal variation in both electric and magnetic field vectors
( £ and 5 ) occurs, simultaneously. As a result, they attain the maximum and minimum values at the same
place and at the same time. The amplitudes of the electric and magnetic fields in free space are related by ,
—»

5. In em waves, the oscillations of E and B are in the same phase.

ww
—)

6. The cross product ExB always tells the direction in which the wave travels.

FF loo
7. The directions of variation of electric and magnetic field vectors are perpendicular to each other as
well as perpendicular to the direction of propagation of waves. Therefore, electromagnetic waves are
transverse

ree
in nature like light waves.
8. The velocity of electromagnetic waves depends entirely on the electric and magnetic properties of the

rFee
medium in which these waves travel and is independent of the amplitude of the field vectors.
9. The velocity of electromagnetic wave in dielectric is less than c (= 3 x 10^ ms *).

F
oor r
rur
10. The electromagnetic waves carry energy which is divided equally between electric field s ff and magnetic
field vectors.

In vacuum, the average electric energy density and average magnetic energy density {ug) due to
k
YYoou
static electric field E and magnetic field B which do not vary with time are given by
ookos

1 1 B^
BBo

and
2^^
u
Ur B
2 M-o
re

1 1 B-
Due to both the fields, total average energy density is u =Ur+u B £2 +-
ouur

2^0
ad

2h 0
Yo

In electromagnetic waves, the electric and magnetic fields vary sinusoidally with space (a:) and time (f).
The above expressions will be valid for electromagnetic waves if £ and B are replaced by their rms values.
dY
Re

Therefore,
idn

I b}0
FFin

I B^ I
1
r- = —
—-y
E-
1
^^ = -!-e £2 and u
B
^ nns — Ex ^ = —
2^0 2 4 2 2 2\Xq 4 Pq
u
E 2 ^

B^ 1 E- 1 E
rms
1
rms _ rms
and c —
Here, Uq =
x
I171S rms
2 ft 0 2|i 0 c~ 2 ft 0 c

.-. Total average energy density is


£2rms 1
ii
av = «£ + = 2»£ = 2hj =2xis„ — G £^
“ ^0 ^rms
^^0 2 2

11. The electric vector is responsible for the optical effects of an electromagnetic wave and is called the
Light Vector.”
12. The electromagnetic waves being unchiu'ged are not deflected by electric and magnetic fields.
8/12
Fundamental Physics (XII) VOL.II

13. Intensity of electromagnetic wave


Intensity of electromagnetic wave ot a point is defined as the energy crossing per second per
unit area normally around that point during the propagation of electromagnetic wave, i.e.,
energy
Intensity =
area X time
Consider the propagation of electromagnetic wave with speed c along the X-axis. Take an imaginary

w
cylindei of area of cross-section/4 and length c A /, where At is the time of propagation of wave, and the wave
crosses the area A normally. Fig. 8.8. Let u be the average energy density of electromagnetic wave.
tia

The energy of electromagnetic wave f U) crossing the area of cross-section at P normally in time A t is
the energy of wave contained in a cylinder of length c A / and area of cross-section A. It is given by

e
U = (c At) A

row
FIGURE 8.8

re
The intensity of electromagnetic wave at P is, H- cAt ■w
U u . cAiA
av
/ =

FFllo
= u c

eeF
av
A At AAt 1
i A
■►c
In terms of maximum electric field.
I

u
I
I

r
= -€ E-

sFr
u
2 0 ^0 ’
so
av 0 0 p
2 mix
Q

kro
In terms of maximum magnetic field,
uor
U 1^ SO / = —
I ,B
offo
^c =
2
I
B- c
2 4o'
av
ruts
Ho
kos
Y
Yo
Retain in Memoiy
eerBB
oo

1. The electric and magnetic field vectors £ and B are always perpendicular to the direction of
rY

propagation of the electromagnetic wave. Hence the electromagnetic wave is of transverse wave.
u

2. The electric field vector is always perpendicular to the magnetic field vector in electromagnetic
wave. ®
ou
ad
do

3. The cross-product ExB always gives the direction of propagation of electromagnetic wave.
nY

^ ^

4. {Ex B) /fio gives the normal energy flow per unit area per unit time in an electromagnetic wave
nid

called Poynting vector (5). Its Si unit is Wm"“.


Re

5. The two fields, {i.e. electric and magnetic) must vary sinusoidally, with the same frequency and in
F
Fi

phase with each other in electromagnetic wave.


6. The ratio co//: for a travelling wave is its speed.
7. Electromagnetic wave has linear momentum as well as energy. That is why electromagnetic wave
exerts radiation pressure on the surface of object, on which it falls.
8. All the electromagnetic waves of different radiations move with same speed in vacuum but they
move with different speeds in a medium.

14. Momentum and Radiation Pressure of electromagnetic wave. The electromagnetic waves carry
energy and momentum. If a portion of electromagnetic wave of energy U is propagating with speed c, then
linear momentum of electromagnetic wave is given by
U SxA
p = —
c c

where S is the Poynting vector {i.e., normal energy flow per unit area per unit time) and A is the area of
cross-section of wave.
ELECTROMAGNETIC WAVES 8/13

If the incident electromagnetic wave is completely absorbed by a surface, it delivers energy V and
momentum V/c to the surface. Tf the electromagnetic wave is totally reflected, then the momentum delivered
to the surface is 2 U/c as the momentum of electromagnetic wave changes from p to -p. This shows that the
electromagnetic wave exerts a force on the surface on which it is incident.
This force exerted by electromagnetic wave on unit area of the surface is called radiation pressure.
It means, electromagnetic wave exerts radiation pressure on the surface it falls,
force change in momentum/lime change in momentum
Radiation pressure = area x time
area area

If p is the change in linear momentum of electromagnetic wave over area A in time Ar, then radiation
pressure is

ooww
P Ulc U 1 /
P = — X- = —
AA; AA/ AAr c c

U
Because, p = Ulc and = / = intensity of em wave.
AAt

e
In 1903, the American scientists Nicols and Hull measured radiation pressure of visible light which was

ere
found to be of the order of 7 x 10"^ N/m^. It means the force due to radiation pressure of visible light on the

rFl
Fre
surface of area 10 cm^ is = (7 x 10"^) x (10 x 10^) = 7 x 10“^ N.

rrF
Sample Problem In an electromagnetic wave, the amplitude of electric field is 10 V/m. The
frequency of wave is 5 x 10 Hz. The wave is propagating along Z-axis, find (i) the average energy
14

sffoo
ouur
density of electric field (li) the average energy den.sity of magnetic field (iii) the total average energy
density of e.m. wave.
kosk
Sol. Here, £„ = 10 Vm'' ; k = 5 x 10^^ Hz.
Yo
(i) Average energy density due to electric field is
oo
Y

1 (E ^
BB

1
— — rt- ie e}0
E ~ 2^^ 2^0 V2 4^0
u
rre

\ ' /

:ix(8-85xl0"i2)x(l0)2 = 2-21 X 10-^0 j/n,3


ouu
Y
ad

(it) Average energy density due to magnetic field is


dY

e}0
1 Blmx 1 B 1 gp _ 1 (£q/c)^
innd

_ 0
=“£
Re

u
B
2 4 1^0 ~ 4 ^^0 4lip c- 4po(l/^pep) 4
Fi

= 2-21 X 10-*® J/m^


F

(Hi) Total average energy density of e.m. waves


„ »g=2-21 X 10-'®+ 2-21 X 10-'® =4-42 x 10"*® J/m^
8.11. HERTZ EXPERIMENT
Maxwell in 1865 predicted electromagnetic waves from theoretical considerations and their existence
was confirmed experimentally by Hertz in 1888.
Hertz experiment was based on the fact that an oscillating electric charge radiates electromagnetic
waves and these waves caiTy energy which is being supplied at the cost of kinetic energy of the oscillating
charge. The detailed study revealed that the electromagnetic radiation is significant only if the distance to
which the charge oscillates is comparable to the wavelength of radiation.
For illustration, if a charge is oscillating with a frequency 1000 Hz, it would radiate electromagnetic
SxlO''
waves of wavelength - 3 X 10^ m = 300 km.
V 1000
8/14
'PniuUe^'^ Fundamental Physics fXinrrgm
It shows that the charge should oscillate over a distance of 300 km in order to radiate sufficient amount
of energy. Hertz developed a system of oscillating charges in order to produce electromagnetic waves in
laboratory.
Hertz Apparatus. The experimental set up used by Hertz for FIGURE 8.9
the production and detection of electromagnetic waves is shown in
Fig. 8.9. Here A and B are two large square metal plates of copper or
zinc, placed about 60 cm apart. They are connected to two highly A

polished metallic spheres 5, and ^2 through thick copper wires. A METAL


high potential difference of .several thousand volts is applied across PLATE
the spheres using induction coil. Due to high potential difference across
and 52, the air between the spheres gets ionised and provides a
path for discharge of plates. Due to it, the large number of successive i
TO INDUCTION /
. kS
tiny sparks are jumping across the gap between 5, and ^3. This sends COIL
out train of tiny waves which inium are received by the detector (or

ww
T
receiver). This train of tiny waves is the electromagnetic wave, which
is radiated out by the transmitter. DETECTOR
OR
Here the two plates act as a capacitor having a small capacitance

Flo
RECEIVER
C and the connecting wires provide the low inductance L. The high
frequency of oscillations of charges between the plates

ee
generate
B
electromagnetic waves of high frequency (i>) which is given by

rere
METAL

rFF
1 PLATE

2tcVZc
uurr
foor
The frequency of these electromagnetic waves produced by Hertz is of the order of 5 x 1Hz. So the
wavelength (k) of electromagnetic waves produced is given by
ks s
c 3x10^ ms
-1
X = -
Yoo
= 6 m
oook

5x10^ Hz
The detector or receiver is also shown in Fig. 5.9. It is in the form of unclosed metallic ring having small
eBB

metallic spheres C and D with some gap. It is held in position such that the two gaps are parallel to each other
and the magnetic field produced by the oscillating current is perpendicular to the plane of ring. The oscillating
magnetic field linked with the ring produces large induced emfs which causes tiny sparks to appear at
uurr

the
spheres C and D, since the resistance of metallic ring is very small, Thus the electromagnetic waves
ad

arc
Yo

detected or received.

Hertz conclusions about electromagnetic waves


dY

1. Hertz demonstrated that the electromagnetic waves can be reflected, refracted and show the phenomena
Re

of interference and polarisation. This establishes that the em waves has wave nature.
innd

2. In Hertz experiment, the electric field associated with the electromagnetic waves radiated from the
FFi

Iransmittei is parallel to the two gaps. This shows that the direction of electric field associated with
electromagnetic wave is perpendicular to the direction of propagation of electromagnetic waves. This
establishes that electromagnetic waves are of transverse nature.
3. Hertz observed that the maximum sparks are produced across the gaps when the gap between C and
D is parallel to the gap between 5, and 82- If these two gaps are perpendicular to each other, no sparks are
observed across the gap between C and D and hence no electromagnetic waves are detected. This shows that
electromagnetic waves can be polarised.

8.12. ELECTROMAGNETIC SPECTRUM


After the expenmental discovery of electromagnetic waves by Hertz, many other electromagnetic waves
were discovered by different ways of excitation.
The orderly distribution of electromagnetic radiations according to their wavelength orfrequency
is called the electromagnetic spectrum.
8/15
ELECTROMAGNETIC WAVES
The electromagnetic spectrum has much wider range with wavelength variation ~
m to

6x lO^m Xhe whole electromagnetic spectrum has been classified into different parts and subparls in order
of increasing wavelength, according to their type of excitation. There is overlapping in certain parts of the
spectrum, showing that the corresponding radiations can be produced by two methods. It may be noted that
the physical properties of electromagnetic waves are decided by their wavelengths and not by the method of
their excitation.

A table given below shows the various parts of the electromagnetic spectrum with approximate wavelength
range, frequency range, their sources of production and detections.
Production Detection
Type Wavelength range (m) Frequency range (Hz)
Radio waves > 0-1 m 5 X 10^ to 3 X 10^ Oscillating circuit or Receivers aerials
Rapid acceleration
and retardation of

llowow
electrons in aerials

0-1 m to I mm 3x 10^to3x 10” Klystron valve or Point contact diodes


Micro wave
magnetron valve
Infrared 1 mm to 700 nm 3x 10” to4x 10^*^ Vibrations of atoms Thermopile, Bolo
and molecules meter, Infrared

ee
photographic films

Fr
Visible light 700 nm to 400 nm 4x 10*^ to8x 10'^
r FF Excitation of Human eye, photo
valency electrons in cells, photographic
atoms plate

r
8x 10'^ to3x lO'^ Photo cells. Photo
Ultra violet 400 nm to 1 nm
forfFore
u
Excitation of atoms, graphic film
s
1 nm to 10 ^ nm 3 X lO'"^ to 3 X lO-O .spark and arc lamps Photographic film,
X-rays '
ok
YYour o
X-ray tubes or Geiger tubes
excitation of inner
o

shell electron Photographic film.


Gamma rays < lO*-^ nm 3 X 10'^ to 5 X 10“^
eeBoBks

Nuclear origin Ionization chamber


r

Only a small part of the complete electromagnetic spectmm is visible to human eye, as the radiations ol
our u
ad

this part can produce some sensation on the retina of the eye. This part of the spectrum is called visible
Yo

spectrum. The radiations of different wavelengths in the visible part of the spectrum produce different colours
on the retina of our eye. This is given in the following table.
d
Re

I (m) X 10-7 Colour I (m) X10 ^ Colour


in
FFind Y

5.7 to 5.9 Yellow


4 to 4.5 Violet/Indigo
4.5 to 5.0 Blue 5.9 to 6.2 Orange
Green 6.2 to 7.5 Red
5.0 to 5.7

8.13. MAIN PARTS OF ELECTROMAGNETIC SPECTRUM AND THEIR USES


The electromagnetic spectrum has been broadly classified into following main parts ; mentioned below
in the order of increasing frequency.
1. Radiowaves
These are the electromagnetic waves of frequency range from 5 x 10^ Hz to 3 x 10 Hz. These waves
are produced by oscillating electric circuits having an inductor and capacitor.
USES
The various frequency ranges are used for different types of wireless communication systems as mentioned
below :
8/16
'P’uieU^'A Fundamental Physics (XII) VOL.ll

(/) The electromagnetic waves of frequency range from 530 kHz to 1710 kHz form amplitude modulated
(AM) band. It is used in ground wave propagation.
(/7) The electromagnetic waves of frequency range 1710kHz to54MHzare used for short wave AM band.
U is used in sky wave propagation.
{ill) The electromagnetic waves of frequency range 54 MHz to 890 MHz are used in television waves.
(iv) The electromagnetic waves of frequency range 88 MHz to 108 MHz form frequency modulated
(FM) radio band. It is used for commercial FM radio,
(v) The electromagnetic waves of frequency range 300 MHz to 3000 MHz form ultra high frequency
(UHF) band. It is used in cellular phones communication.
2. Microwaves

Microwaves are the electromagnetic waves of frequency range 3 x 10^ Hz to 3 x lO'^ Hz. They are
produced by special vacuum tube.s, namely ; klystrons, magnetrons and Gunn diodes etc.
USES

ww
(/) Microwaves are used in Radar systems for air craft navigation.
(//) A radar using microwave can help in detecting the speed of tennis ball, cricket ball, automobile while
in motion.

Flo
(Hi) Microwave ovens are used for cooking purposes.

ee
(/V) Microwaves are used for observing the movement of trains on rails while sitting in microwave operated
control rooms.

rere
rFF
3. Infrared waves
Infrared waves were discovered by Herschell. These are the electromagnetic waves of frequency range
uurr
3x10 Hz to 4 X 10 Hz. Infrared waves sometimes are called as heat waves because their absorption

molecules. These waves are not delected by human eye


foor
causes the heating effect in the bodies and surroundings. Infrared waves are produced by hot bodies and
but snakes can detect them
ks s
USES
Yoo
oook

Infrared waves are used

(/) in physical therapy, i.e., to treat muscular strain.


eBB

DO YOU KNOW 7
(//) to provide electrical energy to satellite by using solar cells
(Hi) for producing dehydrated fruits To measure the temperature of
uurr

human body, ear is one of the best


ad

(iv) for taking photographs during the conditions of fog, smoke etc. places because it is close to hypo
Yo

(v) in green houses to keep the plants warm and in maintaining Earth’s thalamus, the area at the bottom
warmth through the green house effect.
dY

of brain that controls body


(vi) in revealing the secret writings on the ancient walls. temperature. It is due to this
Re
innd

reason, now the doctors use ear-


(vi7) in solar water heaters and cookers
thermometer (like pyroelectric
FFi

(vn7) in weather forecasting through infra-red photography thermometer) to measure the


(ix) in checking the purity of chemicals and in the study of molecular temperature of human body.
structure by taking infra red absorption spectrum,
(.r) in eye surgery.
4. Visible light
It IS the narrow region of electromagnetic spectrum, which is detected by the human eye. Its frequency
is ranging from 4 x lO’^ Hz to 8 x 10^^ Hz. It is produced due to atomic excitation.
The visible light emitted or reflected from objects around us provides the information about the world
surrounding us.
USES

Visible light is used


(0 to see the beautiful world around us as it excites our sense of vision
8/17
ELECTROMAGNETIC WAVES

(ii) in photography to take the picture of objects


{Hi) in astronomy to track the movement of heavenly bodies
(iv) in optical microscopy which involves the study of minute objects.
5. Ultraviolet rays
The ultraviolet rays were discovered by Ritter in 1801. The frequency range of ultraviolet rays is
8 X 10^‘^ Hz to 3 X 10*^ Hz. The ultraviolet rays are produced by sun. special lamps and very hot bodies. Most
of the ultraviolet rays coming from sun arc absorbed by the ozone layer in the eaith’s atmosphere. The
ultraviolet rays in large quantity produce harmful effect on human eyes.
USES

Ultraviolet rays are used


(/) for checking the mineral samples through the property of ultraviolet rays causing flourescence.

ww
(//) in the study of molecualr structure and arrangement of electrons in the external shell through ultraviolet
absorption spectra.
{Hi) to destroy the bacteria and for sterilizing the surgical instruments.

FF loo
(iV) in burglar alarm,

ree
(v) to preserve the food stuff.
{vi) in Lasik eye surgery

reFe
{vH) to kill germs in water purifier.

oroFr
6. X-rays
rur
The X-rays were discovered by German physicst W. Roentgen. Their frequency range is 3 x 10 Hz to
s ff
3 X 10"*^ Hz. These are produced when high energy electrons are stopped suddenly on a metal of high atomic
k
number. X-rays have high penetrating power.
YYouo
okso

USES
BBoo

X-rays are used


(/) In surgery for the detection of fractures, foreign bodies like bullets, diseased organs and stones in the
r ee

human body.
(i7) In Engineering (i) for detecting faults, cracks, flaws and holes in final metal products {ii) for the
ad
ouur
Yo

testing of weldings, castings and moulds.


{Hi) In Radio therapy, to cure untracable skin diseases and malignant growths.
{iv) In detective departments (0 for detection of explosives, opium, gold and silver in the body of smugglers,
d
Re
idnY

(v) In Industry (/) for the detection of pearls in oysters and defects in rubber tyres, gold and tennis balls
FFin

etc. (//) for testing the uniformity of insulating material.


(vi) In Scientific Research (i) for the investigation of structure of crystals, arrangement of atoms and
molecules in the complex substances.
7. y-rays
y-rays are the electromagnetic waves of frequency range 3 x 10‘^ Hz to 5 x 10^^ Hz. y-rays have nuclear
origin. These rays are highly energetic and are produced by the nucleus of the radioactive substances.
USES

y-rays are used


(z) in the treatment of cancer and tumours.
{ii) to preserve the food stuffs for a long time as the soft y-rays can kill microorganisms easily.
{Hi) to produce nuclear reactions,
(zv) to provide valuable information about the structure of atomic nucleus.
8/18
^ Fundamental Physics rxinrosTWn
8.14. MICROWAVE OVEN

It is a device used in the kitchen for heating or cooking the food. The basic principle of working of
microwave oven is to create microwave radiation of suitable frequency in the working space of oven where
the food to be cooked, can be kept. This radiation may match the resonant frequency of rotation of water
molecules which is about 3 GHz. In this situation, energy from the waves is transfeired efficiently to the
kinetic energy of molecules. The microwave of this frequency is of energy equivalent to heating up water.
When microwaves of this frequency fall on food item containing water like fruit, vegetables, meet, cereals,
etc., placed in oven, the water molecules absorb these radiations. Their energy increases. These molecules
share their energy with neighbouring food molecules. Due to it, the entire food gets heated.
In a microwave oven one should use porcelain vessel and not metal container, for cooking the food
items. It is required to avoid the danger of getting an electric shock from accumulated electric charges and to
avoid the melting of metal from heating.
In a microwave oven, the porcelain container containing food for cooking remains uneffected and cool

ww
whereas only the food items gets heated. It is so because the large molecules of porcelain container vibrate
and rotate with much smaller frequency than that of microwave and thus cannot absorb microwaves.

FF loo
In thi.s situation energy from the waves is transferred efficiently to the kinetic energy of molecules.

ree
Curiosity Questions
r
Q. 1. Suppose the eyes of an alien being are sensitive to microwaves. Do you expect the alien

reFe
being to have larger or smaller eyes than ours ?

oroFr
r ur
Ans. The wavelength of microwaves is much larger than that of visible light. The microwaves can
s ff
enter the eyes and produce the sensation of sight if the size of the eyes and the ratina are
comparable to the wavelength of microwaves. Therefore, the alien being should have much
larger eyes than ours.
k
YYouo
Q. 2. It is a common belief that the exposure of radiations for long time is dangerous to health.
koso

Are the meals cooked in microwave oven not dangerous to health ?


BBoo

Ans. As is known, the exposure of high energy radiation like -j^rays, X-rays and ultraviolet rays for long
r ee

time is dangerous to health as they spoil the blood cells of the body. Microwaves are low energy
waves with energy much less than that of yrays and X-rays. The microwaves do not have the
energy necessary to ionise molecules in the way, that y-rays, X-rays can do. Therefore the meals
ad
ouur

cooked in microwave oven are not dangerous to health.


Yo

J
d
Re
idnY
FFin

I TYPE I. DISPLACEMENT CURRENT,


AMPERE-MAXWELL’S LAW

Formulae used.
I (») |?.rf7=Ho(/+/„)
(fVr) Magnetic field induction at a point outside a

.straight wire at a perpendicular distance r from a


dE straight wire carrying current / is given by
= €»
0
/t
dt dt
B =
47C r

0
df\d) d dt where €q = 8.85 x lO"*^ m"“,
dV
m, = 47cx lO-"^ Wb A-' nv or (TA"' m)
or U = C V e„ A
D
dt \E = — and C = 0 Units used. V in volt, A in nr, C in afrad, dorr 'm
d
) m, I in ampere and B in tesla or Wh m~^.
8/19
ELECTROMAGNETIC WAVES

Example Q There is a parallel plate FIGURE 8.10

capacitor of capacitance 2*0 p,F. The voltage


between the plates of parallel plate capacitor is
changing at the rate of 6*0 V s”^ What is the >

displacement current in the capacitor ?


Solution. Here, C = 2-0 pF = 2 x 10^ F,
dV
= 6 V s'* (b) At what distance from the axis is the
dt
m agnetic field due to displacement current
Displacement current. greatest ? Obtain the maximum value of the field.
Solution. Here, /? = 0-12 m, / = 0-15 A
dE d rv'
Area of the plate. A = k R~ = iix (0-12)” m-.

loow w
=e.0 A —
dt di[d , («) Consider a loop of radius / between the two
circular plates, placed coaxially with them.
gp A dV = C
dV
Then area of the loop. A' = 7Z
d dt di —»

By symmetry magnetic field induction B is

ree
= (2x 10-^) X 6 = 12x KT^A equal in magnitude and is tangentially to the circle at
every point.

rree F
= 12 pA r FF In this case, only displacement cun'ent Ip will
Example Q A. parallel plate capacitor cross the loop. Therefore, using Ampere’s Maxwell
C = 0*2 pF is connected across an a.c. source of law, we have
fofr oF
u
angular frequency 400 rad s“^. The value of
ks
conduction current is 2 niA. Find the rms value of (J) B dl =Pp//)
YYouro
the voltage from the source and the displacement
2 71 r B = Pq X (current passing
soo

current in the region between the two plates.


through the area A')
BBook

Solution. Here, C = 0-2 pF = 0-2 x 10"^ F


r ee

7tr“
= 2 X 10“'^ F,
= ^‘o' D tiR-
for r<R
co = 400 rad/s, /nn.s = 2mA = 2x 10-'^ A
ouru
ad

= Mo h for > >R


V, rms = / rm.s xX c
Yo

= / X
1
= (2xl0"^)x
1
Thus, B = Ml) _ M() l[)'' ...(l) (If/-<«)
d

400x(2xl0"‘^) R-lKr 2kR-


Re

rms
coC
iYn

M()
FFind

= 25 V
and B = ...(«) (If r > R)
Inr
Displacement current = conduction current
= 2 mA (/) On the axis, r = 0.
B = 0
Using (/), we get,
Example g (a) Fig. 8.10 shows a capacitor
made of two circular plates each of radius 12 cm (/7) For a point 6-5 cm/ro/« the axis,
r='6-5 cm = 6-5 x 10"^ m.
and separated by 5*0 mm. The capacitor is being
charged by an external source {not shown in the Using (i), we have,
figure). The charging current is constant and equal 471x10 ^ x015x6-5xl0 "
to 0*15 A. Use Ampere’s law (modified to include B =

displacement current as given in the text) and the 27cx(12xl0-')2


symmetry in the problem to calculate magnetic = 1*35 X 10-^ T
field between the plates at a point (i) on the axis (iii) For a point 15 an from the axis.
(ii) 6*5 cm from the axis (iii) 15 cm from the axis. r= 15 cm = 015 m.

I
8/20
Fundamental Physics fXinPTSTWn
Using (//), wc have,
Example § A parallel plate capacitor of
4t:x10“'^x0I5 area 50 em^ and plate separation 3*0 mm is
5 = = 2 X 10"’ T
27CXO-15
charged initially to 80 |iC. Due to a radioctive
source
nearby, the medium between the plates gets
{b) From equations (/) and (//) we note that B ISi. lightly conducting and the plate loses the charge
maximum \f r-R = 12 cwt = 0-12 m initially at the rate of 1-5 x lO"^ Cs"*. (/) What is
the magnitude and direction of displacement
B
max
^ 4jrxlQ-'^xO-l5 current ? («) What is the magnetic field between
2nR 271x012 the plates ? 'NCEHT Solved Example)
= 2-5x 10-’T Solution. Conduction current within the plates
Example 0 (a) Usc the Biot-Savart law to is from the positive plate to negative plate of parallel
plate capacitor.
determine the magnetic field due to conduction

w
(/) Displacement current.
current outside the plates (refer to Fig. 8.10) at
points 6*5 cm, 12 cm and 15 cm from the wire. Do r/0 E d dE
the answers match with those in Ex. 3 ? Explain, 'd =^0 =6„ -(£A) =e„ A

Flo
dt dt
(b) If the conduction wire has a radius of 1.0

eeee
mm, what is the maximum value of magnetic field = A—
d( q 1 dq
= e.0 A
due to the conduction current ? [When you 0

Fr
dt Gn0 A A dt
0
compare the answer to Ex. 3 (b) and 4 (b), you
will appreciate why it is not easy to notice magnetic
/» =^ = l*5xl0-« A
for
field due to the displacement current],
ur
or
dt
(c) Suppose the thin wire in Fig. 8.10. is dq
replaced by rods each of radius 12 cm (i.e. we now Since charge is decreasing with time, so
ks
dt
have two long cylindrical rods separated by a small dE
Yo
and hence < 0- It shows that the direction of
oo

gap). Will magnetic field configurations for r > R dt

be identical for the regions between the plates and is opposite to that of electric field and hence opposite
eB

outside the plates ? to the conduction current. But the magnitude of


Solution. («) Using Biot-Savart law, magnetic displacement current is same as that of conduction
ur

field induction at a point outside a straight current current. The net current between the plates is zero.
ad

carrying wire is given by So total current, /' = / + /^ = 0


Yo

B =
2nr 07) Using Ampere’s Law, ^ dt =0
d
Re

Now. since / = /^. this formula is the same as in Hence magnetic field within the plates is Zero.
in

Ex. 3 for r > R. Therefore, for r = 12 cm and


F

15 cm. answers are the same as in Ex. 3. For TYPE II. VELOCITY, AMPLITUDE AND ENERGY
r = 6-5 cm (r < R), the two formulae differ. Here DENSITY OF ELECTROMAGNETIC WAVES
B = 4 6 X 10^^ T, greater than B in Ex. 3.
Formulae used. (/) Energy of a photon,
(/;) B is maximum at the surface of the wire.
E = hv = hc/X
B =
^0^ 47txl0 ^x015
id)
= 3-0x 10-^ T Eq = BqC
27tr 271x10“^
where Eq = Amplitude (or maximum value) of
[Here, r = 1.0 mm = 10“'^ m] electric field.

This is much greater than the maximum value Bq - Amplitude (or maximum value) of
of B in Ex. 3. Due to this reason, it is not easy to magnetic field
notice magnetic field due to displacement current,
1
(c) Yes, since / = /^ , magnetic field Ud) u erx0 E- =
r-'>

^ 2
configuration are identical for r>R. 2 %V2
8/21
ELECTROMAGNETIC WAVES

c-2 (3x10^)2
and uB
I
B 2 _(5q/V2)^ _ bI _ or
(2-2xKr)“ xl-0
= 1-86

0 2^0 ^^^0 ’
(iv) e = e(j/V2
Example B A plane electromagnetic Is
propagating in the x-direction has a wavelength
where u^, Ug = average energy density of electric of 5-0 mm. The electric field is in the ^-direction
and magnetic field
E. B = r.m.s. value of electric field and
and its maximum magnitude Is 30 Vnr*. Write
magnetic field respectively
the suitable equations for the electric and magnetic
fields as a function of x and t.
£q, = amplitude of E and B respectively. Solution. Here, X = 5-0 mm = 5 x 10“^ m ;

ooww
Total average energy density = + ug = 2 ug £n0 = 30 Vm
-1

1 B.?0 27tc’
= 2 II B £.?0 =
~
2^^ 2^ 0 Angular frequency o) = 271 v = --—
A.
-1

e
Units used. X is taken in m, c in ms , V in Hz, E in
2x3-142x(3xl0*^)

ere
NC“^ B in tesla, w^and Ug in Jm~^.

rFl
fix 10-3

Fre
Example 0 A plane electromagnetic wave = 3-77 X lO" rads-‘

rrF
of frequency 25MHz travels in free space along Max. magnitude of magnetic field,
the x-direction. At a particular point in space and
sffoo
ouur
£ 30
time the electric vector is, E = 6*3 V/mj . B
_ -^0 _ = 1-0x10''^ T
0 "
c 3x10*^
kosk
Equation for electric field along y-axis can be
Yo
Calculate B at this point.
oo
Y
written as
[CBSE 2006 (C)j (NCERT Solved Example)
BB

E= = £q sin CO (/ - xk)
E 6-3Vlm
rre

-1
Solution. B = — - —-o " = 2-1 X 10-^ T = 30 sin 3-77 x 10** (/ - xk) Vm
c 3x10°/n/5
Equation for magnetic field along z-axis can be
ouu
Y
ad

written as
As E is along y-direciion and wave is travelling
dY

B = B^ = Bq sin CO (/ - xk)
along .Y-direction, therefore, B is along z-direction, = (1-0 X 10"^) sin 3-77 x 10** (/ - x/C)
innd
Re

Tesla
U., B =2-1 X 10“* k tesla.
Fi

Example Light with an energy flux of


F

Example Q Electromagnetic waves travel


in a medium at a speed of 2*2 x 10® ms"*. The 18 watt/cm^ falls on a non-reflecting surface at
normal incidence. If the surface has an area of
relative permeability of the medium is 1*0. Find
the relative permittivity of the medium. 20 cm^, find the average force exerted on the
surface during a 30 minute time span, when no
Solution. Here, t) = 2-2 x 10® ms-*, = 1-0 ;
incident light is reflected. How will your result be
modified if the surface is a perfect reflector ?
1 1 1 1 Solution. Total energy falling on the surface.
V -
U = 18 X 20 X 30 X 60 J = 6-48 x 10^ J
^ Vm* 0

r V^O^O
Total momentum delivered to the surface is
c
U 6-48 xlO*^ -1
P= — = 216 X 10"3 kg ms
c 3x10®
8/22
Fundamental Physics (XII)EEIHD
The average force exerted on the surface is
Solution. Here, v = 6 x lO'^* Hz, Eq = 2 V m"'
F = E.
P 216x10-3 {/) Average energy density of the electric field
= l*2xlO-^N
t 30x60 1
= -e E~ -
4 ^0 ^0 =“ -4 x(8-85x 10-‘3)x23
u

If the surface is a perfect reOecior. the change E ^

of momentum will hc= p-(-p) = 8-85 X 10-*2 J m-3


= 2/7 = 2 X 2-16 X lO'-^kg ms"’

w
(fV) Average energy density of magnetic field
Now average force.
2x2-16x10-3 u
B
= 2-4 X 10“* N 0 0
30x60

e
Exam
p'® [E Calculate the peak values of

re
row
2-
= - X
electric and magnetic fields produced by the 4 (4t:x10-'^)x(3x10^)“
radiation coming from a 100 watt bulb at a

eeF
distance of 3 m. Assume that the efliciency of the = 8-85 X 10-*2 j m-3

ullo
FF
bulb is 2‘5% and it is a point source ? Exam
P.e[0 A laser beam has intensity
(NCERT Solved Example)
3*0 X 10^^ W m”^. Find the amplitudes of electric
Solution. Useful Intensity,

srr
and magnetic fields in the beam.

roF
Solution. Here, / = 3-0 x 10^^* Wm~'^,

k
/ = power _ l00x(2-5/100) _ 2-5 Wm-2
uor
ai-ea
4t:(3)2 36 7t

Half of this intensity (/) belongs to electric field


ofof Eo = ?,
Intensity of the plane electromagentic
B^ = ?
wave IS
Y
kos
and half of that to magnetic field. Therefore, 1
Yo
! = u c = -S(,£2c
B oo

av

, I
rY
eerB

2 4^^ 21 2x3x 10*4


E
u

or ,E0
21 £« C
0 Y(8-85x]0-'“)x(3x10*^)
ou
d
o
ad

^0 = 4*75 X 10* V m
-!
nY

^■f,0 4-75x10*
2 X (2-5/36 7C) B, = 1-58 T
nid

3x10*
Re

I
F

x(3x10*)
1 4tex9x10^ pie m A beam of light travelling
Fi

Exam

along x-axis is described by the magnetic field,


= 4 08 Vm"^
=5 X T sin co (/ - x/c)
B gp _ 4-08 Calculate the maximum electric and
0 “ = 1-36 X 10“* T
c 3x10* magnetic forces on a charge , i.e. alpha particle
moving along y-axis with a speed of 3x10"^ m/s,
Exam
pie [Qa plane electromagnetic wave charge on electron = 16 x 10"*^ C
in the visible region is moving along /-direction. Solution. Here, Maximum magnetic field,
The frequency of the wave is 6 x 10*4 Hz, and the B,) = 5 X 10-9 -p .
electric field at any point is varying sinusoidally
with time with an amplitude of 2 V m“*. Calculate charge on alpha particle, q = + 2 e
(i) average energy density of the electric field and = 2x i-6x 10-'9=3.2x i0-‘9c,
(ii) average energy density of the magnetic field. 17 = 3 X lo’ ms
-1
ELECTROMAGNETIC WAVES 8/23

Maximum electric field, Temperature range is 7*73 x 10^ K to


4-83x lO^^K
Eq = cfi() = (3 X 10^) X (5 X 10“'^)
= 150 Vm-^ Example
m A parallel plate capacitor
Maximum force on alpha particle due to electric made of circular plates each of radius 10 cm has a
field = ^ X E(, capacity 200 pF. The capacitor is connected to a
= (3-2x 10-*'^)x 150 230 V a.c. supply with an angular frequency of
= 4-80x10“^’N 400 rad s“^

Force on alpha particle due to magnetic field (i) What is the rms value of the conduction
current ?
= clvB^y

ooww
= (3-2 X IQ-*^) X (3 X lO"^) X (5 X lO"^) (/i) Find the amplitude of B at a point 2*0 cm
= 4-80 X 10-^^N from the axis of the plates.
Solution. Here, /? = 10 cm = 0-10 m ;

e
TYPE 111. TYPICAL EXAMPLES
-12

ere
C = 200pF=200x 10 F.

rFl
Fre
CO = 400 rad
Example Obtain the temperature

rrF
ranges for ultraviolet part of radiation of e.m. Vnns = 230V.
waves. Use the formulae T = 2*9 x 10“^ mK.

sffoo
ouur
Take frequency of ultraviolet part of radiations rms _
V
rms

as 8 X 10“* Hz to 5 X 10‘^ Hz.


(0 I rms = Vnils X coC
X l/coC
kosk
c
Solution. The corresponding wavelength to the
= 230 X 400 X (200 X 10“*^)
Yo
frequency 8 x Hz is
oo
Y
= 184 X 10^ A
BB

c 3x10^
X,= — = 3-75 X lO-"^ m
I
8x10'** = 18-4 pA
rre

V
1

(//) Magnetic field at a distance r from the axis


ouu

The corresponding wavelength to the frequency


Y

of plates is
ad

5 X lO'^ is
dY

3x10^ 1^0 r
c B = I [See Solved Example 3]
Xj - = 6x 10"'^ 2k /?2
innd

m.

V, 5x10'2
Re
Fi
F

As, X,„T=2-9x 10-3 Amplitude of B is given by

2-9x10-3
or T = B.0 “
X
111
2k 0 27C R- nns

For, X] =3-75 X 10-2 m; [●-● I nns =v^i


2-9x10-3
^1 = 3-75x10-2 = 7-73 X 103 K Here, r = 2-0 cm = 2-0 x 10-^ m,R = 0-10 m

For, ^2 = 6 X 10
-10
m :
B
(471x10-2) X 2-0x10-2 x(18-4xl0-^)
0 “
2tc (0-10)2
2-9x10-3
^2 = 6x10“'^ = 4-83 X 10*^ K
= 1-04 X 10-‘* T
8/24
Fundamental Physics (Xn)CEIHD

Example [0 Suppose that the electric field part of an electromagnetic wave in vacuum is

E - 3*1 N/C cos [(1*8 rad/m) y + (5*4 x 10** rad/s) t] i


(a) What is the direction of motion ?
(b) What is the wavelength X ?
(c) What is the frequency v ?
(d) What is the amplitude of the magnetic field part of the wave ?
(e) Write an expression for the magnetic field part of the wave.
Solution, (a) From the given equation, it is clear that the direction of motion of e.m. wave is along
negative y direction i.e. along -j .

ww
(b) Comparing the given equation with the equation E = £qCos (ky + cor), we have
^ = 1-8 rad/m ; o> = 5-4 x 10^ rad/s ; £q = 3-1 N/C
2jc _ 2x(22/7)

Flo
1 = = 3-492 m = 3-5 m
k

ee
rere
to 5-4 X10**
= 85-9x10^ = 86 MHz

rFF
(c) v =
2tc 2x(22/7)
uurr
(cl) So =
£
0 _ 3-1

3x10'
= 1-03 x 10"^ T = 10-3 nT
foor
ks s
Yoo
(e) B = Bq cos (ky + m) k = (10-3 nT) cos [(1-8 rad/m) y + (5-4 x 10® rad/s) /] k
oook
eBB

/Mr /Ts.
uurr
ad

1. Conduction current. It is a current in the electric circuit which arises due to the flow of electrons in the
Yo

connecting wires of the circuit, in a definite closed path. When a capacitor is connected to the battery, it
dY

starts storing the charge, due to conduction current. When the capacitor gets fully charged, the conduction
current becomes zero in the circuit. Conduction current exists even if the flow of electrons is at uniform
Re
innd

rate.

2. Displacement current. It is that current which comes into play in the region, whenever the electric field and
FFi

hence the electric flux is changing with time. The displacement current is given by the relation.
40
=^0
di

where eg = Absolute permittivity of space, 40£/4r= Rate of change of electric fiux.


In case of a steady electric flux linked with a region, the displacement current is zero,

3. Ampere-Maxwell law. This law states that the line integral of magnetic field B over a closed path is
equal to jig times the sum of the conduction current (/) and displacement current (/£>), i.e..

f £.4/ =Pg(/H-/^)=Pp / + dt

The sum of conduction current and displacement current has the property of continuity along any closed
path although individually they may not be continuous.
ELECTROMAGNETIC WAVES 8/25

4. Maxwell’s equations are as follows :—

(I
(/) OE.ds = — (Gauss’s law in electrostatics)
s ^0

(ii) .()B.ds = 0 (Gauss’s law in magnetostatics)


5

-d —»

(Hi) U E.dl = B.ds (Faraday’s law of electromagnetic induction)

oww
r d t
(/v) y B.dl = |Jy / +)iy Sy — E.ds (AmpeFc-Maxwcl 1 law)

5. Electromagnetic waves. These are those waves in which there are sinusoidal variation of electric and

e
magnetic fields at right angles to each other as well as at right angles to the direction of wave propagation.

ree
rFl
Velocity of electromagnetic wave in free space is given by

Fre
rr F
1
= 3x10^ ms -1
ouur
sfoo
Examples of electromagnetic waves are radio waves, microwaves, infrared rays, light waves, ultraviolet
rays. X-rays and y-rays.
kks
Yo
oooo

The scientists associated with the study of electromagnetic waves are Hertz, Jagdish Chander Bose and
Marconi. Electromagnetic waves are of transverse nature and their speed in vacuum is the same as that of
eBB

light.
6. The amplitudes of electric and magnetic fields in free space, in electromagnetic waves are related by
urr

Ey = C^O-
ad
YYo

7. The electric vector is responsible for the optical effects of an electromagnetic wave and is called as “Light
vector.
dd

8. Production of electromagnetic waves. The oscillating or accelerated charge produces electromagnetic


Re
inn

waves. This concept was used by Hertz in his Hertz oscillator.


F

9. Electromagnetic spectrum. The orderly distribution of electromagnetic radiations according to their


wavelength or frequency is called the electromagnetic spectrum.

QUESTIONS

Based on NCERT Book

I. Multiple Choice Questions B. = 4x 10"^ sin (Ji X 10-^ .X + 3 Ji X 10^^ 0 T


1. Which colour of light has the shortest wavelength ? Calculate the frequency of electromagnetic wave.

(a) Violet (b) Blue


(a) 3 X 10^^ Hz
(b) 1-5 X 10“ Hz
(c) Green (d) Red
(c) I-Ox 10“ Hz
2. The magnetic field in a plane electromagnetic wave
is given by (d) 1-5 X 10'2 Hz
8/26 ^nadeefi, ^ Fundamental Physics (XII) VOL.II

3. In the above question, the direction of electric field 12. The dielectric constant of air is 1-006. The speed
variation in electromagnetic wave is : of electromagnetic wave travelling in air is a x 10^
(a) jc-axis ib) y axis ms"*, where a is about
(c) - X axis (a) 3 ib)2
id)-y axis
ic) 3-5 id) 2-5
4. The electromagnetic waves of all wavelengths
(a) travel with the speed of sound
13. An electromagnetic wave going through vacuum
is described by, E = Eq cos ikx-oat) ;
(Jb) travel with the same speed in all media
B=Bq cos ikx-oat), which of the following equation
(c) travel in free space with the speed of light is true ?

id) do not travel through a medium (a) EQ(0 = BQk ib) EQk = BQ(0
5. The speed of electromagnetic waves in a medium ic)EQBQ=0)k id) EQ = 0)kBQ

ww
is same
14. Photon energy in units of eV, electromagnetic
id) for all intensities ib) for all wavelengths wavesof wavelength2 cm is :
ic) for all frequencies id) in all media [use h = 6'6x 10"^ Js, c = 3 x 10* ms"*]

Flo
6. Electromagnetic wave travels in a medium which (a) 2 ib) 3-2 X 10"*9

e
has relative permittivity 2-2 and relative ic) 6-2 X 10"5 id) 0-62

reree
permeability 1-4. The speed of electromagnetic 15. Radiation of intensity 0-6 Wm“^ are striking a metal

r FF
wave is :
plate. The pressure on the plate is :
id) 1-7 X 10^ ms"* ib) 3-4 X 10* ms -1 (a) 2 X 10"* Nm"2 ib) 0-2 x 10"* NnT^
uurr
ic) 1-7 X 10* ms"* id) 3-4 X 10^ ms"‘
7. The velocity of an electromagnetic wave in free foor
ic) 2 X 10"*o Nm"2 id) 0-2 x 10"*** Nm"^
16. If the total energy of electromagnetic wave falling
ks s
space can be changed by changing on a surface is U, then the total momentum
Yoo
ooook

id) its amplitude ib) its frequency delivered (for complete absorption) is :
(c) its intensity id) none of these ia) Ulc ib) Ulc'^
eBB

8. What is the peak value of electric field vector for ic)Uc id) -Jm
sunlight striking the earth with an intensity 2-0 cal 17. A plane electromagnetic wave is incident on a
uurr

cm"^ min"* ?
material. If the wave delivers momentum p and
ad

(a) 102-6 Vm"* ib) 10-26 Vm"* energy E, then


Yo

ic) 205-2 Vm"* id) 204-0 Vm"* ia)pi^0;E = Q ib)p = 0,E:^0


dY

9. If a source of radiation of power 1 kW produces (c) p = 0, £ = 0 id)p^Q,E^0


Re
innd

10^ photons/second, the radiation emitted belongs 18. What is wavelength of signal weather frequency
to the region : [Use h = 6-6x 10"^ Js, c = 3 x 10* of 300 megahertz ?
FFi

ms"*] (a) 2 m ib) 20 m


ia) y-rays ib) X-rays ic) 10 m id) 1 m
ic) U-V rays id) microwaves 19. If Xy represents wavelength of X-Rays,
10. Which of the followings is the infrared wavelength ? microwaves and visible rays then
(a) 10-^ cm ib) 10"5 cm (i>)K>K>K
(c) 10-* cm id) lO""^ cm (c'>K>K>K (‘0K>K>K
20. Human body radiate
11. Which of the following relation is correct for the
ia) microwave ib) X-rays
propagation of em wave in space ?
(c) infrared rays (^0 gamma rays
id) Eq = Bq ib) ^0 ^0 ^0 “ ^0 21. EM waves can be produced by a charge :
ia) An accelerated charged particles
ic) Eq = ^\\.Q 6q Bq id) .JiI^Eq = Bq ib) A charged particles moving with constant speed
ELECTROMAGNETIC WAVES 8/27

(c) at rest
II. Assertion-Reason TVP^ Questions
id) either at rest or moving with constant velocity
22. In EM spectrum minimum wavelength is of: Direction. In each of the following questions two
statements are given, one labelled Assertion (A)
(a) gamma rays ih) radio waves
and the other labelled Reason (R). Select the
(c) visible rays (d) microwave correct answer to these questions from the codes
23. Properties of EM radiation are identified by using (fl), (/>), (c) and (rf) as given below :
there :
(a) Both, A and R are true and R is the correct
(a) colour ib) their use explanation of A.
(c) speed (h) Both A and R are true and R is not the correct
id) frequency or wavelength explanation of A.
24. Light wave constitutes : (c) A is true, but R is a false.

w
(a) mechanical waves (d) A is false, and R is also false.
ib) magnetic waves
29. Assertion. A charged particle moving in a circular

Flo
(c) electromagnetic waves orbit can not produce electromagnetic wave.
id) longitudinal waves

reeee
Reason. Electromagnetic wave can be produced
25. Which of the following transport by EM waves : by non-accelerated motion of charged particle.

FFr
(a) charges and momentum 30. Assertion. The frequencies of incident, reflected
ib) frequency and wavelength and refracted beam of monochromatic light

for
ur
incident from one medium to another are same.
(c) energy and momentum
id) wavelength and energy Reason. The incident, reflected and refracted rays
kkss
26. In an electromagnetic wave, the phase difference are coplanar.
Yo
between electric field and magnetic field is :
oo

31. Assertion. Electromagnetic waves are transverse


ia)0 ib) n/4 in nature.
eB

ic) 7t/2 id) n Reason. The electric and magnetic fields of an


27. In a plane electromagnetic wave, the electric field electromagnetic wave are perpendicular to each
r

oscillates sinusoidally at a frequency of 2-5 x 10


10
other and also perpendicular to the direction of
ou
ad

Hz and amplitude 480 V/m. The amplitude of the wave propagation.


YY

oscillating magnetic field will be ; 32. Assertion. Radiowaves can be polarised.


(a) 1-52X 10-^ Wb/m2
ndd

Reason. Sound waves in air are longitudinal in


Re

ib) 1-52X 10-^ Wb/m^ nature.


Fi

ic) 1-6 X 10-^ Wb/m2 33. Assertion. Dipole oscillations produce


id) l-6x 10-^ Wb/m“ electromagnetic waves.
28. Which of the following statements is not true about Reason. Accelerated charge produces
the properties of electromagnetic waves, electromagnetic waves.
(rt) These waves do not require any material 34. Assertion. X-ray astronomy is possible only from
medium for their propagation satellite orbiting the eanh.
ib) Both electric and magnetic field vectors attain
Reason. Efficiency of X-rays telescope is lai’ge as
the maxima and minimum at the same time.
compared to any other telescope on earth.
ic) The energy of electromagnetic wave is divided
equally between electric and magnetic fields
35. Assertion. X-rays can be deflected by electric field.
id) Both electric and magnetic field vectors are Reason. X-rays are electromagnetic waves. Its
parallel to each other. electric field vector will be affected by electric
field.
(CBSE Sample Paper 2022-23)
8/28 Fundamental Physics fXinwsm
A A A A
36. Assertion. Light with energy flux of 18 W/cm- by (i+2j) and the direction of
falls on a non-reflecting surface at normal propagation of electromagnetic wave is negative
incidence The pressure exerted on the surface is ^-direction.
6 X 10-^ N/m^.
Reason. The propa^ ilion of e.m. wave is along
Reason. Radiation pressure = energy flux
X velocity of light the direction of E x B.

37. Assertion. A changing electric field gives rise to


42. Assertion. The ratio of amplitude of magnetic field
magnetic field.
to the amplitude of electric field for an em wave
Reason. Electromagnetic wave is the sinusoidal

oww
propagating in vaccum is equal to speed of light in
variation in both electric and magnetic field vectors vacuum.

with the same frequency,


Reason. Bq=c £q, where c is the velocity of light.
38. Assertion. The frequency of the electromagnetic
wave remains constant when electromagnetic wave 43. Assertion. Blue coloured light is more energetic

e
passes from one medium to another. than green coloured light.

re
Reason. The velocity of the electromagnetic wave Reason. The wavelength of blue coloured light is

FFrllo
does not change when it passes from one medium more than that of green coloured light.

reF
to another.
44. Assertion. Ultraviolet part of electromagnetic

e
39. Assertion. 10 cm is a wavelength corresponding spectrum has temperature range 7-3 x 10^ K to 2-9
ouru
to the spectrum of infrared rays. X 10^ K

osrF
Reason. Infrared rays have wavelength range Reason. The frequency range of ultraviolet part of
O-I m to 1 mm. radiation is 8 x 10*'^ Hz to 3 x lO'^ Hz.

ffor
k
40. Assertion, y-rays are more energetic than X-rays. 45. Assertion. An oscillating circuit can produce
kso
Reason. Wavelength of y-rays is less than that of radiowavesof very high frequency which is more
ooo
X-rays. than infrared range frequency.
Yo
Y
BB

41. Assertion. In an electromagnetic wave, electric Reason. Infrared rays frequency is less than that
field vector and magnetic field vectors are given of radiowaves.
r ree
Y
uu

ANSWERS
ad

I. Multiple Choice Questions


doo
Y

l.{«) 2. {b) 3. id) 4. (c) 5. ia) 6. (c) 7. (d) 8. ib) 9. (c) 10. (a)
nidn

11. ib) 12. (a) 13. (b) 14.ic) 15.(b) 16. ia) 17. (d) 18. id) 19. ib) 20. (c)
Re

21. ia) 22.(a) 23. id) 24. ic) 25.(c) 26.ia) 27.(c) 28. id)
F
Fi

II. Assertion-Reason Type Questions


29. id) 30. ib) 31. (a) 32. ib) 33. ia) 34. ic) 35. (d) 36.ic) 37. ib) 38. ic)
39. id) 40. (n) 41. ia) 42. id) 43. ic) 44. ib) 45. id)

HINTS/EXPLANATIONS For Difficult Questions

I. Multiple Choice Questions = 4 X 10 sin f;t X 10'^ jc -i- 3 tu x lO” t]T


with the relation
1. Out of the given options, violet light has the
shortest wavelength. 271
B. = B,] sin .v-i-27n'r T
2. Comparing the given magnetic field in a plane X
electromagnetic wave 2 7t V = 3 71 X lO” or v= 1-5 x lOi* Hz
ELECTROMAGNETIC WAVES 8/29

3. The propagation of given electromagnetic wave is 10. The wavelength of infrared region is 8 x 10“^ cm
along - X direction. The magnetic field variation to 3 X 10“^ cm. So minimum wavelength of infrared
of electromagnetic wave is along z-direction. As rays is 8 x 10“^ cm « 10“^ cm.
the direction of propagation of electromagnetic 11. Velocity of em wave in space is
A A

wave is given by (E x B), so the direction of


1 _ ^0
electricfield is along -y axis.
4. The em waves of all wavelengths travel in free
^^0 ^0 B.0

space with the same speed which is equal to speed


of light. or ^0 “ ^oV^*'0 ^0
5. The speed of em waves in a medium is same for all 12. The speed of the e.m. wave in vacuum is

ww
intensities but is different for different frequencies
or different wavelengths. 1
= 3 X 10^ ms -1
6, Here, e ^ = 2-2 and

Flo
Velocity of em wave in a medium is
Air acts almost as vacuum, therefore

e
1 c

ere
V =
3x 108 = ax 10*

FFr
or 0 = 3 (approximately)
3x10*
uurr = 1-7 X 10* ms-l
” Vl-4x2-2

orr
7. The velocity of em wave in free space is a constant
13.
^ = c = ^ or £o* = Bo<»
sfo
Brx
b
k

quantity (=3x10* ms“*), which is independent of


kks
amplitude, frequency and intensity of em wave. he (66 X10-34) X (3x10*)
Yo
oooo

14. E = —
8. Here, I = 2 0 cal cm"^ min“' X (2x10-2)
eBB

2x4-2
J m~2 = 9-9 X 10-24;
10^x60
urr

= 14 X 10r2 Wm-2 9-9x10-24


eV
ad

” 1-6x10-19
YYo

As. I = ^SgElc or £„ = pZ = 6-2 X 10-^ eV


dd
Re
inn

0.6
15. Radiation pressure, = -
c 3x10*
^ _ 2 X (14x10-2)
F

° ~ \ (8-85x10-12) X (3x10*) = 0*2 X 1(T* Nm”2


= 10-26 Vm-l
U
9. Here, p = 1 kW = 10^ W, n = 102® photons/s 16. Momentum of incident em wave, p. = —. Final
* c

. nhc momentum of em wave (for complete absorption)


As, p = —
Py=0
Momentum delivered = Change in momentum of
or X =
nhc 1020 X (66 X10-34) X (3x10*)
u U
1()3
P em wave
= Pi-Pf = j-° — c
= 1-98 X 10-* m
17. An electromagnetic wave has both momentum and
This wavelength radiation belongs to ultraviolet
energy.
rays.
8/30 T^'todeefi, ’4, Fundamental Physics (XII) IV.OL.II
38. Here Assertion is true but Reason is false as
3x10^
18. X = - = 1 m velocity of electromagnetic wave is different in
V 300X106 different media.

480 39. Both Assertion and Reason are false as, 10 cm


27. = = 1-6 X 10-^ Wb/m2 is a wavelength corresponding to the spectrum of
c 3x10^
microwave which have wavelength range 01 m to
28. Knowledge based questions. 1 mm.

40. Both Assertion and Reason are true and Reason


II. Assertion-Reason Type Questions
is the correct explanation of Assertion, because.
29. Here, both Assertion and Reason are false, as
the electromagnetic waves can be produced by he 1
E = i.e., £o=-
accelerated motion of charged particle. X X

ww
30. Both Assertion and Reason are true, but Reason
A A A A
is not the correct explanation of Assertion. 41. As, ExB = (i+2j)x{2i-j)
31. Both Assertion and Reason are true and Reason

Flo
_ A A A A A A A A
is the correct explanation of Assertion, because = 2(ix i)-iixj) + 4(j xi) - 2(j x j)

e
in the propagation of electromagnetic waves, the

rere
electric and magnetic field vectors show variation = 2i0)-k+4{-k)-2(0)

r FF
perpendicular to each other as well as perpendicular
to the direction of wave propagation. A

= -5k
uurr
32. Both Assertion and Reason are true but Reason
can not explain the Assertion, since radiowaves foor
which is negative z-direction.
kss
are transverse waves and can be polarised. Thus both Assertion and Reason are true and
Yoo
33. Both Assertion and Reason are true and Reason
ooook

Reason is the correct explanation of Assertion.


is the correct explanation of Assertion because
42. Both Assertion and Reason are false
eBB

dipole oscillations are accelerated charges.


34. Here, Assertion is true but Reason is false as
as
Eq = cBq
earth’s atmosphere absorbs X-rays and hence
uurr

or E(^Bq = c
ad

X-ray astronomy is possible only from satellite


43. Wavelength of blue coloured light = (4-5 to 5 0) x
Yo

orbiting the earth at a height 36000 km, where the


earth’s atmosphere is quite thin. On earth. X-rays
10"^ m, wavelength of green coloured light
dY

telescope can not function well. = (5-0 to 5-7) X 10“^ m, i.e., Xf^ < X^
Re
innd

35. X-rays are the streams of high energy photons he 1


which are neutral and can not be deflected by .*. Energy of light, E = — i.e., E oc
Fi

X X
electric field. Thus both Assertion-Reason are
false.

8
energy flux > 1 or Efj>E8
36. Radiation pressure = . Thus E
8 h
velocity of light
Reason is false but Assertion is true. Now Therefore, Assertion is true but Reason is false.

ISxlO^W/m^ 44. Both Assertion and Reason are true but Reason
Radiation pressure = is not the true explanation of Assertion. For that
(3x10*) ms *
Wien’s displacement law is to be used.
= 6 X 10^ Nm-2
45. Both Assertion and Reason are false as frequency
37. Both Assertion and Reason are true but Reason of infrared rays is greater than the frequency of
can not give correct explanation of Assertion. radio waves.
ELECTROMAGNETIC WAVES 8/31

iS/VS

I. Displacement current 4. The four Maxwell’s equations and the


and Maxwell’s equations Lorentz force law (which together constitute
the foundations of all the classical
of electromagnetic waves
electromagnetism) are listed below :
1. Induced electric field due to changing
magnetic flux are more readily observed than (i) y E.ds = ql€^^ (ii) ^ B.ds={)
induced magnetic field due to changing
d
electric field- Why ? (iii) () E.dl =- B .(Is
'> (It ●'
Sol. The changing electric field produces
displacement current, which is very small and C ^ d e

w
hence the magnetic field set up by it is also (iv) () 5.J/ = u., / + u„ — B .ds
small, the same cannot be observed easily. In —) —>

an a.c. circuit displacement current can be Lorentz force law : F =q(E+ vxB)

Flo
increased by increasing the angular frequency
Answer the following questions regarding

e
of current. This would increase the induced
these equations :

ree
electric field. On the other hand, the induced
electric field due to changing inangetic llux can (a) Give the name (s) a.ssociated with some of

Fr
the four equations above,

rF
be increased by taking more number of turns of
the coil. The induced e.m.f. in different turns of (b) Which equations above contain source of
uurr
the same coil are added up, resulting in induced

for
^

electric field which is easily observed. E and B and which do not ? What do the

2. A capacitor of capacitance C, is being equations reduce to in a source-free region ?


s
kks
charged up by connecting it across a d.c. (c) Write down Maxwell’s equations for
Yo
voltage source of voltage V. How do the steady (i.e. time independent) electric and
oooo

conduction and displacement currents in this magnetic fields,


set up compare with each other (a) during
eB

(d) If magnetic monopoles existed, which of


the charging up process ? (A) after the the equations would be modified ? Suggest
capacitor gets fully charged ? (CBSE 2013) how they might be modified ?
ur

Sol. From the property of continuity, (e) Which of the four equations show that
ad

(a) Conduction current equals to displacement magnetic field lines cannot start from a point
YYo

currents. During charging up process, there nor end at a point ?


is a conduction current in connecting wires and (f) Which of the four equations show that
dd

equivalent di.splacement current in the region electrostatic field lines cannot form closed
Re

between the two plates of capacitor.


in

loops ?
(A) After the capacitor gets fully charged, the
F

(g) The equations listed above refer to


conduction current in the connecting wires
becomes zero. Now displacement current in a integrals of E and B over loops/surfaces.
region between the two plates is also zero due
to property of continuity. Can we writedown equations for £ and B
3. A varibale frequency a.c. source is connected for each point in space ?
to a capacitor. Will the displacement current (h) Are the equations listed above true for
increase or decrease with increase in different types of media : dielectrics,
frequency ? conductors, plasmas etc. ?
Sol. The increa.se in frequency of a.c., will decrease (i) Are the equations true for arbitrarily high
the reactance of the capacitor = 1/(2 n v C)] —> —>

and hence will increase the conduction current. and low values of E,B,q,I ?
Since the displacement current is equal to the Sol. (a) (i) Gauss’s law in electrostatics, (ii) Gauss’s
conduction current, therefore, the displacement law in magnetostatics, (iii) Faraday’s law of
current will increase with the increase in electromagnetic induction, (iv) Ampere’s
frequency of a.c. circuital law with Maxwell’.s modification.
8/32 4 Fundamental Physics (XII)

(b) Equations (/) and (iv) contain the sources q, macroscopically but large enough to contain a
I : equations (//) and {Hi) do not. To obtain very large number of atoms etc. The resulting
equations in source free region, simply put macroscopic Maxwell’s equations are of great
q = 0 and 1 = 0. Then practical use.
(0 Maxwell’s equations are the basic laws of
->

() E.ds =0 [From cqn. (/)]


classical electromagnetism. They are true in all
d media and for any value of £, 5, q. / etc. within
and () B.dl E.ds
the domain of validity of classical
[From eqn. (/v)] electromagnetism. The precise domain of
(c) Maxwell’s equations will be time
validity is hard to specify and need not concern
us here.
independent if the derivative of the physical
quantity involved with time is zero. Putting this II. Electromagnetic waves
concept on the right hand side of equations (Hi) and its characteristics

w
and (/V), we have Maxwell’s equations as :
5. Consider a plane wavefront of electro
O E.ds = () B.ds = 0 magnetic Helds travelling with a speed c in

Flo
^0 the right (say+Z) direction, it is given that
f —> —^
^ B.dl = flo /

e
() E.dl = 0 E and B are transverse to each other and

rree
(d) Equation (//) and (in) would be modified. uniform throughout the left of the wavefront

r FF
Equation (//) is based on the fact that monopoles and zero on the right of the wavefront. [This
do not exist. If the monopoles exist, the irght is contrived, but not incorrect, configuration
uurr
chosen for simplicity. In the usual
hand side would contain a term say q,„ repre
senting magnetic dipole strength, analogous to
Gauss’s law, in electrostatics, we would have
for
monochromatic plane wave, £ and B vary
—> —>
kss
sinusoidally in space and time], (a) Use
Faraday’s law to show that E == cB.
ooook

() B.ds =q X constant.
Yo

(b) Use Ampere’s law (with displacement


Further equation (/7/) would also be modified. current included) to show that
eB

An additional term /„ representing the current


due to flow of magnetic charge would have to c = l/^„
be included on the right hand side of equation
urr

—>

(Hi) analogous to the electric charge current of


ad

Sol. Let E be in the X-direction and B in the Y-


Yo

equation (iv), we would have direction.


dY

(a) Consinder the rectangular loop in the XZ


() E.dl = I X constant B .ds
ni
dt
plane with one side of length / parallel to E .
Re
innd

All this is of course, based on the expectation At the instant under consideration, the rectangle
of symmetry of form of the equations for £ and
Fi

is partially on left and partially on the right of


£. Nature may never show up monopoles or else the wavefront.
even if monopoles exist, the actual modification Rate of change of magnetic flux = Blc
of Maxwell’s equations might be very different.
—> j. —> —j
(e) Equation (ii) only The line integral of £ = () E.dl = El .From
(/) Equation (/) shows that the electrostatic field Faraday’s laws of electromagnetic induction
lines cannot form closed loops, as electrostatic El = Blc or £ = Be ...(/)
field lines cannot pass through conductors,
(b) Consider a similar rectangle in the IZ plane.
(g) Yes, we can. Maxwell’s equations can be
Rate of change of electric flux = E I c. The line
cast as differential equations valid at every point
in space and at every instant. integral of £ is £ /. From Ampere's law
ib) Maxwell’s equations are true in all media. or £ / = Po ^ 0
But in macroscopic media, it is usually
convenient to write down equations for average
or £ = Pq e Q Ec
or £ = PoSoc(£c) [from (/)]
of E and B over regions which are small
ELECTROMAGNETIC WAVES 8/33

We know £q = cBq or Bq = E^^^c


or c~ - or
c = V,j^^
Al.so 0

6. How are electromagnetic waves produced! by Putting values in (//), wc get


oscillating charges ? Why is it not possible to
produce e.m. waves in the visible region with
modern electronic circuits in the laboratory ? ll
li ~
jlEI_ 1
= «£
What is the method of production of x-rays ? 4Ro c- 4^0
Sol. When a charge q is oscillating with a frequency
V, it producesan oscillatingelectric field, which III. Electromagnetic spectrum
in turn produces oscillating magnetic field. This 8. Electromagneticwaves with wavelength

ww
oscillating magnetic field produces oscillating
electric field and so on. This simultaneous (/) Xr] are used to treat muscular strain
oscillations of electric and magnetic fields (h) ^2 2re used by a FM radio station for
produces electromagnetic wave of frequency v. broadcasting.

Flo
In fact electromagnetic waves are those waves (//{) are used to detect fracture in bones

e
in which there are sinusoidal variation of electric
(iv) X4 are absorbed by the ozone layer of the

ree
and magnetic field vectors at right angles to each atmosphere.

Fr
other as well as at right angles to the direction
Identify and name the part of electro

rF
of propagation of wave.
magnetic spectrum to which these radiations
uurr
In order to produce e.m. wave in the visible region belong. Arrange these wavelengths in
say yellow light of frequency 6 x 10*^* Hz. For
this we have to set up an oscillatory circuit of
frequency 6 x lO'^ Hz. The modem electronic
s for
decreasing order of magnitude.
Sol. (/) Xf belongs to Infrared radiations
kks
(//) X, belongs to VHF radiowaves
Yo
circuit widi latest techniques can hardly produce
oooo

oscillations of frequency lO’ * Hz. That is why e.m. (Hi) X3 belongs to X-rays
wave in the visible region cannot be produced with (iv) X4 belongs to ultraviolet rays.
eB

modem electronic circuits. The arrangement of wavelengths in decreasing


X-rays are produced, when fastly moving order of magnitude are X> > X[ > X4 > X3.
ur

electrons are suddenly stopped by the metal 9. Find the wavelength of electromagnetic
ad

target of high atomic number. waves of frequency 6 x 10*^ Hz in free space.


YYo

7. Show that the average energy density of the Give its two applications.
3x10^
dd

electric field E equals the average energy Sol. X = - = = 5x10"^


Re

m
6x10'-
in

density of the magnetic field B , in


F

electromagnetic waves. (CBSE 2019) These waves belong to infrared rays, which
are used
Sol. Average energy density of electiic field is given
by. (/) in green houses to keep the plants warm
(ii) in revealing the secret writings on the ancient
( p \2 I walls.
= -!-€ £2
1 ^0
U = — e
E 2^ 4 ^0 ^0 10. What causes nuclear winter ?

Sol. During nuclear war, thick clouds are produced


...(/) which cover most past of the sky. Due to it, the
Average energy density of magnetic field is given sun light is prevented from reaching that part
by. of the earth where nuclear explosion has taken
place. This would cause a “nuclear winter.”
1 1 B^0 )- 1 ^0 II. Why are infrared radiations referred to as
u
B ~
£2 ...(«)
heat waves also ? Name the radiations which
2p 0
rms
4H 0
are next to these radiation in electromagnetic
8/34 “Ptadee/t ^ Fundamental Physics (XII) LV*3WII
spectrum having (i) shorter wavelength energy and temperature of the material. Due to
(ii) longer wavelength. (CBSE 2018, 2013) it, the material gets healed. That is why infrared
radiations are often referred to as heat waves.
Sol. Infrared radiations falling on material are easily
absorbed by the atoms, molecules present in (/) The radiations, next to infrared radiations,
them. As a result of it, the thermal motion of having shorter wavelength will be visible light.
the atoms, molecules present in material (ii) The radiations next to infrared radiations,
increases which in turn increases the internal
having longer wavelength will be microwaves.

● Very Short Answer


● Short Answer
● Long Answer

w
VERY SHORT ANSWER QUESTIONS Carrying 1 mark

1. Displacement current Displacement current,

Flo
and MaxwelPs equations diq/G^) = 0
of electromagnetic waves “^0 "^0

ee
dt dt

Fr
1. Give difference between displacement current Let C be the capacity of capacitor and V be the
and conduction current. pot. diff. between two plates of capacitor when
capacitor is fully charged, then q = CV

for
Ans. Conduction current is due to flow of elections
ur
in the circuit. It exists even if the flow of diCV) dV
Conduction current, / - = C = 0
electrons is at uniform rate.
di di
ks
Displacement current is due to time varying
Yo
as Vis constant, when capacitor is fully charged.
oo

electric field. It does not exist under steady


condition. 4. Write down Maxwell’s equations for steady
eB

electric field.

2. Why is the quantity € 0 called the


dt Ans. (/) oE.d.s'=— (//) o E.dl =0 .
ur

^0
displacement current ? Where d^^/dt is the 5
ad
Yo

rate of change of electric flux linked with a 5. If you find closed loops of in a region in
region or space. space, does it necessarily mean that actual
charges are Rowing across the area bounded
nd

dt?
Re

r*

Ans. The quantity e 0 — has the dimensions of by the loops ?


di
Fi

current and this curreni exists in a region Ans. Not necessarily. A di.splacemeni current (such
between the two plates of capacitor when as that between the plates of a charging
displacement of charges occurs there, i.e., capacitor) can also produce loops of B ■
during charging or discharging of capacitor.
6. A plane electromagnetic wave travels in
3. A capacitor has been charged by a d.c. source. vacuum along z-direction. What can you say
What are the magnitudes of conduction and about the direction of electric and magnetic
displacement currents, when it is fully field vectors ? (CBSE 2011)
charged? (CBSE 2013)
Ans. In e.m. wave the electric and magnetic field
Ans. When capacitor is fully charged, let q be the vectors are perpendicular to each other and also
charge on the plate of capacitor, which is perpendicular to the direction of propagation of
constant.
e.m. wave. As plane e.m. wave is propagating
Electric flux through the plates of capacitor, along z-axis, .so the electric field vector is along
-v-axis and magnetic field vector is along y-axis.
<ti£; = <?/Sq= a constant
ELECTROMAGNETIC WAVES 8/35

7. Write the formula for the velocity of light in is the phase relationship between these
a material medium of relative permittivity e ^ fields ?

and relative magnetic permeability |i^ Ans. The oscillations in electric and magnetic fields
(CBSE Sample Paper 2013) of an electromagnetic wave are in same phase.
1 c 15. An em wave exerts pressure on the surface
Ans. V =
on which it is incident. Justify. (CBSE 2014)
Ans. Since electromagnetic waves carry energy and
momentum, therefore, they exert pressure on the
surface on which they arc incident.
'^0 ^0 16. Find the energy stored in a 90 cm length of a
8. What are the basic sources of electromagnetic laser beam operating at 10 mW.
waves ? (Raj. Board 2012) Ans. Time taken by laser beam to move through a
Ans. The basic source of electromagnetic waves is distance 90 cm is.

w
the time varying electric field produces magnetic 90 90 cm
field and vice versa. = 3xl0"^s
c 3x10*^ cm/s
9. Write the relation for the speed of electro

Flo
magnetic wave in terms of the amplitudes of The energy contained in 90 cm length of laser
beam is
electric and magnetic fields. (CBSE 201?)

ee
Ans. Velocity of light c = U = Pr=(10w W)x(3x lO-'^i)
= (10x 10-^Js'')x(3x 10-^9)

Fr
10. Do electromagnetic waves carry energy and
momentum ? (CBSE 2017) = 30 X 10-*^,I
Ans. Yes, e.m. waves carry energy and momentum. 17. If the intensity of the incident radiowave of

for
ur
11. What do you understand by the statement, 1 watt/m^ is reflected by the surface, find the
Electromagnetic waves transport pressure exerted on the surface.
Ans. Pre.ssure exerted by reflected wave on the
ks
momentum ?” (CBSE 2018)
surface is.
Yo
Ans. We know that electromagnetic waves carry
oo

energy and momentum. The momentum of 2x1


= 6-67 X 10-^ N/m^
eB

electromagnetic waves can be transferred to the


surface on which it is incident. Hence electro
c 3x10*
magnetic waves transport momentum. 18. If the intensity of the incident radiowave is
r

2 watt/m^ is absorbed by the surface, find


ou
ad

II. Electromagnetic waves the pressure exerted on the surface.


and its characteristics
Ans. Pressure exerted by absorbed wave on the
Y

surface is.
12. How are electromagnetic waves produced by
nd

accelerating charges ? (CBSE 2019)


Re

2
Ans. When a charge is accelerated, it produces
vJ- = 6-67 X 10"*^ N/m^
c 3x10*
Fi

varying electric field, which in turn produces a


varying magnetic field. This simultaneous III. Electromagnetic spectrum
varying electric and magnetic fields produces
19. If the earth did not have atmo.sphere, would
electromagnetic waves.
13. What evidence is there to establish that sound
its average surface temperature be higher or
lower than what it is now ? Explain.
is not an electromagnetic wave in nature ?
(CBSE 2014)
Ans. Sound waves require material medium for their
propagation. It shows that the sound waves are Ans. If the earth did not have atmosphere, the average
mechanical waves in nature and not surface temperature of earth will be lower,
electromagnetic waves. because there will be no green house effect in
the absence of atmosphere.
Sound waves are longitudinal mechanical waves
whereas electromagnetic waves are transverse 20. Give the ratio of velocities of light rays of
waves in nature and can travel in vacuum. wavelength 4000 A and 8000 A in vacuum.
14. An electromagnetic wave consists of Ans. Ratio = 1 ; because the velocity of both the wave
oscillating electric and magnetic fields. What length in vacuum is same (= 3 x 10* ms“*).

4
8/36 4 Fundamental Physics (XII)EEIHn

21. How are radio waves produced ? 27. Name the electromagnetic radiation to which
{CBSE 2011) waves of wavelength in the range of 10"^ m
Ans. Radiowaves are the electromagnetic waves of belong. Give one use of this part of EM
frequency range from 5 x 10^ Hz to 3 x 10^ Hz. spectrum. (CBSE 2009)
These waves are produced by oscillating electric Ans. Microwaves ; These waves are used in Radar
circuits having an inductor and capacitor. system for aircraft navigation.
22. How are X-rays produced ?
28. Name the electromagnetic waves used for
Ans. When fastly moving electrons are stopped studying crystal structure of solids. What is
suddenly on a metal target of higher atomic its frequency range ? (CBSE 2009)

oww
number, then X-rays are produced. The X-rays
will also be produced when an electron jumps Ans. X-rays are used for studying crystal structure of
from higher orbits to a vacancy on the inner solids. Their frequency range is 3 x 10^^ Hz to
3 X 10-^ Hz.
complete orbits in an atom of the element.
23. How are microwaves produced ? 29. Which part of electromagnetic sectrum is

e
(CBSE 2011) absorbed from sunlight by ozone layer ?

re
Ans. Microwaves are produced due to oscillating (CBSE 2010)

FFrllo
currents in special vacuum tubes, namely Ans. Ultraviolet rays.

rF
klystrons and magnetrons.
30. Name the part of electromagnetic spectrum

ee
24. By which way, the X-rays and y-rays can be whose wavelength lies in the range of 10
-10

distinguished ?
ouru (Manipur Board 2012) m. Give its one use. (CBSE 2010)

sor rF
Ans. By the method of production and the energy they
Ans. The part of electromagnetic spectrum whose
possess.
wavelength lies in the range of 10"’® m belongs
25. Why are microwaves considered suitable for
radar systems used in aircraft navigation ?
(CBSE (D), 2016)
os kffo
to X-ray region. X-rays are used as a diagnostic
tool in medicine and in a treatment for certain
forms of cancer.
ook
Yo
Ans. Microwaves arc of smaller wavelengths than
Y
31. Name the electromagnetic radiations used for
radiowaves. They can pass through the
Bo

(a) water purification, and (b) eye surgery.


atmosphere without any deviation.
reeB

26. What is the time period of the light for which (CBSE 2018)
ouY

the eye is most sensitive ? Ans. (a) U.V. radiations {b) Infrared radiations
ur

Ans. Eye is most sensitive to the light of wavelength 32. Give one use of electromagnetic radiations
ad

k = 5600 A,
Yo

in nuclear disintegrations. [CBSE 2018 (C)]


d

1 k 5600x10"’® Ans. Gamma rays are used in nuclear disintegration :


Time period. T = -
3x10
nidn

V c
Gamma rays are used in treatment of cancer
= 1.87 X ir’5 s
Re

tumours.
F
Fi

SHORT ANSWER QUESTIONS Carrying 2 marks

L Displacement current value on the two sides of a plate of charged


capacitor. The inconsistency of Ampere’s
and Maxwell’s equations
circuital law was removed by Maxwell by
of electromagnetic waves predicting the presence of displacement current
1. A parallel plate capacitor is being charged in the region between the plates of capacitor,
by a time varying current. Explain briefly when the charge on capacitor is changing with
how Ampere’s circuital law is generalised to lime. Maxwell also predicted that the sum of
incorporate the effect due to the displacement conduction current and displacement current has
current. (CBSE 2011) the property of continuity.
Ans. While dealing with the charging of a parallel The generalised form of Ampere’s circuital law.
plate capacitor with varying current, it was modified by Maxwell states that
found that Ampere’s circuital law is not logically
() B.dl
consistent, because q has not same
= \i^{l^lo) = \XQ Z+Sq
\ dt
c
ELECTROMAGNETIC WAVES 8/37

2. How does Ampere-Maxwell law explain the become zero. That is why galvanometer shows
flow of current through a capacitor when it a momentary deflection at the time of charging
is being charged by a battery ? Write the or discharging. The expression to explain this
expression for the displacement current in —»

terms of the rate of change of electric flux. observation is : (h B .dl = [IqU + .


(CBSE 2017) 6. Give one example each to illustrate the
Ans. Ampere-Maxwell law explains the flow of situation where there is (i) displacement
current through a capacitor when it is being current but no conduction current and
charged by a battery, using the concept of (i7) only conduction current but no
displacement current. According to Ampere- displacement current. USE 2018(01
maxwell law. the line integral of magnetic field
Ans. When a capacitor is charged by connecting it
over a closed loop is [Xq times the sum of
across a d.c. voltage source through wires, then
conduction current (I) and displacement current
(![)). Displacement current arises due tochange (/) in the region between the plates of capacitor,

w
in magnetic flux in between the two plates of there is a change in the electric field and hence
capacitor which was not considered by Ampere. electric flux with time till the capacitor is fully
Expression for displacement current is charged but there Is no flow of charge in this

Flo
region. Therefore, in this region, there is
d<i?^

e
"^0 displacement current but no conduction current.

ree
dt
(ii) The conduction current arises due to flow

FFr
3. What is displacement current ? Explain its
cause. of electrons in the conductors. During charging,
a capacitor starts storing the charge due to flow
urr
Ans. Displacement current is that current which is

for
of electrons (charges) in the connecting wires
produced in a region, whenever the elecuic field
but there is no change in the electric field or
and hence the electric flux is changing with time.
electric flux in the connecting wires. Therefore,
kkss
The displacement current is given by,
there is a conduction current in the connecting
Yo
ooo

wires but no displacement current in the wires


where Sy = absolute permittivity of space, outside the capacitor.
eB

d^fddt = rate of change of electric flux. 7. A parallel plate capacitor with plate area A
4. Why did Maxwell introduce the concept of and plate separation d, is charged by a steady
current /. Let a plane surface of area A/3
displacementcurrent ? Explain.
ur

parallel to the plates and situated


ad

Ans. Maxwell found that Ampere circuital law is symmetrically between the plates. What is the
YYo

logically inconsistent. To make this law logically displacement current through this area ?
consistent. Maxwell introduced the concept of Ans. Let q be the charge on capacitor plates at any
displacement current, which is known as
d

instant t and o be the surface density of charge.


Re

Ampere Maxwell’s law as


in

Then electric field between the plates of

^ B.dl= |Xy(/+/^)
F

o
capacitor will be £ = —
e. A
where / is the conduction current and is the ^0 0

displacement current. Here, /^j = Electric flux through area A/3 will be
5. Why does galvanometer show a momentary _ A q A 1
deflection at the time of charging or ^ 3 Gn0 A 3 3g 0
discharging a capacitor ? Write the nece.ssary
expression to explain this observation ? The displacement current will be
(CHSE 2016)
/=e d =_fo_
^/[sGo J 3Gy dt 3
Ans. During charging or discharging of a capacitor, 0
dt
= ^0 37
increasing or decreasing current in a circuit with
lime flows due to conduction current in wire 8. We find closed loops of magnetic field £ in a
and displacement cuirent between the plates of region in .space, does it necessarily mean that
the capacitor. When capacitor gets fully charged actual charges are flowing across the area
both conduction and displacement current bounded by the loops ?
8/38 Fundamental Physics fXinCTOWTI

Ans. Not necessarily so. because a displacement other as well as at right angles to the direction
current (which appears in between the plates of of wave propagation.
capacitor during charging slate or discharging Electromagnetic waves are produced by the
state) can also produce loops of B. where oscillating charge in L-C circuit or by
actually there is no flow of charge. accelerated charge.
13. How are electromagnetic waves produced by
II. Electromagnetic waves
and its characteristics
oscillating charge ?
Draw a sketch of linearly polarised em waves
9. The electromagnetic waves are the radiations propagating in z-direction. Indicate the
of large range of wavelengths. What are their direction of oscillating electric and magnetic
velocities (i) in vacuum and (u) in a medium ? ncld.s. (CBSE (D), 2016)
Ans. The wavelength of electromagnetic waves Ans. An electric charge at rest has electric field
ranges from 6 x 10"’*^ m to 6 x 10^ m. These around it but no magnetic field. A moving charge

w
waves travel with the same velocity (= 3 x 10^ with a constant velocity produces both electric
ms“*) in vacuum but with different velocities field and magnetic field of constant values. But
in a medium. In fact, the velocity of an if a charge is moving with a finite acceleration

Flo
electromagnetic wave is less in a medium than or oscillating, both magnetic and electric field
vacuum and a medium provides different values will change in space with time. Due to it, e.m.
of refractive index to the electromagnetic waves

ee
waves are produced.
of different wavelengths. A linearly polarised em waves propagating in

Fr
10. Write the relation between the following : Z-direclion are shown in Fig. 8(Q).l.
(a) Direction of propagation and directions FIGURE 8(Q).1
of oscillation of the electric and magnetic field
for
ur
vectors in an electromagnetic wave.
{b) Velocity of electromagnetic wave in
ks
vacuum and the permeability and
Yo
permittivity of free space. [CBSE 2007 (C)]
oo

Ans. (fl) The direction of propagation of electro


eB

magnetic wave is given by the direction of cross

product of electric field vector E and magnetic Y


r

-» -» ●
ou

field vector B, i.e., direction of (£ x B) 14. For a plane electromagnetic wave, pro
ad

ib) Velocity of electromagnetic wave in vacuum pagating along the z-axis, write the two
YY

possible pair of its oscillating electric and


IS magnetic fields. How are the peak values of
nd
Re

these (oscillating) fields related to each


other ? (CBSE 2016 (C)]
11. How does a charge q oscillating at certain
Fi

frequency produce electromagnetic waves ? Ans.


Sketch a schematic diagram depicting
E = E^, j = Eq sin co(r - c/f) j
A A

electric and magnetic fields for an electro B = B^ i = sin (o(r - z/c) i


magnetic wave propagating along the E(^Bq = c = velocity of electromagentic wave.
x-direction. (CBSE 2009)
15. What does the cross product of electric field
Ans. Oscillating charge produces sinusoidal variation —> —>

of electric and magnetic fields, which inturn vector (E) and magnetic field vector (B)
produces electromagnetic wave.
U.C., Ex B ) indicate ?
For the schematic diagram of electromagnetic
wave. Refer to Fig. 8.4. Ans. The cross product (ExB) always gives the
12. What are electromagnetic waves ? How are direction of propagation of electromagnetic
they produced ? wave.

Ans. Electromagnetic waves are those waves in 16. What is intensity of electromagnetic wave ?
which there are sinusoidal variation of electric
Give its relation in terms of electric field E
and magnetic field vectors at right angles to each and magnetic field B. (Bihar Board 2011)
ELECTROMAGNETIC WAVES 8/39

15
Hz. Name
Ans. Intensity of electromagnetic wave is defined as wave ^2 ® frequency of 10
the energy crossing per second per unit area held these two types of waves and write one useful
perpendicular to the direction of propagation of application for each. (CBSE (C)]
electromagnetic waves. The intensity of Ans. F| is microwave : It is used in radar communi
electromagnetic wave at a point is cation. ^2 ultraviolet rays; It is used to kill
-ie genns in water purifier.
/ = ji flV„c' where u «v
2 “ » 2n„ 20. Identify the electromagnetic waves whose
wavelenths vary as : (a) 10
-12
m < X<10"*m
and c is the velocity of electromagnetic wave.
(b) 10"^ m< Xr< 10“* m. Write one use each.
i
(CBSE 2017)
/ = -€y
2 ^ E,TC=-
0
2 1*0 Ans. (a) X-rays ; used in surgery and detective
Here £(, and Bq are maximum values of electric department
field and magnetic field respectively.

ww
(h) Microwaves ; used in radar communication
17. State any four properties of electro and microwave oven.
magnetic waves. (CBSE 2007)
21. Identify the electromagnetic waves who.se
Ans. The electromagnetic waves consist of wide

Flo
wavelengths lie in the range
range of radiations, but all these radiations
have the following common properties. (a) 10"*^ m < >, < 10"^“* m {b) 10"^ m<'k<

e
10*^ m. Write one u.se of each. (CBSE 2017)

reree
1. They arc transverse in nature. 2. They do
not require any material medium for Ans. (a) Gammarays : It is used in treatment of

r FF
propagation. 3. They travel with the same cancer and tumor.
speed of 3 X 10^ m/s in vacuum. 4. They {b) Infrared rays: It is used in physical therapy,
uurr
consist of mutually transverse varying electric
and magnetic fields.
18. Hertz in his historical experiment, produced
foor
i.e., to treat muscular strain and in green house.s
to keep the plants warm.
ks s
stationary electromagnetic wave.s and mea 22. Gamma ray.s and radiowaves travel with the
Yoo
ooook

sured the distance between two successive same velocity in free space. Distinguish
nodes. Explain how this measurement between them in terms of their origin and the
eBB

enabled him to show that electromagnetic main application. (CBSE 2020)


waves travelled with the same speed as the
[CBSE 2007 (C)]
Ans. 1. Gamma iy) rays have nuclear origin and are
speed of light.
uurr

produced due to disintegration of radioactive


Ans. Hertz by measuring the distance {= X/2) between
ad

nuclei.
the two successive nodes in his experiment
Yo

determined the wavelength (X) of electro Radiowaves are produced by oscillating electric
dY

magnetic wave. The frequency (v) of this circuits having an inductor and capacitor.
electromagnetic wave was equal to that of the 2. The frequency range of y-rays is 3 x lO'^ Hz
Re
innd

oscillator given by, to 5 X 10-3 Hz.


1
The frequency range of Radiowaves is 5 x 10^ Hz
FFi

V =

2n^^LC to 3 X 10^ Hz.


Then the speed of electromagnetic wave was 3. are used in medicine (radio-therapy),
determined by the relation, industry (sterilization and disinfection) and the
1 nuclear industry.
V = vX =
The radiowaves are used in communication and
2TtVIc
broadcasting system.
It was then found that this value agrees with
23. Which constituent radiation of the electro
the speed of light. Therefore, it was eoncluded
that electromagnetic waves travel with the same magnetic spectrum is used (/) in radar («) to
speed as the speed of light. photograph internal part of a human body,
and (Hi) for taking photographs of the sky
III. Electromagnetic spectrum during night and foggy conditions ? Give one
19. An electromagnetic wave T,, has a wave reason for your answer in each case.
length 1 cm while another electromagnetic (CBSE 2004, ^19)
8/40
Fundamental Physics fXinPZSTTl
Ans. (/) Microwave are used in the operation of radar («7) A3 is used to improve visibility in runway
because microwaves have small wavelenths and during fog and mist conditions.
are not diffracted or bent by objects of normal Identify and name the part of the
dimensions coming in its path. Due to it. these electromagnetic spectrum to which these
waves can be sent in a particulardirection as a radiations belong. Also arrange these
beam signal. wavelengths in ascending order of their
(n)A:-rays are used to photograph internal parts magnitude. (CBSF. Sample Paper 2022-23)
of a human body because j:-rays are of quite Ans. A.,-microwave. ?i2-uUraviolet, A.3-infrarcd.
short wavelength. They can easily pass through Ascending order A,> < A3 < A. 1-
flash but can not pass through the bones. 27. Which waves are used in Radar system ? Give
(Hi) Infrared rays are used for taking their frequency range and mention the
photographs of the sky during night and foggy source of their production. (CBSE 2014)
conditions because these radiations are readily Ans. The electromagnetic waves, which are used in

w
absorbed by water molecules present in foggy Radar system are microwaves. Their frequency
conditions. After absorption of heat, the.se foggy range is I GHz to 300 GHz. They are produced
by special vacuum lubes, namely, klystrons,

Flo
particles are heated up and there by surrounding
magnetrons etc.
is heated. Due to it, the infrared rays can locate

e
reee
the object present there. 28. The following table gives the wavelength of
some constituents of the electromagnetic

FFr
24. Name the electromagnetic waves used for the
spectrum.
following and arrange them in increasing
order of their penetrating power : S. No. Wavelength Range

for
ur
(fl) Water purification 1. 1 mm to 700 nm
(b) remote sensing 2. 0*1 ni to 1 mm
kkss
3. 400 nm to 1 nm
(c) treatment of cancer.
Yo
4. < 10-'^
oo

nm

(CBSE Sample Paper 2011)


eB

Ans. (a) For water purification, ultraviolet rays are Select the wavelength range and name the
used. (associated) electromagentic waves that are
used in
(b) For remote sensing, microwaves are used
ur

(0 Radar system for Aircraft navigation


ad

(c) For treatment of cancer, y-rays are used.


YYo

(ii) Earth satellites to observe growth of crops


Order of penetrating power of these raditions
(CBSE Sample Paper 2013)
microwaves < ultraviolet rays < y-rays.
Ans. (0 The wavelength range used in Radar system
d
Re

25. Find the wavelength of electromagnetic for aircraft navigation is 0*1 m to 1 mm, named
in

waves frequency 4 x 10^ Hz in free space. microwaves.


F

Give its two applications.


(ii) The wavelength range used in Earth satellites
3x10* to observe growth of crops is 1 nim to 700 nm,
Ans. A = -
V 4x10^
= 0*075 m. This wavelength named Infrared.

29. Identify the part of the electromagnetic


belongs to microwave region. The microwaves spectrum used in (/) radar and (ii) eye
are used (1) in microwave ovens (ii) in radar surgery. Write their frequency range.
system for the navigation of aircraft.
(CBSE 2019)
26. Electromagnetic waves with wavelength Ans. (/) Microwaves : its frequency range is
(0 A, is suitable for radar systems used in 3x 10-‘^Hzto3x 10" Hz.
aircraft navigation.
(//) Infrared ray.s : its frequency range is
(ii) Aj is used to kill germs in water purifiers. 3x 10“ Hz to 4 X 10'^ Hz
ELECTROMAGNETIC WAVES 8/41

SHORT ANSWER QUESTIONS Carrying 3 marks

1. What is displacement current ? Show that 8. How are infrared waves produced ? What role
conduction current and displacement currents does infrared radiations play in (/) maintaining
individually are discontinuous, but their sum is the earth warmth and (//) physical therapy ?
continuous. [Arts. 8.2., 8.3] (CBSE 2015) [Art. 8.13(/>)]
2. Give the brief history of electromagnetic waves. 9. Name the constituent radiation of electro

[Art. 8.6] magnetic spectrum which


(a) is used in satellite communication
3. What do you understand by electromagnetic
(b) is used for studying crystal structure
wave. Show that electromagnetic wave is of
transverse wave. (HP Board 2011) [Art. 8.7] (c) is similar to the radiations emitted during
decay of radioactive nuclei
4. (o) How are electromagnetic wave produced ?

ww
Depict an electromagnetic wave propagating in (J) has its wavelength range 390 nm to 770 nm
(e) is absorbed from sunlight by ozone layer
z-direction with its magnetic field B oscillating (/) produces intense heating effect.

Floo
along X-direction. [Ans. (a) Microwaves (b) X-rays
(b) Write two characteristics of electromagnetic (c) Gamma rays id) visible light

e
waves. (CBSE 2020) (e) Ultraviolet radiation (/) Infrared rays]

eere
[(a) Art. 8.9 and Fig. 8.4, (/>) Art. 8.10] 10. (fl) Arrange the following electromagnetic

FFr
5. Draw a labelled diagram of Hertz’s experimental radiations in the ascending order of their
set-up to produce electromagnetic waves. frequencies ;

oorr
uur r
Explain the generation of electromagnetic waves X-rays, microwaves, gamma rays, radiowaves.
s ff
using this set up. [Art. 8.11] (b) Write two uses of any two of these radiations.
6. What do you understand by electromagnetic (CBSE 2022)
sk
YYoo
wave ? Give its four properties. [Ans. (a) Radiowaves < Microwaves < X-rays
ooko

(Jharkhand Board 2012 ; Chhatisgarh Board < Gamma rays (6) Refer to Art. 8.13]
eBB

2011; HP Board 2011) [Arts. 8.6 & 8.10] 11. What are the uses of X-ray, ultraviolet ray and
7. What is electromagnetic spectrum? Name the infrared ray ? (Jharkhand Board 2011)
[Art. 8,13]
uurr

important parts of the electromagnetic spectrum.


ad

(HP Board 2013) [Art. 8.12] 12. Write a note on microwave oven. [Art. 8.14]
Yo
dY

LONG ANSWER QUESTIONS Carrying 5 or more marks


Re
nind

1. Find a relation for the velocity of electro radiations. State the range of their frequencies,
wavelength and source of origin. [Art. 8.12]
FFi

magnetic wave and show that light is an electro


magnetic wave. [Art. 8.8] 3. State the uses of electromagnetic radiations.
2, Name the various known electromagnetic [Art. 8.13]

CASE-BASED VERY SHORT/SHORT QUESTIONS

CASE 1. Electromagnetic waves are those waves product of electric field vector (£■) and magnetic field
in which there are sinusoidal variation of electric and
vector {B) ii.e.,ExB) tells the direction ot
magnetic field vectors at right angles to each other as propagation of e.m. wave.
well as at right angles to the direction of wave Read the above paragraph carefully and answer
propagation. In e.m. waves, the electric field and the following very short and short answer
magnetic field vectors must vary sinusoidally with the questions :
same frequency and in phase with each other. 1. Electromagnetic waves can travel through vacuum
Electromagnetic wave is of transverse nature. The cross but sound waves can not. Why ?
8/42
'P%eid€ef.i’4^ Fundamental Physics (XII)EEH]]
2. Show with the help of diagram the variation of 6. Electromagnetic waves transport momentum.
electric field vector and magnetic field vector in Comment.
the propagationof e.m. waves. 7. An e.m. wave is propagating along z-axis. It has
3. What is the phase difference between electric field the amplitude of electric field 10 V/m. What is the
vector and magnetic field vector in e.m. wave ? amplitude of magnetic field ?
4. The magnetic field in a plane e.m. wave is given by 8. In the above question, find the average energy
density of magnetic field in the e.m. wave.
10“^ sin (0-5 x 10-j: + 1-5 x 10'° t) Tj-
CASE 3. When a plane electromagnetic wave of

w
What are the wavelength and frequency of this
wave ? light of intensity / falls on a non-reflecting surface, then
average force on the surface is
CASE 2- Electromagnetic waves are produced by
the accelerated or oscillating charge. The electro _ total change in nomentum of incident light

e
roow
total time taken
magnetic waves carry momentum and energy. The

re
energy carried by electromagnetic waves is equally If Eq, Bq are the amplitudes of electric and
divided between electric field and magnetic field magnetic field vectors of incident electromagnetic wave,
vectors. The total average energy density of electro

reF
uFFll
1
magnetic waves is then
f = -e^Eh=-

e
0
1 2 ^lu
u = — e E- =
av
2 0 ^0 2 It() A plane electromagnetic wave of light beam of

sFr
where, Eq, Bq are the maximum values of electric intensity 3 0 x lO’^^ watt/cm- falls on a non-reflecting

foro
surface of area 2 0 m^ at normal incidence for 30
field and magnetic field vectors respectively. € q,
fk
uor
are minutes.
the absolute permittivity and absolute permeability of
okso
the space. The magnitude of electric field vector (£q) is Read the above paragraph carefully and answer
related with the magnitude of magnetic field vector (Bq) the following very short and short answer
Y
Yo
questions :
by the relation, Eq = cBq. Intensity of the electro
oo
reeBB

9. What is the total momentum delivered to the


magnetic wave, / = energy density x speed of e.m. wave.
The electromagnetic waves are not deflected by electric surface, when no light beam is reflected.
uurY

and magnetic fields. 10. In the above question, what is the average force
exerted on the surface ?
Read the above paragraph carefully and answer
the following very short and short answer 11. In the above question, if the surface is perfectly
ad

questions : reflecting find the average force on the surface.


doo
nY

5. The electromagnetic waves are not deflected by 12. What is the amplitude of electric field vector in
electric and magnetic fields. Why ? the incident beam of light ?
nid
Re
F

ANSWERS
Fi

1. Electromagnetic waves are transverse waves. They


are associated with the sinusoidal variation of FIGURE 8(Q).2
electric and magnetic field vectors, perpendicular Yi.
ENVELOPE OF ELECTRIC
to each other as well as perpendicular to the INTENSITY VECTOR
direction of wave propagation, which is possible E
in vacuum also. That is why e.m. waves can travel B
in vacuum. But sound waves are mechanical waves
and are longitudinal in nature. They can travel 0
r
through material medium only and not through
vacuum.

B B
2. In diagram e.m. wave is propagating along .v-axis. Z
E
The electric field vector is varying along y-axis and ENVELOPE OF MAGNETIC
INDUCTION VECTOR
magnetic field vector along z-axis.
ELECTROMAGNETIC WAVES 8/43

3. There is no phase difference in the electric field As, average energy density due to magnetic field
vector and magnetic field vector in em wave as the = average energy density due to electric field, so
oscillations in electric field and magnetic field average energy density of e.m. wave due to
vectors in em wave are in the same phase. magnetic field
4. Comparing the given relation with the general
equation of e.m. wave as 1 1 gp 1 _ I ..(3-3x10-^)^
4\Iq 4^ (47CXI0-'')
= -X
2 2 p0
(2n
By = Bq sin x + lnvt , = 2-2 X 10-^” Jm"^
9. Here,/=30x 10“’W/cm“ = 3-0 x lO’'* W/m^
2k
we have; = 0-5 X 10- A = 2 0 m-. / = 30 X 60 s.
X
Total energy falling on the surface, U= lA t
2k Total momentum delivered to the surface
= 4 7C X 10 ^

ww
or X =
O'SxlO-
U lAi (30 X10*^) X 20 X (30x60)
= 4x3-14x 10-2 = 0-126 m P- —
c c 3x10^
10
and 2 71 v= 1-5 X 10 -1
= 3-6 X 10’ kg

Flo
ins

1-5x10“^
p _ 3-6x10’

e
= 2-39 X lO’ Hz 10. Force, F = — = 2 X 10* N

e
or V =
27t 30x60

reer
t

rFF
5. The em waves being uncharged can not be deflected 11. For a perfectly reflecting surface, the change in
by electric and magnetic fields. momentum of incident light will be
uur r
-1
= /7 - (-/?) = 2p = 2 X 3-6 X lO’ kg ms
6. We know that e.m. waves carry energy and
momentum. The momentum of em wave can be ffoor 2p _ 2x3-6x10’
sks
transferred to the surface on which it is incident. Average force, F = 30x60
i
YYoo
Hence e.m. waves transport momentum.
= 4x 10*N
ooko

E 10
= 3-3 X 10-^ T
eBB

7. Eq = cBq or S = -0 - 21
c 3x10^
12. 1 = U^^c = ^EqE^c or E^^ = 0
c

8. Total average energy density of e.m. wave


uurr

2x(30xl0‘-^)
ad

' = i e f2 ‘’~^(8-85xI0-'2)x(3x10^)
Yo

u = —

2 Mo 2 “ “ = 4-75 X 10* Vm"*


dY
Re
innd

CASE-BASED MCQs AND ASSERTION-REASON QUESTIONS


FFi

Suppose, the electric field vector of an em wave


CASE l.If E and B are the electric field vector
at an instant is given by
and magnetic field vector of an em wave, then the
direction of propagation of em wave is given by £ x B.
F =6-1 N/Csin [(3-6 rad/m) .r + (5-4 x 10* rad/s) r];

If an electric field vector of an electromagnetic


Based on the above paragraph, answer questions
no. 1 to 4 :
wave at an instant is given by
E = Eq sin (o)f ~ kx) 1. What is the direction of propagation of
Then magnetic field vector at that instant is given electromagnetic wave ?
by B = Bq sin (cor - Lx), where Bq = Fq/c (h) -t
(a) i
Here, co is the angular frequency of electro
magnetic wave, k is the propagation constant and c is (c) k id) -k
the velocity of light.
8/44
Fundamental Physics (Xll)ESISD
2. Wha*. is the direction of magnetic field vector of (a) ultraviolet region
the given electromagnetic wave ?
(h) X-ray region
(a) k ib) -k (r) microwave region
A (d) visible region
(d j Id) -j
6. Ultraviolet rays arc used
For Question No. 3 and 4, we have given two («) in photography to take the picture of objects
statementseach, one labelledas A.ssertion(A) and
other labelled as Reason (R). (h) in revealing the secret writings on the ancient
walls
Choose the correct option out of the four options
given below: (c) to preserve the food stuff

(«) Both A and R are true and R is the correct (d) in ail the above purposes

w
explanation of A. For Question No. 7 and 8, we have given two
(/;) Both A and R are true and R is not a correct statements each, one labelled as Assertion (A) and
explanation of A. other labelled as Reason (R).

Flo
(c) A is true, but R is false. Choose the correct option out of the four options
(d) A is false and R is also false.
given Ik*1ow ;

ee
(fj) Both A and R are true and R is the correct
3. Assertion. The wavelength of the given e.m. wave

Fr
is 1-75 m. explanation of A.

Reason. Propagation constant, k=2 nX. (h) Both A and R are true and R is not a correct

for
ur
explanation of A.
4. Assertion. Amplitude of the magnetic part of the
given electromagnetic wave is 203 n T. (c) A is true, but R is false.
ks
Reason. Bn0 = c En.
0
id) A is false and R is also false.
Yo
7. Assertion. The blue coloured light is more
oo

CASE 5. The different frequencies of electro


energetic than green coloured light.
magnetic radiations belong to different regions of
B

electromagnetic spectrum, starting from radiowaves to Reason. The wavelength of blue coloured light is
re

gamma rays. The different ranges of electromagnetic more than that of green coloured light.
waves are utilised for different purposes. 8. Assertion. An oscillating circuit can produce
ou
ad

Based on the above paragraph, answer questions radiowaves of very high frequency which are more
Y

no. 5 to 8 : than infra red range frequency.


5. The electromagnetic radiations of frequency range Reason. Infrared rays frequency is less than that
nd

3 X 10^ to 3 X 10** Hz belongs to


Re

of radiowaves.
Fi

ANSWERS

1. (b) 2. (b) 3. (c) 4. un 5. (c) 6. (c) 7. (c) 8. (d)

HINTS/EXPLANATIONS For Difficult Questions

1. The direction of motion of e.m. wave is along Comparing the equation


A

negative .v-direction i.e., along-/. £= 6' 1 N/C sin f(3-6 rad/m) x + (5-4 x 10** rad/s) /]
2. As wave is propagating along negative A:-axis, i.e.. with, E = Eq sin (£v + to/), we have
—> —>

(£ X B) is along negative x-axis and electric field k = 3-6 rad/m

vector is along positive y axis, therefore the 2rc 2x22/7


magnetic field vector is along negative z-axis i.e., Wavelength, X =
k 3-6
A

along ~k. = I-746= 1'75 m

3. Here Reason is false as, /t = 2 nfX. The Assertion is true.


ELECTROMAGNETIC WAVES 8/45

4. Here, Reason is false as 7. Wavelength of blue coloured light


6! = (4-5 to 5-0) X 10-^
Bq - - = 2-03 X 10"'“^ T
3x10^ Wavelength of green coloured light
= 20-3x l()-‘^T = 20-3 /jT
= (5 0 to 5-7) X lO-"^ m.

Thus Assertion is also false. he


Energy of light, ^ = — ^
S. The electromagnetic radiation.s of frequency range Ar A

3 X 10^ to 3 X 10*’ belongs to microwave region. As Xf, < Xg so £y, > Eg. Thus Assertion is true but
6. Ultraviolet rays are used to preserve the food stuff, Reason is false.

whereas visible light is used in photography to take 8. Both Assertion and Reason are false because
the picture of objects and infrared rays are used in frequency of infrared rays is greater tlian frequency
revealing the secret writings on the ancient of radiowaves and oscillating circuit can not
walls. produce ultra High frequency signals.

w
Flo
e
ree
TYPE 1. DISPLACEMENT of wavelength 300 pm.
CURRENT, AMPERE-MAXWELL’S LAW Given, h = 6-6 x 10“^'’ is.

FFr
1. A parallel plate capacitor has circular plates, each
[Ans. {/) 1-6 X 10-22 cal. («) 1-83 x 10"2‘^ Wh
uurr
(Hi) 4-125 X 10-^ eV]

orr
of radius 8 cm. It is being charged so that the
electric field between the gap of two plates irses 6. A magnetic field in a plane electromagnetic wave
sf
steadily at the rate of 10'2 Vm"’ s"’. Find the is given by, = 3 x 10“^ sin [(1-5 rad/m) x
+ (5 X 10^ rad/s) tj tesla.
kks
value of displacement current. [Ans. 1-78 A]
Yo
(a) What is the wavelength and frequency of the
oooo

2. A capacitor consi.sts of two circular plates each wave ? (b) Write down an expression for the
of radius 8-0 cm and separated by 3-0 mm. The electric field, {x is in metre and l is in second).
eB

capacitor is being charged by an external battery. [Ans. (a) 4-19 m ; 7-95 x 10^ Hz
The charging current is constant and is equal to (b) = 90 sin (1-5 x 5 x 10* t) V/m]
0-3 A. Calculate (/) capacitance of capacitor,
ur

7. An electric field in an electromagnetic wave is


(//) the rate of change of potential difference
ad

given by
YYo

across the plates of capacitor and {Hi) the 2k


displacement current. E = 200 sin {ct-x)NC-K
X
[Ans. (0 59-3 pF (i7) 5-06 x 10^ Vs"’ (Hi) 0-3 A]
dd

Find the energy contained in a cylinder of cross-


Re

3. A parallel plate capacitor of capacitance 0-2 jaF


in

section 20 cm2 and length 40 cm along the


is connected across an a.c. source of angular
[Ans. 1-42 X 10"*® J]
F

.t-axis.
frequency 500 rad ,v“’. The value of conduction
current is 2 mA. What Is the rnis value of the 8. A plane electromagnetic wave is propagating in
voltage from the source ? What is the the x'-direction has a wavelength 5-0 mm. The
displacement current across the capacitor plates ? electric field is in the y-direction and its
[Ans. 20 V,2/nA] maximum magnitude is 50 VnrK Write the
equation for the electric and magnetic fields as a
TYPE II. VELOCITY, AMPLITUDE function of x and i.
AND ENERGY DENSITY OF
ELECTROMAGNETIC WAVES [Ans. Ey - 50 sin 3-77 x 10^’ [i - x/c] Vm"’ ;
- 1-67 X 10“^ sin 3-77 x lO’’ (t-x/c) tesla]
4. A laser beam has intensity 4 0 X lO'^’Wm 2. Find 9. An observer is 1 -8 m from an isotropic point light
the amplitudes of electric and magnetic fields in source whose power is 250 W. Calculate the rms
the beam. (Ans. 5-5 X 10* Vm"’ ; 1-83 T] values of the electric and magnetic fields due to
the source at the position of the observer.
5. Find the photon energy in (/) calories (ii) watt-
hour (Hi) electron volt, for electromagnetic wave [Ans. 48-1 V/m ; 1-6 x 10“^ T]
8/46
Fundamental Physics fXinPPTWn

10. Jn a plane electroma^etic waye, the electric field


oscillates with amplitude 20 Vm“*. Find III. TYPICAL PROBLEMS
(a) energy density of electric field {b) energy
density of magnetic field.
13. A beam of light travelling along X-axis is
described by the electric field.
[Ans. (a) 8*85 x lO”*® J/m^
Ey = (600 Vm“’) sin co {/ - xlc)
(6) 8-85 X 10-‘® J/m-l] Calculate the maximum electric and magnetic
11. A plane eiecti-omagnetic wave propagating in the forces on a charge = 2 e, moving along v-axis
.r-direclion has a wavelength of 6-0 mm. The electric with a speed of 3 0 x 10^ ms”’,
field is in the y-direction and its maximum where e= 1-6 x 10"*^ C.
magnitude is 33 Vm”’. Write suitable equations for [Ans. 1-92 X 10”^^ N ; 1-92 x ir^"^ N]
the electric and magnetic fields as a function of 14. In a plane electromagnetic wave, the electric field
X and t.
varies with time having an amplitude 1 Vm”’.
[Ans. Ey = 33 sin % x lO” (/ - x!c)y The frequency of wave is 0-5 x lO'^ Hz. The

w
wave is propagating along Z-axis. What is the
Sj = 11 X 10”1* sin jc X lO” {t - jr/c)]
average energy density of (/) electric field (i7)
12. A plane electromagnetic wave is propagating in magnetic field {in) total (h’) what is the amplitude

Flo
the A-direction has a wavelength 5-5 mm. The of magnetic field ?
electric field is in the y-direction and its
[Ans. (i) 2-21 X 10-*2 J nr^, («) 2-21 x 10”*2 j

reeee
maximum magnitude is 36 Vm”’. Write suitable
m 4*42 X 10-12 J (/j,) 3.3 ^ iq-9 jj
equations for the electric and magnetic fields as

FFr
15. The average energy flux of sunlight is 1-0 kW
a function of .r and / and find average energy m”-. This energy of radiation is falling normally
density of e.m. wave.
on the metal plate surface of area 10 cm- which
[Ans. Ey = 36 sin 1-14 x lO” (^ ~ xk) V/m ;
for
completely absorbs the energy. How much force
ur
= 1-2 X KT’ sin 114 x lO” {t-xk) T is exerted on the plate if it is exposed to sunlight
and 5-7 x 10”’ Jni”^] for 10 minutes ? [Ans. 3-3 X lO”’^ N]
kkss
Yo
oo
eB

For Difficult Questions

1. Here, r=8cm=8x 10 ^m;


r

(«) Charge at any instant on the capacitor plate is.


ou
ad

dE
= 10’2 Vm”’ s”’
q^C Vand / = ^ =— (CV) = C
dV
YY

dt
dt dt di
Displacement cuiTent,
ndd
Re

d(^^ dV _ / 0-34
/
D “^0 =^0 4-(FA) or

dt C ” (59-3x10”’2)F
Fi

dt dt

dE dE = 5 06 X 10*^ Vs”‘
=e.
0
A =£r. nr-
0
dt dt (///) From the property of continuity.
= (8-85x 10”’2)x3-14x(8x 10”2)2 x IO'^ Displacement current = conduction current
= 1-78 A = 0-3 A

2. Here, r = 8 x ; (/ = 3 x lO”^ / = 0-3 A, 3. Here, C = 0-2 pF= 0-2 X IO”^F = 2x 10”^ F,


Plate area of capacitor, (D = 500 rad i”’, 7^,^^ = 2 mA = 2x 10”^ A
A = 7tr2 = 7c(8x I0”2)2 //r
2xl0”3
(0 Capacitance of capacitor, ^nns “ ● /
500x(2xl0“’^)
nns
oiC

= e.A _ (8-85xl0”'2)xTCx(8xl0”2)2
0
C = 20V
d SxlO”"-^
Displacement current = conduction cunent
= 59-3x 10”’2f = 59-3 pF = 2 mA
ELECTROMAGNETIC WAVES 8/47

4. Here, 7 = 4 0 x 10*^ Wm-2 Energy contained in cylinder is


Intensity of plane em wave is U = volume x energy density

/ = u_..
av
c=
1
=''4^0
Amplitude of electric field. = (8 X 1(H) X ^ X (8-85 X l(r‘2) X (200)2
= 1*42 X 10-1® j

i
2x4x10^^
8. Here, X = 5-0 mm = 5 x 10“^ m ; Eq = 50 Vm
-1
^0 =
\ (8-85 X10-12 )X (3X10®) Angular frequency, ® = 2 7C v
= 5*5 X 10® Vm-l 2jrc 2jtx3x10®
= 3-77 X 10” rad/s
Amplitude of magnetic field, A 5x10-3

ww
D _
E
0 _ 5-5x10® E
_ ^0 _ 50
^0 - — = 1-83T B0 “ = 1-67 X 10-1 j
e 3x10® c 3x10®

Flo
5. A. = 300 p m = 300 X 10-^ m = 3 X lO*^ m ; The equation for the electric field along y-axis
is Ey = Eq sin (a {t- xlc)

e
Ac _ 6-6x10-34 x3xl0®j = 50 sin 3-77 x lO” (/ - xlc) Vnr^

ree
X 300x10-6 The equation for the magnetic field along z-axis

Fr
rF
= 6-6 X 10-22 j s= Bq sin ® (/ - xlc)
= 1-67 X ir’ sin 3-77 x lO” {t-x!c) Tesla
uurr
6-6x10-22 6-6x10-22

6-6x10-22
4-2
cal =
60x60
watt hour
s for
9. Intensity of electromagnetic wave is,

7 = -^
47cr2
= u
av
c
kks
electron-volt.
Yo
1-6x10-1®
oooo

1
6. (a) Comparing the given relation with the where, « av
eB

equation of
P 1 ( E
'2n 2nt^ -e £2__p fo
B = Bq sin 47cr2 2 “
[x
x+
ur

T )
ad

= ^0 £2r.m.s. c
YYo

f2n
= sin x + 2nvt
0
p
or E
dd

r.m.s

we have, Bq = 3 x 10-i T ; 4nr- 6q c


Re
in

2ti
F

— = 11-5c or -V
X = = 419m 250
A 1-5
4 7C X (1-8)2 X 8-85 X 10-'2 x 3 x 10®
5x10® = 481 V/m
2 7C V = 5 X 10® or V = = 7-95x10’Hz
2ji
E 48-1
B r.m.s. _
= 1-6 X 10-’ T
(A) Eq = cBq = (3 X 10®) X (3 X 10-’) = 90 V/m r.m.s.
C 3x10®
^2tix , 2nt ^ 10. (a) Average energy density of electric field,
E^ = Eq sin
^ X T ) -
1
c c-2
or = 90 sin (1-5 jc -f 5 x 10® t) V/m. rms
2
-“^0
4
^0
7. Here, Eq = 200 NC~\ (b) Average energy density of magnetic field
A = 20 cm2 = 20 X 10^ m2 ; / = 0-40 m
Volume of cylinder, V=Al = (20 x 10~4) x 0-40
= 8 X 10-4 m2 2H„0 2(l„ 4^„ 4 “ »
8/48 Fundamental Physics (XII) VOL.II

11. Here, X, = 6-00 mm = 6 x 10“^ m ; „ ^0 600 ,


£q = 33 V/m,
- Inc 2jcx3x10« which is along Z-axis.
CO=27tV = ——
X 6x10-3 Thus maximum electric force,

F^= ^Fo = 2e£o= 2x 1-6 X 10-1^x600


= 7txl0’^rad/s. = 1-92 X 10-1<^N
The maximum magnetic field. Maximum magnetic force,
£ 33
£0 “ 0 _
= llxl0“^T = 2 X 1-6 X 10-‘9 X 3 X 10^ X 2 X 10-^
c 3x10®
= 1-92 X 10-17 N
The equation for the electric field, along y-axis
14. (/) Average energy density of electric field,

w
in the electromagnetic wave is
x\ 1 1 ( E.0 ^2 ( _ 0
= £,0 sin 0) t — :.E
2^0
u
£ “ 2 I -ir]
rots

Flo
rots
C

= 33sin7cxl0ii(t-x/c)

e
1

-4^0 ^0 - -x(8-854x10-'2)xi2
——e E^ —

ree
The equation for the magnetic field along z-axis 4

FFr
in the electromagnetic wave is = 2-21 X 10-1^ Jm-3
f (it) Average energy density of magnetic field,
= sin CO t —

for
ur
c
Ug = average energy density of electric field
= 11 X 10-® sin 7C X lOll (t - x/c) = 2*21 X 10-12 jm-3
kkss
12. Here, X = 5-5 mm = 5-5 x 10^3 m; Eq = 36 Vm-^ (Hi) Total average energy density,
Yo
U = Ug + Ug = 2Ug = 2 X 2'21 X 10“*2
oo

0)=27CV =
271C 2x314x(3xl0®) = 4-42 X 10-12 Jm-3
eB

X 5-5x10-3 (iv) Amplitude of magnetic field,


- 1-14 X 10*^ rad s-^ Bq = £(/c = 1/(3 X 10®) = 3-33 x ir’ T
ur

15. Here, energy flux, = 1-0 kW m-2


ad

^o_ 36 = 1-2x10-’T = 10xl03Wm-2;


YYo

«0 = c 3x10®
A = 10cm2= lOx 10*^m2= I(r3m2
The equation of electric field along y-axis will f = 10 min = 10 X 60 s = 600 s.
d
Re

be
in

Total energy falling on the plate,


E-Ey-E^ sin co (f - x/c)
F

U = energy flux x area x time


= 36 sin 1-14 x lOH (/ - x/c) Vm"!
= (1 X 103) X (10-3) X (600) J = 600 J
The equation of magnetic field along z-axis will Momentum of the radiation.
be

B = fij = £o sin CO (r - x/c) _U ^ 600 = 2x 10-^kgms *


c 3x10®
= 1*2 x ir7 sin 114 X 10^ (t - x/c) T
Since the plate absorbs the entire energy, so
Average energy density of electromagnetic wave
is momentum delivered to the plate = change in
momentum (A p) = 2 x lO"^ kg ms-*

ieo £2 ^ ^ X (8-85 x 10-*2) x (36)2


Force on the plate,

= 5*7 X 10-’ Jm-3


F=^ = 2X10-® = 3*3x10-’ N
t 60
ELECTROMAGNETIC WAVES 8/49

WITH
SOLUTIONS

w
Q. 1. Fig. 8(N).l shows a capacitor made of two circular plates each of FIGURE 6(N).1
radius 12 cm and separated by 5.0 mm. The capacitor is being
charged by an external source (not shown in the figure). The
charging current is constant and equal to 0.15 A.

e
(a) Calculate the capacitance and the rate of change of potential

row
re
difference between the plates,
(b) Obtain the displacement current across the plates,

FFllo
eeF
(c) Is Kirchhoff’s first rule valid at each plate of the capacitor ? Explain.
Given = 8.85 x 10"'^ N"* nr^
Sol. Here

u R = 12 cm = 0.12 m, = 5.0 mm = 5 x 10"^ m, /= 0.15 A, Eq = 8.85 x 10"'- N"' nr-

r
sFr
Area, A = = 3.14 X (0.12)2 m“

kro
(a) We know that capacity of a parallel plate capacitor is given by
uor
c =
£/. A
0
offo
8-85x10"‘2x3.14x(0-I2)2 = 80.1 X 10- 12
F=80.1 pF
5x10-2
kos
cl
Y
Yo
eerBB
oo

Now
dq dV dV
q = CV or — = C X— or I = C
dt
rY

dr dt dt

dV__l_ 0.15
u

or = 1.87 X 10’ Vs-*


dt C 80.1X10 -12
ou
ad
do

(b) Displacement current is equal to conduction current i.e. 0.15 A


nY

(c) Yes, Kirchhoff’s first rule is valid at each plate of the capacitorprovided we take the current to be the
sum of the conduction and displacement currents.
nid
Re

Q. 2. A parallel plate capacitor made of circular plates each of radius R = 6.0 cm. has a capacitance
F

C = 100 pF. The capacitor is connected to a 230 V a.c. supply with an angular frequency of 300 rad. s“^
Fi

(a) What is the r.m.s. value of the conduction current ?


(b) Is the conduction current equal to the displacement current ?
(c) Determine the magnitude of B at a point 3.0 cm from the axis between the plates.
Sol. Here, R = 6.0 cm, C= 100 pF = 100 x 10"^2 p
FIGURE 8(N).2
CO = 300 rad. s"*, = 230 V

£ £
A
nils rms
(a) I nns = £ xcoC
X I rms
c

coC \J
/
rms
= 230x 300x 100 X 10"*2
= 6.9 X 10-^ A
= 6.9 p A
8/50 Fundamental Physics (XII) VOL.II

(b) Yes, I = Ijy, whether / is a steady d.c. or a.c.

This is shown below ^D = (V (1)£ = £A)

or
~^0 ^ “^0 ^
d( Q fvE = ^ Q ^
dt
dt Gq A I ^0 ^0^

or
Ip=egAx Sn1 A dQ_dQ_j
dt dt
0

(c) We know, s =
2tir^ ^
1^0
The formula is valid even if is oscillating. As /^ = /, therefore B =

w
2nR^
If / = /q, the maximum value of current, then

Flo
rms
Amplitude of B = max. value of B =
2nR^ 2nR'^

eeee
471X10"'^ X0.03xV2x6.9x10"^
= 1.63 X 10-11T

Fr
2 X 3.14 X (0.06)2
Q. 3. What physical quantity is the same for X-rays of wavelength 10“1® m, red light of wavelength 6800 A

for
ur
and radiowaves of wavelength 500 m ?
Sol. The speed in vacuum is same for all the given wavelengths, which is 3 x 10^ ms“*
Q. 4. A plane electromagnetic wave travels in vacuum along Z-direction. What can you say about the
ks
direction of its electric and magnetic field vectors ? If the frequency of the wave is 30 MHz, what is
Yo
oo

its wavelength?
Sol. In electromagneticwave, the electric field vector E and magnetic field vector B show their variations
eB

perpendicular to the direction of propagation of wave as well as perpendicular to each other. As the
electromagnetic wave is travelling along Z-direction, hence E and B show their variation in x-y plane.
ur
ad

3x10*/n/s
Yo

Q
Wavelength, X = - = 10 m
V 30x10^5“^

Q. 5. A radio can tune to any station in the 7*5 MHz to 12 MHz band. What is the corresponding wavelength
d
Re
in

band ?

Sol. X., = 3 X 10*/(7-5 X 10^) = 40 m and ^2 = 3 X 10*/(12 x 10^) = 25 m


F

Thus wavelength band is 40 m to 25 m


Q. 6. A charged particle oscillates about its mean equillibrium position with a frequency of 10^ Hz. What
is the frequency of electromagnetic waves produced by the oscillator ?
Sol. The frequency of electromagnetic wave is the same as that of oscillating charged particle about its
equilibrium position ; which is 10^ Hz.
Q. 7. The amplitude of the magnetic field part of a harmonic electromagnetic wave in vacuum is
Bq = 510 nT. What is the amplitude of the electric field part of the wave ?
Sol. Here, Bq = 510 nT = 510 x 10“^ T , £„ = c Bq = 3 x 10* x 510 x 10-^ = 153 NC^
Q. 8. Suppose that the electric field amplitude of an electromagnetic wave is = 120 N/C and its frequency
is V = 50*0 MHz. (a) Determine Bq, Q), k and X. ib) Find expressions for £ and B ●

Sol. (a) B«0 =


^0 _ 120 = 4 X 10“2 T ; (0 = 2 7t V = 2 X 314 X (50 X 10^) = 3*14 x 10* rad/s
c 3x10*
ELECTROMAGNETIC WAVES 8/51

(0 344x10^ 3x10^
k=- = 1*05 rad/m ; X = - = 6*00 m
c 3x108 V 50x10^

N/C) sin [(1-05 rad/m) x - (3-14 x 10* rad/s) t] j


(b) Expression for E is, £ = Eq sin (fcc - cor) = (120
Expression for B is, B = Bq sin (kx - (Ot) = (4 x 10“^ T) sin [(1*05 rad/m) x - (3-14 x 10* rad/s) t] k
Q. 9. The terminology of different parts of the electromagnetic spectrum is given in the text Use the
formula E = hv (for energy of a quantum of radiation : photon) and obtain the photon energy in
units of eV for differentparts of the e.m. spectrum.In what way are the different scales of photon
energies that you obtain related to the sources of electromagnetic radiation.
hv
Sol. Energy of a photon of frequency v is given by ; £ = /iv joules = eV ;
1-6x10-*^
where h = 6-6 x 10"^'^ J. The energy of photon of different parts of electromagnetic spectrum in joules and

w
eV are shown in table below , along with their sources of origin.
Part of Mean frequency Energy of Photon Source of electromagnetic

Flo
Spectrum (V) radiation
m Goule) (eV)

ee
y-rays 3 X 1Q20 19-8 X 10-‘4 1-24 X 10^ Nuclear origin

Fr
X-rays 3 X 10'* 19-8 X 10"^^ 1-24 X lO'^ Sudden retardation of high
energy electron

for
ur
Ultraviolet 10'5 6-6 X 10-'9 4-125 Excitation of atom
Visible 6 X 10'^ 39-6 X 10-20 2-475 Excitation of valency electron
Infrared 10'2 6-6 X 10-2' 4-125 X 10-2 Excitation of atom and
ks
molecule
Yo
oo

Microwaves lO'O 6-6 X 10-24 4-125 X 10-5 Oscillating current in special


vacuum tube
eB

Radiowaves 3x 10* 19-8 X 10-20 1-24 X lO-O Oscillating current

Q. 10. In a plane electromagnetic wave, the electric Held oscillates sinusoidally at a frequency of
ur

2*0 X 10^0 Hz and amplitude 48 Vm”^. (a) What is the wavelength of the wave ? (b) What Is the
ad
Yo

amplitude of the oscillating magnetic field ? (c) Show that the total average energy density of the
electric field equals the average energy density of magnetic field, (c = 3 x 10* m/s)
Sol. Here, v = 2.0 x lO'® Hz ; £q = 48 Vm-', c = 3 x 10* ms-'
d
Re

c 3 X10*
in

(a) Wavelength of the wave, X. = - = ttt = 1.5 X10-2 m


® V 2.0 xlO"'
F

£ 48
0 _
(b) Amplitude of the oscillating magnetic field, 5^ = = 1.6 X10-2 T
c 3x10*
(c) For average energy density, see Art. 8.10, point 10.
8/52 'PxadeK^ 'a Fundamental Physics (XII)

iM
1 1
WITH ANSWERS,
HINTS AND SOLUTIONS

MULTIPLE CHOICE QUESTlONS-l


1. One requires lleV of energy to dissociate a distance is E. The electric field intensity
carbon monoxide molecule into carbon and produced by the radiations coming from 50 W
oxygen atoms. The minimum frequency of the bulb at the same distance is

ww
appropriate electromagnetic radiation to (a) E/2 {b)2E
achieve the dissociation lies in

(a) visible region {b) infrared region


(c) E/Vi {d) ViE

FF loo
(c) ultraviolet region {d) microwave region 5. If E and B represent electric and magnetic

ree
2. A linearly polarized electromagnetic wave field vectors of the electromagnetic wave, the
A

given as £ = Eg i cos {kz - co/) is incident direction of propagation of electromagnetic

reeF
wave is along
normallyon a perfectly reflectinginfinitewall -*
at z = a. Assuming that the material of the {a) E {b) B

oroFr
r ur
wall is optically inactive, the reflected wave -»

will be given as (c) BxE (^0 Ex B


s ff
—* /\
6. The ratio of contributions made by the electric
(a) =- Eq I cos {kz - cor)
k
field and magnetic field components to the
YYouo
—* A
koso

intensity of an EAf wave is


(b) E^ = Eq i cos {kz + cor)
(a) c : I {b)c^-: 1
BBoo

(c) E^ =-Eq i cos {kz + cor) (c) 1 : I (<^) Vc : 1


r ee

—* A

{d) = Eq i sin {kz - cor) 7. An EM wave radiates outwards from a dipole


antenna, with Eq as the amplitude of its
ad

3. Light with an energy flux of 20 W/cm" falls


ouur

electric field vector. The electric field Eq which


Yo

on a non-reflecting surface at normal inci


transports significant energy from the source
dence. If the surface has an area of 30 cm^ falls off as
the total momentum delivered (for complete
d
Re

absorption) during 30 minutes is


idnY

(«)
{a) 36 xl0“5 Kg m/s {h) 36 x 10^ Kg m/s r
FFin

(c) 108 x1Q4 Kg mJs{cf) 1-08 x Kg m/s


4. The electric field intensity produced by the (c) -1- {</) remains constant
radiations coming from 100 W bulb at a 3 m r

LTIPLE CHOICE QUESTIONS-ll

8. An electromognetic wave travels in vacuum (Z?) The associated magnetic field is given as
along z direction : 1 A A
B =
-c (£, i -Ef j) cos {kz - cor)
E = (E’j I + E^J) cos {kz - cot). Choose the
correct options from the following : (c) The given electromagnetic field is circularly
(«) The associated magnetic field is given as polarised
1 A A {(/) The given electromagnetic wave is plane
B =
-- (E| / + E2 y) cos {kz - cor) polarised
ELECTROMAGNETIC WAVES 8/53

9. An electromagnetic wave travelling along (a) will have frequency of 10^ Hz


(b) will have frequency of 2 x 10^ Hz
—>

z-axis is given as : E =
(c) will have a wavelength of 0-3 m
Choose the correct options from the
(d) fall in the region of radiowaves
following ;
12. The sources of electromagnetic waves can be
{a) The associated magnetic field is given as
a charge
I A
B=-kxE (a) moving with a constant velocity
c
(b) moving in a circular orbit
(b) The electromagnetic field can be written in (c) at rest
terms of the associated magnetic field as
—♦ —* A (d) falling in an electric field
E =c(B X k) 13. An EM wave of intensity / falls on a surface
kept in vacuum and exerts radiation pressure
(c) Le =0,k.B =0 (d) kxE =0.kxB = 0
A A

ww
p on it. Which of the followings are true ?
10. A plane electromagnetic wave propagating (a) Radiation pressure is I/c if the wave is totally
along j: direction can have the following pairs absorbed

of E and B {b) Radiation pressure is I/c if the wave is totally

Flo
reflected

e
(a)E,,By (b)E,,B^

ree
(c) Radiation pressure is 2 l/c if the wave is
(c)B,,Ey (d)E.^.By totally reflected

Fr
11. A charged particle oscillates about its mean
{d) Radiation pressure is in the range I/c < p

rF
equilibrium position with a frequency of 10^ < 2I/c for real surfaces
uurr
Hz. The electromagnetic waves produced :

ANSWERS
s for
kks
l.(c) 2. ib) 3. ib) 4. (a) 5. id) 6. (c) 7. (c) 8. (a, d)
Yo
oooo

9. (a, b, c) 10. {b, d) n.{a,c,d) 12. (b,d) 13. (a, c, d)


eB

HINTS FOR DIFFICULT MULTIPLE CHOICE QUESTIONS


ur
ad

Multiple Choice Questions -1


YYo

-19 -19
11x1-6x10 11x1-6x10
1. £ = 11 eV = II X l-6x 10-^^J = /iv or v = = 2-65x10'^ Hz
h -34
6-62x10
dd
Re

This frequency radiation belongs to ultraviolet region.


in

2. When a wave is reflected from denser medium, the refiected wave is without change in type of wave but
F

with a change in phase by 180° or n radian. Therefore, for the refiected wave we use z=-z, i = -i and
additional phase of k in the incident wave. The incident em wave is, E = Eq icos[kz - (O/)
The reflected em wave is

= Eg (“ ^) cos [A: (- z) - tor + 7c] = - Eq i cos [- (kz + cor) + tc]


A

= £q i cos [- (kz + cor)] 1 cos (6 -f 71) = - cos 0]


= Eq I cos (kz + tor) [ cos (- 9) = cos 0]
3. Here, energy flux, ^ = 20 W/cm^ ; Area, A = 30 cm^ ; time r = 30 min = 30 x 60 s
Total energy falling on the surface in time / is, f/ = 0 A / = 20 x 30 x (30 x 60) J
U 20 X 30 X (30x60)
= 36 X 10^ kg
-I
Momentum of the incident light = — ms
c 3x10^
Momentum of the reflected light = 0
Momentum delivered to the surface = 36 x 10“^ - 0 = 36 x 10“^ kg m/s
8/54 7^n<ideep, ’4. Fundamental Physics (XII) VOL.II

U
4. Electric field intensity on a surface due to incident radiation is, E = where — = P = power
At a’ /

P (for the given area of the surface)


E' P' 50 \ E
Hence, or £' =
E~ P~ m~ 2 2

5. The direction of propagation of electromagnetic wave is perpendicular to both electric field vector E and
—^ —> —>
magnetic field vector B , i.e., in the direction of E xB .
6. Intensity of em wave, I =u^^c

In terms of electric field, ^ Gq £q ; In te rms of magnetic field, u av

ww
2 Pq
1^0
Now Kv)electricfield = 2^0^0‘ i€o(cB„)2 =ie„x
1 1
e} = 0
B}=-
0 (*^av^due to magnetic field
2 M^o

Flo
Thus the energy in em wave is divided equally between electric field vector and magnetic field vector.

e
Therefore, the ratio of contributions by the electric field and magnetic field components to the intensity of

ree
an em wave is 1:1.

Fr
7. From a dipole antenna, the em waves are radiated outwards. The amplitude of electric field vector (Eq)

rF
which transports significant energy from the source falls off the electric intensity inversely as the distance
uurr
(r) from the antenna, i.e., £q «= 1/r.

Multiple Choice Questions - II


s for
kks
8. In em wave, the electric field vector is given as, £ = (£j ? + £2 y )cos (fe - cor)
Yo
oooo
eB

In em wave, the associated magnetic field vector, cos (fe-cor)


c c

Thus, option (a) is correct. As £ and B are perpendicular to each other and the propagation of em wave
ur

—> —>
ad

is Ir to £ as well as £ , so the given em wave is plane polarised. Thus, option (d) is true.
YYo

9. Suppose an em wave is travelling along negative z-direction. Its electric field is given by
dd

£ = £QCos(fe-cor)
Re
in

which is perpendicular to z-axis. It acts along negative y-direction.


F

The associated magnetic field B 'mem wave is along x-axis, i.e., along kxE ●

As fl _^o -»
B=-{kxE)
1 A -»

»"T c

Thus, option (a) is correct


—> —> A
The associated electric field can be written in terms of magnetic field as £=c(£ xk)
Thus, option (ft) is correct

Angle between k and £ is 90° and between k and B is 90°. Therefore, it.£ = l£cos90°= 0
A —> —>
and A:. £ = 1 £ cos 90° = 0. Thus, option (c) is correct
—^ ^
10. As electric and magnetic field vectors £ and £ are Jj* to each other as well as ±r to the direction of
propagation of em wave, hence options (ft) and {d) are correct
ELECTROMAGNETIC WAVES 8/55

11. Here, y = 10^ Hz, X = c/v =3 x 10^/10^ = 0-3 m.

This wavelength radiation (= 0-3 m) or frequency radiation 10^ Hz falls in the region of radio waves.
Thus, options (a), (c) and (d) are correct.
12. An em wave can be produced by accelerated or oscillating charge. In options (b) and (d), the charge is in
accelerated state, hence will be a source of em wave.
13. Radiation pressure (p) is the force exerted by em wave on unit area of the surface, i.e., rate of change of
momentum per unit area of the surface.
intensity /
Momentum per unit time per unit area =
speed of wave c

oww
Change in momentum per unit time per unit area = AI/c = radiation pressure (p), i.e., p = AI/c.
Momentum of incident wave per unit time per unit area = I/c.
When wave is fully absorbed by the surface, the momentum of the reflected wave per unit time per unit
area - 0.

ee
M I I
Radiation pressure {p) = change in momentum per unit time per unit area = --0 =-

FFrlo
c c c

r
When wave is totally reflected, then momentum of the reflected wave per unit time per unit area = - Uc.

rF
ee
Radiation pressure {p) =L ( I 21

c
<■' J c
ouru
rF
Here, p lies between He and 2 He. Thus, options (a), (c) and {d) are correct.

ffosor
VERY SHORT ANSWER QUESTIONS
os k
14. Why is the orientation of the portable radio with respect to broadcasting station important ?
ook
Yo
Ans. The em waves emitted from a broadcasting station are plane polarised. They can be received properly by
Y
our portable radio if the receiving antenna is parallel to the electric/magnetic part of the wave.
Bo

Why does microwave oven heats up a food item containing water molecules most efficiently ?
reeB

15.
Ans. Because the frequency of the micro-wave matches the resonant frequency of water molecules.
oouY

16. The charge on a parallel plate capacitor varies ^sq-q^ cos 2jtv/. The plates are very large and tio.se
ur

together (area = A, separation = d). Neglecting the edge effects, find the displacement current through
ad

the capacitor ?
dY

Ans. Displacement current, Iq = conduction current 1q


nidn

/r, = /^ = — = —[o,, cos27tvr] 2 V sin 2 n vf.


Re

^ ^ dt dt^
17. A variable frequency a.c. source is connected to a capacitor. How will the displacement current
F
Fi

change with decrease in frequency ?


E E
Ans. Current through capacitor, i ts — = coC E =2iivC E or /_ os V.
1/03C

Therefore, decrease in frequency v of a.c. source decreases the conduction current. As displacement current
= conduction current, hence decrease in v decreases displacement current in circuit.
18. The magnetic field of a beam emerging from a filter facing a floodlight is given by
B = 12 X 10-* sin (1-20 x 10^ z - 3-60 x 10*^ t) T.
What is the average intensity of the beam ?
Ans. Here, S= 12x 10"^ sin (1-20 x 10'^z-3-6x 10^^ t)
Comparing it with. B = Bq sin (kz - co/), we have
0
12 X 10"^ T

/
_— —^ ^0 c — “ X
(12x10'8)2x(3x10^) «l-71W/m2
(4kx10-^)
av

2 Ho 2

*
6/56 ^n4ieU&fi. ^ Fundamental Physics (XII) MSlWIi

19. Poynting vector 5 Is defined as a vector whose magnitude is equal to the wave intensity and whose
direction is along the direction of wave propogation. Mathematically, it is given by _L B ●
^0
Show the nature of S vs / graph.
—i

Ans. In em wave let E be varying along y-axis, B is along z-axis and propagation of wave be along x-axis.
Then E x B will tell the direction of propagation of energy flow in e.m. wave, along x-axis.

Let £ = sin(tor - Lc) y ;


A

B = BQsiniOit-kx)k

ww
S =
— (£x £) = — £p Bq sin^ ((0/ -kx)[j xk]
^0 ^^0

Flo
_^0^0
sin^ (cor - cfc) t

ee
^0

rere
r FF
The variation of I 5 I with time r will be as shown in Fig. 8(N).3 by thick solid curve.

20. Professor C.V Raman surprised his students by suspending freely a tiny light ball in a transparent
uurr
vacuum chamber by shining a laser beam on it. Which property of EM waves was he exhibiting ?
Give one more example of this property. foor
Ans. A liny ball can be suspendedfreely in a transparentvacuum chamber by shinning a laser beam on it. It is so
ks s
because e.m. wave {i.e., laser beam) exerts radiation pressure. The tails of the comets are due to radiation
Yoo
ooook

pressure.
eBB

SHORT ANSWER QUESTIONS

21. Show that the magnetic field £ at a point in between the plates
uurr
ad

FIGURE 8(N).4

of a parallel-plate capacitor during charging is --**^1* - —


Yo

dt b
(symbols having usual meanings).
dY

i
»
Ans. Let be the displacement current in the region between two
Re
innd

plates of parallel plate capacitor, Fig. 8(N).4. The magnetic field


induction at a point in a region between two plates of capacitor at
FFi

a J.r distance r from the axis of plates is

B =
^^0 xe
47C r 2nr D ~
2nr
0
dt D ~^Q di

1^0 ^0 d 1 dE
{Enr^) = nr
[v =Ekt~]
2nr dt 2nr dt 2 dt

22. Electromagnetic waves with wavelength


(i) Xj is used in satellite communication.
(/() germs in water purifier.
(/{() X3 is used to detect leakage of oil in underground pipelines.
(iV) X4 is used to improve visibility in runways during fog and mist conditions.
ELECTROMAGNETIC WAVES 8/57

(a) Identify and name the part of electromagnetic spectrum to which these radiations belong.
(b) Arrange these wavelengths in ascending order of their magnitude,
(c) Write one more application of each.
Ans. (a) - microwave, - ultraviolet, ~ X-rays, X^ - Infrared.
(/>) < X2 < X^ < A.|.
(c) Microwave is used in radar. UV in LASlK-eye surgery. X-ray is used to detect a fracture in bones.
Infrared is used in physical therapy, i.e., to treat muscular strain.

w
23. Show that average value of radiant flux density over a single period‘T ’ is given by S =
Ans. We know that radiant flux density S (called Poynting vector) is given by
1
5 =
(ExB) = c^e^ (exB)

e
Ho

row
re
Let the e.m. wave be propagating along jc-axis. The electric field vector of e.m. wave be along y-axis and
magnetic field vector be along z-axis. Therefore,

FFllo
eeF
E = Eq COS (kx-(at) j and B = Bq cos (kx-(at) k

u
E X B =(EqBq) cos^ (kx-(at) 7

r
5 = €q (£ X B) = Eq (Eq Bq)cos^ (kx - (at) 7

sFr
kro
Average value of the magnitude of radiant flux density over complete cycle is
uor
1 ^ offo
5... =c2eo£oBo — J cos^(foc- <at)dt
av

0
kos
Y
Yo
T j
eerBB

T 1 T
f cos^ (toe - (at) dt = —
oo

= c GqEqBqX —X —
T 2 [ i \
rY

[As c = Bq/Bq]
u

or
ou
ad
do

1 1 1
or €
0“
nY

El0
nid
Re
F
Fi

24. You are given a 2 pF parallel plate capacitor. How would you establish an instantaneous displacement
current of 1 mA in the space between its plates ?

= lmA=10-3A;C = 2pF = 2xl0"^F, /^ = / = ^(CV) = C


dV
Ans. Here,
dt

D _ 10-3
Therefore, = 500 V/s
dt ~ C 2x10-^
Therefore, applying a varying potential difference of 500 V/s would produce a displacement current of
desired value.

25. Show that the radiation pressure exerted by an EM wave of intensity / on a surface kept in vacuum
is//c.

force F Ap
Ans. Pressure P = = —(Ap/Ar)x- V F = = rate of change of momentum
area A A c At

A
w
8/58 ^xttdeefa. ’4. Fundamental Physics (XII) kWII

1 1
F = xAt/

r
AcAr AcAr
...(1)

(where c A^ = A(7 = energy imparted by wave in time A/)


energy imparted At/
Intensity, / = = Pc [From (/)]
area X time AAi

u
P = Hc (proved)

sF
o
26. What happens to the intensity of light from a bulb if the distance from the bulb is doubled ? As a
laser beam travels across the length of a room, its intensity essentially remains constant. What
geomatrical characteristic of LASER beam is ersponsible for the constant intensity which is missing
in the case of light from the bulb ?
Ans. Intensity of light is reduced to one fourth because the light beam spreads as it approaches into a spherical

llow
FF
region of area 4 7i r“, i.e., 1« !//●“. But laser beam does not spread, hence its intensity remains constant.

o
k
Laser beam is unidirectional, monochromatic and coherent light, whereas the light from a bulb does not
possess the above properties.

r
—» —♦

27. Even though an electric field E exerts a force qE on a charged particle yet the electric Held of an

o
ree
EM wave docs not contribute to the radiation pressure (but transfers energy). Explain.

foY
Ans. Electric field of an electromagnetic wave is an oscillating field. Due to it, the electric force caused by

rfF
o
electric field in e.m. wave on a charged particle is an oscillating one. This electric force averaged over an
Y
integral number of cycle is zero, since its direction changes every half cycle. Hence, electric field is not
responsible for radiation pressure.
ur oB
LONG ANSWER QUESTIONS
ks
or
Yo
eBd

28. An infinitely long thin wire carrying a uniform linear static charge FIGURE 8(N).S
density X is placed along the z-axis (Fig. 8(N).5). The wire is set
u
e

into motion along its length with a uniform velocity v = vk . AZ


on

—> 1 ^ ^ a

Calculate the poynting vector 5 = _ (£x B) ●


Yio
ur

^^0
ad

Ans. E =
X /
^
’ B =— —i
Mo 2/ A Mq Xv a
F

2 Tie,,0 a 4n a 2ti a ( X I .
nd
Re
Fi

1 Xj ^^0 1
5 = —[Ex5] = — X Xvi
^^0 27re,.a
0
2Tia
X

X-v
« 70XO
1
47l"e,^^7-
0

X~v A
ir

29. Sea water at frequency v = 4 x 10® Hz has permittivity e 80 Co, permeability M ® Mo n


resistivity
p = 0’25 £2 - m. Imagine a parallel plate capacitor immersed in sea water and driven by an alternating
voltage source V (r) = Vq sin (2 n v/). What fraction of the conduction current density is the displacement
current density ?
ELECTROMAGNETIC WAVES 8/59

Ans. Let d be the distance between the plates. The applied voltage is given by, V (t) = Vq sin 2 n vt.
V(0 V'o
The applied electric field, E = — sin27i:vf
d d

The conduction current density, 7^ = — sin Invt = sin liivt


P pd

where
'^0 = Joc = max. conduction current density.
pd
Displacement current density is given by
e^G^d(EA) dE
— — sm2jivt
= ^0^r dt

ww
A Adt
dt[d
v;0
27Cvcos27cvf = 7q£, cos 2 71 vr

FF loo
>'o

ree
where 0
g„ — x 2tiv = 7 OZ) = max. displacement current density
" d

rFee
dpD gpgr ^o27Cv/d
x(4xl0^)x80x0-25/2 = ^
1
= 27tveQe^p =47ceove^p/2 = 9x10^

F
oor r
rur
dpc VoKpd) s ff
30. A long straight cable of length I is placed symmetrically along z-axis and has radius a (« [).
The cable consists of a thin wire and a co-axial conducting tube. An alternating current
k
I (t) = Iq sin (2 n vt) flows down the central thin wire and returns along the co-aidal conducting tube.
YYoou
ookos

The induced electric field at a distance s from the wire inside the cable is

-> fs ^ ^
BBo

E (s» 0 = m /q ^ (2rev/) In — A:
re

(i) Calculate the displacement current density inside the cable,


ouur
ad

(ii) Integrate the displacement current density across the cross-section of the cable to find the total
Yo

displacement current Ij).


(iii) Compare the conduction current Iq with the displacement current Jq^.
Ans. (/) Given, the induced electric field at a distance r from the wire inside the cable is
dY
Re
idn

-> fs V
= |1q/q vcos(27Cvt)log^ — k
FFin

aj
Displacement current density.

dE d
= ^o Ho/ovcos(2rtw)log^ - k
dt ya)

d ('i V 1 . (s\^
c2
/q v^27r[-sin27tv/] log^ — k
^ ^ ^ dt ^\a)
v2
^-kY
— 2 7c/q sin 2ti vf log^ —
. s .
●: log - = -log -
^ a
a

^ s.

1 fa A 2nl 0 o ^
27c/nlog^ —« sin27cvr^ log„ — sin27cv/ k
^2 A.2
U e
V /
® s
8/60 7^no4{eefi, ^ Fundamental Physics (XII) VOL.II

a 2n a

(if) / D
= J Jj^sdsdQ= J sds
f/0= J Jj^sds.ylTt
s=0 0 s=0

' (1,K^ ^
a
2k
—" sdssinlKvt x2n = —
^
/„u j
log - sdssin2nvt
5=0 J \ ^ J 5=0 V‘« j

12 f «- 2k \2 /^\ ^ \2
/ a
2 /
r« V 2^ ● o
a

/
log^ - -d{s^).sm2%vt = — /fjSin27cv? log — J —
uJ 0
5=0 J 2 2 A ^ 5 =0 \s)

oww
Y^"f =-—f—T /„sin27cvf f r^f r5f
a
a

4U j /Qsin2ji:v/ J log^ — 5=0 UJ . 5=0J


log — d —
Uj
a
\2

e
log
1
-d- = -i
/Qsin27uv/x(-l) \a) {a)

re
4U j 5=0

FFrllo
( 2jc \2 27ia \2
/

eF
/«sin2jcv/ = /„sin27cv/
D ~
T[TJ 0 0

e
ouru < 2A, >

'27ta \2

r
osrF
(Hi) The displacement current, / D ~ /q sin 27cvt = /qo sin 2 7C vr
. 2A. j
27ca \2
ffor
f aitf ,
/

where IOD “
[2X) ^0
k
^0 = UJ
< ^ >
»
^OD
/ __\2
a7C
kso
31. A plane EM wave travelling in vacuum along z direction is given by
ooo
Yo
Y
—¥ ^
BB

E =EQ^(kz-(at) i and B = ^0 sin (kz - eoO y .


« —> —4

(i) Evaluate ^ E .d / over the rectangular loop 1234 shown in Fig. 8(N).6.
Y
r ree

(ii) Evaluate ^ £ .d s over the surface bounded by loop 1234.


ouu
ad
Ydo

(iii) Use equation ^ E ,dT= to prove = c.

dt B
MJq
nidn

● —> —>
'E 1
(iv) By using similar process and the equation ^ B.dI = \IqI + e 0 prove that c =
Re

dt ’
F
Fi

^4
FIGURE 8(N).6
FIGURE 8(N).7

<●«.
4
UJ
II 4 dl 3
4
tiu
A ''dl
1 2
>- >z
■►z ^2
dl

y
ELECTROMAGNETIC WAVES 8/61

-i

Ans. (i) During the propagation of electromagnetic wave along z-axis, let electric field vector (£) be along
—* ^ A
jf-axis and magnetic field vector B be along y-axis, i.e., £ = £q / and B = j ■ Refer Fig. 8(N).7.
Line integral of £ over the closed rectangular path 1234 in x-z plane is
■> 3 4
-» -)
E.dl = E.dl + E.dl + E.dl + E.dl
I 2 3 4

2 4 1

£f//cos90“+
£J/cosO‘’ + j £(//cos90° +J £f//cosl80'’
3 4

= 0 + £ /h-0-£ h = E -£
...(0
= £q A [sin (A:z2 “ oV) - sin (/:zj - (or)l

ww
'2 '! ^2

(/i) For evaluating o B.ds over the surface bounded by FIGURE 8(N).8

Floo
loop 1234, let us considera rectangle 1234 be made of small
strips. One small strip of area ds = h dz 'is shown in Fig.
8(N).8.

ee
o
UJ

eer
II 4 3
() B .ds = oBdscosO° = oBds
tin

FFr
Surface bounded
by strips 1234
h dt/
oorr
uur r
*-*> 1 2
■►z
s ff
j £Qsin(fe-(0/)/idz dz ^2
sk
YYoo
ooko

B.h

^ [cos (^^2 “ 02T) - COS (A:Zj - 03/)] ...(«)


eBB

/C

● —* —» d^ B d r
{Hi) Given, O E .dl= ~ — i)B.ds
dt dt ^
uurr

Putting the values from (/) and («), we get,


ad
Yo

£q h [sin (kzj - o)/) - sin (kz^ - (Ot)


d B.Ji
dY

dt
-^[cos(kz2
k
-to/)-cos{/:zj -co/)}
Re
ind

Br.h
FFin


k
03 [sin(/tz2 -03/)-sin(/tZ| -03f)]
Br.0 03 (0
or E = B..c -c or £(/£o = c
k
0
I ^
(iv) For evaluating (> B .dl , let us consider a loop 1342
iny-z plane as shown in Fig. 8(N).9.
3 4

() B.dl =
B.dl +J B.dl +J B.dl + B.dl
! 4 2

B dl cos 0 + Bdl cos 90° + Bd! cos 180° + Bdl cos 90° Bdl + 0- Bdl+0
4

- h B_^ h - /*(£.^ ) = BqH [sin (A:z, - 03/) - sin {kzj - 03/)] ...{Hi)
8/62 Fundamental Physics (XII) VOL.II

f —> —)
FIGURE 8(N).10
To evaluate <|>£ = y E.ds , let us consider the rectangle
1342 to be made up of strips, each of area ds = h dz. Fig.
8(N).10. <—

UJ

4>£=^ E.ds £f/scosO®=^ Eds II


tuj
^2

= I £q sin (fej -(Ot)hdz

w
^2
■►z
Z| t;
1 TV2
E^h ^d^//h
—^[cos (fe, - ®f)-cos (fe, - ©r)] IL
k ^ ‘ 3‘ 4

e
d^P E(.h(0

row
—[sin (fej - ®r)-sin (kz2 - cor)] m

re
dt y

I B.dt=\i^

FFllo
eeF
In the relation,

u
r ^ —>
d^E
/is the conduction current. The value of / = 0 (in vacuum). y B.dl = |1qGq dt

r
sFr
Putting values from («0 and (tv), we get

kro
Ef.h(o
uor
Bq h [sin (fei - ©f) - sin {kz2 - cor)] = [Iq “T— ^^2 ~
offo K

1 ^0
kos
or or as = c and (H = ck
Y
B^k B0
Yo
eerBB
oo

ck 1 1 1

"" k PqGq
or = or
rY

M^o^o 0

32. A plane EM wave travelling along z direction is described by


u
ou

£ = £q sin {kz - cor) i and B = Bq sin {kz -tat) j . Show that


ad
do
nY

(i) The average energy density of the wave is given by u av = ie +


4 » » 4(1, ●
nid
Re
F
Fi

(ii) The time averaged intensity of the wave is given by /av

Ans. (i) The electromagnetic wave carry energy which is due to electric field vector and magnetic field vector. In
electromagnetic wave E and B vary from point to point and from moment to moment. Let E and B be their
time averages. The energy density due to electric field E is.
1
u £2
£ “ 2^0

Uj>
B = —
1^
The energy density due to magnetic field B is, A
- 2m„
Total average energy density of em wave
1 1 £2
U =Ur.-\-U £2 + -
av £
B 2 ^0 (0
ELECTROMAGNETIC WAVES 8/63

Let the em wave be propagating along z-direction. The electric field vector and magnetic field vectors be
represented by
E = Eq sin {kz - (Of)
B = Bq sin {kz - (Of)
The time average value of E^ over complete cycle = £q / 2.

and time average value of B^ over complete cycle =BqH.

w
f
1 El0 I
U
av

e
1
(i‘0 We know that Eq-cBq and c =

row
re
Bl \El!<-P El

FFllo
1

eeF
4^0 4 4H(, x^‘o®o=:J^o^o

u >‘B = ‘‘E

r
sFr
is £2+i^ = ie E2+- 1 1

=2®0^0 =-21^0

kro
4 » » 4h„ 4^0*0 +
u
Hence, av
uor
Time average intensity of the wave offo
2^0^o‘'=^«o4®0
1
kos
Y
L. =u^..c = -
Yo
av av
eerBB
oo
rY
u
ou
ad
do
nY
nid
Re
F
Fi
8/64
Fundamental Physics fXinCTgwn

|r

NEET/JEE
SPECIAL

For ultimate preparation of this unit for competitive examinations, students should refer to

ww
● MCQs in Physics for NEET
Pradeep's Stellar Series.... ● MCQs in Physics for JEE (Main)
separately available for these examinations.

Floo
ee
Multiple Choice Questions (with one Correct Answer)

eer
(1) the wavelength X is 188.4 m.
I. Displacement current,

FFr
(2) the wave number k is 0.33 rad/m.
electromagnetic waves

oorr
uur r
and its characteristics (3) the wave amplitude is 10 V/in.
(4) the wave is propagating along + x direction.
s ff
1. A parallel plate capacitor of capacitance 20 pF is Which one of the following pairs of statements is
being charged by a voltage source whose potential
sk
correct ?
YYoo
is changing at the rate of 3 V/s. The conduction
ooko

(a) (3) and (4) (6)(l)and (2)


current through the connecting wires, and the
(c) (2) and (3) (d)i\)and (3)
eBB

displacement current through the plates of the


capacitor, would be. respectively, (AIPMT Main 2010)
ia) zero, 60 pA {b) 60 pA, 60 \iA 4. An electromagnetic wave of frequency v = 3 0
uurr

(c) 60 pA, zero (d) zero, zero MHz passes from vacuum into a dielectric
ad

medium with permittivity 6 = 4 0. Then


(NEET 2019)
Yo

{a) wavelength and frequency both become half


2. A plane electromagnetic wave of frequency
ib) wavelength is doubled and frequency remains
50 MHz travels in free space along the positive
dY

unchanged
Re

AT-direction. At a particular point in space and time,


ind

—> ^ (c) wavelength and frequency both remain


FFin

E = 6-3 j V/m. The corresponding magnetic unchanged


{d) wavelength is halved and frequency remains
field B at that point will be :
unchangedd
A

(a) 18-9 x\Q-^ k T (b)2-l X 10'® ft T 5. A plane electromagnetic wave travelling along the
j:-direction has a wavelength of 3 mm. The
(c) 6-3 X 10-^ ft T (d) 18-9 X 10® ft T variation in the electric field occurs in the
(JEE Main 2019) y-direction with an amplitude 66 V m“’. The
3. The electric field of an electromagnetic wave in equation for the electric and magnetic fields as a
free space is given by function of x and / are respectively

E = 10 cos (10^ / + tr) j V/m, (a) £ = 33 cos 71 X 10' * t -—


V cj
where i and x are in seconds and metres
respectively. It can be inferred that B.= 11 X 10'^cos n X 10*' (r-.v/c)
ANSWERS
1. (h) 2. (h) 3. (d) 4. (d)
ELECTROMAGNETIC WAVES 8/65

(/?) = 11 cos 2 71 X 10 ” (r - xlc) e e


'‘i 1
1 _
= 11 X 10“^ cos 2 TU X 10** (7 - xlc) (c) £ r-, (d) Ir-, ■7
4
(c) £'^= 33 cos 7t X 10** (J ~xic)
(JEE Main 2018)
By= 11 X lO’^cos Ji X 10** (i-x/c)
{d) E^. = 66 cos 2 n X 10*' (/ - x/c) 9. The magnetic field of an electromagnetic wave is
B. = 2-2 X 10“^ cos 2 71 X 10** (t-x/c) given by :

(e) Ey- 66 cos tc x 10*' (t-x/c) ~B = l-6x 10-^’cos(2 X 10'^z + 6x 10*5


B^.= 2-2 X 10“^ cos 7C X 10* * (t-x/c)
(Kerala PET 2008) (It'-^-j) Wb/m~
6. The electric field part of an electromagnetic wave The associated electric field will be
in a medium is represented by =0 ;
N
(a) E = 4-8 X 10“ cos (2 x 10 z + 6 x 10*5
E =2-5 —
(-f+2j) V/m

ww
y C
/
rad rad' (b) E = 4-8 X 10^ cos (2 X 10"^ z + 6 X 10*5 t)
X cos 2:txl0^ t- TtxlO - x

FF loo
(-2j'+2f) V/m
m s
L\

E. = 0. The wave is

ree
(a) moving along the x-direction with frequency (c) E = 4-8 X 10- cos (2 X 10'' z + 6 X 10*5
10^ Hz and wavelength 100 m
(1 + 2/) V/m

rFee
(b) moving along the .v-direction with frequency
10^ Hz and wavelength 200 m (d) E = 4-8 X 10- cos (2 X lO'' z + 6 X 10*5

F
oor r
rur
(c) moving along the - .v direction with frequency
10^ Hz and wavelength 200 m
s ff (2/ +j') V/m
(d) moving along the v-direction with frequency (JEE Main 2019)
2 7C X 10^ Hz and wavelength 200 m 10. The energy contained in a small volume through
k
YYoou
(AIPMT 2009) which an electromagnetic wave is passing
ookos

oscillates with
7. An eni wave is propagating in a medium with a
BBo

A (fl) double the frequency of the wave


velocity V = V i. The instantaneous oscillating
(b) zero frequency
re

electric field of this em wave is along + y axis. (c) the frequency of the wave
Then the direction of oscillating magnetic field
ouur
ad

(d) half the frequency of the wave.


of the em wave will be along
Yo

11. In an electromagnetic wave, the electric and


(a) -y direction (b) + z direction
magnetic fields are 100 V/m and 0-265 A/m. The
(c) - z direction (d) - X direction
maximum energy flow per second per unit area
dY

(NEET 2018)
Re

will be
idn

8. An EM wave from air enters a medium. The (b) 13-2 W/m^


(a) 79 W/m^
FFin

A ("2 (c) 53 W/m2 (J) 26-5 W/m^


electric fields are E^ = Eqj x cos 27tv —t
c 12. A small metallic ball is charged positively and
negatively in a sinusoidal manner at a frequency
in air
and £2 = Eq2-v cos[/:(2z-cr)] in of 10^ cps. The maximum charge on the ball is
medium, where the wave number k and frequency 10”* C. What is the displacement current due to
V refer to their values in air. The medium is non the alternating current ?
magnetic. If e and e refer to relative (b) 3-8 A
(a) 6-28 A
permittivities of air and medium respectively, (c) 3-75 X 10^ A (d) 122-56 A
which of the following option is correct ? (DCE 2005)
£ 13. In an electromagnetic wave, the electric and
£r.I
= 4 magnetising fields are 200 V/m and 0-365 A/m.
(«) i r-, (^) ^ r-,
= 2
The maximum rate of energy flow is
ANSWERS

5. (d) 6.(b) 7.(b) 8. (c) 9. (a) 10.(a) 11.(d) 12. (a)


8/66
Fundamental Physics fXIDtTOirn
(a) 73-0 W/m^ (h) 36-5 W/m^
(b) XWE and kWExB
(c) 54-7 W/m- (cl) 77-8 W/nr. -» -*

14. Light with an energy flux of 25 x 10^ Wm"’ falls (c) X II 5 and kWExB
on a perfectly reflecting surface at normal —* —» —) —»

incidence. If the surface area is 15 cm^, the (d) X\\ E and k\\ BxE (AIEEE 2012)
average force exerted on the surface is 20. The electric field associated with an e.m. wave in
(a) 1-25 X 10"* N (b) 2-50 X 10-6 N vacuum is given by
(c) 1-20 X 10-6 N (d) 3-0 X 10-6 N E = i 40 cos - 6 X 10^ t),
(AIPMT 2014)
where E, z and t are in vo!t/m, metre and seconds
15. A plane electromagnetic wave of wave intensity respectively. The value of wave factor k is :
6 W/m^ strikes a small mirror of area 30 cm“, held (a) 2 'm
-1
(b) 0-5 m
-1

perpendicular to the approaching wave. The -1


(c) 6 m (d) 3 m“'
momentum transferred in kg ms"' by the wave to

w
the mirror each second will be [AIPMT (Prelim) 2012]
(a) 1-2 X lO-'O (b) 24 X 10"^ 21. A red LED emits light of 0-1 watt uniformly
(c) 3-6 X I0-« around it. The amplitude of the electric field of
(fO 4-8 X lO-"^.

Flo
the light at a distance of 1 m from the diode is
16. A lamp emits monochromatic green light (a) 1-73 V/m (h) 2-45 V/m

e
uniformly in all dirctions. The lamp is 3%

reee
(c) 548 V/m (d) 7-75 V/m
effecient in converting electrical power to

FFr
electromagneticwaves and consumes 100 W of (JEE Main 2015)
power, The amplitude of the electric field 22. The ratio of contributions made by the electric
associated with the electromagnetic radiation at field and magnetic field to the intensity of an

for
ur
a distance of 5 m from the lamp will be electromagnetic wave is : (c = speed of
(a) 1-34 V/m (b) 2-68 V/m electromagnetic waves)
(a) 1 : 1 (b) \ :c
kkss
(c) 4-02 V/m (d) 5-36 V/m.
(c) I ; c2 (d) c: I (NEET 2020)
Yo
17. In an electromagnetic wave in free space, the root
23. For a plane electromagnetic wave propagation in
oo

mean square value of the electric field is E rms


= 6 V/m. The peak value of the magnetic field is a:-direction, which one of the following
eB

combination gives the correct possible directions


(a) 141 X 10“^ T (b) 2-83 X 10-^ T
for electric field (£) and magnetic field (B)
(c) 0-70 X 10-^ T (d) 4-23 X 10-^ T respectively ?
ur

(NEET 2017) , , A A A A A A

(a) - j + k,- j+ k (b) j+kj+k


ad

18. A point source of electromagnetic radiation has


YYo

/ , A A A A . A A A A
an average power output of 1500 W. The (c) -j+k,-j-k (d) j+k,-j-k
maximum value of electric field at a distance of
(NEET 2021)
3 m from this source in Vm"* is
d

24. The mean intensity of radiation on the surface of


Re

(a) 500
in

(b) 100 the Sun is about 10^ W/m-. The rim value of the
500 250 corresponding magnetic field is clo.sest to :
F

(O- (d)- (a) 10-T (b) 10-^ T


(c)lT (d) 10-2 T
(e) loS (Kerala PET 2009) (JEE Main 2019)
19. An electromagnetic wave in vacuum has the 25. Light is incident normally on a completely
electric and magnetic fields £ and B , which absorbing surface with an energy flux of
are always perpendicular to each —» other. The
25 W cm-^. If the surface has an area of 25 cm^,
the momentum transferred to the surface in 40
direction of polarization is given by X and that minutes time duration will be

of wave propagation by k . Then : (a) 3-5 X 10-6 Ns (b) 6-3 X 10-^ Ns


(c) 5-Ox 10-^ Ns (d) 14 X 10-6 Ns
(a) XWB and TwBxE (JEE Main 2019)

nrccsia
13. 14. (/,/) 15. 16. (P) 17, (b) 18.(6) 19.(6) 20. (ai 21.(6) 22. (n) 23. (c)
24.(6) 25.(f)
ELECTROMAGNETIC WAVES 8/67

26. A plane electromagnetic wave having a frequency, Its magnetic field will be given by :
V = 23-9 GHz propagates along the positive £ A A
z-direction in free space. The propagation (a) —(JC+y)sin(/:z-co/)
constant of electromagnetic wave is : c

(c = 3 X 10^ ms-') En A A
ib) 0-5 X 10^ m (/>) -^(A.--y)sm (Aiz-cor)
-1
(a) 0-5 X 10^ m -1 c

(d)2x IQ-2 m
-1
(c) 2 X 10-2 ^
-1

(JEE Main 2020)


ic) — (x-y)cos(kz-(iii)
27. A plane electromagnetic wave of frequency c

25 GHz is propagating in vacuum along the


£
z-direction. At a particular point in space and time, _o (_.^+J)sin(/l:z-a)r) (JEE Main 2020)
—^ rt A c
the magnetic field is given by B =5x10-® i T ●

ww
31. If and represents relative permeability of
The corresponding electric field is given by E medium and relative permittivity of the medium
is : (Speed of light, c = 3 x 10^ ms"') and speed of light in medium is c/p, where p

Floo
,-16 (refractive index) will be
(a) l-66xl0"'^ V/in (h) -1-66x10 i V/m
A
(b)

ee
(c) ISi'V/m (d) -15 i V/m
(JEE Main 2020)

eer
FrF
28. A plane electromagnetic wave is propagating (c) (d) none of these
A A

oor r
U + j)
ur r
along the direction , with the polarisation s ff (JEE Main 2021)

A
32. An electromagnetic wave is propagating in
along the direction k . The direction of magnetic medium where p = = 2. If speed of light in
sk
medium is x: x 10' m/s. Find the value of x :
YYoou
field vector of electromagnetic wave would be :
oooko

(a) 30 (b) 15
A A A A
eBB

(j-n U + j) (c)8 (J) 7 (JEE Main 2021)


(«) ib)
V2 33. An electromagnetic wave is propagating along
uurr

A A
x-axis, such that E = 800 sin {kx - cot) j (in V/m).
ad

U-j) i-i-j) If an electron is projected with velocity


(c) id)
Yo

VI VI 3 X 10^ ms"', along jc-axis, then the maximum


(JEE Main 2020) magnetic force experienced by electron is
dY

-17 -16
(a) 1-28 X 10 N (b) 2-4 X 10 N
Re

29. In a plane electromagnetic wave, the directions -18


(c) 3-2 X 10 N (d) None
ind

of electric and magnetic field are representedby


FFin

(JEE Main 2021)


k and (27-2 j), respectively. What is the unit
34. When light propagates through a material medium
vector along the direction of propagation of the of relative permittivity e ^ and relative
wave ?
permeability p^ the velocity of light u is given by
1 A A 1 A A (c-velocity of light in vacuum)
(a) (/ + ;●) ib) i2i + j)
VI V5
(o)v =c (b) V =
1 A A 1 A A

ic) U + 2J) id) (k + J)


VI VI
e c
(JEE Main 2020) (c) v = id) v =
30. The electric field of plane electromagnetic wave Pr
A A (NEET 2022)
is given by ; E = Eq(x+ y)sm(kz-(oi) ;

26. (b) 27.(c) 28.(c) 29.(nl 30.(d) 31.(b) 32.(b) 33.(a) 34.(rf)
8/68
Fundamental Physics CXII)
35. A plane elecu-omagnetic wave travels in a medium Which of the following statements are correct ?
of relative permeability 1-61 and relative (0 Only radiowaves can be used to transmit audio
permittivity 6-44. If magnitude of magnetic information,
intensity 4-5 x 10”^ Am~‘ at a point, what will be (n) Only infrared radiation is emitted by very hot
the approximate magnitude of electric field object.
intensity at that point ? (Given, permeability of (Hi) Only visible light can be detected by humans.
free space, |Jq = 4 tu x NA"-, speed of light (a) Only (i) is correct (h) Only (ii) is correct
in vacuum, c = 3 x 10^ ms"’) (c) Only (iii) is correct
(fl) 16-96 Vm-’ (b) 2-25 X 10-2 Vm"' (cf) None of the above is correct
(c) 8-48 Vm"' (d)4-2x 10^ Vm-' 40. The magnetic Qeld in a plane electromagnetic
(JEE Main 2022) wave is given by
36. The electric field in an electromagnetic wave is B^ = 2x 10-2 gij, ^0-5 X 10^ 1-5 X 10'^).
given by £ = 56-5 sin co (/ - x/c) NC-'.Find the This electromagnetic wave is

ww
intensity of the wave if it is propagating along (a) a visible light (b) an infrared wave
-v-axis in the free space. (Given, Sq = 8-85 x 10-'2 (c) a microwave (d) a radiowave
C2 N-' m-2]

Floo
(DUMET 2010)
(a) 5-65 Wm“2 (b) 4-24 Wm-2 41, If a source of power 4 kW produces 10^^
(c) l -9x 10-2 Wm-2 (d) 56-5 Wm-2 photons/second, the radiation belongs to a part

ee
(JEE Main 2022) of the .spectrum called

eer
(a) ultravioletrays (b) microwaves
II, Electromagnetic spectrum

FFr
(c) 7-rays (d) X-rays
(AIEEE 2010)

oorr
37. The energy of the e.m. waves is of the order of
uur r
15 KeV. To which part of the spectrum does it 42. The condition under which a microwave oven
s ff
belong ? (Use h = 6-64 x lO'^^ Js) heats up a food item containing water molecules
(a) y-rays (b) x-rays
most efficiently is ;
sk
YYoo
(c) Infra-red rays (d) Ultra-Violet rays (a) Infra-red waves produce heating in a micro-
ooko

wave oven
(RE-AIPMT 2015)
(b) The frequency of the microwaves must match
eBB

38. The frequency of e.m. wave which is best suited the resonant frequency of the water molecules
to observe a particle of radius 3 x 10“^ cm is of
the order of (c) The frequency of the microwaves has no
uurr

relation with natural frequency of water


(a) lO'^Hz (h) 10“'Hz(£) 10'^ Hz (d) lO'^Hz.
ad

molecules
Yo

39. Consider the following types of electromagnetic id) Microwaves are heat waves, so they always
radiation : radiowaves, infra-red, visible light. produce heating (AIPMT 2013)
dY

m
Re

Multiple Cfic ice Questions (with One or More than One Correct Answers)
ind
FFin

43. An electromagnetic wave is travelling through a 44. An electromagnetic wave going through vacuum
medium of refractive index p,, and is incident on is denoted by £ = £q sin (kx - (at). Which of the
the boundary of a medium of refractive index \i-,. following is/are independent of wavelength ?
If the wave reflects at the boundary. («) k (b) (0
(a) The wave undergoes a phase change of 180”, (c) k/(a (d) k(a
ifPl <P2
(b) The wave undergoes a phase change of 180°, 45. If /<£, are the eneigy density of electromagnetic
ifPi>Po wave due to electric and magnetic field vectors,
are the rms value of electric and
(c) The wave undergoes no phase change, if
rms

magnetic field vectors in electromagnetic wave,


<F2
then the total energy density of a sinusoidal
(d) The wave undergoes no phase change, if electromagneticwave is
1^1 >^2
(«) i/£ (h) u^+ u
(National Standard Exam, in Physics, 2010)
m

ANSWERS

35. (d) 36. (b) 37. (b) 38. (a) 39. (d) 40. (c) 41. (a) 42. (b) 43. (a.d) 44. (c)
ELECTROMAGNETIC WAVES 8/69

—*

I nus {d) O B .ds = 0


w 2^0 + 0
S

48. Which one of the following groups of electro


46. Which of the following are not electromagnetic magnetic waves is/are in order of increasing
waves ?
frequency ?
(fO cosmic rays (b) gamma rays (a) microwaves, ultraviolet rays, X-rays
(c) p-rays (</) .r-rays (b) radiowaves, infrared radiation and visible
47. Which of the following Maxwell’s equations have light,
sources of E and B ? (c) gamma rays, visible light, ultraviolet rays
r ^ a (d) gamma rays, ultra violet rays, radio waves.
(a) O E .ds = — 49. In terms of potential difference V, electric current
^0
s
/, permittivity Eq, permeability Py and speed of

ww
light c, the dimensionally correct equation(s)
ib)^ B.dl = py/ + P(, E ^j>E.d7 0 is(are):
5 dt ^
(a) Po/2 = Gq {b) e Q / - Po V

FF loo
f ^ » d (
(c) () E.dl =- —() B.ds (c) l = eQcV {d) PoC/ = Eo V
(JEE Advanced 2015)

ree
s s

QQ Multiple Choice Questions (Based on the given Passage/Compreh ension)

reFe
Each comprehension given below is followed by some multiple choice questions. Each question has one

oroFr
r ur
correct option. Choose the correct option. s ff
LgjpmPreTTerTs?^ \ When a capacitor of Answer the following questions :
k
capacitance C after charging with a charge
YYouo
50. If Pq, Pr, e 0 and e ^ as the absolute permeability,
koso

Q is connected to inductor of self inductance relative permeability, absolute permittivity and


L, the oscillations of charge takes place with relative permittivity of the medium, then the
BBoo

time between the two plates of capacitor. If


velocity of electromagnetic wave in a medium is
one plate of capacitor is connected to antenna
r ee

and other plate is earthed, then electro 1 1


magnetic waves are produced, which are ia) ib)
ad
ouur

sinusoidalvariationof electric and magnetic


Yo

field vectors, perpendicular to each other as


well as perpendicular to the direction of 1
propagation of wave. The velocity of these
d

(c) id)
Re
idnY

waves depends upon the electric and mag 0


netic properties of the medium.
FFin

The electromagnetic waves were produced 51. In electromagnetic wave, the average energy
experimentally by Hertz in 1888 using Hertz density due to magnetic field is
Oscillator, which were of wavelength 6;n. (a) 8-85 X 10-*'* Jm-3 (b) 4-42 x 10“"^ Jm“^
Jagdish chander Bose in 1895 produced these
waves which were of wave length 5 mm to (c) 2-21 X 10"“^ Jm-^ (d) 6-63 x 10-“^ Jm"^
25 mm and in 1896, G. Marconi established a 52. Sun also sends electromagnetic waves to earth.
wireless communication between two stations Which one of the electromagnetic waves out of
50 km apart using electromagnetic waves. the visible portion, from sun will be reaching the
In an electromagnetic wave, the amplitude surface of earth earlier than others :
of electric field is 10 Vm“*. The frequency of
wave is 5 X 10*“* Hz. The wave is propagating (a) violet waves (b) green waves
along z-axis. (c) yellow waves (d) red waves

ANSWERS

45. (b,c) 46. (a.c) 47. (a.b.c) 48. (a.b) 49. (a,c) 50. (c) 51. (c) 52. (d)

i
8/70
‘Pr^^dee^'4. Fundamental Physics (XII)EEMD
(c) 5-6 X l()26w (d) 5-6 X 10^0 VV
[sBmpre R^s i
The sun delivers 10^ W 54. Total electromagnetic power that is incident on a
of electromagnetic flux to the earth’s surface, roof of dimensions (8 m x 10 m) on the surface
which is at a distance of 1*5 x 10^^ m from of earth is
the sun. The whole incident electromagnetic (a) 2-56 X 10^ W (b) 64 X 10^^ W
flux is absorbed by the earth. (c)4x 10“^ W (d)Sx 10-* W
Answer the following questions : 55. The radiation force on the roof is

53. The total electromagnetic power of the sun is (a) 2-67 X 10^ N (&)5-34x 10^ N
(fl)5-6x 10-^^W ib) 5-6 X 10--W (r) 2-33 X 10^ N (d) 1-33 X 10^ N

09 Matching Type Questions

ww
DIRECTIONS. In each of the following questions, match column I and column II and select the correct
match out of the four given choices.

FF loo
56. Column I Column II

ree
(A)
(f B.dl (p) Gauss’s law in electrostatics

rFee
(B)
^ B.dl V
/+6q di
i^) Faraday’s laws of electromagnetic induction

oor rF
rur
(C) j> E.dT=-—j> ~B.d7 (r) Ampere Maxwell’s law
k s ff
-> ->
(D) E. ds =— cE pds (5) Ampere’s circuital law
YYoou
ookos

^0 s
BBo

(a) As, h-p, C-q, D-r (h) As, B-q, C-r, D-p (c) As, B-r, C-q, D-p {d) As, B-r, C-p, D-q
re

SI. Various electromagnetic waves are given in column I and various frequency ranges in column II
Column I Column II
ouur
ad
Yo

(A) Radiowaves ip) 1 X 10^^ to 3 X 10“’ Hz


(B) y-rays (Cl) 1 X 10^ to 3 X 10” Hz
Yd

(C)
Re

Microwaves (r) 3 X lO”* to5 X 10^2 Hz


idn

(D) X-rays is) 5 X 10^ to 10^ Hz


FFin

(a) A-p, B-q, C-r, Ds (b) A-q, B-r, Cs, D-p (c) A-r, B-5, C-p, D-q id) As, B-r, C-q, D-p
58. Various electromagnetic waves are given in column-I and various applicationsin column-II.
Column 1 Column II
(A) Infrared waves iP) To treat muscular strain
(B) Radiowaves (^) For broadcasting
(C) X-rays ir) To detect fracture of bones
(D) Ultraviolet rays is) Absorbed by the ozone layer of the atmosphere
ia) A-r, B-q, C-p, Ds ib) A-p, B-q, C-r, Ds (c) As, B-r, C-q, D-p id) A-p, B-q, Cs, D-r

(JEE Main 2014)


ANSWERS

53.(c) 54.id) 55.(a) 56.(c) 57.{d) 58. ib)


ELECTROMAGNETIC WAVES 8/71

K9 Matrix-Match Type Questions


p q >■ s

DIRECTIONS. Each of the following questions contains statements given c;-.-.. j r.---, I'—--1

in two columns, which have to be matched. The answers to these questions A @j Oil©
have to be appropriately bubbled. If the correct matches are A-p ; \-q ;
B-<7 ; B-r ; C-r ; C-5 and D-s, then the correctly bubbled matrix will look
B
i©ll©!l©l©
like the one shown here. C:
©iOilOii©
The various electromagnetic waves are given in column 1 and their uses
©!!@!i0li©
59. D
in column II

Column I Column I!

(A) Ultraviolet ip) To destroy living tissues

w
(B) Infrared (^) Radar system
ir) Eye surgery

Flo
(C) X-rays
(D) Microwaves is) Heating the body muscles

reeee
60. The wavelengths are given in column I and the electromagnetic waves in column II

FFr
Column I Column It

(A) 10-2 ^ ip)

for
Y-rays
ur
(B) 10"^ m (^) X-rays
kkss
II
(C) 10- m ir) UV rays
Yo
(D) lO-'** m is) Microwaves
oo

VI. Integer Type Questions A B C D


eB

© ®®@
DIRECTIONS. The answer to each of the following questions is a single digit
®®oo
r

integer, ranging from 0 to 9. If the correct answers to the question numbers


ou
ad

A, B, C and D (say) are 4, 0, 9 and 2 respectively, then the correct darkening ® ©®®
YY

of bubbles should be as shown on the side :


©©®©
© ©©©
ndd

61. The electric field associated with a propagating electromagnetic wave is described by
Re

the equation £= £q sin (4 Ji x 10^.x- 1-2 TC x !0’^ t) volt/meter, where x is in metre and © © © ©
t in second. The wavelength of the wave in the unit of 10"2 m is :
Fi

© ® ©©
62. The speed of electromagnetic wave in the unit of 10*^ m/s, in a medium of dielectric
constant 2-25 and relative permeability 4 is : ©@©®
63. The dielectric constant of air is 1 006. The speed of electromagnetic wave travelling in © ®© ©
air is fl X 10^ ms-^ where a is about: © ® ©©
64. A flood light will be covered with filter that transmits red light. The electric field of the emerging beam is
represented by a sinusoidal plane wave
£^ = 38-8 sin(T2x lO^z-S-bx lO'^ r) V/m.
The average intensity of beam in watt/m^ will be :

ANSWERS

59. A-p, r ; B-r ; C-p ; D-q 6U. As ; B-^. /●; C-p, q ; D-p 61.(5) 62.(1) 63.(3) 64.(2)
8/72
Fundamental Physics fXiniyzsTTt

VII.
Assertion-Reason Type Questions
FOR MEDICAL STUDENTS FOR ENGINEERING STUDENTS
DIRECTIONS. In each of the following questions DIRECTIONS. In each of the following que.stions,
read the two statements and choose if read the two statements and choose if
(A) both Assertion and Reason are true and the Reason
(A) Statement-1 is true ; Statement-2 is true ; Slale-
is correct explanation of the Assertion.
ment-2 is a correct explanation of Statement-1.
(B) both Assertion and Reason are true, but Reason
is not the correct explanation of the Assertion. (B) Statement-1 is true, Statenient-2 is true ;
(C) Assertion is true, but the Reason is false. Statement-2 is not a correct explanation of
Statement-1.
(D) both, Assertion and Reason are false.
(C) Statement-1 is true, Statemcnt-2 is false.

ww
65. ASvSertion. Displacement current is a fictitious
current and has nothing in common with the (D) Statement-1 is false, Statement-2 is true.
conduction current.
70. Statement-1. Short wave band are used for

FF loo
Reason. It was proposed by Maxwell only to
satisfy Kirchoff’s function rule. transmission of radiowaves to a large distance.

ree
Statement-2. Short waves are reflected from
(a) A (/))B (c)C (</)D
ionosphere.
66. Assertion. Environmental damage has increased
(a) A

reFe
the amount of ozone in the atmosphere. (b)B (c)C (d)D

oor rF
Reason. Increase of ozone increases the amount
71. Statement-1. In an electromagnetic wave, the
rur
of ultraviolet radiation on earth.
direction of the magnetic field induction g is
s ff
(a) A {b)B (c)C (d)D
parallel to the electric field E .
67. Assertion. A changing electric field produces a
k
magnetic field. Statement-2. Electric field vector E and mag
YYoou
okso

Reason. A changing magnetic field produces an netic field vector 5 , have the same frequency.
BBoo

electric field.
(fl)A 0)B (c)C (d)D
r ee

(a) A (b)B (c)C (^D 72. Statement-1. Magnetic field lines cannot start
68. Assertion. Like light radiation, thermal radiations from a point nor end at a point.
ouur
ad

are also electromagneticradiation. Statement-2. The line integral of magnetic field


Yo

Reason. The thermal radiations require no induction over a closed path is not zero.
medium for propagation. (AIIMS 2010) (a) A (b)B (c)C {^/)D
73. Statement-1. Out of the following radiations ;
d

(a) A (b)B (c)C (d)D


idnY
Re

69. Assertion. A charge moving in a circular orbit microwave, ultraviolet and A:-rays, microwave has
the shortest wavelength.
FFin

can produce electromagnetic wave.


Statement-2. The microwave do not deviate from
FIGURE 8(CF).1 the obstacles in their path while going from one
location to another.
(a) A (b)B (c)C (d)D
74. Statement-1. Gamma rays are more energetic than
X-rays.
Statement-2. Gamma rays are of nulcear origin
but X-rays are produced due to sudden
Reason. The source of electromagnetic wave
should be in accelerated motion. (AIIMS 2015) deceleration of high energy electrons while falling
on a metal of high atomic number.
(a) A {b)B (c)C id)D (a) A (b)B (c)C (d)D

ANSWERS

65. (d) 66. (d) 67. (b) 68. (b) 69. (a) 70.(a) 71. (d) 72. (c) 73.(d) 74. {b)
ELECTROMAGNETIC WAVES 8/73

For Difficult Questions

Multiple Choice Questions (with one correct Answer)

1. Here, C = 20 x 10“^ F ; dV/dt = 3 V/s 4. The frequency of e.m. wave remains unchanged
but the wavelength of electromagnetic wave
As, q = CV
changes when it passes from one medium to
another.
or ^=c dV

dt dt I I 1
c « and V CC

Conduction current,

ww
/ =^ = C dV = 20xl0"^x3
c
dt dt
c
= F-2
\1

Floo
V
= 60 X 10-^ A = 60 nA ro
Displacement current, lj = = 60 p.A

ree
c
^=2 or
6-3 vX' X' 2
2. =^
V
= 2-1 X 10”^ T

rFee
c 3x10^ 5. X = 3 mm = 3 x 10“^ m ; = 66 V m“*

F
-4

oor r
E, 66
rur
B=Bk = 2-lxlQ-^kT B, = 2-2 xlO""^!
3x10“
s ff
3. Electric field of an electromagnetic wave in free
space is given by As electromagnetic wave is propagating along
osk
.x-axis and electric field variation is along
YYoou
E = 10cos (\0^t + kx)j y-direction, the magnetic field variation is along
oook

z-direction.
eBB

which is acting along y direction. As E is vai7ing


Using the relation for hannonic wave
with X and t, hence propagation of electromagnetic
2?t
wave takes place along - x axis. Thus statement E = cos — {ct - X)
uur r

0
(4) is wrong. >' K
ad

Comparing the relation. £ = 10 cos (10^ t + kx)


Yo

2nc
with standard equation of electromagnetic wave E =
0
cos (t-xlc)
X
2%
dY

E = EqCos—(ur + x:)
Re

271x3x10’
X
idn

Ey = 66 cos (t-x/c)
3x10“^
FFin

2kv 2k
= Eq cos f +—X = 66 cos 2 7t X 10^^ (t-x/c)
X X I
2k c
We have, Eq = 10 V/in. Thus statement (3) is and B^ = BqCos (t-x/c)
correct. X

= 2*2 X 10"^ cos 2 7C X 10^^ (/ - x/c)


= 10”^ or 27tx(3xl0°) = 10"^
27Tt>
Thus, option (d) is correct.
X X
N

or 3. = 60x =60x— = 188-4 m


22
6. Given, E^ = 0, E^ - 2-5-^
7

Thus, statement (1) is correct. rad \ (- rad


X cos 2tcx10^ t- TtXlO"^ X ,
m s
27t _ 1 = 0-033 rad/m
X 60k 30 Ej = 0. This shows that the wave is propagating
Thus, statement (2) is wrong. along positive x-axis.

I
8/74
^>uide^'4. Fundamental Physics (XlI)ESSZm
C V V c
For Difficult Questions or V = —
X0 X XJ2 2
0
Comparing the given equation with
1 I 1 I
E^. = £q cos (o)/ - kx) or or

we have, co = 2 ;t x 10^ ^2 4
2 7C V’ = 2 7C X 10^
9. Here, £o= 1-6 x IQ-^T;
or

V = 10^ Hz
£o = cSy = (3x 10*^) X I-6x 10“^
or

2k = 4-8x iO-T
and = A = 71X10 ^
X
2tc
From the given relation for B , the
or X = = 200m electromagnetic wave is propagating along
Tix 10 - A

negative ::-axis, -k direction.

ww
7. The velocity of propagation of em wave in a
medium is given by
In electromagnetic wave, £ and B are
V =ExB peqjendicular to each other, so

Flo
-i A -4 —> A A
Here V =V f and £ = £j £●£ = 0 or E B =0

e
—*

vt=(Ej)xB

eree
The direction of unit vector of B is
A
A ^
This relation is true, if B=Bk. i.e., the

FFr
B={2i+j). Let unit vector of £ be
direction of oscillating magnetic field of the

oorr
uur r
electromagnetic wave will be along £ = {ai +hj)-
+ Z-directioii.
sf
8. When electromagnetic wave is travelling in air, then As, £■£ = 0. So, {at+bj) ■ (2? + j) =0
sk
or 2a + b = 0.lf will be so, if (i)a= \ and b = -2
Yoo
£
A (Z
= £q, xcos 271V - -r
oook

I A A A
ora = - 1 and b = + 2. If means £ = {i - 2j) or
eBB

A
vz (~U2j)
£j=£qjJ:cos 2ti 2tcv/
c
The direction of wave propagation will be parallel
uurr

A
(az
to (£x5). Thus ExB = (-/ + 2/) x (2f+;)
ad

= £a,a:cos
01
0)7 [ ●●● 0) = 2 Tcv]
Yo

. c
A A A A
A
= ~k - 4k =-5k , i.e., in the direction of -k.
= £Qj.xcos[itz-0)/]
dY

Thus, option (a) is true.


Re

A 0)

= £q| .rcos[/^2 - ket] ...(/) vk = - 10. Energy contained per unit volume through which
innd

c
electromagnetic wave is propagating is sum of
FFi

When electromagnetic wave is travelling in energy of electric field and energy of magnetic
medium, then
1 B^
A field, i.e., u -Ue+Uq =-€a £-+-
0
£3 = £q2-'^cos[/: (2z -c7)] 2
2 fto
A
where £ and B are sinusoidal function
= £(p xcos[2kz-kct] i.e., £ = £q sin 0) (/--c/c)
A
and B = Bq sin O) (7 - x/c)
= £q2Jccos[A: z-ket] ...(.ii)
From (/) and (ii), we note that
271
u
£ =^^0 ^oSin^to(7-x/c)
k’ =2 k or or
1
X 2
2 1-cos2o)(7-a:/c)
- 2 ^0 ^0 2
During refraction, frequency remains unchanged,
the frequency of is double than that of
whereas wavelength gets changed. Let v, c be the
velocity of em wave in medium and air electromagnetic wave. Similarly, the frequency of
respectively. Then Uq is double than that of electromagnetic wave.

t
ELECTROMAGNETIC WAVES 8/75

18. Average intensity of electromagnetic waves is


For Difficult Ouostions

I =
11. Here, £q = 100 V/m. //q = 0-265 A/m 47tr2 2^^^ °
As maximum energy flow persccond per unit area 1/2
in electromagnetic wave will be or £■
P
0 “
27t/-“€«C
^0^0 0
= £q//q =(100) (0-265)
^^0 iI/2
1500
= 26*5 W/m^
12. Here, v = 10^ cps, = lO"^ C, /j^ = ? _2rt(3)^x[l/(4nx9xl0^)]x3xl0^
= 100 V/m
dq d
As, ID ~ — iqQ sm cor) = £7o CO cos tor —>
dt dt

ww
19. Direction of polarization x is parallel to electric
For Iq to be maximum, cos tor = 1 —j

So, maximum value oi 1^ = qQ(Si = q^ (2 7t v) fielci E , i-e., X W E and direction of wave


= 10"^x2x3-14x 10^

Flo
= 6-28 A propagation k is perpendicular to both E as

e
13. Here, amplitude of electric field £q = 200 V/m ;
—> ^

ree
well as B , i.e., parallel to £ x £ .
amplitude of magnetising field Hq - 0-365 A/m.
20. Here, £ = 40 cos (fe - 6 x 10^ t)

Fr
We know that the maximum rate of energy flow

rF
S = £()6o/|r = £q X //q = 200 x 0-365 Comparing it with the relation
uurr
= 73-0 W/m^

for
2n
14. Average force = average pressure x area s E = En cos -(z-.O

21 2x(25xl0^)x(15xl0^)
kks
● F = —X A 27t 2te
= En cos vt
8
T^"T
av
Yo
c 3x10
oooo

= 2-5 X 10-® N
2n
15. Momentum transferred in one second {Le., force) we have.
eB

= k
by e.m. wave to the mirror is
2kv
2IA _ 2x6x(30xl0~^) = 6xl0®s“‘ or )ci> = 6 X 10^
ur

and
P = X
3x10^
ad

c
YYo

= 1*2 X 10"*® kg ms"* 6x10


8
6xl0^s *
or k = = 2 ni"*
-1
p 1 V 3x10 ms
16. Average intensity =
dd

4kR-
= -eo£o"Xc
2
Re

P
in

21. Intensity of light, / = = u_ xc


03'
47Cr“
F

P
or £
0 ~
y2nR^SQ c Where -€ £2 .
47ir2 2
3 2P 2x0-1x9x10^
V2x3-14x25x8-85xl0"'-x3xl0**
or
^0 = 47tr"e„ c 1-x3xl0^
0
= 2*68 V m"‘
= 2-45 V/m
^0
17. Here, £j,^. = 6 V/m; or - 6 or £(, = 6 -</2 V/m 22. The intensity of electromagnetic wave is equally
shared with electric field and magnetic field
As, Eq = c Bq components.

^0 6V2 23. Direction of propagation of electromagnetic


or B
0 “ = 2V2xlO"**T
c 3x10® waves is along (£ xB). So taking E xB for
= 2*83 X 10"® T all the given options, we have
8/76
"PtAdetp- 4- Fundamental Physics (XII) PTsm

For Difficult Questions


Direction of propagation of electromagnetic waves
A A
A A
' + /
(a) E xB = (-j+k)><(-j +h = 0
= ExB = [Given]
V2
(b) ExB=U + k)x(.J + h = 0 Using B for the given options and taking cross-
A A

product of £ and we have


(c) ExB={~j + k)x(-j-k) = 2‘i
A
A A A I
- i
(d) E xB ={j + k)x[(-j-k)] = 0 (a) Ex B = kx ^ - J

V2

oww
It is given in the question that the propagation of
electromagnetic waves is along .v-axis, so option A A
(c) is true. ~'+j
(h) Ex B = kx
24. Here. /=10^W/m- : [ Vi Vi Vi

ee
A A

c or B rms
/Ho A

(c) Ex B = k X
A
t-J + J+‘ i+J

FFrlo
^ j Vi
rms
2
^0 c

r
rF
ee
A A

10^x(47txl0"'^) A A A
-‘-J -j + i i-J
B = 647x 10^ T (d) Ex B = kx
[ Vi J
rms
3x10' ^ ■ Vi
ouru
rF
= 10-^T Thus option (c) is true.

ffosor
25. Energy falling per second = 25 x 25 W os k 29. The direction of propagation of electromagnetic
wave is
Energy reflected per second = 0
A A A A A A a A a
Change in energy per second = (25 x 25) - 0 ExB = kx{2i-2j) = 2(kx i)-2{kx j)
ook
= 25 X 25 W
Yo
A A A A
Y
= 2(;)-2(-/) = 2j-H2/
Change in energy in 40 minutes time.
Bo

A A

A£ = (25 X 25) X (40 x 60) Unit vector along ExB is a unit vector along
reeB

Momentum transferred, A A

(2/+2y).
oouY

AE _ 25 X 25 X 40 X 60
ur

A A

P = = 5 X 10"^ Ns If, A =2i+ 2 j ,


3x 10*
ad

c
A A A A

26. Here , v = 23-9 GHz = 23-9 x 10^ Hz


then. A^^ = 4im a+j)
dY

CO = 2 7t V = 2 X 3-142 X (23-9 x 10^) s"' V2^ +2^ V2


nidn
Re

Propagation constant, k = —
CO
Eq
c 30. = — ; For the given electric field of a plane
F
Fi

2 X 3-142 X 23-9x10^ electromagneticwave, the direction of electric


= 0-5 X 10^ m
-I
3x10® A A

field is (●*^+>’). The direction of propagation of


27. £q = c^o = (3 X 10®) X 5 X 10-® = 15 V/m. electromagnetic wave (which is given by ExB)
A A

—♦ ,
Direction of E is perpendicular to both B and is along + ^-direction. The direction of magnetic
^ —> —> —^ -»

ExB .Here B is along+ivey-direction, ExB field which is ±r to (x+y) and (ExB) must be

in the direction (-!t+ y) as per right hand rule.


—>

is along + z-direction. So £ is along -five


The magnetic field of a plane electromagnetic
^--direction, i.e., along i' . Hence, wave is given by
—» A
£ = / =15? V/m
B^^ = -(-x+ y)n\n(kz-(i)i)
28. Direction of polarisation =k = E Thus option (d) is true.
ELECTROMAGNETIC WAVES 8/77

Velocity of em wave in a medium


For Difficult Questions

1
V -
1
31. Speed of light in vacuum, c =

1 I
1 X
Speed of light in medium, v =

c c £
Refractive index, ~
V B

oww
!
X
=^|r^ c (4-5 X10“^) X (3x10^)
or E = B
32. Refractive index of medium is Vl-61x644

e
FFrlo
c
13-5x10^

re
fx = -
V = 4-2 X 10^ Vm"'
3-22

rFee
■\jl^r^r ~ -2 36. Given, £ = 56-5 sin to (/ - x/c) ...(1)

rF
C 3x10^ Comparing it with, E = Eq sin to (f - x/c), we
have :
V = —
2
= xx lO"^ (Given)

fsoor
ouur
£^0 = 56-5 NC-i
30
X = — = 15
skf
So, 1
2
Intensity, I = -F
— e=0 E^c
ooko
Yo
33. Given, I ^o ' = 800 V/m
Y
Bo

Maximum magnetic field vector = - (8-85 X 10"*2) x (56-5)2 x (3 x 10^)


reB

2
£ 800 8
B
0
= -xl0-^T = 4-24 Wm-2
0
3
oY
uur

c 3x10
37. £=15 KeV= 15000 eV= 15000 x 1-6 x IQ-’^J
ad

Max. force on electron in magnetic field is


6-64x10"2‘^x3x10®
dY

F max = ev B 0 he
Wavelength X = —
£ 15000xl-6xl0-*^ J
= (l-6x 10"‘^)x(3 X 102)x -xlO ^
innd
Re

= 0-083 X 10-^ m « 10“' nm


Since, X for x-rays is in between 1 nm to IQ"^ nm,
Fi

= 1-28 X ir*^ N
F

34. Velocity of light in a medium of magnetic so the given radiation is x-rays.


permeability p and electric permittivity e is 38. To observe a particle, the wavelength of light used
1 1 must be smaller than the radius of the particle.
V =
If X is radius of the particle then
>0 ^ ^0 = 3 X 10^ X 10-2 m = 3 X 10-^ m
1 1
X Frequency of e.m wave,

c _ 3x10^ = 10
14
c 1 X~ 3x10“^
Thus to observe the particle, the frequency of
wave should be more than lO''^ Hz i.e. 10*^ Hz.
35. Given, = 1 -61, e ^ = 6-44,
or smaller value of wavelength.
B = 4-5x 10-2 Am-^£=?
8/78
'«j. Fundamental Physics (XII)
II
For Difficult Questions _ CO _ 1-5x10
= 2-39x 10*^ Hz.
27t ~ 2x3-14
39. (0 Audio information can be transmitted in the
Range of microwave is 10^ Hz to 10*^ Hz.
form of optical waves through optical fibres,
41. Given, P = A kW = 4000 W ; n = 10^^ photons/s.
(ii) Very hot objects emit not only infrared
P = nx hv
radiation but also emit visible radiation.

(///) Humans can detect infrared radiations by or


P 4000

feeling the heat on the portion of body exposed nh 1020x6-63x10-34


by radiations.
= 6x 10*^ Hz.
Therefore, all the given statements are not
completely correct. Hence the option (d) is This frequency radiation belongs to ultraviolet
correct. rays.

ww
40. Here, = 2x1sin (0-5 x 10^x+ 1-5 x lO" 0 42. For efficient working of microwave oven, the
frequency of microwave must match the resonant
Comparing it with. By = Bq sin (fcc + cor), we have frequency of the water molecules, then maximum

Floo
(0= 1-5 X 10** : heating is produced.

e
m

eere
Multiple Choice Questions (with One or More than One Correct Answers)

FFr
43. Note that an electromagnetic wave undergoes a 49. Dimensions of eg = [M L~^ I-] ;

oorr
uur r
phase change of 180® on reflection. From a denser
P0=[AfLr-2/-2];
medium, i.e., a medium that has a larger refractive
s ff
v=[ml2t'-3 r’]
index than one in which it is travelling. However,
sk
(a) L.H.S., /2 = {MLT-- H) fi = [MLT~^
YYoo
there is no phase reversal if the case is opposite.
ooko

44. Here, /c = 2 7i/iV;a)=27tv = 2 KdX R.H.S., eg ^-3


eBB

k lull 1 x(A/2L4r-^r2)
so = constant
(0 2jtv c = \MLT -2]
uurr

So, it is independent of wavelength. [L.H.S.] = [R.H.S.], hence option (a) is true.


ad
Yo

45. Total energy of electromagnetic wave is (b) L.H.S., e 0 / = (M -* L-3 7 4 /


M = ȣ + Uq
dY
Re

R.H.S., Po V= [M77“2 r2] X [ML^T~^ r‘]


nind

1 1^0
ms _
= \M~L^ 7-5 /-3]
4^0 ^0 4-
+
FFi

rms
2 ^0
[L.H.S.j [R.H.S.J, hence option (b) is wrong,
46. Cosmic rays and P-rays arc not electromagnetic (c) L.H.S.,I/I = /
waves.
R.H.S.. eg CV= [M-* L~^T^ [77-*]
47. The equations {a) and (c) are related to source of
x[ML2 7-3r*] = /
electric field and equation (/>) is related with
source of magnetic field. [L.H.S.J = [R.H.S.], so option (c) is true.

48. E = /jv or E oc V : Since the energy of microwave, id) L.H.S., Po a = [MLT-^ r2] [77-*J [/]
ultraviolet rays and X-rays is increasing order, = [M72 7-3r']
hence the frequency for them is also of increasing R.H.S.,Gg V=[M-'7-3j-4/2]
order. Similarly, the frequency for radiowaves,
infrared radiations and visible light is in the = L~' Tf]
increasing order. [L.H.S.J ^ [R.H.S.J, thus option (d) is wrong.
ELECTROMAGNETIC WAVES 8/79

r-y'rr. ■ fvi'’
For Oifficutt Questions

DU Multiple Choice Questions (Based on the given Passage/Comprehens ion)

w
50. Velocity of e.m. wave, in medium, c c ^ B C
-and li=A + ^+^
1 1 52. p = - or V =
V = V

As > X.J, so Pr < IV ’ therefore

e
> v.
51. Average energy density due to magnetic field in
53. Total electromagnetic power of sun

e
e.m. wave is

wr
B \2
1^0 = 103x4tix(1-5x 10^')^

lloo
1

r
U
ms _ 0
= 5*6 X 10^^ W
2 1^0 ^^0 J ^ ^^0

F
1(£q/c)^_ 54. Total electromagnetic power incident on roof on

FFu
the earth = 10^ x area of roof
4 Pq 4\i^c^
1 E}0 1
= 10^ X (8x10) = 8xlO‘*W
4^0 ^0 55. Radiation force on the roof

rese
uro
k
Fr
= ix(8-85xl0 -12
)x 102=2-21 X 10-10 Jm-3 total power on roof _ 8x10'^

o
4
= 2-67 X 10-^ N
velocity of light 3 x 10^
09
foo
fr
Matching Type Questions
kso
Y
56. Knowledge based question. y-rays ; 3 x 10*^ to 5 x 10^2 Hz
Y
B
Yo
57. The frequency rangCvS of various waves are as Microwaves ; 1 x 10^ to 3 x 10*1 ;
oo

under:
X-rays ; I x lO’^ to 3 x lO^* Hz.
eBr

Radiowaves; 5 x 10^ to 10^ Hz ;


e

58. Knowledge based question.


uru
od

Matrix-Match Type Questions


Yo
ad
n

59. Knowledge based question. 60. Knowledge based question.


ndi

VI.
Re

Integer Type Questions


F
Fi

61. Here, E = £Qsin (4 7t x 10^ a: - 1-2 t: xlO*^ /) 63. The speed of etn wave in vacuum is
Comparing it with the equation 1
£ = £q sin {kx - o)/), we have = 3x 10^ ms-i
0

= 4tcx 10^ Air almost acts as vacuum, therefore


X
3x 10^ = (2X 10^
2n or
a = 3 approximately
or X= = 5 X 10-2 m
47CX10®
1 2
64. Average intensity, I = -BqEqXC
62. The speed of em wave in a medium is given by
av

1 1 c 3x10“
i> =
_ (8-85X10~‘2)X (38-8)^ x (3x 1Q*)
^/iH >o^re0^r \4^r^r ”^4x2-25 2
= 1 X 10* ms
-1
= 2 W/m2
8/80 Fundamental Physics (XII) twiil

For Difficult Questions

VII. Assertion-Reason Type Questions

FOR MEDICAL STUDENTS FOR ENGINEERING STUDENTS

65. Displacement current is not a fictitious current at 70. Both Statement-1 and Statement-2 are correct.
all but is a fundamental fact of nature. It produces Here the Statement-2 is correct explanation of

oww
Statement-1.
magnetic field in the same way as the conduction —>

current does. Thus Assertion is false. 71. In electromagnetic wave, the direction of B is
Displacement current was not proposed only to perpendicular to E ■ Hence, Statement-1 is
satisfy Kirchoff’s junction rule. However the wrong but Statement-2is true.

e
discovery of its existance by Maxwell satisfies
72. The magnetic lines of force form a closed path,

re
FFrlo
the Kirchoff’s junction rule. Hence, Reason is —^

also False. hence the line integral of B over the closed path

rF
ee
66. Both Assertion and Reason are wrong. is zero. Thus, Statement-1 is true and Statement-
2 is false.
67. A changing electric field produces a magnetic

rF
ouru
field. It is accounted by Ampere-Maxwell’slaw. 73. The energy of X-rays is maximum as compared
Here, Assertion and Reason are correct but the microwave and ultraviolet rays. Since energy
Reason is not the correct explanation of
fosor E = hv =
he
—, so X is least for X-rays. Therefore,
Assertion.
skf
X
Statement-1 is wrong. The Statemeni-2 is correct.
ooko
68. Both Assertion and Reason are true. But the
Yo
Reason is not the correct explanation of 74. The Statement-1 and Statement-2 are true but
Y
Assertion.
the Statement-2 is not correct explanation of
Bo

69. A charge moving in a circular orbit is in


reeB

Statement-1. In fact the energy of gamma ray


accelerated state. Hence it emits electromagnetic
is more than X-rays because the frequency of
wave. Thus both Assertion and Reason are true
gamma ray is higher than that of X-rays as
ooY
uur

and the Reason is the correct explanation of


E = hv.
ad

Assertion.
dY
nind
Re
F
Fi
w
e
row
re
FFllo
eeF
u
r
sFr
kro
iir
f.
uor
offo
kos
Y
Yo
eerBB

CHAPTER 9.
oo

RAY OPTICS AND OPTICAL INSTRUMENTS


rY
u

CHAPTER 10.
ou

WAVE OPTICS
ad
do
nY
nid
Re
F
Fi
w
RAY OPTICS AND

Flo
OPTICAL INSTRUMiiNTS

e
reee
A .

FFr
9.1. INTRODUCTION

for
ur
Light, as we all know, is a form of energy that produces in us the sensation of sight. It is mainly through
light and sense of vision that we know and interpret the world around us.
kss
Two basic facts about light are that it travels along straight lines with enormous speed, which is measurable.
The presently accepted value of speed of light in vacuum is c = 2-99792458 x 10^ m/s. It is suffice to take
Yo
oo

c = 3 X 10® m/s, which is a universal constant. Further, the speed of light in vacuum is the highest speed
eB

attainable in nature.

The nature of light has been a subject of controversy. Newton was the first to suggest that light energy is
concentrated in tiny particles, called corpuscles. Corpuscles theory could explain the phenomenon of reflection
ur

of light. Later on, wave theory of light was put forward by Huygens, modified duly by Fresnel. It could
ad
YYo

explain the phenomena like Refraction of light. Interference of light and Diffraction of light. Maxwell
electromagnetic theory of light could explain the phenomenon of Polarization of Light. The phenomena of
photoelectric effect and Compton effect were explained by Planck on the basis of Quantum Theory. Finally,
d
Re

de-Broglie postulated Dual Theory of light combining corpusculartheory and wave theory, which accounted
in

for all observed phenomena in light. In this unit, we shall study the phenomenaof Reflection,Refractionand
F

Dispersion of Light. Finally, we shall describe the construction and working of some important optical
Instruments. A brief account of the phenomena of Interference, Diffraction and Polarizationis given in the
last two chapters of this unit.

9.2. REFLECTION OF LIGHT

1 Reflecii- is the phenomenon of change in the path of light without any change in medium.

A smooth and well polished surface which reflects regularly most of the light falling on it is called a
mirror. By regular reflection, we mean that the reflected light goes in one particular direction (and not in all
directions) corresponding to one particular direction of incidence. Regular reflection is also called Specular
Reflection. A surface acts like a smooth surface only when height and separation of irregularities on the
surface are small compared to the wavelength of light falling on the surface. The surfaces which are not smooth
9/1
9/2 “P^euUe^ 'a Fundamental Physics (XII)
reflect light incident in one direction into many
directions. This type of reflection is called diffuse
reflection. In day to day life, we observe our
surroundings through diffuse reflection. In this
chapter, we shall study mainly the specular reflection.
In Fig. 9.1, Mj is a plane mirror. A ray of light
falls on the mirror along AO at ZAON = /, where ON
is nonmil to the mirror at O. It is reflected along OB
at ZNOB = r. Here, i is called angle of incidence
and r is called angle of reflection.
According to the laws of reflection,

ww
(/) Angle of incidence is equal to angle of reflection i.e.
Zi = Zr

Flo
(//) Incident ray (AO), reflected ray (OB) and normal (ON) to the mirror, all lie in the same plane.
For normal incidence, Zi = 0®. Therefore, Zr = 0°.

e
reree
Hence a ray of light falling normally on a mirror, retraces its

r FF
path on reflection.
Note that in a plane mirror, image formed is virtual, erect and at
uurr
the same distance behind the mirror as the object is in front of the
mirror. Also, image in a plane mirror is always laterally inverted.* foor
ks s
When a point object is held inbetween two plane mirrors A/j and
Yoo
ooook

M2, inclined at Z0, more than one image are formed on account of multiple
reflections of light from the mirrors, In general, total number of images
eBB

(n) formed is given by


uurr

360® 360 . 360 360


n - -1 , if is even integer and is an odd integer
ad

n =
e e 0 0
Yo

360°
dY

For example, when angle between two plane mirrors is 0 = 60°, n = -I=6-l=5
60
Re
innd

i.e., five images of one point object held inbetween and M2 are seen.
And when 0 - 40°
FFi

360
n -
40
y, i.e., nine images are seen ; and so on.

Retain in Memory
1. For a given incident ray, when a plane mirror is turned
clockwise or anticlockwise through an angle 0 ; the reflected
ray turns in the same direction, clockwise or anticlockwise,
through an angle 2 0.
2. When a ray of light is incident on a plane mirror at an angle 0,
the reflected ray deviates through an angle 6 = fjc - 2 0). This
is illustrated in Fig. 9.3.

*It means left side of object becomes right side of image and vice-versa.
RAY OPTICS AND OPTICAL INSTRUMENTS 9/3

Sample Problem A point object is held between two plane mirrors held at (/) 24° (h) 30°.
What is the number of images formed in the two cases ?
Sol. (/) When 0 = 24°
n -
360 _ 360 = 15
0 ~ 24
(ii) When 0 = 30°
360 360
n — -1 = -1 =12-1 = 11
0 30

9.3. SPHERIGVL MIRRORS

A spherical mirror is a part of a FIGURE 9.4


hollow sphere, whose one side is

ww
reflecting and other side is opaque. Two Mi MiV
/ /\ \
types of spherical mirrors are : /
/ s
✓ \ \
1. Concave mirror, whose / ✓

s
s
\
reflecting surface is towards the centre

Flo
P P \
of the sphere of which the mirror is a \ /

ee
N /
s
part, Fig. 9.4(a). \
V ✓
/

rere
s ✓

2. Convex mirror, whose


s ✓
\

r FF
/
reflecting surface is away from the centre
of the sphere of which the mirror is a © —- ©
uurr
part. Fig. 9A{b).
foor
The centre of curvature (C) of spherical mirror is the centre of the sphere of which the mirror forms
R

a
ks s
part.
Yoo
oook

The middle point or centre of the spherical mirror is called vertex or pole of the mirror. It is represented
by .P
eBB

Radius of curvature (R) of the spherical mirror is the radius of the sphere of which the mirror forms a
part. Thus PC = R.
uurr

Normal to the .spherical mirror at any point is the line joining that point to the centre of curvature C of
ad

the mirror.
Yo

The diameter M2 of the .spherical mirror is called aperture or linear aperture of the mirror.
dY

Angular aperture of a spherical mirror is the angle Mj CM2, subtended at C by the diameter of the
Re
innd

spherical mirror.
The straight line joining the pole and centre of curvature of spherical mirror extended on both sides is
FFi

called principal axis of the mirror.


A section of the spherical mirror cut by a plane passing through pole and centre of curvature of the
mirror is called principal section of the mirror.

9.4. THE NEW CARTESIAN SIGN CONVENTIONS

In dealing with reflection at spherical mirrors, we shall adopt the following system of sign.s-called the
New Cartesian Sign Conventions :
1. The principal axis of the mirror is taken as X-axis and the pole is taken as origin. All the distances
are measured from the pole (P) of .spherical mirro.r
2. The distances measured in the direction of incidence of light are taken as positive, and the distances
measured in a direction opposite to the direction of incidence of light are taken as negative.
3. The heights measured upwards and perpendicular to the principal axis of the mirror are taken as
positive and vice-versa.
9/4 “P-Kuice^ ^ Fundamental Physics (XII) PTSTini
These are illustrated in Fig. 9.5.
FIGURE 9.5

Real M Concave
Incident
object 4 ^ mirror
light
-R
/J2(+)
+ »
0 c F P(Origin) Virtual image
-f

Real image h2(-)M ~v +v >1


K - u M’
o Distances opposite > Distances along
to direction of direction of
incident light (+)

w
incident light (-)

Real A- Incident Convex mirror * Virtual


object light image

Flo
* R

hi(+) *f
hzi*)

e
+ +

ree
O (Origin) P F C

FFr
O K - U
●H +u
M'
urr
Distances opposite -4 > Distances along

or
to direction of direction of
incident light (~) incident light (+)
sf
kks
Retain in Memory
Yo
ooo

1. As an object is held in front of a spherical mirror, the distance of the object (u) is always negative.
2. In a spherical mirror, real image is formed in front of the mirror. So its distance (u) from the
eB

mirror is taken as negative. Again, virtual image is formed at the back of the mirror. So its distance
(v) from the mirror is taken as positive.
ur

3. On the same basis, focal length and radius of curvature of a concave mirror are taken as negative.
ad

The focal length and radius of curvature of a convex mirror are taken as positive.
YYo

9.5. PRINCIPAL FOCUS, FOCAL LENGTH AND


dd

RADIUS OF CURVATURE OF A SPHERICAL MIRROR


Re
in

Principal Focus (F) of a spherical mirror is a point on the principal axis of the mirror at which, rays
incident on the mirror in a direction parallel to the principal axis actually meet or appear to diverge after
F

reflection from the mirror.


In case of a concave mirror, F is a real point where the rays after reflection from the mirror actually
meet, Fig.9.6(a)
FIGURE 9.6

Concave mirror
RAY OPTICS AND OPTICAL INSTRUMENTS 9/5

In case of a convex mirror, Fis a. virtual point, wherefrom, the rays after reflection appear to diverge.
Fig.9.6(b).
The distance of principal focus Ffrom the pole P of the spherical mirror is called focal length (f) of the
mirror

i.e., PF=f
A vertical plane passing through the principal focus and
perpendicular to the principal axis is called focal plane.
In Fig. 9.7, we have shown a parallel beam of light falling on a
concave mirror at a small angle to the principal axis. It converges to a
point F in the focal plane of the mirror.
The distance of the centre of curvature C of the spherical mirror
from its pole P is called Radius of curvature of the mirror. It is

w
represented by R
i.e., PC = R

Flo
9.6. RELATION BETWEEN/AND R

e
{a) Concave Mirror

rree
In Fig. 9.8, P is pole, C is centre of curvature and F is principal focus of a concave mirror of small apertuie.

r FF
Let a ray of light AB be incident on the mirror in a direction parallel to the principal axis of the mirror. It gets
reflected along BF. Join CB. It is normal to the mirror at B.
uurr
ZABC = /, angle of incidence
./CBF = r, angle of reflection for
kss
Now ZBCF = ZABC = i (alternate angles)
ooook

In A CBF, as i = r (law of reflection)


Yo

CF=FB
eB

But FB - FP (aperture is small)


CF = FP, i.e., F is the centre of FC
urr

I
PF = - PC , Using sign conventions, PF = -/and PC = -R.
ad
Yo

Therefore, -f=-RI2 or I/= R/2


dY

i.e., focal length of a concave mirror is equal to half the radius of curvature of the mirro.r
Re
innd

(J?) Convex Mirror


In Fig. 9.9, P is pole, C is centre of curvature and F is the
Fi

principal focus of a convex mirror of small aperture.


Let a ray of light AB be incident on the mirror in a direction
parallel to the principal axis of the mirror. It gets reflected along
BD and on producing back, it appears to come from the principal
focus Fof the mirror. Join CB and produce it to N. This is normal
to the mirror at B.

ZABN= i, angle of incidence


ZDBN = r, angle of reflection
Now ZFBC = ZDBN = r (opposite angles)
ZBCF = ZNBA = i (correspondingangles)
In A CBF, as i - r (law of reflection)
CF=FB
But FB = FP ('.● aperture is small)
CF = FP or F is middle point of PC
9/6 'a Fundamental Physics (XII) tviwii
1
PF = -PC ,
2

Using sign conventions, /= RH


ie., focal length of a convex mirror is equal to half its radius of curvature.
Note that both,/and R are positive for a convex mirror and negative for a concave mirror.
Sample. Problem (a) What is focal length of a convex mirror of radius of curvature 20 cm ?
(b) What is radius of curvature of a concave mirror of focal length - 50 cm ?
Sol. (a) Here, /=?/? = 20 cm
R 20

^ 2
— = 10 cm
2
{b) Here, /=-50 cm,/? = ?

w
/? = 2/= 2 (-50) = - 100 cm

Flo
9.7. MIRROR FORMULA FOR CONCAVE MIRROR

Mirror formula is a relation between focal length of the mirror and distances of object and image from

reeee
the mirror.

FFr
Let P be the pole, ,F the principal focus and C, the centre of curvature of a concave mirror of small
aperture. Let PF =f be focal length and PC = R he radius of curvature of the mirror.
Depending on the position of the object, the image formed may be real or virtual,
for
ur
(a) Real Image*^
When the object is held in front of a concave mirror beyond the principal focus F of the mirror,
kkss
image formed is real. In Fig. 9.10, an object AB is held L to the principal axis of the mirror beyond C.
Yo
oo

A ray of light starling from A and incident on the mirror along AD parallel to the principal axis, gets
reflected from the mirror and passes through .F Another ray of light incident along AP is reflected along PA'
eB

such that ZAPB = Zi = ZBPA' = Zr. The two reflected rays actually meet at A' which is real image of the
point A. A third ray starting from A and incident on the mirror along AC falls normally on the mirror and
r

retraces its path ; meeting the two reflected rays at A'. From A' draw A'B^ X on the principal axis. Therefore,
ou
ad

A'W is real, inverted image of AB formed by reflection from the concave mirror.
YY

As As ABC and A'B'C are similar

AB _ CB
ndd

...(1)
Re

A'B' ~ CB'
Fi

Again as As ABP and A'B'P are similar


AB _ PB ...(2)
A'B' ~ PB'

FromU) nd (2),
CB _ PB ...(3)
CB' ~ PB'
Measuring all distances from ,P we have
CB = PB- PC
CB' = PC- PB'

.●. from (3), PB-PC _ PB ...(4)


PC - PB' ~ PB'
*A real image is one through which light passes actually or it is the image formed by actual intersection of
light rays. A real image can be taken on a screen held at the location of the image.
RAY OPTICS AND OPTICAL INSTRUMENTS 9/7

Using New Cartesian Sign Conventions,


PB = - u, distance of object
PC = -R

PB' = - u, distance of image


-u + R — u
we get from (4),
-R + v — V

or ■¥uR-uv = uv-vR or iiR-\-vR-2uv


Dividing both sides by « n /?, we get

- + - --
V u R

loow w
As R = 2f i+1-^ 2 1 , which is the required mirror formula.
V u R f
(b) Virtual Image*
When the object is held in front of a concave mirror between the pole P and principal focus F of the

ree
mirror, the image formed is virtual, erect and magnified as shown in Fig. 9.11.
Proceeding as above, we get from (3),

rree F
CB _ PB
CB' ~ PB'
r FF ...(5)
fofr oF
u
Measuring all distances from P, we get
ks
CB = PC- PB
YYouro
soo

CB' = PC + PB'

PC-PB PB
BBook

From (4),
PC + PB' ~ PB'
r ee

Using New Cartesian Sign Conventions,


ouru
ad

PB = - u, PB' = + v
Yo

PC = -R

— R + It — u
d
Re

-R + v
iYn

V
FFind

or uR -14 v=-vR + uv
uR+vR-2uv

Dividing both sides by uv R, we get

1+i-^ 2 1 which is the required mirror formula.


V u R 2f f

9.8. MIRROR FORMULA FOR CONVEX MIRROR

The image fanned in a convex mirror is always virtual and erect, whatever be the position of the object.
Let P be the pole, F, the principal focus, and C, the centre of curvature of a convex mirror of small
aperture, Fig. 9.12.
Let PF = /, the focal length and PC = R, the radius of curvature of the mirror.
*A virtual image is that which cannot be captured on a screen. This is because light rays do not pass actually
through it. They only appear to be coming from it.
9/8 Fundamental Physics (XII)MtsiaiJ

AB is an object held in front of mirror


perpendicular to its principal axis.
A ray of light starting from A and incident
on the mirror along AD in a direction parallel to
the principal axis is reflected along DE. On
producing back, DE appears to come from F.
Another ray from A incident on the mirror along
AP is reflected along PG such that ZAPB = ZA =
ZBPG = Zr. On producing back, PG meets DF at
A'. Therefore, A'is virtual image of A. The third ray
incident on the mirror along AC falls normally on
the mirror and retraces its path on reflection. This

ww
also appears to come from A'. Draw A'B' _L to
principal axis. Therefore, A'B' is virtual image of
the object AB formed after reflection at the convex

Flo
mirror.

e
As As ABC and A'B'C are similar

eree
AB _ CB

FFr
...(6)
A'B' ~ CB'

oorr
uur r
Again, as ABP and A'B'P are similar
sf
AB _ PB
...(7)
sk
ICB' ~ Jb'
Yoo
oook

CB _ PB
eBB

From (6) and (7), ...(8)


'CB'~ PB'
Measuring all distances from the pole ,P we can write
uurr
ad

CB = PB + PC
Yo

CB' = PC- PB'


dY

PB^PC _ PB
Put in (8), ...(9)
Re

PC -pb'^Tb'
innd

Using New Cartesian Sign Conventions,


FFi

PB = ~u, distance of object


PB' = + v, distance of image
PC= + R

-u + R — u

from (9),
R-v V

-uv + vR=--uR + uvov V R + uR = 2uv

Dividing both sides hy uv R, we get


vR uR 2uv
H — ■■ ■
uvR uvR uvR

2 2 1
, which is the required mirror formula. ...(10)
U V R 2f f

f
RAY OPTICS AND OPTICAL INSTRUMENTS 9/9

lin in Memory

1. Mirror formula or mirror equation, i.e., — = i- + — is same for both, the concave and convex
/ u

mirrors. The equation remains unaffected whether the image is real or virtual.
2. Every part of a mirror, howsoever small forms a complete image of object. Therefore, even if any
part of minor is painted black, it will form full size image of the object.
3. Brightness of image is determined by the amount of light reflected by the minor, i.e., by the
aperture of the minor.

Sample Problem An object is placed 18 cm in front of a mirror. If the image is formed at

ww
4 cm to the right of the mirror, calculate its focal length. Is the mirror convex or concave ? What is the
nature of the image ? What Is the radius of curvature of the mirror ? ●. nSE 2011'

Sol. Here, M = - 18 cm, V = 4 cm,/= ?

Floo
From 1 1 1 I 1 _ 9-2 _ 7
/ V u 4 18"36~36

ree
36
/ = — = 5-14 cm

rFee
7
As focal length is positive, the minor must be convex. The image is virtual, erect and smaller in .size.

F
oor r
rur
R = 2/= 2 X 5-14 cm = 10*28 cm s ff
9.9. LINEAR MAGNIFICATION OF A SPHERICAL MIRROR
k
YYoou
> Linear magnification or simply magnification of a spherical mirror is the ratio of the size of the
ookos

^ ^ image formed by the mirror to the size of the object.


BBo

It is represented by m
re

Thus,
size of image (/ij) a'B'
ouur

m =
ad

size of object (/ij) AB


Yo

We assume that is always positive. Magnification, (m) will be + when /12 is + ii.e., image is erect) and
dY

(m) will be negative, when /12 is negative {i.e. image is inverted).


Re
idn

A'B' _ PB'
FFin

From eqn. (2),


AB ~ PB
In case of concave mirror, when image formed is real as shown in Fig. 9.10, using new cartesian sign
conventions, A'B'=-h2, AB = + h^
PB'=-v,PB=-u

. zh- -V V
m = Zh- — , V
i.e., m is negative.* or m = —
V
...(11)
— u u u u

When image formed is virtual as shown in Fig. 9.11, using new cartesian sign conventions,
A' B' = + /t2, AB = + h,
PB' = + v,PB = -u

from m = A'B' _ PB' V


we get, m = — , i.e., m is positive.
AB ~Tb -u

*The positive or negative sign of m is determined from the sign of h-jh^.


9/10 'Pna.eUefi, 4. Fundamental Physics (XII) kV»1WII
In case of convex mirror, Fig. 9.12, using new cartesian sign conventions,
A'fi'= +/i2, A5 = +/i,
PB' = + u, PS = - u

^*2 +V hi V
m = — ~ m = —
— , i.e., m is positive.
'h -u u

Other formulae for magnification :

From mirror formula. 1 + 1- ^


u V /
Multiply both sides by v

oww
V V
-+1 = -
u /
V V v-f

e
rFFlo
u f f

re
V _ f-V

ree
For virtual images in both types of mirrors. m = — ...(12)

F
u f

rF
Again, multiplying both sides of mirror formula by u, we get

fsfoor
ouur
V f
kosk
u u u-f
V f f
Yo
oo
Y

f
BB

V
...(13)
u u-f
rre

V V f
As m = - m = -
oYuu

u u f-u
ad

Sample Problem An erect image 3 times the size of the object is obtained with a concave
dY

mirror of radius of curvature 36 cm. What is the position of the object ?


innd

Sol. Here, m =+3 (image is erect)


Re

P = -36cm, M = ?Letu--x
Fi
F

As
hi
w = — = —
+v = 3 v=-3u=3x
-u

As
1 l-l-l 1 1 2

u v~ f~ R -X 3x -36

-3 + 1 1
or 3x = 36 x= \2 cm.
3x 18

M = - 12 cm

9.10. RELATION BETWEEN THE SPEEDS OF OBJECT


AND IMAGE FORMED BY A SPHERICAL MIRROR

The mirror formula is 1 + 1- ^ ...(14)


V u /
RAY OPTICS AND OPTICAL INSTRUMENTS 9/11

Differentiating both sides w.r.t. time (t), we get


1 dv 1 du
— =0
dt u~ dt
(●.● /= constant)
n2
1 dv 1 du dv V du
or or .(15)
V u 2 dt dt u dt
dv du
As = V., speed of image, and —= > speed of object
dt ● J*
dt V

\2
V
From (15), V. =-
I
V
0
...(16)
II

ww
\2
V / K =-
/
From (13), I
V,0 ...(17)
u U-f M-/
This is the required relation.

Flo
e
9,11. PRACTICAL APPLICATIONS OF SPHERICAL MIRRORS

ere
Following are some of the practical applications of spherical mirrors :

FFr
(a) Convex Mirrors
1. A convex mirror is used as a reflector in street lamps. As a result, the light from the lamp diverges
uurr over

orr
a large area.
2. A convex mirror is used as a driver’s mirror in all
vehicles like cars and scooters etc., for looking at
sfo
the traffic at the rear of the vehicle. Such a mirror
kks
Yoo
has a much wider field of view as compared to a
oooo

plane mirror or a concave mirror. The images


fonned are smaller and erect.
eBB

Fig. 9.13 illustrates larger field of view of a convex


mirror as compared to the field of view of a plane
urr

mirror.
ad

{b) Concave Mirrors


YYo

1. A concave mirror is used as a reflector in search light, head lights of motor vehicles, telescopes, solar
cookers etc.
dd

2. A concave mirror is used in the ophthalmoscope, for reflecting light on to the retina of the eye.
Re
inn

3. A concave mirror is used as head mirror by ENT specialists.


4. A concave mirror is also used as a shaving mirror/make up mirror as it can form erect and magnified image.
F

9.12. BASICS OF REFRACnON

When a beam of 1 ight travelling in one transparent


medium falls on another transparent medium, a part of
light gets reflected back into the first medium and the
rest of light enters into the other medium, as shown in
Fig. 9.14. The direction of propagation of an obliquely
incident ray of light that enters the other medium
changes at the interface of two media. This phenomenon
is called refraction of light. Hence,

Refraction of light is the phenomenon of change


in the path of light, when it goes from one
medium to another.
9/12 ‘Pxeuieefr ^ Fundamental Physics (XII)
Following are the three laws of refraction :
1. VJhenever light goes from one medium to anothe,r the frequency of light, and phase of light do not
change. Howeve,r the velocity of light and the wavelength of light change.
2. The incident ray, the refracted ray and normal to the interface at the point of incidence, all lie in the
same plane.
3. The product of refractive index and sine of angle of incidence/refraction at a point in a medium is
coit^nant, i.e., |X X sin i = constant
}j.j sm ij =\i2 sin »2
If /j = / and i2 = r, then jij sin i = p-2 ^

^^2
or

ww
smr

where ^2 represents refractive index of medium 2 w..rt. medium 1.

Floo
This law is called Snell’s Law.

e
eere
More about Refractive Index

Refractive index is a characteristic property of the medium, whose value depends only on nature of

FFr
material of the medium and the colour or wavelength of light. We define

oorr
uur r
(absolute) refractive index (\l) of a medium as the ratio of speed of light in vacuum/air to the
s ff
speed of light in the medium. It is commonly referred to as the refractive index or index of
refraction.
sk
YYoo
ooko

speed of light in vacuum/air (c)


eBB

speed of light in the medium (u)

Refractive indices of some of the substances are given in Table 9.1.


uurr
ad

TABLE 9.1. Refractive indices of some substances


Yo

(a) Solids at 20»C (b) Liquids at 20^C (c) Gases at O^C, 1 atm.
dY
Re

1. Diamond 2-42 1. Water 1-33 1. Air 1 -000293


nind

2. Crown Glass 1-52 2. Ethyl Alcohol 1-36 2. Oxygen 1-000271


FFi

3. Quartz (crystalline) 1-54 3. Carbon disulphide 1-63 3. Hydrogen 1-000139

4. Sodium chloride 1-54 4. Benzene 1-50 4. Carbon Dioxide 1-00045

5. Ice(0°C) 1-31

When light passes from one medium I to another medium 2, the refractive index of medium 2 with
respect to medium 1 is written as and is called relative refractive index.

1 ^2 _ _ V
1 _ speed of light in medium 1
^^2 = c/v
1 i»2 speed of light in medium 2

Similarly, V,=
^^2 dV2
RAY OPTICS AND OPTICAL INSTRUMENTS 9/13

A medium with higher refractive index is said to be optically denser compared to the medium
with lower refractive index. Optical density is the ratio of the speeds of light in two media. However,
optical density should not he confused with mass density, which is mass per unit volume.
It is possible that mass density of an optically denser medium may be less than that of an optically rarer

w
medium. For example, mass density of turpentine oil is less than mass density’ of wate,r but optical density of
turpentine oil is more than optical density of wate.r
Cause of Refraction of Light
The refraction of light occurs because speed of light changes when it goes from one medium to another.

e
Light travels faster in a rarer medium than in a denser medium. When light goes from a rarer to a denser
medium, then from

e
wr
1^2 _ sin i V

oo
I

r
sin r ^2

F
FFllu
Ui >^2 sin t > sm r or i> r or r <i

ray of light bends towards normal.


When light goes from a denser to a rarer medium, then from the same relation.
Uj<U2 ; sin/<sinr or i<rorr>i

rese
ray of light bends away from nonnal.
uro
Fkr
o
IMPORTANT NOTE

foo
1. In going from a rarer to a denser medium, light bends towards normal, i.e., Zr < Zi.

fr
2. In going from a denser to a rarer medium, light bends away from normal, i.e., Zr > Zi.
kso
Y
3. For a ray of light incident normally, there is no change in the path of ray of light, i.e., refraction of
Y
B

light does not occur.


Yo
4. Refractive index of a medium is also represented sometimes by n instead of p.
oo
eBr

5. Relation between wavelength of light in vacuum/air and wavelength of light in any other medium.
rue

Let Xfl be wavelength of light in vacuum/air, be wavelength of light in any other medium,
oud

V be frequency of light, which remains unchanged.


no

If c is velocity of light in vacuum/air, V is velocity of light in medium, then


ad

c = V and u = v A,m
Y
ndi

c vX 0 X X0
0
As P = - X As p> 1, X,„<Xq
Re
F

V vX MI X m
m
Fi

Thus wavelength of light decreases as it goes from vacuum/airto any other medium.

Ssmple Problem Monochromatic light of wavelength 589 nni is incident from air on a water
surface. If p of water is 1*33, find the wavelength, frequencyand speed of refracted light (CBSE2017)

Sol. Here, Xy = 589 nm = 589 x 10"*^ m, p = 1-33


589
x = :::2. = 442*85 nm
1-33

c 3x10^
V — = 5-09 X 10^** Hz
X.0 589x10-^

c 3x10^
i» = — m/s - 2*25 X 10* m/s
p F33
9/14 ‘P'lctdee^ ^ Fundamental Physics (XII) kwii
9.13. REFRACTION OF LIGHT THROUGH A RECTANGULAR GLASS SLAB

In Fig. 9.15, ABCD is a rectangular glass slab. A ray of light is incident along KL on the face AB of the
slab at Z/|. It is refracted along LM at Zrj.
The refracted ray LM falls on face CD at Z»2 and emerges
out along MN at Zr2-
In going from a rarer medium to a denser medium, the ray
bends towards the normal, and in going from a denser to a rarer
medium, the ray bends away from the normal.
Applying snell’s law at L,
sin i^ = ^g X sin r
1

ooww
sin i^ s _ a
or ...(18)
sin ,r1 ■ a

Again applying Snell’s law at M,


\ig X sin i2 = [i-a si" ^2

e
sin

ree
rFl
o _
a ...(19)
sinr2

Fre
rrF
According to the principle of reversibility of light, when final path of a ray of light after any number
of reflections and refractions is reversed, the ray retraces its entire path.
ouur
sffoo
Imagine a plane mirror P held normal to MN so that on reflection from mirror, path MN is reversed. The
ray would retrace its entire path. For the reversed ray, application of Snell’s law at M gives
]X^ X sin r2 = PgX sin
okks
Yo
ooo

sinr^
.(20)
eBB

sim g
a 2

sm .r 1
uurr

Multiplying (19) and (20), we get x = 1 .'. a

g ^^8 =
sinr2 sm ^2
8
ad
YYo

For example, if refractive index of glass w.r.t. air is 3/2, then refractive index of air w.r.t. glass is 2/3.
dd

a
Similarly, if =4/3, then =3/4.
Re

a
iinn

smi
1 _ sm r2
F

From (18) and (20), ...(21)


sm ,r sm 1,
1

Fig. 9.15. shows that '2=''l


sm i2 = sm Tj
From (21), sin r2 = sin iy or
^2“'l
Hence, the emergent ray MN is parallel to the incident ray KL as shown in Fig. 9.15. We observe that the
incident ray KL is displacedlaterally, on suffering two refractions through a glass slab.
Expression for lateral displaccment/shift
Lateral displacement/shift on passing through a parallel sided glass slab is the perpendicular distance
between the directions of incident ray and the emergent ray. To find an expression for it, draw MN' _L KL
produced, Fig. 9.15.
.●. Lateral displacement of the ray on passing through the parallel sided slab = MN.
Let ZMLN' = 8 = deviation on first refraction.
RAY OPTICS AND OPTICAL INSTRUMENTS 9/15

MN'
InALA/A^', sin 5 = MN' - LM sin 5 ...(22)
LM

rw
LL' LL' t
LM =
In A LL'M, cos rj = LM cos ,r
I
cos .r
1

where t = lL' = thickness of glass slab.

From (22), Mf^' =


t
sin 5 or MN' =
rsin (/j -r,) ...(23)

r
cos cosr.
1 I

wo
llou
F
This is the expression for lateral displacement, which is obviously proportional to thickness (f) of
glass slab. Further, lateral displacement will increase with increasing angle of incidence /].
Lateral shift will be maximum, when

FF
s
sin (/| - r|) = maximum - 1 - sin 90°
or
(/, -r|) = 90° If r, =0°./j =90°
uro
ker
e
txl
Also, maximum lateral shift, MN' = = / = thickness of glass slab.

For
1

ro
fof
9.14. REAL DEPTH AND APPARENT DEPTH OF A TANK

o
Y
Y
A water tank appears shallowe,r i.e., less deep than what it actually is. This is on account of refraction
of light.
B
ks
To prove this, suppose O is a point object at an actual depth OA below the free surface of water XY in a
Yo
roo

tank. Fig. 9.16.


eB

A ray of light incident on XY, normally along OA passes straight along OAA'. Another ray of light from
e
u
rd

O incident at Z/ on XY, along OB deviates away from normal. It is refracted at Zr along BC. On producing
back, BC meets OA at /. Therefore, / is virtual image of O, i.e., when seen through water, O appears at /.
o
n

Therefore, apparent depth = lA, which is less than the real depth OA.
ou
ad

ZAOB - ZOBN'= i (alt. Zs)


iY

ZAIB = ZNBC = r (corres. Zs)


nd
F
Re

. . AB
In A OAB, sin i = —
OB
Fi

AB
In A lAB, sm r -
IB

As light is travelling from denser medium (water) to


rarer medium (air)

a
sin r AB OB OB
—X = —
w
sin i IB AB IB

When angles are small, B is close to A


a
OA
w
lA

a _ real depth X
...(24)
w
apparent depth y
9/16 ‘Pn^ctdet^'4^ Fundamental Physics (XII)CZsZ9II
A- A FIGURE 9.17
i.e.. J = = (3/4) A
a
4/3
vt

Le., apparent depth is only (3/4)th of the real depth RARER


of the tank of water. X
DENSER
Note that this relation is valid strictly for normal I*-*'
incidence of light. The course of rays for oblique incidence
is shown in Fig. 9.17. For calculation of p, the same relation
(24) is valid only approximately.
O'
IMPORTANT NOTE
Normal shift in the position of the object
Al 1 >
= AO-AI =AO 1-

w
= f 1
AO

where t = AO = real depth and p is absolute refractive index of the medium.

Flo
DO YOU KNOW ?

e
reee
In Art. 9.14; we have assumed that llie rays go from denser medium to rarer medium. However, the formula

FFr
is valid equally for other combinations too. For example, in Fig. 9.18(a), observer is in water when virtual
image I is formed, of a real object O in air. In Fig. 9.18(/?), observer is again in water, when real image I is

for
formed of a virtual object at O. In Fig. 9.18(c), observer is in air, when real image is formed at /. of a virtual
ur
object at O.
kss
Yo
oo
eB
ur
ad
YYo
d
Re
in
F

Retain in Memory
FIGURE 9.19
When we hold a coin P in air, directly above a swimmer’s
P'
eye in water, the under water swimmer perceives the rays M
I 1
I I
as originating from a point P', that is farther above the I I
Apparent hi h'
'/\*
surface than the actual point P. This is because rays from iJ Actual ht. h
i Air
P refract towards the normal as they enter the water Water

surface. Fig. 9.19. Apparent height h' is greater than actual


height h of the coin.

Swimmer's eye
RAY OPTICS AND OPTICAL INSTRUMENTS 9/17

Sample Problem A Small Ink dot on a paper is viewed through a glass slab of thickness
10 cm ; and refractive index 1‘5. By what distance would the dot appear to be raised ?
Sol. Here, real thickness of glass slab, a: = 10 cm ; = 1 -5
X

If apparent thickness of glass slab is y, then from [i = —\


X 10
>’ = — — = 6-67 cm
A 1-5

Distance through which the dot appears to be raised = x-y = 10- 6-67 = 3*33 cm

9.15. REFRACTION EFFECTS AT SUN RISE AND SUN SET


The sun is visible to us before actual sun rise and after actual sun set. This is because of atmospheric

w
refraction of light.
The refractive index of air of normal density is about 1.0003 relative to vacuum. Further, the density of
air is higher near the surface of earth*. In Fig. 9.20, S is position of the sun just below the horizon (i.e. before

Flo
sunrise). The rays from the sun (5) coming through space enter at the lop of earth’s atmosphere and travel
from rarer to denser medium. They bend very slightly towards normal at each refraction and appear to come

ee
from S\ the apparent position of sun. Therefore, the sun appears above the horizon. Hence the sun appears to
rise a few minutes before the actual irse and for the same reason, it continues to be seen a few minutes after

Fr
it has actually set. This time difference is of the order of 2 minutes each. Hence the day becomes longer by
about 4 minutes due to refraction effects.

for
ur
Some other phenomena related to refraction of light
are :
ks
(i) Bending of an immersed portion of an object.
Yo
(ii) Visibility of two images of an object floating
oo

inside water.
eB

Still some other phenomena, which can be explained


in terms of refraction of light through the atmosphere are :
ur

(0 Twinkling of stars.
ad

(ii) Oval shape of sun at the time of sun rise and sun set.
Yo

9.16. REFRACTION OF LIGHT THROUGH A COMPOUND PLATE


d

Consider a compound plate made of transparent


Re
in

media b and c bounded by parallel faces as shown in Fig.


9.21. Let it be held in a medium a (say, air).
F

A ray of light is incident on the compound plate along


OA at .Zi. It is refracted along AB at Zrj.
According to Snell’s law
a sin i
\^b= —
sm ,r
...ii)
1

The refracted ray AB is incident on the face separating


media b and c at Zr^. It is refracted along BC at Zr2-
sin .rI
...(»)
sin ^2

*Here, we are considering the whole of the atmosphere of earth. Density of air is higher near the surf ace of
earth.
9/18 Fundamental Physics (XII) orsTWl

Finally, the refracted ray BC falls at on boundary of media c and a. It emerges along CD. As
the various interfaces of media are parallel, the angle of emergence will be equal to angle of incidence i; i.e.
CD 11 OA.

sin

sini

Multiply (/), {//) and (///)


sin .r
i-x sm rj
sm i
X = l
sin .r sin sin i
I

=x

oww
...(25)

Note that in passing through a glass plate obliquely, a ray of light neither converges nor diverges, but is
displaced parallel to itself i.e. it undergoes lateral displacement.

e
9.17. TOTAL INTERNAL REFLECTION OF LIGHT

eree
rFl
To understand the phenomenon of total internal reflection, suppose an interface XY separates a rarer
medium (a) i.e. air from a denser medium {b) say water. Fig. 9.22.

FFr
D is a point object in the denser medium. A
ray of light starting from O and incident normally

orr
ouur
along OA on XY passes straight along AB. sfo
Another ray incident along DAj deviates away
from normal and is refracted along A |S|. Clearly,
kks
Yo
angle of refraction is greater than the angle of
oooo

incidence and angle of refraction increases with


eBB

increase in the angle of incidence. For particular


value of i = C, the critical angle, the incident ray
OAj is refracted at Zr = 90° and goes grazingly
urr

along the interface, along A2B2- When / > C as


ad

for the incident ray OA^, the ray goes along ^3^3,
YYo

as if it is reflected from interface XY. No portion


of light is refracted into medium a. This
dd

phenomenon is called Total Internal Reflection.


Re
inn

We may define
F

Total internal reflection is the phenomenon of reflection of light into a denser medium from an
interface of this denser medium and a rarer medium.

Total Internal Reflection follows the common laws of reflection.


Two essential conditions for total internal reflection are :

(/) Light should travel from a demer medium to a rarer medium, (ii) Angle of incidence in denser
medium should be greater than the critical angle for the pair of media in contact. We may define
Critical angle for a pair of media in contact as the angle of incidence in the denser medium
corresponding to which angle of refraction in the rarer medium is 90“^.

It is represented by C and its value depends on the nature of media in contact.


Hence, we conclude that when a ray of light travelling from an optically denser medium to an optically
rarer medium is incident on the interface at an angle greater than the critical angle for the pair of media
in contact, the ray is totally reflected back into denser medium.
RAY OPTICS AND OPTICAL INSTRUMENTS 9/19

Relation between Refractive index and Critical angle


When i=C,r= 90°
Applying Snell’s law at A2, sin C = sin 90° = x 1
1

sinC
fl

a
1
...(26)
sin C

If is wavelength of light in vacuum, is wavelength of light in medium,

ww
X0
a
then
m

Flo
1 ^m
sin C = —

e
\L X0

ree
Fr
As p depends on wavelength, therefore critical angle for the same pair of media in contact will be

rF
different for different colours. For example, as X^ > Xv, therefore critical angle for a given pair of media will
uurr
be greater for red colour than that for violet colour.
Table 9.2 shows values of p and C for some common media for X = 5900 A.
s for
kks
TABLE 9.2. Reftractive index p and critical angle C for some media
Yo
oooo

Medium p of medium w.r.t. air Critical angle C


eB

1. Ice 1.31 49.76°

2. Water 1.33 48.75°


ur

3. Alcohol 1.36 47.33°


ad
YYo

1.47 42.87°
4. Turpentine oil
5. Crown glass 1.52 41.14°
1.65 37.30°
dd

6. Dense flint glass


Re

7. Diamond 2.42 24.40°


in
F

Sample Problem In the ray diagram shown here, calculate the speed of light in the liquid of
unknown refractive index. (CBSE 2017 (O)

Sol. As shown in Fig. 9.23, ray OA is sulfering total internal reflection at A, and going along AB.
ZOAN = C, the critical angle.

1 1 OA V302 + 402 ^_5


p =
sin C CA/OA CA 30 30 “3

As p = —, therefore, v = —
V

V - -^^10^ =?xl0»m/s =l-8xl0*m/s


5/3 5
9/20 ‘P%<xdecjb. <t Fundamental Physics (XII) kV»Tn
9.18. EXPERIMEm-AL DEMONSTRATION OF TOTAL INTERNAL REFLECTION
Take some clear water in a plain glass beaker. Stir a piece of soap cake a few times in water so that the
water becomes a little turbid.

Take a laser pointer and shine its beam through the turbid water. The path of the laser beam inside water
shines brightly.
When we .shine the beam from below the beaker, we observe that the beam undergoes partial reflection
and partial refraction at the free surface of water in the beaker, [Fig. 9.24(a)]. The reflected beam forms a
spot on the table below and the refracted beam forms a spot on the roof.
When we direct the laser beam from one side of the
beaker, and adjust its direction so that refraction above
the water surface is totally absent, we observe that the
beam is totally reflected back to water from its upper
surface. This is total internal reflection ; as shown in

w
Fig. 9.24(^).
If we repeat this experiment by taking turbid water in
a long test lube and shine the laser beam from the lop at a

Flo
suitable angle, the beam is totally internally reflected

e
repeatedly from the walls of the test tube, as shown in Fig.

rree
9.24(c).

r FF
9.19. SOME APPLICATIONS OF TOTAL INTERNAL REFLECTION
uurr
1. The brilliance of diamond
The brilliance of diamond is due to total internal for
reflection of light, p for diamond is 2.42, so that
kss
critical angle for diamond air interface as calculated from eqn. (9) is 24.4°. The diamond is cut suitably so
that light entering the diamond from any face falls at an angle greaterthan 24-4°. Therefore,it suffers multiple
ooook
Yo

total internal reflections at various faces, and remains within the diamond, failing to come out. Hence the
diamond sparkles.
eB

2. Shining of an air bubble in water


As is known, critical angle for water air interface is 48-75°. When
urr

light propagating from water (denser medium) falls on the surface of an air
ad

bubble at an angle greater than 48-75°, as shown in Fig. 9.25, it suffers


Yo

total internal reflection. Therefore, the air bubble in water shines brilliantly.
dY

3. Mirage
Re
innd

Mirage is an optical illusion of water observed generally in deserts


on hot summer days. An inverted image of an object such as tree is observed
Fi

alongwith the object as if the tree were on the bank of a pond of water.
On a hot summer day,
temperatureof air near the surface of
earth is maximum. The upper layers
of air have gradually decreasing
temperatures. Therefore, density and
refractive index of air goes on
increasing slightly* with height above
the surface of earth. Profiles for the
temperature and refractive index of air
for the phenomenon of mirage are
shown in Fig. 9.26.
*Here, we are considering only a few metres’ height of atmosphere from the surface of earth. Hence density
of air close to surface of earth is smaller due to higher temperature.
RAY OPTICS AND OPTICAL INSTRUMENTS 9/21

In Fig. 9.27, a ray of light from the top O of a


tree goes from denser to rarer medium bending
successively away from normal. At a particular
layer, when angle of incidence becomes greater than DECREASING

the critical angle, total internal reflection occurs,


and the totally reflected ray reaches the observer AIR
along AE, appearing to come from /, the mirror GROUND

image of O. Thus inverted image of tree creates the


impression of reflection from a pond of water, Fig.
9.27. Similarly, in the mirage of a car, the car appears
upside down due to total internal reflection of light
in the hot air close to the ground.

w
4. Totally reflecting glass prisms
Totally reflecting glass prisms are right

Flo
angled isosceles prisms which turn the light
through 90° or 180°. They are based on the

e
ree
phenomenon of total internal reflection

FFr
of light. )x for glass is 1.5 so that critical angle
for glass-air interface is 42°. In totally
urr
reflecting glass prisms, angle of incidence is
made 45° (> Q. Hence light suffers total
internal reflection on face QR as shown in for
kkss
Fig. 9.28(a).
Yo
A'B' is image of AB seen through the
ooo

prism PQR after total internal reflection of


eB

light. In Fig. 9.28(a), turning of light rays is through 90° on account of one total internal reflection on face QR
of the prism. In Fig. 9.28(/?), turning of light rays is through 180° on account of two total internal reflections,
one on face PQ and other on face PR of the prism.
ur
ad

IMPORTANT NOTES
YYo

1. A right angled isosceles prism can also be usedfor inverting the image ofan object without changing
its size. This is shown in Fig. 9.29. Rays from an object AB fall normally in face PQ of the prism,
d

travel undeviated falling on face QR at an angle of 45°, which is greater than the critical angle for
Re
in

glass-air interface. The.se rays suffer total internal reflection on face QR of prism. The totally reflected
rays emerge undeviated from the face PR of the prism forming an inverted image A' B' of the object.
F

Such a right angled isosceles prism is called erecting


prism. It is often used in binoculars.
2. In a totally reflecting prism, only one bright image
of an object is formed, whereas in a plane mirror,
number of faint images may accompany the main
image. Further, in total internal reflection, there is no
loss in intensity of reflected light. That is why image
formed is brighter than the image formed in a plane
mirror.

5. Optical Fibres
Optical fibres are used as a guided medium for transmission of optical signals over long distances of
several kilometers.
9/22 ^>tadeeft-'4r Fundamental Physics (X11)S!SI9D

Construction. An optical fibre consists of three main parts : FIGURE 9.30

(i) Core. The central cylindrical core is made of very high


quality glass/silica/plastic of refractive index |ij. The core has a n
-A:
Q
diameter ranging from 10 micron to 100 micron.
(iV) Cladding. The core is surrounded by a glass/plastic jacket
Core

(Pl) Cladding
-xz
of refractive index ^2 < l^i- {P2)<(Fl) Buffer Coating

w
In a typical optical fibre, Jij = 1-52 and ==
(ill) Buffer coating. The core-cladding of optical fibre is enclosed in a plastic jacket called buffer
coating. This would provide necessary safety and strength to the optical fibre.
A bundle of optical fibres is called a light pipe.

e
Working. The working of optical fibres is based on the phenomenon of total internal reflection.

row
re
Light incident on one end of the fibre at a small
angle enters the fibre after refraction and strikes the

eeF
Fllo
core/cladding interface at an angle of incidence
greater than the critical angle and is reflected back

Fu
into the core. Light thus travels inside the optical fiber
along a zig-zag path as shown in Fig. 9.31. It finally

srr
roF
comes out of the fibre at the other end, even if the

k
fibre is bent or twisted in any form. And there is almost
uor
ofof
no loss of light through the sides of the fibre or due
to absorption by the core.
kos
The only condition is that angle of incidence of light must be greater than the critical angle for the fibre
Y
Yo
material w.r.t. its coating.
eerBB
oo

Applications of optical fibres


rY

(0 Optical fibres are used to transmit light without loss of intensity over long distances of several
kilometers.
u

(i7) Optical fibres are used in the manufacture of medical instruments, called endoscopes for visual
ou
o
ad
d

examination of stomach and intestines, etc. of the patients,


nY

(ill) Optical fibres are used in telecommunication, i.e., in transmitting audio and video signals to long
distances.
nid
Re

(/v) In the form of photometric sensors, optical fibres are used in measuring the blood flow to the
F
Fi

heart.

(v) Optical fibres are used to measure refractive indices of liquids.

9.20. SPHERICAL REFRACTING SURFACES


FIGURE 9.32
A refracting surface which forms a part of a
X
sphere of tran jarent refracting material is called a X

spherical refracting surface. There are two types of Rarer / Denser Rarer\ Denser
spherical refracting surfaces : M1 M2 P2

(0 Convex spherical refracting surface, which P P


is convex towards rarer medium side as shown in Fig. Principal
Axis
9.32(a).
(li) Concave spherical refracting surface, CONVEX Y CONCAVE
which is concave towards the rarer medium side as
e o
shown in Fig. 9.32(b).
RAY OPTICS AND OPTICAL INSTRUMENTS 9/23

Pole (P) of a spherical refracting surface is the mid point/centre of this surface.
Centre of curvature (O of the spherical refracting surface is the centre of the sphere of which this
surface is a part. Radius of the sphere of which the spherical refracting surface is a part is called Radius of
curvature of the surface. It is represented by R. Thus PC = R. The line passing through the pole and centre
of curvature of spher ical refracting surface, extended on either side is called Principal axis of the surface.
The aperture of spherical refracting surface is the effective diameter of the surface exposed to incident
light. In Fig. 9.32 vertical distance between points X and Y is the aperture of the spherical refracting surface.
In dealing with refraction at spherical surfaces, wc use New Cartesian Sign Conventions as detailed
below :

1. All distances are measured from the pole of the spherical refracting surface.
2. The distances measured in the direction of incidence of light are taken as positive and the distances

ww
measured in a direction opposite to the direction of incidence of light arc taken as negative.
3. The distances measured in transverse direction above the principal axis are taken as positive. And the
distances measured in transverse direction below the principal axis arc taken as negative.

Flo
Assumptions :

e
In dealing with refraction at spherical refracting surfaces, we assume the following :

ree
1. The object consists only of a point lying on the principal axis of the spherical refracting surface.

Fr
2. The aperture of spherical refracting surface is small.

rF
3. The incident and refracted rays make small angles with the principal axis of the surface, so that
uurr
sin i =» i and sin r * r.

9.21. REFRACTION FROM RARER TO DENSER MEDIUM AT


s for
kks
A CONVEX SPHERICAL REFRACTING SURFACE
Yo
oooo

Two cases arise. The image formed may be real or virtual.


(a) Real image
eB

Let a spherical refracting surface XY separate


a rarer medium of refractive index |0.j from a denser
ur

medium of refractive index Suppose the surface


ad

is convex towards rarer medium side. Let P be the


YYo

pole, C be the centre of curvature and R ~ PC be


the radius of curvature of this surface.
dd

Consider a point object O lying on the


Re
in

principal axis of the surface, Fig. 9.33.


A ray of light starting from O and incident
F

normally on the surface XY along OP passes straight.


Another ray of light incident on XY along OA at Zi
is refracted along Al at Zr, bending towards the
normal CAN. The two refracted rays actually meet
at /, which is the real image of O.
From A, draw AM _L 01.
Let ZAOM = a, ZAIM ■- p
and ZACM = y
Aj' external angle of a triangle is equal to .sum of internal opposite angles, therefore, in A lAC
y=r+P or r = y-p
Similarly, in AOAC, « = a + y ...(27)

^^2 sin i

According to Snell’s law. sm r ~^ ('.● angles are small) ...(28)


9/24 ^●utcUcfr ^ Fundamental Physics (XII) tltdMll
1^1 ' = ^2''
Using (27), we get, ia-1 (a + 7) = }i2(T-P)
As angles a, |3 iind y are small, using 0 = //r, we get
AM AM \ AM AM
...(29)
MO MC ) n MC MI )
As aperture of the spherical surface is small, M is close to P. Therefore, MO ~ PO, MI ~ PI, MC *= PC
{ 1 1
/J 1
-1-
From (29), ^^1 PO PC PI PO PI PC

Using new cartesian sign conventions, we put


P0 = - it, PI=+v,PC = R

ww
|i| M2
...(30)
-u V R

FF loo
This is the relation governing refraction from rarer to denser medium at a convex spherical refracting
surface.

ree
(b) Virtual Image
The point object O in this case, lies on the

rFee
principal axis close to the pole of the refracting

oor rF
surface. A ray of light OA starting from O suffers
rur
refraction at A, bends towards the normal CAN and
s ff
is refracted along AB. Another ray of light OP falling
normally on the refracting surface travels along PC
k
undeviated. The refracted rays AB and PC do not
YYoou
okos

meet actually at any point but appear to come from


BBoo

a point I. Thus / is the virtual image of the point


object O, Fig. 9.34.
r ee

From Snell’s law.


sini _ M-2 ...(31)
ouur
ad

sin r
Yo

or sin / = Pi / = P2^ ...(32) (i and r are small)


Draw AM ± 01. Let ZAOM = a, ZAIM = p and ZACM = y
Yd
Re

In A OAC, i = y + a and in A lAC, r = y + P


idn

Putting the values of i and r in (32), we get Pi [y + a] = P2 [y+ p]


FFin

I
As angles are small, using 9 = - , we get,
r

1 1 1 1
AM AM
AM . ^ P-i + =1^9
or
^^1 MC MO MC MI
or
'LmC MOj - MC MI

As aperture is small, M is close to P


1
1
JJ. _l^
Ml PC PO PC PI
or
.PO PI PC

Using new cartesian sign conventions, PO = -u,PI = ~v,PC = + R

Ml _M2 _1^2~M| or ...(33)


-It -V R It V R

Note that this relation is the same as equation (30), when image formed is real.
RAY OPTICS AND OPTICAL INSTRUMENTS 9/25

9.22. REFRACTION FROM RARER TO DENSER MEDIUM


AT A CONCAVE SPHERICAL REFRACTING SURFACE

Let O be a point object lying on the principal


axis of a spherical refracting surface XY which is
concave towards the rarer medium. A ray of light
OA starting from O suffers refraction at A. bends
towards the normal CAN and is refracted along AB.
Another ray of light OP, falling normally on the
refracting surface suffers no deviation and passes
along PD. The two refracted rays AB and PD do not
meet at any point on the right side but appear to come
from a point 1 on the left side. Thus / is virtual image
of the point object 0, Fig. 9.35.

w
In A OAC, / = y - a and in A lAC, /● = y - j3
Putting these values of i and r in the relation

Flo
Pi i = H2we have Mj (y-a) = P2 (Y-P)
As angles are small, using 6 = l/r, we get

ee
AM AM AM AM 1 1 1 I

Fr
●●● P, MC MO
or
= ^2
.MC Ml . MC MO MC MI

As M is close to P because of small aperture


for
ur
1 P2
^1 or
.PC PO PC PI PI PO ~ PC
ks
Yo
Using new cartesian sign conventions,
oo

PO=-u,PI = -v,PC-~R
B

li.-P 1
-Pi , P2 _P2~^^l
re

-V ~u -R
or which is the desired relation.
u V R
ou
ad

IMPORTANT NOTE
We have taken the object as a point object lying on the principal axis of the refracting surface. However,
Y

when the object has a finite size, we can calculate the size of the image from the linear magnification
produced by the spherical refracting surface.
nd
Re

The course of rays is as shown in Fig. 9.36.


Fi

Compare equation (33), i.e.,

p, ^ P2 _P2~Pi
u V R

with the mirror equation

i 1-1
u f
-u

We find that u corresponds to ~


V
and V corresponds to
^2
9/26 ^ Fundamental Physics (XII) wiwii

m - —
-V ^l\^2
u -u /)t 1

m = —
u

This relation is valid for any single refracting surface, convex or concave.

Sample Problem Light from a point sourcc in air falls on a convex spherical glass surface
(|i = 1-5 and 7? = 20 cm). Calculate position of the image when the light source Ls at 1 m from the glass surface.

ooww
Sol. Here. = 1 (for air), |i2 = 1-5 (for glass), /? = + 20 cm. h = -1 m = - 100 cm, v = l
As refraction occurs from rarer medium to denser medium, therefore

u V R

e
1 , 1-5 _ 1-5-1 _ 1

ere
rFl
-100 V 20 40

Fre
3 1 _5-2_ 3

rrF
2v 40 100 ~ 200 ~ 200
V -100 cm

sffoo
ouur
Image is formed at 100 cm from the pole of the surface in the direction of incidence of light.
kosk
9.23. REFRACTION FROM DENSER TO RARER
Yo
MEDIUM AT A SPHERICAL REFRACTING SURFACE
oo
Y

(a) At A convex spherical surface


BB

Let P be the pole and C be the centre of curvature of a spherical refracting surface XY, Fig. 9.37. This
rre

surface is convex towards the rarer medium and separates a denser medium of refractive index P2 from a
rarer medium of refractive index Pj. Clearly, P2 > 1^1
ouu
Y
ad

Let (9 be a point object lying on the principal


axis of the spherical surface. A ray of light OA
dY

starting from O meets the refracting surface at A.


On refraction, it bends away from the normal CAN
innd
Re

and moves along AI. Another ray of light OP falling


normally on the refracting surface goes undeviated
Fi
F

along PI. The two refracted rays Al and PI actually


meet at /, which is, therefore, the real image of the
point object O. If i and rare the angles of incidence
and refraction, then from Snell’s law, we have
1^2 _ sin r
^^1 sm i

Retain in Memory

Pt sin r . . ,. sm t
Note this step. Here, — and not equal to — , because refraction occurs from denser to
sin i sm r
rarer medium.

or
P-, sm i = Pj sm r ...(34)
As i and r are small. sin i ~ i and sin r = r
RAY OPTICS AND OPTICAL INSTRUMENTS 9/27

(34) becomes, 112^=^!'- ...(35)


In A OAC i = y-a In A lAC, r=y+p
from (35), H2 (7-a) = l^i (y+P)
From A, draw AM perpendicular to 01
As angles are small, using 0 = l/r, we gel.
AM AM AM , AM 1 I 1 1
^^2 MC MO
or
^^2
MC MI MC MO MC Ml

Since aperture of the refracting surface is small, M is close to P.


1 1 1 1
MC * PC, MO = PO and MI« PI ^^2
.PC PO. PC PI

ww
Applying new cartesian sign conventions, PO - - u, PI = v, PC = - R

Floo
^^2 , _^-^2 ...(36)
-u V -R R

ee
This is the relation governing refraction from denser to rarer medium at a convex spherical refracting

eer
surface.

FFr
(b) Refraction from denser to rarer medium at a concave spherical refracting surface

oorr
uur r
Let (9 be a point object lying on the principal axis of a spherical refracting surface XY which is concave
s ff
towards the rarer medium. Fig. 9.38. A ray of light OP falling normally on the surface goes undeviated along
PC, whereas another ray OA, bends away from the normal and proceeds along AB after refraction. The
sk
refracted rays PC and AB do not actually meet at any point, but appear to come from a point I. Thus / is the
YYoo
ooko

virtual image of the point object O.


eBB

In A OAC, i = y+a
and in A lAC, r = y+p
uurr

Putting these values of i and r in the relation


ad

P2 i = r, we have M2 (y+ tx) = Mi (y + P)


Yo

As angles are small, using 0 = l/r, we get


dY

AM . AM AM , AM
Re

or M2
ind

MC MO MC MI
FFin

1 1 1 1
or
^^2 = 1^1
IMC MO. MC MI
Since aperture is small, M is close to P
1 1 1 1
M2
PC PO PC PI

or M2 Ml ^ M2 ^ Ml „-(M2-^^i)
PO PI PC PC PC

Using new cartesian sign conventions, so that PO = -u,PI = -v, PC = R

~u -V R
or -M2 , Ml ^ Ml-M2 ...(37)
u V R

which is the same as (36).


9/28 Pna.deef.i'A Fundamental Physics (XII)E^HE
in in Memory
When the object is placed in a denser medium and real or virtual image is formed on refraction at
a convex or concave spherical refracting surface, we have proved that

,.(38)
-u V R

The formula governing refraction from rarer to medium (from eqn. 33) is
...(33)
u V R

On interchanging p.| and p-2 in (33), we get u


4. V R
^

ww
which is the relation (38). Hence, we find that when refraction occurs from a denser medium to a
rarer medium, we have to interchange |ij and )i2 in relation (33) to obtain relation (38). The
reverse is also true.

Floo
Sample Problem A small point object is placed in air at a distance of 60 cm from a convex

ee
spherical refractive surface of |i = 1*5. If radius of curvature of spherical surface is 25 cm, calculate the

eer
position of the image and the power of the refracting surface.
Sol. Here, « = - 60 cm, |4.j = 1, jj.2 = 1 -5, R = + 25 cm, n = ?

FFr
As refraction occurs from rarer to denser medium, therefore

oor r
ur r
-Pi P2 _ P2~l^i
s ff
u V R
sk
YYoou
-1 ^ 1-5 _ 1-5-1
oooko

-60 V 25
eBB

J__ J 1 _ 1
2v ~ 50 60 " 300
uurr
ad

300x3
V = = 450 cm
Yo

As V is positive, image formed on the other side of the object, i.e., in refracting denser medium at 450
dY

cm from the pole.


Re
ind

1-5-1 0-5
FFin

Power of the refracting surface, P = hzh = 2D


R 0-25 0-25

9.24. LENSES

A lens is a portion of a transparent refracting medium bound by two refracting surfaces out of
which atleast one is curved.

The commonly used spherical surfaces may be convex or concave surfaces and medium generally used
is glass. Lenses are divided into two classes:
(0 Convex or converging lenses, and (n) Concave or diverging lenses,
(i) Convex or Converging lenses.
These are of three types : (a) Double convex lens. Fig 9.39(/),
{b) Plano convex lens. Fig. 9.39(/i) and (c) Concavo convex lens. Fig. 9.39(//0
The distinguishing feature of a convex lens is that it is thicker in the middle and thin at the edges.
RAY OPTICS AND OPTICAL INSTRUMENTS 9/29

(ii) Concave or Diverging lenses.

ww
These lenses are also of three types: (a) Double concave lens, Fig. 9.40 (/),
0?) Plano concave lens. Fig. 9.40 {ii) and (c) Convexo concave lens, Fig. 9.40 {Hi)
A concave lens is thinner in the middle and thicker at the edges.

FF loo
ree
rFee
oor rF
rur
k s ff
YYoou
ookos
BBo

We shall confine our study to thin spherical lenses only. FIGURE 9.41
re

The surfaces of the lens are spherical in nature ; and radii of


curvature of the surfaces are veiy large in comparison to the
ouur
ad

thickness of the lens.


Yo

A
Some important terms related to lenses : Ri
1. Principal axis. Ii is defined as a straight line passing
Pi c\ P2 Cl
dY

C2
Re

through the centres of curvature of two surfaces of a lens. B


idn

In Fig. 9.4\{a), CjC2 represents the principal axis of


FFin

the lens. o
2. Optical centre. It is a point lying on the principal
axis of the lens so that a ray of light whose refracted path
passes through this point will have its emergent path parallel
to the direction of the incident ray. The ray will suffer only
some lateral displacement, as shown in Fig. 9.41(a). C2 Cl
The optical centre C, Fig. 9.41(a) of the lens divides its
thickness of the lens in the ratio of the radii of curvature of o
two surfaces of the lens.

Thus,
CP2 ^
/?j and Ro being the radii of curvature of the two surfaces of the lens.
9/30
"PnAdee^ ’4. Fundamental Physics (XII) P75T»n
For a thin lens, howeve,r the optical centre is taken as a point lying on the principal axis so that a ray
of light passing through it does not suffer any deviation from its path, i.e., it goes undeviated, without any
lateral displacement even, Fig. 9Al{b).
3. Principal Focus. When a beam of light is
incident on a lens in a direction parallel to the
principal axis of the lens, the rays after refraction
through the lens converge to (in case of convex lens)
or appear to diverge (in case of concave lens) from a
point on the principal axis. Fig. 9.42. This point F is
known as the principal focus of the lens.
CF =f \s principal focal length of the lens.
4. Aperture. Aperture of a lens is the effective diameter of its light transmitting area. Therefore, the

w
brightness i.e. intensity of image formed by a lens-which depends on the amount of light passing through the
lens will vary as square of the aperture of the lens i.e. I« {aperture)^.

Flo
9.25. LENS MAKER'S FORMULA FOR CONVEX LENS
Lens Maker’s formula is a relation that connects focal length of a lens to radii of curvature of the two

eeee
surfaces of the lens and refractive index of the material of the lens.

Fr
The lens maker’s formula is useful in designing lenses of desired focal length using suitable material
and surfaces of suitable radii of curvature.

In deriving this formula, we use New Cartesian Sign Convention.s, which are :
for
ur
\.All distances are measured from the optical centre of the lens.
2. All the distances measured in the direction of incidence of light are taken as positive, whereas all the
kss
distances measured in a direction opposite to the direction of incidence of light are taken as negative.
Yo
oo

3. For a convex lens, f is positive and for a concave lens, f is negative, as is clear from Fig. 9.42.
Tht assumptions made in the derivation are:
eB

1. The lens is thin so that distances measured from the poles of its surfaces can be taken as equal to the
distances from the optical centre of the lens.
r

2. The aperture of the lens is small.


ou
ad

3. The object consists only of a point lying on


YY

the principal axis of the lens.


4. The incident ray and refracted ray make
nd
Re

small angles with the principal axis of the lens.


Derivation of lens maker’s formula for
Fi

convex lens

A convex lens is made up of two convex


spherical refracting surfaces. The final image is
formed after two refractions. In Fig. 9.43, Pj, P2
are the poles, Cj, C2 are the centres of curvature of
two surfaces of a thin convex lens XY with optical
centre at C. Let [I2 be the refractive index of the
material of the lens and |i,j be the refractive index
of the rarer medium around the lens.
Consider a point object O lying on the principal axis of the lens. A ray of light starting from O and
incident normally on the surface XPj Y along OPy passes straight. Another ray incident on XP^ Y along OA is
refracted along AB. If the lens material were continuous and there were no boundary/second surface
XP2Y of the lens, the refracted ray AB would go straight meeting the first refracted ray at f. Therefore, f
would have been a real image of O formed after refraction at XP^Y.
RAY OPTICS AND OPTICAL INSTRUMENTS 9/31

If C/, =Pj/, = v 1
*
and CC 1 P,C, =/?i* ; = =
II, ...(39)
then from (33),
-u V R
1 I

Actually, the lens material is not continuous. Therefore, the refracted ray AB suffers further refraction
at B and emerges along BI, meeting actually the principal axis at I. Therefore, / is the ifnal real image of O,
formed after refraction through the convex lens.
For refraction at the second surface XPf^, we can regard /, as a virtual object, whose real image is
formed at I.

Therefore, h = C/, = P2 ^I = 1 ’ Let Cl = P2I -v


Let Rr, be radius of curvature of second surface of the lens.
As refraction is now taking place from denser to rarer medium, therefore, using eqn. (21), we get

ww
M-7
- +
...(40)
V V
R, -R,

FF loo
Adding (39) and (40), we get

ree
«2j
R
1

1 1 1 1 ^ 1
ijb. -1 1
...(41)

rFee
or
ll,^ =(^l2-Il,) —- or
R,1 R-,
V u ^ R
«2j
V u

oor rF
1 / V
rur
^2
s ff
Put = |i = refractive index of material of the lens w.r.t. surrounding medium.
i
k
When object on the left of lens is at infinity, image is formed at the principal focus of the lens
YYoou
okso

when » = 00, V =/= focal length of the lens.


BBoo

= {p-l) ^
I 1
From (41), or ...(42)
r ee

/ 00 R
1 f R
1 /?2
This is the lens maker’s formula.
ouur
ad

9.26. LENS MAKER'S FORMULA FOR CONCAVE LENS


Yo

A concave lens is made up of two concave spherical refracting surfaces. The final image is formed after
two refractions, one at each surface as shown in Fig. 9.44.
Yd
Re
idn

P^, ?2 are the poles ; C,, C2 are the centres of


curvature of the two surfaces of concave lens (not shown)
FFin

with optical centre at C. Let P2 be the refractive index of


the material of the lens and Pj be the refractive index of
the rarer medium around the lens.

Consider a point object O lying on the principal axis


of the lens. A ray of light starting from O and incident
normally on the surfaced,PjK, along OP, passes straight.
Another ray incident on X, P, T, along OA is refracted along
AB. If the lens material were continuous and there were

no boundary/second surface X2P2X2 of tbe lens, the


refracted ray AB would go straight and appear to come
from /]. Therefore, /, would have been a virtual image of
O formed after refraction at X|P|F,.
If C/, =» P| /] = ; CO = P^O = u, and P, = radius of curvature of surface X^P^Y^.
*Sign convendons need not be reapplied here, as the same have already been applied in the formulae being used.
9/32
'4. Fundamental Physics (XII) LV«1WII
then as refraction occurs from rarer to denser medium, therefore, from (33)
Ml- +
\^2 _ ^^2
U V R ...(43)
1 1

Actually, the lens material is not continuous. Therefore, the refracted ray AB suffers further refraction
at B and emerges along BD. It appears to come from I. Therefore, 1 h final virtual image of point object O,
formed after refraction through the concave lens.
For refraction at the second surface assume /j as an object whose image is formed at I.
u = C!^ = P2 /|= V
P2 I ^ Cl = v
Let be the radius of curvature of second surface of the lens.

ww
As refraction is now taking place from a denser to a rarer medium, therefore, using eqn. (38), we get

_ _ Ml ~ M2 _ M2 “ Mj
R, ...(44)

Flo
V
1
V
-^2

e
Adding (43) and (44), we get

ree
1

Fr
rF
U V
uurr - /

or M1
1

V
1
7«J =(M2-Mi) —“
R
^ 1

1
s ^2
1
for
kks
l__f^_Mi
Yo
1
J 1_
oooo

or
V u
Ml M R, R^
I ;
eB

“ /

M2
Put - M = refractive index of material of the lens w.r.t. the surrounding medium.
M|
ur
ad
YYo

...(45)
R
V u
«2 1
dd

When object on the left of the lens is at infinity, image is formed at the principal focus of the lens.
Re
in

i.e. when u = oo,v =f= focal length of the lens,


from (45),
F

1 1 I A 1
or — = (p.-l) -—
/ 00

fi, R, / R
I
“ /
- /

This is the required lens maker’s formula.


Sample Problem The radii of curvature of the surfaces of a double convex lens are 20 cm and
40 cm respectively, and its focal length is 20 cm. What is the refractive index of the material of the lens ?

Sol. Here, R^ = 20 cm, /?2 = - 40 cm,/= 20 cm, =?


1 1
As = (M-I) —-
/ R
1 R,- y

= (p-l) ~ + —'' = (m-d4


1 2 5
20 1,20 40 40 or (p-l)=^ or fi=-+l = —
3 3
9/33
RAY OPTICS AND OPTICAL INSTRUMENTS
9.27. DERIVATION OF LENS-MAKER'S FORMULA FROM AB-INITIO
The sign conventions and the assumptions arc the same as already stated in case of a lens,
(fl) Convex Lens
Let P^, P2 be the poles and C,, C2 be the centres of curvature of the two spherical surfaces 7 and
X?2>'of a convex lens with optical centre at C, Fig. 9.45. Let 1I2 be the refractive index of the material of the
lens and |ij that of the medium surrounding the lens, where fi2 > |i].
Consider a point object O lying on the principal axis of the lens.
(0 Refraction at the surface XP^Y. Let a ray of light OA starting from the point object O strike the
surface XP^ Y‘<i\. A making an angle /, with normal C^AN^. On refraction, it bends towards the normal and
proceeds along AB at an angle /'j with the normal.
Another ray of light from O incident on the surlace along OP^ passes through the optical centre C and
hence goes undeviated along P\i\- If the lens material were continuous, the relracled rays AB and P|/| would

loow w
ac
tually meet at /j. Thus would be the real image of the point object O after refraction at the surface XP, Y.
^2 _
From Snell’s law. (/j and T] are small angles)
sin .r
1 "1
...(46)
or Pi /i-P2''l

ree
Draw AM; 1 01.

ree F
Let ZAOM] = a, ZAIiM^ = ^^ and ZAC|Mi=Yi
r FF
fofroF
u
ks
soo
YYouor
BBook
r ee
ouru
ad
Yo
d
Re
inY
FFind

In A OACj, /p Yi+ In A/j AC|, ^1 - Yi - Pi


Putting these values in (46), we gel
p, (Yi+a) = P2(Yi - Pi)
As angles are small, using 0 = Ur, we get
I I 1 1
AM.1 AM 1 AM^ AM 1 ...(47)
= P or P] = ^2
AY,C, M,OJ ■ - M,Ci MJ IM M,C, M.O
1 MjC, MJ
11

Since the aperture of the lens is small, is close to Pj. Further, as the lens is thin, P] is close to C.
jWiC, = CCi, M^O^CO and Af,/i = C/,

I
9/34
Fundamental Physics fXinPTSTWn

1 1 1 1
From (44), - 1^2 ^1
or
...(48)
CC CO CC,1 Cl CO CL CC
1 1 i
' /
(ii) Refraction at the surface XP2F. Actually, the lens material is not continuous. It is bound by the
surface XP2Y. The refracted ray AB suffers further refraction at B and proceeds along BI, meeting the other
refracted ray along the principal axis at /. Thus I is the final real image of the point object O after refraction
through the lens.
Let ZABC2 = and ZN2B/ ~ r2
Ml sin i
From Snell’s law.
M-2 ^2 ^2 (*.● /2 and r2 are small angles)
or
M2*2“Mi ^2 ...(49)
InA^/iC2, /2“’y2'*'Pi in A B/C2, t*2 = Y2 + P

w
From (49), P2 (72 + Pi) = Mi (72 + P)
From B, draw BM2 perpendicular to OI^

Flo
As angles are small, using 9 = l/r, we get

e
BM^ BM^ BM^ BM. > 1 1 1

rree
M2 = M, —I =
or M2 = Mi
M2C2 M2C2 M^I ^2^2 M^l
M,C2

r FF
2*1 2'1

As aperture of the lens is small and lens is thin


uurr
●●● M2
( 1
CC2
_1
Cl I
= Mi
C^ Cl
1 1
Ml
Cl
for M2 ^M2~Mi
CL1 CC2
...(50)
kss
Adding (48) and (50), we get
ooook
Yo
MI 1 1

CC CC^
eB

I
*■ /

Using new cartesian sign conventions,


CO = - u, C/ = + u, CCi = R^, CCj = - /?2
urr
ad

M, Ml 1 I A
Yo

+ —
= (M2"Mi) —-
R
-It V
«2
dY

Dividing both sides by Pj, we get


Re
innd

1 1 ^2 J 1_
Fi

4- - = -1
-u V
Ml R,1 R-y-■ ...(51)
/ V y

M2
Let M — refractive index of the material of the lens w.r.t. the surrounding medium.
M
1 1
- + - = (p-I) ^±__L ...(52)
-u V R. R^

Since,
-u
—^ ~ ~ y ’ where/is the focal length of the lens.
1
from (52), ^ = (p-l) ~~ * ...(53)
/ R
i R.- ’
y

which is the required lens-maker’s formula.


9/35
RAY OPTICS AND OPTICAL INSTRUMENTS

(b) Concave lens


The course of rays leading to formation of image in a concave lens is shown in Fig. 9.46. Proceeding
exactly as in Art. 9.27, we can obtain lens maker’s formula for concave lens. It is being left as an exercise for
students.

w
Flo
eee
Fr
for
ur
Retain in Memory
ks
1. Focal length of a lens and nature of lens depends on medium surrounding the lens.
Yo
oo

1
For example, focal length of a glass lens in air is — - -1)
eB

fa ^1 ^

If glass lens is immersed in a liquid, then


r
ou
ad

1 1
1 g
-1 J I_
g R
ft yV ^1 ^
YY

1 1 I
nd
Re

ft R
1 /?2
Fi

If |i.; < [1^, fi is positive, i.e., convex lens continues to behave as convex, though its focal length
increases.

If ^1/ > fi becomes negative, i.e., convex lens in air starts behaving as concave lens in liquid.

If
Pl = \Lg. f- = 0Ji = OO

fi
It implies that glass lens behaves as plane glass.
2. Number of images formed by a lens depends on number of materials used to form the lens. If a lens is
made of single material and it is free from the defects of spherical aberration and chromatic aberration,
then single image will be formed, of a given object.
If a lens is made of more than one material, then for a given object number of images formed by the
lens = number of materials used.
9/36
Fundamental Physics (XII)

3. Focal length of a lens depends on wavelength of light used.


1 1
From lens maker’s formula — = ()J -1) ; / OC

/ R
i Rl

As |l OC
(Cauchy’s formula) .% f
X-

As X^ > X^J, therefore,/^ >/„ i.e. focal length of lens for red colour is greater than the focal length of
lens for violet colour.

9.28. PRINCIPAL FOCAL LENGTHS OF A LENS


FIGURE 9.47
There are two principal focal lengths of a lens.

lowow
(/) When the object is so situated on the principal ●>
axis of a lens that its image, after refraction through the
lens, is formed at infinity, then the position of the object Fi >
represents ihe first principal focus or object focus, Fj.
The distance of the first principal focus F, from the optical
7- ^

ee
centre Cof the lens is called first principalfocal length t4 f1

Fr
1 N- f.1 H
of the lens. It is denoted by /,, Fig. 9.47. Thus when, r FF
D = oo, = CFy = -/j
Substituting these values in (52), we get

rer
fofr Fo
u
1 1 1 1 ^
- + - = (P-I) —- or
...(54)
ks
/l 1
OO
A R R
YYouro
(ii) When the object is situated at infinity, the position of the image on the principal axis of the lens, after
oo

refraction through the lens represents the second principal focus or Image Focus, Fj .The distance of the
BBo ks

second principal focus from the optical centre of the lens is called the secoitd principal focal length of ±e lens.
r ee

It is denoted by />, Fig. 9.48.


Thus, when « = oo, y = CF2 =fi FIGURE 9.48
ouru
ad

Substituting these values in (42), we get


Yo

1 I
- = (P_1)
-^2 R F2
^2 F2^
OO
d

4,
c
Re
iYn

I 1 1 >■
— = (p-l) —- ...(55)
FFind

R
^2 >4 ^2 >1 ^2
1 1
From (54) and (55), — = i.e.. f\ -fi ’ provided p is the same.
A
Hence, when a lens is surrounded by the same medium (or two media of equal refractive indices) on

either side, the two principal focal lengths of the lens are equal
However, if media on two sides of a lens have different refractive indices, the two principal focal
lengths of the lens would be unequal.
1 1 1
As
r
A
/● = 7J ’ therefore, from (53), (54) and (55), we obtain
h

r 1 1
- =(P-1) —-
f~ A~ fi V u R
I
...(56)
R^OPTICS AND OPJICAL INSTRUMENTS 9/37

IMPORTANT NOTE
Intensity or brightness of the image is proportional to square of the aperture of the lens. Therefore,
when a lens is half painted black, intensity or brightness of image reduces to (l/4)th. However, size of
image remains the same, because every part of lens forms a complete image of the object.
9.29. LENS FORMULA

It is a relation between focal length of a lens and distances of object and image from optical centre
of the lens.
To derive this formula, we use the same New Cartesian
Sign Conventions as described in Art. 9.25.
(a) Convex Lens. The image formed may be real or
virtual, depending on the position of the object.

ww
I. Real Image
Let C be the optical centre and F be the principal focus
of a convex lens of focal length CF=f
AB is an object held perpendicular to the principal axis

Flo
of the lens at a distance beyond focal length of the lens. A real,

e
inverted and magnified image A'B' is formed as shown in Fig.

eree
9.39. As A'B'C and ABC are similar.

FFr
A'B' CB'
...(57)
AB CB

oorr
uur r
Again A-^ A'B'F and CDF are similar,
sf
A'B' _ FB'
CD " CF
sk
Yoo
oook

But CD = AB. A'B' _ FB'


...(58)
eBB

AB ~ CF
CB' _ FB' _CB'-CF
From (57) and (58),
uurr

CB~ CF CF
ad

Using New Cartesian Sign Conventions, let CB = - u, CB' = + v,CF = ■¥ f


Yo

V _v-f
dY

-u /
Re
innd

vf= - uv + uf or
uv = uf- vf
Divide both sides by uvf,
FFi

uv w/ vf
or ...(59)
uvf uvf uvf f V u
This is the lens formula.
II. Virtual Image
When the object AB is held close to the lens,
between C and F, the image A'B' formed by convex
lens is virtual, erect and magnified, as shown in Fig.
9.50.

As A'* A'B'C and ABC are similar.

A'B' _ CB' ...(60)


AB ~ CB
9/38 'Pn4}uUefr U Fundamental Physics (XlDiiigijd
Again as A'^F and CDF are similar
MB' ^ B'F
CD ~ CF
But CD=AB

MB' _ B'F ...(61)


AB ~ CF

From (60) and (61),


C^_^ CB' + CF
CB ” CF CF

Using New Cartesian Sign Conventions, let CB = -u, CB' = ~v, CF = +/

oww
-V -u + /
— u f
uv - uf= - vf
uv - uf- vf

e
Divide both sides by uvf

ree
rFl uv uf l = i-i

Fre
or

uvf uvf uvf f V u

rr F
which is the lens formula. Note that lens formula is the same whether image is real or virtual,
ouur
{b) '^.'oncave Lens.
sfoo
In a concave lens, the image formed is always virtual, whatever be the position of the object. Let C be
the optical centre and F be the principal focus of a concave lens of focal length/
kks
oooo
Yo
AB is an object held perpendicular to the principal axis of the lens. A virtual, erect and smaller image
MB' is formed due to refraction through concave lens as shown in Fig. 9.51.
eBB

As A^ MB'C and ABC are similar,


MB' _ CB'
uurr

...(62)
AB ~ CB
ad
YYo

Again as M A'B'F and CDF are similar,


MB' _ B'F
dd

CD ~ CF
Re
iinn

But CD = AB,
F

MB' _ B'F ...(63)


AB ~ CF
From (62) and (63),

CB' _ B'F CF - CB' ...(64)


CF

Using New Cartesian Sign Conventions, let


CB = - u, CB' = -v , CF = -f
-v -f+v
— u -f
vf= uf - uv
uv = uf-vf
RAY OPTICS AND OPTICAL INSTRUMENTS 9/39

Divide both sides by uvf


uv
uf vf
uvf uvf uvf

l-i_i
f V u

which is the lens formula. Note that lens formula is same whether the lens is convex or concave.

Retain in Memory

Note that lens formula is — = ^ —* , which is different from mirror formula : — = i- + - and
/ V u
/ ^ u

ww
1 I 1
lens maker’s formula : — = (li-1) —
/ R
I

Floo
Sample Problem A double convex lens is made of glass of refractive index 1-55 with both

e
eere
aces of same radius of curvature. Find the radius of curvature required, if focal length is 20 cm.
(CBSE 2017)

FFr
Solution. Here, = 1-55,/= 20 cm

oorr
uur r
Let
R^=-R2 = R s ff
1 1 1 1
From = 4i-i) -;r-
R
^
20
= (1.55-1) 1R + 1R 0-55x2

f R
sk
I
YYoo
ooko

/? = 20 X 0-55 X 2 = 22 cm
eBB

J.30. LINEAR MAGNIFICATION PRODUCED BY A LENS

The linear magniifcation produced by a lens is defined as the ratio of the size of the image (Aj)
uurr

to the size of the object (h j). It is represented by m. Thus


ad
Yo

in
_ size of image (A"gp _ 1^2
size of object (AB)
dY
Re
nind

In a convex lens, when image formed is real, Fig. 9.49, from (57),
FFi

A'B' _ CB' ~l^ +V V

AB ~ CB
m ~ or m = ...(65)
— u — u

When image formed is virtual, Fig. 9.50,


A'B' CB' -V V
from (60), or or m = — - ...(66)
AB ~ CB — u h u
1

Thus in a convex lens, lear magnification is positive, when image formed is virtual and linear
tagnification is negative, when image formed is real.
In a concave lens, image formed is always virtual, Fig. 9.51. From (62),
A'B' CB' . -v _v
, i.e.. or m =
AB CB — u u

i.e., linear magnification in case of a concave lens is always positive.


9/40 Fundamental Physics (XlI)KigiaiJ

Linear magnification in terms of u and/

From lens formula,


V u /

u u u+f
+ \-
Multiply both sides by w, or
/
V /

V
m =
f
As m = —
u u+f
Linear magnification in terms of v and/
From lens formula, 1-1 ^

ww
V li /
Multiply both sides by v
1 —
V V V V f-v

Flo
or
II / u / /

ee
f-v

rere
V
ni = —

r FF
It f

A diverging lens of focal length 15 cm forms an image 10 cm from the lens


uurr
Sample Problem

Calculate the distance of the object from the lens,


image?
foor
given \l = 1*5. What is the linear magnification of thi
ks s
Sol. Here,/= - 15 cm, u = - 10 cm, n = ?, fi = 1-5
Yoo
oook

As
i_i ^ I 1 1 J_ 1 ^-3 + 2_-l u = - 30 cm
eBB

V u / u t' / -10 15 30 30

V -10 _1
Now,
uurr

m- —
u -30 ~ 3
ad
Yo

9.31. POWER OF A LENS


dY

Power of a lens is deifned as the ability of the lens to converge a beam of light falling on the
Re
innd

lens.
FFi

It is measured as the reciprocal of focal length of the lens


1
i.e., P = - ...(67
f

-=(1-1-0 4--'
1
According to lens maker’s formula, R
f 1 Rl

1 1 ^
from (67), F = 4l-1) ...(68
R
1 «2

For a converging lens, power is positive (as / is +) and for a diverging lens, power is negative (as/i
negative).
The SI unit of power is dioptre (D)
RAY OPTICS AND OPTICAL INSTRUMENTS 9/41

1 1
When/ = 1 m.
P = y = j = 1 dioptre
Hence, one dipotre is the power of a lens offocal length one metre.
] 100
When /is in cm, P =
//lOO /

9.32. COMBINATION OF THIN LENSES IN CONTACT


In various optical instruments, two or more lenses are often combined to
(0 increase the magnification of the image,

ww
(ii) make the final image erect w.r.t. the object,
(Hi) reduce certain aberrations (i.e. defects of images by single lens).
The position, size and nature of the final image produced by a combination of thin lenses can be obtained

Flo
'yy drawing the ray diagrams and applying the lens formula (even without the diagram) to the successive
enses. We first find the image of the object, formed by the first lens. This image acts as an object for the

e e
second lens and we locate its image. The image formed by the second lens serves as the object for the third

reer
ens and so on.

rFF
9.33. FOCAL LENGTH OF EQUIVALENT LENS
uur r
(a) Both the lenses are convex
Let C], C2 be the optical centres of two thin
ffoor
sks
:onvex lenses and L2 held co-axially in contact with
YYoo
;ach other in air. Suppose f^ and/2 are their respective
ooko

deal lengths. Fig. 9.52.


eBB

Let a point object O be placed on the common


)rincipal axis at a distance OC, = u. The lens Lj alone
vould form its image at /' where Cf' = v'. From the
uurr

ens formula.
ad
Yo

...(69)
v' u
dY
Re

/' would serve as a virtual object for lens L2, which


innd

orms a final image I at distance C2I-V.


FFi

As the lenses are thin, therefore, for the lens u = C2f ^ C^I' = v'
1 1 1
From the lens formula for ...(70)
V
/2
1 1 I 1
Adding (69) and (70), we get V u ...(71)

Let the two lenses be replaced by a single lens of focal length F, which forms image / at distance v, of an
bject at distance u from the lens. For this lens.
1 1

V u F ...(72)

From (71) and (72), we get j__J_ _1_


ff fl ...(73)
9/42 Fundamental Physics (Xll)gx^iJ^H

or
F=hlh. ...(74)
/1+/2

{b) One lens is convex and the other is concave


lIYi is focal length of convex lens and/, is focal length of concave lens, then their equivalent focal
length F would be given by
1 1 1 j I
F /, /i fi

...(75i
fl-A

ww
Three cases ari;- .

(a) If =/2, then from (75), F= The combination would behave like a plane glass plate.
ib) If /| >/2- then from (75), F is negative. Therefore, the combination would behave as a concavt

Flo
lens, when focal length of convex lens is larger (i.e., when power of convex lens is smaller),

e
(c) If /] < /2, then from (75), F is positive. Therefore, the combination would behave as a conve?

rere
lens, when focal length of convex lens is smaller (i.e., when power of convex lens is larger).

r FF
IMPORTANT NOTE
1. The above derivation can obviously be carried through several thin lenses of focal lengths
uurr
h'fl'h- for
contact. The effective focal length of the combination would be given by
kss
1 1 1 1
— — —I 1 1-...
ooook

/ fx fl h
Yo

2. As each lens magnifies the image formed by the preceding lens, therefore total magnification of the
eBB

lens combination is the product of the magnifications produced by individual lenses, i.e..
m = Wj X /«2 X W3 X ... ...(76)
urr
ad

3. If the lenses of focal lengths /i and/2 separated by a finite distance d, the focal length F of the
Yo

equivalent lens is given by


dY

J__J_ J ^ ...(77)
Re
innd

^ /l /2 f\f2
Fi

In terms of power, we can write this relation as


P = P^+P2~dxPiP2
Suitable lens combinations help in increasing the magnification and sharpness of the image.
A system of combination of lenses is commonly used in the design of objectives/eye pieces of cameras,
microscopes, telescopes and other optical instruments.

Rmin in Marmwy
When two thin lenses of focal lengths/j and/2 are held in contact with eachother, the focal length
1 1 1
F of the combination is given by — h—
^ /l /2
When two thin lenses of equal and opposite focal lengths (i.e., one convex and other concave) are
placed in contact, their equivalent focal length is given by
RAY OPTICS AND OPTICAL INSTRUMENTS 9/43

1=1 J_ = 0 ox

and power, P = ;? - p = 0. Such a combination of lenses would behave as a plane glass plate.
In terms of power: P = P, + P2

In general, P = P,-^P2 + P^+...


i.e. total power of any number of ■ jnses in contact is equal to algebraic sum ofth -j pov f
individual lenses.

If the sum of the terms on right side is positive, the system of lenses would behave as convex. If the

ww
sum of the terms on irght side is negative, the system of lenses would behave as concave.

Sample Problem Two thin lenses are in contact and the focal length of the combination is

Floo
80 cm. If the focal length of one lens is 20 cm, then what would be the power of the other lens ?
Sol. Here, P'= 80 cm,/j = 20 cm, P2 ~ ^

ee
eer
100 100
P =
100 _ 100 p = = 5D
= 1-25D,
F “W
1
/l 20

FFr
oorr
As Pl + P2 = P
uur r
P2 = P-P, = l-25-5 = -3‘75D s ff
9.34. COMBINATION OF A LENS AND A MIRROR
Suppose we have a coaxial arrangement of a lens and a mirror held suitable distance apart. Let an object
sk
YYoo
ooko

be placed in front of the lens. To study the nature and size of image formed by the combination, we proceed
as follows:
eBB

(i) Using refraction formulae, we can calculate the position, size and nature of image of the given
object, formed by the lens alone {i.e., as if the mirror were not there).
uurr

(») The image so formed may act as real or virtual object for the given mirror. Using mirror formula, we
ad

can calculate the position, size and nature of the image formed by the mirror.
Yo

The calculations so made will reveal the final position, size and nature of image of the given object
formed by the combination of lens and mirror.
dY
Re

The following example illustrates the procedure we have outlined above :


ind
FFin

Example. A convex lens of focal length 20 cm has a point object placed on its principal axis at a
distance of 40 cm from it. A plane mirror is placed 30 cm behind the convex lens. Locate the position of
image formed by this combination.
Solution. Here, /= 20 cm, ir = - 40 cm, u = ?
I 1 1
As
V u f

1 1

" V f u

^J 1__ J_
20 40 ~ 40
V =40 cm,
i.e., C/j=40 cm, Fig. 9.53.
9/44 Fundamental Physics (XII)Ltsiaij

The plane minor is at M, where CM = 30 cm. If there were no mirror, lens would form a real image at .
As the mirror intercepts the rays refracted from the convex lens, the real image is formed at / on account of
reflection of light from the plane mirror.
Now, /] acts as a virtual object for plane mirror, where M/| = C/j - CM = 40 - 30 = 10 cm. Therefore,
a
real image of /j will be formed by the plane mirror at /, where M/ = M/] = 10 cm.
Hence, final image is formed at 10 cm in front of plane mirror.

935. DISPLACEMENT METHOD TO FIND POSITION OF


IMAGES (CONJUGATE POINTS): NEWTON'S RELATION
Suppose /is focal length of a convex lens. It is found that when the distance between an object and
screen, D is greater than 4/ then there are two positions of the lens for which the image of the object on the

ww
screen is distinct and clear. In these two positions of the lens, the distances of object and image from the lens
are interchanged.
In Fig. 9.54, Lj is first position of the lens andZ.2 second position of the lens. C,L2 = x = displacement

Flo
of the lens. 0 is size of an object A5. is size of image A^B^ in the first position of lens Lj. I2 is size of image

e
^2^0 in the second position of lens Z^.

eree
= D = distance between object and screen.
Distances of object and image, i.e., u and v are just interchanged in the two positions of the lens as

FFr
shown in Fig. 9.54.
uurr
A,5,

orr
V
In the first position of lens, linear magnification, w, = -
sfo
u AB

u
O

A/^ B^ ^2
kks
and in the second position Ly of lens, linear magnification,
Yoo
m2 = — ~ AB ~ O
V
oooo
eBB
urr
ad
YYo
dd
Re
inn
F

l.I
[‘2
;n^.m2 = = 1 orO = ^
m
1 _
v/u
Again, 2
u/v
m2 u

D-x
From Fig. 9.54 ; U+X + u = D u =
2

D~x D + x
v=D-u=D-
Again, 2 2
RAY OPTICS AND OPTICAL INSTRUMENTS 9/45

Taking, u as negative as per sign convention and using lens formula :


2 2 4D
l-i-I
/ =
f V u D-\-x D-x AD

This is called Newton’s relation for focal length/of the lens. The points B and (or B’^) are called
conjugate points. Corresponding to the conjugate points, distances of object and image from the lens are
interchanged.

9.36. PRISM

A prism is a portion of a transparent medium bounded by two plane

ooww
faces inclined to eachother at a suitable angle. In Fig. 9.55. ABQP and
A CRP are the two refractingfaces. Angle A between them is the refracting
angle or angle of prism.
The line AP where the two refracting faces meet is called the

e
refracting edge of the prism. A section ABC of the prism made by a plane

re e
at right angles to the refracting edge of the prism is called Principal

rFl
Fre
section of the prism. Fig. 9.55.

rrF
9.37. REFRACTION THROUGH A PRISM

A ray of light suffers two refractions on passing


sffoo
ouur
through a prism and hence deviates through a certain
angle from its original path.
oksk
(a) Calculation of angle of deviation
Yo
In Fig. 9.56, ABC is principal section of a prism
oo
Y
BB

with angle of prism = A


A ray of light KL is incident on the face AB of
rre

the prism at Z/j. It is refracted along LM at Zr|


bending towards the normal A^jO. The refracted ray
ouu
Y
ad

LM is incident at Zr2 on face AC of the prism. It


bends away from normal N2 O and emerges along
dY

MN at Zi/- In passing through the prism, ray KL suffers two refractions and has turned through an
innd

ZQPN = b, which is the angle of deviation.


Re

At the face AB, the angle of incidence is /] and angle of refraction is /●[. So the deviation at this surface
Fi
F

IS 6, = /i-r. (clockwise)
At the second face AC, the angle of incidence is rj and angle of refraction is h- So the deviation at this
surface is 62 - h - ^2 (clockwise)
.●. Total deviation produced in the ray in passing through the prism is
5 = §1 + 5^ = (/ - + (*2 - ^2) = ('1 '2) “ (^1 + '■2) ...(78)
In Fig. 9.56, sum of angles of AALM is 180°, i.e.,
A + (90"-r,)+(90“-r2)= 180"
A = (r, + T2) ...(79)

From (78), 5 - (/j + z/) - A ...(80)

Applying Snell’s law at L and M, we get


1 sin /j = p. sin r, and p sin T2 = 1 sin ^2
Here, p is refractive index of the material of the prism.
9/46 “PnAdte^ 4. Fundamental Physics (XII) ar»nm

From known values of /[ and i2, we can calculate and r2 and FIGURE 9.57

hence A using (79). Therefore, 5 is calculated from (80). The graph


of angle of deviation 5 against the angle of incidence (/j) is plotted
as shown in Fig. 9.57.
We observe that the graph is not symmetric about the
minimum. Further, the same deviation occurs for two angles of
incidence /j and i2* Here, is often called the angle of emergence.
From the principle of opticd reversibility, we know that a ray incident
at angle I2 will emerge at angle i^ and the angle of deviation would
remain the same.
«1 /1-/2 /2

(h) Angle of Minimum Dr vl'^tion and Prism Formula

ww
From Fig. 9.58*, we find that at the minimum deviation, there is only one angle of incidence. We may
say that when 5 = 5j^jn ; / | = 12- Snell’s Law would give us 1 sin /| = sin and p sin r2 = 1 sin h
As /| - 'f2 = f, therefore, r| = r2 = r, say

Flo
From(79), A-r+r = 2r, r = AJ2 and

e
A+S -

eree
min.
From (80), = 2/-A or / =
2

FFr
sin;
As
uurr |J = -

orr
sinr

P- =
sfo
...(81)
kks
sin A/2
Yoo
oooo

This is called prism formula. It is used for accurate determination of refractive index of a transparent
medium, of which the prism is made. Note that in the position of minimum deviation, as t j = i2, therefore, the
eBB

incident rays passes symmetrically with respect to the refracting faces of the prism, and the refracted ray in
the prism is parallel to the base of the prism as shown in Fig. 9.58.
urr

(c) Angle of Maximum Deviation


ad

In Fig. 9.59, we have redrawn the graph between angle of


YYo

FIGURE 9.59
incidence and angle of deviation. From the graph, we find that
deviation is maximum (= angles of incidence,
dd

and *max- *max “ angle of incidence, angle of emergence


Re
inn

will be and vice-versa. For i < , the angle of incidence K2


F

on second face AC of prism will be greater than critical angle for


prism air interface. The ray will undergo total internal reflection
and will not emerge from the prism. Usually, can be any value
depending on angle of prism and refractive index of the material of
^min i ^max "
the prism. Thus angle of maximum deviation is obtained from the
graph of Fig. 9.59.

*Instead of referring to the graph, we can obtain the same result mathematically :

From (80), 8 = (i| + 12)-A +(.^)^ - A = -2.,jIJ^ + 2^JiJ^-A


5 = (^^ - 7^)^ + 2 (.J/^) - A
5 will be minimum, when whole square on R.H.S. is minimum = 0, i.e.,
ot or i.e.. t\-12
RAY OPTICS AND OPTICAL INSTRUMENTS 9/47

938. THIN PRISM

A thin prism is that whose refracting angle or angle of prism is small. As proved in Art. 9.37, the angle
of deviation of a ray on passing through a prism is
5 = /1 +12 ~ ^ ...(82)
According to Snell’s Law, applied at L, Fig. 9.56, sin / j = |i sin
For small angle of incidence ; / j = p rj
Since i is small, r^ is also small.
From A = r, + T2, T2 would also be small. /2 = P ^2
From (82), 5=pr, + pr2“A = p (r, +r2)-A=pA-A

ooww
5 = (p-l)A ...(83)

Clearly, when angle of incidence is small, the angle of deviation 6 is independent of tlie angle of incidence,
i.e., all the rays (at small angles of incidence) undergo the same deviation on passing through the prism.

ee
Sample Problem A ray of light falling at an angle of 50" is refracted through a prism and
suffers minimum deviation. The angle of the prism is 60". Find the angle of minimum deviation and

r
rFFl
ree
refractive index of the material of the prism.

F
Sol. Here, = 50°, A = 60°, 6 = ?,P = ?

rF
m

In position of minimum deviation, I2 = i\ = 50°


From A + 5,„ = /, + i2 = 50 + 50 = 100°
fsfoor
ouur
5...
m
= 100° - A = 100 - 60° = 40°
kosk
sin (A + 5^j)/2 sin (60°+40°)/2 sin 50° 0-7660
= 1*532
!4 =
Yo
sin A/2 sin 60°/2 sin 30° 0-5000
oo
Y
BB

9.39. DISPERSION OF LIGHT


rre

Dispersion of light is the phenomenon of splitting of a beam of white light into its constituent
oYuu

colours on passing through a prism. The band of seven colours so obtained is called the (visible)
ad

spectrum.
dY

The order of colours from the lower end of the


spectrum is Violet (V), Indigo (/), Blue (B), Green
innd
Re

(G), Yellow (y), Orange (0) and Red (R) and can be
Fi

recollected by the word VIBGYOR.


F

As is clear from Fig. 9.60, different colours turn


through different angles on passing through the prism.
Violet colour deviates through maximum angle and
red colour deviates through the minimum angle.
Cause of dispersion. Each colour has its own
characteristic wavelength (X).
According to Cauchy’s formula, refractive
index (p) of a material depends on wavelength (X) of
light falling on it as
B C
p = A+ ...(84)

where A, B, C. are constants.

.*. p of material of prism is different for different colours/wavelengths.


9/48 'pfuidee^ ^ Fundamental Physics (XII) PZSTWTl
The variation of \x with X in case of fused quartz is shown
in Fig. 9.61.
For prisms of small refracting angle, we have shown that
angle of deviation is

5 = (n-l)/l
As p of prism is different for different colours, therefore,
different colours deviate through different angles on passing
through the prism. Hence they are seen as separate. This is the
cause of dispersion.
As
“^violet < X red ' therefore, ^violet ^ ^red
Hence, ^violet >s red

ww
i.e. max. deviation is of violet colour. That is why violet colour is at the lower end of the spectrum and
red colour is at the upper end.
The refractive indices for crown and flint glass for five typical wavelengths are shown in Table 9.3.

Flo
TABLE 9.3, Refractive indices for crown and flint glass for different wavelengths

e e
Colour Wavelength A Crown Glass (p.) Flint Glass (^)

reer
rFF
1. Violet 3969 1.532 1.663
2. Blue 4861 1.523 1.639
uur r
3.
4.
Green
Yellow
5461
5890
ffoor
1.519
1.517
1.635
1.627
sks
5. Red 6563 I.5I5 1.622
YYoo
ooko

9.40. ANGULAR DISPERSION


eBB

Angular dispersion produced by a prism for white light is the difference in the angles of deviation
of two extreme colours i.e., violet and red colours.
uurr
ad

In other words, it is the angle in which all colours


Yo

of white light are contained. Fig. 9.62.


dY

If 5^, = deviation of violet colour,


Re

5^ = deviation of red colour,


innd

then. Angular dispersion = - 5^


FFi

Now, the deviation 5 through a thin prism of


refracting angle A is
5 = (p-l)A
5^, = (p,,- 1)A ...(85)
and 6, = (p,-l)A ...(86)
Subtracting, (86) from (85), we get
6j,-5^=(p„- l)A-(p^- l)A = (p„- 1 -p^+ I)A
.●. Angular dispersion. (6^-6,) = (p^,-fir)^ ...(87)

Obviously, angular dispersion produced by a prism depends upon (i) angle of prism, (ii) nature of
material of the prism. Note that chromatic dispersion occurs right at the first surface of the prism and
the .same is increased at the second surface of the prism.
RAY OPTICS AND OPTICAL INSTRUMENTS 9/49

Angular dispersion and wavelength


Angular dispersion, (61;- 5^) = 0, say is also defined as the ratio of change of angle of deviation 8 with
dd
wavelength X, i.e.. 0 =
dX

As5 = (|i-l)A = }iA-A ...(0


d[l
B
From Cauchy’s relation, - A' + X2 where A' and B are constants

^__2S ...(h)
dX ~

ooww
Multiplying (/) and (/V), we get
db d\x 2B\ db 1
— X — = A or = 0 OC

dX dX X3

e
i.e., angular dispersion of a prism is inversely proportional to the cube of the wavelength of colour.

ree
rFl
Fre
9.41. DISPERSIVE POWER

rrF
The dispersive power of a prism is defined as the ratio ofangular dispersion to the mean deviation
produced by the prism. It is represented by (a
sffoo
ouur
The mean deviation produced by the prism is 5 = (p-l)A
Angular dispersion produced by the prism is
okks
Yo
As dispersive power = angular dispersion
oo
Y

mean deviation
BB

0) =
5.-5. ...(88)
rre

8
ouu
Y
ad

(0 =
(H-DA
dY

(0 = ...(89)
innd

M-1 |X-1
Re

where [1,^ - d\y {i.e., difference in refractive indices of prism material for violet and red colours).
Fi
F

Clearly, co depends only on nature of material of the prism.


From (88), 8^ - 8^ = to x 6, i.e..
Angular dispersion = dispersive power x mean deviation
We can suitably combine two prisms of different materials so as to produce :
(0 Dispersion without deviation, and (ii) Deviation without dispersion.
Sample Problem Calculate the dispersive power for crown glass from the given data :
= 1-5230, 1-5145.
Sol. Here, = 1-5230, p, = 1-5145, to = ?
1-5230+ 1-5145
Mean refractive index, 4 = = 1-5187
2 2
0-0085
co =
p,, -P^ _ 1-5230-1-5145 r = 0-0163
p-1 1-5187-1 0-5187
9/50
Fundamental Physics (XII)CEIHD
9.42. SCATTERING OF LIGHT

Most of the beautiful phenomena like ‘Blue colour of sky\ ‘white colour of clouds', ‘ red hues of sunrise
and sunset', have been explained in terms of scattering of light.
As sunlight travels through the earth’s atmosphere, it gets scattered by the large number of molecules
and numerous small panicles present in the atmosphere. Scattering represents basically change in the direction
of light.
Lord Rayleigh was the first to study scattering of light from air molecules.

w
If A = amplitude of incident light, X = wavelength of incident light,
V - volume of the scattering particle,
(I = amplitude of scattered light at a distance r from the scattering particle,

e
then according to Rayleigh,

row
re
When size of scatterer is much smaller than wavelength of light, then
AV

FFllo
eeF
a =
r X"

u
As intensity of scattered light varies directly as square of amplitude of scattered light (a\ therefore,

r
sFr
1
/ oc _

kro
uor
offo
ue. intensity of scattered light varies inversely as the fourth power of the wavelength of incident light.
These conclusions were verified experimentally by Tyndall.
kos
Y
Yo
Rayleigh established further that the rays do not undergo any change in wavelength on scattering.
eerBB
oo

However, in case of scattering of light by transparent media, the scattered light might contain some
rY

frequencies or wavelengths, which might not be present in the incident radiations.


When some energy from incident light is imparted to the scattering molecule, the scattered light will
u

have lesser energy, i.e., smaller frequency and longer wavelength. On the contrary, when the scattering molecule
ou
ad
do

is in a higher energy state before the collision than after the collision some energy is received from the
scattering molecule. The scattered light will have higher energy, i.e., longer frequency and shorter wavelength.
nY

However, when there is no exchange of energy between the incident light and the scattering molecule,
nid

there is no change in wavelength or frequency on scattering, as postulated by Rayleigh.


Re

Another important factor in scattering is relative size of the wavelength of light (X) and the scatterer
F
Fi

(of typical size, say, x). For x « X, Rayleigh scattering is valid, i.e., when size of scatterer is much
smaller than wavelength of light, OC
1/X^. But for X » X, Rayleigh scattering is not valid and all
wavelengths are scattered nearly equally.
SOME APPLICATIONS OF SCATTERING OF LIGHT ARE :
(fl) Blue colour of sky is due to scattering of sunlight. Light from the sun, while travelling through
earth’s atmosphere, gets scattered by large number of molecules in the earth’s atmosphere. As .r « X, Rayleigh
scattering is valid. The intensity of scattered light vjiries inversely as the fourth power of wavelength of light.
As blue colour has a shorter wavelength than red, therefore, blue colour is scattered much more strongly.
Hence when the sky is clear, it looks blue.
{b) White colour of clouds. The clouds are at much lower height. They are seen due to scattering of
light from lower parts of the atmosphere, which contains large dust particles, water droplets, ice particles etc.
In this case, size ol scatterer a- » X. Therefore, all wavelengths are scattered nearly equally. All colours
scattered equally merge to give us the sensation of white. Hence clouds generally appear white.
RAY OPTICS AND OPTICAL INSTRUMENTS 9/51

(c) The sun looks reddish at the time of sun FIGURE 9.63
rise and sun set. At the time of sun irse and sun set, SUN NEARLY

the sun is near the horizon. The rays from the sun OVERHEAD;

have to travel a larger part of the atmosphere, Fig.


” LESS BLUE
9.63. As Xfj« and intensity of scattered light BLUE SCATTERED SCATTERED
1 AWAY : SUN-REDDISH

, therefore, most of the blue light is scattered


oc
I

X^
:* 1*

n Alrhosphere^^vv.
away. Only red colour, which is least scattered enters SUNiNE/^R-.
ij

our eyes and appears to come from the sun. Hence HORtZON^;^ OBSERVER
(at sunrise^nd'
the sun looks red, both at the time of sun irse and sun ON EARTH
sun set)
set.

id) Danger signals are red. Because wavelength of red colour is large and intensity of scattered light

ww
varies inversely as the fourth power of wavelength, therehire, red colour is least scattered. It can be seen from
maximum distance.

That is why danger signals are red.

Flo
e
9.43. RAMAN EFFECT

eree
Raman Effect is the phenomenon of scattering of light of frequency v by a substance. The scattered light
contains the original frequency v and a series of lines of lower and higher frequencies. The phenomenon was

FFr
discovered by Indian Physicist, Sir C.V. Raman in the year 1928. He was awarded Nobel Prize in 1930 for
uurr
this discovery.

orr
Usually, a mercury lamp provides the required radiation. The spectrum of radiations from this lamp
sfo
contains several intense lines. A spectrograph is used to obtain a photograph of the scattered radiation spectrum.
The spectrograph is held at right angles to the incident light.
kks
Yoo
In the scattered radiation spectrum; about each excited line of incident radiation, there appear identical
oooo

groups of considerably weaker lines spaced symmetrically to the irght and to the left ot original line. The
eBB

scattered lines, however, differ in intensity.


This spectral pattern can be explained as follows : Basically, a photon of energy fiv is scattered by a
molecule elastically, i.e., without loss of energy. Therefore, its frequency v on scattering remains constant.
urr

However, in addition to such scattering, it is also possible to have scattering with some loss of energy.
ad
YYo

Such energy is spent up in the transition of scattering molecule from one level to another. If is energy lost
by incident photon = energy spent in raising the scattering molecule from one vibrational level to the other,
then frequency v' of scattered photon is given by liv' = (hv - £j). Clearly, v' < v. The scattered line is towards
dd

lower frequency or higher wavelength, i.e., towards red end of the spectrum.
Re
inn

Again, when an incident photon hits a scattering molecule in excited state, the scattered photon gains
F

the extra energy E2 due to transition of scattering molecule to a lower level. The frequency v of scattered
photon is given by : hv" = (hv + E2). Clearly, v" > v, i.e., the scattered line is towards higher frequency or
lower wavelength, i.e., towards violet end of the spectrum.
The ratio of intensities of violet lines to the red lines is proportional to the ratio of number of atoms in
the first excited state to the number of atoms in ground state. At low temperatures, violet lines practically
disappear because most scattering molecules are at the zero level.
Raman spectra are usually obtained by scattering from liquids.

9.44. RAINBOW
A rainbow is a spectrum of sun's light in nature. It occurs in the form of concentric coloured circular
arcs in the sky, when the sun shines on rain drops during or after a shower.
The essential conditionfor observing a rainbow is that the observer must stand with his back towards
the sun. The common centre of concentric circular arcs lies on the line joining the sun to the eye of the
observer.
9/52 Fundamental Physics fXintTOWn

Generally, there are two kinds of rainbows:


1. Primary rainbow, 2. Secondary rainbow.
The inner of the two bows seen is the primary
rainbow. It is brighter of the two and subtends an
angle of 42® at the eye of the observer. It shows all
the colours of the solar spectrum with red colour on
the outside (niciking an angle of 43®) and violet colour
on the inside (making an angle of 41°). As shown in
Fig. 9.64. The primary rainbow is formed by the rays
which undergo one internal reflection and two
refractions before emerging finally from the water
droplets.

ww
The outer of the two bows is called secondary
rainbow. It is not as bright as the primary one. It
subtends an angle of about 52.5® at the eye of the

Flo
observer. It also shows all the colours of the solar
spectrum but in reverse order i.e. red on the inner edge making an angle of 51° and violet on the outer edge

ee
making an angle of 54°, as shown in Fig. 9.64. Secondary rainbow is formed by the rays which have undergone

rere
two internal reflections and two refractions before emerging finally from the water droplets.

rFF
9.45. OPTICAL INSTRUMENTS
uurr
foor
The optical instruments are the devices which help human eye (0 in observing highly magnified images
of tiny objects, for detailed examination, (ii) in observing very far off objects— terrestrial or astronomical.
ks s
Microscopes and Telescopes are common examples of optical instruments. The essential optical
Yoo
components of these instruments are lenses and mirrors. The images are formed by refraction through lenses
oook

and by reflection at mirrors. In dealing with the optical instruments, we shall continue to use the ray
eBB

approximation and assume that light travels in straight lines. Thus the optical instruments belong to the
field of ray optics or geometrical optics. We start with the eye and then go on to describe the principles of
working of a microscope and a telescope.
uurr
ad

9.46. THE EYE


Yo

The eye is far more delicate and perfect than even


dY

the finest photographic camera designed so far. The


Re

anatomical features of the eye are shown in Fig. 9.65.


innd

The outer part, called the eye ball is nearly spherical


FFi

with a diameter of about 25 mm. Light enters the eye


through a transparent white membrane, called cornea. It
covers a clear liquid called aqueous humor, behind which
there is a diaphragm, called iris with a circular aperture,
called the pupil of the eye. Behind the iris, is a crystalline
lens held in position by the ciliary muscles. The region
behind the crystalline lens is filled with a jelly like substance,
called the vitreous humor and finally a light sensitive part
of the eye, called retina which covers the curved back surface of the eye. The retina is a film of nerve fibers
consisting of millions of structures called rods and cones. The rods sense light intensity and cones sense the
colours. When stimulated by light, these structures send electrical impulses to the brain via the optic nerve.
The brain finally interprets the image on the retina.
The least sensitive spot on the retina is called the blind spot. There is another spot at about the centre of
the retina, which is most sensitive to light. This is called yellow spot.
RAY OPTICS AND OPTICAL INSTRUMENTS 9/53

The line joining the centre of the crystalline lens to yellow spot is called visual axis of the eye, and the
line joining the centre of crystalline lens to the centre of cornea is called the optic axis of the eye.
Note that iris is the coloured portion of the eye, which controls the amount of light reaching the retina.
This control comes due to variable opening at its centre. This opening is called pupil and its diameter varies
from 2 mm to 7 mm, decreasing in bright light and increasing in dim light.
Note that in reaching the retina, light travels through five different media with different refractive indices : tur

(p = 1-00), the cornea (p = 1-38), the aqueous humor (p = 1-33), the crystalline lens (p = 1-40) and the vitteous
humor (p = 1 -34). Each time light passes from one medium to another, it is refracted at tlie boundary. About 70%
of refraction occurs at the air/comea boundary. The lens accounts for 20-25% of the total refraction.
One of the remarkable properties of the eye is the ability of the eye to observe distinctly the objects
situated at widely different distances from the eye. This property of the eye is called accommodating power
or accommodation of the eye. It becomes possible by changing the curvature of crystalline lens and hence its

ww
focal length by a change in tension in the ciliary muscles holding the lens. For example, when the eye looks at
a very far off object, the ciliary muscle is not tensed. The crystalline lens has its least thickners and longest
focal length. The eye is said to be fully relaxed. Focal length of the lens is about 2-5 cm and objects at infinity
are in sharp focus on the retina.

Flo
When the object moves closer to the eye, the ciliary muscle tenses automatically, thereby increasing the

e
thickness and decreasing the focal length of eye lens. The most distant point which an eye can observe clearly

eree
is called Far Point (F) of the eye. For a normal eye, F lies at infinity.
The closest distance for which lens can focus light on the retina is called the least distance of distinct

FFr
vision or the distance of Near point (N) of the eye. For a normal eye, value of least distance of distinct

oorr
uur r
vision, d = 25 cm. This distance increases with age, because of decreasing effectiveness of the ciliary muscles
and loss of flexibility of the eye lens.
sf
The distance between near point and far point of the eye is called Range of vision.
sk
Yoo
9.47. DEFECTS OF VISION
oook

Inspite of all precautions and proactive action, our eyes may develop some defects due to various
eBB

reasons. Some of the common optical defects of the eye are


1. Myopia or short sightedness. FIGURE 9.66
uurr

2. Hypermetropia or long sightedness.


ad

3. Presbyopia.
Yo

4. Astigmatism.
R Normal Eye
dY

5. Phorias. {Sharp image of


Re

We discuss here the first two defects in some objecl at infinity)


innd

detail,
FFi

(a) Myopia or short siglitedness


Myopia or short sightedness is that defect of
human eye by virtue of which, the eye can see clearly
the objects lying near it, but the far off objects R Myopic Eye
[Blurred image of
cannot be seen distinctly i.e. objects lying beyond a object at infinity
particular distance cannot be seen clearly by the eye. and sharp image
of object at F
In other for a myopic eye, the far point shifts (far point)]
towards the eye. It is no longer at infinity. It is at F.
In Fig. 9.66(a), parallel rays from infinity are
focussed on the retina by the normal eye. In Fig.
9.66(h), the myopic eye focusses the parallel rays F
R Corrected
myopic aye
from infinity at P in front of the retina. However, the (Sharp image of
defective eye focusses rays from a point F on the >4-
object at infinity)
retina. Thus F is the far point of the myopic eye. It
cannot see clearly beyond F.
9/54 'pfuuUefi^ ^ Fundamental Physics (Xll) lagiafll
The two possible causes of this defect are :
(/) Increase in size of the eye ball, i.e., distance of retina from the eye lens increases.
(fi) Decrease in focal length of the eye lens, when the eye is fully relaxed.
To correct a myopic eye of this defect, the person has to use spectacles with a concave lens of suitable
focal length. Infact, parallel rays of light from infinity, after refraction through the concave lens, should
appear to come from F, the far point of the defective eye. This is shown in Fig. 9.2 (c).
Let a: be the distance of far point of the myopic eye.
and/ be the focal length of concave lens to be used.

w
Now, for concave lens, u = o°,v = -x.
1 1 1 1 -1 1 „
As or — = or/=-A:
/ ^ U / OO

roow
e
Hence focal length of concave lens put in front of the eye should be equal to distance offar point of

re
the defective eye.
(£>) Hypermetropia or Long Siglitedness

reF
uFFll
Hypermetropia or long sightedness is that

e
defect of human eye, by virtue of which the eye can
see clearly the far off objects, but the nearby objects

sFr
cannot be seen clearly, i.e., objects which are

foro
lying nearer than a certain distance cannot be seen
uor
by the eye. In other words,/or a hypermetropic eye,
the near point shifts away from the eye. It is no longerfk
okso
at 25 cm, the least distance of distinct vision for the
Y
normal eye.
Yo
oo
BB

In Fig. 9.67 (a), rays from the near point (AO


are focussed on the retina by the normal eye. N is at
the least distance of distinct vision (25 cm).
rYree
ouu

In Fig. 9.67 (b), the hypermetropic eye focusses


the rays from at F', a point beyond the retina.
ad
Ydo

Therefore, light rays from N, would form a sharp image


behind tlie retina, if they could do so. In reality, the image
nidn

formed on the retina is blurred. However, the defective


Re

eye focusses rays from another point A^'on the retina.


F

Thus the near point of hypermetropic eye has shifted


Fi

away from N to N'.


The two possible causes of this defect are :
(/) Contraction in the size of the eye ball, i.e., distance of retina from the eye lens has decreased.
(«) Increase in the focal length of the eye lens, when the eye is fully relaxed.
To correct -j hypermetropic eye of this defect, the person has to use spectacles with a convex lens of
suitable focal length. Infact, the rays starting from N, after refraction through the convex lens, should appear
to come from N', so that they are focussed on the retina. Fig. 9.67 (c).
Suppose .r' = the distance of near point N' of the defective eye,
d = the least distance of distinct vision (of normal eye), / = focal length of convex lens to be used.
For the correcting lens, u = -d,v = -x'.
x'd
As
I 1 j__J_ 1 _-d + x' / =
J~-x''^d~ x'd
or
x'~d
f y u

As x' > d,f \s +, the correcting lens must be convex.


RAY OPTICS AND OPTICAL INSTRUMENTS 9/55

(c) Presbyopia (old sight)


With increasing age, the ciliary muscles holding the eye lens
weaken and the lens loses some of its elasticity. Therefore, power of DO YOU KNOW ?
accommodation of the eye decreases with age. This defect is called In hypermetropia, length of eye
presbyopia. This defect (presbyopia) is similar to hypermetropia, but bail gels shortened or focal length
the causes of presbyopia and hypermetropia are different as explained. of fully relaxed eye lens
To remove presbyopia, converging spectacle lenses are employed (as increases. But in presbyopia, the
in the case of hypermetropia). The person needs to wear these specs for eye ball has normal length. There
reading or similar close work. A person suffering from presbyopia will is increase in focal length of fully
normally be prescribed bifocal specs. The upper half of each lens is relaxed eye lens.
diverging and corrects the myopia, when the wearer is looking ahead at

oww
distant objects. The lower half of each lens is convex and corrects the
defect when the wearer looks through this part while reading.
(d) Astigmatism
U is yet another common defect of vision, which occurs when cornea has unequal curvature in different

e
rFFlo
directions. For example, cornea may have larger curvature in the vertical plane than in the horizontal plane or

re
vice-versa. Therefore, such a person is not able to focus simultaneously on horizontal and vertical lines. If

ree
horizontal lines are well focussed, the vertical lines appear distorted or vice-versa, as shown in Fig. 9.68(a).

F
rF
fsfoor
ouur
kosk
Yo
oo
Y
BB
rre
oYuu
ad

From the distribution of the intensity vai iation, an


optometrist deduces the variations in the curvature of
the cornea. Astigmatism is corrected by using a cylindrical
dY

lens of suitable radius of curvature and suitable


axis. To correct astigmatism only, one surface of each lens will be flat and the other surface of each lens will
be a section of circular cylinder, with the axis of cylinder horizontal. Rays passing through the lens in a
innd
Re

horizontal plane are unaffected, and rays in vertical plane are converged by the circular surface. The extra
Fi

convergence is such as to compensate for the lack of curvature of the cornea in that direction.
F

The corrected a.stigmatic eye is shown in Fig. 9.68(h).


However, when eye is suffering from some other defect along with astigmatism, the spectacle lens
surface is made foroidal. The two radii of curvature are chosen as to compensate for that defect and the
astigmatism at the same time.
Sample Problem Q A man’s shortest distance of distinct vision is 20 cm. What will be the type
and power of the spectacle lens which he would require to enable him to read a book at a distance of 60 cm ?
Sol. Here, V =-20 CD ' = ?, « = - 60 cm.

As l-i-i. J__ 1_ J_ _ -3 + 1 ^ -2 1_
f V u / ~ 60 “ 60 “ 30
/ = - 30 cm. Therefore, lens must be concave.

100 ^ 100
/ _ 2Q = - 3‘3 dioptre
9/56 ‘Pntuieefi- ^ Fundamental Physics (XII) EEgD
Sample Problem 0 A person’s far point is at 2 m. Find nature, focal length and power of the
lens he must use to see distant objects clearly.
Sol. Here, the distance of far point, a: = 2m.
This defect can be corrected by using a concave lens of focal length
/=-jc = -2m
1 1
P = — - — Diaplre = -05D
/

oww
9.48. SIMPLE MICROSCOPE OR MAGNIFYING GLASS
A simple microscope is usedfor observing magnified images of tiny objects. It con.sist.s of a converging
lens of small focal length. A virtual, erect and magnified image of the object is formed at the least distance of
distinct vision from the eye held close to the lens. That is why the simple microscope is also called a magnifying

e
re
glass. ,

FFrllo
A magnifying glass, allows us to view small or

rF
distant objects because it produces a larger image on

ee
the retina of our eyes than would be possible otherwise.
Thus basically, an optical instrument magnifies the
ouru
sor rF
anguhir size of the object. Larger the angular size, larger
is the image on the retina and hence larger the object
appears to be.
I he course of ray.s through a simple microscope kffo
is shown in Fig. 9.69. ,FFarc two principal foci and C is
os
ook
the optical centre of the convex lens. An object AB is
Yo
Y

held between optical centre C and principal focus F of


Bo

the lens perpendicular to the principal axis. A virtual,


reeB

erect and magnified image A'B' is formed as traced in Fig. 9.69. The eye is held close to the lens, and CB
ouY

= d, least distance of distinct vision for the normal eye.


ur

Magnifying power or Angular Magnification of a simple microscope is defined as the ratio of the
ad
Yo

angle subtended at the eye by the final image to the angle subtend at the eye by the object, when
d

both the final image and the object are situated at the least distance of distinct vision from the eye.
nidn

Now, the image A'B' is already at the least distance of distinct vision (d) from the eye. Let
Re

ZA'CB' = (3. Imagine the object AB to be displaced to A^B' at distance d. Let ZA^CB' = a
F
Fi

P ...(90)
By definition. Magnifying power m = —
a

For small angles expressed in radians, tan 0 - 9 a = tan a and p « tan P


tanp
m = ...(91)
tana

AB A, S' _ aB
In A ABC, tan p = In AAjS'C, tana =
CB'
CB

AB CB' _ CB' -V V
or
V
...(92)
Putting in (91), we get m = m = —

CB AB ~ CB -u u u

where, CB' = -v, distance of image from the lens, CB = - u, distance of object from the lens
1 1
From lens formula. V u /
RAY OPTICS AND OPTICAL INSTRUMENTS 9/57

Multiply both sides by v


V V
1--
u /
V V
using (92), \~m = ~ or m = l
/ /
d '1
But v=~d. m= 1+ —
...(93)
/

This is the expression for magnifying power of a simple microscope or (magnifying glass).
Also, as/decreases, m increases, i.e., smaller the focal length of the lens, greater is its magnifying power.
A simple microscope or magnifying glass is used :

ww
(0 by watch makers and jewellers to see magnified view of tiny parts of watch and fine jewellery work.
(//) by students in Science laboratories for reading vernier scales, etc.
The image is very bright and sharp because object is held close to the lens.

FF loo
IMPORTANT NOTE

ree
1. When image formed by a simple microscope is at infinity, the object must be at focus, i.e., u =f In
that case, we can show that

reFe
d
m = —

ororF
f
rur
s ff
Nonnally, magnifying power of a simple microscope is < 10.
2. As explained already, a single lens suffers from the defects of spherical aberration and chromatic
k
aberration. Therefore, image formed by a single lens is often blurred and coloured. To get a sharp and
YYouo
okso

bright image from a simple microscope, we often use a suitable lens combination.
BBoo

Sample Problem Calculate the maximum magnifying power of a simple microscope consisting
r ee

of a convex lens of focal length 5 cm. Distance of distinct vision is 25 cm.


Sol. Here, /= 5 cm, m = 2 d=25 cm
ad
ouur
Yo

/
d ^ 25 ^
Maximum magnifying power, m = 1 + — 1+— =6
fj 5 j
d
idnY
Re

9.49. COMPOUND MICROSCOPE


FFin

A compound microscope is an optical instrument used for observing highly magnified images of tiny
objects.
Construction. A compound microscope consists of two converging lenses (or lens systems); an objective
lens O of very .small focal length and short aperture and an eye piece E of moderate focal length and large
aperture. The two lenses are held co-axially at the free ends of a tube, at a suitable fixed distance from
eachother. The distance of the objective lens from the object can be adjusted by rack and pinion
arrangement.

The course of rays through a compound microscope is shown in Fig. 9.70.


AB is a tiny object held perpendicular to the common principal axis, in front of the objective lens
beyond its principal focus b\y A real, inverted and enlarged image A'//' of this object is formed by the objective
lens. Now A'B' acts as an object for the eye lens, whose position is so adjusted that A'B' lies between optical
centre C2 of eye lens and its principal focus F^. A virtual and magnified image A" B" is formed by the eye
lens. This image is erect w.r.i. A'B' but inverted w.r.t. AB. The final image A" B" is seen by the eye held close
to eye lens. The adjustments are so made that A" B" is at the least distance of distinct vision (d) from the eye,
i.e., C-, B = d.
9/58 ‘Pn^ut€4p>'4, Fundamental Physics (XII)Ecaau

loow w
ee
Magnifying power or A.ngular Magnification of a compound microscope is defined as the ratio

Fr
of the angle subtended at the eye by the ifnal image to the angle subtended at the eye by the
r FF
object, when both the final image and the object are situated at the least distance of distinct
vision from the eye.

rer
In Fig. 9.70, Co B” = d. Imagine the object AB to be
fofr Fo
shifted to A j B" so tliat it is at a distance d from the
u
eye. If ZA" Cj B" = p and ZA, C2 B" = a, then by definition.
ks
YYouro
...(94)
s oo

Magnifying power. m = —
a

a «tan a and [3 « tan p


BBook

For small angles expressed in radian,


r ee

tanP
From (94),
m - ...(95)
tana
ouru
ad

Aj5 AB
ft
A" B
Yo

In A A" B" Co, tan P = In A A^B" C2,


tana = tf

C2 B" ' C2 B" qB


d
Re

C^B" a"B A"B A'B'


ft n ft
A"B
iYn

X —
Putting in (93), we get m -
ft
X
AB AB A'B' AB
C^B
FFind

m = X mo ...(96)

ft
A" B
where =
A'B
j, magnification produced by eye lens.

A' B'
and m
0
AB
, magnification produced by objective lens.
d >
Now, m
e
l + — (proved in case of simple microscope]
fe
where d is C2 B" = least distance of distinct vision, 4 is focal length of eye lens. And
C,B’
A'B' _ distance of image A' B' from C|
V
1 0
m
0 “
AB distance of object AB from C,B
1
-u
0
RAY OPTICS AND OPTICAL INSTRUMENTS 9/59

Pulling these values in (96), we get


V
0 d ^ V
0
m = 1+—
fe I a. I ...(97)
— U
0 0 fe

As the object AB lies veiy close to Fq, the focus of objective lens, therefore,
uq = C|5 « CjFq =/q = focal length of objective lens.
As A'B' is formed very close to eye lens whose focal length is also short, therefore,
V
0 = length of microscope tube.

Putting in (97), we get m =


...(98)
fe) l/o' fe

w
1 1 1
We know that
U
0
ll
0 /r0

Flo
V V
1- 0 _ ‘"0 V V
Multiplying both sides by Vq, we get 0

e
or
U
●^0

rree
0 u
0 /o

r FF
V f' j ^

Putting in (97), we get m= 1-


0
1+^
f,0 ...(99)
fe)
uurr
Magnifying Power of Compound Microscope when final image is at infinity
/ V

for
kss
V
_ "0
As is known, magnifying power of objective lens, m0
ooook

~
Yo
u
0
eB

When final image is at infinity, magnifying power of eye lens, =—


fe
rr

V d
Magnifying power of compound microscope is
ou
ad

m = m^.ymg —
fe u
YY

When the object is very close to the principal focus of the objective, and the image formed is very close
to the eye lens, then
nndd

m-,0 = /q and = length of microscope tube


Re

-L d
Fi

m = —X —

/.0 fe ...(100)

In this case, the microscope is said to be in normal adjustment.


Discussion. (/) As magnifying power (m) is negative, the image seen in a microscope is always inverted
w.r.t. object, i.e.. upside down and left turned irght.
(/7) As intermediate image is between the two lenses, a cross-wire (or a measuring scale) can be used.
{in) For large magnifying power,^ and^ both have to be small. Also,/g is taken to the smaller than/
so that field of view may be increased. ^
(/v) As aperture of both the lenses in a microscope is small, the defects of images particularly, spherical
aberration is minimised,
(v) In a compound microscope of good quality, objective lens and eye piece, both are formed of a
number of lenses. This is done to avoid defects of images formed by a single lens.
(vO Various other factors such as illumination of the object contribute to the quality and visibility of the
image.
9/60 'P'uluieefi- ^ Fundamental Physics (Xll)fcagUH
IMPORTANT NOTE —
Note that in a compound microscope, objective lens and eye lens are fixed distance apart. For focussing
an object, we adjust distance of objective lens from the object by moving rack and pinion arrangement.

Sample Problem If we need a magnification of 375 from a microscope of tube length 15 cm


and an objective of focal length 0-5 cm, what focal length of eye lens should be used ?
Sol. M = -375,L=15cm, /q = 0-5 cm,/^ = ? 25 cm

1+^
V
0
M =
-u
0 fe
As focal length of objective lens is small Uq-/q- Also, as focal length of eye lens is small, Vq~~ L

ww
-375 =
15 (,1 + —

25
M =
-/o fe
-0-5
fe

Flo
25 375 25
-1=11-5 = 2*2 cm
fe =

ee
30
fe 11-5

rere
r FF
9.50. ASTRONOMICAL TELESCOPE
An astronomical telescope is an optical instrument which is used for observing distinct images of
uurr
heavenly bodies like stars, planets etc. foor
A refracting type astronomical telescope consists of two lenses (or lens systems) : the objective lens,
ks s
which is of large focal length and large aperture and the eye lens, which has a small focal length and small
Yoo
aperture. The two lenses are mounted co-axially at the free ends of the two tubes. The distance between these
oook

lenses can be adjusted using a rack and pinion arrangement.


eBB

In normal adjustment of telescope, the final image is formed at infinity.


1 he course of rays in normal adjustment of telescope is shown in Fig. 9.71. A parallel beam of light
uurr

from an astronomical object (at infinity) is made to fall on the objective lens of the telescope. It forms a real,
ad

inverted and diminished image A'S' of the object. The eye piece is so adjusted that A'B' lies just at the focus
Yo

of the eye piece. Therefore, a final highly magnified image is formed at infinity. The final image is erect
w.r.t.,
dY

A'B' and is inverted w..rt. the object.


Re
innd
FFi
RAY OPTICS AND OPTICAL INSTRUMENTS 9/61

However, in astronomical telescope, final image being inverted w.r.t. the object does not matter, as the
astronomical objects are usually spherical.

Magnifying Power or Angular Magniifcation of an astronomical telescope in normal adjustment


is defined as the ratio of the angle subtended at the eye by the final image to the angle subtended

w
at the eye, by the object directly, when the final image and the object, both lie at infinitedistance
from the eye.

As the object lies at very huge distance, therefore, angle subtended by the object at C2 (where eye is

e
held) is almost the same as the angle subtended by the object at (because C] is close to C2). Let it be a, i.e.
/A' Cj B' - a. Rays coming from the final image at infinity make ZA'C^ 5' = P on the eye. Therefore, by

e
wr
definition.

oo
r
P
Magnifying power, m = — ...(101)

F
a

FFllu
As angles a and p are small, therefore, a = tan a and p « tan p.
tanp
From (101), m -
...(102)
tan a

rese
uro A'B' A'B'

Fkr
In A A'B'Cj, tan p = In AA'B'C,,

o
tan a =
C^B' CB'
1

foo
A'B' C^B' _C^B'
fr
/o
kso
m =
Put in (102), ...(103)
C^B' A'B' ~ C^B'
Y
or m =

-4
Y
B
Yo
where
C^B' =/o = focal length of objective lens, C2B' = -f = focal length of eye lens.
oo

Negative sign of m indicates that final image is inverted w.r.t. the object.
eBr

Thus, to increase magnifying power of an astronomical telescope in normal adjustment, focal length of
rue

objective lens should be large and focal length of eye lens should be small.
oud

Retain in Memory
no
ad

(/) In normal adjustment of telescope, distance between the objective lens and eye lens = (/q +fj).
Y

(ii) Angular magniifcation produced by the telescope = p/a. Clearly, visual angle p is much larger as
ndi

compared to a.
Re
F

Fig. 9.72 shows the course of rays in an astronomical telescope, when the final image is formed at
Fi

the least distance of distinct vision (d) from the eye.

Magnifying power or Angular Magnification of an astronomical telescope is defined as the


ratio of the angle subtended at the eye by the final image at the least distance of distinct vision
to the angle subtended at the eye by the object at infinity, when seen directly.

As the object is very far off and C], C2 are close by, angle subtended by the object on the eye at C2 is
almost the same as the angle subtended by it at Cj. Let it be a.
ZA'Cjfi' = a
Further, let ZA" B" = P,
where C2 B" = d

P
By definition. Magnifying power. m = — ...(104)
a
9/62 “Ptaticcfr Fundamental Physics (XII) kV*mil

ww
FF loo
ree
As angles a and p are small, therefore, P = tan P and a * tan a

rFee
tanP ...(105)
From (104), m =

oor rF
tana
rur
s ff
A'B' A'B'
In A A'B'C-,, tanp = In A A'B'C 1’ tan a =
C,B'
C^B'
k
1
YYoou
okos

A'B'
BBoo

Putting in (105), we get m =

C^B' A'B'
r ee

C,B'
1
ouur
ad

m —
...(106)
C^B' -u
Yo

where C\ B' =fQ = focal length of objective lens


Yd

C2B' = - Ug, distance of A'B', acting as the object for eye lens.
Re
idn

1 1 1
FFin

Now, for eye lens.


V u /
Taking Vg = -d,u~-Ug and /= +/g, we get
1 1 1

-d

J_-J_ 1--L fe
ue f
●' e d f
■' e
d

m = —^
f
1+^
fe
Putting in (106), we get
d
fe
RAY OPTICS AND OPTICAL INSTRUMENTS 9/63

Discussion :

(/) As magnifying power is negative, the final image in an astronomical telescope is inverted i.e. upside
down and left turned irght.
(//) As intemiediate image is between the two lenses, cross wire (or a measuring device) can be used.
(///) In normal setting of telescope, final image is at infinity. Magnifying power is minimum.
When final image is at least distance of distinct vision, magnifying power is maximum. Thus

(M.P.) .
/o f0 fe
' 'min. (M.P.) max. 1 +
fe fe d

oww
(iv) To have large magnifying power,^ must be as large as possible and/^ must be as small as possible,
(v) In a telescope, aperture of objective lens is made large to increase magnifying power and resolving
power of the telescope.
(vi) The largest lens objective in use has a diameter of 102 cm. This telescope is at the Yerkes Observatory

e
in Wisconsin (U.S.A.).

FFrlo
re
Retain in Memory

ree
F
1. In a refracting astronomical telescope, distance between objective lens and eye lens is adjusted

rF
by moving the rack and pinion arrangement. This focusses the object situated at infinity.
2. A telescope of good quality must have three characteristics:

fsoor
ouur
(/) large magnifying power,
skf
(//) large resolving power {i.e., ability to observe distinctly two closely spaced objects). For
details, refer to chapter 6(f).
ooko
Yo
(Hi) Large light gathering power (oc where D is diameter of objective lens).
Y
Bo

3. Two limitations of refracting type telescope are ;


reB

(0 Defects of spherical aberration and chromatic aberration in single lenses used as objective and
eye piece.
uur
oY

(ii) Objective lenses of very large aperture are difficult to manufacture and handle.
ad
dY

Sample Problem D An astronomical telescope of magnifying power 7 consists of two thin


lenses 40 cm apart, in normal adjustment. Calculate the focal lengths of the lenses.
innd
Re
Fi

Sol. Here, m =—=l or


F

f0 = 1fe
fe
In normal adjustment, distance between the lenses, /q +/^ = 40 cm
40
'7/g+/e = 40, 4= —= 5cra

/o = 74 = 7x5 = 35cm
Sample Problem In an astronomical telescope, focal length of eye piece is 5 cm and focal
length of objective is 75 cm. The final image is formed at the least distance of distinct vision {= 25 cm)
from the eye. What is the magnifying power of the telescope ?
Sol. Here, 4 = 5 cm,4 = 75 cm, m = ?, £/= 25 cm
4
From w = - 1 +i 75 r
= - 15 X 1-2 =- 18-0
f.e V
d 5 25
9/64 ‘P'tadecfi.'-i, Fundamental Physics fXlItragTWn
9.51. REFLECTING TYPE TELESCOPE (CASSEGRAINIAN TELESCOPE)
It is an improvement over the refracting type astronomical telescope described above. In such a telescope,
objective lens is replaced by a concave parabolic mirror of large aperture, which is free from chromatic and
spherical aberrations. The image formed is much brighter and the reflecting type telescope has much higher
resolving power compared to the refracting type telescope. Such a telescope is known as Cassegrainian
telescope, after its inventor.
A reflecting type telescope was designed initially by
Newton for observing distant stars. The model has been
modified from time to time. In Fig. 9.73, Cassegrainian
type telescope which was designed by Guillaume
Cassegrain is shown. C is a parabolic concave reflector of

ww
about 200 inch aperture with a narrow hole at the centre.
Parallel rays from a distant star entering the telescope in a
direction parallel to principal axis of the mirror tend to
collect at the focus of the mirror. But these reflected rays

Flo
encounter a secondary convex minor B before meeting at the focus. The convex mirror reflects them onto the

e
eye piece. The final image is seen through the eye piece. The hole in the centre of the objective gives an

eere
advantage that the astronomer looks through the telescope in the direction of the star.

FFr
Note that the final image as seen through the eye piece is inverted w.r.t. the object. But for astronomical

oorr
purposes, it does not matter, as the objects are usually spherical.
uur r
In normal ad justment, magnifying power of a reflecting type telescope is given by
s ff
fo _(R/2)
sk
YYoo
m = —
ooko

f
j g f
jg
eBB

where R is radius of curvature of concave reflector.

Fig. 9.74 shows a Newtonian reflecting type


uurr

telescope. The parallel beam of light coming from the


ad

distant star is reflected by large parabolic concave reflector


Yo

C on to a plane mirror M. This mirror is inclined at an


dY

angle of 45" to the axis of C. The plane mirror reflects the


Re

beam forming a real image / in front of an eye piece E.


innd

The eye piece acts as a magnifier and the final, virtual and
FFi

magnified image of the star is seen distinctly by the eye.


Advantages : Following are the advantages of a
reflecting type telescope, especially for high resolution
astronomy.
(i) There is no chromatic aberration as the objective is a mirror.
{ii) Spherical aberration is reduced using mirror objective in the form of a paraboloid.
{Hi) Image is brighter compared to that in a refracting type telescope,
(iv) Mirror requires grinding and polishing of only one side,
(v) High resolution is achieved by using a mirror of large aperture,
(v/) A mirror weighs much less than a lens of equivalent optical quality. Therefore, mechanical support
of mirror is much le.ss of a problem compared to the support required for lens. Further, mirror can be supported
over its entire back surface unlike the lens which is supported over the rim only.
RAY OPTICS AND OPTICAL INSTRUMENTS 9/65

0 A boy 1.5 m tall with his eye


I I
Example
TYPE I. REFLECTION OF LIGHT
level at 1.38 ni stands before a mirror fixed on a
Formulae used. wall. Indicate by means of a ray diagram how the
1. Number of images of an object held between two mirror should be positioned so that he can view
plane mirrors inclined at Z6 is himself fully. What should be the minimum length
of the mirror ? Does the answer depend on the
360° 360
n = -1, if is even integer eye level ?
9 0
Solution. In Fig. 9.76, eye is at 1-38 m from

ooww
360 360 the foot F. Rays from foot can enter eye after
n — . if is an odd integer
e 0 reflection at M-,, whose height from ground is
1.38
2. When a plane mirror is turned through an angle = 0.69 m .
2
0, the reflected ray (of the given incident ray) turns

e
through an angle 26 in the same direction.

re e
rFl
3. On reflection, there is no change in frequency

Fre
and wavelength of light.

rrF
Units used. 9 is in degrees.

sffoo
ouur
Example a Light incident normally on a

plane mirror attached to a galvanometer coil


oksk
retraces backwards as shown in Fig. 9.75. A
Yo
current in the coil produces a deflection of 3*5° in
oo
Y

the mirror. What is the displacement of the


BB

reflected spot of light on a screen placed 1-5 m


away ?
rre

Solution. In Fig. 9.75, when mirror is turned


ouu

from position M, to position A/, through Z0 = 3-5°,


Y
ad

the reflected ray turns through


Again, eye is at 1.5 - 1.38 = 0.12 m from head
dY

Z20 = 7“ = ZAOB
H. Rays from head can enter eye after reflection at
OA=d=\-5m
innd

Afj, whose height above the eye is = 0.06 m.


Re

Displacement, AB = x = l
Fi
F

FIGURE 9.75
Minimum length of the mirror = 0.69 + 0.06
= 0.75 m.
Ml M2 Minimum length of the mirror is thus always
X
<d! half of the length to be seen irrespective of eye level.
t
/ However, the position of the top and bottom
(
A 0 edges of the mirror would depend on the eye level.
d I

I
/
I TYPE II. REFLECTION
/
/
AT SPHERICAL MIRRORS

tan 20 = — Formulae used.


In AAOB
d
1. In a spherical mirror,/= R/2
x = i/tan20= 1-5 tan 7°= 1-5 x 0-1228
Both, / and R are positive for a convex mirror
- 0*18 metre. and both arc negative for a concave mirror.
9/66 “PtcuCeeft '<*. Fundamental Physics (XII) E*7STT1
V (15)
2. Mirror Formula = - + — = — = - Also, m = -

V u R f u -(5)
The formula is same for both, convex mirror and The image is virtual, erect and magnified
concave mirror, whether the image is real or three times.
virtual.
Example Suppose while sitting in a
3. Linear magnification for spherical mirrors parked car, you notice a jogger approaching
h _ ^ _f-y_ / towards you in the rear view mirror of /? = 2 m. If
m
u f f-u
the Jogger is running at a speed of 5 ms“^, how
fast is the image of the jogger moving, when the
Sign of m is determined from the sign of h2lh^ jogger is (a) 39 m {b) 29 m (c) 19 m and (d) 9 m
Apply the prescribed New Cartesian sign away ? (NCERT Solved Example)
conventions to all distances and heights. R 2

w
Units used./, R, u, v, all are in metre or cm. Solution. Here, /? = 2m, / = — = — = lm
2 2
/?2 is size of image and is size of object (both
1
in metre or cm), m has no units.

Flo
From
V u f
Example El An object is placed (i) 10 cm

e
rree
(h) 5 cm in front of a concave mirror of radius of 1^1 1^u-f
curvature 15 cm. Find the position, nature and V f u fu

r FF
magnification of the image in each case.
fu
uurr
(Raj. Board 2012) (NCERT Solved Example)
Solution. Here, R = -15 cm, for V =
«-/
(a) When jogger is 39 m away, m = - 39 m,
kss
R 15
cm.
fu K-39) 39
ooook

2
Yo
V = = — m

(/) H = -10 cm, V = 1 u-f -39-1 40


eB

r- 11 1 As the jogger moves at a constant speed of


5 m/s, after 1 second, the position of the image (v)
V u f for w = - 39 + 5 = - 34 m is
rr
ou
ad

111^ 1 _-4 + 3 -1 V =
fu l(-34) 34
m
15 10 ” 30 "30
YY

^ / li
u-f -34-1 35
u = - 30 cm. Difference in apparent position of jogger in 1
nndd
Re

.●. The image is at 30 cm from the mirror on the second

same side as object. Also,


39 34 _ 1365-1360 _ 5 _ 1
Fi

m
V (-30) ~ 40 35 ~ 1400 " 1400 "280
magnification, m = — = -3
li (-10) Average speed of jogger’s image
The image is real, inverted and magnified 1
m/s.
3 times.
280
(/■/) M = - 5 cm, V = 2 Similarly, for n = — 29 m, - 19 m and -9m
average speed of jogger’s image is
From
V u / 1 1
m/s; — m/s; — m/s respectively.
150 60
L-l-L-ll 1 _ -4 + 6 _ 2 The speed increases as thejogger approaches the
v~ f u~ 15 5 ~ 30 “^ ca.r This can be experienced by the person in the ca.r
V = 15 cm. Note. We can use eqn. (17) on page 9/11 to
.●. The image is at 15 cm behind the mirror. calculate speed of jogger’s image.
RAY OPTICS AND OPTICAL INSTRUMENTS 9/67

TYPE III. LAWS OF REFRACTION, 8 .


sin r' = — sin r = - X 0-5745 = 0-5107
LATERAL SHIFT, REFRACTION 9 9
THROUGH A COMPOUND PLATE, :,,-l
r - sm (0-5107) = 30-7“
REAL AND APPARENT DEPTHS

Example What is the apparent position


Formulae used.
of an object below a rectangular block of glass
velocityof light in vacuum _ c 6 cm thick, if a layer of water 4 cm thick is on the
1.
velocityof light in medium v top of the glass ? Given "p,g=3/2 and “p^=4/3.
wavelengthin vacuum{^) Solution. Here, real depth of glass, Xj = 6 cm,
real depth of water, X2 = 4 cm.
wavelength in medium(A.') a

Pg = 3/2, X = 4/3

oww
2. When light travels from medium a to medium b,
If V], ^2 ihe corresponding apparent depths.
sini
a
3
Then, _ a
l-i a sin r

yi
1
a
2 2

e
3. = - = -x6 = 4cm
V >1 = 3-, = 3

ree
rFl
a

Fre
4. Lateral shift =
tsin(i^-r^) fl - a
4

rr F
vv
COST. 3
1 >-2
a
5. 3 3
ouur
6. =
real depth (jc)
sfoo
Apparent position of the object
= — X 4 = 3 cm
4
okks
apparent depth (y) = (jj + )>2) = (4 + 3) cm = 7 cm
Yo
Rise in position of the object
ooo

1>
7. Di.splacement of image =x-y = x 1 — = (->^1 + -C2) - Oi -*■ >’2) = (6 + 4 ) - (4 + 3)
eBB

It
= 10-7 = 3 cm
Units used. Angles, i, r, ij, rj in degree; distances, t,
Example B A ray of light is incident at an
uurr

X, y in metre ; |i, has no units, velocities c, v in m/s.


angle of 45** on one face of a rectangular glass slab
ad

of thickness 10 cm and refractive index 1*5.


Example g A rectangular glass slab rests
YYo

Calculate the lateral shift produced.


at the bottom of a trough of water. A ray of light
Solution. Here, /j = 45°, r = 10 cm = 0-1 m
dd

incident on water surface at an angle of 50** passes


p = 1-5, lateral shift = ?
Re
iinn

through water into glass. What is angle of


refraction in glass ? Take \i for water 4/3 and sin I
1
F

for glass 3/2. As |^ = -


sm .r
1
Solution. Here, = 4/3 and = 3/2
a sini
1 sin 45° 0-707
H’
3/2 9 sin .r = 0-4713
I
1-5 1-5
a
4/3 "8
First refraction is in going from air to water
rj = sin-'(0-4713) = 28-14°
a
sin i 4 sin 50°
lateral shift —
fsin (h~r|)
COST,
sin r 3 sin r 1

3 . 0-1 sin (45°-28-14°)


sin r = — sm50° -X 0-766 = 0-5745
4 4 cos 28-14°
Second refraction is in going from water to glass
0-1 sin 16-86° 0-1x0-2900
w sin r _ 9 = 0 033 m
sin / 8 cos 28-14° 0-8818
9/68 ‘P>Kidcefr '4. Fundamental Physics (XII)
Exam
p'®B A transparent cube of side Example BD The critical angle of incidence
210 mm contains a small air bubble. Its apparent in a glass slab placed in air is 45°. What will be the
distance, when viewed from one face of the cube critical angle when the glass slab is immersed in
is 100 mm, and when viewed through opposite face water of refractive index 1’33 ?
is 40 mm. What is the actual distance of the bubble Solution. Here,
from the second face and what is the refractive
1 1
index of the material of the cube ? a

8
= V2 = 1414
Solution. Let the bubble be at a distance x from sin C sin 45°

real depth Refractive index of glass w.r.t. water


face I. As p = , therefore./oryhce/.
apparent depth a

w _ 1414
X 210-;c a
1-33
— md for face II, p = K

ww
100 40

FIGURE 9.77
When glass slab is immersed in water,
100mm 40mm 1 1-33
sin Cf — = 0-9432
p 1-414
u-

Flo
e
C' = sin'l (0-9432) = 70“ 36'

eree
¥ ^1* o *l2 S
Example m Determine the direction in
O
CQ

FFr
which a tish under water sees the setting sun.

oorr
uur r
Given, for water p = 1*33.
Solution. In Fig. 9.78, the setting sun is in the
sf
X
(210-.t)
direction of water surface. A ray of light from setting
sk
sun enters the eye of the fish when apparent position
Yoo
X 210-jc
x= 150 mm
oook

100 40
of the sun makes critical angle C with the vertical,
eBB

Actual distance of bubble from face II


sin C = — = — = 0-7518
= 210-A-= 210- 150 = 60 mm p 1-33
150 C = sin"' (0-7518) = 48-7°
uurr

X
11 = = 1-5
ad

100 100 FIGURE 9.78


Yo

Apparent

I I
pos. of sun
TYPE IV. TOTAL INTERNAL REFLECTION
dY

I
I
0 as
seen by
Re

Formulae used. 1. For TIR, light must travel from


innd

I the fish

denser to rarer medium. Angle of incidence (/) in


FFi

denser medium must be greater than critical angle


I
Setting
-:C:, Sun
(O for the pair of media in contact. I
-t-

1 Fish
2. p =
sin C
Hence angle between apparent position of the
Units used. Angles i and C are in degree, p has no sun and horizontal is 9 = 90° - 48-7° = 41-3°
units.
Example m The refractive index of water
is 4/3. Determine the angle of the cone within
Example E] Calculate the speed of light i.. in
which the entire outside view will be confined for
a medium whose critical angle is 30°. (CBSE 2010) a fish under water. (CBSE 2003)
Solution. Here, u = ?, C = 30° Solution. As is clear from Fig. 9.79, the fish
1
p = -
c
can see the entire outside view within cone angle
sin C
0 = 2 C, where sin C = — ^
V
= 0-75
V = c sin C = 3 X 10^ X sin 30° = 1*5 x 10® m/s 4 4/3

I
RAY OPTICS AND OPTICAL INSTRUMENTS 9/69

Example m Calculate the critical angle


for total internal reflection of light travelling from
(i) water into air (ii) glass into water. Given,
a
- 1.33 and = 1.5
Solution. (/) When light travels from water to
a
1
air. K = sin C

1 1
Casin'! (0-75) =48-7° sin C = = 0.7518
a
1.33
e = 2C = 2x48-7 = 97-4“ w

C = sin-’ (0-7518) = 48" 44'


Example m A point source of light S is {ii) When light travels from glass into water

loow w
placed at the bottom of a vessel containing a liquid a
1 1
of refractive index 5/3. A person is viewing the iv
, i.e.
ft

source from above the surface. There is an opaque sin C'


a
It sin C'
disc of radius 1 cm floating on the surface. The 1.5 1 1.33
centre of disc lies vertically above the source O. or or sin C' = = 0-8866

ee
1.33 sin C' 1.5
The liquid from the vessel is gradually drained

Fr
out through a tap. What is the maximum height r FF C = sin-^ (0-8866) = 62" IT
of the liquid for which the source cannot be seen Example [Q One facc of a prism of
at all. (IIT 2(H)4)

rer
refractive index 1*5 and angle 75" is covered with
Solution. In Fig. 9.80, let OS = h be the max. fofr Fo 3-^2
u
height of liquid above the source for which the source a liquid of refractive index . What should be
ks
cannot be seen at all. the angle of incidence of light on the clear face of
YYouro
ZOSA = C prism for which light is just totally reflected at
s oo

the liquid covered face ? (MP hoard 2011)


Any other ray from S will be totally internally
BBook

reflected because in that case, angle of incidence will Solution. For total reflection at the liquid
r ee

be greater them the critical angle. covered face, ij = C, Fig. 9.81.


ouru
ad
Yo
d
Re
iYn
FFind

OA r 1
In AOSA, sin C =
When y=}h.
sin C = —

1 r~
or

r^
3-s^/4 _ 2->/2 _ 1 = sin 45"
3/2 " 4 "V2
r- + = p? r^ /. C = /2 = 45°,
/i2 = ,2 (^2_ ij Now, + 'i2 = A = 75"
h = ri = 75" - = 75° - 45" = 30"
sini 1
= 1*33 cm As |i = -
3 sm .r
1
9/70 T^uuice^ Fundamental Physics (XII)
3 3
-sin 30^*=-
sm ij = p. sin '1 = 2 4 u V R
= sin-' (3/4) = 48-6
1 1-5 _ 1-5-1 _ 1
-12 V ~ 3 ~6

w
TYPEV. REFRACTION
3 1 1 1
AT SPHERICAL SURFACES
2v 6 12 12
Formulae used. 1. When refraction occurs from n = 18 cm
rarer to denser medium,

e
As V is positive, the image is real.
Example m An empty spherical flask of

e
wr
—u V R
diameter 15 cm is placed in water of p. = 4/3. A

lloo
r
2. When refraction occurs from denser to rarer parallel beam of light strikes the flask. Where does
medium, we interchange P] and P2 it get focussed, when observed from within the

F
u
flask ?
P2 ^ P, P1-P2

r FF
—u V R Solution. Here, R = PC = + 15/2 cm

where symbols have their usual meaning. Values of P2 = 4/3, Pi= 1

rsee
u, V, R are to be used with new cartesian sign For a parallel beam of light, m = «>, Fig. 9.83
uo
conventions.

Fr
k
FIGURE 9.83

o
3. Power of refracting surface p =

foo
R

fr Pl/?
kso
4. First principal focal length, f\ ~~
Y
B
Y
Yo
[i^R
oo

Second principal focal length, /2 ~


1^2
eBr
e

Units used, u, v, R, are in metre ; Pj, P2 have no


uru

units. Power P in dioptre


od

The beam diverges on refraction and appears


Yo

Example [0 A glass dumbbell of length


ad

to come from I.
n

30 cm and refractive index 1*5 has ends of radius PI = v = 2


of curvature 3 cm. A point object is situated at a As refraction occurs from denser to rarer
ndi

distance of 12 cm from one end of dumbbell. Find medium, therefore.


Re
F

the position of the image formed due to refraction


at one end only. P2 ^ Pi P1-P2
Fi

R
Solution. Here, / = 30 cm, p^ = 1-5, P] = 1,
u V

R = 3 cm, M = - 12 cm. 4 1 1-4/3 _ 2


Image formed is at /, as shown in Fig. 9.82, 3X00 V ” 15/2 ~ 45
PI = l 45
V = - = - 22*5 cm
FIGURE 9.82 2
30 cm
Example [0 What curvature must be
given to the bounding surface of p = 1*5 for virtual
image of an object in the medium of p = 1 at 10
cm to be formed at a distance of 40 cm. Calculate
power of the refracting surface and also two
principal focal lengths of the surface.
12 cm V
Solution. /? = ?, P2 = 1-5, Pi = 1
As refraction occurs from air to glass. u - 10 cm, u = - 40 cm
RAY OPTICS AND OPTICAL INSTRUMENTS 9/71

As object is placed in rarer medium,


^2 I ^1 - li, -^2
u V R
u V R
1-5 1 -1-5
1 1-5 _ 1-5 -1 1 15 V -15 30
-10 -40 ~ R ~ 2R
i-_L _L-_1
J_ _ J_ _ J_ _ _5_ _ J_ v~30 io"30
2R~\0 80 ” 80 ~ 16
30
16 v = — = 7-5 cm
R = = 8 cm 4
2
Distance of image I from the centre of sphere
As R is positive, the refracting surface is convex. a =CP2 + P2l=l5+ 7-5 = 22*5 cm

ww
Power of surface.
1-5-1 TYPE VI, LENS MAKER S FORMULA
P = = 6 25 D

FF loo
R 8/100
Formulae used. 1. Lens Maker’s formula is
Note: For power in dioptre, R must be in metre.

ree
1 1 1
1x8
- = (p-I) —- 1 where
Again, /[ =- 1-5-1
= - 16 cm. / R
I - /

reFe
\h_ ref. index of lens material
1^2^ 1-5x8

oor rF
\^ =
and /2 -
rur
= 24 cm ref.index of surrounding medium
1-5-1
s ff
/= focal length of lens in the surrounding medium
Example EE A sunshine recorder globe of
New Cartesian Sign Conventions are to be used for
30 cm diameter is made of glass of p. = 1*5. A ray
k
all variables.
YYoou
okso

enters the globe parallel to its axis. Find the


position from the centre of the sphere, where the Units used, u, v, R, Ri, R2,f, h\,h2 are in metre; m
BBoo

ray crosses the axis. and p have no units.


r ee

Solution. For first refraction at surface AP 1


H = oo, U = P|/j =: ? Example^ A biconvex lens has focal
ouur
ad

2
/? = + 15 cm, po - 1-5, p, = 1 length — times the radius of curvature of either
Yo

Pi ^ P. surface. Calculate refractive index of material of


ii V R the lens.
d

(CBSE 2010)
idnY
Re

1-5 1-5-1 _ 1 u = 45 cm.


Solution. Here, R^-R.Rt =
«./=3
FFin

oc V 15

The image would have been formed at /, if glass p =?


medium were continuous.
1
FIGURE 9.84
- = (P-1) j i_
/ R.1 R^- /

1 1
c
)
I1 2R {R^ Rj
y 3
... p-l = -
4
= 0-75, p = 0-75+ 1 =1-75
For second refraction at surface BPj
/, acts as virtual object. Example m A diverging lens of refractive
li = P2I1 = P]I] - P^P2 = 45 - 30 = 15 cm index 1*5 and focal length 15 cm in air has same
R = - \5 cm, V = P2I- ? radii of curvature for both sides. If it Is immersed
9/72 7^‘ieuUe^ 4 Fundamental Physics (XII) kV«mii
in a liquid of refractive index 1*7, calculate focal Solution. If / is focal length of the lens of
length of the lens in liquid. |i = 1-2, then from
Solution. |^= I-5,/=- 15 cm.
1 1 1
= (1-2-1)
1 1_
Let /?[=-/?, 7^2 = + /?
f R
1 R, R^ R^
“ y

- = (^l-l) _1 1_
1
1 I 1
/
R
I R, (0-2)/
1
'_l_i _1_

w
= (1-5-1) Let /' be focal length of this lens, when
-15 R R R
immersed in a liquid of refractive index = 1 -3.
/? = 15 cm
1 r 1 n
Let/' be focal length of lens in liquid -t^-I
R
^2

roow
r

e
^ / /\ >
1 8 1

re
-1 using (0,
Ri1 R^“ /
! 1-2 1 1
—-1 X

reF
\ /
1-5 1

uFFll
/' 1-3 (0-2)/ 13x0-2/

e
1-7
/ \
-15 15
r = -2-6/
0-2 2 0-4 The lens will behave as concave lens and its

sFr
1-7 15 1-7x15 focal length will be 2*6 times the initial focal

foro
length.
uor
r=
1-7x15
0-4
= 63*75 cm
fk Example Calculate the radius of
okso
The diverging lens will behave as converging curvature of an equiconcave lens of refractive
index 1*5 when it is kept in a medium of refractive
Y
lens in the liquid.
Yo
oo
BB

Example m A magician during a show index 1*4, to have a power of - 5 D. (CBSE2019)

makes a glass lens p = 1*5 disappear in a trough Solution. Here, R^ = l R^= - R, R2- + R.
of liquid. What is the refractive index of the Pi =1-5
rYree

liquid ? Is the liquid water ?


ouu

14,P = -5D
(NCERT Solved Example)
ad
Ydo

/
^ /
Solution. From lens maker’s formula.
1
p = — -1 j 1_'
/ ^^1 / V R,I R'y-
^=(P-I)
1
4-- ^
/
nidn

f r: 1-4
J j_ -0-1 (-2) _ 0-2
Re

1
-5= 1
1-5 -R~ R 1-5 R ~ I-5R
F

^2
Fi

where P =
0-2 2 200 -8
R = m = - cm = cm

1 1-5 (-5) 75 75 3
If = — = 0 or/= 00

/
TYPE VII. LENS FORMULA, LINEAR
The lens in the liquid will act like a plane
MAGNIFICATION AND POWER OF LENS
sheet of glass, when refractive index of the lens and
the surrounding medium is the same. Therefore,
Formulae used
refractive index of surrounding medium,
P2=p, = 1*5 1 _l_\_
1. Lens formula/equation : —
This liquid medium is not water because f V u
refractive index of water = 1 -33.
2. Linear magnification :
Example m Explain what happens when V f f~v
a convex lens of refractive index 1*2 is immersed m = — - -
u f+u f
in a liquid of refractive index 1*3.
RAY OPTICS AND OPTICAL INSTRUMENTS 9/73

1 100 4. Magnification of combination of lenses


3. Power of lens, P =
/ (metre) / (cm) m = /Mj X m2 X m3 X ...

Now cartesian sign conventions are used for all Units used. /],/2, F and d are in metre ; P is in
variables. diopter ; mj, m2, m3,... m have no units.
Units used. u,v, R, /?,, ^2 metre
or cm ; m and |x have no units. P is in dioptre.
Example Use the following ray dia
gram, Fig. 9.86 to calculate focal length of lens L2.
Example ^ jjie graph in Fig. 9.85 shows
the variation of image distance (u) with object
distance (u) in case of a lens. Find focal length of
the lens. What is the nature of the lens, if image

ww
formed is real ? (CBSE 2004)

Flo
e
ree
Solution. For lens L^,f= 15 cm,

Fr
rF
M = “ 40 cm,
uurr
v = l

1 _ 1
s
v~ f
for
1 _ 1
h~15 40 ~ 120’
5
V = 24 cm
kks
Solution. As the image formed is real, the lens
Yo
oooo

must be convex. The image would have been at /',


From the graph, where Ljf - 24 cm.
eB

when I M I = 20 cm, I u I = 20 cm /' serves as object for L-,


M = - 20 cm, u = + 20 cm H = + (24 - 14) = 10 cm, u = 30 cm,/' = ?
ur

i-l_l ^ ’ * I 1 1 I
ad

20^20 10
YYo

f V u /= -15 cm
r V 14 30 10 15

/= 10 cm
Example A concave lens is placed in
dd
Re

contact with a convex lens of focal length 25 cm.


in

TYPE VIII. COMBINATION OF LENSES


The combination produces a real image at a
F

distance of 80 cm, when an object is at a distance


Formulae used. 1. Focal length F of a combination
of 40 cm. What is the focal length of concave lens ?
of two lenses of focal lengths/] and/2 separated by (I’h. B.mrd .;( I .

a distance d is given by
1
Solution. Here,/] = 25 cm,/2 = ?
1 d
For the combination of focal length F,
^ /1/2 u = - 40 cm, V = + S0 cm.

If the lenses are in contact, d = 0 :


j__ I I As
1 1 I

F V u

2. For combination of lenses in contact, power, J__J 1 _ 1+2 _ 3


F = F, + F2 = algebraic sum of powers of individual F~80 -40 " 80 "80
lenses.
1 1 1
3. For combination of two lenses separated by a As —I — —
distance d, power P = P^ + P2 - d x P1P2 /, fi F
9/74 ‘Pnadc^ '4. Fundamental Physics (XII) lviiii

1 1 1 3 1 _ 15-16 1 1-5
1 ^ J_
-1 X —=
/, “ F /, “ 80 25 “ 400 /. »v
1-33 10 1-33 10

I 1
●33x10
h 400 ● /2 = - 400 cm /
j w = = 78-23 cm
0-17

w
Example (f) If/ = + O’S m, what is the Example ^ A real image of an object is
formed at a distance of 20 cm from a lens. On
power of the lens ? (ii) The radii of curvature of
the faces of a double convex lens are 9 cm and putting another lens in contact with it, the image
is shifted 10 cm towards the combination.
15 cm. Its focal length is 12 cm. What is the

e
refractive index of glass ? (Hi) A convex lens has Determine the power of the second lens.
20 cm focal length in air. What is the focal length Solution. Here, as image formed by the lens is

re
o
rw
in water ? (Refractive index of air-water = 1-33, real, the lens must be convex, u = 20 cm. If/j is focal
refractive index of air-glass = 1-5). length of this lens, them from

F
(NCERT Solved Example) 1 1 I

ullo
FF
V u
.. . .
Solution. (/) P_ = —
1 _J_ = +2D

ee
/ ~ 0-5 1 1 I

srr
(//) Here, = 9 cm, /?2 = - 15 cm, ...(/)

oF
20 u
/i

k
/= + 12 cm, p = ?
For the combination (of focal length/),
From lens maker’s formula.
fofr
uor
u = (20 - 10) cm = 10 cm
1
- = (^i-i) —-
I o 1 1 J___l_ J_
Y
kos
/ R
1 10 u
Yo
rBB

~ /
oo

I 1 1 1
eY

1 8 + —
^ = (M-l) - + — =(|i-l)X using (0, 7^ 20
^ 9 15 J ^ 45 u u
12
re

J 1__ J_
u

45 _ 15
^1-1 = 10 20 ■'/2
or

12x8 ~ 32
d
ou
o
ad

or
/2 = 20 cm
nY

, 15 47 , _
100
^ 32 32 P = = 5 dioptre
20
nid
Re

(Hi) Here,/, = + 20 cm,/^ = ?,


Example Rli] Find the position of the
F
Fi

)!„,= 1-33, 1-5 image formed by the lens combination given in


Fig. 9.87. (CBSE 2019)
1
As _1 1_ (NCERT Solved Example)
fa a
y \
FIGURE 9.87
\ /
1 1-5 1 1
f = -H0cm -10cm +30cm
1
20
A U
1 R
1

1 1 1 0

R
I h 10 V ^
K- 30cm -►M- >«— 10cm—H
5cm

Again as L- h -1 J !_
Solution. For 1st lens,
/,w R, Ry
«l =-30 cm. /| =+ 10 cm, Uj = ?
RAY OPTICS AND OPTICAL INSTRUMENTS 9/75

1 1 I Example ED Three lenses Lp L2, each


As
of focal length 30 cm are placed co>axially as
V,
I
u
I /.
shown in Fig. 9.88.
1 1 I An object is held at 60 cm from the optic
V -30 10 centre of lens Lj. The final real image is formed at
the focus of Ly Calculate separation between
1 1 1 (i) Lj and L2 («) Z-2 and L3 ? (CBsr:

w
V
1
10 30 15’ FIGURE 9.88

v^ ~ 15 cm. Li >-2 L3
Image formed by first lens serves as object for

roow
second lens at a distance = (15 - 5) = 10 cm to the

e
+
right of second lens, and the object is virtual. -X-

re
0 I
Therefore, for second lens, Ci C2 C3

Uj - 10 cm, V2 = ?/2 = - 10 cm.

reF
60 cm

uFFll
>l ►I
30 cm

e
1 1 1
As
^2 ^2 «2 Solution. For lens Lj ; « = - 60 cm,

sFr
/= + 30 cm.

foro
_1__ J 1_ IfUj is distance of image/j from Cj, then from
uor
10 " ^2 ” fk
1 I 1
okso
1
V
I
u
A
= 0. U2 =
Y
Yo
^2
oo

1 1 1 J 1_-J_
BB

The virtual image is formed at infinity to the V


1
u 30 60 "60
right of 2nd lens. This acts as an object for 3rd lens.
rYree

Uj = 60 cm = Cj/j
Therefore, for 3rd lens, = ■», U3 = ?
ouu

As final image I is at focus of L3, therefore, rays


/3 = + 30 cm.
falling on L3 must be parallel to commen principal
ad
Ydo

J 1__ J_ axis. For this to happen, /j must be at the focus of


As Hence distance between is
«3 «3~/3
nidn

~ Cj C2 — Cj /j + ■/ ] C2
Re

I 1 I - 60 cm + 30 cm = 90 cm
F
Fi

V
3
00 30 and distance between L2 and L3 can have any
reasonable value, as rays between L2 and L3 are
or
U3 = 30 cm, parallel rays.
The final image is formed at 30 cm to the The course of rays is shown in Fig. 9.89.
right of third lens.
9/76 “Px^xdee^ 4 Fundamental Physics (XII)
distance of 30 cm from the lens. For a point object
TYPE IX. COMBINATION
at a distance of 25 cm from the lens, the final
OF A LENS AND A MIRROR
image due to this combination coincides with the
object itself. What is the focal length of convex
Formulae used. 1. Lens formula to locate image lens ? (NCERT Solved Example)
formed by refraction through the lens.
Solution. Fig. 9.91 shows the given set up.
1 1 1
i.e.,
f V u

2. Mirror formula to locate the image formed by

oww
reflection at the mirror,

i.e.. 1-i + i
f V u
For this, initial image formed by the lens will act as

e
real/virtual object for the mirror. Appropriate sign

re
conventions are used for all: u, v and/. Here, R = PC = 20 cm.

FFrllo
3. For determination of sizes, use usual formulae
CjF = 30 cm, OCi = 25 cm

rF
for linear magnification of lenses and mirrors.
As the final image due to the combination

ee
Units used, u, v,f, all in metres.
coincides with the object itself, the rays reflected from
ouru convex mirror must be retracing their path. This can

sor rF
Example A convex lens is placed in
happen only when rays refracted from convex lens
contact with a plane mirror. An axial point object tend to meet at centre of curvature of convex mirror.
at a distance of 20 cm from this combination, has
its Image coinciding with itself. What is the focal kffo
Therefore, for convex lens,
V = CjC = C^P + PC = 30+ 20 = 50 cm,
length of the convex lens ? (CBSE 2014)
os
when u = OCj = - 25 cm.
ook
Yo
(NCERT Solved Example)
Y

Solution. Fig. 9.90 shows a convex lens placed 1 1 1 3+6 9


Bo

From
in contact with a plane mirror. An axial point object M 50 "^ 25 150 150
reeB

/ ^
i.s placed at (9 at a distance OC = 20 cm. f= 150/9 = 16-67 cm
ouY

As the image / of the object coincides with O,


Example 0] A convex lens of focal length
ur

the rays refracted first from the lens and then reflected
ad

20 cm is placed co-axially with a convex mirror of


Yo

by the plane mirror must be retracing their path. This


radius of curvature 20 cm. The two are kept
d

would happen when rays refracted by the convex lens


fall normally on the mirror, i.e., the refracted rays form 15 cm apart from eachother. A point object is
nidn

a beam parallel to principal axis of the lens. Hence placed 60 cm in front of the convex lens. Find the
Re

position of the image formed by the combination.


the object O must be at the focus of the convex lens.
F

(NCERT Solved Example)


Fi

/= CO = 20 cm

Solution. The set up is shown in Fig. 9.92.


For the convex lens, f =20 cm ;
u - OC = - 60 cm

Example ^ A convcx lens and a convex 1 1 1


As
mirror of radius of curvature 20 cm are placed V u /
co-axially with the convex mirror placed at a
RAY OPTICS AND OPTICAL INSTRUMENTS 9/77

1 _3-l_ 2 Exsmple jj An object is placed 15 cm in


V
/ u 20 60 60 60 front of a convex lens of focal length 10 cm. Find
60 the nature and position of image formed. Where
V = — = 30 cm should a concave mirror of radius of curvature
2
20 cm be placed so that the final image is formed
If image by lens alone is formed at then
on the position of the object itself ? (CBSE 2015)
C/j = 30 cm.
Solution. Here, » = - 15 cm,/= 10 cm, u = ?
For the convex mirror, /j acts as a virtual object. 1 1 1
Therefore, « = P/j = C/j - CP = 30 - 15 = 15 cm, From
/? = 20 cm, t) = ? V u f
From
1-1 + 1
1 + 1-1 1 = 1-1 = 1__1 = 1_ V f u 10 15 "30

w
V u R V R u 20 15 30
V 30
u = 30 cm u = 30 cm, w = — = -2
-15

Flo
Final image by the combination is formed at u

/. where PI = 30 cm Image is real, inverted and magnified formed at

reeee
a distance of 30 cm from lens.
Example ^ A convex lens of focal length
20 cm and a convex mirror of focal length 10 Now, the final image formed by concave mirror

Fr
cm

are placed co-axially 50 cm apart from each other. will be at the position of the object itself only if image
An incident beam parallel to its principal axis is formed by the lens lies at the centre of curvature of

incident on the convex lens. Locate the position of


the mirror.
for
ur
final image formed due to the combination. .●. Distance of mirror from the lens
= 30 + P = (30 + 20) cm = 50 cm
kkss
(CBSE 2014) (NCERT Solved Example)
Yo

I I
oo

TYPE X. REFRACTION THROUGH A PRISM


eB

Formulae used. 1. In refraction through a prism


(0 A + 8 = i| + i2 {ii) r, + r2 = A
r

{Hi) when 8 = 5„,, /, = ^ and Tj = t2


ou
ad

A +8
YY

m
I - and r = —
2 2
ndd
Re

Solution. The set up is shown in Fig. 9.93. When sin (A + 8,„)/2


2. I^ =
sin A/2
Fi

the beam incident on convex lens is parallel to


principal axis of the lens, on refraction, it collects at 3. Mean deviation, 8 = (p - 1) A, when A is small
focus of the lens forming the image 7j, where 4. When A is not small, 8 = /] +I2-A
Cl I = F = 20 cm. /| acLs as a real object for concave

mirror, who.se real image is formed at I. We have to Units used. i|, T|, T2, 8, 6/n, A are in degree ; p
has no units.
find PI =v = 2 For the concave mirror,
H = P/, = CP - C/i = 50 - 20 = (-) 30 cm,
/=- 10 cm. Example EB A ray of light passing
From through an equilateral triangular glass prism from
1
air undergoes minimum deviation when angle of
i l = l-i = J_ ±-__L
V u
f'v f u -lo'^30 " 15 incidence is -th
— of the angle of prism. Calculate
f = - 15 cm speed of light in prism. (CBSE 2008)
Negative sign indicates that final image at I ISi
real. Solution. Here, A = 60°, i = -A=-x60°=45°
4 4
9/78 “pnaxCee^ Fundamental Physics (XII) Kisioa
In the position of minimum deviation,
r = - = 30°,
2

ii = -
sin i
sin 45° _ I/V2 = V2
sin r sin 30° ~ 1/2
c
As ^1 = -,
V

c 3x10^
v = — = 2-12 X 10^ m/s
|i V2

ww
Example!^ A rayincident on face AB
of a prism ABC, as shown in Fig. 9.94, emerges
from the face AC such that AQ = AR. Draw the

Floo
ZLMN' = C, critical angle
ray diagram showing the passage of the ray
through the prism. If the angle of prism is 60° and

ree
As sin C = — = "p C = 45°
refractive index of the material of the prism is -^3 ; V2

rFee
determine the values of angle of incidence and ZAML = 90° - 45° = 45°
angle of deviation. (CBSE 2015) ZALM = 180° - 60° - 45° = 75°

F
oor r
rur
r = 90°-75° = 15°
s ff
sin I
From ^ = -:
sm r
osk
YYoou

= p sin r= -J2 sin 15°


oook

sm i

= 1414x0-2588 = 0-3659
eBB

i = sin-‘ (0-3659) = 21*46°


Example El An equilateral glass prism has
uur r

Solution. As the refracted ray passes such that


a refractive index 1*6 in air. Calculate the angle of
ad

AQ = AR, it must go parallel to the base of the prism


Yo

minimum deviation of the prism when kept in a


suffering minimum deviation.
60 medium of refractive index 4 -^/2/5 ● (CBSE 2019)
dY

r = A/2 = — = 30°
Re

2 4-J2
idn

Solution. Here, A = 60°, ^2 “


FFin

sinz sm i 5 ’
1A = -
sin r sin 30°
From prism formula.
sin i = p sin 30° sin (A + 6^)/2
M =
i = 60° sin A/2

As / -f e = A -f 5 m
^^2 sin A/2
2 / = 60° + 6,„, 6,„ = 2 / - 60° = 120° - 60° sm = p sin A/2 =
2 ^^1
5„, = 60°
1 1
Example ^ In the above example, find -

sin 60°/2=4
the angle of incidence at face AB, so that emergent 4V2
ray grazes along the face AC'. A + 8m
Solution. In Fig. 9.95, refracted ray LM suffers 2
= 45°
total internal reflection on face AC and grazes along
the face AC. 5,„ = 2x45-A = 90°-60° = 30°
RAY OPTICS AND OPTICAL INSTRUMENTS 9/79

TYPE XI. DISPERSION THROUGH A PRISM Example M A crown glass prism of


refracting angle 6® is to be achromatised for red
Formulae used and blue light using a flint glass prism. Find the
1. Angular dispersion = 8^, - 5^ = A, angle of Hint glass prism and also , the mean
when angle of prism A is small. deviation from the following data :
5„+5.. For crown glass : p^ = 1-520, = 1-531 ;
2. Mean deviation, 8 =
2 For flint glass : p/ = 1-662, = 1-684
(HP Board 2012)
3. Mean refractive index, p = Solution. Here, A = 6°, = 1 -520, =1-531
2
A' = ?. p/= 1-662, p,;= 1-684
4. Dispersive power, (co) = ~~ For achromatism, necessary condition is

w
8 p-1
(Pft-p,)A-f (p',^-p',.)A' = 0
5. For deviation without dispersion, condition is ; (1-531 - 1-520) 6°-^(l-684- 1-662)A' = 0
co8 -f to'8' = 0

Flo
0-011x6
Net deviation = 8 + 8' = (p - 1) A -i- (p' - 1) A' A' = - = -3
0-022

ee
6. For dispersion without deviation, condition is :
8-f8' = 0or (p- 1)A -t-(p'- 1)A' = 0 Negative sign indicates that the two prisms mu.st

Fr
Net dispersion = (p^ _ pp ^ a' be Joined in opposition.
In both cases, prisms must be joined in opposition. 1-520 + 1-531

for
ur
Now p- = 1-5255
Units used. 8,8^, 8^ A are in degree : p, p,^, p r
00 2 2
have no units.
1-684 + 1-662
ks
and = 1-673
Example 01 A combination of two prisms,
Yo
2 9
oo

one of flint and other of crown glass produces Net deviation = ( p - 1) A + (p' - 1) A'
eB

dispersion without deviation. The angle of flint = (1-5255 - 1)6“ + (1-673 - 1) (- 3“) = 1-134'
glass prism is 15“. Calculate the angle of crown Example
glass prism and angular separation of red and A crown glass prism of
ur

refracting angle 6“ to be achromatised for red and


violet rays on emergence from the spectroscope,
ad

(p for crown glass = 1-52, p for flint glass = 1-65, blue light using a flint glass prism. Find the angle
Yo

of the second prism and mean deviation from the


CO for crown glass = 0-02, co for flint glass = 0-03). following data.
Solution. Here, A = 15“, A' = ?, co = 0-03
For crown glass ; p^ = 1-520, p^ = 1-531
nd
Re

co' = 0-02. p= 1-65, p' = 1-52


For flint glass ; p^ = 1-662, p^ = 1-684.
Fi

For no deviation, 8 + 8' = 0


(HP Board 2012)
(p- I)A + (p'-l)A' = 0
Solution. For achromatic combination,
(1-65 - 1)15“+ (1-52-1) A'= 0.

A' =
-0-65x15 (P/,-p,)A =-(p;^-^p'Ja'
= -18-75'
0-52
A' = - (p^-p^)A (1-531-1-520)6
Negative sign indicates that two prisms must (nl-p') (1-684-1-662)
be joined in opposition.
0-011x6
Net angular separation = -3°
0-022
= (p,,-p,)A + (p/-p/)A' Mean deviation = (p - 1) A + (p' - 1) A'
= CO (p - i) A + co' (p'- 1) A' 1-53J +1-520 1-662 +1-684
= A
= 0-03(1-65- 1) 15“ + 0-02(l-52- 1)(- 18-75“) 2
-1 X
2
-I

= 0-2925 -0-195
= 6°(l-5255- l)-3° (1-673- l)x(-3)
= 0-0975"
= 3-153 -2-019 = 1-134“
9/80 “Pn^idteft Fundamental Physics (XII)>:giai]

Example Q7J A person wears eye glasses


I TYPE XII. THE DEFECTS OF VISION
I with a power of - 5*5 D for distance viewing. His
doctor prescribes a correction of + 1*5 D for his
Formulae used.
near vision. What is the focal length of his distance
1. Myopia or Short sightedness viewing part of the lens and also for near vision
For observing distant objects, a concave lens of focal section of the lens ?
length/= - A- is to be used, where x is distance of Solution, (fl) For distance viewing,
far point of defective eye. P, =-5-5D
2. Hypermetropia or Long sightedness
100 100
For observing near objects, a convex lens of focal = -18*73 cm
-5-5
1 1 1
length/is to be used, where — =
/ (/?) As power of near vision part is measured

w
u

relative to the main part of lens of power - 5-5 D,


Here, u = d = least distance of distinct vision of
therefore,
normal eye ; v = - x = distance of near point of
^5-5 + P2=1-5

Flo
defective eye.
3. Power of lens. P2= 1-5+ 5-5 = 7-0 D

e
reee
100 1 100 100
P (dioptre) = f.= = 14*3 cm

FFr
/(cm) f (metre) 70
Units used. All distances, u, u,/are in metre, P is
in dioptre. 03 What focal length should the
for
ur
Example

reading spectacles have for a person whose near


ES A person wears glasses of point is 50 cm ? (NCERT Solved Example)
kss
Example

power - 2*5 D. Is the person short sighted or long Solution. The book to be read is at w = - 25 cm
Yo
and the image is needed at f = - 50 cm. Therefore,
oo

sighted ? What is the far point of the person


without glasses ? focal length (f) of the required lens is given by
eB

Solution. Here, P = - 2-5 D


As P is negative, the person is short sighted
1 1 1^1 ^1,1
/ V u ”-50 25 50
ur

100 _ lOQ = 50 cm (convex lens)


ad

=-40cm
YYo

P ”-2-5
Example A short sighted person can
An object at infinity from the corrective lens see objects most distinctly at a distance of 16 cm.
must produce the virtual image at the far point, i.e.,
d

If he wears spectacles at a distance of 1 cm from


Re
in

at 40 cm from the eye. the eye, what focal length should he have so as to
enable him to see distinctly at a distance of 26 cm ?
F

+ —
From Solution. As the person wears spectacles at a
V / U -40 oo

distance of I cm. from the eyes.


t; = - 40 cm
II =-(26- 1) cm = - 25 cm.
and =- (16 - 1) cm = - 15 cm ;/= ?
Example [0 The far point of a myopic
person is 150 cm in front of the eye. Calculate the 1-i ^
As
focal length and power of a lens required to enable f^v
him to see distant objects clearly. (CBSE 2004)
Solution. Here, x = 150 cm,/= ?, P = ? 1 1 _-5 + 3^ 2
To sec distant objects clearly, 7” 15 25 ” 75 75
f=-x = - 150 cm = - 1-5 m 75
/ = cm = -37-5 cm.
2
100 100
P = = - 0*67 D A concave lens of focal length 37*5 cm. is
/ “-150 to be used.
RAY OPTICS AND OPTICAL INSTRUMENTS 9/81

TYPE XIII. SIMPLE MICROSCOPE 1 I 1


From
AND COMPOUND MICROSCOPE V
0
u
0 f,0
Formulae used. 1. Magnifying power of a simple 1 1 1 -7x1-25
or u
d -6u 1-25
0
u 6
microscope, m = 1H— 0 ‘-0

= - 1-46 cm
2. Magnifying power of a compound microscope. Object must be held 1*46 cm in front of
V
0 L objective lens.
ni = X m =
0
^ '“o' 4 f
n fe Exsmple 91 A compound microscope with
an objective of 1-0 cm focal length and an eye piece

ww
For a normal eye, d=25 cm.
of 2-0 cm focal length has a tube length of 20 cm.
Units used. All distances Uy, Vq, d,fQ,f^, L are in Calculate the magnifying power of microscope if
metre, m has no units.
final image is formed at the near point of eye.

Flo
(CBSE 2004)
Example £^1] A child has near point at

e
Solution. Here,/o = 1-0 cm,/^ = 2-0 cm.

eree
10 cm. What is the maximum angular magnifi L = 20 cm, ^/ = 25 cm
cation the child can have with a convex lens of
When final image is formed at the near point of

FFr
focal length 10 cm ? the eye (at least distance d of distinct vision), then
uurr
Solution. For maximum angular magnification,

orr
L
v = -d~- 10cm
m — —
sfo 1.^ 20 r,1-f—

25 =270
/,n 4 1-0 2
From L-l=l l-i_I 1 1

V u f' II V f 10 10 5
kks
Yoo
Example A compound microscope uses
oooo

« = - 5 cm
an objective lens of focal length 4 cm and eye lens
Maximum angular magnification, of focal length 10 cm. An object is placed at 6 cm
eBB

from the objective lens. Calculate magnifying


= :^ = 2
V
m = —
u -5 power of compound microscope if final image is
urr

formed at the near point. Also, calculate length of


the tube of compound microscope. [CBSE 2006 (C)]
ad

Example The focal lengths of objective


YYo

and eye piece of a microscope are 1-25 cm and Solution. Here,/o = 4 cm,/^ = 10 cm.
5 cm respectively. Find the position of the object Uq = - 6 cm, v^ = d = - 25 cm
dd

relative to the objective in order to obtain an 1 1 1


Re
inn

From
angular magnification of 30 in normal adjustment. V
0
u
0 /o
F

(CBSE 2012)
Solution. Here,/Q = 1-25 cm,/^ = 5 cm, «y = L=±+J_ = l_l = —,
^ Un = 12 cm
m = 30 V
0 ●4 u
0
4 6 12 ^
In normal adjustment, magnification produced
by the eye piece, m —
V
0
1-f—
d 12 ,1 +—
25^ =7
6 10
U
4
m = ^ = ^=5 0

' 4 5 1
From
m Ve uV f
■' e
As m = wq X /«y = m 5
1 J J_-_i L-zZ
e

As real image is formed by objective lens.


V
U
e 4 ”-25 10 “50
0
therefore, ,m 0 “ = -6,Uy = -6no 50
u
0
u
e
= -7-14cm
1
9/82 ^ Fundamental Physics (XII)

Length of compound microscope, L 25 6L


30 = 1 +—
L = U{, + 1 = 12 + 7-14 = 19-14 cm 1-25 5 1-25

Example^ A man with normal near ^ 30x1-25 - 6-25 cm


point (25 cm) reads a book with small print using 6

a magnifying glass : a thin convex lens of focal Example A compound microscope has
length 5 cm. an objective of focal length 1 cm and an eye piece
(a) What are the closest and the farthest of focal length 2-5 cm. An object has to be placed
distances at which he can read the book when at a distance of 1-2 cm away from the objective
viewing through the magnifying glass ? for normal adjustment. Find the angular
(b) What is the maximum and the minimum magnification and length of the microscope tube.
angular magnifications (magnifying powers)

ww
iCBSE 2010)
possible using the above simple microscope ? Solution. Here, /g = I cm,/^ = 2-5 cm.
Solution, (fl) w = ? u = - 25 cm,/= 5 cm. u
Q = - 1-2 cm., w = ? L = ?
1 1 I 1 1

Flo
As 1 1 1
/ V / As

e
V u u
V
0
II
0 /o

ree
1 _1 1 _-l-5^-6 I _ 1

Fr
u -25 5" 25 25

rF
V /<0 u I 1-2 1-2
0 0
-25
uurr
u = = - 4-2 cm Un0 = 1-2/0-2 = 6 cm.

the book.
6
This is the closest distance at which he can read
As
s for m -
\uJ
V
0 d
1 +—
4
kks
0
For the farthest distance, // = ?,
Yo
oooo

6 f ,1 25
v' = «»./= 5 cm m = — + = 55
1-2 2-5
eB

As
1-1_1 ^ ^ u' = - 5 cm
/ 00 5 L-VQ+fg-6+ 2-5 = 8-5 cm
This is the farthest distance. Example The total magnification
ur

(h) Maximum angular magnification produced by a compound microscope is 20. The


ad
YYo

d 25 magnification produced by the eye piece is 5. The


= 6
u 25/6
microscope is focussed on a certain object. The
distance between the objective and eye piece is
dd

Minimum angular magnification


Re

observed to be 14 cm. If least distance of distinct


in

““ ““ O
vision is 20 cm, calculate the focal length of
F

u 5 objective and eye piece. iCBSE 2014)


Solution. From
Example ^ You are given two converging
m = X ,

20
lenses of focal lengths 1-25 cm and 5 cm to design m
m
— =4
0
a compound microscope. If it is desired to have a m
e
5
magnification of 30, find out separation between
^ = 4,
V
_ 0
the objective and eye piece. (CBSE 2015) Now, m
0
u
0 /,0
Solution. Here.^o = 1‘25 cm,/^ = 5 cm,
d = 15 cm,L=?,M = 30 . _ L 14 = 3.5 cm

When final image is formed at the near point of
■^0 " 4 4
eye, the magnifying power of compound microscope Also, m = 1 + —= 5
is given by
e
fe
L d \
M =— 1+ — d ^ d 20 = 5 cm

/,0 fe fe
1 = 4, 4 = 4= 4
RAY OPTICS AND OPTICAL INSTRUMENTS 9/83

Example An angular magnificati on 100


a = tana = - — radian ...(0
(magnifying power) of 30 X is desired for a 3x10^ 30
compound microscope using an objective of focal
If h is the height of image of tower, then angle
length 1‘25 cm and eye piece of focal length 5 cm. subtended by the image must also be a.
How will you set up the compound microscope ? h h
(CBSE Sample Paper 2019-20) a=itana = —
.(«)
/r0 150
Solution. Here, M = 30, /q = 1*25 cm,
1
/^ = 5 cm. From (/) and {ii), — —, h = 5 cm.
150 30

From M=— \+ — Magnification produced by eye piece


fe

ww
0

m
e

30 =
L
1+ —
25 L
x6
fe 5

1-25 5 1-25
Height of final image, h' = h x = 5x6

Flo
= 30 cm

e
1-25x30
L = = 6*25 cm

ree
Example The diameter of the moon is
6
3‘5 X 10^ km and its distance from the earth is
i"

Fr
Hence, the objective and eye piece should be 3-8 X 10® km. It is viewed by a telescope which

rF
set at a distance of 6-25 cm from eachother. consists of two lenses of focal lengths 4 m and
uurr
I TYPE XIV. TELESCOPE
I final image.
s for
10 cm. Find the angle subtended at the eye by the

Solution. Here, / = 3-5 x 10^ km = 3-5 x 10^ m.


kks
Formulae used. 1. Astronomical Telescope /-=3-8x 10® km = 3-8 x 10*^
Yo
m
oooo

{a) in normal adjustment, m =


0
/q = 4 m = 400 cm,/^ = ! 0 cm, p = ?
-fe
eB

/o 400
{b) when final image is at the least distance of m = — = = 40
fe 10
-fo fe
ur

distinct vision, m = 1 + ^
P
ad

fe d ) Also, m = —
YYo

a
(c) Distance between the two lenses in normal
adjustment = length of telescope tube, x =/q +/^ 3-5x10^
dd

2. Reflecting type telescope p = ma = 40x - = 40x rad


Re

r 3-8x10^
in

f0 R!2
m = —
180°
F

fe fe = 36-84x10-2 X = 2M°
where is focal length of objective mirror. K

Units used. Alt distances, u, are in metres,


m has no units.
Example A telescope has an objective
of focal length 50 cm and eye piece of focal length
5cm. The least distance of distinct vision is 25 cm.
Example^ A Small telescope has an The telescope is focussed for distinct vision on a
objective lens of focal length 150 cm and an eye scale 200 cm away from the objective. Calculate
piece of focal length 5 cm. If his telescope is used (i) the separation between objective and eye piece
to view a 100 m high tower 3 km away, find the (ii) the magnification produced.
height of the final image when it is formed 25 cm
away from the eye piece. (CBSE 2012)
Solution. Here,/o = 50 cni,/^ = 5 cm,
d = 25 cm, Uq = - 200 cm, m = l L = 7
Solution. Here, /q = 150 cm,/^ = 5 cm
1 1 1
Angle subtended by 100 m tall tower at 3 km As
away is V
0
u
0 fo
9/84 “Pnade^ Fundamental Physics (XII)

1 1 1
1
J 1 _4-l _ 3 From
V
0 f0 u
0
50 200 ~ 200 ~ 200 V
e
u
e 4
200
cm
1 1 1 I L=A
V
5 -25 25
0 3 4 '’e
Linear magnification produced by objective -25
lens, u = -4-17 cm
e
6
V
0
200/3 I
Distance between the two lenses,
'”o= —
u 200 3
0
L =/„ + I H, 1 = 75 + 4-17 = 79-17 cm
Now, v^ = d = -25 cm

I I

w
TYPE XV. TYPICAL EXAMPLES
J 1__ J_
From
"s “e 4

Flo
Example An object is placed at a
-I 1
I 1 5+1 distance of 36 cm from a convex mirror. A plane

e
5 25 mirror is placed inbetween so that the two virtual
fe

rree
images so formed coincide. If the plane mirror is

r FF
25 at a distance of 24 cm from the object, find the
u cm
e
6 radius of curvature of convex mirror.
uurr
Linear magnification produced by eye piece
V -25
for
Solution. The course of rays for image formation
is shown in Fig. 9.96. Clearly, image / formed by plane
kss
m
e _
= 6 mirror is such that MI = OM = 24 cm.
e
-25/6
ooook

u
Yo
e

Net magnification produced.


eB

1
m = m^xm --x6=-2
0 e
3
urr

Distance between objective and eye piece


ad
Yo

200 ^ 25 _ 4(K) + 25 ^ 425


dY

L = I Uq 1 +1 u J = -^ 6 6 6
Re
innd

= 70-8 cm

Example ^ In an astronomical telescope,


Fi

As convex mirror forms the image at / itself,


focal length of objective lens is 75 cm and that of
therefore, v = PI = MI - MP
eye piece is 5 cm. Calculate the magnifying power
and the distance between the two lenses, when final = MI-(OP-OM)
image of distant object is seen at a distance of 25 = 24-12= 12 cm
cm from the eye. u = OP = ~ 36 cm

Solution. Here,/o - 75 cm,/^ - 5 cm, m = ?,


L = ? = 25 cm 1 1 1 j 1 3-12 1
/ V u 12 36 ” 36 “ 36 ~ 18
m = -
fo 1+A /= 18 cm ; /? = 2/= 2 x 18 = 36 cm.
f Exsmpie The sun (diameter d)

75 7
subtends an angle 0 radian at the pole of a concave
5
1 + A'1=-15x-
25 5
= -18 mirror of focal length /. What is the diameter of
the image of the sun formed by the mirror ?
RAY OPTICS AND OPTICAL INSTRUMENTS 9/85

Solution. For the sun, u = o»,v = l diameter of (0 Formation of image by concave mirror
image,d = A'B' = ?,Fig. 9.97.
1 J__J_ 1 1 -1
V
1 /, “i -20 -25 100

=- 100 cm

As VI is negative, image of object AB is real


and invert^, formed in front of concave mirror at
A'B', where = 100 cm

oww
(//) Formation of image by convex mirror M2
The real image A'B' acts as virtual object for
convex mirror.

u2 = P2B' = PiB'-P^ P2 = (100-50)


From

e
= 50 cm
V u /

re
The final image is A" B"; where P2 B" = V2

FFrllo
- + ---
.-. v=f J__J 1_ 1 1 7

rF
V 00
f

ee
15 50 150
i.e., image is formed at the focus Fof concave ^2 -^2 ^2
ouru
mirror. If A' B' is the size/diameter of the image of

sor rF
150
the sun, then from V = 21*43 cm
2- 7
0 = A'B' ^ d Example^ The power of a thin convex
d=ef
PF f kffo
lens of glass is 5 dioptre. When it is immersed in a
os
liquid of refractive index p, it behaves like a
ook
Yo
Y
Example A concave mirror of focal divergent lens of focal length 1 m. Calculate p of
liquid, if p of glass = 3/2.
Bo

length 20 cm and a convex mirror of focal length


reeB

15 cm are placed 50 cm apart, such that the two Solution. Here, Power of glass lens in air P = 5
dioptre
ouY

mirrors face eachother. An object is placed exactly


focal length of lens in air,
ur

midway between them. Find the nature and


ad

position of image formed by reflection, frrst at


Yo

100 100
= 20 cm
concave mirror and then at convex mirror. fa p
d

Focal length of lens in liquid,


nidn

Soluion. Here,/j = - 20 cm,/2 = 15 cm


Re

50
/; = -1 m = - 100 cm
M,1 = cm = -25 cm
F

P/ = ?,p^ = 3/2.
Fi

2
From lens maker’s formula.
/ \
1 1 1
^-1
4 l,M-„ «2j
-
20
= (- -1 ^
/?2^ 2 /?J R2
or J-_—= — .(0
^10
1 g
-1
1 1 ^
Again,
A / \
R
1

P
9/86 ‘Pn^^de^ ^ Fundamental Physics (XII) kV»lWli
1 3 \ 1 Solution.
1 X—
using (/), -100
2[L, 10 ’ Pf-xf _ Q-82^-Q-128^
/i = 4D, 4 X 0-82
= 0-2 m ,
which gives = 5/3

Example ^ One face of a glass cube of P] - 4 _ 0-9^ - 0-3^ = 0-2 ni


fl =
side 0’06 m is silvered. An object is placed at a
distance of 0*07 m from the face opposite to the
4d/- 4x0-9

silvered face. Looking from the object side, the Average focal length.
image of the object appears to be O'll m behind
/ = —
/, +/2 _ 0-2+ 0-2 = 0-2 m
the silvered face. Calculate the refractive index of ^ 2 2
material of glass. Now, /?j = 0-2 m, /?2 = - 0-2 m, [I = ?

ww
Solution. In Fig. 9.99, AB is silvered face of
the section of the cube shown. When seen from the
1 ( 1 1 )
From - = (^l-l) —-
side of O, let AB appear at A'B'. / R
1 R,
- /

Flo
FIGURE 9.99 1 ( 1 1 )
= 01-1)

e
0^ 0-2 1,0-2 0-2 0-1

ree
1 1

Fr
0.07m
= 0 = -+’ =1-5

rF
? D C
uurr
for A beam of light of wavelength
Exam
X

400 nm is incident normally on a right angled


s
A' B'
prism as shown in Fig. 9.100.
kks
Yo
oooo

A B
eB

0.11m
ur
ad
YYo

Let PA'=x
Also PA - 0-06 m.

As image in plane mirror A'B" is as far behind


dd
Re

the mirror as the object is in front, therefore


in

007+x = 0-11, x = 0-11-0-07 = 004m. It is observed that light just grazes along the
F

surface AC after falling on it. If refractive index p,


real depth DA DA 0-06
p = = 1-5 of the material of prism varies with wavelength X
apparent depth DA' x 0-04

Example^ Using the following data, as 11 = = 1-2+-^


calculate refractive index of the material of convex
Calculate the value of b and p of prism
lens. Radii of arvature of both the surfaces are
material for X = 500 nm. Given 0 = sin“^ (0*625).
0*2 m. Data recorded in the displacement/shift (CBSE Sample Paper 2014)
method to determine focal length of convex lens Solution. As the ray goes grazingly along AC,
is shown here. (Karnataka Board 2012) 0 must be the critical angle.
S.No. D X 1 1
From p - = 1-6
sin C sin 0 0-625
1. 0*82 m 0*128 m

2. 0*9 m 0*3 m As 11 = = 1-2+-!;

k «
RAY OPTICS AND OPTICAL INSTRUMENTS 9/87

1-6 = 1-2 + —^
(400)2
::Exam pile A ray of light is incident at
an angle of 60° on one face of a 30° prism. The
= 04 X (400)2 = 64000 nm2 emergent ray from the prism makes an angle of
For X = 500 nm, the refractive index of the 30° with the incident ray. Show that the emergent
material of the prism is ray is normal to the surface from which it emerges.
Calculate the refractive index of the material of
^ = 1-2 + 4?l2 = 1-2 +
64000
= 1.2 + 0-256 the prism.
(500)2
Solution. Refer to Fig. 9.102.
\i = 1-456
An equilateral glass prism
(p = 1-6) is immersed in water (p = 1*33). Calculate
the angle of deviation product for a ray of light

ww
incident at 40° on one face of the prism.
Solution. For an equilateral glass prism,
A = 60°, =1.6 and "p^ = 1.33

Flo
s

e
«p 8 16

eree
_
-p 8 = 1-203
«p w 1-33

FFr
oorr
uur r
sf
Here, i>60°,A = 30°,8 = 30°, *2=?
sk
Yoo
ooko

As /| + /2 = A + 6
60°+ 1*2 = 30°+ 30° = 60°
eBB

1*2 = 0
Hence emergent ray is normal to the surface,
uurr

For refraction on face AB, Fig. 9.101. When *2 = 0, T2 = 0


ad
Yo

i*i=40°, r, = ? As A = rj + r2
sin i1 30 = ri1 + 0 or r,1 = 30°
dY

**'p
^8
= 1-203
Re

smr.
sin I
1 _ sin 60°
1
innd

Now P = — =s
sin 30° 2x1/2
sin^ sin 40° 0-6428 sin .r
FFi

1
= 0-5343
^ 1-203 1-203 1-203

ri=32°18' , Example CT In a Spectrometer experi


ment, the angle Of minimum deviation was found
As ri + r2 = A, r2 = A - ri = 60°- 32° 18'
to be 48-6°. What is the percentage accuracy in
= 27° 42'
the measurement of refractive index of the prism ?
u; Sini2 Given least count of spectrometer = 0-2° and angle
Again ‘^p. = O Cl
= \r -
of prism = 60°
sm T2
sin 1*2 = 1-203 sin 27° 42' = 1-203 x 0-4648 Solution. Here, A = 60°, 6^ = 48-6°,
= 0-5592 sin(A + 6^^)/2 = 1-6242
P =
1*2 = 34° sin A/2

Angle of deviation, As least count of spectrometer=0-2°, Therefore,


5 = 1*1 + 1*2 - A = 40° + 34° - 60° = 14° error in measurement of angle A = ± 0-2°
9/88 "Pfuidecfi. ^ Fundamental Physics (XII) tvjni

Taking only + sign A + 6',„ = 70^ 8'


A' = (60 + 0-2) = 60-2"; 6',„ = 70“8'-A = 70“ 8'-60“
8 mr = 48-6 + 0-2 = 48-8" = 10" 8'

sin(A^+S,/)/2
Exam pie m Determine the value of the
= 1-6233
sin A' 12 angle of incidence for a ray of light travelling from

d\x=\ -6242 1-6233 =0-0009 a medium of refractive index p.j = V2 into the
% age accuracy medium of refractive index II2 ~ ^
grazes along the surface of separation.
d\i 00009
xl00 = xlOO [CBSE (F) 2017, CBSE 2017 (C)]
It 1-6242

Solution. Here, |ij = ^ ; }i2 = 1

w
= 00554%
If i is the angle of incidence of ray KO, it would
Try yourself, taking negative sign of the error
just graze along OA, as shown in Fig. 9.103, when it
in measuring A and 5m'

Flo
suffers total internal reflection at O. This would
Example A prism is made of glass of happen when Z/ = C. the critical angle for the pair of

reeee
unknown refractive index. A parallel beam of light media in contact.

is incident on a face of the prism. By rotating the

FFr
sin C =
prism, the minimum angle of deviation is measured
to be 40". What is the refractive index of the P/P2 !^| V2
for
ur
prism ? If the prism is placed in water (p. = 1*33), C = 45"
predict the new angle of minimum deviation of the
kkss
parallel beam. The refracting angle of prism is 60".
Yo
Solution. Here, 8,„ = 40", p = ?, A = 60"
oo

sin(A + 5m )/2
eB

As ^=
sin A/2

sin(60"+ 40")/2 sin50° _ 0-7660


r
ou

P =
ad

sin 60°/2 sin 30" ” 0-5000


YY

.-. p = 1-532
When prism is immersed in water, we have to
ndd
Re

take refractive index of glass w.r.t. water :


Fi

Exsmple m A telescope has an objective


a

w
1-532
= 1-149
(I
P 1-33 of focal length 30 cm and an eye piece of focal
length 3-0 cm. It is focussed on a scale distant
If is angle of minimum deviation in 2-0 m. For seeing with relaxed eye, calculate the
water, then
separation between the objective and eye piece.
»r sin(A+y^„)/2 Solution. Here,7g = 30 cm./^ = - 3-0 cm.
sin A / 2 7g = - 2-0 m = - 200 cm
sin (A + 6',„)/2 = "’p^ x sin A/2 1
— +
1

= M49xsin30" =
1-149
= 0-5745
V
0 /o u
0

2
J 1 _ 17
A+ 8' m 30 200 ~ 600
= sin (0-5745) = 35" 4'
2
RAY OPTICS AND OPTICAL INSTRUMENTS 9/89

1 1 I
600 As
V = 35-3cm
0 “
17 V
0
u
0 fo
For seeing the scale with relaxed eye, final 1 1 I
J 1 _ 6-1
image should be formed at inllniiy. This would happen ^~l50”l50

w
V
0 fo u
0
when image formed by the objective lens lies at the
150
focus of eye piece. = 30 cm
Distance between the objective and eye piece “0=-?

e
= ^0+fe 1 1 1
Again, as

e
= 35-3+ 3-0
Ve u„e /.

wr
e

lloo
= 38‘3 cm

r
1_ _ J 1 1 1
Example^ The focal lengths of the . 7

F
u -25 2-5

u
e

objective and eye piece of an astronomical


_ -1-10 _-11

r FF
telescope are 25 cm and 2*5 cm respectively. The
telescope is focussed on an object 1*5 m from ~~25 ~'25

rsee
objective, the final image being formed 25 cm from -25
eye of the observer. Calculate the length of the
uo = -2-27cm

Fr
u

k
e
II

o
telescope.
Solution. Here./Q = 25 cm./^ = 2-5 cm. Length of telescope, L = I ijq I + I I
Wq = - 1-5 m = - 150 cm.,
foo
fr
= 30 + 2-27
kso
Y
t; e = - 25 cm. L = ? = 32-27 cm
B
Y
Yo
oo
eBr
e
uru

1. Reflection of light is the phenomenon of change in the path of light without any change in medium.
od

A spherical mirror is a part of a hollow sphere whose one side is reflecting and other side is opaque. Two
Yo
ad

types of spherical mirrors are : concave mirror and convex mirror. The centre of curvature (O of spherical
n

mirror is the centre of the sphere of which the mirror forms a part. Principalfocus (F) of a spherical mirror
is a point on the principal axis of the mirror at which rays incident on the mirror in a direction parallel to the
ndi

principal axis actually meet or appear to diverge after reflection from the mirror.
Re
F

2. While dealing with reflection at spherical mirrors, we use the following New Cartesian Sign Conventions.
Fi

(0 All distances are measured from pole of spherical mirror.


(ii) The distances measured in the direction of incidence of light are taken as positive and vice-versa.
(Hi) The heights measured upwards and perpendicular to the principal axis of the mirror are taken as positive
and vice-versa.

3. In case of both the spherical mirrors, convex and concave,/ = RH. Also, the mirror formula for both the

mirrors is 1 \__ 2

/ u V R

where u is distance of the object and v is the distance of the image from the pole of the mirror.
Linear magnification in case of a spherical mirror is defined as the ratio of size of image (/i^) to the size of
the object (/?]). In a convex mirror, linear magnification is positive, because image is always virtual
+ /L, + U
m —
—u
9/90 ‘P'utdet^ 4^ Fundamental Physics (XII) PTsrwn
In a concave mirror, magnification is positive when image formed is virtual and magnification is negative,
when image formed is real. In general, we write

m = —
/12 V

h u
1

The sign of m is determined from the signs of /ij and /i^.


Other formulae for magnificationare

m =
/-v_ /
/ f-u
4. Spherical mirrors have several applications. A convex mirror is used as reflector in street lamps. It is also

ww
used as a driver’s mirror. A concave mirror is used as a reflector in search light, telescopes, solar cookers,
ophthalmoscope. They can also be used as trick mirrors.
5. Refraction of light is the phenomenon of change in the path of light, when it goes from one medium to

FF loo
another.

In going from a rarer to a denser medium, a ray of light bends towards normal and in going from a denser to

ree
a rarer medium, a ray of light bends away from normal. This is because light travels slower in a denser
medium than in a rarer medium.

rFee
On account of refraction of light, a tank of water appears to be shallow i.e. less deep than what it actually is.

F
oor r
rur
real depth
It is found that P- =
s ff
apparent depth

If i is angle of incidence, r is angle of refraction and |i is refractive index of denser medium w.r.t.
k
rarer
YYoou
medium, then according to Snell’s law.
ookos
BBo

Sint
li = - (as light goes from rarer to denser medium)
Sin r
re

sin r
ouur
ad

and ^ = - (when light goes from denser to rarer medium)


Yo

sin 1

Further, refractive index of medium a w.r.t. medium b is represented by


dY
Re

1
idn

Also. v.=
FFin

and 'P-b ^ Vc =
6. Total Internal Reflection is a phenomenon of reflection of light into denser medium from the boundary of
denser medium with rarer medium. Two essential conditions for the phenomenon of total internal reflection

are ; (/) Light should travel from a denser to a rarer medium, (i7) Angle of incidence in denser medium
should be greater than the critical angle for the pair of media in contact.
The critical angle for a pair of media in contact is defined as the angle of incidence in the denser medium
corresponding to which angle of refraction in the rarer medium is 90“. It is represented by C. If |X is refractive

index of denser medium w.r.t, rarer medium, then [i =


sin C

Obviously, C would depend on colour of light.


Some of the important applications of total internal reflection are brilliance of diamond, totally reflecting
glass prisms, optical fibres, mirage (false appearance of water in deserts in hot summer season) etc.
RAY OPTICS AND OPTICAL INSTRUMENTS 9/91

7. A surface which forms a part of a sphere of transparent refracting material is called a spherical refracting
surface. It may be convex or concave. In dealing with refraction at such surfaces, we use the same new
cartesian sign conventions as in the case of spherical mirrors.
The formula governing refraction at a spherical surface when itght travels from a rarer to a denser medium

IS
U V R

where u and v are distances of object and image respectively from the pole of the spherical surface and R is
radius of curvature of the surface.

When itght travels from a denser to a rarer medium, we have to interchange pj and ^ in the above
formula. The relation becomes

ww
-h.+}!i = hl}h
u V R

Flo
or

ee
V u R

eer
FFr
V u R

oorr
uur r
8. A lens is bound usually by two spherical surfaces. Therefore, a ray of light suffers two refractions on
s ff
passing through the lens. The lens maker’s formula for both, convex and concave lenses is
sk
YYoo
1 M M
- = (u-l) —- —
ooko

/ ^
eBB

where /?j and /?2 are radii of curvature of the two surfaces of the lens and p is refractive index of material
of lens w.r.t. me^um in which lens is placed.
uurr

1 1 1
ad

The relation governing u, v and / in case of both the lenses is — = —. This is called lens formula or
Yo

/ V u
lens equation.
dY

Remember that sign of m is the determined from the signs of and /12.
Re

9. Linearmagnification(m) producedby a lens is the ratio of size of image (/t2) to the size of the object (/i])
innd

For a concave lens, m is always positive.


FFi

-
-V V
m =
-u u

For a convex lens, m is positive (when image is virtual) and m is negative (when image is real).
Remember that sign of m is determined from the signs of and h2-
10. Power of a lens is definedas the ability of the lens to converge or diverge a beam of light falling on the
lens. Power of lens is given by reciprocal of focal length of the lens, i.e., P = — When /= 1 m, P = 1
/
dioptre
For a converging lens or convex lens, P is + and for a diverging lens or concave lens, P is negative.
9/92 ‘P'uuCee^ 4- Fundamental Physics (XII) I^AIWII
II, When two lenses of focal lengths/j,y2 and linear magnifications and m-, are placed in contact with each
other, then for the combination,focal length F, power P and magnificationm are given by

1~J_ _L
fi
P = P, + P^ (sum has to be taken with proper sign), and
wi = mj X m2
12, In passing through a prism, a ray of light suffers two refractions. The net deviation (5) suffered by a ray in
passing through a prism of small angle/i is 8 = (p - 1) A. When A is large, 5 = (i| + 12-^)-
The deviation through the prism is minimum (8,„). when ij = I2 and r, = r2
A + 5..
m
From /] + i2 = A + S , i + / = A + 8

ww
or i =
m
2

From r, + r2 = A, r + r = A, r =A/2

Flo
sin/ sin(A + 8jj|)/2
If p is refractive index of material of prism, then from Snell’s law M=-
sin A/2

ee
sin r

rere
This formula is called prism formula.

rFF
13. Dispersion of light is the phenomenon of splitting of white light into its constituent colours on passing
through a prism. The band of seven colours so obtained is called visible spectrum. The cause of dispersion
uurr
prism. As foor
is : Different colours having different wavelengths deviate through different angles on passing through the
; therefore, Py > p^ From 8 = (p - 1) A ; 8y > 8^. That is why violet colour is at the lower
end of the spectrum.
ks s
Yoo
14. Angular dispersion produced by a prism = 8y - 8^ = (Py - p^) A, whereas
oook

angular dispersion
eBB

Dispersive power of prism. (0 =


mean deviation 8 (P-I)A p-1
uurr

5.+5.
Deviation (8) for yellow colour is mean of 8y & 8,„ i.e. Mean deviation, 8 =
ad

2
Yo
dY

Similarly, mean refractive index of material of prism for yellow colour is p =


Re

2
innd

15. The blue colour of sky is due to scattering of sun light from molecules of earth’s atmosphere.
FFi

As X/, < and intensity of scattered light varies inversely as fourth power of wavelength, therefore, maximum
scattering is of blue colour.
At the time of sun irse and sun set, light from sun has to pass maximum distance through earth’s atmosphere.
As Xij<X^, red colour is scattered least and can enter into our eyes. That is why sun looks red at the time
of sunrise and sunset. For the same reason, danger signals are red.
Clouds are seen due to scattering of light from lower parts of atmosphere which contains large dust particles.
All colours are scattered equally. Hence clouds look white.
16. A rainbow is spectrum of sunlight through rain drops suspended in air. It is seen in the sky usually after
rain when back of observer is towards the sun.

Primary rainbow is much brighter with inner edge violet and outer edge red, subtending 41° - 43° angle
on observer’s eye. Secondary rainbow is fainter with inner edge red and outer edge violet, subtending an
angle 51" - 54° on observer’s eye.
RAY OPTICS AND OPTICAL INSTRUMENTS 9/93

17. Human eye is far more delicate and perfect than even the finest photographic camera. It can observe
distinctly the objects situated at widely different distances ranging from near point (A/) to far point F. This
property is called accommodating power of the eye. For a normal eye, least distance of distinct vision is
25 cm and far point F lies at infinity.
Two m^or defects of vision are: 1. Myopia or short sightedness 2. Hypermetropia or Long sightedness.
Both the defects can be corrected by using spectacle lenses of suitable power.
A myopic person can see clearly the objects lying near it only. To correct a myopic eye, the person has to use
spectacles with a concave lens of focal length/= - jc, where x is the distance of far point of myopic eye.
A hypermetropic person can see clearly only the far off objects. To correct a hypermetropic eye, the person

oww
has to use spectacles with a convex lens of focal length / where ^f = —V - -u = —:
-X
+ ^.
d
Here, u = -d = least distance of distinct vision of normal eye (= 25 cm)
v = -x', where x' is the distance of near point of defective eye.

e
re
FFrlo
18. A simple microscope is used for observing magnified images of tiny objects. It consists of a converging
lens of small focal length. Object is held between principal focus and optical centre of the lens. The image

rF
formed is virtual, erect and magnified.

ee
Magnifying power of a simple microscope is defined as the ratio of the angles subtended at the eye by the

rF
image and the object when both are situated at the least distance of distinct vision (d) from the eye. It is
ouru
given by
/

1.^
fosor
m =

f)
skf
In normal adjustment, when final image is at infinity, m = d/f.
ooko
Yo
19. In a compound microscope, the images are highly magnified. The objective lens forms a real, inverted and
Y

magnified image of the object. This acts as an object for eye lens, which forms a virtual, erect and magnified
Bo

image seen by the eye held close to the eye lens. Magnifying power of a compound microscope is given by
reeB

V
0
ooY

m =
uur

-u
0 fe ^
ad

where Uq and Vq are distances of object and image from optical centre of objective lens,/^ is focal length of
dY

eye lens.
20. An astronomical telescope is used for observing heavenly bodies like stars and planets etc.
nind
Re

The objective lens forms a real, inverted and smaller image of distant object in its focal plane. This image
serves as the object for eye lens, which forms a virtual, erect and magnified image seen by the eye held
F
Fi

close to the eye lens. In normal adjustment, final image as seen by the eye is at infinity. The magnifying
power of astronomical telescope is given by

m =
0 _ 4
-4 '4'
When final image is at the least distance of distinct vision from the eye, the magnifying power is given by

m =
-fo fe\
1 + -^
fe V d)
21. In a reflecting telescope, the objective lens is replaced by a concave parabolic mirror.

4 (fi/2)
Magnifying power of a reflecting type telescope is m=
4 4 ■
9/94 ‘P’uxdce^*^ Fundamental Physics (XIi)BS29D

QUESTIONS

Based on NCERT Book

I. Multiple Choice Questions 9. Two lenses of focal length 20 cm and - 40 cm are


held in contact. The image of an object at infinity
1. For which of the following media, with respect to
will be formed by the combination at
air, the value of critical angle is maximum?
(fl) (b) 20 cm
(a) Crown glass (b) Flint glass
(c) 40 cm id) 60 cm
(c) Water id) Diamond
10. What focal length should the reading spectacles
(CBSE 2022)
have for a person whose near point is 50 cm ?
2. The critical angle for a pair of two media A and B (a) 25 cm ib) - 50 cm
of refractive indices 2*0 and 10 respectively is

w
(c) 50 cm id) 25 cm
ia) 0° ib) 30°
11. An equiconvex lens of focal length 15 cm is cut
(c) 45° id) 60° (CBSE 2022)
into two halves as shown in Fig. 9(Q).l. What

Flo
3. The critical angle of a pair of a medium and air is would be focal length of each part ?
30°. The speed of light in the medium is

e
rree
ia) 10^ ms-^ ib) 1-5 X 10^ ms-‘ FIGURE 9(Q).1

r FF
(c) 2-2 X 10^ ms-^ id) 2-8 X 10^ ms->
(CBSE 2022) Ri A R2
uurr
4. A biconvex lens of glass having jj. = 1-47 is
immersed in a liquid. It becomes invisible and
for
kss
behaves as a plane glass plate. The refractive index
of the liquid is
ooook
Yo

(a) 1-47 ib) 1-62


(a) - 30 cm ib) - 20 cm
eB

(c) 1-33 id) 1-51 (CBSE 2022)


(c) 30 cm id) ~ 15 cm
5. A ray of light on passing through an equilateral
glass prism, suffers a minimum deviation equal to 12. What is the magnification and focal length of a
urr

angle of prism. The value of refractive index of plane mirror ?


ad
Yo

the material of prism is ia)+l, oo


ib)-U OO

{c)-l,0 (ii)+ 1,0


dY

ia) ^^2 (b) V3


13. The angle between incident ray and reflected ray
Re
innd

ic) I/V2 id) I/V3 (CBSE 2022) from a plane mirror is 60°. Through what angle is
the reflected ray deviated with respect to incident
Fi

6. Light of wavelength 6000 A falls on a plane


reflecting surface. The reflected wavelength is ray side plane mirror.
(a) 150° ib) 120°
(a) 6000 A ib) < 6000 A
(c) 90° id) 60°
ic) > 6000 A id) cannot say
14. The final image formed in an astronomical
7. Refractive index of glass w.r.t water is 9/8. What
telescope w.r.t. the object is
is the speed of light in water ? Given speed of light
in glass is 2 x 10® m/s. (a) real, inverted ib) real, erect
(a) 2 X 10® m/s ib) 3 X 10® m/s (c) virtual, erect id) virtual, inverted
ic) 2-25 X 10® m/s id) none of these (CBSE Sample Paper 2019-20)
8. The focal length of a double convex lens is equal 15. An equilateral prism is made up of a material of
to radius of curvature of either surface. The
refractive index What is the angle of minimum
refractive index of the material is
deviation of light passing through the prism ?
ia) 3/2 ib) 1
ia) 60° ib) 45°
(c) 4/3 id) 2/3
ic) 30° id) 90°
RAY OPTICS AND OPTICAL INSTRUMENTS 9/95

16. An object is placed at a distance x from the 25. In the diagram of a prism of angle 30“ is used. A
principal focus of a concave mirror of focal length ray PQ is incident as shown in Fig. 9(Q).2. An
/. What will be the magnification of the image ? emergent ray RS emerges perpendicular to the
(a)x/f (b)f/x second face. The angle of deviation is :
(c) 1 +f/x id) 1 -f/x
17. A convex mirror forms an image which is l/« times
the size of the object. If the focal length of the
mirror is/, then the distance of the object is
(a)nf ib)f/n
(c)(n+l)f

oww
18. The linear magnification of a convex mirror is

(a) always positive


(b) always negative
(c) sometimes positive and sometimes negative

ee
rFFlo
(d) cannot predict

r
19. Two thin convex lenses of focal lengths/| and/2 (^7)60“

ree
(a)0“

F
are placed at a distance d between them. If the
(c) 30“ (d)45“
power of combination of lenses is zero, then the

rF
26. One cannot see through fog, because
separation between the lenses is :
{a) fog absorbs the light

fsfoor
ouur
(^) V’A-^2 (*) {b) light suffers total reflection at droplets
(c) refractive index of fog is infinity
kosk
(c)(f,-/2) (^0(^l+/2>
id) light is scattered by droplets
20. A small ink dot on a paper is seen through a glass
Yo
27. The angle of minimum deviation for prism of angle
oo

slab of thickness 4 cm and refractive index 1-5.


Y

7t/3 is n/6. The refractive index of the material of


BB

The dot appears to be raised by


the prism is
(a) 1 cm {b) 2 cm
rre

(c) 3 cm id) 1-33 cm (a) S Q>) ^


oYuu

21. Optical fibres are based on the phenomenon of (c) 3/2 (d) 2/3
ad

(a) reflection (b) refraction 28. The lens used for correcting myopia is
dY

(c) dispersion (d) total internal reflection (a) concave (b) convex
22. Two lenses of focal lengths 20 cm and - 20 cm are (c) piano concave (d) none of these
innd
Re

held in contact. The image of an object at infinity 29. The focal length of an equiconvex lens in air is
will be formed by the combination at equal to either of its radii of curvature, the
Fi
F

(a) 00
{b) 20 cm refractive index of the material of the lens is

(c) 40 cm id) 60 cm (a) 1.25 ib) 2.5


23. In vacuum, which colour travels fastest ? (c) 1-33 (^01-5
(a) red 30. The final image in an compound microscope (w.r.t.
(jb) violet object) is
(c) yellow (a) virtual and erect (b) real and erect

id) all colours have the same velocity (c) real and inverted (d) virtual and inverted
24. 31. A thin lens made of glass of refractive index 1-5
A glass slab (|i = 1 ’5) of thickness 3-0 cm is placed
on an ink spot. A person looks at it from a distance has a focal length equal to 12 cm in air. It is now
immersed in water of refractive index 4/3. Its new
6-0 cm above the ink spot. The distance of the spot
will appear to be : focal length is
(a) 2-0 cm (b) 10 cm (a) 12 cm (b) 24 cm
(c) 4-0 cm (d) 5"0 cm (c) 36 cm (d) 48 cm
9/96 Fundamental Physics fXIIlETOWTl

32. An astronomical telescope has a magnifying power


of 10. In normal adjustment, distance between the
objective and eye piece is 22 cm. The focal length
of objective lens is
(a) 10 cm (b) 22 cm
(c) 20 cm (d) 2 cm
33. A diver at a depth of 12 cm in water of refractive
index 4/3 sees the sky in a cone of semi vertex angle

-! 3
(a) 90“ (b) sin
\4

-1 3 -1 (a) 1 (b) 2
(c) cos (cf) tan

w
V3 ic) 3 (d) 4
34. How does the focal length of a convex lens change 39. The characteristic feature of light which remains
if mono-chromatic red light is used instead of violet unaffected on refraction is

Flo
light ? (a) speed (b) frequency

reeee
(t2) Focal length is increased when red light is used (c) wavelength id) velocity of light
(b) Focal length is decreased when red light is used

Fr
(c) Focal length remains same when red light is II. Assertion-Reason Type Questions
used
Direction. For question numbers 40 to 48, two
(cf) Not depends on color of light
for
statements are given, one labelled Assertion (A)
ur
35. A glass lens is immersed in water. What will be and the other labelled Reason (R). Select the
the effect on the power of lens ? correct answer to these questions from the codes
kkss
(a) increase (b) decrease ia), ib), ic) and id) as given below :
Yo
oo

(c) constant {d) not depends (a) Both, A and R are true, and R is correct
36. How does the magnifying power of a telescope explanation of A.
eB

change on increasing the linear diameter of its ih) Both, A and R are true, but R is not the correct
objective ? explanation of A.
r

(a) Power increases on increasing diameter (c) A is true, but R is false.


ou
ad

(b) Power decreases on increasing diameter id) A is false, and R is also false.
YY

(c) Power remains constant on increasing diameter 40. Assertion. Mirrors used in search lights are
id) Power doesn’t depend on diameter parabolic.
ndd
Re

37. An object approaches a convergent lens from the Reason. Parabolic mirrors are free from spherical
left of the lens with a uniform speed 5 m/s and aberration.
Fi

slops at the focus. The image


41. Assertion. The image formed by convex mirror
(a) moves away from the lens with a uniform speed cannot be obtained on the screen.
5 m/s.
Reason. The image formed by the convex mirror
(b) moves away from the lens with a uniform
is always virtual.
acceleration,
42. Assertion. Lateral shift produced by a glass slab
(c) moves away from the lens with a non-uniform
acceleration.
increases with the increase in thickness of glass
slab.
id) moves towards the lens with a non-uniform
acceleration. Reason. Angle of refraction increases with the
increase of the thickness of glass slab.
38. The optical density of turpentine is higher than that
43. Assertion. Refractive index of a medium can be
of water, while its mass density is lower. Fig. 9(Q).3
shows a layer of turpentine floating over water in less than unity.
a container. For which one of the four rays incident Reason. The angle of incidence is equal to the
on turpentine is the path shown correct ? angle of refraction.
RAY OPTICS AND OPTICAL INSTRUMENTS 9/97

44. Assertion. The air bubble shines in water. Reason. Dispersive power = angular dispersion
Reason. Air bubble in water shines due to X mean deviation.

refraction of light. 47. Assertion. A convex mirror cannot be used as a


45. Assertion. A convex lens always behaves as a driver’s mirror.

converging lens. Reason. Field of view of convex mirror is small.


Reason. Focal length of converging lens is positive. 48. Assertion. Two lenses of power 1-5 D and 2-5 D
46. Assertion. The dispersive power of the material are held co-axially 1 metre apart. The power of the
of the prism depend on the shape and size of the combinalion is 4 D.

prism. Reason. P = + P-,.

ooww
ANSV/ERS

I. Multiple Choice Questions


1. ic) 2. (b) 3. (b) 4. (a) 5. (£>) 6. (a) 7. (c) 8. (a) 9. (C-) 10. (c)
11. ic) 12.(a) 13. ib) 14. (d) 15.(a) 16. ib) 17. (d) 18. io) 19. (d) 20. (d)

e
ree
21. id) 22.(a) 23. id) 24. (d) 25.(c) 26. id) 27. (c) 28. ia) 29. id) 30. (d)
31. id)

rFl
32. (c) 33. ib) 34. (a) 35. (b) 36. id) 37.(c) 38. (/;) 39. (b)

Fre
rrF
II. Assertion-Reason Type Questions
40. ib) 41. ia) 42. (c) 43. id) 44. ic) 45. ib) 46. id) 47. (</) 48. (i/)
ouur
sffoo
okks
HINTS/EXPLANATIONS For Difficult Questions
Yo
ooo

I. Multiple Choice Questions


BB

sin A 2 sin A/2 cos A/2


= 2 cos A/2
1 sin A/2 sin A/2
rr e

1. As p = C will be maximum when |i is


sin C ’
s
ouu

minimum. Water is the right choice. = 2 cos 30° = 2 X


ad
YY

2. sin 9 C
)i rarer _ _ 1 6. The reflected wavelength is 6000 A only because
"
)i denser 2 there is no change in the wavelength of light on
dd

reflection.
Re
iinn

0c = sin-* (1/2) = 30°


c/v
^ _9 V V
F

g
1 I w

7.
3. From, p = = 2 c/v w V 2x10'
sin C sin 30° g

Again, from p = —,
c
v=~
c 3x10* V = 2xl0^x? =2-25x10* m/s
ft 2

= 1-5 X 10* m I
8. From — = (p-l) —-
4. In liquid the lens becomes invisible. Threfore, / R
1
refractive index of liquid = refractive index of
lens = 1-47 Here,/= P, Pj = + P, Ro=- R, then
5. Here,A = 60°,6„, = A.p=? 1 2
From prism formula, - = (p-l) -+-
p VP P p

sin(A + 5^^^)/2 sin (A + A)/2 1 1 3


p = + 1 = -
sin A/2 sin A/2
^^ = 5
or
2 2
9/98 7^>uideefi, ^ Fundamental Physics (XII) VOL.II

1 1 1 \ l__J_ ; F = 40cm
15. Here, A = 60», n = ^, 5^ = ?
^ fl^fl 20 40 ■ 40
sin (A+ 8^)/2
From
For the combination of lens, sin A/2

1 = 1-1 —=1 ^ ^
s= sin(60 + 5^)/2
or
F V u 40 " u — oo V
sin 60V 2
or V = 40 cm
10. If </ = least distance of distinct vision of normal
eye = 25 cm.
sin (60"+5 m
)/2 = ^sin30°=^ = sin 60®

y = distance of near point of defective eye = 50 cm 60®+8 m


= 60®
/= focal length of convex lens to be used

ww
2

From / = x'd ^50x25 = 50 cm 8^= 120®-60® = 60®


x'-rf "50-25 16. Here « = -(/'+ x), u = ?,/= -/

Flo
11. For equiconvex lens. . 1 1 1 u . u u u
As — = — + — or — = 1 + — or 1

ee
/
1 1 ^ / / V /
j__ 1
u V

eer
/~15 If m is the magnification produced by concave

FFr
mirror, then
For equiconvex lens, Ri = + R, /?2 = - F

oorr
uur r
I u u -(f + x)
-1
1 ,(U-1),,fi—+ —n =(n-i)4R
s ff
— = m V f -/
15 ^ U r)
sk
= /±^_l = l + £_l = £
YYoo
ooko

or
(l-l-l) 1 f / /
R 30
eBB

/
For each half of lens, Fj = /?, /?2 = — oo
or m = —
X

1 {I
uurr

1 1
17. Here, v = + u/n, u-- u,f= +f
ad

f \R oo/
Yo

1 1 1 1 1 1 n-1
/' = 30cm - = - + — or — +
u uln
f u V f u
dY

12. In a plane mirror, image is virtual, erect and of


u = (n-l)f
Re

or
same size as the object,
innd

18. In a convex mirror, image is always erect.


size of the image x Therefore, linear magnification is positive.
FFi

fn = 2- = +- = +l
size of the object X
19. If F is the focal length of the combination of two
For plane mirror u = -o°yV = + °o lenses placed distance d apart, then

1 ^
f u u — oo + 00 f*k f,fi
or /= 00
D „ 1 1 1
.*. Power, /> = — = —h — = 0 (Given)
13. Here, i = r and i + r = 60®
f, fi f,k
or r + r = 60® or r = 30®

.-. Angle through which the reflected ray is then


deviated w.r.t., to incident ray side plane mirror
= 90®+ 30® =120®.

14. In an astronomical telescope, the final image or J _flfl I f\fl =/2+/l


(w.r.t. the object) is virtual and inverted. A fi
RAY OPTICS AND OPTICAL INSTRUMENTS 9/99

real depth (x) 29. Here/?i= + /e,/?2 = -^./=^>|i = ?


20. As \i =
app. depth ()') 1 f \ 1 ^
As
/
1.5 = 1

4
y = — = 2-67 cm
1-5 1
1
or - = Li-l or Ll = l + - = 1*5
The dot appears to be raised by (x - y) 2 2
= 4 - 2-67 = 1*33 cm
31. Heie")Aj= l-5,/=12cm
22. 1 __1_ J_
— = 0 1
/

±_J_'

w
F~ /2 20 20 As
f 8
\ R,-R,j
F = oo cm.
when lens is in air, then.

Flo
The image of object at infinity will be formed at
the focus i.e., at oo cm from the combinatioii. 1 r 1 1 ^

e
- = (1-5-1) —

rree
23. In vacuum, all colours travel with the same 12 R,)

r FF
velocity.
24. Distance of person from upper face of glass slab 1 1

R^j 12(1-5-1) 6
uurr
or
= 6-0 - 3-0 = 3-0 cm.

Here, li = 1 -5, real depth = 3-0 cm. for


When lens is immersed in water, then refraction
kss
Apparent depth = ?
is taking place from water to glass.
ooook
Yo
Real depth
As, H =
Apparent depth (x) 8 _
eB

w.

4/3 "8
3-0
1-5 =
If/' is the focal length of lens when immersed in
urr

X
water, then
ad
Yo

3-0
or = 2cm
dY

1-5

.●. Distance of observed spot from person = 2 -f 3


/' ‘ rJ
Re
innd

= 5*0 cm
= f£_llxi = —
Fi

25. Here, / = 90“ - 30“ = 60“, A = 30“, e = 0“, 5 = ? "U ^J6 48


As I + e = A + 8 or 8 = (/ + e) - A
or /' = 48cm
8 = (60“ + 0)-30“ = 30“
26. One cannot see through fog as light is scattered
by droplets.
32. Here, M = -^ = 10 ●●● /„=10/,.
4
27. From, 8„ = Cu-l)A
Distance between objective and eye piece
n n
/«+/« = 22
^=(.-1)3 10/e-»-4 = 22 .*. 4 = 2 cm
jc/6 1 /o= 104= 10x2 = 20 cm
p-l = = - or p = 3/2 33. A diver in water at a depth d sees the world outside
tc/3
lying within a cone of semivertex angle called
28. For correcting myopia, a concave lens is to be critical angle, given by
used.
9/100 <*. Fundamental Physics (XII) VOL.II

43. Here Reason is false as the ratio of the sine of


1
sin C = — or C = sin~* (l/|i) angle of incidence (i) to the sine of angle of
refraction (r) is constant for two medium, i.e.,
refractive index of second medium w.r.t. first, i.e.,
= sin"‘ (l/(4/3)) = sin-* (3/4)
sin I
|i = - = a constant
1 1 1 ^ sinr
34. - = (n„-i) .
V ^2j The refractive index of a medium stands for the
rays of light going from rarer to denser medium in
1 f I which case sin i > sin r, so ^ of a medium cannot
2J
be less than unity. Hence Assertion is false.
44. Here Assertion is true but Reason is false because

w
the shining of bubble in water is due to total internal
reflection of light.
Jy Jf
45. Here, both Assertion and Reason are true but

Flo
i.e., focal length is increased, when red light is Reason cannot explain the Assertion, because
used. convex lens converges or concentrates the rays of

eee
light passing through it at a single point.
Vi n

Fr
1
35. P = — 8
-1 46. The dispersive power of the material of the prism
●'a R, R^) is independent of the shape and size of the prism.
Thus Assertion is false. As dispersive power
for
ur
/
u. V = angular dispersion/mean deviation. Hence,
p
1 ^8 -1 i___r Reason is false.
ks
W
47. Both Assertion and Reason are false as convex
Yo
mirror has large field of view and can be used as
oo

1 1 driver’s mirror.
Clearly, — (or P^) < — (or P^)
eB

J1 4
w
48. When two lens of focal length/j and/2 placed
i.e., Power of lens will decrease.
distance d apart then total focal length is
r

37. As the object is moved closer to the lens from the J ^


ou
ad

...(0
left, the image moves away from the lens with a ///2 flfz
YY

non-uniform acceleration.

38. As turpentine is optically denser than water, so If Pj, P2 are the power of lenses and P is the total
power of combinationof lenses, then
nd
Re

the ray of light going from denser to rarer medium


(water) bends away from the normal. Thus option 1
Fi

P = —
(b) is true. F ’
39. The fi^uency of light is not affected on refiaction.
1
II. Assertion-Reason lype Questions ,P1 = —
40. Both Assertion and Reason are true but Reason
cannot explain the Assertion. 1
41. Both Assertion and Reason are true and Reason and ^2=-r
is the correct explanation of Assertion as virtual ●^2
image cannot be obtained on screen. From (/), P = Pl+P2-dPiP2
42. Here, Reason is false as angle of refraction Thus Reason is false.
decreases with the increase of the thickness of glass Now P= 1-5+ 2-5-1 X 1-5 X 2-5
slab. Lateral shift, d is given by = 4-0-3-75
d = tsin(i- r) sec r, i.e., doc t. = 0-25 D
Thus Assertion is true.
Thus Assertion is also false.
RAY OPTICS AND OPTICAL INSTRUMENTS 9/101

PROBLEMS

1. Prove that spherical mirror formula is sees a number of images of the candle. What
applicable equally to a plane mirror. is the origin of these multiple images ?
Sol. The spherical mirror formula is Sol. The origin of multiple images is the multiple
reflection of light between the front and back
1 i-1 ...(/) surfaces of glass. Infact, at the front surface of
u
glass, light is partially reflected and partially
For a plane mirror, R = oo refracted. The refracted light gets reflected at
the back surface and then multiple reflections
R
follow within the thickness of glass. They are

w
= OO

responsible for multiple images.


5. If you were driving a car, what type of mirror
From (0,

Flo
U V OO
would you prefer to use for observing traffic
at your back ?

e
reee
1 1 Sol. For observing traffic at our back, we prefer to
or
use a convex mirror. This i.s because a convex

FFr
V u

or V = - u
mirror has a much larger field of view than a
plane mirror or a concave mirror.

for
As u is negative, v becomes +.
ur
Also, the nature of the image formed does not
Hence, image is formed behind the mirror at change with changing distance of the object. It
the same distance as the object is in front of
kss
is always virtual, erect and smaller.
it. This is what happens in a plane mirror. Hence
Yo
the desired result. 6. Suppose that the lower half of a concave
oo

mirror’s reflecting surface is covered with an


2. The wall of a room is covered with a perfect
eB

opaque non-reflecting material. What effect


plane mirror and two movie films are made,
will this have on the image of an object placed
one recording the movement of a man and the in front of the mirror ? {CBSK 201-^:
ur

other of his mirror image. While viewing the


Sol. As laws of reflection are true for all points of
ad

film later, can an outsider tell which is which ?


YYo

the mirror, the image will be of the whole object


Sol. Nature exhibits left right symmetry i.e. physical
(and not of half object as expected erroneously).
laws are the same for an object and its mirror
However, as area of the reflecting surface has
image. Therefore, an outsider cannot distinguish
d
Re

between the two films.


been reduced, the intensity of the image will
in

reduce.
However, the distinction can be made if there
F

7. A section of a sphere has a radius of curvature


is any initial asymmetric information.
of 0'80 m. Both, inside and outside surfaces
3. Why are mirrors used In search lights have a mirror like polish. What are the focal
parabolic and not concave spherical ? lengths of the inside and outside surfaces?
Sol. In search lights, we need an intense parallel Sol. When inside surface has mirror like polish, the
beam of light. If a source is placed at the focus
0-8
of a concave spherical mirror, only paraxial rays mirror is convex. f=RI2 = + - = 0*4 m
are rendered parallel. Due to large aperture of
mirror, marginal rays give a divergent beam. When outside surface has mirror like polish, the
But in case of parabolic mirror, when source is 0-8
at the focus, beam of light produced over the mirror is concave, f=R/2 = - = - 0*4 m
2
entire cross-section of the mirror is a parallel
beam. 8. Will the reflected rays converge at a point when
4. A man holding a lighted candle in front of a a parallel beam of light is incident on a concave
mirror of large aperture ? (CBSI':2001)
thick glass mirror and viewing it obliquely
9/102 ‘Pnadeefi. ^ Fundamental Physics (XII) LV«1WII
Soi. No, this occurs because of spherical aberration 1 ^
of the mirror. Paraxial rays are focussed at the Sol. The apparent normal shift = / I —
principal focus and marginal rays are focussed
between pole and principal focus of the mirror. As is lowest among visible colours, the red
9. Give three basic difTerences between real coloured letter appears to be raised minimum.
image and virtual image. 15. The critical angle for glass air interface is C
and for glass water interface is CHow are
Sol.
C and C' related ?

Real Image Virtual Image Sol. sin C =


1 1 _ —,
2 and
a
3/2 ” 3
1. Rays meet actually at l.Rays appear to diverge g

ooww
the image point. from the image point. 1 1 _ 8
sin C' =
2. It can be taken on a 2. It cannot be taken on a w
9/8 " 9
screen. screen.
As sin C' > sin C C'>C.
3. It is always inverted. 3. It is always erect. 16. Why does a diamond shine ? (CBSR 2009)
Sol. The brilliance of diamond is due to total internal

e
reflection of light. )i for diamond is 2.42, so

ree
I. Refraction at plane surfaces

rFl
and Total Internal Reflection that critical angle for diamond air interface is

Fre
C = 24.4® (from sin C= 1/p). The diamond is cut

rrF
10. Why does a ray of light bend towards normal suitably so that light entering the diamond from
as it passes from air to glass ? any face suffers multiple total internal reflec
tions at the various faces, and remains within
ouur
Sol. As p = —
smr
sini c
sffoo
= —, and speed of light in glass
V
the diamond. Hence the diamond sparkles.
17. Explain why (a) A diamond glitters in a
okks
(v) < speed of light in air (c) brightly lit room, but not in a dark room,
Yo
.'. sin r < sin i or r < i. Hence a ray of light bends (b) A crack in a window pane appears silvery,
ooo

towards normal as it passes from air to glass. (c) The bubbles of air rising up in a water
BB

11. Can the relative refractive index of a medium tank appear silvery when viewed from top.
w.r.t. another medium be less than unity ? Sol. (a) A diamond glitters in a brightly lit room
rr e

because light entering the diamond from any


Sol. Yes, for example, the relative refractive index
face suffers multiple total internal reflections
ouu

4/3 8 and does not come out. The diamond appears


ad
YY

of water w.r.t. glass = = -<l.


3/2 9 illuminated from inside. This would not happen
in a dark room.
12. Can the absolute value of refractive index of
dd

a medium be less than unity ? (b) A crack in a window pane appears silvery
on account of total internal reflection of light
Re
iinn

Sol. No, this is because absolute refractive index in the crack,


(c) The bubbles of air rising up in a water tank
F

c _ speed of light in vacuum


R =- appear silvery when viewed from top again on
V speed of light in medium
account of total internal reflection of light from
and V is always less than c. the bubble.

13. When light comes from air to glass, the 18. Path of a ray of light passing through three
refracted ray is bent towards the normal. liquids of refractive indices P|, P2’ 1^3
Why ? (Manipur 2015) shown in Fig. 9(Q).4. Which liquid has the
smallest index of refraction ?
Sol. According to Snell’s law,
FIGURE 9(Q).4
It g
sm I
1.5 = 1
sm r 2

sin r < sin / or r < /

refracted ray bends towards normal. !^2


14. If a plane glass slab is placed on letters of
different colours, then red coloured letter ^3
appears to be raised minimum, why ?
RAY OPTICS AND OPTICAL INSTRUMENTS 9/103

Sol. As is clear from Fig. 9(Q).4, ingoing from liquid FIGURE 9(Q).6
A to liquid B, the ray of light bends away from
normal. Therefore, ji2 <
Again, in going from liquid B to liquid C, the
ray of light bends towards normal. Therefore,
H2 < ^3-
Hence the smallest index of refraction is |i.2.
II. Refraction at spherical surfaces
and Refraction through lenses
(b) When object is placed at the focus of a
19. A ray of light after refraction through a concave lens of focal length f u = +f,v = 1
concave lens becomes parallel to the principal

ww
axis after refraction through the concave lens. '
From lens formula, — -I '
Explain with a ray diagram when this can V u /
happen.
1-1 + 1

Flo
Sol. A ray of light directed towards first principal
focus of the concave lens emerges after V f u

ee
refraction through the concave lens in a As/is negative for a concave lens,

rere
direction parallel to the principal axis of concave I 1

r FF
lens as shown in Fig. 9(Q).5. + - = 0
V -/ /
FIGURE 9(Q),5
uurr
= oo

foor
Image is formed at infinity. The course of rays
is shown in Fig. 9(Q).7.
ks s
I
Yoo
i
oook

C Fi

LA
eBB

20. The surfaces of the sun glasses (goggles) are


uurr

curved, yet their power may be zero. Why ?


ad

Sol. Both the surfaces of sun glasses are curved.


Yo

Also,
dY

/?] — /?2
Re

1
innd

As F = (u-1)
[^1 ^2 22. The refractive index of the material of a
FFi

P = 0 concave lens is n. It is immersed in a medium

21. Calculate the position of the image of an of refractive index n j. A parallel beam of light
object when placed at is incident on the lens. Trace the path of
(a) focus of a convex lens emergent rays in each of the following cases :
(a) > n (b) < n (c) = n.
(b) focus of a concave lens.
Sol. The paths of the emergent rays in the three cases
Sol. (a) When object is placed at the focus of a are shown in Fig. 9(Q).8.
convex lens of focal leng'"' f f = ?

I
(a) Aj «] > n, light goes from denser to rarer
From lens fonnula medium. Therefore, in passing through a
V u / concave lens, it converges instead of diverging,
as shown in Fig. 9(Q).8(a).
-=-+l- 1
— = 0, f = «●
V / u ./■ / (b) /Jj < n, light goes from rarer to denser
medium. Therefore, in passing through a
Image is formed at infinity. The course of
concave lens, it diverges as shown in Fig.
rays is shown in Fig. 9(Q).6. 9(Q).8(/>).
9/104 Fundamental Physics (XII)SSSi5Il

A2=7i(J/2)-7i(*i/4)-
FIGURE 9(Q).8
Hi
Hi <-P tP- 3nP
ni

W
= K - K
n n1 4 16 16

^2 Pi 37cr/-4 _ 12 _
16 I4 ●● ^2 4
As

7^
1

i /[ A| 16Tif/^
(a) n,>n (b) Oi<n 25. What happens to focal length of a convex lens
Hi Hi when it is immersed in water ? (CBSE 2(104)
w Sol. From lens maker's formula

1 1 1
- = 4i-i) ●●● /
/ R (H-i)

ww
I
(c)rii=n
a
As
(c) As Hi = n, there is no change in medium on
passing through a concave lens. Hence there is fw ^fa’ length of lens will increase.

Flo
no bending/refraction. The rays continue along 26. The radii of curvature of both the surfaces

ee
their original path, as shown in Fig. 9(Q).8(c). of a lens are equal. How will its focal length

rere
23. Following data was recorded for values of and power change if one of the surfaces of

rFF
object distance and the corresponding values the lens is made plane ?
of image distance, in the study of real image Sol. Let /?!=/?-,=/?
uurr
formation by a convex lens of power + 5D.
One of these observations is incorrect.
Identify and give reason :
foor
From lens maker’s formula

1
I 1_
ks s
- = (p-l)
/ R,! /?-,“
Yoo
S. No. 1 2 3 4 5 6
oook

= (|i-l)f-
R +-
2(|i-l)
eBB

Object distance (cm) 25 30 35 45 50 55


^ R R

Image distance (cm) 97 60 37 35 32 30 When one surface is made plane, R2 = 00


uurr

1 1 p-1
- = (p-l) - =
ad

Sol. Here, P = + 5 D
/' R R
Yo

100 ^ 100 Clearly/' = 2/ i.e., focal length becomes twice.


= 20 cm
dY

~ P ~ 5
As />«—. power becomes half.
Re

In observation3, where u = 35 cm, object lies


innd

/
between/ and 2 ./ The image must be beyond 27. A biconvex lens made of a transparent
FFi

2/ i.e., u > 40 cm. But v = 31 cm, i.e. image material of refractive index 1*25 is immersed
also lies between/and 2/, which is wrong. in water of refractive index 1*33. Will the lens
24. A thin lens has focal length f and its aperture behave as a converging or diverging lens ?
has diameter d. It forms an image of intensity
Justify your answer.
/. Now the central part of the aperture upto
(CBSE Sample Paper 2022-23)
diameter dll is blocked by an opaque paper.
What will be the focal length and image 1 ( 1 1 A
Sol. As, - = (|i-l) ---
intensity now ? / R
I
Sol. On blocking the central part of the lens, its focal
length does not change. It remains /only. m
-1
1

Intensity of image is directly proportional to the R


^2
area of the lens through which light passes. Now,
initial area Aj = 7C {dH)^ = 7t d^lA As ^-125 < 1, /is negative
P- IV 1-33
On blocking the central part of the aperture upto
diameter dll, the area left out is The lens will behave as diverging lens.
RAY OPTICS AND OPTICAL INSTRUMENTS 9/105

28. Why is there no dispersion of light refracted equally nearly. They merge to give us the
through a rectangular glass slab ? sensation of white.

Sol. After refraction at two parallel faces of a glass 33. The sun looks reddish at the time of sunrise
slab, a ray of light emerges in a direction parallel and sun.set. (CHSi: 20K.1
to the direction of incidence of white light on Sol. At the time of sunrise and sunset, the sun is near
the slab. As rays of all colours emerge in the the horizon. The sun rays have to travel a larger
same direction (of incidence of white light), part of atmosphere to reach the observer. As
hence there is no dispersion, but only lateral 1
displacement. — and is largest in the visible region,
X^
29. A ray of light falls normally on one face of a
red colour is scattered the least, and enters our
prism of angle 45". If critical angle for
eyes.
material of the prism is 45", trace the course
34. What is the cause of blue colour of ocean ?
of rays.

ww
Sol. Blue colour of ocean is due to preferential
Sol. The incident ray PQ falling normally on face
scattering of light by water molecules.
AB of prism, goes straight striking face AC at
45" as shown in Fig. 9(Q).9. As C = 45", 111. Human eye

Flo
therefore, the refracted ray QR suffers total
35. A girl is using specs of /= - 50 cm. Name the

e
internal reflection and goes along RD.

eree
defect of her vision and calculate power of
lens to be used. (CllSr. .itil),

FFr
Sol. As/is negative, the lens is concave. The girl is

oorr
suffering from myopia.
uur r
sf
100
= -2D
/ -50
sk
Yoo
oook

36. Why has nature given us two eyes instead of


one ?
eBB

Sol. Nature has given us two eyes, which give us a


pretty good idea about the distance of an object.
uurr

For example, if we were to put a thread into a


ad

needle’s eye with one of our eyes closed, the


Yo

thread goes either too far behind the needle or


30. What colour do you observe when white light stops short in front of it. However, when we
dY

passes through a blue and yellow filter ? focus both our eyes on the needle’s eye,
Re

Sol. When white light passes through a blue and threading gets done instantly. Thus nature has
innd

yellow filter, we get green colour with a tinge given us two eyes so that we can have an idea
FFi

of blue and yellow. This is because blue and of solidity and distance of the object we observe.
yellow colours transmitted from these filters mix 37. The diameter of the sun is « 10^ m, but it
to produce green colour. appears to be a small disc, why ?
31. Why does clear sky appear blue ? Sol. This is because the distance of the sun from the
Sol. This is because blue/violet colour has least
earth is very large (= 10*' m).
wavelength in the visible region. It is scattered
.'. Angle subtended by the sun at our eye
the most (●.● Rnyleigh’s criterion). « lO'/lO" = 10-2 The same angle is
subtended by a disc of diameter 1 cm at a
32. Why do clouds generally look white ? distance of 100 cm. from the eye. Hence the
sun looks like a small disc.
Sol. The clouds are at much lower height. They are
seen by light scattered from lower parts of 38. The diameter of sun is several hundred times
atmosphere containing dust particles, ice bigger than the moon, still at the time of solar
particles, water droplets etc. As size of scatterer eclipse, the entire sun is covered by the moon.
» wavelength of light, all colours are scattered How ?
9/106 7\aeUe^'4. Fundamental Physics fXII^rTSTim
Sol. This is because distance of the sun from earth 44. (a) List some advantages of a reflecting
is 1000 times greater than the distance of the telescope, especially for high resolution
moon from earth. At the time of solar eclipse, astronomy, (Jharkhand Board 2012)
moon comes between sun and earth. The angles (b) A reflecting? 'ype telescope has a large
subtended by the sun and the moon at the eye mirror for its objective with radius of
are nearly equal. curvature equal to 80 cm. What is the
39. What are anastigmatic lenses ? magnifying power of telescope if eye piece
used has a focal length of 1.6 cm ?
Sol. The lenses which have been duly corrected for
Sol. (a) (i) There is no chromatic aberration as the
most of the optical defects of images formed
by them are called anastigmatic lenses. objective is a mirror,
These lenses arc used in good quality optical (if) Spherical aberration is reduced using mirror
instruments like camera, microscope, telescope, objective in the form of a paraboloid.

ww
binoculars, etc. (Hi) Image is brighter compared to that in a
refracting telescope,
IV. Microscopc.s and Tele-Si (iv) Mirror requires grinding and polishing of

Flo
only one side,
40. A telescope has been adjusted for relaxed eye.
(v) High resolution is achieved by using a mirror

e
How will you change the distance between

ree
objective lens and eye lens if final image is to of large aperture.
ib) Here, R = S0 cm.

Fr
be seen at the least distance of distinct vision ?

rF
Sol. For relaxed eye, L =fQ + fg (normal adjustment) Focal length of objective mirror
uurr
To form the image at the least distance of distinct
/^0 .^=!«=40cm
vision, L' =/q + Ug and Ug < fg.
L' < L. Hence the distance between objective
s for
and
2 2
f^= 1.6 cm.
kks
and eye piece has to be reduced.
-A)
Yo
oooo

41. How will you distinguish between a As Magnifying power, m = —

compound microscope and a telescope just 4


eB

by looking at them ? 40
m = = 25
Sol. As aperture of objective of a microscope is much 1.6
smaller and that of a telescope is much larger,
ur

45. The objective of telescope A has a diameter 3


therefore, front end of a microscope is narrow
ad

times that of the objective of telescope B. How


YYo

and front end of a telescope is much wider. much greater amount of light is gathered by
42. How does magnifying power change with A compared to 5 ? Show that range of A is
three times the range of B.
dd

change in length of tube for a given telescope ?


Re

Sol. For a telescope, magnifying power Sol. As diameter of objective of A is 3 times that of
in

B, its area of cross section is 9 times that of B.


F

Therefore, light gathering capacity of A is 9


—, and L = (/o + «e) times that of B.
fe u

In a telescope,/q is fixed. When L is increased, Now, range tells us how far away a star of some
Ug increases. Hence magnifying power will standard brightness can be spotted by the
decrease. telescope.
43. How does magnifying power change with 1
As Intensity OC
therefore, when
change in length of tube for a given (distance)^
microscope ?
distance of star is 3 times, the intensity would
Sol. In case of a microscope, magnifying power
1
V
0 reduce to ~ . As light gathering capacity of A
lml = —X—,
ii
where L = Uq + Mg.
0
is 9 times that of B, the star whose distance is 3
When L is increased, Ug increases as Uq is fixed. times can just be seen by A. Hence range of A is
.●. I m I will decrease. three times that of B.
RAY OPTICS AND OPTICAL INSTRUMENTS 9/107

● Very Short Answer


mu
I
● Short Answer
i>
● Long Answer

VERY SHORT ANSWER QUESTIONS Carrying 1 mark

1. Define principal axis of a spherical mirror. 11. Can a virtual image be photographed ?
Ans. A line joining the pole P to centre of curvature Ans. Yes, it can be photographed.
of the mirror, produced on both sides is called 12. A person moves vrith a velocity v towards a
principal axis of mirror. plane mirror. With what velocity docs his
2. A ray incident along normal to the mirror image move towards him ?
retraces its path. Why ? Ans. The image moves towards the person with a

w
Ans. This is because Z/ = Zr = 0°.
velocity 2 v.
3. What is the number of images of a point 13. What are focal length and power of a plane
object held inbetween two plane mirrors mirror ? (Uttarakhand 2012)

Flo
inclined at an angle d** ?
Ans. Focal length of a plane mirror is infinity.
360*’ 360 .

ee
Ans. Number of images, n = -1 . if IS Power, /> = — = — = Zero
0 6 / OC

Fr
360 360 14. If the wavelength of Incident light on a
even integer and n = , if is an odd concave mirror is increased, how will the
6 e
focal length of the mirror change ?
for
ur
integer.
Ans. The focal length of the mirror does not change
4. A mirror is turned through 15°. Through
on changing the wavelength of light incident
what angle will the reflected ray turn ?
ks
on it. This is because/= RH, i.e.f depends only
Ans. 30°, as the reflected ray turns through twice the
Yo
on radius of curvature of the mirror.
oo

angle through which mirror is turned.


5. Can we obtain the image formed by a convex 15. To which wavelength of light is our eye most
eB

mirror on a screen ? If not, why ? sensitive ? In which region does this


wavelength lie ?
Ans. No, because the image formed by a convex
Ans. Our eye is most sensitive to light of wavelength
ur

mirror is virtual.
5500 A. This wavelength lies in the yellow green
ad

6. Which mirror is divergent, convex or


Yo

concave ? region of visible spectrum.


Ans. Convex mirror is divergent. 16. What is the minimum size of a plane mirror
7. Can a convex mirror form a magnified which can enable a person to see his full size
nd
Re

image ? image ?
Ans. The minimum size, i.e., vertical height of plane
Fi

Ans. No, it cannot.


8. When does a concave mirror form a virtual minor should be equal to half the height of the
image ? person.

Ans. When the object is placed between the pole and 17. How many images of himself can a person
principal focus of the mirror. see in a room whose ceiling and two adjacent
walls are mirrors ?
9. What is mirror formula ? (Rajasthan 2015)
Ans. Six images. This is because two adjacent walls
Ans. The mirror formula is
are at 90°. They will produce 3 images
1
—+—= where u is distance of object, 360
V u / 1 . The ceiling mirror will repeat the
90
V is distance of image and/is focal length of three images taking the total to six images.
the mirror.
18. Which spherical mirror has a real focus and
10. What is the relation between / and R of a which one has a virtual focus ?
spherical mirror ? (MP Board 2015)
Ans. A concave spherical mirror has a real focus and
Ans. /= R/2. a convex spherical mirror has a virtual focus.
9/108 “Pfuxdee^'^ Fundamental Physics fXlDPPMTI
19. Which spherical mirror forms a virtual, erect 1
and smaller image of an object ? Ans. \x =
sin C
Ans. A convex spherical mirror forms a virtual, erect 29. Define Snell’s law of refraction.
and smaller image of an object.
sm/
20. Where should an object be held so that a Ans. \i = — , where symbols have their usual
concave mirror forms a real, inverted and sm r

magnified image ? meaning.


Ans. The object must lie between principal focus 30. What is the cause of refraction of light ?
F and centre of curvature C of the concave Ans. Change in the velocity of light on change in the
mirror. medium.

31. What is critical angle for a material of


I. Refraction at plane surfaces
and Total Internal Reflection
refractive index V2 ? (CBSE 2010)

w
21. What is refraction ? (Hr. Board 2001) Ans. From sin C- = —
^ - J_ , C = 45"
Ans. Refraction is the phenomenon of change in the It V2
32. A beam of light is converging towards a

Flo
path of light in going from one medium to
another. certain point. A parallel sided glass plate is
introduced in the path of the converging

ee
22. State the factors on which refractive index
of a medium depend ? [CBSE 2017(01
beam. How will the point of convergence be

Fr
shifted ?
Ans. Refractive index of a medium depends on
Ans. The point of convergence will be shifted away
{/) nature of medium
from the glass plate.

for
ur
(//) wavelength of light used
33. Determine refractive index of a substance if
(iii) temperature
critical angle is 45”. (Kerala Board 2012)
(/v) nature of surrounding medium.
ks
Ans. Here, C = 45®, p = ?
23. For which medium is refractive index
Yo
1 1
oo

minimum ?
p = = V2
Ans. p is minimum (= 1) for vacuum. sinC sin 45° I/V2
eB

24. When does Snell’s Law of refraction fail ? 34. What is the ratio of velocities of two light
Ans. Snell’s Law of refraction fails when light is waves travelling in vacuum and having
incident normally on the surface of a refraePng wavelengths 4000 A and 8000 A ?
ur
ad

medium. In that case, / = 0, r = 0. The ratio sin (MP Board 2011)


Yo

//sin r is meaningless. Ans. The ratio is one. This is because in vacuum, all
25. On what factors lateral shift of a ray on colours travel with the same velocity.
passing through a glass slab depend ?
nd
Re

Ans. The lateral shift depends on II. Refraction at spherical surfaces


(/) angle of incidence and Refraction through lenses
Fi

(//) refractive index of the medium 35. What type of lens is an air bubble inside
(///) thickness of refracting medium. water ? (HP Board 2011)
26. Can total internal reflection occur when light Ans. It behaves as a concave lens because water is
goes from a rarer to a denser medium. denser than air.
(CBSE 2007) 36. A lens of glass is immersed in water. How is
Ans. No. Total internal reflection cannot occur. power of lens affected ?
27. Which one has a greater critical angle- (Manipur Board 2012)
diamond or glass ?
Ans. Power will decrease as “’p^ < ‘^p^,.
I
37. A glass lens of refractive index 1*45 when
.Ans. ^'P£, = 246 and "p^, = 1-5. As p = sinC immersed in a transparent liquid becomes
therefore, invisible. Under what condition does it
28. What is the relation between refractive index happen ? (CBSE 2010)
and critical angle for a given pair of optical Ans. When refractive index of the liquid is equal to
media ? (CBSE 2009) refractive index of the material of the lens = 145.
RAY OPTICS AND OPTICAL INSTRUMENTS 9/109

38. Tw'o thin lenses of power + 6 D and -2D are 50. What is the relation between angle of prism
in contact. What is the focal length of the A, angle of incidence i and angle of minimum
combination ? (CBSE 2009) deviation ? (CBSE 2013)
Ans. Ans. = 2 i-A.
P = />, +^2 = 6-2=40
100 100 51. For which colour, red and blue, is the
F = = 25 cm refractive index of glass greater ?
P 4
39. What is the basis of an optical fibre ?
Ans. Refractive index of glass is greater for blue
colour than that for red colour.
(Pb. Board 2002)
Ans. It is based on the phenomenon of total internal 52. A glass prism is immersed completely in
water. How does angle of minimum deviation
reflection of light.
change ?

oww
40. A lens forms a virtual, erect and diminished
image whatever be the position of the object. Ans. Angle of minimum deviation decreases because
Which type of lens is this ? sin (A+ 5 m )/2
w
Ans. The lens must be a concave lens. and
sin A/2
41. What is one dioptre ? (CBSE 2002)
53. When does a ray passing through a prism

e
Ans. One dioptre is the power of a lens of focal length

FFrlo
deviate away from the base of the prism ?

re
one metre.

42. What is focal length of a lens of power Ans. This happens when prism is immersed in a

ree
F
- 2-5 D ? (CBSE 2004) medium of refractive index higher than the
refractive index of material of the prism.
100

rF
Ans. = - 40 cm 54. In the spectrum of white light through a
~ P ~ -2-5
prism, violet colour is seen at the botton.

fsoor
43. For which colour, {i. of material of a prism is
ouur
Why ? (CBSE 2012)
(i) minimum (k) maximum ? (CBSE 2010)
skf Ans. Violet colour is seen at the bottom because its
Ans. |i is minimum for red colour and p is maximum wavelength is minimum and its deviation on
ooko
for violet colour.
passing through the prism is maximum.
Yo
44. Which colour deviates (i) most (n) least on
Y
55. What is the purpose of adding “blue” to
Bo

passing through a prism ? (CBSE 2017. 2010) clothes ?


reB

Ans. Violet colour suffers maximum deviation and


Ans. On washing, clothes get a yellowish tint. Blue
red colour suffers the least deviation.
and yellow are complimentary colours giving
45. What are the factors on which angular
uur

us a sensation of white.
oY

dispersion of a prism depend ?


ad

56. Give the formula that can be used to deter


(CBSE 2004) mine refractive index of material of a prism
dY

Ans. (i) Angle of prism in minimum deviation position. (CBSE 2010)


(//) nature of material of the prism. sin (A + S^^)/2
innd
Re

46. On what factors does the dispersive power Ans. p =


sin A/2
Fi
F

of a prism depend ? (ISCE 2003)


Ans.
Symbols have usual meaning.
Nature of material of the prism.
47.
57. How does the speed of light in glass change
What is angle of prism ?
(fl) on increasing the wavelength of light ?
Ans. It is the ang^e between two refracting faces of
the prism. ih) on increasing the intensity of light ?
48. What is angle of deviation through a prism ? Ans. (0 Speed of light in glass incrca.scs on increasing
Ans. It is the angle between the incident ray on the the wavelength
prism, and emergent ray from the prism. (n) There is no effect of changing intensity on
49. On what factors does the deviation produced speed of light in glass.
by a prism depend ? 58. How is speed of light in vacuum affected by
Ans. Deviation produced by a prism depends on change in wavelengtli/intensity of light ?
(i) angle of incidence (n) angle of prism Ans. Speed of light in vacuum is an ab.solute constant.
{Hi) nature of material of prism It is not affected by change in wavelength/
(/v) colour/wavelength of light. intensity of light.
9/110 'Pnetdee^'^: Fundamental Physics (XII)SEIHD
59. Which one, crown glass or flint glass, has a 69. What is meant by depth of focus ?
larger refractive index ? Ans. Depth of focus indicates the range of the object
Ans. Flint glass has larger refractive index. distances over which the focussing is reasonably
60. Refractive indices of glass for blue, red and good.
70. What is visual angle ?
yellow colours are p.^and |i^,. Write them
in decreasing order of values. Ans. Visual angle is the angle subtended by an object
at the eye.
Ans. Values of p in decreasing order are p^,, p^, p,.
61. What is the ratio of speed of IR rays and UV IV. Microscopes and Telescopes
rays in vacuum ? (Manipur Board 2012)
71. An astronomical telescope uses lenses of
Ans. 1 : 1 i.e. speed in vacuum is the same for both power 10 D and 1 D. What is its magnifying
IR and UV rays. power In normal adjustment ?
62. Red light is incident on a converging lens of /n0 P. 10

w
focal length /. State with reason how / will Ans. -

change if red light is replaced by blue light. fe Po 1

72. Why is the aperture of objective lens of a

Flo
1
Ans. From lens maker’s formula. — OC telescope taken large ? (CBSE 2017, 2013)
f Ans. This is done to increase the light gathering

e
reee
capacity and hence brightness of image.
As Pi,>P;.,
73. Why is power of objective lens of a telescope

FFr
■■■ h<fr kept as small as possible ?
Hence/willdecrease. Ans. Small power implies large focal length and

for
ur
hence large magnifying power of the telescope.
in. Human eye
74. Why does Galilean telescope have a smaller
63. What is blind spot ? field of view ?
kss
Ans. It is the least sensitive spot on the retina of eye. Ans. This is because eye lens in such a telescope is
Yo
oo

concave.
64. What is yellow spot ?
75. In which, microscope or telescope, the
Ans. It is a spot at about the centre of the retina, which
eB

difference in the focal lengths of the two


is most sensitive to light. lenses is larger ?
65. WTiat is meant by accommodation of the eye ? Ans. In a telescope, the difference in the focal lengths
ur

Ans. It is the ability of the eye to observe distinctly of two lenses is larger.
ad

76. Why do we prefer a magnifying glass of


YYo

the objects situated at widely different distances


from the eye. smaller focal length ?
66. A short sighted person may read a book Ans. This is because the magnifying power is more
d

when focal length is short.


Re

without specs. Comment.


in

77. What do you understand by normal


Ans. The statement is true, because a short sighted
adjustment of a telescope ?
F

person has difficulty in observing far off objects.


Ans. In normal adjustment, the final image is formed
67. A hypermetropic person has problem in at infinity.
driving without specs. (Pb. Board 2010)
78. What is the distance between objective and
Ans. No, the statement is false. A hypermetropic eye lens of telescope in normal adjustment ?
person can observe far off objects without any Ans. a:= (/o +/^).
problem. 79. Can a terrestrial telescope be used for
68. A person looking at a mesh of crossed wires observing astronomical objects ?
is able to see the vertical wires more distinctly Ans. Yes, the final image will be erect w.r.t. the object.
than the horizontal wires. Why ? How can it It may be less bright as compared to the image
be corrected ? in the astronomical telescope, because of
Ans. This problem is due to astigimtism of the eye. absorption of some light by erecting lens.
80. What is the main limitation of Galileo’s
This defect is removed using a cylindrical lens
with appropriate axis and suitable radius of telescope ?
curvature. Ans. Its field of view is small.
RAY OPTICS AND OPTICAL INSTRUMENTS 9/111

81. What is the eye ring of a telescope or 84. Does the magnifying power of a microscope
microscope ? depend on colour of light used ? Justify your
Ans. The image of the objective lens formed by the answer. (CBSE (F), 2017)
eye piece is called eye irng. This is the ideal Ans. Magnifying power of a compound microscope
position of the eye for observing the image. IS

82. Is angular magnification of a telescope equal


to ratio of diameters of objective and eye lens ? M = —
Ans. Yes, it is true. h) , fej

w
83. What is meant by focal plane of a lens ?
Ans. A plane perpendicular to the principal axis and As /q and both depend on colour of light,
at a distance equal to focal length of the lens is therefore, magnifying power of a microscope
called focal plane of the lens. does depend upon colour of light.

roow
e
re
SHORT ANSWER QUESTIONS Carrying 2 marks

1. How can you distinguish between a plane Ans. In a plane mirror, virtual image is of same size

reF
uFFll
mirror, a concave mirror and a convex as the object. In a concave mirror, the virtual

e
mirror, Just by looking at them ?
image is magnified and in a convex mirror,
virtual image is always diminished in size.

sFr
(Hr. Board 2011. HP Board 2011) 6. What is the advantage of using a parabolic

foro
Ans. In a plane mirror, image formed is virtual, erect concave mirror over ordinary spherical
uor
fk
concave mirror ?
and of same size as the object. In a convex
mirror, image is erect and smaller in size. Ans. A parabolic concave mirror is free from spherical
okso
aberration as well as chromatic aberration.
In a concave mirror, the size and nature of the
7. A parallel beam of light strikes a (i) plane
Y
Yo
image change with change in position of the mirror (») a convex mirror and (Hi) a concave
oo
BB

object in front of it. mirror. What is the deviation produced in


2. A concave mirror of small aperture forms a each case in terms of the angle of incidence (/) ?
sharper image. Why ? Ans. In each case, angle of deviation = 180“ - 2 /.
rYree

For example, for a ray striking a plane mirror


ouu

Ans. This is because a concave mirror of small


normally, / = 0, r= 0, angle of deviation = 180®
ad

aperture is free from the defect of spherical


Ydo

i.e., the ray retraces its path on reflection.


aberration.
8. Use the mirror equation to show that an
3. How can we see a virtual image when it object placed between/and 2/ of a concave
nidn

cannot be obtained on a screen ? mirror forms an image beyond 2/.


Re

(MP Board 2011) (CBSE (F) 2017)


F
Fi

Ans. True, a virtual image cannot be taken on screen. 3/.


Ans. Let u — —
But our eye lens forms a real image of (virtual 2
image acting as virtual object) on the retina of when object lies between /and 2/
our eyes.
From mirror equation
4. Why is a concave mirror preferred to a plane
mirror for shaving ? 1 + 1-1
V u /
Ans. A concave miiTor forms a magnified and erect
image of the face when it is held close to the 1 1 1
2 _ -3 + 2 1_
face. That is why a concave mirror is preferred V / H / 3/“ 3/ ■ 3/
over a plane mirror for shaving.
For concave mirror,/is neg.
5. What is the difference between the virtual
v=-3f
images produced by
(/) plane mirror (//) concave mirror and i.e., distance of image from the concave mirror
(Hi) convex mirror ? is 3/ i.e., image formed is beyond If.
9/112
‘P’uuUeft Fundamental Physics (XII) W»!Wll

I. Refraction at plane surfaces 15. The focal length of an equiconvex lens placed
and Total Internal Reflection in air is equal to radius of curvature of either
surface. Is it true ?
9. An ink mark on a sheet of paper is viewed Ans. Yes,/= R only when ji = 1 -5. This follows from
through a glass slab of thickness t and
1 ( 1
refractive index p. Through what distance the - = (p-l) — . Ifp?i 1-5, then/?t/e
mark appears to be raised ? / Ai A-)
V ‘ *-7
Ans. Normal shift in the position of mark
16. Within a glass slab, a double convex air
1 > bubble is formed. How would the air bubble
= / 1--
behave ?

oww
10. A candle flame is held 2 metre above the Ans. The air bubble would behave as a diverging lens,
water level in a tank 4 metre deep. If p of because p of air is less than p of glass. However,
the geometrical shape of the air bubble shall
water is 4/3, where will the image of candle resemble a double convex lens.
flame be seen ?

ee
Ans. The image of candle flame is seen due to 17. Why does a convex lens of glass p = 1*5
reflection of light from water surface in the tank. behave as a diverging lens when immersed

FFrlo
r
The image will be at a depth of 2 metre from in carbon disulphide of p = 1-65 ?

rF
the surface of water in the lank.

ee
Ans. This is because p = = ±1 < 1
11. An air bubble in a jar of water shines 1-65
ouru
brightly. Why ?

rF
Ans. Light entering water in the jar is totally reflected 1 1 1
From — = (p -1)
from the air bubble. The observer feels this light
ffosor / R R.
1
is coming from the bubble. Sc the bubble
os k
appears to shine. / becomes negative. Therefore, the lens behaves
as a diverging lens.
12. For the same angle of incidence, the angles
ook
18. A diverging lens of focal length F is cut into
Yo
of refraction in media ,P Q and R are 35",
Y

25", 15" resp. In which medium will the two Identical parts, each forming a piano
Bo

velocity of light be minimum ? (CBSE 2012) concave lens, Fig. 9{Q).10. What is the focal
reeB

length of each part ? (CBSK 2008)


Ans. For medium R. angle of refraction (r) is
oouY

. sin/ . FIGURE 9(Q).10


ur

minimum. Therefore, p = IS maximum.


ad

sin r
c
dY

As p-—, therefore, v is minimum, i.e.,


V

velocity of light is minimum in medium R.


nidn
Re

13. Why does the rising sun appear oval shaped ?


Ans. Light from the rising sun reaches the earth’s
F
Fi

surface obliquely through different parts of the


earth's atmosphere, suffering unequal
refractions. The vertical diameter appears
shorter and the horizontal diameter appears
bigger. Therefore, the rising sun appears oval.
Ans. For single diverging lens,
II. Refraction at spherical surfaces f I 1 ^
and Refraction through lenses - = (p-l) —
14. Where should an object be placed from a
F ^ [ft, ^2
Let /?[=-/?, /?2 =/?
convex lens to form an image of the same size ?
Can it happen in case of a concave lens ? 1 -2(p-l)
- = (P-1)
Ans. The object has to be placed at a distance 2 / F -R R R
from the lens. No, it cannot happen in case of a
For each half (which is piano concave), as
concave lens, as image formed is always smaller .shown in Fig. 9(Q).I0,
in size.
RAY OPTICS AND OPTICAL INSTRUMENTS 9/113

Ans. As the lens is made of two different materials,


/?, = -/? and /^2 = oo
it has two refractive indices and hence two
1 1 f 1 1 different focal lengths. Hence two distinct
— = (!J-1) images will be obtained.
-R R
F, F,
OO

22. How does focal length of a convex lens change


if violet light is used instead of red light.
IF (CBSE 2010)

F, = F^=1F \ 1_
Ans. — = (^.-1)
19. Draw a plot showing the variation of power /V R.1 R-y
of a lens with the wavelengthof incident light.
{CBSE 200«) \ ^
fr R Ry
- /
1

w
Ans. From F = — = (ii -1) — As fv<fr
F /?, R, i.c’., focal length decreases.
23. In Fig. 9(Q).13, line represents a lens

Flo
As X increases, \i decreases.
Power P decreases.
through which course of rays is as shown. Is
this lens convex or concave ?

eee
The variation of P with X is as shown in Fig.
9(Q).1I.

Fr
for
ur
ks
Yo
oo

Ans. The line AB represents a concave lens because


eB

rays converging towards 0, diverge to meet at I.


20. An equiconvex lens of focal length 15 cm is 24. Explain with reason how the power of a
cut into two equal halves in thickness. What diverging lens changes when
r

is the focal length of each half ?


ou
ad

(/) it is kept in a medium of refractive index


( 1 1 greater than that of the lens.
YY

Ans. From — = (ji -1) (//) incident red light is replaced by violet
/ R
1 ^2
light. (CBSE 2017 (O)
nd
Re

taking /?, = /? and /?2 = - /? Ans. (/) Using standard notations, power of lens in
air
Fi

15 R
p
1 8
-1
\ i_
a
R.
f. 1
For each half (piano convex) lens Fj = F, /?2 = OO a a - /

when the lens is immersed in a medium


1 1 1 p-1 1
N / 1
1 1
R OO R 30 P -1
m
fm m
R, Rl
/'=:30cm
As lens is diverging, P^ is negative.
21. A lens shown in Fig. FIGURE 9(Q).12
9(Q).12 is made of two As |i,„ > )i^, therefore F,„ becomes positive.
different materials. A Hence power of the same lens immersed in
medium becomes positive, i.e., diverging lens
point object is placed on
behaves as convarging lens of smaller power.
the principal axis of the
lens. How many images (ii) When incident red light is replaced by violet
will be obtained ? light, wavelength decreases. Refractive index
increases (|i,, > ji^)- Power of the lens increases.
9/114
Fundamental Physics (XII)C2filSQ
25. A glass slab is placed over a page in which 33. What is Rayleigh’s criterion of .scattering ?
lettei^ are printed in different colours. Will Ans. When size of scatterer is much smaller than
the image of all the letters lie in the same wavelength of light, then intensity of scattered
plane ?
light varies inversely as fourth power of
Ans. As refractive index |x of glass for different
colours is different, therefore, images of wavelength of light, OC

different coloured letters will not be raised


equally and hence they will not lie in the same 34. What happens when size of scatterer is much
plane. bigger than the wavelength of light ?
26. In the above question, which letter will Ans. When size of scatterer is much bigger than
appear to be raised maximum ? wavelength of light. Rayleigh .scattering is not
Ans. As }iy > \x^ therefore violet letter images will valid. In that case, all wavelengths/colours are
be raised maximum. scattered equally.

ww
27. The intensity of light at a distance r from the 35. What is the essential condition for observing
axis of a long cylindrical source is inversely a rainbow ?
proportional to r. Comment. Ans. The observer must stand with his/her back

FF loo
Ans. True. This is because towards the sun.

ree
36. In a primary rainbow, what is the order of
e=L I
E « — colours ? And what is true for secondary
A Inrxl r
rainbow ?

rFee
28. What is the essential difference between
Ans. Outer arc of primary rainbow is red and inner

F
fluorescence and phosphorescence ?

oor r
arc is violet. In secondary rainbow, the order of
rur
Ans. Fluorescence is the phenomenon of emission of colours is just the reverse of colours of primary
s ff
energy in visible region by a substance on rainbow.
absorbing radiaiion.s of higher frequency. If the
37. Why are sodium lamps used for illumination
k
process of emission continues even after the
YYoou
in foggy conditions ?
ookos

exciting radiations arc stopped, the phenomenon


is called phosphorescence. Ans. Sodium lamps give yellow light which has larger
BBo

1
29. Why does a secondary rainbow have inverted
wavelength. As 1^ , it is scattered little
re

OC

colours ?

Ans. This is because in the formation of a secondary and produces sufficient illumination.
ouur
ad

rainbow. light enters from the bottom of the


Yo

drop, instead of entering from the top as in case III. Human eye
of primary rainbow and suffers two total internal 38. Where does a myopic eye focus the parallel
dY

reflections instead of one.


Re

rays falling on it ?
idn

30. Why does one prefer a black umbrella to a Ans. The.se rays are focussed in front of the retina.
FFin

white one, even in summer ?


39. Give one possible cause of hypermetropia.
Ans. A black umbrella provides a better shade than a
white umbrella. Absorption of more light energy Ans. Increase in focal length of eye lens when eye is
from the sun by the black umbrella does not fully relaxed.
matter because our body is not in touch with 40. What is the difference between hyper
the umbrella. metropia and presbyopia ?
31. Why are danger signals red in colour ? Ans. Hypermetropia is long sightedness. It may be
(HP Hoard 2U14) due to contraction in size of the eye ball or
Ans. As A.^is largest in the visible region, therefore, increase in focal length of eye lens, when fully
red colour is least scattered. Hence it can be seen relaxed. Presbyopia is long sightedness due to
from large distances. That is w'hy danger signals old age when eye ball has normal length. Only
are red. the focal length of eye lens in fully relaxed
32. What is meant by scattering of light ? position has increased.
Ans. Scattering of light represents basically the 41. A myopic person prefers to remove his
change in the direction of light. spectacles while reading a book. Why ?
RAY OPTICS AND OPTICAL INSTRUMENTS 9/115

Ans. A myopic person has to use spectacles with 45. Can a microscope function as a telescope by
concave lens. He may have normal near point inverting it. Can a telescope function as a
(s= 25 cm). To read with specs, he has to hold microscope ?
the book at a distance greater than 25 cm. As Ans. No, in a telescope, objective lens has much
angular size of object at a distance > 25 cm is larger focal length than the eye lens. In a micro
less than the angular size of object at 25 cm, scope, both the lenses have short focal lengths.
therefore, the person prefers to remove his 46. How does the magnification of a magnifying
spectacles while reading. glass differ from its magnifying power ?
size of image _ v
IV. Microscopes and Telescopes Ans. Magnification, ni =
sizeof object u
42. Through a simple microscope, an object is Magnifying power,
seen in red light first and then in violet light.
, _ Jingle subtended at eye by image

ww
In which case is magnifying power more ? m

angle subtended at eye by object


Ans. m = I + — . As /r > /y, therefore, magnifying when both the object and image are located at
f
the least distance of distinct vision (d) from the

Flo
power increa.ses when violet light is used.
43. The diameter of objective of a telescope is eye. m

e
u

rree
doubled. What is its effect on intensity of Clearly, m ^ m, unless v = d.
image seen ?

r FF
47. What is meant by range of a telescope ?
Ans. When D is doubled, area of cross section, Ans. Range of a telescope tells us how far away a
A = 7t D^/A becomes 4 times. Therefore, light star of some standard brightness can be spotted
uurr
gathering power and hence intensity of image
becomes 4 times.
48.
for
by the telescope.
Can we increase the range of a telescope by
kss
44. In the above question, how does the increasing the diameter of the objective lens ?
ooook

magnifying power change ? Ans. Yes, because objective with larger diameter will
Yo

Ans. As m =f(Jfe> therefore magnifying power does collect more light and even the distant objects
eBB

not change. can be seen.


urr

SHORT ANSWER QUESTIONS Carrying 3 marks


ad
Yo

1. Establish relation between/andR of a concave 7. A microscope is focussed on a dot at the bottom


dY

mirror. (MP Board 2011) of the beaker. Some oil is poured into the beaker
[Art. 9.6(a)] to a height of y cm and it is found necessary to
Re
innd

2. Establish relation between/and R of a convex raise the microscope through a vertical distance
mirror. [Art. 9.8] of X cm to bring the dot again into focus. Express
Fi

refractive index of oil in terms of .r and y.


3. Give some practical applications of spherical
mirrors. [Art. 9.11] [Art. 9.141
4. A ray of light suffers lateral displacement on 8. What is meant by power of a lens ? What is one
passing through a parallel sided glass slab. What dioptre ? [CBSE 2018 (C), HP Board 2015,
is the maximum possible value of lateral Hr. Board 2012, J & K Board 2011]
displacement ? [Art. 9.311
(Karnataka Board 2012) 9. How do magnification and focal length change
[Art. 9.13] for a combination of thin lenses ? [Art. 9.33]
1 10. What is a rainbow ? What is the essential
5. Prove that [i - , where C is the critical condition for observing it ? [Art. 9.44]
sin C
11. Define the term angular dispersion. Draw the
angle. (Hr. Board 2012, IIP .1 2011)
[Art. 9.17]
path of a ray of white light-passing through a
prism and mark angular dispersion on it.
6. What are optical fibres ? Give three applications
[Art. 9.40]
of these fibres. [Art. 9.19]
9/116
‘P’uKUe^ 4 Fundamental Physics (XII)
12. Why does sky look blue and clouds look white ? For a microscope, lens I3 is chosen as objective
'IIP Board 20i4, CUSE 2010) [Art. 9.42] because its focal length is smallest. Lens Lj
13. The sun looks reddish at the lime of sunrise and serves as eye piece because its focal length is
sunset. Why ? (HP Board 2015, CBSE 2010) not largest. This is required to secure greater
magnifying power of the microscope.
[Art. 9.42]
14. Violet colour is seen at the bottom of the
19. Four double convex lenses with following
specifications are available
.spectrum, when white light is dispersed by a Lens focal length aperture
prism. Explain. (CBSE 2010) [Art. 9.39]
A 100 cm 10 cm
15. What is the cause of dispersion ? B 100 cm 5 cm
(Jharkhand 2015) [Art. 9,39]
C 10 cm

ww
2 cm
16. Name the factors on which brightnes.s of D 5 cm 2 cm
image in a camera depends and how ?
Which of the given four lenses should be
Ans. Brightness of image in a camera is selected as objective and eyepiece to construct

Flo
(/) directly proportional to time of exposure (0, an astronomical telescope and why ? What

e
(ii) directly proportional to the square of will be the magnifying power and length of

eree
diameter (d) or aperture of the lens system, the tube of the telescope ?
(Kerala Board 2012)

FFr
(Hi) inversely proportional to the square of focal
length (/) of the lens system. Ans. The objective of a telescope must have large
uurr
apertureto gathermore light. It should also have

orr
17. Define magnifying power and resolving
power of a telescope.
(CBSE Sample Paper 2019-20)
sfo
large focal length . Therefore, lens A
4
kks
Yoo
Ans. Magnifying power of a telescope is defined as
oooo

is selected as objective lens.


the ratio of angle p subtended at the eye by the
The eye piece should have small aperture and
final image, as seen through the telescope to
eBB

small focal length. Therefore, lens D is selected


the angle a subtended at the eye by the object, as eye lens. In normal adjustment, Magnifying
as seen directly.
urr

Resolving power of a telescope is its power to /o _100 = 20


power, m =-p
ad

see separately, the images of two stars with small J£


YYo

angular separation. Resolving power of


^=/o+/g = 100 + 5 = 105 cm.
telescope is reciprocal of angular separation of
dd

20. From the data of four lenses given in Q. 19,


two stars, which can just be seen as separate
Re

which one will you select as objective of a


inn

through the telescope.


compound microscope and which one as eye
F

18. Using the data given below, state which two lens ? How can the magnifying power of such
of the given lenses will you prefer to construct a microscope be increased ?
a best possible (i) telescope (ii) microscope. Ans. The objective of a compound microscope should
Also, indicate which of the selected lenses is have small focal length and small aperture.
to be used as an objective and as an eye piece Therefore, lens D serves as the objective lens.
in each case. For larger magnifying power, focal length of eye
Lenses Power (P) Aperture (A) lens should be small (though greater than the
6D 1 cm focal length of objective lens). Therefore, lens
C is selected as eye lens.
^2 3D 8 cm

^3 10 D 1 cm M ^ (i d ]
Magnifying power, m = — 1 +—
(CBSE 2017, 2016, 2009) L0 fe
Ans. For a telescope, lens L2 is chosen as objective,
as its aperture is largest. The lens is chosen m = ~
L(,1 +—
25 IL

as eye piece as its focal length is smallest. 5 10 10


RAY OPTICS AND OPTICAL INSTRUMENTS 9/117

Clearly, m can be increased by increasing the / will be positive. The lens will behave as
length L of the microscope tube. converging lens.
21. State the condition under which a large 23. A ray of light incident on face AB of an
magnification can be achieved in an equilateral prism of glass shows minimum
astronomical telescope. (CBSE (F), 2017)
deviation of 30°. Calculate speed of light
Ans. Magnifying power of an asuonomical telescope through the prism.
IS
iCBSK Simipli-
-f,0 f Ans. Here, A = 60°, = 30°
M = 1+^
4 d
sin (A+S^^)/2 sin (60°+30°)/2
.●. Two conditions for large magnification are ; From, p =
sin A/2 sin 60°/2
(i) focal length of objective lens (Jq) should be
c sin 45° _ I/V2 _ 1 x2=V2

ww
large
4 = -
(ii) focal length of eye lens (^) should be small. V sin 30° ~ 1/2 ~ V2
22. A biconvex lens made of a transparent

Floo
c 3x10^
material of refractive Index 1*50 is immersed V - = 2-12x 10^ m/s
in water of refractive index 1 -33. Will the lens ^|2

ee
behave as converging or diverging lens? 24. An object is placed in front of a converging

eer
Justify your answer. lens. Obtain the conditions under which the
magnification produced by the lens is

FFr
1 1 1
(0 negative («) positive.

oorr
Ans. As, — = (p - 1)
uur r
/ R
1 (CBSE Sample Paper 2022-23)
- y
s ff
V
V 1 Ans. (/) Magnification of lens, ni = — . This must be
sk
-i
YYoo
II

M /?, /?,-
ooko

/ negative. As u is always negative, v must be


positive. Image formed will be real u >f
eBB

150 1 1
-1
—> positive
(ij) To make m positive, when u is negative, v
1-33 ; R.I R2 > must be negative. Image formed will be virtual.
uurr
ad
Yo

LONG ANSWER QUESTIONS Carrying 5 or more marks

1. Name two types of spherical mirrors. Define 6. Draw a ray diagram to show image formation
dY

pole, centre of curvature and angular aperture when the concave mirror produces a real,
Re

of the mirror.
nind

[Art. 9.3] inverted and magnified image of the object.


2. Define principal focal length and radius of (CBSK 20181 [Art. 9.7]
FFi

curvature of a mirror. Establish relation between


them for (/), concave mirror and (/7) convex 7. What is meant by linear magnification of
mirror. (MP Board 2011, HP Board 2011) spherical mirrors ? Deduce the fonnulae for the
same. (HP Board 201 \' [Art. 9.9]
[Arts. 9.5 & 6]
3. State and derive mirror formula for a concave 8. Establish relation between the speeds of object
mirror with the help of suitable ray diagram. and image fonned by a spheric :1 mirror.
State the sign conventionsused. [Art. 9.10]
(CBSE 2011) [Art. 9.7] 9. An object AB is kept in front of a concave mirror
4. Establish mirror formula in case of a convex as shown in Fig. 9(Q).14.
mirror. State the sign conventions used. (/) Complete the ray diagram showing the image
(CBSE Sample Paper 2019-20, formation of the object.
Jharkhand Board 2012) [Art. 9.8] (i7) How will the position and intensity of the
5. Obtain the mirror formula and write the image be affected if the lower half of the
expression for linear magnification. mirror’s reflecting surface is painted black ?
(CBSE 2018) [Arts. 9.8 and 9.9] (CBSK 2012) [Art. 9.7(a)]
9/118 Fundamental Physics (XlUromn

the focal length of a convex lens changes with


FIGURE 9(Q).14
A
increase in wavelength of incident light.
FIGURE 9(Q).15

^ P
B C F

DENSER H2
10. With the help of a ray diagram, show how a
concave mirror is used to obtain an erect and
(Ktr da Board 2012) [Art. 9.21]
magnified image of an object.
[CBSE 2018 (C)] [Art. 9.7(b)]
19. Discuss briefly refraction from rarer to denser

w
medium at a concave spherical refracting
11. Using the above ray diagram, obtain the mirror surface. [Art. 9.22]
formula and the expression for linear
20. Discuss refraction from denser to rarer medium

Flo
magnification. [CBSE 2018 (C)] [Art. 9.9]
at a convex spherical refracting surface.
12. What is meant by refraction of light ? State the
(Bihar 2015) [Art. 9.23]
laws of refraction. Show that emergent ray from

ee
21. Derive lens maker’s formula for a thin convex
a glass slab is parallel to incident ray. [Art 9.12]

Fr
lens. (Chhatisgarh Board 2012,
13. Explain the phenomenon of total internal
reflection. What are the conditions for the Bihar Board 2012, J & K Board 2012, 2011,
Jharkhand Board 2011) [Art. 9.25]
phenomenon ? Explain the meaning of critical

or
ur
22. Derive lens maker’s fonnula from ab-initio for
angle.
sf
(CBSE 2019, 2013, 2010, Bihar Board 2012,
a convex lens. (CBSE 2011) [Art. 9.27]
23. Derive lens formula for a convex lens.
Manipur Board 2011) [Art. 9.17]
ok
Yo
14. Show by drawing ray diagrams how a totally (CBSE Sample Paper 2019-20) [Art. 9.29]
reflecting glass prism can be used to deviate a 24. Draw a ray diagram to show the formation of
Bo

ray of light through (i) 90” (//) 180° and invert the image of an object placed between the
it. (CB.-' (r’), 2017) [Art 9.19] optical centre and focus of a convex lens.
re

15. Define total internal reflection. State its Deduce the relationship between object
conditions. How do optical fibres transmit light distance, image distance and focal length of
ou
ad

lens. [Art. 9.29]


without absorption.
25. Derive lens formula for a concave lens.
Y

(J & K Board .:014, A.P. Board 2012)


[Art. 9.29]
[Arts. 9.17 & 19] 26. Trace the rays of light showing the formation
nd
Re

16. Describe briefly any three applicationsof total of an image due to a point object placed on the
internal reflection. [Art. 9.19] axis of a spherical surface separating the two
Fi

media of refractive indices //| and /i^. Establish


17. Prove that when
V R
the relation between the distances of object,
u

refraction occurs from rarer to denser medium


image and radius of curvature from the central
point of the spherical surface. Hence derive the
at a concave spherical refracting surface.
expression for the lens maker’s formula.
(CBSE 2016, J & K Board 2014, Manipur
(CBSE 2009) [Arts. 9.22 and 9.25]
Board 2012, Chhutisgarh Board 2011)
[Art. 9.21]
27. Obtain an expression for focal length of a
combination of thin lenses in contact.
18. A spherical surface of radius of curvature R,
(Jharkhand 2015, Uttarakhand Board 2012)
separates a rarer and a denser medium as shown
[Art. 9.33]
in Fig. 9(Q).15. Complete the path of the
incident ray of light, showing the formation of 28. Discuss the phenomenon of refraction through
a real image. Hence derive the relation a prism. Provetiiat5=(p-)A where the symbols
connecting object distance u, image distance v, have their usual meaning.
radius of curvature R and the refractive indices (Karnataka Board 2012, Uttarakhand Board
n 1 and n-, of the two media. Briefly explain how 2012, Manipur Board 2011) [Art. 9.37]
RAY OPTICS AND OPTICAL INSTRUMENTS 9/119

29. State and prove prism formula. 39. Describe a simple microscope or a magnifying
(Raj. Board 2012, Kemla Board 2012, glass. Derive an expression for its magnifying
Manipur Board 2011, CBSE 2011) [Art. 9.37] power. [Art. 9.48]
30. Draw the ray diagram showing refraction of light 40. Draw course of rays through a compound
through a glass prism and hence obtain the microscope. Deduce an expression for its
relation between the refractive index p of the magnifying power. How can the magnifying
prism and angle of minimumdeviation. power be increased ?
(CBSE (F), 2017) [Art. 9.37] (J & K Board 2015, CBSE 2014, 2010,
31. Define refractive index of a transparent medium. Hr. Board 2014, Raj. Board 20111
A ray of light passes through a triangular prism.
[Art. 9.49]

ww
Plot a graph showing the variation of angle of
deviation with the angle of incidence. Hence 41. Derive an expression for the total magnification
derive prism formula. of a compound microscope. Explain why both
[CBSE 2017 (C), 2009] [Art. 9.37] the objective and the eye piece of a compound

Flo
microscope must have short focal lengths.
32. Explain the terms angular dispersion and

e
dispersive power. How arc the two related ? (CBSE 2013, 2009, 2008, Jharkhand 2012,

eree
[Arts. 9.40 & 41] Hr. Board 2012, J & K Board 2012,

FFr
33. Briefly explain the phenomenon of scattering Chhatisgarh Board 2011)
uurr
of light giving some examples. [Art. 9.42] [Art. 9.49]

orr
34. What is Raman Effect? Give some of the salient
features of this effect. [Art. 9.43]
sfo
42. Describe an astronomical telescope. Derive
expression for its magnifying power when final
35. What is a rainbow ? What are its two types ?
kks
image is
Yoo
How are they formed ? Discuss briefly.
oooo

(/) at infinity
(Chhatisgarh Board 2012, J & K Board 2012)
eBB

(CBSE 2017, 2016, 2010, 2009, 2008)


[Art. 9.44]
(;7) at least distance of distinct vision.
36. Draw a labelled sketch of the eye and describe
urr

its important parts. [Art. 9.46] (CBSE 2019, HP Board 2013, Raj. Board 2012,
ad

37. Name common optical defects of eye. How are Manipur 2015, Hr. Board 2012, 2011)
YYo

they removed ? [Art. 9.47] [Art. 9.50]


38. Explain what is meant by myopia and 43. Describe a reflecting type telescope. What are
dd

hypermetropia. How are they caused ? Briefly


Re

its advantages over the refracting telescope ?


inn

explain their removal.


(CBSE 2018, 2017 (F), 2012, 2009,
F

(CBSE 2013, Jharkhand Board 2011)


MP Board 2011) [Art 9.51]
[Art. 9.47]

CASE-BASED VERY SHORT/SHORT QUESTIONS

CASE 1. Reflection of light is the phenomenon u = distance of object and v is distance of the
of change in the path of light v> thout any change in image from the pole of the mirror. / is focal length of
medium. A plane mirror and a .splicrical mirror are used the mirror and R is radius of curvature of the mirror
for reflection of light. Spherical mirrors are of two New carterian sign conventions are used in all cases.
types : concave mirror and convex mirror. Mirror Linear magnification (m) of a spherical mirror
formula for both the mirrors is the same, i.e.,
m =
size of image (ky)
^ =
V
if-v)_ f
1 1
—, where size of object (/?j) -u f (/-«)
f u V R
9/120 “Pniidee^ Fundamental Physics (XII)

Read the above paragraph carefully and The lens maker's formula for both, convex and
answer the following very short and short concave lenses is

answer questions :
1 1
1. Describe the nature of image of a person standing - = (p-l) —-
before the plane mirror at some distance. / R
I - /

2. Which front mirror do you prefer above the seat of where Rj, /?2 are the radii of curvature of the two
motor car driver and why ? surfaces of the lens, and p is refractive index of the
3. A concave mirror forms the real image of an object material of the lens w.r.t. the medium in whieh lens is
which is magnified 4 times. The object is moved placed.
3 m away, the magnification of image is 3 times. The relation governing u, v and/in case of both
What is focal length of the mirror ?
1 1 I

ww
4. Name two applications, each of convex mirror and the lenses is — =
concave mirror.
/ V u

This is called lens formula.

FF loo
CASE 2. Refraction of light is the phenomenon
Linear magnification (w) produced by a lens
of change in the path of light in going from one medium

ree
to another. As light travels faster in a rarer medium than size of image (/j2) v
in a denser medium, therefore, in going from a rarer to ni -

size of object (/i|) u

rFee
a denser medium, light bends towards normal and vice-
versa. On the same basis, a tank of water appears to be

F
For convex lens, m is positive only when image

oor r
rur
less deep than what it actually is. is virtual and erect. For concave lens, m is always
s ff
positive.
real depth
p = Power of a lens is reciprocal of focal length of
k
apparent depth
YYoou
the lens.
ookos

We can show that


1
BBo

P = —
1
/
^ =
re

a
when / - 1 meter. P = 1 diopter
ouur
ad

For a combination of two lenses in contact,


Yo

and
_L-_L _L ●
Total internal reflection is the phenomenon by
dY
Re

virtue of which a ray of light while going from denser


idn

medium to rarer medium reflects back, into denser P-Pl + Pl


FFin

medium from the boundary of denser medium with rarer and m = /«] X
medium. Two essential conditions for TIR are :
Read the above paragraph carefully and
(i) Light should travel from denser to rarer answer the following very short and short
medium, answer questions :
(h) Angle of incidence in denser medium should 5. Critical angle for a pair of media in contact is
be greater than critical angle C for the pair of media in 45®. What is the refractive index of denser
contact. We can show that medium ?

1 6. What are the essential conditions for total internal


p = reflection of light ?
sinC
7. What is power of a lens of focal length 25 cm ?
C will depend on colour of light.
8. A convex lens of focal length 2 m is held in
A lens is bound by two spherical surfaces.
contact with a concave lens of focal length 4 m.
Therefore, a ray of light suffers two refractions on
What are focal length and power of the
passing through the lens. combination ?
RAY OPTICS AND OPTICAL INSTRUMENTS 9/121

CASE 3. In passing through a prism, a ray of light The blue colour of sky is due to scattering of sun
suffers two refractions. The net deviation (6) suffered light from molecules of earth’s atmosphere. Intensity
by the ray is 5 = ()i - 1) A, when A is small,
of scattered light varies inversely as fourth power of
when, A is large, 5 = /. + - A ; rj + r2 = A wavelength. Therefore, maximum scattering is of blue
when. /| = i2 = i and rj = r2 = r, we get colour.

sin/ sin(A + 8^)/2 A rainbow is spectrum of sunlight through rain


\x = = ^—
sin r sin A/2 drops suspended in air. It is seen in the sky usually after
This is called prism formula. rain, when back of observer is towards the sun.

Dispersion of light is phenomenon of splitting Read the above paragraph carefully and
of white right into its constituent colours on passing answer the following very short and short
through a prism. Different colours having different answer questions :
wavelengths deviate through different angles on passing

w
9. The angle of prism is 5° and its refractive indices
through a prism. Angular dispersion produced by a prism for red and violet colours are 1-5 and 1-6
= 6„ - 5, = A
respectively. Find the angular dispersion

Flo
Dispersive power of prism,
produced by the prism.
angular dispersion
(0 = 10. What is the basic cause of dispersion of light ?

ee
mean deviation
11. What is the condition for observing a rainbow ?

Fr
(0 = 12. Why does clear sky look blue ?
S (p-l)A p-1

for
ur
ANSWERS
ks
1. The image of the person formed due to reflection
Yo
--- + -
of light from the plane mirror is virtual, erect and
oo

f u V

of the same size as that of the object. It is fonned


eB

as far behind the mirror as object is in front of it 1 1 _ 4-1 _ 3 _ 1


and there is a lateral inversion in the image of the 9 36 " 36 " 36 ~ 12
r

person.
/= 12 m
ou
ad

2. A convex mirror is used as a front mirror above


4, A convex mirror is used as a reflection in street
the seat of motor car driver so that the driver may
Y

lamps and also as a driver’s mirror. A concave


look the vehicles approaching him from behind and
mirror is used as a reflection in search lights,
nd

may also estimate the distance of approaching


Re

vehicle behind him. tele.scopes. solar cookers etc.


Fi

1
3. From
V
m = — ; 4 =
V
5. ft = = V2 = 1-414
sinC 810 45®
II — u

6. The essential conditions for total internal


V refleciion of light are as follows ;
3 =
-(«+3) (/) Light should travel from denser to rarer
medium.
4 u+3
Divide : — (//) Angle of incidence in denser medium should
3 u
be greater than the critical angle for the pair of
or 4u = 3 u + 9 media in contact.

u = 9 1
7. P = - = 4 diopter.
v=-4u=-4x9=-36 f 25/100
9/122 Fundamental Physics (XlI)BZs29D

8. /j = 2 m and/2 = -4m, F=?, P=? 10. Different colours have different wavelengths.

1 1 I 11. Back of observer must be towards the sun.

F A 2~4~ 4' 12. Blue colour of sky is due to scattering of sunlight


from molecules of v urth’s atmosphere. Since the
1 1
F = 4 m, P = — — diopter intensity of scattered light varies inversely as
F
fourth power of wavelength and blue colour is
9. Here, A = 5°, p,. = 1 -5, = 1 -6, of lower wavelength than other prominent
Angular dispersion = (5,, - 8,.) = ((ij, - ji^ A colours. As a result of it, maximum scattering of
blue colour takes place.
= (1-6- l-5)x5 = 0-5"

w
CASE-BASED MCQs AND ASSERTION-REASON QUESTIONS

CASE 1. Human eye is far more delicate and Based on the above paragraph, answer

Flo
perfect than even the finest photographic camera. For a questions no. 1 to 4 :
normal eye, least distance of distinct vision is 25 cm

e
1. A person has visual problem in driving a car. He

ree
and far point F lies at infinity. This is called is suffering from
accommodating power of the eye. Two major defects

FFr
{a) Myopia
of vision are ;
(b) Hypermetropia
urr
orr
1. Myopia or short sightedness
(c) Either (a) or (b)
2. Hypermetropia or long sightedness
sf {d) Both (a) and (fc)
A myopic person can see clearly the objects lying
kks
2. A child cannot read the book in a class. He is
near it only. To correct a myopic eye the person has to
Yo
ooo

suffering from
use spectacles witli a concave lens of focal length/= - x,
where .x is distance of far point of myopic eye. (a) Myopia
eB

(b) Hypermetropia
A hypermetropic person can see clearly only the
far off objects. To correct a hypermetropic eye, the (c) Either (a) or (b)
ur

person has to use spectacles with a convex lens of focal {d) Both (fl) and (b)
ad
YYo

length/, where For Question No. 3 and 4, we have given two


1 1 1 1 I .
statements each, one labelled as Assertion (A)
+ —, where and other labelled as Reason (R).
dd

f -x' d
Re

V u

Choose the correct option out of the four options


in

ii = -d = least distance of distinct vision of normal eye given below:


F

V = -x', (a) Both A and R are true and R is correct


where x' = distance of near point of defective eye. explanation of A.
(b) Both A and R are true and R is not the correct
A simple microscope is used for observing
magnified images of tiny objects. It consists of a convex explanation of A.
lens of small focal length (/). The object is held between (c) A is true but R is false.
principal focus and optical centre of the lens. The image {d) Both A and R are false.
formed is virtual, erect and magnified. The magnifying 3. Assertion. All human eyes have some
power {ni) of a simple microscope is accommodating power.
d Reason. Every person can see clearly objects
m ~
lying at different distances.
For different persons, the distances may be
where d is least distance of distinct vision from the eye. different.
RAY OPTICS AND OPTICAL INSTRUMENTS 9/123

4. Assertion. The magnifying power of a simple V


microscope using a convex lens of focal length (C)
0

5 cm is 6. —u
0 1. fe
cl ^
Reason, m = 1H— . u
0
(d)
. f V
n 4J
CASE 2. In a compound microscope, the images 6. The magnifying power of an astronomical
are highly magnified. The objective lens of microscope telescope is
forms a real, inverted and magnified, image of the
object. This image serves as the object for eye lens, (a) m =
fe
which forms a virtual, erect and magnified image as fo

ww
seen by the eye held close to the eye lens. Magnifying
power of a compound microscope is given by Wtn=/o-/c
V
●^0

Floo
0
m — , where Ur, and Vr,0 are distances of (c) m =
— u
n f.j '41

ree
object and image from optical centre of objective lens (cf) none of these
and fg is focal length of eye lens.

rFee
For Question No. 7 and 8, we have given two
An astronomical telescope is used for observing statements each, one labelled as Assertion (A)

F
heavenly bodies like stars, and planets etc. The objective
oor r
rur
and other labelled as Reason (R).
lens forms a real, inverted and smaller image of distant
s ff
Choose the correct option out of the four options
object in its focal plane. This image serves as an object
given below:
for eye lens, which forms a virtual, erect and magnified
k
YYoou
(a) Both A and R are true and R is correct
image .seen by the eye held close to the eye lens. In
ookos

explanation of A.
normal adjustment, final image as seen by the eye is
BBo

at infinity. The magnifying power of astronomical (b) Both A and R are tme but R is not a correct
telescope is given by explanation of A.
re

(c) A is true, but R is false.


f0 _ fo
ouur
ad

m =
(d) Both A and R are false.
-4 'fe'
Yo

7. Assertion. A compound microscope can be used


Based on the above paragraph, answer for viewing far off objects.
dY

questions no. 5 to 8 :
Re

Reason. An astronomical telescope can be used


idn

5. The magnifying power of a compound for viewing tiny objects.


FFin

microscope is
8. Assertion. Magnifying power of an astronomical
V
(«)
0
fo
telescope is m =
-u
0 fe '41
Reason. In normal adjustment of astronomical
ib) telescope, final image is seen by the eye is at
u
0 4 infinity.

ANSWERS

1. («) 2.{b) 3. (a) 4. (a) 5. (a) 6. (c) 7. 8. (b)


9/124 ^'taxUcfi U Fundamental Physics (XII)

HINTS/EXPLANATIONS For Difficult Questions

1. Myopia as he can not see the distant objects Both, the Assertion and Reason are true and
clearly. Reason is correct explanation of the Assertion.

2. Hypennetropia because he can not see the nearer 5. M =


V
0

objects clearly. -H
0 4
3. Both, the Assertion and Reason are true, and
0
Reason is correct explanation of the Assertion. 6. m =

4. For a simple microscope,


'4'
7. Both, A and R are false.
d 25
m = l+ — = 1 + — = 6. 8. Both, A and R are true, but R is not a correct

ww
I 5 explanation of A.

P ISQ'PiLEiiSiiriS [F®[^

Flo
e
eree
TYPE I. REFLECTION OF LKJHT of object A is four times that of B. If object A is
placed at a distance of 50 cm from the mirror,

FFr
1. Light of wavelength 6000 A falls on a plane what should be the distance of B from the

oorr
reflecting surface. What are the wavelength and [Ans. - 20 cm]
uur r
mirror ?
frequency of reflected light. If angle between 7. An object of size lO cm is placed at a distance
sf
incident ray and reflected ray is 60°, what is the of 50 cm from a concave mirror of focal length
angle of incidence ? 15 cm. Calculate location, size and nature of
Yoo
ks
[Ans. 6000 A, 5 X lO^^* Hz ; 30°] the image. [Ans. - 21*4 cm, - 4*28 cm ; Real]
oook

2. A ray of light is incident at an angle of 60° on a 8. An object 2 cm high is placed at a distance of


eBB

horizontal plane mirror. Through what angle 16 cm from a concave mirror, which produces
should the mirror be tilted to make the reflected a real image 3 cm high. What is the focal length
ray horizontal ? [Ans. 15°] of the mirror ? Find the position of the image ?
uurr

3. A point object is held between two plane mirrors [Ans./- -9*6 cm, ti = - 24 cm]
ad
Yo

inclined at 45°. What is the number of images 9. An object is placed in front of a concave mirror
seen ? [Ans. 7] of focal length 20 cm. The image formed is three
dY

4. A 12 m tall tree is to be photographed with a times the size of the object. Calculate two
Re

pin hole camera. It is situated 15 m away from possible distances of the object from the mirror.
innd

the pin hole. How far should the screen be 40 80


FFi

placed from the pin hole to obtain a 12 cm tall U & K Board 2014) [Ans. "y and "y]
image of the tree ? [Ans. 15 cm]
10. A square object is placed 15 cm from a convex
mirror of radius of curvature 90 cm. Calculate
i y peh. reflection
the position of the image and its areal
AT SPHERICAL MIRRORS
magnification.
5. An object is held in front of a concave mirror [Ans. 11.25 cm behind the mirror : 9/16]
of focal length 15 cm. The image formed is 3 11. The image formed by a convex mirror of focal
times the size of the object. Calculate two length 30 cm. is a quarter of the object. What is
possible distances of the object from the mirror. the distance of the object from the mirror ?
(CBSE Sample Paper 2007) [Ans. - 90 cm]
[Ans. - 10 cm ; - 20 cm] 12. Calculate the distance of an object of height h
6. Two objects A and B, when placed one after from a concave mirror of focal length 10 cm so
another in front of a concave mirror of focal as to obtain a real image of magnification 2.
length 10 cm, form images of same size. Size (CBSE 2008) [Ans. - 15 cm]
RAY OPTICS AND OPTICAL INSTRUMENTS 9/125

13. An object is placed in front of a concave mirror 19. A point source of monochromatic light ‘S' is kept
of radius of curvature 40 cm at a distance of at the centre of the bottom of a cylinder of radius
10 cm. Find the position, nature and magni 15-0 cm. The cylinder contains water (refractive
fication of the image. (Pb. Board 2013) index 4/3) to a height of 7-0 cm. Draw the ray
[Ans. 20 cm, virtual, erect behind diagram and calculate the area of water surface
the mirror, m = + 2] through which the light emerges in air.
(CBSE 2015J [Ans. 198-0 cm^]
TYPE III. LAWS OF REFRACTION,
20. A right prism is to be made by selecting a proper
LATERAL SHIFT, REFRACTION
material and the angles A and B(B<A), as shown
THROUGH A COMPOUND PLATE,
in Fig. 9(Q).17. It is desired that a ray of light
REAL AND APPARENT DEPTHS
incident normally on AB emerges parallel to the
14. A mark is made on the bottom of a beaker and a incident direction after two internal reflections.

ww
microscope is focussed on it. The microscope is {a) What should be the minimum refractive index
raised through 1 -5 cm. To what height water must }i for this to be possible ? {b) For ^ = 5/3, is it
be poured into the beaker to bring the mark again possible to achieve this with the angle A equal to
into focus ? Given that |i for water is 4/3. 60 degrees ? [Ans. ^ ; No]

Flo
[An.s. 6 cm]

e
15. Calculate the speed and wavelength of light

rere
{/) in glass (//) in air. when light waves of

r FF
frequency 6 x 10 Hz. travel from air to glass of
\i=\-5. [Ans. (/) 2 X 10® m/s ; 3-3 x 10“^ m
uurr
(«) 3 X 10® m/s ; 5 x 10“^ m]
16. A ray of light is incident at an angle of 60° on
one face of a rectangular glass slab of thickness
for
kss
01 m, and refractive index 1-5. Calculate the
ooook
Yo

lateral shift produced. [Ans. 0-0513 m]


21. In Fig. 9(Q). 18, light rays of blue, green and red
eBB

TYPE IV. TOTAL wavelengths arc incident on an isosceles right


INTERNAL REFLECTION angled prism. Explain with reason which ray of
light will be transmitted through the face AC. The
urr

17. In Fig. 9(Q).16, find the maximum angle i for


ad

which light suffers total internal reflection at the


refractive index of the prism for red, green and
Yo

vertical surface. blue light are 1-39, 1-424 and 1-476 respectively.
[Ans. 48-6"]
dY

(CBSE 2009, 2008)


Re
innd
Fi

18. A small bulb (assumed to be a point source) is


placed at the bottom of a tank containing water
to a depth of 80 cm. Find out the area of the [Ans. Red rays will be transmitted]
surface of water through which light from the 22. Velocity of light in a liquid is 1-5 x 10® m/s and
bulb can emerge. Take the value of refractive in air, it is 3 x 10® m/s. If a ray of light passes
index of water to be 4/3. (CBSE 2013) from this liquid to air, calculate the value of
[Ans. 25837-7 sq. cm] critical angle. [HP Board 2010 (C)] [Ans. 30°]
9/126 ’a Fundamental Physics (Xll)tviwii
illuminated filament at a distance of 6 cm.
TYPEV. REFRACTION
Calculate the radii of curvature of its surfaces.
AT SPHERICAL SURFACES
[Ans. 4*5 cm ; - 9-0 cm]
23. A small air bubble in a glass sphere of radius 30. A converging lens has a focal length of 20 cm in
2 cm appears to be 1 cm from the surface when air. It is made of a material of refractive index
looked at, along a diameter. If the refractive index 1 -6. If it is immersed in a liquid of refractive index
of glass is 1-5, find the true position of the air 1-3, what will be its new focal length ?
bubble. [Ans. 1-2 cm] (CBSE2011) [Ans. 52 cm]
31. The radii of curvature of each surface of a convex
lens is 20 cm and the refractive index of the
material of the lens is 3/2 (0 Calculate its focal
length (//) If this is cut along the plane AB. What
will be the focal length of the each of the two
halves so formed ? (Hi) What happens if the lens

w
is cut along CD ? [Ans. 20 cm ; 40 cm ; 20 cm]
32. A convex lens made up of glass of refractive
index 1 -5 is dipped in turn

Flo
(i) in a medium of refractive index 1-65

e
24. An object is placed 50 cm from the surface of a (i7) in a medium of refractive index 1-33.

rree
glass sphere of radius 10 cm along the diameter. (a) Will it behave as converging or diverging lens

r FF
Where will the final image be formed after in the two cases ?
refractionat both the surfaces ? of glass = 1-5. (b) How will its focal lengtli change in the two
uurr
media ? (CBSE 2011)
[Ans. 20 cm]
25. A convex refracting surface of radius of curvature
20 cm separates two media of refractive indices
for
[Ans. (a) diverging lens in medium of ref. index 1-65;
converging lens in medium of ref. index I *33
kss
4/3 and 1 -60. An object is placed in the first (*)/,=:-5-5/,/2 = 3-9/]
ooook
Yo
medium (p = 4/3) at a distance of 200 cm from 33. A converging lens of refractive index 1-5 and of
the refracting surface. Calculate the position of focal length 15 cm in air, has the same radii of
eB

image formed. (CBSE 2016) curvature for both sides. If it is immersed in a


[Ans. At 234*15 cm in denser medium] liquid of refractive index 17, find the focal
26. A sphere of glass (p = 1 -5) is of 20 cm diameter. length of the lens in the liquid.
urr

A parallel beam enters it from one side. Where (CBSE 2008) [Ans. - 63-75 cm]
ad
Yo

will it get focussed on the other side ? 34. The radii of curvature of the surfaces of a double
[Ans. 5 cm] convex lens are 20 cm and 30 cm. What will be
dY

27. A beam of light strikes a glass sphere of diameter its focal length and power in air and water
Re

respectively ? Refractive indices for glass and


innd

15 cm converging towards a point 30 cm behind


the pole of the spherical surface. Find the position water are 3/2 and 4/3 respectively.
Fi

of the image, if p of glass is 1-5. (Uttarakhand Board 2012)


[Ans. 15 cm from the pole] [Ans. 24 cm, 4-17 D ; 96 cm, 1-04 D]
28. One end of a horizontal cylindrical glass rod 35. A convex lens made up of glass of refractive
(p = 1 -5) of radius 5 cm is rounded in the shape index 1-5 is dipped, in turn in (t) medium A of
of a hemisphere. An object 0-5 mm high is placed refractive index 1 -65, («) medium B of refractive
perpendicular to the axis of the rod at a distance index 1-33. Explain giving reasons, whether, it
of 20 cm from the rounded edge. Locate the will behave as a converging lens or a diverging
image of the object and find its height. lens in each of these media. (CBSE 2006)
[Ans. 30 cm ; 0-5 mm] [Ans. (0 diverging («) converging]
36. A double convex lens of glass of refractive index
TYPE VI. LENS MAKER’S 1-6 has its both surfaces of equal radii of
FORMULA curvature of30 cm each. An object of heights cm
is placed at a distance of 12-5 cm from the lens.
29. The radii of curvature of a double convex lens of
Calculate the size of the image formed.
glass (p = 1-5) are in the ratio of 1 : 2. This lens (CBSE 2007) [Ans. 10 cm]
renders the rays parallel coming from an
RAY OPTICS AND OPTICAL INSTRUMENTS 9/127

37, Convex lens is made of glass of refractive index 44. A convex lens is used to throw on a screen 10 rn
I -5. If the radius of curvature of each of the two from the lens, a magnified image of an object. If
surfaces is 20 cm, find the ratio of the powers of the magnification is to be 19, find the focal length
the lens, when placed in air to its power, when of the lens. (CBSE2010) {Ans. 0-5 in]
immersed in a liquid of refractive index 1-25. 45. A screen is placed 80 cm from an object. The
[Ans. 5/21 image of the object on the screen is formed by a
38. A convex lens of focal length 20 cm and made convex lens at two different locations separated
of glass (p = 1-5) is immersed in water of p = by 10 cm. Calculate the focal length of the lens
used. fAns. 19’7 cm]
1 -33. Calculate change in focal length of the lens.
46. The image obtained with a convex lens is erect
(CBSE 2011) [Ans. 58.23 cm]
and its length is four times the length of the
39. A thin converging lens made of glass of refractive
object. If the focal length of the lens is 20 cm.
index 1-5 acts as a concave lens of focal length Calculate the object and image distances.

w
50 cm, when immersed in a liquid of refractive
[Ans. - 15 cm, - 60 cm)
index 15/8. Calculate the focal length of
converging lens in air. 47. An illuminated object and a screen are placed
(Pb. Board 2010)
90 cm apart. Determine the focal length and

Flo
[Ans. 20 cm]
nature of the lens required to produce a clear

e
40. Find the radius of curvature of the convex surface
image on the screen twice the size of the object.

ree
of a piano convex lens, whose focal length is (CBSE 2010) [ Ans. 20 cm, convex lens]

FFr
0-3 m and refractive index of the material of the
48. The radius of curvature of the faces of a double
lens is 1-5. (CBSE 2010) [Ans. 0-15 m]
convex lens are 10 cm and 15 cm. If focal length
urr
41. A converging lens has a focal length of 20 cm in
for
of lens is 12 cm, find the refractive index of the
air. It is made of a material of refractive index material of the lens. (CBSE 2010) [Ans. 1*5]
1 -6. If it is immersed in a liquid of refractive index
kkss
2
1-3, what will be its new focal length ? 49. A biconvex lens has a focal length — times the
Yo
ooo

(CBSE 2017 (F), 2011) radius of curvature of either surface. Calculate


[Ans. 52 cm] the refractive index of the lens material.
eB

42. A converging lens of refractive index 1-5 and


(CBSE 201II [Ans. 1*75]
focal length 15 cm in air has same radii of
curvature for both sides. It is immersed in a liquid TYPE VIII. COMBINATION OF
ur

of refractive index 1 -7, calculate the focal length


ad

LENSES
YYo

of the lens in liquid. (CBSE 2008)


[Ans. - 63-75 cm]
50. Find the focal length and power of a convex lens,
which when placed in contact with a concave lens
d

of focal length 25 cm forms a real image 5 times


Re

TYPE VII. LENS FORMULA, the size of the object placed 20 cm from the
in

LINEAR MAGNIFICATION combination. [Ans. 10 cm, 10 D]


F

AND POWER OF LENS


51. Find the focal length and nature of lens which
should be placed in contact with a lens of focal
43. From the ray diagram shown in Fig. 9(Q).20,
calculate the focal length of concave lens. length 10 cm so that the power of the combination
becomes 5 dioptre, [Ans. - 20 cm. ; concave]
FIGURE 9(Q),20 52. Two lenses, one diverging of power 2 dioptre and
/= 20 cm the other converging of power 6 dioptre are
combined together. Calculate focal length and
power of the combination. (Bihar 2015)

>
[Ans. 25 cm, 4D]
-► I'
c 53. Two lenses of power + 10 Z) and -5 D are placed

r in contact, (0 Calculate the focal lengtli of the


combination (tV) where should an object be held
60 cm 10 cm ^ from the combination so as to obtain a virtual
image of magnification 2 ? (CBSE 2008)
[Ans. - 20 cm] [Ans. (/) 20 cm (ii) - 10 cm]
9/128 “Pn<ieteefi- ^ Fundamental Physics (XII)

TYPE IX. COMBINATION


60. Show that the angle of deviation produced by a
thin prism is reduced to one fourth (w.r.l. air)
OF A LENS AND A MIRROR
when it is immersed in water. Given "fig = 3/2
and = 4/3.
54. A point object is placed 60 cm in front of a convex
lens of focal length 15 cm. A plane mirror is 61. The refractive index of the material of a prism of
placed 10 cm behind the convex lens. Where is 60° angle for yellow light is ^ . Calculate angle
the image formed by the system ? of minimum deviation, angle of incidence and
(NCERT Exercise) angle of refraction. [Ans. 30°, 45°, 30°]
[Ans. At optical centre of convex lens] 62. A glass prism whose refractive index is 1 -53 and
55. A convex lens of focal length 15 cm, and a refracting angle is 60° is held in a liquid of
concave mirror of radius of curvature 20 cm are refractive index 1-33. Calculate the angle of
placed co-axially 10 cm apart. An object is placed minimum deviation in this case.

ww
in front of convex lens so that there is no parallax [Ans. 10° 12']
between the object and its image formed by the
63. A ray of light is inclined to one face of the prism
combination. Find the position of the object.
at an angle of 60°. If angle of prism is 60° and

FF loo
(NCERT Exercise) the ray is deviated through an angle of 42°, find
[Ans. At 30 cm from lens] the angle which the emergent ray makes with the

ree
56. Fig. 9(Q).21 shows a plane mirror M placed at a second face of the prism. [Ans. 18°]
distance of 10 cm from a concave lens L. A point 64. A glass prism has a refracting angle of 60°. The

rFee
object is placed at a distance of 60 cm from the angle of minimum deviation is 40°. Find the
lens. The image formed due to refraction by the

F
refractiveindex, At what angle should the ray be

oor r
rur
lens and reflection by the mirror is 30 cm behind incident so as to suffer minimum deviation ?
the mirror. What is the focal length of this lens ?
s ff
[Ans. 1*532, 50°]
FIGURE 9(Q).21 65. A glass prism of angle 72° and refractive index
k
YYoou
M
1 -66 is immersed in a liquid of p = 1 -33. Calculate
ookos

the angle of minimum deviation. [Ans. 22*38°]


BBo

TYPE XI. DISPERSION


re

THROUGH A PRISM
ouur
ad

66. The refractive indices of a prism for red, violet


Yo

10 cm ●H and yellow lights are 1-52, 1-62 and 1-59 resp.


What is the dispersive power of the prism ? If
(NCERT Exercise) [Ans. - 30 cm]
mean deviation is 40°. What is angulardispersion
dY
Re

produced by the prism ? [Ans. 0*169 ; 6*76°]


idn

TYPE X. REFRACTION
67. Find the angle of flint glass prism which produces
FFin

THROUGH A PRISM
the same angular dispersion for c and F
57. Calculate the refractive index of the material of wavelengths in 10° crown glass prism.
an equilateral prism for which angle of minimum For crown glass : |i/:-= 1-5230. = 1-5145
deviation is 60°. (CBSE 2001) [Ans. -J3] For flint glass : |i'^ = 1-6637 , p'. = 1-6444 .
58. A ray of light suffers minimum deviation, while [Ans. 4*4°]
passing through a prism of refractive index 1-5 68. The refractive indices of crown and flint glasses
and refracting angle 60°. Calculate the angle of for violet and red light are 1-523, 1-513, 1-773
deviation and angle of incidence. and 1-743 respectively. Find the dispersive
[Ans. 37*2", 48*6°] powers of the glasses. [Ans. 0*019, 0*04]
59. A glass prism has a refracting angle of 60°. The 69. The minimum deviations suffered by red, yellow
angle of minimum deviation is 40°. If velocity of and violet beams passing through an equilateral
light in vacuum is 3 x 10^ m/s. Calculate the transparent prism are 38-4°, 38-7° and 39-2°
velocity of light in glass. What is the angle of respectively. Calculate the dispersive power of
incidence ? [Ans. 1*958 x 10* m/s, 50°] the medium. [Ans. 0*018]
RAY OPTICS AND OPTICAL INSTRUMENTS 9/129

70. Determine the angle of flint glass prism, which the final image seen by the eye is 25 cm from the
should be combined with a crown glass prism of eye piece. What is the magnifying power ?
5° so as to give dispersion, but no deviation. Given (CBSE 2008) {Ans. «o = “ cm, m - 41-5]
for crown glass, = 1-523, = 1-515. For flint 79. The focal lengths of the eye piece and objective
glass, 1-688, [i;= 1-650. [Ans. - 3-88°l of a compound microscope are 5 cm and I cm
71. Calculatethe angle of a prism of dispersivepower respectively, and the length of the tube is 20 cm.
0-021 and refractive index 1-53 to form an Calculate magnifying power of microscope when
achromatic combination with prism of angle 4-2°. the final image is formed at infinity. The least
and dispersive power 0-045, having refractive distance of distinct vision is 25 cm. {Ans. 70]
index 1-65. Find also the net deviation. 80. A compound microscope is made using a lens of
[Ans. - 11-04°, - 3-12°] focal length 10 mm as objective and another lens
of focal length 15 mm as eye piece. An object is

ww
TYPE XII. THE DEFECTS OF held at M cm from the objective and final image
is obtained at oo. Calculate distance between
VISION
objective and eye piece. [Ans. 12-5 cm]
81. A compound microscope uses an objective lens

Flo
72. The far point of a myopic person is 150 cm in
of focal length 4 cm and eye lens of focal length

e
front of the eye. Calculate the focal length and
10 cm. An object is placed at 6 cm from the

ree
power of the lens required to enable him to see
distant objects clearIy.[Ans. - 1-5 m, - 0-67 D] objective lens. Calculate the magnifying power

FFr
of compound microscope. Also, calculate the
73. A short sighted person is wearing specs of power
length of microscope.
- 3-5 D. His doctor prescribes a correction of
uurr
orr
+ 2-5 D for his near vision. What is focal length (CBSE 2011) [Ans. 7 ; 22 cm]
of his distance viewing part and near vision part ?
[An.s. - 28-5 cm ; 16-7 cm]
sfo
82. The total magnification produced by a compound
microscope is 20, while that produced by eye
kks
74. A person cannot see the objects distinctly, when piece alone is 5. When the microscope is focussed
Yo
on a certain object, the distance between
oooo

placed at a distance less than 100 cm. What is


the power of the spectacles that he should use to objective and eye piece is 14 cm. If distance of
distinct vision is 20 cm. what are the focal lengths
eB

see clearly, the objects placed at 25 cm ?


[Ans. 3 D]
of objective and eye piece ? [Ans. 2 cm ; 5 cm]

75. The distance of distinct vision of a person is TYPE XIV. TELESCOPE


ur

50 cm. He wants to read a book placed at 25 cm.


ad

What should be the focal length of the 83. The magnifying power of an astronomical
YYo

spectacles ? (CBSE 2009) [Aas.+ 50cm] telescope in the normal adjustment position is
76. A person has normal for point (infinity) and 100. The distance between the objective and eye
dd

normal near point (25 cm). He intends to read a piece is 101 cm. Calculate the focal lengths of
Re
inn

book using a magnifying glass of/= 5 cm. What objective and eye piece. (CBSE 2004)
is the (/) Closest and (ii) farthest distance at which [Ans. 100 cm and 1 cm]
F

he can read the book through the magnifying 84. A refracting telescope has an objective of focal
glass. [Ans. - 4-17 cm ; - 5 cm] length I m and an eye piece of focal length
20 cm. The final image of the sun 10 cm in
TYPE XIII. SIMPLE MICROSCOPE diameter is formed at a distance of 24 cm from
AND COMPOUND MICROSCOPE eye piece. What angle does the sun subtend at
the objective ? [Ans. 0 0455 rad]
77. An object is to be seen through a simple 85. A giant refracting telescope at an observatory has
microscope of power 10 D. Where should an an objective lens of focal length 15 m. If an eye
object be placed to produce maximum angular piece lens of focal length 1 cm is used, find the
magnification ? Least distance of distinct vision angular magnification of the telescope.
is 25 cm. [Ans. - 7*1 cm] If tliis telescope is used to view the moon, what is
78. The focal lengths of the objective and eye piece the diameter of image of moon formed by objective
lens ? The diameter of the moon is 3-42 x 10^’ m
of a microscope are 2 cm and 5 cm respectively,
and the distance between them is 20 cm. Find
and radius of lunar orbit is 3-8 x 10^ m.
the distance of the object from the objective when (CBSE 2011,19) (Ans. 1500 ; 13-5 cm]
9/130 ‘Pxadeefa. 4. Fundamental Physics (XII) LV«1WII
86. A refracting telescope has an objective of focal which it gets reflected along a direction parallel
length 30 cm and an eye piece of focal length 3 to the plane mirror A/j. Find the value of ZX.
cm. It is focussed on a scale distant 2 m. For [Ans. 40'*]
seeing with relaxed eye, calculate the separation
between the objective and eye piece.
[Ans. 37*97 cm]
87. A telescope consists of two lenses of focal lengths
20 cm and 5 cm. Obtain its magnifying power
when final image is (i) at infinity (//) at 25 cm
from the eye. (Uttarakhand Board 2012)
[Ans. (1) - 4 Hi) - 4*8]

ww
88. A reflecting type telescope has a concave reflector
of radius of curvature 120 cm. Calculate focal
length of eye piece to secure a magnification of
20. [Ans. 3 cm]

Flo
89. How would you combine two lenses of focal

e
lengths 25 cm and 2-5 cm to make a telescope ? 96. A rod AB = 10 cm in length is placed along the

ree
What is the magnifying power of this telescope ? principal axis of a concave mirror having focal

FFr
[Ans. Lensc.s should be held at a distance of length equal to 10 cm as shown in Fig. 9(Q).23.
27*5 cm from eachother, A/ = - 10] The distance PB = 20 cm. What is the length of
uurr
90. Two boys, one 52 inches tall and the other 55 the image of the rod AB ? [Ans. 5 cm]

orr
inches tall, are standing at distances 4 m and 5 m sfo FIGURE 9(Q).23
respectively from an eye. Which boy will appear
taller ? [Ans. First boy]
kks
Yo
91. The angular magnification of a tele.scope is 300.
oooo

10Cm
What should be the diameter of the objective, if A B 20 Cm
our eyes at the eye ring, are just able to collect
eB

P
&
all the light refracted from the objective. Take
diameter of pupil of eye = 3 mm. [Ans. 90 cm]
ur

92. The image of the moon is focussed by a


ad

converging lens of focal length 50 cm on a plane


YYo

screen. The image is seen by an unaided eye from 97. A motor car is fitted with a convex driving
a distance of 25 cm. Find the angular magni mirror of focal length 20 cm. A second motor
dd

fication achieved due to the converging lens ? car is 6 m away from the driving mirror of the
Re
inn

[Ans. 2] first car. Calculate (i) position of second car as


seen in the first car mirror,
F

TYPE XV. TYPICAL PROBLEMS


(if) if the second car is overtaking the first car at
93. A concave mirror of focal length 20 cm is placed a relative speed of 15 m/s, how will its image be
at a distance of 50 cm from a wall. How far from moving and in what direction ?
the wall should an object be placed to form its [Ans. {/) 0*1935 m (ii) -1*56 cm/s]
real image on the wall ? [Ans. 16*7 cm] 98. Monochromatic light is incident on the plane
interface AB between two media of refractive
94. An object is placed 0-4 m from a convex mirror
and a plane mirror is placed at a distance of 0-3 m indices Pj and P2 ^1^2 > angle of
from the object. The images formed in the two incidence 0 as shown in Fig. 9(Q).24. The angle
9 is infinitesimally greater than the critical angle
mirrors coincide without parallex. What is the for the two media so that total internal reflection
focal length of the convex mirror ?[Ans. 0*4 m]
takes place. Now, if a transparent slab DEFG of
95. Two plane mirrors are inclined to eachother at unifonn thickness and of refractive index P3 is
an angle 6 = 70”, Fig. 9(Q).22. A ray SO of light introduced on the interface as shown in Fig.
falls at some angle / on the mirror M], falls after 9(Q).25, show that for any value of P3, all light
reflection from it, on the other mirror M2 from will ultimately be reflected back into medium II.
RAY OPTICS AND OPTICAL INSTRUMENTS 9/131

FIGURE 9(Q).24 material of the prism is 45°. What is refractive


D MEDIUM I E index of the material of prism ? [Ans. ]

MEDIUM m (^3)
A G I B

e I
I
MEDIUM II (^2)
I

99. The image of a needle placed 45 cm from a lens


is formed on a screen placed 90 cm on the other
side of lens. Find displacement of image if object
is moved 5 cm away from lens, [Ans. 15 cm]
100. A biconvex thin lens is prepared from glass

ww
(|X = 1 -5), the two bounding surfaces having equal
radii of 25 cm each. One of the surfaces is
silvered from outside to make it reflecting. Where

Flo
should an object be placed before this lens so
that the image is formed on the object itself ? 105. The refractive index of a prism with apex angle

ee
[Ans. -12*5 cm] A is cot A/2. Prove that the angle of minimum

rere
101. A concavo-convex lens made of glass (p. = 1-5) deviation is 5^ = (180° - 2 A).

r FF
has surfaces of radii 20 cm and 60 cm. (a) Locate 106. The refracting angle of a glass prism is 60° and p
the image of an object placed 80 cm to the left of of its material is 1*45. Calculate angle of
uurr
the lens along the principal axis, (b) A similar
lens is placed coaxially at a distance of 160 cm
foor
incidence at the first face that will just reflect
internally the ray at the second face.
ks s
on the irght of it. Locate the position of the image. [Ans. 24*2°]
Yoo
[Ans. 240 cm ; 34*3 cm]
ooook

107. The dispersive powers of crown and flint glasses


102. Rays of light are falling on a convex lens of focal are 0-03 and 0-05 respectively. The difference in
eBB

length 40 cm. as shown in Fig. 9(Q).25. refractive indices for blue and red colour is 0-015
Determine the position of the image. for crown glass and 0-022 for flint glass.
Calculate the angles of the two prisms for a
uurr

deviation of 2° without dispersion.


ad
Yo

[Ans.A = 10°;A = 6*82°]


108. A telescope objective lens has a focal length of
dY

100 cm. When the final image is formed at the


Re

least distance of distinct vision, the distance


innd

between the lenses is 105 cm. Calculate the focal


FFi

length of eye piece and magnifying power of


telescope. [Ans. 6*25 cm ; 20]
109. A compound microscope has lenses of focal
length 10 mm and 30 mm. An object placed at
1-2 cm from the first lens is seen through the
second lens at 0-25 m from the eye lens. Calculate
[Ans. 8 cm] (0 magnifying power («) distance between the
103. One face of prism of refracting angle 30° and
two lenses. [Ans. - 46*7 ; 8*7 cm]
refractive index 1-414 is silvered. At what angle 110. A reflecting type telescope has a concave reflector
of radius of curvature 120 cm. Calculate the focal
must a ray of light fall on the unsilvered face so
that it retraces its path out of the prism ? length of eye piece to secure a magnification of
15. [Ans. 4 cm]
[Ans. 45°]
111. The lens of human eye has a diameter of 0-8 cm.
104. As shown in Fig. 9(Q).26, PQ is a ray incident How much fainter star can be seen through
on prism ABC. Show the correspondingrefracted 508 cm objective of an astronomical telescope
and emergent rays. The critical angle for the at Mount Polomar in USA ? [Ans. 403,225]
9/132 ^t4xdee^ U Fundamental Physics (XII)
■/■●if

/V
Q

For Difficult Questions

1. As wavelength and frequency remain unchanged


on reflection, therefore, V = \ = 6000 A
/!,' -10-«2
-10 + 50
I
c 3x10^
V = V = — = 5 X 10'^ Hz
X 6000x10““^ _ 40 - 4 «2 = 40
- 4 «2 = 80. = - 20 cm

oww
Now, Z/ + Zr = 60"
As Z/-=Z( 2Z/ = 60"
8. Here, /j j = 2 cm, m = - 16 cm,
i = 30” /i2 = - 3 cm (image is real and inverted)
2. To make the reflected ray horizontal, it must be As
~^2 _ V

e
turned further through 90" - 60° = 30°. Therefore, 14

re
FFrlo
the mirror must be turned through 30/2 - 15"
3. Here, 9 = 45° V = —-u = —x(-16) = -24cm

rF
h 2

ee
360' 360 1
n = _j =8-1 =7
0 45
J 1 _-2-3^ 5

rF
---+--
ouru
4. Here, /jj = 12 m, n = 15 m / V u -24 16" 48 48
d = ?/j2= 12cm = 012m

fosor
48
/=- = — 9‘6cm
i.- V
5
skf
As
h u
1 9. Here,/= - 20 cm. As the image may be real/
ooko
Yo
>h 0-12
Y
V = -=-xw = xl5 = 0-15 m = 15 cm virtual, therefore, m = ±3 = — or v = ±3 u
h 12
Bo

u
I
reeB

5. Here,/ = - 15 cm, m = ± 3, « = ? 1 1
As
V V u f
ooY

As m = — = +'3 ,v = -3 u
uur

u
1 1 I
ad

+ - =
I,
From
111
—+- =
1 1
+ - = —
1
±3m u 20
dY

Vu / -3u u -15
+ 1+3 1
or
2 1
nind

3w 20
Re

or = - 10 cm
3ii -15 ’ or -311 = 40 or 80
F
Fi

V
Similarly, when m = - = -3 ; we get -40
u u - cm
3
« = - 20 cm
-80
f or u =
3
cm

6. For object A, m = — - , and


10. Areal magnification = (linear magnification)^
. _ _ / 12. Here, « = ?,/=- 10 cm, m = - 2
for object B, m
/«; /-»2 As m =
/
f-u
m

m h Ih -10 10
1 -2 =
-10-w 10+M
Now, /j| = 4/jj’ ; /i2 =/i^, /= - 10 cm. 10 = -20-2i(

/<j = - 50 cm 2 u = - 30, tt = - 15 cm
RAY OPTICS AND OPTICAL INSTRUMENTS 9/133

13. Here, /? = - 40 cm, « = - 10 cm, v = ?


Lateral shift =
From the mirror formula cosr,
1

1 1-1 01 sin (60°-35 3°) _ 01sin24»7°


V u R
cos 35-3° cos 35-3°

1 = 1-- 01x0418
= 0-0513 m
V R u 0-816
1 1 4
^J2 L = _L 17. sin C = —
1-25 5
“ ^0 -10 20 11

V =: 20 cm
As C-i- r=90“orr = 90°-C

ooww
As V is positive, image is virtual, erect and sin r = sin (90° - C) = cos C
formed behind the mirror
= Vl - sin^ C = . 1- ('if =-
Magnification, m =
-V
= =“ = 2 V
-10

e
u
sin/
From Snell’s Law, — = \L = 1-25

ree
14. Here, apparent shift,

rFl
sinr

d = 1 -5 cm

Fre
sin / = 1 -25 sin r = 1-25 x - = 0-75
Let t be the height through which water must

rrF
5
be poured into the beaker. / = sin-* (0-75) = 48*6°
ouur
This is the maximum value of /, for which
As d = t \-~
sffoo Total Internal Reflection would occur at the
vertical surface.
okks
18. In Fig. 9(Q).27, A = 80 cm, JC =?
Yo
1-5 = / 1- - :r = 4x 1-5
4/3 J 4 ’ 1
ooo

V p = 4/3 =
sinC
BB

= 6 cm

15. Here, v = 6 x 10*4 Hz. \l = 1-5. tanC = -^, r = htanC


rr e

h
(/) In glass,
sin^C
ouu

V 3x10^ n p- = nh^ tan^C =%h^


= 2x10* m/s
ad
YY

speed of light, 1 - sin^ C


11 1-5

2x10* = Kh^x^
dd

-
V
s _ = nil
wavelength of light, = l-l/n^ H^-l
Re
iinn

g
V 6x10*4
= 3-3 X 10"’ m 3-14 (80)2
F

= 25837*7 sq. cm.


(//) In air (4/3)2-1
speed of light, = c = 3 x 10* m/s
FIGURE 9(Q).27
3x10* r
9 P
Wavelength of light, "6x10*4 /
/

16. Here, /j = 60°, / = 0-1 m, |i = 1-5


= 5 X ir’m
h f /

iC /
/

sin /1 /
/

As = 1^ /
sinr.1 /

Sint, sin 60° = 0-5773 19. Here, R = 15-0 cm, \i = 4/3, h = 7-0 cm.
sm r,
1 “
11 1-5 As is clear from Fig. 9(Q).28, light rays emerge
rj = sin-* (0-5773) = 35-3° through a circle of radius r.

A
9/134
Fundamental Physics (XinsTOWn
Red ray will be transmitted.
For green ray, n = 1424,

sin C = — = = 0-702, C = 44-6”


11 1424
green ray will be reflected at face AC.
For blue ray, \i = 1476,
. ^ 1 1
= 0-678, C=42-6“
|i 1476
r = htmC .(/) /. Blue ray will also be reflected at face AC.
22. Here, u = 1-5 x 10* m/s

ooww
1 1 3
where sinC = - c = 3 X 10* m/s
11 4/3 4
c 3x10* ^
cos C = -^l-(3/4)2 4
V 1-5x10*

e
If C is critical angle, then

ree
.^34 3

rFl tanC = -x-= = -=


4 V? V? sin C = 1-i

Fre
H"2

rrF
Area of patch through which light emerges C = 30“
= jur2 = jc (ft tan = nh^ tan^ C 23. Here, = 1, H2 = 1-5, /? = - 2 cm
ouur
22
= —x7x7
7
IV7J
3
\2
sffoo
= 198-0 cm2
Incident ray OA in glass is refracted in air, along
AB, and appears to come from /, Fig. 9(Q).19.
u = PO = 7 V = PI = -I cm
okks
20. In Fig. 9(0). 17, the incident ray is normal to AB.
Yo
As refraction occurs from denser to rarer
ooo

Angle of incidence on AC = 0 and angle of medium.


incidence on BC is 0' = (90" - 0).
BB

For total internal reflection, critical angle (C)


rr e

must be less than smaller of 0 and (90® - 0). U V R


1
ouu

C^45® or sinC<
1-5 ^ 1 _ 1-1-5 1
ad
YY

V2 u -1 -2 4

or is 1
or ll>-^ 1-5
l__5
11 V2
dd

1
u 4“ 4
Re
iinn

Min. value of p =
-4x1-5
F

u = = -1-2 cm
(b) For p = 5/3, C = sin”^ - = 37® 5

The air bubble O lies at 1-2 cm from the


If ZA were 60®, angle of incidence on CB would refracting surface within the sphere.
be 90®- 60® = 30®, which is less than critical angle
(37®). Therefore, total internal reflection cannot
24. Here, « = - 50 cm, /? = 10 cm, P| = 1, P2 = 1-5
take pla< ; from the surface CB. Refraction at surface P|A
21. As ABC is an isoceles right angled prism, angle Virtual image at /j where P^I^ = U|
of incidence of each ray is 45®. If critical angle C
is less than 45®, the ray will be totally internally u V R
reflected at AC. When ZC > 45®, the ray will be
transmitted through the face AC. 1 ^ 1-5 _ 1-5-1 _ 1
-50 V 10 20
1
For red ray, p = 1-39
3 _ 1 1 _ 3
sm C = — = = 0-719, C = 46-0® , Vj =50 cm
p 1-39 2u.1 "20 50 “ 100

K
RAY OPTICS AND OPTICAL INSTRUMENTS 9/135

Refraction at surface

/j acts as virtual object

oww
u — PqI^ — P j/j — P yP2 — 50 — 20 = 30 cm,
^2 = ^2^=?, /? = - 10 cm
28. Taking vertex as the origin, Fig. 9(Q).31.

e
U V R FIGURE 9(Q).31

re
FFrllo
1 _ 1-1-5 1
30 v~ -10 ” 20

rF
ee
O P c

V
ouru ~ 20 60~10’

sor rF
V

u = 10 cm.
M = PO = - 20 cm
Distance of final image I from centre of sphere
a = C?2 + Pil= 10 + 10 = 20 cm
os kffo
R = + 5 cm, fij = 1, |X2=
Refraction from rarer to denser medium

25. Here, /? = 20 cm, p,| = 4/3, = 1-60


ook
Yo
Y
u = - 200 cm, = ? u V R
, we get u = 30 cm.
Bo

As refraction occurs from rarer to denser


Image is formed inside the rod at a distance of
reeB

medium, therefore. 30 cm from the vertex.


ouY

1x30
[ly Magnification, m = = -l
ur

R P2« l-5x(-20)
ad

U V
Yo

Image will be of same height (0*5 mm) as the


d

4/3 1-60 1-6-4/3


object, but it will be inverted.
nidn

-200 20
V
29. Here, |X = 1*5, Let Ry = R, R2 = -2 R
Re

As rays are rendered parallel, 6 cm must be the


1 8 0-27
focal length of the lens, Le.,f= 6 cm
F
Fi

150 5v~ 20
From —/ = (jj.-l) —
R
- ^
_8_^0^ 1 4 05 -2
5v 20 150 300
-
1
- = (1-5-1)
IS ^ —1 + —
1 1
=-x —
3
5 V X 2-05 = 2400 6 fi.. 2R 2 2/?

2400 18
v = — = 234-15 cm /? = — = 4-5 cm
5; 105 4

(in denser medium) Hence, /?, = 4*5 cm, /?2 = - 9*0 cm


27. Refer to Fig. 9(Q).30. Here, « = -f 30 cm, u = ?, 30. /^ = 20cm,[i^=l-6,// = ?,P/=l-3
Pl = 1,1^2 = 1.5, /? = +15/2 cm. Use formula for 1 'Kg .Vl 1
refraction from rarer to denser medium. -1

Solve to get u = 15 cm. fa a


^ 1
R
R,)
9/136 "Pn^tdeefr '4, Fundamental Physics (XII) VOL.II

\
1
A=(i^-i) -L--! A—
f -m\ 2
orfi* = -SSf
20 R, R,
The lens behaves as concave lens of focal
or _1 1___1_ length - 5*5 /
R^ /?2 ~ 12 (m) When the lens is dipped in medium of
\ / 1-33
I 8
Again, -1
// [R, R, 1 f 15 .Vj
-1 LV—f-i L'
fl V
1-33 / R. Ky ~ 1-33 R. R^
'1-6 A I _ 0-3
1-3 12 1-3x12
/2 _ 1/2 133
= 3-9
/ 17/133 2x17
1-3x12
or /2 = 3-9/

w
= 52 cm
0-3 /. The lens behaves as convex lens of focal
length 3*9 /
1 1 1

Flo
31. (0 - = (p-l) 33. Here, = 15,/^ = 15 cm,// = ?, p/ = 1-7
/ R
«2j If /?j — /?, /?2 ~ — R

e
rree
'3_iV —-f
1 —1=— ● 1 f 1
From — = (n -1)
1 ^

r FF
2 20 20 20 ’ / ^ ' R^ R,\ \ * ^ J
/= 20 cm
uurr
, ,Yi n 2(p-)
(h) When lens is cut along the plane AB, Fig.
9(Q).32, focal length of each half is given by
for =(p-i) —+—
^ \ R R^ R
—-
kss
\

1 / 1 1 1 1
1 ^l(v- 8
ooook

- = (p-l) = --1 X- -1
Yo
f R 2 20 40 .(0
fa
00

V
1

/i = 40 cm
eB

8
FIGURE 9(Q).32 -1
.(«*)
A /( R
urr

A
ad

Divide (/) by («),


Yo

c*- ●T' D /
dY

-1
V fl ^ 1 0-5
Re
innd

B \ /

fa 8
-1
-0-2/1-7
(hi) When lens is cut along CD, Fig. 9(Q).32,
Fi

U-7
each half continues to have original focal length
/= 20 cm. However, intensity of image will
reduce. -0-5 X1-7
// = 0-2
x4 =-4-25x 15 = -63*75 cm
32. (0 When the lens is in air, its focal length is
1 35. Here, p^ = 1-5.
- = (1-5-1) —- The focal length of the lens in air is
/ R
1 rn 8 /
L__L'
(«) When the lens is dipped in medium of 1
R
p= 1-65, fa “1 ^2
\( 1 1 ^
1 ri-5 ^Yi 1 ^ 045fj fl5
—-1
A ,1-65 )[r, ■ 1-65 R^ 1 /?j /?2
j L = _?_
~u[r, r^ ^h~fa
RAY OPTICS AND OPTICAL INSTRUMENTS 9/137

(0 When lens is dipped in medium A of 37. Here, = 1-5, /fj = 20 cm = 0*2 m


Ha =1-65, /?2 = - 20 cm = - 0-2 m
\ \(
1 H8
-1
'±_J. 1
P.I = — -1 _l 1
/a Ha A,
A ' ^ f /l II/?,*
/V /?2^ JI
1 ^
/
n.5 . V 2 -0-15x2 1
= 5D
= (1-5-1)
tl-65 y S, 1-65 4 1^0-2 0-2)
When the same lens is placed in liquid,
1-65/, \ /
/a = = -5-5 4

oww
0-15 X 2
1
-1 1__L
^2=ir
In medium A, the lens will behave as a fi H/
diverging lens, of4 = - 5-54 ( 1-5
\ /
1 1 ^ 0-25 2
-1 x— = 2D
(/i) When lens is dipped in medium B of
1-25 j 1^0-2 "^0-2^ 1-25 0-2

e
Hfi=l-33.
\

FFrlo
re
5
1 ^H 8 1 1
P, 2

ree
-1

F
fe i^Hfl
R1
A2J 39. Here, =1-5, // = - 50 cm

rF
1 _ri-55 A 2 _ 0-17x2 P/= 15/8,4 = ?
/b'Ii-33 ) fa
\
^H 8
fsoor
1 1 ^
ouur
1
-1
.(0
fa R
A2
skf
1
1-334
a

fs ~ = 3-914
ooko
0-34 V8 V \
1
J 1_
Yo
-1
m
Y
In medium B, the lens behaves as a conver
fl I,!'/ «1 «2
Bo

ging lens of 4 = 3-914.


reB

\
36. Here, p = 1 -6, /?| = 30 cm, /?2 = - 30 cm 1 ( 1-5 -1
1 1

/ij = 5 cm, M = - 12-5 cm, /12 = ? -50 1^15/8


uur
oY

< 1 1 ^
ad

1
- = (p-l) —-
1 1
_1_ 5__1_
Al “""'^50^1 “10
or
f R, R,j /?2
dY

From (0, -jr = (1-5 - l)f1=


1
1
innd
Re

= (1-6-1)
1,30 30 j 15 1,10
●'o /
20
Fi
F

4 = 20 cm
15
/ = = 25 cm 40. Here, R2 = ?/= 0-3 m, p = 1-5
0-6
For plane surface of lens, Aj = 00
:R2 = -R
l-i = l
\
1 ^ 1 1
As
From lens formula : — = (p -1)
V u f
/ [Ri /?2 /

j__j L-zl 1 f 1 1 1
= (1-5-1) - + -
V"/ M " 25 12-5 " 25 0-3 00
RJ 2R
i> = - 25 cm R = 0-15 m

fh V '*2-25
41. Here, 4 = 20 cm, Pg = 1-6, P/ = 1-3,/, = ?
As m = A = - h^ u f=— = 2
1 K -1-Vl 1 > = 0-6 f I 1 ^
— .(0
/{= 10 cm
fa [h. A»1
R
R,j R R2
9/138
‘P>t4zeteefi. ^ Fundamental Physics (XII) VOL.II

V 45. Here, £) = 80 cm, = 10 cm,/= ?


1 g
-1 L__L' This is displacement method of finding ‘/’
ft /?! /?2
f = D^-d^ ^ 80^-10^ ^ 6^ = 19-7 cm
AD 4x80 " 320
.(«)
^1.3 «2j 47. Here, - « + v = 90 cm,/= ?
Divide (/) by (//) As image on the screen is real, therefore.
// _ 0-6 V
m=-2=-; v=-2u
fa 03/1-3 u

- M - 2 M = 90 cm ; M = - 30 cm, = 60 cm

ooww
0-6x 1-3x20
1 1 1
f,= 0-3
= 52 cm
-_L 1 _ 1+2 3 1
/ V « ~ 60 30 ” 60 "60 "20
42. Here, - 1-5,/^ = 15 cm, R^=R,R2 = -R
/= 20 cm. The lens is convex.
/ 48. Here, /fj = 10 cm, /?2 = - 15 cm,/= 12 cm,

e
As J_= ji=?

re
fa ^ From lens maker’s formula

rFFl
ree
F
1 1 1 >
1 ri-5
—-1.Yi n j_
—+ — /J = 15 cm - = (^l-l) —-
R2)

rF
15 n R R) R' / R
j\ ^ 1
V 1 (1 1 ^ 1

fsfoor
1
ouur
Again
g — = (p-l) —+ — =(u-l)-^
f,
-1 12 ^ [m 15 / ^ '
6
kosk
/
6 1 1,3
(h5 2 -0-4 li-l = 7r
2 2
Yo
1^1-7 J 15 1-7x15
oo

2
Y

49. Here, / = -/?, where /?i = /?, /?2 = - /?, M- = ?


BB

-1-7x15 3
// = 0-4
= - 63*75 cm
\

From f = (|X -1) ^


1
rre

43. For the convex lens :/ = 20 cm, « = - 60 cm,


/?j /?2
oYuu

v = ?
ad

3 2
1 1 1 1 1 3-1 1
dY

~ ~ ~ - t:: ~t:: “■T7r~^»v = 30cm 2R


V f u 20 60 60 30
H-1 = 1=0■75,^=1●7S
innd

Image V would have been formed where


Re

4
cr = 30 cm. On refraction through concave lens, 52. Here,Pi=-2DPi = + 6D
Fi

rays converging at t become parallel.


F

Power of combination.
Therefore, for concave lens,
/> = />j +/>2 = -2-i-6 = 4D
u = (30 - 10) = 20 cm, v = <»
100 100
F = = 25 cm
l = i-i= ^ ^ 1
P 4
f V u 00 20 20
53. (0 Here, Pj = -f 10 D, F2 = - 5 D
/=-20 cm p = Pj +^2 = + 10-5 = 5D
44. Here, u = 10 m, m = - 19,/ = ? ^ 100 100
F = — cm= = 20 cm
f-v P 5
m =

f (ii) m = -f 2, as image is virtual


f
From m =
f-\0 u+f
-19 = or - 19/=/- 10 or 20/= 10
/ 20
2 =
/= 0*5 m m + 20
RAY OPTICS AND OPTICAL INSTRUMENTS 9/139

2m + 40 = 20 PI^ = R = 20 cm.
20-40
For the lens,
= -10 cm
u =
2
V = C/, = CP + PI^ = 10 + 20 = 30 cm.
1 1 1
54. The given set up is shown in Fig. 9(Q).33. For From —
the convex lens, /= 15 cm, u = OC = - 60 cm V u f
1 1 1 1 1 1
1
From —
1 1 1 1 1 = 15 30 30
u f V
V u f' V f u 15 60 20
« = - 30 cm, i.e., CO - 30 cm
u = 20 cm
i.e.y object must be at 30 cm in front of convex lens.
FIGURE 9(Q).33 56. The given set up is shown in Fig. 9(Q).35. Here

ooww
CM=10cm. M = CO = -60cm;/=?

e
ree
rFl
Fre
14 20 cm

rrF
If plane mirror were not there, the lens would
ouur
form a real image at /,, where C/j = v = 20 cm.
Now, /j would act as virtual object for plane
sffoo The rays starting from O, diverge on passing
through concave lens, get reflected from the
okks
mirror, where /jAf = C/j - CM = 20 - 10 = 10
mirror, would form image at /j, where MI^ =
Yo
cm. Therefore, it will form the image at 10 cm in
30 cm. The corresponding position of object
ooo

front of the plane mirror. As MC = CM = 10 cm, from the mirror will be at I where MI - MI^ - 30
eBB

therefore, final image will be formed at C, the cm. Therefore, C/=M - MC=30 - 10 = 20 cm.
optical centre of convex lens. I is the position of object for plane mirror. This
55. The given set up is shown in Fig. 9(Q).34. Here,
uurr

must also be position of virtual image of object


focal length of lens,/= 15 cm ; radius of formed by concave lens alone. Therefore, for
ad
YYo

curvature of concave mirror, /? = 20 cm and CP lens, u = C/ = - 20 cm.


= 10 cm. Let the object be at O, where CO = u. 1 1 1
From
As there is no parallax between the object and
dd

/ u
its image formed by the combination, therefore,
Re
iinn

final image mustbe at O itself. For this to happen, 1=_L J_=_A = _J_
rays erfracted from convex lens L must tend to /“-2o'^60 60 30
F

meet at /j, the centre of curvature of concave /= - 30 cm.


mirror. These rays would fall normally on 57. Here, A = 60®, 5„ = 60®, p = ?
concave mirror and retrace their path on sin (A + 6 m )/2
reflection. sin(60®+60®)/2
V^ =
sin A/2 sin60®/2

sin 60® ^^/2 = V3


"sin 30® 1/2
58. p=l-5,A = 60®,8^ = ?,i = ?
sin (A+ 8 m )/2
As
sin A/2

sin(60®+8^)/2 = 1-5
sin 60®/2
9/140
“Pn4!uCeefr 4. Fundamental Physics (XII) VOL.II

sin (60“ + 8J/2 = 1-5 sin 30“ = 0-75 A +8

2
m

60“+8
m
= sin"' (0-75) = 48-6“
= sin-'0-7333 =47-19“
2 8„ = 2x47-19-A
8..
m = 2 X 48-6 - 60“ = 37-2“
= 94-38“ - 72“ = 22-38“

A+8
m _
60“+37-2 66. Here, \i^= 1-52, = 162 and \iy= 1-59
= 48-6“ Dispersive power (co)
2 2

59. Here, A = 60“, 8„ = 40“, c = 3 x 10^ m/s, v = ?, ^ 1-62-1-52 ^ 0-10 = 0-169


i = l
p^-1 1-59-1 0-59
sin(A+8^)/2 sin(60“+40“)/2 sin 50“ Angular dispersion : 8„ - 8^ = (O x 8
= 0-169 x 40“ = 6-76“
sin All sin60“/2 sin 30“
67. As angular dispersion is same.
0-7660

w
= 1-532
1/2 (H>-p;)A' = (p^-p^)A
(1-6637 - 1-6444) A' = (1-5230 - 1-5145) x 10“
c
c ^3x108

Flo
As M- = —, v = — = 1-958 X 10* m/s 0-0085x10
V
IL 1-532 A' = = 4-4“

reeee
0-0193
A + 8m _
60“+40“
Also, i= = 50“ 180

FFr
2 2 69. As the prism is equilateral, ^ = 60“

60. For deviation in air, p = ^_3/2 (8J, = 38-4“, (8= 38-7“, (8J„ = 39-2“.
Ha 1
for
ur
A + 5m . 60“+38-4“
Sin sin
3/2 9 2 2
and for deviation in water, ji. = ●●● H, =
kkss
w
4/3 ”8 sin A/2 sin 30“
Yo

Use 8 = ()Li -1) A to arrive at the result,


oo

sin 49-2“
= 1-514
sin (A+ 8^)/2 1/2
eB

62. As \i =
sinA/2 . 60“+38-7“
sin
2 sin 49-35“
60 + 8 m = 1-5184
r

60“ n,=
sin sin 30“ 1/2
ou

= pxsin
ad

2 2
YY

. 60“+39-2“
1-53 1 sin
x- = 0-5751 = sin35“6' 2 sin 49-6“
1-33 2 = 1523
ndd

sin 30“ 1/2


Re

60 + 8 m
_ 1-523-1-514 0-009
Fi

2 (0 = = 0-018
-1 1-5184-1 0-5184
8„=10-12-
63. Here i, = 90- - 60“ = 30”, A = 60°, 70. A = 5“, A'= ?,Hy= 1-523, |iir= 1-515
8 = 42“ :(90-i2) = ? \Ji^' = 1-688, \i; = 1-650
Use I| + /2 = A + 8 1-523 + 1-515
= 1-519
Remember that 12 are angles with normal 11 =
2 2
to the faces of the prism.
+ 1-688 + 1-650
sin(A + 8„)/2 H' =
2 2
= 1-669
65. 1L =
sinA/2
For no deviation, 8 + 8' = 0
● + .. . A 1-66 . 72“ (P-1)A + ()li'-1)A' = 0
sin = psin sin
2 2 1-33 2 -(p-l)A -(1-519-1)5 -0-519x5
A' =
1-66 (^'-l) (1-669-1) 0-669
x0-5876 = 0-7333
1-33 = - 3-88“
RAY OPTICS AND OPTICAL INSTRUMENTS 9/141

Negative sign indicates that prisms must be


joined in opposition. From l = l-i
f V u
71. A' = ?,©' = 0 021, \i' = 1-53
1 1 1 _f-v
A=4-2°,a) = 0-045,p= 165
For achromatic combination, the condition is u V f vf
0)5 + o>'5' = 0 25x5 125
u =
0) A (p - 1) + co'A' (p' - 1) = 0 f-v 5-(-25) 30
= - 4*17 cm
A' = - 0) A (p -1) - 0.Q45 X 4-2 x Q.65
to'(p'-l) 0-02 lx 0-53 Again, let u' be the farthest distance at which he

oww
can read the book through the magnifying glass.
A'= -1104“ In that case, image must be formed at infinity,
Net deviation = (p - 1) A + (p' - 1) A' i.e., V =!oo.

= (1-65 - 1) (4-2) + (1-53 - 1) (- 11-04) From u' -


= '>f - f - 5 = 5 cm

e
= - 312“ f-v flv-l 5/eo-l

re
FFrlo
72. Here, u = j: = - 150 cm, « = «>,/= ? m' = - 5 cm (using sign convention)
77. Angular magnification is maximum, when final
i=i_i = _L- ^

rF
ee
From image is formed at the least distance of distinct
f V u~ -150 OO
vision.

rF
/= - 150 cm = -1*5 m
ouru
.'. u = - 25 cm, / = 100^100 = 10cm, « = ?
P 10
P=i=_L
fosor
= -0-67D
f -1-5 From 1-1 = 1
skf
73. For distance viewing part, P\=- 3-5 D V u f
ooko
1_ 1 1^1 1-7
Yo
100 100
Y
●●●
-3-5
cm = _ 28-5 cm
u V f~-25 10 “50
Bo

1
reeB

50
For near vision part, Fj -f ^
^ = - 7*1 cm
u = -

P2 = P-Pi=2-5D-(-3’5 D) = 6-0D
78. Here,/o = 2 cm,/^ = 5 cm, Mq = ?, m = ?
ooY
uur

100 100
As final image is at 25 cm from eye piece,
ad

h - 60 = 16*7 cm
Pi therefore, - -25 cm.
dY

74. Here, u = -25 cm, v = - 100cm,/= ? From J___l__ J = 1 1 _-6


4'-25 5 “25
innd

1 _ 1 1 _ 1 l_-l + 4_+3
Re

f V u -100 25 “ 100 “lOO 25


Fi
F

6
100 1

^=1 Distance of image from objective,


cm = - m
3
on 25 95
p = i = -L = 3D
/ 1/3
1 1 1 6 1 _ 83
75. Here, « = - 25 cm, u = - 50 cm, 95 2“ 2x95
u,
0
V
0 4)
Asl = l-l = J- + l-= —1 /= + 50cm
2x95
= - 2*3 cm
/ V « -50 25 50 “o=- 83 “
76. Here, /= 5 cm
/

Let u be the minimum distance at which he can Magnifying power =nuxm 1+^
f
read the book through the magnifying glass. The n
/
image must be formed at 25 cm in front of the 95/6 25^
1 +— = 41-5
eye. Therefore, v = -25 cm. 2x95/83
\ 5 j
9/142 “Pnadeefi,'^ Fundamental Physics (XII) VOL.II

79. Here,/g = 5 cm,/o = 1 cm, L = 20 cm ; m = ? Magnification produced by objective lens


Vg = oo^d = 25 cm
As final image is at infinity, image formed by u
o

objective lies at the focus of eye piece.


10
i>0 = = 20 - 5 = 15 cm -4 = — o
= - 2-5 cm
U
1 1 1 o

From — =
u
0
V
0
15 I" 15 1 1 1 J 1 _l + 4_5
From — =
10 -2-5 10 10
15 4 u
o
—cm
“0 = 14 10
f = — = 2 cm
As final image is at infinity, ●>0 2
83. m = - 100,/o +4 = 101 cm, /o = ?, 4 = ?

w
V
0
15 25
X—= 70
15/14 5 m--A-= -100 /o=100/.
fe

Flo
81. Here,/Q = 4 cm,/g = 10 cm, Mq = - 6 cm Now /o +fg = 101
1 1 1
1004+4=101,

eeee
From
«0 U
0 /o 4 = 1 cm,

Fr
1 1 1
4 = 1004 = 100 cm
4 6 12
84. Here, 4 = 1 m = 100 cm,4 = 20 cm.
^0
for If Aj = size of image formed by objective lens
ur
Vq = 12 cm and /i2 = size of image formed by eye piece
V
l-iL
ks
0
M = tana =
6{ loj
Yo
l«ol 4J /o 100
oo

Length of microscope = L = Vq +4 d
eB

= 12 + 10 = 22 cm m = — = 1+ —
e
4
82. Here, m = - 20, /Wg = 5
ur

200 50
-20 W_ ^_44 = —cm

h^ ~^^20~20
m
ad

= -4 44 11
Yo

nte 5
d a=tana =
/»! 50 1
= —rad
=1 + — 100 " 1100 22
d

As
Re

fe = 00455 rad
in

1
F

20
5 = 1 + ^4 4=^ = 5cm 85. Here, 4 = 15 m, 4 = 1cm = m
100

15
Angular magnification = = 1500
fe f 1/100
As m^ =
“.+4 Diameter of image of moon formed by objective
lens
5=^ 342 xlO^
= a.4 = 3-8x108 xl5m =0135 m

5-25 = 13*5 cm
= -4 cm
5 86. Here, 4 = 30 cm, 4 = 3 cm, Mq = - 200 cm.
Distance between objective and eye piece Separation between the lenses = (uq + I “e I)
= v^ + \Ug\=\4 From
1 1 1

Up = 14 -1 Mg I = 14 - 4 = 10 cm V
0 “o fo
RAY OPTICS AND OPTICAL INSTRUMENTS 9/143

92. In Fig. 9(Q).37, moon subtends an angle a on


1_^J_^_1_^J1 1 _20-3_ 17 the lens. This would be the angle subtended by
V
0 fo u
0
30 200 600 600
moon on the eye, when viewed directly.
600 As is clear from Fig. 9(Q).37.
V cm = 35-29 cm
0 ~
17 ^=x!d
and a = x/f
From 1 _1__ J_

J l_^_l 1 _ -28

oww
4 ~ -25 3" 75
-75
= -2-68 cm
28
.4 Separation between the lenses

e
= 35-29 + 2-68 = 37-97 cm RAYS

re
FFrlo
87. When final image is at infinity.

rF
_ /q _ -20

ee
m = -4
fe 5

rF
ouru SCREEN
When final image is at f/ = 25 cm ;

m = l4r„4Lz2or,,Al = -4-8 ffosor


Angular magnification
os k
4 I <> ) 5 I 25j m = l = ^ = l = ^=2
90. Angle subtended by the first boy, on the eye a xff d 25
ook
Yo
Y
52 inch 95. As ray reflected from M2 is parallel to A/j, [Fig.
Bo

«l = = 13 inch/m 9(Q).22]
reeB

4-0 m
Z2 = 0 = 70° (Corres. Zs)
Angle subtended by the second boy on the eye is Also, Z1 = Z2 = 70°
ooY
uur

55 inch In A OAC, 0 + Z1 + ZAOC = 180°


a
ad

2 “
5-0 m 70°-F 70°-I-ZAOC= 180°
dY

ZA(9C= 180°-140° = 40°.


As ttj > 0C2, therefore the first boy will look taller
to the eye. According to the laws of reflection,
nind
Re

91. Herem = 300, D = 7,d = 3 nun = 0-3 cm. Z/ = ZAOC = 40°


Refer to Fig. 9(Q).36 96. As is clear from Fig. 9(Q).23, end B lies at
F
Fi

centre of curvature C of the mirror (●.* PC = R


-2 f =20 cm). Therefore, its image is formed
at & itself {i.e., at centre of curvature C only).
For end A, « = - (20 -1-10) = - 30 cm,
/= -10 cm.

As ------ ^
V u /

1=1 ^ ^ 1 -3-H

V f u -10 30 30
_/q P/0 D .4 u=-,15 cm.
fe dto d .4 PA' =\5 cm.

.4 D = mxd = 300 x 0-3 = 90 cm Size of image = A'ff = 20 - 15 = 5 cm


9/144 'P’xitdee^ 4 Fundamental Physics (XII) k.u»jiiii
97. (0 Here,/= + 20 cm = 0-2 m ; u = - 6 m light incident on AB at an angle greater than
From the mirror formula

critical angle sm will suffer total


1 = 1-1 =_L 31

V f u 0-2-6 6 internal reflection and go back to medium II.


O'O
V = -lm= 0-1935 m
31 sin e>
As V is positive, image of second car is virtual, ll2 ^^2
erect and is formed behind the mirror of first As angle of incidence is less than critical angle,
car.
light enters into medium III.
(i7) As is known, the speed of image of second sin 0 _ 1^3
Angle of refraction 6' is given by

ww
car is sin 0' ^^2
sin 0' - ^2 ● o-s. ^2 ^1
= —^sin0>
dv du
^^3 ^^3 ^^2
dt dt

Flo
sin 0' >
^^1

e
du -1
— = 15 ms ^^3

eree
where
dt
i.e., 0' is greater than critical angle. So total

FFr
internal reflection occurs from face DE and the
dv (0-2)2 -xl5
entire light is reflected back to medium II.
uurr
dt (-6-0-2)

orr
100. In Fig. 9(Q).39, let the object be placed at O.
= - 1-56 X 10"^ m/s sfo
The ray OA is refracted along AB at first surface
dv and hits the silvered surface. To get the final
= -1*56 cm/s
kks
image at O, AB should fall normally on silvered
Yoo
dt
oooo

Negative sign implies that as 2nd car approaches surface. Therefore, AB should appear to come
the first car. image of 2nd car moves away from from centre of curvature C2 of second surface.
eBB

convex driving mirror.


98. Refer to Fig. 9(Q).38. When the slab is not inserted.
urr

-I
0 > C = sin
ad

^^2
YYo

sin 0> ...(I)


dd
Re
inn

When the slab is inserted, two cases arise.


F

(0 1^3 ^1^1 Thus V =-25 cm,


R = + 25 cm, iij = 1,1I2 =
We have sin 0 > Refraction from rarer to denser medium
1^2 1^2
-b.+b. = bj:h.
V R

Solve to get w = - 12-5 cm


Thus the object should be placed at a distance of
12-5 cm to the left of the lens.

101. Refer to Fig. 9(Q).40. R^ is +, R2 is +.


1

/ R
1 y

- = (1-5-1)
/ ---| /=60cm.
RAY OPTICS AND OPTICAL INSTRUMENTS 9/145

sin/j
As 1^1 =
sin r.1

sin /j = n sin rj = 1-414 sin 30°


_V2_ 1
2 V2
I, = 4S°
104. As PQ is normal to AB, / j = 0, r, = 0
The ray goes straight along PQR. AtR, ^ = 45®
which is critical angle of material of the prism.
(a) Now, M = - 80 cm,/= 60 cm
Therefore, QR suffers total internal reflection at

lowow
1 1 1 R and goes along RS
As
V u / 1 1 1
V = 240 cm sinC sin 45° 1/^2
First image /j is at 240 cm from first lens.
sinU + S^)/2 A cos A/2

ee
{b) The second lens intercepts the converging 105. 4 = = cot — =
sin AH 2 sin A/2
beam. Image /j acts as a virtud source for second

Fr
lens. r FF A +d
u = 240 - 160 = -1- 80 cm sin
^ = cos A/2 = sin (90° - A/2)
rer
Use i-i= —.
^ to get V = 34*3 cm fofr Fo
A + 8m
u
V u f = 90°--
ks
2 2
102. Refer to Fig. 9(Q).25,
YYouro
5
oo

/= + 40 cm, M = -f 10 cm, t; = ? or
Y =90°-A/2-A/2 = 90°-A
BBo ks

i = l+i=-L^l=A=i or
5„ = 180°-2A
r ee

« / u 40 10 40 8 106. Here, A = 60°, ji = 1-45, /j = ?


1
ouru

u = 8cm
ad

From |il =
sinC’
103. Here, A = 30°, \i = 1-414, r'l = ?
Yo

The ray will retrace its path when it falls 1


sinC = —
1
= 0-6897
normally on silvered face AC of the prism. Fig.
d

[l 1-45
Re

9(Q).41.
iYn

C = sin-‘ (0-6897) = 43-6°


FFind

For total internal reflection at 2nd face of prism,


T2 = C = 43-6°
From rj + T2 = A,
r, = A - r2 = 60° - 43-6° = 16-4°
sinr,
From |i = i-,
sm r.
1

sin ij = |i sin = 1-45 x sin 16-4°


= 1-45 X 0-2823 = 0-4093

ii = sin-^ (0-4093) = 24-2°


Le., t‘2 = 0, T2 = 0 107. Here, for crown glass :
As + V2= A Pj^-p^= 0-015
ri = 30° © = 0-03

»
9/146 “Pn^xdeefi. 4. Fundamental Physics (XII) VOL.II

for flint glass:


n.=Ar„4i 100 1 +
6-25 ^
= 20
n;-n;=oo22 f,[ d ) 6-25 25

<o' = 005
109. Here, /q = 10 mm = 1 cm, fg = 30 mm = 3 cm
Net deviation = (jt - 1) A + (h' - 1) A' = 2” uq = - 1-2 cm, Vg = - 0*25 m = - 25 cm
M = ? L = ?

1 1 1
From
^0 u
0 fo
(0 CO'
_L__i_^_L_i 1 0-2 1
1 1-2 1-2 6
Ax 0015 A'X 0022 ^0 “o

lowow
003 005
Uq = 6 cm
1 11
2° = - A + — A'
2 25
.(0
From
±_J_ = _L
“e 4
For no dispersion,

ee
1 1 1 J 1 ^-28

Fr
-25 3 75
A (0 015) + A' (0 022) = 0 r FF fe
22 -75
A = -—A' .(if)

rer
= -2-7 cm
15
Put in (0
fofr Fo 28
u
25^
ks
\ f' 71 ^ 11 V
0 d\ 6
i _±±A' +—A' = 2 M = 1 +— 1 +— =-46*7
YYouro
2^ 15 J 25 -u
4, -1-2 3 j
oo

n
44 „ -300
L = Vq +1 I = 6 + 2-7 = 8*7 cm
BBo ks

A' = 2, A' = = -6-82°


150 44
r ee

110. Here, R = 120 cm,/^ = ? m =15


From (/i).
f„ = RI2= m/2 = 60 cm
ouru

22 f-300^ = 10°
ad

A = -
15 I 44 4
Yo

From tn = —
Negative sign of A' implies that two prisms must fe
d

be placed in opposition.
Re
iYn

108. Here, /g = 100 cm ;/^ = ?, m = ? f =— =—


fo
= 4 cm
^ A
●'e™ 1C
15
FFind

m
When the final image is formed at the least
distance of distinct vision, distance between
111. Here, d = 0-8 cm, D = 508 cm
objective and eye lens =/g + I I = 105 cm
Ug = 105 -/g = 105 - 100 = 5 cm As light gathering power oc (diameter)^.
For the eye piece, Ug=-5 cm,
Vg = -d = -25 cm
_ (508)^
.-. Brightness ratio P = = 403,225
1 1 1 -1 1 4
d^ ~ (0-8)2
’’'4'^e “,~25'"5"25 It implies that the telescope can see a star 403,
225 times fainter than the faintest star that can
25
/ =—
●'eA = 6*25 cm be seen with naked eye.
4

1
RAY OPTICS AND OPTICAL INSTRUMENTS 9/147

WITH
SOLUTIONS

Q. 1. A small candle 2.5 cm in size is placed 27 cm in front of a concave mirror of radius of curvature
36 cm. At what distance from the mirror should a screen be placed in order to receive a sharp
image ? Describe the nature and size of the image. If the candle is moved closer to the mirror, how
would the screen have to be moved ?

ooww
Sol. Here, size of candle, /ij = 2.5 cm, u=-ll cm, /? =- 36 cm,/= /?/2 =- 18 cm ; v = 1
1
As
u V /

1 1 1 1 _ -3+2 _ 1

e
or u = - 54 cm
V f -18 27 “ 54 * 54

ree
u

rFl
Screen should be placed at 54 cm from the mirror on the same side as the object.

Fre
If /i2 is size of image, then as the image is real

rrF
-V V
m =
h -u u
ouur
1

-^_(-54)_,
2.5 (-27)
sffoo
okks
/i2 = - 5 cm
Yo
ooo

Minus sign indicates that image is inverted.


BB

When the candle is moved closer to mirror, the screen has to be moved away from the mirror. However,
when candle is at a distance less than 18 cm from the mirror, image formed would be virtual and screen is
rr e

not required, as the virtual image cannot be taken on the screen.


Q. 2. A 4.5 cm needle is placed 12 cm away from a convex mirror of focal length 15 cm. Give the location
ouu
ad
YY

of the image and the magnification. Describe what happens as the needle is moved farther from the
mirror.

Sol. Here, /ij= 4.5 cm, w = - 12 cm,/= 15 cm, u = ?, wj = ?


dd

1 1 1
i-_L _L-l±£-_?_
Re
iinn

As
u V
/ V f u~15 12~60~60
F

V = 60/9 = 6.7 cm
Image is virtual, formed at 6.7 cm at the back of the mirror.
V
As m = ~ -
u

6.7 X 4.5
or = 2.5 cm
4.5“ 12 12

Image is erect, and of course virtual.


As needle is moved farther from the mirror, image moves away from the mirror (upto F) and goes on
decreasing in size.
Q. 3. A tank is tilled with water to a height of 12.5 cm. The apparent depth of a needle lying at the bottom
of the tank is measured by a microscope to be 9.4 cm. What is the refractive index of water ? If water
is replaced by a liquid of refractive index 1.63 upto the same height, by what distance would the
microscope have to be moved to focus on the needle again ?
9/148 <t Fundamental Physics (XII)

Sol. Here, real depth = 12.5 cm ; apparent depth = 9.4 cm ; |i = ?


_ real depth 12.5
As = 1.33
apparent depth 9.4

Now, in the second case, j4. = 1.63, real depth = 12.5 cm ; apparent depth, y = ?
12.5
1.63 =

12.5
3' = = 7.67 cm
1.63
.●. Distance through which microscope has to be moved up = 9.4 - 7.67 = 1.73 cm
Q. 4. Fig. 9(N).l (a) and (b) show refraction of an incident ray in air at 60® with the normal to a glass-air
and water-air interface respectively. Predict the angle of refraction of an incident ray in water at 45”

ww
with the normal to a water glass interface. Take = 1-32.

FF loo
ree
reeF
oroFr
r ur
k s ff
YYouo
kos o

Sol. In Fig. 9(N).l(a), i = 60°, r = 35°


sin 60° _ 0.8660
BBoo

a
sin I
= 1.51
r ee

sinr sin 35° 0.5736

In Fig. 9(N).l(b), i = 60°, r= 41°


ad

sin / _ sin 60° _ 0.8660


ouur

= 1.18
Yo

siiiT” sin 47° ~ 0.7314


In Fig. 9(N).l(c)./ = 45°, r = ?
d
Re

a
dinY

sini
w 8 _
n8 a
sin r
FFin

1.51 sin 45° 0.7071 1.32x0.7071


sin r = = 0.6181
1.32 sin r sinr 1.51
r=38.2'

Q. 5. A small bulb is placed at the bottom of a tank containing


water to a depth of 80 cm. What is the area of the surface of
water through which light from the bulb can emerge out ?
Refractive index of water is 1.33. Consider the bulb to be a
point source.
Sol. In Fig. 9(N)-2, the source of light (5) is 80 cm below the surface
of water i.e. SO = 80 cm = 0.8 m.
When Zi = C, for SA and SB, Zr = 90°
.●. Area of the surface of water through which light from the
bulb can emerge is area of the circle of radius

1
9/149
RAY OPTICS AND OPTICAL INSTRUMENTS

AB
r = = OA = OB
2
1
As
sinC

1 1
sin C = — = 0.75
1.33

C = sin-‘ (0-75) = 48.6“


OB
In AOBS, tanC = OB = OS tan C = 0.8 tan 48.6“
OS
r = 0.8 X 1.1345 = 0.907 m
=
Area of the surface of water through which light emerges = 3.14 (0.907)^ = 2.518

ww
Q. 6. A prism is made of glass of unknown refractive index. A parallel beam of light is incident on a face
of the prism. By rotating the prism, the minimum angle of deviation is measured to be 40“. What is
the refractive index of the material of the prism ? The refracting angle of the prism is 60“. If the
prism is placed in water (refractive index 1.33), predict the new minimum angle of deviation of a

Flo
parallel beam of light

ee
Sol. Here, 6^ = 40“, p = ?A = 60“.

rere
sin (A + dJ/2 _ sin (60“+ 40“)/2 sin 50“ ^ 0.766

r FF
|A = = 1.532
As
sin A/2 sin 60“/2 sin 30“ 1/2
uurr
sin(A + y^)/2
When prism is placed in water, 'Vg = sin AH foor
ks s
a

sin(A + 8'J/2= %xsin- = J!i.sm60“/2 = 1.33 xsin 30“= 1.33 x-2 = 0.5759 = sin 35“
. 1.532
A 1.532 1
Yoo
10'
oook

w
eBB

A + 5' m
= 35“ 10'
2
A + 6'„ = 70“ 20'
uurr

5'^ = 70“ 20' - A = 70“ 20' - 60“ = 10“ 20'


ad
Yo

Q. 7. Double convex lenses are to be manufactured from a glass of refractive index 1.55, with both faces
of same radius of curvature. What is the radius of curvature required if local length of lens is to be
dY

20 cm?
Re

Sol. Here, p = 1.55, = /? and /?2 = -/?./= 20 cm.


innd

1 ^
^ = (1.55-1)1Ul.il
FFi

1
As
U'«2j
B = 20x l.l=22cm
20 /? J R

Q. 8. A beam of light converges to a point P, A lens is placed in the path of the convergent beam 12 cm
from P. At what point does the beam converge if the lens is
(a) a convex lens of focal length 20 cm. (b) a concave lens of focal length 16 cm ?
Sol. Here, the point P on the right of the lens acts as a virtual object, .*. m = 12 cm, u = ?
(a)/= 20 cm
1 1 1
As
1_J_ = _L
V u / V 12 20

1 = 3+5 ^ 8
i = J- —
V “ 20 12 60 60
V = 60/8 = 7.5 cm

A
9/150
‘P^a.det^ ^ Fundamental Physics (XII) P7»Wll
Image is at 7.5 cm to the irght of the lens where the beam converges.
1 1 _ 1 -3 + 4 1
(^)/= - 16 cm, w = 12 cm.
v~ f ii~ 16 !2" 48 ~48
u = 48 cm

Hence, image is at 48 cm. to the right of the lens, where the beam would converge.
Q. 9. An object of size 3 0 cm is placed 14 cm in front of a concave lens of focal length 21 cm. Describe the
image produced by the lens. What happens if the object is moved further from the lens ?
Sol. Here, /i|= 3 cm. ; « = - 14 cm,/ = - 21 cm. u = ?
As l_l I
V u /

1=1 1-J_ -2-3 -5

V f u -21 -14 42 42

w
-42
V = = -8.4 cm
5

Flo
.●. Image is erect, virtual and at 8.4 cm from the lens on the same side as the object.
^2 _V

e
h2 ^-8.4

rree
As
u 3 “ -14

r FF
/i2 = 0.6 X 3 = 1.8 cm
As the object is moved away from the lens, virtual image moves towards focus of lens (but never beyond
uurr
focus). The size of image goes on decreasing.
for
Q. 10. What is the focal length of a convex lens of focal length 30 cm in contact with a concave lens of focal
length 20 cm. Is the system a converging or a diverging lens ? Ignore thickness of the lenses.
kss
Sol. Here, f = 30 cm,/2 = - 20 cm,/- ?
ooook
Yo

As 1-1 J_ j__ j i_ 2-3 1

fi^iT
eB

/ “ 30 20 60 60
/= - 60 cm
urr

.●. The combination of lenses behaves as a concave lens. The system is not converging.
ad

Q. 11. A compound microscope consists of an objective lens of focal length 2.0 cm and an eye piece of focal
Yo

length 6.25 cm, separated by a distance of 15 cm. How far from the objective should an object be
dY

placed in order to obtain the final image at (a) the least distance of distinct vision (25 cm) (b) at
infinity ?
Re
innd

What is the magnifying power of the microscope in each case ? (CBSE 2008)
Sol. Here, /q = 2.0 cm,/^ = 6.25 cm., u 0 =” 7●
Fi

(a) Vg ~ -25 cm
1 1 1
As
J 1 _-l-4_-5
Ve ue fe u
e f. -25 6.25 25 " 25
»^ = - 5 cm.
As distance between objective and eye piece = 15 cm. Vq = (\5 - uj = 15-5 = 10 cm.
1 1 1
As
V
0
u
0 f,0
1 I I 1 1 1-5
u
0
V
0 /o 10 2 “ 10
-10
u = -2.5 cm
0" 4
RAY OPTICS AND OPTICAL INSTRUMENTS 9/151

V
0
( A y 10 25 ^ = 20
Magnifying power = 1 +

ImqI ^ fe 2.5 1, 6.25 J


(b)As Ug = oo, - Ug =4 = 6.25 cm v^= 15- 6.25 = 8.75 cm.
1 1 1
As
^0 u
0 ●^0
1 1 1 J 1 2-8.75
"o V
0
8.75 2.0 ” 17.5
-17.5
“o = 6.75 = -2^9

8.75^^ 25
V V
Magnifying power =
e
= 13.51

w
ImqI ImJ 2.59 6.25
Q. 12. A person with a normal near point (25 cm) using a compound microscope with an objective of focal
length 8.0 mm and an eye piece of focal length 2.5 cm can bring an object placed 9.0 mm from the

Flo
objective in sharp focus. What is the separation between the two lenses ? Calculate the magnifying

eee
power of the microscope ?
Sol. Here, d = 25 cm,/o = 8.0 mm = 0.8 cm,/^ = 2.5 cm, Uq = - 9.0 mm = - 0.9 cm.

Fr
As J-—L = -L J 1_ {Vg = -d = -25 cm)

for
ur
1 1 _-l-10_ 11
-25 2-5 “ 25 “ 25
ks
25
Yo
= -2.27 cm
“e=-
oo

11
eB

1 1 1
Again
"o U
0 /o
ur

1 ^0.9-0.8^ 0.1
ad

V
4 u 0.8 -0.9 “ 0.72 ” 0.72
Yo

0 0

0.72
V
0 ~ = 7.2 cm.
0.1
d
Re

Separation between the two lenses = I I + Ug = 2.27 + 7.2 = 9.47 cm


in

V
0 f ^ \ 7.2 25^ =
F

Magnifying power = —^ 1 + — = 1 + 88

i«o'[ fe j 0-n 2.5;


Q. 13. A small telescope has an objective lens of focal length 144 cm and an eye-piece of focal length 6*0 cm.
What is the magnifying power of the telescope ? What is the separation between the objective and
the eye-piece ?
Sol. Here, /o = 144 cm, fg = 6-0 cm, m = ?, L = ?
0 _ -144
In normal adjustment, n ■ = -24
fe 60

L =/o+4 = 144 + 6-0 = 150*0 cm


Q. 14. (a) A giant refracting telescope at an observatory has an objective lens of focal length 15 m. If an eye
piece of focal length 1.0 cm is used, what is the angular magnification of telescope ?
(b) If this telescope is used to view the moon, what is the diameter of the image of the moon formed by
the objective lens ? The diameter of the moon is 3.48 x 10® m and radius of lunar orbit is 3.8 x 10* m.
(CBSE 2008)
9/152
^n<KUe^'4. Fundamental Physics (XTIIPTSTMTI
Sol. Here,/o ~ m,/^ - 1.0 cm = 10“^ ni FIGURE 9(N).3
-L0 _
-15
(fl) Angular magnification = = -1500
4 10-2

(b) If d is diameter of the image, then from Fig. 9(N).3.


/
I
3.48x10^
angle subtended by diameter of moon 0,1 =
r,
1
3.8x10^

and, angle subtended by image = — = — 15^

oww
''2 fo
d _ 3.48x10^ live lens
As 02 - 0,
15 " 3.8x10®

e
3.48x15x10-2
d =

re
3.8

FFrllo
= 13.73 X 10-2 m= 13.73 cm

rF
Q. IS. Use the mirror equation to deduce that:

ee
(a) an object placed between/and 2/of a concave mirror producesa real image beyond If.
ib) a convex mirror always produces a virtual image independent of the location of the object.
ouru
sor rF
(c) the virtual image produced by a convex mirror is always diminished in size and is located between
the focus and the pole.
(d) an object placed between the pole and focus of a concave mirror produces a virtual and enlarged
image.
os
(CBSE 2011)
kffo
1 1
ook
Sol. (a) The mirror equation is
Yo
or
Y
V u / V / u
Bo

For a concave mirror,/is negative i.e.f< 0. As object is on the left, u is negative, i.e., u<0
reeB

As object lies between/and 2/of a concave mirror, 2/< u <f


ouY

1 I
> - > —
1
or
1
— < — <
1 1
or
j_ 1_ < 1 1 <0 or <-<0
ur

2/ u / 2/ u / f 2f f u
2/ V
ad
Yo

1
d

.-. - is negative or v is negative. The image is real. Also v > 2 fie. the image lies beyond 2/
nidn

{b) For a convex mirror,/is positive ie.f> 0. As object is on the left, ii is negative, Le., m < 0.
Re

I 1 1
As
F
Fi

/ u

1
— is positive or v is positive ie. image is at the back of the mirror. Hence image is virtual, whatever
V
be the value of u.
(c) For a convex mirror,/> 0 and w < 0
1 1 (\ ( 1 ^
As , therefore >
Le.,v < f
V f u V
V' / f
Image is located between the pole and the focus. As i» < I u I, the image is diminished.
{d) For a concave mirror,/< 0
1
As object is placed between the pole and focus /< n < 0 >0
1/ u

But L_I'' = —^ > 0 orv is positive. Image is on the right. It must be virtual.
V
RAY OPTICS AND OPTICAL INSTRUMENTS 9/153

I 1
Also, - < i.e.,v > I u I Image is enlarged.
V \u I

Q. 16. A small pin fixed on a table top is viewed from above from a distance of 50 cm. By what distance
would the pin apper.r to be raised, if it be viewed from the same point through a 15 cm. thick glass
slab held parallel to the table ? jj. of glass is 1*5. Does the answer depend on location of the slab ?
Sol. Here, real depth, .v = 15 cm, apparent depth, y = ?, p = 1 -5
X X 15
As P = - y = - — = 10 cm .
1.5

Distance through which the pin appears to be raised = x - y = 15 - 10 = 5 cm


The answer does not depend upon the location of the slab.
Q. 17. (a) Fig. 9(N).4 shows a cross-section of a ‘light
pipe’ made of a glass fibre of refractive index

w
1.68. The outer covering of the pipe is made
of a material of refractive index 1.44. What
is the range of the angles of incident rays with

Flo
the axis of the pipe for which total reflections
inside the pipe take place as shown,

ee
(b) What is the answer if there is no outer covering of the pipe ?

Fr
Sol. Here, p-> = 1.68, Pj = 1.44
1
As p = sin C = — = = 0.8571 C = sin-1 (0-8571) = 59°

for
ur
sin C 1.68
^^2
Total intemal reflection would take place when i > C i.e. i > 59° or when r < r max , where
= 90°-C = 90° - 59° = 31°. [from Fig. 9(N).4]
ks
Yo
sini
oo

max
As = 1.68
Sin r
eB

max

sin i = 1.68 sin .rmax = 1.68 sin 31° = 1.68 X 0.5156 = 0.8662 .-. i
max
= 60°
max

Hence all rays which are incident in the range Z / < 60° will suffer total internal reflection in the pipe.
It should be noted that lower limit of / is not 0°. It will be determined by the ratio of diameter to the length
ur
ad

of pipe.
Yo

i_b) If there is no outer coating of pipe, P2 = 1 -68, Pj = I


1 1
As sin C = — = 0.5952 C = sin-1 (0-5952) = 36.5°
d

1.68
Re

i^2
in

from = p = 1.68
F

Now, I = 90° will have r = 36.5°


sin r

r = 90° 36.5° = 53.5°, which is greater than C. Thus all rays incident at angles in the range zero to
90° will suffer total internal reflection.
Q. 18. The image of a small electric bulb fixed on the wall of a room is to be obtained on the opposite wall
3 m away by means of a large convex lens. What is the maximum possible focal length of the lens
required for the purpose ?
Sol. For a real image (on wall), minimum distance between the object and image should be 4/.

4/= 3 m .● . f = —iX\ = 0*75 ni


4

Q. 19. A screen is placed 90 cm from an object. The image of the object on the screen is formed by a convex
lens at two different locations separated by 20 cm. Determine the focal length of the lens.
Sol. Here, distance between object and screen, D = 90 cm
Distance between two locations of convex lens, d = 20 cm, /= ?
9/154
T^^uidee^’41 Fundamental Physics (XII) VOL.II

D^-d}
As / =
4D

/ =
90^ - 20^ (90 + 20) (90 - 20) 110 x7 = 21*4 cm
4x90 360 36
Q. 20. Determine the ‘effective focal length* of the combination of two lenses of focal lengths 30 cm and
- 20 cm if they are placed 8*0 cm apart with their principal axes coincident Does the answer depend
on which side a beam of parallel light is incident ? Is the notion of effective focal length of this
system useful at all ?
(b) An object 1*5 cm in size is placed on the side of the convex lens in the above arrangement The
distance between the object and convex lens is 40 cm. Determine the magnihcation produced by the
two lens system and size of image.

oww
Sol. (a) Here, /j = 30 cm, /2 = - 20 cm, d - 8-0 cm, /= ?
Let a parallel beam be incident on the convex lens first. If 2nd lens were absent, then
u^ =
OO
and /i=30 cm
1 1 1 1 1 1

e
As or Ui1 = 30 cm
v^ u^ /l 30

eree
V OO

rFl
This image would now act as a virtual object for 2nd lens.
1

FFr
«2= + (30-8) = + 22cm, ^2=? /2 = -20cm
1 1 1 _-11 + 10 -1

orr
As
^2 ” -20 22 " 220
ouur
^2 “2
sfo
V2 = - 220 cm
.-. Parallel incident beam would appear to diverge from a point 220 - 4 = 216 cm from the centre of the
kks
two lens system.
Yo
oooo

(ii) Suppose a parallel beam of light from the left is incident first on the concave lens.
eBB

Mj = - oo,/j = - 20 cm, Uj = ?
1 1 1
As
1__J_ J__J_ 1 -1
urr

Vi fl V -20 — OO 20
ad

= -20cm
YYo

This image acts as a real object for the 2nd lens


«2 = - (20 + 8) = - 28 cm,/2 = 30 cm, ^2 = ?
dd

1 1 1
1 _ 1 ^ 1 _ 1 1 14-15
Re
inn

As
V2 U2 f2 <>2~ fl «2 30 28 420
F

V2 = - 420 cm
.’. The parallel beam appears to diverge from a point 420 - 4 = 416 cm, on the left of the centre of the two
lens system.
From the above discussion, we observe that the answer depends on which side of the tens system the
parallel beam is incident. Therefore, the notion of effective focal length does not seem to be useful
here,

{b) Here, /i|= 1-5 cm, «j = - 40 cm, m = ?, /»2 = ?


1 1 1
For the 1st lens.
V,
1
u
1 /■
1 _ 1 ^ 1 _ 1 1 ^4-3^ 1
V
1 fl u
1
30 40 120 120

u,= 120 cm.


RAY OPTICS AND OPTICAL INSTRUMENTS 9/155

l=J^ =3
V

Magnitude of magnification produced by first lens, m^= —


u 40
1

The image formed by 1st lens acts as virtual object for the 2nd lens
«2= 120-8 = 112cm, /2 = -20cm, ^2=?
J 1__ J_ J--i- + -L= ^ 1 -112 + 20 _ -92
As
V2 «2 /2 V2~ f2 «2 -20''’112“ 112x20 “112x20
-112x20
U2 = 92

. .... ... ^2 112x20 20


Magmtude of magmfication produced by second lens /«2 = — = ^2x112 “ 92

ww
Net magnification produced by the combination /n = mjxm2 0*652
Size of image, h2 = mhi = 0-652 x 1-5 = 0*98 cm

Flo
Q. 21. At what angle should a ray of light be incident on the face of a

e
prism of refracting angle 60% so that it just suffers total internal

ere
reflection at the other face ? The refractive index of the prism is

FFr
1*524.

Sol. /i = ?, A = 60“, 4 = 1-524


uurr
orr
1
As
sinC
and from Fig. 9(N).5, sfo
C = r2 (for total internal reflection at face AQ
kks
Yo
oooo

sin C = sin r = l = _i_ = 0-6561 T2 = 4r


2 p 1-524
eBB

As r j + T2 = A ri = A-r2 = 60“-41®= 19®.


sini,
urr

As
sinr.
1
sin /j = p, sin r,
ad

sin I.1 = 1-524 sin 19® = 1-524 x 0-3256 = 0-4962 /i = sin-* (0-4962) = 29® 45'
YYo

Q. 22. A cardsheet divided into squares each of size 1 mm^ is being viewed at a distance of 9 cm through a
magnifying glass (a convei^ng lens of focal length 10 cm) held close to the eye.
dd

(a) What is the magnification produced by the lens ? How much is the area of each square in the
Re
inn

virtual image ?
(b) What is the angular magnification (magnifying power) of the lens ?
F

(c) Is the magniHcadon in (a) equal to the magnifying power in (b) ? Explain
Sol. (fl) Here, area of each (object) square = 1 mm^, « = - 9 cm,/= 10 cm.
As
1 1 1 1 1 1 1 1_9-10_ 1
V u / / u 10 9“ 90 90
V =-90 cm.

V 90
Magnification, m = — = 10
lul 9

Area of each square in virtual image = (10)^ x 1 = 100 sq. mm


d 23
(b) Magnifying power = — = — = 2-8
u 9
(c) No, magnification in (a) which is (v/u) cannot be equal to magnifying power in (b) which is (d/u)
unless V = d i.e. image is located at the least distance of distinct vision.
9/156 A Fundamental Physics (XII)
Q. 23. (i) At what distance should the lens be held from the card sheet in Q. 29 in order to view the squares
distinctly with the maximum possible magnifying power ?
(ii) What Is the magnification in this case ?
(iii) Is the magnification equal to magnifying power in this case ? Explain.
Sol. (0 Here, v = - 25 cm,/= 10 cm, « = ?
1 _ -2-5
As
V u f u~ V f~-25 10 ~ 50
-50
u = = -7.14 cm
7

25

oww
V
(ii) Magnification, m = = 3.5
l«l 7.14

25
(iii) Magnifying power = — = = 3.5
u 7.14
Yes, the magnification and magnifying power in this case are equal, because image is formed at the least

ee
rFFlo
distance of distinct vision.

r
Q. 24. What should be the distance between the object (in Q. 30) and magnifying glass if the virtual image

ree
F
of each square in the figure is to have an area of 6.25 mm^. Would you be able to see the squares
distinctly with your eyes very close to the magnifier ?

rF
Sol. Here. magnification in area = 6.25 Linear magnification m = = 2.5

fsfoor
ouur
V
As m = — or V = mu = 2.5 u
u
kosk
1 1 1 1 1
As
Yo
/ 2.5u 10
oo

V u u
Y
BB

1-2.5

2.5u 10
rre

2.5 u = - 15. or w = - 6 cm u = 2.5 u = 2.5 (- 6) = - 15 cm


oYuu

As the virtual image is at 15 cm ; whereas distance of distinct vision is 25 cm, therefore, the image cannot
ad

be seen distinctly by the eye.


dY

Q. 25. Answer the following questions :


(a) The angle subtended at the eye by an object is equal to the angle subtended at the eye by the
innd
Re

virtual image produced by a magnifying glass. In what sense then does a magnifying glass provide
angular magnification ?
Fi
F

(b) In viewing through a magnifying glass, one usually positions one’s eye very close to the lens.
Does angular magnification change if the eye is moved back ?
(c) Magnifying power of a simple microscope is inversely proportional to the focal length of the lens.
What then .stops us from using a convex lens of smaller and smaller focal length and achieving
greater and greater magnifying power ?
(d) Why must both the objective and eye piece of a compound microscope have .short focal lengths ?
(CHSE 2010)
(e) When viewing through a compound microscope, our eyes should be positioned not on the eye
piece, but a short distance away from it for best viewing, why ? How much should be that short
distance between the eye and eye piece ?
Sol. (a) It is true that angular size of image is equal to angular size of the object.
By using magnifying glass, we keep the object far more closer to the eye than at 25 cm, its normal position
without use of glass. The closer object has larger angular size than the same object at 25 cm. It Is in this
sense that angular magnification is achieved,
RAY OPTICS AND OPTICAL INSTRUMENTS 9/157

(/>) Yes, the angular magnification changes, if the eye is moved back. This is because angle subtended at
the eye would be slightly less than the angle subtended at the lens. The effect is negligible when image is
at much larger distance,
(c) Theoretically, it is true. However, when we decrease focal length, aberrations both spherical and chromatic
become more pronounced. Further, it is difficult to grind lenses of very small focal lengths.
f
id) Angular magnification of eye piece is 1+— . This increases as decreases.
\
So^ J

V
Further, magnification of objective lens is “ ● As object lies close to focus of objective lens u «/q. To
increase this magnification should be smaller,

ww
(e) The image of objective lens in eye piece is called ‘eye irng’. All the rays fiom the object refracted by the
objective go through the eye ring. Therefore, ideal position for our eyes for viewing is this eye ring only.
When eye is too close to the eye piece, field of view reduces and eyes do not collect much of the light. The
precise location of the eye ring would depend upon the separation between the objective and eye piece,

Flo
and also on focal length of the eye piece.

e
Q. 26. An angular magnification (magnifying power) of 30 X is desired using an objective of focal length

ere
1.25 cm and an eye piece of focal length 5 cm. How will you set up the compound microscope ?

FFr
Sol. In normal adjustment, image is formed at least distance of distinct vision, d = 2S cm.
( d 25^
uurr
Angular magnification of eye piece = 1 + — = 1 +— =6

orr
I, 4J I sfo 5j
As total magnification is 30, magnification of objective lens,
kks
V
30 = 5^
w = — m =
0
= 5
Yo
Uq = - 5 «o
oooo

or
6 -u
0

1 I 1 1 1 1
eBB

As
V
0
u
“0 /o -5 m
0
u
“0
1.25
urr

6 1

5mq 1.25
ad
YYo

-6x1.25
= -1.5 cm
5
dd

i.e„ object should be held at 1.5 cm in firont of objective lens.


Re
inn

As = -5 «o Vq = -5(-1.5) = 7.5 cm
F

From J___l 1
U.
e
M
e 4

4 4 -25 5 25

-25
M = -4.17 cm
e
6
.-. Separation between the objective lens and eye piece = I m^ I + I Wq I = 4.17 + 7.5 = 11.67 cm.
Q. 27. A small telescope has an objective lens of focal length 140 cm and eye piece of focal length 5.0 cm.
What is the magnifying power of telescope for viewing distant objects when
(a) the telescope is in normal adjustment (i.e. when the final image is at infinity)
(b) the final image is formed at the least distance of distinct vision (25 cm).
Sol. Here,/o = 140 cm,/^ = 5.0 cm.
Magnifying power = ?
9/158 Fundamental Physics (XII) VOL.II

/o _140_=
(a) In normal adjustment. Magnifying power = -28
-fe -5
{b) When final image is at the least distance of distinct vision,

Magnifying power = = -33.6


fA -5^ I 25 j

w
Q. 28. (a) For the telescope described in Q. 34, what is the separation between the objective lens and eye piece ?
(b) If this telescope is used to view a 100 m tall tower 3 km away, what is the height of the image of
the tower formed by the objective lens ?
(c) What is the height of the final image of the tower if it is formed at 25 cm ?

e
Sol. (a) Here, in normal adjustment,

e
Separation between objective lens and eye lens =/q +/g = 140 + 5 = 145 cm

o
rw
r
(b) Angle subtended by tower 100 m tall at 3 km

F
100 1
a = = — radian

ullo
3x1000 30

FF
h h

e
If /i is the height of image formed by the objective, then ct = —
fo 140

srre
oF
h 1
, 140

k
or h = cm. = 4.7 cm
140 ” 30 30

fofr
uor
o
(c) Magnification produced by eye piece = 1 + —
d >

fe^
= 1.^=6
5
Y
\ )
kos
Height of final image = 4.7 x 6 = 28.2 cm
Yo
rBB

Q. 29. A Cassegrainian telescope uses two mirrors as shown in Fig. 9(N).6. Such a telescope is built with the
oo
eY

mirrors 20 mm apart If the radius of curvature of lai^e mirror is 220 mm and the small mirror is
140 mm, where Mill the final image of an object at infinity be ?
re

Sol. Here, radius of curvature of objective mirror


u

/?!1 = 220 mm
d
ou
o

radius of curvature of secondary mirror


ad

/?2 = 140 mm ;
nY

, _ «2 _ I'M = 70 mm
/2-y
nid

2
Re

Mirror
F

Distance between the two mirrors, d = 20 mm.


Fi

When object is at infinity, parallel rays falling on objective


mirror, on reflection, would collect at its focus at

_ 220 = 110 mm
●'*2 2
Instead, they fall on secondary mirror at 20 mm from objective mirror.
.*. For secondary mirror, « =/j - = 110 - 20 = 90 mm
From
V u f2

V u 70 90 630 630

630
V =
— = 315 mm = 31*5 cm to the right of secondary mirror.
RAY OPTICS AND OPTICAL INSTRUMENTS 9/159

Q. 30. Light incident normally on a plane mirror attached to a galvanometer coil retraces backwards as
shown in Fig. 9(N).7. A current in the coil produces a deflection of 3-5‘’ of the mirror. What is the
displacement of the reflected spot of light on a screen placed 1*5 m away ?
Sol. Here, e = 3●5^ a:= l-5m,i/ = ?
FIGURE 9(N).7
When the mirror turns through an angle 0, the reflected ray A
turns through double the angle,
771 M'
rad d /
20 = 2x3-5° = 7° = /
180 70

From Fig. 9(N).7, tan 20 = il-A >


/
o
os ~ 1-5 1.5m
/

In I

ww
d= 1-5 tan 20 « 1-5 (20) =l-5x 180
m = 0*18 m M

Q. 31. Fig. 9{N).8 shows an equiconvex lens (of refractive index


1*5) in contact with a liquid layer on top of a plane mirror.
A small needle with its tip on the principal axis is moved

Flo
along the axis until its inverted image is found at the position

e
of the needle. The distance of the needle from the lens is

eree
measured to be 45*0 cm. The liquid is removed and the

FFr
experiment is repeated. The new distance is measured to
be 30*0 cm. What is the refractive index of the liquid ?
uurr
Sol. Let focal length of convex lens of glass = J\ = 30 cm,

orr
focal length of piano concave lens of liquid =/2
Combined focal length, F = 45-0 cm
sfo
kks
Yoo
1 I 1
As —+— J l_ 1
oooo

/l /2 F
fl 90
eBB

/2 = - 90 cm.
For glass lens, let R^ = R, R2 = - R

- = (|a-l)
1
^
urr

As
R
h
ad
YYo

I
3_|Y 1 n_ 1 ^ 2 _ I
30 2 Rj 2 R~ R
dd
Re

R = 30 cm.
inn

For liquid lens. R 1 - - R = - 30 0 cm. /?-> = «>


F

1 1 1 ^ 1 1
= %-l) or = (p,-l)x
fl R
i
-30 oo
-90 -30

30 1

90 3

. I 4
fl/ =!+-3 = 3
9/160 Fundamental Physics (XIDSaSHfl

1 WITH ANSWERS,
II HINTS AND SOLUTIONS

ll !| L, ssiwm

MULTIPLE CHOICE QUESTIONS-I

1. A ray of light incident at an angle 6 on a of yellow light is replaced with that of other
refracting face of a prism emerges from the lights without changing the anige of

w
other face normally. If the angle of the prism incidence ?
is 5'^ and the prism is made of a material of (<?) The beam of red light would undergo total
refractive index 1*5, the angle of incidence is internal reflection

Flo
(a) 7-5° (b) 5° (6) The beam of red light would bend towards

e
(c) 15° (d) 2-5° normal while it gets refracted through the

rree
second medium
2. A short pulse of white light is incident from

r FF
air to a glass slab at normal incidence. After (c) The beam of blue light would undergo total
internal reflection
travelling through the slab, the flrst colour to
uurr
emerge is
(a) blue
(c) violet
(b) green
(</) red
for
(d) The beam of green light would bend away
from the normal as it gets refracted through
the second medium
kss
6. The radius of curvature of the curved surface
3. An object approaches a convergent lens from
ooook
Yo
of a plano-convex lens is 20 cm. If the
the left of the lens with a uniform speed
refractive index of the material of the lens be
5 m/s and stops at the focus. The image
eB

1*5, it will
(a) moves away from the lens with unifomi speed
5 m/s
(a) act as a convex lens only for the objects that
lie on its curved side
urr

(b) moves away from the lens with uniform


ad

(b) act as a concave lens for the objects that lie


acceleration
Yo

on its curved side


(c) moves away from the lens with a non-uniform
dY

acceleration
(c) act as a convex lens irrespective of the side
on which the object lies
Re
innd

(d) moves towards the lens with a non-unifonn


(d) act as a concave lens irrespective of side on
acceleration
which the object lies
Fi

4. A passenger in an aeroplane shall


7. The phenomena involved in the reflection of
(a) never see a rainbow radiowaves by ionosphere is similar to
(b) may see a primary and a secondary rainbow (a) reflection of light by a plane mirror
as concentric circles
(b) total internal reflection of light in air during
(c) may see a primary and a secondary rainbow a mirage
as concentric arcs
(c) dispersion of light by water molecules during
(d) shall never see a secondary rainbow the formation of a rainbow

5. You are given four sources of light each one (d) scattering of light by the particles of air
providing a light of a single colour ~ red, blue, 8. The direction of ray of light incident on a
green and yellow. Suppose the angle of refrac concave mirror is shown by PQ while
tion for a beam of yellow light corresponding directions in which the ray would travel after
to a particular angle of incidence at the reflection is shown by four rays marked 1, 2,
interface of two media is 90°. Which of the
3 and 4, Fig. 9(N).9 Which of the four rays
following statements is correct if the source correctly shows the direction of reflected ray ?
RAY OPTICS AND OPTICAL INSTRUMENTS 9/161

rear and side mirror of his car after every 2 s


till the other car overtakes. If the two cars
were maintaining their speeds, which of the
following statement (s) is/are correct ?
-1
(a) The speed of the car in the rear is 65 km h
{h) In the side mirror, the car in the rear would
1
appear to approach with a speed of 5 km h
to the driver of the leading car
(c) In the rear view mirror, the speed of the
approaching car would appear to decrease as
the distance between the cars decreases
{d) In the side mirror, the speed of the
approaching car would appear to increase as
the distance between the cars decreases

w
ia) 1 {b)l
11. There are certain materials developed in
(c)3 (^0 4
laboratories which have a negative refractive
9. The optical density of turpentine is higher

Flo
index, Fig. 9(N).ll. A ray incident from air
than that of water, while its mass density is (medium 1) into such a medium (medium 2)
lower. Fig. 9(N).10 shows a layer of turpen

reee
shall follow a path given by
tine floating over water in a container. For

FFr
which one of the four rays incident on turpen FIGURE 9(N).11
tine in Fig. 9(N).10, the path shown is correct ?

for 1
ur
2
kkss
Yo
ooo

1
eB

2
r

(.a) 1 {b)2
ou
ad

(c)3 (^4 ic) 1


YY

10. A car is moving with a constant speed of


60 km on a straight road. Looking at the
/////////7
2
nndd
Re

rear view mirror, the driver finds that the car


(d)
following him is at a distance of 100 m and is 1
Fi

approaching with a speed of 5 km h~^. In I! I! i! I! I!


order to keep track of the car in the rear, the " 2
driver begins to glance alternatively at the

MULTIPLE CHOiCE QUESTIONS^MI


12. Consider an extended object immersed in {b) the angle subtended by the image of the
water contained in a plane through. When object at the eye is smaller than the actual
seen from close to the edge of the trough, the angle subtended by the object in air.
object looks distorted because (c) some of the points of the object far away from
{a) the apparent depth of the points close to the the edge may not be visible because of total
edge are nearer the surface of the water internal reflection.

compared to the points away from the {d) water in a trough acts as a lens and magnifies
edge. the object.
9/162 Fundamental Physics (XII) VOL.II

13. A rectangular block of glass ABCD has a (b) there is no light scattered into this region,
refractive index 1.6. A pin is placed midway (c) light is absorbed in this region.
on the face AB, Fig. 9(N).12. When observed (d) angle made at the eye by the scattered rays
from the face AD, the pin shall with respect to the incident light of the sun
lies between approximately 42° and 50°.
FIGURE 9(N).12
A B
15. A magnifying glass is used, as the object to be
viewed can be brought closer to the eye than
the normal near point. This results in
(a) a larger angle to be subtended by the object
at the eye and hence viewed in greater detail.
ib) the formation of a virtual erect image,
(c) increase in the field of view.
0 C (J) infinite magnification at the near point.

ww
16. An astronomical refractive telescope has an
(a) appear to be near A. (b) appear to be near D. objective of focal length 20 m and an eyepiece
(c) appear to be at the centre of AD. of focal length 2 cm.

Floo
(d) not be seen at all. (a) The length of the telescope tube is 20-02 m.
14. Between the primary and secondary (_b) The magnification is 1000.

ee
rainbows, there is a dark band known as
(c) The image formed is inverted.

eer
Alexandar’s dark band. This is because
(d) An objective of a larger aperture will increase
(a) light scattered into this region interferes

FFr
the brightness and reduce chromatic
destructively.
aberration of the image.

oorr
uur r
s ff
ANSWERS
1. (a) 2. id) 3. (c) 4. (h) 5. (c) 6. (c) 7. ib) 8. (b)
sk
YYoo
ooko

9. (b) 10. (d) 11.(«) 12. (a, b. c) 13. (a, d) 14. (a. d) 15. («, b) 16. (a. b, c)
eBB

HINTS FOR DIFFICULT MULTIPLE CHOICE QUESTIONS


uurr

Multiple Choice Questions -1


ad

1. Here, A = 5°, M= 1-5,/ 1 - 9


Yo

- ●

As the ray emerges from the other face of prism nonnally,


H = 0°. r, = 0°,Fig.9(N).13.
dY
Re

As ri+ri = A
ind

=A-r2 = 5-0 = 5'


FFin

sm
From lt = - . sm i
= |i sin r| = 1-5 X sin 5°
sm ,r
1

= 1-5 x0-087 = 0-1305

i, =sin-' (0-1305) = 7-5°


2. In air, all the colours of light travel with the same velocity, but in glass, velocities of different colours are
different. Velocity of red colour is largest and velocity of violet colour is smallest. Therefore, after travelling
through the glass slab, red colour will emerge first.
3. When an object approaches a convergent lens from the left of the lens with a uniform speed of 5 m/s, the
image moves away from the lens with a non uniform acceleration. For example, if / = 20 m and
f = -50 m ; - 45 m. - 40 m and - 35 m ; we get i> = 33-3 m ; 36 m ; 40 m and 46-7 m. Clearly, image moves
away from the lens with a non uniform acceleration. Choice (c) is correct.
4. In an aeroplane, a passenger may observe a prinKU"y and a secondary rainbow as concentric circles.
RAY OPTICS AND OPTICAL INSTRUMENTS 9/163

5. Here, for yellow light, r = 90° when i = C. As i is kept same, C must be smaller for total internal reflection.
1
From |i = C will be smaller, when p is larger. Out of given colours, |i is largest for blue colour.
sinC ’
Critical angle will be smallest for blue colour. Therefore, blue light would undergo total internal reflection.
Choice (c) is correct.
6. Here, p = 1 -5
If object lies on plane side ; = OO
, /?2 = - 20 cm
FIGURE 9(N).14
1 1 1 1
- = (^t-l) —- = (1-5-1) -+—
0
f R Rn OO 20 40
I “ y O
/= + 40 cm. The lens behaves as convex.
If object lies on its curved side, R^ = 20 cm, /?2 = Fig. 9(N).14(/?)

llowow
o
1 1 I '1 1 O
= (1-5-1)
r /?, /?2 20 OO 40

/' = 40 cm. The lens behaves as convex. Choice (c) is correct.


7. The phenomenon involved in the reflection of radiowaves by ionosphere is similar to total internal reflection

ee
of light in air during a mirage (angle of incidence> critical angle). Choice (b) is correct.

Fr
8. In Fig. 9(N).9, PQ is a ray of light passing through focus, and falling on the surface of a concave mirror. On
reflection, from the mirror, the ray becomes parallel to principal axis of the mirror. Choice (b) is correct.
9. In Fig. 9(N).15, the path shown for the ray 2 is correct. The ray
r FF
er
suffers two refractions : At A, ray goes from air to turpentine,
bending towards normal. At B, ray goes from turpentine to water
fofr For
u
{i.e., from denser to rarer medium), bending away from normal.
ks
10. As is known, in the side mirror, the speed of approaching car
YYour o
s oo

would appear to increase as the distance between the cars


decreases. Choice {d) is correct.
eeBoBk

Sint
11. According to Snell’s law. sinr
r
ouru

sm I
ad

or sin r —
Yo

As }i is negative ; sin r is negative. r is negative. The path shown in Fig. 9(N).ll(a) is correct.
d

Multiple Choice Questions - H


Re
iYn

12. Here, an extended object lies immersed in water contained in a plane trough. When seen from close to the
edge of the trough, the object looks distorted on account of refraction of light from denser to rarer medium.
FFind

Therefore, apparent depths of the points close to the edge and nearer to the surface of water is more
comparedto points away from the edge. Further, the angle subtended by the image of the object at the eye
is smaller than the actual angle subtended by the object in air. Again, some of the points of the object, far
away from the edge may not be visible because of total internal reflection.
FIGURE 9(N).16
The choices (a), {b), (c) are correct,
13. in Fig. 9(N). 16, a pin is held at L, mid point of AB. When seen from face AD
(.so long as / < O image of L appears to be at L', closer to A.

From sin C = — — = 0-625, C = sin"' (0-625) = 38-7°.


\i 1-6
So when angle of incidence becomes greater than C (= 38-7°), the rays starting
from L will undergo total internal refection and pin shall not be seen at all.
The choices (a) and (d) are correct.

I
9/164 ^ Fundamental Physics (XII) kV«Tn
14. Alexandar’s dark band between the primary and secondary rainbows is because light scattered into this
region interferes destructively. Further, primary rainbow subtends an angle of 41M3° at the eye of the
observer w.r.t. the incident light, and secondary rainbow subtends an angle of about 5 T to 54° at the eye of
the observer w.r.t. the incident light. Therefore, the region between the angles of 41° to 51° is dark. Choices
(a) and (d) are correct.
15. A magnifying glass is used as the object to be viewed can be brought closer to the eye than the normal near
point. This results in a larger angle to be subtended by the object at the eye and hence viewed in greater
detail. Also, it results in the formation of a virtual, erect image. Choices (a) and (b) are correct.
16. Here,/^ = 20 m and/^ = 2 cm = 0-02 m.
In normal adjustment, length of telescope tube L=f^+f^ = 20-02 ni

Magnification =1^ = 0-02 = 1000.


4

lowow
The image formed is inverted {w.r.t. the object). Choices (<i), (b) and (c) are correct.

S’TIONS
17. Will the focal length of a lens for red light be more, same or less than that for blue light ?

ee
Fr
1
Ans. From lens maker's formula -=(^-1)
r FF j 1_
/ R,1 R-,
- y

rer
As fb<fr or 4 >4
fofr Fo
i.e., focal length of a lens for red light will be more than the focal length of lens for blue light.
u
18. The near vision of an average person is 25 cm. To vievr an object with an angular magnificationof 10,
ks
what should be the power of the microscope ?
YYouro
oo

Ans. Here, d = 25 cm, m = 10, P = ?

25 100
BBo ks

— = 2-5 cm = 40D
Angular magnification, m = — 10 / 2-5
r ee

/ m

19. An unsymmetrical double convex thin lens forms the image of a point object on its axis. Will the
ouru

position of the image change if the lens is reversed ?


ad

Ans. No, the position of image will not change if the lens is reversed. This is because in the lens maker’s formula :
Yo

1 1
_L_ —
● =-(^-1) ^
d
Re

V u / R, Rr, /?.
iYn

i.e., on reversing the lens, values of R^ and R^ are reversed and so are their signs. Hence, for given u
FFind

(position of object), position of image (d) remains unaffected.


20. Three immiscible liquids of densities d^> di> d^ and refractive indices ^ ^ ^
h
beaker. The height of each liquid column is — . A dot is made at the bottom of the beaker. For near

normal vision, find the apparent depth of the dot.


Ans. Here, real depth of the dot under liquid of density d^ is h/3. If Xj is its apparent depth, when seen from air,
hl3 h
then from
^^1 = or =

Similarly, apparent depths of the dot when seen from air through two other liquids are
h h
^2 — and

t
RAY OPTICS AND OPTICAL INSTRUMENTS 9/165

Apparent depth of the dot seen from air through the three liquids is

.r = j:, -hJC2 +-*^3 = h h J_ + _L + jL


^ ^*^2 ^^3
21. For a glass prism (p = -73 ), the angle of minimum deviation is equal to the angle of the prism. Find
the angle of the prism.
Ans. Here p = 5„, = A
sin(A + S^)/2
From prism formula |X =
sin A/2

ooww
sin A 2 sin A/2 cos A/2
3^ = = 2 cos A/2
sin A/2 sin A/2

cos A/2 = - cos 30°


2

e
A/2 = 30“ or A =60'

ree
rFl
Fre
rrF
22. A short object of length L is placed along the principal axis of a concave mirror away from focus. The
object distance is u. If the mirror has a focal length/, what will be the length of the Image ? You may
ouur
take L « I V -/1.
sffoo
Ans. Let the two ends of the object be at distances u i = {u — U2) and U2 = {u + U2) from the pole of concave
mirror, so that \ u^ - ii2 \ = L = size of the object.
okks
Let the images of the two ends be formed respectively at v y and V2 on reflection from the concave mirror.
Yo
ooo

Size of image = L' = I u, - ^2 I


BB

From mirror formula, — + — = ^


rr e

V u /
ouu

I 1 1 u-f
ad
YY

or v =
V f u
u-f
dd

fUy fiu-U2) fU2 f(u + U2)


V
and 1^2 “
Re

1 “
iinn

Uy-f iu-f-U2) Un-f (u-f+m)


F

f{u-U2) f{u + U2)


L'=\Vy-V^ 1=
{u-f-UD {ii-f + U2)

(«-/) (/«-
2 ^ 2 2V 2 ■’ 2

L' = (u-f) (-/L) + |.2> f^L


(«-/)2-L2/4
As the object is short and kept away from focus, therefore, L^/4 « (u -f)^.

Hence, V =
pL
(u-f?
9/166 ‘P’utdce^ 4 Fundamental Physics (Xll)
23. A circular disc of radius ‘if’ is placed co-axially and
horizontally inside an opaque hemispherical bowl of radius
‘a’, Fig. 9(N).17. The far edge of the disc is just visible when
viewed from the edge of the bowl. The bowl is filled with
transparent liquid of refractive index )i and the near edge
of the disc becomes just visible. How far below the top of
the bowl is the disc placed ?

w
Ans. In Fig. 9(N).9, we have shown an opaque hemispherical bowl
of radius a with centre 0.

A circular disc of radius R with centre C is placed co-axially and horizontally inside the bowl. We have to
calculate OC = d.

AMA' is the direction of incident ray before liquid is filled in the bowl.

e
row
When liquid of refractive index p is filled in the bowl, the near edge B of the disc just becomes visible.

re
Here, BM is the incident ray, which is refracted along MA'.
NN’ is normal to the horizontal surface of liquid in the bowl. / is the angle of incidence and r = a is the

FFllo
eeF
angle of refraction.
As refraction occurs at M in going from denser to rarer medium, therefore, according to Snell’s law

u 1 smi sin;

r
...(/)

sFr
sin r sm a

kro
AN' a + R
uor
BN' a-R
Now, smi = offo and sin a = cos (90-a) -
BM
-^d^ + ia-R)^
AM
■^jd^ -l-{a + R)^
kos
Y
Putting in (0, we get
Yo
eerBB
oo

1 a-R Jd'^ +{o + R)^


x-^ or p X {a - R) = {«-(-/;) -Jd^ + ia-R)^
1^ ^d^+(a-R)~ a + R
rY
u

On simplifying, we get
V(fl + /?)^-p (a-R^
ou
ad
do

24. A thin convex lens of focal length 25 cm is cut into two pieces 0*5 cm above the principal axis. The top
nY

part is placed at (0,0) and an object placed at (- 50 cm, 0). Find the coordinates of the image.
Ans. In Fig. 9(N). 18, AB is the principal axis of original
nid

FIGURE 9(N).18
Re

thin convex lens of/= 25 cm. It has been cut into


F

two pieces 0-5 cm above the principal axis. 50 cm 50 cm


Fi

O.^ m
The top part is placed at (0, 0) and object 0 is at
(- 50 cm, 0). X X'
A B ^ 0.5cm
If there were no cut, the object would have been
J(50
I I
cm, - 1cm)
at a height 0-5 cm from principal axis AB. \ I
I
I
I
s

As l_i ^ \
V /
/

V u /
1
L-l+l = -l D = 50 cm
V / u 25 50 50

V 50
Magnification, p = = -l
u -50

Thus, the image would have been formed at 50 cm from the pole on irght side of lens and 0-5 cm below the
principal axis. Hence, w.r.t. the X-axis passing through the edge of the cut lens, the co-ordinates of the
image are (50 cm, - 1 cm).
RAY OPTICS AND OPTICAL INSTRUMENTS 9/167

25. In many experimental set-ups the source and screen are fixed at a distance say D and the lens is
movable. Show that there are two positions for the lens for which an image is formed on the screen.
Find the distance between these points and the ratio of the image sizes for these two points.
1 1 1
Ans. In the lens formula, — = (0
/ u

we find that u and v are reversible. Therefore, there are two FIGURE 9(N).19
positions of the object, for which there will be an image on
the screen. O c i
Let the first position be when lens is at C, Fig. 9(N).19.
t: 1 i
1

oww
u ■> 4- V
.\-u + v = D = distance between source (O) and screen /.
or u = -(D-v) K l£ns_D
Put in (0
1 1 1 D-v+v 1
or Dv-v^ = Df or v^-Dv + Df=0.

e
- + or
V D-v f v{D-v) f

re
FFrllo
D±^D^-4Df _D ^^D^-4Df D
^4p^-4Df

rF
V =
and u = -(D-v) =-
2 2 2

ee
2 2
ouru
sor rF
Thus if object distance is m = —
D
, then image distance is u = —
D Vd^-4D/
2 2 2 2

Again, if object distance is u =


D ^D^-4Df ,
os kffo
then image distance is v = —
D
2 2 2
ook
Yo
Y
The distance between two positions of the lens for these two object distances is
Bo
reeB

^_D^ -4Df D ^ = ^D'^-4Df


2 2 2 2
ouY

D d V D-d
ur

If u = - +- then ^ -T
2’ Magnification, =—
ad

2 2 2
Yo

u D +d
d

T.
If
D d D d V D +d
then ^ ~ —
2 2 2 2 Magnification, m2 = —
nidn

u D-d
Re

26. A jar of height h is filled with a transparent liquid of FIGURE 9(N).20


refractive index p. Fig. 9(N).20. At the centre of the jar on
F
Fi

the bottom surface is a dot. Find the minimum diameter of ♦ B

a disc, such that when placed on the top surface F


i

symmetrically about the centre, the dot is invisible. I


I

Ans. In Fig. 9(N).21, O is a dot on the bottom surface of a jar of --1


T

height h, filled with a transparent liquid of refractive index p.


--4
ih
i.li
AB = d is diameter of a disc such that when placed on the top 4
surface, symmetrically about the centre, the dot is invisible.
I
I i L

This would happen when rays OA and OB suffer total internal


reflection. i

If ZAOC = I, T
4
. 1 .
I

then tan / = AC ^ dll m


OC h
O
9/168 ^>t4xdcep. '<1 Fundamental Physics (XII)
For total internal reflection, / > c,
1 1
when sm i = sin c = — and tan / - ,Fig. 9(N).21.

d 1
From (/) and (ii).
2h

2h

-1

27. A myopic adult has a far point at 0-1 m. His power of accomodation is 4 diopters, (i) What power
lenses are required to see distant objects ? (ii) What is his near point without glasses ? (iii) What is his
near point with glasses ? (Take the image distance from the lens of the eye to the retina to be 2 cm.)

ww
Ans. (0 Distance of far point, « = - 0-1 m, distance of image, u = 2 cm = 0 02 m.
1 1 1 1 1
+ — = 50 + 10 = 60 D
Power of myopic eye, Pj- = / 002 0-1

FF loo
V u

1 1
With the corrective lens, far point shifts to Power required, Pj = — = 50D

ree
0-02 CO

Required power of glasses, P^ = Pj - Pj= 50 - 60 = - 10 D.


{ii) Power of accommodation = AD

rFee
- 60 or P..H = 64 D.
If is power of normal eye for near vision, then 4 = - Pj= P

oor rF
n
rur
1 1
If ;c„ is near point withoiit glasses, then — + = 64 or — = 14, X = — m = 0 07 m
s ff
" X 002 X n 14
n
n

{Hi) With glasses, P„' = F/ + 4 = 50 + 4 = 54


k
YYoou
1
ookos

-L J_ = J = 54 ● = 54-50= 4
.r n/ 0-02 Xn X
BBo

1
re

' = — m = 0*25 m
n
4
ouur
ad
Yo

LONG ANSWER QUESTIONS

28. Show that for a material with refractive index ^ > V2 »light incident at any angle shall be guided
Yd
Re

along a length perpendicular to the incident face.


idn

Ans. In Fig. 9(N).22, we have shown a ray of light incident at Zi on face


FFin

AB. It is refracted at Zr and falls on face AD ± AC at Z(S). This ray


will suffer total internal reflection at K and proceed along KD _L
AB, provided
1
sin^ > —

1 1
sin (90“ - r) > - or cos r > —

1 1
or 1 - cos- r < 1 r or sin- r < 1 ^

1
According to Snell’s law, sin i = p sin r -Lsin2/<1
sin^ ( < ()i^ - 1)
RAY OPTICS AND OPTICAL INSTRUMENTS 9/169

0 will be smallest when i =» 90°

sin^ / = sin^ 90” =1 1< - 1 or \i^>2 or ^ > V2 , which was to be proved.


29. The mixture of a pure liquid and a solution in a long vertical column {i.e., horizontal dimensions «
vertical dimensions^ produces diffusion of solute particles and hence a refractive index gradient
along the vertical dimension. A ray of light entering the column at right angles to the vertical is
deviated from its original path. Find the deviation in travelling a horizontal distance d « h, the
height of the column.
Ans. In Fig. 9(N).23, we have shown a ray of light entering the column at
(x, y) at 90° to the vertical. A portion of ray between .y and (x + dx)
deviates through an angle y/0, emerging at (.y + dx, y + dy) at angle
(6 + dQ), while the angle at the entry is 9.
According to Snell’s law,

ww
p (y) sin 6 = p (y + dy) sin (9 + dQ)

d[i

Floo
= |i(y) + dy (sin 0 cos dQ + cos 0 sin dQ)
dy

ree
As dQ is small, cos dQ ~ 1 and sin dQ = dQ. dx

rFee
d\x
p (y) sin 0 = p (_v) sin 0 + p (y) cos 0^/0 + dy sin 0 (fourth term is negligibly small)
dy

F
oor r
rur
Jp
p (y)cos0 dQ = - dv sin 0
s ff
dy
osk
1 d\i
YYoou
dQ = dy tan0
oook

p dy
eBB

dx
From Fig. 9(N).23. tan 0 = — or d\ tan Q = dx
dy
uur r

1 c/p
ad

dQ = - dx
Yo

p dy
1 , d
1 c/p ● 1 c/p
dY

9 = - dx= - id)
Re

p dy lA dy
idn

0
FFin

This is the required deviation in travelling a horizontal distance d.


30. If light pa.sses near a mas.sive object, the gravitational interaction causes a bending of the ray. This
can be thought of as happening due to a change in the effective refractive index of the medium given
by « (r) = 1 + 2 GM/rc^
where r Is the distance of the point of consideration from the FIGURE 9(N).24
centre of the mass of the massive body, G is the universal
gravitational constant, M the mass of the body and c the speed
of light in vacuum. Considering a spherical object, find the
deviation of the ray from the original path as it grazes the
object.
Ans. In Fig. 9(N).24, we have shown a spherical massive body of mass
M and radius R. Consider two planes one at r and other at
(r + dr) from the centre of massive body. Let the light rays be
incident at an angle 0 on the plane at r and leave the plane at
{r + dr) at an angle (0 + dQ). {r+dr) ^
9/170 ^ Fundamental Physics (XII) VOL.II

According to Snell’s law.


dn
n (r) sin 0 = n (r + dr) sin (0 + dQ) = n (r) + — dr (sin 0 cos dQ + cos 0 sin dQ)
dr

As dQ is small, cos dQ - I and sin dQ = dQ.


Neglecting products of differentials, we get
dn dn
n (r) sin 0 = rt (r) sin 0 + n (r) cos QdQ+ — dr sin 0 or </rsin0 = n (r)cos0 dQ
dr

dn dQ
or tan 0 = n (r) — .(0
dr dr

2GM dn -2GM

ww
As n(r) =1 + 2 2 '>
rc dr r^ c~

2GM f 2GM ,

FF loo
tan 0 = 1+
Put in (0 ,2^2 rc 2 dr dr rc
/

ree
d0 =
2GM tan0 j

reFe
Integrating both sides, we get

ororF
rur
+0O

dQ = 2GM^ tan0f/r _2GA/ r


tan0rc?r
s ff
0
~c^ J J
—oo
r3
k
YYouo
R
okso

Now, from Fig. 9(N).24, = and tan 0 - —


BBoo

4

r ee

2GM r Rxdx
2 r dr = 2 xdx dQ =
0
c2 J jc(jc2+/?2)3/2
ad
ouur

If we put jc = /? tan ([) so that dx = R sec^ 0 #,


Yo

1
lt/2

then dQ =
2GM f /?2 sec^(j)<j(})
c2 J
d
idnY

R^ sec^ ({)
Re

0 -nil
FFin

71/2
2GM 4GM
0
0 " cos(j) d^ =
Rc^ Rc^
-Jt/2

This is the deviation for the ray from the original path as it grazes the massive body.
31. An infinitely long cylinder of radius R is made of an unusual exotic material vdth refractive index
(-1), Fig. 9(N).25. The cylinder is placed between two planes whose normals are along they direction.
The center of the cylinder O lies along they-axis. A narrow laser beam is directed along they direction
from the lower plate. The laser source is at a horizontal distance x from the diameter in they direction.
Find the range of x such that light emitted from the lower plane does not reach the upper plane.
Ans. The set up described in the question is shown in Fig. 9(N).25. The full line circle represents cross section
of infinitely long cylinder of radius R. As the material is of refractive index (- 1), therefore, 0^ is negative
and 0/ is positive.
Also, 10,.1 = 10,1 = 10/1
For the incoming ray along Y-axis from the lower plane, total deviation of the outcoming ray is 4 0,-. The
rays shall not reach the receiving plate if
RAY OPTICS AND OPTICAL INSTRUMENTS 9/171

n 37C FIGURE 9(N).25


_<40. <_
2 2 y Upper (receiving) plane
(angles are measured clockwise from Y-axis)
K 3k
or -<0. <—.
I
8
/

.Y
R a ^
From Fig. 9(N).25, sin6- = — = 0., when small.
R
9'' 'A JC

. Tl ^ X ^3k Rn 3k/? \

or < .Y <
8 " /? “T 8 0.>I

Hence, we conclude that for values of x lying between 1

ooww
X
kR 3nR Lower plane
and . light emitted from the laser source

shall not reach the upper receiving plane.


32. (i) Con.sider a thin lens placed between a source (5) and an observer (O), Fig. 9(N).26. Let the thickness

e
ere
of the lens vary as w (6) = —, where /> is the vertical distance from the pole. Wn is a constant,

rFl
H*
0
a ^

Fre
Using Fermat’s principle, i.e., the time of transit for a ray between the source and observer is an

rrF
extremum, find the condition that all paraxial rays starting from the source will converge at a point
O on the axis. Find the focal length,

sffoo
ouur
(ii) A gravitational lens may be assumed to have a varying width of the form
/'it
*2 ^
kosk
*2
w (/>) = /:, In ~ max H- (ft) = *jln -
Yo
min
oo
Y

Show that an observer will see an image of a point object as a ring about the center of the lens with
BB

u
rre

(«-l)
an angular radius p - jl,
ouu

u + v
Y
ad

Ans. Refer to Fig. 9(N).26. Time required by light to travel in air


dY

from S to P, is

5/^ ^ V»-+ft-
innd

r
Re

1
c c
Fi
F

u
1 ^
c 2 u- ’ assuming b <<u

Similarly, time required by light to travel in air from Pj to O ;


V

Now. thickness of lens varies as M'(ft) = ,W0 ...(0


a

Time required to travel through the lens is


w(ft) w{b) in~l)w(b)
^3 “ c/n c c

Here, n is refractive index of the material of the lens.


9/172 Pxeidee^ '<t Fundamental Physics (XII) VOL.II

Thus, the total time of travel from 5 to O is


b^ V (n-l)w{b)
/
u ^ V
1+
^ ^ ^ c 2«2 C 2u2
/
c

u + v-V-b^ i + - +(n-\)w(b)
1 1

c 2 u V
/

Let us put i + i= ' ,(w)


U V D
/

c
M + U +
2D
+ («-!) Wq a .(*«)
/-I

w
dt
According to Fermat’s principle, / = extremum. Therefore
2(n-l)h = 0 m

Flo
From {iii). or a = 2(n-l)D
db cD cot.

e
Hence, a convergent lens is formed if a = 2 (n — 1) D. This is independent of b. Hence, all paraxial rays

reee
from 5 will converge at O.

Fr
From (ii), — + — = — . Therefore, focal length of lens = D.
u V D

for
ur
(k ^
(«) Now, for a gravitational lens. w (*) = <:, log. f /
:*min max
ks
( *2
k ''
Yo
w (b) = k, log. -
oo

^ min ^
eB

Proceeding as in the above case, from eqn. (iti)


1 1 b^ ^2
ur

U + V + -
c
ad
Yo

dt b k,
— = 0 =
db D b
d
Re

b^ = in-l)k^D
in

b = ^(n-l) k^D
F

Hence, all rays passing through the lens at a height b shall form the image.
The paths of rays would make an angle

b ^(n-l)k^D _ l(n-l)kfUV f as —1 1
= —+ —
n
P=-
V V v^(u+v) I ^ u
vj

in-l)k^u/v
P=
^ (« + v)
Hence, the observer will see an image of a point object as a ring about the centre of the lens with an angular
radius p.
RAY OPTICS AND OPTICAL INSTRUMENTS 9/173

i^d
iMP

ML'
'r-

NEET/JEE
SPECIAL

For ultimate preparation of this unit for competitive examinations, students should refer to

ww
Pradeep’s Stellar Series.... ● MCQs In Physics for MEET
● MCQs in Physics for JEE (Main)
r

separately available for these examinations.

Floo
ee
a Multiple Choice Questions (with One Correct Answer)

eer
FFr
1. Reflection of Light

oorr
uur r
FIGURE 9(CF).2

1. A ray of light travelling in a direction


s ff CO

1a /r a . (b)
— {i + V3 y ) is incident on a plane mirror. After
sk
YYoo
ooko

reflection, it travels along the direction


eBB

1a
— {i - V3 y). The angle of incidence is : (c) (d) CO-4--
uurr

(a) 30° a<45*’


(b) 45°
ad

I
(c) 60° id) 75° j.
Yo

2. A wire is bent in the shape of a right angled


triangle and is placed in front of a concave mirror (JEE Advanced, 2018)
dY

of focal length ,/ as shown in Fig. 9(CF). 1. which 3. In a concave mirror, an object is placed at a
Re

of the figures shown in the four options distance from the focus and the real image is
ind

formed at a distance d2 from the focus. Then the


FFin

qualitatively represents the shape of the image of focal length of the mirror is :
the bent wire ? These figures are not to scale.
(a) ib) d^d2

(c) {d^ + d2)/l (d)

4. A short linear object of length b lies along the


axis of a concave mirror of focal length / at a
P
distance u from the pole of the mirror. The size
of the image is approx, equal to ;
n1/2 xl/2
u~f f
(a) b ib) b
I f li’f

1. («) 2. id) 3. ((0


9/174 ^ Fundamental Physics (XIl)k^«TWH

»-/ / FIGURE 9(CF).4


(c) b id) b
u-f Si S2
P
5. A car is fitted with a convex side-view mirror of
focal length 20 cm. A second car 2-8 m behind 50cm d

the first car is overtaking the first car at relative


speed of 15 m/s. The speed of the image of the (fl) 60 cm {h) 70 cm
second car as seen in the mirror of the first one is (c) 80 cm {d) 90 cm
1 {JEE Advanced 2015)
(a) m/s (ft) 10 m/s
15 9. An observer can see through a pin hole, the top
1 end of a thin rod of height ft, placed as shown in
(c) 15 m/s {d) —m/s Fig. 9(CF).5. The beaker height is 3 ft and its
10
radius is ft. When the beaker is filled with a liquid

looww
(AIEEE 2011)
upto a height 2 ft, he can see the lower end of the
6. An object is placed on the principal axis of a rod. Then the refractive index of the liquid is :
concave mirror at a distance of 1 -5 / (f is focal
length). The image will be at
(«)3/ (ft)-3/

ree
(c) 1-5/ (i/)-1-5/(NEET 2020)

ree F
II. Refraction of Light
7. Consider a concave mirror and a convex lens
r FF
(refractive index 1-5) of focal length 10 cm each fofroF
u
separated by a distance of 50 cm in air (refractive
ks
index = 1) as shown in the Fig. 9(CF).3. An object
is placed at a distance of 15 cm from the mirror.
os o
YYouor

Its erect image formed by this combination has


magnification Afj. When this set up is kept in a
BBook

medium of refractive index 7/6, the magnification


r ee

becomes M2. The magnitude (M2/Mj) is : 5


(a)- (ft) \2
A
ouru
ad

FIGURE 9(CF).3

A
Yo

I
3
(c) (IIT 2002)
d

10. For a given incident ray as shown in Fig. 9(CF).6,


Re

V
cm
inY

the condition of total internal reflection of ray will


FFind

50 cm - be satisfied if the refractive index of the block


will be
(a) 5 (ft) 6 (c) 1 (d) 8
(JEE Advanced 2015)
8. Two identical glass rods and ^2 (refractive
index = 1-5) have one convex end of radius of
curvature 10 cm. They arc placed with the curved
surfaces at a distance d as shown in Fig. 9(CF).4,
with their axes (shown by the dashed line) aligned.
When a point source of light P is placed inside
rod Sj on its axis at a distance of 50 cm from the
curved face, the light rays emanating from it are
found to be parallel to the axis inside S2. The
distance d is
ANSWERS

4.(rf) 5. {a) 6. (ft) 7. (c) 8. (ft) 9. (ft)


RAY OPTICS AND OPTICAL INSTRUMENTS 9/175

^3 +1 yfl +1 3^
(a) ih) (a) cos * (h) sin
-I
2 2 4 4

(C) (iO (AIPMT 2002) _.f 4 -rMl


\6 (c) tan ' —
V2
3 (J) col ' J
II. The graph in Fig. 9(CF).7 shows how the inverse
of magnification I/m produced by a convex thin (AlIMS 2015)
lens varies with object distance u. What was the 14. Four combinations of two Uiin lenses are given
focal length of the lens used ? in List I. The radius of curvature of all curved
surfaces is r and the refractive index of all the
lenses is 1-5. Match lens combinations in List 1
with their focal length in List II and select the

ww
correct answer using the code given below the
lists.
List-I List-Il

Floo
ee
(A) (p) 2r

eer
b h
(«) - ib) ~

FFr
c ca

oorr
uur r
be c
(c) —
a
s ff r
(B)
12. Sunlight of intensity 1-3 kWm"^ is incident i<i) 2
sk
YYoo
normally on a thin convex lens of focal length
ooko

20 cm. Ignore the energy loss of light due to the


lens and assume that the lens aperture size is much TV
eBB

smaller than its focal length. The average intensity


of light, in kWm”-, at a distance of 22 cm from (C) (r) -r
the lens on the other side is
uurr

(a) 130
ad

{b) 13
Yo

(c) 1300 id) 13000


(JEE Advanced, 2018)
dY

13. Consider the ray diagam for the refraction given (D) (,9) r
Re

Fig. 9(CF).8. The maximum value of angle 6 for


ind

which the light suffers total internal reflection at VL\


FFin

the vertical surface, is


Code :
FIGURE 9(CF).8
{a) A-p, B-q, C-r, D-.s (h) A-cp B-i, C-r, D-;;
(c) As, B-p, C-q, D-r (d) A-q, B-p, C-r, D-j
(JEE Advanced 2014)
15. A ray of light falls on the surface of a spherical
glass paper weight making an angle a with the
normal and is refracted in the medium at an angle
p. The angle of deviation of the emergent ray from
the direction of the incident ray is :
(a) (a-P) ih) 2(a-P)
(c) (a-P)/2 id) (P-(X)
ANSWERS
10. (c) 11. (d) 12.(0) 13. f/j; 14. -
9/176 U Fundamental Physics (XII)

16. Light incident on a surface separating two media


is partly reflected and partly refracted as shown
in Fig. 9(CF).9. Then :

(a) Separate the red colour part from green and


blue colours

(b) Separate the blue colour part from the red and
green colours
(c) Separate all the three colours from one another.

w
(d) Not separate the three colours at all
(CBSE Sample Paper 2019-20, RE-AIPMT 2015)
19. A piano convex lens has focal length /= 20 cm.

Flo
(a) sin / = If its plane surface is silvered, then new focal
length will be

ee
(a) 20 cm (b) 5 cm
1^]

Fr
(b) tan i = (c) 10 cm (cf) 25 cm
^2
20. In an optics experiment, with the position of the
(c) sin/ = PiP2 object fixed, a student varies the position of a
for
ur
convex lens and for each position, the screen is
adjusted to get a clear image of the object. A graph
1^2 between the object distance u and image distance
ks
(d) sin / =
V from the lens, is plotted using the same scale
Yo
1^1
oo

for the two axes. A straight line passing through


17. A slab of transparent material is made as shown origin and making an angle of 45° with the j:-axis
eB

in Fig. 9(CF).10. Monochromatic parallel beams meets the experimental curve at P. The coordinate
of light are normally incident on the slab. The of P will be
thickness of C is twice the thickness of B. If the
ur

number of waves in A = number of waves in / /


(b) (f,f)
ad

(a)
combination of B and C, then the refractive index 2’ 2
Yo

of B is :
(c) (4f,4f) id) i-2f,2f)
FIGURE 9(CF).10 (AIEEE 2009)
d
Re

21. A thin convex lens made from crown glass


in

A
*'C-
\.y
(|i = 3/2) has focal length ./ When it is measured
.-■Mb:
F

in two different liquids having refractive indices


4/3 and 5/3, it has the focal lengths/] and
> Ml M2 =1-2 respectively. The correct relation between the
focal lengths is
B C (a) /2 >/;/j becomes negative
(b) /] and /, both become negative
(a) 1.33 (b) 1.8
(c)/i=/2</
(c) 1.6 id) 1.4
(d) /] >/and/2 becomes negative.
18. A beam of light consisting of red, green and blue (JEE Main 2014)
colours is incident on right angled prism. The
refractive indices of the material of the prism for 22. A point source S is placed at the bottom of a
the above red, green and blue wavelengths are transparent block of height 10 mm and refractive
index 2-72. It is immersed in a lower refractive
1-39, 1-44 and 1-47 respectively. The prism will

16. (u) 17. ih) 18.(0) 19. (c) 20. (d) 2hUf)
RAY OPTICS AND OPTICAL INSTRUMENTS 9/177

index liquid as shown in Fig. 9(CF). 12. it is found


that the light emerging from the block to the liquid
forms a circular bright spot of diameter 11 -54 mm
on the top of the block. The refractive index of
the liquid is

FtGURE 9(CF).12

Liquid

Block

S (a) 15° (b) 22-5° (c) 30° id) 45°

w
{JEE Advanced 2016)
(a) 1-21 ib) 1-30 26. A light ray travelling in glass medium is incident
(c) 1-36 (d) 1-42 on glass-air interface at an angle of incidence 6.

Flo
(JEE Advanced 2014) The reflected (R) and transmitted (7) intensities,
23. The speed of light in media A/j and M2 are both as function of 9, are plotted. The correct

reeee
sketch is
1.5 X 10^ m/s and 2.0 x 10* m/s respectively. A
ray of light enters from medium to M2 at an

FFr
incidence angle /. If the ray suffers total internal
reflection, the value of / is

for
ur
-I 2'^
(a) Equal to sin
u
kkss
s'!
Yo
(b) Equal to or less than sin *
oo

5)
eB

-1 (2
(c) Equal to or greater than sin
r

2 'I
ou

(d) Less than sin"’


ad

(AIPMT Main 2010)


YY

24. In an experiment for determination of refractive


index of glass of a prism by /-8 graph, it was found
ndd
Re

that a ray incident at an angle of 35° suffers a


deviation of 40° and that it emerges at an angle
Fi

79°. In that case, which of the following is closest


to the maximum possible value of refractive
index ?

(a) 1-5 (b) 1-6


(c) 1-7 id) 1-8
(JEE Main 2016)
25. A parallel beam of light is incident from air at an
angle a on the side of a right angled triangular
prism of refractive index \i = ^^2 . Light
undergoes total internal reflection in the prism at
the face PR when a has a minimum value of 45°.
The angle 6 of the prism is (IIT 2011)

ANSWERS

22.(c) 23.(c) 24.(a) 25.(a) 26.(r)


9/178 'PnacUe^'4' Fundamental Physics (XIllSSsMO
27. Water (with refractive index = 4/3) in a tank is {a) - 280*0 cm (/?) 40 0 cm
18 cm deep. Oil of refractive index 7/4 lies on (c) 21-5 cm {d) 13*3 cm (1IT2012)
water making a convex surface of radius of 30. An object 2-4 m in front of a lens forms a sharp
curvature /? = 6 cm as shown in Fig. 9(CF).15.
image on a film 12 cm behind the lens. A glass
Consider oil to act as a thin lens. An object S is
plate 1 cm thick, of refractive index 1*50 is inter
placed 24 cm above water surface. The location
posed between lens and film with its plane faces
of its image is at x cm above the bottom of the
tank. Then x is
parallel to film. At what distance (from lens)
should object be shifted to be in sharp focus on
FIGURE 9(CF).15 film ?
S (fl) 7*2 m {b) 2*4 m
(c) 3*2 m (^d) 5-6 m
(AIEEE 2012)
p= 1.0
31. A concave mirror of focal length ‘/|’ is placed at

ww
R = 6cm a distance of 'd' from a convex lens of focal length
A beam of light coming from infinity and
falling on this convex lens - concave mirror
combination returns to infinity. The distance'd'

Flo
fi = 4/3
must equal :

e
(«) /l +/2 (b) -/, +/2

e
ib) 2

reer
(a) 1 ic) 2/i +/2 id) -2/, +/2

rFF
(c) 3 (.d) 4 (IIT 2011) [AIPMT (Prelim) 2012]
28. A converging beam of rays is incident on a 32. Diameter of a plano-convex lens is 6 cm and
uur r
diverging lens. Having passed through the lens, thickness at the centre is 3 mm, as shown in Fig.
the rays intersect at a point 15 cm from the lens
on the opposite side. If the lens is removed the
ffoor
9(CF).17. If speed of light in material of lens is
2 X 10® nVs, the focal length of the lens is :
sks
point where the rays meet will move 5 cm closer
YYoo
FIGURE 9(CF).17
to the lens. The focal length of the lens is
ooko

(a) - 10 cm (b) 20 cm
eBB

(c) - 30 cm (d) 5 cm
[AIPMT (Main) 2011]
.""f\ 3mm
uurr

29. A bi-convex lens is formed with two thin plano


convex lenses as shown in the figure. Refractive *■-
ad
Yo

index n of the first lens is 1*5 and that of the


second lens is 1*2. Both the curved surfaces are
of the same radius of curvature R = 14 cm. For
dY

this bi-convex lens, for an object distance of


Re
innd

40 cm, the image distance will be


(a) 15 cm (b) 20 cm
FFi

FIGURE 9(CF).16
(c) 30 cm (d) 10 cm
n = 1.5 A n = 1.2 (JEE Main 2013)
33. A light ray falls on a glass surface of refractive
4 index at an angle 60". The angle between
the refracted and reflected rays would be
(a) 30" ib) 60"
(c) 90" id) 120°
(NEET 2022)
vy 34. Two similar thin equiconvex lenses of focal length
r\
R = 14 cm
/ each, are kept co-axially in contact with
eachother, such that the focal length of the
ANSWERS

27.0b) 28. (c l 29.(6) 30. id) 31. (c) 32. (c) 33. (c)
RAY OPTICS AND OPTICAL INSTRUMENTS 9/179

combination is F^. When the space between the (a) 3 X 10^ ni/s (b) 1-5 X 10^ m/s
lenses is filled with glycerine (jO, = 1-5), the
3
equivalent focal length is F2- The ratio Fj/F2 is (c) -= X10^ m/s id) V2 X 10^ m/s
ia) 2 : I (b) 1:2 V2
ic) 2 : 3 (d) 3:4 (NEET 2019) (NEET 2020)
35. A convex lens is put lOcmfrom a light source and 40. The power of a biconvex lens is 10 dioptre and
it makes a sharp image on a screen kept 10 cm radius of curvature of each surface is 10 cm. Then
from the lens. Now a glass block (|X = 1-5) of 1-5 the refractive index of the material of the lens is
cm thickness is placed in contact with the light
ia) 3/2 ib) 4/3
source. To get the sharp image again, the screen is
ic) 9/8 id) 5/3 (NEET 2020)
shifted by a distance d where d is equal to
(rt) I-l cm away from lens 41. A piano convex lens of unknown material and
{b) zero unknown focal length is given. With the help of a

ww
(c) 0-55 cm towards the lens
spherometer, we can measure the
ia) refractive index of the material
id) 0-55 cm away from lens (JEE Main 2019)
36. A monochromatic light is incident from air on a ib) focal length of the lens

FF loo
refracting surface of a prism of angle 75° and ic) radius of curvature of curved surface
id) aperture of the lens (NEET 2020)
refractive index |J = Vs. The other refracting

ree
42. Find the value of the angle of emergence from
surface of the prism is coated by a thin film of
material of refractive index n as shown in Fig. the prism. Refractive index of glass is ^3.

rFee
9(CF).45. The light suffers total internal reflection

F
oor r
at the coated prism surface for incident angle of
rur
6 < 60°. The value of is
s ff
ia) 2/3 ib) 1
(c) 3/2 (^)0
k
YYoou
(JEE Advanced 2019)
ookos

37. A biconvex lens has radii of curvature, 20 cm


BBo

each. If the refractive index of the material of the


lens is 1-5, the power of the lens is :
re

ia) + 2D ib) +20D


ouur

ic) + 5 D id) infinity


ad
Yo

(NEET 2022)
(a) 90° ib) 60°
38. Pick the wrong answer in the context with ic) 30° ((/)45° (NEET 2021)
rainbow.
dY

43. A convex lens A of focal length 20 cm and a


Re

ia) When light rays undergo two internal


idn

concave lens B of focal length 5 cm are kept along


reflections in a water drop, a secondary
FFin

the same axis with a distance d between them. If


rainbow is formed
a parallel beam of light falling on A leaves B as a
ib) The order of colours is reversed in secondary
rainbow parallel beam, then the distance id) will be
ia) 30 cm ib) 25 cm
(c) An observer can see a rainbow when his front
is towards the sun (c) 15 cm id) 50 cm (NEET 2021)
id) Rainbow is a combined effect of dispersion, 44. A point object is placed at a distance of 60 cm
refraction and reflection of sunlight from a convex lens of focal length 30 cm. If a
(NEET 2019) plane mirror were put perpendicular to the
39. If the critical angle for total internal reflection principal axis of the lens at a distance of 40 cm
from a medium to vacuum is 45°, then velocity of from it, final image would be fonned at a distance
of
light in the medium is

ANSWERS

34. ib) 35. ic) 36. ic) 37. (c) 38. (c) 39. (c) 40. (a) 41. (c) 42. ib) 43. (c)
9/180 ^ Fundamental Physics (XII)

( W
-1 -1
(ci) 0 > sin fx sin /4-sin

1
(.b) 6 < sin ^ )i sin A- sin '
V /y

y ,
1
(c) 0 > cos * |X sin A-sin -1
/-I

(a) 20 cm from plane mirror, it would be virtual -I -1 (i


id) Q< cos jx sin A - sin
image

ww
(b) 20 cm from the lens, it would be a real image (JEE Main 2015)
(c) 30 cm from the lens, it would be a real image 48. A monochromatic beam of light is incident at 60°
(d) 30 cm from the plane mirror, it would be a on one face of an equilateral prism of refractive

Flo
virtual image (NEET 2021) inder n and emerges from the opposite face

e
45. A medium has relative permittivity 3 and relative

e
making an angle 9 with the normal. For n =
permeability 4/3. The critical angle of the medium

reer
dd

rFF
IS the value of 9 is 60° and = m. The value of
dn
(a) 60° (b) 15° m IS
uur r
(c) 45° (d) 30° (a) 1 (b)2
(JEE Main 2020)
46. A point object in air is in front of curved surface
(c)3ffoor (d)4
sks
(JEE Advanced 2015)
of a piano convex lens. The radius of curvature
YYoo
49. The angle of prism is A. One of its refracting
ooko

of the curved surface is 30 cm and refractive index


surfaces is silvered. Light rays falling at an angle
of the lens material is 1 -5, What is the focal length of incidence 2 A on the first surface return back
eBB

of the lens ? through the same path after suffering reflection


(a) 30 cm (£») 45 cm at the silvered surface, The refractive index p. of
the prism is
uurr

(c) 60 cm id) 90 cm
ad

(JEE Main 2020) id) 2 sin A ib) 2 cos A


Yo

1
III. Dispersion of Light (c) 2^°^^ id) tan A
dY

47. Monochromatic light is incident on a glass prism (AIPMT 2014)


Re
innd

of angle A. If the refractive index of the material 50. A green light is incident from water to the air-
of the prism is p, a ray incident at an angle 6 on water interface at the critical angle (0). Select the
FFi

the face/45 would get transmitted through the face correct statement.
AC of the prism provided ia) The spectrum of visible light whose frequency
is more than that of green light will come out
of the air medium.

ib) The entire spectrum of visible light will come


out of water at various angles to the normal,
(c) The entire spectrum of visible light will come
out of water at an angle of 90° to the norma).
id) The spectrum of visible light whose frequency
is less than that of green light will come out
of the air medium. (JEE Main 2014)

44. id) 45. [d] 46. («■) 47. ia) 48. (a) 49.ib) 50. (d)
RAY OPTICS AND OPTICAL INSTRUMENTS 9/181

51. For the angle of minimum deviation of a prism to 56. The near point and far point of a person are 40
be equal to its refracting angle, the prism must be cm and 250 cm respectively. Determine the power
made of a material whose refractive index : of the lens he/she should use while reading a book
kept at distance 25 cm from the eye.
{a) lies between ^ and 1
(a) 2-5 D ib) 5 0 D
Q>) lies between 2 and ~J2 (c) 1-5 D id) 3-5 D
(c) is less than 1 (AILMS 2015)
(d) is greater than 2 [AIPMT (Main) 2012] 57. Assuming human pupil to have a radius of
0-25 cm and a comfortable viewing distance of
IV. Optical Instruments
25 cm. The minimum separation between two
52. A telescope has an objective lens of 10 cm objects that human eye can resolve at 500 nm
diameter and is situated at a distance of 1 km from wavelength is

ww
two objects. The minimum distance between these (a) 1 jim (b) 30 pm
two objects, which can be resolved by the (c) 100 jxm (d) 300 pm
telescope, when the mean wavelength of light is (JEE Main 2015)
5000 A is of the order of

Flo
58. An astronomical refracting telescope will have
(a) 5 mm (b) 5 cm
large angular magnification and high angular

e
(c) 2-5 m (d) 5 m

e
resolution, when it has an objective lens of

reer
53. If the focal length of objective lens is increased, (a) Large focal length and large diameter

rFF
then magnifying power of
(b) Large focal length and small diameter
(a) microscope will increase but that of telescope
uur r
(c) Small focal length and large diameter
decrease

(b) microscope and telescope, both will increase


ffoor
(d) Small focal length and small diameter
(NEET 2018)
sks
(c) microscope and telescope both will decrease
YYoo
(d) microscope will decrease, but that of telescope 59. An astronomical telescope has objective and eye
ooko

will increase. (AIPMT 2014) piece of focal lengths 40 cm and 4 cm re.spectively.


To view an object 200 cm from the objective, the
eBB

54. A boy is trying to start a fire by focussing sunlight


lenses must be separated by a distance
on a piece of paper using an equi-convex lens of
(a) 46-0 cm (h) 50 cm
focal length 10 cm. The diameter of the sun is
uurr

1 -39 X 10^ m and its mean distance from the earth (c) 54-0 cm (d) 37-3 cm
ad

is 1-5 X 10*’m. What is the diameter of the sun’s (NEET 2016)


Yo

image on the paper ? 60. A lens of large focal length and large aperture is
ia) 9-2 X 10-'* m ib) 6-5 X lO-'* m best suited as an objective of an astronomical
dY
Re

(c) 6-5 X 10"^ m id) 12-4 X 10" m telescope since


innd

(AIPMT 2008) (a) A large aperture contributes to the quality and


FFi

visibility of the images


55. In an astronomical telescope in normal adjust
ment, a straight black line of length L is drawn on (b) A large area of objective ensures better light
inside part of objective lens. The eye piece forms gathering power
a real image of this line. The length of this image (c) A large aperture provides a better resolution
is !. The magnification of telescope is : (d) All of the above (NEET 2021)

L 61. The magnifying power of a telescope with tube


length 60 cm is 5. What is focal length of its eye
(“)y ib) - + 1
I piece ?
L + 7 (a) 30 cm ib) 40 cm
(c) --1 id)
I L-I (c) 20 cm id) 10 cm
(JEE Main 2020)
(RE-AIPMT 2015)

51. ib) 52. ia) 53. (</) 54. («) 55. (a) 56. (r) 57. (/;) 58. («) 59. (r)
60. (d) 61. id)
9/182 4 Fundamental Physics (XII) LWII

62. The aperture diameter of a telescope is 5 m. The surface of moon so that they are just resolved is
separation between the moon and earth is 4 x 10^ close to
km. With light of wavelength 5500 A, the (a) 20 m (b) 600 m (c) 60 m (d) 200 m
minimum separation between the objects on the (JEE Main 2020)

ED! Multiple Choice Questions (with One or More than One Correct Answers)

63. A ray of light travelling in a transparentmedium (a) The distance between the objective and eye
falls on a surface separatingthe medium from air piece is 16-02 tn
at an angle of incidenceof 45°. The ray undergoes (b) The angular magnification of the planet is
total internal reflection.If p is the refractiveindex -800
of the medium w.r.t. air, select the possible value (c) The image of the planet is inverted

oww
(^) of p from the following (^0 The objective is larger than eye piece
(a) 1-3 (b) 1-4 68. Four light waves are represented as :
(c) 1-5 (d) 1-6 (i) y = sin cor
64. A spherical surface of radius of curvature R (ii) y = sin (cor + (j))

e
separates air (refractive index l-O) from glass (Hi) y = sin 2 cor

ree
rFl
(refractive index 1-5). The centre of curvature is
(iv) y = ^2 2

Fre
in the glass. A point object P placed in air is found
Interferencefringes may be observed due to the
to have a real image Q in glass. The line PQ cuts

rr F
superposition of:
the surface at point O and PO = OQ. The distance
(«) (0 and (ii) (b) (/) and (Hi)
PO is equal to :
ouur
(a)5R
(c)2R
(b)3K
(d) 1-5 R
sfoo
(c) (//) and (fv) (d) (Hi) and (iv)
69. A student performed the experiment on
detennination of focal length of concave mirror
kks
65. A short linear object of length b lies along the
hy ii-v method. Using an optical bench of length
Yo
oooo

axis of a concave mirror of focal length / at a I -5 m. The focal length of the mirror used is
distance u from the pole of the mirror. The size
eBB

24 cm. The five sets of (u, u/values) recorded by


of the image is approximately equal to : the student (in cm) are (42, 56), (48, 48),
1/2 n1/2
(60, 40), (66, 33). (78, 39). The data set (s) that
u-f' f
uurr

(a) h (b) b cannot come from experiment and is (are)


f u-f
ad

incorrectly recorded is (are):


YYo

x2 (a) (42, 56) (b) (48, 48)


(c) b u-f] (d) b
f
(c) (66, 33) (d) (78, 39)
f I
dd

u-f
70. A ray OP of monochromatic light is incident on
Re
iinn

66. Astronomical telescope has an angular the face AB of prism ABCD near vertex B at an
magnification of magnitude 5 for distant objects. incident angle of 60° Fig. 9(CF).21. If the
F

The separation between the objective and the refractive index of the material of the prism is
eyepiece is 36 cm and the final image is formed
at infinity. The focal length /q of the objective ^, which of the following is (are) correct ?
and focal length of the eye piece is
(a) /q = 45 cm and /^ = - 9 cm
(b)fQ = 50 cm andfg = 10 cm
(c)/q = 7-2 cm and /^ = 5 cm
(d) /q = 30 cm and/^ = 6 cm
67. A planet is observed by an astronomical refracting
telescope having an objective of focal length
16 m and eye piece of focal length 2 cm. Which
of the following statements is (are) correct ?
ANSWERS

62. (f) 63. (c.d) 64. (a) 65. (d) 66. (d) 67. (a.b.c.d) 68. (a.d) 69. (c.d)

4
RAY OPTICS AND OPTICAL INSTRUMENTS 9/183

(a) The ray gets totally internally reflected at face 73. A transparent thin film of uniform thickness and
CD refractive index = 1-4 is coated on the convex

ib) The ray comes out through face AD spherical surface of radius R at one end of a long
(c) The angle between the incident ray and the solid glass cylinder of refractive index /;■> = 1-5,
emergent ray is 90“ as shown in Fig. 9(CF).23. Rays of light parallel
to the axis of the cylinder traversing through the
id) The angle between the incident ray and the
emergent ray is 120° (IIT 2010) film from air to glass get focusssed at distance/|
from the film, while rays of light traversing from
71. A piano convex lens is made of material of
glass to air get focussed at distance/2 from the
refractive index |J. When a small object placed film. Then
30 cm away in front of the curved surface of the
lens, an image of double the size of the object is
FIGURE 9(CF).23
produced. Due to reflection from the convex /»●) = 1-4

ww
surface of the lens, another faint image is observed
at a distance of 10 cm away from the lens. Which «2 = T5
of the following slatement(s) is (are) true ? Air Glass

Flo
R
(a) refractive index of lens is 2-5
(b) radius of curvature of convex surface is 45 cm

ee
(c) faint image is erect and real

rere
rFF
(d) focal length of the lens is 20 cm ia) I/, 1 = 3/? (b) I/, 1 = 2-8/?
(JEE Advanced 2016) (c) 1/21 = 2/? id) 1/2 I = 1-4 R
uurr
72. A transparent slab of thickness t/ has a refractive
index n iZ) that increases with Z Here Z is the
foor (JEE Advanced 2014)
74. Three observers A, B, C measure speed of light
ks s
vertical distance inside the slab, measured from
from a source in vacuum to be i>^, Vg, Vq
Yoo
the top. The slab is placed between two media respectively. If A was moving towards source, B
oook

with uniform refractive indices and «2 (> was moving away from source with same speed
eBB

as shown in Fig. 9(CF).22. and C was at rest, then

FIGURE 9(CF),22
ia) v^<Vg<Vc ib) v^>vg>vc
uurr

0,1
ad

ni = const. id) V^ = Vg = Vc
Yo

n(z)
75. Mark the correct options :
dY

d
z
ia) If the far point is 1 m away from the eye,
Re
innd

divergent lens should be used,


FFi

/i2 = const. ib) If the near point is 1 m away from the eye.
divergent lens should be used,
(c) If the far point goes ahead, the power of the
A ray of light is incident at angle 9^ from medium divergent lens should be reduced.
1 and emerges in medium 2 with refraction angle id) If the near point goes ahead, the power of the
0^ with a lateral displacement /. Which of the convergent lens should be reduced.
following statement(s) is (r -e) tiuc 7 76. A ray of light from a denser medium strikes a rarer
ia) I is independent of n ( medium at an angle of incidence i. Fig. 9(CF).24.
ib) sin 9y = («2 - /t|) sin 9y The reflected and refracted rays make an angle of
90° with each other. The angles of reflection and
(c) /j, sin 9,’ = sin 0y
refraction are r and r. The critical angle is ;
id) I is independent of nj (JEE Advanced 2016)
ANSWERS

70. ia.b,c) 71. ia,d) 72. ic.d) 73. ia.c) 74. ic.d) 75. ia.c)
9/184 'Pn<tdeep- U Fundamental Physics (XII)
(a) sin ‘ (tan r) (b) sin"’ (tan /)
(c) sin"’ (tan r) (d) tan"’ (sin /).
77. Photographs of the ground are taken from an
aircraft flying at an altitude of 3 km by a camera
with a lens of focal length 50 cm. The size of the
film in the camera is 18 cm x 18 cm. What is the
area of the ground that can be photographed by
this camera ?

(a) 720 mx 720 m {b) 240 m X 240 m


(c) 1080 mx 1080 m (d) 100 mx 100 m

QQ Multiple Choice Questions (Based on the given Passage/Compreh ension)

w
Each comprehension given below is followed by some multiple choice questions. Each question has one
correct option. Choose the correct option.

Flo
pre hWn s i oniC
Light guidance in an (b) NA of 5j immersed in liquid of refractive index

e
optical fiber can be understood by

rree
—= is the same as that of 5-> immersed in
considering a structure comprising of thin Vl5

r FF
solid glass cylinder of refractive index water,
surrounded by a medium of lower refractive
(c) NA of 5j placed in air is same as that of S2
index nj. The light guidance in the structure
uurr
takes place due to successive total internal
reflections at the interface of media and
for
immersed in liquid of refractive index —=
4

^/i5
kss
«2 as shown in Fig. 9(CF).25. All rays with (d) NA of Sj placed in air is the same as that of ^2
the angle of incidence / less than a particular
ooook

placed in water. (JEE Advanced 2015)


Yo
value are confined in a medium of
79. If two structures of same cross sectional area but
refractive index The numerical aperture
eB

(NA) of the stucture is defined as sin different numerical aperatures NA j and NA2 {NA2
< AWj) are joined longitudinally, the numerical
aperture of the combined structure is
urr
ad

NA^
Yo

(a) (b)NA^+NA2
NA^+NA^
dY

(c) NA 1 (d)NA2
Re
innd

(JEE Advanced 2015)


Fi

Power (P) of a lens is given


78. Fortwostrucluresnamely5[ with — and by reciprocal of focal length if) of the lens.
_

3 8 7 Le. P = Ilf. When /is in metre, P is in dioptre.


n-y = —, and St with n, = — and n- = - and For a convex lens, power is positive and for
2 " ’ 5 - 5
a concave lens, power is negative. When a
4 number of thin lenses of powersp^.p2,py...
taking refractive index of water to be — and that
3 are held in contact with one another, the
of air to be 1, the correct option (j) is (are). power of the combination is given by
(a) NA of 5| immersed in water is same as that of St algebraic sum of the powers of all the lenses
16 i.e.. F=Pi+/72+/>3 +
immersed in a liquid of refractive index
3V15 ’

76. {o.h) 77. (c) 78. ia.c) 79. (d)


RAY OPTICS AND OPTICAL INSTRUMENTS 9/185

Answer the following questions :


ray. According to electromagnetism, the
80. Two thin lenses are in contact and the focal length refractive index of the medium is given by
of the combination is 80 cm. If the focal length
of one lens is 20 cm, the focal length of the other the relation, n = (c / u) = ± r ’
where c
would be
is the speed of the electromagnetic waves in
(a) -26-7 cm (b) 60 cm vacuum, v its speed in the medium, and
(c) 80 cm (d) 20 cm are negative, one must choose the negative
81. When a third lens of focal length - 20 cm is placed root of n. Such negative refractive index
in contact with the two lenses, power of the three materials can now be artificially prepared
would be and are called meta-materials. They exhibit
(a) -315 D (b) 3-75 D significantly different optical behaviour,
(c) 5 0 D id) -5-0 D without violating any physical laws. Since n
is negative, it results in a change in the

ww
[G.omp^eh'ensidmR An initially parallel
direction of propagation of the refracted
light. However, similar to normal materials,
cylindrical beam travels in a medium of
the frequency of light remains unchanged
refractive index |i (/) = where

FF loo
upon refraction even in meta-materials.
and \i2 are positive constants and / is intensity
of light beam. The intensity of the beam is Answer the following questions :

ree
decreasing with increasing radius. 85. For light incident from air on a meta-material, the
appropriate ray diagram is

reeF
Answer the following questions :
82. As the beam enters the medium, it will

oroFr
r ur
(a) converge
s ff
(b) diverge near the axis and converge near the
periphery
k
YYouo
(c) travel as a cylindrical beam
koso

(d) diverge (AIEEE 2010)


BBoo

83. The speed of light in the medium is


(a) the same everywhere in the beam
r ee

(b) directly proportional to the intensity /


(c) maximum on the axis of the beam
ad
ouur

(d) minimum on the axis of the beam


Yo

{AIEEE 2010)
84. The initial shape of the wavefront of the beam is
d

(a) concave
Re
idnY

(b) convex near the axis and concave near the


periphery
FFin

(c) planar
(d) convex (AIEEE 2010)
(HI 2012)
D Most materials have the
86. Choose the correct statement,
(a) The speed of light in the meta-material is
refractive index, n > 1. So, when a light ray V = c\n\
from air enters a naturally occurring (b) The speed of light in the meta-material is
sin 6 I «i
c

material, then by Snell’s law V =

’ sin 02 «2
Ini
(c) The speed of light in the meta-material is u = c
it is understood that the refracted ray bends
(d) The wavelength of the light in the meia-material
towards the normal. But it never emerges on
the same side of the normal as the incident (XJ is given by X„, = X.^^^ Ini, where X^^^ is
the wavelength of the light in air (IIT 2012)

80. (n) 81. («) 82. (n) 83. (</) 84. fc) 85. (c) 86. (/)|
9/186 “Pn^teUefr '4. Fundamental Physics (XII) lV»mil

09 Matching Type Questions


DIRECTIONS. In each of the following questions, match column I and column II and select the correct
match out of the four given choices.
87. Match correctly column I and column II :
Column I Column II

(A) Reflection (P) used for reducing glare


(B) Refraction (?) change in path of light without change in medium
(C) Interference ir) p = sin f’/sin r
(D) Polarization (5) light added to light produces darkness

{a) A-q, B-r, C-s, D-p (b) A-p, B-q, C-r, D-i‘ (c) As, B-r, C-q, D-p (d) A-r ; B-s ; C-q ; D~p

ww
Matrix-Match Type Questions

Flo
DIRECTIONS. Each of the following questions contains statements given in p q r s

ee
two columns which have to be matched. The answers to these questions have to
be appropriately bubbled. If the correct matches are A-r, As ; B-^ ; C-p, Cs © © © Ls.

rere
rFF
and D-q, then the correctly bubbled matrix will look like the one shown here: B
© © o©
88. Column I Column II
c
© ®IO ©
uurr
(A)
(B)
concave mirror

convex mirror
(p)
(q)
foor
virtual, erect, smaller image
real, inverted, smaller image D
© ©lO ©
ks s
(C) convex lens (r) virtual, erect, larger image
-I I
Yoo
(D) concave lens (j) real, inverted, larger image
oook

89. Two transparent media of refractive indices pj and P3 have a solid lens shaped transparent material of
refractive index [I2 between them as shown in figures in Column II. A ray traversing these media is
eBB

also shown in the figures. In Column I, different relationships between Pj, p-> and P3 are given.
Match them to the ray diagrams shown in Column IT.
Column I Column II
uurr
ad
Yo

(a) p, <P2 (P) P3 P2 Ml


V
dY
Re
innd

(^) (?) M3 Ml
FFi

(C) p, = P3 ir) M1

●4

id) P2 > P3 is)


M3 Ml

(0 (IIT 2010)

ANSWERS

87.(a) 88. (A-q,,rs ; B-p ; C-q.ns ; D-p) 89. (A-p,r ; B-q.s,i; C-p,rj : D-q.s)
RAY OPTICS AND OPTICAL INSTRUMENTS 9/187

90. Four combinations of two thin lenses are given


in List I. The radius of curvature of all curved
R. 3. - r
surfaces is r and the refractive index of all the
lenses is 1-5. Match lens combinations in List 1 111
with their focal length in List II and select the
correct answer using the code given below the S. 4.r
lists.

List-1 List-II
Code :

P. 1.2r (a) P-l.Q-2, R-3, S-4


(b) P-2, Q-4, R-3, S-1
(c) P-4, Q-LR-2, S-3
r

w
Q. id) P-2, Q-LR-3, S-4
2
(,TEE Advanced 2014)

Flo
VI. Integer Type Questions

ee
Fr
DIRECTIONS. The answer to each of the following questions is a single digit A 6 C D
integer, ranging from 0 to 9. If the correct answers to the question numbers A, B, ^
C and D (say) are 4, 0, 9 and 2 respectively, then the correct darkening of bubbles

for
ur
should be as shown on the side :
oooo
91. In a tank filled with a liquid of refractive index 5/3, a point source of light is placed 2 m
ks
below the surface of water. To cut off all light coming out of water from the source
Yo
; @®@@
oo

what should be the minimum diameter of a disc, which should be placed over the
source on the surface of water ?
® ®®®
eB

92. A drop of liquid is spread over the hypotenuse of a right angled isosceles prism as
shown in Fig. 9{CF).27 and a ray of light is incident normally on face AB of the prism. © ® ® ©
ur

If refractive index of liquid is .^2 - for total internal reflection to occur, refractive C®) (E) CE)
ad
Yo

index of material of prism should be : ©©©©


©©©©
nd
Re

9
Fi

93. The radii of curvature of surfaces of a convex lens of glass are 20 cm each. If |i is glass is 3/2, power of lens
in dioptre is.

ANSWERS

90. ih) 91.(3) 92.(2) 93.(5)


9/188 ‘PfKxdee^ Fundamental Physics (XII) PZ5TMT1

VII. Assertion-Reason Type Questions

FOR MEDICAL STUDENTS 99. Assertion. The sun looks bigger in size ai sunrise
and sunset than during day.
DIRECTIONS. The following questions consist of
two statements each, printed as Assertion and Reason. The phenomenon of diffraction bends
Reason. light rays.
While answering these questions you are required to (a) A (b) B (c) C id) D
choose any one of the following four responses.
FOR ENGINEERING STUDENTS
A. If both. Assertion and Rea.son are true and the
Reason is the correct explanation of the DIRECTIONS. Each of the following questions
Assertion. contains two statements. Read the statements and
B. If both. Assertion and Reason are true but choose any one of the following four responses :

ww
Reason is not a correct explanation of the (A) Statement-1 is true, Statement-2 is true. State
Assertion. ment-2 is correct explanation of Statement-1.
C. If Assertion is true but the Reason is false. (B) Statement-1 is true, Statement-2 is true, but

FF loo
D. If both. Assertion and Reason are false. Statement-2 is not a correct explanation of
Statement-I.
94. Assertion. In optical fibre, the diameter of the

ree
(C) Statement-1 is true, Statement-2 is false.
core is kept small. (D) Statement-1 is false, Statement-2 is true.
Reason. The small diameter of the core ensures

rFee
that the fibre should have inside it an angle greater 100. Statement-1. The rainbow is seen sometimes in

F
than critical angle needed for total internal the sky when it is raining. When one sees a

oor r
rur
reflection. rainbow, one’s back is towards the Sun.
s ff
ia) A ib) B (c) C id) D Statement-2. Internal reflection from water

95. Assertion. A ray of light entering from glass to droplet causes dispersion. The final ray is in the
k
backward direction.
air suffers change in frequency.
YYoou
ookos

Reason. Velocity of light in glass is more than ia) A ib) B (c) C id) D
that in air. 101. Statement-1. A single lens produces a coloured
BBo

ia) A ib) B (c) C id) D image of an object illuminated by white light.


re

Statement-2. The refractive index of material of


96. Assertion. A prism is the source of colours of
light. lens is different for different wavelengths of light.
ouur
ad

Reason. A prism has same refractive index for ia) A (b) B (c) C id) D
Yo

different colours of light. 102. Statement-1. It is impossible to photograph a


ia) A ib) B (c) C id) D virtual image.
97. Assertion. A lens having large aperture will Statement-2. The rays which appear diverging
dY
Re

produce image of a point source not as a point from a virtual image fall on the camera and a real
idn

but as a diffused bright spot. This error of optical image is captured.


FFin

system is called spherical aberration. ia) A ib) B (c) C id) D


Reason. The paraxial rays of light form the image 103. Statement-1. If a convex lens is kept in water, its
at a longer distance from the lens than marginal convergent power increases.
rays, Statement-2. Focal length of lens depends on its
(rt) A ib) B (c) C (f/) D refractive index w.r.t. surrounding medium.
98. Assertion. The frequencies of incident, reflected ia) A ib) B (c) C D
and refracted beam of monochromatic light are 104. Statement-1. A dentist uses a concave mirror to
same.
examine a small cavity.
Reason. The incident, reflected and refracted rays Statement-2. A dentist uses a concave mirror so
are coplanar. as to form a magnified, virtual image of an object.
ia) A ib) B ic) C id) D ia) A ib) B ic) C id) D

94. (f/) 95. (d) 96. Id) 97. (a) 9».(h) 99. (b) 100. (n) 101. (fl) 102. id)
103. (d) 104. ((/)
RAY OPTICS AND OPTICAL INSTRUMENTS 9/189

105. Statement-1. The refractive index of a prism


1
depends only on the kind of glass of which this is
made and the colour of light. f R
1
- j
Statement-2. The refractive index of a prism
(a) A ib) B (c) C (i/) D
depends upon refracting angle of prism and angle
of minimum deviation. 109. Statement-1. The dispersive power of a lens of
focal length 10 cm is 0-08. Longitudinal chromatic
(a) A ib) B (c) C id) D aberration of the lens would be 0-8 cm.
106. Statement-1. A ray of light incident normally on
Statement-2. It follows from/^-/y - co/
a refracting surface does not suffer any refraction.
(a) A ib) B (c) C id) D
Statement-2. The critical angle for total internal
110. A thin air film is formed by putting the convex
reflection is smaller when a ray of light travels
from glass to water than when it travels from glass surface of a plane convex lens over a plane glass
to air. plate. With monochromatic light, this film gives

ww
an interference pattern due to light reflected from
ia) A ib) B (c) C (rf) D the top (convex) surface and the bottom (glass
107. Statement-1. Light travels faster in glass than in plate) surface of the film.

Floo
air.
Statement-1. When light reflects from the air-
Statement-2. Because air is rarer than glass. glass plate interface, the reflected wave suffers a
(a) A ib) B

ee
(c) C (^ D phase change of k.
108. Statement-]. Focal length of an equiconvex lens Statement-2. The centre of the interference

eer
of = 3/2 is equal to radius of curvature of each pattern is dark.

FFr
surface.
(«) A ib) B ic) C id) D
Statement-2. It follows from

oorr
uur r
(AIEEE 2011)
VIII,
s ff
Multiple Choice Questions (Based on Experimental Skills)
sk
YYoo
111. An experiment is performed to find the refractive
ooko

index of glass using a travelling microscope. In V (cm)


this experiment, distances are measured by
t
eBB

(a) a vernier scale provided on the microscope


ib) a standard laboratory scale
uurr

(c) a meter scale provided on the microscope


ad

id) a screw gauge provided on the microscope


Yo

O u (cm)
(AIEEE 2008)
112. A student measures the focal length of a convex o
dY

lens by putting an object pin at a distance ‘m’ from


Re
ind

the lens and measuring the distance ‘u’ of the (AIEEE 2008)
FFin

image pin. The graph between 'u' and ‘u’ plotted 113. The shape of graph between \/u and \/v in case
by the student in Fig. 9(CF).28 should look like : of a convex lens is, Fig. 9(CF).29

FIGURE 9(CF),28

(cm)

ANSWERS

105. (c) 106. ic) 107. id) 108. ia) 109. ia) 110. ia) 111. ia) 112. ic)
9/190 Fundamental Physics (XII)KLtagli

(a) object needle is at the focus of the lens


(fc) object needle is at 2 F
(c) object needle lies between F and 2 F
Mv (d) none of the above
116. A student has drawn the following courses of rays
t through a glass prism. Fig. 9(CF).30. Which one
X X represents the position of minimum deviation ?
0 1/u

114. In the above question, the correct value of focal


length/of lens as calculated from the graph is:

ww
1
(a) f = (b) f = OA + OB
OA
1 1

FF loo
(c) / = (d) f = OB
OA-OB
115. In finding experimentally, the p. of a liquid using

ree
a convex lens and a plane mirror, parallax is
removed when

F rFee
oor r
rur
s ff
For Difficult Questions
k
n Multiple Choice Questions (with one correct Answer)
YYoou
ookos

Angle of incidence,
BBo

1. Here, A = . 180°-e 180°-120°


re

i = = 30°
2 2
2. As is clear from Fig. 9(CF).32, distance of point
ouur

and
ad

A from pipe P of concave mirror, u = -fI2. Let


Yo

image of A be at A'.
A = B= = 2 and A B = -2
^From —1 + —1 =
1 1 1 !_ 1 1.1
dY

/' f u -f -sn f
Re

V u V

A-B ^ -2 _ 1
idn

cos 0 = v=f
FFin

AB 2x2 2
Image of line AB shall be perpendicular to the
0= 120°, Fig. 9(CF).31. principal axis and image of F will be at infinity.
Therefore, the correct image diagram is as shown
in Fig. 9(CF).32.

ANSWERS

113.(a) 114. (d) 115. (a) 116. id)


RAY OPTICS AND OPTICAL INSTRUMENTS 9/191

For Difficult Questions 1 1 1 1 1


Let u = From — =
V f u V -10 15’
3. Here, » = -{/+i/j) and u =-(/+ in

= - 30 cm
1
From —+— =
u V f For the lens in air, /?, =/?,/?,=-/?
1 (3
\ /
1 1 1 I 1 2 ^
- = (^-1) or --1
-if+clO -if + d,) -f / /?. R. - y
10 2
y V
R
/

2f + d-y I
or
i-1
(f + d^)if + d2) f 5~ R
In medium of refractive index 7/6,
or 2f^ + {dy+d2)f=f^ + d^f+d2f+d^d2

w
1 3/2 ,V2^ 2 1 2
f- = d,d2 -1 — X- = —
7/6 R 7 5 35

Flo
f=Kz^
yv“ j
or

For lens in air, /< = - 20 cm,/= + 10 cm

e
5. From miiTor formula (using convex mirror)

reee
I
or V..a = 20 cm

FFr
...(0 V
(/
(-20) 10
U V /
1 1 2
Using H = - 2-8 m = - 280 cm,/= 20 cm For lens in medium.

for
ur
u' (-20) 35
1 1
J_
ni
15

-280^u 20
or
or v' rn = 140 cm
V 20 280 ' 280
kkss
v'
M2 140
Yo
m
280 = 7
oo

or V - cm M V 20
15 I a
eB

Differentiating (/) w.r.t. t, we have 8. for (Refraction from denser to rarer medium)
H = - 50 cm, R = - 10 cm
dv 2/
1 du 1 dv V du
= 0 or
It, ^ ^1
ur

ir dt v~ dt dt u 2 U/f
ad

-u V R
YYo

Relative velocity of image.


3/2 1 1-3/2 1 3 1
+ — = or - +
V
dv v~ ( du\ -(280/15)2 -(-50) V -10 V 100 20
xl5
d

1 “
Atl
Re

dt u
(-280)2
3__ J_
in

V 20 100 ~ 50
F

= - — m/s
u = 50 cm
15

6. Here, m = -1-5/, v = 1, f=-f for 52 Refraction from rarer to denser medium


From mirror formula : .\u->-(d- 50), V = ', /?2 = /?

M-2 _ f*-2 ~~ 1^1


1=1-1 ^ 1
_j_ 1_
V f u -f 3/ f~^3f~ 3/ ~u V R

1 3/2 1
2
id ~50) 00 20
v=-3f
i/ = 70 cm
7. Image formation by concave mirror is unaffected
9. In Fig. 9(CF).5,
by presence of medium as for mirror, 1 +1 = ^
V u f AC = = hS
Here, u= - 15 cm,/ = - 10 cm
BC = ^ih^+h-
9/192 Fundamental Physics fXinPZSTWTl
o / . N
1 1
For Difficult Questions or M + 1

rw
m /
The ray AC incident on water surface at Zr is which is the equation of a st line whose slope
refracted along CD. If \l is refractive index of
1 h
water, then from Snell’s Law, c

f c
sin(90-i) COSJ
BD'/BC _AC ^ h4s 12. In Fig. 9(CF).35, aperture of lens = /?,
sin r sinr AE!AC~ BC /1V2
aperture at 22 cm from the lens = r,
]i=4sii

r
focal length of the lens,/ = 20 cm.

wo
llou
10. As shown in Fig. 9(CF).33, r + C = 90°,

F
FIGURE 9(CF).35
r = (90° - Q

sin { sin 45° 1

FF
As 11 = -: R
sin(90°-C) V2 cos C 'r

s
sin r I
F

uro
er
^—i

k e
<■ ■K >
20 cm 2 cm

For
ro
fof
Clearly, l = 2l. = L
o
Y
R 20 . 10
Y
B
a r- ( 1 1
ks
Ratio of area - —
100
/I ^10;
Yo
roo

If energy incident on the lens is E, then as given


eB
e
u
rd

here. - = l-3kW/m2
A
o
n

E E A
ou
ad

Average intensity of light at 22 cm = — = — x —


a A a
iY

100
= l-3x = 130 kW/m^
nd
F

1
Re

13. Let the light suffers total internal reflection at the


Fi

1 vertical surface where the critical angle is 0^ as


Again, Ji = , sin C = — = -72 cos C shown in Fig. 9(CF).36.
sin C 11

tan C = V2,

sin C =
(2 1 3
or )i =
\3 sin C

1 1 1
11. From
/
u u

V f
I u

m f
RAY OPTICS AND OPTICAL INSTRUMENTS 9/193

1 1 0-5
For Difficult Questions
— = (1-5-1) —
h V r oo r

sin0 = ii = _L = l => /2 = -2r


1-25 5
1

Here, 01 = 90" - 0, / ///2 2r 2r

f=-r
so sin 0| = sin (90°
- 0J.) = cos =J C ^ r

For refraction at the horizontal surface

sin 0 1-25 sin0


or (D)

ooww
sin 0, 1 3/5
1^1
3 \JW
or sin 6 = -xl-25 = — or 0 = sin ^ (3/4) 1 1 1 ^
5 4
- = (1-5-1) -- = 0-5x- =
/i —r r r

e
fl =

ree
14. (A)

rFl I 1 1 0-5

Fre
— = (1-5-1)
fl OO r
fl r

rrF
1 (\ \ (2 1 ^ /2 = -2r
- = (H-1) -+- =(1-5-1) - =
ouur
/
f=r
r r)
sffoo r
J__J_
/“/l‘'/2“r 2r 2r
1_ 1
okks
J__J_ J_
^'4
/=2r
Yo
D —> /?
ooo

A-¥ q
BB

Thus option (b) is true.


15. In Fig. 9(CF).37, AB is incident ray suffering two
rr e

refractions in the spherical glass paper weight and


(B) emerging out along CD. The angle of deviation
ouu

(from AEBC).
ad
YY

1 1 1 0-5
dd

- = (1-5-1)
fl
Re

CO r r
iinn

/l = 2r=/2
F

L=J- _L 1-J_ _L
/ /l''/2 f~2r^ 2r
f=r
B —> s

(C)
6 = ZEBC + ZECB

= (a-p) + (a-p) = 2(a-p)


1 1 1 0-5 16. As the reflected and refracted components are
_ = (l-5-i) perpendicular to each other, therefore, / must be
/] OO
r ) r

the polarizing angle. According to Brew.ster’s law,


/,=-2r
p = tan I
9/194 "PncuUe^ Fundamental Physics (XII) wimn
Since, |i for green colour and violet colour is
For Difficult Questions
greater than 1-414, so they will undergo total
internal reflection. Red colour will not undergo
total internal reflection. Therefore, it will be
separated.
19. When plane surface is silvered, light suffers
refraction at convex surface, then reflection from
plane surface and final refraction at the curved
surface again as shown in Fig. 9(CF).40.
FIGURE 9(CF).40

1.
>
2

ooww
sin/ _ ^2
As = p = tan /
sin r

From Fig. 9(CF).36,

^^2

e
sin I
Power of such a lens,

ree
P = P^ + P2+P^ = 2P^ (V ^2 = 0)

rFl
Fre
0
17. Let thickness of B = I
— = 10D

rrF
thickness of C = 2 / / 0-2
and thicknessofA = / + 2r = 3r 100 100
ouur
If Xq is wavelength in air, then as number of waves
in A = no. of waves in B + no. of waves in C. sffoo New focal length, f -
20. For a convex lens, plot of v versus u is as shown
P 10
= 10cm
okks
3t t 2i in Fig. 9(CF).41.
Yo
A.(j/py
ooo
BB

or
3 ^0 = ^^1 + 2 P2
Pi=3po-2p2
rr e

or

= 3x 14-2x 1-2
ouu

= 1-8
ad
YY

18. In Fig. 9{CF).39, on face AC, Zi = 45° ■►u

For total internal relleclion, i > C, so sin / > sin C


dd
Re
iinn

Let the co-ordinates of P be (j:, x)


F

u = - X, V =^x

1 1 1
From
V u /
1 1 2 1
or x = 2f.
X / ^ /
I 1 Co-ordinates of P ar&i-2f,2f)
As 11 = sin C = —
sin C 21. From lens maker’s formula

1 g 1 1

sin 45° > - .


1 J_.> —1 / It R
1
^ « / \
P-
(3 I
\ ( 1 1

or li> 1-414 2 ) R, " /


2 R, ^2
RAY OPTICS AND OPTICAL INSTRUMENTS 9/195

24. Here, / 1= 35°, 5 = 40°, ?2 = 79°, M = ?


For Difficult Questions
As /\ + 5 = /j + J2
1 .^3/2
\ /
1 1 1 1 A = (/, + h) - 6 = 35° + 79° - 40° = 74°
-1
/, 1*^/3 J[R^ Z V
R,‘ R^■ y ix =
sin(A + 8,J/2 sin(74°+40°)/2
sin A/2 sin 74°/2
and ! ^3/2 -1.Vj sin57°
5/3 0-8386
/?, R,- y
= 1-4
sin 37° 0-6018
I 1

w
10 R
Here, we have assumed that 5(^j„ = 40° which may
I not be true. Hence maximum possible value of jo.
will be closest to 1-5.
I
Clearly, — < —
or /, >/ 25. In Fig. 9(CF).13, applying Snell’s Law at L,
A f

roow
sin 45°

e
sm a
and fi is negative. ^ = - = V2 =

re
sin .r sm .r
Choice (d) is correct. 1 1

22. If 6 is critical angle between the pair of media in


smr,
sin 45° _ I/V2 _ 1 = 30°

reF
contact, then from Fig. 9(CF).42. V2 ~HT~2

uFFll
e
1 1
sin 0^ = ■●■ec = 45°

sFr
In APLM, (0 + 90°) + r, + (90 - 0^,) = 180°

foro
0 + ri - 0^ = 0 or 0 = 0^ - ri = 45° - 30° = 15"
uor
fk 26. When angle of incidence (0) is less than critical
okso
angle (0^.), there will be partial transmission and
partial reflection of light in a glass medium. When
Y
Yo
0 > 0^. there will be 100% reflection of light. In
oo
BB

this situation, the transmitted intensity of light is


zero. Choice (c) is correct.
11-54/2 5-77 27. For refraction at air-oil interface, we have
rYree

tan 6 = = 0-577
10 10
ouu

7
0 = 30° M = - 24 cm, Jl, = 1,1^2 = — 1
4
ad
Ydo

!_ = 9 R = + 6 cm, V =v^ (say)


Now M2 “
sin0 p sin 30 M1
nidn

As
2p = 2-72, p = l-36 u V R
Re

23. Given, U] = 1-5 x 10*^ m/s ; V2 = 2-0 x 10^ m/s 1 7/4 (7/4)-1 3
F
Fi

Refractive index for medium M,I is -24 V 6 24


I
c 3x10^
^^l=- = 2 _7__ j L-A-_L
V
1
1-5x10*^ or
4v.1 ~ 24 24 ~ 24 “ 12
Refractive index for medium M-, is
12x7
c 3x10^ 3 or V
1 - 21 cm.
4
= ~ = r = T
" 1^2 2-0x10^ 2 This image will act as object for oil water
If / is the angle of incidence and C is the critical interface. Taking refraction at oil-water interface,
we have
angle then for total internal reflection
sin / > sin C 7 4
But sin C = M2/Mi w = + 21 cm, V =v->, 1^1 “ 4 ’ 1*2 = 3. /? =

sin / > — or / > sin


-1 f3 As ^^1 I ^^2 _ ^^2 ~ ^^1
2 4 u V R
9/196 Fundamental Physics (XIDEEHD

30. Here, M = - 24 m = - 240 cm, v = \2 cm.


For Difficult Questions
I 1
From

_(7/4) (4/3)- 4Z3-7/4 / u

= 0
21 V CO 1 1 20+1 7 -I
2 = —cm
12 240 240 80
or
i>2 = 16 cm
.X = 18 - 16 = 2 cm As 11=1-5,
Hence,
real thickness of glass plate, x = 1 cm.
28. Refer to Fig. 9(CF).43. Here, 0 acts as a virtual
object and / acts as final image of the converging X 1 2

oww
beam of light passing through a diverging lens. apparent thickness, y-^-rr-rCni
H 1-5 3

. 2
= 1 — = - cm 1
Shift in image =x-y 3 3

e
1 35

re
i;' = 12-- = —cm m'=?
3 3

FFrllo
reF
1 1 1
From

e
f v' U
ouru 3 7 48-49

osrF
u'~ if f 35 80 35x16
Here,
1
« = +10cm,
1 1
1-__L ffor
p = + 15 cm,/= ?
k u' =
-35x16
cm = - 560 cm = - 5*6 m
kso
As — + _
/ u V f~ 10 15 ” 30 Object has to be shifted to 5-6 m from the lens.
ooo
Yo
Y
or /=-30cm 31. In Fig. 9(CF).44,
BB

29. Using lens maker’s formula


1 1 1 >
- = (li-l) —-
r ree
Y

f R R,~
\
uu

For first piano convex lens.


ad
doo

1 1 1 1
- = (1-5-1)
Y

14 OO 28
nidn

For second piano convex lens.


Re

1 1 1 1
F
Fi

- = (1-2-1)
OO -14 70

focal length of concave mirror = /] and focal


If/ is combined focal length of bi-convex lens,
length of convex lens ^f^-
then 1-J_ J_-J_ 1 .5 + 2 ^ 1 A beam of light coming from infinity and
7 ~ /i /2 ~ 28 ^ 70 " 140 ~ 20 falling on convex lens, converges at focus 0 of
convex lens. Therefore, CjO =/2. The rays of light
/= 20 cm
from O will retrace their path on reflection from
For this lens, u = - 40 cm, p = ? concave mirror only when O is at centre of
curvature of concave mirror, i.e., OP = R = 2 J\.
From
p u / C^P = d=C^O+OP = {f2+2f^)
32. Here, d = 6 cm, /i = 3 mm - 0-3 cm,
1 1 1 _i !_ = _L p = 2 X 10* m/s,/= ?
p f u 20 40 “ 40 If R is radius of curvature of the convex surface
V - 40*0 cm of lens, then from
RAY OPTICS AND OPTICAL INSTRUMENTS 9/197

For Difficult Questions


L-1 l_i-I or P2=f
F. f~f
c/f
- =(2R-h)h = 2Rh (V h«R)
J Hence ^I = i
d}/A _ 6-/4 h 2
R = = 15 cm
2h “ 2x0-3 35. Here, w = - 10 cm, t; = + 10 cm,

1111 1 _ 1
c 3x10^ _ 3 _IO-5-/=5cm
H=- / V u 10
V 2x10^ ~ 2
1 ( 2
1
From — = (^-1) ] 1_ Glass plate shift = / 1 — = 1-5 1 — = 0-5
3
/
[R, R^ New u= 10 - 0-5 = 9-5 cm.
\ /
1 1

ww
1 1 1
12 / \
oo 15 30
From
f = 30 cm V u /
1 1 1 47-5
33. Here, ji = -73, ‘ = 60®

Floo
.●. V = cm
V -9-5 5 4-5
Fig. 9(CF).45, Taking refraction at face XT, we have

e
47-5
Shift in the screen -10 = 0*55 cm,

eere
4-5
towards the lens.

FFr
36. At 0 = 60®, ray is incident at critical angle c at the

oorr
second surface.
uur r
sk s ff
YYoo
ooko
eBB

Sint Sint
uurr

F = - or sm r =
73 ~ V3 ~2
ad

sinr
sin 0
= 14 = -^
Yo

or r = 30°
sin .r
1
It means the refracted ray BD goes making
dY

ZN'BD = r= 30®. sin 9 sin 60° _ -^/2 _ 1


Re

sm ,r -
1
ZDSy= 90°-30° =60®
nind

The reflection ray at face XY is BC, where r, =30° : r2 = C=45°


FFi

ZNBC = angle of reflection = 60°.


.-. ZCBy=90°-60® = 30°.
^ sin 45° = nsin90°
Hence angle between reflected ray and refracted 73 sin 45° S
n =
ray = ZCBY + ZDBY = 30° + 60® = 90®. sin 90° " 72
34. Equivalent focal length in air is
fi
2_3
J__J_ J___2 2
F^~ f~ f FIGURE 8(CF).46 37. Here, R^ = 20 cm, /?2 = - 20 cm, )0. = 1-5
/
—/I“ = (n-l) LVq.s-I) —+ —
1 1
or F, = —
^ 2 ^ .4 /
20 20
When glycerine is filled 2 1
inside, glycerine lens be = 0-5 X — =
20 20
haves like a diverging lens
of focal length (-/). or /= 20 cm = 0-20 m.

$ /
9/198 ^ Fundamental Physics (XII)

FIGURE 9(CF).49
For Difficult Questions

1 1
Power of lens, P = — = 5D ♦-
/ 0-20 + -::-F
38. Rainbow cannot be observed when observer faces
towards the sun.

39. Here, C = 45°, y = ? B


A

c c c K ●A
As }x = —, y = — ^i =
V M sinC V

y = csin C=3 X 10^ sin45° = ixlO* m/s Distance between the two lenses = -
V2

ww
= 20 - 5 = 15 cm
40. Here, f* = 10 dioptre, /? = 10 cm, =?
44. For convex lens,
1 1 1 ^ u = -60cm, /=30cm, y=y I = ?
From f’ = — = (p-l)
/ R R

Flo
1 1} 1 1 1
or
10 = (n-l)(10+10) = (|i-l)20 30 60
/

e
y u y
1

eree
1 1 1
^-1= —
10 1
i.i = i y II = 60 cm
^=2 30 60 60 ’

FFr
20 2 2 V
1

41. As p of material is not known, we cannot measure Thus /j is the image formed by the lens, where
oorr
uur r
focal length of the lens. Using a spherometer, L/j = 60 cm.
sf
we can measure radius of curvature R of the Distanceof image I-y from plane mirror = 60-40
curved surface. = 20 cm.
sk
Yoo
42. As is clear from Fig. 9(CF).48.
oook
eBB
uurr
ad
Yo
dY
Re
innd

Taking second refraction from convex lens, we have


FFi

II = - (40 - 20) = - 20 cm, /= 30 cm. y = ?


1 sin 2 sin 30° ^ ^ *
From we get y = - 60 cm
p sine sine y u /’
1 Thus the final image is virtual, at a distance of
sin e - |X sin 30° = X “ = sin 60
(60 - 40) = 20 cm from plane mirror.
e = 60° 4

43. Here, ^ = 20 cm, /g = 5 cm 45. Here, €^=3 and “ 3


A parallel beam of light after refraction from
convex lens A converges at the focus of convex - 2
lens. The beam after refraction from convex lens
passes through concave lens, and on refraction 1
sin C = — - = 0-5
through concave lens becomes a parallel beam. 4 2
Therefore, light refracted from convex lens
virtually meets at focus of concave lens. C= sin-1 (0-5) = 30

«
RAY OPTICS AND OPTICAL INSTRUMENTS 9/199

For Difficult Questions

46. For a piano convex lens,


R , =oo , /?2 = - 30 cm, |i = 1-5
From lens formula.
1 1 ^
- = (p_l)
/ ^ [r^ R, 1 /

= (1.5-l)fl + ±
30
91-1.
30 " 60

oww
f= 60 cm n =
sin i,
or ^ _ sin 60°
sin ,r
sin i sin Tj 1
1
47. Here, }0. = , As ij = 6
sin .r
1 sin .r
V3/2_ I /●, = 30’
1
^ ^2

e
FFrlo
re
As ZO + ^=I80°, ZO= 120°

ZO + r,+r2= 180°, /'l = i'2~

ree
F
sin 6
n =

rF
sin

fsoor
ouur
sin 0 = « sin = n sin 30° = —
2 2
skf
Differentiating w.r.t. n, we get
ooko
sin 6 = p sin
Yo
COS 0
Y
sin 6 sin 9 dn 2
Bo

or sm T] = or /●, = sin
As r, = r2 /, =0 = 60°
reB

As /-j + T2 = A, t/e 1
= 1 or m = 1
sin 0 dn 2 cos 60°
uur
oY

r2 = A - T] or ^3 = A-sin ^
49. As the ray refracted from first surface of prism
ad

returns back through the same path, it must be


dY

-1 m falling on the silvered surface at 90°, as shown in


This shows, >2 Fig. 9(CF).53
innd

V /
Re

A-sin
-1 sin 0
< sin '
rn
Fi
F

-1
1 sin0
or A - sin < sin
1^

r w sin 0
or sin A - sin <

/ /
M

-1 1
or p X sin A - sin < sin 6

1\ r = 90° - (90° - A) = A, when


or 0 > sin ^ /isin A—sin *
\fij i = 2A

48. Refer to Fig. 9(CF).52, sin/ sin 2 A 2 sin A cos A


P = - = 2 CO.S A
Here, /, =60°, n = ^|3. A =60° sin r sin A sin A
9/200 4. Fundamental Physics (XII) twwn

For Difficult Questions


X 10-1
1-39x10^ 1-5x10"
50. As shown in Fig. 9(CF).54, green light incident
at critical angle 0 gets totally internally reflected. 1-39x10^x10-*
X = = 9-2x10-'* m
For frequencies less than green light. Wavelength 1-5x10"
is higher, refractive index is lower ; critical angle 55. FIGURE 9(CF).S5
is higher. Therefore, these frequencies will emerge
out into the air, without suffering total internal Objective Lens Eye Lens
reflection. Choice id) is correct. A A'
/S
V
V

ww
H —7 7 H
d =/o *fe

f
Magnification by eye piece, m =

Floo
f+u
ky -/ fe
As in-~ = —

e
L 4+(-(/o+4))

eere
sin(A + 5m)/2
51. From prism formula, p. =
sin A/2 Iv « = -(/o+/e>]
/ 4

FFr
For8,„=A,
i /o
oorr
uur r
sin A 2 sin A/2 cos A/2
M =
4
s ff
sin A/2 sin A/2 Magnifying power of telescope =
= 2 cos A/2 fe ■'
sk
56. In the eye of the person there is a defect for seeing
YYoo
When A = 0“, p max = 2 cos 0° = 2
ooko

the near objects as well as far off objects. He inten


When A = 90", p„,:„ = 2 cos 45" = ds to read a book keeping it at a distance 25 cm
eBB

from the eye. The lens used should be such that


Choice ib) is correct. the image of this object by lens should be formed
52. Here, D = 10 cm, x = 1 km = 10^ at distance 40 cm from eye lens, so that the final
uurr

cm

image formed by eye lens is very clearly seen.


ad

Distance between two objects, d=l


Yo

X = 5000k = 5x 10-5 cm Taking refraction from the lens used, we have


u = -25 cm, u = - 40 cm ;
dY

Limit of resolution dQ = — = —
X D The lens formula is L__l+1
Re

/
nind

U V

j 1Q5x5x10-5 = 0-5 cm = 5 mm J__ J__J 1 _ 3


FFi

or
D 10
f~ -25 -40 “25 40 “200
V
0 . 200 2
53. For microscope, M =
^=1
or — cm = — m
—u
0 V
/. 3
1
f f f Power of lens P = — = +1-5 D
and for telescope, M = — 1 + -^ / 2/3
fe d
57. Refer to Fig. 9(CF).56
When we increase : M for microscope
decreases and M for telescope increases.

54.
Diameterof sun’simage
Diameterof sun

Distance of image from lens


Distance of sun from lens

0 f
RAY OPTICS AND OPTICAL INSTRUMENTS 9/201

In normal adjustment, distance between objective


For Difficult Questions
and eye piece = Up +/^ = 50 + 4 = 54 cm
60. All the alternatives (a), (b) and (c) hold good.
sin0 - 0-25 ^ 1
25 100 61. Here, tube length. L =/^ +/^ = 60 cm
Resolving power of eye is
Magnifying power, M = — -5 ●●● 4 = 5/,
RP =
1-22X _ l-22x(50QxlQ-^) 4
2psin0 2xlx(l/100)

w
From L =4 +/^ = = 6/,.
= 3-05 X 10“^ m == 30 pni . 60
58. As is known from theory, for a telescope, 60 = 64 or 4 = y
= 10cm

f
angular magnification = , and angular 62. Let the required sepiiration be x.
4

e
D
\-21\

row
resolution = dQ = , where D is aperture diameter

re
\-21X ■
D
For large angular magnification,^ should be large X 1-22X
and for high angular resolution, objective must

FFllo
eeF
d D
have large diameter.
where d - distance between earth and moon

u
Choice (a) is correct.
59. Here, /q = 40 cm, 4 = 4 cm, Uq = - 200 cm \-22\

r
X d

sFr
X =
! D
As

kro
V
0
u
‘‘O 4 l-22(5500xl0“'”)x4xl0'
uor
V f0 u
J
40
1
200 ~ 200 ~ 50
4 offo1
X - 53-68 m
5

0 0
kos
Y
which is closest to choice (c).
Vq = 50 cm
Yo
eerBB
oo

m Multiple Choice Questions (with One or More than One Correct Answers)
rY

●>
1 fh dv _ v~
u

63. As M = and Here, i = C = 45° m = — or m = —


sin C h du h
1
ou
ad
do

1
4 = = V2=1-414 V-
nY

or
sin 45° u

For total internal reflection p > 1-414,


nid

\2
Re

both options ‘c' and ‘d’ are correct. f V /


/»2 = /> as m = —
F

64. Here, Pj = 1, P2 = u-f ii u-f


Fi

Let PO=OQ = x 66. Here, Angular magnification of telescope


As light is travelling from Rarer to Denser medium.
= ^ = 5.
fe
R ...(/)
—u V
4 = 54
1 1-5 1-5-1 Now, /o +4 = ...Hi)

-i-x) X R On solving (0 and {ii), we get

1 1-5 0-5 2-5 0-5


54 +4 =
- + or or X = 5R 4 = 6 cm,4, = 30 cm
R R
X X X
67. (a) distance between objective and eye piece
65. As —+ —= 4^ differentiating we get L =4 +4 = 16 + 0 02 = 16-02 m
u V f {b) magnification of telescope
0
1 du 1 dv dv v~
= 0 or = -800
2
ir dt v^ dt du u
fe -0-02
9/202 ^%<uCee^ '4- Fundamental Physics (XII) QsSD
therefore, the ray PQ meets the face CD at an
For Difficult Questions
angle of incidence greater than critical angle.
(c) final image formed by astronomical telescope Hence, total internal reflection takes place at face
is always inverted CD of prism. The ray goes along QR and meets
(d) The aperture of objective in astronomical the face DA at an ingle of incidence 30®. The
telescope is larger than that of eye piece emergent ray RT will go making an angle of
68. For interference to occur, the two .sources should emergence 60®. The incident ray OP tind emergent
ray RT will be inclined at an angle 90®.
be coherent, i.e., their frequencies should be equal,
Choices (a, b, c) are correct.
CO 2o)
either or . (/) and (ii) have same 71. As image is of double the size of object,
2tc 2tc f = 2 a = - 60 cm.
CO
Here a = - 30 cm
frequencies and (Hi) and (/V) also have same
2h
FIGURE 9(CF).S8

w
20)
frequencies
27t

Iu

Flo
69. As 1 + 1- V =
O
VJ
I

V u /’
■H

ee
putting u - 42 cm, 48 cm, 66 cm, 78 cm, v comes 30 cm 1? = 60 cm

out to be 56 cm, 48 cm, 37-7 cm and 34-6 cm. So,

Fr
1 1 1 1
(a) (h) are correct. And (c), (d) are incorrectly As
recorded. A V u 60*^30 20 /[=20cm.
70. Refer to Fig. 9(CF).57, critical angle of material
for
ur
In 2nd case, convex surface of lens acts as a convex
(I 1 mirror forming a faint image at a distance of 10 cm.
of prism, C = sin"‘ = sin

V3 M = - 30 cm,
ks
FIGURE 9(CF).59
Yo
For refraction at face AB. i; = 10 cm
oo

sin 60® sin60® V3/2_l using mirror formula


eB

|i = — or sin r = 0 I
sin r 1^ ^2 I _ 1 1
H4- ■W
r=30° V u
30 cm 10 cm
r

Z5PQ = 90®+30°= 120®


ou
ad

Hence, refracted ray goes along PQ. It meets the 1 _L^_2__j_


YY

face CD at angle of incidence = 45®. ~-30 10 ~ 30 ~ 15


- 15 cm, R - 2/2 - 30 cm
nd
Re

From —1 = _L_ * -Izl


Fi

A R
h R

(●.● /?2 = CO and /?] - R)


R 30cm
= 1-5
A 20cm

)i= 1-5 + 1 =2-5


Choices (a) and (d) are correct.
72. From Snell’s law, n sin 0 = constant
sin Qj = ;i2 sin 6y
Further, lateral displacement / depends on and
n (Z). But it is independent of Hj.
Choices (c) and (d) are correct.
:..-l
1 1 1 73. As thickness of film is uniform, the effective
As sin > sin
V2 V3 power of the film is zero. Therefore, we can
consider refraction at glass air interface.
RAY OPTICS AND OPTICAL INSTRUMENTS 9/203

length of divergent lens should be increased or


For Difficult Questions
its power should be reduced.
76. Imagine tlie entire path of beam is reversed. There
In case I : Refraction from air to glass
fore, the wave incident at Z/ gets polaiised on reflec
M1
i- +
tion. According to Brewster’s law, Fig. 9(CF).24.
II V R ^ = tan / = tan (90“ - r) = cot r
1-5 1-5-1 1 1
or
/i = 3R But l^ =
sin C
= cot r =
/] R tanr

In case II : Refraction from denser to rarer sin C = tan r, C - sin"* (tan r)


As r = '/ C = sin"* (tan r) = sin"* (tan /)

oww
p, ^ Pi _Pi -p. 77. As ground is at very large distance from camera,
u V R
therefore image of ground can be assumed to be
1-5 1-1-5 formed at the focus of the camera lens,
or
/2=2R t; = 50 m = 0-5 m, M = - 3 km = - 3(X)0 m.
f. R

e
Choice (a) and (c) are correct. If X is length of ground photographed, then

re
FFrlo
74. Speed of light in vacuum is an absolute constant. h. V 018 0-5
I
lml = or
It does not depend upon velocity of observer. As u X 3000

rF
ee
Va = Vq = t^c ’ therefore, Vq = 2
3000 x 0-18
X = = 1080 m

rF
0-5
ouru
75. A divergent lens has to be u.sed when far point is
1 m away. Further, if far point shifts ahead, focal .●. Area of ground photographed

fosor
skf = 1080 m X 1080 m

HH MuBtiple Choice Questions (Based on the given Passage/Compreh ension)


ooko
Yo
Y
78. Here, refractive index of surrounding medium For 5.I in water
Bo

As sin i,„ = n, sin (90'^ - C) ...O)


J_ 14579
reeB

NA =
Also, sin C =
I _ n
‘2
..m
4/3 V16 4 4UJ 16
ooY

ft n
8 7
uur

1
As NA = sin im For $2 in air, = 1, = —, n
2"5
ad
dY

From (0, sin /m = -^sin (90®-C) NA=-.


1 l64 '49 Vl5
n
s U25 25 5
nind
Re

n 4
_ 1
cos C = ~ -^1-sin^ C For 5-1 in water,
^ ^
— —
3
F
Fi

NA = J_ |64 49 _ 3-Jis
fVy
4/3 V25 25 "4 5
1
n 1
.-. NA = -^
n n n
-Jn," - nl [from(/i)]
s 1 s
16
For S2 in =
V45 3 3V15
For S] in air, = 1, tij = 4 ’
rt, =
2
3V15 |64_^
m = - l 45_9_3 16 U5 25“ 16
1\16 4 "4 4
6 For S2 in =
For 5j in = V5T
NA = VI5 M_49 _ 3
NA = Vis l45_9 _ 3Vi5 _ -Vis 4 V 25 25 “ 4
6 \ 16 4 ” 24 8 Hence, options ‘a’ and ‘c’ are correct
9/204 ^ Fundamental Physics (XII) ar»f 11
100 100
For Difficult Questions 81. Here, P = P,-k- =— = 1-25D
F 80
79.
100 100
FIGURE 1.60 ^3 = = -5D
h -20

w
power of the combination of three lenses is
F* = Fj + p2 + F3 = 1-25 - 5 = - 3-75 D
82. As the medium is denser w.r.t. vacuum, therefore,
on entering the medium, it will converge.

e
83. The speed of light in the medium is minimum on
the axis of the beam, because refractive index of

e
The numerical aperture is limited by second slab.

wr
medium is maximum at the axis of beam.

lloo
.●. sin I2 < NA2 [smaller NA]

r
84. The initial shape of the wave front of the beam is
sin ij > sin I2 planar, as the wave front lies in the plane of the

F
.'. NA of combination = smaller NA - NA2

u
beam.

80. Here, /j = 20 cm./2 = ? F = 80 cm. 85.

As

r FF _L J_-J_
As a ray of light travelling in air enters a meta-
material (with negative refractive index), it bends
towards normal, i.e., 62 < Oj. As refractive index

Frrsee
(n) of meta material is negative, the direction of
uo
k
1 1 1 I 1 3 propagation of refracted light is reversed.

o
Therefore, (c) is the appropriate ray diagram.
/2 ~ F ,/ " 80 20 “ SO
fo
86. Speed of light in meta material is given by the

fl-
80
3 = — 26‘7cm
ofr same formula, v = c/n. As n for meta material is
kso
negative, we write v =c/\ n I.
Y
m
B
Yo
eBrY

Matching Type Questions


oo

87. Reflection involves change in path of light without change in medium. Refraction is governed by Snell’s
re

law : fx = sin //sin r. Interference is destructive when light added to light produces darkness. Polarization is
uu

used for reducing glare. Choice (a) is correct.


od

Cl
Yo
ad

Matrix-Match Type Questions


n

88. In a concave mirror, image formed may be real, P2 > P3 as ray bends away from the normal.
ndi

inverted, smaller/larger or virtual, erect and larger In fig. (t) ; Pi > p^, as ray bends away from the
Re

depending upon position of object. In a convex


F

normal.
mirror, image is always virtual, erect and smaller.
Fi

So is true for concave lens. However, in case of a


P2 = P3 as there is no deviation.
Thus options are (a) (p, r) ; {b) (g, s, t) ;
convex lens, image formed may be real, inverted,
smaller/larger or even virtual, erect and larger (c) ^ (p, r, 0 ; (rf) ig, s)
depending upon the position of the object.
89. In fig. (p); fi2 = 1^3. as there is no deviation. 90. (P)
Pi < P2> as ray bends towards the normal.
In fig. (g); Pj > P2, as ray bends away from the
normal.
1 1 1 f2\ 1
- = (p-l) —h- = (1-5-1) - = =>/=r
/ r
\^rj r
P2 > P3, as ray bends away from the normal.
In fig. (r); P2 = P3. as there is no deviation.
_1__J_ J_
Pi < P2 as ray bends towards the normal.
In fig. (j); Pi > P2, as ray bertds away from the
r~ fx^ fl
normal. P^2
RAY OPTICS AND OPTICAL INSTRUMENTS 9/205

1 1 I 0-5
For Difficult Questions _ = (l-5-l) — — => /2 = -2r
fl \ ^
OO

(Q) 1=± _L= J 1_


/ fi^h 2r 2r => f^-r
R->3

^ = 0-5-1) 14^
1
=>/,=2r=/2 /V7
/l OO r r
(S)
vli
1 1 1 1 1 1
1 1 1
/ /l /2 7
/ = :r‘^^
2r 2r =>/=r - = (1-5-1) -- = 0-5x—= - => /| = r
/] -r r r

Q-»4 1 I n 1 0-5

ww
fTH — = (1-5-1) =* — = —-/2 = - 2 r
fl OO r) fl
(R)
m
/■///2~r 2r"2r

Flo
1 1 1'' 0-5
- = (1-5-1) --- => /i = - 2 r 1
/)

e
1,00 r r
Hence choice (b) is correct.

reree
VI. Integer Type Questions

r FF
91. Refer to Fig. 9(CF).61.
uurr
Here, \i = 5/3, OC = 2 m, AB = 2AC=? foor
A5 = 2AC = 2x-m = —m = 3*2m
5 5
ks s
FIGURE 9(CF).61 The correct integral answer is 3 m
Yoo
92. As is clear from Fig. 9(CF).27
ooook

for 77/?, C=i = 45°


eBB

1 I
From — = = V2
^^l sin C sin 45°
uurr
ad

li2 = M-i = X V2 = 2
Yo

93. Here, /?| = 20 cm,


dY

/?2 = - 20 cm, = 3/2, /* = ?


Re
innd

I 1 1
^ = (^-1)
/ /?. /?2
FFi

To cut off all light coming from the source out of


water, total internal reflection should occur at A
(3
\ /
and B, for which
--1 —+ —1= —
1 5 3 1.2 y V
20*^ 20 20
11 = —~~~> sinC = -=sin9
sinC 3 5 /= 20 cm

4 _ AC 4 8 P =
100 _ 100 = 5D
.-. tan 0 = - AC — —x2 = -m
5 ~ OC 5 5 / " 20

VII.
Assertion-Reason Type Questions

FOR MEDICAL STUDENTS internal reflection of light takes place if angle of


94. Both, the Assertion and Reason are true and incidence is greater than critical angle. So, small
diameter ensures that incident light falls at an
latter explains the former correctly. Total
angle greater than critical angle.
9/206 ‘P'uuUe^ 4. Fundamental Physics (XII) wjhi
103. Statement-1 is false, but statement-2 is true.
For Difficult Questions
Focal length of a lens does depend on the
refractive index of lens material w.r.t. the
95. Both, the Assertion and Reason are false.
Frequency is that characteristic property of light surrounding medium.
104. Both the statements are true and statement-2
which does not change when light goes from one
medium to another. So assertion is false. As glass is correct explanation of statement-1.
is a den.ser medium in comparison to air, velocity 105. Statement-1 is true, but the statement-2 is false.
of light in glass is less than that in air. |i does not depend upon A and 5„,.

w
96. Both, the Assertion and Reason are false as 106. Statement-1 is true, but the statement-2 is false.
prism is not the source of colours. It only splits For a ray incident normally,
white light into its constituent colours. Different ^i = 0 Zr = 0.
colours have different wavelengths. The prism has Also, sinC=l/M.

e
different refractive indices for different colours.

row
107. Light travels faster in air than in glass, because

re
97. Both, the assertion and reason are correct and glass is denser than air. Statement-1 is false, but
the latter is correct explanation of the former. statcment-2 is true.

FFllo
98. As.sertion is true, because frequency in all the

eeF
108. For an equiconvex lens, if = /?, /?2 = - /?
three beams is dependent on source of light. The

u
reason is also true, but it is not a correct
explanation of the assertion. ■■■ f\i

r
sFr
99. The sun looks bigger in size at sunrise and sunset /=R

kro
because of refraction of light. So the reason, Hence both the statements are true and state
though correct, is not a correct explanation of the
uor
offo ment-2 is correct explanation of statement-1.
assertion.
109. Longitudinal chromatic aberration
kos
FOR ENGINEERING STUDENTS (/; -y;,) = «/= o-08 x 10 = 0-8
Y
Yo
Both the statements are true and latter is
eerBB

100. Both the statements are true and statement-2


oo

is correct explanation of statement-1. correct explanation of the former.


rY

101. As |i of lens is different for different colours/ 110. As light is entering from air to glass, the reflected
wavelengths, therefore, different colours are ray suffers a phase change of n. Due to it, at the
u

focussed at different points. That is why the image centre, there will be destructive interference.
Thus, both the statements are correct and
ou

is coloured. Choice (a) is correct.


ad
do

102. A virtual image can certainly be photographed. statement (2) is the correct explanation of
nY

Statement-1 is false. Statement-2 is true. statement (1).


nid
Re

VIll. Multiple Choice Questions (Based on Experimental Skills)


F
Fi

111. In the said experiment, all distances are measured 115. Parallax is removed when object needle is at
by a vernier scale provided on the microscope. focus of the lens. On refraction through convex
112. In a convex lens, u is negative and v is positive lens, rays are rendered parallel. They fall normally
(except when image is virtual). As u is increased, on the plane mirror, and retrace their path on
V decreases. Therefore, graph (c) is the correct
reflection. The image of the object is formed at F
choice in Fig. 9(CF).28.
only. That is why parallax is removed.
113. In case of a convex lens, u is negative, v is posi
tive. Also \/v - l/u = 1// = constant. Therefore, 116. Choice (d) in Fig. 9(CF).30 shows the correct
shape of graph between \/u and l/u is as shown course of rays through prism in minimum
in Fig. 9(CF).29(a). deviation position. This is because in this position,
114. In a convex lens, when object is at infinity, image the refracted ray is parallel to the base of the prism
is at focus, i.e.
and incident ray and emergent ray are inclined
1 1
when M = 00, — = 0, — = OB = — ●’● /= l/OB. equally to the two faces of the prism.
u V f
ww
WAVE OPTICS

Flo
e
10.1. INTRODUCTION

ree
The ray optics uses the geometry of straight lines to account for the macroscopicphenomenalike rectilinear

Fr
propagation of light, reflection of light and refraction of light etc. However, the microscopic phenomena like

rr F
interference, diffraction and polarization could not be accounted for by ray optics. To explain these phenomena,
uurr
concept of waves was introduced. The new branch of Physics based on the wave concept of light was called
sfo
Wave optics or Physical optics. The wave theory of light was put forward, first of all by Huygens and, later
on modified by Frensel. According to wave theory of light; light is a form of energy which travels through
kks
a medium in the form of transverse wave motion. The speed of light in a medium depends upon the nature
Yo
oooo

of medium. Huygens supposed the existence of a hypothetical medium called ‘luminiferous ether’ which
filled the entire space. This medium was supposed to be massless with extremely high elasticity and very low
eB

density.

The speed of light in the medium ether was assumed to be given by u = ^£/p , where E is elasticity of
ur

the medium and p is density of the medium. As velocity of light is really large, the hypothetical medium ether
ad
YYo

should have extremely large elasticity and very low density. Such a medium cannot exist and that is why ether
was never detected. This problem was resolved by Maxwell who proposed electromagnetic theory of light.
dd

According to Maxwell, light is an electromagnetic wave, which consists of mutually perpendicular time
Re

varying electric and magnetic fields. The electromagnetic light wave travels in a direction perpendicular to
inn

both, the electric and magnetic fields. The electromagnetic theory could explain satisfactorily the wave
F

phenomena of interference, diffraction and polarization, but it failed to explain phenomenalike photoelectric
effect, Raman effect and Compton effect. This theory could not account for emission and absorption of
radiation. Einstein pul forward Quantum Theory of light, according to which light consists of photons.
Each photon is a packet of energy, E = hv, where h is Planck’s constant and v is frequency of radiation. On the
basis of this theory, the phenomena of photoelectric effect. Raman effect and Compton effect were accounted
for but this theory failed to explain the phenomena of interference,diffractionand polarisationof light. This
situation led De Broglie to suggest that just as matter has dual character, light too has dual nature. His
conclusion was based on the fact that Nature loves symmetry. Our universe is made up of material particles
like electrons, protons, neutrons and radiation. Both, the matter and radiation have dual characte.r If radiation
can behave like a particle (photon), then all material particles can also behave as radiation. With the dual
nature of light, all the observed phenomena in optics could be explained.
In this chapter, we shall study the various phenomena related to Wave Nature of light.
10/1
10/2 ‘P'MeUe^'4. Fundamental Physics (XlDKSSm
10.2. WAVE FRONT

A wave front is defined as the continuous locus of all the particles of a medium, which are
vibrating in the same phase at a given instant.

Infact, a source of light sends out disturbance in all the directions. In a homogeneous medium, velocity
of light waves in all the directions is the same. Therefore,disturbance reaches at the same time, at all such
particles which are at the same distance from the source. These particles will naturally vibrate in phase with

w
one another. The locus of all such particles is being called the wave front.
Depending on the shape of source of light, wave front can be of three types :
(i) Spherical wave front: When the source of light is a point source, the wave front is a sphere with
centre at the source. Fig. lO.l(fl).

e
row
re
FFllo
eeF
u
r
sFr
kro
uor
offo
(//) Cylindrical wave front: When the source of light is linear, e.g. a slit as shown in Fig. 10.1(/>), all
kos
Y
Yo
the points equidistant from the source lie on a cylinder. Therefore, the wave front is cylindrical.
eerBB
oo

(Hi) Plane wave front: When the point source or linear source of light is at very large distance, a small
rY

portion of spherical or cylindrical wave front appears to be plane. Such a wave front is called a plane wave
front, Fig. 10.1(c).
u

In a given medium, we can draw a set of straight lines which are perpendicular to the wave front.
ou

According to Huygens, these straight lines are the rays of light. Thus rays are always normal to the wave
ad
do

front. According to Huygens, light energy flows along rays.


nY

Figures 10.2. (a), (b) and (c) represent wave front and rays of light corresponding to plane wave front
nid

(PWF), diverging spherical wave front (diverging SWF) and converging spherical wave front (converging
Re

SWF) respectively.
F
Fi

Converging
.PWF

>-

>*RAYS

e
WAVE OPTICS 10/3

10.3. H- XENS PRINCIPLE

Huygens gave a hypothesis for geometrical construction of the position of a common wave front at any
instant, during the propagation of waves in a medium. This principle tells how a wavefront is propagated
further in a medium. According to him,
1. Every point on the given wave front (called
primary wave front) acts as a fresh source of new
disturbance, calledsecondary wavelets, which travel
in all directions with the velocity of light in the
medium.

2. A surface touching these secondary


wavelets, tangentially in the forward direction at
any instant gives the new wavefront at that instant.
This is called secondary wave front.

ww
To illustrate Huygens principle, let AB be a
section of the given wavefront (called primary

Floo
wavefront) at any instant. Fig. 10.3(a) shows a
primary spherical wavefront AB and Fig. 10.3(/>)

ree
shows a primary plane wavefront AB. To find the
position of a secondary wavefront after a time /,

rFee
take points 1,2,3,4, ,on the wavefront AS. The
distance travelled by light in r sec = c x t, where c

F
oor r
rur
is the velocity of light in the medium. Taking each point as centre, draw spheres of radius = c x r. These
spherical surfaces represent the position of secondary wavelets at time t. Draw a surface AjSj touching
s ff
tangentially, all the secondary wavelets in the forward direction. The surface A^B^ is the secondary wavefront.
A surface A2B2 touching tangentially, all the secondary wavelets in the backward direction can also be drawn
osk
YYoou
giving a backward secondary wavefront, but it is contrary to observation. Therefore, we conclude that there
oook

is no backward flow of energy during propagation of a wave.


eBB

From the above discussion, it is noted that in a homogenous isotropic medium, during the wave
propagation, the radius of the spherical divergent wavefront increases uniformly with time and a wavefront
is propagated parallel to itself.
uur r
ad

Huygens principle can be used to explain the phenomena of reflection and refraction of light on the
Yo

basis of wave theory.

10.4. KEFl ‘ or IVAVE THEORY


dY
Re
idn

In Fig. 10.4, AB is a
FFin

plane wave front incident on


a plane mirror A/jA/2 at
ZBAA'= Zi. 1,2,3 are the
corresponding incident rays
perpendicular to AB.
According to Huygens
principle, every point on AB
is a source of secondary
wavelets. Let the secondary
wavelets from B strike M1M2
at A' in t seconds.
BA' = c X r ...(1)
where c is the velocity of light in the medium.
10/4 Fundamental Physics fXIIHviwii

The secondary wavelets from A will travel the same distance c x tin the same time. Therefore, with A as
centre and c‘ x / as radius, draw an arc B', so that
AB' = cxt ...(2)
From A', draw a tangent plane A'B' touching the spherical arc tangentially at B'. Therefore, A'S' is the
secondary wavefront after i seconds. This would advance in the direction of rays T, 2', 3', which are the
corresponding reflected rays perpendicular to A'B"
We can show that secondary wavelets starting from any other point D on the incident wavefront AB,
must reach the point D' on A'B', after reflection at P. and that too. in the same time as the secondarywavelets
take to go from B to A' or from A to S'.
Hence A'B' is the true reflected wavefront. In Fig. 10.4,
angle of incidence, / = ZBAA'
and angle of reflection, r = ZB'A'A

ww
In AMA'S and A A'S',
AA' is common. BA' =AB' = c X t,

Floo
and ZB = ZB' = 90°

.●. are congruent .'. ZBAA' = ZB'A'A, i.e.. Zi = Zr ...(3)

ee
which is the first law of reflection.

eer
Further, the incident wavefront AS, the reflecting surface M^M2 and the reflected wavefront A'B' are all

FFr
perpendicular to the plane of the paper. Therefore, incident ray, normal to the mirror A/jM2 and reflected ray

oorr
all lie in the plane of the paper. This is second law of reflection.
uur r
s ff
10.5. REFRACTION ON THE BASIS OF WAVE THEORY

In Fig. 10.5, XY is a plane surface that separates a


sk
YYoo
ooko

denser medium of refractive index p from a rarer


medium. If Cj is velocity of light in arrer medium and
eBB

C2 is velocity of light in denser medium, then by


definition,
uurr
ad

M = — ...(4)
Yo

^2
AS is a plane wave front incident on XTat ZBAA'
dY

= Zi. 1.2,3 are the corresponding incident rays normal


Re
ind

to AS.
FFin

According to Huygens principle, every point on


AS is a source of secondiuy wavelets. Let the secondary
wavelets from S strike XY at A' in t seconds

BA' = c^ X r ...(5)
The secondary wavelets from A travel in the denser medium with a velocity C2 and would cover a
distance (C2 x /) in t seconds. Therefore, with A as centre and radius equal to (C2 x i), draw an arc S'
From A', draw a tangent plane touching the spherical arc tangentially at S'. Therefore, A'B' is the secondary
wavefront after t sec. This would advance in the direction of rays I', 2', 3'. which are the corresponding
refracted rays, perpendicular to A'B'.
We can show that the secondary wavelets starting from any other point D on the incident wavefront AS,
after refraction at P. must reach the point D' on A'B' in the same time in which the secondary wavelets from
S reach A'.
Therefore. A'B' is the true refracted wavefront. Let r be the angle of refraction. As angle of refraction is
equal to the angle which the refracted plane wavefront A'S' makes with the refracting surface AA', therefore,
ZAA'B' = r.
WAVE OPTICS 10/5

Let ZAA'B' - r, angle of refraction.


BA' c.xt AB'
\xiAAAB, sin/ = In AAA'S', sinr =
AA' AA' AA' ~ AA'

sin / Cj
sin r [using (4)]
^2

Sin i
Hence l^ = - ...(6)
sin r

which proves Snell’s law of refraction.


It is clear from Fig. 10.5, that the incident rays, normal to the interface XK and refracted rays, all lie in
the same plane, (i.e. in the plane of the paper). This is second law of refraction.

loow w
Hence laws of refraction are established on the basis of wave theory.

Ssmple Problem A plane surface separates a denser medium of refractive index 1*5 from
air. A plane wavefront travelling in air is incident on the interface at an angle of 30°. What will be the
angle of refraction ?

ee
Sol. Here, i = 30°, r = ? ji = 1-5

Fr
According to Snell’s Law
Sin i
r FF
rree
= 1^
sin r
fofr oF
u
sin / sin 30° _ 1/2 _ -I = 0-3333
ks
sin r =
3/2 ~3^“3
YYouro
r = sin-' (0-3333) = 19-5°
soo
BBook

10.6. WAVELENGTH AND FREQUENCY


r ee

Suppose a source of light is at rest in one medium and an observer is at rest in another medium. Let there
be no relative motion between the two media so that geometry of .source, medium and observer does not
ouru
ad

change with time. Therefore, time taken by light to travel between source and observeris fixed.
Yo

If we imagine two wavefronts separated by time period T, emitted from the source, then they would
arrive at the observer separated by the same time interval T. As frequency (v) is characteristic of the source,
d

^ remains the same as light travelsfrom one medium to another, i.e. Vj = V2- As the speeds
Re

therefore, v =
iYn
FFind

of light V1 and V2 in the two media are different, therefore, wavelengths of light ^|, A2 in the two media must

also be different. As p =
^2 V X
2 '“2

X
As Vj = V2 ...(7)
X2
therefore, wavelength in a medium is directly proportional to the {phase) speed and inversely
proportional to refractive index.
Clearly, greater is the refractive index p of a medium, smaller would be the wavelength of light in that
medium.

Note that if the source or the observer is moving or there is a relative motion of the medium, then the
above reasoning will not hold. The details about changes in frequency and wavelength shall be discussed
under Doppler’s Effect given in chapter 15.

»
10/6 'Pxctdec^'4- Fundamental Physics fXlIYPTSTWm
Sample Problem Light of wavelength 5890 A travelling in air enters water of )i = 4/3. What
will be the frequency and wavelength of light in water ?
Sol. Here, = 5890 A = 5890 x 10"*o m
^ = 4/3, V2 = ? =?
As frequency in water = frequency in air,
c 3x10^
V2=V| = 5-093x10^4^/5
^11 5890xl0"‘0
X,
As

5890x10-^0

loow w
= 4417-5 X 10-10 m = 4417-5 A
4/3

10,7 BEHAVIOUR Of A PRISM, A LENS, AND


A SPHERIC : ● lIRROR TOWARDS PLANE WAVEFRONT

ree
Let us study the behaviour of a prism, a lens, and a spherical mirror, when a plane wavefront is incident

rree F
on them.
r FF
(a) Behaviour of a prism. In Fig. 10.6, ABC is a prism with small angle A, on which a plane wavefront
KB is incident.
fofr oF
Different parts of the wavefront travel different thicknesses of glass prism, maximum at the bottom and
u
minimum at the top. As speed of light in material of prism is less than the speed of light in air, therefore,
ks
YYouro
emerging plane wave front is CL, where
soo

(AA + AL) BC
BBook

dO)
speed of light in air speed of light in prism material
r ee

The corresponding rays have been shown by dotted lines. Clearly, on passing through a prism, the rays
ouru

bend towards the base of the prism.


ad
Yo
d
Re
iYn
FFind

{b) Behaviour of a lens. In Fig. 10.7, a plane wavefront AB is incident on a convex lens, X.Y Different
portions of this wavefront have to travel through different thicknesses of the lens before emerging out, being
maximum at the centre and minimum at the edges. As light travels slower in lens material than in air, therefore,
refracted wavefront is converging spherical wavefront A‘'P2 ^ > such that
AX + AA' BY + YB'
dl)
speed of light in air speed of light in air speed of light in lens material
The corresponding rays have been shown by dotted lines. Clearly, on passing through a convex lens, the
parallel rays tend to converge.

»
WAVE OPTICS 10/7

Again, let a point source of light be at the focus 5 of a convex lens, such that a diverging spherical wave
front is incident on the convex lens. The refracted rays become parallel to principal axis. Therefore, refracted
wave front is plane as shown in Fig. 10.8.
FIGURE 10.8
FIGURE 10.9


Refracted wavefront
(Plane)

ww
wavefront
(diverging)

(c) Behaviour of a spherical mirror. In Fig. 10.9, a plane wave front AB is incident on a concave

Flo
mirror with pole at P. The centre of the wave front has to travel the largest distance to P before it gets

e
reflected. The peripheral portion of wave front has to travel the smallest distance before getting reflected.

eree
Therefore, the reflected wave front is converging spherical wave front A" P" B" as shown. The corresponding
rays are shown by dotted lines. This accounts for the converging action of a concave mirror. Here, AA'A" -

FFr
BB'B" = PP' + P'P".
uurr
orr
Similarly, when a point source of light S is at the focus
of a concave mirror, then a diverging spherical wavefront
falls on the mirror as shown in Fig. 10.10. The reflected
sfo
Reflected
Incident
spherical
kks
4
Yoo
rays are parallel to the principal axis. Therefore, the Plane
oooo

reflected wavefront is a plane wavefront. Wavefront


4
eBB

10.8. SUPERPOSITION PRINCIPLE

Superposition principle enables us to find the resultant of two or more wave motions.
urr

According to superposition principle, when two or more wave motions travelling through a
ad
YYo

medium superimpose one another, a new wave is formed in which resultant displacement (y)
at any instant is equal to the vector sum of the displacements due to individual waves ^...)
dd
Re
inn

at that instant,
F

i.e., y = Vi+y2+ ...(12)

Clearly, each wave contributes irrespective of the others.


For example in Fig. 10.11 (a), when crest
of one wave falls on crest of the other, the
amplitude of the resultant wave is sum of the
amplitudes of two waves,
i.e.. y = >’,+>'2-
In Fig. 10.11 (b), when crest of one wave
falls on trough of the other, the amplitude of
resultant wave is difference of the amplitudes
of the two waves.
I.e.,
y = y\-y2
10/8 “Pnadeep- ^ Fundamental Physics (XII)

IMPORTANT NOTE

If the amplitudes of the two superimposing waves are equal, then on destructive interference, Fig.
10.11(b), the amplitude of resultant wave would be zero and zero will be the resultant intensity.
The superposition principle which was stated first for mechanical waves is equally applicable to the
electromagnetic (light) waves.

■' ,H -R'^’NT INCOHERFr-T addition OF LIGHTWAVES

Light waves are electromagnetic waves.


The electric vector is responsible for the optical effects of e.m. wave. To calculate resultant intensity of light
due to any number of light sources, we must add suitably the electric fields produced by different sources.
Let the oscillating electric field produced at a point P, by two sources be given by

w
£| = flj cos (coj t + 4>j), E2 = O2 cos (0)2 t + 4>2)
The intensities of light produced by the two waves are

l^-kE^=k a^ cos^ (co, / + (]),), l2=kE2=ka\ cos^ ((O21 + <1)2)

Flo
As frequency of light is nearly lO’^ Hz, we usually observe time average of this quantity. ^

ee
It is known that over a full cycle (0 to 2 Tt), average value of cos j: = 0 and average value of cos^ ^~~2

Fr
Average intensity of first wave, a^H and average intensity of 2nd wave, I^^k a|/2
When both the waves illuminate the same point, electric fields are superposed. The instantaneous intensity
for
ur
at a given time t is k(,E^ + E.^)^ =kE^ +kEl+lkE^E^
Averaging the instantaneous intensity over a full cycle, we obtain /=/j + /2+/i2
ks
Yo
Here, I^2 is the average oil kE^E2 and is called the interference term. Now,
oo

/j2 = 2 /fe £|£'2 = 2 Oj 02 cos (COj t + (|>j) cos (0>2 t + (j)2)


B

Using the identity : 2 cos A cos B = cos (A + B) + cos (A - B), we get,


re

f2 = 2kEy £2 =ka^ 02[cos {(cOj + CU2) ^ {(Wi - CO2) t + (<|}i - <|)2)}]


Averaging 2 k E^E2 over a full cycle, we find the first term (containing the sum of two frequencies tOj
ou
ad

and 0)2) becomes zero. The average of 2nd term is also zero if cOj ^ C02-
Y

If two frequencies are the same, the average oflk EyE2 is given by

7,2 = 2 £j£2 = k 0j02 cos (({)i - (|)2) I


12 =2.y7j7^cos((l),-(1)2)
nd
Re
Fi

Hence to obtain interference, we need two sources with the same frequency and with a constant
phase difference. Such a pair of sources is called coherent sources. Thus, when illumination is by two
coherent sources, resultant intensity is

/—/j+/2 + /j2 ~ ^2 ^ 2.^/1 I2 cos ((j)i ^2^


When we Have two independent sources, even of same frequency, their phase difference (4>| - <|)2) does
average of cos (<l)i - (1>2) = 0 over a full cycle. Hence
not have a stable/constant value with time. Therefore,
7,2 = 0. Therefore, I = I^+12
lory
Two such sources of light, which do not emit light waves with a constant phase difference are
called Incoherent sources. Superposition of light waves from two incoherent sources does not
show up interference. Total intensity at any point is simply the sum of time average intensities of
the two sources.
WAVE OPTICS 10/9

Sample Problem
Light waves from two coherent sources having intensities I and 2 / cross
each other at a point with a phase diff. of 60°. What is the resultant intensity at the point ? If the
sources were incoherent, what would be the resultant intensity ?
Sol. Here, /j = / and /2 = 2 /, (|) = 60°
When sources are coherent

Amplitude R of resultant wave is R = +/?- +2abcos<^


R- = a^ + b^+2ab cos (f)
As intensity « (amplitude)^
Resultant intensity, f/? = /i + ^2 + 2 cos
(J> =1 + 21+ 2^Jl x2I X cos 60'
= 31 + 1^=10+^) = 4-4141

ww
If the sources are incoherent, /^ = /j + /^ = / + 2 / = 3 7
10.10. COHERENT SOURCES

Flo
The sources of light, which emit continuous light waves of the same wavelength, same frequency

ee
and in same phase or having a constant phase difference are called coherent sources.

rere
r FF
The phase difference <j) between light waves at a given time and at a given position, is sum of
(/) Initial phase difference between two sources A and B, and
uurr
(«) Phase difference on account of difference in path lengths. The latter does not change with time (for
a given point P).
Hence, initial phase difference between two source.'; must be constant.
foor
ks s
Yoo
oook

Retain in Memory
eBB

Two sources are coherent when the waves they emit maintain a constant phase relation. Effectively,
it means that the waves do not shift relative to one another as time passes.
uurr

Two independent sources of light cannot be coherent. This is because light is emitted from individual
ad
Yo

atoms of the source, when they return to ground state after being excited by heat or electric discharge. Even
the smallest source of light contains billions and billions of such atoms which obviously cannot emit light
dY

waves in the same phase. Such independent sources of light whose light waves do not possess a constant
Re
innd

phase difference are called non-coherent sources or incoherent sources.


Two coherent light sources can be obtainedfrom a single source of light by reflection, refraction etc.
FFi

This is because all the random phase changes occuring in parent source are repeated in the image source so
that the relative phase difference between the two remains constant at all times.
In practice, coherence in interference is obtained by two methods :
(/) By division of wavefront such as in Young’s double slit experiment: Lloyd’s mirror experiment and
Fresnel’s biprism experiment.
(//) By division of ampUtui ' ■ by puitiul reflection and refraction of waves such as in interference of light
in thin films. Newton’s Rings d Michelson’s interferometer.
Conditions for Obtaining Two Coherent Sources of Light
1. Coherent sources of light should be obtained from a single source by some device. In this case, any
phase change in one is simultaneously accompanied by the same phase change in other. Due to it, the phase
difference between the two waves of light reaching at a point from the two coherentsources remains constant
with time and a steady interference pattern is obtained.
10/10 ‘pncidt^a ^ Fundamental Physics (XII) twii

The coherent sources can be obtained either by DO YOU KNOW


(i) the source and its virtual image or That a laser differs from common
(//) the two virtual images of the same source. light sources in that its atoms
(Hi) two real images of the same source. emit light in a co-operative
manner, thereby making the light
2, The two sources should give monochromatic light.
coherent. Laser light is almost
If the sources give white light, consisting of a number of wavelengths monochromatic with little
(colours), then the light of each wavelength gives its own set of spreading. Hence two inde
interference fringes. Since the fringe width is different for different pendent light sources (such as
wavelengths, overlapping of the fringes occurs, resulting in a few coloured lasers) can produce observable
fringes only near the central white fringe. These fringes are hazy, whereas interference on a screen.
those with monochromatic light are quite sharp.

w
3. The path difference between light waves from two sources should be small.
If the path difference between two interfering waves is large, there is intermixing of the two at every

Flo
point, resulting in uniform illumination.

e
ree
10.1 \ INTERFERENCE OF LIGHT

FFr
When there is a single source of light, the distribution of light energy in the surrounding medium is
uniform in all the directions. But when we have two coherent sources of light emitting continuous waves of
urr
same amplitude, same wavelength and in the same phase (or with a constant phase difference), the distribution
for
of light energy does not remain uniform in all the directions.
At some points, where the crest of one wave falls on crest of the other, resultant amplitude is maximum.
kkss
Hence the intensity of light is maximum. At certain other points, crest of one wave falls on trough of the other.
Yo
Resultant amplitude becomes minimum (zero) and hence intensity of light is minimum (zero). This kind of
ooo

modification in energy distribution is called Interference. We define :


eB

-irerfer, of light is the phenomenon of redistribution of light energy in a medium on account


of superposition of light waves from two coherent sources.
ur

At the points, where the resultant intensity of light is maximum, interference is said to be constructive.
ad
YYo

At the points, where the resultant intensity of light is minimum, the interference is said to be destructive.
^0.12. YOUN< S DOOB’ H SLIT f- Tt': RlVrr;i
d
Re

Young, in 1802, demonstrated the phenomenon of interference of light by a simple experiment. The set
in

up used is shown in Fig. 10.12.


F

5 is a narrow slit (of width about 1 mm) illuminated by a monochromatic source of light, S. At a suitable
distance (about 10 cm) from 5, there are two fine slits A and B about 0.5 mm apart placed symmetrically
parallel to S. When a screen is placed at a large distance (about 2 m) from the slits A and B, alternate bright
and dark fringes running parallel to the lengths of slits appear on the screen. These are the interference
hinges. The fringes disappear when one of the slits A or B is covered.
Explanation. The appearance of bright and dark fringes on the screen can be explained on the basis of
interference of light. According to Huygens principle, the monochromatic source of light illuminating the slit
S sends out spherical wavefronts. Let the solid arcs represent the crests and the dotted arcs represent the
troughs. These wave fronts reach the slits A and B simultaneously, which in turn, become sources of secondary
wavelets. Thus the two waves of same amplitude and same frequency with zero phase difference are given out
by A and B. The full line semicircles represent crests and dotted line semi circles represent troughs. These
waves on superposidon produce interference. The dots {●) represent the positions ofconstructive inter in '.
where crest of one wave falls on crest of the other and trough falls on trough. The resultant amplitude and
hence intensity of light is maximum at these positions. The lines joining the dots lead to points C, E, G on
the screen.
WAVE OPTICS 10/11

ww
Flo
e
ree
Fr
rF
uurr
s for
Similarly, the crosses (x) represent the positions of destructive interferent: where crest of one wave
kks
falls on trough of the other and vice-versa. The resultant amplitude and hence intensity of light is minimum
Yo
oooo

at these positions. The lines joining the crosses lead to points D, F on the screen. Thus we have bright
fringes/bands/maxima* at C, E and G and dark fringes/bands/minim a* at D and .F These bright and dark
eB

fringes are placed alternately and they are equally spaced. These bands are called Interference fringes.
If S is a source of white light, interference fringes are coloured and their widths are unequal.
ur
ad

10.13. CONDITIONS FOR CONSTRUCTIVE AND DESTRl *


YYo

●: INTERFERENCE

Let the waves from two coherent sources of light be represented as


dd

yj =a sin CO t ...(13)
Re
in

^2 = /> sin (CO / + (f)) ...(14)


where a and b are the respective amplitudes of the two waves and (j) is the constant phase angle by which
F

second wave leads the first wave.

According to superposition principle, the displacement (y) of the resultant wave at time (r) would be
given by
y = yi + y2 = <3 sin CO t + /) sin (co / + (j)) = fl sin (0 / + /> sin CO / cos ^ + b cos co t sin <|)
y = sm(£it{a + b cos (|)) + cos co t.b sin ({) ...(15)
Put a + b cos (|) = /? cos 6 ...(16)
sin (|) = /? sin 0 ...(17)
y = sin CO t.R cos 0 + cos co t.R sin 0 = /? [sin co t cos 0 + cos co t sin 0]
y = /? sin (CO f + 0) ...(18)
Thus the resultant wave is a harmonic wave of amplitude R.

*Note that maxima and minima refer to the centre of a band.


10/12 “Pn^idccfi- Fundamental Physics (XII)

Squaring (16) and (17) and adding, we get


/?- (cos^ 0 + sin^ Q) = (a+b cos (]))^ + (b sin (j))“
R-x I = a^ + b^ cos^ + 2ab cos ^ + b^ sin^ ^ = + b^ (cos^ (|) + sin" + 2ab cos (f>

R = ^ja~ +b~ + 2 ab cos ^ ...(19)

As intensity is directly proportional to the square of the amplitude of the wave

I^ = Ka-,^2 = ^f’~ and Iff = KR- = K {a^b~ ●¥ 2 ab cos Iff =/j+/2+2^/( I2 cos 4»
For constructive interference
I should be maximum, for which
cos {]) - max = + 1 6 = 0^ 2 re, 4 71, i.e., (j) = 2 ...(20)

ww
where ;? = 0, 1, 2
If-V is the path difference between the two waves reaching point P, corresponding to phase difference (j),
then

Flo
X
X = = — (2nn) = nX, i
i.e., x= nX ...(21)

e
271 ' 271

eree
Hence, condition for cons/r«c/ive interference at a point is that phase difference between the two
waves reaching the point should be zero or an integral multiple of 2 it. Equivalently, path difference

FFr
between the two waves reaching the point should be zero or an integral multiple offull wavelength.

oorr
uur r
For destructive interference,
sf
1 should be minimum cos ^ = minimum = - 1
sk
...(22)
Yoo
<1) = 7t, 3 7T, 5 7T, or (t) = (2 n - 1) 71
ooko

where n = 1, 2
eBB

The corresponding path difference between the two waves :


X X
X =
(2n-l)7C = (2«-l)-~ _
uurr

2tc 27t
ad
Yo

I.e. x = (2n-])X/2 .(23)


dY

Hence, condition for destructive interference at a point is that phase difference between the two
Re

reaching the point should be an odd integral multiple of nor path difference between the two waves
innd

waves

reaching the point should be an odd integral multiple of half the wavelength.
FFi

Retain in Memory
1. From the above discussion, we find that interference is constructive, when cos (j) = 1

From (19), R =
■yja^ +b^ +2abxl
I.e. R = a + b)'^ =a + b = sum of the amplitudes of two waves, which would be maximum.

As 1
max
oc r}max I inax oc
(a + b)^ or l^^=k{a + b)^ ...(24)

Again, interference is destructive, when cos =-I


From (19), R = -^<3^ +b~ +2fl/?(~l)
WAVE OPTICS 10/13

i.e.
R- ~(a-b) = difference of the amplitudes of two waves, which would be
minimum.

R min = a-b

As /
min
.
mm mm
oc
(o - /?)2 or
...(25)
From (24) and (25), we find that

/
max _ {a + b)^
I . (a-*)2

ww
mm

When b = a
R max = (a + b) = a + a = 2a

Flo
^min = ^ ^=0

e
= 0 i.e. dark fringes will be perfectly dark and the contrast between bright and dark

ree
min.

interference fringes will be the best.

Fr
rF
We have proved that /
I^+ I2+2 .^/j I2 COS 0
uurr
R ~

2. If W] and W2 are widths of two slits from which intensities of light 7, and I2 emanate,
r
s for
kks
then
Yo
w>2 I2 b'^
oooo
eB

Sample Problem If two slits in YDSE have width ratio 4; 1, deduce the ratio of maxima and
minima in the interference pattern.
ur
ad
YYo

Sol. Here,
w,

a_l
dd

a = 2b
Re

or
b ~ 1
in
F

(a + b)^ ^(2b + b)^ 9


I
max

7
min
. (a-b)^ (2/7-6)2 “ 1

10.14. INTERFERENCL ; ND ENERGY CONSERVATION


In the interference pattern, if we take 7 max = 6 (a + 6)2 and 7^j„ = k (a - 6)2, then average intensity of
light in the interference pattern
max mm _ k (« +6)2 +6 (a -6)2 2/:(ij2+62)
7
av
2 2 2
= 6(fi(2+i,2)
If there were no interference, intensity of light from two sources at every point on the screen would be
7 = 7, + 72 = 6 (a- + tr), which is the same as 7^^, in the interference pattern.
This establishes that in the interference pattern, intensity of light is simply being redistributed i.e.
energy is being transferred from regions of destructive interference to the regions of constructive
10/14 “PruicUcfr d Fundamental Physics (XII) Eiaau
interference. No energy is being created or destroyed in the process. Thus the principle of energy
conservation is being obeyed in the process of interference of light.

Retain in Memory
1. If white light is used instead of monochromatic light, the interference fringes are coloured and are
of unequal width. The central bright fringe is white surrounded by a few coloured fringes. As \
is minimum, the fringe nearest to either side of central white fringe is violet. And the fringe
farthest on either side of central white fringe is red, as \ is maximum.

oww
2. If the whole apparatus of Young’s double slit experiment is immersed in a transparent liquid of
refractive index )i, interference pattern is maintained, but fringe width decreases to p' = p/|i.

Angular width of fringe, 0 = — = = — also decreases.


D dD d

e
re
10.15. EXPRESSION FOR FRINGE WIDTH IN INTERFERENCE

FFrllo
reF
OR THEORY OF INTERFERENCE OF LIGHT

e
Suppose A and B are two fine slits, a small distance d apart. Let
uoru
them be illuminated by a strong source of monochromatic light of

osFr
wavelength X. MN is a screen at a distance D from the slits. The two
waves starting from A and B superimpose upon eachother, resulting in
fkfor
interference pattern on the screen, placed parallel to slits A and B,
Fig. 10.13.
okso
O is centre of distance d between the slits A and B. Draw AE, BF
Y
Yo
and OC perpendicular to MN.
oo
BB

The intensity of light at a point on the screen will depend upon the
path difference between the two waves arriving at that point. The point
Y

C on the screen is at equal distance from A and B. Therefore, the path


r ree
ouu

diflerence between two waves reaching C is zero and the point C is of


maximum intensity. It is called central maximum.
ad
Ydo

Consider a point F at a distance x from C. The path difference between two waves arriving at ,P
= BP-AP ...(26)
nidn

Now, AB = EF^d, AE = BF = D
Re

In Fig. 10.11 PE = PC-EC^x-dl2


F
Fi

and PF^ PC+CF = x + d/2


-il/2
(x + d/2)^
In A BPF BP = [SF2 + PF-] = [D- + (x + d/2)^] 1/2 = £) \ +
Expanding Binomially, we get

BP=D 1+-
1 (x + d/2)^ ...(27)
2 D-
1/2
ix-d/2)'^
In A APE, AP = [AE^ + PE-] = [D- + {x- d/2f] =D I+

1 (x-d/2)^
AF=D 1 + - ...Binomially ...(28)
2
WAVE OPTICS 10/15

Putting in (26), we get, path difference


{x + dll)^ {x~dH)~ D
BP-AP=D 1+ -1- -[(x + d/2y--ix-d/2)^]
2D- 2D- 2D-

l . d xd
BP~AP = 4x— - —
2D 2 D ...(29)

w
Now the intensity at point P is maximum or minimum according as the path difference (BP - AP) is an
integral multiple of wavelength or an odd integral multiple of half wavelength.
xd
Thus for bright fringes (maxima), Path difference = — nk

e
D

re
D

o
where n = 0, I, 2, 3 X = nX —

rw
or
...(30)
d

F
Hence, bright interference fringes are formed as detailed below :

ullo
FF
XD
for/?=0, -AQ = 0/.e. atC ...central bright fringe ; for n = 1, ...1st bright fringe

ee
srr
2XD XD

oF
for n = 2, ...2nd bright fringe, and so on for « = n, x..=n ~ ...mh bright fringe
d

k
n

Similarly, for dark fringes (minima)


fofr
uor
Path difference.
xd o or a; = (2n-l)
X D
...(31)
Y
D 2 d
kos
Yo
rBB

Hence dark interference fringes are formed as detailed


oo

below :
eY

XD 3XD
for /I = 1, = ...1st dark fringe ; for/? = 2, =
re

2d 2d
...2nd dark fringe, and so on.
u

forn-n, ^n' ~ .../?th dark fringe


d
ou
o
ad

Comparing with the situation of bright interference fringes, we find that dark interference fringes are
nY

situated inbetween bright interference fringes and vice-versa.


The separation between the centres of two consecutive bright fringes is the width of a dark fringe.
nid
Re
F
Fi

XD XD _ XD
P- _ 1 =n ...(32)
d d d
Similarly, the separation between the centres of two consecutive dark fringes is the width of a bright fringe.

...(33)

XD
From (32) and (33), we find P = p' = — ...(34)
d

Hence all bright and dark fringes are of equal width.


Note that angular separation of the fringes is just (XJd). It is independent of position of the screen.

Infact, angular separation of fringes is increase in 9 needed to increase the path difference by X.
Further, at sites of constructive interference, / max (a + b)^ = constant
max
oc

Hence all bright interference bands/fringes have the same intensity.


10/16 “PnouCee^ ^ Fundamental Physics

At the sites of destructive interference, I-


inin
oc rL
min
oc
(a - h)~ - constant = 0, when b = a i.e. all dark
bands/fringes have same (zero) intensity. Fig. 10.14 shows the intensity of double slit interference pattern
FIGURE 10.14

Intensity (I)
on screen 4Iq (Two coherent
sources)

(Two incoherent
sources)

oww
Iq (Single source)
(J) (Phase difference)

e
1

re
i

FFrlo
5ji 4ti 3jt 2n K 0 K 2n 3ti 4k 5rr

rF
ee
as a function of phase difference ^ between the waves at the screen.
DO YOU KNOW ?
The horizontal solid line represents /q, the uniform intensity on the

rF
ouru
screen when one of the slits is covered up. Note that the resultant intensity When phase difference between
I varies from zero at the fringe minima to 4 /q at the fringe maxima. two sources of light changes with

fosor
If the waves from two sources (slits) were incoherent, there would
time, then the intensity of the
skf
resultant wave is
be no fringe pattern and the intensity would have the uniform value 2 Iq
ooko
for all points on the screen as shown in Fig. 10.14. /
= /^ 4-/2 + 2 cos ({)
Yo
R
Y

Below the intensity pattern in Fig. 10.14, we have shown a


If /| =/2 =/q, then
Bo

photograph of the intensity distribution in Young’s double slit


reeB

^r = ^o + ^o + 2/oCos(!)
experiment.
= 2/q(1 +cos(l>)
Note that when we use a source of white lighty containing light of
ooY
uur

different colours (i.e. different wavelengths X), then at a particular point,


ad

condition for constructive interference may be satisfied only for some


dY

particular value of X. Therefore, colour corresponding to this value of X As phase difference (j> changes,
alone shall be visible at that point. Hence the interference fringes will be changes rapidly with time.
nind
Re

coloured. However, centre of coloured fringe pattern will be white. The time average of intensity is
given by
F
Fi

XD
As X^ > X„ and P = d ' therefore, i.e. red fringe will be = 4 Iq [cos^ (TV

wider compared to the violet fringe.


= 21,0
Sample Problem Two slits 0*125 mm apart are illuminated
- 4 /() X ^
i.e., is sum of the intensities
by light of wavelength 4500 A. The screen is 1 m away, from the
plane of the slits. Find the separation between the 2nd bright fringe screen
cf two waves. It implies that the
will be illuminated
on both sides of the central maximum.
uniformly with intensity (2 Iq). In
Sol. f/= 0125 mm. = 0-125 x 10“^ m, other words, there will be no
X = 4500 A = 4500 X 10"^^ m = 4-5 x lO”'^ m, D = 1 m, n = 2 interference when phase
If X is distance of 2nd bright fringe on each side, then separation difference between the two light
between them changing with time.
D 2x2x4-5x10-^ xl
= 2x = 2nX— = m = 144 X 10^ m = 14*4 mm
d 0-125x10-^
WAVE OPTICS 10/17

10.16. CONDITIONS FOR SUSTAINED INTERFERENCE OF LIGHT


Following are some of the important conditions for obtaining sustained interference of light:
(/) The two sources of light must be coherent i.e. they should emit continuous light waves of same
wavelength or frequency, which have either the same phase or a constant phase difference.
(ii) The two sources should be strong with least background.
(Hi) The amplitudes of waves from two sources should preferably be equal.
(/V) The two sources should preferably be monochromatic,
(v) The coherent sources must be very close to eachother.
(vz) The two sources should be point sources or very narrow sources.
(vH) The distance of the screen from coherent sources of light should not be small. If this distance is
small, fringe width decreases. The fringes may not remain distinguishable.

ww
10.17. FRINGE SHIFT

If a transparent sheet of refractive index p and thickness t is introduced in one of the paths of interfering

Flo
waves, the optical path length of this path will become p t instead of/, increasing by (p - 1) /.

ee
If present position of a particular fringe is

eer
D

FFr
y = — (Ax),
d

oorr
uur r
the new position of the same fringe will be given by
s ff
y'=^[Ax + (\x-\)f]
sk
d
YYoo
ooko

D
eBB

Lateral shift of fringe


d

XD
uurr

As
ad
Yo

D_p
dY

d X
Re
innd

P
FFi

As this expression is independent of n, therefore, each fringe or the entire fringe pattern is displaced by
yg. The shifting is towards the side in which the transparent plate is introduced without any change in
fringe width as shown in Fig. 10.15.
10/18 it Fundamental Physics (XlI)CZsI9D

Curiosity Questions
I Q. 1. In a moving car, radio signals are interrupted sometimes. Why ?
Ans. While we are in a moving car, there are periodic interruptions in a radio signal that we hear. It
occurs on account of interfering radio waves. This is a common form of interference, called
multipath interference between a radio wave and its reflection.
Q. 2. Why two independent light bulbs fail to produce interference ?
Ans. The light from an ordinary electric lamp undergoes about 100 million (10®) random changes
every second. Therefore, light from two independent light bulbs, the given set of interference

w
conditions last for only about 10"® second. As human eye cannot follow such a rapid change,
therefore, no interference effects are observed from two independent light bulbs.
Q. 3. What is responsible for streaks of coloured light from a compact disc ?
Ans. The streaks of coloured tight reflected from a compact disc (CD) resemble the colours that

roow
e
appear when white light passes through a prism. However, a CD does not separate colours on

re
account of refraction. Instead, the light waves undergo interference here.
The digital information (alternating pits and smooth reflecting surfaces on the CD) forms closely
spaced rows. These rows of data do not reflect nearly as much light as the thin portions of the

reF
uFFll
disc that separate them. So light reflected from them undergoes constructive interference in

e
certain directions and destructive interference in certain other directions. The constructive
interference would depend upon wavelength of light, orientation of CD and direction of incoming
light. Thus when white light is reflected from the CD, each wavelength of light would be seen at

sFr
a particular angle w.r.t. the surface of CD. That is how we observe streaks of coloured light from

foro J
the CD.
uor
fk
okso
10.18. DIFFRACTION OF LIGHT
Y
Yo
oo
BB

Diffraction of light is the phenomenon of bending of light around corners of an obstacle or


aperture in the path of light. On account of this bending, light penetrates into the geometrical
shadow of the obstacle or aperture. The light thus deviates from its linear path as shown in
rYree

Fig. 10.16(a) and (b)


ouu
ad
Ydo
nidn
Re
F
Fi

This deviation becomes much more pronounced when the dimensions of the aperture or the obstacle
are comparable to the wavelength of light. Fig. 10.17. Out of the three cases shown, diffraction is most
pronounced when .slit width (a) is smaller than wavelength (X) i.e. a < X
Thus the nanower we make the slit, the greater is the spreading. Note that geometrical optics holds only
when slits or other apertures that might be located in the path of light do not have dimensions comparable to
or smaller than the wavelength of light.
In principle, the phenomenon of diffraction is common to all types of waves. In case of sound
waves and radio waves, diffraction is observed readily because wavelength of these waves is large, and
WAVE OPTICS 10/19

obstacles/apertures of this size are readily available. For visible light, X is very small (« lO”** m).
Therefore, diffraction of \isible light is not so common, as obstacles/apertures of this size are hardly
available.

FIGURE 10.17

Diffracted Diffracted
wave V wave
Incident Diffracted Incident Incident
wave wave wave wave

-H h- X -H K- ^ “H h" X

-■*
1
a

w
/

Flo
Screer
a > X a = X a < /.

ee
Fr
Experimental Demonstration of Diffraction
Hold two razor blades so that their sharp edges forni a

for
ur
very narrow slit inbetween, Fig. 10.18. Keep this slit right in
front of the eye and close to it. Look through the slit on the
straight filament of a clear glass electric bulb. A diffraction
ks
pattern with its bright and dark bands is seen clearly by slight
Yo
oo

adjustment of the width of the slit.


B

Types of Diffraction of light


re

The diffraction of light is of two types :


ou

1. Fresnel Diffraction is that in which the source of light or


ad

Screen
the screen or both are at finite distance from the obstacle/aperture
Y

causing the diffraction of light, as shown in Fig. 10.19(a). For


example : Diffraction at a straight edge, diffraction at a thin wire,
nd
Re

diffraction at a small opaque disc.


Aperture
Fi

2. Fraunhoffer Diffraction is that in which the source of


light and the screen are effectively at infinite distance from the
obstacle/aperture causing the diffraction of light, as shown in Fig.
I0.19(£>). Light from the source at infinity, after diffraction is
focussed on the screen using a convex lens.
For example, diffraction at a single slit, diffraction at a double
slit and diffraction at a diffraction grating.
10.19. DIFFRACTION OF LIGHT AT A SINGLE SLIT
The experimental set up used for the study of diffraction of light at a single slit is shown in Fig. 10.20.
S' is a monochromatic light source held at the focus of a collimating lens Lj. A parallel beam of light
emerging from the lens with a plane wavefront WW* is made to fall on a single slit AB. As width of slit AB =
a is of the order of wavelength of light, therefore, diffraction occurs on passing through the slit. The diffraction
pattern is focussed on to the screen XY with the help of a convex lens L-,.
10/20 ‘Pn<tdee^'4. Fundamental Physics (X1I)EZS15D

llowow
The diffraction pattern obtained on the screen consists of a centra! bright band, having alternate dark
and weak bright bands of decreasing intensity on both sides of the central bright band.
TIulur;.
The set of parallel rays falling on the slit form a plane wave front, WW'. According to Huygens principle,

ree
each point on the unblocked portion (shown dotted) of plane wave front AS sends out secondary wavelets in
all the directions.

rF
The secondary waves, from points equidistant from the centre C of the slit lying in the portion CA and
r FF
CB of wave front travel the same distance in reaching O, and hence the path difference between them is zero.
These secondary waves reinforce each other, resulting in the maximum intensity at point O.
u
forfFreo
Positions of Secondary Minima
s
Let us now consider the secondary waves travelling in a direction making an angle 6 with CO. All the
ok
YYour o
secondary waves travelling in this direction reach a point P on the screen. The intensity at P will depend on
o

the path difference between the secondary waves emitted from the corresponding points of the wave front.
Draw AN perpendicular to BK. Path difference between the secondary waves reaching P from A and B = BN
eeBoBks

= A5 sin 0 = a sin 6

If this path difference is X, (the wavelength of light used), then P will be point of minimum intensity.
r
our u

to be divided into two equal halves CA and CB and if


ad

This is because the whole wavefront can be considered


the path difference between the secondary waves from A and B is X, then the path dilference between the
Yo

secondary waves from A and C reaching P will be X/2, and path difference between the secondary waves from
B and C reaching P will again be X/2. Also, for every point in the upper half AC, there is a corresponding point
d
Re

in the lower half CB for which the path difference between the secondary waves, reaching P is X/2. Thus,
in

destructive interference takes place at P and therefore, P is a point of first secondary minimum.
FFind Y

Similarly, if path difference BN = 2X, the point P will be the position of second secondary minimum,
and so on.

In general, therefore, we have for nth secondary minimum


nX
Path diff. = a sin 0„ = nX or sin 9 ...(35)

where 0„ gives the direction of the n\l\ secondaiy minimum and n = 1,2, 3...., an integer.
Positions of Secondary Maxima
3X
If any other point P^ (not shown) is such that the path difference BN = a sin 0 = 2 '
then P^ will be the position of first secondary maximum. Here, we can imagine the unblocked wave
front to be divided into three equal parts, so that the path difference between secondary waves from
corresponding points in the 1st two parts will be X/2. They will give rise to destructive interference. The
WAVE OPTICS 10/21

secondary waves from the third pan, however, remain unused and. therefore, they reinforce each other and
produce first secondary maximum.
Similarly, if BN = 5 X/2, wc get second secondary maximum of lower intensity, and so on.
In general, wc have for nth .secondary maximum,

a sin 6,
= (2. + I)f; where n = I, 2, 3 , an integer.

Retain in Memory

oww
Note that mathematical expressions for diffraction minima and maxima are exactly reverse of
mathematicalexpressions for interferenceminima and maxima.

The diffraction pattern due to single slit consists of a cenmd bright maximum at O alongwith alternate

e
secondary minima and maxima on either side. The intensity distribution on the screen is represented in Fig. 10.21.

re
The point 0 corresponds to the position of central bright maximum and the points with path difference,

FFrlo
a sin 0 = X, 2^, are secondary minima. The secondary maxima are the points in between secondary

rF
minima and are of rapidly decreasing intensity. If intensity at O is /q, then the intensity at fmst secondary

ee
maximum is found to be 1^12 and at second secondary maximum is /q/61 and so on. The secondary maxima
ouru
and minima are shown by dotted curves in Fig. 10.21. The central maximum is shown by full line curve.

rF
ffosor
os k
ook
Yo
Y
Bo
reeB
oouY
ur
ad
dY
nidn
Re

IMPORTANT NOTE
F
Fi

That intensity of central maximum is due to wavelets from all parts of the slit exposed to light. In the
first secondary maximum (n = 1), the first two parts of the slit send wavelets in opposite phase, which
cancel out. Therefore, intensity of first secondary maximum is due to wavelets from only one third
part of the slit. Similarly, the intensity of second secondary maximum is due to wavelets only from one
fifth part of the slit as the first four parts of slit send wavelets in opposite phase. Hence intensity of
secondary maxima goes on decreasing with increase in the order of maxima (n).

Width of central maximum

The width of central tnaximum is the distance between first secondary minimum on either side of O.
If P is the position of 1st secondary minimum and OP = x, then from (1)
a sin 0 = 1 X.

sin 6 = X/a ...(36)


If/is focal length of lens L2 which is held very close to the slit, then/= D = distance of the slit from the
screen.
10/22 Fundamental Physics (XII)DSHD

If 6 is small. sin0«e = - = -
/ o
0 is obviously half ihe angular width of central maximum of diffraction pattern of single slit.
.V D\
From (36), or
D a a

Width of central maximum = 2,y = 2DX_2fX ...(37)

w
a a

This relation reveals that as the slit width (a) increases, width of central maximum decreases. This is
shown in Fig. 10.22.
Note that angular width of central maximum = 2 0. When 0 is in radian and small, sin 0 ~ 0. Therefore,

roow
e
from (36),

re
X 2X
0 = — . Therefore, angular width of central maximum, 20 = a
a

reF
uFFll
e
sFr
foro
uor
fk
okso
Y
Yo
oo
BB

Diflerence in diffraction pattern of single slit due to monochromatic light and white light.
rYree
ouu

When source of light is monochromatic, the diffraction pattern consists of alternate bright and dark
bands of unequal widths. The central bright fringe has maximum intensity. The intensity of successive secondary
ad
Ydo

maxima falls off rapidly.


When source is emitting white light, the diffraction pattern is coloured. The central maximum is white,
nidn

but other bands are coloured. As band width « X, therefore, red band with higher wavelength is wider than the
Re

violet band with smaller wavelength.


F
Fi

Sample Problem D Red light of wavelength 6500 A from a distant source falls on a slit
0*50 mm wide. Calculate the distance between first two dark bands on each side of central bright band
in the diffraction pattern observed on a screen placed 1*8 m from the slit
Sol. Here, X = 6500 A = 6-5 x lO"”^ m, a = 0-50 mm = 5 x 10^ m, D = 1-8 m
Distance between first two dark bands on each side of central maximum is the width of central maximum

2XD 2x6-5x10-'^ X18


i.e., 2x = = 4*68 X 10-3 ^
a 5x10“^
Sample Problem Determine the angular spread between central maximum and first order
maximum of the diffraction pattern due to a single slit of width 0*25 mm, when light of wavelength
5890 A is incident on it normally ?
Sol. Half the angular spread 6 between central maximum and first order maximum is given by

asin 0 = (2« +1)^ = (2x1 +1) — = —


WAVE OPTICS 10/23

3^ 3X 3x5890x10"’^
When 6 is small, sin 0 = 9 a6 = = 3-534 X 10-'^ rad.
2a 2 X 0-25 X10“^
tola! angular spread = ± 3*534 x 10 rad

10.20. DIFFERENCE BETWEEN INTERFERENCE AND DIFFRACTION OF LIGHT

Following are the important points of difference between interference and diffraction of light.
1. Interference is due to superposition of two distinct waves coming from two coherent sources. Diffraction
is produced as a result of superposition of the secondary wavelets coming from different (unblocked) parts of
the same wavefront.

2. In interference pattern, all the bright fringes (or bands) are of same intensity. In diffraction pattern, all
the bright bands are not of the same intensity.

ww
3. In interference pattern, intensity of minima is generally zero or very small and there is a good contrast
between bright and dark fringes. In diffraction pattern, the intensity at minima is never zero and there is poor
contrast between bright and dark bands.
4. The width of the interference fringes may or may not be equal. But the width of diffraction bands is

Flo
always unequal.

ee
5. In interference, fringes or bands are usually large in number. In diffraction, the fringes or bands are

rere
only a few in number.

rFF
6. In diffraction pattern, width of central maximum is double the width of secondary maxima. It is not so
in interference.
uurr
10.21. FRESNEL DISTANCE
foor
ks s
From the theory of diffraction of light at a single slit, the angle of diffraction (bending) for central
Yoo
X
oook

maximum is 0«sin 0 =
a
eBB

(ZX\
In travellinga distance Z, the half angularwidth of beam would become , due to diffractionof light.
a
2
uurr

zx a
This beam width will become more than width of the slit (a) i.e. >a only when Z > —
ad

a X
Yo

2
a
We call the Fresnel distance.
dY

X
Re
innd

Fresnel distance is the minimum distance a beam of light can travel before its deviation from
FFi

straight line path due to diffraction becomes significant/notice able. Fresnel distance varies
directlyas the square of size of slit and inversely as wavelength of light.

Thus for distances smaller than Fresnel distance (Z^), spreading due to diffraction of light is smaller as
compared to the size of the beam. The spreading becomes comparable to the size of the beam when the
distance travelled is equal to Fresnel distance (Z^). For distances much greater than Zp, Sj. reading of light
beam due to diffraction dominates over that due toray optics.
Now, the average wavelength of visible light is X = 6000 A = 6 x 10“^ m, and diameter of pupil of
2 (10-^ 10
human eye, a = 1 mm - 10'^ m. = — = lo7 m
^ X 6x10-^ 6
This means width of the beam due to diffraction does not become more than 1 mm, unless distance
travelled by the beam is more than 1-67 m. Hence we can ignore broadening of beam by diffraction upto
distances as large as a few metres, i.e. we can assume that light travels along straight lines. Hence ray optics
can be taken as a limiting case of wave optics. This is primarily because wavelength of light is small.
10/24 Fundamental Physics (X1I)B2SMD

If X, ^ 0, Z/r —> oo. It implies that ray optics will be valid for all distances.
That is why, while prescribing eye glasses for patients, the optometrists never take into consideration
the wave nature of light. They predict the path of light through the glasses to the retina of the eye only on the
basis of ray optics, assuming that light travels along straight lines.
Sample Problem Light of wavelength 5000 A Is diffracted by an aperture of width 2 mm.
For what distance travelled by the diffracted beam does the spreading due to diffraction become greater
than the width of the aperture ?
Sol. Here, X = 5000 A = 5 x 10"^ m, a = 2 mm = 2 X 10 ^ m ; Z/.- = ?
(2x10-^ f 40
— = 8ni
As Fresnel distance, = —
^ X SxlO""^ 5

ww
10.22. CONCEPT OF RESOLVING POWER

According to ray optics, the image of a point object formed by an ideal


lens is a point only. However, because of the diffraction effects, the image of

Floo
a single point object formed by a spherical lens will actually be a bright
central circular region known as Airy Disc surrounded by alternate dark and

e
bright concentric rings, as shown in Fig. 10.23.

eere
When two point objects are close to each other, their images i.e.
diffraction patterns as seen through optical instruments, will also be close

FFr
and overlap each other. If the overlapping is small, both the point objects are

oorr
uur r
seen separate in the optical instrument, i.e., the optical instrument is able to
resolve the two point objects. However, if the overlapping is large, the two point objects will not be seen as
s ff
separate, i.e., the optical instrument is not able to resolve them. Hence
sk
YYoo
Resolving power of an optical instrument is the power or ability of the instrument to produce
ooko

distinctly separate images of two closely spaced objects i.e. it is the ability of the instrument to
resolve or to see as separate, the images of two closely spaced objects.
eBB

When two objects are placed very close to each other, every optical instrument has its own limit upto
uurr

which it can produce distinctly separate images of these objects.


ad

According to Rayleigh, two point objects A and B will he just resolved, when central maximum of
Yo

diffraction pattern of B lies on first secondary minimum of diffraction pattern of A. This is called
Rayleigh's criterion for resoivability and is shown in Fig. 10.24 and 10.25.
dY
Re
nind
FFi

The minimum distance between two objects which canjust be seen as separate by the optical instrument
is called the Limit of Resolution ofthe instrument. Obviously, smaller the limit of resolution of the optical
instrument, greater is its Resolving Power and vice-versa.
WAVE OPTICS 10/25

10.23. RESOLVING POWER OF MICROSCOPE

The resolving power of a microscope is the ability of the microscope to show as separate, the images of
two point objects lying close to each other.
The limit of resolution of the microscope is measured by the minimum distance (d) between two
point objects, whose images in the microscope are just seen as separate.
The value of d varies (i) directly as the wavelength (X) of light used and
X
(ii) inversely as the cone angle of light rays from any one object entering the microscope, i.e., d oc

20
where 26 is the angle of the cone of light rays entering the objective of the microscope, as shown in Fig.
10.26. i.e., limit of resolution of microscope (replacing 0 by sin 6)
X
d =
...(38)

ww
2 sin 6

If medium between the object and objective lens of microscope is a transparent


medium of refractive index \i. instead of air, then

Flo
X
(38) can be rewritten as d = ...(39)

e
2p.sin0

eree
Obviously, smaller the value of d, greater will be the resolving power.

FFr
RP of microscope = — = ^ ...(40)

oorr
uur r
d X sf
sin 0 is called numerical aperture of the microscope.
sk
To increase resolving power of microscope, we decrease X, i.e., often use ultraviolet light of smaller
Yoo
ooko

wavelength. We cannot increase 6 because in that case aperture of the lens would increase. Thus UV light
permits finer details to be examined than would be possible for the same microscope using visible light.
eBB

10.24. RESOLVING POWER OF TELESCOPE


uurr

The resolving power of a telescope is its ability to show distinctly the images of two distant objects, say
ad

stars having small angular separation.


Yo

The limit of resolution of telescope is measured by the angle (dQ) subtended at its objective, by those two
distant objects {say, stars) whose images through the telescope are just seen as separate, Fig. 10.27.
dY
Re
innd
FFi
10/26 “P^eidec^’A Fundamental Physics (XII)EEHD

The value of (c/0) varies (/) directly as the wavelength (A.) of light used, and (/V) inversely as the aperture
i.e. diameter D of the objective of the telescope.
1-22
I.e., c/0 OC or dQ = ...(41)
D D

Obviously, smaller the value of c/6, greater will be resolving power of telescope, and vice-versa.

1 D
R.P. of telescope = ...(42)
c/0 122X

A telescope is used to observe distant objects, which are generally seen in sunlight. Therefore, there is

ooww
no control on X. Hence to increase the resolving power of telescope, we must use the objective lens of large
aperture (D). An additional advantage of large objective is that it will collect greater amounts of light and
images seen will be much brighter. The stars, which are extremely far away can also be seen using telescopes
of larger aperture.
Resolving Power of human eye

e
As eye lens is a converging lens, therefore limit of resolution of human eye is same as that of the

ere
rFl
objective lens of a telescope.

Fre
1-22X
0 = where D is diameter of pupil of eye, and X is wavelength of light used.

rrF
D

sffoo
ouur
The limit of resolution of human eye is one minute or — degree. It implies that if mo distant objects
60
kosk
subtend on the eye, an angle which is greater than or atleast equal to one minute, their images will be seen
as separate by the eye.
Yo
oo

What i.s the angular resolution of a 10 cm diameter telescope at a wavelength


Y
Sample Problem
BB

of 0-6 p. m ?
Sol. Here, c/0 = ?D=lOcm = Ol m, X = 0-6 pm = 6x 10 ^m.
rre

\-22X l-22x6xl0-'^
ouu

c/6 = c/0 = = 7-32 X 10-® radian


Y

As
ad

D 01
dY

Retain in Memory
innd

1. Resolving power of a microscope depends upon wavelength of light and the refractive index of
Re

the medium between the object and objective lens of the microscope.
Fi
F

2. Resolving power of a telescope increases with the increase in the diameter of objective lens and
also with the decrease in wavelength of incident light.

10.25. DOPPLER'S EFFECT IN LIGHT

According toDoppler’s effect, whenever there is a relative motion between a source of light and
observer, the apparentfrequency oflight received by observer is differentfrom the true frequency
of light emitted actually from the source of light.

The apparent frequency of light increases when the distance between source of light and observer is
decreasing and the apparent frequency of light decreases if the distance between source of light and observer
is increasing.
Expression for apparent frequency of light
Suppose a source of light emits waves of frequency v and wavelength X. If c is velocity of light in

vacuum, then X=^ ...(43)


v
WAVE OPTICS 10/27

Let the source and observer approach eachother with a velocity v along the direction of propagation of
light. In one second, the two come closer by a distance v.
Apparent frequency = number of light waves received per second by the observer i.e.
v' = number of light waves emittcd/scc by source plus number of light waves contained in a distance v
V V V
v' = v+ — = v + = V + V.—
c/v c

v' = v 1 +-
...(44)
cj
Clearly, v' > v '
Similarly, when the source of light and observer are moving away from eachother with a velocity u, then
the apparent frequency is obtained by replacing i> by - f in Eqn. (44)

w
V
v' = V 1 — ...(45)
c J

Flo
Clearly, v' < v
V

ee
We may combine eqns. (44) and (45) and rewrite v' = v 1+ - ...(46)
cJ

Fr
+ sign when source and observer are approaching eachother, and - sign when source and observer are
receding away from eachother.
for
ur
Note that a correct treatment of Doppler’s principle needs the application of special theory of relativity.
We can show that eqn. (46) is modified to
ks
\±v/c
Yo
v‘
oo

...(47)
V
^h~v^ fc^
eB

This is what is called Astronomical Doppler’s effect.


Calculation of velocity of Star
r
ou
ad

t;v
From (46), we find that v'-v;=± —
YY

V Av
Av= ± —V or v=± ...(48)
nd
Re

c V

c
Fi

From (43). i’= —


X

civ c
Differentiating w.r.t. X, we gel or dX = - dv
dX c

X2 X2 V V
I.e., AX = - X Av ±-v = + -X ...(using 6* = V X)
c C \ C J c

Hence, in terms of wavelength, we can rewrite (48) as

V
AX
AX = + —X or c
...(49)
c

When source and observer approach eachother^ Av is +, i. e., apparent frequency increases or apparent
wavelength decreases. This is called Blue shift.
When source and observer recede away from eachother, Av is negative i.e. apparent frequency
decreases or apparent wavelength increases. This is called Red shift. {●: X^>X^)
10/28 Fundamental Physics (XII)EEHD

By measuring blue shift/red shift, we can calculate velocity of star from eqn. (49).
IMPORTANT NOTE

On the same basis, we can calculate velocity of a rocket/submari ne from the blue/red shift. As the
rocket/submarine is in motion, cqns. (14) and (15) are modified as

^ 1 Av 1 AX
V - ± c - T c
2 V 2 X

Retain in Memory
The speed of star determined from measuring Doppler shift is only the radial speed of the star,
i.e., only the radial component of the star’s velocity relative to us.

ww
Ssmple Problem The spectral line for a given element in the light received from a distant star
is shifted towards longer wavelength side by 0-025%. Calculate the velocity of star in the line of sight.
AX 0-025

Flo
Sol. Here. — = 0-025% = = 2-5 X 10-^
X 100

e
V AX AX

rere
As V xc = 2-5 X 10"^ X 3 X 10^ = 7-5 x lO'* ms -1
X X

r FF
c

As shift is towards longer wavelength side, the star is moving away.


uurr
10.26. DOPPLER'S EFFECT IN LIGHT IS SYMMETRICAL
BLIT THE SAME EFFECT IN SOUND IS ASYMMETRICAL foor
kss
Doppler’s effect in sound is asymmetrical because a source of sound moving through a material medium
Yoo
in which listener is at rest is physically a different situation from that of a listener moving through that
ooook

medium, when the source is at rest.


eBB

In case of light waves it is not possible to identify a medium relative to which the source of light or
observer is moving. Therefore, ‘source moving towards an observer’ and ‘observer moving towards a source’
are physically identical situations. That is why Doppler’s effect in light is symmetrical.
uurr

This statementcan also be establishedmathematically.


ad
Yo

10.27. APPLICATIONS OF DOPPLER'S EFFECT IN LIGHT


dY

Some important applications of Doppler’s effect in light are in


(/) measuring the speed of a star and speed of galaxies,
Re
innd

(ii) measuring speed of rotation of the sun, about its own axis which is about 2 km/s,
{Hi) measurement of plasma temperatures in thermo-nuclear reactions,
Fi

(/v) Doppler’seffect is used in RADAR (Radio detectionand Ranging) to estimate the speed of aeroplanes,
rockets, etc.
(v) Doppler’s effect is used in SONAR (sound navigation and ranging) to estimate the velocity of
submarines, ships, etc. and in estimating the depth of sea.
(v/) Doppler’s effect confirms the hypothesis that universe is expanding.
10.28. POLARIZATION OF LIGHT

(fl) Unpolarised Light


According to Maxwell, light is an electromagnetic wave in which electric and magnetic field vectors vary
sinu.soidally, perpendicular to each other as well as perpendicular to the direction of propagation of wave of light.
In common light sources, e.g.. sun or a bulb, atoms are the elementary radiators of light. As these atoms
act independently, light propagated from such sources in a given direction consists of many independent
waves whose planes of vibration are randomly oriented about the direction of propagation as shown in Fig.
10. 28(a). Such light waves are said to be impolarized. or polarized randomly.
Light is generally represented by electric field vector £ , called Light Vector.
WAVE OPTICS 10/29

In principle, each electric field of


Fig. 10.28(a) can be resolved into y and z
components. We can then find the net
electric field along the v-axis and along
the ^:-axis separately as shown in Fig.
10.28(6). Thus unpolarized light can be
thought of as the superposition of two
polarized waves whose planes of vibration
are perpendicular to each other. The
symbol of unpolarised light is shown in
Fig. 10.29. The arrow head represents
vibrations in the plane of the paper and
dot represents vibrations in a direction

ww
perpendicular to the plane of the paper.
FIGURE 10.30
FIGURE10.29

Flo
A Plane of Vibration
Unpolarised -1 D

e
light . K Plane polar zed light

eree
■►X A A i ' 7n
» T

FFr

Plane of
I Polarization

oorr
Arrows : Vibs in the plane of paper
uur r
I
Dots : Vibs perpendicular to plane of paper :c
B
sf
When unpolarized light is passed through a tourmaline crystal cut with its face parallel to its
sk
Yoo
crystallographic axis AB, Fig. 10.30, only those vibrations of light pass through the crystal, which are parallel
ooko

to AB. All other vibrations are absorbed. That is why intensity of light emerging from the crystal is reduced.
eBB

The emergent light from the crystal which contains vibrations only in one direction is said to be plane
polarized light.
uurr

This phenomenon of restricting the vibrations of light DO YOU KNOW ?


ad

(electric vector) in a particular direction, perpendicular to


Yo

Very high frequency {VHF)


the direction of wave motion is called polarization of light. television antennas in U.K. are
dY

The tourmaline crystal acts as a polarizer. oriented vertically, but those in


Re

U.S.A. are oriented horizontally.


innd

Thus electromagnetic waves are said to be polarized when their


The difference is due to the
electric field vectors are all in a single plane, called the plane of
FFi

direction of oscillation of the


oscillation/ vibration. Light waves from common sources are
unpolarized or randomly polarized. electromagnetic waves carrying the
TV. signals. In U.K., these waves
The plane ABCD in which the vibrations of polarized light are
are polarized vertically i.e. their
conifned is called the plane of vibration/oscillation. The plane KLMN
electric field oscillates vertically.
which is perpendicular to the plane of vibration is definedas the plane
Therefore, for the electric field of
of polarization, Fig. 10.30.
the incident television waves to
Retain in Memory drive a current along the antenna
When originally unpolarized light is passed through a and provide signal to a televi.sion
set, the antenna inu.st be vertical.
polarizing sheet, the transmitted intensity is ideally half
In U.S.A., these waves are
the original intensity. In actual practice, the transmitted
intensity may even be somewhat less than half the original polarized horizontally.
intensity. For proof, see Important Note, pp. 10/32.
10/30 it Fundamental Physics fXin rmn
10.29. EXPERIMEIVfTAL DEMONSTRATION OF POLARIZATION OF
LIGHT AND TRANSVERSE CHARACTER OF LIGHT WAVES

The phenomenon of polarization oflight can be demonstrated experimentally by a simple arrangement,


shown in Fig. 10.31. Here and T2 are two thin plates of tourmaline, cut with their faces parallel to the axis
of the crystal. A fine pencil of ordinary light from the source 5 is passed through the plate Tj ; and the light
transmitted is observed by the naked eye.
FIGURE 10.31
^2 Ti T2
A A A
A A

No Light

w ^
V V
w 7 7

ww
Polarizer Polarizer Analyser
Analyser
e o
When the plate T, is rotated about the direction of propagation of light as axis, the intensity and character

Flo
of transmitted light remain the same (except for slight change in colour due to selective absorption within

ee
the crystal). Let the second plate T2 be placed in the path of light transmitted from Tj. We observe that

rere
intensity and character of light transmitted by Tj remain unaffected only when Tj and T2 are set with

r FF
their axes parallel, Fig. 10.31 (a). When T2 is rotated gradually, intensity of light transmitted from T2 goes
on decreasing. As soon as the axes of the two crystals are at 90° to each other, light is completely cut off, Fig.
10. 31(Z?). On rotating T2 further, light reappears. Intensity oflight transmitted from T2 starts increasing, till
uurr
it is maximum when axes of the two crystals are parallel again. foor
If both. T] and T2 are rotated with the same angular velocity in the same direction, no change in intensity
ks s
of transmitted light is observed.
Yoo
ooook

The phenomenon can be explained only when we assume that light waves are transverse*. Now the
unpolarized light falling on Tj has transverse vibrations of electric vector lying in all possible directions. The
eBB

crystal Tj allows only those vibrations to pass through it, which are parallel to its axis. When the crystal T2 is,
introduced with its axis kept parallel to the axis of Tj, the vibrations of electric vector transmitted by Tj are
also transmitted through T2. However, when axis of T2 is perpendicular to axis of T^, vibrations of electric
uurr

vector transmitted from Tj are normal to the axis of T2. Therefore, T2 does not allow them to pass and hence
ad
Yo

eye receives no light.


When both the polarizer and analyzer are rotated with the same angular velocity in the same direction,
dY

there would be no change in intensity of transmitted light.


Re

Light coming out of the crystal Tj is said to be polarized, i.e., it has vibrations of electric vector which
innd

are restricted only in one direction (i.e., parallel to the optic axis of crystal Tj).
FFi

Since the intensity of polarized light on passing through a tourmaline crystal changes with the relative
orientation of its crystallographic axes with that of pola-
riser, therefore, light must consist of transverse waves.
10.30. NICOL PRISM

A nicolprism is an optical device, which is


used for producing plane polarized light
and analysing the same.

Fig. 10.32 shows the principal section ACGE of


the calcite crystal. The nicol prism consists of two
calcite crystals cut at» 68° with its principal axis joined
by a glue called Canada balsam. The refractive index
of this glue is I -55.
*This is why Fresnel had to modify Huygens theory which assumed light waves to be longitudinal.
WAVE OPTICS 10/31

When a beam of unpolarised light is passed through a calcite crystal, it breaks up into two parts :
(/) Ordinary ray (0-ray), which has its electrical vector perpendicular to the principal section of the crystal.
(//) Extraordinary ray (£-ray), which has its electric vector parallel to the piincipai section of the crystal.
As refractive index of calcite for 0-ray is 1 -658, and for £-ray is 1486, therefore, Canada balsam (with
p = I -55) acts as a rarer medium for O-ray and it acts as a denser medium for £-ray. Therefore, when 0-ray
passes from a portion of crystal into the layer of Canada balsam, it passes from a denser to a rarer medium.
When angle of incidence is greater than the critical angle (/^ = 69°), the 0-ray is totally internally reflected
and is not transmitted. The £-ray is not aifected and is transmitted as such through the nicol prism.
This is how unpolarized light passed through a nicol prism becomes plane polarized.
10.31. DETECTION OF POLARIZED LIGHT

A naked eye cannot distinguish between polarized and unpolarized light. A crystal can be used for

ww
making this distinction. A calcite crystal, quartz crystal, a nicol prism (made from calcite crystal) can be used
as polarizer as well as analyser of polarized light.
When unpolarized light is seen through a single crystal (polaroid) intensity of transmitted light decreases,

FF loo
on account of polarization. On rotating the crystal, intensity of polarized light does not change.
However, when light transmitted from polaroid P| is .seen through another polaroid P2 and is rotated,

ree
the transmitted fraction of light from P2 falls from maximum to zero as the angle between £, and P^ varies
from 0“ to 90° respectively. Here, £| is called polarizer and P2 is called analyser.
Ideiitiflcation of given light beam

reFe
For this, we pass the given light through a polaroid (called analy.ser). rotate the polaroid about the

oroFr
r ur
incident light as axis and examine the emergent light.
{/) if there is no change in intensity of emergent light, incident light is unpolarized.
s ff
(/V) If there is change in intensity of emergent light with minimum not equal to zero, the incident light is
partially polarized.
k
YYouo
okso

(///■) If intensity of emergent light changes with minimum equal to zero, the incident light is plane polarized
or linearly polarized.
BBoo

Retain in Memory
r ee

When angles between the principal sections of two nicols are 0° and 180°, they are referred to as
ad
ouur

Parallel nicols. When this angle is 90°, they are said to form Crossed nicols.
Yo

10.32. LAW OF MALUS

This law was discovered by a French engineer Lousis Malus.


d
Re
idnY

According to law of Mains, when a beam of completely plane polarized


FFin

light is incident on an analyser, the resultant intensity of light (1)


transmitted from the analyser varies directly as the square of the
cosine of the angle (0) between plane of transmission of analyser and
polarizer.
i.e.. I co.s^ 0 ...(50)

This rule is also called cosine squared rule, and we can use it only when the
light reaching a polarizing sheet is already polarized. The transmitted intensity / is
a maximum and equal to the original intensity /q when the original wave is polarized
parallel to the polarizing direction of the sheet {i.e. 0 = 0° or 180°). I is zero, when
the original wave is polarized perpendicular to the polarizing direction of the
sheet {i.e., when 0 = 90°).
To prove this law, suppose is amplitude of electric vector transmitted by the polarizer, along OP. The
plane ol analyser OA makes an angle 0 with O.P Fig. 10.33. The two rectangular components of ^ are
10/32 ‘PfuuCeef.a'^ Fundamental Physics (XIl)EZslSD

are {a cos 6), parallel to plane of transmission of analyser and {a sin 0), DO YOU KNOW ?
perpendicular to plane of transmission of analyser.
Light passes through a
As the component transmitted through analayser is only a cos 0
therefore, intensity of light transmitted from analyser is polarizing sheet only when
electric field of the light wave
I = k{a cos 0)- = k cos^ 0 has a component parallel to the
/ = /,) cos^ 0 transmission axis of the

where Iq = ka^ = intensity of light transmitted from polarizer material.

= intensity of light incident an analyser


= constant

Hence / cos- 0 ,
which proves the law of Malus.

ww
When polarizer and analyser are parallel, 0 = 0° or 180°
so that cos 0 = ± I, / = ,/0

Floo
When polarizer and analyser are perpendicular to each
other, 0 = 90° ; cos 6 - cos 90° = 0

e
/=0

eere
FIGURE 10.34

The variation of intensity of polarised light through the


analyser with angle 0 between the polariser and analyser is shown

FFr
in Fig. 10.34

oorr
uur r
Important Note. We can prove that when unpolarized
s ff
light of intensity Iq gets polarized on passing through a
sk
YYoo
Polaroid, its intensity becomes halfie. I - —1^.
ooko

In unpolarized light, vibrations are probable in all the directions in a plane perpendicular to the direction
eBB

of propagation. Therefore, 0 can have any value from 0 to 2 7t. Therefore


uurr

2n 2n
n2n
1 1 r (1 +cos 20) i/0 1 sin 20
[cos^ 0] cos^QdQ =
ad

0 +
2 2 Jo
ai'
2k 2n
Yo

27tx2
0 0
dY

I
Re

(cos- 0) av
nind

Using Law of Malus, I = Iq cos^ 0, we write


FFi

1 1
/ = Lx- = -I
pol. 0 2 2 0'

Sample Problem Two polarising sheets are placed with their planes parallel, so that light
intensity transmitted is max. Through what angle must either sheet be turned so that light intensity
drops to half the maximum value ?
Sol. According to law of Malus, I = Iq cos^ 0
! 1
cos^ 0 = — cos 0 = ±
I
0
2'

0 = ± 45° or ± 135°

The effect will be same, when any of the two sheets is turned through 0 in any direction.
WAVE OPTICS 10/33

10.33. POLARIZATION BY SCATTERING

When a beam of white light is passed through a medium containing particles whose size is of the order
of wavelength of light, then the beam gets scattered. When the scattered light is seen in a direction perpendiculaj'
to the direction of incidence, it is found to be plane polarized (as detected by the analyser). The phenomenon
is called polarization by scattering.
FIGURE 10.35
In Fig. 10.35, a beam of unpolarised light is incident Z

along Z axis on a scatterer at O. As light waves are transverse


in character, all the possible directions of vibration of electric
vector in unpolarised light are confined to XY plane.
When we look along X-axis, then because of transverse Scatterer
0
nature of light, we can see only the vibrations of electric

w
vector which are parallel to K-axis. Similarly, when we look zr~7 /
along K-axis, the vibrations of electric vector seen are _L to X

Flo
K-axis or parallel to X-axis. Hence lighi scattered in a
direction perpendicular to the incident light is always plane
polarized.

ee
Fr
10.34. POLARIZATION OF LIGHT BY REFLECTION
When unpolarized light is reflected from a surface, the reflected light may be completely polarised,
for
ur
partially polarized or unpoiarized. This would depend on the angle of incidence.
If angle of incidence is 0“ or 90“, the reflected beam remains unpolarized. For angles of incidence
ks
between 0° and 90®, the reflected beam is polarized to varying degree.
Yo
oo

The angle of incidence at which the reflected light is completely plane polarized is called
polarizing angle or Brewster’s angle.
eB

It is represented by The value ofi^, depends on the wavelength of light u.sed apart from the material
r

of the reflecting sutface. Therefore, complete polarization is possible only for monochromatic light. In
ou
ad

Fig. 10.36, unpolarized light is incident along AO at Z/^ on the interface XK separating air from a medium of
refractive index )i. The light reflected along OB is completely plane polarised.
Y

FIGURE 10.36
nd
Re

DO YOU KNOW ?
Fi

You can change the glare you see


in sunlight that has been reflected
from, say. water, by looking
through a polarizing sheet (e.g. a
sun glass lens) and rotating the
sheet’s polarizing axis around the
line of sight. This is possible
because reflected light is fully or
partially polarized by reflection
from a surface.

Infact, the unpoiarized light has two electric field components, one perpendicular to the plane of incidence
(represented by dots) and the other in the plane of incidence (represented by arrows).
The vibrations of electric vector perpendicular to the plane of incidence remain always parallel to the
reflecting surface, whatever be the angle of incidence. Therefore, condition of their reflection is not changed
10/34 “Pnittiee^ '4. Fundamental Physics (XII) IkTOWlI
with the change in the angle of incidence. However, the other set of oscillations of electric vector in the plane
of incidence make different angles with the reflecting surface, as angle of incidence of unpolarized light is
changed. At the polarizing angle (/^), most of these vibrations of electric vector get transmitted and are not
reflected, Hence light refracted along OC is a mixture of polarized light and unpolarized light, i.e., it is
partially polarized.
The reflected light therefore, contains vibrations of electric vector perpendicular to the plane of
incidence. Hence the reflected light is completely plane polarized in a direction perpendicular to the plane
of incidence.

10.35. BREWSTER'S LAW

According to this law, when unpolarized light is incident at polarizing angle, ip on an interface

ww
separating air from a medium of refractive index p, then the reflected light is fully polarized
(perpendicular to the plane of incidence), provided
...(51)
^ = tan ip

FF loo
This relation represents Brewster’s Law.

ree
Note that the parallel components of incident light do not disappear, but refract into the medium, with
the perpendicular components.

reFe
It has been observed experimentally that when light is incident at polarizing angle, the reflected

oroFr
rur
component along OB and refracted component along OC arc mutually perpendicular to eachother. Thus, in
Fig. 10.36. ZBOY+ZYOC = 90°
s ff
...(52)
(90" - ip) + (90" - /●) = 90", where r is the angle of refraction, or
k
YYouo
sin /
okso

According to Snell’s law, M = —


sin r
BBoo

sin i
r ee

— = tan /
When
i = ip, r = (90^-ip) .-. ^ sin (90°- cos I
p
p
ad
ouur

This proves Brewster’s law.


Yo

Using Brewster’s law, we can calculate polarizing angle for any two media in contact. For example
a
(/) For air-glass interface = 3/2
d
idnY
Re

3
ip = tan ’ (1-5) = 56-3°
_ a
tan i II =- = 1-5
P 2
FFin

Thus, light reflected in air from a plane surface of glass will be completely polarized, when angle of
incidence of unpolarized light on surface of glass is 56-3".

-3 = 1-3333 /p = tan-’(I-3333)" = 53-1


a
(//) For air-water interface ll„, = 4/3 tan / <’

Thus light retlected in air from surface of water will be completely polarised, when angle of incidence
of unpolarized light on the surface of water is 53-1".
We can prove that in case of reflection at Brewster angle, reflected and refracted rays are mutually
perpendicular.
sin t
p
According to Brewster’s law. jl = tan = ...(53)
cost
/>

sin i
sin i P
According to Snell’s law, M = - ...(54)
sm /● sin /●
WAVE OPTICS 10/35

From (53) and (54), sin r = cos = sin (90" - ip) r. /●


= W-ip or
/^ + r=90-
In Fig. 10.36, r ~ i r' + r = 90°
p

[90" - ZBOY\ + (90" - ZCOY) = 90‘ .-. ZBOY+ ZCOY =90° or ZBOC = 90°

i.e., angle between reflected rays (along OB) and refracted rays (along OC) is 90".
Sample Problem Light reflected from the surface of glass plate of refractive index 1*57 is
linearly polarised. Calculate the angle of refraction in glass.
Sol. Here, |i=l-57, r = ?
According to Brewster's law, tan /^ = p = I -57 ip = tan ' 1-57 = 57-5°

As /● = 90" - i r = 90"-57-5" = 32-5'


p

10.36. POLAROIDS

ww
A Polaroid is a material which polarises light. Tourmaline is a DO YO^J KNOW ?
natural polarising material. Polaroids are now artificially made. It was
Apart from plane polarized (i.e.
discovered that small needle shaped crystals of quinine idosulphate have
linearly polarized) and partially

Floo
the property of polarising the light. A number of these crystals with their
polarized ; light can also be
axes parallel to one another are packed inbetween two sheets of plastic. circularly or elliptically polarized

e
Such a sheet serves as the polaroid.

eere
(and that too left handed and irght
Uses of plane polarized light and polaroids handed).

FFr
1. One of the major uses of polaroids is to avoid glare of light. Elliptically polarized and circu
Most of the light reflected from glazed surfaces is partially plane polarized larly polarized lights are obtained

oorr
uur r
with vibrations in the horizontal plane. When we use polarized sun glasses when two plane polarized lights
s ff
with their vibration planes-vertical, the most of the polarized light reflected 71
differing in phase by — overlap.
from glazed surfaces is cut off. 2
sk
YYoo
2. To avoid the dazzling light of a car approaching from the opposite The light is circularly polarized
ooko

when the two plane polarized


side during night driving, polaroids are fined on the wind shield and on
lights have equal amplitudes.
eBB

the cover glasses of head lights of each ca.r The arrangement is so made
And the light is elliptically
that polaroids on the wind shield of one car and those on cover glasses of
polarized when two plane
head lights of other car become ‘crossed’. Therefore,the glare is avoided.
uurr

polarized lights have unequal


However, each driver can see the other car from the head lights of his
ad

amplitudes.
Yo

own car.

3. The objectives of microscopes are fitted with polaroids to avoid glare in observing very minute particles.
dY

4. Clear photographs of white clouds are obtained by fitting polaroids in front of the camera lens.
Re

Scattered light present in the atmosphere,which is partially polarised is cut off by the polaroid. Only the light
nind

reflected from the clouds, being unpolarised, enters the camera and we obtain the distinct picture of clouds.
FFi

5. Polaroids are useful in three dimensional motion pictures, i.e., in holography.


6. Polaroids are used to improve colour contrast in old oil paintings.
7. Polaroids are also used in optical .stress analysis.
8. In calculators, watches, T.V., computers, etc. letters and numbers are formed by liquid crystal display
(LCD) through polarization of light.
9. In CD players, polarized laser beam acts as needle for producing sound from compact disc, which is
in encoded digital formal.
10. Polarization of scattered sunlight is used for navigation in .solar compass in polar regions, where
magnetic compass becomes inoperative.
11. By determining the polarizing angle (ip) and using Brewster's law, refractive index of dark tramsparent
substances can be determined.

12. Polarization is also used to study asymmetries in crystals and molecules using the phenomenon of
optical activity.
10/36 Fundamental Physics (XIDESSU

w
TYPE I. HUYGENS PRINCIPLE,
REFLECTION AND REFRACTION OF LIGHT
V
H c 3x10^
X
^ V li h .V l-SxSxlO'*^

r
Formulae used

1. Speed of light in vacuum, c = v X = 3 x 10* m/s = 40x 10-"^ m

2. Refractive index of a medium


= (4-445 X lO-"^ - 4-0 x 10'^) m

r
c vel.of lightin vacuum vX _ X = 0-445 X lO"^ m

llowu
o
^=-
V vel.of light in medium = 445 A

F
1

Wavelength in medium, X' = — (freq. v is same) TYPE II. INTENSITIES


OF MAXIMA AND MINIMA
3. When light travels from medium 1 to 2, then

s
FF
smf Formulae used

o
according to SnelTs law. =~

rek
er
sm r 2
w
1 _ a /,
1.
4. Optical path in vacuum = |i x path in medium.
W2 ^2

Fo
Units used

ro
2. When sources are coherent, interference occurs.
r, V in m/s ; X. X' in metre ; v in Hz ; p has no units.

of
I ia+h)-
kY
o
max _
Y
pte D The absolute refractive index
ur
Exam
L.
min {a-bf
of air is 1*0003 and wavelength of yellow light in
sf
3. Intensity at any point, where phase difference
B
vacuum is 6000 A. Find the thickness of air column
is ^
which will contain one more wavelength of yellow
Yo
/ = IcR- = k{a~ + b~ + 2 ab cos $)
r

light than in the same thickness of vacuum.


oo

Solution. Let thickness i of vacuum contain n


I = k [!^ +12+ 2^/j cos (J>J
d
eB
e
ru

waves and same thickness of air contain (n +1) waves.


t t
If /j = /2 = /, then
, and = L 2/(1 +COS i|)) = cf.4/cos2({i/2
n

n= —
o

X 6000
ou

4. When sources are incoherent, no interference


ad

t t lit l-0003t
n + \ = occurs. Resultant intensity at any point
iY

X' X!\x X 6000 / = /| + /2 = ^ + b~)


Units used. Often we deal with ratios of identical
F

l-00()3r
nd

/
Re

+ 1 =
6000 6000 quantities. They have no units.
Fi

r + 6000= 1-0003/
0-0003 / = 6000 Example |] Two plane monochromatic
6000 waves propagating in the same direction with
/ = = 2 X 10^ A = 2 mm amplitudes A and 2 A and differing in phase by
00003
7c/3 superimpose. Calculate the amplitude of the
Example A light wave has a frequency resulting wave. (CBSE Sample Paper 2003)
of 5 X 10^** Hz. Find the difference in its Solution. Here, a = A and /? = 2 A, (f) = tc/3,
R = 2
wavelengthsin alcohol of refractiveindex 1*35 and
glass of refractive index 1*5.
Solution. Here, v = 5 x 10*"^ Hz
R =
■yja- +b^ +2flZjcos(t)
V c 3x10* +44“ +2A.2ACOS7T/3
Xa o _

u .V 1-35x5x10’^
5A^+4A^x-^ = A-^
i’

= 4-445 X 10"'^ m
10/37
V^VE OPTICS
Example E In a Young’s double slit (//) path diff. X/2. pha.se difference ~
27t
- 7C
experiment, the intensity of light at a point on the
screen where the path difference is X\s K units.
Find the intensities at a point, where path diff. is /y^^=/,+ /2+ 2^/7^ cos (j)"
(/) x/4 m m m) ui. {CBSE 2014, 2012)
= /o + /o + 2/y(-l) = Zero.
/
Solution. From / = /| + /2 + 2 cos = infinite
Zero
if /j = /2 = /q, then / = /q + /q + 2 /y cos <j) R

= 2 /y { 1 + cos (j)) =: 4 /y COS" (])/2


TYPE III. YOUNG’S
When path difference is ^, (|) = 2 7t
DOUBLE SLIT EXPERIMENT
/ = 4 /y cos^ n = 4 1q= K
27t
= tU1 Formulae u.sed

w
(/) When path difference = X/4, ‘t> = —
4
D
\2
1 K 1. Position of bright fringes is given by x = nX — ,
= 21
1 = 4 Iq cos^ k/4 0
V2 « ■ 2

Flo
where n=l,2,3... for 1st, 2nd, 3rd... bright fringes.
(I'O When path difference = ?l/3, (j) = 2 7t/3 2. Position of dark fringes is given by

eeee
( 1 K X D

Fr
/=4/yCos-(2 7C/3) =4/y - = / x = (2n-\) ,
0 4 2 d

X where n = 1, 2, 3,... for first, second, third,... dark


{Hi) When path difference = -^, ([) = 7t
for fringes.
ur
I = 4 If,0 cos^ 7c/2 = 0 XD
3. Width of each bright/dark fringe ^ =
ks
d
Example S In Young’s double slit experi
Yo
4. When the entire apparatus is immersed in a
oo

ment using monochromatic light of wavelength X, transparent medium of refractive index )i, fringe
the intensity of light at a point on the screen where
eB

width,
path diff. is X is K units. Find the intensity of light
at a point where path difference is X/3.
d \id ]i
ur

(CBSE 2014)
ad

5. Angular width of interferencefringes.


Yo

Solution. Here /y = K, where path diff. is X.


1 = 2. where path diff. = X/3 e=P=A
D d
2;r
d
Re

Phase diff., ^ ~ = 120° Units used. Distances x:, d, D, wavelength X, fringe


in

width p all are in metre ; 6 is in degrees :


From / = /y cos^ (j)/2 n is a number.
F

120' \2
I = K cos = K = K/4 Example B Two slits are made 1 mm apart
2 I \2)
and the screen is placed 1 m away. What is the
Example 0 Light waves from two fringe separation when blue green light of
coherent sources arrive at two points on a screen wavelength 500 nm is used ?
with path differences 0 and X/2. Find the ratio of (NCERT Solved Example)
intensitiesat the points. [CBSE 2017 (C)l Solution. Here, d - I mm - 10'^ m D = 1 m,
Solution. Let /, = l2 = h) X = 500 nm = 500 x 10"^ m = 5 x 10“^
{/) path diff. = 0, phase difference <|) = 0 Fringe separation = fringe width, P = ?
I= /j + /") + 2.^/j”/^ cos (j) XD 5xl0"^xl
= 5x10 m = 0*5 mm
= /y + /y + 2 /y COS 0° = 4 / 0 10-3
10/38 '4. Fundamental Physics (XII) kV«iWII
Example ^ In Young’s double slit TYPE IV, FRESNEL DISTANCE
experiment, the two slits 0*15 mm apart are
illuminated by light of wavelength 450 nm. The Formula used. Fresnel distance,
screen is 1*0 m away from the slits. Find the
a~
distance of second bright fringe and second dark
fringe from the central maximum. How will the X
fringe pattern change if the screen is moved away Units used. Aperture a, wavelength X and Fresnel
from the slits ? (CBSE 20101 distance Zp, ail are in metre.
Solution. Here, d = 0-15 mm = 1-5 x 10^ m
X = 450 nm = 450 x I0“‘^ m, Example m For what distance is ray
Z)= lOm optics a good approximation when the aperture is

ww
Distance of 2nd bright fringe 3 mm wide and wavelength is 500 nm ?
IXD 2x450x 10-*^ xl-O (NCERT Solved Exdr. -
=
Solution. Here, a = 3 mm = 3 x 10"-^ m,
d 1-5x10^
X - 500 nm = 5x10"^ m
= 6 X 10”^ m = 6 mm

Flo
Fresnel distance, Zp = ?

e
Distance of 2nd dark fringe

eree
3 XD
- 4*5 mm
Cl
^ _ (3x10-3)^ - 18m
^2 =

FFr
2 d X 5x10"'^
uurr
As D is increased, fringe width of each fringe

orr
XD \ .
increases.
sfo
1 TYPE V. DIFFRACTION OF
i LIGHT AT A SINGLE SLIT
Example B In a double slit experiment, the
kks
Yoo
angular width of a fringe is found to be 0*2” on a
oooo

Formulae used.
screen placed 1 m away. The wavelength of light used
1. Condition for diffraction minima is
is 600 nm. What will be the angular width of the
eBB

fringe if the entire experimental apparatus is a sin 0 = n X, where « = 1, 2, 3, ....for 1st,


immersed In water ? Take refractive index of water 2nd, 3rd dark bands respectively.
to be 4/3.
urr

2. Condition for diffraction maxima is


Solution. Here, 0, = 0-2”, D = 1 m, = 600 nm,
ad

fl sin 0 = (2« + 1) XJ2, where n = I, 2, 3, for 1st,


YYo

02=?, P = 4/3 2nd, 3rd, bright bands respectively.


2DX_2fX
3. Width of central maximum, 2x=-
dd

. 0j |i 4 a a
Re

1
inn

4. Angular width of central maximum in radian,


0, =-0, =-x0.2‘ = 0*15'
F

2 4 1 4 2X
20 =
Example DO In double slit experiment using Cl

light of wavelength 600 nm, the angular width of a 5. In diffraction at a circular aperture, angular
fringe formed on a distant screen is 0*1”. What is
1-22^
the spacing between the two slits ? spread, 0=
Solution. Here, X = 600 nm = 600 x 10"^^ m d
= 6 X 10"^ m. Units used, a, x,f, D and X are in metre and n is a
01 TC pure number.
0 = or= rad., d = l
180
X
From 0 = - Example [0 A slit of Width ^cV is illumi
d
nated by light of wavelength 5000 A. For what
X OxlO""^
d = ~ = 3-44 X 10-* m value of ‘d* will the first maximum fall at an angle
0 K
of diffraction of 30”.
xO-1 (CBSE 2006)
180
WAVE OPTICS 10/39

Solution. Here, X = 5000 A = 5 x lO ^ m,


3 X 3X 3x5890x10"'^
^/= ?, n= 1, 0 = 30“ e = 0 =
2 (I 2d 2x0-25x10“-'
For maxima ol’ diffraction,
= 3*534 X 10“*' radian
X
cl sin0 = (2» + 1) —
2
Example m A laser beam has a
wavelength of 7 x 10“^ ni and aperture 10”“ m.
The beam is sent to moon at a distance of 4 x 10^
^ _ (2n + V)X _ 3x5x10"'^ = 1*5 X 10”^ m
km from earth. Find the angular spread and areal
” 2 sin 9 2 sin 30°
spread of the beam on reaching the moon.
Example m A parallel beam of light of Solution, Here, X = 7 x 10”^ m, a = 10““ m
600 mil falls on a narrow' slit and the resulting D = 4x 10-‘' km = 4x lO*' m

diffraction pattern is observed on a screen 1*2 ni For diffraction at a circular aperture,

ww
away. It is observed that the first minimum is at a
1-22?. i-22x7xl(r'^
distance of 3 mm from the centre of the screen. angular spread = 8 =
Calculate the width of the slit. ●i; 2013' d 10-2

FF loo
= 8*54 X 10 *■’ radian
Solution. Here, X = 600 nm = 6x10“^ m,
Areal spread = (D

ree
D = I -2 m, « = 1, X = 3 mm = 3x10“’ m, a = ?
For first minimum, a sin 6 = ;j ^ = (4 X 10*' x 8-54 X IO“-‘')2 = M97 x lO'-' in^
Example [Q Two spectral lines of sodium

reFe
A'
a ● = \X
Dj and Dj have wavelengths of approximately
oroFr
r ur
D
5890 A and 5896 A. A sodium lamp sends incident
s ff
XD 6x10-2 xl-2 plane wave on to a slit of width 2 micrometre. A
a - = 2-4x lO^m screen is located 2 m from the slit. Find the spacing
3xl0”'-'
k
YYouo
X
between the first maxima of two sodium lines as
koso

= 0*24 mm measured on the screen. [CTkStZOIJ, 2(M)6(C)1


Solution. Here,
BBoo

Example Determine the angular


X| = 5890 A = 5890 X 10”'*’ m,
r ee

separation between central maximum and first


a = 2 p m = 2 X 10“^’ m
order secondary maximum of the diffraction
^2= 5896 A = 5896 x 10“'^ m, D = 2 m
ad
ouur

pattern due to a single slit of width 0*25 mm when


For the ifrst secondary maxima.
Yo

light of wavelength 5890 A falls on it normally.


33.
Solution. Here, 0 = ? sin 6 =
1 ■^1
d

r/ = 0-25 mm = 0-25 X 10”^ m 2a D


Re
idnY

X = 5890 A = 5890 x lO”*^ m 3X, D 33., D


FFin

and -'■T
As is clear from Fig. 10.37,
or
'^1 = 2 a 2a

Central max. FIGURE 10.37 .●. Spacing between the first secondary maxima
of two sodium lines

3D
= ^2 - A, =
2(7
(3., -3,|)
First Oder
-lO
3x2 (5896-5890)xl0
= 9x10-* m
2x2x10"'^

Example m In a single slit diffraction


experiment, first minimum for red light (660 nm>
angularseparationbetween centra! maximumand first coincides with first maximum of some other
order secondary maximum is wavelength X'. Calculate 3,'.
10/40 Fundamental Physics (Xll)EZsl9D
Solution. Here, X,. = 660 nm ; X' = ? Solution. Here, « = 3 mm = 3 x lO ^ m
For diffraction minima, X = 620 nm = 620 x lO-^'^ m. D = 1-5 m
nX For first order minimum,
a sin 0 - nX, sin 0 = nXD \XD
a

a a
IX
For first minima of red light, sin 6 = —-
a
For third order maxima,
X D
=(2n + I)
For diffraction maxima, asin 6 = (2« +1) — 2 2 a
2
XD IXD
for first maxima of X\ a'2 =(2x3 + 1) 2a 2a
3X' 3X'
■ sin 0' = As the two locations are on the same side of the

w
a sin 0' =
2 ’ 2a
screen, therefore, required distance
As the two coincide, therefore, sin 0' = sin 0 IXD XD XD (1
— 1

Flo
X = ^2 -X|
3X' _ 'X, 2a Cl a 2 >
or X' = -X

e
3 '■

reee
2a a
5XD 5 620xI0-‘^xl-5
= - X

FFr
2 a 1
3x10-3
X' =
j (660) = 440 nm
or
= 7-75 x 10"^ m
Example n>] What should be the width of Example ^ A sHt of width d is illuminated
for
ur
each slit to obtain 10 maxima of the double slit by white light. For what value of d is the first
minimum, for red light of X = 650 nm, located at
kss
interference pattern within the central maximum
of single slit diffraction pattern ? point P at 30". For what value of the wavelength
Yo
oo

(NCERT Solved Example) of light will the first diffraction maxima also fall
atP? (CBSE Sample Paper 2011)
eB

Solution. If a is width of single slit, then for


central maximum of single slit diffraction pattern, Solution. Here, X = 650 nm, d = 2
For first minimum, n= 1,0 = 30®
2X
ur

Total angular width. 20 = ...(/) d sinQ = riX


ad

a
YYo

n X 1x650
For 10 maxima of double slit interference d = = 1300 nm
sin 0 sin 30®
pattern, total angular width,
For first maximum to lie at P, we have
d

X
Re

20=10- ..(»)
in

X' 3X'
d d sinQ = {2n + 1) —
F

2 2
2X lOX
From (/) and (ii),
a d X' = - c/sin 9 = —xl300x- = 433*3 nm
3 3 2

« = — = — = 0*2 mm
5 5
Example m A monochromatic light of
wavelength 500 nm is incident normally on a single
Example [R In the diffraction due to a slit of width 0*2 nim to produce a diffraction
single slit experiment, the aperture of the slit is pattern. Find the angular width of central
3 mm. If monochromatic light of wavelength maximum obtained on the screen, 1 ni away.
620 nm is incident normally on the slit, calculate Estimate the number of fringes obtained in
the separation between the first order minima and YDSE with fringe width 0*5 mm, which can be
3rd order maxima on one side of the screen. The accommodated within the region of total angular
distance between the slit and screen is 1*5 m. spread of the central maximum due to a single slit.
(CBSE 2019) (CBSE 2017)
WAVE OPTICS 10/41

Solution. Here, X = 500 nm = 5 x 10“^ m ;


TYPE VI. RESOLVING POWER
a = 0-2 mm = 2 x 10"^ m, D = 1 m OF MICROSCOPE AND TELESCOPE
Angular width of central maximum
Formulae used.
2X 2x5x10-'^ 1. Limit of resolution of a compound microscope
a 2x10"' X
cl =
= 5 X 10"^ radian 2|xsin6
2XD 2. Resolving power of compound microscope
Width of central maximum =
a 2psine
d X
= 5 X lO-^x 1 m

Note. The factor (|i sin 9) is called numerical


As P = 0-5 mm = 5 x 10^ m
aperture {N.A)

ww
Number of interference fringes in the central 3. Limit of resolution of an astronomical telescope
5x10"-' \-22X
= 10 r/e =

FF loo
maximum. n =
5x10"^ D

4. Resolving power of an astronomical telescope


Example ^ Two wavelenths of sodium

ree
D
light 590 nm and 596 nm are used in turn to study JQ 122X

rFee
the diffraction at a single slit of size 4 mm. The
distance between the slit and screen is 2 m.
Units used. X, d, D in metre ; 6 , i/6 in degrees ;

F
p has no units.

oor r
rur
Calculate the separation between the positions of
s ff
first maximum of the diffraction pattern obtained
in the two cases. [CBSE 2017(01 Example ^ Calculate the resolving power
k
of a microscope if its numerical aperture is 0*12
Solution. Here, Xj = 590 nm = 590 x 10“^
YYoou
m

and wavelength of light used is 6000 A.


ookos

^2 = 596 nm = 596 x 1 m
Solution. Here, N.A, = p sin 6 = 012,
fl = 4 mm = 4 x 10’^ m, D = 2m
BBo

X = 6000 A = 6 X 10“^ m
For first secondary maxima
re

R.P. of microscope,
sin6 = (2n + l) — 1 _ 2psin6 _ 2x0-12 = 4 X 10^ m -1
ouur
ad

2a
Yo

d X 6x10-'^
X 3X
sin 9 = 0 = — , when 0 is small Example Calculate the separation of
D 2a
dY

two points on moon that can he resolved using 600


Re

3XD
idn

vY = cm telescope. Given distance of moon from earth


= 3-8 X 10*® cm. The wavelength most sensitive to
FFin

2a

For two wavelengths. eye is 5*5 x 10“^ cm.


Solution. Here, x = 2 D- 600 cm,
3X.Z) 3X^D
Aj =
1
, a'2 —
X = 5-5x 10--^ cm
2a
Limit of resolution.
Separation between the positions of first
1-22 X 1-22 X 5-5x10-5
secondary maxima d% =
D 600
D = M X lO-'^rad.
Aj — 3 (X2 X|) —
If .Y is separation of two points on the moon that
can be resolved and d is distance of moon from
3(596 - 590) xl 0-^x2 V

2x4xI0"-^ objective of telescope, then dQ = —,


-7 10
A2-.Y1 = 4-5 X 10-^ x=(dQ)d= I I X 10
m X 3-8 X 10 cm

= 4180 cm

)
10/42 ‘P>tetcUefi-'4. Fundamental Physics (XII)EEIHD
Example^ A laser beam has a wave Example m A radar wave has frequency
length of 7 X 10“^ m and aperture 10“^ m. The of 8'1 X 10^ Hz. The reflected wave from an
beam is sent to moon, the distance of which from aeroplane shows a frequency difference of 2*7 x
earth is 4 x 10^ km. Find the angular spread and 10^ Hz on the higher side. Calculate velocity of
areal spread of the beam when it reaches the moon. aeroplane in the line of sight.
Solution. Here, X. = 7 x 10“^ m, « = 10“^ m ; Solution. Here, v = 81 x I0‘^ Hz
D=4x 10^ m Av = 2-7x IO-’Hz
For the circular aperture, angular spread
Velocity of aeroplane, ^ 12 —V xc

VllX l-22x7xl0“^
9 =
2-7 X10-^

ooww
10-2 1
a
— ^ x3xl0^ - 50 m/s
= 8-54 X lO-^’ rad 2 81x10'^
Areal spread = (D Q)~ Example 0s] With what speed should a
= (4x 10^ X 8-54 X 10-^)2 galaxy move with respect to us so that the sodium
= M97 X 10‘^ m2 line at 589-0 nm is observed at 589-6 nm ?

e
re
(NCERT Solved Example)
Example The diameter of human eye

rFFl
ree
Solution. Here, u = ?.>. = 589-0 nm

F
lens is 2 mm. What should be the minimum
A X = 589-6 - 589-0 = 0-6 mn
separation between two points situated at 50 m

rrF
from eye, to resolve them. Take wavelength of light AX 0.6
= 5000 A. (CBSE 2002) From V = xc = x3xl0® m/s.

sffoo
ouur
X 589-0
Solution. Here, D = 2 mm = 2 x 10~2 m, t/ - 9

V - 3-06 X 10^ m/s = 306 km/s


= 50 m, X = 5000A = 5x lO""^ m
kosk
Resolving power of eye The galaxy is moving away from us, as apparent
Yo
X increases.
oo

D _ 1 _ 1
Y

>’
p>e ^ A Star is moving towards the
BB

Exam
~ \-22l~ dQ~ d/y d
earth with a speed of 4 x lO’ m/s. If the wavelength
rre

,
d
1-22 Xxv=- emitted by the star is 500 nm, what would be the
change in wavelength received on earth ?
ouu

D
Y

Solution. Here, u = 4 x 10^ m/s. X = 500 nm


ad

1-22x5x10-2x50 AX = ?
dY

2x10-2 V 4x102
d=\52-5 xl0^m= 1-525 cm From ^ =- x500 nm
innd
Re

c 3x10*^
AX = - 66-7 nm
Fi

I I
F

TYPE VII. DOPPLER’S EFFECT IN LIGHT Negative sign implies decrease in the observed
wavelength.
Formulae used.
TYPE VIII. BREWSTER’S
Av V
LAW AND LAW OF MALUS
(0
V c

Formulae used.
AX V
where v is velocity of star. 1. p = tan ip . where is polarizing angle
(/o ^ c ...Brewster's law

(Hi) velocity of aeroplane/rocket/submarine : When polarization occurs on reflection, the reflected


and refracted components are mutually
_1 AX c , 1 Av
= ± c perpendicular.
^”■^2 X 2 V
2. / = /q cos2 9 ...Law of Mains
Units used, v is in hertz.; v and c in ms-^ and X in
metres.
Units used, ip and 8 are in degrees or radian ; p has
no units.
WAVE OPTICS 10/43

Exsmple iS] If the angle between the pass Solution. Here,

axis of polariser and analyser is 45®, write the ratio /j = intensity of light transmitted by Fj
of intensities of original light and the transmitted = intensity of light transmitted by F3
light after passing througl analyser. (CBSE 2009) = /| cos“ p
Solution. Let original intensity of unpolarised I2 = intensity of light transmitted by P2
light be /q - /j cos- (0 - P)
Intensity of polarised light transmitted from
As I2 = /3 cos- (0 - p) = 1

oww
polariser = -2. cos (0 - p) = I = cos 0® 9=p
2
As 0 = 45°.
Also, /j = /t = /j COS^ P cos^ (0 - p)
= /] cos^ p cos^ 0
Intensity of polarised light transmitted from
cos^ P = 1, Hence 0 = p = 0 or 7C rad.

e
I
analyser = cos^ 45°. = 0
x 0
pie g] A narrow beam of unpolari

re
Exam
2 yn, 4
sed light of intensity is incident on a polaroid

FFrllo
I
0
Py The light transmitted by it is then incident on

reF
The required ratio = = 4: 1
I.JA a second polaroid P2 with its pass axis making an

e
u

angle of 60° to the pass axis of P^. Find intensity


uoru
Example m Two nicols are so oriented of light transmitted by p2> (CBSE 2017)

osFr
that the maxipium amount of light is transmitted. Solution. Intensity of transmitted light polarised
To what fraction of its maximum value is the
intensity of transmitted light reduced when the
analyser is rotated through (i) 30" (ii) 60" ?
fkfor
by F[ is /| = /q/2.
As 0 = 60°. tliercfore according to Law of Malus,
okso
intensity of polarised light transmitted by P^ is
Solution. Net intensity transmitted is
Y
Yo
/ = /qCOS-0
= /, cos- 60° = f 11 - ~8
oo
BB

(/) Here 0 = 30° 2^2j


Y
r ree

I
/ = /jj (cos 30°)- = 0 9 TYPE IX. TYPICAL EXAMPLES
ouu
ad

3
Ydo

/=-/,,= 0.75 /„0 = 75% I 0 Example ES A beam of light consisting of


4 «
Intensity transmitted is reduced to 75% of two wavelengths 800 nm and 600 nm is used to
nidn

the maximum intensity. obtain the interference fringes in YDSE on a


Re

(//) Here. 0 = 60° screen held 1*4 m away. If the two slits are
separated by 0*28 mm, calculate the least distance
F
Fi

/ = /q ® ^ from the central bright maximum, where the


bright fringes of the two wavelengths coincide.
= ^=0.25/q =25%/„ 0

Solution. Here,
(CBSE 2012)
= 800 nm and Xo = 600 nm,
Intensity transmitted is reduced to 25% of
the maximum intensity. £> = 1.4 m. ^ = 0-28 mm = 0-28 x 10“'' m
The bright fringes of two wavelengths will
Example m Unpolarised light of intensity coincide at the least distance x from the central
/„ passes through two polaroids F| and P2 such D D
that pass axis of P2 makes an angle 6 with the pass maximum when x = nX, — = (« +1)X^ —
' d ~ d
axis of Fj. A third polaroid F3 is placed between
Fj and P2 with pass axis of F3 making an angle P n X 800 = (n + 1) 600 or 4 n = 3 « -1- 3 ; n = 3
with that of Fj. If /j, I2, represent intensities of 1-4
light transmitted by Pv Pv determine the .r = 3x800xl0-^x
values of angle 0 and p for which 12- ly 0-28x10"3
(CBSE 2014) = 12 X 10"^ m = 12 mm
10/44 Fundamental Physics (XII)EEIHD

Example In YDSE With monochro


I = I^+ I2+ ~ .^/j ^2
matic light, fringes are obtained on a screen placed
at some distance D from the slits. If the screen is / max U cos 0
= /| + /t’*' I '2
moved 5 x 10“^ m towards the slits, the change in
fringe width is 3 x 10“^ m. If the distance between
the slits is 10“^ in, calculate the wavelength of light For n identical waves, each of intensity (/q).
used. {CBSE 2006 (C)]
Solution. Here, (D' - D) = 5 x I0“- m, I + times)-
max

(Y - p = 3 X K)--'* m, d = 10"^ m, X = ?
=ri~I «)
XD XD’
and P' = —

ww
When interference is due to incoherent sources,
d
0 varies randomly with time,
X{D'-D)
{cos (|))^,, = 0
d

Flo
=h+h
I max

(P^-P)d _ 3x10-5 xl0“3

e
For n identical waves, each of intensity /q,

ree
iO'-D) 5x10-- r
= /() + /q
max n times

Fr
= 6x lO""^ m = 6000 A I max = nl 0

rF
Example §0 Find the ratio of the intensity Example m] In a Young’s double slit
uurr
at the centre of a bright fringe to the intensity at
the point one quarter of the distance between two
fringes from the centre.
s for
experiment, fringes are obtained on a screen placed
a certain distance away from the slits. If the screen
is moved by 5 cm towards the slits, the fringe width
kks
changes by 30 |im. Given that the slits are 1 mm
Yo
Solution. Total intensity at any point is given
oooo

apart. Calculate the wavelength of light used.


by I = kR^ = k {cP- + b~+2 ab cos 4>)
[CBSE 2018 (C)]
eB

when b = aj = k {a^ + 2 cos


Solution. Let D be the distance of screen from
= 2 ka-(\ + cos (f>) the slits. Here, d = 1 mm = 0-1 cm, X = ?
ur

At the centre of a bright fringe, (f) = 0°


XD
ad

/| = 2 ka~ (I + cos 0°) = 4 ka~ Fringe width, p = ...(/)


YYo

d
Distance between two fringes = p, which is
When D' = (D - 5) cm.
proportional to wavelength (X)
dd

P' == P - 30 pm
Re

X
in

Now — corresponds to a phase diff. = (p-30x 10"^) cm


F

XD'
_ 2n _ 7T P' = ...Hi)
d
“T“ 2
X
J2 = 2kcP{\ + cos Tt/2) = 2 P-P' = - {D-D')
d

/i _Aka- _2 30x10^ =
Xx5
/-, 2ka^ 1 0-1

Example EB Find the maximum intensity , 30x10^x01


in case of interference of n identical waves each 5
of intensity /q if the interference is (a) coherent X = 6 X 10"5 cm = 6 X 10"^ m
(b) incoherent.
Solution. Intensity of each of n waves = /q Example m The objective of an astro
When interference is due to coherent sources. nomical telescope has a diameter of 150 mm and
WAVE OPTICS 10/45

focal length of 4*0 ni. The eye piece has a focal 1


From (0 and (//), tan p =
length of 25*0 mm. Calculate the magnifying sinC
power and resolving power of telescope. What is If r is angle of refraction, then p = 90°- r
the distance between objective and eye piece ? 1 1
Take X = 6000 A. tan(90®-r) = cot/- =
tan r sin C
Solution. Here, D = 150 mm = 150 x 10"-^ m,
Jq = 4-0 m. /p = 25-0 mm = 25 x 10“^ m or tan r = sin C r = tan”* (sin C)
X = 6000 A = 6 X 10“^ m
Example m Two polaroids are placed at
If we assume that final image is formed at 90'‘ to eachother and the transmitted intensity is
infinity, then magnifying power, zero. What happens when one more polaroid is
0 _
4-0 placed between these two bisecting the angle
/« = —
= 160 between them ? What will be the direction of
4 25x10”^
polarization of outcoming beam ? (CBSE2008)

w
D 150x10”^ Solution. If E is amplitude of electric field
Resolving power =
hl2X l-22x6xl0"'^ component emanating from 1st polaroid, then from

Flo
= 2-05 X 10^ 2nd Polaroid at 45®, the amplitude of electric field
component is
Distance between objective and eye piece

reeee
= /y+/^, = 4 0 + 25 x lO-"-' E' = Ecos 45® =E/^|2

FFr
= 4*025 m Again amplitude of electric field component
coming from 3rd polaroid at 45® to 2nd polaroid
Example Gj] The critical angle between a would be

for
ur
given transparent medium and air is denoted by
E 1
C. A ray of light in air enters this transparent E" = E' cos 45° = — —> half of E
2
kkss
medium at an angle of incidence equal to
polarizing anglep. Deduce a relation for the angle
Yo
As Intensity =>= E~
oo

of refraction in terms of C.
Intensity transmittedfrom three polaroids
Solution. According to Brewster’s law,
eB

will be l/4th of the intensity transmittedfrom theifrst


p = tan /? ...(0 Polaroid. The direction of polarization of the
1
Also,
outcoming beam will be same as that coming from
...(«)
r

sin C the ifrst Polaroid.


ou
ad
YY
ndd
Re

1. Wave optics is based on wave theory of light put forward by Huyghen and modified later by Fresnel.
Fi

According to wave theory, light is a form of energy which travels through a medium in the fonn of transverse
waves.

2. A wave front is defined as the locus of all the particles of a medium, which are vibrating in the same phase.
When source of light is a point .source, the wave front is spherical. When source is linear, the wave front is
cylindrical. At very large distances from the .source, a portion of spherical or cylindrical wave front appears
to be plane. A wave front travels parallel to itself and perpendicular to the rays.
3. Huygens principle of geometrical construction of a wave front at any instant says :
(/) Every point on given wave front (called primary wavefront) acts as a fresh source of new disturbance,
called secondary wavelets.
(ii) The secondaiy wavelets travel in all the directions with the speed of light in the medium.
(Hi) A surface touching these secondary wavelets tangentially in the forward direction at any instant gives
the new (secondary) wave front at that instant.
Using Huygens principle, we can prove the laws of reflection and laws of refraction on the basis of wave
theory.
10/46 Fundamental Physics (XII)ESIHD

4. Superposition principle says that when two or more wave motions travelling through a medium superimpose
one another, they lose their individual identity. A new wave is formed in which resultant displacement (y)
at any instant is equal to vector sum of the displacements due to individual waves at that instant i.e.

y -.V[+.y2+>’3'*‘
This principle applies to mechanical waves as well as electromagnetic waves.
5. Interference of light is the phenomenon of redistribution of light energy on account of superimposition of
light waves from two coherent sources. At the points where resultant intensity is maximum, interference is
said to be consiriictive. At the points where the resultant intensity is minimum, interference is said to be

oww
destructive.

6. Thomas Young’s double slit experiment was the first to demonstrate the phenomenon of interference of
light. Using two slits illuminated by monochromatic light source, he obtained bright and dark bands of
equal width placed alternately. These were called intetference fringes. All bright fringes have same intensity
and all dark fringes are perfectly dark.

e
FFrlo
re
If M-|, W2 are w idths of two slits : /j, are intensities of light coming from two slits ; a, h are the amplitudes
I 2

ree
U’

F
a
I
of light from these slits, then
^2 /t

rF
7. The condition for constructive interference is that path difference between two waves should be zero or
an integral multiple of full wavelength. In that event, crest of one wave would fall on crest of the other and

fsoor
ouur
trough would fall on trough. The resultant amplitude = (« + h)
skf
8. The condition for destructive interference is that path difference betw'een two waves should be an odd
ooko
integral multiple of half the wavelength. In that event, crc.si of one wave would fall on trough of the other
Yo
and vice-versa.The resultant amplitude= (« - h).
Y
Bo
reB

1 (a + b)-
Therefore, in the interference pattern, we have
max

I .. («-/?)-
mill
uur
oY
ad

For nth bright fringe, path difference = x— = nX where n = 0 for central bright fringe.
D
dY

n =1 for first bright fringe, n = 2 for second bright fringe & so on. Here x is distance of nth bright fringe
from the centre.
innd
Re
Fi
F

For nth dark fringe, path difference = .r


D 2

where n = 1 for first dark fringe. /j = 2 for 2nd dark fringe and so on.
D
The width of each of brighl/dark interference fringes is found to be = A.—
where D is distance of screen from coherent sources and d is distance between the two coherent sources.
When source gives white light, interference fringes arc coloured but centre of pattern is white. Red fringes
are wider compared to violet fringes as > X V.

Q. Coherent sources are those which emit continuous light waves of same amplitude, same wavelength/
frequency in same phase or having a constant pha.se difference. Two independent sources can never be
coherent. Coherent sources are usually the object-image sources i.e. they are produced from a single source
of light.
Coherent sources are an essential requirement for interference of light. They should be strong, with least
back ground, lying close to eachother.
WAVE OPTICS 10/47

10. Diffraction of light is the phenomenon of bending of light around comers of an obstacle or aperture in the
path of light. On account of this bending, light penetrates into the geometrical shadow of an obstacle. The
diffraction pattern due to a single slit consists of a central bright band having alternate dark and weak
bright bands of decreasing intensity on both sides.
The condition for nth secondary minimum is that path difference = a sin 9„ = n X, where n = 1, 2, 3... And
the condition for nth secondary maximum is that path difference = a sin 0„ = (2 n +1) — where « = 1,2,3,...
2
Width of central maximum is

2x = 2DX_2fX
a a

Here, a is width of slit and D is distance of screen from the slit ;/is focal length of lens for diffracted light.

ww
Diffraction is supposed to be due to interference of secondary wavelets from the exposed portion of wavefront
from the slit.

Whereas in interference, all bright fringes have same intensity ; in diffraction, successive bright bands are
of decreasing intensity.

Flo
11. Resolving power of an optical instrument is the ability of the instrument to resolve or separate the images

e
of two point objects lying close to each other.

eree
Resolving power of a microscope is defined as the reciprocal of the minimum distance {d) between two

FFr
point objects, which can just be seen as separate through the microscope

oorr
uur r
j^ p ^ 1 _ 2jisine
d X
sf
where |i is refractive index of the medium between object and objective lens, 0 is half the angle of cone of
sk
Yoo
light from the point object, d represents limit of resolution of microscope.
ooko

Resolving power ofa telescope is defined as reciprocal of the smallest angular separation (dQ) between two
eBB

distant objects, whose images are just seen as separate in the telescope

R.P.=
1 _ D
uurr

dQ 1.22X
ad

where D is diameter or aperture of the objective lens of the telescope. dQ represents limit of resolution of
Yo

telescope.
12. According to Doppler’s effect in light, whenever there is a relative motion between a source of light and
dY

observer, the apparent frequency of light received by observer is different from the true frequency of light
Re
innd

emitted from the source of light. The apparent frequency v' is given by
FFi

v' = v 1+-
V cj

where v is velocity of star and c is velocity of light in vacuum, v being the actual frequency of light emitted
from the source + sign is used when source moves towards the observer and vice-versa. We can, show that
V V
Av = ±—V and AX=+-X
c c

When a star is approaching earth, v' > v, A v is +. Accordingly, X <X and A X is negative.
The spectrum of the star shifts towards violet end (lower wavelength side). Similarly when a star moves
away from earth, the spectrum of the star shifts towards red end, (higher wavelength side).
Doppler’s effect in light is used in measuring speed of a star/galaxy, satellites, aeroplanes, submarines, etc.
13. Polarization of light is the phenomenon of restricting the vibrations of light (electric vector) in a particular
direction ; on passing ordinary light (unpolarized) through certain crystals like tourmaline crystal. This
crystal acts as a polarize.r
10/48 '4- Fundamental Physics (XII)
The plane in which vibrations of polarized light are confined is called plane of vibration. A plane
perpendicular to the plane of vibration is called plane of polarization.
Our eye cannot detect whether light is polarized or not. To delect polarization, we have to use another
crystal, which acts as the analyse.r When axes of polariser Tj and analyser T2 are parallel, the intensity and
character of light transmitted by Tj and T2 remain unaffected. However when axes of the two crystals are at
90° to each other, light is completely cut off. This establishes that light waves are transverse in character.
When unpolarized light is seen through a single crystal (polaroid), intensity of transmitted light decreases
on account of polarization. However, on rotating the crystal about the direction of propagation as axis,
intensity of polarised light does not change.
On the contrary, when polarized light is passed through another crystal called analyser and the analyser is
rotated, the transmitted fraction of light changes from maximum to zero. This is how, we detect polarized
light.

ww
14. According to Law of Malus, when a beam of completely plane polarised light is incident on an analyser,
the resultant intensity of light (/) transmitted from the analyser varies directly as the square of the cosine of
the angle (0) between plane of transmission of analyser and polariser i.e. / « cos^ 0.

Flo
15. According to Brewster’s law, when unpolarized light is incident at polarizing angle (1^) on an interface
separating a rarer medium from a denser medium of refractive index (i, such that p = tan ip, then light

ee
reflected in the rarer medium is completely polarized. The reflected and refracted waves, in this case are

rere
perpendicular to each other. Obviously polarizing angle depends on nature of media in contact, and on the

rFF
colour of light.
16. Main uses of polaroids are in wind shields of automobiles, sun glasses etc. They reduce head light glare of
uurr
foor
cars and improve colour contrast in old paintings. They are also used in three dimensional motion pictures
and in optical stress analysis.
ks s
Yoo
oook

QUESTIONS

Bdsed on N E Book
eBB

I. Multiple Choice Questions 5. The shape of interference fringes in YDSE, when


distance between the slit and screen is very large
uurr

1. A plane wave passes through a convex lens. The


ad

as compared to fringe width is


geometrical shape of the wavefront that energy is
Yo

(a) straight line (b) parabolic


(a) plane
dY

(c) circular (d) cylindrical


(b) diverging spherical
(CBSE Sample Paper 2019-20)
Re
innd

(c) converging spherical


6. For an aperture of size (d) illuminated by a parallel
(d) none of these
FFi

beam of light having wavelength (A.), the Fresnel’s


2. In YDSE, when separation between two slits is distance is
increa.sed, fringe width
{a)Z=d^/X ib)Z=dlX
(a) decreases (b) increases
{c)Z=dJX^ {d)Z=dX
(c) remains the same (d) none of these 7. Unpolarised light is incident on a plane glass
3. Two light beams of intensities /, and I2 produce
surface having refractive index . The angle of
an interference pattern. The contrast between the
fringes is the best when incidence at which reflected and refracted rays
become perpendicular to each other is
(a) /| = Ijn (b) /] - I2J2
(a) 15° (b) 30°
(C) /i=/2/4 (^/)/l= I2
(c) 45° (d) 60°
4. The wavefront due to a source of light situated at
infinity is (CBSE Sample Paper 2019-20)
(a) spherical (b) plane 8. The path difference between two waves at the place
of destructive interference is
(c) rectangular (d) cylindrical
WAVE OPTICS 10/49

(a) integral multiple of X. the two interfering waves of light, then the path
(b) odd integral multiple of X/2 difference changes by
(c) even integral multiple of 7J2 (a) (^i+l)r ib)
(d) cannot say (c)(|Ll+l)/? id) i\i-l)/t
9. For an aperture of 2 mm and wavelength of500 nm,
19. When diameter of objective of an astronomical
Fresnel distance is telescope is doubled, its limit of resolution is
(a) 5 m (/>)8m (a) doubled {b) quadrupled
(c) 10 m (d) 40 m (c) halved (d) unaffected
10. Polarising angle for a medium is 60°. Its refractive
20. Which one of the following cannot be polarized ?
index is (a) X-rays (b) radio waves
(a) 1-732 ib)\ (c) sound waves (d) light waves
21. Which one of the following phenomena confirms

w
(c) 1-414 id) 2
11. Colours appearing on a thin Him of a soap bubble that light waves are transverse ?
are due to (a) interference ib) diffraction

Flo
(a) dispersion ib) interference (c) dispersion id) polarization
(c) refraction id) dispersion
22. A ray of light falls on a transparent slab of |i = 1 -0.

ee
12. If reflected and refracted rays are mutually
The angle between pass axis of a polarizer and
perpendicular, what is the angle of incidence ?

Fr
analyser is 45°. The percentage of polarised light
passing through analyser is («)45° ib) 60°
(a) 100% ib) 50% (c) 30° (d)90°

for
ur
(c) 25% id) 75%
23. Wavelength of light used in an optical instrument
13. What is the ratio of slit widths when amplitudes of are X,j= 4000 A and %2 = ^^^O A, then ratio of
their respective resolving powers (corresponding
ks
light waves from them has a ratio 1: -V2 ? to wavelengths X,| and X^) is :
Yo
oo

(a) 16:25 ib)9:\


(a) (^) V2 :1
(c)4:5 id)5:4
eB

(c) 1 : 2 id)2:\ 24. Unpolarised beam of light of intensity Iq is inci


14. When a wave undergoes reflection at a denser dent on a polariser Pj. Another polariser P2 is held
ur

medium, its phase changes by parallel to it such that its pass axis is oriented at an
ad

(a)0° (b) 45° angle 60°, then what percentage of light will emerge
Yo

(c)90° (d) 180° from the system:


15. The ratio of intensities of two waves is 16 : 9. The (a) 30% ib) 100%
d

interference of two waves produce maximum and (c) 12-5% id) 37-5%
Re
in

minimum intensities ratio 25. In a Young’s double slit experiment, the separa
(a)5:7 ib) 25 :7 tion between the slits is 0-1 mm, the wavelength
F

(c)7:l id) 49: I of light used is 600 nm and the interference pat
16. Three lights, red, violet and green are used tern is observed on a screen 1 m away. Find the
successively in YDSE. Fringe width will be separation between bright fringes.
maximum for id) 6-6 mm ib) 6-0 mm
(fl) red ib) violet (c) 6 m id) 60 cm
(c) green id) same for all 26. In the Young’s double slit experiment, the fringe
17. In YDSE, the intensity of central maximum is /. pattern as seen on the screen is :
What will be the intensity at the same place when (a) Parabola ib) Hyperbola
one of the slits is closed ? (c) Ellipse id) Spiral
ia)I ib)I/2 27. The light source used in Young’s double slit ex
(c) 7/4 id)2I periment are
18. (a) Incoherent ib) Coherent
When a thin transparent plate of thickness t and
refractive index p, is placed in the path of one of (c) White light id) Blue-green-red light
10/50 “pKode^ ^ Fundamental Physics (XII) W»!Wll
28. What is the effect on the angular width of interfei - II. Assertion-Reason Type Questions
ence fringes in a Young's double slit experiment
when the screen is moved near to the plane of slits. Direction. For question numbers 37 to 45, two
statements are given, one labelled Assertion (A)
(a) Increases {b) Decreases
and the other labelled Reason (R). Select the
(c) Constant {d) Not defined
correct answer to these questions from the codes
29. The phase difference between two waves at the (a), (h), (c) and (d) as given below :
place of constructive interference is given as a (a) Both, A and R are true, and R is correct
(a) multiple of n (6) multiple of (2/i - 1) ;t explanation of A.
(c) even multiple of n (d) odd multiple of k (b) Both, A and R are true, but R is not the correct
30. The path difference between two waves at the place explanation of A.
of destructive interference is given by : (c) A is true, but R is false.
(a) integral multiple of X (d) A is false, and R is also false.
(b) integral multiple of X/2

w
37. Assertion. The colour of oil film on the surface of
(c) integral even multiple of X/2 water continuously changes.
(d) integral odd multiple of X/2 Reason. The colour of the light reflected by the

Flo
31. Resolving Power of Microscope depends upon oil film depends upon its thickness.
(a) Focal Length {b) Wavelength

reee
38. Assertion. We should have narrow sources to
(c) Diameter produce good interference fringes.

FFr
(d) Wavelength, Diameter of lens Reason. A narrow source is equivalent to a large
32. Angular width of interference fringe depends on number of sources which produce interference.
urr
39. Assertion. In interference, intensity of light at a
(a) Distance Between Slit and Screen
(b) Wavelength of light
(c) Ratio of the wavelength and Slit width
for
point changes with time periodically.
Reason. Light energy is propagated exactly in
kkss
(d) Width of Slit straight line.
Yo
oooo

40. Assertion. Soap bubble shows beautiful colours


33. Resolving Power of the telescope depends upon
the in sunlight.
eB

(a) Diameter of circular aperture Reason. Light waves reflected from upper and
lower surface of soap bubble interfere.
(b) Focal length
41. Assertion. The diffraction of light at a slit is more
r

(c) Magnification power


ou
ad

clearly visible when slit width is increased.


(d) Refractive index
Reason. On increasing the slit width the width of
YY

34. In the phenomena of Diffraction of light when the central maxima increases.
violet light is used in the experiment instead of
42. Assertion. Ultraviolet light is used to achieve high
nndd
Re

red light then


resolving power of microscope.
(a) Fringe width increases 1
Reason. Resolving power of microscope «
Fi

(b) No change in fridge width X


where X is the wavelength of light used.
(c) Fringe width decreases
43. Assertion. Out of microwaves and radiowaves, the
(d) Colour pattern is formed radiowaves show more diffraction.
35. A beam of light of intensity /j is incident on a Reason. Radiowaves have smaller wavelength as
tourmaline crystal and then on another tourmaline compared to microwaves.
crystal. The transmission axes of both crystals are
parallel to each other. If the intensity of light
44. Assertion. Through any medium, the light of all
wavelengths travels with the same speed (= 3 x
transmitted through the .second crystal is I2, then
10^ m/s).
(a) /, = I2 (b) /, = 2 /2
Reason. The velocity of light is too high to measure
(c)/|=4/2 (d)li=l2j2 small changes.
36. Colours appearing on a thin film of a soap bubble
are due to 45. Assertion. Light from any source of light is
naturally polarised.
(a) Diffraction (b) Refraction
Reason. Light gets unpolarised on account of
(c) Dispersion (d) Interference
scattering in the earth’s atmosphere.
WAVE OPTICS 10/51

ANSV\/ERS

I. Multiple Choice Questions


l.(c) 2. ia) 3. 4. ib) 5. ia) 6. ia) 7. (</) 8. (Z>) 9. (b) 10. ia)
11. (/>) 12. (/>) 13.ic) 14. 15. id) 16. 17.(f) 18. (fe) 19. (c) 20. (c)
21. id) 22. (a) 23. (t/) 24. ic) 25.ib) 26. ib) 27. ih) 28. (c) 29.(c) 30. id)
31. (./) 32. (c) 33. (a) 34. (c) 35. (/?) 36. (J)

II. Assertion-Reason Type Questions


37. ib) 38. (c) 39. id) 40. ia) 41. W) 42.ia) 43. ic) 44. UD 45. id)

lowow
HINTS/EXPLANATIONS For Difficult Questions

I. Multiple Choice Questions Taking refraclion at face XY, we have


sin /
1. The geometrical shape of the wavefront emerging sin/ sin i
= tan /
\^ = -

ee
from a convex lens will be converging spherical. sin/j sin(90°-/) cos/

Fr
XD
2. In YDSE, p = r FF V3 = tan / or / = 60”
d
8. At the place of destructive interference.
when d is increased, P decreases.

rer
Path difference = odd integral multiple of }J2.
3. In interference pattern, the maximum intensity is fofr Fo (2x1Q-3)-
u
(/j = It) and minimum intensity is (/| - I-,). So
9. Fresnel distance, Z = = 8 m
the contrast is best when /j= l2- X ~ 500x10-^
ks
YYouro
5. When distance between slits and screen (D) is very
10. p = tan ip - tan 60” = V3 = 1*732
oo

large, interference fringes become straight lines,


because increase in D increases p. 11, Colours on a thin film of soap bubble are due to
BBo ks

interference of light.
d^
r ee

6. Fresnel’s distance, Z = . . 12. According to Brewster’s law,


( 1 f /
ouru

7. RefcrtoFig. 10(Q).l, let for the angle of incident I = /q cos2 45” = I 0


ad

=50%/ 0
/, the reflected ray BC and refracted ray BD be 2
Yo

perpendicular to each other. Then


13. Given, ^ = I:V^
b
d
Re
iYn

\2
w
1 _ a 1 1
FFind

b 0 ●
^2 \

14. On reflection at a denser medium, phase changes by


180”.

1
15. Here, = If a and h are the amplitudes of
^2
two waves, then

/. a 16
or
a _4 ox b = 3al4
h b 9 ~b~3
I ia+h)-
(90”-r) + (90”-ri) = 90° Now,
max _

or
180”-(r+/-i) = 90”
I
mm
- {a-b)^
or
r + rj = 90” ia + 3al4)^ _ (7/4)^ _ 49
or
ri =(90”-r) = 90”-/ (v Z/= Zr) ia-3aJ4)~ ~ (1/4)^ “ 1

I
10/52
7^n4ideefr 4- Fundamental Physics (XII) VOL.II

30. At a place of desrtructive interference, path difference


16. As, P = , /.£?., P oe A,, and A, is maximum. between two waves is odd integral multiple of A/2.
d
fringe width will be maximum for red colour. 31. Resolving power of microscope depends upon
wavelength of light and diameter of lens.
17. When one of the slits is closed, amplitude is halved,
intensity becomes 7/4. p
32. Angular width, 0 = -!-
18. Here, geometrical path = r, optical path = p D~ d
Therefore,path difference Angular width of interference fringes depends on
= optical path - geometrical path = \it-t ratio of wavelength of light and slit width.
= (H-1)/ 33. Resolving power of telescope depends on diameter
D
19. When diameter of objective of an astronomical of circular aperture. As, R.P. =
telescope is doubled, its resolving power becomes 1-22 A,
2 times and limit of resolution is halved because
34. As wavelength of violet light is less than wavelength
D

ww
R.P of telescope = of red light, therefore, fringe width decreases, as
1-22X
XD
and limit of resolution = d0 =
1-22 A.
D
P=T- i.e., P oe A,.

35. Here, 0 = 0°,

Flo
20. Only transverse waves can be polarised. The sound
waves, being longitudinal waves can not be polarised. =-^COS^ 0 = -^COS^ 0=^x1 = —

ee
21. That light waves are transverse is confirmed by the 2 2 2 2 2

eer
phenomenon of polarization. or /j — 212

FFr
22. As reflected and refracted rays are mutually 36. Colours of a thin film of a soap bubble are due to
perpendicular, therefore, refer to hint question 7,

oorr
interference of light.
uur r
p. = tan / or 1 = tan / or / = 45"
II. Assertion-Reason lype Questions
s ff
23. We know that resolving power (JR) of an optical
instrument is inversely proportional to wavelength 37. Both Assertion and Reason are true but Reason
sk
(A.) of light used in an optical instrument, i.e., can not explain the Assertion. The oil film has
YYoo
ooko

R X non-uniform thickness on the surface of water.


R oc 1/A,. Therefore, — = = 5000 ^5 Light waves reflected from the upper and surface
4000 “4
eBB

«2 ^1 of film interfere. For different wavelenths of light,


24. Here, 0 = 60° the conditions of constructive and destructive
interference are satisfied at different positions. As
Intensity of polarised light from R^ =^.
uurr

2 a result of it, the colour of the oil film changes


ad

Light intensity emerging from the system. continuously.


Yo

38. Here Reason is false as a broad source is


fi V _ /''o
^l(cos 60" )2 =L 0 equivalent to a large number of sources which
dY

I =
2 2 8 produce interference. Narrow sources give fine
Re

beam of light which produce good interference


innd

/ 1 1
= -xl00% = 12-5%. fringes. Thus Assertion is true.
FFi

I
0
8 8 39. Here, both Assertion and Reason are false,
25. Here, d = 0 l mm = 10^ m because in interference, there is no change in the
A, = 600 nm = 6 X 10~^ m ; D = 1 m intensity of light at a point with time. The
propagation of light energy exactly in straight line
Separation between bright fringes = width of each
is just an approximation.
^ ^
dark fnnge = B = — =
„ AD 6x10-7x1
:
40. Both Assertion and Reason are true and Reason
^ d 10-4 is the correct explanation of Assertion, because,
= 6 X 10"^ m = 6*0 mm. for different wavelengths conditions for
constructive interference are satisfied at different
26. In F D S £■, the fringe pattern seen on the screen is
position. Due to it, the beautiful colours are seen.
hyperbola.
41. Here Reason is false as on increasing the slit
27. The light sources used in F D S £ are coherent. width, the width of central maxima decreases as
28. In FD S £, when screen is moved near to the plane of 1
the slits, angular width of fringes remains constant. width of central maximum oc . As a
slit width
29. At the place of constructive interference, phase result of it, the diffraction pattern will not be
difference between two waves is even multiple of n. clearly visible. Thus Assertion is also false.
WAVE OPTICS 10/53

42. Both Assertion and Reason are true and 44. Here both Assertion and Reason are false as
Reason is the correct explanation of Assertion. light of different wavelengths travel with different
43. Here Reason is false as radiowaves have smaller speeds in a medium and velocity of light is though
frequency or greater wavelength as compared to too high but can be measured accurately.
microwaves. Therefore, radiowaves undergo more 45. Here, both Assertion and Reason are false as
diffraction as the size of obstacles is of the order light from any source of light is unpolarised and
of wavelength of radiowaves, which is the it gets polarised due to scattering in the earth’s
requirement for diffraction. atmosphere.

PRdBLEMS

1. When monochromatic light travels from one I max would decrease, and
medium to another, its wavelength changes, /min would increase.

ww
but its frequency remains the same. Why ? The contrast between maxima and minima will
(CBSE 2011) decrease.

Sol. Frequency is the characteristic of the source. 5. Why does an excessively thin film appear

FF loo
That is why it remains the same. But wavelength black in reflected light ? (Hr. Board 2004)
is characteristic of the medium. So wavelength Sol. When t« X, path difference in light reflected

ree
and velocity both change. from two surfaces of thin film = 2 |i/ cos r + X/2
2. In YDSE, two slits are illuminated by two reduces to X/2, which is the condition for
light sources of same wavelength. Will you destructive interference. Hence the film appears

reFe
observe interference pattern ? black.

oroFr
r ur
[CBSE 2017 (C), 2008] 6. Show that maximum intensity in interference
s ff
Sol. No, because two independent sources of light pattern is four times the intensity due to each
cannot be coherent, even when light emitted by slit. Hence show that interference involves
k
only redistribution of energy.
YYouo
them is of same wavelength.
koso

3. Two identical coherent waves each of (Manipur Board 2012)


intensity Iq are producing interference Sol. Suppose the amplitude of waves from each slit
BBoo

pattern. What are the values of resultant is a. Therefore,intensity due to each slit = a~.
r ee

intensity at a point of (i) constructive When interference is destructive, the resultant


interference («) destructive interference ? amplitude = a - a = 0
ad
ouur

[CBSE 2004 (C)l Minimum intensity = 0


Yo

Sol. Here, I\= ^2~ h When interference is constructive, the re.sultant


amplitude = a + a = 2 a
^^1 ■'■^2 +2.^/j 12 cosil)
d

Maximum intensity = (2 «)“ = 4 = 4 times


Re
idnY

At a point of constructive interference, 0 = 0 the intensity due to each slit, which was to be
proved.
FFin

R = ;„+/„ + 2^cos0«=4/ 0
Also, average intensity in the interference
At a point of destructive interference, 0 = 180“ 0 + 4a^
pattern = = 2a^ = sum of intensities
■■■ '0 + 277^cosI80“ due to two slits.
2

= 2/o-2/o = 0 Hence we conclude that interference involves


4. In YDSE, one of two slits is covered by a
only redistribution of energy keeping the total
transparent paper which transmits only half energy fixed.
the light intensity. How will the intensity of
7. Light from two coherent .sources is reaching
maxima and minima change ?
a point where path difference for yellow light
(CBSE Sample Paper 2011) is 3 X/2. What will be the colour of the fringe
Sol. As is known, if a and b are amplitudes of two at that point ?
waves, then = {a + b)^, and = (a - b)^ Sol. If light is monochromatic, a dark fringe will be
When second slit is covered by a transparent formed at the point. However, if light is while,
paper b decreases. the fringe seen will be coloured. However, the

?
10/54
Fundamental Physics fXITIPTCTMn
yellow colour and iis neighbouring colours shall 13. If one of the two identical slits producing
be absent.
interference in Young’s experiment is covered
8. What is the difference in the origin of colours with glass, so that light intensity passing
of a soap bubble seen in sun light and the through it is reduced to 50%, find the ratio
colours emerging from a prism ? of the maximum and minimum intensity of
Sol. The colours of a soap bubble are seen due to the fringes in the interference pattern.
intet-ference of light. In the prism, colours appear (CBSE 2018)
due to dispersion of light.
Sol. To start with, b = a
9. Why is interference pattern not detected,
when the two coherent sources are far apart ? /2 = /i i.e. b^- = a^
(Hr. Board 2001) When one of the slits is covered with glass, so
1 that b^ = 50 a
2 = -
I 2
Sol. As fringe width —, therefore, when r/is so 100 2

ww
d

large, the width may reduce beyond the visible


b = ~
region. Hence the pattern will not be seen.
V2

FF loo
10. No interference pattern is detected when two
coherent sources are infinitely close to each I
{a-¥b)^ _ (fl + a/V2)^

ree
max
other. Why ? (CBSE 2003)
Sol. When d is negligibly small, fringe width [i which
J-
nun {a-bf- {a-al-Ji)^
is proportional to Hd may become too large.

reFe
Even a single fringe may occupy the field of _ (fl + 0-7Q7g)^ _ (l-707fl)^ = 3394
"(a-0-707a)2 “ (0-293a)2
oroFr
view on the screen. Hence the pattern cannot
rur
be detected.
s ff
14. Two coherent sources of intensity ratio a
11. Two independent light sources cannot
produce interference. Comment.
interfere. In interference pattern, what is the
k
YYouo
(HP Board 2014, 2015, CBSE 2014)
value of/„^ + ^min ^
okso

Sol. Two independent sources are not coherent (CBSE Sample Paper 2022-23)
BBoo

sources. That is why they cannot produce a


2

interference. For details, refer to the text. Sol. Here, —1. = a = —:r or or a = b-yfa
r ee

^2 b^
12. Find the Intensity at a point on a screen in
YDSE, where the interfering waves of equal ^max = =b^~(-J^ + 1)2
ad
ouur

intensity have a path difference of (i) X/4


Yo

Hi) m. (CBSE (F), 2017) /min = = (b^Ia - bf = b^ (Va -1)2


Sol. //; - ? /| - /2 - /q
. ^max ~ ^min _b^j^+ 1)^ -b^(^~ 1)2
d
idnY
Re

(0 path diff. = — ’ ^ +^min b^i^+l-f+b'^(-Ja-l)-


4 max
FFin

_ 4-Va _ 2>/a
phase diff. ^ = ~ = ^ ~ 2(a + l) " a + I
^2 cos (j) 15. A narrow slit is illuminated by a parallel
beam of monochromatic light of X. = 600 A
— /q + /q + 2.^/y /q cos 7C/2 = 2 /q and angular width of the central maximum
is measured. When the slit is next illuminated
(ii) Here, = ~ = 120° by light of wavelength X, the angular width
decreases by 30%. Calculate the value of
= /j -I- /2 + 2^/j 1^ cos (|) wavelength X.
(CBSE Sample Paper 2022-23)
= /q + /q + 2.^/q /^, cos 120°
0
Sol. Here, = 6000 A, X.2 = 2
1
= 2/.0 +210 2X.J 2% 1
2j=^o Angular width = —
d ’ d

K
WAVE OPTICS 10/55

2X,I 2X2 As both, X and d remain unchanged, angular


Decrease in angular width = — separation between fringes remains the same.
d d
21. In single slit diffraction pattern, why is
30 f2X'
intensity of secondary maximum less than the
100 w . intensity of central maximum ?
2X^ 2X, (CBSK 2(I(IV)

d d Sol. The central maximum is due to constructive


interference of secondary wavelets from all parts
Xj = 0-7 = 0-7 X 6000 = 4200 A

w
of the slit. With increase in n (order of
16. How is spacing between fringes in a double
spectrum), the wavelets from lesser and lesser
slit experiment affected if :
parts of the slit produce constructive
(0 the slit separation is increased, interference to form secondary maxima. That is

e
(ii) colour of light is changed from red to why the intensity decreaes.

roow
blue?

re
22. Why do we not encounter diH'raction effects
(/») the whole apparatus is submerged in oil of light in everyday observations ?
of refractive index 1*2 ? Justify your answer. (CBSE 2010)

reF
uFFll
(CBSE 2022) Sol. This is because objects around us are much

e
XD bigger in size compared to the wavelength of
Sol. Fringe width is given by p = visible light (=; 10“^ m).
d

sFr
23. Draw a graph showing the intensity distri
(0 When slit separation id) is increased, fringe

foro
bution of fringes due to diffraction at a single
width P decreases.
ofk
uor
slit. [CBSE 2018 (C)l
{ii) As A./, < X.^, therefore, p decreases i.e., fringe Sol. Refer to Fig. 10.21.
width decreases.
24. What is the relation between magnifying
kos
{Hi) When apparatus is immersed in oil, of ji =
Y
power and resolving power of a telescope ?
Yo
1-2 angular fringe width will decrease.
reeBB
oo

Sol. Magnifying power


Therefore, number of fringes will increase.
Resolving power of eye
uurY

1. Diffraction of light Resolving power of telescope


17. What is Fresnel’s distance ? (HP Board 2006) 25. Estimate the distance for which ray optics is
good approximation for an aperture of 4 mm
ad

Sol. Fresnel’s distance is the distance from a slit, at


doo

which diffraction spread of a beam becomes and wavelength 400 nm.


nY

equal to width of the slit. Sol. Ray optics is good approximation upto Fresnel
18. What is meant by angular resolution of a distance.
nid
Re

telescope ? [CBSE 2003 (C)]


Z _ (4xI0~^)^ 16xl0~^
F

Sol. Angular resolution of a telescope is the = 40 m


Fi

^ ~ X ~ 400x10“^ ~ 4x10"''
reciprocal of the smallest angular separation of
two distant point objects, whose images can just 26. For a single slit of width a, the first minimum
be resolved by the telescope. of diffraction pattern of a monochromatic
19. What is the essential condition for diffraction light of wavelength X occurs at an angle of
of light ? (CBSE 2011) Xia. At the same angle of X!a, we get a
maximum for two narrow silts separated by
Sol. Diffraction of light occurs only when size of
a distance a. Explain. (CBSE 2014)
obsiacle/aperture is of the order of wavelength
of light. Sol. In the interference pattern due to two narrow
20. In single slit diffraction pattern, how does the slits separated by distance a, path difference =
a sin 6. For constructive interference, a sin 0 =
angular separation between fringes change
when distance of screen from slit is doubled ? 1 A., sin 0 = 0 = Xia, we get a maximum. In the
(CBSE 2012, HP Board 2005) diffraction pattern due to a single slit, when path
difference from secondary waves from the end.s
P
Sol. Angular separation = — ^X of wavefront is A., then for every point in one
D d half of the wavefront, there is a corresponding
10/56 ‘P^adeefr'A Fundamental Physics rxiIH^»1Wll
point in the lower half for which path diff. Obviously, we shall prefer to use the first
between secondary waves is "hJ'l. Therefore, astronomical telescope.
interference is destructive and we obtain first
minimum at the same angle. II. Polarization of Light
27. Coloured spectrum is seen when we look 31. What is the angle between the plane of
through a muslin cloth. Why ? polariser and that of analyser to reduce the
Sol. The coloured spectrum is due to diffraction of transmitted light intensity to half ?
white light on passing through fine slits made
by fine threads in the muslin cloth.
28. Diameter of the objective (or aperture) of an
Sol. From Law of Malus, / = /q cos^ ^ “ "2
astronomical telescope is doubled. How does 1 I
COS^ 0 = — ; COS 9 = , e = 45-
it affect resolving power of the telescope and 2

w
intensity of the image ? 32. When a given light beam is passed through a
Sol. The resolving power is doubled Polaroid and poiaroid is rotated about the
D incident light as axis, what are the three

Flo
●: R.P. =
1-221 possibilities ?
Sol. (/) There may be no change in intensity of

ee
and intensity of image becomes 4 limes as area emergent light, the incident light is unpolarised,

Fr
of the objective lens becomes 4 times. (n) There may be change in intensity of emergent
29. The objective lenses of two telescopes have light with minimum not equal to zero. The
the same apertures but their focal lengths are incident light is partially polarised.
for
ur
in the ratio 1 : 2. Compare the resolving
(Hi) There may be change in intensity of light
powers of the two telescopes.
with minimum equal to zero. The incident light
ks
[CBSE 2017 (O)
is plane polarised.
Yo
Sol. Resolving power of a telescope is
oo

33. What happens on polarisation by reflection ?


D (Pb. Board 2007)
eB

PP =
\-22X Sol. The reflected light is completely plane
As aperture D is same and X. is same, therefore, polaroised containing vibrations of electric
ur

resolving powers of the two telescopes are vector perpendicular to the plane of incidence.
ad

same. Difference in their focal lengths does not The light refracted is partially polaroised. being
Yo

affect their resolving power. a mixture of polarised light and unpolarised


30. Two convex lenses of same focal length but light.
d

of aperture Aj and A2 (A2 < A|) are used as 34. Unpolarised light is passed through a
Re
in

the objective lenses in two astronomical Polaroid Pj. When this polarised beam passes
telescopes having identical eye pieces. What through another poiaroid and if the pa.ss
F

is the ratio of their resolving power ? Which axis of P2 makes an angle 6 with pass axis of
telescope will you prefer and why ? Give P|, then write the expression for the polarised
reason. (CBSE 2011) beam passing through Pj. Draw a plot
D showing the variation of intensity when 6
Sol. As is k .own, R.P = which varies varies from 0 to 2 7C. (CBSE 2017)
1-22X.’
Sol. If /() is intensity of unpolarised light, then
directly as aperture D of objective lens.
intensity of polarised light from Pj is
/,=/(/2
Therefore,
RP2 A2 When this is passed through another poiaroid
P2, intensity of polari.sed light from P2 is
As A2 < Aj or AI > A2, therefore, the first
astronomical tele.scope (with aperture Aj) has
I2 = /| cos^ 0
higher resolving power than the second When 9 is varied from 0 to 2 7i, we obtain I2
astronomical telescope (with aperture A2). versus 0 plot as shown in Fig. 10(Q).3.
WAVE OPTICS 10/57

35. Distiaguish between unpolarised light and of light pass through the crystal, which are
linearly polarised light. How does one get parallel to the axis. All other vibrations are
linearly polarised light with the help of a absorbed and we get linearly polarised tight. For
Polaroid ? (CBSE 2017) details, refer to Art. 10.28.
Sol. An unpolarised light consists of many
independent waves, whose planes of vibration III. Doppler’s Effect of light
are randomly oriented about the direction of 36. What is meant by blue shift ?
propagation as shown in Fig. 10(Q).2. Sol. When source of light and/or observer approach
FIGURE 10(Q).2 eachother, the apparent frequency of light
increases or apparent wavelength decreases.
This is called blue shift as wavelength of blue
colour is smaller.
>

37. The observed spectrum of a star shows red

w
shift. What do you conclude ?
Sol. Red shift in the spectrum of star indicates

Flo
increase in apparent wavelength or decrease in
Light is generally represented by electric field apparent frequency of light reaching the

e
observer. Therefore, the star is receding away

rree
—^

vector E, called light vector. from earth.

r FF
Linearly polarised light is that in which 38. Name any three applications of Doppler’s
vibrations of electric field are confined in one effect ?
uurr
direction in a plane perpendicular to the
direction of propagation of light. for
Sol. (/) Measuring the speeds of stars and galaxies.
(ii) Measuring speed of rotation of sun around
kss
When unpolarised light is passed through a its own axis.
tourmaline crystal cut with its face parallel to
ooook
Yo
(Hi) Measuring velocities of satellites,
the crystallographic axis, only those vibrations
aeroplanes, rockets and submarines.
eB

Very Short Answer


0 ● Short Answer
urr

● Long Answer
ad
Yo

VERY SHORT ANSWER QUESTIONS Carrying 1 mark


dY

1, A and B are two points on water surface 3. What is the geometrical shape of wavefront
Re
innd

where waves are generated. What is the phase emitted from a source in the form of narrow
Fi

dilf. if slit ? (Hr. Board 2002)


(/) A and B are on same wavefront separated Ans. Such a wavefront is cylindrical in shape.
by distance X. 4. A plane wavefront is incident normally on a
(h) A and B are on successive crests separated convex lens. Sketch the refracted wavefront

by distance 2 X. emerging from the lens. (CBSE 2016, 2009)


(}7() A and B are on successive troughs Ans. The refracted wavefront is spherical converging
separated by distance 3 X. wavefront as shown in Fig. 10(Q).3.
Ans. (0 Zero {ii) 2 k radian
(///) 2 7C radian.
2. What is a wavefront ?

ICBSE 2018. 2016, 2012, 2004 (C)]


Ans. A wavefront is the locus of all such panicles of
the medium, which are vibrating in the same
phase.
10/58 Fundamental Physics (XII)00190

5. What is the relation between path diff. and As c>>v, > > P„i
wavelength for constmetive interference of 13. Whena wave undergoesreflectionat a denser

rw
two waves ? (Hr. Board 2002) medium, what happens to its phase ?
Ans. Path diff. = « X, where n = 0, 1,2 Ans. Reflection of waves at a denser medium causes
i.e., path difference should either be zero or an a phase change of n or 180°.
integral multiple of wavelength. 14. In Young’s double slit experiment, what is the
6. What is the relation between path diff. and effect of following operations on interference
wavelength for destructive interference ? fringes ?
(Hr. Board 2003) (0 The screen is moved away from the plane

r
Ans. Path diff. - (2 n - 1) X/2, where n = 1,2, 3 of the slits.

wo
llou
i.e., path diff. should be an odd integral multiple (I'O The monochromatic source is replaced by

F
another monochromatic source of smaller
of half the wavelength of light.
7. What is the shape of Interference fringes in wavelength.
YDSE? (Hi) The monochromatic source is replaced

FF
s
Ans. The fringes are nearly straight line fringes by a source of white light. (CBSE 2018)
parallel to the slits. (iV) The width of the source slit is made wider
uro
ker
8. What is the ratio of slit widths when (v) The separation between the two slits is

e
increased,
amplitudes of light waves from them have

For
(vi) The distance between the source slit and

ro
ratio V3 : V2 ?
the plane of the two slits is increased,

fof
(vii) The width of each of the two slits is of
I

o
Y
w 3
a
Y
Ans. L = _L =
the order of wavelength of light source.
m'2 I2 ^ 2'
V /
(Meghalaya 2013, Uttarakhand Board 2012)
B
ks
9. What is the geometrical shape of the XD
Yo
wavefront when a plane wave passes through Ans. As fringe width is given by [3 = — , therefore
roo

d
a convex lens ? (CBSE 2009, 2008)
(i) fringe width will increase when D is
eB

Ans. When a plane wave passes through a convex


e

increased
u
rd

lens, the refracted wavefront is converging (ii) fringe width will decrease when X is
spherical wavefront. For details, refer to Art. decreased
o
n

10.7(b).
ou

(Hi) coloured fringes will be obtained when light


ad

10. Out of speed, frequency and wavelength, consists of different wavelengths


iY

name the parameter (5) which remain the However, the centre of the interference pattern
same on reflection.
will be white,
nd
F

Ans. All the three parameters remain the same on


Re

(iv) & (vi) Fringes are no longer distinct due to


reflection.
overlapping of the interference fringes,
Fi

11. Out of speed, frequency and wavelength,


(v) Fringe width will decrease when d is
name the parameter (s) which remain the increased.
same on refraction.
(vii) The single slit diffraction effects will
Ans. Only frequency remains the same on refraction. superimpose the interference pattern.

12. Out of electric Held vector, E and magnetic 15. What type of wavefront will emerge from (i)
a point source and (21) distant light source ?
Held vector, B in an electromagnetic wave, (CBSE 2009)
which is more effective and why ?
Ans. (i) From a point source, the wavefront is
—»

Ans. E is more effective than B ■ This is because diverging spherical wavefront.


when a charge q moving with a velocity v (i7) From a distant light source, the wavefront
encounters an e.m. wave, is plane wavefront.
16. The refractive index of glass is 1*5 for light
E
_ qE E _c waves of X = 6000 A in vacuum. Calculate
B
Fm qvB vB V their wave length in glass.
WAVE OPTICS 10/59

c vX, Ans. Yes, the statement is true. Secondary wavelets


Ans. As It = — from unobstructed part of incident wavefront
V \X
g
interfere to produce diffraction pattern.
25. What should be the order of size of obstacle/
.-. X
K 600C = 4000A^
aperture for diffraction of light ?
^ It 1-5
Ans. Size of obstacle/aperture should be of the same
17. What will be the effect on the fringes, if order as that of wavelength of light.
Young’s double slit experiment set up is
immersed in water ? (CBSE 2010) 26. How are resolving power and limit of
resolution of an optical instrument related ?
Ans. The fringes become narrower
(HP Board 2001)
p' = p/p.
Ans. Smaller is the limit of resolution, higher is the

oww
18. Bubbles of colourless soap solution appear
resolving power.
coloured in sun light. Why ?
27. How does the resolving power of telescope
Ans. This is due to interference of light. change when the aperture of the objective is
19. Differentiate between a ray and a wavefront. increased ? (CBSE 2007)
{CBSE 2009)
Ans. The resolving power of telescope increases on

e
Ans. A ray is a .straight line path along which light

ree
increasing the aperture of objective lens (Infact,

rFl
travels. A wavefront is continuous locus of all

Fre
R.P. « D).
the particles of a medium, which are vibrating

rr F
in the same phase. II. Polarization of Light
20. The phase difference between two waves 28. Can electromagneticwaves be polarised ?
ouur
reaching a point is idl. What is the resultant
amplitude, if the individual amplitudes are
3 mm and 4 mm ?
sfoo
Ans. Yes, because electromagnetic waves are
transverse in nature.
kks
29. Is blue light from the sky polarised ?
Yo
oooo

Ans. R = +b~ + lab cos k/2 = +4^ Ans. Yes, it is polarised.


30. Unpolarised light of intensity I is passed
eBB

= 5 mm
through a polaroid. What is the intensity of
21. How would the angular separation of
emerging polarised light ?
interference fringes in Young’s double slit
uurr

ICBSE 2018 (C), 2009]


experiment change when the distance of
ad

separation between slits and screen is Ans. Intensity of polarised light = 7/2.
YYo

doubled ? (CBSE 2012, 2009) 31. Unpolariscd light is incident on plane surface
of a medium of refractive index p at an angle
P ^X
dd

Ans. Angular separation, 9 = — i. If the reflected light is completely polarised,


Re

D~ d
iinn

how are p and i related ? Name the Law.


It does not depend upon D, the distance of Ans. p = tan i. This is Brewster’s Law.
F

separation between slits and screen. Therefore,


6 remains unaffected.
32. Will ultrasonic waves show any polarisation ?
Ans. No, as they are longitudinal in nature,
I. Diffraction of light 33. Which field vector is used to represent the
polarisation of an e.m. wave ?
22. In diffraction due to a single slit, what is the Ans. Electric field vector.
condition for first minimum ?
Ans. If a is width of slit, then the condition for first
34. In a plane polarised light, name three para*
minimum is (7 sin 0 = X.
meters which are mutually perpendicular.
(ICSE 2002)
23. How is resolving power of an optical
Ans. Plane of vibration, plane of polarisation and
instrument related to wavelength of light ?
direction of propagation of e.m. wave.
Ans. The resolving power of an optical instrument
35. What is the value of refractive index of a
varies inversely as the wavelength of light used.
24. Diffraction is interference between different medium of polarizing angle 60° ?
parts of the same wavefront. Comment. Ans . p = tan jp = tan 60° = ^ .
10/60 “PnAdee^'A Fundamental Physics (XII)CSI9D
36. Is head light of a car or an air plane polarized ? c c c
From ji = — or ?; = —
Ans. No, it is unpolarized. V
tan” p
37. Name two commonly used devices which use 3 X10^ m/s
polarized light. = ,3 xlO^ms
tan 30'^
Ans. Sunglasses (dark glasses) and Liquid crystal
45. Which phenomenon leads us to the conclu
display (LCD).
sion that light is transverse in nature ?
38. Give three examples of laevo rotatory
substances. Ans. Polarization of light.
46. What is the angle between the reflected and
Ans. (/) Nicotine (//) Turpentine {Hi) Fruit sugar
refracted rays at polarizing angle ?
39. A ray of light falls on a transparent slab of p. Ans. 90°.
- 1*732. If reflected and refracted rays are
47. Sound waves can not be polarized. Why ?
mutually perpendicular, what is the angle of
(CBSE 2013)
incidence ? (CBSE 20091

w
Ans. Only transverse waves can be polarized. Since
Ans. 60°, using tan = p = 1-732 = ■y/3 sound waves are longitudinal, they cannot be
40. How does one demonstrate using a suitable polarized.

Flo
diagram, that unpolarized light, when passed 48. If the angle between the pass axis of polarizer
through a Polaroid gets polarized ? and analyser is 45°, write the ratio of the

ee
Ans. Refer to Fig. 10.30. (CBSE 2014) intensities of original light and the trans
41. Which among X-rays, sound waves and radio mitted light after passing through analyser.

Fr
waves can be polarized ? (Hr. Board 2007) tCBSE 2009)

Ans. X-rays and radio waves can be polarised because Ans. From Law of Malus, / = /q cos" 0

for
ur
they are transverse waves. /
0 1 1 1

42. Unpolarizcd light is incident on a plane


surface of glass of refractive index p at angle
/ cos^ 0 (cos 45°)^ (1/V2)2 ^
ks
i. If the reflected light gets totally polarized, III. Doppler’s Effect in light
Yo
oo

write the relation between the angle i and


refractive index p. (CBSE 2009) 49. Does the speed of light in vacuum depend
eB

Ans. According to Brewster’s Law, p = tan = tan i upon relative motion between source and
observer ?
43. Define the term ‘linearly polarized light’.
(CBSE 2009) Ans. No, speed of light in vacuum is an absolute
r
ou

constant.
ad

Ans. Linearly polarized light is that in which


vibrations of electric field are confined in one 50. What does red shift in spectra of galaxies
YY

indicate ?
direction in a plane perpendicular to the
direction of propagation of light. Ans. The universe is expanding.
nd
Re

44. What is the speed of light in a denser medium 51. What is the basis of measurement of plasma
of polarising angle 30° ? (CBSE 2019) temperatures in thermonuclear reactions ?
Fi

Ans. Here, i; = ? = 30° Ans. Doppler’s effect in Light.

SHORT ANSWER QUESTIONS Carrying 2 marks

1. What are the essential conditions for two light 3. What happens to light energy when light
waves to be coherent ? (Hr. Board 2004) waves interfere destructively at a point ?
Ans. (/) light waves must have same frequency/ Ans. Energy is not lost. It gets transferred from
wavelengths regions of destructive interference to the regions
of constructive interference.
(//) they should have zero or constant phase diff.
4. In YDSE, when the entire apparatus is immer
2. What happens to the interference pattern if sed in water, what happens to fringe width ?
phase difference between two light sources
(CBSE 2009, 2012)
Ans. Fringe width decreases to p/|i, where p is
varies continuously ? refractive index of water.
Ans. The positions of bright and dark fringes would 5. Why does a soap bubble show beautiful
change rapidly. The interference pattern shall colours, when illuminated by white light ?
not be sustained. (Pb. Board 2004)
WAVE OPTICS 10/61

Ans. Light waves reflected from outer and inner (//) When distance (d) between them becomes
surfaces of soap bubble interfere. For different too large, fringe width (3 becomes too small and
wavelengths, conditions for constructive may no longer be in the visible region. Hence
interference are satisfied at different positions. no interference pattern will be observed.
That is why beautiful colours are seen. 10. Explain the statement. Might added to light
6. Do interference effects occur for sound can produce darkness.’ (CBSE 2001)
waves ? Recall that sound is a longitudinal Ans. When light waves from two coherent sources
mechanical wave while light is transverse and superimpose such that at any particular point,
non-mechanical ? crest of one wave falls on trough of the other
Ans. Yes, because interference is a wave and trough falls on crest, the amplitude of the
phenomenon, which takes place for waves resultant wave is zero. Hence resultant intensity
which may be longitudinal or transverse ; is zero. This is the phenomenon of destructive
mechanical or non- mechanical. Waves should interference. Thus light added to light produces

ww
darkness.
be of same type and coherent.
7. Oil floating on water looks coloured due to
I. Dil^raction of light
interference of light. What should be the
approximate thickness of the Him for such

Flo
11. A diffraction grating has 5000 lines per cm.
effects to be visible ? What is the grating element ?

e
Ans. For interference effect to be visible, thickness I

rere
of oil film must be of the order of wavelength Ans. Grating element = = 2x 10-^ cm
5000
of visible light, which varies from 4000 A to

r FF
8000 A. 12. How can we increase the resolving power of
a microscope ?
uurr
8. Mark the statement true or false :
(a) In Young’s double slit expt, performed
with a source of white light, only black and
for
Ans. (/) By using light of smaller wavelength.
(//) By increasing refractive index of the medium
kss
white fringes are observed. between object and objective lens of
microscope.
ooook

(b) Two slits in Young’s double slit expt are


Yo

illuminated by two different sodium lamps 13. How does the angular width of principal
maximum in the diffraction pattern vary with
eBB

emitting light of same wavelength. No the width of slit ?


interference pattern will be obtained.
(CBSE 2014) Ans. Angular width of principal maximum
urr

21
Ans. (a) False. With a source of white light central
ad

= 20 =
maximum is white. The fringes are coloured i.e.,
Yo

bright bands are coloured and dark bands are Clearly it is inversely proportional to width (a)
dY

dark. The centre of pattern is white. of the slit.


Re

(b) Tnie. Two different sodium lamps cannot 14. Diffraction is common in sound but not
innd

be coherenteven when they emit light of same common in light waves. Why ?
Fi

wavelength. Therefore, no interference pattern (Hr. Board 2011)


will be obtained.
Ans. For diffraction of a wave, an obstacle or aperture
9. What are coherent sources of light ? Why no of the size of wavelength of the wave is needed.
interference pattern is observed when two As wavelength of light is of the order of 1 m
coherent sources are {/) too close (ii) very far and obstacle/aperture of this size are rare,
apart ? (Jharkhand Board 2012)
therefore, diffraction is not common in light
Ans. The sources of light which emit light waves of waves. On the contrary, wavelength of sound is
same wavelength,same frequency and in same of the order of 1 m and obstacle/apertureof this
phase or having a constant phase difference are size are readily available, therefore diffraction
called coherent sources. is common in sound.
(/) When these sources are too close, distance 15. How does the resolving power of a micro
{d) between them tends to zero. Fringe width scope change on
XD (i) decreasing wavelength of light
tends to infinity. Hence no
(ii) decreasing diameter of objective lens ?
interference pattern will be observed. (CBSE 2010)
10/62 Fundamental Physics (XII)EZS15Q

Ans. On decreasing A., resolving power of microscope 22. What is a polaroid ?


increases and on decreasing diameter of Ans. A Polaroid is a device that produces an intense
objective lens, resolving power of microscope beam of plane polarised light.
decreases.
23. Name two commonly used devices which
16. In a single slit diffraction experiment, the make use of polaroids.
width of the slit is made double the original
width. How does this affect the size and
Ans. (i) Sun glasses
intensity of the central difTraction band ? (») Liquid crystal Displays (LCD).
(CBSE 2012) 24. A partially plane polarised beam of light is
passed through a polaroid. Show graphically
2DX
Ans. Width of central maximum is 2x = the variation of transmitted light with angle
a
of rotation of polaroid.
when width of slit {a) is doubled, central max. (CBSE Sample Paper 2012)

looww
width is halved. Its area becomes l/4th. Hence
intensity of central diffraction band becomes 4 Ans. According to Law of Malus, Ip = Iq cos^ 0
times. As the beam is originally partially polarised,
17. When light travels from a rarer to denser therefore resultant intensity of transmitted light
IS / = ,/ + L0 cos^ 6.
medium, it loses some speed. Does the unpol
The variation of transmitted light with angle of

ree
reduction in speed imply a reduction in the
energy carried by the light wave ? rotation of polaroid is shown in Fig. 10(Q).4.

ree F
(CBSE 2013, 2010)
Ans. We know that total average energy density
associated with an electromagnetic wave (i.e.
r FF
b}0 fofroF
u
i1 r’’’
light wave) is u flV ““^0^0
ks
2)1 0 2

This shows that depends upon Bq and Eq


kos o
YYouor

{i.e. the amplitude of magnetic vector and


BBoo

electric vector), u^^, is independent of the speed


r ee

of wave propagation. Therefore, the wave may


lose speed, but not the energy.
ouru

25. When an object is seen through a calcite


ad

II. Polarization of Light crytal, we observe two images. Why ?


Yo

18. What is meant by circularlypolarised light ? Ans. The two images are seen on account of the
Ans. A light wave is said to be circularly polarised if
phenomenon of double refraction by the calcite
d

crystal.
Re

the tip of the vibrating electric field vector traces


inY

a circle. 26. If the polarising angle for air glass interface


FFind

is 56*3", what is the angle of refraction in


19. What is elliptically polarised light ?
glass ?
Ans. A light wave is said to be elliptically polarised
when the tip of the vibrating electric field vector
Ans. As is known, /^ + = 90°
56-3° = 33*7‘
traces an ellipse.
27. At what angle of incidence should a light
20. What is polarising angle ?
Ans. It is the angle of incidence of unpolarised light beam strike a glass slab of p » -Js, such that
on a transparent refracting medium, corres reflected and refracted rays are perpendi
ponding to which, reflected light is plane cular to eachother ?

polarised completely. Ans. The reflected and refracted rays will be


21. What is the value of refractive index of a perpendicular when i = i P'
medium of polarising angle 60° ? where
tan ip = |i = V3 .-. ip = 60“
(CBSE 2000)
28. For a given medium, the polarising angle is
Ans. According to Brewster’s law, 60“. What will be the critical angle for this
|i = tan ip = tan 60° =-j3 = 1-732
medium ?
WAVE OPTICS 10/63

Ans. Here, = 60°, C = ? 4 cos^ 9 sin^ 6 = 1


(2 sin 0 cos 6)2 = 1 or (sin 2 6)2 = I
\i = tan ip = tan 60° = ^ sin 2 0 = 1 = sin 90°
. ^ 1 1
sm C = — = —^ = 0-5773, 90°
^ s 9 = = 45'
2
C = sin“‘ (0-5773) = 35° 16'
29. Two polaroids A and B are kept in crossed IIL Doppler’s Effect in light
position. How should a third polaroid C be
placed between them so that the intensity of 30. What is the speed of star in the line of sight if
polarized light transmitted by polaroid B a spectral line shifts towards longer
wavelength side by 0*032% ?
1
reduces to —th of the intensity of AX 0-032
8 Ans. V = - xc = - X 3 X 10^ m/s

ww
unpolarised light incident on A ? X 100

(CBSE 2012) = - 9*6 X 10** m/s


Ans. Let the third polaroid C be kept at Z6 with the Negative sign of v indicates the star is moving
axis of Polaroid A. away from earth.

Flo
Intensity of polarised light from C 31. The radial speed of a galaxy is 1*2 x 10^ m/s

ee
/[ = /q cos2 0, where Iq is intensity of polarised receding away from earth. What is the
percentage change in wavelength in the

rere
light from A ...(/)

rFF
Angle between axis of B and C = (90° - 0) observed spectrum ?
Ans. Here, v = 1-2 x 10^ m/s
Intensity of polarised light from B
uurr
Ij = /j cos2 (90° - 9) = /[ sin2 6 1-2 xlO*^

Using (/), /t = Iq cos2 0 sin2 0 =


21
0
(given)
As v =
foor
X
X c
"
AX

X
V

c 3xl0»
ks s
= 0-4x 10-2
Yoo
Note that 2 Iq is intensity of unpolarised light Percentage increase in wavelength
oook

incident on A = 0-4x 10-2 X 100 = 0*4%


eBB

SHORT ANSWER QUESTIONS Carrying 3 marks


uurr

1. What is a wavefront ? State its relation with ray experiment with the diffraction pattern obtained
ad

of light. due to a single slit. [CBSE 2018 (C), 2017]


Yo

(Uttarakhand Board 2012) [Art. 10.2] [Art. 10.20]


2. State the conditions which must be satisfied for
dY

6. Distinguish between interference and diffraction


two light sources to be coherent of light. [Art. 10.20]
Re
innd

(CBSE 2008) [Art. 10.10] 7. Doppler’s effect in light is symmetrical but the
3. State and explain Huygens principle. Name the same effect in sound is asymmetrical. Explain.
FFi

type of wave front that corresponds to a beam [Art. 10.26]


of light. (CBSE 2016) 8. Write two points of difference between
(/) coming from a convex lens when point interference and diffraction patterns of light.
source is placed at its focus, [Art. 10.20]
(//) coming from a very far off source, 9. Define limit of re.solution of a microscope. Give
(Hi) diverging radially from a point source. the expression for it.
[Art. 10.2] (Karnataka Board 2012) [Art. 10.23]
4. What is meant by the term ‘interference of 10. What is the phenomenon of polarization ?
light’ ? Write any two conditions necessary for Derive the relation connecting the polarising
obtaining well defined and sustained angle of a medium and its refractive index.
interferencepattern of light. (Hr. Board 2014, 2012, MP Board 2011)
[Arts. 10.11 and 10.16] [Arts. 10.28 and 34]
5. Explain two features to distinguish between the 11. Define plane of polarisation and plane of
interfrence pattern in Young’s double slit vibration. (Raj. Board 2011) [Art. 10.28]
10/64 ^>Mdeefr'A Fundamental Physics (XlI)BiS15D

LONG ANSWER QUESTIONS Carrying 5 or more marks

1. What is meant by wavefront ? Explain three 12. What do you understand by fringe width ?
types of wave front. (CBSE 2019) [Art. 10.2] Derive an expression for fringe width in the
2. State Huygens principle and prove the laws of interference pattern.
reflection on the basis of wave theory. (Chhatisgarh Board 2012, Hr. Board 2012,
Bihar Board 2012, Jharkhand Board 2011,
(CBSE 2019, 2018, Jharkhand 2006)
CBSE2011) [Art. 10.15]
[Arts. 10.3 and 10.4]
3. Prove Snell’s laws of refraction on the basis of
13. Explain what is meant by diffraction of light.
Describe a simple experiment to demonstrate
Huygens principle. ‘CBSE 2013, 2011,
diffraction at a single slit. [Art. 10.18]
Uttarakhand Board 2012. K Board 2012,
14. In a single slit diffraction pattern, how is the
2011, Raj. Board 2011) [Art. 10.5] angular width of central bright maximum

w
4. Based on Huygens principle,draw the incident changed when (/) the slit width is decreased.
and refracted wavefronts AB and CD when light (it) the distance between the slit and screen is

Flo
travelling from air to a denser medium incident increased.
at an angle of 45° is refracted at an angle of 30°.
(Hi) Light of smaller wavelength is used. Justify

e
(CBSE 2019, Manipur Board 2012)

rree
your answer.
[Art. 10.5]
(Raj. Board 2012 ; J & K Board 2012, 2011 ;

r FF
5. A plane wavefront is incident on (a) a prism CBSE 2010) [Art. 10.19}
(b) a convex lens (c) a concave mirror. Draw 15. State three characteristic features which
uurr
the shapes of the refracted/reflected wavefront
in each case. [Art. 10.7] for
distinguish the interference pattern due to two
coherently illuminated sources as compared to
kss
6. Explain superposition principle. Does it apply that observed in a diffraction pattern due to a
to electromagnetic waves ? [Art. 10.8]
ooook

single slit. (CBSE 2007) [Art. 10.20]


Yo

7. What are coherent sources of light ? Why are 16. Draw the intensity pattern for single slit
coherent sources required to obtain sustained
eB

diffraction and double slit interference. Hence


interference pattern ? slate two differences between interference and
(CBSE 2007) [Art. 10. 10] diffraction patterns. (CBSE 2019, 2017)
urr

8. What is meant by interference of light ? Describe [Arts. 10.15 and 10.20]


ad

briefly Young’s double slit experiment to


Yo

17. State the essential condition for diffraction of


demonstrate interference of light. light to take place. Use Huygens principle to
dY

(J & K Board 2012. Jharkhand Board 2011) explain diffraction of light due to a narrow
Re

single slit and the formation of a pattern of


innd

[Arts. 10.11 and 10.12]


9. Derive the conditions for constructive and fringes obtained on the screen. Sketch the
Fi

destructive interference. pattern of fringes formed due to diffraction at a


single slit showing variation of intensity with
(CBSE 2012, Karnataka Board 2012)
angle 0 .
[Art. 10.13]
(Manipur Board 2011, Raj. Board 2011,
10. Light waves each of amplitude and frequency CBSE 2007) [Art. 10.19]
to emanating from two coherent light sources
18. A monochromatic light of wavelength X is
superpose at a point. If the displacements due
incident normally on a narrow slit of width a to
to these waves are given by yj = cos tor and
produce a diffraction pattern on the screen
yo = a cos (tor + ^), where (]) is the phase
difference between the two, obtain the placed at a distance D from the slit. With the
help of a relevant diagram, deduce the condition
expression for the resultant intensity at the point. for maxima and minima on the screen. Use these
(CBSE 2019. 2014) [Art. 10.13]
conditions to show that angular width of central
11. Prove that the law of conservation of energy is maximum is twice the angular width of
obeyed during interference of light. secondary maximum. (CBSE (F), 2017)
[Art. 10.14] [Art. 10.19]
WAVE OPTICS 10/65

[Hints. From Fig. 10.20 representing diffraction 25. Describe an experiment to demonstrate
pattern of a single slit, we find that central transverse nature of light.
maximum extends from path diff. - A. to path (J & K Board 2012. 2011, CBSE 2011) [Art. 10.29]
diff. + X. And secondary maxima around central 26. Can a naked eye detect polarization of light ? If
maximum range from path difference X, to 2
not, how is polarization of light detected ?
Obviously angular width of secondary
[Art. 10.31}
maximum is half the angular width of central
maximum.] 27. Describe briefly the construction and working
of a nicol prism. [Art. 10.30]
19. Define resolving power of a compound
microscope. How does the resolving power of 28. State and explain law of Malus.
a compound microscope change when (CBSE 20161 [Art. 10.32]
(/) refractive index of the medium between the 29. Explain polarisation (/) by scattering (//) by
object and objective lens increases ? reOection. [Art.s. 10.33 and 34]

w
{»■) Wavelength of the radiation used is 30. Stale and explain Brewster’s law of polarization.
increased ? (CBSE 2007) [Art. 10.23] (CBSE Sample Paper 2019-20, Karnataka Board

Flo
20. Explain the concept of resolving power. Briefly 2012, Manipur Board 2011,
discuss resolving power of a microscope and a J & K Board 2014, 2011) [Art. 10.35]

reeee
telescope. 31. A beam of unpolarized light is incident on a
(Uttarakhand Board 2012) [Arts 10.22. 23 and 24}

Fr
glass air interface. Show, using a suitable ray
21. Slate Doppler’s effect in light. Derive an diagram that light reflected from the interface
expression for the apparent frequency of light is totally polarized when |a = tan /g, where p is

for
refractive index of glass with respect lo air and
ur
waves when source and observer move relative
to each other. Mention the applications of is Brewster’s angle. (CBSE 2014)
kkss
Doppler’s effect in light. 32. What is an unpolarized light ? Explain with the
help of suitable ray diagram how an unpolarized
Yo
[Art-s. 10.25 and 10.27]
oo

22. What do you understand by polarization of light can be polarized by reflection from a
transparent medium. Write expression for
eB

light ? What arc plane of polarisation and plane


of vibration ? (Hr. Board 2012) [Art. 10.28] Brewster’s angle. (Chhatisgarh Board 2012,
CBSE 2012, 2011, 2010) [Arts. 10.34 & 35]
23. What is meant by plane polarized light ? What
r

33. What are polaroids ? Mention some of their


ou

type of waves show this property ? Describe a


ad

method for producing a beam of plane polarized practical uses,


YY

light. (CBSE 2012, HP Board 2011) (Bihar Board 2012, MF Board 2011.
[Art. 10.28] HP Board, 2013. 2011 ■ [Art. 10.36]
ndd
Re

24. Differentiate between polarised and unpolarizcd 34. Define polarising angle. How is it related with
light. How are they represented ? critical angle ?
Fi

(J & K Board 2012) [Arts. 10.28 & 10.30] (HP Board 2013) [Arts. 10.34]

CASE-BASED VERY SHORT/SHORT QUESTIONS

CASE 1. Wave optics is based on wave theory cylindrical. At very large distance from the source, a
of light put forward by Huyghen and modifiedlater by portion of spherical wavefront appears to be plane. A
Fresnel. According to this theory, light is a form of wavefront travels parallel to itself and perpendicular to
energy, which travels through a medium in the form of the rays.
tran.sverse waves.
According to Huyghen’s principle of geometrical
A wavefront is defined as the locus of all the construction of a wavefront.
particles of a medium, which arc vibrating in the same (/) Every point on given wavefront (called
phase. For a point source of light, wavefront is primary wavefront) acts as a fresh source of new
spherical. For a line source, the wavefront is disturbance, called secondary wavelets.
10/66 ‘pftadccfi. 'a Fundamental Physics (XII) WSTWl
(//) The secondary wavelets travel in all the Condition for constructive interference is path
directions with the speed of light in the medium. difference between two waves = zero or integral multi
(Hi) A surface touching these secondary wavelets ple of full wavelength. In that event, crest of one wave
tangentially in the forward direction at any instant gives would fall on the crest of the other and trough would
us the new secondary wavefront at that instant. fall on trough. The resultant amplitude = (o + b).
Using Huyghen’s principle, we can prove the laws When path difference between two waves is an
of reflection and the laws of refraction on the basis of odd integral multiple of half the wavelength, crest of
wave theory. one wave would fall on trough of the other and vice-
Read the above paragraph carefully and versa. The resultant amplitude = (a - h). Therefore,
answer the following very short and short /
max _ (a + W-
answer questions : r.
mm
{a-b9
1. What is the shape of wavefront for a point

ww
source ? Read the above paragraph carefully and
answer the following very short and short
2. For a line source of light, what is the shape of
answer questions :
wave front ?

FF loo
5. Does superposition principle apply only to
3. How does a wavefront travel ?
mechanical waves or e.m. waves ?
4. What is Huyghen’s principle of geometrical

ree
6. What is meant by interference of light ?
construction of wave front ?
7. How is intensity of light related to the amplitude

rFee
CASE 2. According to Super position principle, of light wave ?
when two or more wave motions travelling through a

oor rF
8. What are the conditions for constructive and
rur
medium, superimpo.se on one another, they lose their destructive interference ?
s ff
individual identity. A new wave is formed in which
CASE 3. In interference experiment, let
resultant displacement (y) at any instant is equal to
k
d = distance between two slits,
vector sum of the displacements due to individual waves
YYoou
ookos

at that instant, i.e., D = distance of screen from the slits,


X = distance of a point on screen from
BBo

y = yi+y2 +>●?
the centre of interference pattern.
re

This principle applies to both, mechanical waves Path difference between two waves arriving at the
and electromagnetic waves. xd
ouur
ad

given point on screen = —


Interference of light is the phenomenon of
Yo

D
redistribution of light energy on account of xd
superimposition of light waves from two coherent For nth bright fringe, ^— ~nX, where n = 0 for
dY
Re

sources. At the points where resultant intensity is central bright fringe and for nth dark fringe,
idn

maximum, interference is said, to be constructive. At xd X


FFin

the points, where resultant intensity is minimum, = (2n-D-


D
interference is said to be destructive.
Width of each bright/dark interference fringe is
Thomas Young was the first to demonstrate the
XD
phenomenon of interference of light. Using two slits
illuminated by monochromatic light source, he obtained
bright and dark bands of equal width placed alternately. When source gives white light, interference
These were called interference fringes. All bright fringes are coloured, but centre of pattern is white. As
fringes have same intensity and all dark fringes are X^ > Xy, therefore red fringes are wider compared to
perfectly dark. violet fringes.
Two sources which emit continuous light waves
If M’j, W2 are widths of two slits ; /|, I2 are
intensities of light from two slits, a, b are the amplitudes of same amplitude, same wavelength/frequency in same
of light from these slits, then pha.se or having a constant phase difference are called
coherent sources. Two independent sources can never
w I a~ be coherent. Coherent sources are usually object image
L = _I =
vv., b^ sources.
WAVE OPTICS 10/67

Read the above paragraph carefully and 11. How is interference pattern affected when we use
answer the following very short and short a source of white light instead of monochromatic
answer questions : light ?
9. How is fringe width in interference pattern 12. Light of wavelength 500 nm is incident on
affected when distance of screen is doubled ? two slits separated by 1 mm. What is the width
10. Can two independent sources of light be of each dark band on the screen 1 m away from
coherent ? slits ?
ANSWERS

1. For a point source of light, wavefront is spherical. path difference between two waves = odd integral
2. For a line source of light, wavefront is cylindrical. multiple of half the wavelength.
3. A wavefront travels parallel to itself and XD
perpendicular to the rays.
9. As B = — , therefore, when D is doubled,
^ cl

w
4. (i) Every point on given wavefront acts as a fringe width P becomes double.
source of new disturbance, called secondary
wavelets.
10. No, two independentsources of light can not be

Flo
coherent sources because in that case, the two
(ii) The secondary wavelets travel in all the waves of light coming from those two sources of
directions with the speed of light in that medium.

e
light, while reaching at a point on screen will

rree
(Hi) At any instant, a surface touching the not have constant phase difference with time. Due

r FF
secondary wavelets tangentially in the forward to it, the condition of constructive and destructive
direction gives us the new secondary wavefront interference will be changing with lime, at that
at that instant.
uurr
point, consequently interference pattern will
5. Superposition principle applies to both,
mechanical waves and e.m. waves.
for
disappear from screen.
II. Instead of monochromatic light, when we use a
kss
6. Interference of light is the phenomenon of source of white light, the fringes become
ooook
Yo
redistribution of light energy on account of coloured. However, the centre of pattern becomes
superimposition of light waves from two coherent white.
eB

sources.
12. Here, X = 500 nm = 500 x 10-^ m = 5 x 10"^ m
7. Intensity (/) of light is directly proportional to d = 1 mm = 10“^ m, D = 1 m, P = ?
urr

square of the amplitude of the wave.


XD 5xl0"^xl
ad

8. For constructive interference, path difference


Yo

10-3
between two waves = zero or integral multiple
dY

of full wavelength. For destructive interference. = 5 X 10"^ m = 0*5 mm


Re
innd

CASE-BASED MCQs AND ASSERTION-REASON QUESTIONS


Fi

CASE 1. Diffraction of light is the phenomenon Note that in interference, all bright bands have
of bending of light around comers of an obstacle or same intensity. In diffraction, successive bright bands
aperture in the path of light. The diffraction pattern due are of decreasing intensity.
to a single slit consists of a central bright band with Resolving Power of an optical instrument is the
alternate dark and weak bright bands of decreasing ability of the instrument to resolve or separate the
intensity on both sides. The width of central maximum images of two point objects lying close to eachother.
2DX_2fX Resolving power of a micro.scope is reciprocal
is 2.r = , where
a a of the minimum distance (d) between two point objects
a = width of slit
lying close to eachother. which can just be seen as
separate through the microscope.
D = distance of screen from the slit

/= focal length of the lens for diffracted light. R.P=-


1 2|isin0
d X
Diffraction is supposed to be due to interference
of wavelets from the exposed portion of the wavefront Here, p is refractive index of the medium between
from the slit. object and objective lens, 6 is half the angle of cone of
10/68
^ Fundamental Physics (XII)
light from the point object and d represents limit of Reason. In the diffraction pattern of a single slit,
resolution of the microscope. IXD
width of central maximum is , where
Resolving Power of a telescope is reciprocal of a
the smallest angular.septiration{d 9) between two distant
symbols have standard meaning.
objects, whose images are just seen as separate in the
telescope. CASE 2. According to Doppler's effect in light,
1 D
whenever there is relative motion between a source of
R.P = , where
r/0 1-22 X
light and observer, the apparent frequency of light
received by the observer is different from the true
D is diameter or aperture of the objective lens of frequency of light emitted by the source.
telescope and d 0 represents the limit of resolution of The apparent frequency is given by
the telescope.
V
Based on the above paragraph, answer \' = v 1±- ,

w
c
questions no. 1 to 4 :
1. In diffraction at a single slit, width of central where v is velocity of star and c is velocity of
maximum is light in vacuum. Note that + sign is used when source

Flo
moves towards the observer and vice-versa. We can
2XD XD show that
(a) (b)

reee
a a V V
Av = ±—V and A>. =+—X
Xa 2^ a

FFr
c c
(c) (d)
D D Doppler’s effect in light is used in measuring
Symbols have standard meaning. speed of star/galaxy, satellites, aeroplanes, submarines
urr
2. Resolving power of a microscope is
jisin 0 2psin9
etc.
for
Polarization of light is the phenomenon of
kkss
(a) (b) restricting the vibrations of light (electric vector) in a
X X particular direction, on passing ordinary light
Yo
oooo

3|isin9 sin 0 (unpolarised) through certain crystals like tourmaline


(c) (.d) crystal. This crystal acts as a polarizer.
eB

X p X
The plane in which vibrations of polarised light
All symbols have standard meaning.
are confined is called plane of vibration. A plane
For Question No. 3 and 4, we have given two
r

peipendicular to the plane of vibration is called plane


ou
ad

statements each, one labelled as Assertion (A) of polarization.


and other labelled as Reason (R).
YY

Our eye can not detect whether given light is


Chot»e the correct option out of the four options polarized or not. To detect polarization, we have to use
nndd

given below : another crystal which acts as analyser.


Re

(a) Both A and R are true and R is correct According to Law of Malus, when a beam of
Fi

explanation of A. completely plane polarised light is incident on an


(b) Both A and R are true and R is not the correct analyser, the resultant intensity (/) transmitted from
explanation of A. analayser is directly proportional to cos^ 0, where 0 is
(c) A is true but R is false.
angle between plane of transmission of analyser and
polariser.
(d) Both A and R are false.
Accordingto Brewster’slaw ; when unpolarised
3. Assertion. Resolving power of a telescope is light is incident at polarising angle (ip) on an interface
D separating a rarer medium from a denser medium of
. where symbols have standard meaning.
1-22;^ refractive index p. such that p = tan ip. then light
reflected in the rarer medium is completely polarised.
Reason. The same formula can be used for
Based on the above paragraph, answer
resolving power a microscope. questions no. 5 to 8 :
4. Assertion. The angular width of central 5. When a source of light moves towards an
maximum in the diffraction patlcm of a single observer witij a velocity v, the apparent frequency
slit is 2 X/a. of light received by the observer is
WAVE OPTICS 10/69

(fl) V {1 ± v/c) (fc>) V (I - v/c) (h) Both A and R are true but R is not a coiTect
(c) V (1 + v/c) {d) none of these explanation of A.
6. Which one of the following can be used as a (c) A is true, but R is false.
polariscr 7 (d) Both A and R are false.
(a) Water {b) glass 7. Assertion. The plane in which vibrations of
(c) diamond (d) Tourmaline crystal polarised light are confined is called plane of
vibration.
For Question No. 7 and 8, we have given two
statements each, one labelled as Assertion (A) Reason. Plane of vibration is the same as plane
and other labelled as Reason (R). of polarisation.
Choose the correct option out of the four options 8. Assertion. Naked eye can not detect whether

oww
given below : given light is polarised.
(a) Both A and R are true and R is correct Reason. Some suitable crystal has to be used as
explanation of A. analyser.

e
ANSWERS

FFrlo
re
1. (a) 2. ib) 3. (c) 4. (a) 5. (c) 6. (d) 7. (c) 8. (a)

ree
F
HINTS/EXPLANATIONS For Difficult Questions

rF
2XD Thus Assertion is true and Reason is the
1. Width of central maximum =

fsoor
ouur
a skf correct explanation of Assertion.
- „ , . f ● 2u.sin0 5. Frequency of light received by the observer is
2. Resolving power of a microscope =
X v'= V (1 +v/c).
ooko
Yo
3. Assertion is true, but Reason is false. 6. Toumaline crystal can be used as a polariser,
Y

4. Here Reason is true. Angular width of central


Bo

7. Assertion is true, but Reason is false because


reB

maximum = 2 9, where 0 is in radian and small.


the plane of vibration is perpendicular to the plane
Therefore, angular width of central maximum of polarized light.
DX
uur

8. Both, the Assertion and Reason are true and


oY

2x 2X
= 20 = X =
ad

D a a Reason is correct explanation of the Assertion.


dY
innd
Re
Fi
F

TYPE I. HUYGENS PRINCIPLE, TYPE II. INTENSITIES


REFLECTION AND OF MAXIMA AND MINIMA
REFRACTION OF LIGHT
3. Find the ratio of intensities at two points on a
1. The optical path of monochromatic light is same screen in Young’s double slit experiment, when
if it travels 2 cm thickness of glass or 2*25 cm, waves from two slits have path diff. of (/) 0 and
thickness of water. If refractive index of glass (//) XJA. (CBSE2003) [Ans. 2:1]
is 1-5, what is the refractive index of water ? 4. The ratio of intensities at maxima and minima
is 25 : 16. What will be the ratio of the widths
[Ans. 1'33]
of two slits in YDSE ?
2. Red light of wavelength 750 nm enters a glass
(CBSE2006) [Ans. 81 ; I]
plate of refractive index 1 -5. If velocity of light
in vacuum is 3 x 10^ m/s, calculate velocity, 5. The intensity ratio in the interference pattern is
1 ; 9. What is the amplitude ratio and the ratio
wavelength and frequency of light in glass. of widths of two slits ?
[Ans. 2 X 10* m/s ; 500 nm ; 4 x 10*“* Hz] (Hr. Board 2011) [Ans. 2 : 1 ; 4 : 1]

a
10/70
4 Fundamental Physics (XII) LV*1MII
6. Two interfering sources have an intensity ratio 1-5 m away. Find the distance of fourth bright
16 ; 1. Deduce amplitude ratio and ratio of fringe from the central maximum.
intensity between the maxima and minima in (CBSE 2010) [Ans. 0*1 ni]
interference pattern. (Uttarakhand Board 2012)
14. A double slit is illuminated by light of X= 6000
[Ans. 4 :1 ; 25 : 9]
A°. The slits are 01 cm apart and the screen is
7. The ratio of the intensities at minima to the
placed 1 m away. Calculate (i) angular position
maxima in YDSE is 9 : 25. Find the ratio of
of lOlh maximum in radian (ii) separation of two
widths of two slits. (CBSE 2014) [Ans. 16 : 1] adjacent minima.
TYPE III. YOUNG’S [Ans. (i) 6 X 10"^ rad. (ii) 6 x 10^ m]
DOUBLE SLIT EXPERIMENT 15. In a Young’s double slit experiment, the angular
width of a fringe formed on distant screen is 0-1®.
8. Yellow light of wavelength 6000 A produces The wavelength of light used is 6000 A. What is
fringes of width 0-8 mm in YDSE. What will

ww
the spacing between the slits ? (CBSE 2015)
be the fringe width if the 1 ight source is replaced [Ans. 3*44 x 10”* m]
by another monochromatic source of
wavelength 7500 A and the separation between 16. In Young’s experiment, two coherent sources are
1-5 mm apart and the fringes are obtained at a

Flo
the slits is doubled ?
distance of 2-5 m from them. If the sources
(CBSE Sample Paper 2005) [Ans. 0*5 mmj

e
produce light of wavelength 589-3 nm, find the

eree
9. The fringe width in YDSE is 2-4 x lO”^ m, when number of fringes in the interference pattern,
red light of wavelength 6400 A is used. By how which is 4-9 x 10"^ m long. [Ans. 5]

FFr
much will it change, if blue light of wavelength
4000 A is used ? 17. A central fringe of interference pattern produced

oorr
uur r
(Hr. Board 2002) [Ans. 9 x lO”^ m]
by light of wavelength 6000 A is shifted to the
position of 5th bright fringe by introducing thin
sf
10. In Young’s double slit experiment, using light film of )i = 1-5. Calculate thickness of the film.
of wavelength 400 nm, interference fringes of
sk
[Ans. 6 micron]
Yoo
width X are obtained. The wavelength of light
ooko

is increased to 600 nm and the separation 18. In Young’s double slit experiment, monochro
matic light of wavelength 630 nm illuminates the
eBB

between the slits is halved. If one wants


observed fringe width in the screen to be the pair of slits and produces an interference pattern
same in the two cases, find the ratio of distance in which two consecutive bright fringes are
uurr

between the screen and the plane of the separated by 8-1 mm. Another source of
ad

interfering sources in the two arrangement. monochromatic light produces the interference
Yo

(CBSE 2004) [Ans. 3: I] pattern in which the two consecutive bright


11. In YDSE, the width of the fringes obtained with fringes are separated by 7-2 mm. Find the
dY

light of wavelength 6000 A is 2 mm. Calculate wavelength of light from second source.
Re
innd

the fringe width if the entire apparatus is What is the effect on interference fringes if
immersed in a liquid medium of refractive index monochromatic source is replaced by a source
FFi

133. (CBSE 2003) [Ans. 1-5 mm] of white light ? (CBSE 2009)

12. In YDSE. using light of wavelength 400 nm, [Ans. 560 nm ; coloured fringes
interference fringes of width X are obtained. The with white centre]
wavelength of light is increa.sed to 600 nm and 19. In YDSE, light of wavelength 5000 A is used.
the separation between the slits is halved. If we The third bright band on the screen is formed at
want fringe width on screen to be the same, in a distance of 1 cm from the central bright band.
the two cases, find the ratio of the distance If the screen is at a distance of 1-5 m from the
between the screen and the plane of the centre of narrow slits, calculate the separation
interfering sources in the two arrangements. between the slits.

(CBSE 2004) [Ans. 3: 1] (Pb. Board 2008) [Ans. 2-25 x lO^* m]


13. The two slits in Young’s double slit experiment TYPE IV. FRESNEL DISTANCE
are separated by a distance of 0-03 mm. When
light of wavelength 5000 A falls on the slits, an 20. Calculate the distance which a beam of light of
interference pattern is produced on the screen wavelength 500 nm can travel without significant
WAVE OPTICS 10/71

broadening, if the diffracting aperture is 3 mm screen. What is the linear diameter of {i), first
wide. [Ans. 18 m] minimum (») second maximum ?
21. For what distance is ray optics a good approxi [Ans. 2-4 mm ; 6-0 mm]
mation when the aperture is 2 mm wide and the 29. A slit of width ‘a’ is illuminated by red light of
wavelength is 600 nm ? [Ans. 6*7 m] wavelength 6500 A. For what value of ‘a’ will
TYPEV. DIFFRACTION OF the {/) first minimum fall at an angle of diffraction
LIGHT AT A SINGLE SLIT
of 30" ? (»■) first maximum fail at an angle of
diffraction of 30" ? (CBSE 2009)

22. A parallel beam of light of wavelength 600 nm [Ans. (i) 1-3 X 10-^ m («) 1-95 x 10“^ m]
is incident normally on a slit of width'd'. If the 30. A 0-02 cm wide slit is illuminated at normal
distance between the slit and screen is 0-8 m and incidence by light having wavelength 6 x 10“^ m.
distance of 2nd order maximum from the centre
(0 Find the width of the central band maximum
of the screen is 15 mm, calculate the width of on the screen which is I m away from slit.

w
the slit. (CBSE 2008) [Ans. 80 |im] (ii) What would be the width of central
23. Light of X. = 550 nm is incident as parallel beam maximum, if the apparatus is immersed in water
on a slit of width 0-1 mm. Find the angular width whose refractive index is 4/3 ?

Flo
and linear width of the principal maximum in [Ans. (0 0*6 cm («) 0-45 cm]
the diffraction pattern on a screen at a distance

eeee
31. Two wavelengths of sodium light 590 nm and
of 11 m from the slit. Which of these widths
596 nm are used, in turn, to study the diffraction

Fr
will not change if the screen were moved to a taking place at a single slit of aperture 2 x
distance of 2-2 m from the slit ? 10"^ m. The distance between the slit and screen
(CBSE Sample Paper 2008) is 1-5 m. Calculate the separation between the

for
ur
[Ans. 1-1 x 10"^ rad ; 12*1 mm. positions of first maxima of diffraction pattern
obtained in the two cases.
Former will not change]
kss
(CBSE 2013) [Ans.9nm]
24. Light of wavelength 500 nm falls from a distant
Yo
source on a slit 0-5 mm wide. Find the distance 32. A parallel beam of light of wavelength 500 nm
oo

between the two dark bands on either side of falls on a narrow slit and the resulting diffraction
pattern is observed on a screen 1 m away. The
eB

central maximum, if diffraction pattern is first minimum is at a distance of 2-5 mm from


observed on a screen at 2 m from the slit.
the centre of the screen. Calculate the width of
[CBSE 2004 (C)] [Ans. 4 mm] (CBSE 2013) [Ans. 0*2 mm]
r

the slit.
ou
ad

25. A slit of width d is illuminated by a


TYPE VI. RESOLVING POWER
monochromatic light of wavelength 700 nm at
YY

normal incidence. Calculate the value of d for


OF MICROSCOPE AND
tff.ESCOPE
position of (/) first minimum at an angle of
nd
Re

diffraction of 30° {ii) first maximum at an angle


of diffraction of 30". (CBSE 2007) 33. The smallest object detail that can be resolved
Fi

with a microscope using light of wavelength


[Ans. 14 X 10 ^ m ; 21 X 10~^ m] 600 nm is 3-5 x 10”^ cm. Find N.A of objective
26. Light of wavelength 600 nm is incident normally when used dry and when immersed in an oil of
on a slit of width 3 mm. Calculate linear width 11= 1-6. [Ans. 0-86 ; 1-376]
of central maximum on a screen kept 3 m away 34. Calculate the resolving power of a tele.scope
from the slit. (CBSE 2001) [Ans. 1-2 mm] when light of wavelength 540 nm is used.
27. A parallel beam of light of wavelength 600 nm Diameter of objective lens is 6 cm.
is incident normally on a slit of width d. If [CBSE 2006 (C)i [Ans. 9-1 x 10^]
distance between slit and screen is 0-8 m and
35. Calculate wavelength of light used when limit
distance of 2nd order minimum from the centre
of resolution of the microscope is 5 x 10“^ m
of the screen is 9-6 mm, calculate the width of and cone angle of light falling on the objective
the slit. (CBSE 2008) [Ans. 0*1 mm] is equal to 90°. Given for air, ji = 1.
28. A plane wavefront (X = 6 x 10“^ m) falls on a slit [Ans. 0-707 x 10“^ m]
04 mm wide. A convex lens of focal length 0-8 m 36. What is the minimum angular separation between
placed behind the slit focusses the light on a two stars, if a telescope is used to observe them
10/72
‘P'iadec^'4' Fundamental Physics fXIIltTOTl
with an objective of a^rture 0-2 m. Wavelength 1
of light used is 5900 A. transmitted by polaroid B reduces to th of the
[Ans. 3‘66 x 10"* rad] intensity of unpolarised light incident on A ?
(CBSE 2013) [Ans. 45”]
TYPEVIL DOPPLER’S
EFFECT IN LIGHT 45. The polaroids and P-, are placed in crossed
position. A third polaroid F3 is kept between Fj
37. Light from a galaxy, having wavelength and F2 such that pass axis of F3 is parallel to that
6000 A is found to be shifted towards red by of F|. How would the intensity of light {I2)
50 A. Calculate velocity of recession of the transmitted through F2 vary as F3 is rotated ?
galaxy. [Ans. 2*5 X 10^ ms“^] Draw a plot of intensityA versus angle 0 between
38. The spectral line for a given element in light pass axis of F| and F3. In which orientation will
received from a distant star is shifted towards the transmitted intensity be minimum and
longer wavelength by 0-024%. Deduce the
maximum ? (CBSPi 2015, 2010) [Ans. 0”, 45"J
46. Two polaroids are placed at 90° to eachother and

w
velocity of star in the line of sight.
[Ans. 7*2 X lO'^ ms”^ moving away] the transmitted intensity is zero. What happens
39. The red shift of radiation from a distant nebula
when one more polaroid is placed between these

Flo
two, bisecting the angle between them. How will
consists of light known to have a wavelength of
the intensity of transmitted light vary on further
434 nm. In the laboratory, this wavelength

e
rotating the third polaroid ? (CBSE 2008)

rree
appears to be 6562 A. What is the speed of the
nebula in the line of sight relative to the earth ? [Ans. /q/4 ; and Fig. 10{Q).6]

r FF
Is it approaching or receding ? TYPE IX. TYPICAL PROBLEMS
[Ans. 1-2 X 10* ms -1.; receding]
uurr
TYPE VIII. BREWSTER’S
LAW AND LAW OF MALUS
for
47. In YDSE, the two slits 0-15 mm apart are
illuminated by monochromatic light of
wavelength 450 nm. The screen is 1 m away from
kss
the slits. Find the distance of the second (/) bright
ooook

40. A ray of light is incident on the surface of a


Yo
fringe («) dark fringe from the maximum.
glass plate of reflective index 1-536 such that
the reflected and refracted rays are mutually (CBSE 2010) [Ans. 6 mni ; 4-5 mm]
eB

perpendicular. What is the angle of refraction ? 48. In Young’s experiment, interference pattern is
obtained on a screen at a distance of 1 m from
[Ans. 33-n
urr

slits separated by 0-05 cm and illuminated by


41. The polarising angle of a medium is 60°.
ad

Calculate refractive index of the medium and sodium light of wavelength 5893 A. Calculate
Yo

angle of refraction. (CBSE 2016) distance between 4th bright fringe on one side
dY

and 3rd bright fringe on other side of central


[Ans. 1-732 ; 30°]
bright fringe. [Ans. 8-25 x 10"^ mj
Re

42. Two polaroids A and B are set in crossed


innd

49. In a two slit experiment with monochromatic


positions. A third polaroid C is placed between
light, fringes arc obtained on a screen placed at
Fi

the two making ZQ with the pass axis of first


some distance from the plane of slits. If the screen
Polaroid. Write the expression for intensity of
light transmitted from second polaroid. In what
is moved by 5 x 10"^ m towards the slits, the
orientations will be transmitted intensity be
change in fringe width is 3 x 10“^ m. If the
distance between the slits is 10"^ m. calculate
(/■) minimum (ii) maximum ? (CBSE 2010)
the wavelength of the light used.
Ans.
^/Qsin^20; 0“ ; 45° (Karnataka Board 2012) [Ans. 6000 A]
50. A beam of light consisting of two wavelengths
43. A ray of light falls on a transparent glass slab of 6500 A and 5200 A is used to obtain interference
refractive index 1 -62. If the reflected ray and the fringes in a Young’s double slit experiment.
refracted rays are mutually perpendicuhir, what (a) Find the distance of the third bright fringe on
is the angle of refraction ? [Ans. 31-7°] the screen from the central maxima for the
44. Two polaroids A and B are kept in crossed wavelength 6500 A. (b) What is the least distance
position. How should a third polaroid C be placed from the central maxima where the bright fringes
between them so that intensity of polarised light due to both the wavelength coincide ? The
WAVE OPTICS 10/73

distance between the slits is 2 mm and the 55. A parallel beam of monochromatic light of wave
distance between the plane of the slits and screen length 450 nm passes through a long slit of width
is 120 cm. [Ans. 1*17 x 10"^ m ; 15*6 x 10^ m] 0-2 mm. Find the angular divergence in which
51. A transparent paper (refractive index = 145) of most of the light is diffracted.
thickness 0 02 mm is pasted on one of the slits [Ans. 4*5 X 10 ^ rad.]
of a Young’s double slit experiment which uses
56. Unpolarized light of intensity 32 Wm"^ passes
monochromatic light of wavelength 620 nm.
through three polarizers such that the
How many fringes will cross through the centre
transmi.ssion axis of the last polarizer is crossed
if the paper is removed. [Ans. 14-5]
with the first. If the intensity of emerging light is
52. In a Young’s double slit experiment using
3 Wm“^. what is the angle between the

oww
monochromatic light, the fringe pattern shifts by
a certai n distance on the screen when a mica sheet
transmission axis of the first two polarizers ? At
of refractive index 1-6 and thickness 1-964 what angle will the transmitted intensity be
maximum ? [Ans. 30" ; 45"]
micron (1 micron = 10"^ m) is introduced in the
path of one of the interfering waves. The mica 57. Angular width of central maximum in the

e
sheet is then removed and the distance between Fraunhofer diffraction pattern of a slit is

re
FFrlo
the screen and the slits is doubled. It is found measured. The slit is illuminated by light of
that the distance between the successive maxima wavelength 6000 A. When the slit is illuminated

rF
ee
now is the same as the observed fringe-.shift upon by light of another wavelength, the angular width
the introduction of the mica sheet. Calculate the decreases by 30%. Calculate the wavelength of

rF
ouru
wavelength of the monochromatic light used in this light. The same decrease in the angular width
the experiment. [Ans. 5892 A] of central maximum is obtained when the original

fosor
53. In YDSE, the central bright fringe produced by apparatus is immersed in a liquid. Find the refrac
tive index of the liquid.
skf
light of wavelength 5600 A shifts to the position
of 5th bright fringe when a thin transparent film [Ans. 4200 A ; 1*43]
ooko
of refractive index 1-28 is introduced in the path
Yo
58. Critical angle for a certain wavelength of light
Y
of light from one of the two slits. Find the in glass is 40". Calculate the polarizing angle and
Bo

thickness of the film. [Ans. 10"^ m] the angle of refraction in glass corresponding to
reeB

54. In YDSE, slits are 0-2 mm apart. The interference it. [Ans. 57-3°, 32-7"J
fringes for light of wavelength 6000 A are formed 59. Two polaroids P] and P2 are placed with their
ooY
uur

on a screen distant 1 -5 m from the slits. Calculate


pass axes perpendicular to eachother. An
ad

(/) angular position of 3rd maxima, unpolarised light of intensity Iq is incident on


dY

(ii) angular position of 5th minima, P^. A third polaroid P3 is kept in between Pj and
such that its pass axis makes an angle of 60°
(///) fringe width
nind

with that of Pj. Determine the intensity of light


Re

[Ans. (1) 0-009 rad, (i7) 0-0135 rad


transmitted from P2.
(in) 4-5 mm]
F
Fi

(CIISIC 2014) [Ans. 3/(/32]

For Difficult Problems

1. As optical path (in vacuum) = |i x path in medium 750


= 500 nm
m, X 2-25 = X 2 00 = 1 -5 X 2-00 1-5

1-5x2-00
= 1-33
c 3x10^
2-25 = 4x 10^** Hz
~X~ 750xl0-^
2. Here, X - 750 nm, p = 1 -5, c = 3 x 10^ m/s 3. When path diff. = 0,
c 3x10^ phase diff. 4>| = 0
v = — = 2x10^ m/s
4 1-5 When path diff. = X/4,
10/74 Fundamental Physics (XII) VOL.II

2n
1
phase diff. ^2 ~ ^
= 71/2
and P2 =
^2
/j _ 4/qCos^<1)i/2 _ (cos 0)2 2
^2 4 /q cos^ <j>2/2 (cos 7c/4)2 1 Pi ^2 ^1
/
4. Here, max. _ ^ w,
1
= ? 7500 1
I. 16 X- =0*5 mm
nun. W2 6800 2

/
max.
25 9. Here, Pi = 24 x IQ-^ m, X, = 6400 A,
= 4000 A
nun. (a-6)2 16
,"1 £> D _ Pi _ 2 4x10^
-+ 1
Pi-^'7’ d"?l, “6400X10-10

looww
g+6 _ 5 or V.6 y_£ 1

a-b 4 a
--1
^ 4 f 24x10-4
\

/ \
Again, P2 = ^2 7 = 4000x10-1° 6400x10-1°
a a
5 --1 =4 -+1
a
-=4+5=9 = l-5x 10-^m

ree
or
\b b
Pi - P2 = 24 X 10-4 - 1-5 X 10-4
2 (9^ = 0-9 X 10-4

ree F
W2
1 a

62 U,
= 81:1
r FF = 9 X 10“® m
10. Here,A. = 400nm, p = X,
5. nun _ (a-6)2 _ 1 fofroF
u
I (a+ 6)2 9 r = 600nmd' = fr/2,P' = p, — = ?
ks
max
D'

a-6 _ 1 XD X'D'
os o

or 3a-3b-a + b
From P = = X
YYouor

7T6“3 d d'
BBook

2a = 46 or a = 2 b
d ^ 600 x2 =3:1
*’* D' d'^X 400
r ee

a 2
Amplitude ratio : —
6 1 11. Here, X-i = 6000 A, Pi = 2-0 mm,
ouru
ad

p=l-33, P2 = ?
L = iL = £^ /'2f = 4:1
Yo

^2 ^2 „ ^1 _ 6000
^2 1-33
11
d
Re

^min ^ w
1
inY

7. Here, = ? D D
/
25’ W2 P2=A.2- and Pi =>-17
FFind

max

(a-b)^ 9 X2 6000 x 2-0 mm


P2=^Pl = = 1*5 mm
(o + i>)2 25 1-33 x6000

a-5 3 d

a +6 5
12. Here X^ = 400 nm, 7^ = 600 nm, ^2 “
5a-56 = 3fl + 36 D,
1
2a = 86 or a =4 6 =? Asp2 = Pi
^2
g2 _ (46)2 _ 16
W2 I2 62 62 1 ^2 d
1

8. Here, X-i = 6000 A, Pi = 0-8 mm,


^2 = 7500 A, ^2 = 2^1, p2 = ?
^ -.4x^2 = 2x
600
= 3:1
^2 “d2^ 1
400

«
WAVE OPTICS 10/75

14. Here, X = 6000 A = 6 x lO""^ m 19. Here, X = 5000 A = 5 x lO""^ m, n = 3


d = 0*1 cm = 10"^ m, D = 1 m, n = 10 = 1 cm = 10“^ m, D = 1-5 m, J = ?

jc = /iX—-If®is angle of diffraction, then ^3 = n


d d

nX 10x6x10-’^ .^2
10"2 o c
=3x5x10 1-5 =
' X— 22.5x10-'^
sin 0 = — = = 6x10-3
D d 10-3 d d

As sin 9 is small, ^
d =
22*5x10-^
T— = 22-5 X10-3 m
0 « sin 0 = 6 X 10“^ rad 10-2
(i7) Separation of two adjacent minima = 2-25 X 1(H m

XD 6x10-^ xl 20. Z^=?,X = 500nm = 500x 10-^m = 5x lO-'^m


P= = 6 X 10“^ m a = 3 mm = 3 x 10~3 m

w
d 10-3

n
(3x10-3)2 = 18m
15. 0 = 01’’ = 01 X radian ^ ~ X ~ 5x10-'^

Flo
180
22. Here,X = 600nm = 6x 10-'^m,d=?

e
X = 6000A = 6x 10-’m,i/=?

ree
D = 0-8 m, n = 2,2: = 15 mm = 15 x 10"3 m

FFr
As angular width, 0 = — For secondary maxima,
d
f/sin0 = (2«+l)A/2
uurr
orr
6x10-”^ 180x6x10-'^ f x^ 5^
d - =
0 O-lxji/180 ~ 01x314 2 ’
sf
= 3-44 X 10^ m
kks
5XD 5x6x10-2x0-8
16. Here, = 1-5 mm = 1-5 x 10-3 ^ 2.5 m
Yo
a =
2x15x10-3
ooo

lx
X= 589-3 nm = 589-3 X 10-^
= 0-8 X 10“^ m = 80 pm
eB

r^- ... Q 589-3x10-9x2-5 23. Here, X = 550 nm = 5-5 x 10-2 m,


Fnnge width, p = —— = z a = 0-1 mm = 10^ m, D = 1-1 m
d 1-5x10-3
Angular width of principal maximum,
ur

= 982-17 X 10-6 m
ad

IX 2x5-5x10-2
YYo

4-9x10-3 26 — — — z - 1*1 X 10"2 rad


Number of fringes = - =
X
= 4-99 a 10-4
P 982-17x10-6 Linear width of principal maximum
dd

x = (2 0)£)= 11 X 10-2 X 1-1


Re

O 5
in

17. Here X = 6000 A = 6 x IO-2 m = 1-21 X 10-2 m = 12-1 mm


F

n = 5,p= 1-5,/ = ? When the screen is moved. D is changed linear


The basic relation used is that path difference width changes, angular width does not change
introduced by film = Path diff. for the shift, 24. Here, X = 500 nm = 5 x 10-2 m,
(p- 1) f = nX a = 0-5 mm = 5 X lO"^ m
nX D = 2 m. Width of central max. (2 x) = ?
t = = 6x10-6 m = 6 micron From a sin 0 = n X,
(P-1)
f 2XD 2x5x10-2x2
X' 3' - =a, 2x =
18. As — = ^
a
D a 5x10-4
X p
= 4 X 10-3 nj = 4 mm
7-2
x630 = 560nm 25. Here, ^ = 700 nm = 7 x 10-2 m, =?
P 8-1
(i) For first minimum, at 0 = 30"
When monochromatic source is replaced by sin 9 = nX
source of white light, fringes are coloured with
white centre. dsin 30"= 1 X 700 x 10-9 ; fr= 14 x ir2 m
10/76 Fundamental Physics (XII) VOL.II

(h) For first maximum 32. Here, X = 500 nm = 5 x 10~^ m, n = 1, a = ?


D = 1 m, jc = 2-5 mm = 2-5 x 10"^ m
flfsin0 = (2n + l)- = —
2 2 For diffraction minimum
a sin 0 = rCk
3X 3x7x10-'^
d = = 21xl0"’m
2sin0 2 X sin 30“ a = nX

w
26. Here, X = 600 nm = 6 x 10"^ m.

fl = 3 mm = 3 X 10“^ m, D = 3 m ; (2 jc) = ? nXD 1x5x10"'^ xl


a =

2XD 2x6xlQ-'^x3 X 2-5x10-3


2x =
3x10-3 = 2 xlO-* m = 0*2 mm

e
a

= 12 X 1(H m = 1*2 mm 33. Here, X = 600 nm = 6 x 10"^ m,

e
d = 3-5x 10“^ cm = 3-5 x 10-^ m

o
27. Here, X = 600 nm = 6 x 10"^ m, D = 0-8 m

rw
r
rt = 2, X = 9-6 mm = 9-6 x 10“3 m,« = ? When objective is used dry,

F
For 2nd order minimum, a sin 0 = n X X _ 6x10-“^

ullo
N.A = = 0-86
“2x3-5x10-^

FF
af-l=2X,
D When objective is immersed in an oil of |i = 1-6.

e
srre
MA = |ix(dry)M/l
2XD_2x6xlQ-^xO-8

oF
a = = 1-6 X 0-86 = 1-376

k
X 9-6x10-3
34. Here, R.P = ?, X = 540 mm = 540 x 10-^ m.
= 10-^ m = 0-1 mm

fofr D = 6 X 10"^ m
uor
28. Here, X = 6 x 10"^ m, a = 0-4 mm = 4 x 10“^ m
/= 0-8 m
o D 6x10-2
= 9-1 X 10^
Y
R.P =
kos
(0 Linear diameter of 1st minimum 1-22X 1-22 x 540x10-9
Yo
rBB

35. Here, X = ?,
oo

2X/ 2x6x10-'^ xO-8


eY

= 2x = Limit of resolution, d=5x 10“2 m


a 4x10-^
re

2 0 = 90“ or 0 = 45“, ^ = 1
= 2-4 X 10"3 m = 2-4 mm
u

X
(a) Linear diameter of second maximum, (n = 2) As d =
d

2|xsin0
ou
o
ad

2x^ = 2(2a + l)X/ _2x5x6xlQ-^xO-8 X = 2 |i, sin 0 = 2 X 1 X 5 X 10~2 sin 45“


nY

2a 2x4x10^
1
= 6 X 10-3 _ 5.0 imjj X = 10-^x = 0-707 X10"^ m
s
nid
Re

30. (ii) Width of central band maximum on


F

immersing the apparatus in water becomes l/|i 36. Here, cf0 = ? £> = 0-2 m,
Fi

time, because X' = X/p X = 5900 A = 5900 X KT^O m


31. Here, X = 590 nm and X' = 596 nm
1-22 X 1-22 x 5900x10- 10
a = 2xl0‘^m and D=l-5m J0 =
D 0-2
For secondary maxima, the condition is dQ = 3-66 X 10"* radian
X 37. Here, X= 6000 A, AX = 50 A, u = ?
a sin 0 = (2n-fl)-
n
2 AX 50
V = xc = x3x108
First maxima of diffraction pattern corresponds X 6000
to a = 1.
= 2-5 X 10^ m/s
Separation of first maxima of diffraction
X' 3X 38. Here, AX _ 0-024 = 2-4x10^
pattern of two wavelengths =(2x1+1) X " 100
2 2
AX
^ c =2-4x 10-^x3x 10®
V =

= 1(V-X) = -2 (596-590) = 9ran


2 = 7-2 X lO'* ms-l
WAVE OPTICS 10/77

As A, increases, star must be moving away from This light falls on polaroid C at Z9.
earth Intensity of light transmitted through C is
39. Here, k = 434 nm = 434 x 10"^ m
r = 6562 A = 6562 X lO-'^m.

I -x>/c
/2 = /, cos^ 6= ^cos^e
(l-v/c)
As v' - t V Light transmitted from C falls on polaroid B at
Vl - lc~ Y 1 + u/f an angle (90 - 9). Therefore, intensity of light
transmitted from B is

l-v/c (v'
n2 434x10"^ /3 = I2 cos^ (90 - 9)
l + v/c ) l^6562xl0-'<^ J /
l-v/c
= —cos- 9 sin- 9 = — (sin 9 cos 9)^
2 2
= 0-437.
l + t;/c
On solving, we get v = 1-2 x 10* ms
-1

w
^3 == -2-(sin29)-
8
As apparent wavelength increases, the nebula
must be receding away from earth. As 73 = 7(/8, therefore, therefore (sin 2 9)^ = 1

Flo
40. Here, |l = 1 -536. When reflected and refracted sin 2 0= I,or2 0=9O"or0 = 45'’
rays are mutually perpendicular, i = /^, where tan

e
45. The situation is shown in Fig. 10(Q).5.

rree
1-536

/^ = tair' (I-536) = 56-9°

r FF
FIGURE 10(Q).5
/● = 90‘
ip = 90 - 56-9° = 33-r Pl P3
41. Here, (^ = 60", M = ?, r = ?
uurr
|i ~ tan ip - tan 60° = ^ = 1-732 for
kss
r = 90" - ip = 90" 60" = 30‘
ooook

42. By lawofMalus, intensity of light emerging from


Yo

middle polaroid C; 7| = 7q cos- 0. If 9 is angle between P, and F3, the angle between
eB

This is the intensity of light falling on polaroid B. F3 and P2 = (90 - 0)


Light emerging from second polaroid B, By Law of Malus, = 7j cos^ 0 and
I2 = l\ cos^ (90" - 9) = 7q cos- 0 [cos (90 - 9)1^ I2 = I2 cos^ (90 - 9) = 73 sin^ 9
urr
ad

1 .

= 7j cos^ 0 sin^ ® ^ ®
Yo

= 7q cos^ 9 sin- 9 “ ^ /Q(sin29)-


dY

1
I.=-hsin^2d 7,“ = -4 7,‘ sin^ 2 0
Re

0
2 4
innd

72 will be minimum, when sin 2 0 = 0, 0 = 0 The plot of 7, versus 0 is as shown in Fig. 10(Q).6.
Fi

I2 will be maximum,
when sin 2 0 = max. = 1 = sin 90° FIGURE 10.(Q).6
I2.1
90"
0 = = 45'
2

43. tan /^ = p = 1-62


= tan-* (1-62) = 58-3" / ^6
58-3" = 31-7"
r = 90" - ip = 90' 0 7u/4 nl2 K 3n 2k

44. Let the angle between the pass axis of polaroids 2

A and C be 0. As polaroids A and B are crossed,


the angle between the pass axes of polaroids C Transmitted intensity I2 will be minimum. When
sin 2 0 = 0 or 0 = 0".
and B is (90 - 9). When unpolarised light of
intensity 7^ falls on first polaroid A, intensity of Transmitted inensity will be maximum, when
polarised light through A is /, = Iq/2. sin2 0= 1,2 0 = 90", 9 = 45"
10/78
7^'uideefr ^ Fundamental Physics (Xn) VOL.II

46. If /q is intensity of polarised light from 1st (b) If n is the least number of fringes of
Polaroid, then intensity of light transmitted from Xj (= 6500 A), which coincide with (« +1) fringes
3rd Polaroid placed in between (for 0 = 45®) is of ^ (= 5200 A), then a:' = n P = (« + 1) (J'
/j = /q cos2 e = /q (cos 45®)2 = ^ n +1 _ P _ A,j
n

Intensity of polarised light from 2nd polaroid


already there
or 1 + - =-i.
J ^2
/, =/,cos245»=^xJ^J=i
n

n =
^2 _ 5200
= 4
On further rotating the third polaroid, intensity Xj-^2 6500-5200
of transmitted light will vary as shown in Fig.
X,D 4 x 6500x10-*° X1-2
10(Q).6.
.'. y = np = n-Ld

w
47. Here, d = 015 mm = 015 x lO'^ m, 2x10-3
X = 450 nm = 450 x IQr^ m, D = 1 m = 15*6xl0^m

Flo
Distance of 2nd bright fringe from central max. P
51. From
^XD 2 x 450x10-^x1

ee
^ d 015x10-3 No. of fringes that cross when paper is removed

Fr
= 6 X 10~3 m = 6 mm _ yp _ (p-l)t _ (1-45-l)x 0-02x10-3
Distance of 2nd dark fringe from central max. P X 620x10^

for
ur
A,P 3x450x10-^x1 = 14-5
4=(2n-l) 2d 2x015x10-3 P
52. Shift, yp=^ai-l)/
s
= 4-5 X 10"3 m = 4*5 mm
ok
Yo
48. Here, D = 1 m, d = 0-05 cm = 5 x 10"^ m
= (1-6-l)x 1-964x10-^ m
Bo

X = 5893 A = 5893 x lO"*® m


Distance between 4th bright fringe on one side _ 11784 X10^ p
re

and 3rd bright fringe on other side of central max.


X
XD
Distance between successive maxima
ou

is equal to width of 7 fringes = 7 —-


ad

P = -^, i-e., P<>=P


Y

7x5893xl0-*°xl d
m = 8*25 X 10-3 m
5x10-4 As D is doubled, P becomes twice.
nd
Re

XD
49. As
As per question, yp = 2p = —x 11784x 10"^
Fi

A*

Ap = ^(AD)
d ^ 11784x10-^ m = 0-5892 x 10-^ m = 5892 A
2
^_(Ap)xd 53. Here, X = 5600 A = 5600 x lO"'® m
AD
p= 1-28,/=?
3 X10"^ X10-3 5XD
= 6x10-’ m
Shift, x = —
5X10-’ d
= 6000 A
, D 5XD
D As x = (|x-l)/x—= ——
50. (a) x = nX— d d
d
5X 5x5600x10-*°
_3x(6500xlQ-*°)xl-20 t = = 10-Sm
= 117 X 10-3 „ p-l (1-28-1)
2x10-3
WAVE OPTICS 10/79

54. Here, d = 0-2 mm = 2 x 10^ m I2 = I\ cos^ 0 and = I2 cos^ ^


A, = 6000A = 6x 10-?m,D= l*5m I2 = /i cos^ 0 cos^ (j>
(0 For angular position of 3rd maxima = cos^ 0 cos^ (90® - 0)
●<3 _ «A,_ 3x6x10"“^ = /j cos^ 0 sin^ 0 = 16 cos^ 0 sin^ 0
0 = = 9 X 10-^ rad
D d 2x10-^ 3 = 4 (sin 2 0)2
(if) For angular position of 5th minimum
sin 20 = ^= sin60®.
0' = ii = (2n-l)^ 4
D d 0=30“

6x10-’ 73 will be maximum when


= (2x5-l)x
2x2x10“^ sin 2 0 = max. = 1 = sin 90“ 0 = 45“

= 13-5 X 10-2 = 0-0135 rad 57. (a) As angular width of central maximum is

ww
directly proportional to and angular width
XD
(ii’O Fringe width, P = —d decreases by 30%, therefore, wavelength must
have decreased by 30%. The new wavelength

FF loo
would be 70% of original wavelength.
6x10-’X1-5
= 4-5x10-2 m =4-5 mm

ree
2x10-4 .-. X' = —X = — X 6000 A = 4200 A
100 10
55. Most of the light is diffracted between the first

rFee
order minima on either side of central maximum, (b) When the apparatus is immersed in liquid,
for which X changes to X/|i.

oor rF
rur
. ^ _^X ^450x10"^- = ± 2-25 X 10-2 1
s ff
sin0 = ±— = + Angular width becomes — times.
d 0-2x10-2 It
k
As sin 0 is small, therefore, sin 0 = 0
YYoou
10
1 ^ 70
okos

0 = ± 2-25 X 10-2 radian.


\i 100 or p. = y = 143
BBoo

Angular divergence = 20 = 4-5 x 10"2 rad


59. Intensity of polarised^light from Pj = 7(/2
r ee

56. Let 0 = angle between transmission axis of


and ?2 Intensity of polarised light from P3
<{> = angle between transmission axis of P2 and
ouur
ad

f I \ /
= _0 cos2 60“ = -^
^3-
Yo

.-. 0 + ([) = 9O“ or <t) = (9O“-0) ..(/)


I 2 /
8

Here Iq = 32 Wm-2 Intensity of Polarised light from P2


Yd
Re

3/0
= :^cos2 (90“-60“)=^(V3/2f =
idn

7j =i/„
2 ° =16Wm"2
FFin

32
10/60 Fundamental Physics (XII)ESm

WITH
SOLUTIONS

Q. 1. Monochromatic light of wavelength 589 nm Is incident from air on a water surface. What are the

w
wavelength, frequency and speed of (a) reflected and {b) refracted light ? \i of water is 1*33.
Sol. Here, X = 589 nm, c = 3 x 10^ m/s, |i = 1-33
(£3) For reflected light
3x10’
wavelength, X - 589 nm = 589 x 10“ ® m, v = —K

roow
14

e
= 5 09 X 10 hertz
589x10"^

re
speed, u = c = 3 X 10* m/s
X 589x10“^
= 4-42 X 10“ ^ m
{b) For refracted light X' = — ^

reF
uFFll
e
As frequency remains unaffected on entering another medium, therefore, v' = v = 5*09 x 10^** hertz
3x10’

sFr
speed, v' = — = 2-25 X 10* m/s
p 1-33

foro
Q. 2. What is the shape of the wavefront in each of the following cases ?
uor
(a) light diverging from point source, fk
okso
(b) light emerging out of a convex lens when a point source is placed at its focus,
Y
(c) the portion of the wavefront of light from a distant star intercepted by earth.
Yo
oo
BB

Sol. (a) The geometrical shape of the


wave front would be diverging 10(N).1
spherical wave front, as shown in
^PWF
rYree

Fig. 10(N).l(a)
ouu

ib) When a point source is placed at


the focus of a convex lens, the rays
ad
Ydo

>
emerging from the lens are parallel. Rays Rays
Therefore, the wave front must be
nidn

plane, as shown in Fig. 10(N).1(/j)


Re

(c) As the star {i.e. source of light)


is very far off i.e. at infinity, the wave
F
Fi

front intercepted by earth must be a o


plane wave front as shown in Fig.
10(N).1(6).
Q. 3. (a) The refractive index of glass is 1-5. What is the speed of light in glass ? (Speed of light in vacuum
is 3 X 10* ms“*).
(b) Is the speed of light in glass independentof colour of light ? If not, which of the two colours, red
and violet travels slower in a glass prism ?
Sol. (a) Here, p = 1-5, u = ?, c = 3 x 10* ms“*

As p=-
c
V = —c 3xjf = 2 X 10* ms *
V p 1-5
{b) No, the refractive index and the speed of light in a medium depend on wavelength i.e. colour of light.
We know that p^ > p^. Therefore Vvoiiet ^red- Hence violet component of white light travels slower than
the red component.
WAVE OPTICS 10/81

Q. 4. In Young’s double slit experiment, the slits are separated by 0.28 mm and the screen is placed 1.4 m
away. The distance between the central bright fringe and the fourth bright fringe is measured to be
1.2 cm, determine the wavelength of light used in the experiment.
Sol. Here, c/ = 0.28 mm = 0.28 X 10“^ m, D= 1.4 m, .r = 1.2 cm = 1.2 x 10"2 m,/? = 4, X = ?
D xd 1.2x10-2x0.28x10“^
For constructive interference x — nX— or ^ = = 6 X 10-2 m
d nD 4x1.4
Q. 5. In Young’s double slit experiment using monochromatic light of wavelength X, the intensity of light
at a point on the screen where path difference isXis K units. What is the intensity of light at a point
where path difference is X/3 ?

oww
Sol. Let /| = /2 = /. If (]> is phase difference between the two light waves, then resultant intensity,
/^ = /j + /^ + 2 ^/j cos 6
When path difference = X, phase diff. (]) = 0“ /^ = / + / + 21^ cos0°= 4/ = K

ee
X , 2tu 2n
When path difference = — , phase diff. $ - — rad. I'
R ~ ! + i + 24Tl cos
3

FFrlo
r
1

rF
I' R =21 + 21 = / =m

ee
2;
Q. 6. A beam of light consisting of two wavelengths 650 nm and 520 nm, is used to obtain interference
ouru
rF
fringes In a Young’s double slit experiment, (a) Find the distance of the third bright fringe on the
screen from the central maximum for wavelength 650 nm. (b) What is the least distance from the

ffosor
central maximum, where the bright fringes due to both the wavelengths coincide ? {CBSE 2012)
A.2 = 520 nm = 520x 10"^
Sol. Here, 3.1 = 650 nm = 650x 10" ^ m.
os k m

Suppose d = distance between two slits. D = distance of screen from the slits.
ook
D D
Yo
x = nX, — = 3x650
Y
{a) For third bright fringe. « = 3 ●c/ d
nm
Bo

Let «th fringe due to X2 = 520 nm coincide with (n - 1 )th bright fringe due to X.[ = 650 nm.
reeB

.●. n 3.T = (/I - 1) X,]


nx520 = (n- 1)650
oouY
ur

4n = 5n-5 or n = 5
ad

D D D
The least distance required, x = nX^- — = 5x520— = 2600—wm
dY

d d d
nidn
Re
F
Fi
10/82 Fundamental Physics (XII)ES29Q

I
WITH ANSWERS,
HINTS AND SOLUTIONS

w
MULTIPLE CHOICE QUESTIONS-I
xl/2
1. Consider a light beam incident from air to a And 1
{b) 1—Lsin^e
glass slab at Brewster’s angle as shown in Fig. n~
10(N).2.

roow
e
U/2
And (
l-4rsin20 K

re
(C) + -

^ I 2

^1/2

reF
And 1

uFFll
(d)
X
1—:^sin^6 + 2tc

e
n~

4. In a Young’s double slit experiment, the


source is white light. One of the holes is

sFr
covered by a red filter and another by a blue

foro
filter. In this case
uor
A Polaroid is placed in the path of the emer
gent ray at point P and rotated about an axis fk (a) there shall be alternate interference patterns
of red and blue
okso
passing through the centre and perpendicular
(b) there shall be an interference pattern for red
to the plane of the polaroid.
Y
Yo
distinct from that for blue
oo

(a) For a particular orientation there shall be


BB

(c) there shall be no interference fringes


darkness as observed through the polaroid.
(d) there shall be an interference pattern for red
(b) The intensity of light as seen through the mixing with one for blue
rYree

Polaroid shall be independent of the rotation, 5. Figure I0(N).3 shows a standard two slit
ouu

(c) The intensity of light as seen through the arrangement with slits Sj, 52- P\, Pi are the
Polaroid shall go through a minimum but not
ad

two minima points on either side of ,P Fig.


Ydo

zero for two orientations of the polaroid. 10(N).3.


(d) The intensity of light as seen through the At Pj on the screen, there is a hole and behind
nidn

polaroid shall go through a minimum for four P2 is a second 2-slit arrangement with slits
Re

orientations of the polaroid. .S3, S4 and a second screen behind them.


2. Consider sunlight incident on a slit of width
F
Fi

FIGURE 10(N).3
10*^ A. The image seen through the slit shall
(a) be a fine .sharp slit white in colour at the centre
(b) a bright slit white at the centre diffusing to
zero intensities at the edges
(c) a bright slit white at the centre diffusing to
regions of different colours
Second
(d) only be a diffused slit white in colour Screen

3. Consider a ray of light incident from air onto


a slab of gla.ss (refractive index «) of width d, (a) There would be no interference pattern on
at an angle 6. The phase diflerence between the second screen but it would be lighted
the ray reflected by the top surface of the glass (b) The second screen would be totally dark
and the bottom surface is
(c) There would be a single bright point on the
l second screen
2nnd
id) + 7t (d) There would be a regular two slit pattern on
A. n- the second screen
WAVE OPTICS 10/83

MULTIPLE CHOICE QUESTIONS-U

6. Two sources 5j and S2 of intensity and I2 7. Consider sunlight incident on a pinhole of


are placed in front of a screen, Fig. 10(N).4(a). width 10^ A. The image of the pinhole seen
on a screen shall be
The patteren of intensity distributionseen in
the central portion is given by Fig. 10(N).4(i). (a)a sharp white ring
(b)different from a geometrical image

w
FIGURE 10(N).4 (c)
a diffused central spot, white in colour
(d) diffused coloured region around a sharp
● S
A central white spot
8. Consider the diffraction pattern for a small

e
●S2 pinhole. As the size of the hole is increased

re
\LAL (a) the size decreases

row
O O x = 0
(b) the intensity increases
(c) the size increases

eeF
ullo
In this case which of the following statements (d) the intensity decreases

FF
are true, 9. For light diverging from a point source
(a) the wavefront is spherical
(a) 5j and Sn have the same intensities
(b) the intensity decreases in proportion to the

srr
roF
(b) 5[ and So have a constant phase difference distance squared

k
(c) 5j and $2 have the same phase (c) the wavefront is parabolic
uor
(d) 5[ and ^2 have the same wavelength ofof (d) the intensity at the wavefront does not depend
on the distance
Y
kos
ANSWERS
Yo
eerBB
oo

1. (c) 2. (a) 3. (fl) 4. (c) 5. (d) 6. (a, b. d) 7. (/?, d) 8. {a, b) 9. {a, b)


rY

HINTS FOR DIFFICULT MULTIPLE CHOICE QUESTIONS


u

Multiple Choice Questions -1


ou
d
o
ad

1. According to Brewster’s law, the light reflected from the top of glass slab gets polarised, Fig. 10(N).2. The
nY

light refracted into the glass slab and the light emerging from the glass slab is only partially polarised.
Therefore, when a polaroid is held in the path of emergent light at P, and rotated about an axis passing
nid
Re

through the centre and perpendicular to plane of polaroid, the intensity of light shall go through a minimum
F

but not zero for two orientations of the polaroid.


Fi

Therefore, choice (c) is correct.


2. Here, the width of the slit is 10"^ A, i.e., 10000 A. The wavelength of (visible) sunlight varies from 4000 A
to 8000 A. As width of slit a>X (wavelength of light), therefore, no diffraction occurs. The image seen
through the slit shall be a fine sharp slit white in colour at the centre. Choice (a) is correct.
3. In Fig. I0(N).5, a ray of light AB is incident from air onto glass slab
of width at angle 0. It is reflected partially at B and refracted partially
at B along BC at Zr. At C. the ray is partially reflected along CD and
partially refracted (not shown). To calculate phase difference between
rays reflected from B and C, we find
time difference, AT = time taken to travel BC in glass

BC id/cosr) nd

V c/n ccosr
10/84
Fundamental Physics (XII) VOL.II

sin0
From Snell’s law, n =—
sin r

sin0
sinr =
n

/ xl/2
sin2 0
cosr = Vl - sin^ r = 1- 2
n

oww
-1/2

A7’ =
nd nxra {,1- sin^0^
...{c = XIT)
V/2 X 2

c \1- sin2 0 n

2
n

e
FFrlo
re
x-l/2
2nAT 2%nd ( sin2 0

ree
Phase diff. = A<1) = —

F
1-
T X n2

rF
As reflection at C is from medium of higher refractive index, additional phase diff. of n is introduced.

fsoor
ouur
x-l/2
2nnd ( sin2 0
Hence required phase difference = —- 1-
skf + JC
2
n
ooko
Yo
Choice (a) is correct
Y

4. As light from two slits of YDSE is of different colours/wavelengths/frequencies, therefore, there shall be
Bo
reB

no interference fringes. Choice (c) is correct


5. As there is a hole at minima point P2, the hole will act as a source of fresh light for the slits S3 and S4, Fig.
10(N).3. Therefore, there will be a regular two slit patternon the second screen. Choice (d) is correct
uur
oY
ad

Multiple Choice Questions -11


dY

6. From the pattern of intensity distribution seen in the central portion. Fig. 10(N).4, we find that
(0 as intensity of successive maxima is the same, S, and S2 have the same intensity.
innd
Re

(if) as width of successive maxima appears to increase slightly, Sj, S2 must have a constant phase difference,
(ill) as minimum intensity is zero, 5j, ^2 must have the same wavelength.
Fi
F

The choices (a), (h), (d) are correct.


7. As width of pinhole is 10^ A = 1000 A and wavelength of sunlight ranges from 4000 A to 8000 A, therefore,
sunlight is diffracted on passing through the pin hole. The image of the pinhole seen on the screen shall be
different from a geometrical image. Infact, the image of pinhole will consist of a diffused coloured region
around a sharp central white spot. The choices (b) and (d) are correct

8. As size of hole (a) is increased, width of central maximum of diffraction pattern of hole L2/^1
= decreases.
a
\

As the same amount of light emergy is now distributed over a smaller area, the intensity increases. Choices
(c) and (h) are correct
9. For light diverging from a point source, the wavefront is (diverging) spherical wavefront. The intensity
varies inversely as the area of the wavefront (= 4 jc r^), i.e., intensity decreases in proportion to the distance
squared.

9
WAVE OPTICS 10/85

VERY SHORT ANSWER QUESTIONS


10. Is Huygen’s principle valid for longitudinal sound waves ?
Ans. Yes, Huygen’s principle is equally valid for longitudinal sound waves.
11. Consider a point at the focal point of a convergent lens. Another convergent lens of short focal length
is placed on the other side. What is the nature of the wavefronts emerging from the final image ?
Ans. The focal point of a convergent lens is the position of the real image formed by the convergent lens, when
object is situated at infinity.
When another convergent lens of short focal length is placed on the other side of the first convergent lens,
the combination will fonn a real point image at the combined focus of the two lenses. The wavefronts
emerging from the final image will be spherical.
12. What is the shape of the wavefront on earth for sunlight ?

loow w
Ans. The wavefront on earth for the sunlight (treated as point source) will be spherical. However, as distance of
earth from the sun is huge (== 10^* m), portion of the wavefront on earth can be treated as plane wavefront.
13. Why is the diffraction of sound waves more evident in daily experience than that of light wave ?
Ans. For diffraction to occur, size of obstacle/aperture must be of the order of wavelength of the waves to be
diffracted.

ee
Wavelength of light waves varies from 4 x 10"^ m to 8 x 10“^ m. Obstacles/apertures of this small size are

Fr
hardly available. Therefore, diffraction of light waves is not so common. On the contrary, wavelength of
r FF
sound waves varies from 15 m to 15 mm. Obstacles/aperturesof this size are commonly available. Therefore,
diffraction of sound waves is more evident in daily life.

rree
14. The human eye has an approximate angular resolution of 0 = 5*8 x 10"^ rad and a typical photoprinter
fofr oF
prints a minimum of 300 dpi (dots per inch, 1 inch = 2-54 cm). At what minimal distance z should a
u
printed page be held so that one does not see the individualdots ?
ks
Ans. Here, angular resolution of human eye, (j) = 5-8 x 10^ rad.
YYouro
soo

The linear distance between two successive dots in a typical photoprinter is


2-54
BBook

/ = cm = 0-84 X 10"^ cm.


300
r ee

At a distance of z cm, the gap distance / will subtend an angle


ouru

/ 0-84 xl0~- cm
ad

/
z = — - 14*5 cm
5-8 X10*^
Yo

15. A Polaroid (/) is placed in front of a monochromatic source. Another polaroid (11) is placed in front
of this Polaroid (/) and rotated till no light passes. A third polaroid (III) is now placed in between (/)
d
Re

and (//). In this case, will light emerge from (II). Explain.
iYn

Ans. As per the given question, monochromatic light emerging from polaroid (I) is plane polarized. When
FFind

Polaroid (II) is placed in front of this polaroid (I), and rotated till no light passes through polaroid (II), then
(I) and (11) are set in crossed positions, i.e., pass axes of I and II are at 90°.
When a third polaroid (HI) is placed inbetween (I) and (II), no light will emerge from (II), if pass axis of
(III) is parallel to pass axis of (I) or (II). In all other cases, light will emerge from (II), as pass axis of (II)
will no longer be at 90° to the pass axis of (III).

SHORT ANSWER QUESTIONS


16. Can reflection result in plane polarised light if the light is incident on the interlace from the side with
higher refractive index ?
Ans. Yes, refiection can result if the light, is incident on the interface from the side with higher refractive index.
As is known, polarization by reflection occurs when angle of incidence is the Brewster’s angle (i^), i.e..
>h
tan i
P n
where > «]●
1
10/86
’4. Fundamental Physics (XQ) VOL.II

When light travels in such a medium, the critical angle c is such that
1
= ii
smc = -
n
where «2 >
"2

As I tan I > I sin c I for large angles, therefore, ip>c


Hence polarization by reflection shall occur definitely.
17. For the same objective, find the ratio of the least separation between two points to be distinguished
by a microscope for light of 5000 A and electrons accelerated through 100 V used as the illuminating

w
substance.

Ans. The limit of resolution of a microscope is d = —-—


2sin0

roow
e
where 2 0 is the angle of cone of light rays entering the objective of the microscope.

re
For light, X,| = 5000 A
12-27 .

reF
For electrons accelerated through 100 V, de Broglie wavelength, Xj =

uFFll
= 1-227 A.
-Jm

e
d] _ \ _ 5000

sFr
As (2 sin 0) is same in the two cases, therefore, — = 4075
^2 ^2 " 1*227

foro
uor
fk
18. Consider a two slit interference arrangement, Fig. 10(N).6 such that the distance of the screen from
the slits is half the distance between the slits. Obtain the value of D in terms of X such that the Hrst
okso
minima on the screen falls at a distance D from the centre O.
Y
Yo
Ans. As is clear from Fig. 10(N).6,
oo
BB

FIGURE 10(N).6
T2P = T20 + OP = S2C + OP = D + X
TiP = OT^-OP=CSi-OP = D-x Si Ti
rYree

P
Now, = .^5,r,2 + r,/>2 = [Z)2 + (£> _;p)2jl/2 Source
ouu

O
S C
ad
Ydo

and 52^ = = [D^ +(D + x)2]>/2 S2 T2


^ Screen
OP = x
nidn

Path diff. between the waves reaching P from 5| and S2 CO = D


Re

= 52P-5jP S-jC = CS2 - D


= [£>2 + (Z) -f a:)2]‘/2 _ [£)2 + (£) _ ;p)2jl/2
F
Fi

For first minimum to fall at P, n = 1


1 X
Path diff. = 52P-5,P=-

or [D2 + (£> + JC)2] 1/2


2

If x = D

[D^ + 4D^f^-D = -
2

D(V5-1) = | or D (2-236-l) = X/2 or £> = —^


2-472
WAVE OPTICS 10/87

LONG ANSWER QUESTIONS

19. Fig. 10(N).7 shows a Cwo slit arrangement with a source which emits unpolarised light. Pisa polariser
with axis whose direction is not given. If Iq is the intensity of the principal maxima when no polariser
is present, calculate in the present case, the intensity of the principal maxima as well as of the first
minima.

Alls. As is known, resultant amplitude is the sum of amplitudes of either FIGURE 10(N).7
beam in perpendicular and parallel polarization,
A=A + A

oww
i.e.,
perp. parallel
Si
Now A — A* + A^ 1
perp. ~ ^»rp. perp.
0 0 S<-
= A
perp.
sin {kx - cor) + A perp. sin (kx~(£tt + (J))

e
A — + A^ P

re
Similarly, ^parallel ~ parallel parallel S2

FFrllo
= ^Iparallel doc~(at + if))

reF
e
Intensity = a^perp. + A^parallel
„ , = l'^e”rp. ~ + S'"" + *!>)] average
uoru
osFr
I
= [A°^ ,] — 2 (l + cosi|»)

fkfor
perp. parallel 2 y

q2
okso
Q2
I A
As
perp. average “ ' parallel ' average , therefore,
Y
Yo
0^
oo
BB

Without P, Intensity = 2 lA perp 1(1 + COStj)) ...(0

With polariser P, suppose A^perp is blocked.


Y
r ree
ouu

1
|2xi
Intensity = a^ + A.2parallel 2 ●●●(«)
ad
Ydo

arallel

We are given that without polarizer, intensity of principal maximum is


nidn

...{in)
Re

From (ii), intensity of principal maximum with polariser would be


F
Fi

/= =|'< - n>.

Using {Hi), we gel /=if^ =1/ 0


2 4 8
V y

Again, intensity at first minima with polarizer [From (ii)]

X—
1 V4 I,
2 2 2 8

20. A small transparent slab containing material of p = 1*5 is placed along A52, Fig. 10(N).8. What will
be the distance from O of the principal maxima and of the first minima on either side of the principal
maxima obtained in the absence of the glass slab.
AC = CO= />, SjC = S2C = d«D

*
10/88 “Pn/uieep, 4- Fundamental Physics (XII) VOL. 11

Ans. As is clear from Fig. 10(N).8, path difference between waves reaching Fj from A is
= 2d sin 0 + (|i, - 1) /.
For principal maximum^ path diff. = 0,
i.e.. 2</sin0 + (^-l)/ = O

1
2dsin0 + (l-5-l) - = 0.; sin0 = —
-1

4 16

/
1 ^
OPi = (CO) tan 0 D
I 16J
For the first minimum, at angle 0|, say,
path difference = 2 d sin 0| + 0*5 / = ± A/2

ww
±A./2-05/
sin 0| =
2d

±A./2-A/8

Flo
1
As diffraction occurs when d = X, sin 0 1 - =±i-
2A. 4 16

e
ree
3 ' 1 _J__-5
on the positive side, sin 0, =+--— = —
16 ; on the negative side, sin 0j = —4

Fr
1 4 16 16 " 16

rF
The first principal maximum on the positive side is at distance (above O)
uurr
= Dtan0i =D
sin0 1 jD.3/16s for
3D
kks
On the negative side, distance of first principal maximum (below O) will be
Yo
oooo

= Dtan 0/
Dsin0j' D (-5/16) -5D
1 =
eB

.^l-sin2 0j' ^1-(5/16)2 ^162-52


21. Four identical monochromatic sources A, B, C, D 2& shown in the Fig. 10(N).9 produce waves of
ur

the same wavelength A, and are coherent Two receivers and R2 are at great but equal distances
ad

fromB.
YYo

(i) Which of the two receivers picks up the larger signal ?


(ii) Which of the two receivers picks up the larger signal when B is turned off ?
dd

(iii) Which of the two receivers picks up the larger signal when D is turned off ?
Re
in

(iv) Which of the two receivers can distinguish which of the sources Bor D has been turned off ?
F

Ans. In Fig. 10(N).9, A, B, C, D are four identical mono FIGURE 10(N).9


chromatic coherent sources producing waves of same
. ¥R2
wavelength A,.
AB = BC = BD = A/2.
Rl and /?2 are two receivers held as shown such that =
R2B = d (» X). d
Let the wave at Rj because of source A be
= a cos (Of

Path diff. of signal at from B and A = A/2 so that phase


diff. is 7C.
R., A ^ B A/2 C
.●. The wave at because of source B is K d
yg-a cos (cof - it) = - a cos (Of
A/2
Similarly, path diff. of signal at /?i from C and A = A, so that
phase diff. = 2 k D

Ih <1
WAVE OPTICS 10/89

The wave at /?, because of source C is


= a QOS (jaa - 2 n) = a cos tor
Again, path diff. of signal at from D with that of A is

/ n1/2 \
I? ^2
= d 1 +
(V d»X)
' V y V I 2 2

Therefore, phase diff. is jc

oww
yj) = a cos (0)/ - It) = - a cos ©r
Hence the signal picked up at /?, from all the four sources is

)'/?j ~ y a'^ yB'^ =a cos (Ot-a cos (Ht + a cos m-a cos ©/ = 0. .(/)

e
Let the signal picked up at R2 from B be yg = Uj cos oat

re
Path difference of signals at /?2 from D to that from B is X/2. Therefore, y^ = Uj cos (oat - n)

FFrllo
= -ai cos oat.

reF
e
Again, path diff. of signals at /?2 from B and A is
ouru
osrF
sl/2
I f >2
= -yJd^ + (X/2f-d = d 1 + d ^
-‘‘ = 17^
As d»X, therefore this path diff. ffor
k
0, and phase diff. ([) -» 0 y^^^i cos (oat - <j>)
kso
Similarly,
ooo
Yo
Y
Hence the signal picked up at /?2 from all the four sources is
BB

>'/?2 “ >^>1 -^Z) = fli cos (©r - <j>) + flj cos ©/ + cos (©/ - 0) - cos ©r
r ree
Y

yp^ =2<2jCOS(©t-<l)) .(«)


uu
ad

Hence receiver R2 picks up the larger signal,


Ydoo

(ii) If source B is switched off, then from (/),


nidn

fl2
= a cos oat < /„ > = fl2 < cQgZ (Of > =
Re

1
2
F
Fi

a^ a2
From (ii). </ = a?1 < cos2 ©r > = -1-
y^ = a, cos (©/ - (j)) = flj cos ©t, (]) -> 0 «2 ^ 2 2

Hence /?| and Rj pick up the same signal.

(Hi) When source D is switched off. y^^ 1 =acos©/

2
a
and = 3a cos oat
</
^*2 (Taking (j) -> 0)
/!, > - 2
<I
> = 9a^ < cos^ iat> = ^a^ picks up larger signal compared to Rj.
(iv) Hence a signal at R^ indicates that source B has been switched off and an enhanced signal at /?2
indicates that source D has been switched off.
10/90 Fundamental Physics fXinPZSWn
22. The optical properties of a medium are governed by the relative permittivity (e^ and relative

permeability The refractive index is defliied as = n . For ordinary material > 0 and |X^
> 0 and the positive sign is taken for the square root. In 1964, a Russian scientist V. Veselago postulated
the existence of material with < 0 and ji.^ < 0. Since then such 'metamaterials’ have been produced
in the laboratories and their optical properties studied. For such materials « = . As light
enters a medium of such refractive index the phases travel away from the direction of propagation,
(i) According to the description above show that if rays of light enter such a medium from air (refractive
index = 1) at an angle 9 in 2"^ quadrant, then the refracted beam is in the quadrant,
(ii) Prove that Snell’s law holds for such a medium.
Ans. Suppose the given postulate is true.

ww
In that event, two parallel rays entering such a medium from air (retractive index = 1) at an angle 6,- in 2nd
quadrant, will be refracted in 3rd quadrant as shown in Fig. 10(N).10(fl)

FF loo
FIGURE 10(N)

ree
F rFee
oor r
rur
k s ff
YYoou
ookos
BBo
re

Let AB represent the incident wavefront and DE represent the refracted wavefront. All points on a wavefront
ouur

must be in same phase and in turn, must have the same optical path length.
ad
Yo

^e = bc- FFr~ or
BC = ,J^^ (CD-AE)
dY

If BC > 0, then CD > AE, which is obvious from Fig. 10(N). 10(o).
Re
idn

Hence the postulate is reasonable.


FFin

However, if the light proceeded in the sense it does for ordinary material, (going from 2nd quadrant to 4th
quadrant) as shown in Fig. 10(N).I0(/>), then proceeding as above,

^E=BC- CD or (CD-AE)

As AE > CD, therefore. BC < 0 which is not possible. Hence the given postulate is correct.
(i7) In Fig. 10(N). 10(a), BC = AC sin 8, and CD - AE = AC s\n 8,

As
BC = ^e^\i^ (CD-AE)
AC sin 0,- = |i^ AC sin 0^

sin 0..
■FTr^
t —
or = n
, which proves Snell’s law.
sin 8^
WAVE OPTICS 10/91

23. To ensure almost 100 per cent transmittivity, photographic lenses are often coated with a thin layer
of dielectric material. The refractive index of this material is intermediated between that of air and
glass (which makes the optical element of the lens). A typically used dielectric film is Mgp2
(n - 1*38). What should the thickness of the film be so that at the centre of the visible spectrum
(5500 A) there is maximum transmission ?
Ans. In Fig. 9(N).ll, we have shown a dielectric film of thickness d deposited on a glass lens. Refractive index
of film = 1-38 and refractive index of glass = 1-5.
Given, \ = 5500 A.
A ray is incident on the film at angle i. It is reflected partially along AB (as rj) and refracted partially along
AD in the film at angle r.

ww
FF loo
ree
reFe
oroFr
rur
The refracted ray is reflected partially from film glass interface along DC and partially transmitted through
s ff
the glass lens. At C. the ray is reflected partially in the film and refracted into air as r~, parallel to r,.
k
As amplitude of wave goes on decreasing during successive reflections, therefore, rays rj and rj dominate
YYouo
okso

the behaviour.
BBoo

For maximum transmission through the centre of the lens, i-j and r2 should interfere destructively.
r ee

As both the reflections at A and D are from lower to higher refractive index, there is no phase change on
reflection.
ad
ouur

The optical path difference between and r, = n {AD + CD) - AB ...(0


Yo

DE _ d
In AADE, cos r =
AD ~ AD
d
Re
idnY

d
FFin

AD =
cos r

Similarly, CD = -^
cosr

Again, tan /● =
AE 2"^
DE d

AC = 2d tan r

AB
InAA^C, sini =
AC

AB - AC sin / = 2d tan r sin i


10/92 7>%eideefi^ '<*. Fundamental Physics (Xn) VOL.II

Putting in (i), we get Optical path dilference between r>i and T|

x = n '2d_^ -2</tanrsini
cosr^

sin/ 2d sinr .
-2d sin/
sinr cosr COST

0 \
1 - sin^ r
= 2d sin /
sm rcosr

ww
sin/
x = 2d cos r =2dn cos r
sin r

The waves K2 and r, will interfere destructively when their path difference

Flo
X X X

e
X = —
i.e.. 2 dn cos r= — dn cos r - — m

eree
2 ’ 2 4

For photographic lenses, the sources are normally in vertical plane. i = r = (y

FFr
uurr X X 5500 A
d = =^1000A

orr
From (//), dn cos 0“ = —
4 4/1 "4x1-38
sfo
kks
Yoo
oooo
eBB
urr
ad
YYo
dd
Re
inn
F
WAVE OPTICS 10/93

m1

i-. r I'A
ft
ii

i U NEET/JEE
SPECIAL

For ultimate preparation of this unit for competitive examinations, students should refer to

ww
● MCQs in Physics for NEET ● V';
Pradeep's Stellar Series.... ● MCQs in Physics for JEE (Main) ^
Ml
separately available for these examinations,

Flo
e
Multiple Choice Questions (with one correct Answer)

e
reer
rFF
I. Interference of Light 4. A Young’s double slit interference arrangement
with slits 5| and S2 is immersed in water
uur r
1. On a hot summer night, the refractive index of (refractiveindex = 4/3) as shown in Fig. lO(CF).1.
air is smallest near the ground and increases with
height from the ground. When the light beam is are given by
ffoor
The positions of maxima on the surface of water
np- - S, where X is the
sks
directed horizontally, the Huygen’s principle leads wavelength of light in air (refractive index = 1),
YYoo
ooko

us to conclude that as it travels, the light beam 2 fr is the separation between the slits and m is an
{a) becomes narrower interger. The value of p is
eBB

(b) goes horizontally without any deflection FIGURE 10(CF).1

(c) bends downwards


uurr

(d) bends upwards (JEE Main 2015) Si


ad
Yo

2. In a YSDE experiment the wavelength of light d P Air


X

u.sed is 5890 A. The separation between the slits d iWater::r


dY

is 0-5 mm and the distance between the slits and


Re

screen is 0-5 m. Find the distance between 1st S2


innd

and 3rd bright fringes on the screen?


FFi

(a) 589 pm (b) mi pm


(a) I (b)2
(£●) 1178 pm {d) None of these
(c)3 id) 4
(JEE Main 2021) (JEE Advanced 2015)
3. In a double slit experiement, the two slits are 5. A light source, which emits two wavelengths
1 mm apart and the screen is placed 1 m away. A Xj = 400 nm and X2 = 600 nm, is used in a Young’s
monochromatic light of wavelength 500 nm is double slit experiment. If recorded fringe widths
used. What will be the width of each slit for
for X] and X2 are [5| and P2 and the number of
obtaining ten maxima of double slit within the fringes for them within a distance y on one side
central maxima of single slit pattern of the central maximum are W] and m2, respec
(a) 0-1 mm ib) 0-5 mm tively, then which of the following is incorrect ?
(c) 0-02 mm id) 0-2 mm (a) P2 > P 1 (b) in I > ni2
ANSWERS

1. id) 2. (c) 3. (d) 4. (c)


10/94 T^euUejk'A Fundamental Physics (XII)SS2HD
(c) From the central maximum, 3rd maximum of nX nX
^2 overlaps with 5th minimum of Xj (c) sin- (d) cos
2d 2d
(d) The angular separation of fringes for Xj
is greater than 10. A ray of light of intensity I is incident on a parallel
glass-slab at a point A as shown in Fig. 10(CF).3.
(JEE Advanced 2014, Paper 1)
It undergoes partial reflection and refraction. At
6. Two coherent sources of intensity ratio P interfere. each reflection 25% of incident energy is
Tlien the value of
reflected. The rays AB and A'B' undergo
(/,max max + ^min) ‘S : interference. The ratio is :

I+P 1+P
(i)
VI p J
1 +P 2^0

ww
(d)
1 +P
7. A transparentsheet of refractive index 1.5 is kept

Floo
near one of the slits of the YDSB apparatus. The
intensity at the centre of the screen (where the

ee
central maximum was located before the
introduction of the sheet) is half of the previous

eer
value. The minimum thickness of the sheet should

FFr
be (wavelength of the monochromatic light used
in the experiment is 6000 A):

oorr
uur r
(a) 2x10“"^ m (b) 7.5 X 10"^ m (a) 4 : 1 (/7) 8 : 1
s ff
(c) 3x10“^ m (^0 2 X 10-’ m (c) 7 : 1 (d) 49 : 1.

8. Young’s double slit experiment is first performed in 11. In a double slit experiement, the two slits are
sk
YYoo
air and then in a medium other than air. it is found 1 mm apart and the screen is placed 1 m away. A
ooko

that 8^^ bright fringe in the medium lies where 5


ih
monochromatic light of wavelength 500 nm is
eBB

dark fringe lies in air. The refractive index of the used. What will be the width of each slit for
medium is nearly obtaining ten maxima of double slit within the
(a) 1-25 (b) 1-59 central maxima of single slit pattern ?
uurr

(c) 1-69 id) 1-78 (NEET2017) (a) 0-1 mm (b) 0-5 mm


ad

9. A beam with wavelength X falls on parallel (c) 0-02 mm (d) 0-2 mm


Yo

reflecting planes with separation d. Fig. 10(CF).2. (AIPMT 2015)


The angle 6 that the beam should make with the
12. In the ideal double slit experiment, when a glass
dY

planes so that the beams reflected from successive


Re

planes may interfere constructively is (where n = plate (p = 1-5) of thickness t is introduced in the
ind

path of one of the interfering beams (wavelength


FFin

1,2, ...):
X.), the intensity at the position where the central
maximum occurred previously remains unchan
ged. The minimum thickness of the glass plate is

(a) 2X (b) X
2X
(c) X/3 id) -

13. In a Young’s double slit experiment, slits are


separated by 0-5 mm, and the screen is placed
150 cm away. A beam of light consisting of two
(nX ' fiX wavelengths, 650 nm and 520 nm, is used to
(a) sin (b) tan
d d obtain interference fringes on the screen. The least

5. id) 6. id) 7. (cl 8. id) 9. (c) 10. id) 11. id) 12. (n)
WAVE OPTICS 10/95

distance from the common central maximum to


/_ f 1
the point where the bright fringes due to both the
wavelengths coincide is
(c) -f 1 + 4 cos f (d)
2)' ' 9
1 + 8 cos
2

(a) 9-75 mm (b) 15-6 mm (AIEEE 2012)


(c) 1-56 mm (d) 7-8 mm 19. In Young’s double slit experiment, the slits are
(JEE Main 2017) 2 mm apart and are illuminated by photons of two
14. n identical waves each of intensity Iq interfere wavelengths X] = 12000 A and X2 = 10000 A. At
each other. The ratio of maximum intensities if what minimum distance from the common central

interference is (/) coherent and (//) incoherent is bright fringe on the screen 2 m from the slit will
a bright fringe from one interference pattern
(a) n (i>) coincide with a bright fringe from the other ?
1 (a) 3 mm (h) 8 mm
(c)

ww
id) — (c) 6 mm (d) 4 mm
n

(AIPMT 2013)
15. In Young’s double slit expt., the intensity is / at a
point where the path difference is X/6, where X is 20. The intensity at the maximum in a Young’s double

Floo
wavelength of light used. If Iq denotes the slit experiment is Iq. Distance between two slits
isd = 5k, where X is wavelength of light used in
maximum intensity, then I/Iq is equal to

ree
the experiment. What will be the intensity in front
(a) 3/4 (b) I/V2 of one of the slits on the screen placed at a distance

rFee
D= lOrf?
1
(c) V3/2

F
(d) - (AIEEE 2007) I

oor r
3
rur
16. In a double slit experiment, when light of w i'o
s ff
wavelenght 400 nni was used, the angular width
(c) IqI2 /o (NEET 2016)
of first minimum formed on a screen placed 1 m
osk
21. Two coherent sources produce waves of different
YYoou
away, was found to be 0-2°. What will be the
intensities which inteiiere. After interference, the
oook

angular width of first minimum if entire apparatus


ratio of maximum intensity to minimum intensity
eBB

is immersed in water (|i = 4/3). is 16. The intensities of waves are in the ratio
(a) 0-266° (b) 0-15°
(a) 16:9 (b) 25:9
(c) 0-05° (d) 0-1° (NEET 2019)
uur r

(c) 4 : 1 (d) 5:3


ad

17. At two points P and Q on screen in Young’s (JEE Main 2019)


Yo

double slit experiment, waves from slits 5| and 22. Two coherent sources of light interfere and
X
^2 have a path difference of 0 and — respectively, produce fringe pattern on a screen. For central
dY

4 maximum, the phase difference between the two


Re
idn

the ratio of intensities at P and Q will be waves will be


FFin

ia) 3:2 (b) 2: \ (a) n/2 {b) zero


(C) 71 (d) 3 k/2 (NEET 2020)
(c) V2:l {d)A:\ (AIEEE 2011)
23. In Young’s double slit experiment, if the
18. In Young’s double slit experiment, one of the slit separation between coherent sources is halved,
is wider than other, so that the amplitude of the and the distance of screen from coherent sources
light from one slit is double of that from other is doubled, then the fringe width becomes.
slit. If 7„, be the maximum intensity, the resultant (fl) half {h) four times
intensity / when they interfere at phase difference
(c) one fourth (d) double
(j) is given by
(NEET 2020)

ia)
-^(4 + 5 cos P) (6) 1 + 2 cos
2
24. The two light beams having intensities / and 9 I
to produce a fringe pattern on a screen. The phase
ANSWERS

13. (</) 14. (a) 15. (a) 16. (b) 17. Xb) 18. id) 19. (c) 20. (c) 21. (b)
22. ib) 23. (b)
10/96 ‘Pna^eUe^'A Fundamental Physics (XI1)EEI5D
difference between the beams is 7t/2 at point P and
K at point Q. Then the difference between the (a) io
2
ib) ^ 4
resultant intensities at P and Q will be :
I /
(a) 21 W6/ 0 0
(0 ^ (d) TT
16
(NEET 2017)
8
(0 5/ {d)ll
(JEE Main 2022) 30. At the first minimum adjacent to the central
maximum of single slit diffraction pattern, the
25. In a Young’s double slit experiment, light of
phase difference between the Huygen’s Wavelet
500 nm is used to produce an interference pattern.
from the edge of the slit and the wavelet from the
When the distance between the slits is 0 05 mm,
midpoint of the slit is
the angular width (in degree) of the fringes formed
7t
on the distant screen is close to
(a) — radian (b) — radian
4
(a) 0 07° (b) 0-17“

ww
71
(c) 1-7° (d) 0-527° (c) — radian (d) K radian
2
(JEE Main 2020)
(RE-AIPMT 2015)

Floo
n. Diffraction and Polarization of Light 31. Unpolarised light is incident from air on a plane
surface of a material of refractive index At a

ee
26. Assuming human pupil to have a radius of
0-25 cm and a comfortable viewing distance of particular angle of incidence it is found that

reer
25 cm, the minimum separation between two the reflected and refracted rays are p>erpendicular

rFF
objects that human eye can resolve at 500 nm to each other. Which of the following options is
wavelength is correct for this situation ?
uur r
(a) 1 pm
(c) 100 pm
(b) 30 pm
(d) 300 pm
ffoor
(a) I = sin
-I
sks
(JEE Main 2015)
YYoo
(b) Reflected light is polarised with its electric
27. For a parallel beam of monohromatic light of
ooko

vector perpendicular to the plane of incidence


wavelength X, diffraction is produced by a single
eBB

slit whose width ‘a’ is of order of wavelength of (c) Reflected light is polarised with its electric
light. If 'D' is the di.stance of the screen from the vector parallel to the plane of incidence
slit, the width of central maxima will be.
1 j
uurr

-1
DX Da
{d) i - tan (NEET 2018)
ad

(a)
Yo

a
32. Two white dots are 1 mm apart on a black paper.
2Da 2DX They are viewed by eye of pupil diameter 3 mm.
dY

(0 id) What is the maximum distance at which these dots


X
Re

a
can be resolved by the eye ? A. = 500 nm.
ind

(AIPMT 2015)
FFin

(a) 1 m (b) 5 m
28. A parallel beam of light of wavelength 5000 A is
incident normally on a single stil of width 0-001 (c) 3 m (c/) 6 m
mm. The light is focussed by a convex lens on a 33. A beam of unpolarised light of intensity Iq is
screen placed in focal plane. The first minimum passed through a polaroid A and then through
is formed for the angle of diffraction equal to another pohuoid B which is oriented so that its
(a) 0“ (b) 15“ principal plane makes an angle of 45“ relative to
(c) 30“ (d) 60“. that of A. The intensity of emergent light is
29. Two Polaroids Pi and P2 are placed with their («) k (b) /q/2
axis perpendicularto eachother.Unpolarisedlight (d) /()/8 (JEE 2013)
iO /q/4
/q is incident on A third polaroid P^ is kept in
34. Two beams, A and B of plane polarized light with
between P| and P2 such that its axis makes an
angle 45“ with that of P[. The intensity of mutually perpendicular planes of polarization are
transmitted light through P2 is seen through a polaroid. From the position when
ANSWERS

24. (b) 25. (d) 26. (/?) 27. (d) 28. (c) 29. (c) 30. (d) 31. {b) 32. (h) 33. (c)
WAVE OPTICS 10/97

the beam A has maximum intensity, (and beam B 37. The Brewster angle for an interface should be
has zero intensity), a rotation of polaroid through (a) 30° < ii, < 45' (h) 45° < i,, < 90°
30° makes the two beams appear equally bright. (c)/,, = 90° id) 0° < < 30°
If the initial intensities of the two beams are (NEET 2020)
and respectively, then equals. 38. Visible light of wavelength 6000 A falls normally
(a) 1 (h) 1/3 on a single slit and produces a diffraction pattern.
It is found that .second diffraction minimum is at
(c) 3 (d) 3/2
60° from the central maximum. If the first
(JEE Main 2014)
minimum is produced at 0j, then 6| is close to
35. Unpolarized light of intensity I passes through an (n) 20' ih) 45'
ideal polarizer A. Another identical polarizer B is (c) 30' id) 25° (JEE Main 2020)
placed behind A. The intensity of light beyond B
39. A polariser-analyser set is adjusted such that the
/
is found to be —. Now another identical polarizer intensity of light coming out of analyser is just

ww
10% of original intensity. Assuming that there is
C is placed between A and B. The intensity beyond no stray absorption of light, the angle by which
the analyser needs to be rotated further to reduce

Floo
B is now found to be 8
The angle between the output intensity to zero is
polarizer A and C is (a) 28-4° (h) 71-6°

ee
(c) 90° (d) 45° UEE Main 2020)
(a) 0° ih) 30°

eer
40. In a young’s double slit experiment, a student
(c) 45° id) 60°
observes 8 fringes in a certain segment of screen

FFr
(JEE Main 2018) when light of wavelength 600 nm is used. If the

oorr
uur r
36. The angular width of the central maximum in a wavelength of light is changed to 400 nm, number
single slit diffraction pattern is 60°. The width of of fringes he will observe in the same region of
s ff
screen is
the slit is 1 pm. The slit is illuminated by
monochromatic plane waves. If another slit of (a) 6 (b)^
sk
YYoo
same width is made near it. Young’s fringes can (c)9
ooko

id) 12 (NEET 2022)


be observed on a screen placed at a distance 50 cm 41. In free space, an elcctromagentic wave of 3 G Hz
eBB

from the slits. If the observed fringe width is 1 cm, frequency strikes over the edge of an object of
what is slit separation distance ? size X/lOO, where X.is the wavelength of the wave
ii.e., distance between the centres of each slit.) in free space. The phenomenon which happens
uurr

there will be :
(a) 25 pm ih) 50 pm
ad

(fl) Reflection ih) Refraction


Yo

(c) 75 pm id) 100 pm


(c) Diffraction (d) Scattering
(JEE Main 2018)
(JEE Main 2022)
dY

m
Re

Multiple Choice Questions (with One or More than One Correct Answers)
ind
FFin

42. If the source of light used in Young’s double slit minimum of the diffraction pattern, the phase
experiment is changed from red to violet: difference between the rays coming from the two
(a) the fringes will become brighter. edges of the slit is :
(h) con.sccutive fringes will come closer, (a) zero ih) k/2
(c) the intensity of minima will increase. (c-)K (d)2n
id) the central bright fringe will become a dark 44. Young’s double slit experiment is performed using
fringe. a source of white light,
43. A parallel monochromatic beam of light is (fl) The central fringe will be white
incident normally on a narrow slit. A diffraction ib) There will be no completely dark fringe
pattern is formed on a screen placed perpendicular (c) The fringe next to central fringe will be violet
to the direction of the incident beam. At first id) The fringe next to central fringe will be red.
ANSWERS
34. ib) 35. ic) 36. ia) 37. ib) 38. id) 39. ia)
40. id) 41. id) 42. ih) 43. ic) 44. ia,b.c)
10/98 “Pmuiccfr 'a Fundamental Physics (XII) orsTWTl

nil Multiple Choice Questions (Based on the given Passage/Comprehens ion)


Each comprehension given below is followed by some multiple choice questions. Each question has one
correct option. Choose the correct option.

iG.ompreffensimil When waves from two co 45. The ratio of slit widths of the two sources is
herent sources, having amplitudes a and b («) 9 : 1
superimpose, the amplitude R of the resultant
wave is given by (b) 81 : 1
(c) 1 : 9
R - -yja^ +b^ + lab cos^ id) 1 : 81

w
where is the constant phase angle between
the two waves. The resultant intensity I is 46. The ratio of maxima and minima in the
directly proportional to the square of the interference pattern is

Flo
amplitude of the resultant wave, i.e., I« R^,
(a) 9 : 1
i.e., I ix (a^ + b^ + lab cos ((►)

ee
For constructive interference, ^ = 2 n n, (6) 81 : 1

Fr
(c) 25 : 16
For destructive interference, (|) = (2 n - 1) jc id) 16:25

for
ur
If 11,12 are intensities of light from two slits 47. If two slits in Young’s experiment have width ratio
I, 0)
1 a
2 I : 4, the ratio of maximum and minimum
of widths CO,1 and 0)i, then — = ~
ks
b^ intensity in the interference pattern would be
a>2
Yo
ia) 1:4
oo

Light waves from two coherent sources of


intensity ratio 81 : 1 produce intererence. ib) I : 16
eB

With the help of the passage given above, Cc) 9 : 1


choose the most appropriate alternative for
id) 9:16
ur

each of the following questions :


ad
Yo

m
nd

Matching Type Questions


Re
Fi

DIRECTIONS. In each of the following questions, match column I and column II and select the correct
match out of the four given choices.

48. Match correctly column I and column II :


Column I Column II
(A) Interference of light (P) / = /q cos^ 0
(B) Brewster’s Law iq) Obstacle/aperture of size * 2,
(C) Diffraction of light (r) p = tan ip
(D) Law of Malus is) Coherent sources

ia) A-r, B-5, C-q, D-p ib) A-p, B-q, C-r, D-.? (c) As, B-r, C-p, D-q id) As ; B-r ; C-q ; D~p

EM.'
45. (h) 46.(c) 47. (c) 48. (</)
WAVE OPTICS 10/99

□ Matrix-Match Type Questions p q s


—1 r~—n f ! I

DIRECTIONS. Each of the following questions contains statements given in A © @ O!©


two columns which have to be matched. The answers to these questions have to
be appropriately bubbled. If the correct matches are A-r, As ; ; C-p, C-s
B
!©!|© Ol©
and D-q, then the correctly bubbled matrix will look like the one shown here:
c © @ OlO
49. Column I Column II
D
© © ©I©
(A) Interference (P) Corpuscular theory A B C D

(B) Compton effect (9) Longitudinal waves


(C) Diffraction (r) Transverse waves ® © ® ®

ww
(D) Polarization (s) Dual theory ® O 0 o
VI. Integei "'ypr Questions ® ® ® ®

Floo
® ® ® ®
DIRECTIONS. The answer to each of the following questions is a single digit

ree
integer, ranging from 0 to 9. If the correct answers to the question numbers A, B,
0 0 0 0
C and D (say) are 4, 0, 9 and 2 respectively, then the correct darkening of bubbles 0

rFee
should be as shown on the side :
© ®© ®

F
oor r
rur
50. A man wants to distinguish between two pillars located at a distance of 11 km. What
©0® ©
s ff
should be the minimum distance between the pillars ?
51. In Young’s double slit experiment, the widths of two slits are in the ratio 4:1. The ratio © ©© ®
osk
YYoou
of maximum and minimum intensity in the interference pattern will be :
© ©® ©
oook

VII. Assertion-Reason Type Questions


eBB

FOR MEDICAL STUDENTS Reason. Objective lens of large diameter collects


uur r
ad

more light,
DIRECTIONS. The following questions consist of
Yo

(fl) A (b) B (c) C id) D


two statements each, printed as Assertion and
Reason. (AIIMS 2005)
dY

53. Assertion. The clouds in the sky generally appear


Re

While answering these questions you are required to


idn

to be whitish.
choose any one of the following four responses.
FFin

Reason. Diffraction due to clouds is efficient in


A. If both, Assertion and Reason are true and the
Reason is the correct explanation of the equal measures its all wavelengths.
Assertion. (a) A ib) B (c) C id) D
B. If both. Assertion and Reason are true but (AIIMS 2005)
Reason is not a correct explanation of the 54. Assertion. Colours can be seen in thin layers of
Assertion. oil on the surface of water.

C. If Assertion is true but the Reason is false. Reason. White light is composed of several
colours.
D. If both. Assertion and Reason are false.
(a) A ib) B (c) C (d) D
52. Assertion. The resolving power of a telescope is
55. Assertion. No diffraction is produced in sound
more if the diameter of the objective lens is more.
waves near a very small opening.

ANSWERS

49. A-q,,rs ; B~p.s ; C-q.r.x ; D-nr 50. (3) 51. (9) 52. (a) 53. (c) 54. {h)

i
10/100 Fundamental Physics fXHIPTSTWTl

Reason. For diiTraction lo take place, the aperture (B) Statement-1 is true, Statement-2 is true, but
of opening should be of same order as wavelength Stalement-2 is not a correct explanation of
of waves,
Statement-1.
(a) A (h) B (c) C (d) D (C) Statement-1 is true, Statcment-2 is false.
56. Assertion. Light shows the phenomena of (D) Statement-1 is false, Statement-2 is true.
interference, diffraction and polarisation.
58. Statement*!. Energy is created during
Reason. Because light behaves as corpuscles.
constructive interference and it is destroyed during
ia) A {b) B (c) C (d) D destructive interference.

57. Assertion. All bright interference bands have Statement-2. Because in constructive
same intensity. interference, the fringes are bright and in
Reason. Because all bands do not receive same destructive interference, the fringes are dark.

loow w
light from two sources. («) A (b) B (£●) C (d) D
(a) A {h) B (c) C (d) D 59. Statement-!. On viewing the clear blue portion
of the sky through a Calcite Crystal, the intensity
FOR ENGINEERING STUDENTS of transmitted light varies as the crystal is rotated.
Statement-2. The light coming from the sky is

ee
DIRECTIONS. Each of the following questions
polarized due to scattering of sun light by panicles

Fr
contains two statements. Read the statements and
in the atmosphere. The scattering is largest for
choose any one of the following four responses :
(A) Statement-1 is true. Statement-2 is true. State
r FF blue light.

rer
(a) A (b) B (c) C (d) D
ment-2 is correct explanation of Statement-1. fofr Fo (AIEEE 20!1)
u
ks
VIII. Multiple Choice Questions (Based on Experimental Skills)
YYouro
s oo
BBook

60. In an experiment, electrons are made to pass FIGURE 1Q(CF).5


r ee

through a narrow slit of width ‘d' comparable to fy


their de-Broglie wavelength. They are detected
ouru
ad

on a .screen at a distance D from the slit, see Fig.


Yo

10(CF).4. Which of the graphs in Fig. 10(CF).5 N-* d


can be expected to represent number of electrons
d
Re

N detected as a function of the detector position


iYn

y (y = 0 corresponds to the middle of the slit)


FFind

o o
FIGURE 10(CF).4 fy fy

N-4- d
-d >●=0

^ - - D ♦
O o

(AIEEE 2008)

ANSWERS

55. (fl) 56. (c) 57. (c) 58. (d) 59. (a) 60. (d)

i
WAVE OPTICS 10/101

For Difficult Questions

Multiple Choice Questions (W'th one correct Answer)

1. Consider a plane wave front travelling horizon Optical path length S2P = +d~
tally. As it moves, its different parts move with
for constructive interference A .r = m \
different speeds. So, its shape will change as
shown in Fig. 10(CF).6.
= mX
FIGURE 10(CF).6

llowow
40
= m X.
0>
3
flj - ►
<0
= 9 nr - d~
c

On comparing with .r = nr Xr - d~

ee
we get, /? = 3

Fr
XD
r FF 5. P = — => P2>Pi ('●' ^2 > X,[)
Light bends upwards.
2. Here, >. = 5890 X

er
m ; y
Also, => mj >m2
d=0-5x 10-3 m ;£) = 0-5 m forfFor
u
D
Distance between 1st and 3rd bright fringes is 3rd maxima of Xo lies at 3 (600 nm) x —
s
d
ok
YYour o
3XD XD
"3-"'= —“T = (1800 nm) —
so

5th minima of X,, lies at


eeBoBk

2XD 2x(5890xl0-*^)x0-5
D D
d 0-5x10-3 (2x5- 1), (400)x = (1800nm) —
r

2d
ouru
ad

= I178x 10-6 m= 1178 ^im B X


Yo

3. Here, d = 1 mm, D = 1 m, X, = 500 nm Angular separation

P ^ It is more for X>.


d

In double slit experiment, angular width = —


Re

D Therefore, choice (4) is incorrect.


in
FFind Y

XD
6.
XD . P __d_ _ X
-

as h
" D~ D ~d
2X.
In single slit experiment, angular width =
d'
I {a+b)--{a-b)~ Aab
max min _
According to Question, I
max mill {a + Z?)2 + {a- b)- 2{a^+b^)
IPX. _ 2X. or d' = 0-2 d [\- d = \ mm]
d ^ d' 2ab 2b.bS
d' - 0’2 mm (a^+b^ ~ b~+^h^ ~ I-l-p
7. As per the question,
4. Optical path length,
/0 7t
/.cos2^
= r^JTd^ =-^177^7
cos — = = cos —

S2P
0
2 2 2 s 4
3

1
10/102 <t Fundamental Physics (XII) r7ST»n
GdC'irCSuTlCACQ© ■)
/
For Difficult Questions I a~
_ / 16 fl _ 4
h " 4 91 ~ 9 " ~b~ 3
^ _ n K

2 4 or f|5=2-
2ti 71
/
max _ ja + b)^ _ (4+3)^ _ 49
As
mm (a-/?)2 (4-3)2 1
X 11. Here, d - I nim, D = 1 m, X = 500 nm
A.x = - = ([i-l)f
4 In double slit experiment,
X 6000x10-'^ P XD
t = = 3 X 10-’ m angular width = — as P =
4(^t-l) 4(1.5-1) D d

8. Let X^ be wavelength of light in air and be XD

ww
wavelength of light in medium.
As 8ih bright fringe in medium coincides with
1 = -^ = A
D D d
5th dark fringe in air.

FF loo
2X
D X,.a D In single slit experiment, angular width =
8?., - = (2x5-1) d'

ree
d 2 d

9 According to Question,
8 X...
m
— — Xa

reFe
9
IPX _ 2X or d' = 0-2 d
d ~ d'
oroFr
X..Cl 8x2
rur
16
It=- = 1-78

or or
X 9
s ff d' - 0’2 mm [●.● d = \ mm]
m

9. Refer to Fig. 10(CF).7. Path diff. between the 12. Path difference due to slab should be an integral
k
multiple of X.
YYouo
beams reflected from successive planes = AM
okso

+ AN = 2 c/ sin 6. i.e.. A.t=(p,-l)r = nX


BBoo

nX
t =
r ee

For minimum value of 7, n = 1


ad
ouur

IX
Yo

t = = 2X
1-5-1

13. Here, d = 0-5 mm = 0-5 x IQ-^


d

m.
Re
idnY

£> = 150 cm = 1-5 m


FFin

for constructive interference, 2 c/ sin 6 = n X


Xj = 650 nm X, = 520 nm, y = ?
For common maxima
nX nX
sin 6 = e =sm /ij Xj — ^<2 X^
2d 2d
n
1 _ X2 520 4
10. Intensity reflected along AB, /] = 1/4 «2 X, ” 650 ” 5
Intensity transmitted along AC = 3 1/4
d
1 3/ From .V — = n X
Intensity reflected along CA' = —x— = 3//!6 D

1 3/ 3/
D 4 X (650x10-^) 1-5
y = nX — =
Intensity reflected at A' = — X— = — d aSxlO"^
4 16 64
= 7-8 X lO"^ m = 7*8 mm
Intensity transmitted along
14. When interference is coherent,
A'B' = X IL-IL-IL
16 64 ~64 I
max = (^ + ^+-rttimes)2 =/i2/0
I
WAVE OPTICS 10/103

For Difficult Questions


I _ +A2 +2 A[ A2 cos (])
hn A^ + A2 + 2 A| A2
When interference is incoherent,
/' max = /q +/y +... n times - n I0 _ qq +(2ao)^ +2(ao)(2flQ) cos (f)
I
max
n^l 0 Oq + (2 fly)” + 2(flQ)(2aQ)
= n
nl 2
max 0 _ 5oq + 4 qq cos $
9^0-
15. As /R — /] + /2 ^ 2 I'2^ cos i|)
I 5 + 4cos(|) _ 5 + 4(2cos^(j)/2-l)
^ “ ^max = / + / + 2-^/x/ cos 0° = 4 / / 9 9
For path difference, X/6

ww
phase difference 0 =
360'
= 60”
/ = ^s.(i
9
+ 8cos^((,/2)
6
19. Here, d=l mm = 2 x 10 ^ m,

Floo
●. /'=/+/ + 2yfn cos 60”= 3/ = 12000 A, X^= 1000 A

e
Z> = 2 m, jc = ?

eere
Let n|th bright fringe from X| coincide with
L~'aJ~4
0
fi2th bright fringe from "Ko

FFr
16. In air, angular fringe width, Oy = |3/Z) = 0-2”. ^2 _ IQOOQ _ 5

oorr
n
uur r
1 _

In water, angular fringe width. fj] X] = fi2 ^2 or


s ff ~12000 6
0-2 For minimum distances, n[ = 5 and n~, = 6
= 0*15°
e„, = (3/iiD =
sk
YYoo
4/3
>.,Z>_5x(12000xI0"'*^)x2
ooko

17. At point P on the screen, path diff. between two 2x10-3


eBB

waves reaching there, x = 0. = 6xl0 3m = 6 mm


2n
Ar = 0. 20. Here, d = 5 X, I =1 D = \0 d,x = d/2
Therefore, phase diff. (j) =
X
uurr

xd
i -1^ = ^
ad

Path diff. —
Here, /] = /2 = Iq (say)
D 2'(\0d)~4
Yo

As / = /j + /2 + 2 cos ^ 2n X n
Phase diff. = * = —x- = - = 90”
dY

.’. Ip — /q + /y + 2 /g cos 0 — 4 Iq. ^ X 4 2


Re
nind

At point Q on the screen, path diff. between two ^2


waves reaching there x = X/4. Therefore phase
7
/ = /„cos^- = /
7 t ( cos —
^
FFi

diff.
0 2 ‘3 4

2n X _n = Lx
1 /
0
T^4 " 2 0
2
K 2
I
^G = ^0 + ^0 + 2/gCOS-=2/y 21. Let l _ a

2Iq 1
I
max ja + b)^ _ 16
/ . (a-b)^ ~ 1
IS* L^ct <A j — Qq ● ● <^2 ” 2 Qq
min

At phase difference (j), the resultant intensity a+b 4

would be / = A]^ + a| + 2 Aj A2 cos , and a-b 1

4a-4b = a-k-b
^max =^m= + A| + 2 Aj A2 ('.■ COS ([) = 1) 3a = 5b

«
10/104 ^>utcUc^'4. Fundamental Physics (XII)EEIBD

For Difficult Questions


1-22x500x10-^
d =
2x1 X
o _5 100
~b~3
d = 3 05 X 10 ^ ni = 30*5 ^im = 30 jim
I
I _
a 2 f5-\^ 25 X
27. As angular width, 0 = —
A b- \-
3 9 a

11. As is known from theory, for central maximum,


phase difference between two light waves = Zero
XD
23. As fringe width, P = d
, , therefore, when d is

w
halved and D is doubled, p would become four
times.

Flo
24. Here, /j = /, /2 = 9 /, Gj = ti/2, 63 = rt

As, / = /j + 73 + 2.^/j I2 cos 6

reeee
FFr
Ip = I +91 +2^1x91 cos 71/2 = 10/

Iq = I+91 + 2^Ix9I cos 71

for
ur
= 10/-6/ = 47
kkss
In Fig. 10(CF).9,
/p-/g= 107-47 = 6/
Yo
Linear width
oo

25. Here, A, - 500 nm = 500 x 10“^ m 20 =


arc

d = 0-05 mm = 5 x 10“^ m radius D


eB

Linear width of central maxima = 2 D 0


XD
P = 1DX
r

d width of central maxima =


ou
ad

P - ^ - 500x10"^ 28. Here, 1 = 5000 A = 5 x lO'^ m,


YY

Angular width of fringes = — a = 0-001 mm= 10“^ m


D~ d~ 5x10-5
ndd

For first minimum, (n = 1)


Re

180° fl sin 6 = n A.= 1 X.


= 10-2 rad =10-2 x = 0*527'
Fi

71
X 5x10-2
sin0 = — = 0-5, 0 = 30-
1-22^ a 10-6
26. Limit of resolution, d =
2 ^ sin 0
7()
29. Intensity of polarised light from

Intensity of polarised light from

^(cos2
2
45°)=^4
Now, angle between axes of P2 and P3
= (90° - 45“) = 45°
Intensity of polari.sed light transmitted from Pj
arc
0-25 _ 1
Now, 0 =
radius
* sin 0 =
25 ~100 ■^(cos45°)2=i
4 8
0

1
WAVE OPTICS 10/105

33. Here, polaroid A polarises light.


For Difficult Questions
/
30. Path difference between AP and MP for the first Intensity ol‘ polarised light from A =
minima
According to law of Malus,
A. Intensity of light emerging from B,
MP-AP = - (v «= 1)
2
/ / I I
FIGURE 10(CF).10 I
1) ^ (cos45°)2=Ji _
2 ) 2 [V2 4

34. As and are initial intensities, therefore, on


rotation of pokiroid through 30°,
cos2 30°
and /'g = /g cos“ 60°

ww
As ^'a = 1'b
● con- 30° = /g cos^ 60°
271 _ cos^60 (1/2)^ 1

Flo
Phase difference (|) = — X path diff.
X /s cos -30° (n/3/2)^ ■ 3

e
2tc X 35. Polaroids A and B are oriented with parallel pass

eree
— X— = JC radian axis.
X 2

FFr
Let the optic axis of polaroid C be making an
31. In Fig. 10(CF).ll, we have shown that refiected
angle 0 with the optic axis of A. Then, it makes
rays are perpendicular to the refracted rays.

oorr
uur r
an angle 0 with B also. Therefore
FIGURE 10(CF).11
sf
- = - cos^ 0 xcos- 9
8 2
sk
Yoo
1 cos'* 0 1
oook

or or cos'* 9 = —
8 0 4
eBB

1
or cos^ 0 = - or cos 0 =
2 VI
uurr

or 0 = 45°
ad

36. Here, = 1 pm = 1 x 10“^ m,


Yo

P = I cm = 10“^ m :
dY

For central maxima in a diffraction pattern,


0 = 30°
Re

According to Brewster’s law, the reflected light


innd

d .sin 0 = X
is polarised with its electric vector perpendicular
X = d sin 0 = 10“^ X sin 30°
FFi

or
to the plane of incidence as shown in figure. Also,
p = tan/ or / = tan-'(p)
= 10“^x - =5000 x 10-'^ m
Choice (b) is correct.
I-22X
32. Limit of resolution of eye = 0 = FIGURE 10(CF).12
D
1-22x5x10“^
= 2-03x10^ rad
3x10-3
If X is the maximum distance at which dots are
just resolved, then d t -cii;
''J60°
1 mm 10-3
0 = = 2-03x10-^
X X

10-3
= 5m
^ 203x10^

f
10/106 ^x4!uCeeft’4^ Fundamental Physics fXinPISTWTl
TiTrw'v We are given.
For Difficult Questions

/
10% of / 0 _ 0
= / = — cos^ 0
Let d' be the slit separation for Young’s fringes. “
10 2
Then fringe width
COS“"a0 = 2 1
\D
10 5

-10 1 I
or 10-2 ^ (5000x10
)x0-50 COS0 = = 0-4472
d' Vs 2-236

oww
5000xl0"'“x0-5 0 = cos-' (0-4472) = 63-44°
or d' = = 25 X 10"<^ m
10-2 To reduce the output intensity to zero, angle by
= 25 |ini which analyser is to be rotated
37. Brewster’s angle (if,) is the angle of incidence at = 90° 63-44° = 26°56'

e
which the reflected light is completely plane

re
The closest answer is (a).
polarised. This angle must be less than 90° but

FFrllo
40. Here, /ij = 8, X| = 600 nm
more than 45°. Hence 45° < z^< 90°.

reF
ih = ?, = 400 nm
Choice (ft) is correct.

e
38. Here, X = 6000 A = 6000 x lO-*** m In the same region of .screen
ouru0., = 60°, n = 2, 2nd diffraction minimum =.^2

osrF
0,”=?m= 1, _ A., _ 8 X 600
For second diffraction minimum, /JI — II') X'), ^2 — = 12

sin0 =
2X
= sin 60°= —
ffor
k 41. Here, v = 3 G Hz = 3 x 10^ Hz
400
kso
a 2
ooo
Yo
3x10'
Y
c
X X = - = 0-1 m
BB

V 3x10^
a 2X 2 4
X 0-1
For first diffraction minimum, Size of the object = 0-001 m = 1 mm
r ree
Y

100 100
uu

sinO
X S _ 1-732 = 0-433
For reflecuon, the size of obstacle must be much
ad

1 -
4 “ 4 larger than wavelength of light. For diffraction,
doo

a
size of the obstacle should be of the order of
Y

01 = sin-* (0-433) = 25° wavelength. Since the object is of size 1 mm,


nidn

. 39. If /q is intensity of unpolarised light, then intensity much smaller than wavelength (= 0-1 m), so
Re

of polarised light = I^/l. Scattering will occur.


F
Fi

01 Multiple Choice Questions (with One or More than One Correct Answers)

XD 2n
42. As P= — phase diff. —> X path diff.
d

^o.X 2k X
— X— =K
X 2
or !->\
PV X
V
44. When source is of white light, the central fringe
is white as all colours meet there in phase. No
or
P.<P. fringe is completely dark as some value of X
So, fringe width will decrease, hence consecutive (colour) may satisfy condition for constructive
fringes will come closer. interference. As P ^ and < X^ therefore, the
43. For first minimum, path diff. —> X/2 fringe next to central fringe will be violet.
WAVE OPTICS 10/107

For Difficult Quostfons

QQI Multiple Choice Questions (Based on the given Passage/Comprehens ion)

45. If vvj, Wj are the slit widths, then w 1 a


2
1
w. /.1 81 47. Here,
1 4
"^2
1
VV2 /2
£2 _ 2
46.
1
max _ (a + b)^
I IIUD
. or b = 2a

(9fl+/>)^ _ (lOfc)^ _ 25
/
(<v + b)^ _(a + 2a)^_9

w
/ max
As £2 = 9/2.'. max _

I.. (9a~/>)2 ~ (8/2)2 " 16


I min
. {a-b)^ 1
min

Flo
09 Matching Type Questions

e
ree
48. For interference of light, essential requirement is aperture must be of the size of wavelength (X) of
light used. According to Law of Malus,

FFr
of coherent sources. ji = tan represents
Brewster’s law. In diffraction of light, obstacle/ / = /q cos2 0. Choice {d) is correct.

for
ur
Matrix-Match Type Questions
kkss
49. Interference and diffraction are basically wave explained on dual theory and corpuscular theory.
Yo
phenomena, which cannot be explained on Polarization can be explained in terms of dual
oo

corpuscular theory. Compton effect can be theory as it is a transverse wave phenomenon only.
eB

VI. Integer Type Questions


ur

50. As the limit of resolution of human eye is


ad

vv, 4
51. Here, —L = -
YYo

1 71 VV. 1
2
e = i' = — radian
60 60x180 ●2
/ a~
vVi _ 4
d

1 _
Re

If-V is minimum distance between the pillars, and 1


in

1. h- vv.,

£/ = 11 km = 11 X 10^ m, then from 0 = —


F

£2 2
d
b 1
7C
^ = £/x0 = llxlO^x m = 3-2 m
60x180
I
max _ (a + hf _{2 + \)~ = 9
The correct integral answer is x = 3 in I.
min
(fl-/2)2 (2-1)2

VII. Assertion-Reason Type Questions

FOR MEDICAL STUDENTS where D is the diameter of the objective lens.


Moreover, large objective lens collects more light
52. Both, the Assertion and Reason are true and
and hence image becomes brighter.
latter explains the former correctly.
53. Here. Assertion is true but the reason is false
D whitish colour of cloud is due to the fact that all
As R.P. =
\-22X' the colours of light are equally scattered by the
10/108 'Pn^uCeA^'4. Fundamental Physics (XII)EEIHIl

For Difficult Questions

FOR ENGINEERING STUDENTS


droplets in the clouds. Hence, assertion is true
but reason is false. 58. Energy can ncithc; oe created nor be destroyed.
54. Colours are seen in thin layer of oil on the surface Energy is simply ledisiribuled in constructive and
of water, because of interference of light. Reason destructive interference. Statement-1 is false, but
is otherwise correct, but not a true explanation statement-2 is true.
of the assertion.
59. When light from blue portion of sky is passed
55. Wavelength of sound is of the order of one metre; through a calcite crystal, the emergent light gets
and size of opening is much smaller. Therefore, polarised and its intensity varies with rotation of
sound is not diffracted, Choice («) is correct.
Polaroid. Thus statement-1 is true.
56. The phenomenon of interference, diffraction and
The light coming from the sky is polarised due to

w
polarisation are explained on the basis of wave
scattering of sun light by particles in the
nature of light. The assertion is true, but the
reason is false. 1
atmosphere. As intensity of scattered light

Flo
OC

57. Assertion is true, but the reason is false. Bright


bands have same intensity, because

eeee
hence blue colour is most scattered. Thus
/ max OC
(fl + b)^, statement-2 is true. And statement-2 is the

Fr
which is constant.
correct explanation of statement-].

for
ur
ks
Yo
oo
eB
ur
ad
Yo
d
Re
in
F
w
e
e
o
wr
r
F
ullo
r FF
e
rsre
uo
koF
DUAL NATURE OF
oofr
sf
ko
Y
RADIATION AND MATTER
BB
Yo
Y
oo
er

CHAPTER 11.
re
uu

DUAL NATURE OF RADIATION AND MATTER


od
Yo
ad
n
ndi
Re
F
Fi
DUAL NATUKE OF

ww
RADIATION AND MATTER

Flo
e
ree
n.l. INTRODUCTION

Fr
The discovery of phenomena like interference, diffraction and polarisation of light established that the

rr F
light IS of wave nature. The Maxwell’s equations of electromagnetism and Hertz experiment on the production
uurr
and detection of electromagnetic waves in 1886 strongly, supported the concept of wave nature of light. The
study of discharge through gases at low pressure led to discover many historic discoveries. The discovery of
sfo
cathode rays by William Crookes in 1879, of X-rays by Roentgen in 1895 and of electrons by J.J.
Thomson in 1897 were very important a.s these discoveries helped to understand the atomic structure. J.J.
kks
Thomson predicted that the cathode rays are nothing but the stream of fast moving electrons. J.J. Thomson,
Yo
oooo

using the mass spectrograph, experimentally determined the velocity and the specific charge (charge/mass)
of cathode rays. He concluded that the speed of cathode rays ranges from 0-1 to 0-2 times the speed of light
eB

and the accepted value of specific charge of cathode rays is 1-76 x 10“ C/kg, which is independenrof
(0 nature of material used as cathode and (n) nature of gas introduced in the discWge tube. This shows that
ur

cathode ray particles are universally alike.


ad

In 1887, it was also found that if certain metals were irradiated by ultraviolet light, the electrons were
YYo

emitted having small velocities. It was also found that if certain metals are heated to high temperature, the
electrons were emitted. These studies led scientists to think that electrons are fundamental particles and are
dd

universal constituent of matter. Later on Millikan measured experimentally the charge on the droplets and
Re
inn

predicted that the charge on a droplet is always integral multiple of elementary charge 1-602 x 10“^^ C,
which is the integral multiple of charge of an electron (= 1-602 x 10“’^ C). This shows that the charge on a
F

body is quantised. The knowledge of values of charge (e) and specific charge {ehn) of electron, helped to
1-602x1Q-'9
determine the mass of the electron. m =
= 9-11 X 10-31 kg
ehn I-76X10’*
In 20th century, the discoveries of photoelectric effect by Hertz, Compton effect by Compton, Stark
effect by Stark were explained by quantum theory of light. According to which, the light consists of the
packets of energy which travel in straight line, with the speed of light. Each packet of energy is called photon
or quantum of light (= hv = hd'k) where h is planck’s constant, v is the frequency of light, c is the velocity of
light and X is the wavelength of light. Thus, the particle nature of light was established.
As some phenomena of light were explained by wave theory of light and some by particle nature of
light, hence it was concluded that light is of dual nature. In this unit, we shall study about the dual nature of
radiation and matter.

11/1
11/2 p>utdee^ '4. Fundamental Physics (XII) EAaiMi
n.2. FREE ELECTRONS IN METALS
In metals, the electrons in the outer shells (called valence electrons) of the atoms are loosely bound.
They are quite free to move easily within the metal surface but can not leave the metal surface. Such loosely
bound electrons are called free electrons.
The free electrons in metal can move about freely inside the metal but can not leave its surface. The
moment an electron comes out of the metal surface, the metal .surface acquires positive charge and pulls it
back. Thus, the free electrons are held inside the metal, by attractive forces of the surface called restiaining
forces, causing potential barrier. It means there is a potential barrier on the metal surface which has to be
overcome by the electron to just escape from the metal surface. In order that an electron may just leave the
metal surface, a certain minimum amount of energy has to be supplied to the electron which depends upon
the properties of the metal and the nature of the surface and is termed as the work function of the metal.

ww
Thus,

Work function of a metal is the minimum energy required by an electron to just escape from the
metal surface so as to overcome the restraining forces at the surface.

Floo
Work function of a metal is generally denoted by <1)q and it is usually measured in eV.

ree
Electron volt (eV) is a unit of energy commonly used in atomic and nuclear Physics. The approximate
values of work function of some metals are given in table 11.1.

rFee
TABLE 11.1. Work function of some metals

F
oor r
rur
Work function 4>o (eV) Metal Work function <|>o (eV)
Metal
s ff
2-14 A1 4-28
Cs
osk
YYoou
K 2-30 Hg 449
oook

Na 2-75 Cu 4-65
eBB

Ca 3-20 Ag 4-70

Mo 4-17 Ni 5-15
uur r

Pb 4-25 Pt 5-65
ad
Yo

From table we note that the work function is lowest for caesium (t})o = 2-14 eV) and highest for platinum
((})(j = 5-65 eV). It means metal caesium can eject electrons with the radiation of lower energy and platinum,
dY

with higher energy.


Re

of metal (ii) the nature of metal and


idn

The work function of a metal depends upon (/) temperature


FFin

{in) the impurities present in the metal surface.


11.3. ELECTRON EMISSION

The phenomenon of emission ofelectrons from the surface of a metal is called electron emission.
The free electron in the metal at room temperature can not leave the metal surface due to restraining
forces and the energy possessed by electron is less than work function of the metal. Therefore, the electron
emission from a metal surface is possible if the energy possessed by free electrons is more than the work
function of the metal. The required energy for the electron emission from the metal surface can be supplied to
the free electrons by one of the following physical processes:
(i) Thermionic emission. It is the phenomenon of emission of electrons from the metal surface
when heated suitably.

Here the energy required for the emission of electrons from metal surface is being supplied by thermal
The emitted electrons are called thermal electrons or thermions. The number of thermions emitted
energy .
depends on the temperature of the metal surface.
DUAL NATURE OF RADIATION AND MATTER 11/3

(il) Photoelectric emission. It is the phenomenon of emission of electrons from the surface of
metal when radiations of suitable frequency fall on it.
Heie tlie energy to the tree electrons for their emission is being supplied by the photons of incident
radiations. The emitted electrons are called photoelectrons. The number of photoelectrons emitted depends
on the intensity of the incident light.
(Hi) Field emission or cold cathode emission. It is the phenomenon of emission of electrons
from the surface of a metal under the application of a strong electric field.
When a very strong electric field (~ 10^ Vnr') is applied to metal, it emits electrons,

(iv) Secondary emission. It is the phenomenon of emission of electrons from the surface of
metal in large number when fast moving electrons (called primary electrons) strike the metal

ww
surface.

The fast moving electrons are high energy electrons. As they fall on metal surface, they transfer their
energy to the free electrons of the metal by which their energy becomes more than the work function of the

Flo
metal and so they are emitted from the metal surface. The electrons so emitted are called secondary electrons.

e
eree
n.4. PHOTOELECTRIC EFFECT

It is the phenomenon of emission of electrons from the surface of metals, when radiations of

FFr
suitable frequency fall on them.

oorr
uur r
The emitted electrons are called photo-electrons and the current so produced is called photo
sf
electric current.

Different materials emit photoelectrons when they are exposed to radiations of suitable frequencies or
sk
Yoo
wavelengths. For example, alkali metals like lithium, sodium, potassium, caesium etc. show photoelectric effect
ooko

with visible light, where as the metals like zinc, cadmium, magnesium etc. are sensitive only to ultraviolet light.
eBB

n.5. HERTZ, HALLWACHS AND LENARD'S OBSERVTIONS ON PHOTO ELECTRIC EFFECT


[A I Hertz Observations
uurr

The phenomenon of photoelectric emission was discovered by Heinrich Hertz in 1887, when he was
ad

working with his electromagnetic wave experiment. Heilz while studying experimentally the production of
Yo

electromagnetic waves by means of spark discharge, found that the high voltage sparks across the detector
dY

loop were enhanced when the emitter plate was illuminated by ultraviolet light from an arc lamp. It was
Re

accounted as follows :
innd

When suitable radiations fall on a metal surface, some electrons near the surface absorb enough energy
FFi

from the incident radiations. Due to it, they are able to overcome the attraction of the positive ions in the
material of the surface and escape to the surrounding space.
[B] Hallwachs and Lenard’s Observations
Hallwachs and Lenard made a detailed study of photoelectric effect during 1886 to 1902.
Lenard observed that if a potential difference is applied across the two metal plates enclosed in an
evacuated tube no current flows in the circuit. When one plate (called emitter plate) enclosed in the evacuated
tube, kept at negative potential is exposed with ultraviolet radiations, current begins to flow. As soon as
ultraviolet radiations failing on the emitter plate are stopped, the current flow is also stopped. These
observations indicate that when ultraviolet radiations fall on emitter plate, electrons are ejected from it which
are attracted towards the other metal plate (called collector plate) kept at positive potential. The flow of
electrons through the evacuated glass tube results in the current flow in the external circuit. Thus, light falling
on the surface of emitter caases current in the external circuit Hallwachs and Lenard studied the variation
of photoelectric current with collector plate potential and with frequency and intensity of incident light
In 1888 Hallwachs connected a negatively charged zinc plate to a gold-leaf electroscope and observed
that the zinc plate lost its charge when it was illuminated by ultraviolet radiations. He also observed that
11/4 “Px^uicef:Fundamental Physics (XII)

uncharged zinc plate became positively charged when it was exposed by ultraviolet radiations. The positive
charge on the positively charged zinc plate was further enhanced when it was more illuminated by ultraviolet
radiations. From these observations Hallwachs concluded that negatively charged particles were ejected out
from the zinc plate under the action of ultraviolet radiations.
After the discovery of electron in 1897, it became clear that the exposure of emitter plate with incident
light causes the electrons to be emitted. The emitted electrons (called photoeleclrons) are accelerated towards
the collector plate under the effect of electric field applied. Hallwachs and Lenard also observed that there
was no emission of electrons from the emitter plate if the frequency of the incident light was smaller than a
certain minimum value, called threshold frequency.
h the minimum frequency of incident rcidiations required to emit the electrons

oww
' »●
Thn rrc'iULP.

from an emitter (i.e. a metal surface) without any kinetic energy.


Thus the photoelectrons are emitted from an emitter if it is exposed with the incident radiations of
frequency greater than threshold frequency. The value ofthresholdfrequency depends on the nature ofmaterial
of the emitter plate.

e
FFrlo
re
11.6. EXPERir — TTUDV OF ^^HOTOELECTPIC EFFECT

ree
The apparatus consists of an evacuated glass or quartz

F
FIGURE 11.1

tube which encloses a photosensitive plate C(called emitter) SOURCE

rF
and a metal plate A (called collector). A transparent window W .
QUARTZ FILTER
W is sealed on to the glass tube which can be covered with a

fsoor
filter for a light of particular radiation. This will allow the
ouur
light of particular wavelength to pass through it. The plate A
skf
e : A
can be given a desired positive or negative potential with C
ooko
respect to plate C, using the arrangement as shown in Fig.
Yo
11.1. Voltmeter (V) measures the potential difference between
Y
Bo

the plates A and C, and microammeter (pA) measures the


reB

photoelectric current in the circuit.


When a monochromatic radiations of suitable frequency
obtained from source 5, after being filtered by a filter attached
uur
oY

the window W. fall on the photosensitive plate C, the 'V’VV-'v'‘r V « V V


ad

on

photoelectrons are emitted from C, which get accelerated


dY

towards the plate A (collector) if it is kept at positive potential. <> i -I


These electrons flow in the outer circuit resulting in the K B
innd
Re

photoelectric current. Due to it, the microammeter shows a


deflection. The reading of microammeter measures the
Fi
F

photoelectric current.
If the potential of plate A is kept zero with respect to plate C, some of the emitted electrons get stuck up
in between C and A, constituting, what is known as space charge. The creation of space charge decreases tlie
probability of the emitted photoelectrons reaching the collector. Due to it. at photoelectric
current is small but not maximum.
This experimental arrangement can be used to study the variation of photoelectric current with (a) the
intensity of the incident radiation, (b) the potential difference between the plates C and A, (c) the frequency of
the incident radiations, and (d) the nature of the materia! oi the plate C.
ITfcct; i .'idejii raiit;-- on. Maintain the collector A at a definite positive potential
w.r.t. to emitter C so that the electrons emitted from emitter are accelerated towards collector A. Using the
incident radiations of a fixed frequency and keeping the plate A at a fixed suitable potential, note the
photoelectric current from the reading of microammeler. Vary the intensity of the incident radiation, keeping
the radiations source at different distances w.r.t. to plate C and note the resulting photoelectric current each
time. It is found that the photoelectric current varies linearly with the intensity of the incident radiation
as shown graphically in Fig. 11.2. As the photoelectric current is directly proportional to the number of
DUAL NATURE OF RADIATION AND MATTER 11/5

photoelectrons emitted per second, so the number of photoelectrons


emitted per second is directly proportional to the intensity of the incident U H
radiations.
cc
o
(h) KITccl of poll iii it'. A vv.r.t. to platt
I:
Keep the plate A LU
c.c

at some accelerating positive potential w.r.t. to plate C. Illurninate the plate LU O

C with radiations of fixed frequency v and fixed intensity l^. Vary the positive o

potential of plate A gradually and note the photoelectric current each time. o
I
It is found that the photoelectric current increases gradually with the increase 0.

in positive potential of plate A. At one stage, for a certain positive potential


of plate A. the photoelectric current becomes maximum or saturates. After o
INTENSITY
this, if we increase the positive potential of plate A, there will be no increase
in the photoelectric current. This maximum value of photoelectric current is called n,, J UMi . rtn Tlie
saturation current corresponds to the state when all the photoelectrons emitted from C reach the plate A.

w
Now apply a negative (retarding) potential on plate A w.r.t. to plate C. We will note that the photoelectric
current decreases. It is so because the photoelecirons emitted from C are repelled and only energetic
photoeleclrons are reaching the plate A. Increase the negative potential of plate A gradually, we will note that

Flo
the photoelectric current decreases rapidly and becomes zero at a certain value of negative potential Vq , on
plate A. At this stage even the fastest photoelectrons {i.e., photoelectrons of maximum kinetic energy, K ) max

ee
can not reach the plate A.

Fr
This minimum negative potential Vq , given to the plate A w.r.t. to plate C at which the photoelectric
current becomes zero is called lopoiti^ potential orcf-- ufi tHu-.-iiai If e is the charge on the photoelectron,
1

for
■)
FIGURE 11.3
ur
then K = eV^ = — mv~max
max
0 2 I3 > I2 > Ii
O
where m is the mass of photoelectron and v is the
max ir
^3
ks
maximum velocity of emitted photoelectron. Z h
Yo
LU q]
The variation of the potential on plate A w.r.t. to plate C
oo

cc
111 I1
and photoelectric current is shown by curve marked f in Fig. Q 5
oc
eB

11.3. Repeat the experiment with the radiation of same frequency


but of higher intensity I2 and Ij, we note that the saturation current I
CL

becomes more with the increase in the intensity of the incident


r

radiations but the value of stopping potential remains the same


ou
ad

as that for radiations of intensity / 1-


-Vo POTENTIAL
Y

From above we conclude the following :


(0 All the photoeleclrons emitted from metal C are not having the same kinetic energy.
nd
Re

(//) For the radiations of a given frequency and material of plate C, the value of stopping potential is
independent of the intensity of the incident radiation. It means, the maximum kinetic energy of emitted
Fi

photoelectrons depends on the radiation source and nature of material ofplate C but is independent of the
intensity of incidentradiation.
li tain in Menory
1.
It is important to note that on doubling the intensity of the incident light, the number of photons
incident on the plate C per unit time becomes double. Due to it, the number of photoelectrons emitted
per unit time would not be doubled because, all the incident photons would not be successful in
photoelectric emission. But only some fraction of photons is able to cause the photoelectron emission.
2. The photoelectric effeciency (r)), is defined as :
number of photoelectrons emitted per second jdNIdt)
number of photons falling per second (dNp/dt)
In photoelectric emission, the value of q is less than 1.
3. During photoelectric emission from the metal plate C, the photoeleclrons can be emitted in any
direction with any velocity inbetween zero to maximum velocity.
11/6 '4- Fundamental Physics (XII)

(c) EtTect of frequency of the incident


radiation.Take the radiations of different frequencies
but of same intensity. For each radiation, study the
variation between photoelectric current and potential
of plate A w.r.t. to plate C. We get the variation of
the type as shown in Fig. 11.4. From the graph we note
that

(/) the value of stopping potential is different for


radiation of different frequency.
(//) the value of stopping potential is more
negative for radiation of higher incident frequency.
(Hi) the value of saturation current depends on

ww
the infen.sity of incident radiation but is indepenent of
the frequency of the incident radiation,

FF loo
(d) If \vi plot a graph between stopping potential and the B

frequency of the incident radiation for two different metals A

ree
and B, we get the graph as shown in Fig. 11.5.
From the graph we note that

rFee
(i)for a given photosensitive material, the stopping potential

F
varies linearly with the frequency of the incident radiation,

oor r
rur
(ii) for a given photosensitive material, there is a certain RADIATION
s ff
minimum cut off frequency Vq (called threshold frequency) for
which the .stopping potential is zero.
k
YYoou
(Hi) the higher is the work function for a photosensitive
ookos

material, the greater is the value of thre.shold frequency,


BBo

4) 0 -hv
_ 0
(iv) the intercept on the potential axis = - . Therefore, work function (j)Q = e x magnitude of
re

e e
ouur

intercept on the potential axis.


ad
Yo

It means for a given photosensitive material, there is a certain incident radiation of frequency Vq (threshold
frequency) for which the emission of photoelectron is just possible, but for the incident radiation of frequency
less than threshold frequency, no emission of photoelectron is possible, even if intensity is high.
dY
Re
idn

From above experiments, it is found that, if the incident radiation is of higher frequency than that of
FFin

threshold frequency, the photo-electric emission starts instantaneouly, even if the incident light is very
dim. The time lag between the incidence of radiation and emission of photo-electrons is less than 10“^ second.
Laws of photo-electric emission
1. For a given metal and frequency of incident radiation, the number of photoelectrons ejected per
second is directly proportional to the intensity of the incident light.
2. For a given metal, there e.xLsts a certain minimum frequency of the incident radiation below which no
emission of photoelectrons takes place. This frequency is called threshold frequency.
3. Above the threshold frequency, the maximum kinetic energy of the emitted photoelectron is independent
of the intensity of the incident light but depends only upon the frequency {or wavelength) of the incident
light.
4. The photoelectric emission is an instantaneous process. The time lag between the incidence of
radiations and emission of photoelectrons is very small, less than even 10"^ second.
DUAL NATURE OF RADIATION AND MATTER 11/7

11.7. EINSTEIN'S PHOTOELECTRIC EQUATION ; ENERGY QUANTUM OF RADIATIONS


Einstein explained the various laws of photoelectric emission on the basis of Planck’s quantum theory.
According to Planck’s quantum theory,
light radiations consist of tiny packets of energy called quanta. One quantum of light radiation
is called a photon which travels with the speed of light.
The energy of a photon is given by. E = hv

where h is Planck’s constant and v is the frequency of light radiation.


Einstein assumed that one photoeleclron is ejected from a metal surface if one photon of suitable frequency
of radiation falls on it.
FIGURE 11.6
Consider a photon of radiation of frequency v, incident on a
photosensitive metal surface (Fig. 11.6). The energy of the incident 2 ^^iTiax

w
photon (= hv) is spent in two ways :
. ,<l>0
(0 A part of the energy of the photon is used in liberating the
electron from the metal surface which is equal to the work function 4>q

Flo
of the metal.

reee
Hi) The rest of the energy of the photon is used in imparling the maximum kinetic energy max
to the
emitted photo electron.

FFr
If V max. is the max. velocity of the emitted photoeleclron and m is its mass, then
uurr
Max. K.E. of the photoeleclron. K
max
— —mv
2
for
max
kkss
or K ...(4)
Yo
max
oooo

max

This equation is called Einstein’s photoelectric equation. If the incident photon is of threshold frequency
eB

Vq, then the incident photon of energy /jvq is just sufficient to eject the electron from the metal surface without
imparling it any kinetic energy, hence, Hvq must be equal to work function 0Q of the metal, i.e.,
r

Hvq = (|)o
ou
ad

1
YY

From (4), — mv~


max = /jV-/M’q = h(V- Vq) ...(5)
2
nndd
Re

Kmux
or
= h(V- Vq) ...(6)
Fi

Explanation of laws of photoelectric emission.


(/) Since one incident photon may eject one photoeleclron from a metal surface, therefore, number of
photoelecirons emitted per second depends upon the number of photons falling on the metal surface per
second which intum depends on the intensity of the incident radiation. If the intensity of the incident radiation
is increased, the number of incident photons increases, which results in an increase in the number of photo-
electrons ejected. This is the first law of photoelectric emission.
(//) From (6), we note that if v < vq, max. K.E. is negative, which is impossible. Hence, photoelectric
emission does not take place for the incident radiation below threshold frequency. This is the second law of
photoelectricemission.
(Hi) From (6), we note that if v > Vq, max. K.E. V. This means, maximum kinetic energy of photoeleclron
OC

depends on the frequency (or wavelength) of incident radiation. If the intensity of the incident light radiation
is increased, the number of incident photons falling per second on the metal surface increases but the energy
of each photon remains the same. This is the third law of photoelectric emission.
(/V) The phenomenon of photoelectric emission has been conceived as an effect of an elastic collision
between a photon and an electron inside the metal. As a result of it, the absorption of energy by the electron
11/8 ‘P'utdee^ ’a Fundamental Physics (XII)

of metal from the incident photon is a single event which involves transfer of energy at once without any time
lag. Due to it, there is no time lag between the incident photon and the ejected photoelectron. This is the
fourth law of photoelectric emission.
DO YOU KNOW ?
I onset, . -s of photoelectricemission
The photoelectric emission is possible only if the incident light is Accordingto Einstein’stheory of
photoelectric emission, one
in the form of packets of energy, each having a definite value, more
than the work function of the metal. This shows that light is not of incident photon of light can eject
one photoeleciron from the
wave nature but of particle nature. It is due to this reason that surface of metal. But it does not
photoelectric emission was accounted by quantum theory of light., mean that the number of photo
i.e., Particle nature of light. electrons emitted from the metal

ooww
surface will be equal to the
Retain in Memory. number of photons incident on it.
1. When the radiation interacts with matter it behaves as if it In fact all the photons falling on
is made of particles called photons. During interaction, the the metal surface are involved not

energy and momentum of photon are U'ansferred to the only in ejecting the electrons

e
matter. from a metal surface but in many

ere
other processes also. As a result
2. Einstein’s photoelectric equation establishes the particle

rFl
of it, the ratio of number of
nature of light, i.e., light is of quantum nature.

Fre
photoelectrons ejected to the
3. KE of the slowest pholoelectron is zero. number of the photons falling on

rrF
the metal surface is less than one.
This ratio can be increased by

sffoo
ouur
11.8. RELATION BETWEEN CUT OFF POTENTIAL
coating the surface of the metal
FREQU’"MCY C F THE INCIDENT PHOTON with a layer of barium oxide or
kosk
ANDT-i:^ESH :LD FREQUENCY/WAVELENGTH stonsium oxide, due to it, the
As already established, from Einstein’s photoelectric equation, work function of the metal
Yo
oo

surface is lowered.
Y

Max. kinetic energy is K = hv-(^Q ...(7)


BB

max

If Vq is the cut off potential or stopping potential and e is the charge on the electron, then
rre

K max 0
...(8)
ouu

If Vq is the threshold frequency, the work function


Y
ad

<j)0 = /ivq ...(9)


dY

From (7), (8) and (9) we have


eVr.0 = .Kmax = hv - hvQ - h (v - Vq ) ...(10)
innd
Re

If X, = wavelength of the incident radiation, X.q = threshold wavelength for the metal surface,
Fi
F

c = velocity of light
c c

Then, V = — and V,
X 0

Putting values in (10), we get


c c 1 1
eV^0 = K max ...(11)
= ' IT0
or
X~X 0
J

11.9. DETERMINATION OF PLANCK'S CONSTANT AND WORK FUNCTION OF A METAL


From Einstein’s photoelectric equation K max = /iv - (|)o
If Vq is the cut off potential, then K,max = eV,0

hv 90
eVQ = hv - (t)o or Vq = — ...(f)
e e
DUAL NATURE OF RADIATION AND MATTER 11/9

This equation is similar to the equation of straight line, y = mx+C-, where m is the slope of straight line
and C is the intercept on y-axis. Therefore the graph between Vq and v is a straight line AB as shown in
Fig. 11.7.
FIGURE 11.7
. h AK0
Slope of this straight line, tan 0 = — Av
Vq A ●t

or /i = e tan e = e X slope of Vq versus v graph / j


;AVo
Thus the value of Planck’s constant can be determined. Av
A 0
O ■►V
From this graph, the intercept on Vq axis is
oc = -
e c
e

or I (|)q I = e X (OC)

w
This relation helps to determine the work function of metal.
Sample Problem □ The work function of caesium is 2*14 eV. Find (a) the threshold frequency

Flo
for caesium, and (b) the wavelength of the incident light if the photo current is brought to zero by a
stopping potential of 0*60 V. Given h = 6*63 x 10"^ Js.

eeee
Sol. Here, Vq = 0-60 V, (Jiq = 2-14 eV = 2-14 x 1-6 x 10"*^ J.

Fr
2-14xl-6xl0"*^
= 5-16 X Hz
I rt) Threshold frequency, Vq = —
h 6-63x10-^^
for
ur
he he 6-63x10-3^ x(3xl0^)
<t>0 or ^ -
ks
(l-6xl0
-19
x0-6 + 2-14xl-6xi0"‘^)
Yo
oo

a 471 X 10“^ m - 471 nm


eB

Sample Problem Using the graph FIGURE 11.8


Vq (volt)-.
shown in Fig. 11.8 for stopping potential the
ur

incident frequency of photons, calculate Planck’s 1-6-'


ad

constant. (C»SK 2015 (Cm /


Yo

1-23" 71
Sol. Here, AVq = 1-23 - 0 = 1-23 V ;
Av = {8-5)x lO'^^Hz.
d
Re

0-8--
From Einstein’s Photoelectric equation
in

eVQ = hv-%
F

0-4-- 14
5 10 Hz /
On differentiating it, we get
eAVft0 = hAv *■
0 2 4 6 8 10
19
or h =
cAVq _ (1.6x10- )xl-23 > v(IO^'^Hz)
Av (8-5)xl0‘^
= 6-56 X 10-^ Js
11.10. IMPORTANT GRAPHS RELATED TO PHOTOELECTRIC EFFECT
(0 Frequency (v) and Stopping potential (Vq) graph
h 0
So Vn0 « V
As, eVQ = hv~<^Q or
11/10
4 Fundamental Physics (XII) VOL.II

Therefore, v - Vq graph is a straight line as shown in Fig. 11.9. Vo. FIGURE 11.9
Comparing the above relation with y = wx + C, we note that the B
slope of V - Vq graph is hie. The jr-intercept, OA = Vq (threshold
frequency)

0
and y-intercept, OC - - or
I (t>o I = ^ X (OC)
e
O
Thus work function can be estimated. /

Frequency (v) and stopping potential (Vq) graphs for different /


/

materials will be represented by parallel straight lines as shown in H>o

oww
C
Fig. 11.10. It is so because, the slope of each straight line is hie but
e

the threshold frequency for different materials is different.


Vo
(ii) Frequency (v) and maximum kinetic energy (K ) max

graph.

e
As, K max ~hv-%

FFrlo
re
So, K max OC V

ree
F
The V- K max
graph is a straight line as shown in Fig. 11.11.
Comparing the above relation with y = /tu: + C, we note that slope

rF
of V - AT
graph is h, x-intercept is Vq and y-intercept is - 0Q.
max

♦9

fsoor
FIGURE 11.11
ouur
K max

I FIGURE 11.12
skf
ooko
Yo
Y
Bo
reB

/
ToT
uur
oY

/
/
ad

-00 ■>>0
dY

(Hi) Frequency (v) and photoelectric current (I) graph.


innd
Re

It is shown in Fig. 11.12. This graph shows that the photoelectric current I is independent of frequency
of the incident light, till intensity remains constant,
Fi
F

(/v) Intensity and stopping potential (Vq) graph.


It is shown in Fig. 11.13. This graph shows that the magnitude of the stopping potential is independent
of the intensity of the incident radiation, till the frequency of the incident radiation remains constant.
FIGURE 11.13
Vo I FIGURE 11.14

Intensity O 10-9$

*
DUAL NATURE OF RADIATION AND MATTER 11/11

(v) Time (t) and photoelectric current (I) graph.


It is shown in Fig. 11.14. This graph shows that the photoelectric current attains the saturation value in
10“^ s from the start of radiation falling on photosensitive material, irrespective of the intensity of incident
radiation.

11.11. PARTICLE NATURE OF LIGHT (The photon)


Photoelectric effect established the fact that light in interaction with matter behaves as if it was made of
quanta or packets of energy, each of energy liv, where h is Planck’s constant and v is the frequency of light.
Einstein also made an important conclusion that the light quantum can be associated with momentum hv/c,
where c is the velocity of light. A definite value of energy as well as momentum gives a strong indication that
the light quantum is a particle which was later named as ‘photon’. The particle like behaviour of light was
later on confirmed by the experiments of A.H. complon. on scattering of X-rays from electrons.

w
The photon picture of electromagnetic radiations and the characteristic properties of photons arc
given below :
(!) //I interaction of radiation with matter, radiation behaves as if it is made of particles like photons.

Flo
hv
(2) Each photon has energy E (= hv = hc/k) and momentum p —- YA ) , where h is Planck’s constant,

e
reee
V ^
V and X are the frequency and wavelength of radiation and c is the velocity of light.

Fr
(3) Irrespective of the intensity of radiation, all the photons of a particular frequency v or wavelength
hv h \

for
ur
X have the same energy E (= hv = hc/X) and same momentum p = —
X
V c

(4) By increasing the intensity of radiation of a given frequency or wavelengh on a metal surface, there
ks
is only increase in the number of photons per second falling on that surface, while each photon is having the
Yo
oo

same energy. It shows that the photon energy is independent of the intensity of radiations.
(5) All the photons emitted from a source of radiations travel through space with the same speed c
eB

(equal to the speed of light).


(6) The frequency of photon gives the radiation, a definite energy (or colour) which does not change
ur

when photon travels through different media.


ad

(7) The velocity of photon in different medio is different which is due to change in its wavelength.
Yo

(8) The rest mass of a photon is zero. According to theory of relativity, the mass m of a ptulicle moving
with velocity v, comparable with the velocity of light c is given by
d
Re
in

m
0
m = or m„ = m ...(12)
0
F

where m^ is the rest mass of particle. As a photon moves with the speed of light i.e., u = c, so from (12),
niQ = 0. Hence photon has zero rest mass.
(9) Photons are not deflected by electric and magnetic fields. This shows that photons are electrically
nentrual.

(10) In a photon-particle collision (such as photo-electron collision), the energy and momentum are
conserved. However the number of photons may not be conserved in a collision. One photon may be
absorbed or a new photon may be created.
(11) In « photons, each of frequency v are emitted per second from a source of light, then power of light
radiated out by the source of light is

Power =
energy _ nhv = n hv
time

where h is Planck’s constant.


11/12
U Fundamental Physics (XII) W«>Wll
Sample Problem
Find the number of photons emitted per minute by a 25 W source of
monochromatic light of wavelength 5000 A. Use h = 6-63 x 10“^ Js. I'Pb

Sol. Time r= 60 s ; P = 25 W ; >. = 5000 A = 5000 x lO’^^ m


DO YOU KNOW ?
,-34
he (6-63x10 )x(3xi0^)
Energy of a photon, E = — ● Photon is not a material
5000x10-'^’
particle but it is a packet of
= 3-97x 10-'^ J energy.
Energy emitted by 25 W source per minute = P/ = 25 x 60 = 1500 J ● The intensity of radiation
Number of photons emitted per minute depends upon the number of
1500
= 3-78x102J photons present in it.

ooww
3-97x10-'^ ● The energy of photon depends
upon the frequency of the
Retain in Memory
radiation and is independent
1. The total energy of a single photon of frequency v is of the intensity of the
E = hv. Since a photon always moves with the speed of

e
radiation.
light, the above relation shows that the photon is a relativistic

ere
● The equivalent mass of a
particle.

rFl photon = /iv/(T or h!\c.

Fre
2. When a material particle of rest mass mQ, moves with
velocity v which is comparable with the speed of light ● The momentum of a photon of

rrF
c,
energy E is Elc and of
then its momentum is, wavelength X is h/X.

sffoo
ouur
● The velocity of photons in
3. Total energy of a particle of rest mass while moving
kosk
different media is different
with velocity v (comparable to c) having momentum p is due to change in wavelength
Yo

E = yjp^ c^+tn^c"^ with the change in medium.


oo

given by,
Y
BB
rre

11.12. FAILURE OF WAVE THEORY OF LIGHT TO EXPLAIN THE PHOTOELECTRIC EFFECT


Huygen’s wave theory of light could not explain the photoelectric emission due to the following main reasons.
ouu
Y
ad

1. According to wave theory of light, the energy carried by a beam of light is measured in ternis of
dY

intensity of the beam. If the intensity of the beam of light is increased, there will be increase in amplitude of
waves and hence energy carried by waves will increase. When beam of light falls on a metal surface, the
energy carried by light is distributed uniformly over the entire surface of metal. All the free electrons present
innd
Re

in the surface of metal will receive the light energy and they will be ejected out of metal surface. When the
Fi

waves of light of higher intensity falls on a metal surface it will impart more energy to the electrons in metal.
F

Due to it, the kinetic energy of the ejected electrons will increase. This is against the experimental facts that
the maximum kinetic energy of the ejected photoelectrons is independent of the intensity of incident light.
2. According to wave theory of light, the photoelectric emission is possible from a metal surface by the
waves of light of all frequencies, provided the intensity of light is sufficient to provide the required energy to
the electron for its emission. It means, by increasing the intensity of the incident wave of light, the energy can
be increased and photo-electric emission will take place even if the frequency of the incident light is less than
threshold frequency. But it is against the experimental fact that no photoelectric emission takes place from
a
metal surface if the frequency of the incident light is less than the threshold frequency, no matter
whatsoever may be its intensity.
3. When the wave of light falls on a metal surface, the energy of the wave of light will not go to a
particularelectron in metal but will be distributed to all the electrons present in the illuminated portion of
metal surface. As a result of it, the electrons will take sometime in accumulating the energy required for their
emission from metal surface. This is also against the experimental fact that the emission of photoelectrons
takes place immediately after the light is incident on the metal surface.
DUAL NATURE OF RADIATION AND MATTER 11/13

11.13. PHOTOELECTRIC CELL FIGURE 11.15

FF
INCIDENT
Photoelectric cell. It is a device which converts light
7^:>. -LIGHT
energy into electrical energy. It is also called an electric eye.

Y
Photo electric cell (al o called photo tube) is shown in

o
Fig. 11.15.

d
It consists of an evacuated glass or quartz tube. Inside

o
the tube, there is a semi-cylindrical photo-sensitive metal plate
C and a wire loop A which serve as cathode and anode J

ll
respectively. The tube is provided with insulating base on which
metallic pins are projected. The elements inside the tube are
connected to these pins and through these pins, photocell is

B
er
fitted into a socket and is connected to the external circuit

ow
having a battery (5), and microammeter (fiA), and a load

ef
TO

n
resistance R in the circuit.
B » AMPLIFIER
Working. When light of suitable frequency is allowed
to fall on the cathode, photoelectrons are emitted. These are
attracted by anode which is kept at positive potential w.r.l.

ku
cathode. So a current starts flowing in the circuit. Due to it, micro-ammeter shows a deflection. The photoelectric

Fr
current is very small, so it is to be amplified first before it is used for some useful purpose. When the light is
cut off, no photoelectrons are ejected from the cathode and hence there is no current in the external circuit.
Applications of photo electric cells.

or
ooi
ur
1. Photoelectric cells are used in television camera for telecasting scenes and in phototelegraphy.
sf
2. Photocells are used for the reproduction of sound recorded on films along with pictures in the
re

movie theatre.
Yo
3. Photocells are used in counting devices, e.g., to count the persons entering the hall provided they come
Yo
F

one by one.
eB

4. Photocells are also used in burglar alarm and fire alarms.


5. Photocells are also used to measure the temperature of stars and to study the spectrum of the heavenly
bodies.
6. They are used to switch on and oH the street lighting system at dusk and dawn, without any
ou
ad

manual attention.
7. They are used in photometry to compare the illuminating powers of two sources.
8. They are used in industries for locating minor flaws or holes in metallic sheets.
nd
Re

9. They are used as photoelectric sorters ; to sort out the materials of different shades.
10. They are used to determine the opacity of solids and liquids.
Fi

II. They are used to control the temperature and chemical reactions.
12. They are used for the determination of Planck’s constant.

Curiosity Question
Q. Many public buildings have doors that open automatically as people approach. What
is the basic concept involved in it ?
Ans. The automatic opening of the doors in a building is often operated by “electric eyes.” On
the door of a public building, a light beam from a strong source of light shines across the
door opening on to a photo voltaic cell. It causes the ejection of electrons, resulting in
photoelectric current in the cell circuit which keeps the door closed.
When some person approaches the door for entering into the building, the light beam is
prevented from falling on photovoltaic cell. Due to it, the current stops in the cell circuit. A
mechanism is triggered that opens the door automatically.
11/14
j^tade^’4. Fundamental Physics fxrnPTSTWn
11.14. DUAL NATURE OF RADIATION
The phenomena like interference, diffraction and polarisation of light were explained successfully by
considering that the radiation is of wave nature. On the other hand the phenomena like, photoelectric effect,
Compton effect etc. could be explained on the basis of quantum nature of radiatic , Le., radiation is of particle
nature. For example, in photoelectric effect, when a single photon of radiation of sufficient energy strikes a
metal surface, it gives up all its energy to a single electron in an atom and the electron is knocked out of the
metal. The emitted electron carries part of the energy of incident photon. In compton effect when an X-ray
photon is incident on a metal of light element. like aluminium, the free electron of the metal recoils along a
definite direction with some energy depending upon the direction along which the incident photon is scattered.
From above two effects, it is clear that a particle (photon of radiation) is colliding against another particle
(electron). It is due to this reason it was concluded that in photo electric effect and compton effect, the
radiation possesses particle nature.
It means radiation sometimes behaves as a wave and sometimes as a particle, i.e., radiation has dual

ww
nature. It is to be noted that in a particular experiment, the radiation has a particular nature, i.e., either it
possesses wave nature or particle nature.
The various phenomena related to radiation can be divided into three categories.

Flo
1. There are the phenomena like interference, diffraction and polarisation etc., which can be explained on

ee
the basis of electromagnetic (wave) nature of radiation only.

rere
2. There are the phenomena like photo electric effect, compton effect etc., which can be explained on the

r FF
basis of quantum (particle) nature of radiation.
3. There are the phenomena like rectilinear propagation, reflection, refraction etc., which can be explained
uurr
foor
on the basis of either of the wave nature or of the particle nature of radiation.

11.15. DE-BROCLIE DUALISTIC HYPOTHESIS


ks s
Yoo
Dual nature of matter. Since radiation has dual nature, Le., it possesses properties of both wave and
ooook

particle and universe is composed of radiation and matter, therefore de Broglie concluded that the moving
material particle must also possess dual nature, since nature loves symmetry.
eBB

De-Broglie hypothesis. According to de-Broglie a moving material particle sometimes acts as a


wave and sometimes as a particle or a wave is associated with moving material particle which
uurr
ad

controls the particle in every respect. The wave associated with moving material particle is
Yo

called matter wave or de-Broglie wave whose wavelength called de-Broglie wavelength, is given
by
dY

h
X =
Re

mv
innd

where m and v are the mass and velocity of the particle and li is a Planck’s constant.
FFi

Derivation of de-Broglie wavelength.


According to Planck’s quantum theory, the energy of a photon of a radiation of frequency v and wavelength
X is

E = hv ...(13)
where h is a Planck’s constant. If photon is considered to be a particle of mass m, the energy associated
with it, according to Einstein mass energy relation, is given by
r- 9
t - m c- ...(14)

hv
From (13) and (14), we get hv = mc^ or m = —r ...(15)

Since each photon moves with the same velocity c, therefore, momentum of photon,
p = mass X velocity
DUAL NATURE OF RADIATION AND MATTER 11/15

hv hv ^
i.e. p = —xc = —
c2 c cfv X

h
or X = ~ ...(16)
P

De-Broglie assumed that the equation (16) be equally applicable to both the photons of radiation and
other material particles.
If a material particle of mass m, is moving with velocity v, then momentum of the particle p =
According to de-Broglie hypothesis, the wavelength of wave associated with the moving material particle
IS

h
X=^ ...(17)

ww
p mv

This is de-Broglie wave equation for material particle


Conclusions from de-Broglie hypotheses

Flo
e
(/) From (17), A. oc — ; It means if a material particle moves faster, the de-Broglie wavelength associated

reree
V
with it is smaller and vice-versa.

r FF
(//) From (17), Xoc 1/m : It means for a heavier moving material particle, the de-Broglie wavelength
associated with it is smaller and vice-versa.
uurr
foor
(///) If = 0, from (17), X = ~ and if u = «>, then X = 0. It means the waves are associated with the
material particles only if they are in motion,
ks s
(/v) The de-Broglie waves associated with a moving material particles are independent of charge and
Yoo
ooook

nature of particles.
(v) The de-Broglie waves are not the electromagnetic waves because the electromagnetic waves are
eBB

produced by the motion of charged particle.


(vi) If a material particle is moving with a velocity v which is comparable to the velocity of light c, then
uurr

the mass m of moving material particle of rest mass mg is given by


ad
Yo

m
0
m -
dY
Re
innd

So de-Broglie wavelength associated with this particle is,

h^|l-v^/c^
FFi

h
X =
mv nu V
0

(v/i) Since the position of a wave cannot be located


exactly, therefore, the wave nature of material
particle introduces the problem of particle location. It
means the wavelength of a wave associated with
moving particle defines a region of uncertainty, within
which the whereabouts of particle are unknown. A
schematic diagram of a de-Broglie wave is given in
Fig. 11.16.
(viii) De-Broglie wave hypothesis led Heisenberg to predict Heisenberg’s Uncertainty principle which
states that it is impossible to measure both the position and momentum of a moving subatomic particle at a
time very accurately.
11/16
'Pn4^dee^ 'a Fundamental Physics (XII) VOL.II

If Ax, A/7 are the uncertainty in the determination of position and momentum of a moving subatomic
particle at a time, then according to Heisenberg’s uncertainty principle
h
AxA
27C

Relation for de-Broglie wavelength and temperature


From kinetic theory of matter, the average kinetic energy of particle at a given temperature 7 kelvin is
3
K = — kT where k is Boltzmann constant.
2

oww
If particle of mass m, moving with velocity v, then its kinetic energy is, ^

Momentum of particle, p-mv


= -^2mK =^2mx^kT =^3mkT

e
re
h

FFrlo
De-Broglie wavelength.
P ^3mkT

rF
ee
Sample Problem What is the de-Broglie wavelength associated with (a) an electron moving
ouru
with speed of 5-4 x lO'* ms-\ and (b) a ball of mass 150 g travelling at 30*0 ms"* ? h = 6*63 x 10"^ Js,

rF
mass of electron = 9*11 x 10"^* kg. (NCERT Solved Example)
Sol. (,
ffosor
For the electron ; m = 911 x 10"^’ ; u = 5-4 x 10^ ms -1
os k
6-63x10-34
= 0-135x 10-^m = 0-135 nm
(9-1Ix10"3')x(5-4x10^)
mv
ook
Yo
Y
{b: For a ball; m - 150 g = 150 x 10“3 kg ; u = 30-0 ms“*
Bo
reeB

h 6-63x10-34
X = = 1-47 X 10-34
mv
(150x10-3)x300
ouY
ur
ad

' 16. DE-BROGLIE WAVELENGTH OF AN ELECTRON


Yo

Consider an electron of mass m and charge e. Let v be the velocity acquired by electron when accelerated
d

from rest through a potential difference of V volt. Then


nidn
Re

2 .
Gain in kinetic energy of electron = ~ FIGURE 11.17
F
Fi

Work done on the electron = eV X- > X-.

1 2eV
— mv^ =eY or v =
2 m
\
If X is the de-Broglie wavelength associated with
the electron, then
VorK
h h h O ■fV
X =
...(18)
mv m
-yj2eV/m ■^2meV Xi.

Substituting the standard values in (18) we get


6.63x10-34
V2x9xl0- 3>xl.6xl0-^9^y
¥
12.27 12.27
xlO ^^m A o
1

4T
DUAL NATURE OF RADIATION AND MATTER 11/17

For a moving electron the variation between X and V (or X and kinetic energy K) is shown in graph,
Fig. 11.17(a) between X and Vv is shown in Fig. 11.17(/?) and between A. and U-Jv in Fig. 11.17(c)
Importance of wave properties of electrons
The wave nature of electron has been utilised in the working of an electron microscope, which has a
very high resolving power than that of optical microscope.
Sample Problem Find the de-Broglie wavelength of an electron in a metal at 127°C. Given,
mass of electron = 9*H x 10“^^ kg; Boltzmann constant = 1*38 x 10“^^ J mole"' K"'; Planck’s constant
= 6-63 X 10-^ Js.
Sol. Here, T = 127 + 273 = 400 K ; m
= 9-11 X 10'-^* kg.
De-Broglie wavelength of electron is
h 6-63x10-^4

ww
X = -i^= --==_^=^==^= - 6-63x10-^4 = 5*3x10 ^ m
pmkT ^3X (9-11 x 10’^') x (1-38x 1) x400 " 12-28x 10“-^
11.17. EXPERIMEINTTAL DEMONSTRATION OF WAVE NATURE OF ELECTRON

Flo
The wave nature of slow moving electrons has been established experimentallyby Davissonand Germer

ee
in 1927.

rere
The apparatus as shown in FIGURE 11.18

rFF
Fig. 11.18, consists of a filament B
A A A -
F of tungsten coated with barium
uurr
oxide, which on heating with
current from low tension battery foor
ks s
emits large number of electrons.
Yoo
C
C is a hollow metallic cylinder
I I 1'
oook

with a hole along the axis, it F


eBB

sun-ounds the filament and is kept I I


at negative potential, so that the
electrons emitted from filament
uurr

may form a convergent beam of L |_ |_vvv,Mr-


ad

/?/;
electrons. It acts as a cathode. A L..T
Yo

is a cylinder with fine hole along


its axis. It is kept at positive
dY

potential w.r.t. cathode and is called anode. The cathode and anode form an electron gun, by which a fine
Re
innd

beam of electrons can be obtained under different accelerating potentials applied between cathode and anode,
is a nickel crystal cut along cubical diagonal. D is an electron detector. It can be rotated on a circular scale
FFi

and is connected to a sensitive galvanometer which records the current.


Working. A fine beam of accelerated electrons obtained from
electron gun is made to fail normally on the surface of nickel crystal.
The incident electrons are scattered in different directions by the
atoms of the crystal. The intensity of the electron beam, scattered in
a given direction is found by the use of detector. By rotating the
electron detector on circular scale at different positions, the intensity
of the scattered beam is measuredfor different values of scattering
angle the angle between the incident and the scattered electron
beam.

The experiment was performed by varying the accelerating


voltage from 44 V to 68 V. It was noticed that at accelerating voltage
54 V, the variation of intensity (I) and scattering angle (4>) is of the
type as shown in Fig. 11.19.
11/18 ^n^uUeib, ^ Fundamental Physics (XII) LV«TWI1
From the graph, it is noted that at accelerating voltage 54 V, there is a sharp peak in the intensity of the
scattered electrons for scattering angle <J) = 50".
The appearance of peak in a particular direction is due to constructive interference of electrons scattered
from different layers of regularly spaced atoms of the crystal, i.e., the diffraction of electrons takes place.
This establishes the wave nature of electron.

From Fig. 11.15 we note that for the scattering angle (f) = 50°, the angle of glancing 0, (angle between the
scattered beam of electron with the plane of atoms of the crystal), for the electron beam will be given by
0 + (l) + e= 180°
1 I
or 0 = - (180° -(!)) = -( 180° -50°) = 65°

Now for the nickel crystal, the interatomic separation is

ww
d = Q.9l A

According to Bragg’s law for first order diffraction maxima (n = 1), we have

Floo
2 (/ sin 0 = 1 X X
X= 2 X 0.91 X sin 65°= 1.65 A

ee
According to de-Broglie hypothesis, the wavelength of the wave associated with electron is given by

eer
12.27 12.27

FFr
= 1.66 A

oor r
ur r
s ff
This shows that there is a close agreement with the estimated value of de Broglie wavelength and the
experimentalvalue determined by Davisson and Germer. This proves the existence of de Broglie waves for
sk
YYoou
the slow moving electrons.
oooko

The wave nature of fast moving electrons was established by G.P. Thomson with his experiment, named
eBB

G.P. Thomson’sexperiment.
Note that de-Broglie hypothesis gave a way to the development of quantum mechanics by which many
complicated problems faced by classical mechanics were solved. It has also led to the field of electron optics
uurr

which helped in preparing electron microscope with higher resolution as compared to optical microscope.
ad
Yo

Retain in Memory
dY

Matter waves cannot be electromagnetic in nature because electromagnetic waves are produced
Re

by accelerated or oscillating charged particles, whereas matter waves are associated with every
ind
FFin

charged or uncharged moving particles or bodies.

11,18. PROBABILITY INTERPRETATION OF MATTER WAVES

The concept of matter waves associated with a moving material particle was accounted for by quantum
mechanics, which involves probability interpretation of matter waves. In this direction. Max Bom, proposed
the probability interpretation to the matter wave amplitude, stating that the intensity (i.e., square of the
amplitude) of the matter wave at a point determines the probability density of the particle at that point.
Probability density means probability per unit volume. If at a point, the amplitude of wave is A. then
probability of the particle being found in a small volume dV around that point is = IA 1“ AV. It means, if the
intensity of the matter wave is large in a certain region, there is a greater probability of the particle being
found there as compared to the region where intensity is less.
DUAL NATURE OF RADIATION AND MATTER 11/19

I TYPE I. PHOTOELECTRIC EFFECT


I (b) eV^=--(^
X
he
0
or X =
he

Formulae used.

(/) Work function. (6-63 X 1 Js) X (3 X 1m/s)


x =
(j)y = /iVy = /jC/Xy (fxO-6V + 2-I4eV)
(/<) K.E. of emitted photoelectron, I9-89x10"2<^ Jm
454 nm
1 ^ 2-74 X1-6 xiO-'^J
K = —mv^ = eV^ =/iv - <1)„
max 2 ^ 0
Example S The work function of a metal

ww
he he is 2*31 eV. Photoelectric emission occurs when
= hv - hv
0 T~X 0 light of frequency 6*4 x 10^** Hz is incident on the
Units used. <{)q in joules, Vy is stopping potential in metal surface Calculate : (i) the energy of the

Flo
volt, h is Planck’s constant (= 6.63 x Js) c incident radiation (ii) the maximum kinetic energ}’
and V max in ms
,-i
, m is in kg and X is wavelength in of the emitted electron and (m) the stopping

e
potential of the surface.

eree
metre.

[Use h = 6-626 x 10"3^ Js] (CBSE 2022)

FFr
Example D The work function of a metal Solution. Here, (})() = 2-31 eV,

oorr
= 6-4x IQl^Hz
uur r
is 4-50 eV. Find the frequency of light to be used V

to eject electrons from the metal surface with a (/) Energy, E = hv


sf
maximum kinetic energy of 6-06 x 10^^ J. = (6-626 X 10-^4) X 6-4 X lO'^^ J
sk
(Use h = 6-63 x 10”^ Js)
Yoo
(CBSE 2022)
ooko

6-626x10"^^ x6-4xl0‘‘^
Solution. Here, 0q = 4-50 eV eV
1-6x10-'^
eBB

= 4-50x 1-6 X 10"'^ J


Kmax = 6-06 X 10'*^ J = 2-65 eV
uurr

As, Kmax = £-4>0 in) Maximum KE, .Kmax = £-4)0 = 2-65-2-31


ad

E=K max = 0-34 eV


or
+ *1>0
Yo

= 6-06 X 10"'^ + 4-5 X 1-6 x lO"'^ K 0-34 eV


max

iiii) Stopping potential, Vq =


dY

= 13-26 X lO-'^J e e
Re

E 13-26x10"’^
innd

= 0-34 V
Frequency v = —
h 6-63x10-3*^
FFi

= 2x 10*5 s"l Example El Photoelectrons are emitted


Example The work function of caesium
from a metal surface when ultraviolet light of
wavelength 300 nm is incident on it. The minimum
is 2-14 eV. Find (a) the threshold frequency for negative potential required to stop the emission
caesium and (b) the wavelength of the Incident of electrons is 0-54 V. Calculate :
light if the photocurrent is brought to zero by a
(i) the energy of the incident photons
stopping potential of 0*60 V.
(ii) the maximum kinetic energy of the
Given h - 6-63 x Js. (CBSE 2004)
photoelectrons emitted
(NCERT Solved Example)
(iii) the work function of the metal
2-14 eV
Solution, (a) y0 ~ Express all answers in eV.
h 6-63x10 -34 Js (ISCE 2003)
(Use h = 6-63 x Js)
2-14x1-6x10"*^ J Solution. Here, X. = 300 nm = 300 x 10“^ m
= 5-16 X 10*“* Hz
6-63 X10-54 Js = 3x10-"^ nu F„ = 0-54 K
11/20 ‘P%adee^ 4. Fundamental Physics (XII) rzsTim
(/') Energy of the incident photon, Maximum KE of the emitted photoelectron
he
E = — _ (6-63xl0-^'*)x(3xl0^) he (6-6xl0~^‘*)x(3xl0^)
K
k SxlO-"^ max -i
k ^0 “ 18x10“^
= 6-63 X l0-‘9 J -4-2x 1-6 X 10"*^
6-63x10“'^ = 11 X 10-^^-6-72 X 10-*^ = 4-28 x
eV - 414 eV
1-6x10-1^ Stopping potential,

(ii) Max. K.E. of emitted photoelectron is K


max _
4-28x10-^^
= 2-67S V
K.max. = 0
~ ex54 V = 0-54 eV ^0 = / 1-6x10"’^
he he (h) Stopping potential will be zero if the incident
(Hi) As, ,Kmax. = T--t>o™'l’o=V -Kmax
light is of threshold wavelength In this case

w
k
K.max = 0
= 4-14 eV-0-54 eV=:3-6 eV
he he he
Then 0 = 4>o or — = $0 or >. = —

Flo
Example § By how much would the
stopping potential for a given photosensitive

eeee
surface go up if the frequency of the incident k0
(6-6x10-3‘^)x(3x10*)
radiations were to be increased from 6 x 10^^ Hz “
4-2x1-6x10"19

Fr
to 16 X 10^® Hz ? Given k = 6-4 x 10"^** Js, = 2947 X 10 -10 A
A

e = 1-6 X 10“^^ C and c = 3 x 10* m/s. = 2947 A

for
ur
Solution. Here, V] = 6 x 10*^ Hz,
V2 = 16 X 10*^ Hz. V|02-^)!=? Example B In an experiment on photo
ks
electric effect, the graph between maximum
As eVQ = /iv-(j)Q
Yo
kinetic energy and frequency v of emitted
oo

0 - <j)o and eVQ2 = hv2 - photoelectron from metal surface is found to be a


1
eB

straight line as shown in Fig. 11.20. Calculate


FIGURE 11.20
h
ur

or ^02-^01 = -(v, -‘^1) 4Kmax(’c10-^^)J


ad

4-
Yo

3--
-34
(6-4x10 )
●●●^02-^01- 1-6x10-'^ x(16x 10^^-6x 10*^) 2-
d
Re

1 -
in

= 40 V 0 I >
4 6 8
10 v{x10'''*)s"''
F

-1 -

Example (a) The work function for the -2-


C
surface of aluminium is 4*2 eV. Find potential -34
difference to be required to stop the emission of
maximum kinetic energy photoelectrons emitted (a) threshold frequency,
by light of 1800 A wavelength ? (b) work function of metal in electron volt,

(b) Determine the wavelength of that incident (c) Planck’s constant and
light for which stopping potential will be zero ? (d) maximum kinetic energy of the emitted
ih = 6-6 X 10"^** Js) electron by light of frequency v = 8 x 10^“* s~*.
Solution, (a) Here, Solution. («) The maximum kinetic energy
of the emitted photoelectron is zero for the
% = 4-2 eV=4-2x 1-6 x lO''*^!;
k = 1800 A = 1800 X 10-*'’m= 18 x 10“*
incident light of threshold frequency Vq, which is for
m.
point A on graph. So the threshold frequency
= ? V = 3 X lO*'^ s"‘
0
DUAL NATURE OF RADIATION AND MATTER 11/21

{b)K max = hv-(^Q he


When, v = 0,^:,max = ot<^q = -K max
or (|)0 = /iv-e'VQ=—-eVo
X
= _(_2x 10-‘^) = 2x e.esxio-^'^xSxio^
-1.6x10"^^ xl.3
2x10"’^ 2271xl0"‘^‘
eV = 1-25 eV
1-6x10“''^ 6.7x10“*^
(c) Planck’s constant, h = slope of straight line or <1)0=6.7x10-1^1 = 1.6x10"’^
eV = 4.2 eV

3x10-*^
AB = 6*0 X 10-^ Js If I’o is ihe threshold frequency, then hv^ = <{»o
”(8-3)xlO'^ '0 6.7 xlQ-'^
== 1.0 X 10*= Hz
(d) For V = 8 X 10^“^ s“*, the value of maximum 6.63x10-2'*
kinetic energy from graph is 3 x 10“*^ J
For red light, X^ = 6328 A = 6328 x 10-*° m.

Example B When a piece of metal is

lowow
c 3x10^
illuminated by monochromatic light of wavelength we have, frequency =—
6328x10"*°
X then the stopping potential for photoelectric r

current is 2*5 Vq. When the same surface is = 4.74 X lO’^iRz.


illuminated by light of wavelength 1'5 X, then the
Since, v,. < vq , therefore, the photocell will

ee
stopping potential becomes Find the value of
not respond to this red light, howsoever strong its

Fr
threshold wavelengthfor photoelectricemission. r FF intensity may be.
he
Solution. As, “T
X
0 ’ Example IE Monochromatic radiation of

rer
wave length 640.2 nm (Inm = IQ-** m) from a neon
he fofr Fo
lamp irradiates a photo-sensitive material made of
u
so
2-5V,. = --<!>,
X calcium or tungsten. The stopping voltage is
ks
he measured to be 0.54 V. The source is replaced by an
YYouro
and V^e = -<1). 0 ...(«)
oo

0 iron source and its 427.2 nm line irradiates the same


1-53.
photocell. Predict the new stopping voltage.
he
BBo ks

eV.
0 Solution. Here, for neon lamp,
From (0 ; 0 “
X = 640.2 nm = 640.2 x 10-^ m, Vq = 0.54 V.
r ee

2-5 X 2-5
From (li) and (m). he

We know, ^
ouru
ad

he he <1>,0
'o “
Yo

he
\-5X 2-5X 2-5
.●. Work function, (|)q = -^-eV,
A.
0

fw J 1_ 1 1-5 3
d

2-5 = 4*0 I- _ 6.63x1Q-2‘*x3x10^


or

X Li-5
Re

0
2-5 2-5 -1.6xl0-*^x0.54
iYn

640.2xir^
3 he he 4 3 he
= 3.1 X 10-'^ - 0.864 X 10-*^ J
FFind

or
X Li5 25.
or
5X
0
TiTs 5X
0
2.236x10"*^
= 2.236 xl0-‘9j = eV = 1.4 eV
or X ^-X = 2-25 X 1.6x10“*^
0-4 For iron source, X = 427.2 nm =427.2 x 10"^ m.
Example B Ultraviolet light of wavelength he
.-. eV
2271 A from a 100 W mercury source irradiates a '0
photocell made of molybdenum metal. If the
stopping potential is 1.3 V, estimate the work 6.63x10-^“* x3x 10^
-2.236x10-*^
function of the metal. How would the photocell 427.2x10-9
respond to a high intensity (- 10^ Wm"^) red light of = 4.649 X 10-*9 - 2.236 x I0"*9 = 2,413 x IQ-'^ J
wave length 6328 A produced by a He-Ne laser ? .●. Stopping potential,
Solution. Here, >. = 2271 A = 2271 x 10"*° m. -19 -19
2.413x10 2.413x10
Vo = 1.3V, (Do=? V
0 - -19
= 1.55 V
e 1.6x10
We know. eVQ = hv- (j),)

1
11/22 7^nadeeft.'4^ Fundamental Physics fXinrosTWn

I I
TYPE II. PHOTONS
Energy emitted by source in lime /,
£] = P/ = 66 X 2 X 60 J
Formulae used. No. of photons emitted by the source in 2
(0 Energy of photon, E = hv = hcfk minutes
,, = —!- = —
Pt = 66x2x60
hv h E E 3-3x10-19
(//) Momentum of photon, p=—
X
c
= 2-4 X 10^^ photons,
{Hi) Equivalent mass of photon,
E _ hv _ h
Exam
pie [g The wavelength of light in the
m =
visible region is about 390 nm for violet colour,
c2 ‘ ■a
about 550 nm (average wavelength) for yellow-
where, h is Planck’s constant (= 6-63 x Js),
green colour and about 760 nm for red colour.
c is the velocity of light (= 3 x 10* m/s), v is the
frequency of photon and X is the wavelength of photon. What are the energies of photon in eV at the

ww
Units used. £ is in Joules, p is in kg ms"’ and m is (0 violet end (ri) average wavelength yellow-green
in kg. colour and (Hi) red end of the visible spectrum ?
(Take k = 6-63 x 10"^ Js and 1 eV = 1-6 x J)

FF loo
Example m Monochromatic light of freq (NCERT Solved Example)
Solution. Energy of the incident photon,

ree
uency 6*0 X lOi^* Hz is produced by a laser. The
power emitted is 2*0 x 10”'^ W. (a) What is the £ =
he

energy of a photon in the light beam ? (h) How X

reFe
many photons per second, on the average, are (/) For violet light.

oroFr
rur
emitted by the source ?
^ _hc (6-63x1Q-34)x(3x10*) eV
s ff
Given h = 6*63 x 10“^ Js. tCBSE 2014)
‘ X.I (390x10-9)x1-6xI0“‘9
(NCERT Solved Example)
k
YYouo
= 3-19 eV
Solution, (u I E~hv
okso

(ii) For yellow-green light.


= 6-63x 10-34 X 6-Ox jqM
BBoo

3-98x10-’9 (6-63x10-^^)x(3x10^) eV
^2 =
r ee

= 3-98x 10-’9j = (550x10"9)x(1-6x10-^9)


1-6x10-’9
= 2-26 eV
= 2-49 eV
ad
ouur

{Hi) For red light.


Yo

(/;) No. of photons emitted per second,


P 2-Qx1Q-3w (6-63xI0-34)x(3xI0*)
^3 = = 1-64 eV
(760x10-9)x(1-6x10"'9)
d

n = —
Re
idnY

£ 3-98x10-’9j
5‘0 X 10^^ photons per second
I I
FFin

gs

TYPE III. de-BROGLIE HYPOTHESIS


Example m A monochromatic source,
emitting light of wavelength, 600 nm, has a power Formulae used,

output of 66 W. Calculate the number of photons (i) Energy of a photon, E = hv = hc/X


emitted by this source in 2 minutes. Use h = 6*6 x
10-34 jj._ (ii) De-Broglie wavelength, X = hhnv = h/^2 m
(^T{^ Sample Paper 2013)
{Hi) De-Broglic wavelength of electron accelerated
Solution. Here, X = 600 wn =6x 10"^ m ; 12-27 .
P = 66W,N=U = 2x60s under a pot. diff. V (volt) is A =
Energy of each photon,
(iv) De-Broglie wavelength of a moving particle of
he (6-6x10“44)x(3x10*) mass m at temp. T is X = h/yj3mkT , where k is
£ = — Boltzmann constant.
X 6x10-^
Units used. E and £^ in Joule, h in Js, c or v in ms-',
= 3-3 X 10-’9 J m in kg and X in m.

I
DUAL NATURE OF RADIATION AND MATTER 11/23

Example An electron is accelerated Solution.

from rest through a potential difference of 100 V.


m V m 1
Find (0 the wavelength associated with it (ii) the t' _
^x-
momentum of electron and (Hi) the velocity "k e hl(m
' e
V e') m
p
3

acquired by the electron.


[// = 6*6 X 10“^ Js, mass of electron = 9*1 m
m 911xl0"-^’
X 10"^* kg]
or
(CBSE 2022)
P 3(X^/X^) 3x1-813x10-^
Solution. Here, V= 100 volt,
= 1-675 X 10-27 kg
12-27 12-27
(/■) As, X= A = = 1-227 A It is the mass of a neutron or proton. Hence the
emitted particle is a neutron or proton.
Example [Q What is the de-Broglie wave

loow w
(//) or p = —
h _ 6-6x10~3^Js length associated with an electron, accelerated
P X ~ 1-227x10-*^’ m
through a potential difference of 100 volt ?
= 5-37 X 10-2^ kg ms-‘ (CBSE 2010) (NCERT Solved Example)
Solution. Here, V = 100 volt

ree
p 5-37x10-24 i^g De-Broglie wavelength,
(Hi) p = mv or v = —
m
9-lxl0-3^kg

rree F
r FF 12-27 , 12-27
X = A = = 1-227 A
- 5-5 X 10^ ms -1
VIoo
In this case the wavelength associated with an
Example m An electron, an a-particle, fofroF
electron is of the order of wavelength of X-ray.
u
and a proton have the same kinetic energy. Which EEl Find de-Broglie wavelength
ks
Example
of these particles has the shortest, de-Broglie of neutron at 127”C. Given Boltzmann constant,
soo

wavelength ? (NCERT Solved Example)


YYouor

k = 1-38 X 10-23 j K-l, h = 6-63 x IQ-^** Js,


Solution. Kinetic enrgy,
BBook

mass of neutron = 1*66 x 10-27


r ee

Solution. Here, 12TC = 127 -h 273


= 400K
2 2 m 2 m
ouru
ad

Energy of neutron at 127“ C is,


Yo

or
p - .^2 m K E = -^/:r = -^xl-38xl0-23x400
2 2
De-Broglic wavelength.
d
Re

= 8-28x 10-21 J
inY

h h 1 h
x=- X o«
FFind

or
Now, X=
P ^2mK ■Jm ^2niE
Since proton is 1836 times heavier than electron 6-63x10-24
and a-particle is 4 times heavier than proton, so ■72x1-66x10-27 x 8-28x10-2>
m^ : nip : m^ = I : 1836 ; (1836 x 4) = 1-264 X 10-‘0„i = 1-264 A”
X is shortest for a-particle. Example [0 X-rays of wavelength 0.82 A
Example [Q A particle is moving three fall on a metal plate. Find the wavelength
times as fast as an electron. The ratio of the de-
associated with photoelectron emitted. Neglect
work function of the metal.
Broglie wavelength of the particle to that of the
Given h = 6.6 X 10“34 Js ; c = 3 x 10^ ms-^.
electron is 1-813 x 10"4. Calculate the particle’s
Solution. Here, X = 0.82 A = 0.82 x lO'^'^
mass and identify the particle. Mass of electron
= 9-11 X 10-31 kg. (NCERT Solved Example) 6o = 0
From Einstein’s photoelectric equation.

i
11/24 Fundamental Physics fXin Wsm

I ^ According to gas equation for 1 mol of a gas


K.E. of electron, —mv~ =hv-<^ 0
2 R
PV = RT or PV = kNT
2 he N
mvr = 2 /jv =

w
or
X (V (J>o = 0)
or
2 he m N~ P
or mv =

Mean separation between two atoms of a gas is


de-Broglie wavelength associated with the xl/3
^i/3
V' kT
electron is given by

wr
e
r =

h hX
N P J

r
oo
●il/3
mv
■Jlhcm/X 2cm (1-38x10"23)x300 = 34 X 10-‘0 m

F
llu
I-01xl0=
6.6x1Q~'-^^xQ.82x10~'Q
X 0-73
\ 2x3x10^x9.1x10"^^ <1 or ^ < r.

FF
r 34
= 0.099 X 10"’^ m = 0.099 A

ees
This shows that the wave packets associated

rr
Example^ Find the typical de-Broglie
uro with helium atoms do not overlap. Therefore, the

Fk
wavelength associated with a helium atom in helium atoms can be seen separately.

oo
helium gas at room temperature 2TC and 1 mm
pressure and compare it with the mean separation
I TYPE IV. TYPICAL EXAMPLES
I
fofr
between the atoms under these conditions. Atomic
so
Y
weight of helium = 4, Avagadro’s number
oY
= 6 X 1023 per gram mole^ Boltzmann constant Example m A stopping potential of 0*82
B

= 1-38 X 10-23 J.mole-l K"', h = 6-63 x lO-^** Js. volt is required to stop the emission of
ok
Yo
(NCERT) photoclectrons from the surface of a metal by light
eBr

Solution. Mass of one atom of helium is of wavelength 4000 A. For light of wavelength
3000 A, the stopping potential is 1*85 volt. Find
rue

atomic weight 4
oud

m - the value of Planck’s constant


g
Avogadro’s number 6x1023 [1 eV = 1*6 X 10-1^ J].
no
ad

4x10-3 (ii) At stopping potential if the wavelength


kg = jXlO 26 kg of light is kept fixed at 4000 A, but the intensity of
iY

6x1Q23
light increased two times, will photoelectric
T=27 + 273 = 300K
current be obtained ? Give reason for your answer.
nd
Re
F

Average K.E. of a helium atom at absolute Solution. (/) Here,


temperature T is X] = 4000 A = 4 x lO""^ m ; V, = 0-82 V ;
Fi

1 3
— mv- ~—kT
X2 = 3000 A = 3 X lO""^ m ; ¥5 = V
2 2 he he
As
^ = <t>o+‘'V|
or mv- = 3 kT and
^2
or mv =
^3mkT
h X~f X1
de-Broglie wavelength, X = — =
mv
V3 m kT
or he
^1 “^2
663xI0"34 - =e(V2-~V^)
^1 ^2
/ 0 \ ^1/2
3x -xl0"26 x1-38x10"23x300
or It = e(V^_-V^)X^ X2
3

= 0-73 X 10-^0 m = 0-73 A


(Xj - X^lc
DUAL NATURE OF RADIATION AND MATTER 11/25

(1-6x10-19)x(1-85-0-82)x4x10-'^x3x10-'^ Example Light is incident on the


cathode of a photocell and the stopping voltages
(4x10"'^ -3xl0-'^)x3xl0^ are measured for light of two different wave
= 6*592 X Js lengths. From the data given below determine the
(/7) No. because the stopping potential is work functions of the metal of the cathode in cF
independent of intensity of incident light. and the value of universal constant hde.

Example m Calculate the strength of the Wavelength (A) Stopping Voltage (in volt)
transverse magnetic field required to bend all
4000 1-3
photoelectrons within a circle of radius 50 cm,
when light of wavelength 4000 A is incident on a 4500 0*9
barium emitter. Work function of barium is
2-5 eV. Given, m^ = 9*1 x 10“^^ kg ; Solution. As, ,Kmax = eV.
he

w
0 “ 1 '0
e = 1-6 X 10"*^ C ; h = 6-6 X Js. X
Solution. Here, B =?;/■= 50 cm = 0-50 m ; he <i>,0
X = 4000 A = 4000 X lO"^*^ m = 4 x lO-"^ m
or
>^0 =

Flo
; eX e
4>0 = 2-5 eV = 2-5x 1-6 x
AFo = (Fo)i-(Fo)2

ee
As. 1
— mv
2 he
he <l> 0 he (!) 0
'0

Fr
max
2 X
eX.j e eX^ e
or V
2 2 [ he , y V

he I 1 he ^2 “ ^1
for
max
ur
e X
2 L X'^ 1 C ^2
9-lxlO"^^ AVy-Xj ^2
ks
he
or
Yo
-34
)x(3xl0®) -2-5xl-6xl0"’^ e iX^-X^)
oo

(6-6x10
X
4x10"'^ (l-3-Q-9)x(4000xlQ~*<^)x(4500xlO~'^^)
eB

= 20-88 X lO^® 500x10-'^


V
max
= 4-57 X 10^ tfi/s = 1*44 X 10-^ Vm
ur

= miii tr
ad

Now, Bev max he 0 '() he


Also, Vq = -V
Yo

max or
0
eX e e e X

or B =
mv
max _ (9-lxl0"^’)x(4-57xi05)
'0 1-44x10*^
(l-6xl0-^^)x0-50
nd

e r
-1-3 = 3-6-1-3 = 2-3 F
Re

e 4000x10"^^
= 5*2 X 10-^ T
Fi

(!)0 = (2-3xe)F=2*3 eV

RsMWliS7l€i»l!4t ZiST za, GilL-A'^NlGsEi


1* Free electrons in metals. These are loosely bound electrons of the atoms, which can move freely within
the metal surface but cannot leave the metal surface at room temperature.

2. Work function of metal. It is the minimum energy required by an electron to just escape from the metal
surface so as to overcome the restraining forces at the surface of metal. Work function of a metal is generally
denoted by 6q and it is usually expressed in electron volt (eV).
3. Electron emission. It is the phenomenon of emission of electrons from the surface of a metal.
The electron emission can be obtained from the following processes :
(/) Thermionic emission
(i7) Photoelectric emission
11/26 ‘P'uiuUtfr'4. Fundamental Physics (X1I)CSI9D
(m) Field emission, and
(jv) secondary emission.
4. Photoelectric effect. It is the phenomenon of emission of electrons from the surface of metais, when light
radiations of suitable frequency fall on them. The emitted electrons are called photo-electrons and the
current so produced is called photoelectriccurrent. Alkali metals ; like lithium, sodium, potassium, caesium
etc. show photoelectric effect with visible light, whereas metals like zinc, cadmium, magnesium etc. are
sensitive only to ultra-violet light for photoelectric effect.
5. Laws of photoelectric emission
(1) For a given metal and frequency of radiation, the number of photoelectrons ejected is directly proportional
to the intensity of the incident light.
(2) For a given metal, there exists a certain minimum frequency of the incident radiation below which no
emission of photoeleStrons takes place. This frequency is called threshold frequency.

ww
(3) Above the threshold frequency, the maximum K.E. of the emitted photoelectrons is independent of the
intensity of the incident light but depends upon the frequency (or wavelength) of the incident light
(4) The photoelectric emission is an instantaneous process.

Flo
6. Einstein’s photoelectric equation is

e
eree
1

FFr
where v is the frequency of the incident photons, Vq is the threshold frequency of metal, v max is the maximum

oorr
uur r
velocity of ejected photo-electron from the metal surface and m is the mass of photo-electron.
sf
7. Cut off potential or stopping potential. It is that minimum negative potential given to anode in a photo
cell for which the photo-electric current becomes zero. It is denoted by Vq.
sk
Yoo
The value of stopping potential is different for different metals but is independent of the intensity of the
ooko

incident light.
eBB

8. Photoelectric cell. It is a device which converts light energy into electrical energy.
Photoelectric cells are of three types : (i) Photo emissive cell (/i) Photo voltaic cell {Hi) Photo conductive
uurr

cell.
ad

9. Photons. These are the packets of energy (or energy ptirticles) which are emitted by a source of radiation.
Yo

They travel in a st. line. The energy of a photon, E ■= hv — hefk. The photons emitted from a source, travel
through space with the same speed (equal to the speed of light). The frequency of a photon does not change
dY

when it travels through different medium but its wavelength changes in different medium as the velocity of
Re
innd

a photon in different media is different. The rest mass of photon is zero. Its momentum is hvtc or hl\.
Dual nature of matter. Since radiation has dual nature i.e. it possesses properties of both ; wave and
FFi

?a.

particles and universe is composed of radiation and matter, therefore de-Broglie concluded that the matter
must also possess dual nature, since nature loves symmetry.
11. De-Broglie hypothesis. According to de-Broglie, a moving material particle some times acts as a wave and
some times as a panicle or a wave is associated with a moving material particle which controls the particle
in every respect. The wave associated with moving material particle is called matter wave or de-Broglie
wave, whose wavelength is given by, X = himv where m is the mass of the particle moving with velocity v
and h is Planck’s constant. De-Broglie wavelength associated with electron accelerated under a potential
difference V volt is given by
12-27 A
X =

The wave nature of electron i.e. de-Broglie hypothesis was established experimentally by Davisson and
Germer in 1927 for slow electrons and by G.P. Thomson in 1928 for fast electrons.
DUAL NATURE OF RADIATION AND MATTER 11/27

QUESTIONS

Based on NCERT Book

I. Multiple Choice Questions 6. The photoelectrons emitted from a surface of


sodium metal are such that
1. Light of wavelength 3500 A is incident on two
metals A and B, A of work function 4-2 eV and B {a) they are all of same velocity
of work function 1-19 eV respectively. The photo- [b) they have the same kinetic energy
electrons will be emitted by (c) they have the same de-broglie wavelength
{a) metal A id) they have their speeds varying from ;itero to a
certain maximum
(b) metal B
7. If the intensity of incident radiation falling on

ww
a
(c) both metals A and metal B photo-cell is increased three times, the number of
(d) neither metal A nor metal B photo-electrons emitted and the energy of photo
electrons emitted become
2. If a surface has work function 3-2 eV, the longest

Flo
wavelength of light which will cause the emission (a) doubled, remains unchanged
(b) three times, remains unchanged

e
of electrons is : (/j = 6-6 x lO"^** Js)

ree
(a) 386-7 A ib) 3867 A (c) remains unchanged, three times

Fr
(d) three times each
(c) 193-4 A id) 1939 A

rF
8. The threshold frequency of a certain metal is
3. The threshold wavelength for photoelectric
uurr
3-3 X lO*'* Hz. If light of frequency 8-2 x 10^*^ Hz

for
emission from a material is 4800 A. Photoelectrons
is incident on the metal, the cut off voltage for
will be emitted from the material when it is
photoelectric emission is (Given Planck’s constant,
illuminated with light from a
s
/j = 6-62x 10-34 jj.)
kks
(a) 40 W green lamp {b) 100 W red lamp (a) 1-61 V (b) 2-03 V
Yo
oooo

(c) 100 W yellow lamp((/) 40 W blue lamp (c) 2-63 V (d) 2-93 V
4. For photoelectric emis.sion, Tungten requires light 9. Radiations of two photon energies three and five
eB

of wavelength 2300 A. If light of wavelength 1800 times the work function of metal are incident
A is incident, then emission. successively on the photosensitive metal surface.
ur

{a) takes place The ratio of the maximum velocity of the


ad

photoelectrons emitted in the two cases will be


YYo

{b) don't take place


V3
(c) may or may not take place ia) - ib)
5 V5
dd

id) depends upon frequency


Re
in

1 1
5. The correct curve between the stopping (c) - {d)
2
potential(V^) and intensity of incident light(I) is :
F

10. Ultraviolet radiations of 6-2 eV falls on an


aluminium surface (work function 4-2 eV). The KE
in joules of the fastest electron emitted is
approximately,
{a) 3-2 X 10-'3 (/>)3-2x 10
17

fc) 3-6 X 10-'^ (rf)3-2x 10


-21

11. The threshold frequency for a metallic surface


corresponds to an energy 5-5 eV and the stopping
potential for a radiation incident on this surface is
4-5 V. The incident radiation lies in :

{he = 1240 nm eV)


{a) X-ray region {b) ultraviolet region
(c) infrared region {d) visible region

I
11/28 ^>utdee^’4> Fundamental Physics (XII) VOL.II

12. The theory, on the basis of which, Photoelectric (a)/l>/2 (i>)/l=/2


effect can be explained : i.c)h<h (t0/i</2
(a) Corpuscular theory 19. Which of the following has maximum stopping
(b) Wave theory potential when metal is illuminated by visible
(c) Electromagnetic theory light ?
(d) Quantum theory (a) Blue (b) Yellow

13. The photoelectric work function for a metal surface (c) Violet (d)Red
is 4* 14 eV. The cutoff wavelength for this is : 20. The slop of frequency of incident ray and stopping

oww
(fl) 4125 A {b) 2062-5 A potential for a given surface will be
(c) 3000 A (d) 6000 A {a)h (b) hie
(c) eh {d)e
14. If £j, E2, £3, £4 are the respective kinetic energies
of electron, deuteron, proton and neutron having 21. The threshold wavelength for a metal having work

e
same De-Broglie wavelength. Select the correct function is Xq, what is the threshold wavelength

re
order in which those values would increase : for a metal whose work function is ^^2.

FFllro
(a) £,, £3, £4, £2 (£>) £2* £4* ^1* ^3 (a)4Xo {b)2XQ

reF
(c) £2* £4. £35 £j (d) £3', £j, £2* £4 . (c) V2 V4

e
15. When radiation of given frequency is incident upon 22. Maximum kinetic energy of emitted electron
uoru
different metals, the maximum kinetic energy of depends on the frequency of incident photon when

osFr
electrons emitted frequency of incident photons is
(a) decrease with increase of work function (a) equal to the threshold frequency
(b) increase with increase of work function
fkfor (b) half of threshold frequency
okso
(c) remains same with the increase of work function (c) greater than threshold frequency
(d) does not depend upon work function (d) one third of thresholdfrequency
Y
Yo
23. Two particles have equal momentum. What is the
oo

16. A proton, a neutron, an electron and alpha particle


BB

have same kinetic energy, then their De-Broglie ratio of their de-Broglie wavelength ?
wavelengths compare as (a) 2 (b)l
Y
r ree

(a) — — — X^ {b) Xg>Xp>Xf^> X^ (c)3 (d)0^5


ouu

(c) Xg<Xp<X^< Xg id) Xp = X^ and Xg > X^ 24. Identify the correct relation for the given diagram
ad
Ydo

17. The monochromatic beams A and B of equal for frequency


intensities /, hit a screen. The number of photons
hitting the screen by beam A is twice than that by FIGURE 11(Q).3
nidn

beam B. The ratio of their frequencies will be


Re

4 Photoelectric current
(a) 1 : 2 ib)2:l
F
Fi

(c) 1 ; 1 id)\:3
18. Following graph shows the variation of
photoelectric current with anode potential for two >
light beam of same wavelength but different V03V02V0, Anode potential
intensity. Find the correct relation :
FIGURE 11(Q).2
■c (a) vi = V2 = V3 (b) Vj > V2 > V3
0 c
o § (C) Vj < V2 < V3 (^/) V| = 2 V2 = 3 V3
(I2. V)
oO 25. What is de-Broglie wavelength associated with
Q.
dl. v) electron moving under a potential difference of
lO'^V
Stopping
Potential (a) 12-27 nm ib) 1 nm
(c) 0-01227 nm id) 0-1227 nm
O Anode Potential
DUAL NATURE OF RADIATION AND MATTER 11/29

26. Two photon, each of energy 2-5 eV are (a) 7750 nm (b) 775 nm
simultaneously incident on the metal surface. If the (f) 77-5 nm (d) cannot be determined
work function of the metal is 4-5 eV, then from the
33. The work function for a metal surface is 2-07 eV.
surface of metal
The threshold wavelength for this metal surface
(o)One electrons will be emitted with energy IS :
0-5 eV
(fl)4125 A (b) 2062-5 A
(h)Two electrons will be emitted with energy (c) 3000 A (d) 6000 A
0-25 eV
(CBSE Sample Paper 2022-23)
(c) more than two electrons will be emitted
34. The kinetic energy of a proton and that of an
(d) not a single electron will be emitted
a-particle are 4 eV and 1 eV, respectively. The ratio
27. How does the energy of a photon change if the of the de-Broglie wavelengths associated with
corresponding wavelength increases ? them, will be :

w
(a) Increases (b) Decreases («) 2 : 1 (b)l:l
(c) May increase or decrease (c) I ; 2 (d)4:l (CBSE 2020)
(d) Does not depend on wavelength

Flo
35. A photocell connected in an electric circuit is
28. A proton and lithium (Li^'*') are accelerated through placed at a distance d from a source of light. As a

reeee
same potential difference. If mass of lithium is 8-3 result, current / flows in the circuit; What will be
times of the mass of proton, then ratio of the de- the current in the circuit when the distance is

FFr
Broglie wavelength of lithium and proton will be reduced to dll.
(a) 0-1 (/;) 0-2 (a) I {b)21
(c) 0-3
for
ur
(d) 0-4 (c)4/ (d) in (CBSE 2020)
29. If the de-Broglie wavelengths of an alpha particle 36. The graph showing the correct variation of linear
kkss
and a proton are the same, then the ratio of their momentum(p) of a charged particle with its
velocities is :
Yo
de-Broglie wavelength(>.) is
oo

4
eB

(«) 4 (b) -
r

(0- (d) 1
ou
ad

30. X-ray photon of wavelength 10 A and a particle of


YY

mass x/1 have the same energy. What is the value


of a: ? [/r-Planck’s constant] (use E = me-),
ndd
Re

(a) \0h (h)h/]0


Fi

(c)/i/100 (d) 100 h


31. Velocity of particle is 4 times velocity of electron
and the ratio of de-Broglie wavelength of particle
and electron is 2 : 1. What is the ratio of mass of
the particle and mass of the electron ? (CBSE 2020)
(a) 8 (b) 1/8 37. If photons of frequency v are incident on the
(c)4 id) 1/4 surfaces of metals. A and B of threshold frequncies
32. For a certain incident wavelength on a metal v/2 and v/3 respectively, the ratio of the maximum
surface, the maximum kinetic energy of the kinetic energy of electrons emitted from A to that
from B is :
photoelectrons is 4-8 eV. If the incident wavelcngtli
is doubled, then the maximum kinetic energy (a) 2-.3 (/;) 3 : 4
changes to 1-6 eV. The threshold wavelength for
the metal surface is : (c) I : 3 id)

[U.se he = 1240 nm eVj (CBSE 2020)


11/30 "pfuicUt^’A Fundamental Physics (XII)EEHD

II. Assertion-Reason Type Questions 46. Assertion. In photoelectric effect, on increasing


the intensity of light, the kinetic energy of the
Direction. For each of the following questions,
emitted photoeleclrons will be increased but the
two statements are given, one labelled As.sertion photoelectric cuncn' mains unchanged.
(A) and the other labelled Reason (R). Select the
Reason. The phoiooectric current depends only
correct answer to these questions from the codes
on wavelength of the incident light.
(o), (h) (c) and (d) as given below’:
47. Assertion. If frequency of the incident radiation
{a) Both A and R are true, and R is the correct
on a photosensitive surface is increased, then the
explanation of A
value of stopping potential will increase.
(b) Both A and R are true, but R is not the correct
Reason. Stopping potential (Vq) is directly pro
explanation of A
portional to the frequency of incident radiation.
(c) A is true, but R is false
48. Assertion. The work function of aluminium is
(d) A is false, and R is also false

w
4-2 eV. Emission of photoelectron will not be
38. Assertion. It is harder to remove free electron from possible if two photons each of energy 2-5 eV strike
copper than from sodium. the surface together.

Flo
Reason. Copper is a hard metal as compared to Reason. Photoelectric emission is explained by
sodium metal. Quantum theory of light.

reeee
39. Assertion. Ultraviolet light is incident on two 49. Assertion. When a monochromatic light is incident
photosensitive materials having different work

FFr
on a photosensitive surface, all the photo- electrons
functions. The kinetic energy of the emitted emitted do not have the same energy.
photoelectron will be greater for photosensitive Reason. The energy of photoelectrons emitted from
material having low work function.
for
ur
inside the metal surface is lost in collision with
other atoms in the metal.
kkss
Reason. K.E. of a moving particle = - mu“.
2 50. Assertion. If the work function of a metal is 4>o
Yo
and frequency of the incident light is v, then there
oo

40. Assertion. A photon cannot exist at rest.


Reason. Rest mass of a photon is zero.
eB

is emission of photoclcctrons if v < — .


41. Assertion. A photon and electron have same de-
Broglie wavelength, the energy of electron is Reason. For no photoelectric emission, the freq
uency of incident light is equal to the threshold
r

greater then that of photon.


ou
ad

frequency.
Reason. Photon is not a pciriicle but electron is a
51. Assertion. In a photoelectric experiment, the
YY

material particle.
42. Assertion. The currecl in the phototube increases stopping potential (Vq) plotted against the
frequency (v) of the incident light, the resulting
ndd
Re

with the increase in frequency of incident radiation.


graph is a straight line whose slope is independent
Reason. The photoelectric current is directly
Fi

of Vq
proportional to the frequency of incident radiation.
Reason. The slope of straight line between Vq and
43. Assertion. There is no difference between light
V is dh, where h is Planck’s constant and e is the
waves and matter waves.
electronic charge.
Reason. Light waves and matter waves travel with
52. Assertion. A particle which has zero rest mass and
the same speed in vacuum.
non-zero energy and momentum must travel with
44. Assertion. The de-Broglie wavelength of a certain the speed of light in vacuum.
charged particle is X. If the accelerating potential
Reason. A particle with zero rest mass is a photon.
(V) applied on a charged particle is doubled, then
the associated de-Broglie wavelength becomes 53. Assertion. The photoelectrons produced by a
monochromatic light beam incident on a metal
surface have a spread in their kinetic energies.
Reason. The energy of electrons emitted from
Reason. de-Broglie wavelength X Vv. inside the metal surface, is lost in collision with
the other atoms in the metal.
45. Assertion. A photon has zero rest mass.
(CBSE Sample Paper 2022-23)
Reason. A photon can move with infinite velocity.
DUAL NATURE OF RADIATION AND MATTER 11/31

ANSWERS

I. Multiple Choice Questions


1. (h) 2. (h) 3. UD 4. {a) 5. ib) 6.(^ 7. (/?) 8. (b) 9. (<0 10. (c)
11. (/’) 12. id) 13. (c) 14. (c) 15.(a) 16. (/?) 17. (a) 18.(c) 19. (c) 20. (6)
21. W 22. (c) 23. (h) 24. (c) 25.(c) 26. id) 27. (/.) 28. (&) 29.(a) 30. (a)
31. (b) 32. (b) 33. (^/) 34. (b) 35.(c) 36. (fc) 37. (fc)

II. Assertion-Reason Type Questions


38. (/;) 39. ib) 40. ia) 41. ic) 42. id) 43. (^0 44. (i/) 45. ic) 46. (iO 47. (fl)
48.(h) 49. (a)

ww
50. (^/) 51. (c) 52. (fl) 53. ic)

HINTS/EXPLANATIONS For Difficult Questions

Floo
ee
I. Multiple Choice Questions 8. eV,0
= hiv-VQ) or V'y = -(v-vy)

eer
1. Energy of incident photon, e

FFr
^ he 12400eVA ,, 6-62x10-3'* 14
^) = 1-6x10-'^ X [8-2 X 10 -3-3 X lO*'*]

oorr
uur r
\ 3500 A s ff
As incident light energy £ > <()g but £ < , so = 2-03 V
photo-electrons are emitted from metal B. 1
2 = /iv = (1)(,
sk
9.
YYoo
2. <|)o = 3-2eV = 3-2x I-6x 10-‘^J — mv
2
max
ooko

he he _ (6-6x1Q-34)x(3x10^)
0,, = — or Xf.= —
^mvf = 3 00 -‘}>o = 2 (|)o
eBB

Case (0,
" <t>o 3-2x1-6x10-'9
= 3-867 X 10-^ m = 3867 x 10-“^
^mvl =500-00 = 400
m
Case (h).
uurr

= 3867 A
ad

3. The wavelength 4800 A belongs to blue light. So,


Yo

2 V

photoelectric are emitted from a material if ^_2_2 or

illuminated by light from 40 W blue lamp.


u?2 ~ 4 “ 2 ^2 ^
dY
Re

he I 10. Max. KE of emitted photoelectron


nind

4. £ = — or £ « —. As the energy of photon of


X Am
= £-({)q = 6-2-4-2 = 2-0 eV
FFi

wavelength 1800 A is more than that of 2300 A,


= 2-0 xl-6 X 10-'9 J = 3-2 X 10-*® J
so photoelectric emission takes place. 11. Here, % = 5-5 eV, = 4-5 V
5. Stopping potential is independent of the intensity From Einstein’s photoelectric equation
of the incident light. Hence, graph between Vy he
and I is a st. line parallel to I-axis. - = eVy 00 = e X 4-5 V -H 5-5 eV = 10-0 eV
6. The KE of emitted photoelectrons varies from zero
he 1240 eV nm
or X= = 12 nm
to certain maximum value, so does the velocity of lO-OeV 10-0 eV
emitted photoelectrons.
7. The number of photoelectron emitted is directly The radiation of this wavelength belongs to
ultraviolet region.
proportional to the intensity of the incident
radiation and energy of photoelectrons emitted 12. Photoelectric effect can be explained by Quantum
theory of light.
depends upon the frequency of the incident
radiation and not on the intensity of incident 13. X0
he 12420eVA
= 3000 A

radiation. Hence, option ib) is true. 'o 4-14 eV
11/32 pfuzelee^’4. Fundamental Physics (Xll) VOL.II

he
14. Kinetic energy, K = - mv^ or mv
= pmK 27. Energy of a photon, ^ or E « l/X. As X
2
increases, energy E decreases.
h
De-Broglie wavelength, X, = — = 28. Charge on lithium is 3 times that of proton
mv
^j2mK i.e., qi^i/qp = 3 ; my/mp = 8-3.
1 1
or K = Koc qV = -
- mv^ or mv = ^Iq^Vm
X'^xl m m ^ 2
h
As mQ>mf^>mp>mg 1 _ ^ _
,% De-Broglie wavelength,
so Kq < < Kp < Kg ^2qVm

ooww
15. Max KE, = hv - iJiq. As <|)o increases 1
decreases. or X oc ('.● h and V are constant)
Vi m

16. X = -f^
1
or X OC

pmK 4m
Vi = —=1

e
As mg<mp< mf^ < m^, Xg>Xp>X„> X

re
a 24-9 5
17. Let n be the no. of photons striking the screen per

rFFl
ree
= 0-2

F
second ; The intensity of the incident beam of
photons of frequency v is 29. Given, X^ = Xp

rF
I 1
I=nhv or v = or V ec —
h
V m
-P _ 1^

fsfoor
h a _
ouur
nh n or or
Up 4
kosk
V In 2
30. Here, X = 10 A = 10 x m = 10"’ m
Yo
B
Mass of particle, m = x/3
oo

18. Knowledge based question.


Y
BB

19. Kmax = eVQ = hv-^Q


Stopping potential Vq is maximum for the incident
Energy of particle, E = mc^ - i-V
rre

light of maximum kinetic energy. Which is for volet


he _(x)
oYuu

light. C2
As per question, T" ~ VT
ad

A
hv <i> 0
20. eVo = hv-% or =—
dY

e 3h 3h
= 10h
The slope of the graph of Vq ^ is h/e. ^ Xe 10-9x(3x10«)
or
innd
Re

he he 1
or X0 - or X0 OC 2
Fi
F

0 <t>o <!> 0 31. Here, Vp = 4Vg\ Xp/Xg


x: h
= 2 or X'o = 2X 0 As, or m = —

^0 V2 mv Xu

h
X — — i.e., X
1 m
P_ _ \ -yVg _ 1 J__l
XpXVp “2^4~8
23. OC

P p me
he he he he
32. KE^max 4-8eV = V ...(0
X2 Pi p ~X~X 0 X X0
12-27 12-27
25. X = A = = 0-1227 A = 0-01227 nm
1-6 eV =
he he
4v and
2X 1,0
26. As incident photon of energy 2-5 eV is less than,
work function 4-5 eV of the metal surface, so no he 2 he
or 2xl-6eV = -r- ...(«●)
photoelectron will be emitted. 1
DUAL NATURE OF RADIATION AND MATTER 11/33

Subtracting («) from (0, we get Assertion is true. Here Reason is also true but it
he cannot explain the Assertion.
(4-8-2X l-6)eV = —
39. K.E. of emitted photoelectron is, KE = hv -
^0
smaller is the value of (])o, greater is the value of
he he K.E. Thus Assertion is true. Here, Reason is also
or
4-8-3-2 1-6 eV true but cannot explain the Assertion.
40. Both Assertion and Reason are true and Reason
, _ 1240(nmeV) is the correct explanation of Assertion.
^0 ■ 1-6 = 775 nm
or

41. Here, Assertion is true but Reason is false. As


33. Here, = 2 07 eV
photon is also a particle showing a packet of energy.
X he For Photon, energy £j = hc/X .(1)
Xq For an electron moving with velocity v and

ww
effective mass m de-Broglie wavelength (X) is
he 12400 eV A given by
or Xq = — 6000A
2-07 eV
h

Flo
or m =
1 mv Xv
34. As, K = -mv^ or = ^2mK

ee
2
Total energy of electron

rere
X=± h 1

r FF
or X oe he^
mv
pmK ^mK E2 = nuP' = .(2)
Xv
uurr
m^K
a a =
¥ m
4m 1
X- =- =1:1
4
1
1
- -
foor
From (1) and (2), we have
ks s
^2 _ he^fXv
Yoo
= -V > 1 or ^2 >
ooook

1 hefX
35. Intensity of light OC and
(distance)^
eBB

42. Both Assertion and Reason are false as photo


current oc intensity of light electric current is independent of frequency of
incident radiation but depends upon the intensity
uurr

= 4 or /' = 4I of incident radiatio.


/ (d/2)2
ad

43. Both Assertion and Reason are false because


Yo

velocity of light in vacuum is constant, whereas


36. Since, de-Broglie wavelength, X = —
dY

P the velocity of matter waves in vacuum depends


upon the wavelength of matter waves.
Re
innd

h 1
or P = - or 44. Both Assertion and Reason are false because
FFi

Thus the variation between X and p is a rectangular eV = —2 mv^ or mv = ^2meV


hyperbola, t.e.. Option (b) is true.
37. Max. KE, K,max = hv = hvQ = h(y- Vq) h h
de-Broglie wavelength, X =
. _ h(v - v/2) _ v/2 _ 3
mv
^2meV
B h(v-v/3) ~2^ ~ 4 1
i.e., X OC —

V
II. Assertion-Reason Type Questions
X
38. Higher is the work function of a metal surface,
V 1 ^
or X|=-=
. _

larger will be the energy spent to eject the X ^|2V


photoelectron from it. As work function of copper 45. Here, Assertion is true but Reason is false as the
is more that of sodium, so it is harder to remove maximum velocity of a photon is the velocity of
free electron from copper than from sodium. Thus light (= 3 X 10^ ms“^) in vacuum.
11/34 Fundamental Physics (XlDCZsISD

46. Both Assertion and Reason are false as photo


electric current depends on intensity of incident As, (j>U = /IVq. So. Vq - h
light and not on wavelength of light. Moreover, Thus Both Assertion and Reason are false.
the KE of emitted photoelectrons remains unchan
h
ged with increase in intensity of incident light. 51. As, eVQ = /iv- 00 or
0

e
47. Both Assertion and Reason are true and the
Rea.son is the correct explanation of Assertion. which is similar to equation of a straight line
48. Both Assertion and Reason are true but Reason y = na+C, whose slope m = h/e.
Thus Assertion is true but Reason is false.
can not explain correctly the Assertion. There
52. Both Assertion and Reason are true and Reason
will be emission of phoioeleclrons if the energy of
an incident photon is greater than work function is the correct explanation of Assertion because
of metal. photon travels with the .speed of light in vacuum.

ww
53. Here, Assertion is true but Reason is false,
49. Both Assertion and Reason are true and the
because the photoelectrons prodoced by a
Reason is the correct explanation of Assertion.
monochromatic light beam incident on a metal

Floo
50. There will be photoelectric emission if the surface have KE from zero to certain maximum
frequency (v) of the incident light is equal to or value as different electrons in a metal surface are

ee
greater than threshold frequency Vq, i.e., v > Vq. bound by different energies.

eer
FFr
oorr
uur r
s ff
3. If the intensity of light falling on the emitting
I. Photoelectric Effect
substance of a photoelectric cell be increased
sk
YYoo
1. It is easier to remove an electron from sodium then what will be the effect on (i) current
ooko

than from copper. Which metal has higher flowing from the cell, (ii) potential difference
eBB

value of threshold frequency ? required to stop the current completely ?


Sol. The minimum energy required to remove an Sol. (0 On increasing the intensity of the incident
electron from the surface of metal without light, the number of photoelectrons emitted per
uurr

giving any kinetic energy is called work second and hence the photoelectric current will
ad

function. The work function of sodium is less increase, (ii) Since, the slopping potential is
Yo

than that of copper. Therefore, it is easier to independent of the intensity of the incident light
remove an electron from sodium than from but depends on the frequency of the incident
dY

copper. light, hence the stopping potential will remain


Re
nind

As, work function, 0q = /iVq, therefore, unchanged.


threshold frequency, Vq = 0q//j 4. When a metal surface is exposed with a
FFi

It shows, the threshold frequency of copper is monochromatic light, then all the photo
electrons are not emitted from the metal
more than that of sodium.
surface with the same kinetic energy. Why ?
2. Blue light can eject electrons from a photo
sensitive surface while orange light can not. Sol. In photoelectric emission, the maximum K.E.
Will violet and red light eject electrons from is acquired by that electron which is most
the same surface ? loosely bound to the metal. Since other electrons
Sol. The photoelectrons can be emitted from a metal of metal require different energies for their
emission, hence the different photoelectrons are
surface if the frequency of incident radiation is
more than the threshold frequency, i.e., more emitted with different kinetic energy.
than that of blue light for the given surface. As 5. For three different materials, the variation
frequency of violet light is more than that of of the stopping potential Vq and the wave
blue light, hence violet light will eject length X, of the incident light Is shown by
photoelectrons. But the frequency of red light curves a, b and c in Fig. 11(Q).5. Which
material has maximum work function and
being less than the blue light, can not eject
which one has least work function?
photoelectrons from the given surface.
DUAL NATURE OF RADIATION AND MATTER 11/35

Vo A FIGURE 11 (Q}.5 AV h
slope of the line AB =
Av e

Here h is Planck’s constant and e is the


electronic charge.
Intercept on potential axis = -
Work function (})q = e x magnitude of intercept
on the potential axis
7. Fig. 11(Q).7, shows the variation of stopping
Sol. From the graph, we note that for the given potential with the frequency r of the inci
incident light of wavelength X, the material a dent radiation for two photosensitive metals
has highest stopping potential and material c FanAQ:

oww
has lowest stopping potential. As FIGURE 11(Q).7
he
'0

and stopping potential is the negative potential

e
applied to collector w.r.t. emitter to stop the

ree
rFl
fastest photoelectron to reach the collector.

Fre
Hence, Vq is negative.
(i) Explain which metal has smaller threshold

rr F
Therefore, for the given value of X, Vq « (J)q (in
magnitude). Hence, the work function is wavelength
maximum for metal a and least for metal r. (ii) Explain, giving reason, which metal emits
ouur
6. Draw a graph showing the variation of sto
pping potential with frequency of the incident
sfoo
photoeicctrons having smaller kinetic energy,
for the same wavelength of incident radiation,
kks
radiation. What does the slope of the line with (iii) If the distance between the light source
Yo
oooo

frequency axis indicate ? What information and metal P is doubled, how will the stopping
can be obtained from the value of intercept potential change. (CBSE 2008)
eBB

on the potential axis ? (CBSE 2006) Sol. (/) Threshold wavelength,


Sol. The variation between stopping potential and c
or X
1
frequency v of the incident radiation on a 0
uurr

^0
photosensitive surface is a st. line AB as shown
ad

in Fig. 11(Q).6.
YYo

> 1
(Vq)p
dd

So, (Xq)p > (Xq)^ or < (X^p


Re
iinn

It means metal Q has smaller threshold


wavelength.
F

(ii) From Einstein’s photoelectric equation


he he
K.E. = —
X X. 0

For given value of X, (hdX) is constant. From


above relation, we note that if X.Q is small, then
K.E. emitted is small. Thus metal Q with
smaller Xq will emit photoelectrons of smaller
Here, slope of line AB = K.E.
Av
From Einstein photoelectric equation, (iii) If the distance between light source and
metal P is doubled then intensity of light
hv <1),0
eV^= or ,V becomes one- fourth (as / « 1/r^) but frequency
e of incident light remains unchanged. As
It is an equation of straight line as shown by stopping potential of a metal is independent of
line AB in figure intensity of incident light but depends upon the
11/36 “Pna-deep- 'a Fundamental Physics (XII)
frequency of the incident light, hence, stopping Sol. (/) Since the saturation cunem is same for all
potential of metal plate P remains unchanged the three curves, so the intensity of the incident
by changing the distance between source of radiation is same for all the three radiations.
light and metal plate. (i7) From graph, the value of stopping potential
8. Fig. 11(Q).8. shows variation of stopping is highest for the radiation of frequency Vj, so
potential (Vq) with frequency (v) for two the frequency V] is highest.
photosensitive materials and M^. II, Photttns

10. A source of light is emitting photons. Does


all the photons emitted have the same energy?
Is the source monochromatic ? Explain.
Sol. A monochromatic source of light emits photons
of single frequency (v). Energy of each photon

ww
is hv. The photons emitted by a source of light
have different energies as the different atoms
(i) Why is the slope same for both lines ? of the source of light are excited to different
(ii) For which material will the emitted

Flo
states. Therefore, we can say that a source of
electrons have greater kinetic energy for the light is not a monochromatic source of light.

e
incident radiations of the same frequency ?
U. A lamp mainly emits light of wavelength X.

rere
.Justify your answer. (CBSE 2009)
The lamp is rated at P watt and 8% of the

r FF
Sol. (i) From photoelectric equation, we have energy is emitted as visible light,
(i) How many photons of light are emitted
eVo = /n’-<t>o or V =.^-5o
uurr
by the lamp per second ?
e e

It is an equation of straight line with slope file


(= constant). It means the slope of Vq - v graph
foor
(ii) How many photons are falling per second
on a square whose length of each side is u,
kss
(= hie) is same for both metals Mj and M2. held perpendicular to the incident photons
Yoo
ooook

at a distance r from the lamp.


(//) Also, K max. = hv hvQ. For the given
frequency of incident light, the smaller is tlie he
eBB

Sol. (i) Energy of each photon, E = —


value of Vq, the larger is the value of K max.
and A
vice versa. Since material M, has lower value Light energy emitted per second by lamp
of threshold frequency Vq, so metal M j will emit
uurr
ad

photoelectrons of greater K.E. P


100
Yo

9. The graph of Fig. 11(Q).9, shows the vari


ation of photoelectric current with collector No. of photons of light emitted by lamp per
dY

8 P/100 8P;
plate potential for different frequencies of second. N =
Re

incident radiations. hclX ~ 100/ic


innd

FtGURE 11(Q).9 (n) No. of photons falling per second on square


Fi

N 2 _
%PX a~
-xa X
47ir" lOO/ir 47tr- 50 7thcr“

12. (a) Calculate the energy and momentum of a


photon in a monochromatic beam of
wavelength 331*5 nm.
(h) How fast should a hydrogen atom travel
in order to have the same momentum as that
of the photon in part (fl). [Use h = 6*63 x
10-'^ J.S, c = 3 X 10^ ms-‘] (CBSE 2020)
Sol. {a) Energy of a photon.
(i) Which physical parameter is kept constant
for the three curves ?
E = —
he (fr63xl0-^^)x(3x!0^)
(ii) Which frequency (vj, V2 or V3) is the X 331-5x10-9
highest ? (CBSE 2019, 2009)
= 6x 10*9 J
DUAL NATURE OF RADIATION AND MATTER 11/37

Momentum of photon in terms of wavelength Sol. Since, work function of a surface can be neg
IS
lected so, (|)q = 0. From photoelectric equation
_ 6-63x10-^^
h 1 2 ....
^~X~331-5xI0-'-' = 2 X 10-27 kg
Kmax. = E^, + 4>o = £■,, and K max.
= -mv
max.
2

(/)) For hydrogen atom, mass,/n= l -66x 10'“^ kg. {mv max.
If the hydrogen atom is to have the same
2 m
momentum as that of the photon in part (a),
then, 2 x 10““^ = mv = 1-66 x 10“-^ X V
or (m V max max

2x10--'^
V = = 1-2 ms”* Wavelength of the emitted electron is
-27
1-66 xl0‘
h h
X = , i.e., X«
III. De-Brogiie hypothesis mv

w
max

13. X-rays of wavelength X fall on photosensitive Therefore, a graph of X versus will be a


surface, emitting electrons. Assuming that the
parabola as shown in Fig. I1(Q).10.

Flo
work function of the surface can be neglected,
prove that the de-Broglie wavelength of FIGURE 11(Q).10
Xa

e
rree
hX
electrons emitted will be .(CBSE2004)
y 2mc

r FF
Sol. As work function of a surface can be neglected
uurr
.so, <1)q = 0. Therefore, kinetic energy of emitted
photoeleciron for Ev
kss
1
2 _ he he 2 m he
— mv or mv = 16. A photon and an electron have got same de-
ooook

2 X X V X -10
Yo
Broglie wavelength (« 10 m) which has
de-Broglie Wavelength of emitted photoelectron greater kinetic energy ? Explain.
eB

Sol. For electron. Let X be the de-Broglie


h h hk
wavelength of electron.
(Proved)
mv
■Jimhc/X ) 2mc
urr

1
K.E. of electron, E, = —
- mv^ or mv^ = 2 E 1
ad

14. If an electromagnetic wave of wavelength X ' 2


Yo

is incident on a photosensitivesurface of
negligible work function. If the photo or mv - ^2£j m
dY

electrons emitted from this surface have the


h
Re
innd

de-Broglie wavelength X|, prove that As X =


mv m

( 2mc \
Fi

x = X}. (CBSE 2008)


h
1
h^-
or Ej = ...(0
Sol. From solution of above problem, we have 2X^ m

/ imc
7 For photon of wavelength X., Energy is given
hX hX
X.I = or XrI = or X = 2?1 he
2mc 2 me / by. ^^2=T-
X
(Proved)
15. X-rays fall on a photosensitive surface to E2 _hc ^2}e^m 2cX m

cause photoelectric emission. Assuming that E


1
X. /|2 h
the work function of the surface can be
neglected, find the relation between the de-
2x3xl0^xl0-‘bx9xl0-3>
Broglie Wavelength (X) of the electrons 6.6x10-3'^
emitted to the energy (£^.) of the incident 2x3x9x10 90
photons. Draw the nature of the graph for X = —>1
as a function of . [CBSE 2014 (C)l 6.6 1.1
11/38 “P^uidee^ Fundamental Physics (XII) tWII
Hence, E-, > £] This is as stated by Bohr about the stationary
It means kinetic energy of photon is greater orbits.

than that of electron. 19. In the nth orbit of hydrogen atom, find the
ratio of the radiu.s of the electron orbit and
17. A proton and an electron have same de-
Broglie wavelength which of them moves fast de-Broglie wavelength associated with it.
and which possesses more K.E. .Justify your Sol. In rjth orbit of hydrogen atom, the velocity v n

answer. of electron is given by


Sol. Kinetic energy /C of a particle of mass m having nh nh
or V
●'n
■ f ^
n
2ji Inmr
or /? = ^2 m K
n
momenium p is, K =
2 m de-Broglie wavelength,
h h /. 2jt m r, 2Ttr. n

De-Broglie wavelength, X. =— 2 = — = —X -■= ^

w
p ^2mK m V, m nh n

r n
h /»2 ● JL —
...(/) and K = ...Hi)
”=x X 2n

Flo
2 m %?
20. If the intensity of the incident light (whose

e
If X is constant, then from (/), p = a constant,

reee
frequency is greater than threshold
V
frequency) is increased then explain its

Fr
i.e.. or
effect on (i) the maximum momentum that
V m
e p photoelectron could have and (ii) the
minimum de-Broglie wavelength that

for
ur
or
^p<^e
If X is constant, then from (ii), K « \lm photoelectron could have.
K„ Sol. (0 Maximum K.E. of the emitted photo- electron
ks
m
of momentum is given by
or
Kp<K,
Yo
K m 2
oo

e P
K
Pmax
It means the velocity of electron is greater than max
2 m
eB

that of proton. Kinetic energy of electron is 1/2


or
Pmax = [2 (hv - 4)o)]
greater than that of proton.
where 00 the work function of the metal
18. Show that de-Broglie hypothesis of matter
ur

surface used on which light falls. The above


ad

wave supports the Bohr’s concept of relation is independent of intensity of the


Yo

stationary orbit. (CBSE2011)


incident light {i.e., number of photons falling
Sol. According to de-Broglie hypothesis, the per second per unit area of light) but depends
wavelength of the wave associated with electron on the frequency of the incident light.
d
Re

while moving with velocity v is given by


If intensity of the incident light is increased
in

without changing its frequency, the maximum


F

...(0 momentum of the emitted photoelectrons


mv
remains unchanged.
According to de-Broglie, stationary orbit is that
orbit whose circumference is integral multiple (ii) The minimum de-Broglie wavelength of the
of wavelength of wave associated with electron emitted photoclectron of maximum momentum
in that orbit. h
Fmax IS, X_:
If X is the de Broglie wavelength of electron mm
P max
while revolving in nth orbit of radius r. then
2 TCr = nX or X = 2 nrhi It is also independent of intensity of incident
...(//■)
light. Thus, minimum de-Broglie wavelength
From (0 and (ii).
of emitted photoelectron remains unchanged
2 nr h nh with the increase in intensity of the incident
or mvr =
n mv 2n light.
21. Fig. ll(Q).ll shows the variation of de-
( h \
i.e.. Total angular momentum =n Broglie wavelength (X) verses \fyjv (where
2n) V is the accelerating potential) for two
DUAL NATURE OF RADIATION AND MATTER 11/39

charged particles A and B. Which particle Sol. When a charged particle of charge q, mass iii is
have the heavier mass ? Explain. accelerated under a potential difference V, then
de-Broglie wavelength associated with particle
h
IS,
^=n==
yjlmqV
If V is constant, then

w
X oc

From graph, so It means the


particle A is of heavier mass.

e
roow
re
● Very Short Answer
● Short Answer

reF
● Long Answer

uFFll
e
VERY SHORT ANSWER QUESTIONS Carrying 1 mark

sFr
foro
I. Photoelectric Effect 2. The threshold wavelength for photoelectric

fk
uor
emission for a material is 5200 A®. Will the
1. The graph Fig. 11(Q).12 shows the variation photoelectrons be emitted when this material
okso
of photoelectric current {/) versus applied is illuminated with monochromatic radiation
voltage (V) for two different photosensitive
Y
from 1 watt ultra violet lamp ?
Yo
materials for two different intensities of the
oo
reeBB

Ans. Yes ; because wavelength of ultra violet light is


incident radiation. Identify the pairs of less than the threshold wavelength 5200 A“.
curves that correspondto differentmaterials 3. How will the photoelectric current change on
rY

but same intensity of incident radiation. decreasing the wavelength of incident radi
ouu

(CBSE 2013) ation for a given photo-sensitive material ?


ad

Ans. Photoelectric current is not affected on


Ydo

FIGURE 11(Q).12
decreasing the wavelength of incident radiation
provided its intensity remains unchanged.
nidn

4. The Fig. I1(Q).13 shows a plot of three curves


Re

a, b, c showing the variation of photocurrent


F

3 versis collector plate potential for three


Fi

different intensities /|, Ii and having


1 4
frequencies Vj, V2 and V3 respectively, incident
on a photosensitive surface. Point out the two
■►V

3
Ans. The similar materials will have same slopping
potential and the different materials will have ●2
different stopping potential whatever may be the I
intensity of incident radiation.
For the same intensity of two different incident
radiations on two different materials, there will
be same value of photoelectric current. So the
pair of curves (I and 3) or (2 and 4) correspond
»
to different materials but having same intensity COLLECTOR
of incident radiations. PLATE POTENTIAL
11/40 ‘Pnadee^ 4^ Fundamental Physics (XII)
curves for which the incident radiations have 11. What determines the maximum velocity of
same frequency but difierent intensities. the photoelectrons ?
(CBSE 2009) Ans. The frequency of the incident radiations and
work function of the metallic surface.
Ans. Since the value of stopping potential is same
for curves a and b and have different saturation 12. Define threshold frequency. Is it a constant
current, hence frequency Vj and v-, are the same quantity for a metal surface ? Comment.
but their intensities are different. (CBSE 2019)
5. Why arc alkali metal surfaces most suited as Ans. For a given metal, there exists certain minimum
photo-sensitive surfaces ? frequency of the incident radiation below which
Ans. The work function of alkali metal is very low. no emission of photoelecirons takes place. This
Even the ordinary visible light can bring about frequency is called threshold frequency. It is a
emission of photoelectrons from these metals. constant quantity for a given metal surface.
Due to this reason, alkali metal surfaces are most 13. Define space charge. What is its effect on the

w
suited as photosensitive surfaces. emission of photoelectrons from a metal
6. A source of light is placed at a distance of surface ?
Im. from a photocell and the cut off potential Ans. Space charge is the region or space around a

Flo
is found to be V^. If the distance is doubled metal surface (i.e., cathode) in which the photo
what will be the cut off potential ? electrons emitted from it get struck up when

reee
(CBSE 2008) anode (i.e., collector) is not given any positive
Ans. Vq. When distance is doubled, the intensity of potential. The creation of space charge decreases

FFr
incident light becomes one-fourth (I« l/r“), but the probability of the emitted photoelectrons
the frequency of incident light remains reaching the anode (or collector).
unchanged. The value of cut off potential of a
for
14. Is photoelectric emission possible at all
ur
material only depends upon the frequency of frequencies ? Give reason for your answer ?
the incident light and is independent of the Ans. No, the photo-electric emission from a metal
kkss
intensity of incident light. surface is possible if the frequency of the
Yo
ooo

7. Give an example each of a metal from which incident light is greater than the threshold
photoelectric emission takes place when frequency (vq) for that metal or the energy of
eB

irradiated by (i) UV light («) Msible light. the incident photon is greater than the work
(CBSE 2022) function of metal, i.e., v > vq, where
Ans. (0 Zinc (») Alkali metals, e.g., sodium IS. Two metals A and B have work functions
r
ou

4 eV and 10 eV respectively. Which metal has


ad

8. The work function of lithium and copper are


2-3 eV and 4*0 eV respectively. Which of these higher threshold wavelength ? (CBSE 2004)
YY

metals will be useful for the photo-electric he


cell working with visible light ? Explain. Ans. Work function, (J)g =—, where Xq is the
nndd
Re

Ans. Lithium, because its work function is less than


the energy of the visible light photons. threshold wavelength. So, « l/<|)o. It means a
Fi

9. It is harder to remove free electron from metal A with lower work function has higher
copper than from sodium. Which has higher threshold wavelength.
work function ? 16. Does the stopping potential in photoelectric
Ans. The work function of copper is higher than that emission depend upon the intensity of the
of sodium. incident radiation in a photo cell ? Comment
on it. (CBSE 2005)
10. The work function of copper is 4.0 eV. If two
photons, each of energy 2.5 eV strike with Ans. No, the slopping potential does not depend upon
some electrons of copper, will the emission the intensity of incident radiations but depends
be possible ? on the nature of photosensitive surface and
Ans. One photon can eject one photoelectron from frequency of the incident radiation.
the surface of a metal provided its energy is not 17. What is the effect of decrease in frequency
less than the work function of metal. Since each of incident radiation on the stopping
photon has energy less than the work function potential in photoelectric emission ?
of copper, hence, photoelectric emission is not Ans. The value of stopping potential in photo electric
possible. emission from a metal surface decreases with
DUAL NATURE OF RADIATION AND MATTER 11/41

the decrease in frequency of incident radiation, Ans. The graph is shown in the Fig. 11(Q).14.
provided the frequency of radiation is greater
than threshold frequency.
When frequency of incident radiations becomes
equal to threshold frequency, stopping potential
becomes zero.

18. When the frequency of incident radiation is


less than threshold frequency for a metal
surface, what is value of stopping potential
for that and why ?
Ans. When the frequency of incident radiation
becomes equal to threshold frequency, the value
of stopping potential becomes zero. When the 24. In an experiment on photoelectric effect the

w
frequency of incident radiation becomes less graphs as shown in Fig. 11(Q).15 were
than threshold frequency no photoelectric obtained between the photoelectric current
emission takes place. Hence, the involvement {/) and the anode potential (V). Name the

Flo
of stopping potential does not arise. characteristic of the incident radiation that

19. The photo electric current at distances rj and was kept constant in this experiment.

ee
r2 of light source from photoelectric cell are
/j and I2 respectively. Find the value of/j//2.

Fr
1
Ans. / OC

for
ur
\2
I
I
So,
ks
'i
Yo
oo

20. How does the maximum kinetic energy of


electrons emitted vary with the work function
eB

of the metal ?
(Pb. Board 2011)
Ans. As, (K.E.) max = hv — $0, so maximum K.E. of Ans. As the value of stopping potential is same for
r

emitted electrons decreases with the increase in


all the curves, so the frequency of incident
ou
ad

work function of the metal.


radiations is kept constant but their intensity is
21. The frequency (v) of the incident radiation is
Y

different.
greater than threshold frequency (vq) in a 25. Work function of sodium is 2’75 eV. What
photocell. How will the stopping potential will be KE of emitted electron when photon
nd
Re

vary if frequency v is increased, keeping other


factors constant.
of energy 3*54 eV is incident on the surface
(CBSE Sample Paper 20201
Fi

of sodium ?
Ans. As, eVQ = h (v ~ Vq), therefore, the value of
Ans. KE = hv-%= 3-54 - 2-75 = 0-79 eV
stopping potential (Vy) increases with the
26. How does one explain the emission of
increase in frequency (v) of the incident
radiation. electrons from a photosensitive surface with
the help of Einstein’s photoelectric equation ?
22. If the intensity of the incident radiation in a (CBSE (D), 2017)
photocell is increased, how does the stopping
potential vary ? Ans. According to Einstein’s photoelectric equation
the maximum KE of emitted photoelectron is
Ans. As, eVQ= h (v - Vq), so stopping potential Vq
remains unchangedby the increase in intensity
K
of the incident radiation. max

23. Plot a graph showing the variation of


If the frequency of incident light v > Vg, then
photoelectric current with anode potential for K max is positive, i.e., the emission of photo
two light beam of same wavelength but
electrons takes place. If v < Vq, then A’,,,ax
different intensity. Mark the graph for the
(CBSE 2018) negative, no photoelectric emission lakes place.
radiation of higher intensity.
11/42 ‘P>uuiec^'4. Fundamental Physics rxinPTSTWl
n. Photons 34. Two particles have equal momenta. What is
the ratio of their de-Broglie wavelengths ?
27. What is the energy of a photon in eV corres (CBSE Sample Paper 2018)
ponding to the visible light of maximum
wavelength ? Ans. De-Broglie wavelength, X = — or X oc

Ans. Maximum wavelength of visible light (i.e. of p p


red light) is 7800 A.
Energy of red light, So, h P'y P
— =1 Iv =/J2=/’(say)j
P\ P
12400eV A
1*6 eV 35. What is de-Broglie wavelength of an atom of
X 7800
mass m, moving at absolute temperature TK.
28. What is the momentum of photon of energy

ooww
h h h
3 MeV in kg ms“^ ? Ans. X -
^ImK 2 m
r3
- kT ■yjs mkT
E 3x1-6x10“‘3 2
Ans. P = —
c 3x10^
= 1-6 X 10-21 ,^g ^-1
where k is Boltzmann constant.
29. Which photon is more energetic : A red one

e
36. What inference was drawn from Davisson

ree
or a violet one ? (Hr. Board 2002) and Germer experiment regarding the nature

rFl
Ans. Since frequency of violet light is more than that of electrons ? (ISCE 2002)

Fre
of red light, i.e., and energy of a Ans. This experiment established the wave nature of

rrF
photon = hv. So photon of violet light is more moving electrons as predicted by de-Broglie
energetic than that of red light. hypothesis.
ouur
III. De-Broglie hypothesis sffoo 37. The most probable kinetic energy of thermal
neutrons at a temperature of T Kelvin, may
be taken equal to kT, where k is Boltzmann
okks
30. What consideration led de-Broglie to suggest constant. Taking the mass of a neutron and
Yo
that material particles can also show wave its associated de-Brogliewavelength as m and
ooo

property ? Xjf respectively, state the dependence of Xg


BB

Ans. Nature loves symmetry, and matter can be on m and T. (CBSE Sample Paper 2011)
converted into energy and vice versa, i.e, matter
rr e

Ans. De-Broglie wavelength.


and energy are two forms of the same entity. h h h
ouu

31. Why are de-Broglie waves with a moving foot X


ad
YY

B ~
ball not visible ? P pmK 1
2m X
(3
—kl'
^
(CBSE 200: Hr. Board 2007, 2008) U J
1
dd

Ans. Dc-Broglie wavelength associated with a body «


B
of mass m, moving with velocity v is given by -JmT
Re
iinn

h
38. If the temperature is increased from 27°C to
F

X = 127"C. Find the ratio of the de-Broglie


mv
wavelength of wave associated with a
Since, the mass of foot ball is quite large, hence material particle at these two temperatures.
the de-Broglie wavelength associated with it is Ans. De-Broglie wavelength.
quite small, therefore it is not visible.
h h h
32. If an el ctron behaves like a wave, what
should determine the wavelength and P ^2mK 2m hr ^3 mkT
frequency of this wave. 2
Ans. Momentum and Energy.
i
33. Is there any difference between light waves or A cc
and matter waves ?

Ans. Yes, the velocity of light waves in vacuum is


constant, whereas the velocity of matter waves X 27 127 + 273 f4 2V3
in vacuum depends upon the wavelength of
matter wave.
" X 127 “V 27 + 273 "Vs " 3 = 1-155
DUAL NATURE OF RADIATION AND MATTER 11/43

39. Name the experiment used to establish the h


wave nature of electron (i) for slow moving Ans. X= — = or Xoc

electrons (ii) for fast moving electrons. p ^2mK VF


Ans. (0 For slow moving electrons, the wave nature
of electrons was established by Davisson and A.4
FIGURE 11(Q).17
Germer experiment.
(ii) For fast moving electrons, the wave nature
of electrons was established by G. P. Thomson
experiment.
40. In Davisson-Germer experiment, if the angle
of diffraction is 52°, then find the glancing
KE
angle.
Ans. Glancing angle, 0 = 90° - (}>/2 = 90° - 52/2
Thus the graph between X versus K.E. (K) is of

w
= 64"

41. de-Broglie wavelength associated with an the type as shown in Fig. 11(Q).17.
electron accelerated through a potential 44. A proton and an electron have same velocity.

Flo
difference V is X. What will be its wavelength Which one has greater de-Broglie wavelength
when the accelerating potential is increased and why ? {CBSE 2012)

e
to4V?

rree
1
Ans. De-Broglie wavelength X =

r FF
h I
Ans. X = or X OC - mv m

■yjl meV
uurr
-HI < 1 or
K~
X ~')j4V ~ 2
-1 or X' = -
for
V is constant. Then
'I. m
p
< X^.
kss
2
i.e.. De-Broglie wavelength of electron is more
than that of proton.
ooook

42. Show graphically, the variation of the de-


Yo

Broglie wavelength (A.) with the potential (V) 45. On what principle is an electron micro- scope
through which an electron is accelerated based ?
eB

from rest. (CBSE 2011) Ans. An electron microscope is based on de-Broglie


h hypothesis. According to it, a beam of electrons
urr

Ans. We know that X= behaves as a wave which can be converged or


ad

72 meV ’ diverged by magnetic or electric field lenses like


Yo

a beam of light using optical lenses.


dY

1
so X 46. Is the de-Broglie wavelength of a photon of
Vv an electromagnetic radiation equal to the
Re
innd

The variation of V and X will be a curve as wavelength of the radiation ?


shown in Fig. 11(Q).16.
Fi

Ans. Yes, de-Broglie wavelength for a photon is given


by
FIGURE 11(Q).16
h h
X =
mv p

where p is the momentum of photon and h is


the Planck's constant.

The momentum of a photon of frequency v is


given by
V
hv h h
P = —
or X = ~ ...(ii)
c X P
43. Draw a plot showing the variation of de-
Broglie wavelength of electron as a function It is the same wavelength as de-Broglie
of its K.E. (CBSE 2015 (C)l wavelength of the photon.
11/44 T^'tacUcft.'^. Fundamental Physics (XH)EHHD

SHORT ANSWER QUESTIONS Carrying 2 marks

the value of Planck’s constant be determined


I ■ Pimtoelectrlc Effect
from this graph ? (Raj, Board 2011)
1. Number of ejected photoelectrons increases Ans. The variation of stopping potential with the
with an increase in intensity of light but not frequency of radiation, incident on a metal plate
with the increase in frequency of light. Why ? is a straight line AB as shown in Fig. 11 (Q). 18.
Ans. One incident photon can eject one photoelectron
FIGURE 11(Q).18
from a photosensitive surface. Therefore the no.
of photoelectrons ejected per second depends
upon the no. of photons falling per second on
it. which in turn depends upon the intensity of
the incident light. The increase in the frequency

w
of the incident light simply increases the energy
of the incident photon but one photon of high
energy can not eject more than one photo

Flo
electron from a photosensitive surface.

reeee
2. An increase in the frequency of the incident
light increases the velocity with which photo

Fr
electron is ejected. Explain how ?
Take two points C and D on the graph. Note
Ans. In photoelectric emission, —mv^ =hv — ^Q.
for
down the corresponding frequency of radiation
ur
The increase in frequency v of incident light (vj and V2) 3nd stopping potential (Vj and V-,).
increases the energy of the incident photon. Then,
kkss
Since the work function <|)q of a given photo eV^= /n'l - and eV2= hv2-%
Yo
sensitive surface being fixed, therefore the
oo

kinetic energy of the photoelectron increases e(V2-VQ


e (Vj- Vj) = /j (v2~ Vj) OT h =
eB

with increase in the frequency of incident light ^2 -V,

due to it, the velocity v of photoelectrons Thus Planck’s constant can be determined.
increases.
r

3. Explain the term stopping potential and 5. Radiations of frequencies Vj and V2 are made
ou
ad

to fall in turn, on a photosensitive surface.


threshold frequency. (CBSE 2011)
The stopping potentials required for stopping
YY

Ans. Stopping potential. It is the minimum negative the most energetic photoelectrons in the two
potential given to the anode in a photocell for
cases are Vj and V2 respectively. Obtain a
ndd

which the photoelectric current becomes zero.


Re

formula for determining the threshold


If Vq is the stopping potential, then maximum frequency in terms of these parameters.
Fi

K.E. of emitted photoelectronis


(CBSE Sample Paper 2003)
hv 4>, 0 Ans. If Vq is the threshold frequency, then from
{K.E.) mux = eVo = Av - 0Q. ^0 = e photoelectricequation, we have
Threshold frequency. It is the minimum =/iVj - (J)q and eV2 = hv2~^Q
frequency of the incident radiation for which
e(V2-V^)
just emission of photoelectrons takes place from V,)=/j(v2-Vj) or/i =
a metal surface without any K.E. If Vy is the (V2-V,)
threshold frequency, then using Einstein’s Now, eVj = /ivj - il)o = - hvQ
photoelectric equation
eV,1 V- — V
2 1
= v, -eV.1
0 = H~4'o or
e(V2-V^)
4. Draw a graph to show the variation of
^1^^2-^l)
stopping potential with frequency of = V|-
radiation incident on a metal plate. How can (V2-V,)
DUAL NATURE OF RADIATION AND MATTER 11/45

1 2 he 2 2hc 2$ 0
y-'t’o
●. — m V
2
max
ink m

1 1
V1V2-V2V, or V
2 oc
or V oc
max max
VI
(V2-V1)
It means, as the wavelength (\) of the incident
6. When a given photosensitive material is
irradiated with light of frequency v, the
light decreases, the maximum velocity of
emitted photoelectrons increases.
maximum speed of the emitted photo
electrons equals Vmax* square of is 8. For a photosensitive surface, threshold
observed to vary with v, as per the graph wavelength is Xq. Does photoemission occur
shown in Fig. 11(Q).19. Obtain expressions if the wavelength (X) of the incident radiation
for (i) Planck’s constant and (ii) the work is (i) more than Xq (ii) less than Xq ? Justify
function of the given photosensitive material, your answer.

w
in terms of the parameter /, 11 and the mass Ans. The maximum kinetic energy of the emitted
m of the electron. (CDSE 2018 (O) photoelectron from a metal surface is given by
X.-X"l

Flo
FIGURE 11(Q).19 I ■> he he 0
2 K = —mvz = he
V max max
2 X X0 XX

e
0 ;

rree
(/) When X > Xq, is negative, v max
IS

r FF
imaginary. Thus photoelectric emission will
not occur.
uurr
(ii) When X < Xq, is positive, v max IS
for
positive. Thus photoelectric emission will take
place and K.E. of photoelectron increases as
kss
X decreases.
ooook

Ans. According to Einstein’s photoelectric equation 9. If we go on Increasing the wavelength of light


Yo

1 incident on a metal surface, what changes in


K the number of electrons and the energy take
eB

= —mv
max 0
place ?
2 0 Ans. If we increase the wavelength of incident light
urr

V V —
or
max then the energy of incident photon £ = he/X
ad

, m , m

will decrease. Due to it, the energy of photo


Yo

Thus the graph between and v will be a electron emitted decreases, but the number of
dY

electrons emitted will not change. When the


2h I
straight line. Slope of st. line = — wavelength of the incident light becomes greater
Re
innd

m n
than the threshold wavelength (i.e. the energy
. 24) 0 of the incident photon becomes less than the
Fi

Intercept on axis, = / work function of metal) then no photoelectron


m
will be emined.
[in magnitude]
10. (a) Calculate the frequency of a photon of
(/) Planck’s constant, h =
ml energy 6-5 x 10"^^ J.
2/1 (b) Can this photon cause emission of an
electron from the surface of Cs of work
m I
(ii) Work function, <[)q = function 2*14 eV ? If yes, what will be maxi
2
mum kinetic energy of the photoelectron ?
7. What is the effect on the maximum velocity [Use h = 6-63 x 10“^ Js]
of the emitted electron if the wavelength of
Ans. (a) Here, £ = 6-5 X 10“*^ J. (j)o = 2-14 eV
the incident light is decrea,sed ?
Ans. According to Einstein’s photoelectric equation ^ £ 6-5xl0-'^J
Frequency, v = —
he I h 663xl0-^^Js
K
-—(})q. But K = - mu?max
max
K
max
2 = 0-9 X 10'® Hz
11/46 Fundamental Physics (XIl)CHHD
(h) Energy of incident photon is (/) the stopping potential remains unchanged
6-5 xlQ-'^ (//) the photoelectric current gets doubled.
£= 6-5 X 10-’^ = eV = 4-068 eV (b) If the frequency of the incident radiation is
1-6x10-'^
doubled,
As. 4-068 eV > 2-14 eV. so the incident phulon
can cause the photoelectric emission from Cs.
(/) then the value of stopping potential becomes
more than double as explained below :
Maximum KE. ,Kmax
= 4-08-2-14= 1-928 eV
eVo = hv - (})o or liv = cVq + (J)o
and cVq = h 2v- (J)q
II. A source of light of frequency greater than
the threshold frequency, is placed at a or eVQ = 2 (eV^ + 0q) - t[)Q = 2 c’Vq +
distance d from the cathode of a photocell. or Vq' = 2 Vq + i^o/e. Thus stopping potential
becomes more than double.
The stopping potential is found to be V. If
the distance of the light source is reduced to (//) the photoelectric current remains
uneffected.

ww
dhi (where n > 1), explain the changes that
are likely to be observed in the (i) photo 13. Write two characteristic features observed in
electric current and (i7) stopping potential. photoelectric effect which support the photon
(CBSE Sample Paper 2018) picture of electromagnetic radiation.

Flo
Ans. (f) Photoelectric current depends on the (CBSE 2019, 2012)
Ans. The following features observed in photoelectric

ee
number of photoeleclrons emitted per second
which in turn depends on the number of effect helped to establish the photon picture of

rere
photons of light falling per second on a photo electromagnetic radiation.

rFF
sensitive surface i.e., the intensity of light. (/) The maximum kinetic energy of the emitted
When the distance of light source is reduced photoelectron is independent of the intensity of
uurr
from photocathode, the intensity of incident
light will increase [as intensity <= (distance)"^].
foor
the incident light but depends upon the
frequency of the incident light.
ks s
Therefore, photoelectric current will increase. (ii) For every metal, there is a certain minimum
Yoo
(t7) The value of stopping potential depends on frequency of the incident light (called threshold
oook

the max. kinetic energy of the emitted frequency) below which, no photoelectric
emission takes place.
eBB

photoelectrons, which is related with the


frequency of the incident light. (///) The photoelectric emission is an
When the distance of the light source from instantaneous process.
uurr

photocalhode is reduced, the energy of the 14. The threshold frequency of a metal is /q.
ad

incident photon will remain unchanged. Hence When the light of frequency 2 /q is incident
Yo

max. kinetic energy of the pholoelectron will on the metal plate, the maximumvelocity of
remain the same. So stopping potential will electrons emitted is Vy When the frequency
dY

remain unchanged. of the incident radiation is increased to 5 /q,


Re
innd

12. State how in a photo-cell, the work function the maximum velocity of electrons emitted is
of the metal influence the kinetic energy of V2- Find the ratio of V| and V2- (CBSE 2004)
FFi

the emitted electrons,


Ans. As/o is the threshold frequency, so (^Q = hfQ.
(a) If the intensity of the incident radiation Using Einstein’s photoelectric equation, we
is doubled, what changes occur in (i) the have
stopping potential and (ii) in photoelectric I
current? —mv
2 2=;,x2/o-()>„ = ;,x2/o-A/„ = A/„
(b) If the frequency of the incident radiation
I
is doubled, what changes occur in the
(i) stopping potential and (ii) photoelectric
and -tnv
2 2 =/,x5/„-(]>o =5/i/o-/i/o = 4*/o
current. (CBSE 2001) 2
_ 1
V V.
1
Ans. Maximum K.E. of the emitted photoelectron is or

given by = e Vq = /iv - 0Q. vl 4


From above relation we note that if 4>q is more, 15. Explain how Einstein’s photoelectric
then is less and vice versa. equation enables us to understand the
(a) If the intensity of the incident radiation is (0 linear dependence, of the maximum K.E.
doubled. of the emitted electrons, on the frequency of
DUAL NATURE OF RADIATION AND MATTER 11/47

the incident radiations (ii) existence of a Ans. (0 As the threshold frequency is higher for metal
threshold frequency for a given photoemitter. Q than that of meia! P, hence the work function
Ans. According to Einstein’s photo electric equation, (|)q = Hvq, is greater for metal Q than that of P.
(K.E.) nm = hv - hv 0 hv
(/) It means, above threshold frequency Vq, (//●) eV=hv-% or V = — e
°
e

maximum K.E. is directly proportional to v, i.e,

w
h AV h
(K.E.) max depends linearly on the frequency of AF = - Av or
the incident radiation. e Av e

{ii) When v < V(), (K.E.) maxbecome.s negative. AV h


The negative K.E. has no physical meaning. Slope of the line =
Av e

e
Hence there is no photoelectricemission below
where h is the Planck’s constant and e is the
the threshold frequency Vq.

e
electronic charge.
16. Green light ejects photoelectrons from a

o
wr
19. The work function of Cs is 2*14 eV. Find

r
given photosensitive surface where as yellow
light does not. What will happen in case of (fl) threshold frequency for Cs. (A) wave

F
violet and red light ? Give reason for your length of incident light if the photocurrent is
brought to zero by stopping potential of 0-6 V.

ullo
[CBSE 2001 (C)]

r FF
answer.

Ans. The pht)ioelecirons can be emitted from a metal (CBSE Sample Paper 2020)

e
surface if the frequency of incident radiation is Ans. (a) Threshold frequency,

rsre
more than threshold frequency, i.e, more than 2-14 X 1-6x10"'^
= 5-17 X 10^** Hz

oF
uo
that of green light for the given surface. As

k
h 6-62 xlO--^'^
frequency of violet light is more than that of
ib)K, = eVr,0 = 0-6 eV

oofr
green light, hence violet light will eject max

phoioelectrons. But the frequency of red light Energy of photon,


sf
ko
is less than that of the green light, hence red E = Kmax + 00 = 0-6 eV + 2-14 eV = 2-74 eV
Y
light can not eject photoelectrons from the given Wavelength of photon.
BB
Yo
surface.
Y
oo

17. Ultraviolet light is incident on two photo


he (6-62 xlQ-^^^jx (3x10^)
X = —
2-74 X 1-6x10-*^
er

sensitive materials having work functions 0{ E


re

and 02 (4^1 ^ H’hich case will the K.E. = 4530x 10-'°m = 4530A
uu

of the emitted electrons be greater ? Why ? 20. Why is a photo-electric cell also called an
od

Ans. Max. K.E. of photoelectron, K^.^ = hv - 0q ; electric eye ?


Yo
ad

As 0j > 0-,, so the max. K.E. of the emitted Ans. A photoelectric cell is called an electric eye as
n

photoelectrons will be greater for the photo the photoelectric current set up in the
sensitive material having work function 02- photoelectric cell corresponding to incident light
ndi

18. In a photoelectric effect experiment, the provides the information about the objects as
Re

graph betweenthe stoppingpotential (VO and


F

— - -Tias been seen by our eye in the presence of light.


frequency (v) of the incident radiations on
Fi

21. Why it is the frequency and not the intensity


two different metal plates P and Q are shown of light source that determines whether
in the Fig. 11(Q).20. (i) Which of the two emission, of photoelectrons will occur or not?
metal plates, P and Q has greater value of Explain. (CBSE 2022 i
work function ? (ii) What does the slope of
Ans. According to Einstein’s photoelectric equation
the lines depict ? K max = h{V- Vq)
FIGURE 11(Q).20 Where v is the frequency of the incident
V
radiation and Vq is the threshold frequency. From
this equation, it is clear that if v < Vq, no
photoelectron will be emitted. Thus the kinetic
energy of the emitted photoelectron depends on
the frequency of incident light and is
independent of intensity of incident light.
Therefore, on increasing the intensity of incident
>v
light, the number of photoelectrons emitted will
11/48
“Pn^ideefi. <’t. Fundamental Physics (XII) WWIWII
increase but not the kinetic energy of emitted will the stopping potential for the electrons
photoelectrons. emitted from it is effected ? Justify your
22. If the frequency of light incident on the answer. (CBSE 2020)
cathode of a photocell is increased, how will
FIGURE 11(Q).21
the following be affected? Justify your Vq- .
answer :

(0 Energy of the photo-electron A B

(i7) Photocurrent (CBSE 2020)


Ans. From Einstein’s photoelectric equation
K. max = /l (v - Vq)
Where v is the frequency of incident light and O ►1
Vq is the threshold frequency of the metal
surface :

w
(/) From above, it is clear that increases as
V increases. Ans. (a) We know that
(//) The increase in v will not effect on the
FIGURE 11(Q),22

Flo
photoelectric current as photoelectric current Vo 4
can be increased by increasing the intensity of
the incident radiation. Moreover the increase in A B

ee
the frequency of incident light increases the

Fr
maximum kinetic energy of the photoelectrons
emitted but not affecting the photoelectric
current. .e Ae ►1
for
ur
23. Light of same wavelength is incident on three X

photosensitive surfaces A, B and C. The


following observations are recorded.
ks
(0 From surface A, photoelectrons are not
Yo
he
oo

emitted he
eV. 0
...(/)
(u) From surface B, photoelectrons are just
0 “ or Vq = e\ e
eB

emitted
It is an equation of straight line between Vq and
{iii) From surface C, photoelcctrons with he
some kinetic energy are emitted. \fk with slop — as shown in gragh. Fig. 11 (Q).22
r

e
ou
ad

Compare the threshold frequencies of the he


three surfaces and justify your answer. slope of line = tan 6 = —
YY

e
(CBSE 2020)
Ans. (/) For surface A, threshold frequency (Vq) is ,
h =
e tan 0
nd

...(»)
Re

or
more than frequency (v) of the incident light, c

so no photoelectric emission takes place Thus the value of Xcan be determined by putting
Fi

(/7) For surface B, threshold frequency is equal the values of the slope of graph {tan 0), speed
to the frequency of incident light. So of light (c) and electronic charge in relation (ii).
photoelectric emission just takes place (b) Stopping potential only depends upon the
{Hi) For surface C, threshold frequency is less frequency of the incident light but is
than the frequency of incident light so emitted independent of intensity of the incident light, If
photoelectrons have some kinetic energy. the distance is increased intensity will decrease
Therefore, > Vgg > but there will be no change in the value of
24. Fig. ll(Q).21 shows the stopping (V^) for the stopping potential.
photo-electron versus (l/X.) graph, for two 25. In case of photoelectric effect experiment,
metals. A and B, X being the wavelength of explain the following facts giving reasons.
the incident light, (a) The wave theory of light could not explain
(a) How is the value of planck’s constant the existence of the threshold frequency
determined from the graph? (b) The photoelectric current increases with
{b) If the distance between the light source increase of intensity of incident radiation.
and the surface of metal A is increased, how (CBSE 2020)
DUAL NATURE OF RADIATION AND MATTER 11/49

Ans. (a) According to wave theory of light, the energy (ii) Momentum of photon,
of light would be represented by the amplitude 1
hv h .
of the light wave. A more intense light source, p = — i.e.. P OC —
even if it will be of lower frequency, than c X
threshold frequency of a metal surface would
P' P
be able to eject out photoelectrons from the or
2X 2
surface of metal. Due to which the photoelectric P

current would be generated. But actually a light Thus the momentum of photon becomes half
below a certain threshold frequency could not when the wavelength of radiation is doubled.
produce any photoelectric current, whatever
may be its intensity. III. De>Broglic hypothesis
(b) When the intensity of incident light on a 28. State two laws of photoelectricemission.Are
metal surface is increased, whose frequency is cathode rays waves or particle ?
more than threshold frequency, then more

w
Ans. For two laws of photoelectric emission refer to
number of photoelectrons will be emitted. This
Art. 11.6. Cathode rays are not waves but are
results in more photoelectric current.
streams of fast moving electrons.

Flo
II. Photons 29. Why is the wave nature of matter not more
apparent to our daily observations ?
26. What is a photon ? Show that it has zero rest

ee
Ans. De-Broglie wavelength associated with a body
mass or photons can not exist at rest. Explain.

Fr
of mass m, moving with velocity v is given by;
(West Bengal Board 2012) ^ = h/nw. Since the mass of the objects used in
Ans. Photons are packets of energy (or energy our daily life is very large, hence the de-Broglie

for
particles) which are emitted by a source of wavelength associatedwith them is quite small
ur
radiation. The energy of a photon is, and is not visible. Hence the wave nature of
E= hv = hc/X ; where h is a Planck's constant, matter is not more apparent to our daily
s
v,X= frequency and wavelength of photon. observations.
ok
Yo
We know that the mass m of a particle moving 30. A particle of a mass M at rest decays into two
with velocity v comparable with the velocity of
Bo

particles of masses and m2 having non


light c, is given by zero velocities. What is the ratio of the de-
Broglie wavelengths of the two particles ?
re

m
0
m =
Ans. Let Pj and V2 be the velocities of the two
particles of masses and m2 respectively.
ou
ad

where is the rest mass of the particle.


According to law of conservation of linear
Y

m^ = m Vl - v^/c^
momentum, m-,V2 = A/xO = 0
As a photon moves with the velocity of light,
nd
Re

—»

i.e, V = c, then, = m a/|-c2/c2 = 0 or m,v,=-m2V2 or


I w,u, I = I I
Fi

Hence, a photon has zero rest mass. It shows h


that photon cannot exist at rest. and X2 =
^^2
27. If the wavelength of an electromagnetic
radiation is doubled, what will happen to X
1 _ '”2^2
(i) the energy of photons and (ii) the or = 1
momentum of photon ? ^2 m,Ui
Ans. (/) Energy of a photon. 31. Calculate the ratio of the accelerating
he ! potential required to accelerate (i) a proton
E = hv = — i.e., £ oc and (ii) an a-particle to have the same de-
X
Broglie wavelength associated with them.
[CBSE 2009 (C)]
or £ = —
E~ 2X~ 2 2 Ans. If a charged particle of mass m, charge q is
Thus the energy of a photon becomes half accelerated under a pot. diff. V, the velocity
acquired by particle is v. Then
when the wavelength of radiation is doubled.
11/50 'a Fundamental Physics (XII) kvjwii
1.1
qV = — mv- =
m V
^2
—^ =
hc^/X V c
= — > 1
,
2 2 m 2m E.1 hc/X V

where p is momentum of particle Hence, > £|


or
p = -yllrngV It shows that total energy of an electron is
greater than that of photon.
h h
De-Broglie wavelength, \ z= 34. A particle with rest mass ihq is moving with
p ^2mqV velocity c. What is the de Broglie wavelength
associated with it ?
h h
and X a

oww
Now, X
j2maV„ Ans. X =
h hy\T - V^k'^ hyj\-c^/c- = 0
\ P^P P mv
m^^v m,^c
0
Given, a
35. A proton and an alpha particle are
h h
So. accelerated through the same potential.

ee
●y
l2m q p Vp J2m
\
q V
a^a i a
Which one of the two has (/) greater value of
de-Broglie wavelength associated with it, and

FFrlo
r
V
P _ Va. 4m X 2e (k) less kinetic energy ? Justify your answers.

rF
or = 8
(CBSE 2017, 2010, 2009)

ee
Ka mxe
Ans. (0 When a charged particle of charge q mass m
32. An electron, an a-particle and a proton have is accelerated under a pot. diff. V, let v be the
ouru
rF
the same kinetic energy. Which of these velocity acquired by the panicle, then
particles has the largest de-Broglic

ffosor
1 2
wavelength ? (CBSE 2007) qV = — mv or nw=[2 mqV\
1/2
os k ^ 2
Ans. Kinetic energy K = —mv 2 _1 _ }_pI h h
2 2 m 2 m or X,«
ook
Yo
^ImqV y[mq
Y
mv
or
p = ^2mK
Bo

I 4m 2e
reeB

h
De-Broglie wavelength, X = — =
Hence, X = 2>/2> 1
p .jlmK K ^ m e
oouY
ur

I
(when K is constant)
So X^, > Xf^ , i.e. de-Broglie wavelength
Xoc -J=
ad

associated with proton is greater than that of


Vm
alpha particle,
dY

It means smallest is the particle mass, largest is (ii) Kinetic energy of charged panicle
the de-Broglie wavelength associated with it.
nidn

Since the mass of electron is least out of the Ef^ = qV , i.e, Ej^o^^ q
Re

given particles, hence de-Broglie wavelength


F
Fi

P _
is largest for electron and least for a-particle. — = — < 1 or < E.
2e 2 p k
33. A photon and electron have got same de- h

Broglie wavelength. Which has greater total i.e., proton particle has less kinetic energy
energy ? Explain. than a-particle.
Ans. For a photon, energy, 36. An electron and a proton are accelerated
E^=hv = hetX ...(I) through the same potential. Which one of the
For an electron moving with velocity v, and two has (/) greater value of de-Broglie
effective mass m. de-Broglie wavelength (X.) is wavelength associated with it and ((7) less
given by momentum ? Justify your answer.
(CBSE 2009)
h h
X.= or m = ...(«) Ans. When a charged particle of charge q. mass m is
mv Xv
accelerated under a pot. diff. V, let v be the
Total energy of electron. velocity acquired by particle. Then

from (/i) 1/ ^
£., - me — qV = - mv or mv = m
Xv 2
DUAL NATURE OF RADIATION AND MATTER 11/51

So momenium is same for them . So (-P is


h h I
(0 ^= or X oc
same for them. But rest mass of proton is greater
mv m m
than that of electron, therefore, the total energy
of a proton is greater than that of electron.
X a ni ex 1837 m 39. An electron and a proton have the same de-
IP P ^ >1
Broglie wavelength. Which one of these has
■■ I 1 exm
higher kinetic energy ? Which one is moving
faster ?
So, Xg > Xp, i.e. greater value of de-Broglie
wavelength is associated with electron as Ans. Let m and K be the mass and kinetic energy of a

compared to proton, particle. Its de-Broglie wavelength is

{ii) Momentum of particle, P ~ mv = m h iP


^ = -T— or X^ =
P -Jqm ; ^2mK 2m K

ww
OC

IP
P. q m
e _
e m
or K =
Hence -X <1
2 m A.“
p \ q m ■ye 1837

Flo
t'p i>
1
So, p^ < Pp, i.e, lesser momentum is associated

e
I.e., K oc
X is same for panicles

eree
with electron as compared to proton. m

37. A proton and a dcuteron are accelerated

FFr
K m
through the same accelerating potential. e _
^>1
Which one of the tw'o has (a) greater value of K
or
K^>Kp

oorr
uur r
rn
P
de-Broglie w-avelength associated with it, and
(b) less momentum ? Give reasons to justify
sf
Thus kinetic energy of electron is greater than
your answer. (CBSE 2014) that of proton. So the electron is moving faster.
sk
Yoo
Ans. (a) De-Broglie wavelength is given by 40. Plot a graph showing variation of de-Broglie
oook

h 1 wavelength X versus 1/Vp , where V is


X = X
eBB

or
accelerating potential for two particles A and
OC

yl2qVm m
B carrying same charge but of masses and
/«2 (p*i > "*2)* Which one of the two
uurr

represents a particle of smaller mass and


= %)
ad

why ? How does this graph give us the


Yo

information regarding the magnitude of the


As
md>mp. so
Xp>X^ charge of the particle ? (CBSE (D), 2016,2019)
dY

h Ans. When a charged particle of charge q, mass m is


Re

,ib) Momentum of a particle, P=--r


innd

X accelerated under a potential difference V, let v


be the velocity acquired by the particle. Then
FFi

1
. Pp _
I.e.,
— <1 orpp<p^ 9^ = — or mv ~ ^2 q ni V
Pd
38. An electron and proton have same de-Broglie De-Broglie wavelength,
wavelength. Which one possesses greater
energy ? h h h 1
X = — ^
Ans. The total relativistic energy of a particle of rest mv
.yj2qmV .yj2qm
mass Wq, momentum p is
7- r~2 ^ T
Xoc
E = ^jp^c- I.e.

■Jv
As de-broglie wavelength is same for electron
and proton, i.e.. Therefore graph between X versus U-Jv is a
h
Jl-Jl straight line whose slope is = . This
K = \ or so p^=Pp.
-J2qm
Pe Pp
11/52 U Fundamental Physics (XII) IWII
shows that lower is the value of mass m, the
larger is the slope of graph and vice-versa. As.
the slope of graph of mass m2 is greater than
that of mass mj as shown in Fig. 11(Q).23,
therefore, > m2.
It
From graph, slope .v =
^2 qm

.-. q = ; can be calculated.


2
2tnx

w
SHORT ANSWER QUESTIONS Carrying 3 marks

1. Describe photoelectric effect and state the laws Ans. The three characteristic features in photoelectric
of photoelectric emission. effect are given below ;

Flo
(HP Board 2013) [Arts 11.5 and 11.6] (0 One incident photon can eject one

e
photoelectron.

reee
2. Draw a plot showing the variation of
photoelectric current with collector plate (/;) On increasing the frequency of the incident

FFr
potential for two different frequencies V[ and light, the kinetic energy of photoelectrons
increases but not the emission of number of
V2 (vj > V2) of incident radiations having the
same intensity. In which case will the stopping photoelectrons,

for
ur
potential be higher ? Justify your answer. (m) On increasing the intensity of the incident
(CBSE 2011) light, the number of photoelectrons increases
kss
[Refer Fig. 11.6 and Art. 11.6 part (c)] but not the increase in the kinetic energy of the
Yo
3. Light of intensity / and frequency v is incident emitted photoelectrons.
oo

on a photosensitive surface and causes 8. Why photoelectric effect cannot be explained


on the basis of wave nature of light ? Give
eB

photoelectric emission. What will be the effect


reasons. (CBSE 2013)
on anode current when (0 the intensity of light
is gradually increased, (//) the frequency of Ans. The wave nature of radiation can not explain
ur

incident radiation is increased and (///) the anode the followings :


ad

potential is increased ? In each case, all other (/) The instantaneous ejection of photoelectrons.
YYo

factors remain the same. Explain, giving justi (/7) The existence of threshold frequency for a
fication in each case. (CBSE 2015) [Art. 11.6] metal surface.
d

4. Write Einstein's photoelectric equation. State (///) The fact that KE of the emitted
Re
in

clearly the three salient features observed in photoelectrons is independent of the intensity
photoelectric effect, which can be explained on of the incident light but depends upon the
F

the basis of the above equation. frequency of the incident light.


(CBSE 2010) [Art. 11.7] For reasons refer to Art. 11.12.

5. Derive Einstein’s photo-electric relation. Prove 9. Sketch a graph between frequency of incident
intensity law on the basis of Einstein relation. radiations and stopping potential for a given
(Chhattlsgarh Board 2012, MP Board 2011) photosensitive material. What information can
[Art. 11.7] be obtained from the value of the intercept on
the potential axis ?
6. Find a relation between cut-off potential,
A source of light of frequency greater than the
frequency of the incident light and threshold
threshold frequency is placed at a distance of
frequency. [Art. 11.8]
1 m from the cathode of a photocell. The
7. Write the three characteristic features in
slopping potential is found to be V. If the
photoelectric effect which cannot be explained distance of the light source from the cathode is
on the basis of wave theory of light, but can be reduced, explain giving reasons, what change
explained by using Einstein’s equation. will you observe in the (i) photoelectric current
(CBSE (D), 2016] (n) stopping potential ? (CBSE 2010)
DUAL NATURE OF RADIATION AND MATTER 11/53

Ans. For graph refer to Fig. 11.5. Intercept on the (i) the saturation current vary with anode
potential axis = - potential for incident radiations of different
If the distance of the light source from the frequencies but same intensity ?
cathode is reduced, then (/) the photoelectric (//) the stopping potential vary for incident
current increases because the intensity of radiations of different intensities but same
incident radiation increases. (//) the stopping frequency ?
potential Vq remains the same because it is (///) photoelectric current vary for different
independent of the intensity of the incident intensities but same frequency of incident
radiations.
radiations ?
10. Sketch the graphs, showing the variation of Justify your answer is each case. (CBSE 2007)
stopping potential with frequency v of the
Ans. (/) In photoelectric emission from a given metal
incident radiations for two photosensitive
surface, the saturation current depends on the
materials A and B having threshold frequencies
intensity of the incident radiation but is

w
Vy> Vq respectively. independent of the frequency of the incident
(0 Which of the two metals A or B has higher radiation. Therefore, for the radiations of
work function ? different frequency but of same intensity, the

Flo
(//) What information do you get from the slope value of saturation current will be the same.
of the graphs ? (n) The value of stopping potential for a given

reeee
(///) What does the value of the intercept of metal surface depends on the frequency of the

FFr
graph A on the potential axis represent ? incident radiation but is independent of the
(CBSE 2010, 2016) intensity of the incident radiation. Therefore, for
Ans. The variation of stopping potential (V^^.) with the incident radiations of the same frequency

for
but of different intensity, the value of stopping
ur
frequency(v) of the incident radiations for two
photosensitive materials is shown in Fig. potential will be the same.
kkss
11(Q).24. {Hi) The photoelectric current depends on the
number of photoelectrons emitted, which in turn
Yo
oo

FIGURE 11(Q).24 depends on intensity of incident radiations but


o> is independent of frequency of the incident
eB

METAL B, METAL A
Z -4
E< radiations provided the frequency is more than
the threshold frequency for the given metal
O 2
surface. Therefore, the photoelectric current for
r
ou
ad

CL
the radiations of the same frequency but of
different intensity will be different.
YY

O /
/ Vo
;)
/ Vo frequency of
►«) 12. (/) Stale two important features of Einstein’s
photoelectric equation.
ndd

’ / INCIDENT RADIATION
Re

{ii) Radiation of frequency lO’^ Hz is incident


on two photosensitive surfaces P and Q. There
Fi

is no photoemission from surface P. Photo


“ e
emission occurs from Q but photoelectrons have
zero kinetic energy.
Explain these observations and find the value
(0 Metal A has higher work function, of work function for surface Q.
as, (f)Q = /jVQ and =/u'q', therefore, Vy> Vy'. (CBSE (D), 2017)
(//) Slope of V^- V graph = h/e. [For discussion Ans. (0 Einstein's photoelectric equation is
see conceptual problem 6| I
(Hi) Intercept of graph A on the potential axis K
max
= - mV ^
max
=hv—hv 0 ...(/)
2

^0 hv
0
where Vq is threshold frequency of metal and v
e e is the frequency of incident radiation. Important
11. In a plot of photoelectric current versus anode features of Einstein’s photoelectric equation are
as follows :
potential, how does
11/54 “Pn^idte^ 4 Fundamental Physics (XII) orsmu
1. The phoioeleclric emission lakes place if the (c) Max KE = bVq = hv- (})q. It is independent of
energy of incident radiation is greater than the intensityof incident radiation till v is constant.
work function of metal.
14. The following graph shows the variation of
2. The photoelectric emission is independent of photoelectriccurrentfor a photosensitivemetal.
intensity of incident radiation but depends on
frequency of incident radiation.
3. No. photoelectric emission takes place if
V < V,).
4. Photoelectric emission is an instantaneous
process.

(//) Energy of incident radiation

oww
E = /n, = (6-63 X 10-^4) x 10^^ J
_6-63xlQ~'^ eV =A-\AeV
" 1-6x10-'^ ■►X

e
As no photoemission takes place from metal P,

FFrlo
re
so work function of metal P is greater than (a) Identify the variable Xon the horizontal line.
4-14 eV. As photoemission takes place from ib) What does the point A on the horizontal axis

ree
F
metal Q with zero kinetic energy, so work represent ?
function of Q = energy of incident radiation (c) Draw this graph for three different values

rF
= 4-14 eV
of frequencies of incident radiation V|, V2 and
13. Explain giving reasons for the following : V3 (vj > V2 > V3) for same intensity.

fsoor
ouur
(a) Photoelectric current in a photocell increases (d) Draw this graph for three different values
skf
with the increase in intensity of the incident of intensities of incident radiation /j, I2 and
radiation. /s (/] > /2 > 73) having same frequency.
ooko
(CBSE Sample Paper 2020. CBSE 2017)
Yo
{b) The stopping potential (Vq) varies linearly
Y

with the frequency (v) of the incident radiation Ans. {a) From experimental study of photoelectric
Bo

for a given photosensitive surface with the slop emission, we conclude that X is a potential on
reB

horizontal axis.
remaining the same for different surfaces.
(c) Maximum kinetic energy of the photo- (b) Point A denotes the stopping potential.
uur
oY

electrons is independent of the intensity of (d


FIGURE 11(Q).26
ad

incident radiation. (CBSE 2017)


dY

Ans. (fl) For the radiation of frequency (greater than


threshold frequency of metal) photoelectric Vi > ^'2 > i’3
innd

current is directly proportional to the intensity


Re

of incident radiation. So the increase in intensity


Fi
F

of incident radiation increases the photoelectric


current.

hv 00 ■►V
(/?) As, eV„ = hv - 0y or =— ...(/)
e e

So stopping potential Vq is directly proportional (d)


to frequency v. So the graph between Vq and v
Photoelectric
is a straight line. Thus equation (/) is a st. line
Current' >
equation. Comparing it with a straight line
equation
1
y = ni\ + C, we have

Slope of this line = — = a constant


e

So slope of Vq and v graph =— which is


e ■►V
constant for all surfaces.
DUAL NATURE OF RADIATION AND MATTER 11/55

15. Radialion of frequency lO’^ Hi is incident on 18. Explain de Broglie dualistic nature of matter and
three photosensitive surfaces A, B and C. derive de-Broglie relationship for wavelength
Following observations are recorded : of matter waves.

Surface A. no photoemission occurs. (Jhark u.ud Board 70U) [Art. 11.15J


Surface B. photoemission occurs but the 19. What are matter waves ? Show that de-Broglie
photoclectrons have zero kinetic energy. wavelength associated with an electron of
Surface C. photoemission occurs and energy, V'-electron volt is approximately
photoclectrons have some kinetic energy. 12-27 «
A.
Using Einstein’s photoelectric equation, explain
the three observations.
(Uttarakhand Board 2D12) [Art. 11.16J
(CBSE Sample Paper 2022-23) [Art 11.7]
20. How can de-Broglie wave hypothesis be verified
16. Using Einstein’s photoelectric equation show
how the cut off voltage and threshold frequency experimentally ?

w
Or
for a given photosensitive material can be
determined with the help of a suitable Describe Davission and Germer experiment to
establish the wave nature of electrons.
plot/graph. (CBSE 2012) [Art. 11.10(0]

Flo
17. Write the basic features of photon pictures of [Art. 11.17]
electromagnetic radialion on which Einstein’s 21. Why is wave theory of electromagnetic radiation

reeee
photoelectric equation is based. not able to explain photo electric effect ?
(CBSE 2013) [Art. 11.11] (CBSE 2019) [Art. 11.12]

Fr
LONG ANSWER QUESTIONS Carrying 5 or more marks

for
ur
1. What is photoelectric effect ? Explain Einstein’s photoelectric equation and use it to
kss
experimentally the variation of photoelectric explain (/) independence of maximum energy
current with (/) the intensity of the incident light of the emitted photoelectrons from intensity of
Yo
oo

(//) the potential difference between the plates incident light (ti) existence of a threshold
and (m) the frequency of the incident light and frequency of the emitted photoelectrons from
eB

hence state the laws of photoelectric emission. intensity of incident light. [Arts. 11.4, 11.7]
(J & K Board 2007) [Art. 11.4, 11.6] 4. {a) Derive the expression for the de-Broglie
r

2. State laws of photo electric emission. Establish wavelength of an electron moving under a
ou
ad

Einstein photo-electric relation. Explain the potential difference V volt. (Hr. Board 2012,
YY

laws of photo-electric emission on the basis of MF Board 2012, HP Board 2011)


this relation.
(b) Describe Davisson and Germer experiment
to establish the wave nature of electrons. Draw
ndd

(Hr. Board 2011, Pb. Board 2007)


Re

[Art. 11.6,11.7] a labelled diagram of the apparatus used.


Fi

3. What is photoelectric effect ? Give any two (Hr. Board 2011)


practical applications of this effect. Write [Arts. 11.16, 11.17]

CASE-BASED VERY SHORT/SHORT QUESTIONS

CASE 1. Photoelectric effect is the phenomenon h is a Planck’s constant. For photoelectric emission,
of emission of electrons from the surface of a metal the visible light is used for alkali metals and ultraviolet
when a light of suitable frequency is incident on it. The light is used for alkaline metals. The work function of
emitted electrons are called phoioelectrons. For every every metal is constant and does not change with the
metal, there is a certain minimum frequency of the change in intensity of the incident light. The increase in
incident light (called threshold frequency) for which the intensity of incident light on a photo.sensitive surface
there is a just emission of phoioelectrons from the increases the photoelectric current and increase in the
surface of metal without any kinetic energy. The energy frequency of the incident light increases the maximum
of the incident photon of threshold frequency (vq) is KE of the emitted phoioelectrons which is given by
equal to work function (4>q) of metal. So, (t>o = bvQ, where Einstein’s photoelectric equation as, (KE)^^^. = /jv -
11/56 "Pn^ieCctf.i 'a Fundamental Physics (XII)
Lower is the work function of a metal larger is the 3
K = — kT where k is Boltzmann constant.
maximum KE of the emitted photoelectrons and vice versa. 2 ’
A minimum negative potential (V'q) given to anode Putting this value in (/), we have
w.r.t.. cathode in the photocell for which the h h
photoelectric current becomes zero is called cut off k =

w
potential or stopping potential V'q, /.e., 2m x — kT ^j3mkT
2
K = eV.
niax
= —
0
mv-
inax 0 Read the above paragraph carefully and
answer the following very short and short

e
The value of cut off potential depends upon the answer questions :
frequency of incident light and nature of the metal

e
5. Is there any difference between light waves and

wr
surface but is independent of intensity of incident light. matter waves ?

lloo
r
Read the above paragraph carefully and 6. Show graphically, the variation of de-Broglie
answer the following very .short and short wavelength (A.) with potential (V) through which

F
FFu
answer questions : an electron is accelerated from rest.

1. Two metals A and B have work functions 4 eV 7. A proton and electron have same velocity. Which
and 10 eV. Which metal has higher threshold one has greater de-Broglie wavelength and why ?
wavelength ? 8. If the temperature is increased from 27°C to

rese
2. How will the photoelectric current change on 127°C, find the ratio of the de-Broglie wavelength
uro of wave associated with a material particle at these

k
Fr
decreasing the wavelength of incident radiation

o
two temperature.
for a given photosensitive surface ?

foo
3. What determines the maximum velocity of CASE 3. According to Planck’s Quantum Theory,

fr
photoelecirons ? light is a packets of energy (called photons) which travel
kso
Y
4. Plot a graph showing the variation of photoelectric in a straight line with a speed of light. Energy of a
current with anode potential for two light beams photon, E = h\' = hcfk where v and X are the frequency
Y
B

and wavelength of light used.


Yo
of same wavelength but different intensity. Mark
oo

the graph for radiation of higher intensity. A source of light emits large number of photons
eBr

per second. For a source of light of power P, the number


e

CASE 2. De-Broglie from the study of dual nature of photons emitted per second is
uru

of radiation {i.e., radiation possesses properties of both P P P


od

n = —
wave and particle) concluded that the moving material E hv hc/X
Yo
ad

particle must also possess dual nature, since nature loves If a light of frequency v from a point source falls
n

symmetry. The wave associated with moving material normally on a surface placed at a distance r from the
particle is called matter wave or de-Broglie wave whose source of light, then the intensity of light on the surface
ndi

wavelength was called de-Broglie wavelength, given


Re

^ _ Energy falling per second


F

h IS
by X = Area of surface
Fi

mv
A 100 W ultraviolet light source of wavelength
where h is Planck’s constant, v is the velocity of 2480 A illuminated a magnesium (Mg) surface placed
the moving particle of mass m. De-Broglie waves are 2 m away. [Use h = 6-62 x 10"^'^ Js; c = 3 x 10^ms"*]
not electromagnetic waves. Read the above paragraph carefully and
If K is the KE of material particle of mass m, answer the following very short and short
moving with velocity v, then answer questions :
9. What is the rest mass of a photon of wavelength
1 2480 A ?
K = ~ mv^ or mv
= pmK
2 10. What is the equivalent mass of a photon of
de-Broglie wave as.sociated with material particle is wavelength X ?
11. In the given statement, determine number of
h h
X = ...(/) photons emitted per second by the given source
mv
^2mK of light.
12. In the above question, what is the energy of light
According to kinetic theory of matter, the average falling per second on the area 2 m square of the
KE of the particle at a given temperature Tkelvin is magnesium surface ?
DUAL NATURE OF RADIATION AND MATTER 11/57

ANSWERS

Thus variation of Vand \ will be a curve as shown


1. Work function, (|)q = hclX^, where Xg is the
threshold wavelength. So (})q 1/Xg. It means the in Fig. ll(Q).29.
metal with lower work function (i.e. metal A) has ft 1A 1
higher threshold wavelength than metal B. 7. de-Broglie wavelength, X = mv
— or oe
-.if
m

2. Photoelectric current is not affected on decreasing V is constant.


the wavelength of incident radiation provided its k m

w
intensity remains unchanged. Therefore, —^ < 1 or
Xe
3. The maximum velocity (v max ) of the photo- m
p

electrons is decided by the frequency of incident Le., de-Broglie wavelength associated with
radiation and work function of photosensitive electron is more than that of proton.
8. de-Broglie wavelength,

e
1

roow
surface, as — mv ^max = hv - (1)^ 0 ●
2

re
h h 1
X = - or X oc

4. The graph is shown in Fig. 11(Q).28, where /], I2 P ^3mkT


represent the intensity of two light beams of

reF
wavelength Xj and X2, respectively. X
1 127+273 _ nr _ 2V3

uFFll
27
= 1155
^~y 27+ 273 ~y 3 ~ 3

e
9. According to theory of relativity, the mass m of a

sFr
particle moving with velocity v, comparable with

foro
the velocity of light c is given by
ofk
uor
m
0
m = or nir, = m
0
Vl - v^lc^
kos
Y
Here, is the rest mass of the particle.
Yo
reeBB
oo

As a photon moves with the speed of light.


i.e.,
u = c, so ihq = m -\f - c^/c“ = 0
uurY

i.e., photon has zero rest mass.


5. Yes, the velocity of light waves in vacuum is
constant, whereas the velocity of matter waves in 10. According to mass energy relation
£ = mc^ or m = £/c2
ad

vacuum depends upon the wavelength of matter


doo

wave. For a given photon, £ = hc/X


nY

6. KE acquired by electron when accelerated under hc/X h


a potential difference V is Equivalent mass, m = —
nid

cX
Re

c~

eV = - mv^ or mv = J2meV 11. No. of photons emitted per second.


F
Fi

2 ^
P 100
de-Broglie wavelength associated with electron n =

h h
hc/X (662 X10-3*^) X (3 X10^)/(2480 x 10"'^)
is X = i-e., X«l/Vv. 19
= 12-5 X 10
mv
pmeV ’
12. Intensity of light falling on the surface at distance
FIGURE 11(Q).29 P
ris, / =
471

Energy of light falling per second on the area A is


P
E' = IA =
4 k I-

Here, r=2m;A = 2x2 = 4sqm, £ = 100 watt


100x4 ia)x7 -1
.-. £' = = 7 95 Js
V 4 X (22/7) X 2- 4x22
11/58 ^n<tdee4i, 4- Fundamental Physics (XII) PTSTTl

CASE-BASED MCQs AND ASSERTION-REASON QUESTIONS

CASE 1. Einstein’s photoelectric equation for the 3. Assertion. The inaxiinuiii velocity of the emitted
photoelectric emission is given by photoelectron from a metal surface is independent
of the work function of the metal surface.
1
K
max =“
2 = /tv' - <l)o = - /jvo Reason. Maximum velocity of the emitted
photoelectron from a metal surface depends upon
where K
max
is the maximum KE of the emitted the frequency of the incident radiation only.
photoelectron, is the maximum velocity of the 4. Assertion. The stopping potential in photoelectric
emitted photoelectron, v is the frequency of the incident emission depends upon the intensity of the incident
radiation, Vq is the threshold frequency of the metal on radiation in a photo cell.
which radiation is falling, h is Planck’s constant and (|)q Reason. The value of stopping potential changes

w
is the work function of the metal surface. If 0
is the with the change in intensity of the incident
radiation.
stopping potential, then

Flo
he he C/ >E 2. Photoelectric emission tiikes place from
eVn0 = = AV - (t)y =
max a metal surfacewhen the energy of the incidentradiation

eeee
X X
0 is more than the work function of that metal surface.

Fr
The minimum frequency of the incident radiation (called
where X and Xy are the wavelength of the incident
radiation and threshold wavelength of the metal surface. the threshold frequency Vy) is just able to eject
photoelectron without any kinetic energy. The energy
The work function of a metal is 4-2 eV. Light of
for
ur
of the radiation corresponding to threshold frequency
wavelength 2000 A is incident on it. [Use h = 6-6 x
is equal to work function ((1)q) of metal surface. So
10-3“^ Js, m^ = 9x 10-3' kg, c = 3 X 10^ ms"*]
ks
Based on the above paragraph, answer % = /'I'D-
Yo
From Einstein’s photoelectric equation
oo

questions no. 1 to 4 :
1. What is the threshold frequency of the metal
eB

he
surface ? eVr.0 = Kmax = - (j)o = — '0
(a) 102 X 10'“^ Hz
where Vy is the stopping potential, v is the
r

(b) 102 X 10‘3 Hz


ou
ad

frequency of the incident radiation.


(c) 2-04 X 10'4 Hz
YY

Based on the above paragraph, answer


(d) 2-04 X lO'^Hz questions no. 5 to 8 :
2. In the above question, what is the maximum kinetic 5. The variation of frequency (v) of the incident
nd
Re

energy (in eV) of the emitted photoelectrons ? radiation and maximum kinetic energy of
Fi

(a) 1-67 (b) 1-99 the emitted photoelectrons can be represented by :


(c) 3-28 id) 5-27
For Question No. 3 and 4, we have given two
statements each, one labelled as Assertion (A) and
other labelled as Reason (R).
Choose the correct option out of the four options
given below :
(a) Both A and R are true, and R is the correct
explanation of A
(b) Both A and R are tiaie, but R is not the correct
explanation of A
(c) A is true, but R is false
(d) A is false, and R is also false
DUAL NATURE OF RADIATION AND MATTER 11/59

6. The variation of frequency (v) of incident radiation Choose the correct option out of the four options
on a photocell and photoelectric current (/) for a given below :
constant intensity of incident radiation can be (a) Both A and R are true, and R is the correct
represented by explanation of A
FIGURE 11(Q).31 (Jb) Both A and R are true, but R is not the correct
lA explanation of A
(c) A is true, but R is false
{d) A is false, and R is also false
(b)
7. Assertion. When the intensity of the incident
■►v radiation falling on a photosensitive surface
becomes double, then the number of photoelectrons
14 14 emitted becomes double.

w
Reason. Photoelectric current is directly pro
(c) id) portional to the intensity of the incident radiation.
8. Assertion. In photoelectric cell, the photoelectrons

Flo
■►V > V
can be emitted in any direction with velocity
ranging from zero to maximum value.

ee
For Question No. 7 and 8, we have given two
Reason. The photoelectrons from a metal surface

Fr
statements each, one labelled as Assertion (A) and
other labelled as Reason (R). will be emitted with maximum velocity only in one
direction.

or
ur
ANSWERS
sf
1. ib) 2. ib) 3. id) 4. (d) 5. (b) 6. (d) 7. (a) 8. (c)
ok
Yo

HINTS/EXPLANATIONS For Difficult Questions


Bo

1. Here, = 4-2 eV = 4-2 x 1-6 x 10"^^ J upon the nature of photosensitive surface and
re

Threshold frequency. frequency of the incident radiation.


ou

5. Maximum KE, Kmax = hv - <|)q, i.e., Kmax


ad

oc V.
4-2 X 1-6x10-’^
V, The given relation between Kmax and V is similar
Y

6-6x10-3^
to an equation of straight line, y = mx + c, where
= 1018 X 10l= Hz the slope of the straight line is h and photo electric
nd
Re

he emission starts if the frequency of incident


2. K n)ux '0 radiation is greater than the threshold frequency.
Fi

X
Thus option ib) is true.
(6-6 X 10-^**) X (3x10^) 6. Photoelectric current depends upon the intensity
-4-20 eV
(2000xl0“^^)xl-6xl0"‘^ of incident light and not on the frequency of the
= 619-4-20 = 1-99 eV incident light provided it is greater than the
threshold frequency. If the intensity of incident
3. Both Assertion and Rea.son are false because,
radiation is constant and the frequency of the
2 incident radiation is increased, there will be no
— tmi
2 nuix = /iv-(1)q change in the photoelectric cunent. Thus option
id) is true.
Maximum velocity of the emitted photoelectrons
7. Both Assertion and Reason are true and the
depends upon the frequency of the incident
radiation and work function of metal surface. Reason is the correct explanation of Assertion.
4. Both Assertion and Reason are false. The 8. Here, Assertion is true but Reason is false, as
stopping potential does not depend upon the photoelectrons can be emitted in any direction
intensity of the incident radiation but depends with velocity ranging zero to maximum value.
11/60 “Prutdeefr ^ Fundamental Physics (XII) lvihi

P'’^©)©LE!SiS {F©[^
ryPE I. PHOl OELECTRIC 5. Light of wavelength 2000 A® falls on an
aluminium surface. In aluminium 4-2 eV are
EFFECT
required to remove an electron. What is the
1. Given in Fig. 11(Q).32, is the graph between kinetic energy of («) the fastest (b) the slowest
frequency (v) of the incident light and maximum emitted photoelectrons, (c) what is the stopping
kinetic energy (Ej.) of the emitted photoelectrons. potential ? (d) what is the cut off wavelength for
Find the values of (/) threshold frequency and aluminium ? Planck's constant h = 6-6 x 10”^ Js
(i7) work function from the graph. and speed of light, c = 3 x 10^ ms“’.
(CBSE Sample Paper 2011)
[Ans. (a) 2 eV (*) zero (c) 2V (d) 2946-4 A]

w
6. The work function for caesium is 1-8 eV. Light
of 4500 A is incident on it. Calculate (i) the

Flo
maximum kinetic energy of the emitted
photoelectron (//) maximum velocity of the

reeee
emitted photoelectron (///) if the intensity of the
incident light is doubled, then find the maximum

FFr
kinetic energy of the emitted photoelcctron.
Given. /i = 6-6 x lO’^-^ Js, = 91 x IQ-^' kg,

for
c = 3 X 10^ m s"*]
ur
[Ans. (i) 10‘^ Hz («) 4 eV] [Ans. (0 1*52 X 10"''' J («) 5-78 x 10^' m s"’
kkss
(m) 1-52 X 10-'^ J]
2. When light of wavelength 400 nm is incident on
Yo
the cathode of a photocell, the stopping potential 7. Find the frequency of light which ejects electrons
oo

recorded is 6 V. If the wavelength of the incident from a metal surface, fully stopped by a retarding
eB

light is increased to 600 nm, calculate the new


potential of 3-3 V. If photoelectric emission
14
begins in this metal at a frequency of 8 x 10
stopping potential. [Given, h = 6-6 x 10“^'^ Js ;
Hz, calculate the work function (in eV) for this
c = 3x 10^m/s;e= l-6x lO-'^CJ
r

metal. [CBSE 2018 (C)l


ou
ad

(Pb. Board 2007)


[Ans. 15-96 x 10‘-^ Hz, 3-315 eV]
YY

[Ans. 4-97 V]
8. The electric field associated with a light wave Is
3. If light of wavelength 412-5 nm is incident on given by £= £q sin [(1-57 x 10^ m“’) (cr - .r)]
ndd
Re

each of the metals given below, which ones will


Find the stopping potential when this light is used
show photoelectric emission and why ? in an experiment on photoelectric effect with the
Fi

Metal Work function (eV) emitter having work function 21 eV.


h = 6-63 X 10-^'* Js. [Ans. I-O VI
Na 1-92
9. A photon of wavelength 3310 A falls on a photo
K 2-15 cathode and an electron of energy 3 x 10“*^ J is
Ca 3-20 ejected. If the wavelength of the incident photon
Mo 4-17 is changed to 5000 A, the energy of the ejected
electron is 9-72 x 10"^^ J. Calculate the value of
(CBSE 2018) [Ans. Na and K] Planck’s constant and threshold wavelength of
4. Ultra-violet light of wavelength 800 A and 700 A the photon. [Ans. 6-62 x Js ; 6620 A]
when allowed to fall on hydrogen atoms in their 10. Radiation of wavelength 180 nm ejects
ground state is found to liberate electrons with photoelectrons from a plate whose work function
£.£., 1-8 eV and 4-0 eV respectively. Find the is 2-0 eV. If a uniform magnetic field of flux
value of Planck’s constant. density 5-0 x 10“^ T is applied to the plate, what
[Ans. 6-57 x 10
-34
Js] should be the radius of the path followed by
DUAL NATURE OF RADIATION AND MATTER 11/61

electrons ejected nonnally from the plate with of the photon and (Hi) the kinetic energy of
maximum energy ? electron.

use/( = 6-6 X Js ; c = 3 x 10*^ ms“'. Given h = 6-6 x Js ;


[Ans. 0'149 cm] c = 3 X 10^ m/s (i I:*

TYPE II. PHOTONS [Ans. (/) 6-6 x 10“^^ kg ms“', 6*6 x 10“’^ kg nis“*
(«) 1-98 X 10-"^ J (m) 2-39 x 10“^^ J]
11. Monochroamtic light of frequency 6-0 X 10'“* Hz
18. An electron is accelerated through a potential
is produced by a laser. The power emitted is
2-0 X 10"^ W. Calculate the (/) energy of a photon difference of 64 volts. What is the de-Broglie
in the light beam and (//) number of photons wavelength associated with it ? To which part of
emitted on an average by the source. the electromagnetic spectrum does this value of
[CBSE 2018 (C)] wavelength correspond ?
[Ans. (0 39-78 x 10--“ J (CBSE 2010) [Ans. 1*53 A, x-rays]

w
(I'O 5*03 x 10*^ photons/s] 19. From a cliff that is 10 0 m above a lake, a boy
12. The minimum light intensity that can be per (mass 40 kg) jumps from rest, straight down into

Flo
the water. At the instant, he strikes the water, what
ceived by the eye is about 10“*“ Wm“^. Find the
number of photons of wavelength 5-84 x 10“^ m is his de-Broglie wavelength ? (g = 10 ms"^)
that must enter the pupil, of area 10^ m- s“' for /i = 6-6x 10-^-* Js) ,-36

ee
[Ans. 1-17 X 10 m]
vision. Given, h = 6-6 x 10“^“* Js. 20. The de-Broglie wavelength of an electron moving

Fr
[Ans. 3 X 10"* photons] with a velocity 1-5 x 10** ms"’ is equal to that of
13. Find the number of photons emitted per minute a photon. Calculate the ratio of the kinetic energy

for
ur
by a 25 W source of monochromatic light of of the electron to that of photon.
wavelength 5000 A. Given h = 6-63 x 10'^“* Js. [Ans. 1/4]
(Pb. Board 2004) 21. Calculate the energy of an electron having
ks
[Ans. 3-77 x 10^‘] de-Broglie wavelength 5500 A. Given,
Yo
oo

14. A parallel beam of light is incident nonnally on h = 6-6 -t-10-^“* Js. m, = 9-1 X IQ-^' kg.
eB

a plane surface absorbing 40% of the light and (EAM( . ■


reflecting the rest. If the incident beam carries [Ans. 4-9 X 10-“ eV]
10 watt of power, find the force exerted by it on
22. An electron and a photon each have a wavelength
r

the surface. [Ans. 5-33 x 10“® N]


ou
ad

2 nm. Find (/) their momenta (//) the energy of a


15. Find the number of photons emitted per second photon and {Hi) the kinetic energy of electron.
Y

by a 40 W source of monochromatic light of Given ft = 6-63 X IO--*^Js. ((


wave- length 6000 A. What is the photoelectric -25
[Ans. (/) 3-3 X 10 kg ms-‘ Hi) 6-2 x lO'^ eV
nd

current assuming 5% efficiency of photoelectric


Re

effect ? (Hi) 0-377 eV]


Fi

Given ft = 6-6 x IO-^“* Js, c = 3 x 10* ms


-1 .
23. The wavelength of a photon is 1 -4 A. It collides
e=l-6x I0-'“C. [Ans. 0-97 A] with an electron. Its wavelength after collision
is 4 A. Calculate the energy of scattered electron,
16. An X-ray lube produces a continuous spectrum ft = 6-63 X 10-34 Js. -16
[Ans. 9-23 x 10 J]
of radiation with its short wavelength end at
0-42 A. What is the maximum energy of a photon 24. Find the energy that should be added to an
in the radiation ? What is the order of accelerating electron of energy 2 eV to reduce its de-Broglie
voltage (for electrons) required in such a case ? wavelength from 1 nm to 0-5 nm. [Ans. 6 eV]
ft = 6-63 X 10-34 Js ; £ = 1-6 X lQ-'“ C. 25. The de-Broglie wavelength associated with
[Ans. 29-6 keV, 30 kV] proton changes by 0-25%. If its momentum is
changed by 9 x 10“3“ kg ms-*, find the initial
TYPE III. de-BROGLIE momentum of electron.
HYPOTHESIS -23
[Ans. 3-6 X 10 kg ms“*]
17. An electron and photon each have a wavelength 26. An electron is accelerated through a potential
I-00 nm. Find (/) their momenta (ii) the energy difference of 200 volts. What is the de-Broglie
11/62 4 Fundamental Physics (XII) kwii

wavelength associated with it ? To which part of 30. 10-6 eV photons of intensity 2-0 W/m^ falls on a
the electromagnetic spectrum does the value of platinum surface of area 1 -0 x 10“^ and work
wavelength correspond ? (CBSE 2010) function 5-6 eV, 0-53% of the incident photons
[Ans. 0*87 A ; A-rays] eject photo elec v>ns. Find the number of
27. Find the de-Broglie wavelength (in A) photoelectrons emitted per second and their
asscociated with a proton moving with a velocity minimum and maximum energies (in eV).
0-5 c, where c- = 3 xlO*^ m/s, rest mass of proton lAns. 6-25 x 10“ s"*,0, 5-0 eV]
= 1-675 X 10-27 ^ 5.6 X 10-2^^ Js.
31. When a surface is irradiated with light of
-15
[Ans. 2-27 x 10 m] wavelength 4950 A, a photo-current appears
28. Find the de-Broglie wavelength of an electron in which vanishes if a retarding potential greater

oww
a metal at 127“C and compare it with the mean than 0-6 volt is applied across the photo-tube.
separation between two electrons in a metal When a different source of light is used, it is
which is about 2 A. Given h = 6-63 x Js, found that the critical retarding potential is
= 9-1 X 10"2’ kg. Boltzmann constant changed to M volt. Find the work function of
/:= 1-38 X 10-23 j K-l. [Ans. 5*4 nm, 27]

e
the emitting surface and the wavelength of second

FFrlo
re
TYPE IV. TYPICAL PROBLEMS source. If photoelectrons (after emission from the
surface) are subjected to a magnetic field of

ree
F
29. Given that a photon of light of wavelength 10 tesla, what changes will be observed in the
10,000 A has an energy equal to 1 -23 eV. When above two retarding potentials.

rF
light of wavelength 5000 A and intensity /q falls Given, /? = 6-6 x 10,-34 Js.
on a photoelectric cell and the saturation current
[Ans. 1-9 eV, 4125 A ; no change]
fsoor
ouur
is 0-40 X 10"^ ampere and the stopping potential
32. What amount of energy (in eV) should be added
skf
is I -36 volt, then (0 what is the work function ?
(//) If intensity of light is made 4 /q, what should to an electron to reduce its de-Broglie wavelength
ooko
from 100 to 50 pm ? Given,
Yo
be the saturation current and stopping
Y

/t = 6-63x 10-2^ Js, w^ = 9-l X 10


-31
potential ? kg-
Bo

[Ans. (0 IT eV (m) 1*6 x lO”^ A, unchanged]


reB

[Ans. 452 eV]


uur
oY
ad

For Difficult Questions


dY

1. (0 Threshold frequency, 6-63x10-2^x3x10^


innd
Re

^’0 = lOx 10^^ Hz = 10*^ Hz


eV = 3-0 eV
412-5xI0-^xl-6xl0-‘^
(//) At, V = 0, £;, = ft X 0 - (j)Q = - 00
Fi
F

Metal will show photoelectric emission only


or
0y = -£^=-(-4eV) = 4eV when energy of incident photons is greater than
he work function of that metal. As the energy of the
2. As, eV.0 - 00 ’SO
X incident photon (= 3 eV) is greater than the
he work function of Na and K, so only these two
ex6 = ...(/)
400x10-9
'0 metals will emit photoelectrons.

he 4. K.E. of photoelectron, K = (ftc/A) - 0q >


and exV^'
0
= '0 ...(H) case (0, K=l-Sx 1-6 X 10-'9j;
600x10-9
^=800x 10-"^ m ;
Solving (/) and (ii) for Vq ,
V0 ' = 4-97 V case (i7). A = 4-0x l -6x 10-*^ J,
3. Energy of the incident photon.
X=700x 10-"2 m

-34 ft X 3 X10^
he
E = —
(6-63x10
)X(3xlQ^) J 1-8xI-6x10-’9 =
800x10-1^
'0 ...(0
X (412-5x10-9)
DUAL NATURE OF RADIATION AND MATTER 11/63

/ix3xl0* 7. Here, V’o = 3-3V;v = ?;


40xl-6xl0"‘^ = '0 .(«) Vq = 8x 10^^Hz;(t)o = ?
700xl0-*0
Subtracting (/) from (ii) we get 6-63x10-34x8x 10’^
eV
1-6x10-*9
(40-1-8)1-6x10-19 =Ax3x10*6 ---
.7 8. = 3-315 eV

On solving we get, h = 6*57 x 10“34 Js As, hv = hvQ + K^^ = hvQ + eV^


5. Energy of the incident photon, eV. 1-6x10-'9x3-3
6-6x10-34x3x10^ or v = Vq + —^ =8x10*4 +
663x10-34
£=^= J=9-9x 10-'^ J
X 2000x10-^0 = 8 X 10*4 ^ 7.953 1Q14
9-9x10-*^ = 15-963 X 10*4 Hz

w
eV » 6-2eV
1-6x10-*^ 8. Here, E = Eq sin [(1-57 x lO’m-*) (ct - x)]
(a) K.E. of the fastest electron. Comparing it with the equation of harmonic wave

Flo
2tc
iir=£-(l)Q = 6-2-4-2 = 2eV in electric field ; E = E,.0 sin

e
{b) K.E. of the slowest electron = zero. As the

ree
emitted electrons have all possible energies from 27t 2ti
We have: = 1-57x10^ or A,= m.

FFr
0 to certain maximum value K. ’ X 1-57 xlO^
(c) If Vq is the stopping potential, then
^ _ he <l>o
urr
Vq = Kle = 2 eV/e = 2 V ^^~Xe e
(d) If is the cut off wavelength for aluminium.
or
sf 6-63 X10-34 X 3 X10* X 1.57 ^ io7
he -2-1
kks
then, <t>o = r- 2x3-142x1-6x10-*^
^0
Yo
ooo

= 1-0V
Ac 6-6x10-34x3x10*
he he _ he
eB

or
4-2x1-6x10-*^ 9. As, E =
*>0 X A(q a»
= 2946-4 X 10-*** m For first case:
ur

= 2946-4 A
ad

he he
3x10-*^ =
YYo

6. Here, ^Q=hS eV, X = 4500 A = 4-5 x lO*"^ m .(0


3310x10-*** Xq
(0 Max. K.E. of emitted photoelectron is
For second case:
dd

he
Re

K he he
in

max
X "'** 9-72x10-2** = Hi)
5000x10-*** Xq
F

(6-6x10-34)(3x10*)
-l-8x 1-6 X 10-*^ Subtracting (ii) from (1), we get
4-5x10-2
he 1 1
= 4-4 X 10-*^ - 2-88 X 10-*^ = 1-52 x ir*’ J (3 - 0-972) X l(r*9 =
10-*** [3310 5000.
(ii) Max. velocity of emitted photoelectron
2-028 X 10-*9 =
Ax(3xl0*)r5000 - 3310'
V
2K
max
2x1-52x10-*’
10-*** [3310x5000.
^ 9-lxlO
max -31
On solving, h = 6-62 x 10"34 Js.
m

= 5-78 X 10^ m S-* he


(Hi) Kinetic energy of the emitted photo-electron Now, *J*0 “ Y ~ ^
is independent of the intensity of the incident
light. Hence, if intensity of incident light is (6-62x1Q-34)x(3x10*) -3x10-*’
doubled, the max. K.E. of the emitted photo (3310x10-***)
electron remains unchanged (= 1-52 x ICT*’ J) = (6 - 3) X 10-*’ = 3 X 10-*’ J
11/64 ^fuidee^ Fundamental Physics (XII) VOL.II

Threshold wavelength, Energy of a photon,

he (6-62x10-34) x(3x1Q8) E = —
/»c _(6>63x 10-34)X(3x10^)
^0~T~ 3x10-1’ X SxKT"^
= 3-978 X 10-1’ J
= 6-62 X 10-"^ m = 6620 A
Energy emitted by source perminute = E^ = Pt
10. Here, A. = 180 x 10"’ m = 1-80 x 10-“^ m, or = 25 X 60 = 1500 J.
B = 5-0 X 10-3 No. of photons emitted per minute
i|)0 = 2-0 eV = 2 X 1-6 X 10-1’J 1500
N = = 3-77 X lO^l
= 3-2 X 10-1’ J 3-978x10-1’

ooww
1 2 14. Momentum of incident light, per sec.,
As, -mv^ = —
X
E^ 10
(6-6x10-34)x(3x1Q8) -3-2x10-1’ c ~3xlQ^
1-8x10-7 Momentum of the reflected light per sec.,

e
= 11 X l(r‘’-3-2x 10-1’ 60 E 60 10

re
= 7-8 X 10-1’ J ^2 100 c 100 3x108

rFFl
ree
F
.●.Force on the surface = change in momentum

I _ [2x7-8x10-1’
2x7-8x10-1’

rF
V =
per sec
m 9-1x10-31 = P2-(-Pl)=P2+Pl

fsfoor
= 1-309 X 10^ -I
ouur
ms 60 10 10
= X 4" . = 5-33 X 10-8 N
The electron moving in a perpendicular magnetic 100 3x108 3x1()8
kosk
field describes a circular pa& of radius r, then 15. No. of photons emitted per second by a source is
2
Yo
mv P PX
oo

evB = power
Y

energy of each photon ~ hc/X he


BB

mv (9-1 xlQ-l’)x (1-309x10^) 40x (6000x10-1’)


rre

or r =
eB ~ (l-6xl0-l’)x(5-0x10-3) = 12-12 X 10'’
(6-6x10-34)x(3x1()8)
oYuu

= 1-49 X 10-3 m
As one photon can eject one photoelectron only,
ad

= 1*49 mm and efficiency of photoelectric emission is 5%


dY

11. (0 Energy of a photon = hv so

= (6-63 X 10-34) X (6-0 X 10l4) _ no. of photoelectrons emitted per sec (n)
innd
Re

= 39-78 X 10-20 J
no.of photons falling per sec (12-12x lO' ’)
(ii) No. of photons emitted per second
Fi
F

5 n

_ total energy/sec 100 12-12x10'’


energyof one photon
5x12-12x101’
or n = = 60-60 X 10*7 s-i
2-0x10-3 100
“ 39-78x10-2’ ne ne

= 5-03 X 10i3 photons/sec Photoelectric current, 1 = —


t 1
he I AX
12. n— = I.A. or n = = 60-6 X 10i7 X 1-6 X 10-1’ = 96-96 x 10"2 A
X he
= 0-97 A
10-i’xl0-4x5-84xl0-7 16. Here. X = 0-42 A = 0-42 x 10-'’ m;
n =
= 3x1©4
(6-6 X10-34 )x (3x10*) Max. energy of photon,
13. Here, P = 25 W, / = 60 s,
X = 5000 A = 5 X 10-7 m:
^ /rc ^ (6-63 X10-34 )x (3 x1Q8)j
X 0-42x10-1’
DUAL NATURE OF RADIATION AND MATTER 11/65

663 x 3x10-26 1 2 h \ 2 hv
£ = eV K.E. of electron, = -mv
(0*42x10-*0)x1-6x10-^9 2 v% 2X
\ y

« 29-6 X 103 he
K.E. of photon, ^p-~:r
K
= 29^keV
K 1-5x108 1
It shows that the maximum energy of X-ray kvnx V

w
e _

photon is 29-6 keV. As accelerating voltage hclX 2c ”2x(3x108) " 4


provides eneigy to the electron for the production 21. Here, X = 5500 A = 5500 x lO”*® m
of X-rays, so to get the X-rays of eneigy 29-6 eV,
= 55xl(Hm
the incident electron must possess the least

e
energy 29*6 keV. Therefore the accelerating de-Broglie wavelength associated with electron is

e
voltage of the order of 30 kV is required for

wr
lloo
producing X-rays. or £ =
X= I

r
h 6-6x10-^ .J2^

eF
17. (0 Momentum of photon, p=—=
1 [’6-6x10-3'*?

u
X ixlO"®

r FF = 6-6 X 10-25 kg ms-i


66x10"^^
2x(9-1x10-3^)L 55x10-8 .
= 7-9 X 10-25 J

rrse
Momentum of electron, p =
1x10“® 7-9x10-25
uo
koF
= 4-9x10-6 eV
= 6-6 X 10-25 kg 1-6x10-1®

oofr
he
22. Here, X. = 2 nm = 2 x 10"® m

(I'O Energy of photon, E- —


sf
ko
h
X.
Y
(0 As A. = — or P = :r. since X, for both electron
p X
_(66xl0'^)x(3xl0^)
B
Yo
= 1-98 X irl6 J and photon is same so they have equal
BY
oo

1x10“® momentum, given by


er

2
h 6-63x10-5^
re

(ii'O Kinetic energy of electron, K=-^ = 3-315 X 10-25 kg m s-i


uu

2m 2X10-®
od

he
(66x10"25)2 (ii) Energy of photon, E = —
Yo
ad

= 2-39 X 10-1® J X
2x(9-1x10"31)
n

(6-63x10-5^)x(3x108) J
12-27A
18. X.= = 1-53 A 2x10-®
ndi

V64
Re

9-945x10-12
F

This wavelength belong to x-rays. eV = 6-22 X Wr2


Fi

1-6x10-1®
19. Here, y = 10-0 m ; m = 40 kg ;
(in) Kinetic energy of electron,
Velocity of boy before entering the water is
i)->^lgy =-^/2x 10x10 = 10^2 m/s £ = p^ _ (3-315x10-25)2 j
2m 2x9-1x10-31

de-Broglie wavelength, X, = (3-315x10-25)2


mv eV = 0-377 eV
2x9-1x10-31x1-6x10-1®
6-6x10-3^ 23. Energy of incident photon.
= 1-167 X10-36 m
40x10-,^
20. Here, v = 1-5 x 108 ms -1 ^_hc _ 6-63X10-34 X3x 1Q8 ^
X. 1-4x10-1®
de-Broglie wavelength of electron. Energy of scattered photon.
h
or m = P _ he 6-63x10-3^x3x1Q8
mv vX ' X, 4x10-1®
11/66 Fundamental Physics (XII) VOL.II

Energy of scattered electron = £ - 28. Here, T = 127 + 273 = 400 K.


1 3
6-63x10-34 xSxlO^r 1 1
Energy of electron = -mv^=-kT
10-10 1-4 4 2 2

w
= 9*23 X J or mv
= ^3mkT
h 1 de-Broglie wavelength of electrons is
24. As, X = so K = or K o: —
^2mK 2mX^ A2 h
mv
■yjSmkT
K
> 'OSl^ ^ i

e
wr
K2 .^1. . 1 J ”4 6-63x10-34

r
oo
or K2 = *K^=4x2eV=SeV 73x(9-1x10-31)x(1-38x10-23)x400
Increase in energy = K2~K^=S-2 = 6e\

F
6-63x10-34

FFllu
= 5-4 X 10-’m
25. = ..(0 ”12-276x10-36
p Mean separation between two electrons in metal
As p decreases. A, increases.

ees
is r = 2x lO-^^m X _ 5-4x10-^ = 27
Let Pq be the decrease in momentum when

rr
r " 2xlO-*o
uro
Fk
wavelength increases by 0-25%.
A^_^_10000_2

ro
0-25 h 29. (0
Then, X-i- X =
£j hdX 1 5000

ofo
100
P-Po
or £2= 2^1 = 2 X 1-23 = 2-46eV
Y
100-25 h
sof
or X = .(«●) Energy of incident photon, £ = 2-46 eV ;
oY
100
(P-Pq) Given, stopping potential = 1-36 V ; therefore,
B ok
Yo
100-25 _ p kinetic energy of photoelectrons, K= 1-36 eV.
From (i) and («). we have So, work function is
eBr

100 P-Po
%= E-K= 2-46- 1-36 = MeV
rue

On solving, p = 401 Pq = 401 x 9 x 10-36 (11) When the intensity of the incident light is
oud

= 3-6 X 10"33 kg ms"* made four times, the stopping potential remains
26. De-Broglie wavelength.
o

unchanged (i.e. 1-36 V) but saturation current


ad
iYn

1 - ^2-27 12-27 A = 0-87 A will become four times = 4 x 0-40 x 10^


” >/v = 1-6x10-6 A
The value of wavelength belongs to x-rays 30. Energy of photon, £ = 10-6 x 1-6 x 10-*^ J ;
nd
Re

27. Rest mass of proton = 1-675 x 10*33 j^g


F

7=2-0 Wm-3; A = 10-4 m3; <1,^= 5.5 eV ;


u = 0-5 c = 0-5 X (3 X 10^) = 1-5 x 10^ m/s
Fi

Tl = 0-53%
As V is comparable to c, hence mass of proton in
motion will be a relativistic mass. No. of photo-electrons ejected per sec
0-53
mo mo X no. of photons falling per sec
So, m = 100
Vl-t;3/c3 .^1-(0-5c)3/c3
_ mp 2mo 0-53 lA 0-53 2x10-4
X— =

^/3 100 £ 100 10-6x1-6x10*'^


= 6-25 X IQll s-1
6-6x10-34
Minimum energy of ejected photoelectron
mv
(2mQl^/3)xv = zero K.E.
6-6x10*34 x^ Maximum energy of ejected photoelectron
2x1-675x10*33 x(1-5x108) = 10-6 - 5-6
= 2-27 X 10-16 ^ = 50 eV
DUAL NATURE OF RADIATION AND MATTER 11/67

31. ]^mv\=eV^=hv^-ifQ k =
-JlmK

rw
he
-12
or
%=hv^-eV^ =-—eV^ In first case, 100 x 10 .(0
Ai ^2m/sr,
6-6x10"^^ x3x 10* h
‘i'o- -10
-l-6xl0"^^x0-6 In second case, 50 x 10"*^ = ...(«●)
4950x10
^ImK^

e
= 304x 10-‘9y
From (i) and (n)
= 1*9 eV

r
o
he 2 =
^2

llou
F
^2 \^1

w
= 1-6 X lO-l^x M +3 04X 10-‘9 or
K2 = 4Ki
= 4-8 X 10-1^ J Energy to be added = 4 ^2 “ ^i = ^ ^i

FF
s
From (0
he 6-6 X10-34 X (3x10*)
%2 - 4*8x10-19
uro 4*8x10-19 6*63x10-34

k
rere
=
= 6*63 X 10-24
= 4125 A 10-10

oFo
As the magnetic field does not change the speed
of the electrons hence there will be no change ^1 = (6*63x10-24)2 ^
2x9*1x10-31

fofr
in the stopping potential.
o
Y
-12
ooY
32. Here, A,| = 100 pm = 100 x 10 m; Energy added = 3
= 50 X 10 -12
B
m
ks
De-Broglie wavelength. 3x(6*63x10-24)2
Yo
eV
2x9*1x10-31x1*6x10-19
r

X, =-^ and K = —mv^ = 4*52 X 102 = 452 eV


eB
e

mv 2
ru
d
no
ou
ad
iY
nd
F
Re
Fi
11/68 ‘P'KicUcf^ 'a Fundamental Physics (XII) tTOWTl

. f

■ .y

rw
yii
A*
■:

y -f
.s.
WITH
Si : SOLUTIONS

Q. 1. Find the (a) maximum frequency and (/>) minimum wave*length of X-rays produced by 30 kV
electrons. Given, h = 6*63 x 10"^ Js.

r
Sol. Here, V= 30 kV = 30 x 10^ V = 3 x 10^ V

wo
llou
F
eV (l-6xl0-*^)x3xl0'‘
(a) eV=hv or V = = 7-24 x 10^* Hz
h 6-63x10-3'^
he he (6-63x10"3‘^)x(3x10*) = 0-041 X 10"^ m = 0041 nm

FF
(b) eV = or

s
(1-6x10-19)x(3x10‘*)
min
eV
min

2-14 eV. When light of frequency 6 x 10^‘* Hz is Incident on


uro
Q. 2. The work function of caesium metal is

ker
e
the metal surface, photoemission of electrons occurs. What is the (a) maximum kinetic energy of the
emitted electrons, (b) stopping potential and (c) maximum speed of the emitted photoelectrons.

For
Given ; h = 6-63 x Js, 1 eV = 1*6 x lO'*’ J; c = 3 x 10* m/s.

ro
fof
6-63xlO"^'‘x6xlO^‘*
Sol. (a)Max. K.E. =/;v-4>q =
o
-2-14 = 0-346 eV = 0*35 eV
Y
Y -19
1-6x10

(b) cVq = Max. K.E. = 0-35 eV Vn0 = 0-35 V


B
or
ks
Yo
= 0-346 eV = 0-346 X 1-6 x lO"'^!
roo
eB
e
u
rd

-.1/2
-19
0-346x1-6x10 x2
or V = 3-488 X 10^ m/s = 348-8 km/s = 349 km 8"^
max -31
9-lxlO
o
n
ou

Q. 3. The photoelectric cut off voltage in a certain experiment is 1*5 V. What is the maximum kinetic
ad

energy of photoelectrons emitted ? c = 1*6 x 10“*^ C.


iY

Sol. Max. kinetic energy = eVQ = e x 1 -5 V = 1*5 eV.


Q. 4. Monochromatic light of wavelength 632*8 nm is produced by a helium-neon laser. The power emitted
nd
F
Re

is 9*42 mW. (a) Find the energy and momentum of each photon in the light beam. (6) How many
Fi

photons per second, on the average, arrive at a target irradiated by this beam ? (Assume the beam
to have uniform cross-section which is less than the target area), (c) How fast does a hydrogen atom
have to travel in order to have the same momentum as that of the photon, h = 6*63 x 10“^ Js,
1 a.m.u. = 1*66 x 10"^^ kg.
he (6-63xl0-^‘^)x(3xl0*)
Sol. {a) E = — = 314xlO-*®J;
K 632-8x10-9

_h _ 6-63 xlQ-^^
= 1*05 X 10"^^ kg ms"^
632-8x10-9

P 9-42 x 10-^ W
ib)n = -
3-14x10"*9 j = 3 X 10^^ photons/sec
1-05x10-2'7
(c) u = -^ = - 0*63 ms"^
m 1-66x10-27
DUAL NATURE OF RADIATION AND MATTER 11/69

Q. 5. In an experiment on photoelectric effect, the slope of the cut off voltage versus frequency of incident
light is found to be 4*12 x 10~^^ Vs. Given e = 1*6 x 1(T^^ C, estimate the value of Planck’s constant
Sol. Given, slope of graph = 4-12 x 10“^^ Vs ; Slope of graph = h/e
h = ex slope of graph = 1-6 x 412 x = 6*592 x 1(^4 jg
Q. 6. The threshold frequency of a certain metal is 3*3 x 10^"* Hz. If light of frequency 8*2 x 10^^ Hz is
incident on the metal, predict the cut off voltage for photoelectric emission. Given Planck’s constant,
A = 6*62 X 10-34 Js.

Sol.
A(v-Vq) _ 6-62x10~34 (8-2x 10‘4 - 3-3X 10^4) = 202*74 V
eVQ-h(v-VQ) or Vq = e ~ 1-6 X 10-^9
Q. 7. The work function for a certain metal is 4*2 eV. Will this metal give photoelectric emission for
incident radiation of wavelength 330 nm ? Given, charge on electron, e = 1*6 x 10~^^ C, c s 3 x 10^
m/s ; h = 6*62 x 10"34
Sol. (1)0 = 4-2 eV = 4-2 x 1-6 x J = 6-72 x J

w
^ Ac ^ 6-62x10-34 x3xl0^ = 6 018 X 10-^9 J
~ X~ 330x10-9

Flo
As energy of incident photon E < (|>o, hence no photoelectric emission will take place.
Q. 8. Light of frequency 7*21 x 10^4 ^ }g incident on a metal surface. Electrons with a maximum speed

reee
of 6*0 X 10^ ms-^ are ejected from the surface. What is the threshold frequency for photoemission of
electrons 7 h = 6*63 x 10"34 jg^ _ 9.1 ^ iQr3i jjg.

FFr
2
= 7-21x10^4 _ (94x1Q-31)x(6x 10^)3 = 4*74x1014 Hz
mv

|mt;^=Av-Avo or Vq
max
uurr
Sol. = V-
2h 2x(6-63xl0"^^)
for
Q. 9. Light of wavelength 488 nm is produced by an argon laser which is used in the photoelectric effect
kss
When light from diis spectral line is incident on the cathode, the stopping potential of photoelectrons
is 0*38 eV. Find the work function of the material from which the cathode is made. Given ;
ooook
Yo

h = 6*63 X 10-^ Js, 1 eV = 1*6 x l(r'9 j


or d. - (6-63x10-3^)x3x108
eB

he
Sol. eVQ=--% -0-38 =2*17eV
A. ° (488x10-9)x 1-6x10-19
rr

Q. 10. What is the de-Broglie wavelength of (a) a bullet of mass 0*040 kg travelling at the speed of
1*0 km s-l. (A) a ball of mass 0*060 kg moving at a speed of 1*0 ms-i and (c) a dust particle of mass
ou
ad

1*0 X 10-9 i^g drifting with a speed of 2*2 ms-l ? A = 6*63 x 10-34 Js.
YY

6-63x10-34
Sol. (a) A, = —= = 1-66 X 10-35 « 1.7 i(j-3S ^
nndd

nw (0-040) X (1-0 xl03)


Re

6-63x10-34 6-63x10-34
Fi

(A) A.= = 1*1 X 10-33 m (c) X = = 3*0xir“m


(0-060) X1-0 (1-0 X10-9 )x 2-2
Q. 11. Show that the wavelength of electromagnetic radiation is equal to the de-Broglie wavelength of its
quantum (photon).
Sol. The momentum of a electromagnetic wave of frequency v wavelength X is given by.
h
hv _h x = h.
p=— or De-Brotlie wavelength of photon, X = —
c X P P

Thus wavelength of electromagnetic radiation is equal to the de-Broglie wavelength.


11/70 'PnaxCct^ 4. Fundamental Physics (XII) WilWII

WITH ANSWERS,
HINTS AND SOLUTIONS

MULTIPLE CHOICE QUESTI

1. A particleis droppedfrom a height H. The de


(c-) X^<Xp = X„>X

oww
a

Broglie wavelength of the particle as a


function of height is proportional to (d) Xg = Xp = X^ = ,X a

1/2 6. An electron is moving with an initial velocity


(a)H {b)H
0 -1/2 V = V I and is in a magnetic field
{c)H (d)H »

e
A

2. The wavelength of a photon needed to remove B - B^j . Then it’s de Broglie wavelength

ree
rFl
a proton from a nucleus which is bound to

Fre
(fl) remains constant
the nucleus with 1 MeV energy is nearly

rr F
{b) increases with time
(fl) 1-2 nm ib) 1-2 X 10 ^ nm
(c) decreases with time
(c) 1-2 X 10“^ nm (d) 1-2 x 10^ nm
ouur
3. Consider a beam of electrons (each electron sfoo
{d) increases and decreases periodically
7. An electron (mass m) with an initial velocity
with energy £q) incident on a metal surface
kks
—»
kept in an evacuated chamber. Then q?(Vq>0) Is in an electric field
Yo
oooo

V = V

(a) no electrons will be emitted as only photons


E = - E^i (Eq = constant > 0). It’s de Broglie
eBB

can emit electrons


wavelength at time t is given by
(d) electrons can be emitted but all with an
energy, Eq
uurr

X eEr.t
0 0
(c) electronscan be emittedwith any energy, with ia) ib) L,0 1 +
ad

eE.0 1 mv
YYo

a maximum of is the work function) 1 + 0 )


m V
0
{d} electrons can be emitted with any energy,
dd

with a maximum of
(c) \) id) Xq (
Re
iinn

4. Consider Fig. 11.18 in the Fundamental


8. An electron (mass m) with an initial velocity
Physics for Class XII. Suppose the voltage
F

applied to A is increased. The dilTracted beam


will have the maximum at a value of 6 that
V = V
? is in an electric field E = E^j .M
= hlmv^^, it’s de Broglie wavelength at time
(a) will be larger than that earlier value t is given by
(b) will be the same as the earlier value
(c) wilt be less than the earlier value e^ElP-
0

id) will depend on the target (a) \ ib) \ 1 +


m
2
V
2
0

5. A proton, a neutron, an electron and an a-


particle have same energy. Then their de X
0
X0
Broglie wavelengths compare as ic) id)
e'^ElC- e^ElP-
0
(a) Xp = X,j>X^>X
0 1 +
a 1 +
■> 2
m~ \r nr vx
0
(b) X^<Xp = X„ < X^ u
DUAL NATURE OF RADIATION AND MATTER 11/71

MULTIPLE CHOICE QUESTIONS-JL'-i

9. Relativistic corrections become necessary 12. Photons absorbed in matter are converted to
when the expression for the kinetic energy heat. A source emitting n photon/sec of
frequency v is used to convert 1 kg of ice at
- mv^ , becomes comparable with where 0**C to water at 0“C. Then, the time T taken
2
for the conversion
m is the mass of the particle. At what de
Broglie wavelength will relativistic correc (rt) decreases with increasing n, with v fixed
tions become important for an electron ? {b) decreases with n fixed, v increasing
(a) A = 10 nm ib)X= 10-’ nm (c) remains constant with n and v changing such

oww
(r) A = 10^ nm id)X = 10-^ nm that nv = constant

10. 'fwo particles Aj and A2 of masses /nj, m2 (r/) increases when the product nv increases
(m| > m2) have the same de Broglie wave 13. A particle moves in a closed orbit around the
length. Then origin, due to a force which is directed
(fl) their momenta are the same

e
towards the origin. The de Broglie wavelength

FFrlo
re
(b) their energies are the same of the particle varies cyclically between two
(c) energy of Aj is less than the energy of A2 values ^1, Xj > ^2- Which of the

ree
F
(cl) energy of Aj is more than the energy of A2 following statement are true ?
11. The de Broglie wavelengthof a photonis twice id) The particle could be moving in a circular

rF
the de Broglie wavelengthof an electron. The orbit with origin as centre
(/?) The particle could be moving in an elliptic

fsoor
ouur
c

speed of the electron is = . Then


orbit with origin as its focus
100
skf
£
(c) When the de Broglie wavelength is X.j, the
5l = io-2
ooko
= 10 particle is nearer the origin than when its
Yo
value is
Y
r
Bo

{d) When the de Broglie wavelength is the


Pe Pe
reB

ic) = 10-2 id) = 10^ particle is nearer the origin than when its
c m c
e value is A..
uur
oY
ad

ANSWERS
dY

1. id) 2. ib) 3. id) 4. (c) 5. (b) 6. (a) 7. (a) 8. (c)


9. (c. d) 10. {a, c) 11. (b, c) 12. {a, b, c) 13. (b, d)
innd
Re
Fi
F

HINTS FOR DIFFICULT MULTIPLE CHOICE QUESTIONS

Multiple Choice Questions -1

1. Velocity acquired by a particle while falling from a height H is, V

h
or A « W Hence option (d) is correct.
mv m
■JIJh
2. Energy £= I MeV= 10^ eV ; he = 1240 eVnm
he he 1240ey nm
£ =— or A = — = 1’24 X 10“^ nm
A £ lO^eV
3. When a beam of electrons of energy £q is incident on a metal surface kept in an evacuated chamber, electrons
caji be emitted with maximum energy £q (due to elastic collision) and with any energy less than £q. as ptut of
incident energy of electron is used in liberating the electrons from the surface of metal.
11/72 7\eidee^% Fu!hdamental Physics (301) VOL.II

4. In Davisson and Germer Experiment, if V is the voltage applied to anode, then de-Broglie wavelength
associated with electron is

12-27
A (0

If there is a maxima of the diffracted electrons at an angle 6, then


2dsin6sX m
From (0, we note that if V increases. A, decreases.
From (it), we note that as X decreases, the value of sin 6 decreases, Le.y 0 decreases.
1
5. Kinetic energy of particle. K=-^nw^ or mv
= pmK
2

llowow
De-Broglie wavelength.
mv ^2mK
1 1 1 1
For the given value of K, A,« 1/Vm ● ●*.
-J“o

ee
Since hence
mp = m n»

Fr
As «a > therefore r FF K<^i,
As nig < m„, therefore K>K

r
Hence
forfFore
u
6. Here, v = Vq r, B=BqJ
s
ok
^ -4 A A A
YYour o
Force on moving electron due to magnetic field is F = -e(v xB) = -e[vQ i x^j] = -cVq Bq k
o

As this force is perpendicular to v and B , so the magnitude of v will not change, Le.y momentum
eeBoBks

(= mv) will remain constant in magnitude. Hence de-Broglie wavelength A. = h/nw remains constant
r

h
7. Initial de-Broglie wavelength of electron. A,q (0
our u
ad

mU(j
Yo

Force on electron in electric field, f z=-eE = -e[-EQ ?] -eE^^


d
Re
in

Acceleration of electron.
m m
FFind Y

Velocity of electron after time /, i? =s f+ = »0+ £i,lf -4.


m
/ muo )
de-Broglie wavelength associated with electron at time t is

x=± h ^0
[from (i)I
mv
m
Vq 1 + 1 + l£«,
, -""o 1 mv
0 J

8. Initial de-Broglie wavelength of electron, =


mvo

Force on electron in electric field, F --eE =-bEqJ

\
DUAL NATURE OF RADIATION AND MATTER 11/73

Acceleration of electron,
F_
m m

It is acting along negative y>axis.


The initial velocity of election along jc-axis ^ velocity of electron along y-axis, iT^0 = 0


Velocity of election after time t along jc-axis,
(*.* there is no acceleration of electron along x-axis)

Velocity of electron after time t along y-axis, Vy = 0 + j t=- m


tj

Magnitude of velocity of electron after time t is

looww
f-eE
vi* = Ua0 . 1 + 2 2
V m ; m^vl
h ^0
De-Broglie wavelength, V = — =

ree
mv
mv.

ree F
9. Velocity of electron, v = h/(m X)
r FF
Multiple Choice Questions - II

Let /i = 6-6 X 10-34 Js and m


fofroF
= 9 X 10-3J kg
u
(a) When A,] =s 10 nm = 10 X 10"^ m = 10^^ m
ks
6-6x10-34 2-2
xl()3 =103m/s
kos o

V
YYouor
i ~
(9x10"3^)x10"* 3
^2 = 10-1 nm = 10-1 X 10-® m = 10-*o m
BBoo

ib) When
r ee

66x10-34
V »10^ m/s
^ (9xl0"3i)xl0"i°
ouru
ad

(c) When ^3 = 10-4 nm = KH X 10-9 m = 10-13 m


Yo

6-6x10-34
V a 10l9 m/s
d

^ ”(9x10-31 )x 10-13
Re
inY

(rf)When X4 = l(r6 nm = 10^ X 10-9 m = l(ri5 m


FFind

6-6x10-34
V a lOl^ m/s
^ (9 X10-31 )x 10-13
As t>3 and are greater than velocity of light (= 3 x 10^ nd/s), hence relativistic correction is needed for X
= 10^ nm and X = 10^ nm. Thus, options (c) and (d) are rquired wavelengths.
h 1 Pt Xa X ,
10. As X= -
P
or p = —
X
or p oc —
X
. ●- -L = ^ = - = l or Pi=P2
P2 X| X

E = -i£i=_L^ V p =
h\
2 m 2m X^ \

So E oc — 1 . El^Ti <1 or < ^2


m
£2

fl
11/74 ‘P'utdee^ ’4. Fundamental Physics (XII)
h h 100/»
n. For electron, ...(0
rn e V e m e*'(c/100)' m
e
c

1 ●>
Kinetic energy. E
e
= - tn V-
e e
or m
e
V
e
= J2E
y e
me
2

h h /,2
X or E =—- ...(«)
' 2-k]
e

E^ m
e

he he
For photon of wavelength Xp, energy = E^ = X. 2X (●●● \ = 2X^)
P e

E
P he
X
IX] m
e 'Egm^c 100 ft X
"'gC = 100
E
e
1
= 10-2
2X^ ft2 ft m e ft E 100

w
e P

Pe
For electron, Vg = x c/IOO = 10-2
100
mg e

Flo
Thus options (ft) and (c) are correct.

eeee
12. Energy spent to convert ice into water = m L = (1000 g) x (80 cal/g) = 80000 cal
Energy of photons used = nTxE = nTxhv (V £=ftv)

Fr
inL
iiThv = mL or T-
nhv

for
ur
T l/n when v is constant; T oc
1/v when n fixed ; T OC \/nv. Thus T is constant if nv is constant. Thus
options (a), (ft) and (c) are correct.
ks
13. The de-Broglie wavelength of the particle can be varying cyclically between FIGURE 11(N).1
Yo
two values Xj and X2, if particle is moving in an elliptical orbit with origin as
oo

U2
its one focus. Refer Fig. II(N).l. Let t;|, V2 be the speed of particle at A and B
eB

respectively and origin is at focus O. If Xj, X2 are the de-Broglie wavelengths A P


associated with particle while moving at A and B respectively. Then O

ft ft
ur

Ui
and ^2 =
ad

mv
1 m^2
Yo

X
1 ^2
= — ; Since Xi > X^ >v■
X. V| 1 ^ ^ I
d
Re
in

From figure we note that origin 0 is close to P than A. Thus options (ft) and (d) are true.
F

VERY SHORT ANSWER QUESTI

14. A proton and an a-particle are accelerated, using the same potential difference. How are the de
Broglie v welengths Xp and X^ related to each other ?
x2e
Ans. As,
ft . 1 ^P _ ^ =2n/2
X
■yj2 mq V a

15. (0 In the explanation of photo-electric effect, we assume one photon of frequency v collides with an
electron and transfers its energy. This leads to the equation for the maximum energy E max of the
emitted electron as = ftv - <})o E max

where 4>g is the work function of the metal. If an electron absorbs 2 photons (each of frequency m)
what will be the maximum energy for the emitted electron ?

I
DUAL NATURE OF RADIATION AND MATTER 11/75

(h) Why is this fact (two photon absorption) not taken into consideration in our discussion of the
stopping potential ?

Ans. (/) Given. ^mux = - 4'0- When v' = 2 V, £'max


= h{2v)~(pQ =2//v-4>„
(iV) The probability of absorbing 2 photons by the same electron is very low. Hence such emission will be
negligible.
16. There are materials which absorb photons of shorter wavelength and emit photons of longer
wavelength. Can there be stable substances which absorb photons of larger wavelength and emit
liglit of shorter wavelength.
Ans. In the first case, the energy of the incident photon on a material is high and the energy of emitted photon
is low.

In the second case, the energy of the incident photon is low and the energy of emitted photon is high. It
means in second case the material has to supply the energy for the emission of photon. This cannot happen

ww
for stable substances.
17. Do all the electrons that absorb a photon come out as photoelectrons ?
Ans. No, most electrons get scattered into the metal by absorbing a photon. Only a few come out of the surface

FF loo
of metal whose energy becomes greater than the work function of metal.
18. There are two sources of light, each emitting with a power of 100 W. One emits Z-rays of wavelength

ree
1 nm and the other visible light at 500 nm. Find the ratio of number of photons of A'-rays to the
photons of visible light of the given wavelength ?

reeF
Ans. Here, = 100 W, = I nm, X2 = 500 nm
Let/ij, ^2 = no. of photons of X-rays and visible light emitted from the two sources

oroFr
r ur
he he n
1 X,1 1
P = n, — = ru, — or
s ff
or
'X.1 X^ X «2 ^-1 500
k
YYouo
koso

SHORT ANSWER QUESTIONS


BBoo

FIGURE 11(N).2
19. Consider Fig. 11(N).2 for photoemission. How would you reconcile with
r ee

Electron

momentum-conservation ? Note light (photons) have momentum in a different


direction than the emitted electrons.
ad
ouur

Ans. During photoelectric emission, the momentum of incident photon is transferred to


Yo

the metal. At microscopic level, atoms of a metal absorb the photon and its
Light
momentum is transferred mainly to the nucleus and electrons. The electron excited
d

is emitted. Therefore, the conservation of momentum is to be con.sidered as the


Re
idnY

momentum of incident photon transferred to the nucleus and electrons. Metal


FFin

20. Consider a metal expo.sed to light of wavelength 600 nm. The maximumenergy of the electron doubles
when light of wavelength400 nm is used. Find the work function in e.V
Ans. Here. X. = 600 nm, K'nuix = 2K max ' X' = 400 nm, 4>q = ?
he
K -({) 0 ...(/●)
max
X
he
2K ...(»)
mux '0
X'

{heIX') - (|)p 2hc he


Dividing (//') by (/), we gel 2= ^ or
'0
or
-H>0
(/tt’/X)-<j) 0 X X'

= 1240
( -=—
2 1
= l*03eV
0 [●.● he - 1240 eV nm|
600 400

I
11/76 P'lacUefr'^ Fundamental Physics (Xn) VOL.II

21. Assuming an electron is confined to a 1 nm wide region, find the uncertainty in momentum using
Heisenbei^ Uncertainty principle (Ax Ap » ^). You can assume the uncertainty in position Ax as
1 nm. Assuming p ~ ^, find the energy of the electron in electron volts.
Ans. Here, A X = 1 nm = 10:^ m, A p = ?

As, AxAp~h
n _ h 6 62 X10-3^ Js = 1-05 X ir^s kg ms-^
Ax ” 2icAx ” 2X (22/7) 10-^ m

Enew, £ = |i = (^ ^”= (105x10-^)^


2m 2x91x10-31
2m
j^ (105x10-^)^
2x91x10-31x1.6x10-19
eV =3-8xir2eV

(Givenp = A p)
22. Two monochromatic beams A and B of equal intensity /, hit a screen. The number of photons hitting
the screen by beam A is twice that by beam B. Then what inference can you make about their

ww
frequencies ?
Ans. Let /ij, «2 = number of photons falling per second of beam A and B respectively
Energy of falling photon of beam A = hvi ; Energy of falling photon of beam B = hv2

FF loo
Vi ^2 '*2 _ 1^
n

As per question, intensity of A = intensity of B /. "l “ “ “ “ 2n~ ~ 2

ree
Le., Frequency of beam B is twice of beam A.
23. Two particles A and B of de-Broglie wavelengths A.| and combine to form a particle C. The process

reFe
conserves momentum. Find the de-Broglie wavelength of the particle C. (The motion is one

ororF
rur
dimensional).
h h h
s ff
Ans. lpcl = lp^l + lpgl or .(0
B
k
Following cases may arise
YYouo
okso

Case (0 p^ > 0, pg > 0, i.e., both p^ and are positive.


BBoo

From (0, or
Xc -
^A^B
r ee

Case (ii) p^ < 0, p^ < 0, i.e., both p^ and pg are negative.


ad
ouur

h h h h(X^ + Xg) ^A^B


Xc =-
Yo

From (0, or

^B (X^ + Xg)
Case (wi) p^ > 0, pg < 0, Le., p^ is +ve and pg is -ve.
d
idnY
Re

h _Ji h .(^b~^a^^ ^A
From (0, or
Xc -
FFin

^A ^B ^A ^B ^B-'^A
Case (iv) p^ < 0, Pg > 0, Le., p^ is -ve and pg is +ve.
h h h (X^ ^A ^B
From (/) or
Xc -
^A ^B ^A ^B ^A~^B
24. A neutron beam of energy E scatters fixrm atoms on a surface with a spacing d = 0*1 run. The first maxi
mum of intensity in the reflected beam occurs at 6 = 30°. What is the kinetic energy E of the beam in eV ?
Ans. Here, d = 0*1 nm, 8 = 30°, n = 1. According to Bragg’s law
2dsin0 = nX or 2 x 0-1 x sin 30°= 1 x X or X = 0*1 nm = lO-l® m
h 6-62xl0r34
= 6-62x10-24 kg ms-l
^=X = 10-10
(6-62x10-24)2
eV = 0*082 eV
2 m 2 1-67x10-22 2 (1-67 X10-22 )x (1-6 xl0-‘9)

:
DUAL NATURE OF RADIATION AND MATTER 11/77

LONG ANSWER QUESTIONB

25. Consider a thin target (10”^ m square, 10“^ m thickness) of sodium, which produces a photocurrent
of 100 |iA when a light of intensity 100 W/m^ (A. = 660 nm) falls on it. Find the probability that a
photoelectron Is produced when a photon strikes a sodium atom. [Take density of Na = 0*97 kg/m^,
Avogadro’s number = 6 x 10^ kg atom].
Ans. Here, A = m sq = x 10“^ sq m = 10^ d= 10"^ m, i = 100 x 10"* A = lO"^ A
Intensity, / = 100 W/m^, X = 660 nm = 660 x 10”^ m
Probability = ?, = 0-97 kg/m^, Avogadro’s number = 6 x 10^^ kg atom
Volume of sodium target =Axd = lO'^ x 10“^= 10"^ m^
23 3
We know that 6 x 10^® atoms of Na weighs - 23 kg Volume of 6 x 10^^ Na atoms = m
0-97

ww
23
Volume occupied by one Na atom = = 3-95 X 10"-^ m^
0-97x(6x1026)

Flo
lO-"^
No. of Na atoms in the target = = 2-53xlO>^
3-95x10-26

ee
Let n be the number of photons falling per second on the target.

rere
Energy of each photon = hc/X

r FF
Total energy falling per sec on target = = IA
uurr
X

n =
foor
lAX _ 100x10^ x(660xlQ-^) = 3-3x10*6
ks s
he (6-62 xl0"3'^)x (3x10^)
Yoo
Let P be the probability of emission per atom per photon.
oook

The number of photoelectrons emitted per second N = P x nx{n^^ = P x 10*6) x (2-53 x jQis^
eBB

As per question, / = 100pA= 100 X 10-6 = 10-^ A,


Current, i=Ne 10-^ = Px(3-3x 10‘6)x(2-53x 10'^)x(1-6x 10-*^)
uurr

10-^
P = ir = 7-48 X l(r21
ad

or

(3-3x 10'6)x(2-53xI0*^)x (1-6x10- 19 )


Yo

Thus the probability of emission by a single photon on a single atom is very much less than 1. It is due to
dY

this reason, the absorption of two photons by an atom is negligible.


26. Consider an electron in front of metallic surface at a distance d (treated as an infinite plane surface).
Re
innd

9^
FFi

Assume the force of attraction by the plate is given as ~


4 47C€Qrf2 ●
Calculate work in taking the charge to an infinite distance from the plate. Taking d = 0.1 nm, find the
work done in electron volts. [Such a force law is not valid for d < 0*1 nm}.

Ans. Refer Fig. 11(N).3, = 0-1 nm = 10-*** m ; F =


4x47teQ
Let the electron be at distance x from metallic surface. Then force of attraction on it FIGURE 11(N).3
IS

^x= ^ T
4x47C€q;c‘^
4 = 0.1 nm
Workdone by an external agency for a displacement dx against the force of attraction
F,is,
dW = F^ dx
11/78 “Pn^tdee^^ Fundamental Physics (XH)GSESD
Toial workdone by external agency in taking the electron from distance d to infinity is
->

W = F dx =
7 ^ I
X
4x47i:e^0 \_d

(1-6xI0^‘^)^x9x1Q9 j_ (1-6x10"'^)2x(9x10^) eV =3'6eV


4xl0-‘° (4xlO-'0)x(l-6xlO-'^)
27. A student performs an experiment on photoelectric
effect, using two materials A and B. A plot of V^jop vs v
is given in Fig. 11(N).4
(i) Which material A or B has a higher work function ?
{«) Given the electric charge of an electron =s 1*6 x
10“*^ C, find the value of h obtained from the

w
experiment for both A and B.
Comment on whether it is consistent with Einstein’s
theory.

Flo
Ans. (/') Here, threshold frequency of A. = 5 x lO''^ Hz and
o(B. = 10 X 10'^ Hz.

ree
V,
UH

Work function, (})() = hv^ or (|),)«« v 0

F
'0/1 5x!0>^
14
< I or
<!>()A <
10x10

or
ur
Therefore, work function is higher for material B than A.
sf
2 2 Xe 2xl-6xl0-‘-‘^
(//) For metal A, slope = — = or h = = 6*4 X 10-^ Js
ok
14
14
5x10'**
Yo
e (10-5)10 5x10

. 2-5 xe 2-5xl-6xl0'^
Bo

2-5
For metal B, slope = — = or ^ = 77 = = 8 X ir^ Js
e (15-10)10
14 5x10'^* 5x10'**
re

Since, the value of h from experiment for metals A and B is different. Hence, experiment is not consistent
with theory.
ou

28. A particle A with a mass is moving with a velocity v and hits a particle B (mass m^) at rest (one dimen¬
ad

sional motion). Find the change in the de Broglie wavelength of the particle A. TVeat the collision as elastic.
Y

Ans. As collision is elastic, hence laws of conservation of momentum and kinetic energy are obeyed.
According to law of conservation of momentum
nd
Re

ni^ V + nig 0 = + nig V2 or


(v - Uj) = mg V2 ..(/)
According to law of conservation of kinetic energy
Fi

~m.
2 ^
or
m^{v^ ~v\) =
or
(u-n,)(v + u,) = ...(«)

Dividing (//) by (i), we get V +V^ =V2 or


V = V2-V^ ...(i»)

(m^-m B and V f V
Solving (/) and iiu), we gel V V 2 ~
m^+nig m^+nig

h h h{m^ +m^)
^initial X
final
m^v ’

h
-1
^ - Vinal “ ^iniiial “
DUAL NATURE OF RADIATION AND MATTER 11/79

29. Consider a 20 W bulb emitting light of wavelength 5000 A and shinning on a metal surface kept at a
distance 2 m. Assume that the metal surface has work function of 2 eV and that each atom on the
metal surface can be treated as a circular disk of radius 1*5 A.
(0 Estimate no. of photons emitted by the bulb per second. [Assume no other losses] (ii*) Will there be
photoelectric emission ? (iii) How much itme would be required by the atomic disk to receive energy
equal to work function (2 eV) ? (tv) How many photons would atomic disk receive within itme duration
calculated in («i) above ? (v) Can you explain how photoelectric effect was observed instantaneously ?
[Hint: Time calculated in part (Hi) is from classical consideration and you may further the
tai^et of surface area say 1 cm^ and estimate what would happen ?]
Ans. Here, /» = 20 W, X = 5000 A = 5000 x 10"*0 m ; t/= 2 m, <|)o = 2 eV, r = 1-5 A = 1-5 x lO-'^ m.
(0 No. of photon emitted by bulb per second is

n' = P _PX_ 20x(5000x1Q-»Q) »5xl0l*^s-l


(hc/X) he (6-62x10"34)x(3x108)

w
(ii) Energy of the incident photon = — = (6-62x10 ^"*)(3xlQ^) eV = 248 eV

Flo
^ 5000xl0"^‘^x 1.6x10"*^
As this energy is greater than 2 eV (Le., work function of metal surface), hence photoelectric emission

reee
takes place.

FFr
(iii) Let A / be the time spent in getting the energy <i)o (= work function of metal). Refer Fig. 11(N).5,
urr
for
FIGURE 11(N).5

4<|)o</^ 4x(2x1-6x10~^9)x2^ _ 114 s


kkss
or At =
Pj2 20x(1-5x10-'0)2
Yo
ooo

(iv) No. of photons received by atomic disc in time A / is


eB

n'xnr^ n' A r _ (5 X 10‘9) X (1-5 x lO’^O )2 x 114


N = X Ar = = 0-80 »1
4nd^ 4^2 4x(2)2
r
ou
ad

(v) As time of emission of electrons is 11-04 s, hence the photoelectric emission is not instantaneous in this
problem.
YY

In photoelectric emission, there is an elastic collision between incident photon and free electron of the
metal surface, which lasts for very very short interval of time (» 10“® s), hence we say photoelectric emission
nndd
Re

is instantaneous.
Fi
11/80 ’4. Fundamental Physics (XII)

NEET/JEE
SPECIAL

For ultimate preparation of this unit for cprhpetitive examinations, students should refer to

ww
‘i
MCQs In Physics for NEET V
Pradeep's Stellaj^Series? MCQs In Physics for JEE (Main)
'a!
separately availabie for these examinations.

FF loo
a Multiple Choice Questions (with one correct Answer)

r ee
I. Photoelectric Effect lA

reFe
lA

oroFr
1. If and K2 are maximum kinetic energies of
r ur
photoelectrons emitted when light of wavelength
s ff /
A. I and X2 respectively are incident on a metallic
surface. If A.j = 3 X2 then
k
YYouo
(^2 f kr \
koso

o o
(a) > -y (b) «,< Y
BBoo

(JEE Main 2013)


ic) K^=3K2 (d)K2 = ^K^
r ee

4. In a photoemissive cell, with exciting wavelength


2. A and B are two metals with threshold frequencies X, the fastest electron has speed v. If the exciting
1-8 X 10*'* Hz and 2-2 x 10*'* Hz. Two identical
wavelength is changed to 3X/4, the speed of the
ad
ouur

photons of energy 0-825 eV each are incident on fastest electron will be


Yo

them. Then photoelectrons are emitted in


xl/2
(take h = 6-6 X lO"^** J/s) f3
(a) V - (b) V -
(b) A alone
d

(a) B alone
Re
idnY

(c) neither A nor B (d) both A and B 1/2 u/2


(4 ) (4
(Karnataka GET 2007)
FFin

(c) less than v — (d) greater than v —


3. The anode voltage of a photocell is kept fixed. UJ
The wavelength X of the light falling on the (JEE Main 2016, AIIMS 2010)
cathode is gradually changed.The plate current / 5. When light of wavelength 400 nm is incident on
of the photocell varies as follows : the cathode of a photocell, the stopping potential
FieURE 11(CF).1 is 6 V. If the wavelength of incident light is
I /4 increased by 600 nm, the new value of stopping
potential is : [Use he = 1240 eV nm]
(a) 4-97 V (b) 4-76 V
(c) 4-56 V (d) 414 V
6. If the frequency of light in a photoelectric
>X ♦X experiment is doubled the stopping potential will
o o (a) be doubled (b) be halved

1. t/j) 2. (/j) 3. Uh 4. id) 5. (</)


DUAL NATURE OF RADIATION AND MATTER 11/81

(c) become more than double


X (^im) Vo (volt)
id) become less than double.
0-3 2-0
7. Light of wavelength ^ strikes a photo sensitive
04 10
surface and electrons are ejected with kinetic
0-5 04
energy E. If the kinetic energy is to be increased
to 2E, the wavelenegth must be changed to X', Given that c = 3 x 10® ms"’ and e= 1-6 x 10"’^ C.
where Planck’s constant (in units of Js) found from such
(a) V = X/2 ib) X' = 2X an experiment is
-34
{«) 6-0 X 10 (b) 64 X
-34 -34
(C) V>X -<x <x (c) 6-6 X 10 id) 6-8x10
(JEE Advanced 2016)
8. Two identical metal plates show photoelectric 12. The threshold frequency for a certain metal is Vq.
effect by a light of wavelength X^ falls on plate A When light of frequency 2 Vq is incident on it, the

looww
and Xg on plate B. = 2 Xg). The maximum maximum velocity of photoelectrons is
kinetic energy is 4x10^ ms"’. If the frequency of incident radiation
ia)2K^ = Kg ib)K^<Kg/2 is increased by 3 vq, then
-1
the maximum velocity
(c)K^ = 2Kg {d)K^ = Kg/2 of photoelectrons in ms will be

9. A metal surface is illuminated by light of two (fl) (4/5) X 10^ (b)2x lO'^

ree
different wavelengths 248 nm and 310 nm. The (c)4x 10^ (i/)8x 10^
maximum speeds of the photoelectrons 13. The work function of a surface of a photo

eeF
corresponding to these wavelengths are z/j and
ZZ2 respectively. If the ratio zzj : z^2 = 2 : 1 and
r FF sensitive material is 6-2 eV. The wavelength of
the incident radiation for which the stopping

roFrr
he = 1240 eW nm, the work function of material potential is 5 V lies in the
is nearly (a) infrared region (b) X-ray region
s fof
u
(a) 3-7 eW (b) 3-2 eV (c) ultraviolet region (d) visible region
{AIPMT 2008)
k
(c) 2-8 eW (d) 2-5 eV
14. Photoelectric effect experiments are performed
kos o
YYouor
(JEE Advanced 2014)
using three different metal plates p, and r having
10. A metal plate is exposed to light with wavelength
work function, <!^p = 2-0 eV, = 2-5 eV and
BBoo

X. It is observed that electrons are ejected from


r ee

0^ = 3-0 eV respectively. A light beam containing


the surface of the metal plate. When a retarding
uniform electric field E is imposed, no electron wavelengths of 550 nm, 450 nm and 350 nm with
equal intensities illuminates each of the plates.
ad

can move away from the plate farther than a


ouur

The correct l-V graph for the experiment is


certain distance d. Then the threshold wavelength
Yo

(Take /tc=1240eV nm)


Xq for the material of plate is {e is the electronic
charge, h is Planck’s constant and c is the speed FIGURE 11(CF).2
d
Re

of light)
inY

p
FFind

Xe Ed Xhc
(a) (b) y

(e Ed - he) hc-eEdX /V' p,


/ ‘I,

eEdX-hc Xhc —e Ed ■►V


(c) id)
heX heX o o
U
n. In a historical experiment to determine Planck’s
constant, a metal surface was irradiated with light - — r

/ — q
of different wavelengths. The emitted photo- -P
electron energies were measured by applying a // / r '

/.y.
stopping potential. The relevant data for the
wavelength (X) of incident light and the
/// ■►V ●►V
corresponding stopping potential (Vq) are given o o
below in table.
(IIT 2009)
tmm..
6. (c) l.id) 8. (/?) 9. (a) 10, (/>) 11. (/;) 11. (d) 13. (<■) 14. It
11/82 "Pnadee^'4. Fundamental Physics (XlI)B2sMI]
15. Surface of a certain metal is first illuminated with stopping potential becomes Wand respec
light of wavelength Xj = 350 nm and then the tively. The threshold frequency for this metal is
light of wavelength X2 = 540 nm. It is found that (a) 2 V
the maximum speed of the photoelectrons in the (c) 2 v/3 id) 3 v/2 (NEET 2022}
two cases differ by a factor of 2. The work
function of the metal (in eV) is close to : (Energy
21. Let AT] and K2 be the maximum kinetic energies
of photoelectrons emitted when two
1240 monochromatic beams of wavelength and ^
of photon = eV)
X (in nm) respectively are incident on a metallic surface. If
(a) 1-8 ib) 2-5 = 3 X2, then
(c) 5-6 (d) 1-4 (JEE Main 2019) ^2 ib) K.1 <
^2
16. When the energy of the incident radiation is (^) ^ 3
increased by 20%, the kinetic energy of the ^2 K

ww
photoelectrons emitted from a metal surface (c) K, = id) =
2 3
increased from 0-5 eV to 0-8 eV. The work
function of the metal is (JEE Main 2022)

Floo
(a) 0-65 eV ib) 1-0 eV 22. The electric field at the point associated with a

(c) 1-3 eV id) 1-5 eV light wave is given by

ee
(AIPMT 2014) £■=200 [sin (6 x 10*^) r + sin (9 x 10*^) t] Vm"'.
Given, h = 4-14 x 10"*^ eVs. If this light falls on

eer
17. Light described at a place by the equation £ =
(100 V/m) [sin (6x 10*^s“*)/ + sin (8x 10^^ j”*)/] a metal surface having a work function of 2-50 eV,

FFr
the maximum kinetic energy of the photoelectrons
falls on a metal surface having work function

oorr
uur r
will be :
2-28 eV. The maximum energy of the photo
(a) 1-90 eV ib) 3-27 eV
s ff
electrons is : (use h = 6-63 x 10"^“^ Js) id) 342 eV
(c) 3-60 eV
(a) 2-28 eV ib) 3-0 eV (JEE Main 2022)
sk
YYoo
(c) 1-24 eV id) 1-50 eV
ooko

23. The stopping potential for electrons emitted from


18. The magnetic field associated with a light wave a photosensitive surface illuminated by light of
eBB

is given, at the origin, by wavelength 491 nm is 0-710 V. When the incident


B = Bq [sin (3-14 x 10^) ct + sin (6-28 x 10^) c/] wavelength is changed to a new value, the
If this light falls on a silver plate having a work stopping potential is 1-43 V. The new wavelength
uurr

function of 4-7 eV, what will be the maximum is:


ad

kinetic energy of the photoelectrons ? (a) 400 nm ib) 382 nm


Yo

(/j = 6-6x 10-^'^Js) (c) 309 nm id) 329 nm


(a) 6-82 eV (b) 12-5 eV (JEE Main 2021)
dY
Re

(c) 8-52 eV id) 111 eV 24. In photoelectric effect stopping potential is 3 Vq


ind

(JEE Main 2019) for incident wavelength Xg ^nd stopping potential


FFin

19. The electric field of light is given by is Vq for incident wavelength 2 Xq. Find threshold
wavelength.
Inx
£ = 10 ^ cos j-lKx6xl0^*t (fl)3Xg ib)lXQ
5x10-
(c)4Xq (d)SXQ
This light falls on a metal plate of work function (JEE Main 2021)
2 eV. The stopping potential of the photoelectrons 25. Light of frequency 1-5 times the threshold
IS:
frequency is incident on a photosensitive material.
ia) 0-48 V ib) 2-48 V What will be the photoelectric current if the
(c) 0-72 V id) 2 0 V frequency is halved and intensity is doubled ?
(JEE Main 2019) (a) four times ib) one-fourth
20. When two monochromatic lights of frequency v (c) zero id) doubled
and v/2 are incident on a photoelectric metal, their (NEET 2020)
ANSWERS

15. (a) 16. (d) 17.(6) 18. (tO 19. (a) 20. id) 21.(6) 22. id) 23.(6)
24. (c) 25. (c)
DUAL NATURE OF RADIATION AND MATTER 11/83

n. Photons Use, he = 1240 eV nm.


(a) l-!l X 10-^ N (b) 2-22 X 10"^ N
26. Two sources of light emit X-rays of wavelength (c)3-33x 10"® N id) 6-66 X 10-* N
1 nm and visible light of wavelength 500 nm,
32. A beam of light of wavelength 400 nm and power
respectively. Both the sources emit light of the 1-55 mW is directed at the cathode of a
same power 200 W. The ratio of the number
photoelectric cell. If only 10% of the incident
density of photons of X-rays to the number density
photons effectively produce pholoelectron, then
of photons of the visible light of the given find current due to these electrons,
wavelengths is
(given, he = 1240 eV-nm, e = 1-6 x 10~'^ C)
1
(a) (b) (a) 5 pA (b) 40 pA
500 250
(c) 50 pA (d) 114 pA
(c) 500 id) 250
(AIIMS 2015)

ww
(JEE Main 2020)
33. An electron beam is accelerated by a potential
27. The number of photons per second on an average difference V to hit a metallic target to produce
emitted by the source of monochromatic light of X-rays. It produces continuous as well as
wavelength 600 nm, when it delivers the power

Flo
characteristic X-rays. If is the smallest
of 3-3 X 10“^ watt will be {/i = 6-6 x Js) possible wavelength of X-ray in the spectrum, the

e
15
ib) 10'^ variation of log with log V is correctly

e
ia) 10

reer
(c) lOl’^ id) 10
18
(NEET 2021) represented in

rFF
28. Ultra violet light of wavelength 300 nm and
intensity 10 watt/m^ falls on the surface of a
uur r
photo sensitive material. If one percent of the
incident photon produce photoelectron then the
ffoor
sks
number of photoelectrons emitted per second
YYoo
from an area 1-0 cm^ of the surface is nearly.
ooko

ih = 6 6x lO-'^'^Js).
eBB

II 12
(r7) 2-13 X 10 ib) 1-51 X 10
(c) 4-12 X 10'3 id) 9-61 X 10
14

29. We may state that the energy £ of a photon of


uurr

frequency v is £ = hv, where h is a Planck’s


ad
Yo

constant. The momentum p of a photon isp = h/X


where X is the wavelength of the photon. From
dY

the above statement one may conclude that the


Re

wave velocity of light is equal to


innd

ia) 3 X 10*^ m/s ib) E/p


FFi

(JEE Main 2017)


(c) £p id) (£/p)2
30. A sodium lamp emits 3-14 x 10^® photons per 111. de-Broglic hypothesis
second. Calculate the distance from sodium lamp
34. An electron is accelerated through a potential
where flux of photon is one photon per second
per cm^. difference of 10,000 V. Its de-Broglie wavelength
ia) lO'O cm ib) 5 X 10^ cm is (nearly): (m^ = 9 x 10“^* kg)
13 -12
(c) 5 X 10® cm (d) 10^ cm ia) 12-2 X I(T m ib) 12-2 X 10 m
-14
(c) 12-2 X 10 m id) 12-2 nm
31. A parallel beam of monochromatic light of
wavelength 500 nm is incident normally on a (NEET 2019)
perfectly absorbing surface. The power through 35. The energy that should be added to an electron, to
any cross-section of the beam is 10 W. The force reduce its de-Broglie wavelength from 2 x 10"^^ m
exerted by the light beam on the surface is ; to 0-5 X 10“^ m will be
ANSWERS

26, ia) 27. ib) 28. ib) 29. ib) 30. ib) 31. (c) 32. (c) 33. (c) 34. ib)
11/84 “PKuUeft 'a Fundamental Physics (XII) tviwii
(a) M MeV {h) 0-56 MeV
(c) 0-56 KeV {d) 5-6 eV (fl) ^ ih)
X... -i- X.
36. The ratio between masses of two particles is 1 : 2 y

and ratio between their temperatures is also 1 : 2.


The ratio between their de-Broglie wavelengths (c)
is:

(a) 1:2 (b)2:l (JEE Main 2019)


{c) 1:3 (J)3;l 43. The moving proton and a particle are subjected
37. For Bragg’s diffraction by a crystal to occur, then to the same magnetic field so that the radii of their
the X-ray of wavelength X and interatomic paths are equal to each other. Assuming the field
distance d must be
induction B is perpendicular to the velocity
(a) X is greater than 2d vector of the a-particle and proton, the ratio of

llow ow
(b) X equals 2d de-Broglie wavelength of a- particle to that of
(c) X is smaller than or equal to 2d proton is
(d) X is smaller than 2d (AIIMS 2015) (a) 1/4 (/i) 1/2
38. The de-Broglie wavelength associated with proton (c)l (d)2

ree
changes by 0-25% if its momentum is changed 44. The ratio of de-Broglie wavelength of molecules
by Pq. The initial momentum was of hydrogen and helium in two gas jars kept

rF
(a) 100;jo (b) Pq/400 separately at temperatures of 27“C and 127°C
(c) 401/10 (d) Pq/100
39. The de-Broglie wavelength of a particle moving
r FF respectively is
(«) 2/SforFe fo {b) 2/3
with a velocity 2-25 x 10* ms“' is equal to the
ou
wavelength of photon. The ratio of kinetic energy (c) V3/8 (i/) VsTI
ks
of the particle to the energy of the photon is 45. A particle A of mass m and initial velocity v
(velocity of light is 3 x 10* ms"')
oo

collides with a particle B of mass m/2 which is at


rest. The collision is head on, and elastic. The
YourY

(a) 1/8 {b) 3/8


B

(c) 5/8 id) 7/8 ratio of the de-Broglie wavelengths X^ to X^ after


r eBokse

the collision is
40. The energy of a photon is equal to the K.E. of a
proton. The energy of the photon is E. Let Xj be
ou

{d) -
ad

the de-Broglie wavelength of the proton and X2 Xs 3 B

be the wavelength of the photon. The ratio Xj/X2


Your

= 2
is proportional to
B
(a)EP 1/2
d

ib)E
Re

(JEE Main 2017)


in

(c) (J)F-2
46. A free particle with initial kinetic energy £,
41. An electron microscope, the resolution that can
FindYF

de-Broglie wavelength X, enters a region where


be achieved is of the order of the wavelength of
in it has a potential energy V, what is the new
electron used. To resolve a width of 7-5 x 10“^^ m,
de-Broglie wavelength ?
the minimum electron energy required is close to :
(a)X(l -V/E) (b)X{\ +E/V)
(a) 500 keV (£>) 100 keV 1/2
(c) X/(I - V/E) (d))J{l + V/£)2
(c) 1 keV id) 25 keV
(JEE Main 2019) 47. Xg, Xp and X„ are the de-Broglie wavelengths of
electron, proton and a particle. If all are
42. A particle P is formed due to a completely inelastic accelerated by same potential, then
collision of particles x and y having de-Broglie
wavelength X^ and X^ respectively. Ifx and y were (a) Xg<Xp<X a (b) Xg < Xp > X a

moving in opposite directions, then the de-Broglie (c) X^>Xp< ,X a id) Xg = Xp > ,X a

wavelength of F is : (e) X^>Xp> ,X a (Kerala PMT 2009)

35. id) 36.ih) 37. (ri 38.(0 39. (/» 40. i/" 41. (^/i 42. (o 43. '
45. id) 46. I 47. (.'●)
DUAL NATURE OF RADIATION AND MATTER 11/85

48. Two particles move at right angles to each other. 52. Light of wavelength falls on a cathode plate
Their de-Broglie wavelengths are Xj and X2 inside a vacuum tube as shown in Fig. 11(CF).5.
respectively. The particles suffer perfectly The work function of the cathode surface is <[) and
inelastic collision. The de-Broglie wavelength X, the anode is a wire mesh of conducting material
of the final particle, is given by : kept at a distance d from the cathode. A potential
difference Vis maintained between the electrodes.
^1 ^2
(fl) X = ^X^Xj ib)X =
2
If the minimum de-Broglie wavelength of the
electrons passing through the anode is which
2 1 1 , . 1 i i of the following the statement(s) is (are) true ?
1 ^2
FIGURE 11(CF).5
(JEE Main 2019)

ooww
49. Fig. ll(CF).4representsagraphofkineticenergy K'

of most energetic photoelectrons


frequency v for a metal used as cathode in photo
electric experiment. The threshold frequency of
light for the photoelectric emission from the

e
metal is

ere
rFl
FIGURE 11(CF).4

Fre
V
-I
^max. (®^)

rrF
3-- (a) For large potential difference (V » <^/e), X^

sffoo
ouur
is approximately halved if V is made four
2-
times
kosk
1- (b) X^ decreases with increase in (|) and
Yo
(c) Xg increases at the same rate as Ap^ for
oo

0
Y

10''® Hz ?.ph < hc/<?


BB

(d) Xg approximately halved, if d is doubled


(JEE Advanced 2016)
rre

(a) 1 X 10^^ Hz {b) 1-5 X 10


14
Hz 53. An electron of mass m with an initial velocity
ouu
Y

(c) 2-1 X lO*'^ Hz


—> A
(d) 2-7 X 10**^ Hz
ad

V = Vq j (Vq > 0) enters an electric field


(AMU Engineering 2009)
dY

50. After absorbing a slowly moving neutron of mass E = -Eq i (Eq - constant > 0) at f = 0. If Aq is
(momentum - 0) a nucleus of mass M breaks
innd

its de-Broglie wavelength initially, then its de-


Re

into two nuclei of masses m| and 5 mj (6 mj = A/


Broglie wavelength at time t is
+ m^), respectively. If the de Broglie wavelength
Fi
F

of the nucleus with mass mj is A., then de Broglie eE.0


wavelength of the other nucleus will be (fl) A^ t (.b) Aq 1 -I- mV.
(a) 25 X (b) 5 X 0

(c) XJ5 (d)X (AIEEE 2011) X0


(c) 7 (d) Xq (NEET 2018)
51. If the momentum of electron is changed by P, then eE0
1 +
the de Broglie wavelength associated with it mV.0
changes by 0-5%. The initial momentum of
electron will be :
54. The de-Broglie wavelength of an electron moving
(a) 200 P (/?) 400 P with kinetic energy of 144 eV is nearly :
(a) 102 X 10-2 102 X 10-3
(c) (d) loop
200 (c) 102 X 10^ nm (d) 102 x IQ-^ nm
[AIPMT (Main) 2012] (NEET 2020)

48. (J) 49. (</) 50. {£/) 52.00 53. (f) 54. t/ji

»
11/86 ‘^t4xdecfr'4- Fundamental Physics fXinPZSTWTl
55. An electromagnetic wave of wavelength 'V is 57. When photons of energy 4 eV strikes the surface
incident on a photosensitive surface of negligible of a metal A, the ejected photoelectrons have the
work function. If ‘m’ mass is of photoelectron maximum kinetic energy eV and de-Broglie
emitted from the surface, has de-Broglie
wavelength The maximum kinetic energy of
wavelength then
photoelectrons liberated from another metal B by
2h 2 m
(«) ^ = ib) X = photon of energy 4-50 eV is; 7^ = (7^ - 1 -5) eV.
\mc ^ he If the de-Broglie wavelength of these
' 2 me' ' 2mc\ photoelectrons, Xg = 2 then the work function
(c) Xj = {d) X =
, ft , I h of metal B is
(NEET 2021)
(a) 3eV (b) 1-5 eV

ooww
56. An electron (of mass m) and a photon have the
(c) 2cV (d) 4eV
same energy E in the range of few eV. The ratio
of the de-Broglie wavelength associated with the (JEE Main 2020)
electron and the wavelength of the photon is : 58. Kinetic energy of the particle is E and its
(c = speed of light in vacuum).
de-Broglie wavelength is X. On increasing its

e
y]/2 1
kinetic energy A£, its new de-Broglie wavelength

ere
( E \(2E\
(«) ib) E

rFl
2 m becomes X/2. Then A£ is
c m

Fre
1 (a) 3E (b) E

rrF
(c) e(^2mE) id) -1/ £ ^ (c) 2E id) 4 E
1/2
c 2m
(JEE Main 2020)

sffoo
ouur
(JEE Main 2020)

m■'Multiple
kosk
Choice Questions (with One or More than One Correct Answers)
Yo
oo

59. The photo electric effect with incident photon


Y

FIGURE 11(CF).7
BB

wavelength X. the stopping potential is Vq- The


correct variation (^) of Vq with X and l/X are :
rre

FIGURE 11(CF).6
ouu
Y
ad

Vo Vo
O
0|
dY

(a) planck’s constant (b) charge of an electron


innd
Re

X X
(c) threshold frequency
Fi
F

Vo Vq (d) work function of cathode metal


61. Choose the incorrect statement;
o 1 ©T
(a) The velocity of photoelectrons is directly
proportional to the square root of wavelength
1/X 1/X of light
(b) The number of photoelectrons emitted depends
(JEE Advanced 2015) upon the intensity of incident light
60. The maximum kinetic energy (£^ of the emitted (c) The velocity of photoelectrons is directly pro
phoioelectrons against frequency v of the incident portional to the frequency of the incident light
radiation is plotted as shown in Fig. 11(CF).7. This id) The velocity of the photoelectrons is inversely
graph helps us in determination of following proportional to square root of the frequency
quantities. of the light.
ANSWERS
55. (d) 56. id) 57. (d) 58. (n) 59. (a.c) 60. ia,c,d) 61. ia,c,d)

%
DUAL NATURE OF RADIATION AND MATTER 11/87

62. The maximum K.E. of photoelectrons ejected 64. When a monochromatic point source of light is
from a photometer when it is irradiated with at a distance of 0-2 m from a photoelectric cell,
radiation of wavelength 400 nm is 1 eV. If the the cut off voltage and the saturation current are
threshold energy of the surface is 1-9 eV, respectively 0-6 V and 18-0 mA. If the same
{a) the maximum K.E. of photoelectrons when it source is placed 0-6 m away from the photo
is irradiated with 500 nm photons will be electric cell, then
0-42 eV (a) the stopping potential will be 0-2 volt
(b) the maximum K.E. of photoelectrons when it (/?) the stopping potential will be 0-6 volt
is irradiated with 5000 nm photons will be (c) the saturation current will be 6-0 mA
0-42 eV
(d) the saturation current will be 2 0 mA.
(c) maximum K.E. will increase if the intensity 65. Which of the following characteristics of photo

ooww
of radiation is increased
electric effect supports the particle nature of
(d) the longest wavelength which will eject the radiations
photoelectron from the surface is nearly (a) threshold frequency
610 nm
(b) instantaneous photoelectric emission

e
63. The graph between 1/^ and stopping potential V
(c) independence of the velocity of photo

ere
of three metals having work functions <{)j, ^2
electrons on intensity of radiations

rFl
03 in an experiment of photoelectric effect is

Fre
plotted as shown in the Fig. 11(CF).8. Which of (d) dependence of the velocity of photoelectrons
on frequency.

rrF
the following statement(s) is/are correct ? [Here
X is the wavelength of the incident ray] 66. The frequency and intensity of a light source are
both doubled. Which of the following
sffoo
ouur
statement (i) is/are true ?
kosk
(a) The saturation photocurrent gets doubled
(b) The saturation photocurrent remains almost
Yo
oo

the same
Y
BB

(c) The maximum K.E. of the photoelectrons is


more than doubled
rre

(d) The maximum K.E. of the photoelectrons gets


ouu

doubled.
Y
ad

67. If V is frequency, X is the wavelength and v is


dY

the wave number then the energy of a photon can


be represented by
innd
Re

(a) hv (b) he V
Fi
F

(a) Ratio of work functions ; (c) he X (d) hc/X.


01: 02'03 = 1 = 2 : 4 68. In which of the following situations the heavier
(b) Ratio of work functions ; of the two particles has smaller de-Broglie
wavelength ? The two particles
01: 02 : 03 — 4 : 2 : 1
(a) move with same speed
(c) tan 0 is directly proportional to hele where h
is Planck’s constant and e the speed of light (b) move with the same kinetic energy
(c) move with the same linear momentum
{d) The violet colour light can eject photoelectrons
from metals 2 and 3. (IIT) (d) have fallen through the same height.

ANSWERS

62. (a.d) 63. (a,c) 64. {b.d) 65. (a,b,c,d) 66. (a,c) 67. ia.b.d) 68. (a,b,d)

I
11/88 Fundamental Physics (XII)EEIHn

mi Multiple Choice Questions (Based on the given Passage/Comprehens ion)


Each cumprchension given beluw is followed by some multiple choice questions. Each question has one
correct option. Choose the correct option.

fejompr.efi^r^ionlC An experimental set up


Answer the following questions
69. When radiation of suitable wavelength falls on
for the photoelectric effect is shown in Fig. the cathode C, a current of 2 pA is recorded in
11{CF).9. the ammeter. Assuming that the vacuum tube
arrangement follows Ohm’s law, the equivalent
FIGURE 11(CF).9
resistance of vacuum tube when jockey is at P
end is

ooww
(a) lOHi (b)
A (c)4x 10'^ n (</) 8 X 10^ n
70. It is found that ammeter current remains
unchanged (2 pA) when the jockey is moved from

e
the end P to the mid point of rheostat. Assuming

re e
all the incident photons eject electrons and the

rFl power of the incident light is 4-3 x 10“^ W, then

Fre
P [—■'NA/WVSA; Q
the colour of the incident light is
4 Q. 50 cm

rrF
(a) green (b) violet
(c) red (d) yellow
10 V 1

sffoo
ouur
71. When other light falls on the plate C, the ammeter
Here, the voltage across the electrodes (A and reading remains zero till jockey is moved from
oksk
C) is measured with the help of ideal the end P to the middle point of the wire PQ.
Thereafter the deflection is recorded in the
voltmeter V. The voltage across the electrodes
Yo
ammeter. The maximum kinetic energy of the
oo

can be varied by moving jockey J on the


Y

emitted electron is
BB

rheostat working as potential dividing


arrangement. The battery used in potential (fl)2eV (/>)4eV (c)8eV (f/)10eV
rre

dividing arrangement is 10 V and its internal


resistance is 1 Q. The resistance of rheostat
^jbrhPTeh'erisiQh ^
ouu

wire is 4 n and its length is 50 cm. The two Wave properties of elec
Y
ad

electrodes in vacuum tube are plates of trons implies that they will show diffraction
dY

effects. Davisson and Germer demonstrated


platinum. The plate C is coated with
potassium oxide. Each plate is of area 50 cm^ this by diffracting electrons from crystals.
The law governing the diffraction from a
innd

at separation of 0*5 mm. When radiations of


Re

suitable frequency or wavelength fall on plate crystal is obtained by requiring that electron
Fi

C, the photoelectric current is set up which waves reflected from the planes of atoms in
F

is measured with the help of ideal ammeter. a crystal interfere constructively as shown
The photoelectric current in circuit is very in Fig. 11(CF).10.
small so we can treat potential dividing
circuit an independent circuit.
The wavelength of various colours is as
follows :

Light Violet Blue Green Yellow Orange Red

X in 4000- 4500- 5000- 5500- 6000- 6600-


A“ 4500 5000 5500 6000 6600 7800

c
TTTTTTT

69. <r) 70. (</) n.(b)


DUAL NATURE OF RADIATION AND MATTER 11/89

Answer the following questions function of the detector position (y = 0


72. Electrons accelerated by potential V are diff-racted corresponds to the middle of the slit) ?
from a crystal. If d = 1 A and i = 30°, V should be FIGURE 7(CF).11
about (fi = 6-6 X lO"^*^ Js, t-

m^=9-l X 10-3'kg, e= j.gx 10"'^C)


(a) 1000 V ib) 2000 V
(c) 50 V (d) 500 V
i
73. What is the glancing angle in the above -d jv = 0
experiment ?
(a) 30° ib) 15°
(c) 45° (d) 60° 'A
D
74. If a strong diffraction peak is observed when
electrons are incident at an angle i from the normal

ww
to the crystal planes with distance d between them, FIGURE 11(CF).12
de-Broglie wavelength of electrons can be ●y

calculated by the relationship (n is an integer)

Floo
(a) d cos i = n ^dB (b) d sin i = n X^g N- d N d
(c) 2 d cos / = n (d) 2 d sin i = n X^g

e
© o

eere
75. In an experiment electrons are made to pass
through a narrow slit of width d comparable to ●y y

FFr
their de-Broglie wavelength. They are detected
on a screen at a distance D from the slit. Which

oorr
uur r
H d N-*- d
of the following graphs can be expected to
represent the number of electrons N detected as a
s ff
G ©
sk
m
YYoo
Matching Type Questions
ooko
eBB

DljRECTIONS. In each of the following questions, match column I and column II and select the correct
match out of the four given choices.
76. Column I Column II
uurr

(A) Planck’s theory of quanta ip) Light energy = hv


ad
Yo

(B) Einstein’s theory of quanta (?) Angular momentum of electron in an orbit


(C) Bohr’s stationary orbit (r) Oscillator energies
dY

(D) de-Broglie waves is) Electron microscope


Re

(a) A-p ; B-^ ; C-r ; D-5 (b) A-q ; B-r ; C-s ; D-p (c) A-r ; B-p ; C-q ; D-s (d) A-r ; B-;>; C-,s; D-^
nind
FFi

77. Column I Column II


(A) Particle nature of light ip) Davisson and Germer

(B) Wave nature of light (?) G.P. Thomson

(C) Wave nature of slow moving electrons ir) Max. Planck

(D) Wave nature of fast moving electrons is) Huygen


{a) A-p ; B-^ ; C-r ; D-i (b) A-q ; B-r ; C-s ; D-p (c) A-r ; B-i-; C-p ; D-^ (d) A-s ; B-r ; C-q ; D-p

78. Match List I (Fundamental Experiment) with List II (its conclusion) and select the correct option
from the choices given below the list:
List 1 List II

(A) Franck-Hertz experiment (0 Particle nature of light

72. (c) 73. (d) 74. {(■) 75. (^;) 76. (r) 77. (c)

I
11/90 'P'uuUefr’^ Fundamental Physics (XII)EEIHIl

(B) Photo electric experiment («) Discrete energy levels of atom


(C) Davison-Geimer experiment {Hi) Wave nature of electron
(iv) Structure of atom
(a) (A)-(O ; (B)-(/v); (C)-(m) ib) (A)-(«); (B)-(/v); (C)-(/»)
(c) (A)-(ii) ; (B)-(O ; (C)-(//0 UD (A)-(/v) ; (B)-(//0 : iO-di) (JEE Main 2015)

p q
'"ii
Matrix*Match Type Questioni:
A ©1010;©
DIRECTIONS. Each of the following questions contains statements given
in two columns, which have to be matched. The answers to these questions
B
© |©|!©l ©
have to be appropriately bubbled. If the correct matches are A-p, A-q ; C ®j®||©|i^

ww
B-r ; C-r, C-i ; and D-s, then the correctly bubbled matrix will look
like the one shown here :
D
© ©I Oil©
79. In photoelectric effect experiment match the statement of column I with that of column II :

Flo
Column I Coiunm II

e
(A) When intensity of incident light is increased on ip) The value of photo- electric current

eree
photocathode without changing its frequency. increases.

FFr
(B) When the frequency of incident light is increased {q) The value of stopping potential increases.
on photocathode without changing its intensity.

oorr
uur r
(C) When the wavelength of incident light is (r) The value of stopping potential does not
sf
decreased without changing its intensity. change,
(D) When the potential of anode is increased without (i) The value of photo-electric current does
sk
Yoo
changing the frequency and intensity of the not change.
oook

incident light.
eBB

80. Match the statements of column I with that of column


n, where Xp are the de-Broglie
wavelengths of a-particle and of proton, Pp are the momentum of a-particle and of proton
respectively.
uurr

Column I Column 11
ad

(A) a-particle and proton have same K.E. (p) Xp = ,X


Yo

a A B C D
(B) a-particle has one quarter K.E. than that of proton, iq) Xp>X,
® © ® ®
dY

(C) a-particle has four times K.E. than that of proton ('■) Pp=Pa
Re

©® ® ©
innd

(D) a-particle and proton has same velocity (^) Pp < Pa


FFi

® ® ® ®
VI. Integer Type Questions
® ® ® ®
DIRECTIONS. The answer to each of the following questions is a single digit ®) ®) ®) (®
integer, ranging from 0 to 9. If the correct answers to the question numbers A, B,
C and D (say) are 4, 0, 9 and 2 respectively, then the correct darkening of bubbles ® ® © ®
should be as shown on the side :
®® ® ®
81. A silver sphere of radius 1 cm and work function 4.7 eV is suspended from an insulating
thread in free-space. It is under continuous illumination of 200 nm wavelength light.
As photoelectrons are emitted, the sphere gels charged and acquires a potential. (£) (£) (£) (®
The maximum number of photo-electrons emitted from the sphere is A x 10^ (where
I < A < 10). The value of Z is (IIT 2011)
®®®®
ANSWERS
78.(c) 79. (A-p.r ; B-q.s ; C-q.s : D-.rs) 80. (A-q,s : B-p.r : C-q,.s : D-q.s) 81.(7)

\
DUAL NATURE OF RADIATION AND MATTER 11/91

82. The work function of silver and sodium are 4-6 and
2-3 eV respectively. The ratio of slope of the stopping
potential versus frequency plot for silver to that of sodium is ; (JEE Advanced 2013)
83. A 25 watt bulb, which is producing monochromatic light of wavelength 6600 A is used to illuminate a metal
surface. If the surface has 3% efficiency for photoelectric effect, then the photoelectric current produced in
deci ampere is (use /i = 6-6 x 10"^“^ Js) :
84. A parallel beam of monochromatic light of wavelength 660 nm is incident on a totally reflecting plane
mirror. The angle of incident is 60° and the number of photons striking the mirror per second is 2-0 x 10*®.
The force exerted by the light beam on the mirror is a x 10“^ N. What is the integer value of a ? Given
/i = 6-6 X 10-^4
85. A proton is fired from very far away towards a nucleus with charge Q~ 120 e, where e is the electronic
charge. It makes a closest approach of 10 fm to the nucleus. The de Broglie wavelengtli (in units of fm) of
the proton at its start is : (take the proton mass, = (5/3) x 10“^^ kg ; hie = 4-2 x 10”’^ J.s/C ;

ww
I

4 Tie
= 9 X 10® m/F ; 1 fm = 10 m) (IIT 2012)
0

Flo
e
86. What is the energy in eV that should be added to an electron of energy 2 eV to reduce its de-Broglie wavelength

e
from 1 nm to 0-5 nm ?

reer
rFF
VII.
Assertion-Reason Type Questions
uur r
FOR MEDICAL STUDENTS
ffoor
89. Assertion. An electron microscope can achieve
sks
DIRECTIONS. In each of the following questions, better resolving power than an optical
YYoo
microscope.
ooko

read the two statements and choo.se if


(A) both Assertion and Reason are true and the Reason Reason. The de-Broglie wavelength of the
eBB

is correct explanation of the Assertion. electrons emitted from an electron gun with
(B) both Assertion and Reason are true, but the Reason velocity 500 m/s is much less than 500 nm.
uurr

is not a correct explanation of the Assertion. («)A {b)B (c)C (d)D


ad

(C) Assertion is true and Reason is false.


90. Assertion. The de-Broglie wavelength equation
Yo

(D) both. Assertion and Reason are false. has significance for any microscopic or
87. Assertion. Light of frequency L5 times the submicroscopic particles.
dY

threshold frequency is incident on photo-sensitive Reason. The de-Broglie wavelength is inversely


Re
innd

material. If the frequency is halved and intensity proportional to the mass of the object if velocity
is doubled, the photo current remains unchanged. is constant.
FFi

Reason. The photo electric current varies directly (a) A (/j)B (c)C (d)D
with the intensity of light and frequency of light. 91. Assertion. A particle of mass M at rest decays
(a) A {b)B (c)C id)D into two particles of masses mj and m^, having
88, Assertion. Photoelectric effect demonstrates the non-zero velocities will have ratio of de-Broglei
wave nature of light. wavelengths unity.
Reason. The number of photoelectrons is Reason. Here we cannot apply conservation of
proportional to the frequency of light. linear momentum.

(a) A ib)E (c)C id)D (a) A {b)B (c)C id)D


(AIIMS 2004) (AIIMS 2015)

ANSWERS

82.(1) 83.(4) 84.(2) 85.(7) 86.(6) 87. (</) 88. (</) 89. (/>) 90. (a) 91. (c)
11/92 ‘P'iadeefr'4^ Fundamental Physics fXIIirasiim
FOR ENGINEERING STUDENTS ^max- When the ultraviolet light is replaced by
DIRECTIONS. In each of the following questions X-rays, both Vq and increase.
read the two statements and choose if
Statement-2. Photoelectrons are emitted with

(A) Statement-1 is true ; Statement-2 is true ; State


speeds ranging from zero to a maximum value
because of the range of frequencies present in the
ment-2 is a correct explanation of Statement-1.
incident light,
(B) Statement-1 is true ; Statement-2 is true : State
(fl) A (h) B (c) C ((/) D
ment-2 is not a correct explanation of Statement-1.
(AIEEE 2010)
(C) Statement-1 is true ; Statement-2 is false.
(D) Statement-1 is false ; Statemcnt-2 is true.
95. Statement-1. A small metal ball is suspended in
a uniform electric field with an insulated thread.
92. Statement-1. The photoelectrons produced by a If high energy X-rays beam falls on the ball, the
ball will be deflected in the electric field.
monochromatic light beam incident on a metal
surface have a spread in their kinetic energies. Statemcnt-2. X-rays emit photoelectrons and

w
Statement-2. The work function of the metal metal becomes negatively charged.
varies as a function of depth from the surface. (fl)A (b)B (c)C (d)D
96. Statement-1. An electron of mass m when
(a) A {b)B (c)C (d)D

Flo
93. accelerated through a potential difference V, has
Statement-1. A metallic surface is irradiated by a
de-Broglie wavelength X. The de-Broglie

reeee
monochromatic light of frequency v > Vq (the wavelength associated with a proton of mass M
threshold frequency). The maximum kinetic
accelerated through the same potential difference

FFr
energy and the stopping potential are and
Vq respectively. If the frequency of incident light will be X Vm7 m ●

on the surface is doubled, both the and ,V0

for
Statement-2. de-Broglie wavelength
ur
arc also doubled.
h
Statement-2. The maximum kinetic energy and X =
pm eV '
kkss
the stopping potential of photoelectrons emitted
Yo
from a surface are linearly dependent on the (a) A (b)B (c)C (^OD
oo

frequency of incident light. 97. Statement-1. Davisson-Germer experiment


eB

(a) A {b)B (c)C (d)D established the wave nature of electrons

(AIEEE 2011) Statement-2. If electrons have wave nature, they


can interfere and show diffraction,
94. Statement-1. When ultraviolet light is incident
r

(a) A (b)B (c)C (d)D


ou

on a photocell, its stopping potential is Vq and


ad

the maximumkinetic energy of photoelectronsis (AIEEE 2012)


YY
ndd
Re
Fi

For Difficult Questions

Multiple Choice Questions (with one correct Answer)

he 1 _ 1
1. a:, ...(f) .-. K^-K^= he
X1 ^1 ^2
he 1 -2 he
and
^2=^-^0 = hc
X, 3^2 ^2 2X^
he
or
0 ...(«) ^1-^2=- I (^2 ... from («)

riNSWERS
92.(u) 93.(rf) 94.(r) 95. (r) 96.(d) 97.in)
DUAL NATURE OF RADIATION AND MATTER 11/93

From (/) and (//)


For Difficult Questions
6-V'= 3-10-1-24= 1-86
V' = 6- 1-86 = 4*14 V
2 2

^1 = ^2-3^2-J^0 = 3 3
’0 6. eV^ = hv - <{)o

^2 and eV^ = /i x 2u - (J)q = 2 ((●'V'q + (Jiq) - $0


K. <
1 3
Vo=2Vo+V^
hv
0 thus the stopping potential of the photoelectrons
2- ^
eV
will be slightly more than double.
(6-6x10-34)x{1-8x10‘‘^) = 0-74 eV
he he
7. E and 2E These
1-6x10~’9 X
0 k ~
X'
0 ●

w
relations will be satisfied if X' > XII and X' <X
(6-6x10-34)x(2-2x10‘^) = 0-91eV
'OB ~ he
1-6x10-19 8. ...(0

Flo
Since incident energy 0-825 eV is greater than
0-74 eV and less than 0-91 eV, so photoelectrons he

ee
are emitted from metal A only. and
- Kg+(pQ ...(«)
3. As the wavelength A. of the incident light falling

Fr
on cathode is gradually increased,there will be a Dividing (ii) by (i), we gel
value of X above which the photoelectric emission

for
will slop, i.e., the photoelectric current will
ur
A
become zero. As X is decreased, the energy of the B

incident photons increases (£ = /icA). Due to it, 2A


ks
g _
more number of photons will be successful in or or
A
Yo
ejecting the electrons from the surface of cathode. g
oo

As a result of it, the photoelectric current or 2K^ +200 = +00


eB

increases, i.e., I« HX. Thus, option (d) is true.


4. According to Einstein’s photoelectric equation or
Kg=2K^+(Po
he K
r

B
— mv~ =—— 0 0 ...(/) /.e., Kg>2K^ or
>^A
ou
ad

2 A 2
1 he 4 he he
Y

and ~tnv'^ — -00 = -0 0 ...(«) 0 — mu


2 _
2 (3A/4) 3A '0 ;
2 X1
nd

4hc
Re

-0 ■) he
V 3A 0 4 — =
'0
X-.
Fi

From (/) and (//), — = he


> - 2
V 3
A he he
'0
4\‘^^ M,
2
X
I 1^- X1
or or
V > V ■)
he he
1
3; “2 '0
he
5. As eV =
X
0 4 he he 4 he he
or
-4<!)o=7—4>o _ = 3<|) 0
1240 eV run 5^, X
1 X2 X^
Case (0 ^x6 = ’0 ...(0
400 nm
I 4/?c he 1 1240 1240
4x
= (3-1-(I),,) O'-
3 X^ X1 3 "JhT 248
J
1240 eV nm
Case (fV) exV' = '0 ...(«)
(400 + 600) nm - [16-5] =3-7 eV

= (1-240-({lo) eV
11/94 'Pncutecp 'a Fundamental Physics (XII) kviwii
12400 eVA
O
he
For Difficult Questions or ~k = = 1107 A
11-2 eV 11-2 eV
he he
10. Max. K.E. = e Ed = — This wavelength is in the ultraviolet region.
X0
14. Energy of waveleng.n 550 nm is,
e Ed he -eEdX he 1240eV nm
or
£. = — = 2-25 eV
X..0 X he heX 1
A 550nm
I

he X Energy of wavelength 450 nm is,


or
Xq- he — e Ed X 1240eV nm

oww
£,= = 2-75 eV
he 450 nm
11. As cV, '0
Energy of wavelength 350 nm is,
From 1 St observation, 1240cV nm
he ^3 = 350nm
= 3-54 eV

e
ex 2 =
0
...(0
-6

re
FFrlo
0-3x10 Since light of all the three wavelengths are
From 2nd observation. incident on each plate with equal intensities, the
stopping potential will be linked by the light of

rF
ee
he
exl =
'0 ...(») wavelength having maximum energy, i.e.,
0-4x10-^ 3-54 eV.

rF
ouru
From 3rd observation.
Plate-/? is having least work function (= 2 0
he eV). will have maximum value of stopping

fosor
exO-4 =
0 potential as it will emit photoelectrons due to light
0-5 X10"®
skf
of all the three wavelengths of energies E|, £2
Subtracting (//) from (/), we get and £3.
ooko
he he he 1 1 Therefore, for plate-/? saturation current will be
Yo
Y
e =

0-3X10"® 0-4x10“® 10"®L0-3 0-4. maximum and stopping potential will be


Bo

maximum.
reeB

exl0~®x0-12 Plate-^ is having a work function (j) (= 2-5 eV),


or h =
cx(0-4-0-3) will emit photoelcctrons due to light of energy
ooY

£2 and £3. The value of saturation current and


uur

(l-6xI0-*^)xl0"®xQ-12 stopping potential will be less than plate-p.


ad

(3xl0^)x0-10 Plate-r is having a work function (f>^ (= 3 0 eV),


dY

= 6-4 X 10-^ Js will emit photoelectrons due to light of energy


£3. Its saturation current will be minimum and
nind

1
stopping potential will be least. Thus option (a)
Re

12. As =hv-hv0I
2
niiiA
is true.
F
Fi

1 15. Let t?], i>2 be the maximum velocities of the


■■ 2
m X
(4 X 10®)- = h 2vq - hvQ = hvQ ...(0 photoelectrons in two cases. As per question
1 v^=2v and i?2 = v. Then £] = 4 Kand K2 = K
and - ni = /i (2 Vq + 3 Vq) - /ivq = 4 hv^ he
...(H) (££),=--<l.o
X
I
Dividing (//) by (/). we have
n 1240 eV nm
i? “ or 4K =
'0 ...(0
max _ 4 or v'max =2x4x 10® 350 nm
(4xl0®)2
he
= 8 X 10® ms
-1
(A'£), =- 4> 0
13. Here. ([>(, = 6-2 eV, Fq = 5 V ^2
he he 1240 eV nm
As. <’V„ = Y-'t'o '>'■ Y = ^''o + 'l>o or K =
540 nm
'0 ...(H)
= ex5 V + 6-2eV = ll-2eV
DUAL NATURE OF RADIATION AND MATTER 11/95

we have Angular frequency,


For Difficult Questions
Q) = 2 Ji X 6 X 10**^ = 27Cvorv = 6x lO’"^ Hz

Subtracting (//) from (i), we get Energy of photon.

1240 1240 6-6x10-34x6x10*^


3K = = 3-542- 2-296= 1-246 £ = /jw = eV
350 540 1-6x10"‘9
1-246 = 2-475 eV
K = = 0-415
3 {KE) max = (^Vo = £-0o
= 2-475 - 2-0= 0-475 eV
1240 1240
= 0-475 V = 0-48 V
From (//), (|>Q = -K = -0-415 or

oww
0
540 540
20. As, Kmax = eVj, = hv - (j>Q = Av - /ivq
= 2-296-0-415 = 1-881 = 1-8

16. According to Einstein’s photoelectric equation Case (0, — hv - hv


0
...(1)
2
Kmax
= hv - (|)q 0-5 = hv - (j)Q ...(/)

e
hv

FFrlo
20
Case (i7), eV= hv. 0 ...(2)

re
and 0-8 = h v +
'0 s
2
100

ree
Putting value of (2) in (1), we get

F
= 1-2 hv - (t>(, ...(«)
Multiplying (/) by 1-2, we get 1 hv

rF
hv.0 = hv- hv. 0
0-6= 1-2 hv - 1-2 <!),) 2 . 2
Subtracting {in) from (»), we get

fsoor
ouur
hv hv,0
0-2 = 0-2 (1)0 or (t»o = l^V or = hv — hv.0
4 ~ 2
skf
17. The incident light has two different angular
frequencies ; 6 x lO'^ s~^ and 8 x 10*^ s”*. The
ooko
hv.0 hv
Yo
maximum kinetic energy of the emitted photo or hVf, = hv~ —
Y
® 2 4
electrons will be due to larger of the two angular
Bo

frequencies, i.e., (o = 8 x lO'^ s“'. Now


reB

hv0 3/jv
or or vr,0 = 3 v/2
(1) 8x10'^ 2 4
V =
uur
oY

2Tt 27t
he
ad

21. As, K max


Maximum kinetic energy, = hv - ifiQ
dY

E
(6-63x10"34)(8x10*^) eV-2-28eV
he
max Case (0 ...(1)
1-6x10-'^x2tc
innd
Re

1
= 5-28 eV-2-28 eV = 3-0 eV
Fi
F

he
18. Max. angular frequency, Case (li) = '0 ...(2)
CO
max = (6-28 X lO^) c = 2 7C V max
(6-28xl0'^)x(3xl(y^) As, X,| = 3 ^2, SO from (1), we have
V
max
2n he
K,1 = '0
iKE) max = hv max 3^2
(6-6 x 10-34) X (6-28 X10’) X (3 X10^) -4-7
he
2x3-14x1-6x10-'^
or
3 K, =--3(1)0 ...(3)

= 12-37-4-7 = 7*67 eV
Subtracting (2) from (3), we have
19. Comparing the given equation of electric field
with the equation
3K^-K2 = -2% or 3 /f I = /r2 - 2 00
2tc ^2
E = E„0 cos
A
3 K^<K2 OT K.< —
X-03( X, ’ 3
X
11/96 'P’uxdeep- 4 Fundamental Physics (XII) lwwm
Putting value of equation (2) in equation (1), we
V -
^ K ●.'■1 i! ^ '■
For Difficult Quostions
get

22. Given, he he
'0
£ = 200 [sin (6 x 10^^) t + sin (9 x 10*^) /] Vm“*. X0
For maximum kinetic energy, we will take higher
3 he he
9x10'5
frequency used, i.e., = Hz
or
IX -3«|)o = 7 %
27t 0 '0

3 he he
9x10*5
^ 0 - 3 ([>Q - 2 (|)q
or
Kmax = ftv-4)o = (4-14x 10"’5)x 2tc
-2-5 2X
0

= 3-43 eV = 3-42 eV

ww
he
or
he ’o -
4X
23. As, 0
0
If X' is the threshold wavelength, then
he

Flo
Case (i), eV
"1 X '0 ...(1) fw^Jw or A,' = 4 Xfl

e
I X'~ 4X 0

ree
he
Case (n), eV ...(2) 25. When the incident frequency of light becomes

Fr
'0
"2 Xj

rF
1-5
half, i.e., v = — Vq =0-75vq, then the frequency
uurr
Subtracting equation (2) from equation (1), we

for
have
V becomes less than threshold frequency Vq.
he he Therefore, no photoelectricemission will take
eY -eV
s
place whatevermay be the intensityof the incident
kks
^1 ^2
light.
Yo
oooo

he 1 1 'l 26. No density of photons number of photons oc

y -y emitted.
or s
■^2 e X,
eB

^2. If n no. of photons emitted in time /, then


1 1
or 0-710- 1-43 = 1240 he M n
ur

491 X, Power, P = n or — = a constant


X
X ji
ad
YYo

he 1240 eVnm

e e ^ _ Inm 1
or
X| X2 X2 500 nm 500
dd

-0-72 1 1_
Re

or
27. Given. X = 600 x 10“^ m = 6 x 10"^ m,
in

1240 “ 491 X^
P = 3-3x 10-5 W
F

1 0-72
or = 0-00203 + 0-00058 he
^2 491 1240 Energy of each photon = —
A
= 0-00261
Let n photons be emitted in time t from photo
1 sensitive surface.
X,= 0-00261
= 383-14 *=382 nm
Power delivered to photosensitive surface
he n he' PX
24. As, P =- or
n

/ he

he Number of photons emitted per second is


Ca.se (/), e(3 Vq) = -^-(1)q ...(1)
0
n _PX _ (3-3x1Q-5)x(6x1Q-'^)
he t he (6-6 X 10-5**) X (3x10*)
Case (//), e(yQ> = 2X
...(2)
0 = 10*® photons per second.
DUAL NATURE OF RADIATION AND MATTER 11/97

: '‘ivOTCaCO© he
For Difficult Questions 33. In X-ray tube, ^min ~ eV

28. No. of incident photons falling per sq. cm. hi . \

log (^mm) = 'og


/ IX eV )
n =
hc/X he he he
log ) = log — - log V = - log V -i- log —
e e
(10/10^)x(300xl0-^) 10
15
The above relation shows the graph between
(66xl0"^'^)x(3xl0^) 66 log and log Visa straight line with negative
As 1% of the incident photons produce slope and its intercept on log axis IS

oww
phoioelectrons, therefore no. of photoelcctrons positive. Thus option (c) i.s true.
15 34. Dc-Broglie wavelength,
1 10
produced per sec. = n - x
12-27 . 12-27 f 12-27 xlO"'0
100 100 66 X = A = m

= 1*51 X 1012 ^/v ' Vioooo 100

ee
29. E - hv and p - h/X = 12-27 X 10-^2

FFrlo
So E/p - hvl{h/X) = V X = c, i.e., c = E/p h

r
30. Photon flux at a distance r 35. As X^ or E =
yjl m E

rF
2m

ee
no.of photons emitted per sec
4jir2 /|2 /|2
£,1 =

rF
ouru 2 m X}I 2mX\
3-14x102“ 1 10
20
1 =

ffosor
or or r- =
4x3-14xr2 4 /|2 I
os k
or /● =
10>“
or ^ = 5x10^^ cm
2m [X? X~
2
ook
Yo
{6-6x10 -34)2 1
Y
31. Here, >.= 500 X 10-'^m:/^= 10 W; J
Bo

initial momentumper second of the incident light 2x9-lxl0--^' (0-5x10“'^ )2 (2x10"^)2


reeB

P 10
= 3-33 X 10"^ Ns (6-6x10 -34)2 3-75
X eW
3x10^ = 5-6 eV
ooY

c
2x9-1x10-^' xlO-'^ 1-6x10-*^
uur

Final momentum of incident light = 0, as the


ad

surface is perfectly absorbing. 3


36. E~—mv-=—kT
dY

Force = rate of change of momentum 2 2

333x10"^ Ns
= 3-33 X 10-* N
nind
Re

or mv= m
Is mv

32. Energy of the incident photon of wavelength 400


F
Fi

h
he 1240eV nm
nm is : £ = — = 3-1 eV So, ' ^2 ~
X 400 nm r"i
m
3 7'2 "^2
No. of such photons in a beam of light of power
1-55 mW X (2^(2^
1

n =
1-55x10‘2
= 3-125 X 10'5 s"'
n m
i
,T n’lJl' = 2,

-19
3-1x1-6x10 X
1
No. of photoelectrons used to produce -i-= 2: 1

photoelectron per sec = 10% of n


10 37. For bragg’s diffraction
X 3-125 X 10'^ = 3-125 x lO'^ s'
100 2f/sin 0
2 £/ sin 0 = n >. or X=
Current due to such phoioelectrons per second n

= (3-125x 10‘‘^)x(l-6x 10-^^) = 5x 10-^ A As O<sin0<l so X^2d


= 50|iA
11/98 ^●utdee^ 4- Fundamental Physics (XII) tTOWn

For Difficult Questions


.'. K =
(662x10"^'*)^
2x(9lxl0“^')x(7-5xl0"‘-)“
38. X=- ...(0 1
P X eV
1-6 X 10“'^
0-25 h = 26-75 X 10^ eV = 26-75 keV =: 25 keV
X+ X
100
(p-Po'^ —» h /^ X V
I

100-25 h X
● i >
y
or ...(H)

oww
100
P-Po Let k be the de-Broglie wavelength of the particle
P formed. Final momentum of the particle formed
From (/) and (//),
h ^
100-25 P p=- i
k
100

ee
P-Pq According to law of conservation of momentum

FFrlo
On solving, p = 40! pg we get

r
39. Let = mass of particle and m2 = equivalent P = P,+ P,

rF
ee
mass of photon moving with velocity of light
c = 3 X 10*^ ms having the same de-Broglie /j A /j A h A

(-»)

rF
wavelength as that of particle. Then
k k^ ky
ouru
8
'”1 _ ^ 3x10 3
m^v=nhc or
ffosor
2-25x10^ 2-25 or h^Jl h
ni2 V
os k
k~ k X
kV k
.r
k
V

1 2 k
ook
\ /..\2
Yo
-w, V V
^1 _ 2 1 1 m k = —
Y
V or
1
= —x X
2
Bo

£2 nu c
2
2
reeB

43. qv B = mv~/r or mv = q B r
1 3 2-25 f 3 h h
De-Broglie wavelength, k =
ooY

= -X X
uur

2 2-25 3 8 nw qBr
ad

so A.« Mqr
1 ni^v^
dY

2 _
40. For proton, £ = mv
2 2 m
K.^P^P _ ^ yj
■■ k
p r 2
nind
Re

or mv =

44. K.E. of the particle at the given temp. T is given


F
Fi

h h by
A.=
mv
^2 inE 1
2 _
3
~mv
2
— ^5", where ^ is Boltzmann constant
he he
For photon, £ = /jv, = — or k^ =
2 X. “ £ or
mv = ■yjSmkT
h
A| h / ^2 niE 1 £
1/2 De-Broglie wavelength, A =—
a E mv
^3mkT
A-, hclE c \ 2m
I
41. Here, X = 7-5 X 10^'2 m ; i.e.. k oc

Kinetic energy of electron -JmT


k '8
P^ H _
m
He^Hc _ 4m
X
(273-h127)£
K =
●● X He 2 m (273+ 27) A’ 3
2 m 2 mI X
DUAL NATURE OF RADIATION AND MATTER 11/99

oornir?Ty?:;^.T_^rr^fr./i:^vr<5^^ h 1
For OifficuK Questions 47. X= or X OC

.jlmqV
45. Refer to Fig. 11.(CF).13 shown here. According
1 1 1
to law of conservation of linear momentum, we X :X -.X
have e p a
'fVo.
FIGURE 11(CF),13
rest

^ V\ V2
dy^ (ffi)—► (m/2)—►
X^>Xp>X a
A
B 48. Momentum of particle A,
h
Before collision After collision h
h
— i aP-^

ww
I A B
in m
wi X u + — X 0 = w V,1 + — u,
2 2 Momentum of particle B,
h IS
or
2mv = 2 mV] + m V2 P^=^J

Flo
or
2v = 2vi + V2 ...(0 ^2

e
For clastic collision,

e
relative velocity of separation = relative velocity Let P be the momentum of the final particle after

rere
inelastic collision.

rFF
of approach
=t;-0 = tJ Using law of momentum conservation, we have
uur r
Putting this value of v in (/), we gel h A h ^
2 (i>-) - Uj) = 2 + ^2 or V2 = 4vj
From (it). 4i»,-i;]=uor3ui=i'ori;] = i;/3
ffoor
P = Pl + P2 = ^‘+T~J
I
sks
h f + /t f
Yoo
V 4u
I p\ =
ooko

Also l?2=tij+l? = — + D =
3 3
id
eBB

De-Broglie wavelength of particle 4,


h
h h)- +
h
or _L-J_ J_
X^~Xj'^Xl
or
X^^ = — X
uurr

mv 1 m (u/3) \ - I
ad

De-Broglie wavelength of particle 5, 49. From graph, v= 10‘^ Hz,


Yo

Kmax = 3eV = 3x l -6x 10"'^ J


h h ^_2 As, ,Kmux - Iiv - hv 0 or livr,0 = Itv - Kmax
dY

Xg - m ■■ >. B 1

^(4t;/3)
Re

— V-y
2 ^ K 3x1-6x10-*^
innd

or = V -
max
= 10>5-
0
h 6-6x10-34
46. When a particle of kinetic energy E goes to a
FFi

region where it has now a potential energy V, = (10-7-3) X 10'4 =2-7 X 10*4 Hz
then its new kinetic energy, S' = E - V, because 50. Initial momentum of nucleus of mass M, Pj = 0.
the energy V has been taken out of its K.E.. Let Pj and P2 be momentum of broken nuclei of
similar to a body going to a height losing K.E. masses and 5 respectively.
and gaining P.E. According to law of conservation of momentum
h P, = P,+P2
De-Broglie wavelength, X =
^2mE .-. 0 = P, + P2 or P,=-P2.
h
h h Hence X, — —
X' = ' P1
yj2mE' ^2m{E-V) h
and — (in magnitude)
h X P, P1
^2mE(l-V/E) yjil-V/E) ^j — ^
11/100 Fundamental Physics (XH)E3EDEI

53. Initial de-Broglie wavelength.


For Difficult Questions
h
Xq - mV ...(0
0
51. We know that X = — dX = — ~dP ;
P pi Acceleration of electron in electric field

eE.0

dX -4^/> dP dP
a =
m
(in magnitude)
_

X h/P P P Velocity of electron after time t is


eE
dX 0-5 0
Given, ; dP = P. V = u + ot =V^0 +

oww
X 100 m

Let the initial momentum be Pr,. de-Broglie wavelength of wave associated with
0
electron at time f is
™ P dX 0-5 100
or Pr.0 =
Then, — = — = —
Pq X 100 0-5
P =200P
1=^ h

mV eE.

ee
0
m K-h
52. If (KE)j is the KE of electron just after ejection 0
m

FFrlo
from cathode, then

r
h
he

rF
ee
{KE). = eE.0
X mV0 \ +
ph mV0

rF
ouru
Final kinetic energy of electron while reaching X
anode is 0

ffosor
eE
0
[from (/)]
he H-
os k -1}) +eV
{KE)f={KE)- + eV = X
mV.0
ph
54. Given, KE of electron, K = 144 eV
ook
IfV'»(t>/^ or e?V» (j), then (KE)^~ eV
Yo
Accelerating potential difference for electron,
Y
1 V= 144 V
Bo

or = eV or mv .JTmeV
reeB

2 nijx max
de-Broglie wavelength,
De-Broglie wavelength, 1A. = —^
>2-27.A
12-27
xl0"'“m
ooY

Vv
uur

h h
X
ad

e
= 1-02 X 10-'« m = 102 X 10”'^ X lO-'^ m
mv
max ^2meV
= 102 X 10-^
dY

nm

55. Given, = 0. Let K be the maximum kinetic


or X «
nind

‘ # energy of the emitted photoelectron when light


Re

of wavelength X falls on a photosensitive surface.


If V is made 4 V. then
F
Fi

Using Einstein’s photoelectric equation we have


X he
=K+0=K -(0
or X'e = e

X^ ~ll4V ~2 7

Let p be the momentum of the emitted


Thus option (a) is true. photoelcctron, then
If and A.p|, increases, then (AT£)ydecreases. ->

But (KE)j-= eV, so Vdecreases, hence X^ increases. K =


P~
or P = ^2 mK = ^ImhdX
Im
Thus option {b) is incorrect.
Now de-Broglie wavelength is
h dX dX ph
Also X^ = or h h

^2m(KE) f di dt \/=TP
.^2 in he IX
Thus option (c) is incorrect.
' 2mc'
Xg is independent of d, hence option {d) is or x =
incorrect. , h
DUAL NATURE OF RADIATION AND MATTER 11/101

h h
For Difficult Questions
^A = and ^B
a/2^ B

56. —nnP- = E or nw = Jl m E
2 T 1 T,A -1-5
H
or
de-Broglie wavelength for electron is 2 1 7,
Va
h
K =— On solving, we get
...(0
mv
■yjlni E r^ = 2eV
he
7*3 = 2- 1-5 =0-5 eV
For photon, E=™ or
P
= —
E
...(H) 03 = £3 - 7*3 = 4-5 - 0-5 = 4-0 eV
h X
58. x = or X and OC

Vi 2 VeTae

ww
Xe h
E 1 ^2m E
X —

V2«j E he cyim X E + AE
or 4 E = E + AE
X/2"V E

FF loo
57. Given 7^ and 7'3 be the kinetic energy of the
photoelectrons emitted from metals A and B or AE = 4E-E=3E

ree
respectively. Then

III

rFee
Multiple Choice Questions (with One or More than One Correct Answers)

F
oor r
rur
he he he 2-9x400 eV nm
0
Longest wavelength = —
59. As eVy = — '0 - 0 “
s ff I-9eV
A eX e '0

so variation of Vq with X and IfX is given by - 610 nm


k
YYoou
options a and c respectively. 63. From Einstein’s photoelectric equation
ookos

60. The intercept of straight line for a graph between he


BBo

and v on negative energy axis gives the value K.E.= eV = 0 ...(0


X
of work function of cathode metal. The point
re

where the straight line cuts the frequency axis, or V = — ...(H)


gives the value of threshold frequency, where as eX e
ouur
ad

the slope of st. line can help to find the planck’s


Yo

constant. When A. = Xq, the threshold wavelength, K.E. - 0.


he 1
1 9 1 2 _
he
From (/), 0 = — or
-mv^ =hv-%
dY

61. As or —mv
'0 X he X0
Re

2 X 0
idn
FFin

V or i; oc '1
For plate 1, = 0-001 :
Vx he
so options a, c, d are incorrect as no. of
photoelectrons depends on intensity of incident For plate 2, —^ = 0-002 ;
he
light, so, option b is correct.
62. Here, X = 400 nm, = 1 eV, 0q = 1 -9 eV '1 = 0-004
For plate 3,
he he
K
max =^
X
- 4>o or he = {K max + $()) X
0, ; 02: (})3 = 1 : 2 : 4
= (1 + 1-9) eV X 400 nm = 2-9 x 400 eV nm Differentiating (//), we get
X' = 500 nm, then he A V he
A F =—A (1/X) or
he 2-9x400 eV nm e A (1/X) e
K - 1-9 eV
max '0
X' 500 nm AV he
= 2-32 eV- 1-9 eV = 0-42eV
Slope of the graph, tan 9 =
A (1/X) e
11/102 Fundamental Physics fXinPTSTWl

65. Photoelectric effect is explained by quantum


For Difficult Questions
theory of light, which can explain all the
characteristics of photoelectric effect mentioned
For plate 2, threshold wavelength, in the question.
he he
= 500 nm 66. Saturation photoelectric current depends upon the
2 0-002/2c- inlenitty of the incident light.
For plate 3, threshold wavelength, As K max = hv — (pQ . If V becomes double, then
he he
\ (|)3 0-004/ic
= 250 nm K max becomes more than double as

K n^ix
= 2 /iv - (t>o = 2 (K max

Since the wavelength of violet colour is 400 nm = 2Kmax + 4>o


and \ for plate 2 and for 67. Wavenumber v = I/X;

ww
plate 3, therefore, violet colour light will eject he
elections from plate 2 and not plate 3. Energy of a photon = hv = — ~hcv
X
64. Since intensity of incident light/oc 1/r^; therefore
h 1
on increasing the distance 0-2 m to 0-6 m, i.e.,

Floo
68. (/) X = ; X oc — if u is same.
three times, the intensity of the incident light mv m

reduces to 1/9 times the original intensity, i.e.,

ee
h
will become 2*0 mA. But energy of each photon (//) Also X =

eer
will still remain the same.
The value of cut off potential is independent of

FFr
1
so X«_ if E is same.
the intensity of the incident light but depends upon

oorr
uur r
■ym
the frequency of the incident light.
(m) When two bodies fall from the same height,
s ff
The saturation current depends upon the intensity
of the incident light and not on energy of the both acquire equal valocity (/v), so the X will be
less for heavier particle for options {a), {b)
sk
incident light.
YYoo
and id).
ooko

mi Multiple
eBB

Choice Questions (Based on the given Passage/Comprehens ion)


69. When Jockey is at P, the voltage across P and Q 72. Path difference between two electron waves
uurr

10
reflected from the two planes of the atoms in a
ad

V = x4 = 8 V crystal is = d sin (90'* - 30°) + d sin (90° - 30°)


Yo

(4+1) = 2f/cos 30°


Resistance of vacuum tube. For diffraction of these two waves 2 deos 30° = X
dY
Re

R =
8 V
= 4x10^ n or X=2(/cos30° = 2x 10“'‘^x V3/2
ind

2x10"^ A
FFin

= V3xl0-'°A
ne It (2xl0-^)xl h
70. / = — or « = — -xio'3 De-Broglie wavelength, X = —
t e 1-6x10-19 4 mv
-JlV em

n he n he *2
Power, p = or X = or V =
X P 2emX^
(6-6x10"3‘1)2
-xlO'3 (6-6x10-3^)x(3x10^)
or X = X
2x(1-6xI0-'9)x(9-1x10-31)x(3x10'20)2
4 4-3x10“^
«50 V
= 5-74x 10~^=5740A°
73. Glancing angle is the inclination of the incident
Thus, yellow light is incident rays with the plane of the atoms which is,
71. Pol. diff. across the two electrodes when jockey 0 = 90° - 30° = 60"
is at the mid points = 4 V 74. The Bragg’s relation for having an intensity
kinelic energy of the emitted electron maximum for diffraction pattern is
= ex4V = 4eV
2 (/ sin 0 = /z X where 0 is glancing angle.
DUAL NATURE OF RADIATION AND MATTER 11/103

For Difficult Questions

Here, 6 = (90“-/) 75. There will be electron diffraction when the de-

2 d sin (90° - i) = nX or 2d cos i = nX Broglie wavelength of moving electron is


comparable with the width of slit. This will result
FIGURE 11(CF).14
into a diffraction pattern on screen. With bright
central maximum intensity having dark and bright
minimum and maximum intensity alternately and
of decreasing width.
The size of central maxima is more than the width
of slit d. Therefore, the option (a) is correct.

oww
m Matching Type Questions

76. (A) Planck’s theory of quanta is for quantisation Wave nature of slow moving electron was estab
of energies of oscillations. lished by Davisson and Germer. The wave nature

e
(B) Einstein theory of quanta gave the energy of of fast electrons was established by G.P. Thomson.

ree
rFl
photon as /iv. 78. (A) Franck-Hertz experiment is associated with

Fre
(C) Bohr’s stationary orbit in an atom is one in descrete energy levels of atom.

rr F
which the angular momentum of electron is
integral multiple of h/ln. (B) Photoelectric experiment is associated with
particle nature of light.
ouur
(D) De-Broglie waves are used in the working
of electron microscope. sfoo
(C) Davison-Germer experiment is associated
with diffraction of electron beam Le., wave
77. Particle nature of light was given by Max. Planck.
kks
nature of electron.
Yo
oooo

Wave nature of light was given by Huygen.


eBB

K9 Matrix-Match Type Questions


uurr

79. The value of photoelectric current depends on Pa m


a — 4 ^
ad

intensity of incident light and not on frequency


\l- =2;so p^>pp
YYo

Pp
m
of incident light. The value of stopping potential p

depends on frequency of incident light and not


dd

on intensity of incident light. (B) When, E.a =-E


4 P'
Re
iinn

80. K.E., E = -mv^ or mv = ^2mE X m E \xE


F

2 a p p
then

de-Broglie wavelength, X = —
h
^4x(£^/4)
mv
■yj2mE 4xE 14
Pa _ P
= 1
and momentum, P = mv = -^2mE pp i Ix£
p

1
X OC
(C) £„ = 4 E^, then X^ > X^ and p^^ > p^
■yJmE
1 2

P ■yJmE (D) v„a = u.„


£●
2^a m 4
P' then _! a _ a _

£ 1 1
(A) When £^ = £^, then 2Vp
p

X
IL= H=1
m
ct _
So E^ = 4Ej,
m „
a
14 2
or
X^>X a
Hence, X„P > ,X a and
Pa>Pp
11/104 Fundamental Physics fXIT^WSTim

For Difficult Questions

Integer Type Questions

81. Here, r = 1 cm = 10"- m ; ({)q = 4-7 eV, Current, / = 3% of no, of photoelectrons emitted
X = 200 nm. Lei be the stopping potential. per sec x charge of electron
According to Einstein’s photoelectric equation 3 3 25 -19
(using he = 1240 eV nm), we have = fie = X — X 1-6x10
100 100 3x10"'^
he
= 0-4 A = 4 deciampere

ww
X
nhc
\240eV nm 84. Energy falling per second on the miiTor, E =
-A-leV X
200 nm

Flo
Momentum of the incident light per second.
= 6-2 eV-A-1 eV= 1-5 eV

e
£ nh
P=—

rere
■■
c X

r FF
The sphere will stop emitting photoelectrons, Momentum of the incident light per second
when the potential on its surface becomes 1 -5 V.
normal to the plane mirror, P| =pcos 60°, towards
uurr
Let n be the no. of photo electrons emitted from
sphere.
Then charge on sphere, q = ne, = 1-5 V
mirror
for
Momentum of the reflected light per second
kss
normal to the plane mirror. p-> = p cos 60°, away
from mirror
ooook

1
Yo
ne
V =
4ti€q r 47t6Q r Force on mirror = - (-pj) = p-> +pj
eBB

= 2p cos 60°
Vxrx47te
s 0
n —
2nh
urr

e cos 60°
X
ad

_(l-5)x(lQ-^) 1
Yo

X
1-6x10"'5 ''(9x10^) 2x(2xl0‘^)x(66xl0""‘^) 1
dY

X-

= 1-05 X 10'^ 660x10"^ 2


Re
innd

As per question. Ax 10^ = 1-05 x 10^ or Z = 7 = 2 X 10"* N


Fi

85. Here, q\ = e and <?2 = 120 e


82. ^Vq = hv - <1)q or V,
0 “
e e
i-Q = 10 fm = 10 X 10"’^ m = 10"^^ m

Slope of the stopping potential (Kq) versus


frequency (v) is = hie = constant. Therefore, slope
m
p =^xI0"2'^ kg,
of Lq and v plot will be same for silver as well as
for sodium. - = 4-2x10"‘5Js/C
e

83. No. of photons emitted per second by 25 watt


1
source will be = 9xl0^m/F
4tc€ 0
E EX
he IX he Using the principle of con.servation of energy

25x(6600xl0~“^) 25 1
= —mv~ =
->
p‘ _ (hlX)^ _ h^
(6-6xlO“^^)x(3xlO^) 3x10"^^ Akg„ .r
0 '0
2 2m 2m 2mX^
DUAL NATURE OF RADIATION AND MATTER 11/105

'CQ©
For Difficult Questions

h !?
47re(,r„/i2 86. As, X = or EK -
X = ,j^K 2mX~
)l i/j (2 fn)
1
or
Ek oc

(10xl0-l^)/rx3

oww
y9xl0'^e(120e)x2x5xl0
-27
E
_
[kT ro.5f
X, 4
3QxlQ-'-‘^(4-2xlQ-*^)-
'V9xl0‘^xl20xl0xl0'-'' or
^A'2 =4^;^, =4x2f'V=8t'V

e
X = lx 10-‘^m = 7fm

re
Increase in energy = - Ef^^ = 8 - 2 = 6 eV

FFrllo
reF
VII.

e
Assertion-Reason Type Questions
uoru
osFr
FOR MEDICAL STUDENTS FOR ENGINEERING STUDENTS

87. When incident light is of frequency =--x(I-5 r,,)


2 fkfor
92. Both statement-1 and slatement-2 are true and
statcment>2 is the correct explanation of the
okso
statenient-1.
= 0-75 Vq i.e., less than threshold frequency no
Y
Yo
93. K.E. max - hv - hv,-,0 - eV.0 ...(/)
photo electric emission takes place. Hencecurrent
oo
BB

becomes zero.
hv hv0
Thus, Assertion is wrong. Here Reason is also So l/„=-
e e
...(«)
Y
r ree

wrong.
ouu

88. Photoelectric effect demonstrates the particle When i’ is doubled, K.E.max = 2/ji’ - hv,,0 = eV„0
ad
Ydo

nature of light and the number of photoelectrons


emitted is proportional to intensity of incident 2hv hv 0
light. Thus the Assertion and Reason both are and VjJ =
nidn

e e

wrong.
Re

89. Both Assertion and Reason are correct but the k.e' ^^0
F

max
^ 2 and
Fi

Reason is not the correct explanation of the K.E.


max 0
Assertion.

90. Both Assertion and Reason are correct and


From (i) and (//). K.E. max oc V and V,,0 « v

Reason is the correct explanation of A.ssertion. Thus, statement-1 is false, but statement-2 is
true.
91. Using law of conservation of linear momentum,
we have he
94. eV.,n = K mux
Mx 0 = W| U] + n?2 ^2 X
or
I W] Uj I = I nh V21 When X decreases, then both Vq and /f, will max

Ratio of de-Broglie wavelengths X-rays < ^ultraviolet ’ ^O Vq and /r,


increase. As X max

X1 him, V W2 V2 will be more for X-rays than for ultraviolet rays.


1 *^1 _
Hence, statement-1 is true. Here, statement-2
^2 h/m^ v-y '"1^1
is false because the incident ray is not having
Thus Assertion is true. Here Reason is false. range of frequencies, for pluttoelectric etnission.

t
11/106 Fundamental Physics (XII)ESSISD

JOCK
For Difficult Questions

95. When high energy X-rays fall on the ball, the m

metal ball will emit photoeleclrons, leaving the

w
\
I
Vm
positive charge on the ball. As a result of it. the
ball is deflected in the direction of the electric
m
field. Thus, the statement-1 is true but or
statcment-2 is false. 2 'Vm VM

e
Thus, statcment-1 is wrong and statement-2 is
96. eV = -mv^ or mv =-JlmeV

e
wr
correct.
2

lloo
r
97. Both the statements 1 and 2 are true and the
h h 1
. i.e., statement 2 is the correct explanation of

F
■^ImeV 4m

FFu
mv statenicnt-1.

rese
uro
k
Fr
o
foo
fr
kso
Y
Y
B
Yo
oo
eBr
e
uru
od
Yo
ad
n
ndi
Re
F
Fi
Re
F
Fi ad
nid
nY
Y d
o
ou
u

ATOMS

NUCLEI
Yo

CHAPTER 12.

CHAPTER 13.
rY
eerB QJ][MD=ir=©
B
ATOMS
oo
uor
FF kos
ofof
k
roF AND NUCLEI
ullo
o
rw srr
ee
w F
re
e
oww
ATOMS

e
ree
12.1. INTRODUCTION

rFl
Fre
The structure of matter that shapes the world around us has been a subject of study since long. The first

rr F
contribution in this regard came from Dalton, who postulated that matter is made of atoms, which are indivisible.
The word atom comes from the Greek word atomos meaning ‘not cut'. J.J. Thomson proposed a structure for
ouur
sfoo
the atom, which was modified by Rutherford and later by Niels Bohr. In this unit, we shall discuss these
models of atoms in some detail. We shall also study some fundamental information about the atomic nuclei.
kks
The phenomenon of '■Radioactivity' has probably played the most significant role in the development of
Yo
oooo

both, the Atomic and Nuclear Physics. We shall study important aspects of this phenomenon. Further, nuclear
reactions provide useful information regarding interaction of nuclei. This led us to the phenomena of nuclear
eBB

fission and nuclear fusion whereby the vast reservoirs of nuclear energy were discovered.

12.2. THOMSON'S MODEL OF ATOM


urr

According to Thomson’s model, every atom consists of a positively


ad
YYo

charged sphere of radius of the order of 10“**^ m in which entire mass and
positive charge of the atom are uniformly distributed. Inside this sphere, the
dd

elections are embedded like seeds in a watermelon or like plums in a pudding.


Re
iinn

The number of electrons is such that their negative charge is equal to the positive
charge of the atom, Fig. 12.1. Thus the atom is electrically neutral.
F

Limitations of Thomson atom model were the following :


/. It could not explain the origin of spectral series of hydrogen and
other atoms, observed e.xperimentally.
2. It could not explain large angle scattering of a particles from thin
metal foils, as observed by Rutherford.

12.3. RUTHERFORD'Sa-RAY SCATTERINGEXPERIMENT:


DISCOVERY OF ATOMIC NUCLEUS
It is an important experiment, which led Rutherford to the discovery of atomic nucleus.
An alpha particle is helium nucleus containing 2 protons and 2 neutrons. Therefore, an alpha particle
has 4 units of mass and two units of positive charge. Many radioactive elements emit alpha particles.
12/1
12/2 Fundamental Physics (XII)BSSD

The experimental set up used by Rutherford and his collaborators, Geiger and Marsden is shown in Figs.
122 and 123.

FIGURE 12.2

Gold Foil
(10”®m thick)
ZnS
Collimator Screen

^ Most-a
^ Pass
( j

fn a r\
\ > a / Tq ^ Straight
Lead

ww
Cavity

About 1 In 8000-a
/■ Microscope

is Reflected Back (Rotatable


Detector)

Flo
e
S is a speck of a radioactive source* contained in a lead cavity. The alpha particles emitted by the source

e
are collimated into a narrow beam with the help of a lead slit (collimator). The collimated beam is allowed to

rere
rFF
fall on a thin gold foil of thickness of the order of 2-1 x lO"^ m. The a-particles scattered in different
directions are observed through a rotatable detector consisting of a circular zinc sulphide screen and a
uur r
ffoor
microscope. The alpha particles produce bright flashes or scintillations on the ZnS screen. These are observed
in the microscope and counted at different angles from the direction of incidence of the beam. The angle 0 of
sks
deviation of an alpha particle from its original direction is called its scattering angle 6.
Yoo
Observations. A graph is plotted between the
ooko

scattering angle 6 and the number of a-particles N


eBB

(0), scattered at Z0 for a very large number of a-


particles. This is shown in Fig. 12.4.
The dots in this figure represent the data points of
uurr
ad

the actual experiment. The solid curve is the theoretical


Yo

prediction based on the assumption that atom has a small,


dense, positively charged nucleus. We find that
dY

(0 most of the alpha particles pass straight


Re
innd

through the gold foil. It means they do not suffer any


collision with gold atoms,
FFi

(ii) only about 0-14% of incident a-particles


scatter by more than 1°.
{Hi) About one a particle in every 8000 a particles deflects by more than 90^.
Explanation. The scattering of a-particles is due to Coulombian interaction of a-particles with positive
charges and electrons in every atom of the gold foil. As most of the alpha particles pass straight through the
foil without any deflection, it means they did not suffer any collision. From this, Rutherford concluded that
most of the space in an atom is empty.
An a particle is over 7(X)0 times more massive than an electron, and in this experiment, a particle is travelling
al a high speed, therefore, very strong forces alone could have deflected them through large angles.
This led Rutherford to postulate that the entire positive charge of the atom must be concentrated in a
tiny central core of the atom. This tiny central core of each atom was called atomic nucleus.
214
*The source used in this experiment was g3Bi
ATOMS 12/3

As the gold foil is very thin, it can be assumed that a particles will suffer not more than one scattering
during their passage through it. An a particle carries two units of positive charge and has mass of a helium
atom. Charge on gold nucleus = Ze, where atomic number of gold, Z = 79. As gold nucleus is about 50 times
heavier than an a-particle, we assume that it would remain stationary in the scattering process. Therefore, the
trajectory of a particle can be computed using Newton’s second law of motion and coulomb force of repulsion
between a particle and gold nucleus, i.e..

F =
1 (Zc)(2e)
4tC€
0

where r is distance of a particle from the centre of the


nucleus. The magnitude and direction of the force on
an a particle changes continuously as it approaches

ww
the nucleus first and then recedes away from it.
As shown in Fig. 12.5, an alpha particle (1),
tending to collide head on with the nucleus, slows

Flo
down due to repulsive force of the nucleus, finally
stops and is then repelled back. This a particle,

ee
therefore, retraces its path, scattering through 180“.

rere
Alpha particles 2, T tending to hit the nucleus at its periphery, experience strong repulsive forces and

r FF
get scattered through large angles (9 > 90“)
The alpha particles 3, 3' which pass at a distance from the nucleus, experience small repulsive forces
uurr
foor
and get scattered through small angles. The a particles which pass at still larger distances from the nucleus go
almost undeviated.
ks s
We can show that number of a particles scattered per unit area, N (0) at scattering angle 6 varies
Yoo
oook

inversely as sin*^ (0/2)


1
eBB

Nm oc

sin'^ (0/2)
uurr

Note that electrons being very light, do not affect the motion of alpha particles.
ad
Yo

12.4. DISTANCE OF CLOSEST APPROACH (SrE OF NUCLEUS)


dY

The minimum distance from the nucleus upto which an energetic a particle travelling directly towards
the nucleus can move before coming to rest and then retracing its path is known as distance of closest
Re
innd

approach. This distance is represented by Tq. It gives us the order of the size of the nucleus.
FFi

To calculate this distance, Rutherford made the following assumptions.


(/) The atomic nucleus is so heavy that its motion during the impact is disregarded,
{ii) The nucleus and the alpha particle both are taken as point charges having no dimensions.
(Hi) The scattering is due to elastic collision between nucleus and d-particle.
Suppose an a-particle with initial kinetic energy E is directed towards the centre of the nucleus of an
atom. On account of Coulomb’s repulsive force between nucleus and alpha particle, kinetic energy of alpha
panicle goes on decreasing and in turn, electric potential energy of the particle goes on increasing. At a
certain distance tq from the nucleus, K.E of a particle reduces to zero. The particle stops and it cannot go
closer to the nucleus. It is repelled by the nucleus and therefore, it retraces its path, turning through 180“.
Therefore, the distance tq is known as the distance of closest approach. At this distance, the entire K.E. of
CL particle is converted into electric potential energy.
Now, charge on a particle, q^ = -\-2e ; Charge on nucleus, <?2 = +
where Z is the atomic number of material of the foil and + c is charge on a proton.
12/4 Fundamental Physics (X1I)ESS9Q1

Ze
Electric potential at distance tq due to the nucleus = ...(1)
47teo'i,
]
where = 9x lO'^Nm^C-2,
4t:€
0

Potential energy of alpha particle at this distance (/q) from the nucleus = potential x charge
Ze Ze{le)
x(2e) = ..(2)
471Go Tq
1 T
Kinetic energy of alpha particle of mass m moving with velocity u is E = — mv ...(3)

If we neglect the loss of energy due to interaction of a particle with the electrons, then at the distance of

ww
closest approach, as K.E => RE
1 2 Ze(2e)

FF loo
~mv =

47t€ory

ree
Ze(2e)
'0 = 1
...(4)

rFee
4716 0 mv"
2

oor rF
rur
Obviously, the radius of the nucleus must be smaller than the calculated value of as an alpha
s ff
particle cannot touch the periphery of the nucleus on account of strong repulsion.
Sample Problem In the original experiment, Geiger and Marsden calculated the distance of
k
YYoou
closest approach to the gold nucleus (Z = 79) - of a 7*7 MeV a particle before it comes momentarily to
ookos

rest and reverses its direction. What is its value ? (NCERT Solved Example)
BBo

Sol. In the original experiment, K.E. of a particle,


re

E- = 7-7 MeV = 7-7 x 1-6 x 10 joule, i.e.. E= 1-2 x 10 joule


ouur
ad

Z = 79 for gold and e= 1 -6 x 10“'^ coulomb.


Yo

1 (Ze)(2e) 9x10‘^x79x2(1-6x10“'^)2
From (4), '■() = 4716
Yd

I 1-2x10"12
Re

2
0
idn

— mu
2
FFin

-29
9 X 79 X 2 X1 -6 X1 -6 X10
'() = ’ ^0 = 3-0 X m = 30 fermi
1-2x10"'2
Note that this value is considerably larger than the sum of radii of gold nucleus and a particle. Thus
a particle reverses its motion without ever actually touching the gold nucleus.

12.5. IMPACT PARAMETER

Impact parameter w deifned as the perpendicular distance of the initial velocity vector of the
alpha particle from the central line of the nucleus, when the particle is far away from the
nucleus of the atom. Infact, impact parameter determines the trajectory traced by an alpha
particle in passing through the gold foil.

The angle between the direction of approach of the alpha particle and the direction of scattered alpha
particle is called scattering angle, h is denoted by 0.
ATOMS 12/5

Fig. 12.6 shows the path or trajectory of an alpha FIGURE 12.6

particle in the coulomb field of a heavy nucleus. The Velocity a


impact parameter b and scattering angle 0 are also Vector of
a Particle
shown in the diagram. ='Vj:.e
For large impact parameters, force experienced a

by the alpha particle is weak, because F varies as b


+
l/(distance)^. Therefore, when impact parameter is
Central Line
large, an alpha particle will deviate through a much
smaller angle. However, when impact parameter is
small, force experienced is large and hence the alpha
particle will scatter through a large angle. For the case Nucleus

of head on collision, impact parameter/? tends to zero.


The alpha particle will rebound like a ball thrown

w
against a wall, scattering through 180°.
Fig. 12.7 shows theoretically calculated paths of
alpha particles moving with a speed of 1 -63 x 10^ m/s in

Flo
the coulomb field of a gold nucleus. The gold nucleus
is supposed to be at the origin O of the coordinate

e
rree
system. The values of impact parameters chosen for
alpha particles 1, 2, 3, 4 respectively are 2-5 fin, 10

r FF
fin, 20 fin and 100 For b = 0,a particle 0 retraces
its path, turning through 180°, as shown in Fig. 12.7.
uurr
Rutherford calculated analytically, the relation
between the impact parameter h and scattering angle for
kss
0, which is given by
ooook
Yo
1 Ze^ cot 012
b = ...(5)
47CG E
eB

where E = is kinetic energy of alpha particle, when it is far away from the atom.
urr
ad

'isctission.
Yo

(/) For large values of impact parameter b, cot 0/2 is large. Therefore, scattering angle 0 is small. It
dY

means, when impact parameter is large, scattering angle is small, i.e., a particles passing far away from the
nucleus undergo small deflections,
Re
innd

(n) When impact piu-ameterb is small, cot 0/2 is small. Therefore, scattering angle 0 is large. It means
Fi

alpha particles passing close to the nucleus suffer large deflections.


0
(ill) When b = 0, cot 0/2 = 0, — = 90°, 0 = 180°. Therefore, an alpha particle travelling directly towards

the centre of nucleus will retrace its path.


(/V) If KE (£) of alpha particle is large, impact parameter b shall be small for the same deflection.
A given beam of alpha particles has some distribution of impact piu’ameters (/?). Therefore, the beam is
scattered in various directions with different probabilities.
Sample Problem A 4 MeV alpha particle is scattered through 20°, when it approaches a
gold nucleus. Calculate the impact parameterif Z for gold is 79.
Sol. Here, £ = 4 MeV = 4 x 1-6 x lO"*^ J, 0 = 20°, Z = 79, /? = ?

From ^_ Ze^ cot 0/2 _ 9x10^x79(1-6x10-‘^)2 cot 10°


47te. (E) 4xl-6xl0“‘'-'
12/6 ‘P>iet,cUt^’4. Fundamental Physics (Xll)BSZSD

, 9x79x2-56 ,, „
As lan 10*^ = 0-1763; b = xlO"^^ m = 1-61 X 10“*^ m
6-4x0-1763

12.6. RUTHERFORD S ATOM MODEL


The essential features of Rutherford’s nuclear model of the atom or planetary model of the atom are as
follows:

1. Every atom consists of a tiny central core, called the atomic nucleus, in which the entire positive
charge and almost entire moss of the atom are concentrated.
2. The size of nucleus is of the order of J0~^^ m, which is very small as compared to the size of the atom
which is of the order of IO~^^ m.
3. The atomic nucleus is surrounded by certain number of electrons. As atom on the whole is electrically
neutral, the total negative charge of electrons surrounding the nucleus is equal to total positive chatge on

ww
the nucleus.
4. These electrons revolve around the nucleus in various circular orbits as do the planets around the
sun. The centripetal force required by electron for revolution is provided by the electrostatic force of attraction
between the electrons and the nucleus.

Flo
e
12.7.ELECTR( JRBIT^

reree
If = centripetal force required to keep a revolving electron in orbit,

r FF
Fg = electrostatic force of attraction between the revolving electron and the nucleus, then
for a dynamically stable orbit in a hydrogen atom, F^ = Fg
uurr
V
m —
2
ie){e)
4;ue^0 r"
r foor ...(6)
ks s
Yoo
~>
e~ ■)
ooook

r = e~
2 _
4ti€^0 mv~
or V ...(7)
47t€o0 mr
eBB

1 ^
K.E. of electron in the orbit, K = — mv using (6), K =
2
StiSq r
uurr
ad
Yo

Potential energy of electron in orbit, U =


4tcs«0 r 4ne^0 r
dY

Negative sign indicates that revolving electron is bound to the positive nucleus.
Re
innd

e~ e~
.'. Total energy of electron in hydrogen atom. E = K+ U =
FFi

87T€^ r 47CSq r
.2
E = -
87t£,. r ...(8)
0

Therefore, total energy of electron in orbit of hydrogen atom is negative. Hence, the electron is
bound to the nucleus, i.e., the electron is not free to leave the orbit aroundthe nucleus.

12.8. ;> SC SPECTRA

When an atomic gas or vapour at low pressure is excited usually by passing an electric current through
it. the gas/vapour emits radiations of certain specific wavelengths only. A spectrum of this kind is called Line
emission spectrum and it consists of a few bright lines on a dark background.
When white light is passed through the same gas/vapour, we observe a bright background crossed by a
few dark lines signifying the missing wavelengths or the wavelengths that are absorbed by the gas. They
lorm the Line Absorption Spectrum. It was found that missing wavelengths are the same as the wavelengths
present in the emission spectrum of the gas/vapour.
ATOMS 12/7

The fact that every gasivapour has its own characteristic line emission/absorption spectrum shows
that the line spectra serve as finger prints for identification of the gas.
Fig. 12.8 shows emission lines in ihe spectrum of hydrogen.

FIGURE 12.8

E E £
E
eI
c c c
CD CO c
CM CD o h-
●>- CM CD CD CM CO
05 1- CO CD CO

ww
Lynrian Balmer Series Paschen Series
Series

Flo
ee
12.9. SPECTRAL SERIES OF HYDROGEN

A close look at Fig. 12.8 shows that the spectral lines are in groups. Further, the spacing between lines

eer
within certain sets of the hydrogen spectrum decreases in a regular way. Each of these sets is called a spectral

FFr
series. ,

oorr
uur r
FIGURE 12.9
Balmer was the first to observe one such spectral series in ihe
s ff H ■X' Ha Hy H a
visible region of the hydrogen spectrum. It is called Balmer series
and is shown in Fig. 12.9. The spectral line with largest wavelength,
sk
YYoo
656-3 nm in the red region is called ^/^ line, the next line with X =
ooko

486-1 nm in the blue, green region is called line, the next line
eBB

with X = 434-1 nm in violet region is called Hy line and so on. Table


12.1. The spacing of successive lines and their intensity CD CM T- CO
goes on ●
« c
E
5^
M- CD
CD CO in
decreasing.
uurr

CO CO

Balmer Series
ad
Yo
dY

TABLE 12.1. Spectral lines of Balmer Series


Re
innd

S. No. NAME OF LINE COLOUR OF LINE WAVELENGTH


FFi

1. Ha Red 656-3 nm

2. H Green 486-1 nm
P
3. H Blue 434-1 nm
7
4. Violet 410-2 nm
H5

Balmer found an empirical formula to account for these wavelengths:


1 1 1
- = R -
●)
,’ where n = 3, 4, 5,.... ...(8)
X \ -
n~

where R is a constant = 1-097 x 10^ m-1 . It is called Rydberg constant and n is an integer having values 3,4,
5, ...etc.
12/8 Fundamental Physics (XII) kWII

For n - 3, eqn. (8) gives


1 1
- = l-097xl0'^ = 1-5236 X 10^ m-1
X [2^ 3^
X - 656-3 nm

which is the wavelength of //„ line.


Similarly, for « = 4, we get X = 486-1 nm and so on. For n = ; we get X = 364-6 nm, which is the limit
oo

of the Balmer series. Beyond this limit, there are no further distinct lines. Instead, the spectrum becomes
continuous, though faint.
Later on, Lyman series was discovered in the ultraviolet region of the hydrogen spectrum. It is
represented by
1 1 1
- = R — , where n = 2,3, 4,....
X l2 n^

w
Paschen Series was discovered in the infrared region of the hydrogen spectrum. It is represented by

Flo
1 1 I 'I
- = R where n = 4, 5, 6,.

reee
2 ’
X 32 n

FFr
Another spectrum, called Brackett series was discovered in the infrared region of hydrogen spectrum.
It is represented by
urr
1

X
= R
1

42
1
for
— , where n = 5, 6, 7,....
n
kkss
Yo
And in far infrared region of hydrogen spectrum, there was yet another spectral series called Pfund
oooo

series, represented by
eB

1 1 1
- = R — . where n = 6,7, 8,... .
X 52 «2
r
ou
ad

These formulae are useful as they give the wavelengths X and hence
YY

. ( o
frequencies - -r that hydrogen atoms radiate or absorb. However,there
. \ ^J
nndd
Re

IS no reasoning why certain speciOc frequencies/wavelengths alone are


observed in the hydrogen spectrum.
Fi

12.10. LIMITATIONS OF RUTHERFORD ATOM MODEL

In Rutherford atom model, an electron is revolving around the nucleus


and is constantly experiencing a centripetal force. Therefore, the electron
has an accelerated motion. According to classical electromagneti c theory,
the electron must radiate energy in the form of electromagnetic waves.
As the revolving electron loses energy continuously, it must spiral inwards
and eventuUy fall into the nucleus, as shown in Fig. 12.10. Thus,
Rutherford atom model does not explain the stability of the atom.
Further, according to classical electromagnetic theory, frequency of em waves emitted by revolving
electron = frequency of revolution of electron. As the revolving electrons spiral inwards, their angular velocities
and hence their frequencies of revolution would change continuously. Therefore, frequency of em waves
emitted must change continuously. Therefore, atoms should emit continuous spectrum, but what we observe
is only a line spectrum.
ATOMS 12/9

12.11. BOHR MODEL OF HYDROGEN ATOM


In view of the limitations of Rutherford atom model, Niels Bohr came to the conclusion that classical
mechanics and electromagnetism could not be applied to the processes on the atomic scale. Bohr cleverly
combined classical ideas and early quantum concepts (given by Planck) to give what is known as Bohr Model
of hydrogen atom. Following are the three basic postulates of this model:
1. Every atom consists of a central core called nucleus, in which entire positive charge and almost entire
mass of the atom are concentrated. A suitable number of electrons (luiving as much negative charge as the
positive charge on the nucleus) revolve around the nucleus in circular orbits. The centripetalforce requiredfor
revolution is provided by the electrostatic force of attraction between the electron and the nucleus.
If m is the mass of electron moving with a velocity v in a circular orbit of radius r, then the necessary
2
mv

centripetal force is F =
r

loow w
Also, the electrostatic force of attraction between the nucleus of charge (+2e) and electron of charge
(- e) is

F =
1 {Ze){e) KTaP- where K =
1

4tcg r2 r2 47t£
0

ee
0

Fr
mir KZ* e^ r FF ...(9)
r r2

rer
2. According to Bohr, electron can revolve only in certain discrete non radiating orbits, called stationary
fofr Fo
u
orbits, for which total angular momentum of the revolving electron is an integral multiple of h/2%, where
ks
h is Planck's constant.
YYouro
Thus the angular momentum of the orbiting electron is quantised.
s oo

As angular momentum of electron = mvr, :. for any permitted (stationary) orbit


BBook

nh
r ee

mvr = ...(10)
2k
ouru
ad

where n is any positive integer, 1,2, 3


It is called principal quantum number. The quantum condition (10) limits the number of allowed
Yo

orbits. The electron, while revolving in such orbits, .shall not lose energy i.e., its energy would stay constant.
3. The emission/absorption of energy occurs only when an electron jumps from one of its specified
d
Re

non -radiating orbits to another. The difference in the total energy of electron in the two permitted orbits is
iYn

absorbed when the electron jumps from an inner to an outer orbit, and emitted when electron jumps from
FFind

outer to the inner orbit.

If Fj is total energy of electron in an inner stationary orbit and Ej is its total energy in an outer stationary
orbit, then frequency V of radiation emitted on jumping from outer to inner orbit is given by
hv = Ej - Ey ...(11)

On the basis of these three postulates, we calculate the following :


(a) Radii of Bohr’s stationary orbits
nh nh
From (10), mvr — or v =
2k iKinr
2 /
m n^h^ KZe'^ n^h^ n

Put in (9), or ...(12)


r 4tc
A 1 ')
m^r-
“>
t
.2 4n^m KZe^ Z AK?-mKc-

♦For hydrogen atom, Z = 1, as it contains only one proton.

I
12/10
4. Fundamental Physics (XII) P;»Tll

1
This shows that /● o« rr and /● oc —

i.e., radii of stationary orbits are in the ratio P: 2^: 3^: and so on Le. 1:4:9. Clearly, the stationary
orbits are not equally spaced.
The variation of radius (r) of stationary orbit of hydrogen atom with the principal quantum number is
shown in Fig. 12.11.
Putting h ~ 6-6 x 10"^'^'^ joule-sec., m = 9-1 x 10“^' kg, y
FIGURE 12.11

A:=9x 10‘^Nm2(r^f'= 1-6 X 10 19 ^


we get from eqn. (12) r = «2x5.29x 10-'' m

For example, size of 1st orbit (// =1) of hydrogen atom is


/
r= 1 x5-29x 10“" m = 0-529 A = 5-29 X 10“’^ m

loow w
X
Eqn. (12) can be rewritten as 0 -► n

-II n~
=(5-29x10 m) ...(13)
Z

(l») Vehxity of electron in Bohr's stationary orbit

ee
nil

Fr
KZe~
From (9), r = r FF
')
From (10),
mir 2 TC mv

2nKZe^

rer
KZe- nh

mv^ 2nmv
or
fofr Fo V =
nh
...{14)
u
For hydrogen atom, Z = 1
ks
2-jtKe^
YYouro
' V =
s oo

...(15)
nh
BBook

Calculations show that in the ifrst orbit (n = l) of hydrogen atom, orbital


r ee

FIGURE 12.12
velocity of electron is 2'2 x 10^ nils which is roughly of the velocity of
ouru
ad

light in vacuum. V
Yo

Further, the orbital velocity of electron in outer orbits is smaller as


compared to its value in the inner orbits. (●.● V oc l/«)
d

The variation of speed (u) of an electron in a stationary orbit of hydrogen


Re

> n
iYn

atom with the principal quantum number (n) is shown in Fig. 12.12.
FFind

Sample Problem Find the wavelength of electron orbiting in the first excited state of
hydrogen atom. (CBSE 2017)
Sol. Here, X = ? u = 2 for first excited state

2izKe-
V —
nh

. h, nh nh-
mv 2nm Ke~ 2nm Ke^

X =
2(6-6x10-3‘*)2
2x3-14x9x10-3'(9x10^)(1-6x10-"^)2
2x6-6x6-6xi0 ^ m
X = = 6-69x10-'“ m
6-28x81xl-6xl-6

1
ATOMS 12/11

(c) Frequency of electron in Bolir's stutioiiury orbit OO YOU KNOW ?


It is the number of revolutions completed per second by the
We can rewrite eqn. (15) as
electron in a stationary orbit, around the nucleus. It is represented
by V
V =
iKKe- 2k Ke^ 1 c
From v= rt£>- r{lKv) nh ch n

V InKZe^ KZe^ c
V = using (15), v = a —

Inr nh-2nr nhr n

w
I
2nKe-
where a =
KZe^ ch
= constant,
i.e.. V = ...(16)
nhr called fine structure constant.

In the first orbit of hydrogen atom, n=l, r = 0-53 x 10 m 2x314x9x10^ (1-6x10 -19,2
)

roow
a =

e
(3xlO^)(6-66xlO"^'^)
using^=9x 10‘^Nm-(r2,Z= l,/i = 6-6x IQ-^^Js,

re
e = 1-6 X !0“'^C. we get a =
1

137
9xI0^xlx(l-6xl0~^^)^

reF
uFFll
V’ =
-10 = 6‘57 X 10^^ rps In the first stationary orbit of

e
(6-6xl()--’‘^)x(0-53xl0
hydrogen atom, orbital velocity of
1 electron.
As r oc rr, therefore from (16),

sFr
V oz —
3
n
c 1 3x10^

foro
V - a- = X
The frequency of electron in subsequent stationary orbits is 137
uor
fk
n

smaller. = 2-2 X 10^ m/s


okso
(d) Total energy of electron in Bohr’s stationary orbit
The energy of elecrtron revolving in a stationary orbit is of two types :
Y
Yo
Kinetic energy which is due to velocity and potential energy which is due to position of electron.
oo
BB

mxr KZe^
From the first postulate of Bohr’s atom model, —
rYree
ouu

1
— mv^
^ 1 KZe^ i.e., K.E of electron = — mv~ = ...(17)
ad
Ydo

2 2 r 2 2r

KZe
nidn

Potential due to the nucleus, at any point in the orbit in which electron is revolving = r
Re

Potential energy of electron = potential x charge


F
Fi

KZe{-e) ~KZe^ ...(18)


r r

1 KZe- KZe- KZe'^


Total energy of electron in the orbit. E = K.E. + RE. =
2 r r 2r

n 2/i2 2Tt:“m K^e"^


Putting from (12), we get. £=- ...(19)
^ 4K^mKZe^ /i2 n
2

Substituting the standard values, we get


y2 21-76x10"^^z2 Z2
£ = -(21-76xlO-’^)^J = n n 2x1-6x10-'9
eV =-(13-6) —eV
n
...(20)
12/12 Fundamental Physics (XII)EEIHII
For hydrogen atom, Z= 1
13-6
£ = -
n
^eV ...(21)

Eqn. (21) shows thoitotal energy of electron in a stationary orbit is negative, which means the electron
is bound to the nucleus and is not free to leave it.
Further, asn has discrete values (1,2,3,...), therefore energy of an electron in hydrogen atom is quantised.
This is called Bohr’s energy quantisation.

Retain in Memory
In ground state of hydrogen atom (Z = \ ,n= 1), From (17) and (18),
Ke^

ww
and P.E. - - = -2K.E.
2 r r

I Ke^ Ke^ 1 Ke-


Total energy, E = K.E. +££.=- — = -K.E.
2 r 2 r

Flo
r

Thus in a hydrogen atom, K.E. = E; .PE. =-2 E, .TE. - - E

ee
As total energy of electron revolving in any orbit is negative, the revolving electrons are bound to

eer
the nucleus.

FFr
The energy of the atom is least (= largest negative value) when n - 1, i.e., when electron is revolving in

oorr
uur r
an orbit closest to the nucleus. This state of lowest
energy of the atom is called ground state. The energy of
s ff
13-6
this state is, E,1 =- eV = -13-6eV . Therefore, the minimum energy required to remove the electron
l2
sk
YYoo
ooko

from the ground state of hydrogen atom is 13-6 eV. This is called ionisation energy of hydrogen atom.
As n increases, value of negative energy decreases, i.e., energy is progressively larger in the outer orbits.
eBB

These are called Excited States.

Retain in Memory
uurr
ad

Note that stationary orbits of electrons are not equally spaced. Their radii are in the ratio
Yo

l2; 22; 32 .... In outer stationary orbits, velocity of electrons and their frequency of
revolution decrease. Total energy of electron in outer orbits is more than that in inner orbits.
dY
Re
innd

Sample Problem Calculate the radius of the third Bohr orbit of hydrogen atom and the
FFi

energy of electron in that orbit.


Sol. Here, r = ?, n = 3, £ = ?

As r =

mk e-

32 (6-6x10-^)2x9x10^
r —
02 22
= 4-775-x '10-‘Om= 4-775 A"
4x
7
X—
7
x9-lxl0"^^ (1-6x10“'^)2

-2tc2 mk^
E =

On putting the standard values, we get £ = - 2-43 x 10“^^ J


ATOMS 12/13

(o) Origin of spectral lines FIGURE 12.13

At room temperature, most of the hydrogen atoms are in Photon


ground state. When a hydrogen atom receives energy by processes hv
such as electron collisions or heat, the atom may acquire sufficient
energy to raise the electron to higher energy states, i.e., from n = 1
to n = 2, 3, ... . The atom is then said to be in an excited state.
From these excited states, the electron can fall back to a state of
lower energy-emitting thereby a photon of particular energy
(= difference in energies of the two .states). Fig. 12.13.
Suppose
£] = total energy of electron in the inner (n]th) orbit.
£2 = total energy of electron in the outer (n2th) orbit.

ww
When an electron jumps from an outer to an inner orbit, the energy of radiation emitted, according to
Bohr’s third postulate, is hv = E2 - E\

Flo
mK^ 2k^ mK^Z^e^
hv = -
Using (19),

e
7 I
nr /r

ree
1

Fr
he 2nhnK-Z~e‘^ 1 I

rF
/l2 2 2
uurr n n
1 2 J

X ch^
1
__
s
__
1
for ...(22)
nf «2j
kks
Yo
1
oooo

Now, — = V , the wave number of radiation emitted i.e., number of complete waves in unit length.
X
eB

2tcWV I
= /?^ a constant called Rydberg constant.
cl?
ur

Putting the values of various lenns in the above relation, we get the value of Rydberg constant
ad
YYo

£= 1-097 X 10"^ m-^


dd

1 1
Re

.*. From (22), v=£Z2 —


in

2
n n
1 2 J
F

For hydrogen. Z= 1 V =R 1T 1_7 ...(23)


n
1

Eqn. (23) is called Rydberg formula for the spectrum of hydrogen atom. Clearly, wavelengths/
frequencies/wave numbers of radiations emitted by the excited hydrogen atom are not continuous.
They have specific values depending upon the values of Rj and ti2.
Sample Problem In hydrogen atom, wavelength of emitted photon will he minimum in which
of the following transitions ?
(0 n - 2ton = 1; (ii) n = 6 to n = 5 (/«)« = 3 to « = 2 (Mi Vt.T 2(H2)

Sol. Wavelength of emitted photon will be minimum when frequency and energy of photon are maximum.
For maximum energy, the energy difference of the two transitions should be maximum. This occurs between
n = 2 to n= 1.
12/14 ■ ' ■ Fundamental Physics fXin P7STWT1
12.1: BOHH S EXPLAIN \» M>N OE SPECTRA’ SERIES Or HYDROGEN ATOM

Long before Bohr gave his theory of hydrogen atom, various scientists had observed experimentally, the
spectral series of hydrogen atom as detailed in Art. 12.9. Bohr offered a theoretical explanation of these
spectral series as follows :
t I ^ »njlM '● ■

Bohr postulated that Lyman series is obtained when an electron jumps to the first orbit (/ij = 1) from any
outer orbit (fh “ 2, 3, 4
Wave numbers of spectral lines of Lyman series were calculated using eqn.(23), i.e.

v=«i-4
Ll-
where k = 2,3,4.

The longest wavelength of the spectral lines of Lyman series is emitted when transition of electron takes

w
place from /t2 = '^ = 2 to «| = 1
1
J

Flo
= R
22 J 4
I.C.,

reee
4 4
= 1-216 X 10-2 m= ]2I6A

FFr
max
3/? 3x1-097x102
The shortest wavelength of the spectral series of Lyman series is emitted when transition of electron
for
ur
takes place from ii2 = k = to = I
kkss
I 1 1 1 1
I.e., = R
K, = 0-911 X 10-2 111 = 912 A
Yo
l2 R 1-097x102
ooo

cc
min
inin
eB

These values of v lie in the ultra violet region of the spectrum and agree well with the values of v
observed experimentally by Lyman.
' Halmcr S^r li
r
ou
ad

According to Bohr, Balmer series is obtained when an electron jumps to the second orbit {n | = 2) from
any outer orbit (u2 = 3,4,5...).
YY

Wave numbers of these spectral lines were calculated using eqn.(23), i.e.
nndd
Re

1 1
V = R - , where /: = 3, 4, 5
22 k^
Fi

Tliis set of spectral lines lie in the visiblepart of the spectrum.


Proceeding as above, we can prove that
Xr,B =6563A and =3646A
max mm

.V ruHiM ii ' t II

According to Bohr, Paschen series is obtained when an electron jumps to the 3rd orbit (n, =3) from any
outer orbil (/?2 = 4, 5, 6...). Bohr calculated the wave numbers of spectral lines of Paschen series from the
reiatioii ;

1 I
v=R , where k = 4,5, 6.
[32 *2
These values of v lie in the infra red region of the spectrum and agree well with values of v observed
i xpcrimcnlally by Paschen.
ATOMS 12/15

Proceeding as above, we can prove that FIGURE 12.14

^P.
max
= 18751 A

and
^Pmin
= 8220 A

Bohr also predicted two new series in the infra


red region of the speclrum, which were observed later
by Brackett and Pfund. Paschen'Se ries

4. llracketf Series

According to Bohr. Brackett series is obtained


when an electron jumps to the 4th orbit («] = 4) from
any outer orbit («2 = 5, 6, 7...).

w
5.1'fiiiul Series

According to Bohr. Pfund series is obtained when


an electron Jumps to the 5th orbit (/Jj = 5) from any

Flo
n =7
outer orbit (/J2= 6, 7, 8...).

reeee
The various spectral series of hydrogen atom are represented in Fig. 12.14.*
Table 12.2 give.s the longe.si wavelengths shortest** wavelengths (\,j„) of some of the

FFr
spectral series of hydrogen atom.
TABLE 12.2. Wave length limits (X-^ax some spectral series of Hydrogen
for
ur
Spectra! Series Lower state Upper state ^max. Lower state Upper state X min.
kkss
1. Lyman 1 2
Yo
1216 A CO 912 A
oo

2. Balmcr 2 3 6563 A 2 OO 3646 A


eB

3. Paschen 3 4 18751 A 3 OC
8220 A

12.13. ENERGY LEVEL DIAC‘


r
ou
ad

A diagram which represents the total energies of electron in different stationary orbits of an
YY

atom is called the enctgs ' of that atom. In this diagram, total energies of electron
in various stationary orbits are represented by parallel horizontal lines drawn according to
ndd
Re

some suitable energy scale.


Fi

The vertical line connecting any two states represents the transition of tlie electron from one to the other of
these states. Difference of energies of two states gives the amount of energy emitted/absorbed according as the
electron goes from higher to lower energy state or from lower to higher energy level, in an atom.
Total energy of electron in /ith orbit of hydrogen atom is

-In-mK-e''
£ = ...(24)
n h~

On substituting the standard values, we get as shown in Art 12.1 l{d),


13-6
£ = - eV
2 ...(25)
n

*Circiilar orbits shown in Fig. 12.14 are only symbolic. The.se are not drawn to scale as r « n~
**Wavelength emitted will be shortest when 112 = OO

I
12/16 Fundamental Physics (XII) VOL.II

Putting n =: 1,2,3 we get the energies of electrons in various stationary orbits as :


13-6 13-6 13-6
= -l-51eV
^=-32
= -13-6eV = -34eV
^2=-3.
£,1 =-
12
13-6 13-6 13-6
= -0-54eV = -0-37eV
£4=- 42 = -0-85eV
^.=-52 ^6=- 62
13*6
= -0-28eV
72
Clearly, as n increases, E„ becomes less negative until at n = «>,£„ = 0.
The energy levels of hydrogen atom are represented in energy level diagram. Fig. 12.15. The principal

ooww
quantum number (n) labels the stationary states in ascending order of energy. The highest energy state
13-6
corresponds to n =«» and has energy E = — — = 0eV ● This is the energy of the atom, when the electron is
00
2

removed (r = <») from the nucleus and the electron is at rest. As n increases, energies of the excited states
come closer and closer together as shown in Fig. 12.15.

e
ree
Note that an electron can have any total energy above E = 0e.V In such a situation, the electron is free.

rFl
And there is a continuum of energy states above E = 0 e.V

Fre
The various spectral series of hydrogen atom have also been drawn in Fig. 12.15. From this diagram, we can

rrF
calculate the eneigy, frequency, wave number etc. of any line of the various spectral series of hydrogen atom.

sffoo
ouur
FIGURE 12.15
Series limit
oksk
n =6
I-0-8SW
I Brackett series
Yo
n=^
oo

-20 Paschen
Y
I series
BB

-30
,-3-4 eji n=2
rre

Baimer series
-40
I
ouu

-50i
Y
ad

I
-60f
dY

■S- -7 0
I
innd
Re

-80
ui I
Fi
F

I
-100(
- 11-0
I
-120

-130
^^eV, n=l
-140—- Lyman series

’ t *
|s>m1pi^igrio6Tem| Calculate the longest and shortest wavelength in the Baimer series of
hydrogen atom. Given Rydberg constant = 1*0987 x 10^ m~^.
Sol. The wavelength (A.) of different spectral lines of Baimer series is given by
1 1 1 1
- = R
22
X
4.
ATOMS 12/17

Longest wavelenth is of line or 1st line of the series, for which 1x2 = 3

i- = l-097xl0^ -1-4
2^ 3^
= 1-097 xlO'^x —
36

w
36 36x10^^
X = m
A“ = 6563 A"
5x1097x10’ 5x1-097x10’
For shortest wavelength, tij = 00

1 1 1 1-097x10’

wr
e
- = 1097x10’
X 2^ 00
2
4

r
oo
4x10’*^ A'
X = m = = 3646 A"

F
llu
1-097x10’ 1-097x10’

1Z14. DE BROGLIE'S EXPLANATION OF BOHR'S SECOND POSTULATE OF QUANTIZATION

FF
The second postulate of Bohr atom model says that angular momentum of electron orbiting around the

ees
nh
nucleus is quantized, i.e., mvr = , where n = 1, 2, 3. Louis de Broglie explained this puzzle in 1923, ten

rr
uro
Fk
2tu
years after Bohr proposed his model. According to de Broglie, the electron in its circular orbit, as proposed

oo
by Bohr, must be seen as a particle wave.
We know that when a string fixed at two ends is plucked, a large number of wavelengths are excited. But

fofr
only those waves which have nodes at the two ends form the standing waves and survive. It means that in a
so
Y
string, standing waves form when total distance travelled by a wave down the string and hack is any integral
oY
number of wavelengths. Waves with other wavelengths interfere with themselves upon reflection and their
B

amplitudes vanish quickly.


ok
Yo
Hence, according to de Broglie, a stationary orbit is that which contains an integral number of de
eBr

Broglie waves associated with the revolving electron.


rue

For an electron revolving in nth circular orbit of radius r^,


d

total distance covered = circumference of the orbit = 2 7C /●„


o

For the permissible orbit, 2n r^ = nX


ou
ad
n

h
According to de Broglie, X=
iY

mv

where is speed of electron revolving in nth orbit.


nd
Re
F

nh nh
2nr.n =
Fi

or r = = n(/i/2 n)
mv 2tc

i.e., angular momentum of electron revolving in nth orbit must


be an integral multiple of h/2%, which is the quantum condition
proposed by Bohr in second postulate.
Thus, the wave particle duality plays the central role in the structure
of the atom. Fig. 12.16, shows a standing particle wave on a circular
orbit for n = A, i.e.,In r„ = 4 X.
Sample Problem The velocity of electron in inner most orbit of hydrogen atom is 2*2 x 10^
m/s. Use de-Broglie hypothesis to calculate radius of innermost orbit.
Sol. Here, f = 2-2 x 10^ m/s.
de-Broglie wavelength associated with this electron,
h 6-6 xlO-^'^
X =
-31
= 0-333 X 10-^ m
mv (9x10 ) (2-2x10^)
12/18 Fundamental Physics (XII)

If r is radius of the orbit, then according to de-Broglie hypothesis.


X 0-333x10-'^
2 Tir = 1 ; /● = = 0-53 X m = 0-53 A
27t 2x3-14

12.15. LIMITATIONS OF BOHR S THEORY

Following are some of the limitations of Bohr's theory.


1. This theory is applicable only to simplest atom like hydrogen, with Z = 1. The theory fails in case of
atoms of other elements for which Z > 1 .

2. The theory does not explain why orbits of electrons are taken as circular, while elliptical orbits are
also possible.
3. Bohr’s theory does not explain Ihc ifne structure of spectral lines even in hydrogen atom.

ww
4. Bohr’s theory does not say anything about the relative intensities r>/spectral lines.
5. Bohr’s theory does not take into account the wave properties of electrons.

Floo
6. Bohr’s atomic model does not give any indication regarding the tuTangement and distribution of
electrons in an atom.

ee
7. Bohr's model could not explain the splitting of spectral lines under the effect of magnetic field

eer
(Zeeman Effect).

FFr
8. Bohr’s model could not explain the splitting of spectral lines under the effect of electric field {Stark
Effect).

oorr
uur r
s ff
12.16. HYDROGEN LIKE ATOMS

As is known, hydrogen atom has a single proton in its nucleus and a single electron is revolving around
sk
YYoo
ooko

the nucleus in a circular orbit.

An atom consisting of a nucleus of charge + Ze (where Z is the atomic number of the atom) but
eBB

having a single electron revolving around the nucleus is called Hydrogenlike atom.
For example :
uurr

(/) singly ionized Helium atom (2W<^’^) has Z = 2 : and a single electron revolving around the nucleus, as
ad

out of two electrons, one has been removed.


Yo

(//) doubly ionised lithium atom has Z= 3, and a single electron revolving around the nucleus, as
dY

out of three electrons, two have been removed.


Re

We can show that


nind
FFi

r.
{a) Radius (rp of nth orbit of hydrogen like atom r.
Z

For example, for singly ionized ^He"^ atom, radius of first orbit,
(^n)^ 0-53 = 0-265 A.
2

(6) Speed (u') of electron in nth orbit of hydrogen like atom

v'n =Z{v
' /I')

For example, speed of electron in 1st orbit of singly ionised He atom


= Z (Vq) = 2 (2 X 10^) m/s
and speed of electron in 1st orbit of doubly ionised Lithium atom (Z = 3) is
= Z (n„) = 3 X 2 X 10^ m/s
ATOMS 12/19

(c) Energy of electron in nth orbit of hydrogen like atom


13-6Z^
e: = eV
n~

For example energy of electron in 1st orbit of singly ionised He atom


13-6x2-
£o=- eV = -54‘4eV
l2
and energy of electron in first orbit of doubly ionised Lithium atom
13-6x32
£A0 = eV = - 122-4 eV
l2
Note. Some energy levels of singly ionised He atom aie same as that of hydrogen atom. Therefore,

ww
spectrum of light emitted by He"^ atom and hydrogen atom may be same.
Sample Problem D Calculate radiu.s of first orbit of singly ionized He atom, when radius of

Flo
first orbit of hydrogen atom is 0-53 A.

e
0-53 A
Sol. For helium atom, Z = 2 r =^ = = 0-265 A

reree
Z 2

r FF
Sample Problem The energy of electron in 1st orbit of hydrogen atom is - 13-6 eV. What
will be the energy' of doubly ionised 3L12 atom in the first orbit ?
uurr
Sol. For 3Li2. Z = 3 and n - 1 for first orbit E' -I foor
32
= -13.6x—= -122-4eV
ks s
n
Yoo
ooook

12.17. EXCITATION AND lOh


eBB

Whenever an electron revolving in a stationary orbit of an atom absorbs some energy, two things
may occur:
1. The electron may jump over to an outer orbit of higher energy. This process in which absorption of
uurr

energy by an electron takes the electronfrom an inner orbit to some outer orbit of higher energy is called
ad

i xcimthii. The atom is said to be in the excited state. The minimum accelerating potential which provides
Yo

an electron energy sufficient to jump from the inner most orbit (ground state) to one of the outer orbits is
dY

called excitation potential or ; I/ll.I nee p.'y...


For example, in case of hydrogen atom, energies of electron in various orbits are :
Re
innd

£j=-13-6eV. £2 = -3-4eV, £3 = - 1-51 eV and so on, £«, = 0.


FFi

.●. Energy required to raise an electron from ground state (n = 1) to first excited state (n = 2) is
£ = £2 - £j = - 3-4 - (- 13-6) = - 3-4 + 13-6 = 10-2 eV
The corresponding excitation potential = 10-2 volt
Similarly, energy required to raise an electron from ground state (n = 1) to second excited slate
{n = 3) is £ = £3-£| =- 1-51 -(- 13-6) = - 1-51 + 13-6= 12-09eV
The corresponding excitation potential = 12-09 volt and so on .
Thus, excitation potential of an atom is not one. It can have many values, depending on the state to
which the atom is excited.

2. If the energy supplied is so large that it can knock out an electron from the atom, the process is called
Ionisation. Thus, ionisation is the phenomenon of removal of an electron from an atom. The minimum
accelerating potential which would provide an electron energy sufficient Just to remove U from the atom is
called Ionisation potmtiu!.
For example, total energy of electron in ground state of hydrogen atom = - 13-6 eV. To remove this
electron from the hydrogen atom, + 13-6 eV energy is required.
12/20 “Pnadce^ 4 Fundamental Physics (XII) kwwii
Ionisation energy of hydrogen atom = 13-6 eV, Ionisation potential of hydrogen atom = 13-6 volt,
13-6z2
The general expression for ionisation potential of an atom is y = volt, where Z is the charge
n
number of the atom and n is number of orbit from which electron is to be removed.
Note that for a given element in ground state, ionisation potential is fixed. But for different elements
in groundstate, ionisationpotentialsare different.
It should be clearly understood that when an element is not in ground state, ionisation energy of the
element will not be unique. Ionisation energy of the given element will vary with the initial excited state of the
element.

For example, if hydrogen atom is in first excited state, £2 = ~ 3 4 eV. Therefore, its ionisation energy =
E^- £2 = ^) - (- 34) eV = 34 eV. Similarly, if hydrogen atom is in second excited state, its energy
£3 = - 1-51 eV. Therefore, its ionisation energy would be £^ - £3 = 0 - {- 1 -51) eV = 1 -51 eV.

ww
12.18. X-RAY SPECTRA : CONTINUOUS AND CHARACTERISTIC
There are two types of X-ray spectra :

Flo
1. Continuous X-ray spectrum,

e
2. Characteristic X-ray spectrum.

eree
{a) Continuous X-ray spectrum is that which consists
ofall wavelengths larger than a certain minimum wavelength,

FFr
The intensity of X-rays forming continuous spectrum

oorr
uur r
varies with the voltage applied across the cathode and target
of X-ray tube as shown in Fig. 12.17. This spectrum does not
sf
depend on nature of material of the target.
Consider an electron of initial KE = Xq that interacts
Yoo
ks
oook

.collides) with one of the target atoms, as shown in Fig. 12.18.


The electron may lose a small amount of energy AX, which
eBB

would appear in the fonn of X-ray photon. This is radiated


away from the site of collision.
uurr

If we ignore the energy transferred to the recoiling target


ad

atom, the energy of scattered electron would be (Xq - AX).


Yo

The scattered electron may have a second collision with


a target atom generating another X-ray photon and so on. This
dY

electron scattering process continues till electron is


Re
innd

approximately stationary. All the photons generated by such


electron collisions form part of continuous X-ray spectrum.
FFi

If incident electron were to lose all its initial KE (Kq) in


a single head on collision with a target atom, a single X-ray
photon of maximum energy/frequency or minimum wavelength
would be emitted.

he he
Xn0 = hv max
min
min
X,0

This is the eut off wavelength of eontinuous X-ray


spectrum, which does not depend upon the target material,
{b) CharacteristicX-ray spectrum is that which consists
of a few particular wavelengths superposed on the continuous
X-ray spectrum in the form of peaks as shown in Fig. 12.19.
These wavelengths are characteristic of the target material of
X-ray tube.
ATOMS
12/21

The peaks arise when an energetic electron strikes FIGURE 12.20


an atom of the target and knocks out one of the atom’s
deep lying electron, creating a vacancy or hole, in this - N Shell
shell (say AT shell, n = 1). An electron in one of the outer
M Shell
shells having higher energy jumps to K shell, filling
Q)
the
vacancy/hole. During this jump, the atom emits a D5 -T
L Shell
characteristic X-ray photon. If the jump is from L shell a>
c

(n = 2), the emitted radiation is line. If the jump


LU
is
from M shell, the emitted radiation is line and so on. Ka
Similarly we obtain L^, Lp lines when electron P

jumps to L shell from M shell and N shell respectively.


Some of the spectral lines are shown in Fig. 12.20. Their

ww
K Shell

wavelengths would depend upon nature of the target


material of X-ray tube.

Flo
e
eree
FFr
I
TYPE I. DISTANCE OF CLOSEST _9x10^x2x80(1-6x1Q-‘9)2
uurr
APPROACH AND IMPACT PARAMETER

orr
8x1-6x10-^^
Formulae used

1. Distance of closest approach (r^) is calculated


sfo
= 2-82 X m

Example 0 Calculate the impact para


kks
Yoo
1
from K.E. of a particle , 1 2e(2e)
oooo

2 _
—trw
2 47te
0 'b
meter of a 5 MeV alpha particle scatterd by 10"
when it approaches a gold nucleus. Take Z-19
eBB

for gold.
Ze^ cot 6/2
2. Impact parameter, b = 1
1
urr

4t:6 — mv- Solution. Here, K.E. = - tnv- = 5 MeV


\1 2
ad
YYo

Units used. Kinetic energy of a particle in joule ; = 5 X I-6x 10"’^ J,


/■q and b in metre. 0=10", 2 = 79,/;=?
dd

Standard values, e = 1-6 x C ;


Ze^ cote/2
Re
inn

(l/47ceo) = 9x IO^Nm-C-2 As b =
4tc€ 1 2
F

~mv^
0
2
Example Q In a Geiger Marsden
experiment, calculate the distance of closest 9x10^x79(1-6x10- ’^)2cot 5°
approach to the nucleus of 2 = 80, when an a- 5x1-6x10-'^
particle of 8 MeV energy impinges on it before it (tan 5" = 0-0875)
comes momentarily to rest and reverses its
direction.
(CBSE 2012) 9x79x1-6x1-6x10-'^
= 2*6 X 10“'3 m
Solution. Here, Fq = ? 2=80, 8x0-0875

1 2
-mv^ =8MeV=8x l-6x 10-*3j Example 0 In a Gciger-Marsden experi
ment, calculate energy of a particle whose distance
1 {Ze){2e) of closest approach to the nucleus of
'b = 4 Tie 1 2 = 79 is 2-8 x m. How will the distance of
2
0 ~ mv closest approach be affected when the kinetic
2
energy of the a-particle is doubled ? (CBSE 2012)
12/22 pfuicUe^'^ Fundamental Physics (XII)EEIgD
-!4
Solution. Here, = 2-8 x 10 m, Z = 79, KZe^ KZe~
7. K.E.= , P.E. = -
£= ? 2r r

1 iZe) i2e) ~KZe^ -13-6


From £ = 8. Total energy =
4tI£ 0 2r
^eV
'b n

1 1
1 2Ze^ 9 X10^ X 2 X 79(16 x 1Q~'^ 9. V = — = R
X
£ = -14
nr
1 ni
47l£n.
0- ^0 2-8x10
27t2m£V
= 1-300 X 10'^^ J where R - = Rydberg’s constant
ch^
-12
1-300x10
MeV =8-125 MeV 10. Spec tral series of hydrogen atom are explained
1-6x10
-13 on the basts of this formula ;

w
As £ is doubled, tq becomes half (i) Lyman Series: nj = 1, /J2 = 2, 3, 4
(/i) Balmer Series : nj = 2, «2 = 3, 4, 5
^ = 1-4 X lO"*'* m
{Hi) Paschen Series : nj = 3, n2 = 4, 5, 6

Flo
2
(iv) Brackett Series : n| = 4, n2 = 5,6, 7
Example 0 In a Geiger Marsden experi (v) Pfund Series: n = 5, n, = 6, 7, 8

ee
1
ment, what is the distance of closet approach to Units used. X, r are in metre ; £ in joule ; h in J-s ;
the gold nucleus of a 7-7 MeV a-particle before it

Fr
m in kg ; u, c in m/s ; e in coulomb ; n, nj, n2 are
comes to rest momentarily and reverses its numbers with no units.
direction ? [CBSE 2015 (C)l Standard values. K = (1/4 te Eq) = 9 x 10^Nm"CT'',
for
ur
Solution. Here K.E. = 7-7 MeV Z = 1 for hydrogen, m = 9 x 10“'^* kg,
= 7-7 X 1-6 X 10^3 J e= l-6x 10^'^C, c=3 X 10** m/s.
h= 6-6 X 10"^ J.S., 1 eV = l -6x 10"*^ J
ks
Z=79 for gold, fQ = ?
Yo
oo

! (Ze)(2e) Example g The energy of electron in


'0 = 47i:e
B

0
(££)
hydrogen atom is = eV, where n = 1,
re

9xl0^x2x(79) (1-6x10-1^)^ 2, 3, Show that (1) the electron in hydrogen


-13
ou

7-7x1-6x10 atom cannot have an energy of- 6-8 eV. (ii) spacing
ad

= 29-5 X 10"*^ m = 29-5 fermi between the lines (consecutive energy levels) within
Y

the given set of observed hydrogen spectrum


decreases as n increases. [CBSE 2008 (C)]
I
nd

TYPE II. BOHR’S ATOM MODEL,


Re

-13-6
RADIUS OF ORBIT, VELOCITY AND ENERGY Solution. Here, £ eV
2
Fi

OF ELECTRON, IONISATION POTENTIAL n

Putting « = 1,2, 3, , we get


Formulae used.
2 nh _ -13-6
eV =- l3-6eV;
1 !^ = mv
2. mvr =
2n
E.1 =
l2
r2 r

-13-6
4. r = ^2 = eV =-3-4eV
3. /iv = £2 - £1 471^ mKZe^ 2^

5. v =
2kKZ€^ c(2nKZe^ = —a
c

£3 =
-13-6
eV =- 1-51 eV,
nh n ch n 32
2% KZe- -13-6
where ot = = fine structure constant
ch ^4 = eV =-0-85eV;E„ = 0.
4^
2

6. £ =
—me
-a
2
Clearly, an electron in hydrogen atom cannot
n^h^ 2n^ have energy of -6-8 eW.
ATOMS
12/23

(ii) As the value of n increases, energy diff. of the atom, would the eartli be closer to or farther
between two consecutive energy levels decreases. away from the sun than actually it is ? The radius of
Example The energy level diagram of earth’s orbit Is about 1-5 x 10“ m. The radius of the
an element is given here. Which transition sun Is taken as 7 x 10* m.
corresponds to the emission of a spectral line of (NCERT Solved Example)
wavelength 102-7 nm ? (CBSE 2008) Solution. Here,
FIGURE 12.21 radius of electron’s orbit lO-lo
-0-85 eV
= \0^
radius of nucleus 10-*5
A
-1'5eV In that event, radius of earth’s orbit = lO^ X
B C
radius of sun = lO^ x 7 x l()*m = 7 x lO’^ m
-3-4 eV
This is more than 100 times greater than the

ww
D
actual radius of earth’s orbit.
It means that the earth would be 100 times
-13'6eV
farther away from the .sun than it actually is.

FF loo
Solution. As ^ = 102-7 nm = 102-7 x 10"^ m. The result implies thatnw atom contains much
Energy of emitted photon greater fraction of empty space than our solar

ree
system does.
^ ^ _ 6-6x10“^^^ x3xl0* Exam
p'® El According to classical electro

rFee
X 102-7x10-^
magnetic theory, calculate the initial frequency of

F
oor r
the light emitted by the electron revolving around
rur
1-935x10-'*
= 1-935X 10-'* J = eV
a proton in hydrogen atom.
1-6x10-'^
s ff
= 12-1 eV (NCJ,.: Solved Example)
Solution. We have studied in theory that
k
YYoou
Corresponding to Transition D
velocity of electron moving around a proton in
ookos

£ = -1-5-(-13-6) eV=n-l eV hydrogen atom in an orbit of radius 5-3 x 10''' m is


BBo

Example 0 In the ground state of 2-2 X 10^ m/s.


hydrogen atom, its Bohr radius is 5-3 x IQ-”
re

m.
According to classical electromagnetic theory,
The atom is excited such that the radius becomes frequency of em wave emitted by the revolving
ouur

21-2 X 10-*l
ad

m. Find the value of principal electron = frequency of revolution of electron around


Yo

quantum number and total energy of the atom in the nucleus


excited state. (CBSE 2013)
V 2-2x10^
Solution. Here, rj = 5-3 x 10“" m ; =1
dY

V =
= 6-6x10*5 Hz
Re

2nr 22
r2 = 21-2 X 10"" m ; ?, £ = ? 2x —x5-3xl0-"
idn

7
As roijp-
FFin

^ \2 Example [0 A 10 kg Satellite circles earth


«2 _'2 21-2x10-"
= 4, once every 2 hr in an orbit having a radius of
5-3x10-" 8000 km. Assuming that Bohr’s angular
momentum postulate applies to satellites just as it
= 2 n, = 2 X 1=2 does to an electron In the hydrogen atom, find the
E -13-6 quantum number of the orbit of the satellite.
E =~L = -3-4 eV
n“ 2-
(NCERT Solved Example)
Solution. Here, m = 10 kg,
Example [!] jn Rutherford’s nuclear model 7'= 2 hr = 2 X 60 X 60 s,
of the atom, the nucleus (radius about r = 8000km = 8x 10^ m
10-*5 m) is analogous to the sun about which the 2nr 22 8x10^
electron moves in orbit (radius about ir*" m) like V =
r
= 2x—x
the earth orbits around the sun. If the dimensions of 7 2x60x60

the solar system had the same proportions as those = 7 X 10 '-’m/s


12/24 pfuuUeft'4. Fundamental Physics (XlDEEgD

According to Bohr’s postulate, mv r = nh/2 n Example m Which state of the triply


27Cmur 2x344xlOx7xlO^ x8xl0^ ionized Be^ ^ has the same orbital radius as that
n =
h 6-6 xlO-^^ of the ground state of hydrogen ? Compare the
energies of two states.
= 5-3 X 10^*5
Solution. Radius of nth orbit is given by
This is the quantum number of orbit of satellite.
Example m Using the Rydberg formula, i.e. r « —
Z
calculate the wavelengths of the first four spectral 4n- mKZe^

w
lines in the Lyman series of the hydrogen For hydrogen, Z- 1, n = 1 in ground state
spectrum. (NCERT Solved Example)
Solution. From Rydberg’s formula,
Z 1

e
roow
be
— = 13-6xl-6xl0
-19 J
For Beryleum, Z = 4, As orbital radius is

re
X
2
n
he same, — = 1

reF
X = Z

uFFll
1 1
21-76x10-'^ rt^=lxZ=lx4=4

e
2 2

n = V4 = 2

sFr
For Lyman series, ri| = I,
Hence, n = 2 level of Be has same radius as n

foro
,A ” 6-6x10'3‘’x3x10^ metre ~ 1 level of hydrogen.

fk
uor
/ 1
1 1
21-76x10-'^ Now, energy of electron in nth orbit is
okso
1 n:^
2 2tc“ mK'^Z^e'^
£ = -
Y
0'9134xn? n^h^-
Yo
^xlO ^ metre
oo
reeBB

(n^ -1)
2
9134«?2
rY

X = A
ouu

^(Be) [Z^/n^] Be _ 16/4 = 4

Put «2 = 2,3,4,5 to gel X = 1218 A ; 1028 A ;


ad

[Z^/n^ln 1/1
Ydo

E
(H)
974-3 A and 951-4 A
pie[@ Which level of the doubly Example m The ground state energy of
nidn

Exam

ionized lithium has the same energy as the ground hydrogen atom is —13-6, eV, If an electron makes a
Re

state energy of the hydrogen atom ? Compare the transition from an energy level -0-85 eV to - 3-4 eV,
F
Fi

orbital radii of the two levels. calculate the wavelength of spectral line emitted. To
Solution. We know, total energy of electron in which series of hydrogen spectrum does this
nth orbit £ oc Z-/n- wavelength belong ?
As [^]Li = t£lH (CBSE 2012, West Bengal Board 2012)
-13-6
Z^ Solution. Here. £ = - 13-6 eV = -.V,
2 n
n^ Li n H
where n = 1.
9 1 J2 A , 13-6
~>
1 Now, -0-85 eV = —j- "T = 0-85 = 16,
ii~ n
1
or = 9 or n = 3 n.! =4
Again radius of nth orbit, -13-6 . 13-6
and -34 eV = = 4,
n~
n

r^(Li) ^[n^/Z]^. _^_3 M2


2 ■■ "2 "-JA
r oc
Z ^1 (H)~[n^/Z] H 1
ti2 - 2
ATOMS
12/25

In transition from n^ to ih. Total energy = E- 13-6+ 12-5 = - 1-1 eV


he 13-6
hv = — = E^-E^_ =-0-85-(-34) i.e.. E = -
2
= -M
II

= 2-55 eV=2-55x 1-6 x 13-6


2 _
n = )24
M
x =
he 6-6xl0-'-*^x3xl0*
2-55xl-6xl0“‘^ 2-55xl-6xl0"’^ n =
4\2A<A
= 4-852 X 10-^ m = 4852 A « = 3

This wavelength belongs to visible region of It goes over to second excited state.
hydrogen spectrum.
(/) When electron jumps from n = 3 to n = 2,
pie [B The total energy of electron

w
Exam
in
he 13-6 13-6
the first excited state of hydrogen atom is about
- 3-4 eV X ^3 ^2 3^ 1-

Flo
(a) What is kinetic energy of electron in this = - 1-51 + 34= 1-89 eV
state ? = 1-89 X 1-6 X l(r*9j

ee
(b) What is potential energy of electron in this /te¬

Fr
state ? X = m
1-89x1-6x10"^9
(c) Which of the answers above would change

for
if the choice of zero of potential energy is changed ?
ur
6-6x10-3'^ x3x 10^
Solution. We know Kinetic energy of electron I-89xl-6xl0“*^
ks
KZe^ K2£- X = 6-5476 xl0~’
and RE. of electron = _ m
Yo
2r
oo

r
This is first line of Balmer series
P.E. = - 2 (Kinetic energy) (»■) When electron jumps from n = 3 to « = 1.
eB

In this calculation, electric potential and hence 13-6 13-6


- = E
potential energy is zero at infinity. X 3 -^l=- 32 l2
ur

Total energy = RE. + K.E.


ad

= - 1-51 + 13-6= 12-09 eV


Yo

= -2KE + KE = -KE

(a) In the first excited state, total energy he


— = 12-09x1-6x10-^9;
d

= - 3-4 eV K
Re
in

KE = - (-3-4 eV) = + 3-4 eV


he
{b) RE. of electron in this first excited state x =
F

12-09x1-6x10"^9
= - 2 KE = - 2 X 3-4 = - 6-8 eV

(c) If zero of potential energy is changed, K.E. 6-6x10-24x3x108


does not change and continues to be + 3-4 eV. However,
12-09x1-6xI0-19
the P.E. and total energy of the state would change
with the choice of zero of potential energy. = 3-068 X lO-"^ m

Example A 12*5 eV electron beam is uscd This is second line of Lyman series.

I
to excite a gaseous hydrogen atom at room
temperature. Determine the wavelengths and the
corresponding series of the lines emitted.
TYPE III. TYPICAL EXAMPLES
I
Example m A 12-09 eV electron beam ISi:
(CBSE 2017)
used to bombard gaseous hydrogen at room
Solution. Energy of hydrogen atom in ground
state = - 13-6 eV. temperature. Upto which energy level, the hydrogen
atoms would be excited ? Calculate the wavelengths
'4. Fundamental Physics (XII) EESP
12/26 _
of the second member of Lyman series and second \ 42
16
- 4-86 X 10 ^ m
member of Balmer series. (CBSE 2014) 3xl-097xl0^
-13-6 = 486 nm
Solution. As £n , energy required to
n
Example [B Calculate the ratio of the
excite the hydrogen atom from ground state (n = 1) frequencies of radiation emitted due to transition
to second excited state {n = 3) is
of the electron in a hydrogen atom from its (/)
-13-6 |^-13-6^ second permitted energy level to the first level and
A£ = £3-£, = 3“ (11) highest permitted energy level to second
permitted level. [CBSE 2018 (C)]
= - 1-51 + 13-6 = 12-09 eV
Hence hydrogen atom would be excited to third Solution. Here — = ?
energy level (n = 3) or second excited state.

ww
'’2
For second member of Lyman series.
(0 /ivj = £2 - £|
«,1 = 3 and n-) - 1
r-13-6^ -13-6
8

Flo
1 1
= 1-097x10'^ —- . = 1-097 x-xlO"^ 22 J \
12
9
X l2 3^-J

e
.11 = -3-4+ 13-6= 10-2 eV.

eree
5^3, = 1-025 X 10-2 ^ 102-5 nm (jl) /1V2 = £00 - ^2
For second member of Balmer series

FFr
\ /
/I,1 = 4. = 2 f-n-6 -13-6
= 0 + 3-4 = 3-4 eV

oorr
uur r
00
2 22
1 ●7(1 1
= 1-097x102
sf
X
42 22 42 ^ hv.
= 121 = 3
sk
3-4
3 V2 /IV2
Yoo
= 1-097x102 X —
oook

16
eBB
uurr

1 Dalton had postulated that matter is made up of atoms, which are indivisible. Thomson was the first to
ad

suggest a structure for an atom. According to him, an atom is a positively charged sphere of radius
Yo

= 10“^^ m in which the mass and positive charge of the atom are uniformly distributed. Inside the sphere,
electrons carrying equal negative charge are embedded like seeds in a water melon. This model failed as it
dY

could not explain the origin of spectral series of hydrogen atom.


Re

2. Large angle scattering of alpha particles led Rutherford to the discovery ot atomic nucleus, the central
innd

core of every atom. The entire positive charge and almost entire mass of the atom are concentrated in the
FFi

nucleus Electrons carrying equal negative charge revolve around the nucleus in circular orbits. This model
of atom postulated by Rutherford failed because an electron revolving around the nucleus would lose
energy continuously and radius of its orbit would go on decreasing and ultimately electron would fall into
the nucleus, which is not the case. Further, as electron would lose energy continuously, the atoms should
emit electro-magnetic waves over a continuous range. But atoms emit only discrete frequencies /wavelengths
only.
3. From alpha ray scattering experiment, Rutherford calculated distance (ro) of closest approach of a particle
to the nucleus. This gave him the order of size of nucleus. At r = Tq, whole of K.E. of alpha particle is
Ze
1 2 _ (2e),
converted into potential energy i.e. — -
4Tceoro
where (Ze) is charge on nucleus and (2 e) is charge on an alpha particle.
Yet another parameter that determined the angle of .scattering was impact parameter (/?)—which is the
perpendicular distance of the velocity vector of alpha particle from the central line of the nucleus, when
particle is far away from the atom.
ATOMS
12/27

1 Ze^coiQH
b =
47ce 0 (\ 2I
12
4. Bohr improved upon Rutherford model postulating that electrons can revolve around the nucleus in certain
discrete, non radiating orbits, called stationary orbits, for which total angular momentum of ervolving
electron is an integral multiple of Nln, where h is Planck’s constant.

i.e. nh
mvr =
(0

w
2jc

where n = 1,2,3... is called principal quantum numbe.r


Further, when an electron jumps from an outer stationary orbit to an inner stationary orbit, difference in the
total energies of electron in the two orbits is radiated in the form of a spectral line

roow
e
i.e. hv = — -E -K
X "^2 1 m

re
Also, the centripetal force required by electron in moving in a circular orbit is provided by the force of
attraction of nucleus on the electron.

reF
uFFll
2
1 Ze{e) KZe'^

e
mv
i.e.
●(<«)
r 4kEq r2 r2

sFr
From equations (/), («) and (Hi), we can calculate radius of stationary orbit as

foro
uor
fk 4t^mKz^
okso
This shows that r oc fi 2 i.e. radii of stationary orbits are in the ratio 1:4:9: 16...
Velocity of electron in Bohr’s orbit
Y
Yo
oo
BB

V = 27tifZc^ “
c(lnKZe'^\ =-a
c where a =
2'tiKZe'^ 1
nh ch = fine structure constant =
ny n ch 137
Total energy of electron in an orbit
rYree
ouu

„ _ -27i2mA'2z2g4 1 ^2 _i3.g
^ " -3-electronvolt
ad
Ydo

The origin of spectral lines is explained in terms of the formula


nidn

1 1

L"i "IJ
Re

1
F

where v ^ _ wave number of radiation emitted = number of complete waves in unit length
Fi

27c2m^:2^4
R =
= 1.0973 X 10’ m
-1
and is called Rydberg constant.
ch^
The various spectral series of hydrogen atom were accounted for by Bohr as follows :
(0 Lyman series is obtained when an electron jumps to first stationary orbit from any subsequent stationary
orbit (i.e. n, = 1, /ij = 2, 3, 4,...). Similarly, for
(«■) Balmer series, = 2,
«2 - 3,4, 5... (iii) Paschen series, «, = 3, «2 = 4, 5, 6...
(iv) Brackett series, n^ = 4, /J2 = 5, 6, 7... (v) Pfund series, n^ =5, /12 = 6,7,8...
Total energy of an electron in a stationary orbit is calculated from the erlation
-13-6
E = -eV
n

Atomic masses are measured in a.m.u. (atomic mass unit) 1 a.m.u. = 1.66 x 10~2^ kg.
Electron volt is unit of energy. 1 eV = 1.6 x j ^ jq6 eV = 1.6 x IQ-l^ joule.
From E = nup-, we prove that 1 a.m.u. = 931 MeV
12/28 p^tadecfr '4. Fundamental Physics (XlDCglBll

©>@301^' I QL3ESXIONS

Based on NCERT Book

6. The series of hydrogen spectrum which lies in


1. Multiple Choice Questions visible region is
1. The radius of the innermost electron orbit of a (n) Lyman series (b) Balmer series
hydrogen atom is 5-3 x 10“’' m. The radius of /i = 3 (c) Paschen series (d) Brackett series
orbit is 7. Which of the following quantities has the same
-10 -10
(a) 1-01 X 10 m (b) 1-59 X 10 m dimension as those of Planck’s constant ?
-10 -10
(c) 2-12 X 10 m (d) All X 10 m
(a) angular momentum

ww
{CBSE Sample Paper 2022-23) (b) torque
2. The velocity of electron in innermost orbit of (c) energy {d) linear momentum
hydrogen atom is 2 2 x lO*’ m/s. The de Broglie 8. Total energy of elechon in nth stationary orbit of
wavelength associated with electron is

Flo
-10
hydrogen atom is
-10
ib) 6-66 X 10

e
(a) 3-33 X 10 m m
13-6 13-6

ree
(c)2-33 X 10
-10 m {d) 1-33 X 10 m
-10
(«)
n
— Joule {b)
n
^eV

Fr
(CBSE 2022)
13-6

rF
3. In the above question, radius of the innermost orbit (c) eV {d) Joule
uurr ●
n~
IS

(a) 1-06 A
(c) 0-053 A
(b) 0-53 A
(d) 1-53 A {CBSE 2022)
s for
9. The order of velocity of electron in ground state of
hydrogen atom is
(a) 1-097 X 10-'^ ms-* {b) 1-097 x lO'' ms
-1
kks
4. The graph which depicts the results of Rutherford
Yo
(c) 1-097 X 10-"^ ms'* {d) none of the above
oooo

gold foil experiment with a particles is, where 6 is


scattering angle and Y is number of scattered ct 10. The diameter of the first stationary orbit of
eB

particles detected hydrogen atom is of the order of


{a) 0-5 A (b)lA
(c) 1-5 A (d)2A
ur

11. Which one of the following is the famous Bohr’s


ad
YYo

quantum condition for angular momentum (/) ?


nh h
} =
dd

I = (b)
(fl) 2k 9tu
Re
in

h n
I =
F

/ = id)
ic) 2k 2k

12. The minimum energy required to knock out an


electron completely out of the atom is called
(a) Kinetic energy ib) Potential energy
(c) Ionisation energy id) Excitation energy
13. Which spectral series of hydrogen lie in the UV
region ?
(a) Balmer ib) Paschen
5. In the line spectrum of hydrogen atom, difference
between the largest and shortest wavelength of (c) Lyman id) Brackett
Lyman series is 304 A. The corresponding 14. The ratio of the speed of the electron in the first
difference for the Paschen series is Bohr orbit of hydrogen atom and the speed of light
ia) 10553 A ib) 9768 A is equal to (where, e, h and c have their usual
ic) 11686 A id) 12453 A meanings)
ATOMS 12/29

(a) 2 K hc!^ (b) delink (Hi) Rutherford predicted a lot of empty space between
(c) hllKc (d) e^!2 SqHc the atomic nucleus and revolving electrons
15. The orbital radius of the electron in the hydrogen (zv) Total energy of revolving electrons in an atom is
atom changes from r to 4 r. The energy of the orbital positive
electron will change from £ to (a) (0 and (zv) (b) (zz) and (zv)
(a) £/2 (b)m (c) (z), (zz) and (zV) (d) (/) and (zz) and (Hi)
22. For Balmer series if X max is 6563 A, then
(c)2E (d)4E
16. Which of the following statements are true ? wavelength of second line for Balmer series of
(z) Thomson postulated that every atom consists hydrogenspectrum will be :
(a) 4861-5 A (b) 3646 A

oww
-10
of a positively charged sphere of radius =io m

(zz) Thomson model failed to account for large


(c) 4253-2 A (d) None of the above
angle scattering of a-particles from thin metal foils,
23. The relation governing Balmer series is
(zzz) Rutherford postulated that almost entire mass 1 1 1 ^
= R
and positive charge of atom are concentrated in 2 —2 » where/Zj = 2 ; ZZ2 = 3,4,...
«2 j

e
central core of atom, called Nucleus,

re
FFrlo
/
1 1 1
(iv) Size of nucleus is of the order of 10"^^ m. = R
2 -j , where n, = 1 ; «2 = 2, 3, 4,...
(a) (i) and (zv) (b) (ii) and (zv) «2 j

rF
ee
(c) (Hi) and (zv) (d) (i), (ii) and (zzz) 1 f I 1 ^
17. The ratio of size of atom to size of atomic nucleus (c) - = R ——y , where /i, = 3 ; /Z2 = 4, 5, 6,...

rF
ouru
is of the order of A. yni n2j
(a) 10’ (b) 10-5 (d) None of the above
(c) 102 (d) 10-2
fosor
24. The standard value of Rydberg constant is
skf
18. The number of a particles scattered per unit area (a) 1-097 X 102 cm-‘ (b) 1-092 x 10-^ cm"*
ooko
at scattering angle 6 varies as (c) 1-097 X 10’ m-‘ (d) 1-907 x 10-^ m“^
Yo
40
Y
40 25. For an electron revolving around the nucleus of an
(a) cos — (b) sin'^^2
Bo

2 atom.
reeB

(c)
1
(d) None of the above .2 ^2
(a) P.E. = - ; K.E.=
sin 47i:eo r Sjceq r
ooY
uur

<2^
.2
ad

19. The correct expression for impact parameter (b) is (b) P.E.= ; K.E.=
8ji€o^ 4T16o^
dY

1
Ze^ cot-
1 Ze^cot^2
(a) (c) P.E.= : K.E. = -
47ieo (1—mv
innd

4jceo
(i_.) 4jiGor ’
Re

47tGo r
V4
^2 .2
Fi
F

1
Ze^cot-
1 Ze^cot^^2 (d) P.E. = -
87CGo r 4tcgq r
; K.E.=

(C)
47tGo (\—mv 2 4jtGo nw
2 26. The ground state energy of electron in case of
\2 is

20. The wavelength of the first line of Balmer series (a) 13-6 eV (b) - 13-6 eV
of hydrogen atom is A.. What will be the wavelength (c) 30-4 eV (d) - 122-4 eV
of the same line in doubly ionised lithium ? 27. The ratio of radii of orbits corresponding to first
(a) A/2 (b)X!% and second excited states of hydrogen atom is
(c)X!9 (d)mi (a) 1 (b) 1:2
21. Which of the following statements are true ? (c) 2:3 (d) 4:9
(0 Rutherford atom model cannot explain stability of
28. The irmermost orbit of the hydrogen atom has a
the atom diameter 1-06 A. The diameter of the tenth orbit is

(«) Rutherford atom model predicts continuous atomic (a) 5-3 A (b) 10-6 A
spectrum. (c) 53 A (d) 106 A

*
12/30 ^fveuUe^’^. Fundamental Physics (XlI)CZsISD
29. In the Bohr model of hydrogen atom, let R, V and 38. The energy level diagram of an element is given ;
E represent the radius of the orbit, the speed of the which transition corresponds to the emission of a
electron and total energy of the electron spectral line of wave length 102-7 nm
respectively. Which of the following quantities is
FIGURE 12(Q).2
proportional to the quantum number n ?
-0-85 eV
(fl) ElV (b) R/E
A
{c)WR {d)RE -1-5eV
30. The ionisation potential of hydrogen atom is B C
Z -3-4eV
(fl) - 13-6 eV ib) 13-6 eV
D
(c) - 13-6 V (d) 13-6 V
-13-6 eV
31. When alpha particles are sent through a thin gold
foil, most of them go straight through the foil,
because (a) A (b) B (c) C (^/) D

w
(«) Alpha particles are positively charged 39. For the shortest wavelength present in the paschen
(b) Mass of alpha particle is more than mass of 1 1 1
series of spectral lines — = R

Flo
electron 2
n
1
(c) Most of the part of an atom is empty space

ee
{d) Alpha particles move with high velocity (a) «2 = 3, nj = 00
(b) /I2 = <», H] = 3
32. The existence of positively charged nucleus was (c) /l2 = 3, H] = 1 (d) /I2 = 00, H] = 1

Fr
established by
{a) Bohr’s model of H-atom li. Assertion-Reason Type Quest! ms

for
ur
{b) Positive ray analysis Direction. For question numbers 40 to 47, two
(c) a Scattering experiment statements are given, one labelled Assertion (A)
(d) Thomson’s model of atom and the other labelled Reason (R). Select the
ks
correct answer to these questions from the codes
Yo
33. Given the longest wavelength in Lyman series as
oo

1240 A, the highest frequency emitted in Balmer (a), (b), {c) and (rf) as given below :
series is (a) Both, A and R are true, and R is correct
eB

(fl)8x 10^“* Hz ib) 8 X 10*2 explanation of A.


(b) Both, A and R are true, but R is not the correct
(c) 8x10*0 Hz (J) 8 X 10^ Hz
ur

explanation of A.
34. Kinetic energy of electron in hydrogen atom is
ad

(c) A is true, but R is false.


Yo

c2 ^2
(a) ib) (d) A is false, and R is also false.
87ce„0 r
0 40. Assertion. In a-ray scattering experiment, most of
nd

the a-particles go undeflected.


Re

.3 e2
(c) id) Reason. Most of the space in an atom is empty.
Fi

Sree^
0
r 37ce«0 r
41. Assertion. When impact parameter of a-particle
35. Energy required to excite an electron in hydrogen
is zero, the a-particle travelling directly towards
atom from its ground stale to its first excited state is
the centre of the nucleus retraces its path.
(a) 6-2 eV {b) 3-40 eV
Ze- coiG
(c) 10-2 eV (d) -13-6 eV Reason. Impact parameter, b =
36. What is the angular momentum of an electron Atzg
0

revolving in the 3rd orbit of an atom ? .2


(a) 31-5 X 10-^ J.sec {b) 0-315 x 10,-34 J.sec where the various terms have their usual meanings.
(c) 3-15 X 10^^^ J.sec {d) 315 x 10 ●34 J.sec 42. Assertion. If the total energy of electron in the first
37. The ground state energy of Hydrogen atom is excited stale of hydrogen atom is - 3-4 eV, then its
- 13-6 eV. What is the KE of an electron in the 3rd potential energy in this state is - 6-8 eV.
excited state ? Reason. Total energy of electron in the first orbit
{a) - 3-4 eV {b) 1-51 eV is double than that of potential energy of electron
in that orbit.
(c) - -85 eV {d) OeV

4
ATOMS 12/31

43. Assertion. Total energy of revolving electron in Reason. Balmer means visible, hence series lies
any stationary orbit of an atom is negative. in the visible region.
Reason. Energy can have positive or negative 46. Assertion. Rutherford atom model accounts for
value.
stability of atom.
44. Assertion. If the radius of the second Bohr's orbit
Reason. A revolving electron loses energy
of hydrogen atom is 3 A, the radius of the fourth
stationary orbit is 12 A. periodically.
Reason. Radius of inh Bohr’s stationary orbit, 47. Assertion. Stationaiy orbits of electrons are equally

w
where n is Principal Quantum number. spaced.
45. Assertion. Balmer series of hydrogen spectrum lies Reason. Orbital velocity of electron in outer orbits
in the visible region of electromagnetic spectrum. is larger as compared to its velocity in inner orbits.

e
row
re
ANSWERS

I. Multiple Choice Questions

FFllo
eeF
1. (d) 2. (a) 3. {b) 4. (c) 5. (a) 6. (b) 7. (a) 8. (c) 9. id) 10. ib)

u
11. (a) 12. (c) 13. (c) 14. (d) 15. (b) 16. (d) 17. (fl) 18.(c) 19. (b) 20. (c)
21. (d) 22. (a) 23. (a) 24. (c) 25.(a) 26. (d) 27. id) 28. id) 29. (c) 30. id)

r
sFr
31. (c) 32. (c) 33. (fl) 34. (/j) 35.(c) 36. (c) 37. ib) 38. id) 39. (fl)

kro
II. Assertion-Reason Type Questions
uor
40. (a) 41. (c) 42. (c) 43. (6) offo
44. (fl) 45. (c) 46. id) 47. id)
kos
Y
Yo
eerBB

HINTS/EXPLANATIONS For Difficult Questions


oo

I. Multiple Choice Questions


rY

5. According to Bohr’s theory of hydrogen atom,


1. Here, r, = 5-3 x 10"** m fl j = I ; T2 = 7 Il2-3 c
u

2 X =
As rocn 1
ou

^2 = 32
ad
do

X r
1
= 9x5-3x 10"" m
2

= 4-77 x 10-^*> m
nY

For Lyman series, Aj = 1, fl2 = (for shortest


6-6x10-24 wavelength)
2. X= —
nid
Re

mv (9x10"2*)x2-2x10^ c
F

= c
Fi

= 3-33 X 10-*« ni
l2-
,2
3. According to de Broglie hypothesis.
333x10"*® and for largest wavelength of Lyman series
1 X=2nr. r = — (flj = I,fl2 = 2)
2n 2x3-142
c 4
= 0-53x 10"'® m = 0-53 A ^2 =
4. According to Rutherford, number of a-particles 1^-
02
scattered is

4c- c
AX — X-y — X, —
-c = -j =304A;c- = 912A
fl e>c

(sin 0/2)4 2 * 3
rt is maximum, when 0 is small For Paschen series in j = 3, A2 = ~ for shortest A,)
For 0 71, fl tends to zero. c
X. = = 9c
The correct graph that depicts the result is I " J 1
option (c). 32 00“
12/32 ‘P%«zd6e^'4. Fundamental Physics (XII) VOL.II

and for largest wavelength 12. This energy is called Ionisation energy.
c 144
13. Lyman series of hydrogen atom lies in the U.V.
^2 - c region.
1 1 7
14. Velocity of electron in the first Bohr orbit of
32 42 hydrogen atom is
144 81 InKe^ 1 ^2 e
= -X, = c-9c = — c V, = = 2icx
2 1 7 7 * h 2e.h
4jc€q h 0

81
● e2
= yx912 =10553A
V
1

c 2e«0 he
6. Balmer series lies in visible region.
15. For orbital radius of hydrogen atom r «=
E

ww
7. Planck’s constant h = — 1
1
V
and Eoc— Eoc-
n~ r

Floo
Angular momentum = nwr = M (LT~^) L 1
=
Hence
£, 4r 4

ee
8. In nth stationary orbit, total energy

eer
13-6
E, E
or E = — [V E^=E\
£ = -
5-eV

FFr
4 4
n

oor r
ur r
16. Statements (/), («) and (Hi) are correct. Statement
9. Velocity of electron in nth stationary orbit of
(iV) is wrong, because size of the atomic nucleus
s ff
hydrogen atom is
is about 10“^^ m.
iKKe^
sk
YYoou
V Size of atom 10-10
oooko

n
nh 17. = 10^
Size of nucleus 10“^^
eBB

For ground state, n = 1


1
InKe^ 18. V(0) oc

Ui = ● 4fe^
uurr

sin
h
\2j
ad

2x3-14x(9xl0^)x(l-6xl0~^^)2
Yo

1
Ze^cot-
2
(6-63 X10"^^) 19. Impact parameter, b =
dY

= 2-2 X 10® ms-'


Re

U
ind

10. Radius of the first stationary orbit of hydrogen


FFin

atom is 1
20. From Bohr’s theory of atomic spectra X OC

Z2 ■
For lithium, Z = 3.
4jc2 m Ke^
. ^Li (^h)^ 1 1
(6-62x10-34)2
4x(314)2x(9x10-3>)x(9x10^)
. _ ^
x(1-6x10"‘9)2 [V Xh = X]
^Li - 9 - 9
0-5 X 10-^0 m = 0-5 A 21. Statement (1), («) and (Hi) are true.
Diameter = 2 rj = 2 x 0-5 = 1*0 A Total energy of revolving electron is negative.
11. Bohr’s quantum condition is angular momentum 22. From Bohr’s theory of atomic spectra
nh 1 1 1 ^
2n X
^ U? n|2 J
ATOMS 12/33

For Balmer series of hydrogen atom R


2
1 n
= «4.
1 1 1 ^ V l/n E MrP-
where /I2 = 3, 4, 5 n

X 22
V 1 1

X is maximum if «2 = 3 (i.e., first line of Balmer


V xR = -xrp- =n ; RxE = n^x 2=1
n n
series)
Thus option (c) is true.

...(0
30. Ionisation energy of hydrogen atom =13-6 eV.
’ ’ 6563 [22 32 J 36

w
The ionisation potential of hydrogen atom
For second line of Balmer series, «2 = 4. 13*6eV
= 13-6 V
1 _J 1 1^3^ ...(H)
e

■' X [2^ 42 J 16 31. This happens because most of the part of an atom

e
roow
Dividing (/) by (i7), we get is empty.

re
32. Large angle scattering of a particles led to the
1/6563 ^ 5/36 or A. = —X—X6563 existence of positively charg^ nucleus.
l/X "3/I6 36 3

reF
uFFll
= 4861-5 A
33. For longest wavelength in Lyman series /ij = 1

e
23. For Balmer series, the correct relation is and ti2 = 2
1
-r{^
^ 2 ±]

sFr
^-R ,
X *■ X max 2 l2 22 4

foro
^2j n
I "2 j
ofk
uor
where = 2 and «2 = 3, 4, 5 ... 4 4
R =
24. The standard value of Rydberg constant is or

/? = 1097 X 102 m-‘.


3xA,
max 3x(1240xl0-'0)
kos
Y
25. For an electron revolving around the atomic For highest frequency emitted in Balmer series,
Yo
reeBB
oo

nucleus. «j = 2, «2 = 00

.2 1 e2 — = cR X- 1 cR
uurY

● V
P.E. = - K.E. = -P.E.=
L22
max 2
4
4jceo ^ ' 2 87C6„ r
00
nun
0

26. In case of 3Li2, Z = 3. =


(3xl0«)^X 4
ad

For ground state, n = 1 4 3 X (1240x10-^0)


doo
nY

-13-6 Z2 -13-6(3)2 = 8-0 X 10^* Hz


E = eV = eV
2 12 34. KE of electron in ground state hydrogen atom
nid

n
Re

= -122-4 eV
IKe^ \ 1 e2 e2
F
Fi

= —X —

27. As r oc n2 -1 = ^ = 1 2 r 2 47C£q r 87ceQr


T2 32 " 9
28. Radius of the nth Bohr (r„) is given by 35. E=E2-E^ -13-6 r-13-6^
n
2 I J
r oc — = - 3-4 + 13-6 = 10-2 eV
n
z

For hydrogen atom Z = 1, so nh 3x6-62x10-34


36. Angular momentum =
2% 2x3-14
a
10 _
2xr.
10 _ (10)2 = 100
D 2xr,1 12
= 3-15 X 10-34 J.S
1
13-6
.-. Dio = 100 Dj = 100x 1-06 = 106 A 37. K.E. of electron =
29. From Bohr’s model of hydrogen atom, R « n2.
n

1 13-6
V o: l/n and E OC eV =l-51eV
n
2 32
12/34 ‘Pn/ulce^'i. Fundamental Physics (XlDCSm
38. For \ = 102-7 nm = 102-7 x 10"^ m It means VE is double than that of TE. When
electron is in the First excited state then n - 2.
6-6xlO“^‘^x3xlO*y
= 12-leV
13-6eV
102-7x10-^ = -3-4eV
Total energy £ = -
2^
For transition D,
£ = £3 - £,= - 1-5 - (- 13-6) = 12-1 eV. Potential energy is double of total energy
39. For shortest wavelength for the given relation, in = 2 £ = - 3-4 X 2 = - 6-8 eV.

Paschen series, /?2 = 3 and aJ| = o* Thus Assertion is true.

43. Here both Assertion and Reason are true but


II. Assertion-Reason Type Questions
Reason cannot explain Assertion.
40. Both Assertion and Reason arc true and
44. Here, Reason is true. Therefore,
Reason is the correct explanation of Assertion.

w
41. Here, Reason is false becau.se impact parameter
is given by.
^4 _ (2)-
(4)2 - li=4orr4 = 4/-2=4x3=l2A
4

Ze~ cot 0/2 Thus Assertion is also true and Reason is the

Flo
h =
1 2"^ correct explanation of Assertion.

e
4716 —mv
0

reee
V - 45. Here Reason is false but Assertion is true, as
Balmer does not stands for visible.

FFr
Ze- cote/2
When h = 0, then 0 =
1 2 46. Here both Assertion and Reason are false as
4716 0 — mv
2 Rutherford atom model could not explain stability

for
ur
0 0
of atom. Also, a revolving electron being
or cot — = 0 or — = 90® or 0=180“
2
accelerated, loses energy regularly.
2
kss
47. Both Assertion and Reason are false as
Thus Assertion is true.
Yo
stationary orbits are not equally spaced and orbital
oo

42. Here, Reason is false as total energy of electron,


velocity of electron in outer orbit is less as
eB

1 Ke'^ Ke- compared to its velocity in inner orbits because


£ = - and potential energy =- V l/n.
2 r r
ur
ad
YYo
d

3. Why is electron supposed to be revolving


Re

1. Atoms & Nuclei


in

around the nucleus ? (Hr. Board 2011)


F

1. For scattering of a-particles at large angles, Sol. If the electrons were stationary, they would fall
only the nucleus of the atom is responsible, into the nucleus due to electrostatic attraction
explain why ? (CBSE 2004) and the atom would be unstable.
Sol. We know that in an atom, an electron is very
light particle as compared to an a-particle. II. Bohr’s Theory
Hence electron can not scatter the incident
4. Show graphically the nature of variation of
a-particle at large angles, according to law of radius of orbit with the principal quantum
conservation of momentum. Hence the result. number. (CBSE Sample Paper 2011)
2. The K.E. of a-particle incident on gold foil is
doubled. How does the distance of closest n 2*2 2
Sol. As r = ●. r « n

approach change ? (CBSE 2012) 47i2 mKZe^


Sol. As is known, distance of closest approach varies The graph of r versus n is a parabola as shown
inversely as K.E. of a-particle. When K.E. is in Fig. 12(Q).3.
doubled, tq is halved.
ATOMS 12/35

FIGURE 12(Q).3 Sol. Here, total energy of hydrogen atom


E = - 1-51 eV
KE = -£:= I-5I eV
r
PE = 2£' = 2(- 1-51) = -3-02 eV
8. The electron in a hydrogen atom is found
-II
typically at a distance of about 5*3 x 10 m

from the nucleus. Which has a radius of


n
1-0 X 10"^® m. Assuming the hydrogen atom
5. The electron in hydrogen atom is initially in to be a sphere of radius 5*3 x 10"*^ m. What
the third excited state. What is the maximum fraction of its volume is occupied by the
nucleus ? (CBSE 2022)
number of spectral lines which can be
emitted, when it finally moves to the ground Sol. Here, radius of atom (r^) = 5-3 x 10"’ ’ m, and
state ? [CBSE 2013(0] radius of nucleus (r„) = 10 x 10"'^ m

w
Sol. For third excited state, n = 4 and for ground Fraction of volume occupied by the nucleus
state n= 1.
r Y 1x10'’^

Flo
Hence, the possible transitions are :
r -II
iij = 4 to nj-= 3, 2, 1 ^5-3x10

e
reee
3 to /if= 2, 1 = 6-72 X 10-'5 = 6-72 x 10“^^ x 100%
= 6-72x 10-*^%

FFr
Hj = 2 to tif= 1
Hence, total number of transitions = 6 9. What results do you expect if a particle
6. Explain the significance of negative energy scattering experiment is repeated using a thin

for
ur
of electron in an orbit. (Hr. Board 20III sheet of hydrogen in place of gold foil ?
Sol. The energy of an electron in the orbit of an atom (CBSE 2022)
kss
,-27
is negative. It shows that electron is bound to Sol. We know that mass of a particle = 6-64 x 10
kg is much more than the mass of hydrogen
Yo
the nucleus. Greater is the value of negative
oo

energy, more tightly the electron is bound to the


(1-67 X 10“^^) kg. Therefore scattering angle
would not be large enough. Therefore a particle
eB

nucleus. Since the negative energy of electron would not bounce back. The size of nucleus
is maximum in first orbit of an atom, therefore, cannot be determined.
the electron is more tightly bound to the nucleus 10. An electron in a hydrogen atom makes
ur

in the innermost orbit. With the increase in the transitions from orbits of higher energy to
ad
YYo

number of quantised orbit, the negative energy orbits of lower energies. When will such
of electron decreases and hence binding of transitions result in (a) Lyman series
nucleus on the electron decreases. This shows (6) Balmer series ? (CBSE 2021)
d

Sol. (a) Lyman series is obtained when an electron


Re

that the electrons in the outermost orbit of an


in

atom are least bound to the nucleus. jumps to first stationary orbit from subsequent
outer orbits.
F

7. The energy of hydrogen atom in an orbit is


(b) Balmer series is obtained when an electron
- 1‘51 eV. What are KE and PE of electron
in this orbit ?
jumps to second stationary orbit from
(CBSE 20221
subsequent outer orbits.

1 1 ● Very Short Answer


11 ● Short Answer
* Long Answer

VERY SHORT ANSWER QUESTIONS Carrying 1 mark

1. Atoms & Nuclei Ans. This model could not explain scattering of a-
parlicle through large angles.
1. Why did Thomson atom model fail ? 2. How is impact parameter related to the
(Pb. Board 2004) scattering angle ?
12/36 ‘Pnaxieefi.'A Fundamental Physics (XII)EEIHn
11. What is the ground state energy of electron
cot 0/2
Ans. b = in case of 3Li^ ?
I
4Tre 0 — mV
2 Ans. Here, Z = 3 n=2
2

^
E
-13-6Z2
— eV =
-13-6(3)2 = -30*4eV
3. Can a hydrogen atom absorb a photon whose 2
n
n 22
energy exceeds its binding energy ?
Ans. Yes, it can absorb. 12. What is the order of radius of helium atom ?

4. What is the value of Rydberg constant ? Ans. It is of the order of 10"^® m.


Ans. R = 1-097 x 10^ m -I 13. What is the order of velocity of electron in a
hydrogen atom in ground state ?
II. Bohr’s Theory
Ans. 10^ ms-1

ww
5. The ground state energy of hydrogen atom 14. Name the series of hydrogen atom which lies
is -13*6 eV. What are RE. and K.E. of electron
in U.V. region.
in this state ? (CBSE 2011, 2010)
Ans. Lyman series is in U.V region.

Floo
Ans. Total energy = E = - 13-6 eV 15. What is the ratio of radii of orbits corres
Kinetic energy T = - E = 13*6 eV ponding to first excited state and ground state

ee
Potential energy V'=-2 7' = -2x 13-6 in a hydrogen atom ? (CBSE 2010)

eer
= - 27-2 eV
22

FFr
6. Write an expression for Bohr’s radius in rj
Ans. — = = 4:1.
l2

oorr
uur r
hydrogen atom. (CBSE 2010) 1
s ff
n2 16. What is the ionisation potential of hydrogen
Ans. r -
47^2 m KZe- atom ? (CBSE 2011)
sk
YYoo
Ans. 13.6 V
ooko

where symbols have their usual meanings.


17. What are the values of first and second
eBB

7. Name the spectral series of hydrogen atom,


excitation potential of hydrogen atom ?
which be in infrared region.
Ans. 10-2 V ; 12-09 V.
[Pb. Board 2009 (C)]
uurr

18. Name the series of hydrogen spectrum which


Ans. Paschen Series, Brackett Series and Pfund
ad

lies in the visible region of electromagnetic


Series lie in infrared region.
Yo

spectrum ? (Raj. Board 2011)


8. When is a
line of Balmer series in the
Ans. Balmer series lies in the visible region.
emission spectrum of hydrogen atom
dY
Re

obtained ? [CBSE 2013(01 19. The wavelength of some of the spectral lines
ind

obtained in hydrogen spectrum are 1216 A“,


Ans. Hq, line of Balmer series is obtained when an
FFin

6463 A** and 9546 A°. Which one of these


electron jumps to 2nd orbit from 3rd orbit of
hydrogen atom. wavelengths belongs to the Pa.schen series ?
9. In Bohr’s theory of hydrogen atom, what is Ans. X = 9546 A“ belongs to the Paschen series.
the implication of the fact that the potential 20. The radius of innermost electron orbit of a
-II
energy is negative and is greater in magnitude hydrogen atom is 5*3 x 10 m. What is the

than the kinetic energy. radius of orbit in second excited state ?

Ans. The implication is that electron revolving in any (CBSE 2010)


stationary orbit is bound to the nucleus, and
cannot leave the nucleus on its own.
Ans.
10. What Is the energy possessed by an electron n 2-1^
for n = oo ?

Ans. Zero. This follows from E


13-6 rj = 9 T| = 9 X 5*3 x 10“** m
n
n
^eV, = 4-77 X 10“*® m
when n = °o.
ATOMS
12/37

SHORT ANSWER QUESTIONS Carrying 2 marks

I. Atoms & Nuclei


II. Bohr’s Theory
1. Why is that mass of nucleus does not enter 6. In a hydrogen atom. If the electron is replaced
the formula for impact parameter but its by a particle which is 200 times heavier but
charge does ? has the same charge, how would its radius
Ans. The scattering of a-particles occurs due to change ? (CBSE 2008)
electrostatic field of the nucleus. That is why 1
charge on nucleus enters the expression for Ans. As radius, r «
impact parameter and not its mass. m

2. In Rutherford scattering experiment, if a When electron is replaced by a panicle 200

w
proton is taken instead of an alpha particle, 1
then for same distance of closest approach, times heavier, the radius would decrease to
200
how much K.E. in comparison to K.E. of a time the original radius.

Flo
particle will be required ? (CBSE 2017,2009) 7. The energy levels of an atom are shown in
Ans. At the distance of closest approach (rQ),

reeee
Fig. 12(Q).4. Which transition corresponds
iZe){2e) {Ze){e) to emission of radiation of (/') maximum

FFr
KE and KE -
a

0 'b
P
4lteo wavelength (iV) minimum wavelength ?
(CBSE 2013)
Clearly,
for
ur
FIGURE12(Q).4

3. Define distance of closest approach and A B


kkss
OeV
impact parameter. ' ■ C D
Yo
-2 eV
Ans. Distance of closest approach is the distance
oo

between the centre of nucleus and the point from 4 5eV


eB

which the alpha panicle appraoching directly


to the nucleus returns.
v
Impact parameter is the perpendicular distance
r

-10 eV
ou
ad

of the velocity vector of the alpha particle from


the central line of the nucleus, when the particle he
YY

1
Ans. As energy emitted, E=— OC ^

is far away from the atom. X X


4. What is the impact parameter for scattering
ndd

for maximum wavelength. E should be


Re

of a-particle by 180" ?
minimum. Transition A for which energy diff.
Fi

Ans. Zero. This follows from the relation E is minimum corresponds to emission of
Ze-cot 0/2
radiation of max. wavelength.
b =
^ n~mv~O' Simihu’ly, transition D for which energy diff.
4ner.
0 E is maximum, corresponds to emission of
7
radiation of minimum wavelength.
5. For a given impact parameter ft, does the 8. Define ionization energy. How would the
angle of deflection increase or decrease with
ionization energy change when electron in
increase in energy ?
hydrogen atom is replaced by a particle
I Ze^ cot 0/2 200 times heavier than electron, but having
Ans. From ft =
47t€
0
£ the same charge ? (CBSE 2016)

e
Ans. Ionization energy is the minimum energy
Forgiven value of ft, when £is increased, cot — required to knock out an electron from an atom.
2
Its value will be different for different atoms.
9
increases ; tan — decreases ; 0 decreases. Ionization energy will also depend on the orbit
2
from which electron is to be removed.
12/38 Fundamental Physics (XII)EuaiJ

When an electron in hydrogen atom is replaced 12. (fl) The energy levels of an atom are shown
by a particle 200 times heavier than electron, in Fig. 12(Q).5. Which of them will result in
but having the same charge, ionization energy the emission of a photon of wavelength
275 nni ?
will not change, as it depends only on charge
and not on mass of particle.
FIGURE 12(Q).5
9. The energy of electron in ground state of A B
hydrogen atom is -13*6 e\. How much energy O.ev

is required to take an electron in this atom C D


from the ground state to first excited state ? -2ev

oww
(CBSE 2009)
-4.5ev
Ans. Energy in ground state E\=- 13-6 eV.
Energy in first excited state, -lOev
-13-6

e
= -3-4eV.
^2 =

FFrlo
2- (b) Wliich transition corresponds to emission

re
Required energy = £2 “ of radiation of maximum wavelength ?

ree
F
= _3.4_(_ 13-6) = 10*2 eV (CBSE 2011)
10. Show that Bohr’s second postulate “The Ans. (a) When X = 275 nm = 275 x 10“^ m

rF
electron revolves around the nucleus only in
he 6-6x10"3‘^x3x10*^
certain fixed orbits without radiating

fsoor
E = hv = —~
ouur
can be explained on the basis of de-
91 X 275x10-^
energy
skf
Broglie hypothesis of wave nature of electron. -17
19-8x10
ooko
(CBSE 2008) eV =4-5eV
-19
Yo
275x1-6x10
Y
Ans. When an electron of mass m is confined to move
Bo

Transition B will result in the emission of


on line of length / with velocity v, the de-
a
reB

Broglie wavelength X associated with electron photon of X, = 275 nm.


is (b) Maximum wavelength has minimum energy.
Transition A provides energy of 2 eV, which is
uur
oY

h _ h _ nh
ad

X = — = - or P = T
mv p
X ~ lUn ~ 21 minimum.
dY

When electron revolves in a circular orbit of 13. The short wavelength limits of Lyman,
Paschen and Balmer series in the hydrogen
radius r ; then 2 I = 2 nr
innd

spectrum are denoted by Xp and Xg


Re

nh nh
P = or pxr = respectively. Arrange these wavelengths in
Fi
F

2Kr 27t
increasing order.
i.e., angular momentum (p x r) of electron is (CBSE Sample Paper 2011)
integral multiple of hl2n. This is Bohr’s
quantisation condition of angular momentum. 1 f 1
Ans. As — 2
11. The electron in the hydrogen atom passes X
'h "22;
from the « = 4 energy’ level to the n = 1 level.
What is the maximum number of photons 1 1
— oc
('.■ «2 =
that can be emitted ? and minimum number ? X
1

Ans. When an electron in hydrogen atom passes from


n = 4 energy level to n = 1 level, max. number
For Lyman series nj = 1
For Balmer series = 2
of photons = 6, corresponding to transitions
4^3;3_>2;2^1;4^2, 3^1,4->1. For Paschen series /ij = 3.
The minimum number of photons can be one Xf^ < Xg < Xyv
only corresponding to the transition 4 —> 1.
ATOMS
12/39

SHORT ANSWER QUESTIONS Carrying 3 marks

1. What are impact parameter and angle of hydrogen atom using De Broglie’s hypothesis.
scattering ? How are they related to each other ? [CBSE 2017(C)] [Art. 12. 14]
[Art. 12. 5] 4. State Bohr’s postulate to define stable orbits in
2. Derive an expression for the potential energy hydrogen atom. How does de Broglie's
and kinetic energy of an electron in any orbit of hypothesis explain the stability of these orbits ?
a hydrogen atom, according to Bohr's atomic (CBSE 2018) [Arts. 12.11 and 12.14]
model. How does P.E. change with increasing 5. What are the limitations of Bohr’s theory of
n ? hydrogen atom ?
[Art. 12. 11] [Art. 12. 15]
3. Derive Bohr’s quantisation condition for 6. Distinguish between excitation potential and
angular momentum of orbiting electron in ionisation potential. (Jharkhand Board 2012)

w
[Art. 12. 17]

LONG ANSWER QUESTIONS Carrying 5 or more marks

Flo
1. Draw a schematic arrangement of the Geiger radius of orbit and the energy of orbital electron

eeee
Miu-sden experiment. How did the scattering of in hydrogen atom.
a particles by a thin foil of gold provide an

Fr
(CBSE Sample Paper 2019-20, CBSE 2014, 2013,
important way to determine an upper limit on
2008, Raj. Board 2012, Karnataka Board 2012,
the size of nucleus ? Explain briefly.
Manipur Board 2011, Jharkhand Board 2011)

for
(CBSE 2009) [Arts. 12.3 & 12.4]
ur
[Art. 12.11]
2. Explain distance of closest approach and impact
parameter with illustrations. (CBSE 2017) 7. Explain the origin of different spectra] lines of
ks
[Arts. 12.4 & 12.5] hydrogn spectrum on the basis of Bohr’s theory.
Yo
3. Give a brief account of Thomson model of atom.
oo

(CBSE 2015, Kerala Board 2012)


What are its limitations ? [Art. 12.2] [Art. 12.12]
eB

4. Describe Rutherford atom model. What are the


8. Name some hydrogen like atoms. Write
drawbacks of this model ? (CBSE 2017)
expressions for radii of stationary orbits : .speed
[Art. 12.6 and Art. 12.10]
ur

of electrons and energy of electrons in case of


5. Using Bohr’s postulates, derive the expression
ad

such atoms. [Art. 12.16]


Yo

for the orbital period of the electron moving in


the nth orbit of hydrogen atom. 9. Explain the concept of minimum wavelength
(CBSE (F), 2017) [Art 12.11] in continuous X-ray spectra. [Art. 12.18]
d
Re

6. State the basic postulates of Bohr’s theory of 10. Discuss brielly the origin of characteristic X-
in

atomic spectra. Hence obtain an expression for ray spectra. [Art. 12.18]
F

CASE-BASED VERY SHORT/SHORT QUESTIONS

CASE I. Dalton postulated that matter is made Large angle scattering of alpha particles led
up of atoms, which are indivi.sible. Thomson was the Rutherford to the discovery of atomic nucleus, the
first to suggest a structure for an atom. According to central core of every atom. The entire positive charge
and almost entire mass of the atom are concentrated in
him, every atom is a positively charged sphere of radius
the nucleus. Electrons carrying equal negative charge
10 m. The entire mass ;md positive charge of atom revolve around the nucleus in circular orbits. This is
are uniformly distributed in this sphere. Inside the Rutherford model of atom.
sphere, electrons carrying equal negative charge are This model of atom failed because
embedded like seeds in a watermelon. Thomson model (/) Electrons revolving around the nucleus would
failed as it could not explain the origin of spectral series lose energy continuously. Radius of orbit would go on
of hydrogen atom. decreasing and ultimately, the electrons would fall into
the nucleus.
12/40 T^naxUe^ '<i Fundamental Physics (XII) iiiajjli

(ii) As electrons would lose energy continuously, CASE 3. Bohr introduced the concept of
atoms should emit electromagnetic waves over a stationary orbits. According to Bohr, electrons in an
continuous range, But atoms emit discrete atom can revolve around the nucleus only in certain
frequencies/wavelength only. discrete non radiating orbits, called stationary orbits.
Read the above paragraph carefully and In such orbits, total angular momentum of revolving
answer the following very short and short electron is an integral multiple of (W2 Tt), where h is
Planck’s constant, i.e..
answer questions :
h
1. What did Dalton postulate ? mvr = n :.(i)
2. What is Thomson model of atom ? 2k

oww
3. What is Rutherford model of atom ? where ;j = 1,2, 3 is called principal quantum
4. Why did Rutherford atom model fail ? number.

Further, when an electron jumps from an outer


CASE 2. From a-ray scattering experiment, stationary orbit to an inner stationary orbit, the

e
Rutherfordcalculateddistanceof clo.sest approach(rg)
difference in total energies of electron in the two orbits

re
of a particle to the nucleus. This gave him the order of is radiated in the form of spectral line.

FFrlo
size of nucleus (rg).
;,v = :^ = £2-£,

rF
At r=rg

ee
K.E. of a particle = P.E.
Also, the centripetal force required by electron in
ouru
rF
1
— mv
2 _
Ze
{2e)
moving in a circular orbit is provided by the force of
attraction of nucleus on the electron, i.e.,
2 47ceg rgy
ffosor
Yet another parameter that determined the angle
os k mv
2
1 Ze Ze~
...m
of scattering was impact parameter {b). It is the r 4ne
0
perpendicular distance of the velocity vector of
ook
Yo
Y
1
a particle from the central line of the atomic nucleus, where K = . From equations (/), (//) and
Bo

when particle is far away from the atom. 4716


0
reeB

1 cot 8/2
(///), we can calculate radius (r) of stationary orbit as
ouY

b =
4 716 0 ri
ur

— mv-
2 4k? m KZe^
ad
Yo

Read the above paragraph carefully and


d

Clearly, r
answer the following very short and short Read the above paragraph carefully and
nidn

answer questions : answer the following very short and short


Re

5. What happens at the distance of closet approach answer questions:


F
Fi

9. What are stationary orbits ?


6. Is the calculated value of (rg) smaller/greater than 10. What is the condition for an orbit to be
the actual size of nucleus ? stationary ?
7. Is impact parameter same as the distance of 11. How is force required by an electron in moving
closest approach of a particle to the nucleus ? in a circular orbit provided ?
8. How does impact parameter vary with KE of a 12. Are the stationary orbits in an atom equally
particle ? spaced ?

ANSWERS

1. Dalton postulated that matter is made up of sphere. Electrons carrying equal negative charge
atoms, which are indivisible.
are embedded in the sphere like seeds in a
watermelon.
2. Every atom is a positively charged sphere of
3. Rutherford discovered atomic nucleus, the
radius 10"'® m. The entire mass and positive
charge of atom are uniformly distributed in this central core of every atom. The entire positive
ATOMS 12/41

charge and almost entire mass of atom are 8. Impact parameter {b) varies inversely as KE of
concentrated in the nucleus. Electrons carrying a particle Higher the KE of a panicle, smaller
equal negative charge revolvearound the nucleus would be the impact parameter.
in circular orbits. 9. Orbits of electrons around a nucleus, which are
4. (0 Electrons revolving around the nucleus would non radiating are called stationary orbits.
lose energy continuously. Radius of orbit would 10. The angular momentum of revolving electron is
go on decreasing and ultimately, the electrons an integral multipleof {h/2 n), where h is Planck’s
would fall into the nucleus. constant, i.e.. mv r = nh/2 %.

(ii) As loss of energy is continuous, atoms should 11. The required force is provided by the force of
attraction of nucleus on the revolving electron,
emit e.m. waves over a continuous range. But
atoms emit only discrete frequencies/wavelengths.
mv- Ze{e)
i.e.,

5. At the distance of closest approach (/q). KE of a

ww
panicle is convened into RE.
12. No. In an atom, stationary orbits are not equally
6. The calculated value of (rg) is some what greater spaced. It is found that radius (r) of stationary
than the actual size of nucleus. orbit varies as n~ where n is numberof orbit, i.e.,

Flo
7. No. the two parameters are not the same. ri : T2 : r3 = I - : 2^ : 3- = I : 4 : 9

ee
rere
CASE-BASED MCQs AND ASSERTION-REASON QUESTIONS

rFF
CASE I. When an atomic gas or vapour at low 2. What do we observe, when white light is passed
uurr
pressure is excited usually by passing an electric current through some gas/vapour?
through it. the gas/vapour emits radiations of certain
specific wavelengths only. A spectrum of this kind is
foor
(fl) coloured bands
ks s
(b) coloured lines
called Line Emission Spectrum. It consists of a few
Yoo
(c) bright background crossed by a few dark lines
oook

bright lines on a dark background.


(d) none of these
When white light is passed through the same
eBB

gas/vapour, we observe a bright background crossed For Question No. 3 and 4, we have given two
statements each, one labelled as Assertion (A)
by a few dark lines, signifying the missing wavelengths
and other labelled as Reason (R).
uurr

or the wavelengths that are absorbed by the gas. They


ad

form the line absorption spectrum. Choose the correct option out of the four
Yo

options given below :


It was found that missing wavelengths are the
dY

(o) Both A and R are true and R is correct


same as the wavelengths present in the emission
explanation of A.
Re

spectrum of the gas/vapour.


innd

(Z?) Both A and R are true and R is not the correct


The fact that every gas/vapour has its own
FFi

explanation of A.
characteristic line emission/absorption spectrum shows
(c) A is true but R is false.
that line spectra serve as finger prints for identification
of the gas. (d) Both A and R are false.
3. Assertion. There is no difference between line
Based on the above paragraph, answer
questions no. 1 to 4 : emission spectrum and line absorption spectrum
of a gas.
1. Every gas/vapour emits certain specified
wavelengths only when an electric current is
Reason. Line emission spectrum of all gases are
the same.
passed through it. This statement is
4. Assertion. Line spectra serve as fingerprints for
(a) always true
identificationof the gas.
(b) totally false Reason. In the line absorption spectrum, missing
(c) sometimes true wavelengths are the same as the wavelengths
(d) can not say present in the emission spectrum of the gas.
12/42 Fundamental Physics (Xll)iaagU]

CASE 2. When an atomic gas or vapour is excited (a) all wavelength


usually by passing an electric current through it. the (b) all frequencies
gas/vapour emits radiations of certain specific (c) certain specific wavelengths
wavelengths only. A spectrum of this kind is called Line {cl) none of these
emission spectrum. It consists of a few bright lines on
a dark background.
6. Empirical relation governing Balmer series is
Balmer found an empirical relation to account for 1
- = /e where
these wavelengths : X 2^ n~

1 ( 1 {a)n=\ {h) n = 2
- = R — , where n = 3, 4, 5
X 2^ n~ (c) 11 =-3 (cl) n=3,4, 5,....

Here, R is constant = 1 -097 x 10^ m '. It is called For Question No. 7 and 8, we have given two
Rydberg constant, n is an integer having values 3, 4. statements each, one labelled as Assertion (A)

w
5. etc. and other labelled as Reason (R).

For n = 3, we gel X = 656-3 nin. This is line. Choose the correct option out of the four
options given below :

Flo
For n = 4, we get X = 486-1 nm. This is line.
(a) Both A and R are true and R is correct
For n = 03, we get X = 364-6 nm, which is the

e
explanation of A.

rree
limit of Balmer series. Beyond this limit, the spectrum
becomes continuous, though faint. Thus Balmer series (b) Both A and R are true and R is not the correct

r FF
lies in visible region. explanation of A.
Later on, Lyman series was discovered in U.V. (c) A is true but R is false.
uurr
region of the spectrum.
Paschen series was discovered in infrared region.
for
(d) Both A and R are false.
7. Assertion. The wavelength of line of Balmer
kss
Brackett series was discovered in infrared region. series is 656-3 nm.
ooook
Yo
Pfund series was discovered in infrared region. Reason. The value talies with the one calculated

Based on the above paragraph, answer from Rydberg’s formula.


eB

8. Assertion. Paschen series, Brackett series and


questions no. 5 to 8 :
Pfund series all lie in infrared region.
5. When an atomic gas or vapour is excited by
urr

passing an electric current through it, the Reason. Wavelengths of all these spectral lines
ad

gas/vapour emits radiations of are greater than the wavelength of red light.
Yo
dY

ANSWERS
Re
innd

1. («) 2. (c) 3. {d) 4.{u) 5.(c) 6. (rO 7. («) 8. (a)


Fi

HINTS/EXPLANATIONS For Difficult Questions

1. The statement is always true. 5. Radiations of certain specific wavelengths are


emitted.
2. When white light is passed through some gas,
we observe bright background crossed by a few 6. For Balmer scries, n = 3, 4, 5
dark lines. 7. Both, Assertion (A) and Reason (R) are true
3. Both, the Assertion (A) and Reason (R) are and Reason (R) is correct explanation of the
false. Assertion.

4. Both, A and R are true and R is correct 8. Both, Assertion (A) and Reason (R) are true and
explanation of A. Reason is correct explanation of the Assertion.
ATOMS 12/43

TYPE I. DISTANCE OF CLOSEST 9. The ground state energy of hydrogen atom is


-13-6 eV. If an electron makes a transition from
APPROACH AND IMPACT
an energy level -0-85 eV to - 1-51 eK calculate
PARAMETER
the wavelength of the spectral line emitted. To
1. A beam of a-particles of velocity 21 x 10^ m/s which series of hydrogen atom does this
is scattered by a gold foil (Z = 79). Find the wavelength belong ?
distance of closest approach of a-particle to the (CBSE 2012) [An.s. 18750 A, Paschen Series]
gold nucleus. For a-particle, lehn = 4-8 x 10. The ground state energy of hydrogen atom is
10"^ C kg~K [Ans. 2-5 X 10-1^ m] - 13-6 eV (/) What are the potential energy and

loow w
K.E. of electron is 3rd excited state ?
2. In a head on collision between an a particle and
a gold nucleus, the minimum distance of (ii) If the electron jumps to the ground stale from
approach is 4 x lO"*"* m. Calculate the energy of the third excited state, calculate the frequency of
alpha particle. Take Z = 79 for gold. photon emitted. (CBSE Sample Paper 2U11)
[Ans. 5*688 MeV] [Ans. (/) - 1-7 eV ; 0-85 eV ; (n) 3 x Hz]

ee
3. An alpha panicle is scattered through an angle 11. At what speed must an electron revolve around

Fr
of 10° on passing through a thin foil of copper r FF the nucleus of hydrogen atom so that it may not
(Z = 29). If energy of the particle is 5 MeV, be pulled into the nucleus by electrostatic
what is the impact parameter ? attraction ? Given, mass of electron = 91 x

rer
10"^^ kg, radius of orbit = 0-5 x 10"^^ m and
[Ans. 0*95 X 10"^^ m] fofr Fo
e= l-6x 10-^9 C.
u
4. An alpha particle of energy 4 MeV is scattered [Ans. 2*25 x 10^ ms"^]
ks
through 180° by a gold foil (Z=79). Calculate the 12. Calculate the frequency of revolution of electron
YYouro
in the second Bohr’s orbit of radius 2-12 A.
maximum volume in which positive charge of
s oo

the atom is likely to be concentrated ? Given, /j = 6-6 x lO"^"^ Js, m = 9 x 10"^' kg.
[Ans. 8*23 X 10*** Hz]
BBook

[Ans. 7*7 X m^]


13. Determine the radius of the first orbit of hydrogen
r ee

5. The number of alpha particles scattered at 60° is


100 per minute in an alpha particle scattering atom. What would be the velocity and frequency
ouru

of electron in this orbit ? Given : h = 6-62 x


experiment. Calculate the number of alpha
ad

particles scattered per minute at 90°. [Ans. 25] 10“^'*J-s. m = 9-1 X 10“-^' kg ; e = 1-6 X 10"'^ C,
Yo

K = 9x](f Nm-(T“.
6. Calculate the impact parameter of a 5 MeV
particle scattered by 90", when it approaches a [Ans. 0*53 A" ; 2*19 x !0^ ms"* ; 6*6 x 10*^ Hz]
d
Re

gold nucleus (Z = 79). [Ans. 2*27 x ni] 14. The wavelength of line for copper is I -36 A".
iYn

7. What is the impact parameter at which the scatte Calculate the ionisation potential of a shell
FFind

ring angle is 90° for Z = 79 and alpha particle of electron in copper. [Ans. 1*22 X 10-* V]
initial KE 10 MeV ? [Ans. 1*14 X 10"*** m] 15. Calculate shortest wavelength of Balmer series.
Given R = 1-097 x 10^ m '. (CBSE 2016)
TYPE II. BOHR’S ATOM MODEL, [Ans. 3646*8 A"]
RADIUS OF ORBIT, VELOCITY 16. The wavelength of line of Balmer series is
AND ENERGY OF ELECTRON, 4861 A. Calculate the wavelength of line of
IONISATION POTENTIAL the series. (CBSE (F), 2015) [Ans. 6562 A]
17. A 12-9 eV beam of electrons is used to bombard
8. The ground state energy of hydrogen atom is
-13-6 eV. If an electron makes a transition from gaseous hydrogen at room temperature. Upto
an energy level - 0-85 eV to - 3-4 eV, calculate which energy level would the hydrogen be
the wavelength of spectral line emitted. To which excited ? Calculate the wavelength of first
series of hydrogen spectrum does this wavelength member of Paschen series and first member of
belong ? Balmer series.
(CBSE 2014)
(CBSE 2012) [Ans. 4853 A, Balmer series] [Ans. // = 4 ; 18750 A ; 6547 A ]
12/44 ‘Pna.dae^ Fundamental Physics (XII) Jil

18. Fig. I2(Q).6 shows energy level diagram of TYPE III. TYPICAL PROBLEMS
hydrogen atom. Find out the transition which
results in the emission of a photon of wavelength 19. A hydrogen atom rises from its n = 1 .state to the
n = 4 state by absorbing energy. If the potential
496 nm. Which transition corresponds to
emission of radiation of maximum wavelength ? energy of the atom in the n = 1 state be 13-6
eV, then calculate
Justify your answer.
(a) potential energy in the n = 4 state,
FIGURE 12[Q).6 {b) energy absorbed by the atom in transition,
n = 4 (c) wavelength of the emitted radiation if the atom
n = 3 returns to its original state.
[Ans. - 0-85 eV, 12-75 eV ; 970 A]
n = 2
20. If elements with principal quantum number
n> 4 did not exist in nature, what would be the

loow w
possible number of elements ? [Ans. 60]
n= 1
21. Which state of triply ionised beryllium
[CBSE (OD), 2015 (C)] has the same radius as that of the ground state of
hydrogen ? Compare the energies of the two
[Ans. £^2 J 4 —> 3] states. [Ans. 2 ; 4]

ee
Fr
rx
r FF
rer
For Difficult Problems
fofr Fo
u
ks
1. Here, v = 2-\ x lO"^ m/s, Z = 79, ^2 _ (sin60V2)‘* _ (sin30°)^ ^ (1/2)^ 1
YYouro
100 {sin90"/2)^ (sin45‘-)‘* 4
s oo

— =4-8x lO'^Ckg-’.
m 100
BBook

N, = = 25
I (Ze)(2e) 4
r ee

As to “ 4 Tie 1 0 6. Here, b = 1
0 — mv~ -13
ouru

2 /f£=5MeV = 5x 1-6 x 10 J
ad

0 = 90^
Yo

2e
{2Ze) Z=79
1 \ m J

^ Ze-cot e/2
d

2
4 Tie
Re

V
0
“ Atz^AKE)
iYn

9x10^x2x79x1-6x10'’^ (4-8xl0‘^)
FFind

(2-1x10'^)2 9x10^x79 (l-6xl0“’^)2cot 45°


5x1-6x10"’3
tq = 2-5 X 10-’4 m b = 2-27 X lO"’** ni
4. Maximum volume in which positive charge of
7. Here. /? = ? 6 = 90°, Z = 79
atom is likely to be concentrated is Vq = E= 10MeV= lOx l-6x 10 -13 J

Calculate Tq from the usual formula. ^ _ Ze“ cot 6/2


5. It can be shown that number of particles N 4Tie. (E)
scattered per unit area at an angle 0 is directly
1
9x10^x79(1-6x10-'^)2 cot 90°/2
proportional to 10x1-6x10"’^
sin4 (0/2)
-16
, 9x79x2-56x1x10
N, sin 6,1 /21’^ b
16
Nl sin 0, /2 = M4 X 10"’4 m
ATOMS 12/45

8. In the given transition, energy emitted,


£ = £2 - = - 0-85 - (-34) = 2-55 eV. /?Z2 = — (0
3?i
he
— = 2-55 X 1-6 X 10-‘9 J, For ionisation of K-shell electron 1
= 1 and
X
ni =
00

he
X =
j =RZ^
m
2-55x1-6x10-'9 r 00

6-6xl0~^'‘x3xl0*
Ionisation energy, E = — = he RZ~

oww
2-55xl-6xl0“‘^ X'

= 4-853 X lO-"^ m
= hcx
4 (663xl0"'-*‘^)x(3xl0^)x4
=4853 A
3X 3x(l-36xlO-'0)
This wavelength belongs to Balmer series of

e
hydrogen atom. = 19-5 X 10-'^ J

re
FFrlo
9. Here, £q = - 13-6 eV, 19-5x10-'^
In the given transition energy emitted. E = eV = 1-22 X 10‘*eV

rF
ee
1-6x10-19
£ = £2 - £j = -0-85 - (- 1-51) = 0-66 eV
Ionisation potential = 1*22 x 10'* volt

rF
he
ouru
— =0-66x 1-6 X 10-19 J 15. Take /»,= 2 and /I2 = 00

X
16. For //„ line, /ij = 2, ^2 = 3
he 6-6x10-3'* X 3x10^
fosor and for line /i| = 2, «2 = 4
X =
skf
0-66x1-6x10-19 1-056x10-19 17. Total energy of gaseous hydrogen
ooko
= (12-9-13-6) eV = -0-7 eV
Yo
= 18-75x10"'^ m
Y
13-6 2 _ 13-6
Bo

= 18750 A
, =-0.7, n
0-7
= 19-4
reeB

n
This wavelength belongs to Paschen Series of
hydrogen atom. n = Vm<5
ooY

10. Here, £, = -13-6 eV


uur

n =4
For third excited state, n = 4
ad

For the first member of Paschen series.


dY

-13-6
● ^4 = 42 = -0-85eV
£=£^-£3=- 42
13-6
13^6 "i
I 32
nind
Re

/C.£. =-£4 = 0-85eV


= -0-85 + 1-51 =0-66 eV
RE. = - 2 (K.E.) = -2 (0-85) eV
F
Fi

= -1-70 eV he
^ =0-66x 1-6 X 10-19]
X
Energy emitted, A£ = £4 - £j
hv = - 0-85 - (- 13-6) eV = 12-75 eV X =
he
m
0-66x1-6x10-19
12-75x1-6x10-19
V = = 3 X 10*5 jj2
6-6x10-3'* 6-6x10-3'* x3xl0^xl0“i A
0-66x1-6-1-10-19
1
14. From the relation — = 18750 A
2
X n and for first member of Balmer series
ri 2 )
For line, «j= 1, «2 = 2 13-6 ( 13-6^ ev
£ = £3 - £2 =
32 22
-1 = RZ^ 1 1 "i =-RZ^
3 „^2
1

■■ X 2^J 4 = -1-51 -1-3-4= 1-89 ev


12/46 ^ftadtep.'^ Fundamental Physics (XII) VOL.II

he he
^ = 1-89 X 1-6 X 10-*^ J (c) A£ = —
X X

X=
he or ^ _ AE
he _ 6-6x10-34x3x10^
l-89xl-6xl0"*^ 12-75x1-6x10->9
= 970x 10-^0 m = 970 A"
6-6x10-34x3x10® 20. Upto quantum number 4, an atom has K, L, M
m
1-89x1-6x10-*9 and N shells. In any shell, max. no. of electrons
is 2 n^. Therefore,
= 6-5476 X 10-^ m = 6547 A
in K shell, max no. of electrons = 2 (1)^ = 2
18. Energy levels of hydrogen atom are in L shell, max no. of electrons = 2 (2)^ = 8
= - 13-6 eV, £2 = - 3-4 eV in M shell, max no. of electrons = 2 (3)^ = 18
£3 = - 1-51 eV, £4 = - 0-85 eV in N shell, max no. of electrons = 2 (4)^ = 32

ww
Energy of a photon of wavelength 496 nm is Total max. no. of electrons in these shells
= 2 + 8+18 + 32 = 60
^_he _ 6-63x10-34x3x10®

Floo
eV =2-5eV Hence, number of elements would be 60
X 496x10-^(1-6x10-^^)
21. Suppose orbital radius of ground state of

ee
As £4 - £2 = - 0-85 + 3-4 = 2-55 eV = £, hydrogen (« = 1) is rj

ee r
therefore, the transition from /i = 4 to n = 2 level Radius of nth orbit of hydrogen like atom is

FrF
results in the emission of photon of wavelength 2
n
496 nm.

oor r
rur
Z
Further, wavelength emitted will be maximum,
ForBe+++,
when energy emitted is minimum. The transition
s ff Z =4

n = 4 to n = 3 level will give maximum n2


k
wavelength as energy emitted is minimum. As r,
= r,, therefore, “^ = 1 o*" « = 2
YYoou
ookos

19. Here, £j= - 13-6 eV ; n = 4, £4 = ? Let energy of electron in ground state of hydrogen
BBo

£ -13-6eV = £|. Energy of electron in nth state of hydrogen


1 _
(«) ^4=-Tt = -0-85eV like atom
re

42 16
Z2 42
(b) Energy absorbed by the atom in transition ^2 = E, =4£,1
2 ^1 “22 ^
ouur
ad

n
from n = 1 to n = 4 is
Yo

A£ = £4 - £1 = - 0-85 - (- 13-6) ^2 = 4
= 12-75 eV ^1
dY
Re
idn
FFin
ATOMS 12/47

WITH
IM'
?i

naT SOLUTIONS

Q. 1. Choose the correct alternative from the clues given at the end of each statement :

w
(a) The size of the atom in Thomson’s model is the atomic size in Rutherford’s model (niiicii
greater than/no different from/much less than)
(b) In the ground state of. , electrons are in stable equilibrium, while in electrons always
experience a net force (Thomson’s model/Rutherford’s model),

e
(c) A classical atom based on is doomed to collapse (Thomson’s model/Rutherford’s model),

row
re
(d) An atom has a nearly continuous mass distribution in but has a highly non uniform mass
distribution in (Thomson’s model/Rutherford’s model),
(e) The positively charged part of the atom possesses most of the mass of the atom in (Rutherford’s

FFllo
eeF
model/both the models).

u
Sol. (a) No different from.
(b) Thomson’s model. Rutherford’s model,

r
sFr
(c) Rutherford’s model.

kro
id) Thomson's model, Rutherford’s model,
uor
(e) Both the models.
offo
Q. 2. Suppose you are given a chance to repeat the alpha particle scatteringexperimentusing a thin sheet
of solid hydrogen in place of gold foil (hydrogen is a solid at temperature below 14 K). What results
kos
Y
do you expect ?
Yo
eerBB

Sol. The basic purpose of scattering experiment is defeated, because solid hydrogen will be a much lighter
oo

target compared to the alpha particle acting as projectile. According to theory of elastic collisions, the
rY

target hydrogen will move much faster compared to alpha, after the collision. We cannot determine the
size of hydrogen nucleus.
u

Q. 3. A difference of 2*3 eV separates two energy levels in an atom. What is the frequency of radiation
ou

emitted when the atom transits from the upper level to the lower level.
ad
do

Sol. Here, £=2-3 eV = 2-3x l-6x


nY

£ 2-3xl-6xl0"*^
As £ = hv = 5-6xlO*-^Hz
nid

6-6 xlO-^'^
h
Re

Q. 4. The ground state energy of hydrogen atom is - 13*6 eV. What are the kinetic and potential energies
F
Fi

of the electron in this state ?


(Ruj. Board 2011, CBSE 2010)
Sol. Total energy, £ = - 13-6 eV
£.£.=-£=13-6 eV, .PE.=-2 K.E. = 2x 13-6 = -27-2 eV

Q. 5. A hydrogen atom initially in the ground level absorbs a photon, which excites it to « = 4 level.
Determine the wavelength and frequency of photon.
Sol. For ground .state, n, = 1, and iij = 4.
-13-6 -13-6
Energy of photon absorbed, £ = £2 - £| = -> ●>
eV = 13-6 I
= 13-6 eV
/lo nr n 42

15 13-6x15 -19
= I3-6X —eV =
16
X i-6x 10
16
J =2-(Mx 10-'^ joule

6-6xl0-''‘^x3xl0^
From eJ^ X = —
he
= 9-7xI0-*m.
X ' £ 2-04xl0"'^
12/48 ‘^ladeefa. Fundamental Physics (XII) kv/oji^ll

c 3xl0»
= 31x1015 U2
A, 9-7x10-5
Q. 6. (a) Using the Bohr’s model, calculate the speed of the electron in a hydrogen atom in the n = 1,2 and
3 levels, (b) Calculate the orbital period in each of these levels.
Q
Sol. (a) From v =- a , where a = = 0-0073
n ch

3x105 3x10*
V
1 “ X 0-0073 = 2-19 xlO^m/s, Vj = x0-0073 = 1-095 xlO^^m/s
1 2 2

V
_ 3x105 X 0-0073 = 7-3x105 m/s .
5” 3

ww
2%r 271x0-53x10-^^1
{b) Orbital period, T= . As Tj = 0-53 X 10
-10
m,
T = = l-52xl0-l‘^s
V 2-19x10^

Flo
1

e
r2 = 4r, and
As
^2 = 2^ 1

ere
72 = 8 r, = 8 X 1-52 X 10-1^ s = 1-216 x 10-15 s

FFr
1
r3 = 9 r, and
uurr
As =-v
5 3 I

orr
73 = 27 7, = 27 X 1 -52 x 10-'^ s = 4-1 x 10-15 s sfo
Q. 7. The radius of innermost electron orbit of a hydrogen atom is 5-3 x 10-n m. What are the radii of
kks
/} = 2 and n = 3 orbits ?
Yo
= 4 r, = 4 X 5-3 X 10-*i m = 2-12 x 10-1®m
oooo

Sol. As roc n-

and T3 = 9 r, = 9 X 5-3 X lO"" m = 4-77 x 10"l® m


eBB

Q. 8. A 12-5 eV electron beam is used to bombard gaseous hydrogen at room temperature. What series of
wavelengths will be emitted ?
urr

Sol. In ground state, energy of gaseous hydrogen at room temperature = - 13-6 eV. When it is bombarded with
12-5 eV electron beam, the energy becomes - 13-6 -f 12-5 = - 1-1 eV. The electron would jump from n =
ad
YYo

13-6
1 to rt = 3, where ^3 - “ 32 = - 1 -5 eV. On de-excitation the electron may jump from n = 3 to n = 2
dd

giving rise to Balmer series. It may also jump from « = 3 to /i = 1, giving rise to Lyman series.
Re
inn

Q. 9. In accordancewith the Bohr’s model, Bnd the quantum number that characterises the earth’s
revolution around the sun in an orbit of radius 1-5 x IQH m with orbital speed 3 x lO"* m/s. (Mass of
F

earth = 6-0 x 10^ kg).


Sol. Here, r = 1-5 x lO^l m, u = 3 x lO'i m/s, m = 6-0 x 10^'^ kg
nh
According to Bohr’s model, mvr =
2n

2nmvr 22 6-0x1024x3x104x1-5x10>*
n =
h
= 2x — X
1 6-6x10-54
= 2-57 X lO^'l, which is too large.
ATOMS 12/49

ii
K. . WITH ANSWERS,
Li^ j^i
if V
HINTS AND SOLUTIONS

PRO
f
»1 ■1

MULTIPLE CHOICE QUESTIONS-I

1. Taking the Bohr radius as Qq = 53 pm, the be infinitely many orbits with the vector

w
radius of Li'*^ ion in its ground state, on the pointing in all possible directions. In actuality,
basis of Bohr’s model, will be about this is not true.

(a) 53 pm (6) 27 pm (a) because Bohr model gives incorrect values

Flo
(c) 18 pm (d) 13 pm of angular momentum
2. The binding energy of a H-atom, considering (/;) because only one of these would have a

ee
an electron moving around a fixed nuclei minimum energy

Fr
4
f/ie (c) angular momentum must be in the direction
(proton), is B = - of spin of electron
Sn^elh^
0

for
(m = electron mass). id) because electrons go around only in
ur
horizontal orbits
If one decides to work in a frame of reference
where the electron is at rest, the proton would 5. O2 molecule consists of two oxygen atoms. In
ks
be moving around it. By similar arguments, the molecule, nuclear force between the nuclei
Yo
of the two atoms
oo

the binding energy would be


(n) is not important because nuclear forces are
eB

(M = proton mass) short-ranged


(h) is as important as electrostatic force for
This last expression is not correct because
r

binding the two atoms


ou
ad

(a) n would not be integral (r) cancels the repulsive electrostatic force
between the nuclei
{b) Bohr-quaniisation applies only to electron
Y

(c) the frame in which the electron is at rest is id) is not important because oxygen nucleus have
nd

not inertial equal number of neutrons and protons


Re

id) the motion of the proton would not be in 6. Two H atoms in the ground state collide
Fi

circlar orbits, even approximately inelastically. The maximum amount by which


3. The simple Bohr model can not be directly their combined kinetic energy is reduced is
applied to calculate the energy levels of an (a) 10-20 eV
atom with many electrons. This is because (b) 2040 cV
{a) of the electrons not being subject to a central (c) 13-6 eV
force
(^/) 27-2 eV
ib) of the electrons colliding with each other 7. A set of atoms in an excited state decays.
(c) of screening effects (a) in general to any of the stales with lower
id) the force between the nucleus and an electron energy
will no longer be given by Coulomb's law (b) into a lower state only when excited by an
4. For the ground state, the electron in the H- external electric field
atom has an angular momentum = h, (c) all together simultaneously into a lower state
according to the simple Bohr model. Angular (d) to emit photons only when they collide
momentum is a vector and hence there will
12/50 ^●ta.cCee^'4, Fundamental Physics (XIl)EZsISD

MULTIPLE CHOICE QUESTIONS-II

8. An ionised H-molecule consists of an electron 11. The Balmer series for the H-atom can be
and two protons. The protons are separated observed
by a small distance of the order of angstrom. {a) if we measure llie frequencies of light emitted
In the ground state, when an excited atom falls to the ground state
(«) the electron would not move in circular orbits {b) if we measure the frequencies of light emitted
(b) the energy would be (2)*^ times that of a H- due to transitions between excited states and
atom the first excited state

(r) the electrons, orbit would go arround the (f) in any transition in a H-atom
protons {d) as a sequence of frequencies with the higher
(c{) the molecule will soon decay in a proton and frequencies getting closely packed
a H-atom
-1 me^

w
9. Consider aiming a beam of free electrons 12. Let E 7 be the energy of the n
th

towards free protons. When they scatter, an 0


electron and a proton can not combine to

Flo
level of H-atom. If all the H-atoms are in the
produce a H-atom, ground state and radiation of frequency

e
(a) because of energy conservation (£2 - E^)/h falls on it,

rree
(b) without simultaneously releasing energy in (o) it will not be absorbed at all

r FF
the form of radiation
(b) some of atoms will move to the first excited
(c) because of momentum conservation state
uurr
(d) because of angular momentum conservation
10. The Bohr model for the spectra of a H-atom
{a) will not be applicable to hydrogen in the
for
(c) all atoms will be excited to the n = 2 state
((/) no atoms will make a transition to the n = 3 slate
kss
13. The simple Bohr modlc is not applicable to
molecular form He** atom because
ooook
Yo
{b) will not be applicable as it is for a He-atom (a) He^ is an inert gas
(c) is valid only at room temperature (&) He'^ has neutrons in the nucleus
eB

(d) predicts continuous as well as discrete (c) He‘* has one more electron
spectral lines (rO electrons are not subject to central forces
urr
ad

ANSWERS
Yo
dY

1. (c) 2. (c) 3. (a) 4. (a) 5. (a) 6. (<7) 7. (a) 8. ia. c)


9. {a, b) 10. (a, b) 11. (b, d) 12. (b. d) 13. (c. d)
Re
innd
Fi

HINTS FOR DIFFICULT MULTIPLE CHOICE QUESTIONS

Multiple Choice Questions -1

n 2/^2 n
2
1. On the basis of Bohr’s model, = a,-. —
4n^ mKZe~ Z

For Li'^'*’ ion. Z = 3 : n = 1 for ground state


53x12
Given «() - 53 pm r= = 18 pm

2. In a hydrogen atom, electron revolving around a fixed proton nucleus has some centripetal acceleration.
Therefore, its frame of reference is non inertial. In the frame of reference, where the electron is at rest, the
given expression cannot be true for binding energy as the frame in which electron is at rest would not be
inertial. Choice (c) is correct.
ATOMS 12/51

3. The simple Bohr model cannot be directly applied to calculate energy levels of an atom with many electrons.
This is because all the electrons in the atom are not being subjected to one single central force. Choice (a)
Is correct

4. Simple Bohr model, infact does not give correct values of angular momentum of revolving electron. It
gives only the magnitude of angular momentum, which is a vector. So the given statement in the question
is not true. Choice (a) is correct
5. In the given oxygen molecule, nuclear force between the nuclei of two atoms is not important because
nuclear forces being short ranged are confined only within one particular nucleus. The distance between
the nuclei of two atoms may be large. So the nuclear forces between two nuclei may not be operative/
effective. Choice (a) is correct
6. Initial K.E. of each of two hydrogen atoms in ground state = 13-6 eV.

w
.-. KE of both H atoms before collision = 2 x 13-6 eV = 27-2 eV.

As the collision is inelastic, linear momentum is conserved, but some K.E. is lost.
If one H atom goes over to first excited state and other remains in ground state, then their combined K.E.

Flo
after collision = ri3-6^ n3-6^ = 170eV

ee
22 ,2

Fr
.-. Reduction in combined K.E. = 27-2 - 17 0 eV = 10-2 eV.

7. A set of atoms in an excited state decays in general to any of the states with lower energy. Choice (a) is

for
ur
correct.

Multiple Choice Questions - II


ks
8. In an ionised hydrogen molecule, as there are two protons and one electron, therefore, electron’s orbit
Yo
oo

would go around the two protons separated by a small distance (~ A). This orbit shall not be a circular
orbit. Choices (a) and (c) are correct
eB

9. When a beam of fi^ electrons is aimed towards free protons, the electrons get scattered on account of
energy conservation. An electron and a proton can combine to produce a H-atom only if they simultaneously
r

release energy in the form of radiation. Choices (a) and (b) are correct
ou
ad

10. Bohr model for the spectra of H-atom is not applicable to hydrogen in the molecular form. As it is, the Bohr
Y

model is not applicable to a He atom. These are the known limitations of Bohr atom model. Choices (a)
and (b) are correct
nd
Re

11. Balmer series for hydrogen atom can be observed by measuring the frequencies of light emitted due to
Fi

transitions between excited states and the first excited state (n = 2). In these measurements, we would find
the sequence of frquencies with higher frequencies getting closely packed. Choices (b) and (d) are correct
4
-me
12. Here, E =
0

is the energy of nth level of hydrogen atom. If all the H-atom are in ground state, (n = 1), then the radiation
of frequency (£2 - E{)lh falling on it may be absorbed by some of the atoms and move them to the first
excited state (n = 2). All atoms may not be excited to n = 2 state. Further, as (£2 - E{)lh is sufficient only
to take the atom from n = 1 state to n = 2 state, no atoms shall make a tansition to n = 3 state. Choices (ft)
and (d) are correct.
13. 2He"^ atom has two electrons. Simple Bohr model applies only to one electron atom (H-atom). It does not
apply to He'^ because it possesses one more electron. Further, these electrons are not subject to central
forces. Choices (c) and (d) are correct
12/52 Fundamental Physics (XII)EEIHD

VERY SHORT ANSWER QUESTIONS


14. The mass of a H-atom is less than the sum of the masses of a proton and electron. Why is this ?
Alls. According to mass energy equivalence established by Einstein, E = m(r. If B represents binding energy of
hydrogen atom {= 13-6 eV), the equivalent mass of this energy = B/r’.
Hence, mass of a H-alom = + me - B/c^.
It is less than sum of the masses of a proton and an electron.
15. Imagine removing one electron from He'* and He^. Their energy levels, as worked out on the basis of
Bohr model will be very close. Explain why.
Ans. Imagine one electron removed from -,He'* and ,He^. As both the residual nuclei arc very heavy compared to
the mass of electron removed, therefore, their energy levels, worked out on the basis of Bohr model, will
be very close.
16. When an electron falls from a higher energy to a lower energy level, the difference in the energies

w
appears in the form of electromagnetic radiation. Why can not it be emitted as other forms of energy ?
Ans. When an electron falls from a higher energy to a lower energy level, difference in energies appears in the

Flo
form of electromagnetic radiation only. This is because electrons (being charged) interact only
electromagnetically.

reeee
17. Would the Bohr formula for the H-atom remain unchanged if proton had a charge (+4/3) e and
electron a charge (- 3/4) e, where e = 1*6 x 10"*^ C. Give reasons for your answer.

FFr
2
tnv
Ans. In Bohr’s formula, we say centripetal force is provided by Coulomb's force of attraction between

for
r
ur
the nucleus (a proton) and revolving electron. The coulomb’s force (- e) (e) = - If charge on proton
^ 4 ) 4^ U 3 ^
kkss
3 1 0

+ - ^’ and charge on electron = — e , their product —


- e —e =—e", remains the same.
3 4 V 4 4
Yo
/ V
oo

And e = 1 -6 x C in both the cases. Therefore, Bohr formula for H-atom shall remain unchanged.
eB

18. Consider two different hydrogen atoms. The electron in each atom is in an excited state. Is it possible
for the electrons to have different energies but the same orbital angular momentum according to the
Bohr model ?
r

Ans. No. The electrons with different energies cannot have the same angular momentum, This is because according
ou
ad

13-6 ..
YY

to Bohr model, E n -eV.


ti¬
ll ll
When is different, n must be different. And angular momentum, iti v r = must be different for
ndd
Re

2k
different ii values.
Fi

SHORT ANSWER QUESTIONS


19. Positronium is just like a H-atom with the proton replaced by the positively charged anti-particle of
the electron (called the positron which is as massive as the electron). What would be the ground state
energy of positronium ?
4
-me
Ans. In Bohr’s fonnula, En m is called the reduced mass.
8€q Ir
In case of hydrogen atom, rn = = mass of electron.
For positronium : electron revolves around a positron, both of mass m^,. Therefore, reduced mass
m xw m 4
e c e
—m^ e 13-6
III = eV =-6'8eV
+ 2 2

This is the ground state energy of positronium.


ATOMS
12/53

20. Assume that there is no repulsive force between the electrons in an atom but the force between
positive and negative charges is given by Coulomb’s law as usual. Under such circumstances, calculate
the ground state energy of a He-atom.
Ans. In a helium atom (2He^), there are two protons and two neutrons in the nucleus. Two electrons are revolving
around the nucleus in the first orbit. We are given that there is no repulsive force between the electrons.
For He nucleus, charge, Z = + 2 e. As two electrons of charge (- 2 e) ervolve around the nucleus, therefore,
the formula for energy in nth orbit would be

P _ me"*(Z^) 4me"^ = 4E

4
me
The ground state will have two electrons, each of energy E=-
8e2 (1)2^2 =-13-6eV

ww
.-. The total ground state energy of He atom = 4 £ = 4 (- 13-6) eV = - 54*4 eV
21. Using Bohr model, calculate the electric current created by the electron when the H-atom is in the

Flo
ground state.

e
Ans.
In the ground state of hydrogen atom, suppose, Oq = Bohr radius, Vq = velocity of electron in first orbit

reree
2na
.’. Time taken by electron to complete one revolution, 0

r FF
j =
^0
Current created, / = charge (g)
ev
0
uurr
time (T)l7ta^
foor
22. Show that the first few frequencies of light that are emitted when electrons fall to the nth level from
ks s
levels higher than n, are approximate harmonics in the ratio 1:2: 3....) when n » 1.
Yoo
ooook

Ans. When on electron jumps from mth orbit to nth orbit, the frequency of electromagnetic radiation emitted.
eBB

according to Bohr’s formula, is v-cRZ^ —* —— where R is Rydberg constant.


Let us take m = (n + p), where p = 1, 2, 3...,
uurr
ad

\-2
Yo

1 1 1
v = cRZ^
.(« + p)2 2 =c/?z2 2
dY

n n‘‘ n n
Re
innd

v = cRZ^
1 2p\ 1
= -cRZ^
2p {-2cRZ^'
Asp «n, therefore. Z2 1- 2 3 ~ P
FFi

n n 3
y n n n

As p — 1,2, 3...., therefore, first few frequencies of light emitted are approximate harmonics (/.e., in
the ratio 1:2: 3....), when n » p.
23. l^at is the minimum energy that must be given to a 17 atom in ground state so that it can emit an Hy
line in Balmer series. If the angular momentum of the system is conserved, what would be the angular
momentum of such Hy photon ?
Ans. Hy line in Balmer series corresponds to transition from n = 5 to n = 2. Therefore, electron in ground state
(n = 1), must be raised first to the state (n = 5). Energy required for this purpose
13-6 f 13-6^
eV = - 0-54 + 13-6 = 13 06 e.V
[ .
If the angular momentum of the system is conserved, then
angular momentum of photon emitted = change in angular momentum of electron

= L^-L2= 5h-2h = 3h =3 X 106x 10-34 = 3-18 xir^4 kg m2 s-l


12/54 Fundamental Physics (XII)Eiai^

LONG ANSWER QUESTIONS


24. The first four spectral lines in the Lyman series of a H-atom are X = 1218 A, 1028 A, 974-3 A and
951*4 A. If instead of Hydrogen, we consider Deuterium, calculate the shift in the wavelength of
these lines.

Ans. In hydrogen atom, one electron (of mass m^) revolves around one proton (of mass M)
m xM m
e

Reduced mass for hydrogen, = —~ ..


" m +M m
e
l + -£
M

In deuterium, [D^, one electron (of mass /m,,) revolves around nucleus containing one proton and one
neutron (of mass 2M).

llowow
II -
m x2M
'g
2M .m^ — m
m
e

Reduced mass for deutenum, -> a e


2M
2M +m^
m
2M 1 + —£
2M

When an electron jumps from orbit j to orbit i, the frequency of radiation emitted

ree
1
X -■ «
li vj. = {Ej - £;) « |i (reduced mass) ji

rF
If
u
m 1--^
m
r FF
is wavelength emitted in case of deuterium, and Xf^ is wavelength emitted in case of hydrogen.

forfFore \(
\ f rn m
X
D - M , rUe 1-
c
1 + —
then 1-
s
M 2M
m M 2M / V
ok
/ V
YYour o
m
e
2M
o

m m
eeBoBks

m
e
1-
M 2M 2M
/ V
r
our u
ad

m 1 X 1
D _ = 0-99973
1-
As e _
, therefore -r
Yo

M 1840 X 2x1840
H \

Xjy = (0-99973) Xfj


d
Re

Using Xh = 1218 A. 1028 A. 974-3 A and 951-4 A, we get


in

Xfi = 1217*7 A, 1027*7 A, 974*04 A, 951*1 A.


FFind Y

25 Deuterium was discovered in 1932 by Harold Urey by measuring the small change in wavelength for
a particular transition in *H and ^H. This is because, the wavelength of transition depends to a
certain extent on the nuclear mass. If nuclear motion is taken into account, then the electrons and
nucleus revolve around their common centre of mass. Such a system is equivalent to a single particle
with a reduced mass )i, revolving around the nucleus at a distance equal to the electron-nucleus
separation. Here \i = + M) where M is the nuclear mass and is the electronic mass.
Estimate the percentage difference in wavelength for the 1st line of the Lyman series in H and H.
(Mass of *H nucleus is 1*6725 x 10"^’ kg, Mass of ^H nucleus is 3*3374 x lO'^’ kg, Mass of electron
= 9*109 X 10-^* kg).
Ans. If we take into account the nuclear motion, the stationary state energies will be given by
I
L
2
It
8e2/,2
0
n

Let be reduced mass of hydrogen (H') and be reduced mass of deuterium (H ),


ATOMS
12/55

The frequency of 1st line of Lyman series in hydrogen is given by


hv
8eJ/,2[,2-22j= 8^2;,2
H ~

The wave number of transition is XH 3/4 Hh^'*


~
8€q /j3 c
T _3/4Ho«‘’
Similarly, for deuterium.

oww
6X = Xd~^h
The percentage difference in wave number is
AX
xl00 = {Xp A.j^)xl00 ^ M^£) xlOO
>-H

ee
FFrlo
"<e^n

r
bX m +M m +M
Using iLp = and \ip = , we get = xl00 = —^ D e H
XlOO

rF
m^+M H m+M

ee
e D XH

As mg«Mfj<Mp

rF
ouru \ /
AX Mp MH \ + m^lMp m

ffosor
xlOO = -1 XlOO = 1 + 1 + —^ -1 XlOO
M M M
\ \ + mJMp
M
H
os k H
J\ D

m
1 1
ook
Yo
1+— -1 XlOO =m XlOO
Y
M M ^ M M
H D H Dj
Bo
reeB

^X100
Xh
= 91x10-31 .1-6725x10-27
1

3-3374x10-27
1
XlOO
ooY
uur

= 91x10-^ (0-5979 - 0-2996) x 100 = 2-714 x ir^ %


ad

26. If a proton had a radius R and the charge was unifonnly distributed, calculate using Bohr theory,
dY

the ground state energy of a H-atom when (i) R = 0-1 A, and (ii) = 10 A.
2
h . mv -^2
nind

Ans. In ground state of hydrogen atom.


Re

mv rg = n v = —, and
mr
B
F
Fi

m fi2 ^2
^B 47lGo'ij
4kGq —
fP-
^B = =0-51 A
.(/)
g2 m

PE =- g2 J_ = - 27-2 e.V
47CGo [rg
1 1 ^2 ^2
KE =- mv^ =- m. 2 2 = + 13-6 eV
2 2 m r^
B
2mr1B

For spherical nucleus of radius R,


ground state energy remains the same as calculated above.
//■/?» : the electron moves inside the sphere with radius r^ (new Bohr radius)
12/56 Pn^^dee^'^ Fundamental Physics (Xll)^aill

Charge inside, e =

From (0, '●» =


e'2

= (0-51 A)R^ Hi)


" m
* J

oww
=(0-51A)‘/'^x/?6/'^

If/? = 01 A, = (O-SlA)*'"^ xCOl)^/"^ = 0126 A>R

If/?= lOA, r; =(0-51A)‘/'^x(10)6/'^ =7-19 a </?

e
FFr lo
re
/
2 > (0-51A)2
Ik. =(13-6eV)

ree
K.E. = - mv^ = = 0-068 eV

F
Now,
2 2m r;? 2mrlB (7-19 A)2
B

rF
r!I-3R^
B r^-SRh
ffsoor
ouur
RE. = +
4 TUG 0 R^ 47tGorg R^
/ V
kosk
0-51 (719--300)
Yo
= (27-2 eV) = _ 3-44 eV
oo

1000
Y
BB

27. In the Auger process, an atom makes a transition to a lower state without emitting a photon. The
excess energy is transferred to an outer electron which may be ejected by the atom. (This is called an
rre

Auger electron). Assuming the nucleus to be massive, calculate the kinetic energy of an n = 4 Auger
oYuu

electron emitted by Chromium by absorbing the energy from a« = 2ton = l transition.


ad

Ans. As the nucleus is massive, recoil momentum of tlie atom can be ignored. We can assume that the entire
dY

energy of transition is transferred to the Auger electron.


As there is a single valence electron in chromium (Z = 24), the energy states may be thought of as given by
innd
Re

R
Bohr model. The energy of the nth state is =- — where R is Rydberg constant.
Fi
F

n
In the transition from n = 2 to n = 1, energy released, AE - -RZ~ —4 -1 1=14
y

if RZ^
The energy required to eject a n = 4 electron = RZ~ x -
16

3RZ^ RZ^
KE of Auger electron = —- 16

11
KE — (13-6eV)x 24x24
[4 16 J 16 16

= 5385-6 eV
ATOMS 12/57

^2
28. The inverse square law' in electrostatics is IF I = 2 for the force between an electron and a
(47iej)).r
/
1
proton. The - dependence of I FI can be understood in quantum theory as being due to the fact
J
that the ‘particle’ of light (photon) is massless. If photons had a mass wip, force would be modified to
IFI =
\ I X"] „ h
TTrrV Lr r j
irXr) where X - m„clh
●'
and ^ = —.
2n
Estimate the change in the
ground state energy of a H>atom if nip were 10“^ times the mass of an electron.
Ans. Here, we suppose mass of photon, nip = 10"^ electronic mass
i.e..
ip = 10-6 (0-5) MeV = 5 X 10-"^ X 1-6 X lO-*^ J = 0-8 x 10"^^ J.
m

w
1 n tic lO'^'^xSxlO®
Now, = 4 X lO-"^ m
which is much larger than Bohr radius.
X 0-8x10
-19
p p

Flo
c2 1 X . _1 ^ -
As I Fl = exp. (- X r) ...(/) where A = = 4 x 10"' m »
L 7^7

eeee
4 TIG m c
0 P

Fr
1
X« — or X, Tb « 1
'is

for
ur
Now,
exp (-X,r)
U{r) = .(«)
4tcg r
0
ks
Yo
2
n r 1 XI
oo

mv
As mv r= fi, V = . As exp (- X r) -> 1, therefore, from (/) = F =
mr r 4tcg 0
eB

m » f 1 X
^*7
ur

r mr 4 kg
0 L
ad
Yo

tp- _ r + Xr^
m 4jcg
0
d
Re

2
in

If X = 0, r= rg, then — = 4jcg


'b-
F

m
0

As X“* » r5’ put r = rg + 6

From (Hi), = (r^ + 5) -f X (r^ + 6)2 = -f 5 + X (r| -f 5^ + 28r^)


Neglecting 62, we get 0 = X rj + 6 (1 + 2 X )
-Xr2B
6 =
l + 2Xr,B = -Xrl (l-2Xrp)
8 = -X To
B (As X rg « 1)
/

From (h). V(r) =


-e
2 exp. (-X8-Xrp) -^2 1
1-
- (1-Xrp) = -27-2 eV
4tcg
0 (rg+8) ‘'“^0 '■« LI 'b j
i.e., V (r) remains unchanged.
12/58 Fundamental Physics (XII)

(
1* 2 1 r fi ^2 fP- t? 28
- mv^ =
K.E. =- — m —
= (13-6eV) [1 + 2Xrg]
2 2 m (r^ +8)^ 2mr3
5 V

-^2 ^2
Total energy = + -^{\ + 2Xr„) =-27-2+13-6(1+2 Xrg)eV.
47ceo'fl 2mr|
Change in ground state energy of hydrogen atom = 13-6 x 2 X eV = 27*2 X rg eV
29. The Bohr model for the H-atom relies on the Coulomb’s law of electrostatics. Coulomb’s law has not
directly been verified for very short distances of the order of angstroms. Supposing Coulomb’s law
between two opposite charge + - ^2 *s modified to
/
gl2 1

ww
\F\ = r>R 0 , r</?o
(4nzQ) r2 ’ 4lte„ fij 1. r J
Calculate in such a case, the ground state energy of a H-atom, if e = 0-1, Rj = 1 A.

FF loo
1

ree
Ans. We are given IFI = -J ; r>/?0
(47ieo) r2

rFee
^1^2 _L

oor rF
IFI = ; r</?o
rur
(4)ce(,) iq [ r /
s ff
e =01,R„= I A.
G = 2 + 8.
k
Let
YYoou
okos

^1^2
BBoo

F = for r <R,0
(47IGo) r2+5 ^2+5
r ee

9,^2
where a = = 9 X 10^ (1-6 X l(r*9)2 = 23-04 x 10-29
ouur
ad

4jC€
0
Yo

So, F =
mv 2 _ ^ ^0 (0
Yd

or V
1+8
Re

r ^2+8 mr
idn
FFin

nh
(/) As mv r = n fi, r =
mv

-ll/2 1 5
nh m -+-

Using (0, r= . r2 2
m A
0 j

«2^2 1-6
Solving this for r, we get
mAf^
I 1

^2 1-8
1-052x10-^* 2-9
For n = 1, ^1 - = 8 X 10“‘* m = 0-08 nm< 0-1 nm
mAR^ (9-1x10-2^) (23-04x10-29) 10*9
ATOMS 12/59

(i7) From v n
nh
= n h
A
Rl]l-B For n = 1, =
n
= 1-44X lO^m/s
mr mr,
n 1

1
(m) K.E. =-mv1
2
= 943 x J = 5-9 eV RE. (tiU /?o) =
^0

RE. from /?n to r =


dr r 1
nr

_ Aiigr 1 1 A 1

«o
7 2+5 _i_§ ^1+5
%
1+8
l + 8[r^+S

w
-1-9
A 1-9 2-3x10-28
RE. =-
[(0-8)0-^-l-9] joule = -17-3 eV
l + 8[r‘+S Rq Rq -0-9 [ r-0-9 Rq fr9

Flo
Total energy = (- 17-3 + 5-9) eV = - 11-4 eV

eee
Fr
for
ur
ks
Yo
oo
eB
r
ou
ad
YY
nd
Re
Fi
12/60 “piteuUefi.'^^ Fundamental Physics fXIIlPPTim

Competition
hi

NEET/JEE
SPECIAL

ww
For ultimate preparation of this unit for competitive examinations, students should refer to
● MCQs in Physics for NEET
Pradeep's Stellar Series.... ● MCQs in Physics for JEE (Main)

FF loo
separately available for these examinations.

ree
Multiple Choice Questions (with one correct Answer)

rFee
Atoms
4. As an electron makes a transition from an excited

oor rF
stale to the ground state of a hydrogen like atom/
rur
1. Consider 3rd orbit of Helium. Using non- ion.
s ff
relativistic approach, the speed of electron in (a) Its kinetic energy increases but potential
this orbit will be [given K =9 x lO''^ Z = 2 and energy and total energy decrease
k
YYoou
h = 6-6x Js]
ookos

ib) Kinetic energy, potential energy and total


{a) 1-46 X 10^ m/s (b) 0-73 X 10^ m/s energy decrease
BBo

(c) 3 X 10^ m/s (d) 2-92 X 10*^ m/s (c) Kinetic energy decreases, potential energy
re

(AIPMT 2015) increases but total energy remains same


2. In the spectrum of hydrogen, the ratio of the (d) Kinetic energy and total energy decrease but
ouur
ad

longest wavelength in the Lyman series to the potential energy increases


Yo

longest wavelength in the Balmar Series is : {JEE Main 2015)

(.)! 5. An excited hydrogen atom emits a photon of


(a) 4
Yd
Re

wavelength X in returning to the ground state. The


idn

27
quantum number n of excited state is given by
FFin

27
«/)- (R = Rydberg constant)

XR
(RE-AIPMT 2015)
(a) ^XR(XR-l) (b) ^j(XR-l)
3. Consider a hydrogen atom with its electron in the
nth orbit. An electromagnetic radiation of
wavelength 90 nm is used to ionize the atom. If XR-l 1
(C) (cl)
the kinetic energy of the ejected electron is 10-4 y XR ^XR(XR-l)
eV, then the value of n is (he = 1242 eV nm)
6. When an a particle of mass m moving with
(a) 1 (b)2
velocity v bombards a heavy nucleus of charge
(c)3 (r/)4 Ze, its distance of closest approach from the
(JEE Advanced 2015) nucleus depends on m as
ANSWER)3
1. (£/) 2. (cO 3. (b) 4. («) 5. (h)
ATOMS 12/61

1 1 4 9
ia) ib) — (c) r = - Ul) r = ~
-fm m

1 (JEE Main 2017)


(c) m (d) (NEET 2016)
m 11. A diatomic molecule is made of two masses ni^
and which are separated by a distance r. If we
7. Suppose an electron is attracted towards the origin
by a force kJr, where k \s .a constant and r is the calculate its rotational energy by applying Bohr’s
distance of electron from the origin. By applying rule of angular momentum quantization, its energy
Bohr model to this system, the radius of //Ih orbit will be given by (n is an integer)
of electron is found to be and the K.E. of
(nil + nh)^ li~ n^f?
electron is found to be T,,. Then which of the {a)
T 2 2 2
ih)
following is true ? m-, r 2(/»| + ni-))r~

In-}?
0 '7

w
ia) T, OC
1
(c)
(/»! + nh)n- tr
n~ UD
(h) T,, is independent of n ; r,,; oc n (mj +m2)/-^ o
2 //»] /«2 f
2

(AIEEE 2012)

Flo
1
(C) T (d) oe
— and /●„ ec n- 12. An electron of a stationary hydrogen atom passes
n n
from the fifth energy level to the ground level.

ee
(AIEEE 2008)
The velocity that the electron acquired as a result

Fr
8. The wavelength of the first spectral line in the of photon emission will be :
Balmer series of hydrogen atom is 6561 A. The 24 hR 25 hR
wavelength of the second spectral line in the (a) (h)

for
25 m
ur
24 m
Balmer series of singly-ionized helium atom is
(a) 1215 A (h) 1640 A 25 m 24 m
(c) id)
(c) 2430 A {d) 4687 A {IIT2011) 24 hR 25 hR
ks
9. The wavelength of the first line of Lyman series
Yo
[AIPMT (Prelim) 20121
oo

for hydrogen atom is equal to that of the second 13. Consider a hydrogen like ionized atom with
line of Balmer series for a hydrogen like ion. The atomic number Z with a single electron. In the
eB

atomic number Z of hydrogen like ion is emission spectrum of this atom, the photon
(a) 3 ih) 4 emitted in the n = 2 to = 1 transition has energy
ic) 1
ur

id) 2 74*8 eV higher than the photon emitted in the


ad

[AIPMT (Prelim) 2011) n = 3ton = 2 transition. The ionization energy of


Yo

10. Some energy levels of a molecule are shown in the hydrogen atom is 13-6 eV. The value of Z is
the Fig. 12(CF).I. The ratio of the wavelengths ia) I ib)2
r= is given by (c)3
nd

(^4
Re

FIGURE 12(CF).1 (JEE Advanced 2018)


Fi

14. Hydrogen atom in ground state is excited by a


-E monochromatic radiation of X = 975 A. Number
of spectral lines in the resulting spectrum emitted
will be

X
ia) 3 ih) 2
(c) 6 id) lO (AIPMT 2014)

-2E-
15. Hydrogen (jH*). Deuterium (,H-), singly ionized
helium (^He'’)'*' and doubly ionized Lithium
-3E—
(3Li^)'*"'' all have one electron around the nucleus.
Consider an electron transition from n = 2 to
n = 1. If wavelengths of emitted radiation areAj,
ih) r = - ^2, A3, A4 respectively, then approximately which
one of the following is correct ?

6. id) 7.{h) 9. in) 9. id) 10.(6) 11. (f/) 12. ((») 13. (r) 14. (t 1

i t
12/62 d. Fundamental Physics (XII)

(a) A, =A2 = 4A3 = 9A4 ground state. Let be the de Broglie


{b) A ] = 2 A2 — 3 A3 — 4 A4 wavelength of the electron in the nth state and
the ground state respectively. Let A„ be the
(c) 4A| = 2A2 = 2A3=A4
wavelength of the emitted photon in the transition
(d) A j = 2 A2 — 2 A3 — A4. (JEE Main 2014)
from the nth state to the ground slate. For large n,
16. Balmer gave an equation for wavelength of visible {A, B are constants)
kn^ B
radiation of hydrogen spectrum as X.= (a) A «A + ^ {b) A„ « /) + B A,
n^ -4 ^2
The value of k in terms of Rydberg’s constant R is
(a) R {b) 4R (c) Al^A+BXl (d) Al<.X
R 4 (JEE main 2018)
{(]) -r (AIIMS 2014)
R 23. If the series limit frequency of the Lyman series
17. The KE of the electron in an orbit of radius r in is v’l^, then the series limit frequency of the Pfund

w
hydrogen atom is (e = electronic charge) series is
«2 (a) 25 (h) 16
(a) — (b) rY

Flo
r
(c) VfJ]6 (i!) vJ25
(JEE main 2018)
(.) ^

reeee
{d) (AIIMS 2014)
r 2r2 24. For which one of the following, Bohr model is
not valid ?

FFr
18. The ratio of wavelengths of the last line of Balmer
series and the last line of Lyman series is (fl) singly ionised helium atom
(a) 2 (b) 1 (b) Deutron atom

for
ur
(c) 4 id) 0-5 (NEET2017) (c) singly ionized neon atom (Ne'*')
19. The radiation corresponding to 3 —» 2 transition (d) Hydrogen atom (NEET 2020)
kkss
of hydrogen atom fall on a metal surface to
produce photoelectrons. These electrons are made 25. Let r, and Ti be the energy of an electron in the
Yo
first and .second excited states of hydrogen atom,
oo

to enter a magnetic field of 3 x 10^ T. If the radius


of the largest circular path followed by these respectively. According to the Bohr’s model of
eB

electrons is 10 0 mm, the work function of the an atom, the ratio Tj : T2 is


metal is close to : (a) 1 : 4 (B) 4 : I
r

(c) 0-8 eV ib) L6eV (c) 4 ; 9 (r/)9:4 (NEET 2022)


ou
ad

(c) 1-8 eV {d) M eV 26. The total energy of an electron in the nth stationary
YY

(JEE Main 2014) orbit can be obtained by


20. If Aq, is the wavelengdi of K„ X-ray line of copper 13-6
(atomic number 29) and is the wavelength of
ndd

(fl)£„ = -13-6n2eV (/^) £,, =


Re

-eV
the K„ X-ray line of molybdenum (atomic number n"

42), then the ratio A^u/A-mo ‘s close to


Fi

(a) 1-99 {b) 2-14 13-6 13-6

(c) 0-50 id) 0-48


(c) = -
n
^eV id)
n
_eV

(JEE Advanced 2014) (NEET 2021)


21. An electron in an excited state of Li^^ ion has
27. The time period of revolution of electron in
angular i omentum 3h/2n. The de-Broglie its ground .state orbit in a hydrogen atom is
wavelength of the electron in this state is p 71 Aq 1-6 X 10“’^ s. The frequency of revolution of
(where Uq is the Bohr radius). The value of p is electron in its first excited state (in s"') is
(a) 4 ib) 3 (c) 2 (d) 1
(fl)6-2x 10
15
ib) 5-6 X I0‘“
(JEE Advanced 2015) 14 14
(c)7-8x 10 id) ] -6x 10
22. An electron from various excited states of
(JEE Main 2020)
hydrogen atom emits radiation to come to the
ANSWERS
15. ia) 16. id) 17. ih) 18. ic) 19. id) 20.ih) 21. ic)
22. ia) 23. id) 24. ic) 25. id) 26. ic) 27. ic)

k
ATOMS 12/63

m Multiple Choice Questions (with One or More than One Correct Answers)

28. The radius of the orbit of an electron in a hydrogen (a) V X r (b) En


like atom is 4-5 Aq, where «q is the Bohr radius. (c) Er id) V X )i.
Zh
Its orbital angular momentum is - . It is given 32. Let be the area enclosed by the nth orbit in a
IK
hydrogen atom. The graph of In {AJA{) against
that h is plunk’s. Constant and R is Rydberg In in)
constant. The possible wavelength (i), when the (a) will be a circle
atom de-excites is (are)
(b) will be a monotonically increasing non-linear

ww
curve
9 9
(o) —R (b) ^R
32 16 (c) will be a straight line with slope 4
(d) will pass through the origin.

Flo
9 4
(c) -R (d)-R 33. An electron in hydrogen atom first jumps from

e
5

ree
second excited state to first excited state and then
(JEE Advanced 2015) from first excited state to ground state. Let the

Fr
rF
29. In Bohr’s model of hydrogen atom : ratio of wavelength, momentum and energy of
photons emitted in these two cases be a, b and c
uurr
(«) the radius of the Atth orbit is proportional to
respectively. Then
for
2
n

ib) the total energy of the electron is «th orbit is 9


s
^ 5
(b) b = -
kks
(a) a = -
inversely proportional to n 27
Yo
oooo

(c) the angular momentum of electron in /nh orbit


5
eB

(c) c =— (d) c = -
is an integral multiple of — 27 a
2k
34. Highly excited states for hydrogen like atoms (also
ur

(d) the magnitude of potential energy of the called Rydberg states) with nuclear charge Ze are
ad

electron in any orbit is greater than its K.E. defined by their principal quantum number n,
YYo

30. An electron in a hydrogen atom makes a transition where n » I. Which of the following statement(s)
n
1 —> where «| and Ihe principal is (are) true ?
dd

quantum numbers of the two states. Assume the


Re

(a) Relative change in the radii of two


in

Bohr model to be valid. The time period of the consecutive orbitals does not depend on Z
electron in the initial state is eight times that in
F

(b) Relative change in the radii of two


final state. The possible values of /i, and n-i are consecutive orbitals varies as l//i
(a) /ij = 4, «2 = 2 (/;) Uj = 8, Uo - 2
(c) Relative change in the energy of two
(c)/Ji =8,/i2= 1 (r/) «j - 6, /I2 = 3 consecutive orbitals varies as l/n^
31. Which of the following products in a hydrogen
atom are independent of the principal quantum (d) Relative change in the angular momenta of
two consecutive orbitals varies as l/«
number n ? The symb<'’> have their usual
meanings : (JEE Advanced 2016)

ANSWERS
28. (fl.c) 29. (a.c.d) 30. (a.d) 31. (c.d) 32. (c.d) 33. (b.cM) 34. {a.h.d)

i
12/64 P'uuUe^'.i. Fundamental Physics

DQ Multiple Choice Questions (Based on the given Passage/Compreh ension)


Each comprehension given below is followed by some multiple choice questions. Each question has one
correct option. Choose the correct option.

35. The size of atomic nucleus is of the order of


^.G.ompr.ehensi^ According to Thomson’s -15 -10
(fl) 10 m {Jo) lO m
model, every atom consists of a positively
charged sphere of radius 10
-to
m in which (c) 10*^ m id) lO’^m
entire mass and positive charge of the atom 36. The ratio of atomic volume to nuclear volume is
are uniformly distributed. Inside the sphere, of the order of
electrons are embedded like seeds in a (a) I0'5 ih) 10
-12

ww
watermelon. According to Rutherford, entire (c) 10« id) 10-^
positive charge and mass of the atom are
37. Large angle scattering of a-particles could not be
concentrated in a tiny central core of the
explained by
atom, which is called atomic nucleus. Size of

Flo
nucleus « 10“*^ m. The nucleus contains (a) Rutherford model

e
protons and neutrons. Negatively charged {b) Thomson model

eree
electrons revolve around the nucleus in (c) Both Rutherford model and Thomson model

FFr
circular orbits. {d) Neither Rutherford model nor Thomson model

oorr
uur r
09 Matching Type Questions
sf
DIRECTIONS. In each of the following questions, match column I and column II and select the correct
sk
Yoo
match out of the four given choices.
oook

38. Column I Column II


eBB

A. Thomson atom model p. fixed for an atom


B. Rutherford atom model q. charge and mass of atom are distributed uniformly in a sphere
Nucleus
uurr

C. Bohr atom model r.


ad

D. Ionization potential s. stationary orbits


Yo

(a) A-q, B-r, C-s, D-p (b) A-p, B-q, C-r, D-s (c) A-q, B-p, C-r, D-J (d) A-r, B-5, C-q, D-p
dY
Re

Matrix-Match Type Questions


innd

r s
p q

a'^ ©;©: ©
FFi

DIRECTIONS. Each of the following questions contains statements given


in two columns,which have to be matched.The answersto these questions B
1)1©! 101 !0;
have to be appropriately bubbled. If the correct matches are A-r, As ;
B-p, ; C-p ; D-q, D-s, then the correctly bubbled matrix will look like
the one shown here :

Column 11
D ®||©jiO; s'

39. Column I
A. Potential energy of electron in hydrogen orbit P- e"'/87ie n
r

B. Kinetic energy of electron in hydrogen orbit q. - e~/^n


C. Total energy of electron in hydrogen orbit r. - e-/4Tt e Q r
D. Distance of closest approach of a-particle s. proportional to KE of a
E. Impact parameter t. inversely prop, to KE of a
iNSWERS
35. (ri) 36. («) 37. (/>) 38. (a) 39. {A-r. B-p. C-q. D-f. E-f)

I
ATOMS 12/65
A a c t>

VI. 0000
Integer Type Questions
oooo
DIRECTIONS. The answer to each of the following questions is a single digit 0 0 00
integer, ranging from ● to 9. If the correct answers to the question numbers A, B, 0000
C and D (say) are 4, 0, 9 and 2 respectively, then the correct darkening of O ©©©
bubbles should be as shown on the side :
© ©©©
40. The radius of first Bohr orbit is ,y. The de Broglie wavelength of electron in 3rd orbit © ©©®
h n K X where n = ?
©©0©
41.
For an atom of an ion having single electron, the wavelengths observed are X] =2 ©00©
units and X3 = 3 units Fig. 12(CF).2. The value of missing wavelength X2 is : ©000
42. Balmer gave an equation for wavelength of visible radiation of H-spectrum FIGURE 12(CF).2
. The value of k in terms of Rydberg’s constant R is m/R, Ec

ww
as X =
n 2-4 ^1
where m is :
43.
^Eb
A hydrogen atom in its ground stale is irradiated by light of wavelength

Floo
970 A. Taking hc/e = 1-237 x 10"^ eV m, and ground slate energy of ^3 >.2
hydrogen atom as - 13-6 eV, the number of lines present in the emission

ee
spectrum is (JEE Advanced 2016) Ea

eer
VII.
Assertion-Reason Type Questions

FFr
oorr
uur r
FOR MEDICAL STUDENTS FOR ENGINEERING STUDENTS
s ff
DIRECTIONS. Read the following questions and DIRECTIONS. Each of the following questions
choose any one of the following four responses. contains two statements. Check if
sk
YYoo
A. If both. Assertion and Reason are true and the (A) Statement-1 is true, Slatemeni-2 is true and
ooko

Reason is the correct explanation of the Siaiement-2 is correct explanation of Statement-1.


Assertion.
eBB

(B) Statement-1 is true, Statement-2 is true, but


B. If both. Assertion and Reason are true but
Statement-2 is not a correct explanation of
Reason is not a correct explanation of the Statement-!.
uurr

Assertion.
(C) Statement-1 is true, but Statement-2 is false.
ad

C. If Assertion is true bui the Reason is false.


(D) Statement-1 is false, but Statcmcnt-2 is true.
Yo

D. If both Assertion and Reason are false.


47. Statement-1. Large angle scattering of alpha
44. Assertion. Balmer series lies in visible region
dY

particles led to the discovery of atomic nucleus.


Re

of electromagnetic spectrum.
Statement-2. Entire positive charge of atom is
ind

Rea.son. Balmer means visible, hence series lies


FFin

concentrated in the central core,


in visible region.
(a) A (/;) B (r) C {d) D
(a) A (h) B (c) C (d) D
45. Assertion. Isotopes of an element can be 48. Statement-1. Impact parameter for scattering of
a particles by 180° is zero.
separated by using a mass spectrometer.
Reason. Separation of isotopes is possible Statement-2. Zero impact parameter means a
because of difference in electron numbers of particle tends to hit the centre of the nucleus
isotopes. (a) A (h) B (r) C (d) D
(a) A (h) B (c) C id) D 49. Statement-1. Distance of closest approach of a
46. Assertion. Rydberg’s constant varies with mass particle to the nucleus is always greater than the
no. of a given element. size of the nucleus.

Reason. The ’reduced mass’ of the electron is Statcment-2. Strong nuclear repulsion does not
dependent on the mass of the nucleus only. allow a particle to reach the surface of nucleus.
ia) A (b) B (c) C id) D («) A (h) B (c) C id) D

40. 6 41.6 42.4 43.6 44. (c) 45. (r) 46. (J) 47. (i;) 48.00 49. (o)
12/66 ‘P'teidce^ ’a Fundamental Physics (XII) cf.wwii

For Difficult Questions

Multiple Choice Questions (with one correct Answer)


1. Here.Z=2, » = 3 1 ( 1
5. From t ~ ^ ■)
For H-likc atoms, v = — x 2.2 x 10^ m/s X U“
n

2 4-4 X10^
V = -x2-2xl0^ = = 1*46 X m/s X X
3 3

w
2. For Lyman series XR
n =
{XR-\)
I
— = R ^_1 l_'| = -R
3 ...(/)
X 22 4 6. Assuming that there is no loss of energy, at the

Flo
for Balmcr series distance of closest approach from the nucleus.

e
K.E. of a particle = RE. of a particle

ree
1 5
— = R = — R 1 Ze

FFr
02 32 36 ...(») -5

Xi K ~
— mv~ (2e)
2
471^0'b
urr
Dividing (n) and (/)

for
2Ze~ Ze-

A., _ (5/36)/? _ 5 '() = 2


4TU6^mt;^/2
0
rc e.. mv
0
X, ~ (3/4)/? “ 27
kkss
Yo
3. Here, X = 90r\m,K=\0-4eV Clearly, r, oc —
ooo

0
m

13-6 7. Applying Bohr model to the given system,


As En f .V
eB

n-
mv~ k
...(0
he 13-6 cV
r r
Now — = 10-4 eW
ur

n n
X n
ad

nh nh
1242eVnm 13-6t>V
YYo

V =
= 10-4 and mvru =
90 nm n
2 2tc’ 2tiw.n

41-4 13-6 m n~fr k


d

or
2- = 10-4 Put in (0, — X—r—^ = -
Re

3 rn 4TTm-r~ rIt
in

n
n

13-6
F

or 13-8- 10-4 = — -) n 2/,2


n r~ =
n ...(«)
4 71" ntk
13-6
●U 2
13-6
3-4 = or n- = 4 or w = 2
or
n
Y or « = 3-4 2
r*- oc n~
●>
or r„ « w
II n

1 KZe- 13-6Z2
4. As K.E. = -
2 2r /I- K.E. of electron, T, =i:'nv-
2
21 ^ 2/ 2
KZe~ -27-2 Z- 1 n^fr n h
P.E. = - eV r. ■> 2
2 8 7t“ m r
2r n 2 471" m'r"
n n

KZe^ 13-6Z2 Using (ii), we get


T.E. = - eW
2r
n ^Ir.An^mk k
7

As n decreases. K.E. increases, .PE. decreases and


It
871^ mtrh^ 2

T.E. also decreases. T„ is independent of n.


ATOMS 12/67

he
For Difficult Questions
= -E-{-2E) = E ...(I)

8. The wavelength (A.) of a spectral line in the Balmer


series is given by he
= -£-
and ...(«)
- = RZ-
1 ^2 l 3 J 3
A i} n
2
Dividing (/'/) by (/), we get
where n-3,4, 5, 6
In case of hydrogen atom, for first spectral line A-i ^ £73 1
in the Balmer scries, Z = 1, /j = 3. A, = A,pj (say) ^2 £ ~3
1 11. According to Bohr’s rule of quantization of
= Rx(\-f 4- = Rx—.
5
...(/) angular momentum,
XH 2" 32 36

w
nh
In case of singly-ionized helium atom, for mvr = = nn
second spectral line in the Balmer series, Z=2,n 2tc

=4 ^ ^ Hc-^+

Flo
But i> = ro). m (rO)) r = nh

e
1 3 nh

reee
= Rx(2-f — = 4Rx- ...(«) (0 =
2
X 22 42 16 mr

FFr
He*
Rotational energy of molecule
Dividing (/) by (ii), we get
X 1 nh f
for = -lap- =^(mr2)
5/36 5
ur
Hc+
~ 4x3/16 ~ 27
9
X 2 mr
H
kss
= A„x-
= {6561A)x^ =1215 a £ =
Yo
H
27
2mr^
oo

9. For hydrogen atom, the first line of Lyman series


eB

IS
_ mi m2
As m =

+ m2
1=/? 1_± -11
X Ll^ 2^J 4 ...(0
ur

n^h^im^ 4-m2)
ad

E =
For hydrogen like ion, the second line of Balmer
2mj m2r^
YYo

series is
12. For fifth energy level, «2 = 5 and for ground level,
I
— = RZ^ ~ -L =Rz-xl /2j = I. The wavelength A, of radiation emitted is
d
Re

X' 22 42 16 given by
in

According to question, X = X'


F

i-- A
1 24 R
37?
= RZ^xl or z2 = 4 or Z=2 V ‘ «2 J I 1- 5“ 25
4 16

10. As is clear from Fig. 12(CF).3, Linear momentum of photon emitted


h 24 R
FIGURE 12(CF).3 p=-=hx
25


As hydrogen atom is stationary, therefore,
momentum acquired by electron as a result of
±E-~ .>2
3 24 hR
X photon emission = mv = p .=
25
-2£

24 hR
V =

3£-l- 25 m
12/68 Fundamental Physics (XII)QSMn

16. From Bohr's theory of atomic spectra,


For Difficult Questions
1 2ri 1 ^
- = /?z
13. Here, n - 1 X n
2
1 - j

(\ (I For hydrogen atom, Z = 1 and for visible spectrum,


A£ = I3-6Z- 1- = 13-6Z-
2-4 1 4● «, =2
12 ^ /

/ I \2 / 1 >2 1
- = /?
( 1
= /?
(n^-4]
->2
->
AE 3-4 2 I3-6Z- X 4n-
V - n~ 7
Lv 2J 3

4/1“
5 'l

oww
= \3'6Z- — or x = ...(0
/?(/!- -4)
56 j
As (A£)2 I = (AE)^ 2
Given, x = ...(«)
3^ 5 'i /r-4
●●● 13-6 Z2 - =13-6Z2 + 74-8

e
4 36 From (/) and (//),

FFrlo
re
V y

kn~ 4/r 4
or /t= -

ree
F
or 13-6 Z- =74-8 n 2-4 /^(n--4) R
4 36j
17. Refer to Fig. 12(CF).4 shown here.

rF
' 22 '
13-6Z- = 74-8
,36 , FIGURE 12(CF).4

fsoor
ouur
Vn
74-8x36
skf
Z- = = 9
13-6x22
ooko
+ Ze#— ●e
Yo
Z=^f9= 3
Y
Bo

14. Here. >. = 975 A = 975 X 10-‘°


reB

-34 8
he 6-63x10 x3xl0 1

£= — joule mv~ 1 Zexe k Ze~


uur

fl —
oY

X 975x10 ●> 4
r 4 7t€ r" r“
ad

n 0 It It

-16
6-63x3x10
dY

-19
eV = 12-75 eV
mv
2 _kZe-
975x1-6x10 II
r
It
innd

13-6
Re

.. £..n = -13-6 +12-75 =- 0-85 eV=- 2


Kinetic energy of electron in /ith orbit is
II
Fi

^ I . kZe"-
F

which gives n = 4. i.e., hydrogen atom will be K = -mvzIt


1 2rn
excited to n = 4.

4(4-1) Foi‘ hydrogen atom, Z = 1 and in cgs system k = 1


= 6
Number of spectral lines - ●> 2 e~
■>
e~
1

K =
2r 2r
) 3 ^ 4 M

15. From - = RZ~


18. Using the relation
X r- 2-y 4

1
4 4 - = R
=-i;
3/?
X2=—\
3/?
^3 = 3R(2^) \1R
X nr
->

1
■>

●● /

For last line of Balmer series, //| =2, Hi ~ oo

4 4
X4 -- I ^ R
3R{3^) HR = R ...(0
4
Xq \ ~
Choice (a) is correct.
ATOMS 12/69

For Difficult Questions


It means, the electron is in quantum state, n = 3.
From de-Broglie hypothesis
For last line of Lyman series, «j = 1, U2 = «« '■..2
n
_U0«
2
nX = 27tr=27C — a ●/ r
I 1 1 'I
n
Z ° n
Z
— =R = R ...(«)
12 do

or ^ = 27C —On
Dividing («) by (0, we get Z °
I For lithium, Z = 3 and o = 3, so
^ =4

oww
X = 27t-
^flo=2KUo = pttUo (Given)
1
19. -mv'^=eV or mv-yjlmeV So p=2
22. Let and be the momentum of electron in
Radius of circular path of electron nth state and ground state of hydrogen atom.

ee
mv
_ -iJlmeV _ 1 2mV h

Then P„=y I
r =

FFrlo
qB eB B \ e

r
n g

rF
Bhr^

ee
2
or V =
2m Kinetic energy of electron, K = ~
2m
ouru
rF
_ (3xl0^)2x(l-6xlO-'9)x(10xlQ-2)2 Total energy of electron in a state of hydrogen atom
2x(9xl0“^^) (ft2/A.2)
ffosor
P~
os k E=-K=-
= 0-8 V 2 m 2 m 2 m A.2
KE of electron, K = eV = 0-8 eV -/;2 ~h^
E and =
ook
For transition between 3 to 2, we have
Yo
n
2 m A.2n 2 m X?8
Y
Bo

£’ = 13-6 J ^ 13-6x5
r 1 1_ he
reeB

= 1-88 eV
22 32 36 E -E
2 m X.2
n

Work function, ^q = E~K= 1 -88 - 0-8


g n
oouY
ur

= 1-08 eV=l*l e\
hL he
ad

20. For the emission of X-ray line of wavelength X, 2 m X^ X^


dY

8 «

'-A. = R(Z-\A\/Ij
I
X'^
nidn

2
2 me
'*2 8 n
Re

or A
h ^2n _ ^2
n

8
F
Fi

1 1 1

X = /?(Zc,-l)2 ^ 2
...(/●)
2 me A.2 X2
Cu ”2 8 n

h
1 1 1 X^ I - g

-1)2 X2
n

= R(Z Mo ...(«) n

L"i
2
X
Mo "2
n-1
Dividing (ii) by (i), we get 2 me 2 me A.?5
£ j £ 1 +
h h
n n
Cu _ 41
X
Mo (Z(.„-l)2 (29-1)2 28 2 me
8
2 me
8 1
= A + —r
B

= 214 h X2n
21. Given angular momentum of electron.
2 me X^ 2 me X‘i
3h nh where A =
8
and B -
L = —
h h
2k 2k
12/70 Fundamental Physics (Xll)BSlSD

For second excited state, /» = 3


For Difficult Questions
0

23. For the scries limit frequency of Lyman series,


^2 = 3“ 9
n,I = 1 and _
r,
oo

Hence,
/IV
I
- =£ .40 7-2 £„/9 4
L
oe>~
26. From the knowledge of theory, energy of electron
For the series limit frequency Vp of Pfund series, in the nth stationary orbit of hydrogen atom is
II
= 5 and /I2 “ 00 13-6 ,,
£. =n
n
2"
I 1 £
...(») 27. From the knowledge of Bohr’s theory of atomic
hvp = £ 5^ 25
spectrum, time period of revolution of electron

ww
in nth orbit.
Dividing (;'0 by (0. we get
iTir 2rta(/r tZ)
'jL-1 T =
25
or Vp
^ 25 V vAZIn)
‘’l

Flo
3
n
24. Bohr’s model is not valid for singly ionized Neon .-. T OC

ee
atom. Infacl, Bohr’s model is valid only for atoms

rere
having one electron in the outermost orbit.

r FF
25. Energy of electron in nth orbit is,
T^
7-2 = 8ri=8xl-6x 10-*'^s
uurr
For first excited state, n = 2
foor
New frequency, fij ~
1 10
16

●●● 7*1 = ^=5


8x 1-6
ks s
0 7-2
Yoo
2^ 4 = 7-8 X lO*-* Hz
ooook

ni Multiple Choice Questions {with One or More than One Correct Answers)
eBB

When atom dc-excites from 3rd to 2nd shell,


uurr

nh 3/1
28. As mvr = n = 3
ad

2jt 27t 1 1 1 5R
’r
Yo

— = 4/?
2 2 ^2 4 9 9 ^2-5
dY

n n
0
r
" nmZe^ Z Time^ Z
Re

29. RadiusofBohr’sorbitisgivenby r =
innd

4Tt-fn£^-
2
FFi

n
4-5 fln = — a 0
Hence r<^rP'
° Z angular momentum of e~ in nth orbit is integral
nh
.. ^0
Kme
y = £Iq (Bohr’s radius) multiple of —
2k
as mv r =
2k

When Z = 2 KZe'^
magnitude of RE. =
r
1 1 1 1
^X = RZ^ —
n~( n;I
2
n
2 2
nt\ KZe~
/ / and magnitude of K.E. =
2r
When atom deexcites from 3rd to 1st shell
So magnitude of RE. is greater than that of K.E.
1 1 32 R
— = 4R 1 2k r.II
1 9 9 A.,● = —
32 R 30. As « n^. II
II
OC —
, time / =
n V
II
ATOMS 12/71

: vj.rrrT:
E
For Difficult Questions 1
As P = — , therefore, = c
c
Pi ^2
r, n
1
oc -ii- or r,, oc tv'. i.e., —
5
tJ
n T'2 «2 i.e. b = c =
27

n
or
I
or
(2)3 = ^ As £ =—
he
X OC
1

V - / V - >
X E

hence /ij = 2 n-y, \ 27 1


or a = — or c = —
when (a) «| = 4, «2 = 2 and (^/) ;i j = 6, «2 = 3 X^ E.I 5 c a

2%Ke~ 27cii:e2 -)
31. y = vn = n-
y/i /i 34. As radius, /' oc

w
z
.●. vn is independent of n.
Ar (n + l)~-n~ 2n + l 1
1

Flo
OC —

Also E oc
and r °c r n~
1
n
2
n
2
n

Choices (a) and (h) are correct.

reee
E r is independent of n.
A 4 z-

FFr
■> n
32. As r oc n-
n —
As energy E oc
1 ●
n~
/I,
1
urr
In
A

A,
n
= 41n (n). AE

E
for
tr (/t + l)3
l/(n + I)3
(n + \)--n-
«3(„ + l)
kkss
(a
The graph of In —^ against In (n) will be a
Yo
AE 2n+l
oooo

OC —
2
E n n
eB

straight line with slope = 4. Choice (c) is incorrect.


Also, it will pass through the origin. nh
33. First transition is from n = 3 to n = 2. Second Angular momentum, L =
r

271
ou

transition is from n = 2 to n = 1.
ad

AE {n + \)h-nh I AE
YY

OC
E
1
1/2^-1/33 ^ 5/36 5 4 5 E nh n L n

E-y 1/i2-1/23 3/4 36 3 27 Choice id) is correct.


nndd
Re

m Multiple Choice Questions (Based on the given Passage/Comprehens ion)


Fi

35. Nuclear size is of the order of 10"*^ m


10-10 m
= 10«5
Atomic volume 10 m
36. ^atom
Nuclear volume V

nucleus
37. Thomson model of atom could not explain large
angle scattering of alpha particles.

03 Matching Type Questions

38. According to Thomson atom model, charge and stationary orbits around the nucleus. Ionization
mass of atom are distributed uniformly in a sphere. potential is fixed for an atom.
Rutherford atom model introduced the concept Choice (a) is correct.
of nucleus. Bohr atom model gave the concept of
12/72 ’4. Fundamental Physics (XII)

For Difficult Questions

Matrix-Match Type Questions

-e^ Distance of closest approach of a-particle and


39. In hydrogen orbit, PE of electron =
47t€«0 r impact parameter, both vary inversely as KE of
a-particle.
KE of electron = e-/87t e q r
Hence A-r; B-p ; C-q ; D-/; E-r.
Total energy of electron = - €q r

VI. Integer Type Questions

w
40. Here. X. = 2 Tt .x 4n- kn^ where
X =
Radius of 3rd orbit is 3^ x = 9 a: 2-4’
Rin^-4) n

Flo
It must contain 3 de Broglie wavelengths.
3V = 2jc(9a)

e
m

= 6 Tt .X n = 6 k =A!R = —. where m = 4

rree
R
41. As is clear from Fig. 8(CF).2.

r FF
43. Here, X = 970 A
£'j — S') +
he 12375
uurr
he _ he be
X,| ^2 for
£ = —
X 970
eV = 12-7eV

After excitation, let electron jump to nth state


kss
1 _j L-i_i ^
— X2 - 6 units
ooook

13-6
Yo
Xt X,1 Xj 2 3 n
2
= - 13-6-H 12-7 = 0-9
eB

1
42. From ~ = RZ~ —
n
2 _13-6 =151=16
X nr n::
1 2 0-9
urr

For hydrogen, Z = 1, For visible radiation, n = 4


ad

/i2 = n ; =2
Yo

Total number of lines in the emission spectrum


(n2-4)
dY

1 1 1
- = R T =/? n(n-l) 4(4-1) ^
X 22 n~ 4n^
Re
innd
Fi

VII. Assertion-Reason Type Questions

FOR MEDICAL STUDENTS 46. Assertion is false and the reason too is false.

44. Assertion is true, but the Reason is false.


FOR ENGINEERING STUDENTS
1 I 1 ■
For Balmer series, — = R
’ X 22 n
2 ’ 47. Both the statements are correct, and the latter
where n = 3, 4, 5 is a correct explanation of the former.
When we put n = 3,4,5.... and R - 10“^ m“' in the 48. Both the statements are true and the latter is
given formula, the values of X calculated lie correct explanation of the former.
between 4000 A to 8000 A, which is visible region.
49. Both the statements are true and .statement-2
45. Assertion is true, but the reason is false. Number
of electrons in isotopes of an element is same. is correct explanation of the statement-1.
loow w
NUCLEI
13.1. INTRODUCTION

ee
In the last chapter, we learnt that atomic nucleus was discovered in the year 1911 by Rutherford and his
associates from the study of large angle scattering of alpha particles from thin metal foils.

Fr
r FF
Rutherford established that atomic nucleus is the central core of every atom which contains entire positive
charge and more than 99-9% of the mass of the atom. Whereas the size of an atom is of the order of 10"^^ m,

rree
the size of atomic nucleus is of the order of 10“'^ m. Thus, most of the space around the nucleus of an atom
is empty space. fofr oF
u
In this chapter, we shall study the constituents of the nucleus and how they are held together. We shall
ks
also study size, mass, density and stability of nuclei. Finally, we shall have a look at the associated nuclear
YYouro
phenomena such as radioactivity, nuclear fission and nuclear fusion,
soo

13.2. ATOMIC MASSES


BBook
r ee

For an insight into the nuclear phenomena, it is essential to determine accurately, the atomic mass of an
element. The instrument designed for this purpose is called a mass spectrometer. Two commonly used mass
ouru

spectrometers are due to Aston and Bainbridge.


ad

The unit in which atomic and nuclear masses are measured is called atomic mass unit (a.m.u.) or
Yo

unified mass unit (u).


d

One atomic mass unit or uniifed mass unit is defined as 1/12 th of the mass of an atom of
Re
iYn

isotope.
FFind

As Avogadro’s number = 6-023 x 10^^, Mass of 6-023 x 10^^ atoms of = 12 g


12
Mass of one atom of = g
6-023x10^^
1
By definition, 1 a.m.u. = — X mass of one atom of
12
1 12
1 a.m.u. = —X 8 = 1-66 X 10-24 g
12 6-023x1023

1 a.m.u. = 1-66 x 10 2^ kg
As an atom of ^C^2 contains 12 nucleons, therefore, one a.m.u. represents the average mass of a nucleon
and is denoted by u.
13/1

i
13/2 'P.netdce^ 'a Fundamental Physics (XII) ^wii

In tenns of this unit, mass of an electron {m^) = 0-00055 u, mass of proton = 1-0073 u
mass of a hydrogen atom = -f nip = 1 -0078 u, mass of a neutron (m„) = 1 -0086 u.
Atomic masses can also be expressed in terms of unit of energy, which is electron volt.
One electron volt is the energy gained by an electron, when accelerated through a potential
difference of one volt.

As work done = charge x potential


1 eV= 1-602 X 10-'^ (Ox ! (V) = 1-602 x 10"*^ joule.
I MeV = i million eV = 10^ eV = 1-602 x 10"'^ joule
Relation between a.ni.u. and MeV
According to Einstein, mass energy equivalence is represented by £ = mc-
Taking m = 1. a.m.u. = 1 -66 x 10“^^ kg. and c = 3-0 x 10^ m/s

llowow
we get, £ = {l-66x iO-^"^) (3-0 x 10^)2j= 1-49 x 10,-10 j
-10
1-49x10
^= n MeV = 931-25 MeV
1-6x10"’^

ee
Hence, 1 a.m.u. » 931 MeV , which is used as a standard conversion.

Fr
133. COMPOSITION OF NUCLEUS
r FF
The study of radioactive disintegrations indicates the emission of alpha, beta and gamma particles/rays

r
to be of nuclear origin. Further, study of artificial radioactivity has revealed that many other particles like
forfFore
u
proton, neutron, alpha particles, beta particles and sub-particles like meson, positron, neutrino enter into the
s
constitution of the nucleus in one way or the other. Based on these studies, several hypothesis have been pul
ok
YYour o
forward about the structure of the atomic nucleus. Prominent among them are :
o

1. Proton-Electron hypothesis,
2. Proton-Neutron hypothesis,
eeBoBks

(a) Proton Electron hypothesis


r

This was put forward in the year 1930 to account for the emission of alpha and beta particles from the
our u
ad

nuclei of radioactive elements.The positive charge in the nucleus is that of protons. A proton carries one unit
of fundamental positive charge and is stable. An electron carries one unit of fundamental negative charge and
Yo

its mass is negligible compared to the mass of proton.


According to proton electron hypothesis, the nucleus of an atom of mass number A and atomic number
d
Re

Z is made up of A protons and (A-Z) electrons. As every atom is electrically neutral, it must contain Z more
in

electrons. They revolve around the nucleus in circular orbits. However, this hypothesis was rejected later in
FFind Y

view of the following problems :


(/) According to de Broglie hypothesis and Heisenberg’s uncertainty principle, if an electron is to exist
inside the nucleus, it should possess energy ranging from 20 MeV to 200 MeV.
But the energy of electrons emitted during [3-decay is at the most 2 to 3 MeV. Therefore, existence of
electrons inside the nucleus is not justified on the basis of wave mechanics.
(//) The observedvalues of nuclear spin or angular momentum of nuclei rule out the possibility of existence
of electrons inside the nucleus.

{Hi) Experimentally, the values of magnetic moments of nuclei are much smaller than the magnetic moments
of electrons. Therefore, electrons cannot exist inside the nucleus,
(/v) The presence of a few elecQons inside the nucleus and others revolving in orbits around the nucleus
show dual role of electrons in the atomic structure, which is difficult to visualize.
Discovery of Neutron
A neutron is a neutral particle caiTying no charge, and having mass equal roughly to the mass of a
proton. This particle was discovered experimentally by Chadwick in the year 1932 :

1
NUCLEI 13/3

(<) Chadwick observed that when beryllium was bombarded with alpha particles, some neutral radiations
were emitted, which could knock out protons from light nuclei like helium, carbon and nitrogen.
(//) Application of the principles of conservation of energy and momentum showed that neutral radiations
from bombardment of beryllium could not be photons. Chadwick solved this puzzle by assuming that
these neutral radiations consisted of hitherto unknown neutral particles - which were called neutrons.
Chadwick estimated the mass of a neutron being roughly equal to mass of a proton.
yVhereas a free proton is stable ; a free neutron is unstable and has a mean life of 1000 second.
Neutron is however, stable inside the nucleus.
Chadwick was awarded the 1935 Nobel Prize in Physics for his discovery of neutron.
(/») Proton-neutron hypothesis
This hypothesis was put forward by Heisenberg after the discovery of neutron by Chadwick.
According to this hypothesis, a nucleus of mass number A and atomic number Z contains Z protons, and

loow w
(A-Z) neutrons. As an atom is electrically neutral, therefore, number of peripheral electrons must be equal to
Z, the number of protons inside the nucleus.
This hypothesis accounted for all the discrepancies of proton electron hypothesis.
As is known, a neutron is a neutral particle carrying no charge. A proton carries a unit positive charge.

ree
Proton is slightly lighter than a neutron,
mass of proton, = 1-6729 x 10“^^ kg = 1-007825 amu

rree F
mass of neutron, m„ = 1-6743 x 10“^^ kg = 1-008665
r FF amu
A proton and a neutron may be regarded as two different states of the same particle, called nucleon.
As the atom is electrically neutral, number of electrons in an atom is equal to the number of protons
inside the nucleus. fofr oF
u
Atomic Number of an element is the number of protons present inside the nucleus of an atom of the
ks
element. It is also equal to number of electrons revolving in various orbits around the nucleus of the atom. It
YYouro
soo

is represented by Z.
Mass Number of an element is the total number of protons and neutrons present inside the atomic
BBook

nucleus of the element. It is represented by A.


r ee

Thus, in an atom, number of protons = Z number of electrons = Z


number of nucleons = A number of neutrons = (A — Z)
ouru
ad

A Nuclide is a specific nucleus of an atom, which is characterised by its atomic number Z, and mass
Yo

number A. It is also called nuclear species.


It is represented as 2^^ where X is the chemical symbol of the species. For example, a gold nucleus is
represented by It contains 79 protons and 197 - 79 = 118 neutrons.
d
Re
iYn

More about Proton and Neutron


FFind

Proton

Proton is a constituent particle of atomic nucleus. It carries unit positive charge = I -6 x 10"*^ C and a

-27 1
mass equal to 1-6729 x 10 kg = 1 -007825 u. It has spin or intrinsic, angular momentum = - {hll 7t),
where h is Planck's constant.

Proton is the lightest baiyon with a magnetic moment of 2-79 nuclear magneton. Number of protons in
a nucleus gives us atomic number of the nucleus.
Neutron

A neutron is a constituent particle of atomic nucleus. It carries no charge, and has a mass
= 1-6743 X 10“^^ kg = 1-008665 u, which is slightly more than the mass of a proton.
Inside the nucleus, a neutron is a stable particle. However, outside the nucleus, a neutron is unstable
having a mean life time of 1000 s. It decays into a proton, an electron and antineutrino.
o'' > I,//' +_ie^+ V

>
13/4 it Fundamental Physics (XII)
1
Neutron has a spin (intrinsic angular momentum) equal to — (/i/2 Jt), where h is Planck’s constant.
Magnetic moment of a neutron is - 1-91 nuclear magneton. Neutron has low ionising power.
As a neutron carries no charge, it is neither attracted nor repelled by the nucleus. Therefore, a neutron
has a large penetrating power.
A fast moving neutron can be slowed down by certain materials like ordinary water, graphite, heavy
water. These materials are called moderators. Slow neutrons, which are in thermal equilibrium with the
molecules of the moderator are called Thermal Neutrons. Their energy is of the order of 0-025 eV. Thermal
neutrons are most suitable for causing nuclear fission.
13.4. NUCLEAR SIZE

Experimental measurements show that volume of a nucleus is proportional to its mass number A. If/?

ww
is the radius of the nucleus assumed to be spherical*, then its volume OC A
3
1/3 1/3

Floo
or Roc A or R = RrsA
0 ...(1)
-15
where Rq is an empirical constant whose value is found to be 1-2 x 10 m.

ree
As A is different for different elements, therefore, atomic nuclei of different elements have different sizes.

rFee
13.5. NUCLEAR DENSITY

F
oor r
Density of nuclear matter is the ratio of mass of nucleus and its volume.
rur
s ff
If m is average mass of a nucleon and R is the nuclear radius, then, mass of nucleus = mA, where A is the
mass number of the element.
osk
YYoou
4
Volume of nucleus = — nR =
|tc(/?o =^nR^A
oook

0
3
eBB

mass of nucleus mA
As density of nuclear matter = P =
volume of nucleus
uur r
ad

3m
Yo

Thus, P = 5 ...(2)

As m and Rq are constants, therefore density p of nuclear matter is the same for all nuclei.
dY
Re

Using 1-66X 10-2'^kg, /?r,= 1-2 X 10-^5 m


idn

0
FFin

3xl-66xl0~^^
we get. -15^3 = 2-29 X 10*'^ kg/m^,
4x3-14(l-2xl0 )
which is very large as compared to density of ordinary matter. For example, for water, p 10^ kg/m^
and for air, p = 1-293 kg/m^. Hence matter in the nucleus is very densely packed.
Sample Problem Obtain approximately the ratio of nuclear radii of and 92U^^*. What
is the approximate ratio of their nuclear densities ?
Sol. Here,Ai = 56,A2 = 238
R
1 _
A1 f ^ 56
n1/3

= (0-235)^/3 ^ 0-617
238

P]
As nuclear density is same for all nuclei, therefore, — -1
P2
*Though most of the nuclides are spherical, some of them are ellipsoidal, too.
NUCLEI 13/5

13.6. ISOTOPES

Isotopes of an element are the atoms of the element which have the same atomic number (Z)
but different mass number (A).

For example, [//’, are the isotopes of hydrogen. isotopes of helium.


5C‘®, gC*’, gC*^, gC*'^ are the isotopes of carbon.
As isotopes of an element have the same atomic number, they contain same number of protons and
same number of electrons. But as their atomic weights are different, they contain different number of
neutrons.

Isotopes of an element have identical chemical properties. Their physical properties, however, differ.
All the known elements have one or more isotopes. The relative abundance of different isotopes differs

ww
from element to element. The atomic weight of an element is the weighted average of the masses of all its
isotopes based on the occurrence of each isotope in nature. For example, neon has two isotopes of masses 20
and 22 which occur in the ratio, 9:1.

Flo
20x9 + 22x1 180 + 22
Average atomic weight of neon = = 20-2

e
(9 + 1) 10

rere
On the contrary, mass number A is always an intege,r as it represents the number of nucleons in the nucleus.

r FF
13,7. ISOBARS
uurr
for
Isobars are the atoms of different elements which have the same mass number (A), but different
atomic numbers (Z).
kss
Isobars contain different number ofprotons, different number of electrons and aho different number of
ooook
Yo
neutrons. Only the total number of nucleons in them is the same.
For example, jjA/d^^and ^^Ne are isobars. Similarly,
eBB

22

are also isobars.


^nd \gAr^^ are isobars and ig5^'^
Similarly, and 3//^^ are isobars. Also, gC’"* and are isobars.
urr

The chemical properties of isobars are widely different. Their physical properties may be identical. The
ad

isobars occupy different places in the periodic table.


Yo
dY

13.8. ISOTONES
Re
innd

Isotones are the nuclides which contain the same number of neutrons i.e. in their case,
(A-’Z) = N is the same. Their atomic number (Z) and mass number (A) are different.
Fi

For example, and are isotones. This is because number of neutrons in chlorine = 37 - 17 =
20 and number of neutrons in potassium = 39 - 19 = 20.
Similarly, j//^ and are isotones. Also, and gC^'* are isotones.
Some other examples of isotones are :
460^ and 56'° ; gC*^ and ; I! Na^^ and |2Mg^.
Sample Problem Choose One pair of isotopes, isobars and isotones of the following :
18 Ar^O ; :
’ 11 Na23 ;
Sol. (i) 17CP^ and are isotopes with same Z= 17 and different A = 37 & 35
(ii) igAr'^® and 20^3"^® are isobars with same A = 40 and different Z = 18 & 20
{Hi) iiNa^^ and |2Mg^“^ are isotones with same number of neutrons = (A - Z) = 12
13/6 “Pnaideefi- ’4. Fundamental Physics (XII)
13.9. MASS-ENERGY RELATION
Einstein was the first to establish the equivalence of mass and energy through the famous relation
E = mc^

where c = 3 X 10^ m/s = speed of light in vacuum.


The relation means that when a certain mass m disappears, an equivalent amount of energy E appears

w
and vice-versa.

The mass of a particle measured in a frame of reference in which the particle is at rest is called its rest
mass, usually denoted by niQ. The rest mass energy of particle would be «1qC~, which is enormously large on
account of large value of c. If T is kinetic energy of the particle, then its total energy

e
E - mc^ = rest mass energy + K.E. = +T ...{3)

row
re
Here, m is called effective mass of the particle, when it is moving. Clearly, m > ntQ.
T = mc~ - wiq = (m - Wq) (-p

eeF
From eqn. (3),

ullo
FF
7’=(Am)c2
Thus K.E. of a particle = change in mass of particle x (speed of light in vacuum)^.

srr
roF
In nuclear and elementary particle interactions, both the conversion of masH into energy and tlie conversion

k
of energy into mass lake place. Therefore, the two classical laws of conservation of mass and conservation of
uor
ofof
energy have been unified into one \aw-the law of conservation of mass energy. According to this law, the
sum of the mass-energy of a system of particles is the same, before and after an interaction.
kos
Y
This law of conservation of mass energy has been verified in all nuclear reactions.
Yo
eerBB

Express 150 microgram mass into equivalent energy in electron volt.


oo

Sample Problem

m - 150 microgram = 150 x 10^ g = 150 x 10"^ kg, £ = ?


rY

Sol. Here,
From £ = mc2 = (150 X 10-9) (3^ io8)2= 1-35 x 10‘“ J
u

1-35 xlO*^^
ou

eV = 0-843 x 10^9 eV
o
ad
d

£ =
1-6x10"'9
nY

13.10. NUCLEAR BINDING ENERGY


nid
Re

(a) Concept of binding energy


F

An atomic nucleus is a stable structure. Therefore, nucleons in every nucleus are bound together with
Fi

short range interacting forces, called nuclear forces. A definite amount of work has to be done to separate the
nucleons from the nucleus to such a distance that there is no interaction between them. This work done is a
measure of binding energy of the nucleus. Thus,
Binding energy of a nucleus is the energy with which nucleons are bound in the nucleus. It is
measured by the work required to be done to separate the nucleons an infinite distance apart
from the nucleus, so that they may not interact with each other.
The origin of nuclear binding energy has been explained on the basis of Einstein’s theory of mass
energy equivalence. It is found that rest mass of a nucleus is always slightly less than the sum of the rest
masses of free neutrons and protons composing the nucleus. This is as if certain mass disappears in the

formation of the nucleus. This difference between the sum of the masses of neutrons and protons forming
a nucleus and mass of the nucleus is called mass defect. It is this mass defect which appears in the torm of
binding energy, responsible for binding the nucleons together in the nucleus.
NUCLEI
13/7

(b) Expression for Nuclear Binding Energy


In a nucleus
Z = charge number = number of protons.
A - mass number = number of protons plus the number of neutrons
number of neutrons = (A-Z)
Let
= mass of a proton,
= mass of a neturon
m
N = ma.ss of nucleus, ^
Mass defect, Am = [Zmp + (A-Z) HI
m^] ...(3)

Using Einstein’s mass energy equivalence. Binding Energy = Atti.c'^


B.E. = fZm^ + (A-Z) m„ - ...(4)

ww
where c is velocity of light in vacuum.

If m is the mass of the atom ^ containing Z electrons each of mass m^, then we can write

Floo
B.E. of electrons
m
izX^) = ,n {zX^) + Zm^-

ee
N ...(5)

eer
For hydrogen atom of mass containing one electron, we can write

FrF
B.E. of one electron
H =7n^^+m^- ...(6)
m

oor r
ur r
s ff
Now, the binding energy of electrons is much smaller = eV to keV, as compared to the rest mass energy
of a nucleon » 10 MeV, therefore, we can safely ignore in (5) and (6), the terms involving binding energy of
sk
electrons.
YYoou
oooko

Eqns. (5) and (6) reduce to m


(zX^) = m (zX'^) + Zm, ...(7)
eBB

N
and
...(8)
Rewriting eqn. (4), as B.E. = {Zmp + Zm^ + {A - Z) - m
N - Zm^
uurr

B.E. = [Z (mp + mj + {A~Z) - (m^ + Zm^,)] (?■


ad
Yo

Using (7) and (8), we gel B.E. = [Znif^ + (A - Z) -m c~ ...(9)


dY

This is the required expression for B.E.


Re
ind

(c) Average Binding Energy per nucleon of a nucleus


FFin

It is the average energy we have to spend to remove a nucleon from the nucleus to inifnite distance. It
is given by total binding energy divided by the mass number of the nucleus.
The variation of average B.E. per nucleon with mass number A is shown in Fig. 13.1. This curve reveals
the following facts :
1. Average B.E./nucleon for light nuclei like j//', is small.
2. For mass numbers ranging from 2 to 20, there are sharply defined peaks corresponding to
g(9 etc. The peaks indicate that these nuclei are relatively more stable than the other nuclei in their
neighbourhood.
3. The B.E. curve has a broad maximum in the range A = 30 to A = 120 correspoding to average binding
energy per nucleon = 8-5 MeV. The peak value of the maximum is 8-8 MeV/N for
4. As the mass number increases, the B.E./nucleon decreases gradually falling to about 7-6 MeV per
nucleon lor The decrease may be due to Coulomb repulsion between the protons. The heavy nuclei
are therefore, relatively less stable.
13/8 'P'uutee^'4. Fundamental Physics (XII)KiSiSUl

FIGURE 13.1

? 10
56

W
32s Fe
1^0 IQQMo 127i 197
8
12c
<v
o ^He 18q 184^ 238u
D
14n
6
a>
Q.
®Li
>.
cn
0)
c 4
LU
Oi
c 3H
●D
c 2

ww
£0

o> '2h
>
< 0
0 50 100 150 200 250

Flo
Mass Number (A)

e
(cl) Importance of Binding Energy Curve

eree
(/) When we move from the heavy nuclei region to the middle region of the plot, we find that there will
be a gain in the overall binding energy and hence release of energy. This indicates that energy can be released

FFr
when a heavy nucleus (A = 240 ) breaks into two roughly equal fragments. This process is called Nuclear

oorr
uur r
Fission.
(//) Similarly, when we move from lighter nuclei to heavier nuclei, we again find that there will be
sf
gain in the overall binding energy and hence release of energy. This indicates that energy can he released
when two or more lighter nuclei fuse together to form a heavy nucleus. This process is called Nuclear
sk
Yoo
ooko

Fusion.
Hence, the importance of binding energy curve is that it led us to the possibility of release of nuclear
eBB

energy by nuclear fission and nuclear fusion. These phenomena of nuclear fission and nuclear fusion were
actually discovered later.
uurr

Retain in Memory
ad
Yo

When mass defect (A m) is in a.m.u., energy released in MeV can be calculated directly by
multiplying mass defect with 931 (MeV). There is no need to apply ; E = (Am)c^.
dY
Re
innd

Sample Problem Calculate mass defect, binding energy and binding energy per nucleon for
FFi

a lithium nucleus taking its mass = 7*000000 a.m.u., Mass of proton = 1*007825 a.m.u. and mass
of neutron = 1*008665 a.m.u. Take 1 a.m.u = 93T5 MeV.
Sol. Here in ^Lp, number of protons - 3, number of neutrons = 7-3 = 4
.*. mass defect, Am = 3 m^, + 4 m n
-M

A m = 3 X 1-007825 + 4 x 1-008665 - 7-000000


= 7-058135 -7-000000 = 0*058135 amu
Total BE = 0-058135 x 931-5 MeV = 54-153 MeV
54-153
BE/nucleon = = 7*736 MeV/N
7

13.11. PACKING FRACTION

Packing fraction of a nucleus is defined as the mass excess per nucleon.


NUCLEI
13/9

Mass excess is the difference between the mass of a nucleus and mass number of the nucleus. Thus,
Mass excess M - A
Packing fraction = ...(10)
Mass number A

where M is the actual weight of a nuclide (on the physical


atomic weight scale) and A is the mass numbe.r While A is a
whole number, M is not necessarily so.
The variation of packing fraction with mass number
(A) is shown in Fig. 13.2.
Infact, packing fraction measures the stability of a
nucleus. Smaller the packing fraction, larger is the stability

ww
of the nucleus.
From the graph, we find that
(0 for A = 16, packing fraction is zero.

Flo
(») for very light nuclei (A < 16), packing fraction Is large. Therefore, they are unstable.

e
{Hi) As A increases beyond A = 16, packing fraction becomes negative, decreases with increasing mass

eree
number. Beyond certain value of mass number, packing fraction starts increasing till it becomes zero. At
about A > 240, packing fraction becomes large. Thus heavy nuclei are unstable.

FFr
Clearly, nuclei of intermediate mass numbers are relatively stable.
uurr
orr
13.12. NUCLEAR FORCES
sfo
We know that the force that determines the motion of atomic electrons is the Coulomb force of attraction
between electrons and the nucleus. In the previous article, we noted that for nuclei of average mass, binding
kks
Yoo
energy per nucleon is about 8 MeV, which is pretty large. Therefore, to bind a nucleus, there must be very
oooo

strong attractive forces. Such forces must overcome repulsion between positively charged protons in the
eBB

nucleus ; and bind both protons and neutrons into tiny nuclear volume (= IQ-^^ m^). These forces arc called
nuclear forces.
urr

Nuclear forces are the strong forces of attraction which hold together the nucleons (neutrons
and protons) in the itny nucleus of an atom, inspite of strong electrostatic forces of repulsion
ad
YYo

between protons.

These forces are very complex in nature. Some of the important characteristics of the.se forces
dd

are:
Re

1. Nuclear forces act between a pair of neutrons, a pair of protons and also between
inn

a neutron, proton
pair, with the same strength. This shows that nuclear forces are independent of charge.
F

2. Nuclear forces are the strongest forces in nature. The magnitude of nuclear forces is 100 times that
of electrostatic forces and 10'^^ times that of gravitational forces between nucleons. That is why nucleons are
held together in a nucleus inspite of electrostatic force of repulsion between protons.
3. The nuclear forces are very short range forces. They are operative upto distances of the order of a
few fermi, i.e.. they operate only within the nucleus.
4. The variation of nuclear forces with the distance between nucleons is not known exactly. However,
from Fig. 13.3, we observe that
(/) Nuclear forces are negligible, when distance between nucleons is more than 10 fermi.
(ii) when nucleons are brought closer, nuclear force of attraction develops which goes
’ ’' on increasing
'
rapidly with decreasing distance. However nuclear forces do not obey inverse square law.
{Hi) When distance between nucleons becomes less than 0-8 fermi, the nuclear forces become strongly
repulsive.
Fig. 13.4 shows a rough plot of potential energy between a pair of nucleons with distance (r) between
them.
13/10 Fundamental Physics (Xll)Eiaau

The potential energy is minimum at a distance Kq = 0-8 tm. At


this distance, force between nucleons is zero. For distances larger
than 0-8 fm. netative RE. goes on decreasing. Tlie nuclear forces
are attractive. For distances less than 0-8 fm; negative RE. decreases
to zero and then becomes positive. The nucleiir forces are repulsive.
Figs 13.3 and 13.4 are compatible.
5. The nuclear forces show saturation properties i.e. each
nucleon interacts with its immediate neighbours only, rather than
with all the other nucleons in the nucleus.
6. Nuclear forces are non central forces. This shows that the
distribution of nucleons in a nucleus is not spherically symmetric. Y
FIGURE 13.4

7. Nuclear forces are dependent on spin or angular >

ww
momentum of nucleons. Force between nucleons having parallel I 40
spins is greater than the force between nucleons having antiparallel LU
Q.
0.8 fm 10fm
spins. A 0 X

Flo
8. Nuclear forces are due to exchange of n mesons between t> r (fm)
the nucleons, as detailed in Art. 13.13. That is why they are called

ee
-40
Exchange Forces.

rere
Y'
13.13. NATURE OF NUCLEAR FORCES ’

r FF
A Japanese Physicist Yukawa postulated in the year 1936 that nuclearforces between nucleons arise on
account of continuous exchange of particles called mesons between the nucleons. Later on, these particles
uurr
foor
were detected experimentally. They were found to have a mass about 270 times the mass of an electron. Three
types of mesons were detected. Mesons carrying no charge were called neutral pi meson Mesons carrying
ks s
unit positive charge each were called positive k meson (n*) and mesons carrying unit negative charge each
Yoo
oook

were called negative Jt meson (7T).


According to meson theory or Yukawa theory of nuclear forces,
eBB

(0 All nucleons consist of identical cores surrounded by a pulsating cloud of 7i meson.


(//) Mesons or pions may be neutral (TC*^) or carry either charge (Jf*’ or n ).
uurr

{Hi) The difference between a proton and a neutron is essentially in the composition of their respective
ad

meson clouds.
Yo

C/v) The force between neutron and proton is due to exchange of charged meson between them, i.e.,
dY

0
p* + K~ - n
Re
innd

= P

Thus proton and neutron continuously exchange tlieir nature by absorbing and emitting 7C mesons. Thus
FFi

0
¥ p* +7C and
n » n^ + TC'*'
(v) The forces between a pair of neutrons or a pair of protons ore
0
the result of the exchange of neutral
0
meson (7t^) between them, i.e.. P ^ p' + n or p + TZ
0
and n > n + or n + K n

In both ihe cases, a proton or a neutron is converted into a new state after emitting or absorbing the tiP meson.
Thus exchange of n meson between nucleons keeps the nucleons bound together. It is responsible for
the nuclear forces.

13.14. NUCLEAR STABILITY


Atomic nucleus consists of a closely packed collection of protons and neutrons. The very large repulsive
electric forces between protons should cause the nucleus to fly apart. Nucleons are able to stay together
despite the repulsive electric force between protons, because of a much stronger force, called the nuclear
force. It is the attractive force that acts between all nucleons in the nucleus and this force is much stronger
than the Coulomb repulsive force between protons.
NUCLEI
13/11

There are 103 different elements including the


transuranic elements. Z = 1 is hydrogen and Z =
103 is Lawrcncium. All these elements have isotopes
and as such there are 2500 different nuclides. Out
ot these, 280 are stable and the rest are unstable.
Nuclear stability depends on many factors. For
example :
(/) The stability of a nucleus is determined by
the value of its binding energy per nucleon. Higher ■
the B.E/nucleon, more stable is the nucleus.
{ii) The stability of a nucleus is also determined
by its neutron to proton ratio. A plot of neutron

w
number (AO and proton number (atomic number Z) ^
for stable nuclides is shown in Fig. 13.5. The solid
line in the plot shows the locations of nuclei that have an equal number of protons and neutrons (N = Z). Note

Flo
that only light nuclei are on this line i.e. light nuclei are stable only if they contain about the same number of
protons and neutrons. Heavy nuclei, on the other hand, are stable only when they have more neutrons than

ee
protons. Thus, heavy nuclei are neutron rich compared to lighter nuclei. This is as if neutrons help to stabilize

Fr
a nucleus. This is easily understood. More is the number of protons in the nucleus, greater is the electrical
repulsive force between them. Therefore, more neutrons are needed to provide the strong attractive forces
necessary to keep the nucleus stable. The long narrow region in Fig. 13.5, which contains the cluster of short

for
ur
lines representing stable nuclei is referred to as the valley of stability.
(ni) The stability of a nuclide is also determined by the consideration whether it contains an even or odd
number of protons and neutrons. A detailed .study shows
ks
that stable nuclei often contain even number of
Yo
protons or neutrons or both as listed in Table 13.1.
oo

TABLE 13.1. Even/odd Number of protons and neutrons in stable nuclei


eB

Number of stable nuclei Number of protons (Z) Number of neutrons (N) u.


ur

165
ad

even even
Yo

57 even odd
53 odd even

6
nd

odd odd
Re
Fi

13.15. RADIOACTIVITY

Natural radioactivity is a spontaneous and self disruptive activity exhibited by a number of heavy
elements occuring in nature. The word spontaneous means shear out of nature of the element and self disruptive
means that the element disintegrates itself, i.e., no external provocation influences the emission*.

Radioactivity is, therefore, the property by virtue of which a heavy element disintegrates itself
without being forced by any external agent to do so.

The phenomenon was discovered by a French Physicist, Henry Becquerel in 1896. He observed that
uranium salts possessed a peculiar property of affecting a photographic plate even when the plate was in a
light proof package. This he thought, must be due to certain active radiations emitted by uranium salts. These
radiations were called Becquerel rays. The phenomenon of emission of active radiations by an element was
termed radioactivity. The element exhibiting this property was called radioactive element.
*AIso, the emission cannot be controlled by physical or chemical means.
13/12 “PnaiUefa. ^ Fundamental Physics (XII) EjgaiiJ
The total number of radioactive elements known at present is about 40. For instance, natural elements
with atomic number greater than 82 are all radioactive. Their nuclei are thus Unstable nuclei. Some examples
are Radium, Thorium, Actinium, Polonium etc.
Effect of physical conditions
The experimental evidence reveals that the phenomenon of radioactivity, is not at all affected by the
imposed conditions of temperature, pressure, chemical combination etc. Therefore, electrons orbiting the
nucleus were not responsible for radioactivity. Hence, the radioactivity must be property of heavy nuclei
only.
Cause of Radioactivity
nucleus is unstable. We notice from the binding
Emission of active radiations indicates that parent
energy curve [Fig. 13.3] that B£/nucleon goes on decreasing, though slowly, for nuclei beyond A > 140. This

ww
indicates that stability decreases as we move towards heavier nuclei. The heavy nuclei have larger atomic
number Z and hence contain larger number of protons. The mutual repulsion of protons reduces the binding
effect of nuclear forces. This is the main cause of relative instability of heavy nuclei. Radioactivity results
from this instability.

Flo
e
13.16. THREE TYPES OF RADIATIONS

eree
Experimental investigations by Rutherford and others have proved that the radiations emitted by

FFr
radioactive elements consist of three kinds, depending upon their ability to penetrate matter. One kind of
radiations with least penetrating power (i.e. the one which would stop first) was named Alpha rays.
uurr
orr
The other kind with a comparatively larger penetrating power was called Beta rays.
sfo
The last kind with maximum penetrating power was called Gamma rays. It should be clearly noted
that all the three types of radiations need not necessarily be emitted by one radioactive element.
kks
Yoo
NATURE OF a, p, y-RAYS
oooo

A number of attempts were made to study the nature of a, P and y-rays. One of the methods used is
shown in Fig. 13.6. The sample of a radioactive element (say, radium) is placed in a small cavity drilled in a
eBB

lead block. The lead block checks all the a, p and most of the y-rays, except those through the opening. The
radiations coming out of the cavity are subjected
urr

to an electric field provided by two plates as shown


ad

in Fig. 13.6(a). The a-rays are deflected through


YYo

smaller angles towards the negative plate. The


p-rays are deflected through larger angles
dd

towards the positive plate. The y-rays remain


Re
inn

undefiected. As opposite charges attract each other,


it was concluded that a-rays consist of a stream of
F

positively charged particles, whereas p-rays consist


of a stream of negatively charged particles. Since
y-rays were undeflected they could be waves or
uncharged particles. Further analysis failed to
detect any charge on y rays and showed that they
have no mass, y-rays, therefore, were thought to
be waves. Exactly same results were obtained when
these radiations were subjected to the action of a magnetic field, [Fig. 13.6(b)].
Some of the important properties of the three types of radiations (a, P and y-rays) are discussed
below :

a-Rays
1. An (X’particle carries double the positive charge ofproton, which is equal to the charge on the helium
nucleus.
NUCLEI
13/13

2. Mass of an a-particle is roughly four times that of hydrogen atom i.e. it is equal to the mass of the
helium nucleus.

The above two properties establish that an a~particle is equivalent to helium nucleus (or a helium
atom which has lost its two orbital electrons i.e. a doubly ionised helium atom).
3. The velocity of a-particles ranges between 1-4 x lO"^ ms
-1
to 2*1 X 10^ ms depending upon the
source emitting it.
4. Because of large mass, the penetrating power of a-particles is very small, it being 1/100 time that due
to p-rays and 1/10,000 time that due to y-rays. a-particles can be easily stopped by an aluminium foil
only 0-02 mm thick. Air of thickness 7*0 cm absorbs them completely.
5. Because of large mass and large velocity, a-particles have large ionising power. Each a-particle produces
about 2(XX)0 ion pairs before being absorbed.
6. The range of a-particIes in air (distance through which they can travel in air) depends upon the radioactive

w
source producing it. At normal pressure in air, the range of a-particles varies from 3 to
8 cm.

7. a-particles produce fluorescence in certain substances, like barium-plantinocyanide and zinc- sulphide.

Flo
8. a-particles affect photographic plate slightly,
9. a-particles are deflected by electric and magnetic fields.

ee
10. a-p^icles are scattered while passing through thin metal foils. Most of the a-paiticles are scattered at

Fr
small angles, but a few of them are scattered at an angle more than 90° also.
11. a-particles cause bums on human body.

for
ur
12. a-particles on being stopped, produce heating effect.
p-Rays
s
ok
Yo
1. A p-particle carries 1-6 x 10 C of negative charge,/.e. unit negative charge, which is the charge on
an electron.
Bo

2. The rest mass of P-particle is 9-1 x 10“^^ kg, which is the same as that of electron.
3. The velocity of p-particles ranges from 33% to 99% of the velocity of light.
re

The above properties establish that p particles are fast-moving electrons.


ou

4. Because of small mass, the penetrating power of p-particles is very large. They can easily pass through
ad

a few millimeter of aluminium. *


Y

5. The p-pamcles ionise the gas through which they pass, but their ionising power is only 1/lOOth that of
a-particles.
nd
Re

6. The range of p-particles in air is several metres.


7. The p-particles can also produce flourescence m
i certain substances like barium platinocyanide and
Fi

zinc sulphide.
8. They affect a photographic plate.
9. They are deflected by electric and magnetic fields, showing that they carry negative charge.
y-Rays

1. y-rays are not deflected by electric and magnetic fields, showing that they do not carry any charge.
2. The rest mass of a y-ray photon is zero.
3. y-rays travel with the speed of light.
4. y-rays are electromagnetic waves like X-rays. The wavelength of y-ray photon is smaller than that of
X-rays in electromagnetic spectrum.
5. y-rays have very large penetrating power. They can pass through several centimetre of iron and lead.
6. y-rays have got small ionising power.
7. y-rays can produce fluorescence in a substance like willimite.
13/14 P>uuU€^ A Fundamental Physics (XII) EEMD

8. y-rays affect a photographic plate more than |3-particles.


9. y-rays can knock out electrons from the surface of a metal, on which they may fall.
10. y-rays can cause nuclear reactions.
NOTE: Ionizing Power and Penetrating Power
As is known, an ion is an atom with an overall charge. Charge is positive when some electrons are
removed from a neutral atom and charge is negative, when some electrons are added to a neutral atom.
Alpha particles are moving slowly. They have time to interact with nearly all atoms in their path. The
positive charge on alpha particles attracts electrons from atoms in their path and ionise them, losing energy
quickly. Thus, a-particles have high ionizing power but poor penetrating power.
Beta particles are repelled by electrons in atoms. This repulsion causes p-parlicles to bounce between
the atoms. Collisions with atoms may cause ionization of some of the atoms. Therefore, p particles lose

w
energy slowly. Their penetrating power is higher than that of alpha, but ionizing power is smaller than that of
alpha particles,
y-rays have no charge. Therefore, their collisions occur only when a nucleus or electron is directly in

Flo
their path. This is much less likely to happen because of large empty space in an atom. Therefore, gamma rays
have very low ionizing power, and very high penetrating power.

ee
Fr
13.17. LAWS OF RADIOACTIVE DISINTEGRATION
Soddy and Rutherford, from their experimental study regarding the radioactive disintegration formulated

for
the following laws, known as laws of radioactive disintegration :
ur
1. Radioactivity is a spontaneous process which does not depend upon externalfactors like temperature,
pressum etc. i.e. a radioactive element is in a state of disintegration which depends upon law of chance. It
ks
means it is impossible to predict which particular atom of the radioactive element will disintegrate in a given
Yo
oo

lime interval.
2. During disintegration of an atom, either an a-particle or a ^-particle is emitted. Both of these
eB

particles are never emitted simultaneously. Also at a time, an atom will not emit more than one a-particle or
more than one ^-particle, y-rays emission follows the emission of a or ^ particle.
3. The emission of a-particle from an atom will change it, into a new atom whose charge number is
ur
ad

reduced by two and mass number is reduced by fou.r


Yo

4. The emission of a ^-particle from an atom will change it, into a new atom whose charge number is
raised by one, without any change in its mass numbe.r
d

5. The number of atoms disintegrated per second (Le. rate of disintegration of radioactive atoms) at
Re
in

any instant is directly proportional to the number of radioactive atoms actually present in the sample at
that instant. This is also known as radioactive decay law.
F

Thus greater the number of atoms present in a sample of an element, faster will be its decay and vice-versa.
Mathematical Form of Radioactive Decay Law
Let Nq = total number of atoms present originally in a sample at time t = 0.
(t = 0 refers to the time when the radioactive element is freshly separated from its by-products).
N ~ total number of atoms left undecayed in the sample at time t
dN-a small number of atoms that disintegrate in a small interval of time dt
Rate of disintegration of the element

dN
R =- (minus sign indicates that the number of atoms left undecayed decreases with time)
dt
dN
dN
N R = - ...(11)
According to radioactive decay Law,
OC

dt dt

where X, is a constant of proportionality and is called the disintegration constant or the decay constant.
NUCLEI
13/15

dN
Equation (11) can be written as = ~ \ dt
N

r dN
Integrating both sides, we get, N
= - X di or
log, N = -Xi + C ...(12)
where C is a constant of integration.
At
t = 0,N = NQ
From (12), log^, Nq=XxO+C
C = log, A'o
or
...(13)
Put in (12), log,yV=-X/ + log,A^O

ww
N
\og^N-lQg^NQ = -Xt or
log. = -Xt
N
0

N
N=Nr.e~'^‘

Flo
i.e.
0 .(14)
N.0

e
eree
Equation (14) shows that the radioactive decay is exponential
as shown in Fig. 13.7.

FFr
From Fig. 13.7, we find that
= 0 only at t = . This is also clear from eqn. (14).
uurr OO

orr
13.18. DISlISfTECRATION CONSTANT OR DECAY CONSTANT

Substituting / zz
1
sfo
N N
- in (14), we get, N = = N^e
-] _ ‘’0 0
= 0-368 = 36-8% ,A 0
kks
Yoo
0
e 2-718
oooo

Thus, the disintegration constant of a radioactive element is the reciprocal of itme at the end of
eBB

which, the number of atoms left undecayed in a radioactive sample reduces to — time or

36'8% the original number of atoms {Nq) in the sample.


urr
ad

Units of decay constant X are s"’ or min“’ or day' or year“'.


YYo

Significance of Decay Constant


The decay constant of a radioactive element determines the rate of decay of the radioactive element
dd

When X is small, the radioactive substance decays slowly. When value of X is large, the radioactive substance
Re
inn

will decay rapidly.


F

13.19. HALF LIFE OF RADIOACTIVE ELEMENT

Half life of a radioactive element is defined as the time during which half the number of atoms
present initially in the sample of the element decay or it is the time during which number of
atoms left undecayed in the sample is half the total number of atoms present initially in the
sample. It is represented by .T ir-i
/

Expression for Half Life


When t=T,N = Nq/2
N 1
0
From (14), = N.0 e-'^T or = i.e.2 = e^T
2 2

Taking log of both sides, we get,


X T log, e = log, 2 = 2-3026 log,Q 2
XTx I = 2-3026 X 0-3010 = 0-6931
13/16 “P^iteUcfi. 'a Fundamental Physics (Xll)ra*lgD

0-6931
r = ...(15)
X

Thus half life of a radioactive substance is inversely proportional to the decay constant of the
substance.

Significance of Half Life


Half Life of a radioactive substance measures the time for which the substance lasts for one half
of its total number of atoms. If decay constant of radioactive substance is large, its half life is small, and
vice-versa.
Table 13.2 below lists the half lives of some important radioactive substances. Note that half life of a

oww
radioactive substance is not affected by the change in temperature and change in pressure etc.
TABLE 13.2.

S. No. ELEMENT HALF LIFE

e
FFrlo
re
1. Uranium 92
U238 4-5 X 10^ yrs.
Ionium 9qIo"^^ 8-0 X lO'^ yrs.

ree
2.

F
3. Radium 88
Ra226 1620 yrs.

rF
222
4. Radon ggRn 3-82 days
210
5. Polonium a^Po 138-3 days

fsoor
ouur
6. Thorium
skf 90
Th234 24-1 days

Sample Problem Find the half life of if one gram of it emits 1*24 x 10^ a-particles per
ooko
second. Avogadro’s Number = 6-025 X 10^3.
Yo
Y

Sol. Here, atomic wt. of uranium = 238


Bo

23
reB

Avogadro’s number = 6 025 x 10


-dN
Rate of disintegration. = 1-24 X 10"^
uur

dt
oY

6-025x102'-^
ad

No. of atoms of in 1 gram N =


dY

238

~dN 6-025 xl0“3


According to radioactive decay law. = X/V 1-24x10^ =X.
innd

or
Re

dt 238
Fi
F

1-24x10“^ x238
X =
6025x10^^

0-6931 0-6931x6-025x102-^ 0-6931x6-025x10’^


sec = - years
As half life, T = 238x 1-24X 60x 60x 24x365
X 238x1-24x10^
= 4*259 X 10^® years

Retain in Memory
1. As discussed above, in / = T, N = Nq/2
x2
1 N.0 I
0 _
In another half life, (i.e. after 2 half lives), /y/ = — = .N
0
2 2 4 2J
\(N 0 _
N
0
( 1
N = - = .N0
After yet another half life, (i.e. after 3 half lives), 2 4 8
NUCLEI 13/17

and so on. Hence, after n half lives.


1 i V/r
/v = /v.0 = .N
0
2
V “ y
2
V /

where t = nxT = total lime of n half lives.

1 y/r
yj
N 1
or
N.0 9 0 ...(16)
V - y

2. Some radioactive elements like tritium and plutonium have very small half life.

ooww
So these elements have decayed long ago. Hence these elements are not found in nature. However,
they can be produced artificially by means of nuclear reactions.

3. Number of atoms in excited Slate, N^-


where is energy of atom in ground state, and is energy of atom in excited state.

e
ree
rFl
Sample Problem The half-life of Radoii is 3-8 days. Calculate how much of 15 milligram of

Fre
Radon w'ill remain after 38 days.

rrF
Sol. Here, T = 3-8 days, r = 38 days, N^ = 15 milligram, A/= ?
ouur
sffoo
38
No. of half lives in 38 days is n = = 10
3-8
okks
f \"
We know. N = .N 0
Yo
ooo
Y
BB

1 VO
/V=15 = 0-014 mg
rr e
ouu
YY
ad

13.20. AVERAGE LIFE OR MEAN LIFE OF RADIOACTIVE ELEMENT


dd

A radioactive element undergoes spontaneous disintegration. The process of disintegration follows the
laws of probability and continues till infinite time. There are atoms which disintegrate right in the
Re
iinn

beginning. Their life time or age is zero. There are other atoms which would disintegrate at the end. Their life
time or age is obviously infinite. The life time of atoms which disintegrate inbetween, ranges from zero to
F

infinity.

Average I(fe time of a radioactive element can, therefore, be obtained by calculating the total
life time of all the atoms of the element and dividing it by the total number of atoms present
intially in the sample of the element.

Let us consider a radioactive element containing Nq atoms to start with i.e. at / = 0. Let the number of
atoms left at time / be N. Suppose a small number of atoms = dN disintegrate further in a small time dt.
Therefore, the life time of each of these dN atoms lies between i and (r + dt). If dt is taken very small, then the
age of each of the dN atoms can be taken as t.
Total age of dN atoms = t. dN
N.0

Proceeding in this way, total life time of all the atoms in the sample of the element = t.dN
13/18 '4, Fundamental Physics

By definition, average life time of the radioactive element.


A'.0

t.dN
total life time of all the atoms
T = i.e.. T = ...(17)
total no. of atoms N
0

From (11), dN--\N dt

using (14), dN = -X {Nq e-'^ d(


0

-XN^e
0
^'dtxt 90

x = X t.e dt
Substituting in (17), we get, 1 = N.0
or

ww
Note that the limits of integration have been changed in terms of time i.e. from (14), when A/ = 0,
t = 00 and when N=Nq,i = 0.
Integrating by parts, we get

Floo
1 _ 1
00

ree
x = X dt e ^^dt - = 0-
= X 0+ -
-X~ X
-X -X -X
0
X i 0
0

rFee
0 0

F
...(18)

oor r
x = —
rur
X s ff
Hence, average life of a radioactive element is reciprocal of the decay constant of the element.
Relation between average life and Half Life
osk
YYoou
,K 0-6931
oook

From (15), —
T
eBB

T
= 1-44T ...(19)
Putting in (18), we get X -
uur r

0-6931
ad

i.e., the average life of a radioactive element is 1-44 times the half life of the element.
Yo

Probability of decay and Probability of Survival


Probability of decay or survival of a nucleus is relevant because the decay process is governed by
dY
Re

statistics.
idn

dN
FFin

dN
From = XN, = -Xdt
dt N

This is the probability of decay in small time dt. Over a large interval of time t, we know
N
= e-^‘ As N is number of atoms left undecayed.
N.
0

therefore, this is the probability of survival.


N Nr.-N
The probability of decay over this time interval is 1-
0
= l-e~^‘
N N.
0 0

Sample Prokslem A radioactive isotope X has a half life of 3 seconds. At t = 0, a given sample
of this isotope contains 8000 atoms. Calculate (i) its decay constant (ii) average life (iii) the time ,
when 1000 atoms of the isotope X remain in the sample (iv) number of decays/sec in the sample at
r = f,I sec.
NUCLEI 13/19

Sol. Here, 7’=3s,A^o = 8000,


0-6931 0-6931 1 1
(0 x = = 0*231 sec 1
(ii) x =~= = 4*33s m N = 1000, = ?
T 3 X 0-231

1 r
n
N 1000 1 '
As
n =3 , ri=n7’=3x3 = 9 sec.
N
0 8000 l2j
cIN
(/V) = XN =0-231 X 1000 = 231 s"'
dt
M=t^

13.21. ACTIVITY OF A RADIOACTIVE SUBSTANCE


The activity of a radioactive substance is defined as the total rate of disintegration of the substance.
It is represented by A.

ww
According to radioactive decay law,
N = Nn0

FF loo
dN
= -XNQe~^‘^~XN

ree
dt

dN
Activity, A = - = XN

rFee
dt ...(20)

oor rF
rur
If Aq is activity of the substance at r = 0, then Aq = XN^0
s ff
A _XN
k
A) ^^0
YYoou
okos

-Xt
BBoo

...(21)
r ee

Clearly, both A and N decrease exponentially with time.


0-6931 ,
As >.=
ouur
ad

, where T is half life of the substance.


T
Yo

, 0-693 lA^
from (0, A =
...(22)
T
Yd
Re
idn

1
Thus A °c — , i.e., greater the half life of the substance, smaller is its activity.
FFin

\n r, \t!T
Further. A _ N _(i 1

N^~{2
Following are the three units of radioactivity in common use :

(0 Curie (Ci). The activity of a radio active sample is said to be one curie, when 3-7 x
decays take place in every one second. Thus
1 Ci = 3‘7 X 10^^ decays/s.

This is the approximate activity of 1 gram of radium. In practice, smaller units are used :
I millicurie (1 m Ci) = 3-7 x 10^ decays/i' ;
I microcurie (1 p Ci) = 3-7 x 10“* decays/.?.
13/20 "pfUKUefr Fundamental Physics

(«■) Becquerel (Bq). It is the SI unit of activity. DO YOU KNOW ?


Thus 1 Bq- 1 decay/s 1 Ci = 3-7 x 10^^ Bq 1. The level of activity of a
sample of a radioactive substance
varies directly as the mass of the
(Hi) Rutherford (Kd). The activity of a radioactive sample substance. This level of activity
is said to be one rutherford, when 10^ decays take place in is measured using a device called
every one second. Thus, Geiger Muller counter (GM
counter).
1 Rd - 10^ decaysis, j 2. As A=Af^e
-Xf

Dimensional formula of all the three units of radioactivity — ^A^e-'^(-'k)=-XA


dt

ww
= -X(kbf)
Sample Problem A radioactive element decays to l/32th
of its initial activity in 25 days. Calculate its half life. = -X~N =
-(0-693)^ N

Floo
A 1 T2
Sol. Here, t - 25 days
32’ Instantaneous rate of change

ee
r= ? of activity varies inversely as the

eer
square of half life period of the

FFr
substance.
As n = 5
32

oorr
A) ^0
uur r
t 25
s ff
As 7 = - 7 = — = 5 days
n
sk
YYoo
ooko

13.22. ALPHA, BETA AND GAMMA DECAY


The radioactivenuclei are unstableand emit radiationsto achieve states of greater stability. The emission
eBB

of any of the a, |3, y radiations changes the original nucleus, called the, parent nucleus into a new nucleus,
called the daughter nucleus.’^ The daughter nucleus may be radioactive and disintegrate further to form a
uurr

still newer nucleus. The process continues till a stable nucleus of lead (Z = 82) is formed.
ad

Rutherford and Soddy suggested the following rules governing the radioactive decay.
Yo

1. The algebraic sum of charges (atomic numbers) before and after disintegration must be the same.
2. The sum of mass numbers before and after disintegration must also be the same.
dY
Re

Let us now see the three kinds of decay.


ind
FFin

13.23. ALPHA DECAY

It is the phenomenon of emission of an a particle from a radioactive nucleus.


When a nucleus emits an alpha particle, its mass FIGURE 13.8
number decreases by 4 and charge number decreases by 2. Parent Daughter Alpha
For example, when undergoes alpha decay, a Nucleus Nucleus Particle
new element with mass number (238 - 4) = 234 and
charge number (92 - 2) = 90 is formed. This corresponds o - o—►
to Thorium. Thus a new element is formed as a result of
Before Decay After Decay
alpha decay. The transformation is represented by the
equation.
92
f/238

*Often the daughter nucleus is more stable than the parent nucleus.
NUCLEI 13/21

Note that the alpha decay of 92 occurs spontaneously (without any external provocation). The total mass
of the decay products c^^Th-^ and is less than the massof original Thus the total mass energy of the
decay products is less than the mass energy of the original nuclide. The differencebetweenthe initial mass energy
and total mass energy of decay products is called disintegration energy (Q) of the process.
In general, alpha decay is represented as
Z-2 + 2He‘^ + Q ...(23)
where Q is the energy released in the decay. This can be calculated using Einstein mass energy equivalence
relation, £ = (A m).c-.
i.e.,
0 = (m;^-my-mHe) ...(24)
The energy released (Q) is shared by daughter nucleus Y and alpha particle.
We can show that kinetic energy of alpha particle is

ww
(A-4)Q
(K.E\
A

Flo
As the daughter nucleus is very heavy as compared to the a particle, therefore, most of the disintegration
energy (0 appears as K.E. of a particle.

ee
Explanation of alpha decay

rere
rFF
The basic question to be understood is how an a particle, which is bound inside the nucleus on account
of nuclear forces, comes out. The potential energy of such an a particle has two components :
uurr
(0 one component is due to attractive nuclear force-which dominates inside the nucleus making net
potential energy negative. foor
ks s
(«) Other component is due to coulomb repulsive force between the a particle and remaining positive
Yoo
charge of the nucleus.
oook

The nuclear component of potential energy of a particle sharply


eBB

drops as its separation from nucleus increases beyond the radius of


the nucleus. The coulomb component of potential energy dominates
at this stage. With increasing distance from the nucleus, this component
uurr

also decreases to negligible values, Fig. 13.9.


ad

From various alpha emitters, kinetic energy of a particles emitted


Yo

is of the order of 4-9 Me V. Studies reveal that nucleus of alpha emitters


dY

behaves as a potential barrier of height of the order of 25 MeV.


Classically, it is impossible to account for the emission of an alpha
Re
innd

particle of insufficient energy (=» 4-9 MeV), crossing such a high


potential barrier (» 25 MeV).
FFi

In the year 1928, Gamow, Gurney and Condon explained the alpha emission in terms of penetration of
the nuclear potential barrier on the basis of quantum theory. According to their theory :
(i) An alpha particle may exist as an entity within a heavy nucleus.
(ii) The a-particle is in constant motion inside the nucleus with a speed of the order of 10^ m/s.
(Hi) Quantum mechanically, there is a small but finite probability that the particle may pass through the
barrier even if its energy is insufficient to cross the barrier height.
As size of nucleus = 10“^^ m and speed of a-particle « 10^ m/s, the particle takes about 10“^^ second to
move across the nucleus. Thus an alpha particle presents itself 10^^ times in a second against the potential
bamer. As this frequency (v) is very large, probability F of its escaping the potential barrier is F=pxv where
p is probability of escape per collision. As F becomes sufficiently large, the alpha particle may penetrate
through the nuclear potential barrier resulting in alpha emission. This is called tunneling of the nucleus.
We can now visualise why every nuclide in a sample of atoms does not decay at once.
And the half lives for alpha decay of most of the alpha unstable nuclei are very long.
13/22 Fundamental Physics (X11)B&29D
13.24. BETA DECAY

Beta decay is the phenomenon of emission of an electron from a radioactive nucleus.

When a parent nucleus emits a |3-particlc {i.e. an electron), mass number remains same because mass of
electron is negligibly low. However, the loss of unit negative charge is equivalent to a gain of unit positive
charge. Therefore, atomic number is increased by one. For example, when emits a p-particle. the
daughter nucleus has mass number = (234 - 0) = 234 and charge number = (90 + 1) = 91. This is called
91 The reaction can be represented as
90
Tfj234 91 -1 ([3-particle) FIGURE 13.10
In general, we can write

ww
where Q is the energy released in (3-decay.
The graph between energy of |3-panicles and their num ° £
ber is shown in Fig. 13.10. From the graph, we find that n -c End Point

FF loo
E ™ Energy
i 0.
(/) Most of the P particles emitted carry small energies. Z (Si

ree
(//) Only very few p-particles carry maximum energy,
called end point energy.
0.2 0.4
(Hi) The energy spectrum of emitted p-particles is

reFe
continuous indicating that the p particles can carry all possible [3 Particle Energy (Qp MeV)

oroFr
r ur
energies from 0 to (max). s ff
Further studies reveal that energy of nucleus emitting P-particles decreases by an amount equal to the
end point energy- but most of the p particles emitted have energies smaller than the end point energy. Therefore,
k
YYouo
the principle of conservation of energy is violated in ^-decay.
koso

Again, the principle of conservation of angular momentum is also violated in ^-decay. This is because
BBoo

1
1 h
P particle or electron has a spin = — . Therefore, in P decay, spin of nucleus must change by ^ ^^ ■
r ee

2 271
V
But in actual practice, there is either no change or an integral change in the spin of nucleus emitting P particle.
ad
ouur

Neutrino hypothesis
Yo

The violations of the principles of conservation of energy and angular momentum were resolved by
Wolfpang Pauli in the year 1931. He postulated that ^-decay is always accompanied by another particle of
d
Re
idnY

zero rest mass and zero charge. This particle was called antineutrino (v ). Thus P-decay of a nucleus may
be represented as : + jgO + y + Q
FFin

■>
Z+1

The neutrino hypothesis implies that a low energy P-ptuticle is accompanied by a high energy antineutrino
and vice-versa. So the total energy Q of p particle and antineutrino is constant and always equal to end point
energy. Thus the law of conservation of energy holds good in p-decay.
I \
1
Further, antineutrino has a spin = — . If both, the p FIGURE 13.11
2 2tc
\ )
P" P"
particle and antineutrino spin in opposite direction, as shown in
Fig. 13.11 (a), the spin or angular momentum of daughter nucleus
will be equal to spin/angular momentum of parent
nucleus. However, when both, the P-particle and anti- neutrino Nucleus Nucleus
spin in the same direction, as shown in Fig. 13.11(b), there will O
be integral change in spin or angular momentum of the daughter © V O V
nucleus ; as observed experimentally.
Thus, with this hypotehesis, the law of conservation of angular momentum is also not violated in P-decay.
NUCLEI 13/23

Retain in Memory
1. A positron is similar to an electron in all respects, except that it has a charge + e, instead of - e.
The symbol is used for an electron and is used for a positron. Positron is said to be antiparticle
of electron.

2. An excited nuclide (which is proton rich) often returns to a stable nuclide by capturing an orbital
(K shell) electron, i.e. Z-1
yA
This process is called electron capture or K-capture. The vacancy caused in K shell is filled by
transition of electrons from outer orbits-resulting in the emission of characteristicX-rays.
Note that electron capture process is identified by the shell or energy level from which the captured
electron comes. If captured electron is from /(-shell, it is known as /(-shell capture. If captured
electron comes from L-shell, it is called £-shell capture and so on.
3. For beta-minus decay, a neutron transforms into a proton within the nucleus according

w
to: n p + e~ + V
Here, v represents antineulrino.

Flo
And for beta-plus decay, a proton transforms into a neutron within the nucleus according to
p -»« + «■*■ + V

reee
As mass of a neutron is greater than the combined mass of a proton and electron, therefore,
Q value of beta minus decay is positive. The P minus decay process is energetically allowed

FFr
and the decay may take place in nature spontaneously.
On the other hand, Q value of beta plus decay is negative. Therefore, the process of P plus
urr
for
decay is not energetically allowed. It does not take place in nature.
Infact, a free proton is stable. Its life time is greater than the life of our universe. A free proton
cannot change into a neutron but a free neutron can change into a proton.
kkss
However, inside the nucleus, a proton can change into a neutron and a neutron can change
Yo
ooo

into a proton. This is because inside the nucleus, both protons and neutrons are bound. The
bound nucleons behave differently than free nucleons. When we take into account the energies of
eB

parent and daughter nuclei involved in beta plus decay (or positron decay), the decay is always
energetically allowed. For example, \ \Na-^ +e* + v
Infact, lighter radioactive isotopes with Z<10 decay mostly by beta plus decay.
r
ou
ad
YY

Sample Problem After a Series of alpha and beta decays, 94Pu^^^ becomes How
many alpha and beta particles are emitted in the complete decay process ?
nndd
Re

Sol. Here, parent nucleus is 94Pu^^^ and daughter nucleus is


Decrease in mass number = 239 - 207 = 32
Fi

Decrease in charge number = 94 - 82 = 12.


Let number of a particles emitted be x and number of p panicles emitted be y.
Now, one alpha decay causes decrease in mass number by 4 and decrease in charge number by 2. Again
one beta decay causes increase in charge number by 1 and no change in mass number.
32
Therefore, jcx4 = 32, x=—=8 and ->’X 1 + xx2 = 12
4

>’ = 2x- 12 = 2x8 12 = 4

13.25. GAMMA DECAY

Gamma decay is the phenomenon of emission ofgamma ray photonfrom a radioactivenucleus.


This occurs when an excited nucleus makes a transition to a state of lower energy. As nuclear states have
energies of the order of MeV, therefore, the photons emitted by nuclei have very large energies
13/24 ^futdce^'^ Fundamental Physics (XII)CZsMD
(= McV) and hence much smaller wavelength (< 0-01 A°). Such short wavelength electromagnetic waves
emitted by excited nuclei are called y-rays.
Since photons do not have any charge or rest mass, therefore in y-decay, daughter nucleus has the same
charge number and same mass number as those of parent nucleus. The y-decay can be represented as
z + y
After an a decay or a |3 decay, the daughter nucleus is
usually in an excited state and it achieves stability by the
emission of one or more gamma ray photons.
For example, (i) the p-decay of 21^°^ transforms it into
an excited 28^'^ nucleus. This reaches the ground state by
emission of y-rays of energy 117 Me V and 1 -33 MeV. This is
shown in Fig. 13.12 by an energy level diagram.

ww
The nuclear reactions may be represented as
{)
e'‘ -I- V
27 28 -I

28 (=M7MeV)

Flo
28
.●60*
28 M + (= 1-33 MeV)

ee
By studying the energies of gamma rays emitted, we 20
9F

rere
can obtain energy levels of the nucleus.

rFF
(//) decays by emitting p“ particle to give a
daughter nucleus This daughter nucleus is in 20
uurr
10Ne
excited state. It comes to ground state by emitting a y-ray
photon of energy 1-63 MeV. This is shown in Fig. 13.13
by energy level diagram.
foor 1oNe
20
ks s
Yoo
Explanation of gamma decay
oook

It is known that in an atom, electrons revolve around the nucleus in circular orbits, and possess definite
energies. These are called atomic energy levels. Transitions between these energy levels account for emission
eBB

of visible radiations, U.V. radiations and I.R. radiations etc. Studies reveal that similar energy states exist
inside the atomic nuclei. These are called nuclear energy levels. Transition of a nucleus from a higher energy
uurr

state to a lower energy state results in the emission of energy, which lie in the gamma ray region. This
ad

accounts for gamma decay.


Yo

13.26. NUCLEAR REACTION


dY

A nuclear reaction represents the transformation of one stable nucleus into another nucleus by
Re
innd

bombarding the former with suitable high energy particles.


FFi

Symbolically, we can represent a nuclear reaction as


V‘●/t+3
■¥ > z+2*^^ ^
+ ,h' + q
Here, X is the target nucleus, bombarded by an a particle (called the projectile). C is the compound
nucleus which is unstable. It disintegrates to give a product nucleus Y and a product particle (proton).
The reaction is named (a, p) reaction after the name of the projectile (a)and product particle ip). In the
nuclear reaction, Q is the total energy change in the reaction. It is called nuclear reaction energy or Q
value of the reaction.

In all types of nuclear reactions, the following conservation laws tu'e obeyed : 1. conservation of licicar
momentum, 2. conservation of total energy. 3. conservation of charge, 4. conservation of number of nucleons.
Rutherford was the first to perform an experiment in 1919 on artificial transmutation of elements. He
discovered the nuclear reaction :

gO>’+
18*
7/V'^ +
It is (a, p) reaction.
NUCLEI 13/25

Some other examples of nuclear reactions are :


(i) ^lP +1//' > (p, a) reaction
00 5S“ + l^'
12*
o'*’ (p, n) reaction
On) 6C‘^+i^f‘
13*
^■jN 7 />/i3 ^ y (p, y) reaction
OV) 7 (n, p) reaction
(V) i//2 + y_ 1 0 (photo disintegration)
Note that Q value of nuclear reaction is calculated from Einstein’s mass energy equivalence relation,
E= (A m) C-. It may be positive or negative.
2-value of reaction = K.E. of products - K.E. of reactants

oww
or Q-v^tlue of reaction = (mass of products - mass of reactants) (P
A nuclear reaction in which Q value is positive, energy is released. Such a reaction is called exoergic or
exothermic. A nuclear reaction in which Q value is negative, energy has to be supplied before the reaction
occurs. Such a reaction is called endoergic or endothermic.

e
It may be noted that exothermic nuclear reaction takes place spontaneously. Thus, radioactive decay

re
FFrlo
process is always exothermic.

rF
ee
13.27. NUCLEAR ENERGY : ENERGY FROM THE NUCLEUS

To have an insight into the process of energy generation from the nucleus, let us examine carefully,

rF
ouru
the curve of binding energy per nucleon, shown in Fig. 13.10. From A = 30 to A = 170, this curve has a
long flat region, indicating that B.E./nucleon in this region is almost constant. For A < 30 and A > 170, the

fosor
average B.E./nucleon is comparatively low. It means that in the mid mass region 30 < A < 170, the nuclei
skf
are more tightly bound in comparison to the nuclei with A < 30, and A > 170. Therefore, transmutation of
ooko
less stable nuclei into more tightly bound nuclei would provide an excellent possibility of releasing nuclear energy.
Yo
The nuclear reactions involving nuclei with A > 170 constitutenuclearifssion. And the nuclear reactions
Y
Bo

involving nuclei with A < 30 constitute nuclear fusion. Thus two distinct ways of obtaining energy from
reeB

nucleus are (/) nuclear fission and (ii) nuclear fusion. These processes are discussed below in some detail.

13.28. NUCLEAR FISSION


ooY
uur

Nuclear ifssion is the phenomenon of splitting of a heavy nucleus (usually A > 230) into two or
ad

more lighter nuclei.


dY

In the process, certain mass disappears i.e. sum of the masses of final products is found to be slightly
nind

less than the sum of the masses of the reactant components. This difference in masses is called mass defect
Re

(Am). Therefore, as per mass energy relation given by Einstein, energy released in nuclear fission is
F
Fi

E = (Am) c~
In 1938, Hahn and Strassman discovered that when is bombarded with thermal neutrons, it
splits up into and ,26^^^ emission of 3 neutrons alongwith 200 MeV of energy per fission.
The neutrons produced after fission are called secondary neutrons. The reaction is represented as

92
^235 ^ 0« 56

Here, mass of 92
f/235 235-0439 amu
mass of 0n/i' 1-0087 amu

Total mass of reactants 236-0526 amu

Again, mass of 140-9139 amu

mass of 91-8973 amu


mass of 3 0n/j' 3-0261 amu

Total mass of products 235-8373 amu


13/26 'P’MuUe^ 4 Fundamental Physics (XII)
Mass defect (A m) = Total mass of reactants - Total mass of products
= 236-0526 - 235-8373 = 0-2153 amu
As 1 amu = 931 MeV

/. Energy released per fission of 926^^'^^


Q={Am)x93i MeV = 0-2153 X 931 MeV
= 200-4 MeV

Thus about 200 MeV of energy is released in the fission


of a single nucleus. This energy appears in the fonu of
y-rays, kinetic energy of fission fragments and the released
neutrons. Further, such a huge amount of energy is released in
a very short time *= 10"^^ second.

ww
It is relevant to mention here that the fission fragments
of not always Ba and Kr. Other nuclides have also
been obtained. But the energy evolved is almost the same in

Flo
all the cases.

e
Fig. 13.14 represents the distribution by mass number, of the fragments that are found when many
fission events of 926^"^'^ iwe examined. The most probable mass numbers occuring in about 7% of the events

ere
are centred around A * 90 and A « 140. Note that in Fig. 13.14. vertical scale is logarithmic.

FFr
Note that the fragment nuclei produced in fission are highly unstable. They are highly radioactive and
uurr
emit beta particles in succession until each reaches a stable end product.

orr
Also, the disintegration energy in fission events first appears as the kinetic energy of fragments and
sfo
neutrons. Eventually, it is transferred to the surrounding matter in the form of heat. About 5-6% of energy is
associated with neutrinos emitted during [3-decay. It is lost to the system.
kks
Yoo
oooo

13.29. NUCLEAR CHAIN REACTION


A nuclear chain reaction is said to occur when neutrons emitted from the decay of one nucleus are
eBB

free to initiate fission in the surrounding nuclei.


The fission of 926^^^ by thermal neutrons is represented as
urr

i/235 + n ■>
ad

92 0 56
YYo

Now, the three secondary neutrons produced in the reaction may bring about the fission of three more
92 ^235 f^yciei and produce 9 neutrons, which in turn, can bring about the fission of nine nuclei and so
dd

on. Thus a continuous reaction called nuclear chain reaction would start and a huge amount of energy will
Re
inn

be released in a short time {i.e. a few micro seconds). This chain reaction is shown in Fig. 13.15.
Here, FP represents the fission product. If we can
F

have a control over this reaction, the energy


released can be utilised for peaceful purposes. This
led to the construction of Nuclear Reactor or
Nuclear Pile. However, if the reaction cannot be
controlled, the energy released will bring about
disastrous effects. This led to the discovery of an
atom bomb.

The following points should be considered for


a self propagating nuclear chain reaction.
1. Leakage of neutrons from the system.
Some of the secondary neutrons produced may
escape out of the system and will not take part in
further fission. This leakage may be reduced by
designing the system appropriately.
NUCLEI 13/27

2. Absorption of neutrons by impurities. The secondary neutrons


DO YOU KNOW ?
may be absorbed by impurities which are not fissionable. This loss may
be reduced by having a fissionable material free from impurities. For nuclear power plants,
3. Absorption of neutrons by Uranium-238. The natural uranium should be 1-4% and for nuclear

consists of three isotopes ; viz U~^^, iP-^^ and having relative bombs, must be closer to
abundance of 0-006%, 0-714% and 99-28% respectively. It is found that 97%. The process of increasing
is fissionable with fa.st neutrons {i.e. neutrons with energy more the percentage of in a
than 1 MeV), whereas is fissionable with slow neutrons {i.e., sample of uranium is called
neutrons with energy of the order of 0-025 eV). Enrichment of Uranium.
In natural uranium, as the percentage of is much less than that of
lP^^, there is more possibility of collision of neutrons with iP^^. It is found
that the neutrons get slowed on colliding with iP^^. As a result of it, further fission of iP'^^ is not possible. That is

w
why the chain reaction in natural uranium cannot occu.r
To sustain nuclear chain reaction, the percentage of isotope in the uranium ore is increased from
0-7% to 3%. 92 content of increased percentage, i.e., 3% is known as Enriched Uranium.

Flo
4. Critical size. In order to have a sustained chain reaction in a sample of it is required that the

e
number of neutrons lost due to leakage and absorption should be much smaller than the number of neutrons

reee
produced in a fission process. This is possible if the uranium block has a size greater than a certain critical

FFr
value. It is called critical size.

The chain reaction once started will remain steady, accelerate or retard depending upon, what is called
the neutron reproduction factor or multiplication factor. This is the ratio of rate of production of neu

for
ur
trons to the rate of loss of neutrons due to leakage and absorption i.e.
rate of production of neutrons
kkss
Reproduction factor, £ =
rate of loss of neutrons
Yo
oo

If A! = 1. the chain reaction will be steady or sustained. The size of the fissionable material used is said
to be the criticial size and its mass, the critical mass.
eB

This is the stage required for steady power generation, when criticality is said to have been reached.
If A" > 1, the chain reaction accelerates resulting in an explosion. The stage is said to be Super critical.
This is how an atom bomb works.
ur
ad

If AT < 1, the chain reaction gradually comes to a halt. The stage is said to be subcriticai.
YYo

Sample Problem The cuei^y released in a nuclear fission process is 4*5 x 10^^ J, wherein
total mass involved is one gram. Calculate the percentage of mass defect in the process.
d

Sol. Here. £ = 4-5 x 10^^ J ; m = 1 gram


Re
in

Let the mass defect be A/«


F

£ = (A/n)

. E 4-5x10^’
Am = — = 0-5x10"^ kg
(3 X 10^)2
Am 0-5x10"^
xl00 = xl00% =0-5%
m 10'^

13.30. THERMAL NEUTRONS


Thermal neutrons are the low energy neutrons or slow moving neutrons. Their energy = 1/40 eV. The
velocity of thermal neutrons » 2-2 km/s, which corresponds to the velocities of random motion of atoms and
molecules in a gas at room temperature. That is why these neutrons are called thermal neutrons.
To obtain thermal neutrons so that chain reaction is sustained, secondary fast neutrons are made to pass
through substances like paraffin, deuterium or heavy water, which are rich in hydrogen. On colliding against
the hydrogen nuclei or protons of roughly equal mass, their velocities get interchanged with protons. The
13/28 Fundamental Physics (XII)CEOtD

neutrons are thus slowed down, and are therefore in thermal equilibrium with the material. These are thermal
neutrons.

The materials used for slowing down fast neutrons are called moderators.

13.31. NUCLEAR REACTOR

It is a powerful device, wherein the nuclear energy produced is utilised for constructive purposes.
A nuclear reactor is based upon controlled nuclear chain reaction.
Construction. The main components of nuclear reactor are shown in Fig. 13.16.

FIGURE 13.16

Control Rods Coolant Superheated Steam

ww
Shielding

Eiectric
Generator

Flo
Steam
Turbine

e e
Heat

reer
Exchanger

rFF
Water
Used
uur r
Steam

Fission
Chamber
Fuei
Rods
ffoor
Condenser
z:
sks
YYoo
Moderator

'n
Water
ooko

Pump
Cold Water
eBB

(1) Nuclear Fuel. It is a fissionable material to be used for the fission process to take place. Commonly
used fuels in a nuclear reactorare etc. Generally, uranium oxide pellets are inserted end to
uurr

end into long hollow metal tubes constituting the fuel rods. When slow neutrons interact with the fuel, the
ad

fission starts and the energy is released.


Yo

(2) Moderator. Its function is to slow down the fast moving secondary neutrons produced during the
dY

fission. The material of moderator should be light and it should not absorb neutrons. Usually, heavy water,
Re

graphite, deuterium and paraffin etc. can act as moderators. These moderators are rich in protons.
innd

When fast moving neutrons collide head on with the protons of moderator substances, their energies are
FFi

interchanged and thus the neutrons are slowed down. Such neutrons are called thermal neutrons which cause
fission of iP^^ in the fuel.
(3) Control rods. They have the ability to capture the slow neutrons. To control the chain reaction from
becoming violent, rods of boron or cadmium (called control rods) are inserted in the holes of reactor core, up
to a desirable length. As a result of it, the desired number of neutrons are absorbed and only limited number
of neutrons are left to produce fission. These rods can be adjusted from outside the reactor, in order to control
the chain reaction.

In addition to control rods, reactors are provided with safety rods, which when required, can be inserted
into the reactor. They reduce the neutron reproductionfactor (K) to less than unity.
(4) Coolant. A substance which is used to remove the heat produced and transfer it from the core of the
nuclear reactor to the surroundings is called coolant. At ordinary temperatures, water and heavy water serve
as coolants but at high temperatures, generally liquid sodium is used as a coolant.
The coolant takes up the heat energy produced in nuclear fission and passes on this energy to water in
a heat exchange.r As a result of it, superheated steam is produced which drives a turbine coupled with an
electric generator, [Fig. 13.16.]
NUCLEI 13/29

(5) Shielding. The whole reactor is protected with concrete walls, 2 to 2.5 metre thick, so that radiations
emitted during nuclear reactions may not produce harmful effects on the persons working on the reactor.
Working. Fig. 13.17 outlines the working of nuclear reactor. Slow neutrons cause the fission of
nuclei. FP stands for fission product. To start the nuclear reaction, the cadmium rods are slowly removed and
to stop it, they are inserted. This brings out a controlled nuclear chain reaction and hence the energy produced
can be used for constructive purposes.

ww
Flo
e
ere
FFr
uurr
orr
sfo
The nuclear reactors used for Research purposes in India are Apsara, Zerlina, CIR (Canada India
kks
Yoo
Reactor), Dhruva and Purnima.
oooo

Uses, (i) Nuclear reactors are used to produce radioactive isotopes which in turn are used in medicine,
eBB

industry and agriculture,


(ii) They are used in electric power generation.
Four nuclear power plants in India are :
urr

(a) Tarapur Atomic Power Station in Maharashtra


ad
YYo

(b) Rajasthan Atomic Power Station near Kota


(c) Narora Atomic Power Station in U.P.
dd

(d) Kalpakkam Atomic Power Station in Tamil Nadu.


Re
inn

(Hi) They can be used for the propulsion of ships, submarines and air crafts.
F

(iv) They are used to produce neutron beam of high intensity whicli is used in the treatment of cancer
and nuclear research.

Fast Breeder Reactors. They use thorium or natural uranium as fuel elements. When fast neutrons
strike uranium fuel in a reactor, (which is present along with 92^^^^ in natural uranium) absorbs a
neutron and becomes This is radioactive and undergoes P-decay twice to produce as
-P -P
(7238^ „1
-a
V r/239 239 239
represented here : 92 > 92^ > 93/^P > 94^^ » 92 f/235
Now is fissionable and undergoes alpha decay (with T = 24000 years) to produce 92^/“^^- Thus
using an unfissionable material produce fissionable material 94Pw239^ which can be fissioned
even by fast neutrons. Therefore, no moderator is used in such reactors.
13.32. NUCLEAR FUSION

Nuclear fusion is the phenomenon offusing two or more lighter nuclei to form a heavier more
stable nucleus.
13/30 'Pmtdee^'A Fundamental Physics (XII)CZ&ISD
The mass of the product nucleus is slightly less than the sum of the masses of the lighter nuclei fusing
together. This difference in masses (A m) results in the release of tremendous amount of energy, in accordance
with Einstein’s mass energy relation £ = {A m) <?■.
Some of the examples of nuclear fusion are :
1 //I + j//' ^ i//2 + + V + 042 MeV ...(0
^ + 3_27 MeV ...(//)
1 + ,//“ ^ + j//' + 4-03 MeV ...(/«)
e-g-. 4 ,//> ^ +2 + 2 V + 26-7 MeV
In reaction (/), two protons combine to form a deutron and a positron with release of 0 42 MeV energy.
In reaction {ii), two deutrons combine to form a light isotope of helium and a neutron with release of 3-27
MeV energy. In reaction (iii). two deutrons combine to form a triton and a proton with release of 4-03 MeV
energy. In reaction (/v), four hydrogen nuclei combine to form a helium nucleus, two positrons and two

ww
neutrinos with release of 26-7 MeV energy.
In all these reactions, we observe that two positively charged particles combine to form a heavier nucleus.
Coulomb repulsion between these charges prohibits them to come close enough to be within the range of their

Flo
attractive nuclear forces and fuse. Tlte height of the Coulomb barrier depends on the charges and the radii of
the two interactingnuclei.

ee
The essential condition for carrying out nuclear fusion is to raise the temperature of the material so that

rere
particles have enough energy due to their thermal motions alone and they can penetrate the Coulomb barrier.This

r FF
process is called Thermonuclear fusion.
Thus for thermonuclear fusion to occur, extreme conditions of temperature and pressure are required.
uurr
foor
Further, higher density is also desirable so that collisions between light nuclei occur quite frequently. These
conditions are met only in the interior of stars, and cannot be arranged in a laboratory. The energy generation
ks s
in sun and stars takes place via thermonuclear fusion.
Yoo
ooook

13.33. DISTINCTION BETWEEN NUCLEAR FISSION AND NUCLEAR FUSION


eBB

As detailed above, we find that


(/) Both, nuclear fission and nucler fusion are the sources of tremendous energy.
(ii) In both the processes, a certain mass (A m) disappears, which appears in the form of energy as per
uurr

Einstein equation : £ = (A m)
ad
Yo

(iii) In fission, a heavy nucleus splits into two or more lighter nuclei. In fusion, two or more lighter nuclei
fuse together to form a heavier nucleus.
dY

(iv) For carrying out fission, a suitable bullet or projectile like neutron is needed. For carrying out fusion,
Re

the lighter nuclei have to be brought very close to eachother against electrostatic repulsion. For this,
innd

suitable energy is to be made available, often by raising the temperature to the order of 10^ K. This
FFi

justifies nuclear fusion being called thermonuclear fusion. In actual practice, such high temperatures
are generated by nuclear fission. That is why usually, nuclear fission precedes nuclear fusion.
(v) In a nuclear fusion reaction, energy liberated per unit mass of their nuclei is many times larger than the
energy liberated per unit mass in nuclear fission reaction. Therefore, for a given weight, hydrogen
bomb (based on nuclear fusion) is far more dangerous than an atom bomb (based on nuclear fission),
(vi) The products of nuclear fission reaction are radioactive. They produce environmental pollution and
hence require very careful disposal. However, the products of nuclear fusion are not radioactive. They
are harmless and can be disposed off easily.
(vii) While producing nuclear energy from fission, we have learnt how to control nuclear chain reaction. But
we have yet to learn controlling the thermo nuclear fusion reactions. This would be the basis offusion
reactor, which is seen as the future source of unlimited energy without pollution.
Sample Problem The mass defect in a nuclear fusion reaction is 0*3%. What amount of
energy will be liberated in one kg fusion reaction.
NUCLEI 13/31

0-3
Sol. Mass defect, A m = 0-3% of 1 kg =
100
XI kg = 3x10-3 kg
Energy liberated = (A m) -3 x (3 x 10^)^ = 2*7 x 10^** joule
1334 CONTROLLED THERMONUCLEAR FUSION

Controlled thermonuclearfusion is the basis offusion reactor, which is thefuturesource ofunlimited


and unpolluted energy.
The most attractive reactions for terrestrial use are : ^ + 3-27 MeV

1 + ff- + |//i + 4-03 MeV


,//- + ,//3 ^ -,He^ + o«‘ + 17-59 MeV
Deuterium, the source of deutrons for these reactions is available in unlimited quantity in sea water
Other requirements for a fusion reactor are :

w
(/) High particle density. The deutron-deulron collision rate can be high only when the density (number)
of interacting particles is very large. At high temps required, deuterium will be completely ionised, forming
neutral plasma.

Flo
(ii)A high plasma temperature of the order of 10^ K is required so that interacting particles can penetrate

e
the Coulomb barrier and fuse together.

rree
{Hi) A long conifnement time. The hot plasma must be maintained at a sufficiently high density and

r FF
temperature for a sufficiently long time so that fusion of enough fuel occurs. No solid container can withstand
these conditions. Therefore, clever confining techniques, such as magnetic confinement and inertial confinement
uurr
are being explored.
for
Efforts are being made all the world over to achieve controlled thermonuclear fusion in the laboratory.
kss
1335 NUCLEAR HOLOCAUST
ooook
Yo

Nuclear holocaust is the name given to large scale destruction and devastationthat would be
eB

caused by the use of nuclear weapons.

The devastating damage caused by nuclear weapons can be classified as :


urr

(0 Immediate Effects (») Long term Effects


ad
Yo

Immediate Effects include thermal flash causing spontaneous ignition and setting fires ; shock waves
which are high pressure waves moving out at speeds greater than 3000 km h”* ; Electromagnetic pulse
dY

capable of destroying power distribution systems, telecommunicat ions and computer networks and Initial
Re
innd

Nuclear Radiation which may cause instant death of people and animals.
Long Term Effects include Radioactive Fall out especially from small fission bombs and nuclear
Fi

winter that may follow large scale detonations. Sunlight might be blocked from the earth’s surface resulting
in lower temperatures and destruction of plant life. The protective ozone layer could also be damaged.
1336 INDIA'S ATOMIC ENERGY PROGRAMME

The Atomic Energy Programme of our country was launched around 1950 under the leadership of
Homi J. Bhabha. The major milestones achieved so far are :
(0 First nuclear reactor named Apsara went critical on August 4, 1956. It used enriched uranium as fuel
and water as moderator.

(«) Another reactor named Canada India Reactor (CIRUS) became operative in I960. It used natural
uranium as fuel and heavy water as moderator.
(Hi) Indigenous design and construction of plutonium plant at Trombay. It ushered in the technology of fuel
reprocessing.
(tV) Research reactors like Zerlina, Purnima, Dhruva and Kamini were commissioned. The last one uses
U-233 as fuel.
13/32 ^X4xdee^ 4- Fundamental Physics (XII) LVWIWII
(v) The fast breeder reactors which use Plutonium-239 as fuel do not need moderators. They can be used
to produce fissile Uranium-233 from Thorium-232 and to build power reactors based on them.
Considerable work has been done by our scientists in this direction,
(v/) We have mastered the complex technologies of mineral exploration, mining, fuel fabrication, heavy
water production, fuel reprocessing etc.
Elaborate safety measures, both in the design and operation of reactors and stringest standards of
radiological protection are most important hall marks of Indian Atomic Energy Programme.

1
TYPE I. ATOMIC NUCLEUS, 1 joule = eV .
ITS SIZE AND DENSITY l-602xl0-'^

w
Formulae used. 1 joule = 6-242 x eV
(/'O From E=

Flo
3m
1/3
l.R^RnA
0 2. P =
47CR3 E 931x1-602x10-^^

reeee
—7 .-. 1 a.m.u. =
3.1 a.m.u. = (1/12)massofC-12atom= l-66x 10“^^kg C"
(3 X 10^)2
4. E= me? S. 1 MeV =1-6x10-*^ joule

FFr
6. 1 a.m.u. = 931 MeV
= 1-66 X 10-2'^ kg
Units used. R is in metre : p in kg/m^ ; A m in a.m.u. Now, by definition,

for
Mass of = 12 a.m.u.
ur
(or kg); B.E. in joule or MeV.
Standard Values. Rq = 1-2 x lO"*^ metre, average = 12x 1-657 X 10-2'' kg
kkss
mass of nucleon, m = 1-66 x 10“^^ kg. = 1-99 X 10-26 |(g
Yo
Exsmple Given the mass of iron nucleus
oo

Example Q Jhe natural boron is


as 55-85 u and A - 56. Find the nuclear density.
eB

composed of two isotopes 5^^^ and 5^^* having (NCERT Solved Example)
masses 10-003 u and 11-009 h resp. Find the
Solution. Here, mass m = 55-85 u
relative abundance of each isotope in the natural
r

= 55-85 X 1-67 X IQ-^'^kg


ou

boron if atomic mass of natural boron is 10*81 u.


ad

4 ^
Solution. Suppose natural boron contains x% Volume V = ~kR^ =
YY

3
of and (100 -x)% of isotope.
4 ,
ndd

As atomic mass of natural boron = weighted -7C/?gxA


Re

average of masses of two isotopes.


3 m
Fi

xXlO-0O3 + (IOO-A-)ll-OO9 Nuclear density, P = 77


10-81 =
100 V 4nR^A
1081 =-0-996 .r+ 1100-9; 3x55-85x1-67x10-2”^
A = 19-98. 22
10
4x —(1.2x10-1^)3x56
Hence, relative abundance of = 19-98%, 7

and relative abundance of jp


II = 2-29 X lOl”^ kg/m-3
= 100- 19-98 = 80-02%
Example Express one Joule in eV.
Taking 1 amii = 931 MeV, calculate the mass of
Example Expre.ss 1 joule in eV. Taking C- 12 atom. (HP Board 2003)
1 a.m.u. = 931 MeV, calculate the mass of gC^2_ Solution. As I eV = 1 -6 X 10-19 J
Solution. We know
1
1 eV= 1-602 x 10-19 joule 1 J = eV = 6-25 X 10'* eV
1-6x10-19
1-602 X 10-19 joule = i cV
NUCLEI 13/33

Mass of C- 12 atom= 12 amu = 12 x 931 MeV


Example B Calculate the energy
= 12x931 X 1-6 X 10^*^ J equivalent of 1 g of substance.
_12x931xl-6xlQ-‘^ kg
(NCERT Solved Example)
(3x10^)2 Solution. Here, m= \ g = ICH kg, £ = ?
m = 1-986 X 10"^^ kg From E = me- = 10"-^ (3 x 10^)“ = 9 x J

TYPE II. MASS DEFECT AND


Example 0 Find the energy equivalent of
NUCLEAR BINDING ENERGY one atomic mass unit, tirst in joule and then in
MeV. Using this, express the mass defect of
Formulae used.
in MeV/c^.
1. Mass defect. Am = [Z m + (A - Z) m - m J
Given = 1-00727amu,//zn = 1-00866 amu,
2. Total Binding Energy = (Am) c-
= 15-99053 amu

llowow
m
oxy
(Am)c^ Take 1 amu = 933-75 MeV/c^
3. Average B.E./nudeon =
A
(NCERT Solved Example)
M -A
4. Packing fraction =
A
Solution. We know, 1 amu = 1-66 x 10“^^ kg
From E=mc- = (1-66 x (3 x 10^)-

ee
Units used. When Am is in kg., binding energy is
in joule. When A m is in a.m.u., B.E. is in MeV, = 1-494 xl0-‘^J

Fr
where 1 amu = 931 MeV. r FF
Standard Value. c = 3x 10^ ms -1 1494x10"’^
E = MeV = 933-75 MeV
1-6x10"'3

er
Example @ Calculate the binding energy forfFor
u
For oxygen mass defect
per nucleon of 26^®^^ nucleus. Given that mass of
s
= S nip+ S m„ - Afftiy
ok
= 55-934939 u, mass of proton = 1-007825 u
YYour o
26
= 8 X 1 00727 + 8 X 1-00866 - 15-99053
and mass of neutron 1-008665 u and
= 0-13691 amu = 0-13691 x 933-75 MeV/c^
so

lu = 931 MeV. [CBSE 2005 (C), 2004]


= 127-8 MeV/c^
eeBoBk

Solution. In protons 26 ;
no. of neutrons = 56 - 26 = 30 Example Calculate binding energy per
r

Mass defect = 26 m^ + 30 m -MFe nucleon of Given that


ouru

n
ad

= 26 X 1-007825 + 30 x 1-008665 ~ 55-934939 :209


m (jjjBi ) = 208-980388 amu
Yo

= 26-20345 + 30-25995 - 55-934939


m (neutron) = 1-008665 amu
= 0-528461 u.
m (proton) = 1-007825 amu
d
Re

0-528461x931
in

B.£./nucleon = = 8-79 MeV/N (CBSE (OD), 2000)


56
FFind Y

Solution. In 83 number of protons = 83


Example Calculate the B.E7nucieon of
number of neutrons = 209 - 83 = 126
17 Cl^^ nucleus. Given that mass of proton
Mass defect,
= 1-007825 «, mass of neutron = 1-008665 m, mass
of lyCl^^ = 34-980000 m ; 1 u = 931 MeV. A m = 83 X nip + 126 x m„ - M (Bi)
(CBSE 2002) = 83 X 1-007825 + 126 x 1 008665

Solution. In ]7CP^, no. of protons = 17 ; - 208-980388


no. of neutrons = 35 - 17 = 18. = 83-649475 x 127-091790 - 208-980388
Mass defect = 17 m^ -f 18 - Mq = 1-760877 amu
= 17x 1-007825 + 18x 1-008665-34-980000
= 17 133025 + 18-155970 - 34-980000 1-760877x931
R£./nucIeon =
= 0-308995 u 209
0-308995x931
B.£/nucleon = = 8-22 MeV/N = 7-85 Mev/N
35

i
13/34 ^'uidee^'A Fundamental Physics (XII)EEIHIl
il
TYPE III. RADIOACTIVE DECAY LAW, Solution. Here,
N _ 6-25 _ 0-25
HALF LIFE AND AVERAGE LIFE
/Vq " Tm “ ^ ’
dN
r = 16 days. X = ?
Formulae used. 1. = -XN 2.N = N^e~^’
0 N (1 Y' _ 0-25 _ 1 I
dt
As
0-6931 N 2 16 \~ J
7
0 \ - /
3.T = 4. t = - = 1-44T
X X i t 16
n = 4 As « = — T = - — = 4 days
N (I Y' t T ’ n
5. A. - , where ^ = ~
9 ’ r 0-693 0-693
\ — /
Decay constant X =
6. Number of atoms in excited state x, T 4

ww
-i
= 0*173 day
Example m A radioactive material is
where Eq is energy of atom in ground state and 1

Flo
is energy of atom in excited state. reduced to — of its original amount in 4 days.
16
Units used, r, 711 in second, X in sec"*, M Nq are

e
ree
numbers with no units. How much material should one begin with so that
4 X10"^ kg of the material is left over after 6 days
?

Fr
rF
Example [j[j] xhe decay constant for a (CBSE Sample Paper 2008)
uurr
given radioactive sample is 0*3465 day"*. What _ /V 1

for
Solution. Here . t = 4 days.
percentage of this sample will get decayed in a N 16
0
period of 4 days ? 2007 .'f')!
s
1 Y'
Solution. Here, X = 0-3465 day"’, t = 4 days. N I
kks
As n = 4.
Yo
N.0 2 16 2
oooo

0-693 0-693 v^- ^ V “ /


Half life, T = - 2 days
X 0-3465 t 4
eB

Half life T = - = - = I day .


No. of half lives, n = —
{
1=2 n 4 ^
T 2
Now. = 4 X 10 ^ kg, t = 6 days, Nq = ?
ur

N 1 Y' 1 1 n6/1
ad

- = 25% N 1 1 1
YYo

N 2 2 4 As
0 V / V “ / N. 2 64
0
Percentage of sample that will get decayed
= 64 N = 64 X 4 X 10"^ = 0*256 kg
dd

= 100-25 = 75%
Re
in

Example [Q TVitium has a half life of 12*5 TYPE IV. ACTIVITY OF A


F

RADIOACTIVE SUBSTANCE
years against beta decay. What fraction of a
sample of pure tritium will remain undccayed after
dN
25 years ? (NCERT Solved Example) Formulae used. 1. Activity, a = = XN
dt
1 ^l/T
\25/l2-5
N 1 1 2.A=An e ■Xi
Solution.
N.0 UJ 2 4
± = JL^(L'’‘ ,
\ / t
3. where n = —
1
-th of the sample will remain undecayed.
\ /Vo U T

Units used. /, T in second ; X in sec"’ ; N, Nq are


numbers with no units.
Example m It is observed that only Units of activity are :
6*25% of a given radioactive sample is left
1 curie = 3-7 x 10*** disintegrations/sec ;
undecayed after a period of 16 days. What is the
1 Bq = I disintegration/sec
decay constant of this sample in day"* ?
1 rutherford = 10^ disintegrations/sec.
ICBSE (OD), 2007 (C)j

i 1
NUCLEI 13/35

Example [f] The half life of 92^^^* against 0-693x6-023xl0-


23

a decay is 1*5 x 10^^ s. What is the activity of the 142x10^''x238


sample of 92^^* having 2*5 x 10^^ atoms ?
(CBSE 2005) = 1-235 X 1# Bq
Solution. Here, T= !-5 x lO'^ s ;
TYPE V. a, p AND y DECAY
N=2-5 X 1021
Formulae used
0-693 yv 0-693x2-5x1021 a
R = XN =
T 1-5x1012
P
= 11550 disintegrations/sec. 2-zX'' ■>
Z+l Y^ + _i€^+Q
Example m A radioactive sample contains

lowow
2-2 mg of pure which has half life period of
zX'^+y
1224 s. Calculate (i) number of atoms present
initially, (ii) the activity when 5 |ig of the sample Example fB The sequence of decay of
will be left. (CBSE 2005) a P
radioactive nucleus is D ^ D > D2

ee
11 1
Solution. Number of gC atoms in 11 g of a
= 6-023 X 1Q23 > £>3. If nucleon number and atomic number

Fr
sample
of Z>2 are 176 and 71 respectively, what are their
in 2-2 mg sample,
r FF
values for D and ? (CBSE 2017)

rer
6-023x1023
N.0 x2-2x10“3 = 1-2 X 102® Solution. As a is 2He"i and P is -1 e®, therefore,
fofr Fo
u
11
for D, mass no. = 176 + 0 + 4 = 180
ks
and in 5 pg of sample
charge number = 71 - 1 + 2 = 72
YYouro
6-023x1023x5x10'^
oo

/V = = 2-74 X 10
17
for D3, mass no. = 176 - 4 = 172
11
charge no. = 71 - 2 = 69
BBo ks

0-693
Example [Q One MeV positron en
r ee

Activity of the sample = R = XN = N


T
counters one MeV electron travelling in opposite
ouru
ad

0-693x2-74x10'2 direction. What is the wavelength of photons


Yo

1224 produced, given rest mass energy of electron or


= 1-55 X 10*** disintegrations per sec. positron = 0-512 MeV ? Take h = 6-62 x J-s.
d

Solution. Two photons are produced when a


Re

Example {0 The half life of 92^2^^* against


iYn

positron annhilates an electron.


a-decay is 4-5 x 10^ year. What is the activity of
FFind

0
1 g sample of 92^^^* ? (NCERT Solved Example) i.e., -I e + +1 e® = 2 y
Solution. Here, T = 4-5 x 10^ years Total energy involved
= 4-5 X 10^ X 365 x 24 X 60 X 60 s
= Rest mass energy + KE of both
= 1-42 X lO’^s
= 2(0-512+ l) = 2x l-5l2MeV
As number of atoms in 238 g
Energy of each photon
= Avogadro’s no. = 6-023 x 1023
2x1-512
.●. number of atoms in 1 g of sample = l-512MeV
2
6-023x1023
N = E= 1-512 X 1-6 X 10-’3j
238
he
dN 0-693 As £ = —
R = - = XN = N X
dt T

1
13/36 ‘Pn<uUe^'<i. Fundamental Physics (XII)EEIHD

he 6-62x10-3^x3x10^ Energy released, Q = 0-005138 x 931 MeV


X = ~ m
= 4-78 MeV
E 1-512x1-6x10"'3
= 8-21 X 10-13 m Example^ The nucleuS of
decays to by emitting an alpha particle.
Example [Q Calculate the amount of
Calculate the K.E. of a particle. Given atomic
energy released during a decay : mass of = 238-04954 a.m.u., mass of U'^'^
92
U238 90 10234 + ^He^ - 234*04096 a.m.u., and mass of a particle
Given, atomic mass of 921)238 = 238*05079 u = 4 002603 a.m.u.

atomic mass of 9^)Tli234 = 234*04363 u Solution. The decay is represented by the


atomic mass of 2He^ = 4*00260 u nucleai* reaction :

and 1 « = 931*5 MeV. (CBSE 2007) 94


Pl/238 92 f/234 + (q, particle) + Q

ww
Solution. The given nuclear reaction is Mass defect, Am - mass of Pu238 _ (p^yss of
U238 C/234 + mass of He^^)
92 > 9oTh234 +
Mass defect, Am = 238-05079 - 234-04363 = 238-04954 - (234-04096 -f 4-002603)

Floo
= 238-04954 - 238-043563 = 0-005977 a.m.u.
- 4-000260
Q value of the reaction = 0-005977 x 931 MeV

e
= 0-00456 u

eere
= 5-564 MeV.
Energy released = 0 00456 x 931-5 MeV
= 4*25 MeV This energy is shared by 92 and a particle.

FFr
As 926^^^^^ is very heavy compared to a particle,

oorr
uur r
I
therefore, this energy is carried mostly by the alpha
I
TYPE VI. NUCLEAR REACTIONS
particle.
s ff
Formulae used. K.E. of a particle = 5*564 MeV
sk
YYoo
->
Z+2
CV4+4 Z-fl y4+3 + ^ Example m We are given the following
ooko

... (a, p) reaction atomic masses:


eBB

2. In every nuclear reaction, mass number is 92 C/238 = 238*05079 u ;


conserved and charge number is also conserved.
90 J/i234 = 234*04363 u;
3. Q = (Am) c2
uurr

91 Pfl237 ^ 237*05121 u ; = 1*00783 ;


Units used. If Am in kg ; c in m/s; Q in joule. If Am
ad

is in amu, Q in MeV, using 1 amu = 931 MeV 2He‘* = 4*00260 U


Yo

(a) Calculate the energy released during a


Example ^ A neutron is absorbed by a decay of
dY
Re

3Li^ nucleus with subsequent emission of an alpha (b) Calculate the kinetic energy of emitted a
nind

particle. Write the corresponding nuclear particles,


FFi

reaction. Calculate the energy released in this (c) Show that cannot spontaneously
reaction.
emit a proton. (NCERT Solved Example)
m
(3Li<^) = 6*015126 M, m (2He^) = 4*0026044 u Solution, (fl) 92C/238 -> goTh^^^ +
m((/(^) = 1*0086654 m, in (jH3) = 3*016049 u Am = (238-05079 - 234-04363 - 4-00260)
Take lu = 931 MeV. (CBSE 2006) = 0-00456 u
Solution. The nuclear reaction is Energy released, Q = 0-00456 x 931-5 MeV
3U6 + 0
1
n‘ -¥
2He4 + jH3 + Q. = 4*25 MeV

Mass defect, A-4


(b) K.E. of a particle = — xQ
Am - m (3Li^) + m (gn') - m (2He‘^) - m
(iH^) I *4
= 6-015126 -f 1 0086654 - 4-0026044 238-4
X4-25 MeV
- 3016049 238

= 7-0237914 - 7-0186534 = 0-005138 u = 4*18 MeV

1
NUCLEI
13/37

(c) If emits a proton spontaneously, Solution. For the fission reaction,


92
t/238 91 P«237 +
1
7/' f/238 ●f 0.rn 1
92

A m = (238-05079 - 237-05121 - 1-00783) u mass defect, A/n = mass of + mass of n -


Q = - 0-00825 u=~ 0-00825 x 931 -5 MeV (mass of + mass of Ru^^)
= - 7'68 MeV = 238-05079 + 1-00867 - (139-90543 + 98-90594)
As the Q value is negative, the process cannot = 0-24809 u
proceed spontaneously. Q = 0-24809 X 931 MeV = 230*97 MeV
Example Under certain circumstances,
a nucleus can decay by emitting a particle more
TYPE VII. NUCLEAR FISSION
AND NUCLEAR FUSION
massive than an a-partide. Consider the following
decay processes :
Formulae used. 1. Energy released in both, fission

w
88 82 and fusion £=(Am) c2
88
Units used. When mass defect A/n is in kg ;
Calculate the Q-values for these decays and c = 3 X 10* m/s, energy E is in joule. When Am is in

Flo
determine that both are energetically allowed. amu ; £ is in MeV, using 1 amu = 931 MeV
The relevant atomic and particle masses are:

eee
m
(s8/ea223j ^ 223*01850 u ; Example ^ It is estimated that the atomic

Fr
m = 208*98107 u»
bomb exploded at Hiroshima released a total
m
(6C*‘*) = 14-00324 u
energy of 7*6 x lO^^ J. If on the average, 200 MeV

for
and = 219*000948 u.
m
energy was released per fission, calculate
ur
Solution. (/) For the decay process, (i) the number of Uranium atoms fissioned,
..£^223 (ii) the mass of Uranium used in the bomb.
ks
mass defect, Solution. Number of Uranium atoms fissioned
Yo
oo

A m = mass of Ra~^ - (mass of + mass of C^"^) n =


total energy released
eB

= 223-01850 - (208-98107 + 14-00324) energy released/fission


= 0-03419 u
7-6x10'2
Q = 0-03419 X 931 MeV = 31*83 MeV = 2*375 X
r

200x1-6x10-13
ou

(ii) For the decay process


ad

£«223 ■>
gf,Rn^'‘) + 2HeUQ Mass number
YY

88
Mass of Uranium = x n
mass defect, Avogadro’s number
Am- mass of £fl223 _ of Rn^^^ + mass ofHe"^)
nd

235x2-375x1024
Re

= 223-01850 - (219-00948 + 4-00260) = 926*66 g


6-023x1023
Fi

= 0-00642 u
Q = 0-00642 X 931 MeV = 5*98 MeV Example m If200 MeV energy is released
As Q values are positive in both the cases, in the fission of a single nucleus of 92^2^^, how
therefore both the decays are energetically possible. many fissions must occur per second to produce a
power of 1 kw ? (HP Hoard 2010f
Example w Consider the fission of 92^2^8 Solution. Here, energy released/l'ission
by fast neutrons. In one fission event, no neutrons = 200 MeV = 200 x 1-6 x 10"‘3 j
are emitted and the final stable end products, after = 3-2 X 10-^1 J
the beta decay of the primary fragments are
58 Calculate Q for this fission Total energy required/sec = 1 kw = 1000 w
= 1000 J/s.
process. The relevant atomic and particle masses
are : m (^2^^^^) = 238*05079 u ; Number of fissions/sec =
Energy reqd. per sec.
m (sgCe***^) = 139*90543 u ; energy released/fission
m i^Ru^) = 98*90594 u ; 1000
m((^*) = 1*00867 u; = 3*125 X 10*3 s
3-2x10"**
13/38 Fundamental Physics (XIQii^aiJD

Solution. Total targeted power = 2 x lO^ MW,


TYPE VIM. TYPICAL EXAMPLES
I Total Nuclear Power = 10% of 2 x lO^ MW
= 2x10^ MW.
Example m In an experiment, the activity
Energy produced/fission = 200 MeV,
of 1*2 mg of radioactive potassium chloride
(chloride of isotope of K - 40) was found to be Efficiency of power plant = 25%
170 s“^ : Taking molar mass to be 0*075 kg mole“^ Energy converted into electrical energy per
find the number of K - 40 atoms in the same and fission
hence find the half life of K - 40. Avogadro’s
25
number = 6*0 x 10^ arole”^ X 200 = 50 MeV
100
Solution. Here, m = 1-2 mg = 1-2 x 10'^ g -13
A = 170 s-*, M = 0-075 kg = 75 g, T= ? = 50 X 1-6 X 10 joule

ww
Number of molecules present in the sample Total electrical energy to be produced
m
= 2x10^ MW = 2 X 10^* X 10^" watt
/V = —xN.
M ^ = 2 X 10*^^ joule/sec

Flo
= 2 X 10^*^ X 60 X 60 X 24 X 365 joule/year
1-2x10-^

e
X 6 0 X 10^3 No. of fissions in one year

rere
75
18
2xl0'“x60x60x24x365

r FF
= 9-6 X 10
-13
50x1-6x10
, 0-693
As A = X A/ = ——— X N
uurr
^ 0-693/V
T

0-693x9-6x10’^
for= 2x
36x24x365
8
xlO^'^
kss
T = =
A 170 Mass of 6-023 x 10^^ atoms of f/^35
ooook
Yo
= 3*91 X 10’® sec = 235 gm. = 235 X 10-3 kg
Example ^ Suppose India has a target
eBB

2x36x24x365
xlO^'’atoms
of producing by 2020 A.D., 2 x 10^ MW of electric Mass of
8
power, ten percent of which is to be obtained from
urr

nuclear power plants. Suppose we are given that 235x10-3 2x36x24x365x202^


ad

on an average, the efficiency of utilization (i.c.,


Yo

0-023x1023 8
conversion to electrical energy) of thermal energy
dY

produced in a reactor is 25%. How much amount = 3-08 x 10“ kg.


of fissionable uranium will our country need per
Re

Hence, mass of uranium needed per year


innd

year ? Take the heat energy per fission of to = 3-08 X 10“ kg


Fi

be about 200 MeV.

1. Atomic I; *cleus is represented as where Z is charge number - number of protons.


A is mass number = number of protons number of neutrons
Number of neutrons in the nucleus = (A - Z)
-15
Nuclear radius (/?) is given by /? = /?q
1/3
, where Rq is a constant whose value is 1.2 x 10 m

It shows that heavier nuclei are bigger in size,


nuclear mass 3 m
Density of nuclear matter = - = 2.29 X 10‘2 kg/m3
nuclear volume
4^4
This value is constant for all nuclei.
NUCLEI
13/39

2. Binding Energy of a nucleus is the energy with which nucleons are bound in the nucleus. It is measured by
the work required to be done to separate the nucleons an infinite distance apart from the nucleus, so that
they may not interact with each other.
It was discovered that mass of a nucleus (m^) is always less than the calculated mass of nucleons in the
nucleus. This difference in masses is called mass defect (A m). It can be written as
A m = [Znip + (A - Z) - m^]
Total B.E. = (A m) C2 = [Zm^ +
Av. BE/nucleon = [Zm^ +(A-Z)m^
A

mass excess M-A


Packing fraction =
mass number A

w
3. Natural radioactivity is the phenomenon of emission of active radiations by heavy nuclei, on their own.
without any external provocation.

Flo
The three types of radiation are Alpha, Beta and Gamma rays. An alpha particle has 4 units of mass and 2
umts of positive charge. A P particle is an electron emitted from the nucleus. A vray photon has no rest mass

reeee
and no charge.

FFr
dN
According to radioactive decay law, — -XN, N = NQe where X is disintegration constant
dt

for
4. Half life of a radioactive element is the time during which half the total number of atoms in the element
ur
kkss
(A^o) disintegrate i.e. In t=T,N= NfJ2, T = %
Yo
oo

1
5. Average Life or Mean Life of radioactive element is x = — = 1-44T
eB

X 0-6931/T
6. Units of radioactivity are : 1 curie = 3.7 x 10^° disintegrations/sec.
r

1 rutherford = 10^ disintegrations/sec and 1 Becquerel = 1 disintegration/sec., which is the SI unit of


ou
ad

activity.
YY

In alpha decay, mass number decreases by 4 and charge number decreases by 2. In beta decay, mass number
remains unaffected and charge number increases by one. In gamma decay, the mass number and charge
ndd
Re

number both remain unaffected, only the energy changes.


7. A nuclear reaction represents transformation of one stable nucleus into another. In all nuclear reactions,
Fi

linear momentum, total energy, charge and nucleon number are conserved. Q value of nuclear reaction is
calculated from Einstein’s mass energy equivalence E = (Am)
8. Binding energy curve discloses that nuclear fission and nuclear fusion are two distinct ways of obtaining
energy from the nucleus.
9. Nuclear fission is the phenomenon of splitting a heavy nucleus into two or more smaller nuclei. Mass
defect m this process ap^«ars in the form of energy. The fission of by thermal neutrons is represented
as: 92 + + +e
The Q value of the reaction is 200 MeV.
Under suitable conditions, the three secondary neutrons may cause further fission of C/235 nuclei and start
what is known as nuclear chain reaction. The nuclear chain reaction is controlled by
''t

Neutron reproduction factor (K) = production of neutrons


rate of loss of neutrons
K= 1 represents critical stage. K> 1 respresents supercritical stage and 1 represents sub critical stage.
13/40 Fundamental Physics (XII)Migiail

10. A nuclear reactor uses nuclear energy for peaceful purposes. It is based on the phenomenon of controlled
‘ nuclear chain reaction. Moderators like heavy water, graphite, paraffin and deutrerium slow down neutrons.
Rods of cadmium or boron serve as control rods. Ordinary water and heavy water serve as coolants.
11. Nuclear fusion is the phenomenon of fusing of two or more lighter nuclei to form
^ a single heavy nucleus.
Mass defect in the process appears as energy. For example, ,//- + ,//-
Temperatures = 10^ K are required for fusion to take place.
12. Stellar energy is the energy obtained
' from the sun and stars. Our sun is radiating energy at ● the rate of
3.8 X 10^^ joule per second. Nuclear fusion is said to be responsible for stellar energy. The fusion reaction
in the sun can be represented as 4 j//’ ^ ^
Calculations show that Q = 26.7 MeV.
13, The wide spread destruction caused by nuclear weapons like atom bomb, hydrogen bomb etc. is named
nuclear holocaust.

fusion reactor, which is the future source of unlimited and

w
14. Controlled thermonuclear fusion is the basis of
unpolluted energy.

Flo
TP" 1^3 QUESTIONS

e
rree
Based on NCERT Book

r FF
4. Spontaneous decay in the sequence
I. Multiple Choice Questions c z-2^
^Z-1^ ■>
Z-3
uurr
1. The half life of a radioactive nuclide is 100 hours.
The fraction of original activity that will remain
after 150 hours would be
for
where Z is atomic numbei of element X. The
possible decay particles in the sequence are
kss
ia) p-, a, p+ (b) a, r. P""
(c) a, p-^, p- (d) a, p-
ooook

2 1
Yo
(a) (CBSE 2022)
3V2
5. In the nuclear decay given below :
eB

1 2
z-i
2^^
{CBSE 2022) Z-l
(c)
2V2 id) 3 The particles emitted in sequence are ;
urr

2. A nucleus with mass number 240 break into two (a) a, p, Y (b) P. a, Y
ad
Yo

fragments each of mass 120. The binding energy (c) Y, P. a id) p. T a


per nucleon of unfragmented nuclei is 7-6 MeV, 6. Assume that a neutron breaks into a proton and an
dY

while that of fragments is 8-5 MeV. The total gain electron. The energy released during the process is
Re

-27
innd

in the B.E. in the process is (mass of neutron = 1-6725 x 10 kg, mass of


(a) 216 MeV ib) 0-9 MeV proton = 1-6725 x lO'^"^ kg, mass of electron = 9
Fi

(c) 94 MeV id) 804 MeV X 10-3' kg)


(fl) 0-506 MeV ib) 7-10 MeV
(CBSE 2022)
ic) 6-30 MeV (i/) 5-4 MeV
3. Which of the following statements about nuclear
force is not true ? 7. Two spherical nuclei have mass number 216 and
64 with their radii and R2 respectively. The ratio
(a) Nuclear force between two nucleons falls is equal to
rapidly to zero as their distance is more than a ib)l:3
few femlometers.
(u) 3 : 2
(c) 1 : 2 id) 2: 3
(b) Nuclear force is much weaker than coulomb 8. A radioactive nucleus A with half life T, decays
force,
into a nucleus B. At some time /, the ratio of number
(c) Nuclear force is attractive for distances larger of B to that of A is 0-3. Then i is given by
than 0-8 fermi and repulsive if they are (/7)t = r/log(l-3)
(a) t = riog (1-3)
.separated by distances less than 0-5 fermi.
T log 2 log 1-3
id) The nuclear force between n-n : p-n and p-p is id) I =T
(CBSE 2022) log 2
approximately the same.
NUCLEI
13/41

1 16. Two samples X and Y contain equal amounts of


9. After how many days wiU — th of the radioactive
20 1
radioactive substances. If th of sample X and
element remain behind, if half life of the element 16
is 6-931 days ?
(1/256) of sample Y remain after 8 hours, then the
(a) 23-03 days (b) 25-12 days ratio of half periods of X and Y is
(c) 29-96 days (^0 27-72 days (a)2:l (b) 1:2
10. Two radioactive substances A and have decay (c) 1 : 4
constants 5A, and X respectively. At r=0, they have 17. The activity of a freshly prepared radioactive
the same number of nuclei. After a time
what will be the ratio of number of nuclei sample is 10^° disintegrations/second, whose mean
of A and B ? life is 10^ s. The mass of an atom of this isotope is
(a)4X 10 kg. The mass (in mg) of the radioactive
(b)2X sample is

ww
1 1
(c) (a)l (b)2
(b)
2X 4X (c)3 (d)4
11. The intensity of gamma radiation from a given 18. The half life of a radioactive substance is 20

Flo
source is /. On passing through 36 mm of lead, it minutes. The approximate time interval (/2 - /j)
is reduced to //8. The thickness of lead which will 2

ee
reduce the intensity to 7/2 will be between the time ^2 when - of its has decayed

rere
(a) 12 mm (b) 18 mm
and time t, during which (1/3) of it has decayed is

rFF
(c) 9 mm (d) 6 mm (a) 14 min (b) 20 min
12. A radioactive nucleus with initial mass number A (c) 28 min (d) 1 min
uurr
and atomic number Z emits 3 a particles and 2
positrons. The ratio of number of neutrons to that
of protons in the final nucleus will be
foor
19. A radioactive nucleus of mass M emits a photon of
frequency v and the nucleus recoils. The recoil
ks s
energy will be
Yoo
A-Z-4 A-Z-12 {a)M(?-hv (b) /I V/2 Mc^
oook

(a) (b)
Z-8 Z-4 (c) zero (d) hv
eBB

A-Z-4 A-Z-8 20. Which of the following statements is false ?


(c) (d)
Z-2 Z-4 (a) Atomic number (Z) of an element = number of
protons present inside the nucleus of the
13. A parent nucleus disintegrates into a daughter
uurr

element.
nucleus of mass Af which is moving with velocity
ad

(b) Mass number (A) of an element = number of


Yo

V and Am is the mass defect. The speed of daughter


nucleus is protons and neutrons present inside the nucleus
dY

of the element,

/ /
2Am Am
Re

(a) c (c) Number of neutrons inside the nucleus = (A - Z)


innd

M
(b) c
M
{d) In certain cases, Z may be greater than A.
FFi

Am Am 21. In an atom, whose nucleus is represented by


(c) c (d) number of electrons is
M +Am M +Am
{a) A (b)Z
14. The decay constant of a radioisotope is A,. If Aj, A2 (c)A-Z (d)A+Z
are its activities at times /j and t2 respectively, the
number of nuclei, which have decayed during the 22. In the famous relation, R = A^^^ ; Rq is an
time (/j - /2) empirical constant, whose value is
(a) Aj/j — A2/2 (b)A^-A2 (a) 1-2 X 1015 m ib) 1-2 X 10-15 m
(c) 1-2 X 10-m m (rf) 1-2 X IQio m-i
(c) (d)X{A^-A2) 23. Nuclear density is of the order of
X
(a) 10^^ kg/m^ (b) 10-17 kg/m3
15. A radioactive isotope has a half life T years. How
(c) 10-27 kg/m3
long will it take the activity to reduce to 1 % of its (d) 10l5 kg/m5
original value ? 24. Which of the following statements is true ?
(a) 3-2 T years (b) 4-6 T years {a) 17CI27 and are isotopes
(c) 6-6 T years (d) 7-2 T years {b) igAr^lll and are isobars
13/42 Fundamental Physics (XII) VOL.II

33. A radioactive element has half life of 30 seconds.


(c) iiNa^^ and ,2Mg^ are isotones If one of the nuclei decays now, the next one will
(d) All the above are true.
decay
25. Choose the false statement
(fl) any time {b) after 30 s
(a) Total energy of particle = rest mass energy + K.E.
(c) after 60 s (^0 after 30 h
(b) Two classical laws of conservation of mass and
conservation of energy have been unified into 34. A nucleus „X'" emits one a particle and one P
one law-conservation of mass energy particle. The mass number and atomic number of
(c)The equivalence of mass and energy is product nucleus, are
represented by the relation E = mc^. (a) (m - 4), n {b) (m - 4), (n - 1)
(cOHere, c is velocity of light in vacuum (c) (m - 3), n + 1 id) (m - 3), (n - 1)
= 2 X 10® m/s 35. Out of ’ 7 and gO^®, the pair of
26. Which of the following statements are true ? isotones is

ww
13
(a) Nuclear Binding energy is the energy with (a) 6C'^ 8°“ W 7N“'.
which nucleons are bound in the nucleus (d) gO'«
(c) jC'''
(b) Nuclear B.E = work required to separate the 36. The average binding energy per nucleon is

Flo
nucleons, an infinite distance apart from the maximum for the nucleus.
nucleus

e
(c) Mass defect. Am - Zmp + (A-Z) m„ - mf^ (a) ^He (i) ‘2O

reree
where mf^ - mass of nucleus

r FF
{d) All the above statements (d)
and for what
27. The peak value in B.E curve is 37. In the Uranium radioactive series, the initial
uurr
nuclei ?

(a) 8*8 MeV/N for 26^^^ foor


nucleus is ^92^ and the final nucleus is
When uranium nucleus decays to lead, the number
ks s
(b) 6-8 MeV/N for
of a-particle and p-particle emitted are
Yoo
(c) 5-9 MeV/N for Li^
ooook

(a) 8a,6P (b) 6a,7p


(d) None of the above (iO 4 a, 3 P ■ ●
(c) 6a, 8 P
eBB

28. The factors governing nuclear stability are :


38. In gamma rays emission from a nucleus,
(fl) Binding energy per nucleon
(a) only the proton number changes
(b) Neutron to proton ratio in the nucleus
uurr

(c) Whether the nucleus contains odd or even (b) both the proton number and neutron number
ad

change
Yo

number of protons and neutrons


(d) All the above factors (c) there is no change in the proton number and
dY

neutron number
29. Choose .the false statement out of the following :
(d) only the neutron number changes
Re
innd

(a) Nuclear forces are the strongest forces in nature.


7
(b) Nuclear forces are very short range forces,
39. Starting with a sample of pure Cu^, - of it decays
FFi

(c) Nuclear forces are central forces.


(d) They are saturated in character. into Zn in 15 minutes. The corresponding half life
30. The ratio of size of a hydrogen atom to the size of is

its nucleus is (a) 10 minutes (b) 15 minutes


(a) 105 (b) 10-5 (c) 5 minutes (d) 7-5 minutes
(c)l(y (^10-4
31. Two elementary particles which have almost
(1]
40. Activity of a radioactive sample decreases to ^
infinite life time are
(a) electron and neutron(fc) neutron and proton of its original value in 3 days. Then in 9 days its
(c) electron and proton (d) none of the above activity will become
32. Radioactive radiations in order of increasing 1
(a) — of the original value
penetrating power are:
(a) y, P, a (b) Y, a, p 1
(c) a, p, Y id) a, Y» P (b) - of the original value
NUCLEI
13/43

1
(c) — of the original value
II. Assertion-Reason Type Questions
Direction. For question numbers 46 to 53, two
statements are given, one labelled Assertion (A)
{d) ^ of the original value and the other labelied Reason (R). Select the
correct answer to these questions from the codes
41. Which word equation represents p+ decay ? (a), {b\ (c) and {d) as given below :
(a) proton neutron + electron + electron {a) Both, A and R are true, and R is correct
antineutrino
explanation of A.
ib) proton —> neutron + electron + electron neutrino (/?) Both, A and R are true, but R is not the correct
(c) proton —> neutron + positron + electron explanation of A.
antineutrino
(c) A is true, but R is false.

ww
W) proton —> neutron -i- positron + electron (</) A is false, and R is also false.
neutrino

42. In a radioactive decay series, three successive 46. Assertion. Neutrons possess very high penetrating
power.

Flo
decays each result in a particle being emitted. The
first decay results in the emission of a P-particle. Reason. The ionising power of neutrons is very

e
low.
The second decay results in the emission of an

rere
a-particle. The third decay results in the emission 47. Assertion. The density of the nuclei of all the atoms

r FF
of another P'''-particle. is same.

FIGURE 13(Q).1 Reason. The density of the nuclei is independent


uurr
r a p" foor
of mass number.

48. Assertion. Within the nucleus, neutron is a stable


kss
P ●►Q ■►R
particle.
Yoo
■►S
ooook

Reason. Outside the nucleus, the neutron is an


‘Nuclides P and S are compared. unstable particle with half life period 12 seconds.
eBB

Which statement is correct ? 49. Assertion. The atoms which have same mass

{a) P and S are identical in all respects. number are called isotones.
uurr

ib) P and 5 are isotopes of the same element. Reason. jH^ and 2He^ are isotones.
ad

(c) S'is a different element of lower atomic number. 50. Assertion. In nuclear reactor, heavy water is used
Yo

(d) S is a different element of reduced mass. as a moderator.


dY

43. What is the ratio of nuclear radii if the mass


Reason. The controlled chain reaction occurs as
numbers of two nuclei are 4 and 32
the neutrons are absorbed by the heavy water.
Re
innd

(n) 1 : 2 (b) 1:3


(c) 1:4
51. Assertion. A nucelus contains no electrons, but can
Fi

(^0 1 :5 eject them.


44. Which statement about alpha, beta and gamma
radiation is correct ? Reason. The betadecay can take place at a time
from the nucleus as per relation,
(a) Alpha radiation has the greatest ionizing power.
(b) Beta radiation has the greatest ionizing power, On* ■> +
-I e
0
+v .

(c) Gamma radiation has the greatest ionizing 52. Assertion. Natural radioactive nuclei are nuclei of
power.
high mass number.
(d) Alpha, beta and gamma radiation have nearly
equal ionizing powers. Reason. The binding energy per nucleon of heavy
45. The nuclei of the isotopes of an element, all contain nuclei is large as compared to that of stable nuclei.
the same number of a certain particle. What is this 53. Assertion. Neutron-proton ratio increases with beta
panicle ? decay.
{a) electron (h) neutron Reason. During P-decay, number of neutrons
(c) nucleon id) proton increases by one in a radioactive nucelus.
13/44 p>tcuUe^'^ Fundamental Physics (XII)EEIBD

ANSWERS

I. Multiple Choice Questions


5. (/>) 6. (fl) 7. (fl) 8. id) 9.(c) 10. (c)
l.(c) 2. (a) X(b) 4.(d)
15. (c) 16. (fl) 17.(a) 18. ib) 19. (b) 20. {^0
11.(a) 12. (a) 13. (a) 14. (c)
25. {£/} 26. (f/) 27.(a) 28. (cf) 29. (c) 30. (a)
21. (b) 22. (b) 23. (a) 24. (d)
35.(a) 36. (c) 37. (a) 38. (c) 39. (c) 40.(a)
31. (c) 32. (c) 33. (a) 34. (b)
41. (d) 42.(b) 43.(a) 44. (a) 45. (^0

ooww
II. Assertion-Reason lype Questions
50. (c) 51. (a) 52.(c) 53. (rf)
46. ib) 47. (fl) 48. (c) 49. (d)

HINTS/EXPLANATIONS For Difficult Questions

e
7. Here, A] = 216 and A2 = 64

re
I. Multiple Choice Questions 1/3
From R = Rq A

rFFl
ree
1. Here T= 100 hr, r= 150 hr

F
0/3
If Aq is initial activity of radioactive nuclide and
n1/3
/?, A ^216 6 _ 3

rF
A is activity at time t, then a V 64 4 " 2
^2
/ 050/100
f f^
fsfoor
1
ouur
/
A

A) 2; 2; U ■2V2 A/
B
= 0-3 ●●● = 0-3 ,^4
8. At time r|,
kosk
2. Total gain in B.E. = B.E. of products
Yo
~ B.E. of reactants
If Nq is total number of nuclei at r = 0, then
oo
Y

= (120 + 120) X 8-5-240x7-6


BB

^A " ^0
= 240 (8-5-7-6) = 216 MeV N
0
rre

3. Statement 0) is not true. A/^ (14-0-3) = % = 1-3


oYuu

4. On decay, atomic number of product nucleus -Kt


ad

As b!,A = Nr,0 e
decreases by 1 on decay, atomic number of
product nucleus increases by 1. On a decay,
dY

N 1
atomic number of product nucleus decreases by 2. 0
=:Nr.e
0
-Xi
or e
1-3
1-3
Hence correct order of decay will be given by (</)●
innd
Re

;u = ln(l-3)
5. Using conservation of charge number and mass
or
Fi
F

number, we find In (1-3) In 1-3 In (1-3)


t =
xT
Z-t-l + _ie^ {i.e. P) X ln(2)/r In (2)

z+\ z-i 4- 2He‘* (a) 9. Here, T= 6-931 days


z-i z-l + y
0-6931 0-6931
X = = 0-1 per day
Hence the sequence of panicles emitted is P, a, y. T 6-931
6. Here, qb* -4 ]H* 4- -1 N
Mass defect, A m = + m^) - ni„ From = e-^
N
-27 0
= (1-6725 X 10"^'^4-9x 10"^') - 1-6725 x 10
1
= 9 X l(r^‘ kg = or ^o->'=20
20
Energy released. E = (Am)
= 9 X 10“^^ (3 X 10^)^ = 81 X 10"^^ J 0-1 t = 2-303 logio 20 = 2-303 x 1-3010
81x10-15 2-303x1-3010
MeV = 0-506 MeV r = = 29-96 days
-13 0-1
1-6x10
NUCLEI
13/45

10. At f = 0, = ^Vq for both the substances A and B \n


1
15. As
Aq~ Nq ~[2j "100
N,
1 = Nq and
N
I _ £V n (log 1 - log 2) = log 1 - log 100
g-^2^ n (0-0-3010) = 0-2
^2
2
n \4Xt n = = 6-6
= e-^^‘= i 0-3010
\ej t = nT=&6T

ijrf ,
N
As therefore,4X,r = 2orr = — 16. Here, t = S hy = 9

ooww
^2 kCj 23i 7^2
11. As / = /„0 e-H-f A, rv iT'=± ("1
\4

1
= e
1
__g
A) UoJ K2) 16 v2y
n, = 4

e
8 2
1 ri \8

re
\3 \2) 256 \2J /I2 — 8
(I

rFFl
ree
= = e-l^(36)

F
^2J Tj _ tinL n 8
- _2 = - = 2:l

rF
/. -3jlc = -36|X f/«2 n1
36
— = 12 mm 1

fsfoor
JC =
ouur
3 17. Activity A = }JI = - N
X
12. As per question, 3 a and 2 positrons are emitted
kosk
where x is mean life time of given sample
●●● z^'’^^^l^-'^ + 3 2He‘' + 2(,.,«<>) A^=At = (10^® decays/sec) X10^ s= 10*^ atoms
Charge number of final nucleus = (Z - 8)
Yo
oo

Mass of sample, m = Nx mass of one atom


Y
Mass number of final nucleus = (A - 12)
= 10^9 X 10-25 kg = l(r6 kg = 1 mg
BB

Number of protons in final nucleus, = (Z - 8) 18. Here, 7 = 20 min.


Number of neutrons in final nucleus,
rre

n„ = (A-12)-(Z-8) = A-Z-4
In time t^, radioactive sample left behind
oYuu

n
A-Z-4
. 3j“3'
ad

n _

n Z-8
dY

P
In time t2, radioactive sample left behind
13. KE of daughter nucleus of mass M moving with
=1-1 '
innd
Re

vel. V = —Mv^ 3 3
2
Fi
F

t!T

-a
N
Energy released due to decrease in mass (Am) of As
the parent nucleus = (Am) c2 N.
0

= {Am)c'^ 2 1 (1
and
3 <2> 3 \2J

/ 2(Am)c2 _ 4 Dividing, we get


2Ain
or v =
M M
1 ^1
14. Activity at itme : Aj = XJVj or Ni = Aj/A. 2 \2) ^2-^1 = ^=20inin
Activity at itme or - A2/A, 19. Momentum of emitted photon.
Number of nuclei decayed during time interval hv
(^1 - ^2) is /^photon — ~
N,1 -A(2=4-^
2 X X
= Vlil
X
If V is speed of recoil nucleus, then
/^nucleus ~ ^photon
13/46 Fxmdamental Physics (Xll)toaiim

\n
hv hv
Mv = — V =
As , therefore, n = 3.
c Me
Wo 8 UJ u
hh^
RecoD energy of nucleus - -Mv^ = 15
- 2Mc^ From t~nT, T = - — = 5 min
n 3
20. Statement (d) is false. Atomic number cannot be
N 1
greater than mass number. 40. In 3 days, — =
21. Atom is electrically neutral. Number of electrons N.0 3‘
= number of protons = Z.
22. The correct value of empirical constant N 1
/? = 1-2 X 10-*^ m. In next 3 days, — = - x
N,0 3 Nq" 9
23. Nuclear density is of the order of 10^^ kg/m^. V /

w
24. All the three statements (a), ib\ (c) are true. 1 1 i =—
//
N
25. Velocity of light in vacuum is 3 x 10* m/s and
3 [w„J " 3 ^ 9 ” 27
= -X
In next 3 days,
0
not 2 X 10* m/s.

Flo
26. All the three statements (a), {b) and (c) are true, 1
27. The value in B.E curve is 8*8 MeV/N for Hence, V" = = — of original value.

e
27 27
Fe^®.

ree
26
28. All the factors (a), (/?), (c) governing nuclear 41. Proton —> neutron + positron (P'*’) + electron

FFr
neutrino.
stability are correct.
29. Statement (c) is false. Nuclear forces are non 42. In going from nuclide P to S, decrease in mass

for
central forces. number = 4
ur
Size of hydrogen atom _ 10-*0 m = 1()5
decrease in charge number = 2- I- l= 0
30.
P and S must be isotopes of the same element
kkss
Size of its nucleus 10-^5 m
Yo
31. Electron and proton have almost infinite life time.
r (aV (4 >1/3 j
oo

32. Penetrating power of a is minimum and that of y 43. 1= ^ ^


^2 [^) I32j "2
eB

is maximum.
33. As radioactivity depends on the law of chance,
the next nucleus may decay any time. 44. Alpha radiation has the greatest ionizing
power as their energy is maximum.
ur

34. As a = 2He"^ and p =


ad

.*. with emission of one a and one P-particle mass 45. In isotopes of an element, all contain the same
YYo

number becomes (m - 4) and atomic number number of protons,


becomes (n - 2 + 1) = (n - 1)
35. The pair of isotones must have same number of n. Assertion-Reason lype Questions
d
Re

neutrons = (A - Z). This is true for (a) and


in

46. Both Assertion and Reason are true but Reason


8 can not explain the Assertion,
F

36. From the binding energy cu^e average mass of nucleus


BE/nucleon is maximum for 26^®^°- 47. Density of nucleus =
37. 92U^3^ ■> 82 Pb206 volume of nucleus
Decrease in mass number = 238 - 206 = 32
AX 1-66x10-^'^
32 = 2*97 X lO*"^ kg m"5
Number of a-particles emitted = — - o ^3 7C (MX 10-15)3 X A
Decrease in charge number = 92 - 82 = 10 Since, the density of nucleus is independent of
8 a particles decrease charge no. by 16 mass number A, hence density of all nuclei is
Number of P particles emitted = 16 - 10 = 6 same. Thus both Assertion and Reason are true
38. As gamma ray photon has no mass and no charge; and Reason is the correct explanation of
therefore, in gamma emission, there is no change Assertion.
in proton number and neutron number. 48. Outside the nucleus, the neutron is an unstable
39. Here,
N 7 1
-; / = 15 min particle and decays into proton, electron and
N.0 8 8 antineutrino with half life period about 12
NUCLEI
13/47

minutes. Thus Rea.son is false. Here Assertion 51. Both Assertion and Reason are true and
is true since in the nucleus, no. of neutrons Reason is the correct explanation of Assertion.
remains constant due to nuclear exchange forces. 52. Here Reason is false as the binding energy per
49. The atoms whose nuclei have same number of nucleon of heavy nuclei is small as compared to
neutrons, are called isotones. Thus. Assertion is that of stable nuclei. Such nuclei undergo
false. 2 He-^ having same mass number are radioactive disintegration in order to attain greater
isobars. Hence. Reason is also false. value of BC/nucleon. Then they become stable.
Here Assertion is true.
50. Here Reason is false as the heavy water is u.sed
as a moderator to slow down the neutrons so that 53. During [3-dccay from a radioactive nuclei, a
neutron inside the nucleus converts into proton
a sustained chain reaction may take place. Here. and hence neutron-proton ratio decreases. Thus
Assertion is true.
Reason is false. Here. Assertion is also fal.se.

w
PROBLEiVdS

Flo
1. Atomic Nucleu.s 5. The isotope has 8 protons, 8 neutrons

e
and 8 electrons, while has 4 protons, 4

ree
1. Electrons cannot be a part of nucleus but neutrons and 4 electrons. Yet the ratio of their

FFr
protons can be. Why ? atomic masses is not exactly 2. Why ?
Sol. This is because de Broglie wavelength Sol. The ratio of mass of and mass of is
urr
associated with electrons is larger than the size

for
not exactly two because of difference in the
of nucleus. And the de Broglie wavelength mass defect/binding energy of the atomic nuclei
as.sociated with protons is smaller than the size of the two elements.
kkss
of nucleus.

2. Is free neutron a stable particle ? If not, what II. kadi ctivity


Yo
ooo

is its mode of decay ? iPh. Board 2001)


Sol. No, free neutron is not a stable particle. Its mean 6. A radioactive nucleus contains no electrons,
eB

yet it ejects them. How ? {Pb. Board 2003)


life is about 1000 second. It decays into a proton,
an electron and an antineutrino. Sol. A neutron inside a radioactive nucleus is
ur

0«' ^ iH’ +_j£-0+ V unstable. It decays into a proton, an electron and


ad

an antineutrino.
3. Group the following six nuclides into three
YYo

pairs of (i) isotones (ii) isotopes and > |H* +_|e®+ V


(iii) isobars.
198 It is this electron, which is emitted as a p-particle
H3,
d
Re

’ 1 from the radioactive nucleus.


in

2013, 2004 (C)l


7. A radioactive sample having N nuclei has
Sol. (0 Isotones ; 79Au'^'^
F

activity R. Write down an expression for its


[Same number of neutrons, {A - Z)] half life in terms of/? and N.
(ii) Isotopes [Same Z, different A] Sol. Activity, R = XN :.X= RIN
(iii) Isobars ,He^ ,H^ [Same A, different Z] 0-693 0-693
4. The mass of the nucleus is less than the sum Half Life, j =
X R
of the masses of the nucleons forming it, 8. Prove that the instantaneous rate of change
why ? C BSE 2009, 2006)
of acitivity of a radioactive substance is
Sol. In the formation of the nucleus, the neutrons inversely proportional to the square of half
and protons have to collect in a very small space life. (.B.si 'Sample Paper 2008)
whose size is of the order of 10'^^ m. The energy dN
required for this purpose is spent by the nucleons Sol. Activity R = - = XN
dt
at the cost of their masses. As a result of this,
the mass of the nucleus formed becomes less The instantaneous rate of change of activity
than the sum of the masses of the individual dN
nucleons. dt
= X(-XN)=-X- N
dt dt
13/48 “Pn^sidec^’^. Fundamental Physics (XII)naiBIl
When we take into account the energies of
-0-693 0-693^/V parent and daughter nuclei involved in beta plus
N =
T decay (or positron decay), the decay is always
energetically allowed.
dR 13. Two different radioactive dements with half
Clearly OC
, which was to be proved.
di T- lives r, and T, have and N, (undecayed)
atoms respectively present at a given instant.
9. During alpha decay of a nucleus, how does Determine the ratio of their activities at this
the neutron to proton ratio change ? instant.
(CBSE 2006)
dN 0-693/V
Sol. Let us consider a-decay of 92^^^^ Sol. Wc know A = = XN =
dt T

ooww
92
y238 ^ 90 Th234 + 2Hc*^
Neutron to proton ratio before a-decay
0-693/V
1 0-693/V2
1
and ^^2 =
238-92 146
T1 Tl
> and
92 92 ^ N T.
A, _ /V Lxi-=

e
Neutron to proton ratio after a-decay A, 1 ^2 [ ^2 J t J
.T

re
234-90 144
14. Four nuclei of an element fuse together to

rFFl
ree
form a heavier nucleus. If the process is

F
90 90
accompanied by release of energy, which of

rF
144 146
As > . therefore, the ratio increases. the two - the parent or the daughter nucleus
90 92
would have higher binding energy per

fsfoor
ouur
10. During beta decay of a nuclues, how does the nucleon ?
neutron to proton ratio change ? Sol. As lighter nuclei are less stable than the
kosk
(CBSE 2003) intermediate nuclei, therefore, S£/nucleon of
Sol. Let us consider P-decay of g3Bi"'‘^ daughter nucleus will be higher than the BE/
Yo
oo

bP-JO ^ 84 p^210 + _l^0 nucleon of parent nucleus. The difference in the


Y
83
BB

Neutron to proton ratio before beta decay masses of parent nucleus and daughter nuclei
is the mass defect, which is released in the form
210-83 127
rre

of energy.
83 83
oYuu

Neutron to proton ratio after beta decay III. Nuclear Energy


ad

210-84 126 127 15. What is the difference between a photon and
dY

. w'hich is less than


84 84 83 a neutrino ?
Hence, the neutron to proton ratio decreases Sol. A photon is one quantum of eleclromagneiic
innd
Re

in p-decay. radiation. It has zero rest mass, zero charge, zero


11. Name two radioactive elements which are not spin and no antiparticle. Its energy E = hv
Fi
F

found in observable quantities. Why is it so ? depends on its frequency.


Sol. Tritium and Plutonium are two radioactive A neutrino is an elementary particle that
elements which are not found in observable accompanies P-decay. It has zero rest mass,
quantities in the universe. This is because half
life period of each of the two elements is short zero charge and spin = —(h/2n). It has an
compared to the age of the universe. antiparticle called antineutrino. It can have any
12. A free proton cannot decay into (« +e* + v), energy from zero to a value permitted by nuclear
because such decay is not energetically reaction. Its nature is non electromagnetic.
allowed. Yet we observe in nature beta decay
16. Safety of nuclear reactors is an important
with positron emission. How do we under issue. Guess some of the safety problems that
stand the emission of positrons from nuclei ?
a nuclear engineer must cope within reactor
Sol. A free proton cannot decay into a neutron as design.
the Q value of reaction becomes negative. Sol. One of the major safely problems in a reactor is
However, inside the nucleus, both protons and that the nuclear waste from the reactor contains
neutrons are bound. The bound nucleons behave
some long lived radio active isotopes. Further,
differently than free nucleons.
NUCLEI
13/49

accidents due to excessive heating and melting As power = 5 x watt


of the reactor core have to be prevented by = 5 X 10^® joule/.sec.
designing appropriate cooling systems.
17. What are delayed neutrons ? Discuss their 29191473x1045
role. .■. Time required = sec.
5x10^“
Sol. Some neutrons produced in fission are delayed
by some time as they tire produced in subsequent 29-191473x10*^
decays of the initial fission fragments. The.se 5x60x60x24x365
year
are called delayed neutrons. This situation is
crucial to mechanical control of the reactor. If / = 1-85 X 10^ years
all the fission neutrons were produced instantly 20. (a) Write symbolically the process expressing
in fission, there would be no lime for the minute the p'*’ decay of jjNa^^. Also write the basic
adjustment required in a reactor to keep it nuclear process underlying this decay.

ww
critical.
(/>) Is the nucleus formed in the decay of the
18. Calculate the disintegration energy Q for nucleus i|Na^", an isotope or isobar ?
fission of I"'® equal fragments (CBSE 2014)

Floo
21 by bombarding with a neutron. Given Sol. {a) |]Na“- 10 Ne22 + + Q
that
i 4. i 4.

ree
m {^2^0^^) = 97-90541 u, (Parent (Daughter) (p"^) Energy
m (2,Sc4*^) = 48-95002 u, m„ = T00867 u

rFee
nucleus) nucleus)
Sol. The disintegration energy in fission of The ba.sic nuclear process is decay by emission

F
is given by

oor r
of p'*' particle.
rur
Q = (Ml) X 931 MeV = [m {^2^0'^^) {b) The nucleus formed in the decay of nNa^-
s ff
+ m„- 2 m (2,Sc4y)l X 931 MeV is obviously an isobar having same mass
= [97-90541 + I -00867 - 2 x 48-95002] number but different charge number.
osk
YYoou
x931 MeV
21. Two nuclei have different ma.ss numbers Aj
oook

Q = 1-01404 X 931 = 944-1 MeV and A2‘ Are these nuclei necessarily the
eBB

19. A star converts all its hydrogen to helium, isotopes of the same element ? Explain.
achieving 100% helium composition. It then (CBSE 2022)
converts the helium to carbon via the reaction
uur r

Sol. The two nuclei would only be isotopes of the


2//^** + + 2^fe‘* > + l-ll MeV
ad

same element if their atomic numbers were the


The mass of the star is 5-0 x 10'^^ kg and it
Yo

same. The two nuclei having different mass


generates energy at the rate of 5 x 10'^® watt. number may be isotopes of the same element if
How long will it take to convert ail the helium they have the same atomic number and those
dY

to carbon at this rate ?


Re

nuclei will not be isotopes of an element if their


idn

Sol, Mass of star = 5-0 x lO^- kg = 5-0 x 10^-5 g atomic numbers and different.
FFin

Number of atoms in 4 gram of He 22. Two nuclei may have the same radius, even
= 6-023 X
though they contain different number of
Number of atoms in 5-0 x 10^5 gram of He protons and neutrons. (CBSE 2022)
6-023x10^5 x5-0xl(>^^ Sol. Nuclear radius R is given by
= 7-529 X I058 I/.I
4 /? = /?(,4
As each fusion reaction consumed 3 helium where A = atomic mass = (n^, + n„)
atoms to produce 7-27 MeV energy. For same radius, sum of protons and neutrons
.●. Total energy produced by fusion of 5-0 x 10^5 is same. Their individual numbers may vary.
gram of He 23. Calculate and compare the energy released
7-27 by (a) fusion of TO kg of hydrogen deep
x7-529xl0^** MeV
3 within the sun, and (b) the fission of TO kg
of jji 3 fission reactor.
7-27x7-529xl05«xl-6xl0-'5 Sol. In sun, four hydrogen nuclei fuse to form a
3 helium nucleus with the release of 26 MeV
= 29-191473 X 1045 j energy.
13/50 “P^cuice^ ^ Fundamental Physics (XII) LigaUJ
Energy released by fusion of I kg of hydrogen 6x102^x1000
X 200 MeV
6xl0-^xl0^ 235
x26 MeV
4 £2 = 5*1 X MeV
£, = 39 X 102<^ MeV 39xIQ2^
= 7-65,
As Energy released in fission of one atom of £2 5-1x102^
= 200 MeV,
92 f.e., energy released in fusion is 7-65 times the
Energy released in fission of 1 kg of energy released in fission.

● Very Short Answer

ww
● Short Answer
● Long Answer

Flo
VERY SHORT ANSWER QUESTIONS Carrying 1 mark

e
ree
I. Atomic Nucleus Ans. 6£/nucleon =
(Am) c'2

Fr
A
1. How is nuclear size related to its mass

rF
number ? [CBSE 2013(C)] where c = velocity of light in vacuum.
uurr
8. State two characteristic properties of nuclear

for
Ans. The radius R of atomic nucleus is related to mass
1/3
number A of the nucleus as R = RqA where forces. (CBSE 2011, 2008)
Rq = 1-2 X 10"’^ m ; an empirical constant. Ans. Nuclear forces are the strongest forces in nature.
s
kks
2. What is the order of nuclear density ? They ai'e effective only inside the nucleus.
Yo
Ans. Nuclear density is of the order of 10*^ kg/m^.
oooo

9. Name two elementary particles which have


3. Why is nuclear density same for all nuclei ? almost infinite life time.
eB

(CBSE 2013) Ans. Electron and proton have almost infinite life
mass time.
Ans. This is because density = , and volume
ur

volume 10. What is the ratio of kV/h to MeV ?


ad

of nucleus varies directly as its mass number. Ans. 1 kWh = 10^ (Js-‘) 60 X 60 s = 3-6 x 10^ J ,
YYo

4. Why are heavy nuclei usually unstable ? 1 MeV = 1-602 X 10“*^ J


Ans. Heavy nuclei are unstable due to large repulsive
IkWh 36x10^
dd

forces between large number of protons in the = 2-25x10'^


Re

nucleus. IMeV ^ 1-602x10'''^


in

5. If one a.m.u. = 1*66 x lO"^^ kg, what is the


11. What holds nucleons together in a nucleus ?
F

mass of one atom of 7 (ICSE 2002)


Ans. Nuclear forces amongst the nucleons in a
12 nucleus.
Ans. As 1 a.m.u. = — mass of one atom of C
12
12. How many coulomb of charge is carried by
mass of one atom of C'2 = 12 a.m.u. 1 kg of electrons ?
= 12 X 1-66 X 10-2"^ kg = 1-992 x IQ-^^ kg. Ans. 1-77 X 10*^ C. It is the value of of electrons.
6. How many electrons, protons and neutrons 13. Name three nuclei which lie on maxima In
are there in a nucleus of atomic number 11
Binding energy curve.
and mass number 24. ^ 2UU2)
Ans. Three nuclei which lie on maxima arc
Ans. In the nuclide number of protons = 11,
number of neutrons = 24 - 11 = 13. Number of ftC'2 and
electrons = 0, in the nucleus. 14. Name three nuclei which lie on minima in
7. A nucleus of mass number A has ma.ss defect Binding energy curve.
10
{Ant) what is BE per nucleon of this nucleus ? Ans. Three nuclei which lie on minima are 56
[CBSE 2004 (03
and yLi^.
NUCLEI
13/51

15. Proton and neutron exist together in an 24. Two nuclei have mass numbers in the ratio
extremely small space within the nucleus. 1 : 2. What is the ratio of their nuclear
How is this possible when protons repel each densities ? (CBSE 2009)
other ?
Ans. The ratio of nuclear densities is 1. This is
Ans. Due to strong attractive nuclear forces. because nuclear density does not depend upon
16. Write down the names and formulae of the mass number.

three isotopes of hydrogen.


Ans. Hydrogen (,//*), deuterium and tritium II. Radioactk ity
25. Identify the nuclides X and Y in the nuclear
17. Select the pairs of isobars and isotones from reactions
the following nuclei: : —>
4Be« + X ;
Ans. Isobars -,N^\ Isotones (ICSE 2001)

w
-I
18. Select the pairs of isotopes and isotones from
Ans. Using conservation of mass number and charge
the following nuclei: 15/^, 15/*^^ no.

(Uttarakhand Board 2012)

Flo
5+ 1 -4 = 2 + = 4 U.,
Ans. Isotopes 15P'''}, Isotones (gC'^ 6-(-l) = 7'^
14-0= 14
i.e., 7Y
14

e
19. Two nuclei have mass numbers in the ratio

reee
1:8. What is the ratio of their nuclear radii ? 26. Which has greater ionizing power : alpha

FFr
particles or beta particles ? (CBSE 2010)
(Manipur Board 2012, Kerala Board 2012,
CBSE 2009) Ans. Alpha particles have greater ionising power than
beta particles.

for
ur
xl/3

Ans.
R
1 Ja if' 1 27. Out of alpha, beta and gamma radiations,
^2 A, 2 which are affected by electric field and
kss
magnetic field ? (CBSE 2004)
20. The binding energies of deutron (jH^) and
Yo
a-particle (2He'*) are 1-25 and 7-2 MeV/ Ans. Both, alpha and beta radiations are affected by
oo

nucleon respectively. Which nucleus is more both, electric field and magnetic field.
eB

stable ?
(CBSE 2000} 28. The decay constant of a radioactive element
is Give the formula for half life and
Ans. a-particle (2He"^) is more stable, because a life.
mean

nucleus is more stable when value of binding (CBSE 2007)


ur
ad

energy per nucleon is larger. 0-693 1


YYo

Ans. T = and t = —.
21. Four nuclei of an element undergo fusion to X X
form a heavier nucleus, with release of 29. The mean life of a radioactive sample is .
energy. Which of the two - the parent or the What is the time in which 50% of the sample
d
Re

daughter nucleus would have higher binding would get decayed ?


in

energy per nucleon ? (CBSE 2018) (CBSE Sample Paper 2011)


F

Ans. Parent nucleus would have higher binding Ans. Time in which 50% of the sample will get
energy per nucleon. This is because when decayed is half life
daughter nuclei are formed, some energy is
0-693
released. Therefore, daughter nuclei have lesser T = = 0-693 X = 0-693 r„.
m
binding energy per nucleon. X

22. Which one is unstable among neutron, 30. Why do alpha particles have a high ionising
proton, electron and a-particle ? power ? (CBSE 2010)

Ans. Neutron is unstable out of the four given Ans. Alpha particles have a high ionising power
particles. because of their large mass, large velocity and
large K.E.
23. Nuclear forces are short range forces.
Comment. 31. What is the difference between a beta particle
and electron ?
Ans. Nuclear forces operate only within the nucleus Ans. An electron and a p-particle are essentially the
i.e. only upto distances = 10 fermi. Hence they same. We can say that a ^-particle is an electron
are short range forces. of nuclear origin.
13/52 Fundamental Physics (XII)S!Zs29D

32. A nucleus emits one a particle and one 40. Draw the graph showing the distribution of
^-particle. What are the mass number and kinetic energy of electrons emitted during
atomic number of the product nucleus ? beta decay. (CBSE 2006)

Ans. With emission of an alpha particle and a Ans. The distribution of K.E. of electrons emitted
beta particle (_|e®), the product nucleus has during beta decay is shown in Fig. 13(Q).2.
mass number = (m - 4) and charge number
= (n- 1).
33. Arrange radioactive radiations in the order
of increasing penetrating power.
Ans. a, p and Y arys.
34. What is the relation between decay constant
and half life of a radio active element ?

w
Ans. r=0.693/X
35. What are units of activity of radioactive
elements ?

Flo
Ans. 1 curie = 3-7 x 10^*^ disintegrations/sec.
1 rutherford = 10^ disintegrations/sec and
III. Nuclear Energy

ee
1 bccquerel = 1 Bq = 1 decay/sec.

Fr
36. The initial concentration of a radioactive 41. A nucleus 92^^^^ undergoes alpha decay and
substance is and its half life is 12 hours. transforms into thorium. What is mass
What will be its concentration after 36 number and charge number of nucleus

for
ur
hours ? (CBSE Sample Paper 2019-20) produced ? [CBSE 2011 (C)l
Ans. Here, T= \2h,t= 36 h Ans. 92
U235 ^ + 92-2 Xh235^
ks
06/12
Mass number of Thorium = 235 - 4 = 231
y/T-
Yo
/V ( 1 fl 1
Charge number of Thorium = 92 - 2 = 90
oo

From
N.0 2 2 2 8
42. A nucleus emits two a-particles and
eB

N = Nf^S
two beta particles and transforms into a
thorium nucleus. What is the mass number
37. A radioactive material has a half life of
and atomic number of Thorium so produced?
r

1 minute. If one of the nuclei decays now,


ou
ad

when will the next one decay ? (CBSE 2010)


Ans. The next nucleus can decay any time. Ans. Alpha particle is 2He‘‘ and Beta particle is _^e^.
Y

38. The half life of a radioactive element A is on emission of 2 a and 2 [5 particles,


same as mean life time of another radioactive
nd
Re

mass no. of Thorium = 235 -2x4 = 227


element B. Initially, both have same number
Chrage no. of Thorium = 92 - 2 x 2 - 2(~1) = 90
Fi

of atoms. B decays faster than A. Why ?


43. Name the process responsible for energy
Ans. = 1-44 Tq > Tg (CBSE 2004)
productionin the sun.
Therefore,B decays faster than A.
Ans. Fusion of four hydrogen nuclei into helium
39. A radioactive isotope decays in the following nucleus.
0 a
44. Name the absorbing material used to control
sequence : A A A l ^ ^2
the reaction rate of neutrons in a nuclear
If the mass number and atomic number of (CBSE 2008)
reactor.
A2 are 176 and 71 respectively, find the mass Ans. Cadmium or Boron are absorbers of neutrons.
number and atomic number of Aj and A.
Which of the three elements are isotopes ? They serve as controllers of reaction rate of
neutrons in the nuclear reactor.
Ans. As a = therefore, mass number of A,
= 176 + 4 = 180 and charge number of A j =71 45. What do you mean by Q value of a nuclear
reaction ? (Pb. Board 2004)
+ 2 = 73. Similarly, mass number of A = 180 +
1 = 181 and charge no. of A = 73 + 0 = 73. Ans. It is the energy released/absorbed in the nuclear
reaction.
Clearly, elements A} and A are isotopes ?
NUCLEI 13/53

46. What is meant by critical size ? Ans. Applying the principle of conservation of charge
Ans. An uncontrolled nuclear chain reaction can number.
occur only when the lump of uranium (target) Z + 57 = 92
is greater than a certain minimum size. This is Z= 92-57 = 35
called critical size.

47. What is the mass of pion plus (Tf'') ? 50. Neutrons produced in fission can be slowed
Ans. Mass of a pion plus is 273 times the mass of an down even by using ordinary water. Then,
electron. why is heavy water used for this purpose ?
48. What is the mass of muon plus (iJ.'*’) ? Ans. A material which is rich in hydrogen serves as a

oww
Ans. Mass of a muon plus particle is 207 times the better moderator.
mass of an electron.
51. Which physical quantity in a nuclear reaction
49. In a particular fission reaction, a 92 i/235 is considered equivalent to the Q value of the
nucleus captures a slow neutron. The fission reaction ? (CI3SE Sample Paper 2019-20)
142
products are three neutrons, a ^-^La

ee
Ans. The quantity equivalent to Q-value of reaction
nucleus and a fission product 2-X^* What is is the energy absorbed/released in the reaction.
the value of Z ?

FFrlo
r
rF
ee
SHORT ANSWER QUESTIONS Carrying 2 marks
ouru
rF
I. Atomic Nucleus 5. Why heavy stable nucleus must contain more
neutrons than protons ?

ffosor
1. Two nuclei have mass numbers in the ratio os k Ans. Coulomb forces between protons are repulsive
2 : 5. What is the ratio of their nuclear and nuclear forces are ordinarily attractive. For
densities ? (CBSE 2009) nuclei to be stable nuclear forces must dominate
Ans. The ratio of their nuclear densities is 1, as the repulsive forces. Therefore, number of
ook
Yo
neutrons must be greater than the number of
Y
nuclear density is constant for all nuclei.
protons.
2. What is the nuclear radius of Fe^^^, if that of
Bo

6. Which property of nuclear forces is


reeB

Al^^ is 3*6 fermi. (CBSE 2008)


xl/3
responsible for the constancy of binding
125 4/3 5 energy per nucleon ?
oouY

Ans. As
ur

27 3 Ans. Nuclear forces are saturated in character. This


R,
ad

property makes B.E./nucleon constant for most


of the nuclei.
dY

/?. = - R. =-x3-6 = 6*0 fermi


3^3 7. How many electrons, protons and neutrons
nidn

are there in a nucleus of atomic number 11


3. Assuming the nuclei to be spherical in shape,
Re

how does the surface area of a nucleus of and mass number 24 ?

mass number compare with that of a Ans. No. of electrons in the nucleus = 0,
F
Fi

nucleus of mass number A2 ? No. of protons = atomic number =11,


[CBSE 2008 (C)J No. of (protons -t- neutrons) = mass number = 24
No. of neutrons = 24 - 11 = 13
/ RO \2 A1 f 8. Calculate the energy equivalent of 1 a.m.u.
Ans. 1 A
in MeV. Board 2011)
A. ^2 A,- A,-
j V y Ans. From £ = mc~,
4. What is the effect on neutron to proton ratio when m = I a.m.u.= 1-66 x 10“^^ kg
in a nucleus when (/) an electron, (ii) a £ = (1-66 X I0“27) (3 X 10^)2 joule
positron is emitted ?
_ l-66x9xlQ~" MeV =933-75 MeV
Ans. In emission of an electron, a neutron is converted
1-6x10-'^
into a proton. Therefore, number of neutrons
decreases and the number of protons increases. 9. You are given two nuclei and 31’“*.
The neutron to proton ratio decreases. In the Explain giving reasons, as to which one of
emission of a positron, a proton is converted into the two nuclei is likely to be more stable ?
a neutron. Hence the ratio increases.
(CBSE 2006)

a
13/54 "PteuCcef^ U Fundamental Physics (XII) tTOWl
Ans. In case of , of one P particle increases the charge number
neutron number 7-3 by I without affecting mass number, yeniission
= 1-33
proton number 3 affects neither of two. Therefore, for A_^ : mass
In case of
no.= 180-8= 172 charge no. = 72-4+ I =69.
neutron number 4-3 i
14. The sequence of decay of radioactive nucleus
- = 0-33 is
proton number 3 3 a P a a
D ^ D1 ^ ^3 ^ />4
For stability, this ratio has to be close to one.
If nucleon number and atomic number of D2
Obviously, nucleus is more stable than the are 176 and 71 respectively, what are their
nucleus values for D and ? iCBvSE2010)
10. If the nucleons bound in a nucleus are
Ans. As mass number of each a particle is 4 units

ooww
separated apart from each other, the sum of
and its charge number is 2 units, therefore, for
their masses is greater than the mass of the
nucleus. Where does this mass difference
^4
4 = 176-8= 168
come from ? Explain briefly.
Z = 71 -4 = 67
Ans. When nucleons are bound in a nucleus, some

e
of their mass is converted into energy that binds Now. charge no. of P is - 1 and its mass number

re
them together in the nucleus. The energy equal is zero, therefore, for D

rFFl
ree
to binding energy is spent to separate these 4=176 + 0 + 4 = 180

F
nucleons apart from one another. It is this energy Z=71 1 + 2 = 72

rF
which appears in the form of increased mass
15. Define the activity of a radionuclide. Write
[A m = B.E./c2].
its SI unit. Give a plot of the activity of a

fsfoor
ouur
II. Radioactivity radioactive species versus time.
(CBSE 2010, 2009. 2005)
kosk
11. Show that the decay rate of a sample of
radionuclide Is related to the number of Ans. Activity of a radionuclide is defined as the rate
Yo
of disintegration of the radionuclide. It is also
oo

radioactive nuclei N at the same instant by


Y
called count rate and is represented by 4. The
the expression R = XN.
BB

(C.'-- ^005)
activity of the sample at any instant is directly
Ans. According to radioactive decay law.
proportional to the number of atoms left
rre

N = Nr.0 e~^.
undecayed in the sample at that lime. A plot of
Rate of decay.
ouu

activity of the sample with time is as shown in


Y
ad

dN Fig. 13(Q).3. Obviously, activity varies


R = -
exponentially with time and is represented as
dY

dt dt

= XNQe~'^='kN.
innd

12. What % age of a given mass of a radioactive


Re

substance will be left undecayed after five


Fi
F

half life periods ?


Ans. As no. of half lives, n = 5

N (\

^0 25 “3^
= 3-125%

13. A radioactive nucleus ‘A’ undergoes a series


of decays according to the following scheme : A = A,^0 e~^'
a a y
A —> A2 SI unit of activity is Becquerel (Bq).
+ A3 + A4
The mass number and atomic number of A 1 Bq = 1 decay/sec.
are 180 and 72 respectively. What are these
in. Nuclear Energy
numbers for 44 ? (CBSE 2009)
238
Ans. Emission of 2 alpha particles decreases the mass 16. Why is 92 U not suitable for chain
reaction ? (HP Hoard 2001)
number by 8 and charge number by 4. Emission
NUCLEI 13/55

Ans. This is because only fast moving neutrons of Ans. The law of conservation of spin (angular
12 MeV energy can cause fission of 92 U238 momentum) is violated. Each particle on right
nuclei. Such neutrons have less chance of 1
interaction. They escape the fissionable material side has a spin —(hlln) so that the resultant
without causing fission.
spin on the right side is 0 or 1 (h/2 n). And on
17. Nuclear fusion Ls not possible in laboratory.
Why ? [CBSE 2003 (C)] 1
the left side, spin is — (/* / 2 k) .
Ans. This is because nuclear fussion requires
temperatures as high as 10^ - 10^ K. Such high 23. What are thermal neutrons ? Why are
temperature are often generated in nuclear neutrons considered as ideal particles for
fission. That is why fission precedes fusion. nuclear fission ?
These processes cannot be carried out in
Ans. Thermal neutrons are low energy neutrons
laboratory.
18. A fusion reaction is much more energetic than = —eV. Neutrons are considered as ideal

w
40
a fission reaction. Comment. (Pb. Board 2011)
particles for nuclear fission, because they are
Ans. True. This is because energy released per unit
neutral particles on which neither any force of
mass of fuel is much higher in fusion than in

Flo
fission. attraction nor any force of repulsion acts.
24. A heavy nucleus X of mass number 240 and

e
19. Write symbolically the nuclear p"*" decay
binding energy per nucleon 7-6 MeV is split

rree
process of gC’*. Is the decayed product X an into two fragments Y and Z of mass numbers

r FF
isotope or isobar of ?
110 and 130. The binding energy of nucleons
Given the mass values of (^C*') = 11-011434 ii in Y and Z is 8-5 MeV per nucleon. Calculate
uurr
and m (X) = 11-009305 u. Estimate the Q
value in the process.
Ans. gC** ^
(CBSE 2015)
(i.e., p+) + Q
for
the energy Q released per fission in MeV.

Ans. The nuclear reaction is


(CBSE 2010)
kss
Clearly, * is an Isobar of gC *'. X240 ^yll0^zl30 + Q
ooook
Yo
Mass defect, A/n = 11 -011434 - (11 -009305 As per the given data,
+ 0-000545) Q = 110 X 8-5 + 130x8-5 -240x7-6
eB

= 0-001584 u = 240 (0-9) MeV = 216 MeV


Q value of reaction = 0-001584 x 931 MeV Energy released/fission = 216 MeV
urr

= 1-4747 MeV 25. Draw a graph showing variation of potential


ad

20. What is meant by critical mass in a nuclear energy of a pair of nucleons as a function of
Yo

chain reaction ? their separation. Indicate the region in which


dY

Ans. Critical mass in a nuclear chain reaction is the nuclear force is (a) attractive (b) repulsive.
minimum mass of fissionable material that [CBSE 2018 (C), 2012]
Re
innd

would sustain the nuclear chain reaction at a Ans. The variation of potential energy of a pair of
steady rate. The neutron reproduction factor K, nucleons as a function of their separation (r) is
Fi

in this case, becomes equal to one i.e. rate of shown in Fig. 13(Q).4.
production of neutrons is just equal to the rate We find that corresponding to r = OC = 0-8 fm,
of loss of neutrons. P.E. is minimum. At this distance, force between
21. A chain reaction dies out sometimes, why ? nucleons = 0. For distances > OC, negative P.E.
goes on decreasing. The nuclear forces are
Ans. A chain reaction may die out due to any of the
attractive. For distances < OC, negative P.E.
following reasons; decreases, becomes zero and then increases. The
(/) Size of fissionable material may be less than nuclear forces in this region arc repulsive.
the critical size.

(ii) Mass of fissionable material may be less than


the critical mass.

(Hi) Neutron absorbing material (arrestor) might


absorb neutrons at a faster rate than the rate at

which they are being produced.


22. Which law is violated in the following nuclear
iW’ + ?
I
reaction ?
13/56 ‘P'utcCce^ ’4- Fundamental Physics (XII) orTTl

SHORT ANSWER QUESTIONS Carrying 3 marks

1. What do you understand by atomic number and 11. Distinguish between nuclear fission and nuclear
mass number ? Explain giving examples. fusion. (Hr. Board 2012, Manipur Board 2012)
[Art. 13.3] [Art. 13331
2. Show that atomic nuclei of different elements 12. Some scientists have predicted that a global
have different sizes. [Art. 13.4] nuclear war on earth would be followed by
‘nuclear winter’. What would cause nuclear
3. Show that nuclear density is same for all the winter ?
nuclei. [Art. 13.35]
(CBSE 2013) [Art. 13.5]
13. How do you account for energy from the
4. Differentiate between isotopes and isobars. nucleus ? [Art. 13.27]
(Jharkhand Board 2011)
14. Give salient features of controlled

w
[Arts. 13.6 and 13.7] thermonuclear fusion. [Art. 13.34]
5. Define atomic mass unit and electron volt.
15. Draw a plot of binding energy per nucleon as a
Establish a relation between them.
function of mass number for a large number of

Flo
[Art. 13.2] nuclei 2 < A < 240. How do you explain the
6. What is the significance of binding energy per constancy of binding energy per nucleon in the

ee
nucleon ? (Jharkhand Board 2012) range 30 < A < 170 using the property that

Fr
[Art. 13.10] nuclear force is short ranged. (CBSE 2010)
[Art. 13.10]
7. Explain the nature of nuclear forces ?
16. Derive an expression for average life of a radio

for
ur
(.1 & K Board 2012) [Art. 13.13]
nuclide. Give its relationship with the half life.
8. Discuss briefly about nuclear stability.
(CBSE 2010) [Art. 13.20]
ks
[Art. 13.14]
17. In a typical nuclear reaction e.g. jH- + jH^
Yo
9. Define the term decay constant of a radioactive 2He-^ + n + 3-27 MeV, although number of
oo

nucleus. [Art. 13.19] nucleons is conserved, yet energy is released.


eB

10. Define half life and average life of a radioactive How ? Explain. (CBSE 2013) [Art. 13.27]
substance. What is the relation between the two? 18. Write the basic nuclear processes underlying P'*'
(Ml* Board 2012) [Arts. 13.19, 13.20] and p' decays. (CBSE 2017) [Art. 13.24]
ur
ad
Yo

LONG ANSWER QUESTIONS Carrying 5 or more marks


nd

1. What is atomic nucleus ? Who discovered it ? 5. Explain the term mass defect and binding
Re

What are atomic number and mass number ? energy. How are they related ? Draw B.E. curve.
Fi

Explain with illustrations. [Arts. 13.1 & 3] (Uttarakhand Board 2012, J & K Board 2012,
2. Explain nuclear size and nuclear density. Show Karnataka Board 2012) [Art. 13.10]
that nuclear density is same for all the nuclei. 6. Draw a graph showing the variation of potential
[Arts. 13.4 and 13.5] energy between a pair of nucleons as a function
3. What do you understand by isotopes, isobars of their separation. Indicate the regions in which
and isotones ? Explain with illustrations. the nuclear force is (/) attractive (ii) repulsive.
(CBSE 2008) [Arts. 13.6, 13.7 & 13.8] [CBSE 2017 (C), 2009, 2007] [Art 13.12]
4. Explain the concept of nuclear binding energy. 7. Explain the concept of nuclear forces. Discuss
Draw a curve between mass number and average their chaiacteristic properties. Which properties
binding energy per nucleon. Explain the energy distinguish them from electrostatic forces ?
relea.se in the processes of nuclear fission and [CBSE 2017 (C), 2011] [Art. 13.12]
nuclear fusion from this plot. 8. What is meant by natural radioactivity ? What
[CBSE 2018. 2017 (C). 2011. 2009. 2008, type of radiations are emitted ? Explain briefly
Uttarakhand Board 2012] the nature of these radiations.
[Art. 13.10] (Bihar Board 2012) [Arks. 13.15 & 13.16]

r
NUCLEI 13/57

9. State and explain the laws of radioactive 14. Discuss brieny the phenomenon of a-decay.
disintegration. Hence define disintegration How will you account for it ? [Art. 13.23]
constant and half life period. Establish relation 15. Explain briefly the phenomena of ^-decay and
between them. (.1 & K Board 2012.
y-decay.
(CBSE (F), 2017, Hr. Board 2012, HP Board 2011) Raj. Board 2011) [Arts. 13.17]
[Arts. 13.17, 13.18 & 13.19] 16. What are nuclear reactions ? Give examples.
10. State the law of radioactive decay. If Nq is the State the conservation laws obeyed in such
number of radioactive nuclei in the sample at reactions. What is meant by Q value of the
some initial time /q, find out the relation to nuclear reaction ? [Art. 13.26]
determinethe numberN present at a subsequent 17. What is meant by nuclear fission and nuclear
time. chain reaction ? Outline the conditions
(CBSE 2008, 2013, Raj. Board 2012, Karnataka necessary for nuclear chain reaction.
Board 2012, HP Board 2011) [Art. 13.17] [Arts. 13.28 & 29]

w
,-X/f
11. Deduce the expression N = N^e for the law
18. Draw a labelled diagram of nuclear reactor, and
of radioactive decay. discuss the function of each of its main
(CBSE 2014) [Art. 13.17] (A.P. Board 2012)

Flo
components.
12. State the law of radioactive decay. Plot a graph [Art. 13.31]

reeee
showing the number (N) of undecayed nuclei 19. Explain the phenomenon of nuclear fusion,
as a function of time for a given radioactive giving some examples. [Art. 13.32]

FFr
sample having half life Ty2- Depict in the plot, 20. What do you understand by nuclear holocaust ?
the number of undecayed nuclei at (/) t = 3T\i2
and i = 5 T 1/2- (CBSE 2011) [Art. 13.17] [Art. 13.35]

for
21. Differentiate between radioactive decay and
ur
13. What is meant by average life of a radioactive
nuclear fission. (Jharkhand Board 2011)
element ? Derive an expression for it.
kkss
[Art. 13.20] [Arts. 13.15 and 13.28]
Yo
oo

CASE-BASED VERY SHORT/SHORT QUESTIONS


eB

CASE 1. One atomic mass unit or unified mass 149x10-'^


12 E = MeV = 931-25 MeV
unit is defined as (1/12) of the mass of an atom of 1-6x10“'3
isotope.
r

Read the above paragraph carefully and


ou
ad

As Avogadro’s number = 6-023 x 10“-^ answer the following very short and short
YY

.-. Mass of 6 023 x 10^^ atoms of = 12 gram answer questions :


1. Define a.m.u, and state its value in S.I. unit of
12
ndd

Mass of one atom of =


Re

mass.
6023x10-3 “
2. How many units of mass (in a.m.u.) are there in
Fi

I 12 10 gram of ?
1 a.m.u = —X = 1-66 X 10-2“^ g
12 6-023x1023 3. How do you relate a.m.u. to MeV ?
4. Deduce the value of 1 a.m.u. in kWh.
1 a.m.u = i-66 X 10 2/ kg
Atomic masses can also be expressed in terms of CASE 2. After the discovery of neutron by
unit of energy, which is electron volt (eV) Chadwick, proton neutron hypothesis was put forward
As work done = charge x potential by Heisenberg. According to this hypothesis, a nucleus
of mass number A and atomic number Z contains Z
I eV = (1-602 X I0-*^C)x I volt
protons and (A - Z) neutrons. As an atom is electrically
= 1-602 X 10“‘'‘'joule neutral, therefore, number of peripheral electrons must
1 MeV = 10^ eV = 1-602 x 100-’3 J be equal to Z. the number of protons inside the nucleus.
Relation between a.m.u and MeV Mass of proton, = 1-6729 x kg
From, E = mc^ = 1-007825 a.m.u.

= (1-66x10-2^) (3x 10^)2 Mass of neutron, = 1 -6743 x 10~27 kg


= l-49x 10~‘0j = 1-008665 a.m.u.
13/58 7\<}uCee^’4. Fundamental Physics (XII)EE3HD
Atomic number (Z) of an element = number of m A 3m
protons inside the nucleus of an atom. Mass p =
4ji«3
number (A) of element = total number of protons and
neutron.s present inside the nucleus.
.●. Number of neutrons in the nucleus = (A - Z) As m and Rq are c\ istant, therefore, density (p)
of nuclear matter is the same for all nuclei (= 2-29 x
Inside the nucleus, a neutron is a stable particle.
However, outside the nucleus, a neutron is unstable 10^^ kg/m-^)
having a mean life time of 1000 second. It decays into Isotope.s of an element are the atoms of the
a proton, an electron and antineutrino. element, which have same atomic number (Z) but
different mass number (A).
O'' > |H' + V
Isobars are the atoms of different elements, which
A neutron has low ionising power, but a large
penetrating power, as it carries no charge. have the same mass number (A) but different atomic
number (Z).
Read the above paragraph carefully and

w
answer the following very short and short Isotones are the nuclides which contain the same
answer questions : number of neutrons, i.e., in their case (A - Z) = A is the

Flo
5. What is atomic number (Z) of an element ? same.

6. What is number of neutrons in the nucleus ? Einstein was the first to establish the equivalence

e
of mass and energy through the famous relation

rree
7. How many numbers of protons and neutrons are
there in

r FF
8. What do you know about stability of neutron ? E = mc^ where c = 3 x 10^ m/s = speed of light i.i in

CASE 3. Experimental measurements show that vacuum. This relation means that when a certain mass
uurr
volume of a nucleus is directly proportional to mass
number (A) of the nucleus. If R is radius of nucleus, for
m disappears, an equivalent amount of energy £ appears
and vice-versa.
kss
assumed to be spherical, then volume Read the above paragraph carefully and
ooook
Yo
4 T
- Tl/?-’ o: A or /? oc A
1/3 answer the following very short and short
3 answer questions:
eB

or
£ = /?oA*^ 9. What is the ratio of nuclear radii of two elements

where Rq is an empirical constant = 1-2 x 10“^^ m. with mass numbers 8 and 27 ?


urr

For different elements, A is different. Therefore, 10. With usual notation,


ad

atomic nuclei of different elements have different


R = R^A^’^
Yo

sizes.
what is the value of empirical constant Rq ?
dY

Density (p) of nuclear matter is the ratio of mass


of nucleus and its volume 11. Is nuclear density same or different for different
Re
innd

nuclei ? What is its value ?


Mass of nucleus m A
12. What is Einstein mass energy relation ? What
Fi

P =
Volume of nucleus
-7C/?3 does it imply ?
3

ANSWERS

1. 1 a.m.u. is defined as (I/I2) of the mass of one 3. This is done using Einstein mass-energy relation.
atom of isotope. 1 a.m.u. = 1-66 x 10“^^ kg. 4. Given, m = 1 a.m.u. = 1-66 x 10
-27
kg
2. 1 a.m.u. = 1-66 x 10"^^ kg = 1-66 x 10“-"^ gram. As, E = m c^ = (1-66 x 10“^^) x (3 x I0^)“ J

.●. 1 gram of =
1
a.m.u.
_ 1-66x9x10^^' kWh
-24
1-66x10 (36x10^)
[ -.■ 1 k Wh = 10^ X 60 X 60 = 36 X 10^ J]
10
10 gram of = -24
= 4-15 X 10"** k Wh
1-66x10
5. Atomic number (Z) of an element = number of
= 6*02 X 10^ a.m.u. protons inside the nucleus of the atom.
NUCLEI 13/59

6. Number of neutrons inside the nucleus \l/3


= mass number (A) - atomic number (Z) ^1 -fA 8 2

= (A-Z) AJ 27 3
1/3
7. No. of atoms in 12 gram of = 6 023 x 10-^. 10. In the relation R = Rq A , is empirical
Each atom of carbon of atomic weight 12, has 6 constant = 1-2 x 10”*^ m.
protons and 6 neutrons. Therefore, in 12 gram of 11. For different nuclei, nuclear density is the same
carbon, the number of protons or number of 3 m
neutrons = 6 x 6 023 x 10^3 = 36-138 x lO^^ = 2-29x lO^'^kg/m^
= 3-614 X 1024
8. Inside the nucleus, a neutron is a stable particle. 12. Einstein was the first to establish the equivalence
However, outside the nucleus, a neutron is of mass (m) and energy (E) through the relation
unstable having a mean life time of 1000 second. E = mc^
It decays into a proton, an electron and

ww
anlineutrino. where c = 3 x 10^ m/s = speed of light in vacuum.
This relation has been verified in all nuclear
.0
0" - 1 reactions.

Flo
CASE-BASED MCQs AND ASSERTION-REASON QUESTIONS

e
rree
CASE 1. Binding energy of a nucleus is the For Question No. 3 and 4, we have given two

r FF
energy with which nucleons are bound in the nucleus. statements each, one labelled as Assertion (A)
It is measured by the work required to be done to and other labelled as Reason (R).
uurr
separate the nucleons an infinite distance apart from
the nucleus. The origin of nuclear B.E. has been
explained on the basis of Einstein’s theory of mass
given below :for
Choose the correct option out of the four options
kss
energy equivalence. It is found that rest mass of a (a) Both A and R are true and R is correct
explanation of A.
ooook

nucleus is always slightly less than the sum of the masses


Yo

of free protons and neutrons composing the nucleus. (b) Both A and R are true and R is not the correct
eBB

This difference is called mass defect (Am). This mass explanation of A.


defect appears in the form of B.E. (c) A is true but R is false.
In a nucleus
(d) Both A and R are false.
urr

Z = charge number = number of protons 3. Assertion. Binding energy of a nucleus is the


ad

A = mass number = number of protons


Yo

energy with which nucleons are bound in the


+ number of neutrons nucleus.
dY

.●. Number of neutrons = (A - Z) Reason. Binding energy is measured by work to


Re

Mass defect, Arn = [Z + (A - Z) - nif^]


innd

be done to separate the nucleons from the nucleus


B.E = (A?n) = [Z + (A - Z) to infinity.
Fi

Note that B.E of electron is much smaller 4. Assertion. Rest mass of a nucleus is always a
(= eV or keV) compared to rest mass energy of a nucleon little more than the sum of masses of free protons
(= 10^ MeV). In the B.E/nucleon curve, peak value of and neutrons composing the nucleus.
maximum is 8-8 MeV/N for B.E curve led
Reason. This mass excess appears in the form of
us to the possibility of release of energy in the processes
of nuclear fission and nuclear fusion. binding energy.

Based on the above paragraph, answer CASE 2. Natural radioactivity is a spontaneous


questions no. 1 to 4 : and self disruptive activity of certain heavy elements
1. Binding energy of electron is of the order of by virtue of which a heavy element disintegratesitself
(a) eV or keV ib) MeV without being forced by any external agent to do so.
(c) GeV (c/) none of these The phenomenon was discovered by Henry Becquerel
2. In the BE/niicleon curves, peak value is in 1896. About 40 natural elements with atomic number
for . (fill in the blanks). greater than 82 are found to be radioactive. Some
(a) 6-1 MeV for 2He^ (b) 8-8 MeV for 26Fe^^ examples are Radium, Thorium, Polonium, Actinium
12
(c) 6-7 MeV for {([) none of these etc.
13/60 ‘P'utdccft'^ Fundamental Physics (XH)EEMD
Emission of active radiations indicates that parent 6. Choose the natural radioactive element
nucleus is unstable, The heavy nuclei have larger atomic (a) Iron (b) Gold
number Z and hence contain larger number of protons. (c) Radium id) Platinum
The mutual repulsion of protons reduces the binding For Question No. 7 and 8, we have given two
effect of nuclear forces. This is the main cause of statements each, one labelled as Assertion (A)
relative instability of heavy nuclei and radioactivity. and other labelled as Reason (R).
The radiations emitted by radioactive elements are Choose the correct option out of the four options
of three kinds given below :
(/) One kind of radiation with least penetrating {a) Both A and R are true and R is correct
power was named alpha Rays. explanation of A.
(//) Another kind of radiations with comparatively (b) Both A and R are true and R is not the correct

oww
larger penetrating power was called Beta Rays. explanation of A.
(Hi) The last kind of radiations with maximum (c) A is true but R is false.
penetrating power was called Gamma Rays. (d) Both A and R are false.
It was discovered that 7. Assertion. Alpha rays are deflected through

e
smaller angle towards negative plate than p-rays

rFFlo
(0 a rays are deflected through smaller angles

re
towards the negative plate, which are deflected through larger angle towards
positive plate.

ree
(ii) P rays are deflected through larger angles towards

F
Reason. Alpha rays carry positive charge and p-
the positive plate,
rays carry negative charge. And P-rays are much

rF
iiii) y-rays remain undeflected.
lighter than a-rays.
Based on the above paragraph, answer 8. Assertion. In an external electric field, gamma

fsfoor
ouur
questions no. 5 to 8 : rays remain undeflected.
5. Natural radioactivity is
kosk
Reason. Deflection towards positive plate is
(n) spontaneous (b) self disruptive equal and opposite to the deflection towards the
Yo
(c) both (fl) and (h) (d) neither (a) nor (b) negative plate.
oo
Y
BB

ANSWERS
rre

1. (fl) 2.{b) 3. (a) 4. id) 5.(c) 6. (c) 7. (o) 8. (c)


oYuu
ad

HINTS/EXPLANATIONS For Difficult Questions


dY

1. B.E of electron is of the order of eV or keV. 5. Natural radioactivity is spontaneous and self
disruptive activity.
innd

2. Peak value in B.E/nucleon curve is 8*8 MeV for


Re

26
Fe5^ 6. Radium is a natural radioactive element.
Fi
F

3. Both, the Assertion and Reason are true, and 7. Both, the Assertion and Reason are true and
Reason is correct explanation of A.ssertion. Reason Is correct explanation of the Assertion.
4. Both, the Assertion and Reason are false. 8. Assertion is true. However, the Reason is false.

TYPE I. ATOMIC NUCLEUS, 2. Calculate the density of hydrogen nucleus in SI


ITS SIZE AND DENSITY units, given Rq= 1.2 fermi, mass of a proton =
1.007825 a.m.u. [Ans. 2-295 x kg m"^]
1. How many electrons, protons and neutrons are 3. Calculate the equivalent energy of electron and
there in 14 gram of ? Avogadro’s number proton at rest. Given that mass of electron
= 6x102^ per gram atom. = 9-1 X 10"^* kg and mass of proton = 1-673 x
[Ans. 36 X 1Q2^, 36 x 10^, 48 x lO-^"^ kg. [Ans. 0-511 MeV, 941-1 MeV]
NUCLEI 13/61

4. Assuming that protons and neutrons have equal TYPE m. RADIOACTIVE


masses; calculate how many times nuclear
matter is denser than water. Take mass of a DECAY LAW, HALF LIFE AND
nucleon = 1-67 x 10"-^ kg AVERAGE LIFE

and /?o = 1-2 X m. [Ans. 2-307 x 10^^*] 14. Calculate the half life period of a radioactive
5. What is the nuclear radius of if that of
is 3-6 fermi. (CBSE 2008) [Ans. 6 fermi] substance if its activity drops to — th of its
16
6. The nuclear radius of is 3 X 10"*^ m. What
initial value in 30 years.
is its nuclear mass density ?
tCBSE Sample Paper 2011) [Ans. 7-5 years]
[Ans. 2-359X 10*"^ kg/m^] IS. The half life of a radiactive substance is 20 s.

ooww
TYPE II. MASS DEFECT AND Calculate (/) the dacay constant, and (//) time
NUCLEAR BINDING ENERGY
taken by the sample to decay by 7/8th of its initial
value. (CBSE 2009) [Ans. 0-0346 ; 60 s]
7. The binding energy of loNe^^ is 160-6 MeV. Find 16. A radioactive isotope has a half life of 5 years.
its atomic mass. Take mass of proton = 1 -007825 u How long will it take the activity to reduce to

e
and mass of neutron = 1-008665 u. 3-125% ? (CBSE 2008) [Ans. 25 yr]

ree
rFl [Ans. 19-9924 m] 17. Half life of a certain radioactive material is 100

Fre
days. After how much time the undecayed fraction
8. The binding energies of deuteron {^H^) and of material will be 6-25% ?

rrF
alpha particle are 1-25 and 7-2 MeV/ [Ans. 400 days]
nucleon respectively. Which nucleus is more 18. Half life of a sample is 20 minutes. In how much
time will the activity of sample drop to 1/16 of
ouur
stable ? Calculate binding energy per nucleon of
2^Fe^^.m (2f,Fe^^) = 55-934939 amu, m (proton)
= 1-007825 amu, m (neutron) = 1-008665 a.m.u.
sffoo its initial value ? (Bihar Board 2012)
[Ans. 80 min]
okks
[Ans. Helium : 8-79 MeV/nucIeon] 19. The half life of a given radioactive nuclide is
Yo
138-6 days. What is the mean life of this nuclide ?
ooo

9. What energy is needed to split an alpha particle


After how much time will a given sample of this
eBB

to pieces ? Given mass of a particle is 4.0039


nuclide get reduced to only 12-5% of its initial
a.m.u., mass of proton = 1 -007825 u and mass of value ? ICBSE (OD), 2004 (C)j
neutron = 1.008665 u. Use 1 a.m.u. = 931.5 MeV
uurr

[Ans. 27.09 MeV] [Ans. 199-58 days ; 415-8 days]


20. The decay constantfor a radio nuclide, has a value
ad

10. The binding energy per nucleon for is 7-68


YYo

MeV/N and that for gC'^ is 7-47 MeV/N. of 1 -386 day"^. After how much time will a given
Calculate the energy required to remove a neutron sample of this nuclide get reduced to only 6-25%
dd

from (Raj. Board 2011) [Ans. 4-95 MeV] of its present number ? (CBSE (OD), 2004)
Re
iinn

11. The mass of nucleus of is 34.9800 a.m.u. [Ans. 2 days]


Calculate the average energy required to extract
F

a nucleon from this nucleus. Use 1 a.m.u. = 931.5 TYPE IV. ACTIVITY OF A
MeV. Given mass of proton = 1-007825 u, and RADIOACTIVE SUBSTANCE
mass of neutron = 1-008665 u.
21. The half life of a radioactive substance is 5 x 10^
(CBSE 2011) [Ans. 8-22 MeV]
years. In how many years will its activity decay to
12. Calculate the binding energy per nucleon in the 0-2 times its initial activity ? Take log,g 5 = 0-6990.
nuclei of Given m (,5?^^) = 30-97376 u
(Pb. Board 2003) [Ans. 11615 years]
m (gn*) = 1-00865 u ; m (|//1) = 1-00782 u. 22. The count rate of a radioactive sample falls from
(CBSE (OD), 2000) [Ans. 8-47 MeV/N] 4 X lO'’ to 1-0 X 10^ s~^ in 20 hours. What
13. A nucleus of mass number 240 and having B.EJ will be the count rate 100 hours after the
nuclear 7-6 MeV splits into two fragments Y, Z beginning ? [Ans. 3-91 X 10^ s~^]
of mass numbers 110 and 130 respectively. If the 23. A sample of radioactive material has an activity
5£/nucleon of Y, Z is equal to 8-5 MeV each, of 9 X lO’^ Becquerel. The material has a half
calculate the energy released in the nuclear life of 80 s. How long will it take for the activity
reaction. [CBSE 2017 (C)] [Ans. 216 MeV] to fall to 2 X 10*^ Becquerel ? [Ans. 173-6 s]
13/62 Fundamental Physics (XII)iziaiaii

24. A radio isotope with atomic weight 99 has a half 32. One MeV position encounters 1 MeV electron
life of 6 hours. A solution containing 10"'^ g of travelling in opposite direction. What is the
this isotope is injected into the bladder of a wavelength of photons produced ? Given rest
patient. Find its activity at the beginning and at mass energy of electron or positron = 0-512
the end of one hour. and h = 6-62 x Js. [Ans. 8-2 x m]
[Ans. 7-026 x 10* hr"* ; 6-26 x 10* hH] 33. A deuteron strikes nucleus with the
subsequent emission of an alpha particle. Find
25. For a given sample, the counting rate is 47-5 a the atomic number, mass number and chemical
particles per minute. After 5 minute, the count is
name of the element so produced. [Ans.
reduced to 27 a particles per minute. Find the 10
34. A neutron strikes nucleus with the
decay constant and half life of the sample.
subsequent emission of an alpha particle. What
[Ans. 0-0019 s-^ 368-12 s] is the atomic number, mass number and chemical
26. The selling rate of a radioactive isotope is decided name of the remaining nucleus ? [Ans.

w
by its activity. What will be the second hand rate 35. An isotope decays successively to form
of a one month old sample with half life
and
= 14-3 days if it was originally purchased for 800 90

Flo
[Ans. Rs. 182-8] What are the radiations emitted in these five
rupees ?
steps ? [Ans. a, p, p, a, a]

e
27. One milligram of thorium emits 22 a particles

rree
per minute per unit solid angle. Calculate average TYPEVn. NUCLEAR FISSION
life of thorium. Atomic weight of Thorium is 232.

r FF
AND NUCLEAR FUSION
[Ans. 1-783 x 10^® years]
uurr
36. How much Urianium should be consumed
28. A radioactive sample contains 2-2 mg of pure
which has half life of 1224 s. Calculate
(0 the number of atoms present initially (») the
for
per day in a nuclear reactor, for giving power of
1 MW ? Given energy released per fission is
kss
activity when 5 |Xg of the sample will be left ? 200 MeV. [Ans. 1-0575 g]
ooook
Yo
(CBSE (OD), 2005) 37. Calaculate the energy released by the fission of
[Ans. 1-2 X 10^“ ; 1-55 x 10*“* disintegrations/sec] 1 g of in kwh. Energy per fission is
eB

200 MeV. [Ans. 2-278 x 10“* kWh]


TYPE V. a, P AND Y DECAY 38. When undergoes fission, 0-1% of the
urr

original mass is released into energy. How much


29. How many alpha and beta particles are emitted
ad

energy is released by an atom bomb which


Yo

when 92 decays to ? contains 10 kg of [Ans. 9 x lO*"* J]


[Ans. 8 a and 6 P]
dY

30. Calculate the maximum energy that a P particle TYPE Vm. TYPICAL PROBLEMS
Re
innd

can have in the followingdecay ;


39. The activity of a radioactive sample falls from
8o
19
■> 9^*^ +-1^*’ + V 600 s”’ to 500 s"’ in 40 minutes. Calculate its
Fi

Given, m = 19-003576 u, half life. [Ans. 152 min.]


m(9f*^)= 18-998403 u, 40. A radioactive nucleus can decay by two different
m LifO) = 0-000549 u [Ans. 4-3 MeV] processes. The half-life for the first process is fj
and that for the second process is ^2- Show that
TYPE VI. NUCLEAR REACTIONS
the effectivehalf life t of the nucleus is given by
1-1 1
31. When a deuteron of mass 2-0141 a.m.u. and
t (| I2
negligible K.E. is absorbed by a Lithium (3Li^) 41. Calculate packing fraction of a particle from the
nucleus of mass 6-0155'a.m.u. the compound
following data :
nucleus disintegrates spontaneously into two
m, = 4-0028 a.m.u. :
alpha particles, each of mass 4-0026 a.m.u.
Calculate the energy carried by each a particle. nip = 1-00758 a.m.u.
(CBSE 2004) and /«, = 1-00897 a.m.u,

[Ans. 1-8225 x 10
-12
J] [Ans. 7-58 x 10 ^ a.m.uJN]
NUCLEI 13/63

42. A/1 and M2 represent the masses of jQiVe


20
45. Prove that the instantaneous rate of change of
nucleus and 20^°^ nucleus respectively. State activity of a radioactive substance is inversely
whether A^2 = 2 Af, or > 2 Af, or A/^ < 2 A/, proportional to the square of its half life.
[Ans. M2> 2 M[]
(CBSE Sample Paper 2011)
43. Calculate the energy of the following nuclear
reaction : 46. The nucleus of an atom of initially at rest,
I H-^ + 1
decays by emitting an alpha particle. The binding
Given; m = 2-014102 u energy per nucleon of parent and daughter nuclei
are 7-8 MeV and7-835 MeV respectively and that
m 3-016049 u
of a particle is 7-07 MeV/nucleon. Assuming the
m = 4-002603 u
daughter nucleus to be fonned in the unexcited
m (o«*) = 1-008665 u (CBSE 2015) ●State and neglecting its share of energy in the

ww
[Ans. 17-58 MeV] reaction, calculate speed of emitted alpha particle.
44. One milligram of thorium emits 22 alpha particles Take mass of a particle to be 6-68 x 10”-^ kg.
per minute per unit solid angle. Calculate average

FF loo
(CBSE Sample Paper 2013)
life of throrium. Atomic weight of thorium is 232.
[Ans. 1-786 x 10^® years} [Ans. 1-573 x 10^ m/s}

ree
rFee
oor rF
rur
For Difficult Problems s ff
1. Each atom of contains 6 protons, 8 neutrons 5. Here, A, = 27, r, = 3-6 fermi, A2 =125, r-, = ?
and 6 electrons and there are 6 x 10^-^ such atoms
k
YYoou
in 14 gm. of Carbon 14. 4/3
h_{\ 4/3 5
okos

125
As
BBoo

3m 3xl-007825xl-66xl0"2'^
2. p =
I 27 3
4ti/?,^0 22
4x—(1-2x10“'5)3
re

7 ’ 5
/-■, =-.r =-x3-6 = 6-0 fermi
= 2-295 X lO^"^ kg nr^ 2 3 ^ 3
ouur
ad

3. £j = c-2 = 9-1 X 10“^* (3 x lO^)^ J 6. Here, /? = 3 x 10"'^ m, p = ?


Yo

Mass, m = 16 amu = 16 x 1-66 x 10“-^ kg


_ 81-9x10-^^
MeV = 0-511 MeV
Yd

in m 3m
1-6x10-^3
Re
idn

Ej = c- = 1-673 x lO^^? (3 ,q8j2 j


FFin

_ 1-673x9x10^’! 3x16x1-66x10-2'^
MeV = 94M MeV = 2-359 X 10*^ kg/m^
1-6xI0-'3 4x3-14 (3x10-’^)^
4. Density of nucleus (of water) 7. Total BE = 160-6 MeV
160-6
3 m 3x1-67x10--'^ Mass defect (Am) = = 0-17250 a.m.u.
P = 931
471/?.^0 "52
4x —(1-2x10"'^)^ 10 Ne“® has 10 protons and 10 neutrons.
7
Am = 10 X 1 -007825 +10x 1 -008665 - M Nc
7x3x1-67x10’’^ M^^ = 10-07825 + 10-08665 - 0-17250
88x1-2x1-2x1-2
= 2-307 X lO'^kg/m"-* = 19-9924 u

9. An alpha particle is made up of 2 protons and 2


Density of water, p' = 10^ kg/m^ neutrons.

p _ 2-307x10’'^ = 2-307x10^“* 10. Total B.E. of^C‘“= 12 x 7-68 = 92-16 MeV


10-^ Total B.E. of6C'3= 13 x 7-47 = 97-11 MeV
13/64 ‘P'uideefr Fundamental Physics (XII) VOL.Il

As has one excess neutron than t


n = 3 = —
/. Energy required to remove a neutron T
= 9711 -92-16 =4-95MeV r = 3 r = 3 X 138-6 = 415-8 days
12. In number of protons = 15 20. Here, X = 1 -386 day"‘, t=l
number of neutrons = 31-15 = 16 N
= 6-25%
Mass defect,
^0
A m = 15 X 1-00782 + 16 x 1-00865 - 30-97376
0-693 0-693 _ 1
= 15-1173 + 16-1384-30-97376 Half life T = - = 0-5 day
X " 1-386 2
= 0-28194 amu

0-28194x931
B.E./nucleon = = 8-47 MeV/N
No UJ liooj 16 ^2)

ww
31

= 4 = ^, r = 47’ = 4x0-5 =2days


13. Here, n

Energy released = 110 x 8-5 + 130 x 8-5


- 240 X 7-6 21. Here, 7= 5 x 10^ years ; f = ?

Flo
= 935 + 1105 - 1824 = 216 MeV A
— = 0-2 = - 1 ^ XN

e
N 1
A) 5~XNr0

e
14. T=?, = —, f = 30 years

reer
16’

rFF
0 0-693
logg 1 - logg 5 = - X r = -
N fl 1 T
uur r
As
N
0
16
ffoor
0 - 2-303 X 0-6990 = -7—7
5x10^
f
0-693
sks
n = 4 =-, T = - = — =7-5 years
r 4 4
2-303 x 0-6990 x 5x10^
YYoo
ooko

N 7 1 t = years
15. Here,r=20s,X=?/=?; = 1- 0-693
’ ,N0 8 8
eBB

= 11615 years
X =
0-693 0-693
= 00346 S-* 22. Here, Aq = 4 x 10^ s'* ; A = 1-0 x 10^ s~\
20 f = 20hr.
uurr

T
ad

fl T 1 V* 1-0x10®
— = f-
N
Yo

As As = 4 n =2
iV,0 V 2J 8
Aq [2) 4-0x10®
dY

n = 3, r = 37’=3x20s = 60s 20
Re

As n = t/T T = - — = 10 hours
N 3-125
innd

n 2
16. Here, t = 2,T= 5 years
’ N,0 100
FFi

t' 100
= 10
\n S.1IT
Again, n = —
T 10
N (I
As xlO
N,0 \2) \2)
r 1
1 n
^' = A) = 4x10® -
2; UJ
3-125 ^ ^ nV = 3-91 X Id? s"^
100 32
dN N 2x10*2 2
23. Activity ocN
- = 5,f = 5T = 5x5yr = 25yr dt N.0 9x10*2 9
T
Y*
N 12-5 N fl
19. Here, T= 138-6 days, t = ? r = ? From
N,0 100 N.
0 I2j
X = 1.44 T = 1-44 X 138-6 days = 199*58 days N 2
n (log 1 - log 2) = log
N.0 9
N f 1 T _ 12-5 _ 1 it
J J “ 100 “ 8 UJ
From = (log 2 - log 9)
NUCLEI 13/65

n (0 - 0-3010) = (0-3010 - 0-9542) dN 1


Use = XN and t = - minute
n = 0-6532-, 17 dt
0-3010
28. Here, number of atoms present in 11 gram of
From t = / = 2-17 X 80 = 173-6 s
sample = 6-023 x 10^
TT ^ 1 0-693 0-693 hr"* No. of atoms present in 2-2 mg of sample
24. Here, T = 6 hrs., ^ = =-
T 6 initially
No. of atoms in 10~*2 g at t = 0 6023x1023x2-2x10-3
6-023x1023 N.
0 “
= 1-2 X 1020
N xl0-*2 =6-08x 10^ 11
0 “
99
No. of atoms present in 5 pg
Activity of the sample at t = 0
6-023x1023x5x10-^
(dN\ 0-693 A = = 2-74 X 10^“^

llowow
X 6-08 X10^ hr-l 11
V dt ^0
Activity of the sample
= 7-026 X 10* hr-l
0-693.. 0-693x2-74x101’^
From
N / j y/r / j \l/6 A = XN =
T
N
1224
N, 2
A = 1-55 X 10^'* disintegrations/sec.

ee
”0 v*-/
calculate A^= 5-423 x 10^ 29. Let X be the number of a particles and y be the

Fr
0-693
r FF number of P particles emitted in the disintegration
= XN = X 5-423x10^
[dt 6 process of to We may write

r
A
i/238
= 6-26xl0*hi-‘ 92
forfFore
u
From law of conservation of mass number
25. Here, Nq corresponds to 47-5 and N corresponds
s
to 27. 238 = 206+ 4 jc + y (0) = 206-1-4 jc
ok
YYour o
26. Here, T = 14-3 days, f = 30 days 4a: = 238-206 = 32;a: = 8
o

0-693 0-693 -1 From law of conservation of charge number,


X= day
92 = 82-t-2;c-y
eeBoBks

T 14-3

As A=Aq€-^‘ y = 82-92 + 2jc = -10 + 2jc


r

-30x0-693
y = - 10 + 2 (8) = 6
our u

A
ad

14-3
= e _ g-1-454 Hence 8a particles and 6P particles are emitted.
*’ A)
Yo

31. The nuclear reaction involved is


A
log^ — = -1-454 log e = -1-454 3Li^ + iH2 -> 2He'l + 2He'l + Q.
d

A)
Re

Total initial mass = 6-0155 + 2-0141


in

2-3031ogjoA = _i.454 = 8-0296 a.m.u.


FFind Y

A) Total final mass = 2 x 4-0026 = 8-0052 a.m.u.


1-454 Mass defect. Am = 8-0296 - 8-0052
= -0-631 = 1-369
2-303 = 0-0244 a.m.u.

Energy released = (Am) c2


— =0-2285
= [0-0244 X 1-66 X 10-2^] (3 x 10*)2j
A)
= 3-645 X 10-12 J
Sale price = 0-2285 x 800
.-. Energy carried by each a particle
= 182*8 rupees
3-645
27. Number of atoms in 1 milligram of thorium X10-12 J = 1-8225 X 10-12 J
2
6-023x1023
N = XlO-3 32. The nuclear reaction is
232
dN
+l^° + _ieO->2Y
= 22 X 4 7t alpha particles/minute. Total energy before reaction = 1-512 x 2 MeV
dt

i
13/66 7^n,adee^ Fundamental Physics (XII) VOL.II

After reaction, energy of each photon,


^ log, (6/5)
1-512x2 t
E = MeV
Half life, ^
T =0-693 0-693xr
2
he X log,(6/5)
E= 1-512 X 1-6 xl0-*3j = Y 0-693x40
= 152min
r =
he 6-6x10-3^x3x10^ 2-303x0-0792
X =
1-512x1-6x10-13 “ 1-512x1-6x10-13 40. Decay constant for the first process is
= 8*2 X 10-13 m 0-693
X,1 =
33. + ,h2 ■»
2^^ + 2//e^ t
1
Conservation of mass number :
0-693
A-f4=16 + 2=18,A=18-4=14 and for the second process ^2 =

ww
h
Conservation of charge number :
Z+2 = 8-i-1 =9 The probability that an active nucleus decays by
Z=9-2=7 the first process in small time dt is Xj dt. Similarly,

FF loo
Element is
the probability that it decays by the second
process is dt. The probability that it decays
36. Total energy/day = 10^ x 24 x 60 x 60 J

ree
either by first process or by second process is
Energy released per fission = 200 MeV (A,j dt + X2 dt). If effective decay constant is A,

reeF
= 200 X 1-6 X 10-13 J this probability is also equal to X dt.
Number of fissions/day X dt — X.J dt + A.2 tit

oroFr
r ur
10^ x24 x 60 x 60 _ 24x36x1Q1^ s ff X — + A^2
0-693 0-693 0-693
" 200x1-6x10-13 “ 3-2 1-1 1
/. Amount of consumed
t
'2 t ~ t1 ^2
k
YYouo
which was to be proved.
koso

235 24 x 36x101^
X g = 1*0575 g 41. An alpha particle contains two protons and two
6x10^3 3-2
BBoo

neutrons.
37. Number of atoms in 1 gram of
r ee

Mass excess. Am = 2 2 mn - m
a

_ Avogadro’s number _ 6-023x 10^3 = 2 X 1-00758 + 2 X 1-00897 - 4-0028


Mass number 235
ad

= 0-0303 a.m.u.
ouur
Yo

Energy released per fission = 200 MeV. Am 0-0303


Packing fraction = —
Total energy released in fission of 1 gram of A 4
U235 = 7*58 X 10-3 a.m.u7N
d
Re

92
idnY

6-023x1033 42. Let nip be the mass of a proton and m„ be the


mass of a neutron. As some energy is required to
FFin

X 200 MeV
235 bind a nucleus, the mass of the nucleus is always
= 5-126 X 1033 MeV less than the mass of constituent nucleons.
Therefore,
As 1 kWh = 3-6 X 10^ J,
.'. Total energy released Mj < 10 {nip + m„)
5-126x1Q33x1-6x10-13
and
M2 < 20 (nip + m„)
As BE/nucleon of 20^"^ more ±an that of
3-6x10^ the factor
= 2*278 X 10^ kWh
39. FromA = A^e
-Xi M2-20(m^+m„) M,-10(m^+m„)
-Xt 40 20
500 = 600 e

600 40

500
or M2 - 20 (nip + m„) > 20 [M^ - 10 (tHp + m,,)]
X t log, e = log, (6/5) or M2 > 2 Mj

I
NUCLEI 13/67

43. In the given nuclear reaction, Differentiating w.r.t. t


Mass of reactants = m ( +m(
^ = ^ON) = X
dN
= 2014102 + 3016049 (0
dt dt dt
= 5030151 u

Mass of products = m (2//^'*) + m (gn*) = =XNa(-X)e


-h

= 4002603 + 1 008665
= 5 011268 u 0

Mass defect, A w = 5 030151 - 5 011268


— = X^N (leaving neg. sign)
= 0 018883 u dt

Energy released = 0-018883 x 931 MeV


= 17-58 MeV
As X=
T

w
44. Here, m = 1 milligram = 10“^ g
t = ?A = 232
■■
dA
dt
_ 1^0-693 f ^ ^ (0-693)^ N
Number of atoms in 1 mg of Th y T t2

Flo
6-023 xlO^^ dA I
N = xlO"^ =2-595 X 10^® Hence,

ee
^ which was to be proved.
— OC
232
dt
As a particles are emitted by the source in all the

Fr
46. The given reaction is
directions, Le., in a solid angle of 4 71: steradian.
92
y235 -» + 2He"* + energy
dN
therefore.

for
= 22 X 4ti a particles/minute
ur
dt Energy released during the decay process is
E = 7-835 X 231 4 X 7-07 - 7-8 x 235
dN
= XN
ks
From E = 1809-885 + 28-28 - 1833-3 = 5-165 MeV
dt
Yo
= 5-165 X l-6x 10-'3 J
oo

x = dNIdt _ 22x471
N ~ 2-595x10** If V is speed of alpha particle emitted, then
eB

1 2-595x10** 1
x = —
minutes - mv^ =E
X 22x471 2
ur
ad

9-39x10*^
Yo

year 2x5165x1-6x10-*^
60x24x365 v =
6-68x10-2'^
T = 1-786 X 10*® years
d

45. If a radiactive substance contains V atoms at any


Re

/
in

16-528
time t, its activity is xlO^ m/s
6-68
F

A = ^ = ;uv V = 1-573 X 10’ m/s


dt
13/68 7^^Ktdceft-'4. Fundamental Physics (XII)EEIHII

WITH
7 EE SOLUTIONS
!■!

You may find the following data useful in solving the exercises :
e=\-6x l0-‘^C;A^=6 O23x per mole
1

ooww
= 9x lO^Nm-ZC^; /:= 1-38 x 10-^3 JK"'
47ce
0

1 MeV = 1-6 X 10"'^ J ; 1 u = 931 MeV ; 1 year = 3-154 x lO’^s ; m (2He'*) = 4-002603 u
= 1-007825 u ; = 0-000548 u ; m= 1-008665 u

Q. 1. Obtain the binding energy (in MeV) of a nitrogen nucleus Given, m = 14*00307 u

e
re
Sol. nucleus contains 7 protons and 7 neutrons.

rFFl
Mass defect (A m) = 7 m^y -f 7 m,, - = 7 x 1-00783 -f 7 x 1-00867 - 14-00307

ree
F
= 7-05481 + 7-06069 - 14-00307 = 0-11243 u.

rF
Binding energy = 0-11243 x 931 MeV = 104-67 MeV
Q. 2. Obtain the binding energy of the nuclei and in units of MeV from the following data :

fsfoor
ouur
m = 55*934939 a.m.u. ; m = 208-980388 a.m.u.
Which nucleus has greater binding energy per nucleon ? Take 1 a.m.u. = 931*5 MeV
kosk
Sol. (0 26^^ nucleus contains 26 protons and (56 - 26) = 30 neutrons
Yo
Mass of 26 protons = 26 x 1-007825 = 26-20345 a.m.u.
oo
Y

Mass of 30 neutrons = 30 x 1-008665 = 30-25995 a.m.u.


BB

Total mass of 56 nucleons = 56-46340 a.m.u.


rre

Mass of 26^^^^ nucleus = 55-934939 a.m.u.


.■. Mass defect, A/jj = 56-46340 — 55-934939 = 0-528461 a.m.u.
oYuu
ad

Total binding energy = 0-528461 x 931 -5 MeV = 492.26 MeV


dY

492-26
Av. B.E. per nucleon = = 8*790 MeV
56
innd
Re

.-209
(«) 83^/ nucleus contains 83 protons and (209 - 83) = 126 neutrons.
Mass of 83 protons = 83 x 1-007825 = 83-649475 a.m.u.
Fi
F

Mass of 126 neutrons = 126 x 1-008665 = 127-091790 a.m.u.

Total mass of nucleons = 210-741260 a.m.u.

Mass of g3fir^ nucleus = 208-980388 a.m.u.


Mass defect. Am = 210-741260 - 208-980388 = 1-760872
Total B.E. = 1-760872 x 931-5 MeV = 1640-26 MeV.
1640-26
Av. B.E. per nucleon = = 7*848MeV
209
Hence has greater B.E. per nucleon than 83 5,-209
Q. 3. A given coin has a mass of 3*0 g. Calculate the nuclear energy that would be required to separate all
the neutrons and protons from each other. For simplicity, assume that the coin is entirely made of
29 Cu^ atoms (of mass 62*92960 u).
6-023x1023x3
Sol. Number of atoms in 3 g coin = = 2-868 X 1Q22
63
NUCLEI 13/69

Each atom of copper contains 29 protons and 34 neutrons. Therefore, mass defect of each atom
= [29 X 1-00783 + 34 X 1-00867] - 62-92960 = 0-59225 u
Total mass defect for all the atoms = 0-59225 x 2-868 x 10^^ u
Am = 1-6985 x 10^2 u
Nuclear energy required = 1-6985 x 10^2 x 931 MeV = 1*58 x 1025 MeV

w
As 1 « = 931 MeV,
Q.4. Obtain approximately the ratio of the nuclear radii of the gold isotope and silver isotope

Sol. Here, A, = 197 and ^2 = 107

e
nI/3 xl/3
(\91
= 1-225

e
wr
^ [A) {107)

oo
r
Q.5. The Q value of a nuclear reaction
A + b = C+d is defined by g = [m^ + m* c2 where the masses refer to the respective nuclei.

F
FFllu
Determine from the given data the Q value of the following reactions and state whether the reactions
are exothermic or endothermic.
0)
(n) + ^e*

rese
Atomic masses are given to be
uro
m (1^2) = 2-014102 u ; m = 3-016049 u ; m i^C^) = 12-000000 u ; m

Fkr
= 19-992439 u

o
Sol. (0 + i//2

oo
g = A m X 931 MeV = [m -i- m -2m (i//2)] x 931 MeV

fr
= [1-007825 + 3-016049 - 2 x 2-014102] x 931 MeV = - 4-03 MeV
Y
The reaction is endothermic
of
(«●) gC‘2 -f gC‘2 ^ j(jV^20 +
ks
Y
B
Yo
g = A m X 931 MeV = [2 m (gd2) _ ^ - m (2/Ze'^)] x 931 MeV
oo

= [24-000000 - 19-992439 - 4-002603] x 931 MeV = 4-61 MeV


eBr

The reaction is exothermic.


rue

Q.6. Suppose, we think of Hssion of a 26/^e^^ nucleus into two equal fragments 13^/2^. Is the Hssion
oud

energetically possible ? Argue by working out g of the process. Given


m (26^«^) = 55-93494 u, m = 27-98191 u.
no
ad

Sol. g = [m (26pe^^) -2m (,3^/28)] x 931-5 MeV = [55-93494 - 2 x 27-9819] x 931-5 MeV
Y

g = - 0-02886 X 931-5 MeV = - 26-88 MeV, which is negative.


ndi

The fission is not possible energetically.


Re

The fission properties of 34^^^ are very similar to those of


F

Q.7. The average energy released per


fission is 180 MeV. How much energy in MeV is released if all the atoms in 1 kg of pure
Fi

undergo fission.
6-023x1023
Sol. No. of atoms in 1 kg of pure P«239 = xl000 = 2-52x1024
239
As average energy released/fission is 180 MeV, therefore, total energy released
= 2-52 X 1024 X 180 MeV = 4-53 x 102<^ MeV
Q.8. How long can an electric lamp of 100 W be kept glowing by fusion of 2-0 kg of deuterium ? The
fiision reaction can be taken as -♦- jff2 _> -♦-« -f 3-27 MeV
6-023x1023x2000
Sol. Number of deuterium atoms in 2-0 kg = = 6-023 X 1026
2
Energy released when 2 atoms fuse = 3-27 MeV
3-27
Total energy released = — X 6-023 X 1026 MeV = 1-635 x 6-023 x 1026‘x 1-6 x 10-^3 j
= 15-75 X 10*3 J
13A70 4^ Fundamental Physics (XII) iSslBD
Energy consumed by the bulb/sec = 100 J
15-75x1013 15-75 X10* 1
Hme for which bulb will glow = s = years = 4*99 x lO'* years
100 60 x 60 x 24 x 365
Q.9. Calculate the height of potential barrier for a head-on collision of two deuterons. The effective
radius of deuteron can be taken to be 2 fm. Note that height of potential barrier is given by the
Coulomb repulsion between two deuterons when they just touch eachother.
Sol. For head on collision, distance between centres of two deuterons = r = 2 x radius
r = 4fm = 4 X 10“i3ni
charge of each deuteron e = 1 -6 x 10“i^ C
^2 9x10’(1-6x10-19)2 9x1-6x1-6x10-14
Potential energy = joule = keV = 360keV
4jie«0 r 4x10-13 4x1-6x10-16

ww
360
As P.E. = 2 X K.E. of each deuteron = 360 keV K.E. of each deuteron = = 180 keV
2
This is a measure of height of Coulomb barrier.

FF loo
Q. 10. From the relation R - where is a constant and A is the mass number of a nucleus, show
that the nuclear matter density is nearly constant (i.e., independent of A).

ree
Sol. Refer to Art. 13.5.

reFe
oroFr
r ur
k s ff
YYouo
koso
BBoo
r ee
ad
ouur
Yo
d
Re
idnY
FFin

!1
NUCLEI 13/71

WITH ANSWERS,
HINTS AND SOLUTIONS

n Imi Ei
wm t
L m
MULTIPLE CHOICE QUESTIONS-I

1. Suppose we consider a large number of (a) ej=(M,-M,.)c2


containers each containing initially 10000 and =

ww
atoms of a radioactive material with a half
(b)
life of 1 year. After 1 year,
(fl) all the containers will have 5000 atoms of and Q2 = (M^~My)c~
the material (c)

FF loo
(b) all the containers will contain the same and Qo = {M^-M^.+ 2m^)c~
number of atoms of the material but that uo Q]={M,-My+2m,)

ree
number will only be approximately 5000 and Qj = M^.+ 2 ni^,) c~
(c) the containers will in general have different 5. Tritium is an isotope of hydrogen whose
nucleus Triton contains 2 neutrons and 1

reFe
numbers of the atoms of the material but their
average will be close to 5000 proton. Free neutrons decay into p+e + v .

oor rF
rur
(d) none of the containers can have more than If one of the neutrons in Triton decays, it
would transform into He^ nucleus. This does
5000 atoms
s ff
2. The gravitational force between a H>atom and not happen. This is because
another particle of mass m will be given by (a) Triton energy is less than that of a He-^ nucleus
k
YYoou
Newton’s law ;
okso

(b) the electron created in the beta decay process


M.m can not remain in the nucleus
BBoo

F =G , where r is in km and (c) both the neutrons in triton have to decay


simultaneously resulting in a nucleus with 3
r ee

(a) M = m proton + m
electron protons, which is not a He^ nucleus
B (cf) because free neutrons decay due to external
ouur
ad

(b) M = m proton + m
electron —^ {B= 13-6 eV) perturbations which is absent in a triton
Yo

C"
nucleus
(r) M is not related to the mass of the hydrogen 6. Heavy stable nuclei have more neutrons than
atom
protons. This is because of the fact that
d
idnY
Re

I V I (a) neutrons are heavier than protons


{d) M = m proton + m
electron -ili {\V\) =
(b) electrostatic force between protons arc
FFin

c“

magnitude of the potential energy of electron repulsive


in the H-atom) (c) neutrons decay into protons through beta decay
3. When u nucleus in an atom undergoes a (cl) nuclear forces between neutrons are weaker
radioactive decay, the electronic energy levels than that between protons
of the atom 7. In a nuclear reactor, moderators slow down
the neutrons which come out in a fission
(a) do not change for any type of radioactivity
(b) change for a and j3 radioactivity but not for process. The moderator used have light nuclei.
y-radioaciivity Heavy nuclei will not serve the purpose
because
(c) change for a-radioaclivity but not for others
(rt) they will break up
(d) change for ^-radioactivity but not for others
4. and M denote the atomic masses of the (b) elastic collision of neutrons with heavy nuclei
will not slow them down
parent and the daughter nuclei respectively
in a radioactive decay. The g-value for a P- (c) the net weight of the reactor would be
decay is and that for a decay is Qi* unbearably high
denotes the mass of an electron, then which {cl) substances with heavy nuclei do not occur in
of the following statements is correct ? liquid or gaseous stale at room temperature

I
13/72 T^'icidce^'4, Fundamental Physics (XII)QSBII

MULTIPLE CHOICE QUESTIONS-II


8. Fusion processes, like combining two 10. The variation of decay rate of two radioactive
deuterons to form a He nucleus are impossible samples .A and B with time is shown in Fig.
at ordinary temperatures and pressure. The 13(N).l.
reasons for this can be traced to the fact :

(a) nuclear forces have short range


(b) nuclei are positively charged
(c) the original nuclei must be completely
ionized before fusion can take place
(d) the original nuclei must first break up before
combining with each other

ww
9. Samples of two radioactive nuclides A and B
are taken. and arc the disintegration
constants of A and B respectively. In which of
Which of the following statements are true ?

Floo
the following cases, the two samples can
simultaeously have the same decay rate at any (fl) Decay constant of A is greater than that of 6,

e
time ? hence A always decays faster than B

eere
(a) initial rate of decay of A is twice the initial (b) Decay constant of B is greater than that of A
rale of decay of B and but its decay rate is always smaller than that

FFr
of A
(b) Initial rate of decay of A is twice the initial

oorr
uur r
(c) Decay constant of A is greater than that of B
rate of decay of B and > Xf^
but it does not always decay faster than B
(c) Initial rate of decay of B is twice the initial
s ff
rate of decay of A and > Xq (d) Decay constant of B is smaller than that of/I
but still its decay rate becomes equal to that
sk
YYoo
(d) Initial rate of decay of B is same as the rale of /4 at a later instant
ooko

of decay of /4 at / = 2h and X[^ =


eBB

ANSWERS

1. (c) 2. {b) 3. (b) 4. (a) 5. (a) 6. (b) 7. ib) 8. (a, b)


uurr
ad

9. (b. d) 10. (c. d)


Yo

HINTS FOR DIFFICULT MULTIPLE CHOICE QUESTIONS


dY
Re

Multiple Choice Questions -1


nind
FFi

1. Radioactivity is spontaneous self disruptive activity of the radioactive material. In r =1 year = half life of
material : on the average, half the number of atoms will decay. Therefore, the containers will in general
have different number of atoms of the material, but their average will be close to 5000. Choice (c) is
correct.

GMm
2. In the relation f =
r' ’
M = effective mass of hydrogen atom
= mass of proton + mass of electron - B/c^ ; where B is B.E. of hydrogen atom = 13-6 eV.
Choice {b) is correct.
3. As an alpha particle carries 2 units of positive charge, and a beta particle carries one unit of negative charge
and Y (particle) carries no charge, therefore electronic energy levels of the atom change for a and (3 decay,
but not for y-decay. Choice {b) is correct.

f
NUCLEI 13/73

4. p decay is represented as

z + *'+2l
Q\ = [ni^ iz^^) - - m^] c- = [m^ + Zm^ - C^+jF^) - (Z+ I) I
= [m {yX^) - ni (2^1 Y^)] = (M^ - M,)

w
P’^ decay is represented as
zZ^=2_iF’ + ie‘> + v + a2
02 = = [m^(zX'^) + Zm^-mf^(z^iY^)-(Z~ 1) m^-2 mj

e
= [m (zX^) - m (z_i Y^) - 2 «g = (M^-My-2 m,)
.●. Choice (a) is correct.

e
wr
5. Tritium -> |H^. The nucleus contains 1 proton and

lloo
2 neutrons. If one neutron decays n —> /> + c" + v, the

r
nucleus may have 2 protons and one neutron, i.e., tritium will transform into ^He-^ (with 2 protons and one

F
neutron). But this does not happen because triton energy is less than that of 2He^ nucleus, i.e., transformation

u
is not allowed energetically. Choice (c) is correct.

r FF
6. Heavy nuclei, which are stable contain more neutrons than protons in their nuclei. This is because electrostatic
force between protons is repulsive, which may reduce stability. Choice (b) is correct.

rs
ee
7. The moderators used have light nuclei (like proton). When protons undergo perfectly elastic collision with

Fr
uo
the neutrons emitted, their velocities are exchanged, i.e., neutrons come to rest and protons move with the

k
o
velocity of neutrons. Heavy nuclei will not serve the purpose because elastic collisions of neutrons with
heavy nuclei will not slow them down. Choice (h) is correct.

foo
fr
Multiple Choice Questions -11
kso
Y
8. Fusion processes tu'e impossible at ordinary temperatures and pressures. This is because nuclei are positively
B
eBrY

charged and nuclear forces are the strongest forces having short range. Choices (a) and (b) are correct.
Yo
oo

9. The two samples of two radioactive nuclides A and B can simultaneously have the same decay rate at any
time if initial rate of decay of A is twice the initial rate of decay of B and > Xjf.
e

Also, when initial rate of decay of B is same as rate of decay of A at r = 2 /? and Xg < X^. The choices (b)
uru

and (d) are correct.


od

10. From Fig. 8(EP). 1. we find that rate of decay of A is faster than that of B. It means decay constant of A is
Yo
ad

greater than that of B.


n

However, the two curves intersect at P. Beyond ,P B decays faster than A. And at P, decay rate of both A and
B is the same.
ndi

Hence, choices (c) and (d) are correct.


Re
F
Fi

VERY SHORT ANSWER QUESTIONS

11. 2He^ and He^ nuclei have the same mass numbers. Do they have the same binding energy ?
Ans. No. 2He-^ and jHc-^ nuclei have the same mass number, but the binding energy of |Hc^ is greater than that
of 2He-^. This is primarily because ^He-^ has 2p and In, whereas jHe-^ has 1 proton and 2 n. The repulsive
force between protons is missing in ,He-^. FIGURE 13(N).2
12. Draw a graph showing the variation of decay rate with number of
active nuclei.

dN dt
Ans. As - = XN and X is constant for a given radioactive material. O
df
>N
dN
therefore graph between N and is a straight line as shown in Fig.
dt
13(N).2.
Y
13/74 ^ Fundamental Physics (XII) WSIWII
13. Which sample, A or B shown in Fig. 13(N).3 has shorter mean-
life ?

dN \ (dN\
Ans. From Fig. 13(N).3. we find that at r = 0,
dt dt
M JB
Therefore.

dN
<
dN \
At any subsequent time. dt dt
jB \ Ja
Xff Ag < -►

As
Nb < N^ (rate of decay of B is slower)
Xg > X_4
Hence, i.e., mean life time of B i.s shorter than that of A.

w
14. Which one of the following can not emit radiation and why ? Excited nucleus, excited electron.
Ans. Excited nucleu.s can emit radiation, but excited electron cannot.

Flo
This is because energy of electronic energy levels is in the range of electron volt (eV) and not MeV (million
electron volt), y-radiations have energy of the order of Me.V

reee
15. In pair annihilation, an electron and a positron destory each other to produce gamma radiation.
How is the momentum conserved ?

FFr
Ans. When an electron annhilates a position, two y-ray photons are produced, which move in opposite directions.
This leads to con.servation of linear momentum. qC'’’ + qc’ —> 2 y ray photons
urr
SHORT ANSWER QUESTIONS
for
kkss
16. Why do .stable nuclei never have more protons than neutrons ?
Yo
ooo

Ans. As is known, protons are positively charged and repel eachother electrically. In nuclei with more than 10
protons or .so, the force of repulsion becomes too large. Therefore, excess of neutrons is required for
eB

stability. This is because neutrons produce only attractive (nuclear) force amongst them.
17. Consider a radioactive nucleus A which decays to a stable nucleus C through the following sequence :
r

A^B^C
ou
ad

Here, B is an intermediate nucleus which is also radioactive. Considering that there arc A^q atoms of
YY

A initially, plot the graph showing the variation of number of atoms of A and B versus time.
Ans. The given sequence of radioactive decay is
nndd
Re

A^B C
C is stable.
Fi

At t = {), N^= and Ag = 0.


As time passes on, t increases. falls off exponentially
to zero at / = CO. The number of atoms of B goes on
increasing with time, becomes maximum and finally
decays to zero (at t = following exponential decay law.
The variation of the number of atoms of A and B versus
lime is shown in Fig. 13(N).4.
18. A piece of wood from the ruins of an ancient building was found to have a activity of
12 disintegrations per minute per gram of its carbon content. The activity of the living wood is
16 disintegrations per minute per gram. How long ago did the tree, from which the wooden sample
came, die ? Given half-life of is 5760 years.
Ans. Here, 7= 5760 yrs,/ = ?
/?() = 16 disintegrations/minute/gram
R = \2 disintegrations/minute/gram
NUCLEI 13^5

] R T R 5760 16
From 0 0
i -
R
X 2-303 log 10 X 2-303 loseiO
0-693 R 0-693 12

5760 5760x2-303x0-125
t = X 2-303 (0-602-0-477) = yrs = 2392-7 yrs
0-693 0-693

19. Are the nucleons fundamental particles, or do they consist of still smaller parts ? One way to find out is to
probe a nucleon just as Rutherford probed an atom. What should be the kinetic energy of an electron for
it to be able to probe a nucleon ? Assume the diameter of a nucleon to be approximtely l(f m.
Ans. Yes, the nucleons (neutrons and protons) are fundamental particles.
To resolve two objects, say nucleons separated by distance d, the wavelength X. of probing signal must be
less than or equal to d.
Asd= 1(T*^ m, therefore to detect septirate parts, if any. inside a nucleon, the electron must have a wavelength

ww
he 6-63x10"^'^ x3x 10*^
X< 10"'^ m.Now, X~ ~ or p = — and Kinetic energy, K = pc = — joule
KP X io-'5

Flo
19-89x10-'*

e
K = eV = lO^eVslGeV
1-6x10-'^

eree
20. A nuclide 1 is said to be the mirror isobar of nuclide 2 if Zj = N2 and = ^|. (a) What nuclide Is a

FFr
mirror isobar of Na ? (b) Which nuclide out of the two mirror isobars has greater binding energy

oorr
uur r
and why ? sf
Ans. By definition, a nuclide 1 is said to be mirror isobar of nuclide 2, if Z, = and 2^ = Nj.
Now in j,Na2-\ Z, = 11. = 23 - 11 = 12
sk
Yoo
Mirror isobar of jjNa^-^ is j2Mg^^, for which Z-, = 12 and A^2 = 23 - 12 = 11
ooko

As i2Mg"^ contains even number of protons against nNa^^ which has odd number of protons, therefore 12^8^
eBB

has greater binding energy than nNa^


uurr

LONG ANSWER QUESTIONS


ad

21. Sometimes a radioactive nucleus decays into a nucleus which itself is radioactive.An example is :
Yo

38 halt-life halMife
Sulphur ^ ^C1 ^ ^Ar (stable)
dY

=2-48h = 0-62h
Re

Assume that we start with 1000 nuclei at time / = 0. The number of ^Cl is of count zero at f = 0
innd

and will again be zero at r - <». At what value of t, would the number of counts be a maximum ?
FFi

38
Ans. The given decay sequence is Sulphur ^ ^''Chlorine 4 ^''Ar (stable)
r=2-48/i r=0'62/i

At any time t, suppose has (/) active nuclei and have N2 (t) active nuclei.
dN,
di
= -X.| A^i = rate of formation of ^*C1

dNi
and
dt
= -X-2 A^2 + ^1 A'] = net rate of decay of Cl^" = + X.|

Multiplying both sides by e^2‘d! 2nd rearranging: e^2'dN^ + e^2'dt = X., N dt


Integrating both sides, we get =

where C is constant of integration.


13/76 7>nadceft, Fundamental Physics (XII) VOL.II

At t = 0, N2 = 0 C=-

Putting in (i), we get iV2^ -1

^2 = ^0^ rg-v-g-v
Xj-x, L
m
For maximum count; N2 = max, 2. = 0
dt
N^X
on rV
(-^2)

ww
or
x,-x, ^2 A,j Xj e-V
iog,(X,/X2) iog,(rj/r,)
x^ = (Xj-X2) / log^e
or or / =

Flo
(^1-^2) 0-693 f±-l'
T, Tj

e
rree
^ ^ 2-303 log^Q (0 62/ 248) x T2 2 303 (0 - 0-602) x 248 x 0-62

r FF
^ 0-693 (T2-rj) 0-693(0-62 - 2-48)
uurr
2-303 x 0-602 x 2-48 x 0-62

for
22. Deuteron is a bound state of a neutron and a proton with a binding energy B = 2*2 MeV. A y-ray of
0-693x1-86
= 1*65 sec
kss
energy E is aimed at a deuteron nucleus to try to break it into a (neutron -f proton) such that the n
ooook

and p move in the direction of the incident y-ray. If £ = jB, show that this can not happen. Hence,
Yo

calculate how much bigger than B must E be for such a process to happen.
eBB

Ans. Applying principle of conservation of energy in the given situation.

E-B = K„ + Kp = EL+fp ...(0


urr

2m 2m
ad
Yo

From the principle of conservation of linear momentum, p^+Pp = E/c


If E = B, eqn. (/) would give p^^p^ = 0. Hence, the process cannot take place.
dY

For the process to take place, let E = B + X, where X«B.


Re
innd

'-.4.4
1
From (0, using (ii). x =
p2+(p^-E/c)\
Fi

2m L

^£2
2£/c± 4£2/c2_8 --2mX
o 2 2£ (e^
^Pp- — Pp^ -;r-2mX = 0 or Pp - 4

For Pp to be real, the determinant on RHS must be positive.


4 £2 ^£2 4 £2
-8 -r--2/nA, >0 or 16mX =
c2 c2

£^ , B^
4mc2 4mc2
Hence, £ must be bigger than B by X = B^IA m c^, for the given process to happen.
NUCLEI 13/77

23. The deuteron is bound by nuclear forces just as H-atom is made up ofp and e bound by electrostatic
forces. If we consider the force between neutron and proton in deuteron as given in the form of a
/2
1 e
Coulomb potential but with an effective charge e': F =
4ntQ r
estimate the value of (e'/e) given that the binding energy of a deuteron is 2*2 MeV.
4
Ans. The binding energy in ground state of hydrogen atom is E= = 13-6 eV .(0

If proton and neutron (in a deuteron) had charge e' each, and were governed by the same electrostatic force,
then in (/), we replace e and e' and m by m\ the reduced mass of neutron-proton, where
m' = MxM _ M 1836m = 918m

ww
M +M ~ 2 2

918me'4
Here, M represents mass of a neutron/proton. Binding energy, s' = = 2-2 MeV. .(«*)

Flo
0

e
2-2 MeV _ 2-2 xlO^

e
Dividing («) by (/), we get 918

rree
13-6 eV 13-6

r FF
/f 2-2x10^ = 176-21
e 13-6x918
uurr
e'
- = (176-21)1/4 = 3.54
e
for
kss
24. Before the neutrino hypothesis, the beta process was thought to be the transition, n-^p+ e
ooook
Yo
If this was true, show that if the neutron was at rest, the proton and electron would emerge with fixed
energies and calculate them. Experimentally, the electron energy was found to have a large range.
eBB

Ans. We are supporting that p-decay is due to the transition


n-¥p + e~
urr

Suppose before beta decay, neutron is at rest. p„ = 0 and E„ = m„c^


ad

After p-decay, from conservation of linear momentum, we have


Yo
dY

Pn=Pp+Pe=<^ i?pi=ip,i= P, say.


Re
innd

Also,
Fi

and
Ee = (m^ c4 + c2)l/2 _ ^^2 ^4 ^ ^2 (.2)1/2
From conservation of energy, Ep + Eg = E„
i.e..
(m^ + (m^ c4 + ii)
Now, nip c2 = 936 MeV = 938 MeV and = 0-51 MeV
As energy difference between and m„ is small, therefore pc will be small.
pc « nip c^. However, pc may be greater than Therefore, from (/),
2^2
p-c
nipC^~ + - m_c ^-pc
2n,y
n

To first order of approximation, pc= m^c^ -m_c


pC^ = 938 MeV - 936 MeV = 2 MeV
This gives us the momentum of proton or neutron.
13A78 ‘Pnadeefr’^ Fundamental Physics (XII) VOL.II

= V936^ + 2^=936 MeV

and
- {tn^c^ + = ^(0-51)2+22 = 2-06 MeV
25. The activity A of an iinkown radioactive nuclide is measured at hourly intervals. The results found
are tabulated as follows :

0 1 2 3 4
Hh)

100 35-36 12-51 4-42 1-56


R (MBq)

(i) Plot the graph of R versus t and calculate half-life from the graph.
R ^
/

versus t and obtain the value of half-life from the graph.

w
(ii) Plot the graph of In

Ans. In Table 8.1, we have listed R iMBq) and log^ (R/Rq) of a radioactive nuclide at different time intervals.

Flo
TABLE 8.1

reeee
0 1 2 3 4
Hh)

FFr
100 35-36 12-51 442 1-56
R (MBq)

, R
for
ur
- 1-04 -2-08 -3-11 -4-16
kkss
(0 When we plot the graph of R versus t, we obtain an exponential curve as shown in Fig. 13(N).5(a).
Yo
From the graph, we find that activity R reduces to 50% int=OA =40 minutes. This gives us half life of
oo

the nuclide.
eB

(/●/) Fig. 13(N).5(1>) shows the graph of log^ {R/Rq) versus t.


r
ou
ad
YY
ndd
Re
Fi

Slope of this graph = -X.


r-4-16-3-lO = 1-05 h-‘
From the graph, X, = - 1

0-693 0-693
Half life, T = — = 0-66 h = 39-6 min = 40 min.
1-05
NUCLEI 13/79

26. Nuclei with magic no. of proton Z = 2,8,20,28,50,52 and magic no. of neutrons N = 2,8,20,28,50,
82 and 126 are found to be very stable,
(i) Verify this by calculating the proton separation energy Sp for (Z = 50) and Sb = {Z- 51).
The proton separation energy for a nuclide is the minimum energy required to separate the least
tightly bound proton from a nucleus of that nuclide. It is given by
Sp = (Afj.j, ^ + Af^ - Mx^n)
Given l*^Sn = 118*9058 u, ^^Ogn = 119*902199 u , ^^isb = 120*903824 u, = 1*0078252 u.
(ii) What does the existence of magic number indicate ?
Ans. (0 The proton separation energy is given by -(f)
For Sn, ^Z-1, = 118-9058 M, Mff=\ -0078252 u
and M
Z.N = 119-902199 u

ww
From (/), Spsn = (118-9058 + 1-0078252 - 119-902199) <?■ = 0-0114362 (p-
Similarly, SpSb = (^120,70 + ^121.70) = 019-902199 1-0078252 - 120-903824) (P
= 0-0059912 P

Flo
Since Sp^„ > 5^^^, therefore S„ nucleus is more stable than Sb nucleus, which was to be proved.

ee
(h) The existence of magic numbers indicates the shell structure of nucleus similar to the shell structure of

rere
an atom. It also accounts for the peaks in Binding energy/nucleon curve.

rFF
uurr
foor
ks s
Yoo
oook
eBB
uurr
ad
Yo
dY
Re
innd
FFi
13/80 ‘P>vxdee^'4- Fundamental Physics (XII)CS!9Q

I'■'■I

k&i
it
gt

NEET/JEE
SPECIAL

For ultimate preparation of this unit for competitive examinations, students should refer to

ww
● MCQs in Physics for NEET
Pradeep's Stellar Series.... ● MCQs in Physics for JEE (Main) ''

separately available for these examinations.

FF loo
ree
Multiple Choice Questions (with one correct Answer)

rFee
I. Nuclei 3. If a star can convert all the He nuclei completely

F
into oxygen nuclei, the energy released per oxygen

oor r
rur
1. A fission reaction is given by nucleus is [Mass of He nucleus is 4 0026 amu
s ff
and mass of Oxygen nucleus is 15-9994 amu]
236 U
92 Xe + 3^Sr + .v-f y, where x and y (a) 7-6 MeV (b) 56-12 MeV
k
YYoou
(c) 10-24 MeV id) 23-9 MeV
two particles. Considering 92^ U to be at rest,
ookos

are

4. When a slow neutron is captured by a


the kinetic energies of the products are denoted
BBo

nucleus, a fission results which releases 200 MeV


by Kxe, ^sr MeV) energy. If the output of nuclear reactor is 1 -6 MW,
re

respectively. Let the binding energies per nucleon then the number of fissions per second of nuclei
of ^»Xe and Ipr be 7-5 MeV, 8-5 MeV undergoing fission is
ouur
ad

12
(«) 5 X 10“^ ib) 5 X 10
Yo

and 8-5 MeV respectively. Considering different 16


conservation laws, the correct option (5) is (are) ic) 5 X 10*^ id) 5x10

(a) X = 11, y = n, K^j. = 129 MeV, Kj^^. = 86 MeV 5. In the nuclear decay given below
dY
Re

yA ^V Z-1^
rA-4
idn

(b) X = p,y = e~ Kg, = 129 MeV. K^e = 86 MeV ■»


z-fl Z-1
FFin

(c) .V = p,y — ^Sr “ MeV, = 86 MeV The particles emitted in the sequence are ;
{d)x = n. y = /(. Kg^ = 86 MeV. Kj^^. = 129 MeV (a) a, P, Y ib) P, a. Y
(JEE Advanced 2015) ic) y, P, a id) p.Y-O-
2. In the nuclear fusion reaction (AIPMT 2009)
,h2+ |H3->2He-^+»7, 6. Assume that a neutron breaks into a proton and
given that the repulsive potential energy between an electron. The energy released during this
the two nuclei is « 7-7 x 10"'“* J, the temperature process is (Mass of neutron = 1-6725 x 10“22 kg ;
at which the gases must be heated to initiate the mass of proton = 1-6725 x I0“2^ kg ; mass of
reaction is of the order of (given Boltzmcum electron = 9 x 10”3’ kg)
constant k = 1 -38 x 10“2-3 J/K) ia) 0-506 MeV (/;) 7-10 MeV
(fl) lO^K ib) lO^K ic) 6-30 MeV id) 5-4 MeV
ic) 103k id) lO'-’K (AIEEE 2012)

ANSWERS
1. ia) 2. id) 3. ic) 4. UD 5. (/;) 6. (u)
NUCLEI 13/81

7. A nucleus of uranium decays at rest into nuclei (c) 3fiKr>03 {d) 5f,Br‘^
of thorium and helium. Then :
(NKET 2021)
{a) The helium nucleus has less kinetic energy
than the thorium nucleus II. Radioactivity
{b) The helium nucleus has more kinetic energy
than the thorium nucleus 13. A radioactive nucleus -4 with a half life T, decays
into a nucleus S. At f = 0, there is no nucleus B.
(c) The helium nucleus has less momentum than
At sometime t, the ratio of the number of B to
the thorium nucleus
that of A is 0-3. Then, t is given by
{d) The helium nucleus has more momentum than
T
the thorium nucleus (AIPMT 2015) (fl) r = riog (1-3) (h) t =
8. Two spherical nuclei have mass number 216 and log (1-3)
64 with their radii /?] and /?■> respectively. The T log 2 log 1-3
(c) >=- id) l=T

ww
R
i 2 log 1-3 log 2
ratio, is equal to
^2 UEE Main 2017)
ia) 3 : 2 (b) 1 ; 3 14. The compo.sition of uranium ores on the earth at
present is 99-3% of the isotope 92^"-^^ and 0-7%

Flo
ic) 1:2 id) 2:3 (AIIMS 2015)
9. A nuclear power plant supplying electrical power of the isotope 9511-^^. If those isotopes were

e
to a village uses a radioactive material of half life T equally abundant when the earth was formed, then

eree
years as a fuel. The amount of fuel at the beginning what is the age of earth ? (Given that
hn = 4-47 X 10^ years

FFr
is such that the total power requirement of the village
is 12-5% of the electneal power available from the and /j/2 of 95U“^^ = 7-04 x 10^ years.

oorr
uur r
plant at that time. If the plant is able to meet the (a) 4-78 X 10*^ years (b) 5-97 x 10^ years
total power needs of the village for a maximum
sf
period of nT years, then the value of n is (c) 6-78 X 10^ years (d) 7*29 x 10^ years
ia) 1 1
sk
ib) 2
Yoo
IS. After how many days will — th of the radio
oook

ic) 3 id) 4 20

(JEE Advanced 2015) active element remain behind, if half life of the
eBB

10. It is found that if a neutron suffers an elastic element is 6-931 days


collinear collision with deuterium at rest, (a) 23-03 days ib) 25-12 days
fractional loss of its energy is ; while for its
uurr

(c) 29-96 days id) 27-72 days


similar collision with carbon nucleus at rest,
ad

(Kerala PMT 2006)


fractional loss of energy is p^.. The values of p^i
Yo

and p^. are respectively 16. Two radioactive substances A and B have decay
constants 5 X and X respectively. At r = 0, they
dY

ia) (0-89, 0-28) ib) (0-28, 0-89) have the same number of nuclei. The ratio of
Re

ic) (0. 0) id) (0, 1)


innd

number of nuclei of A to those of B will be


(JEE Main 2018) (1/e)^ after a time
FFi

11. A nucleus with mass number 240 breaks into two


ia) 4X ib) 2X
fragments each of mass number 120, the B.E. per
1
nucleon of unfragmented nuclei is 7-6 MeV.
ic) id) 4X (AIPMT 2007)
While that of fragments is 8-5 MeV. The total gain 2X
in B.E. in the process is
17. The intensity of gamma radiation from a given
ia) 216 MeV ib) 0-9 MeV source is I. On passing through 36 mm of lead, it
ic) 9-4 MeV id) 804 MeV
is reduced to - . The thickness of lead which will
(NEET 2021) 8
12. When a uranium isotope is bombarded reduce the intensity to — will be
with a neutron, it generates 35Kr^^, three neutrons 2
and
ia) 12 mm ib) 18 mm
(«) ib) 36
Krioi
(c) 9 mm id) 6 mm
ANSWERS
7. {h) 8. ia) 9.(c) 10. (fl) 11. (a) 12. id) 13. (d) 14. (./;) 15. (c)
16.ic) 17.ia)
13/82 lP>utdee^ ^ Fundamental Physics (XII) VOL.II

18. To determine the half life of a radioactive element, (a) 3-2 T years (/>) 4-6 T years
dN{t) (c) 6*6 T years (fO 9-2 T years
a student plots a graph of In versus t.
dt (DPMT 2010)
dN{t) 23. Two samples X and T contain equal amounts of
Here is the rate of radioactive decay at
dt
radioactive substances. If — th of sample X and
time t. If the number of radioactive nuclei of this 16
1
element decreases by a factor of p after 4.16 years, th of sample Y remain after 8 h, then the
256
the value of p is
ratio of half periods of X and Y is
(a) 2: 1 ib) 1:2
(c) 1 :4 (d) 1 : 16
(e) 4:1 (Kerala CEE 2010)

ww
24. The activity of a freshly prepared radioactive
sample is 10^® disintegrations per second, whose
mean life is 10^ s. The mass of an atom of this

FF loo
radioisotope is 10“^ kg. The mass (in mg) of the
radioactive sample is :

ree
(«) 1 {b) 2
(c) 3 id) 4 (HT 2011)
25. The half life of a radioactive substance is 20

reFe
id) 2 ib) 4
id) 8 (HT 2010) minutes. The approximate time interval

ororF
ic) 6
rur
19. A radioactive nucleus (initial mass number A and (t2 - tj) between the time t2 when ^ of it has
s ff
atomic number Z) emits 3 a-particles and 2
positrons. The ratio of number of neutrons to that decayed and time and ^ of it had decayed is
k
YYouo
of protons in the final nucleus will be {a) 14 min {b) 20 min
okso

A-Z-4 A-Z-12 (c) 28 min {d) 1 min


BBoo

id) ib) (AIEEE 2011)


Z-8 Z-4
26. The half life of a radioactive isotope X is 50 years.
r ee

A-Z-A A-Z-8
ic) id) It decays to another element Y which is stable.
Z-2 Z-4
The two elements X and Y were found to be in the
ad
ouur

(AIEEE 2010)
ratio of 1 : 15 in a sample of a given rock. The
Yo

20. The speed of daughter nuclei is age of the rock was estimated to be
2 Aw Aw id) 150 years ib) 200 years
d

ia) c ib) c
idnY
Re

M M ic) 250 years id) 100 years


Aw Aw [AIPMT (Prelim) 2011]
FFin

ic) c id) c 27. A radioactive nucleus of mass M emits a photon


VM + Aw M +Aw
of frequency v and the nucleus recoils. The recoil
(AIEEE 2010)
energy will be
21. The decay constant of a radio isotope is X. If A j (a) Mc^-hv {b) h^vVlMc^
and A2 are its activities at times and ?2 id) hv
(c) zero
respectively, the number of nuclei which have
decayed during the time (/| - ^2) [AIPMT (Prelim) 2011]

id) Aj/j - A2/2 ib) A,-A2 28. Two radioactive nuclei P and Q, in a given sample
:● (c) (Ai-A2)/X id) A,(Aj-A2) decay into a stable nucleus R. At time / = 0,
[AIPMT (Main) 2010]
number of P species are 4 Nq and that of Q are
Nq. Half-life of P (for conversion to R) is
22. A radioactive isotope has a half life T years. How 1 minute where as that of Q is 2 minutes. Initially,
long will it take the activity to reduce to 1% of its there are no nuclei of R present in the sample.
originalvalue ?
IfANSWERSil
18. (d) 19. (a) 20. (a) 21. (c) 22. (c) 23. (a) 24. (a) 25. (b) 26. (b) 27. {b)
NUCLEI 13/83

When number of nuclei of P and Q are equal, the 33. Half lives of two radioactive elements A and B
number of nuclei of R present in the sample would are 20 min and 40 min respectively. Initially, the
be
samples have equal number of nuclei. After
9N.0 80 minutes, the ratio of decayed numbers of A and
ia) 3N,0 (b) - B will be

5N.0 (a) 1 : 16 (b)4:\


(c) - (d) 2Nq (c) 1 ; 4 id) 5: A
[AIPMT (Main) 2011] (JEE Main 2016)
29. A mixture consists of two radioactive materials 34. An accident in a nuclear laboratory resulted in
/4j and Aj with half lives of 20 s and 10 s deposition of a certain amount of radioactive
respectively. Initially the mixture has 40 g of A ] material of half life 18 days inside the laboratory
and 160 g of A2- The amount of the two in the Tests revealed that radiation was 64 times more

ww
mixture will become equal after : than the permissible level required for safe
(a) 60s (b) 80s operation of laboratory. What is the minimum
(c) 20 s id) 40 s number of days, after which the laboratoiy can

Flo
be considered safe for use ?
[AIPMT (Prelim) 2012]
30. The half life of a radioactive nucleus is 50 days. (a) 64 (b) 90

e e
The time interval - tj) between the time ^2 (c) 108 (d) 120

reer
when ^ of it has decayed and the time /j when

rFF
(JEE Advanced 2016)

of it had decayed is: 35. A radioactive nucleus undergoes spontaneous


uur r
decay in the sequence
(a) 30 days
(c) 60 days
(b) 50 days
(d) 15 days
^ Z-i^ffoorZ-3^ —^ z-2^’ where Z is the
atomic number of element X. The possible decay
sks
[AIPMT (Main) 2012] particles in the sequence are
YYoo
ooko

31. The half life of a radioactive isotope ‘X’ is 20 (a) (3 , a, (b) a, p-, p-^
years. It decays to another element ‘Y’ which is
eBB

(c) a, p- (d) p-^, a, p-


stable. The two elements ‘X’ and ‘Y’ were found
to be in the ratio 1 : 7 in a sample of a given rock. (NEET 2020)
The age of the rock is estimated to be : 36. The half-life ofa radioactive nuclide is 100 hours.
uurr

The fraction of original activity that will remain


ad

(a) 100 years (b) 40 years


after 150 hours would be
Yo

(c) 60 years id) 80 years


2 1
(AIPMT 2013)
ia)
dY

32. For a radioactive material, its activity A and rate 3V2


Re

of change of its activity R are defined as


innd

dN dA
(c)
2V2 (d) I (NEET 2020)
FFi

A = - and R = -
dt dt
37. The half life of a radioactive sample undergoing
where N it) is the number of nuclei at time t. Two a decay is 1-4 x lO'^ s. If the number of nuclei
radioactive sources P (mean life x) and Q (mean in the sample is 2-0 x 10“^ the activity of the
life 2 t) have the same activity at / = 0. Their sample is nearly
rates of change of activities at / = 2 t are R,> and ia) 10^ Bq
n
ib) 10^ Bq
Rq, respectively. If R
—, then the value of n
e
(c) lO^Bq id) 10^ Bq
IS Q
(NEET 2021)
(a) 1 ib) 2
38. What happens to the mass number and atomic
ic) 3 i(l) 4 number of an element when it emits gamma
(JEE Advanced 2015) radiation ?

28. ib) 29. id) 30. (/?) 31. (c) 32. (/)) 33. (^/) 34. (c)
35. id) 36. ic) 37. ib)
13/84 Fundamental Physics (XIl)iL*i*Mn

(a) Mass number increases by 4 and atomic (c) Mass number and atomic number remain
number increases by two. unchanged.
(b) Mass number decreases by four and atomic (d)Mass number remains unchanged while
number decreases by two. atomic number decreases by one.
(NEET 2021)

n Multiple Choice Questions (with One or More than One Correct Answers)
39. Which of the following statement(s) is (are) (c) are central forces, independent of the spin of
correct ? the nucleons

(a) The rest mass of a stable nucleus is less than (d) are independent of the nuclear charge.
the sum of the rest mas.ses of its separated 43. A radioactive sample consists of two distinct
nucleons. species having equal no. of atoms initially. The

ww
(b) The rest mass of a stable nucleus is greater mean life time of one species is T and that of the
than the rest masses of its separated nucleons, other is 5 x. The decay products in both cases are
stable. A plot is made of the total no. of radioactive
(c) In nuclear fission, energy is released by fusing nuclei as a function of lime. Which of the

Flo
two nuclei of medium mass, (approximately following figures 13(CF).2 best represents the

e
100 amu)
form of this plot ?

e
{d) In nuclear fission, energy is released by

reer
FIGURE 13(CF).2

rFF
fragmentation of a very heavy nucleus.
40. During a nuclear fusion reaction ; N
N
uur r
(a) a heavy nucleus breaks into two fragments by
itself

(b) a light nucleus bombarded by thermal neutrons O


ffoor O
sks
breaks up
YYoo
ooko

(c) a heavy nucleus bombarded by thermal


neutrons breaks up
eBB

t t
(d) two light nuclei combine to give a heavier
nucleus and possibly other products
N
uurr

41. A star initially has 10^^® deutrons. It produces ■\ r\


ad

energy via the process


Yo

O
fH + fH -> JH+/?
dY

and ^H + ^H —> ^He + n


Re
innd

If the average power radiated by the star is t t t


10*^ W ; the deuteron supply of the star is
FFi

exhausted in a time of the order of : 44. A sample of a radioactive element has a mass of
[The masses of the nuclei are as follows : 10 g at an instant t = 0. The approximate mass of
this element in the sample after two mean lives is
M (Jh) = 2014m; M(p)= 1-007 u, {a) 1-35 g (b) 2-50 g
M (n) = 1-008 u ; M (^H) = 4-001m] (c) 3-70 g (d) 6-30 g
(a) lO^s (b) 10® s 45. In a radioactive decay chain. 9QTh^^^ nucleus
(c) 10*^ s id) 10‘^ s decays to nucleus. Let and be the
42. The nuclear forces
number of alpha and beta particles respectively
emitted in this decay process. Which of the
(a) are stronger, being roughly hundred times that following statement (s) is (are) true ?
of electromagnetic forces
(b) have a short range dominant over a distance
ia)N^ = 5 (b)N^ = 6 (c)yVp = 2 (rO Wp = 4
(JEE Advanced 2018)
of about a few fermi
ANSWERS

38. (c) 39. ia.(t) 40. (d) 41. (c-) 42. ia.h.d) 43. (d) 44. (a) 45. ia.c)
NUCLEI 13/85

DU Multiple Choice Questions (Based on the given Passage/Comprehens ion)


Each comprehension given below is followed by some multiple choice questions. Each question has one
correct option. Choose the correct option.
46. In alpha decay
The radioactive nuclei are (a) mass number A decreases by 4 and atomic
unstable and emit alpha (2He'*), beta number Z increases by 2.
and gamma (qY*^) radiations to achieve states (b) A decreases by 4 and Z decreases by 2
of greater stability. Rutherford and Soddy (c) A increases by 4 and Z decreases by 2
suggested the following rules governing the
(d) A increases by 4 and Z increases by 2.
radioactive decay : (i) The algebraic sum of
47. In beta decay
charges (atomic numbers) before and after
disintegration must be the same. («) The sum (a) A remains unaffected. Z increases by I.

ww
of mass numbers before and after (b) A is unaffected, Z decreases by I
disintegration must also be the same. (c) A increases by I, Z is unaffected
(d) A decreases by 1, Z is unaffected.

09

Flo
Matching Type Questions

e
eree
DIRECTIONS. In each of the following questions, match column I and column II and select the correct
match out of the four given choices.

FFr
48. Column I Column II

oorr
uur r
A. Nuclear density P- (A-Z) sf
B. Size of nucleus q. Z
C. Number of protons in r. lO^'^kg/m^
sk
Yoo
D. Number of neutrons in s. 10-*^ m
ooko

(a) A-p, B-q, C-r, D-j (b) As, B-r, C-q, D-p (c) A-r, B-i', C-q, D-p (d) A-r, B-5, C-p, D-q
eBB

K9 Matrix-Match Type Questions


uurr

p q r s
ad

''|]©i!®!;©l©I
Yo

DIRECTIONS. Each of the following questions contains statements given


in two columns, which have to be matched. The answers to these questions B
©l©IOIi©l
dY

have to be appropriately bubbled. If the correct matches are A-r, As ;


'='©i®!OI©
Re

B-p, B-q ; C-p ; D-^, D-s, then the correctly bubbled matrix will look like
innd

the one shown here :


D
l®ii©i!0i ©■
FFi

49. Column I Column II j 1

235
A. Nuclear fusion P- absorption of thermal neutrons by 92
U

60
B. Fission in a nuclear reactor. (}■ Co nucleus
27

C. P'decay r.
energy production in stars via conversion of hydrogen to helium
D. y-ray emission s. Heavy water
t. Neutrino emission
50. Column I Column II
A. Atomic nucleus P- nuclear fission
B. mass defect (P nuclear fusion
C. E=m(p- r.
binding energy
D. Stability of nucleus s. much smaller than atom

ANSWERS

46. {h) 47. (a) 48. (c) 49. (A-r ; B-p, s \ C-q. t; D-^) 50. {As ; B-/j. q, r \ C-p. q, r ; D-r)
13/86 Fundamental Physics (XII)EEIHD
51. Column 1 Column II

A. Activity of a sample P- No change in mass number


-1
B. Beta decay sec

C. Gamma decay r. Becquerel


D. Decay constant s.
no change in charge number
52. Column 1 Column II
235
A. Nuclear fusion P- Absorption of thermal neutrons by 92
U

B. Fission in a nuclear reactor ^Co nucleus


C. P-decay .r Energy production in stars via hydrogen conversion to helium
D. y-ray emission s. Heavy water
Neutrino emission

w
t.

(JEE Advanced 2015)

Flo
A B C D
VI. Integer Type Questions
@®@®

e
reee
DIRECTIONS. The answer to each of the following questions is a single digit
oo®o

FFr
integer, ranging from 0 to 9. If the correct answers to the question numbers A, B, © ®© ®
C and D (say) are 4, 0, 9 and 2 respectively, then the correct darkening of bubbles ® ©®®
should be as shown on the side :

for
0000
ur
53. The intensity of gamma radiation from a given source is I. On passing through 27 mm 0 0 0®
kss
of lead, it is reduced to 7/8. The thickness of lead which will reduce the intensity to HI
will be :
0 0 0®
Yo
©0©©
oo

54. The half life of a freshly prepared radioactive sample is 2 h. If the sample emits radiation
of intensity, which is 16 time the permissible safe level, then the minimum time taken
0 0 0®
eB

after which it would be possible to work safely with source is : 000®


55. The number of P particles emitted if a radioactive element decays into 33 by a and P emission is:
ur
ad

VII. Assertion-Reason Type Questions


YYo

Reason. Binding energy per nucleon of 3Li^ is


d

FOR MEDICAL STUDENTS


Re

greater than that of -,He‘*. Hence 3Li^ is less stable


in

DIRECTIONS. Read the following questions and


than ^He'*.
F

choose any one of the following four responses.


A. If both. Assertion and Reason are true and the {a) A {b) B (c) C {(i) D
Reason is the correct explanation of the 57. Assertion, a-parlicle is a helium nucleus.
Assertion.
Reason. In a-decay, both the mass number as well
B. If both. Assertion and Reason are true but as atomic number of the daughter is more than
Reason is not a correct explanation of the that of parent.
Assertion.
{a) A ih) B (f) C id) D
C. If Assertion is true but the Reason is false.
D. If both Assertion and Reason are false. 58. Assertion. If a heavy nucleus is split into two
medium sized parts, each of new nucleus will have
56. Assertion. Lower value of binding energy per more binding energy per nucleon than original
nucleon indicates greater stability. nucleus.

ANSWERS

51. (A-£/, r ; B-f); C-p, s : D-r/) 52. (A-r ; B~p, s ; C-q,i; D*/j. q. r) 53.(9) 54.(8)
55.(1) 56. (d) 57. (f)
NUCLEI 13/87

Reason. Joining two light nuclei together to give FOR ENGINEERING STUDENTS
a single nucleus of medium size means more DIRECTIONS. Each of the following questions
binding energy per neclcon in new nucleus. contains two statements. Check if

(a) A ib) B (c) C id) D (A) Statemeni-1 is true, Stalement-2 is true and Staie-
ment-2 is correct explanation of Statement* 1.
59. Assertion. In the process of nuclear fission, the
(B) Statement-! is true, Siatemenl-2 is true, but
fragments emit two or three neutrons as soon as
Statement-2 is not a correct explanation of
they are formed and subsequently emit particles. Statement-1.
Reason. As the fragments contain an excess of (C) Statement-1 is true, but Statement-2 is false.
neutrons over protons, emission of neutrons and (D) Statement-] is false, but Statemeni-2 is true.
particles bring their neutron/proton ratio to stable
values 65. Statement-1. Nuclei of different atoms have same
size

ww
(a) A (b) B (c) C id) D Statement-2. R =

60. Assertion. Radio activity of 10* undecayed ia) A ib) B (c) C id) D

Floo
radioactive nuclei of half life 50 days is equal to 66. Statement-1. Density of nuclear matter is same
that of 1-2 X 10* no. of undecayed nuclei of some for all nuclei

ee
other material with half-life of 60 days. Statement-2. Density has nothing to do with mass
and size of nucleus.

eer
Reason. Radioactivity is proportional to half life.
ia) A ib) B ic) C (^0 D

FFr
(a) A ib) B (c) C id) t> 67. Statement-1. One kg of coal, on burning,
produces energy = 10^ joule.

oorr
61. Assertion. In a radioactive disintegration, an
uur r
Statement-2. It follows from the relation E = rnc-
electron is emitted by the nucleus.
s ff
(a) A (b) B ic) C id)D
Reason. Electrons are always present inside the
68. Statement-1. After ten half lives, the amount
sk
nucleus.
YYoo
of a radioactive element reduces to about 1/1000
ooko

ia) A ib) B ic) C id) D part.


eBB

62. Assertion. Fragments produced in the fission of 1 \" 1 \I0


t/235 active. Statement-2. N = .N0 = .N
0
2 2
uurr

Reason. The fragments have abnormally high N N


ad

0 0
proton to neutron ratio.
Yo

1024 1000
ia) A ib) B (C-) C (iO D ia) A ib) B (c) C id) D
63. Assertion. Electron capture occurs more often 69. Statement-1. No law is violated in the nuclear
dY

1
Re

than positron emission in heavy elements. reaction q/i -1


ind

Statement-2. Mass number and charge number,


FFin

Reason. Heavy elements exhibit radioactivity. both are conserved


ia) A ib) B ic) C id) D ia) A ib) B ic) C id) D
64. Assertion. Forces acting between proton-proton 70. Statement-1. Half life of tritium is 12-5 years.
The fraction of tritium that remains after 50 years
ifpp), proton-neutron ifp„) and neutron-neutron is 6-25%

if,J are such that^, =/„„. N 1 Y' 1 ^


l/T

Reason. Electrostatic force of repulsion between Statement-2.


N.0 1
V - y
o
V - /
two protons reduces net nuclear forces between
them. n12’5/50
1
— = 6-25% .
ia) A ib) B ic) C id) D 2 16

{ARMS 2015) ia) A ib) B ic) C id) D

ANSWEBSi
58. ih) 59. ia) 60. (c) 61. ic) 62. id) 63. (/;) 64. («) 65. {d) 66. (r ) 67. (c)
68. (a) 69. id) 70. (a)
13/88 ‘Pnadee^'<i. Fundamental Physics (X1I)CQ2MD
71. Statement-1. Energy is released when heavy 72. Statenieiit-1. A nucleus having energy £, decays
nuclei undergo fission or light nuclei undergo by P" emission to daughter nucleus having energy
fusion. E-i, but the P" rays are emitted with a continuous
energy spectrum having end point energy
Statement-2. For heavy nuclei, binding energy (£, - £2)-
per nucleon increases with increasing z, while for Statement-2. To conserve energy and momentum
light nuclei, it decreases with increasing z. in P-decay, at least three particles must lake part
in the transformation,
(a) A (/;) B (c) C (d) D
(a) A (b) B (c) C (d) D
(AIEEE 2008)
(AIEEE 2011)

A A o

ww
For Difficult Questions

Multiple Choice Questions correct Answer)

Flo
e e
1. if U If Xe + Ipn + 0IX+lY Mass defect, Am = 4xw He — m
O,

reer
54 0

rFF
X = y = n = 4 X 4 0026 - 15-9994 = 0 011 a.m.u.

Q = (140 X 8-5 -t- 94 X 8-5) - 236 x 7-5 Energy released


uur r
= {1190-h 799)- 1770 = 219 MeV
As Q= Kj(e + + K^,
ffoor
£ = 0-011 x931 = 10-24 MeV

4. Output power = 1-6 MW = 1-6 x 10^ J/s


sks
= 129+ 86 + 4 = 219 MeV.
Energy released/fission = 200 MeV
YYoo
In options {«) and (d), energy and charge
ooko

= 200x 1-6 X lO-'-^J


conservation are followed.
Number of fissions/sec
eBB

In option (d), > (Psr + P.k +


therefore, conservation of momentum is not
1-6x10^ 16
= 5x10
200x1-6x19-^^
uurr

followed.
ad

Hence option is correct. 5. Z-t-l


yA^ Z-I —>
z-i
Yo

2. The temperature T to which gases must be heated In going from X to K, mass number remains
should provide energy sufficient to overcome the unaffected, charge number increases by 1.
dY

repulsive potential energy between the two nuclei. Therefore, P particle must have been emitted. In
Re
innd

going from Y to B*. mass number decreases by 4


As K.E. of translation/molecule = —kT and charge number decreases by 2. Therefore, an
FFi

2
a particle must have been emitted. In going from

|kT + |kT = repulsive RE. £* to B, both mass number and charge number
remain unaffected. Therefore, y ray photon must
3 kT = 7-7 X lO-'** have been emitted. Choice ib) is correct.
6. The nuclear reaction is
_ 7-7x10-*4 7-7 xlO-'** I 0
T = o»’ ^ \P‘ +-,e
3A* 3x1-38x10-23
Mass defect. Am = (w^, + m^) - m„
7= 1-85 X 10’K = 10^ K = (1-6725 X IQ-2'7 + 9 X 10-3’) - ,.6725 x 10"27
3. When four helium nuclei are fused together, one = 9 X 10-3' kg
oxygen nucleus is formed. The reaction is Energy released = (A/m) t-
8 Qi^+Q
4 X 2 He-^
= 9x 10-3' (3 X io8)2 j
ANSWERS
71. (a) 72. (a)
NUCLEI 13/89

.●. Fractional loss of energy


For Difficult Questions

1 ●>
^2
-15 — tmi~ — m u
81x10 A£ 9 2 V 3 48
AE = MeV = P,=^ = 0-28
-13 E 1 169
1-6x10 — um~
2
= 0-506 MeV
11. Here, mass number of reactant = 240
7. ■>
90 + ^He'’ + Energy
7
B.E per nucleon = 7-6 MeV
P~ Mass number of product nucleus = 120
KE of thorium =
Im
Th B.E/nucleon of product nuclei = 8-5 MeV
7
Total gain in BE = B.E of products
P~
and KE of helium =
- B.E of reactants
2 m

w
He
= (120-1- 120) X 8-5 -240x 7-6
Since < mTh so KE He >KE Th
= 240 (8-5 - 7-6) = 240 x 0-9 = 216 MeV
8. Radius of the nucleus having mass number A is

Flo
determined as, R = A or R OC A
1/3 12. The nuclear reaction can be put as
92 U235 ^ _1 ^ ^ o"' +

ee
xl/3
R, (A 1 (216 As charge number is conserved,

Fr
Z + 0 + 36 = 92 or Z = 92 - 36 = 56
\
A,
~ /
164 4 2
As mass number is also conserved.
9. Let E - power requirement of the village A + 3 + 89 = 235 + 1

for
ur
£' = power of plant A = 236-92 = 144
12-5 E' E' 144
.-. £ = x£' £ = — or £ = — The product generated is
ks
100 8 2^ NB
Yo
No. of half lives = 3 13. At lime /, = 0-3 /Vg = 0-3/^^
oo

So total time required = 3x7 years = vT years


eB

If Vq is total number of nuclei at / = 0, then


n = 3

10. Mass of neutron, m^=m N


ur

0
Mas.s of deuterium, /«-, = 2 m V^(l -fO-3) = Vo, =
ad

1-3
Let H be the initial velocity of neutron before collision
Yo

with deuterium. Let U|, 1^2 be the vekx-iiy of neutron Again, as = Vq


and deuterium after collision. Then N. 0
= N.S0 e~^'' or
nd

Wj = m. /«2 = 2 m, H| = H, I/, = 0. 1-3 1-3


Re

From elastic collision between neutron and or X/=ln(l-3)


Fi

deuterium wc have
. In (1-3) In (1-3)
I =

V
(/n, _(/?i-2nr u =
-u X In (2)/7
1 “
(f«j + m^) «i -I- 2 m 3
In (1-3) log 1-3
xT =T
Fractional loss of energy In (2) log 2

1 7 -II
2 14. Let Nq be the number of atoms of each isotope at
AE
— mii~ — in r = 0 and Vj and N2 be the number of atoms of
2 2 3 ; 8 each i.sotope at r = /.
- = 0-89
2 9
-X, t
and N^=N^^e-h'
— mu
2 Then N, = e
0

For elastic collision between neutron and carbon iV, (Xt—X|) I


= e
nucleus, m^ = w, m-, = 12 m ; Hj = m, »■> = 0 ^2
V
(/n, -W2)H| ’ m - 12 m 11
ti N
1 “ I
(m, -t- m^) //I -I-12 m 13 or
(X,-X^)/ = !n
- /
13/90 ‘Pn.eideefi^ '4. Fundamental Physics (XII) FTSTWIl
I );'5]rK^ir^:^3?c^NiriGSK)®ca© 18. N = M,0 e~‘^
For Difficult Questions
dN -h
= —X Ni\ e
1 NI dt
or t = [n
X-, - dN -h
= X e
dt
99-3
or / = In ...(0
A-2 Xj 0-7 dN
log dt = log^ XNq-Xi
0-693 0-693
h = 4-47 xlO^ and X2 = 7-04x10^ dN
Putting values in (0
log
dt
= -Xt+ log^ {X /^o) ...(0

1 (99-3 Comparing if with the equation of straight line


In

w
t =
0-693 0-693 0-7 y = /fur + C, we get
7-04x10*^ 4-47x10^ 3-4
slope = -X. = (From graph)

Flo
or t = 5*97 X 10’ years 6-4

0-6931 0-6931

e
-1
15. As X. = X = — year
= 0-1 per day or

rree
T 6-931

r FF
N 0.6931
Now = e Half life, T = = 2 xO.6931 = 1.386 yrs.
N X
0
uurr
or
1

20
for ^ = ^0-
(W
Ml
t/r
or
N
N0
(\
\2)
\l/T
kss
4.16
^+01/_ 20 or 0-1 / = 2-303 log IQ 20
ooook

1 Hi .386 1 1
Yo
0-1 r = 2-303 X 1-3010 or p = 8
v2, P 2
eB

t = 29*96 days
19. As per question, the nuclear reaction is
16. At t = 0, N = Nq for both the substances A and B + 3 2He^ + 2
e -A.V
urr

-X.t
N. =N e
1 e
and N
j>2
= N
i'IqC Charge number of final nucleus = (Z - 8)
ad

-X,i Mass number of final nucleus = (A - 12)


Yo

N 1
l.£ - g(^2 ~ ^1) ^ = ^(T.-5 X) t Number of protons in final nucleus, = (Z - 8)
dY

^2 e 2 Number of neutrons in final nucleus,


«^_ = (A- 12)-(2-8)=A-Z-4
Re
innd

sAXt
= = -
n
(A - Z - 4)
Fi

n
p
Z-8
N I
1 _ 2 1 20. K.E. of daughter nucleus of mass M moving with
As 4 X. r = 2 or t = —
^2 4;^ 2X I
-Mv-
velocity V =
2
17. As 0
Energy released due to decrease in mass A m of
1 1
= ^-M'36 and _ g-ilx parent nucleus = A mc^.
2
1
Mv- = Amc-
2

=e = (e 1^)^ or -3 |i.v = - 36 p 2A/nc^ 2Am


or t) = = c.

V M \ M
36
x = — = 12mm 21. Activity at time /|. Aj = XJV] or N^ = A,/X.
3
Activity at time /2. = XjVj or N2 = A2IX
NUCLEI 13/91

Dividing (ii) by (i), we gel


For Difficult Questions

1
Therefore, the number of nuclei decayed during or t2~t\=T= 20 min.
2 2
lime interval (/tI - to) is
A Ao 26. Let at a time /, Nx be the number of X in an
I
X
element and Ny be the number of Y in that element.
X X

A N V' 1 Given, or Nx= 15 Ny


22. From iV, 15
A) N
0
2
V-/
100

It (log 1 - log 2) = log I - log 100 Part of =


[Nx+Ny]
16

ww
n (0-0-3010) = 0-2
2 1 fl
n = = 6-6 or
0-3010 ^x+Ny 16 V"/
2

t = nT=6'6T years

Flo
If n is the number of half lives of isotope X.

e
23. Here, f = Sh,T/T = ?

eree
1 2 V' /1
1 1
f ^ \4 Then or /j = 4
A N
I 1 T‘ I 1
2 2

FFr
«,=4 V *- y \ y

A) N.
0
16 V 2
- y
uurr If r is the age of rock, and T is the half life period

orr
8 of X, then r = /iT" = 4 x 50 = 200 years
( Y- 1 1

A) ^0 2 ) 256 [2J
«2 = 8 sfo
27. Momentum of emitted photon, Pphoton ~ — ●
hv
kks
Yoo
r. t/nj /I, 8 Let V be the speed of recoil nucleus, then
oooo

- = 2:1
7-2 thx^ n 4
/^nucleus "■ /’photon
eBB

.. hv hv
I 1 Mv = — or V =
24. Activity, A = XN =~N ■.■X = - c Mc
X X
urr

The recoil energy of the nucleus


where x is the mean life time of given sample.
ad

\2
YYo

1 1 hv
A^ = Ax = (10’'^ decays/sec) x (10^ s) = -Mty2 =_M
= lO’^ atoms 2 2 \Mc 2Mc^
dd

Mass of the sample, m = N x (mass of one atom) 28. Initially, number of nuclei /* = 4 Nq
Re
inn

= 10^'^ X 10-25 ^ 10-6 kg = I mg number of nuclei Q = Nq


25. Here, 7 = 20 minutes Half life of P,Tp=\ min.
F

In time tj, the radioactive sample left behind Half life of Q, 7^^ = 2 min.
No. of nuclei of P after time r.
=l-i=- Nf/l
3 3 ■ (\
n = 4N = 4N
In time the radioactive sample left behind p 0 2 0
2J
No. of nuclei of Q after time r.
= iA=i
3 3 ■ y/2
Q 1
n =N =N
.i/T Q 0 0
N 2 1 Y|/7' 12
As ...(0
N. 2 Suppose number of nuclei P and Q in given
0 j 3 U
sample are equal after time t, then n..p - n Q
\h/r
1 1 v/i 1 y/2
and ...(H) or 4N = N
3 [2) 0 ') 0
2
13/92 ‘P'uxdee^’^ Fundamental Physics (XII)EEIHD

For Difficult Questions 1 1


Divide
2 I)
(I y/2
\2
or 1 (\
. !i h = 1 or ^2 - f, = 7” = 50 days
V“ /
2 4 2J r

/
A/ X 1
or — = 2 or t -4 min —
r = ?
2 31. Here, 7= 20 years

n4/1
1 N N 1 I N
0
Hp=4Nq 2 4
Now,
+ '+7
\ / 0

x4/2
1 N \n
0 N fl

w
and = N0
Q 2 4
From ,/j = 3
N0 2j
Population of R in the sample is f = « 7 = 3 X 20 years - 60 years

Flo
\ /
N N.0 9A^ I
I
0 0
4Nn- + .N

eeee
0 4 / ^
0 4 2 32. Here, Ap = A^e r Aq = Aq€
2t

29. Here, 7] = 20 s, T2 = 10 s, Nq^ = 40 g,

Fr
I
I

/V(j2= 160 g. R T R
Q ~ 2t
Let the amount of the two radioactive materials ^ T

for
ur
become equal after t sec. At r = 2 X,
■J/T
JL-(L 1
ks
From , we get e~-
R 2
Yo
2 p - X
\ /
oo

R
Q io. e

y/20 f\ Y/20 2t
eB

!
M =N,01 = 40
2 2 n
V y V y
On comparing with —, we get n = 2
ur

R
y/lO f\ y/IO
ad

1
^2 — Nq2 = 160
Yo

u V
2
y
33. Here, «, = — = 4,
80
n
B ~
?0=2
20 40

(\ Y/20 y/iO
d

1
Re

As /V,=/V2 40 = 160 /t^4


I 1 N
in

0
2 2 From — , /V A
\ y
N.0 u 16 16
F

fl V/20 1 y/10 / I N
1 10
2 15
= 4 N.
or
2 2 2
Decayed no. of atoms of A = Nq- M^ = 16
0
V y V y

' V'a n2 /V
/ N 1 1 0
_ 0 or r = 40 s B _ N
20 10 N.0 2 2
V*- y
4 ^■4
v“ y

30. Here, 7 = 50 days Decayed no. of atoms of B = Nq - Np


.t/T N 3
From
N = .N0 ^ = -N,.
4 4 0
N.0

15
— .N
1 1
\h/T
2 f 1 Y' Required ratio =
16 0 _ 15 4^5
3 \1)
and
3"U, 3/4 AL0 "16^3 4
NUCLEI 13/93
r.f
I .
36. Here, T = 100 hours, r = 150 hours.
For Difficult Questions
If Aq is initial activity of the radioactive nuclide
and A is the activity of the radioactivenuclide at
A time i. Then as
34. Here, 7=18 days, /=? — = 64
f 1 A /■
'4 = A) T
N A I
yi J A) u
From
N. 0 nLSO/IOO
Aq v2
A-fl 1

1 /
1
\n
Ay U.*
n = 6 1
64 2 Fraction of original activity at time t =
17
2V2
37. Here, 7= l-4x 10

ww
s
t
As n = — N=2-0x 10^'
7
dN 0-6931
f =/I 7=6x18 = 108 days Activity, A=- xN

Flo
dt 7
35. On P"*" decay, atomic number of product nucleus
0-6931x2-0xl02t

ee
decreases by 1. On P~ decay, atomic number of
product nucleus increases by 1. On a decay, 1-4x10''^

rere
atomic number of product nucleus decreases

rFF
A = lO'^Bq
by 2.
38. Gamma radiation is a packet of energy, which
uurr
Hence correct order of decay will be as given by carries no charge and no mass. Therefore, on
p'*’, a, P“ is choice (d).
foor
emission of y-radiation, charge number and mass
number remain unchanged.
ks s
m
Yoo
Multiple Choice Questions (with One or More than One Correct Answers)
oook
eBB

39. As some mass is converted into binding energy 42. Nuclear forces are strongest forces, are short range
of nucleus, so rest mass of nucleons is greater forces, and are independent of the nuclear charge.
than the mass of stable nucleus and in fission
uurr

reaction, a heavy nucleus breaks down into 43. Both the species of atoms follow exponential
ad

smaller nuclei with the emission of energy. decay law, whatever be their life lime. Therefore,
Yo

40. Nuclear fusion is the process in which two or more option {d) is correct,
light nuclei combine to give a heavy nucleus with 44. / = 2i = 2 x 1-44 7= 2-88 T
dY

the emission of lot of energy.


Re
innd

y/r s2-88
41. As rH + 1,2H
1 ^ fH+p N m 1
As
12
FFi

N. m 2
0 0
?H
1 + 1?H

3?H 2He + ^? + p f\ >^2-88


4
I I
= 10
UJ ^
m = m
Mass defect, Am = 3x2-14-{4-001 + 1-008+ 1-007) 0
UJ
Am = 0-026 u
= 10 X 0-135 g = 1-35 g
Energy released by 3 deutrons
= {0-026 u) (931-5 MeV/i() = 24-22 MeV 45. Here 9QTh^^^ is changing into 32^^^'^. Change
in mass number = 232 - 212 = 20
Energy released by 10*^ deutrons
24-22
X 10^x1-6x10-13] =i2.9xlO-^J 20 ^
3 Number of a particles emitted, =— =5

energy 12-9x1027 J
Time (0 - Due to 5a particles, Z will decrease by 10. Since
power IOI6J/S net change in 2 is (90 - 82) = 8, therefore, number
1-29 X 10*2 S =10*2 sec of P particles emitted = 2
13/94 'Pn^uUc^ 4 Fundamental Physics (XII) k*Amil

For Difficult Questions

on Multiple Choice Questions (Based on the given Passage/Comprehens ion)


46. As mass of an a-particle is 4 units and it carries 2 units of positive charge, therefore, in alpha decay, A

w
decreases by 4 and Z decreases by 2
47. As a beta particle carries unit negative charge and has negligible mass, therefore, in beta decay. A remains
unaffected and Z increases by I.

e
m etching Type Questions

e
I i

o
rw
r
48. Size of atom lO"^^ m ; Size of nucleus ~ lO m. In a nuclide 2^'^, number of protons = Z and number of

F
neutrons = (A - Z).

ullo
FF
Choice (c) is correct.

e
srre
Matrix-Match Type Questions

koF
49. Nuclear fusion is involved in energy production fusion and binding energy. E = mc^ applies in

fofr
uor
in stars via conversion of hydrogen to helium. nuclear fission, nuclear fusion and binding energy.
o
Fission in a nuclear reactor involves absorption Stability of nucleus is related to binding energy
of the nucleus.
Y
of thermal neutrons by Heavy water is
kos
used as a moderator in nuclear reactor. Beta decay 51. Activity of a sample is measured in Becquerel,
Yo
rBB
oo

of 2?Co^® nucleus involves neutrino emission. which is equivalent to sec"^ In beta decay, there
eY

Gamma ray emission is involved in decay of is no change in mass number. In gamma decay,
re

27
Co^ nucleus. there is no change in mass number or charge
u

50. Atomic nucleus is much smaller than an atom. number. The units of decay constant are sec”’.
Mass defect is related to nuclear fission, nuclear
d

52. Knowledge based question.


ou
o
ad
nY

VI.
Integer Type Questions
nid
Re
F

53. From 1= In e 54. Here, r=2h,f=?


Fi

To work safely with the sample, its activity must


-pj; = log — be reduced to — .
^0 16

N 1 Y’
1 1 From n = 4
0
-p(27) = log = 31og - N.0 2) 16‘ 2
8/,0 J
r = ,i7'=2x4 = 8h
238
1 55. 9qX
and -p(.x')=log - As mass number decreases by 238 - 222 = 16,
16
therefore, number of a particles emitted
Dividing, we get. 2Z = 3
.X
The charge number reduces to 90 - 4 x 2 = 82. To
x' -9 mm increase charge no. by (83 - 82) = 1. One beta
particle must be emitted.
NUCLEI 13/95

For Difficult Questions

VII. Assertion-Reason Type Questions

FOR MEDICAL STUDENTS reverse is not true i.e. when electron capture is
energetically allowed, positron emission is not
56. Assertion and the Reason, both are false. Lower
necessarily allowed.
value of binding energy per nucleon indicates
So, the assertion is true, but reason is not a
lesser stability. Binding energy per nucleon of 3Li^

oww
correct explanation of the assertion.
is lesser than that of 2He'^ jLi^ is less stable
than 2He‘*. 64. Here Assertion is true and Reason is also true

57. Assertion is true, but the Reason is false. When and is the correct explanation of Assertion.
Because the effective force between two nucleons
a-decay takes place, both the mass number and

e
FrFlo
atomic number decrease. In general, a-decay is is equal to the algebraic sum of the nuclear and

re
represented as + ^He^^ + energy electrostatic forces acting between them. Between

ree
proton-neutron and neutron-neutron, only the

F
where X is the parent nucleus and Y is the nuclear force of attraction act. But between
daughter nucleus.

rF
proton-proton, the nuclear force of attraction and
58. Both, assertion and reason are true, but the electrostatic force of repulsion also acts. Due to

fsfoor
ouur
latter is not a correct explanation of the former. it, the net force between the two protons becomes
59. Both, the assertion and reason are true and the less than the force of attraction between proton-
kosk
latter is correct explanation of the former. neutron or neutron-neutron.
Yo
oo

dN
Y

60. Radioactivity = - = 'kN FOR EN‘- ● R1N(' .TUDENTS


BB

dt

65. The statement-1 is false, but the relation given


rre

0-693 N 0-693x10' in statemcnt-2 is true. In this relation, Rq is


oYuu

T 50 constant and not R.


ad

66. Statement-1 is true, but the statcment-2 is false.


0-693x1-2x10*
dY

Infact as /? o: volume « A. That is why


60
dependence of density (mass/volume) on A
innd
Re

0-693 X 2 X 10^ vanishes.

1
Fi

67. Statement-1 is true, but the statement-2 is false.


F

So, assertion is true but radioactivity « — , and


Energy, here depends on calorific value of fuel.
not T. Therefore, the reason is false.
68. Both the statements are true and statement-2
61. P particles are emitted from radioactive nuclei as
is correct explanation of the statement-1.
a neutron is converted into a proton. So assertion
69. Statement-1 is false, but statement-2 is true.
is true, but the reason is false.
The law violated is the law of conservation of
62. Fragments produced in the fission of are not
spin or angular momentum.
active. Neither proton to neutron ratio in their
70. Both the statements are true; and statement-2
cases is abnormally high. Both Assertion and
Reason are false. is correct explanation of statement-1.

63. Electron capture occurs more often than positron N 1


*^50/12-5
1 100
emission in heavy elements. This is because if N 2 16 16
0
positron emission is energetically allowed,
electron capture is necessarily allowed, but the = 6-25%
13/96 '4. Fundamental Physics (XII) P7STWT1
72. Here, statement 1 is true and statement 2 is also
n

For Difficult Questions


true because for energy and momentum
conservation, at least three particles ; daughter
71. Both the statements 1 and 2 are true ; and nucleus, |3-panicle and anlineutrino take pan in
statement 2 is a correct explanation of transformation, i.e., statement 2 is a true
statement 1.
explanation of statement 1.

ww
Floo
ee
eer
FFr
oorr
uur r
sk s ff
YYoo
ooko
eBB
uurr
ad
Yo
dY
Re
nind
FFi
ooww
e
ere
[UJK]D u

rFl
o

Fre
rrF
i;
k '' l< |( t

sffoo
ouur
kosk
Yo
oo
Y
BB

CHAPTER 14.
rre

SEMICONDUCTOR ELECTRONICS
ouu

MATERIALS, DEVICES AND SIMPLE CIRCUITS


Y
ad
dY
innd
Re
Fi
F

» i
SEMICOMDUCTOR

oww
ELECTRONICS MATERIALS,
DEVICES AND SIMPLE CIRCUITS

e
ree
rFl
Fre
14.1. INTRODUCTION

rr F
Now-a-days all the electronic devices which we use are based on the controlled flow of electrons.
ouur
Before 1948, mostly the electronic devices were made of vacuum tubes (also called valves) like the diode
sfoo
valve which has two electrodes; anode (often called plate) and cathode, triode valve which has three elctrodes
(i.e., plate, cathode and grid); tetrode valve having four electrodes (i.e., plate, cathode and two grids) and
kks
pentode valve having five electrodes plate, cathode and three grids). In these vacuum tubes, the electrons
Yo
oooo

are provided by heating the cathode using low tension battery. The controlled flow of electrons is achieved by
eBB

varying the voltage between its different electrodes. A vacuum is created between inter-electrodes so that the
moving electrons may not lose their energy on collision with air molecules in their way. In these vacuum
lubes, the electrons can flow only in one direction from cathode to anode), hence they are called as
urr

valves. The vacuum tube devices are bulky, operating at high voltages (more than 100 V), consume more
ad

power, having limited life and low reliability.


YYo

In 1930, it was realised that some solid-state semi-conductors and their junctions can be helpful of
controlling the number and direction of flow of charge carriers through them. The discovery of semi-conductor
dd

junction, i.e., junction diodes and trasistors, replaced the vacuum tubes as they are small in size, operate at
Re
iinn

low voltage, consume small power, having long life and high reliability. The semi-conductor junctions led to
F

the discovery of integrated circuits which have revolutionised the electronic industry as they have been used
in the working of television and computer which are very commonly used in our daily life.
In this unit we shall learn the basic concepts of semi-conductor electronics and their applications.

14.2. CLASSIFICATION OF SOLIDS ON THE BASIS OF THEIR CONDUCTIVITY


On the basis of relative values of electrical conductivity (c) and resistivity (p), the solids can be classified
into following categories,
(i) Metal conductors.
These are those solids which have high conductivity and low resistivity.

For example, Al, Cu, Ag, Au etc. The value of conductivity for such material lies inbetween 10^ to 10^
Sm“’ and the value of resistivity lies in between 10'^ to m. Silver (Ag) is the best metal conductor.

14/1
14/2 Fundamental Physics fXlIIPZsm

(it) Insulators.

These are those solids which have very low conductivity and very high resistivity.
Examples are, rubber, plastic, putty etc. The value of conductivity for insulators is less than 10“^ Sm"*
and resistivity is more than 10^ Q m.
(ill) Semi-conductors.

These are those solids which have conductivity and resistivity inbetween metal conductors
and insulators.

The value ofconductivity for semi-conductors lies inbetween 10 ^to 10^ Sm ' and of resistivity between
10^ to 10° Q m.
Our interest in this chapter is in the study of .semi-conductors , which can be of following types ;

loow w
(a) Elemental scini-conductors.

These are the semi-conductors available in natural form.


Examples are silicon (Si) and germanium (Ge).

ree
(b) Compound semi-conductors.
These are the semi-conductors which are made by compounding the metals.

rree F
Examples are ;
r FF
(i) Inorganic semi-conductors; CdS, GaAs, CdSe, InP
fofroF
etc.
u
(ii) Organic semi-conductors ; anthracene, doped pthalocyanines etc.
ks
(lYi) Organic polymers semi-conductors ; polypyirole, polyaniline, polythiophene etc.
soo

Now a days most of the available semi-conductor devices make use of elemental semi-conductors and
YYouor

inorganic semi-conductors. However after 1990, some semi-conductor devices have been prepared by the
BBook

use of organic semi-conductors and organic polymers semi-conductors. But their detail study is going on.
r ee

Curiosity Question
ouru
ad

r
Yo

Q. The semiconductors prepared by traditional chemical methods leave impurities at


the part per thousand (ppt) level. The semiconductors to be used in electronic Industry
d

must be purified to the part per billion (ppb) level. How can you obtain It ?
Re
inY

Ans. In order to obtain a pure semiconductor of high purity level, we take some length of a rod of
FFind

crystal of silicon or germanium. Heat it to melt the same in vacuum and then allow it to cool
slowly to form the rod again. In this process, the impurities present in the original crystal
will migrate to the surface of the rod, where they can be removed. It is so because,
crystallization is a natural means of purification.
J
14.3. ENERGY BANDS OF SOLIDS (BAND THEORY OF SOLIDS)
According to Bohr’s theoiy of atomic spectra and the concept of electronic configuration, the electrons
in an isolated atom have certain definite discrete amounts of energy corresponding to different shells and
subshells. It means there are well defined energy levels of electrons in an isolated atom. If large number of
atoms are brought close to one another to form a crystal, they begin to influence each other. Due to this
interatomic interaction, there is no appreciable modification in the energy levels of electrons in the inner
shells but there is a considerable modification in the case of energy levels of the electrons in the outer shells.
This is due to the fact that the valence electrons are shared by more than one atom in the crystal.

I
SEMICONDUCTOR ELECTRONICS MATERIALS, DEVICES AND SIMPLE CIRCUITS 14/3

To understand this modification of energy FIGURE 14.1


levels, consider a single silicon (Si) or Germanium
(Ge) crystal containing N atoms. Each atom in the Forbidden

crystal is situated at a lattice site. For silicon atom,


atomic number 14, the electronic configuration is
C.B.
I Energy Gap
L 3p2
2s^ 2p^ 3/?“ and for germanium atom,
3s2
atomic number 32, the electronic configuration is
li-2 2p^ 35^ 3p^^ 3c/‘*^ 4s^ 4p^. Both the atoms
have four valence electrons, /.<?., number of
electrons in the outermost orbit is 4 (2, 5-electrons
and 2, p-electrons). Therefore the total number of
valence electrons in the crystal of Si or Ge is 4 N.
t
>» > 2p®
TO
The maximum number of electrons in the outer orbit

ww
(1>
> 2s^
of silicon atom can be 8 (= 2.v electrons + 6p lU

electrons). It means for the 4N valence electrons Crystal Lattice Spacing


>
7
1s2
there are 8 N available energy states. In Fig. 14.1,

Floo
the interatomic spacing r is shown on X-axis and a b
X
d
energy on K-axis. The distance r = a = I A

ee
corresponds to equilibrium distance between atoms Interatomic Spacing, r

eer
or actual crystal lattice spacing.
The process of splitting of energy levels for silicon (Si) can be understood by considering the different

FFr
situations as discussed below :

oorr
uur r
(/) If the interatomic spacing of the Si atoms is very large (i.e. r-d>>a), there is no interatomic
s ff
interaction. Each atom in the crystal behaves as a free atom. In this situation each of N atoms has its own

identical energy levels, wWch are sharp, descrete and distinct. The electronic configuration of silicon of
sk
YYoo
atomic number 14 is 15 25 2p 35- 3p^. This shows that the outer two subshells of silicon atom contain two
ooko

electrons in 3.5 subshell and two electrons in 3p subshell, whereas six electrons are required to completely fill
3p subshell. Thus, in the silicon crystal under study, there are 2 N electrons completely filling 2 N possible 3.v
eBB

levels, all of which are of the same energy. There are 6 N possible 3p levels, out of which only 2 N levels are
filled and all the filled 3p levels have the same energy. All these facts follow from Pauli’s exclusion principle.
uurr

(«) when the interatomic spacing r is less than d but greater than c (i.e. c<r<d), there is no visible
ad

splitting of energy levels.


Yo

{Hi) When the interatomic spacing r is equal to c, the interaction between outermost shell electrons
(35 and 3p ) of neighbouring silicon atoms becomes appreciable. As a result, the energy of electrons
dY

corresponding to 35 and 3p levels of each atom starts changing, i.e., the splitting of these energy levels
Re
nind

commences, whereas there is no change in the energy levels of electrons in the inner shells.
(iv) When the ineratomic spacing r lies inbetween b and c (Le. b < r < c), the energy of electrons
FFi

corresponding to 35 and 3p levels of each atom gets slightly changed. Instead of a single 35 or 3/; level, we get
a large number of closely packed levels (2 N levels corresponding to a single 3.y level and 6 N levels for a
single 3p level of an isolated atom). This spreading of energy corresponding to 35 and 3p levels reduces the
energy gap that existed between 35 and 3p levels of free atom. Since number N is very large
(* 10 atom/m ) and the energy of 35 and 3p levels is of the order of few electron volt, i. *e levels due to
spreading of the energy of 35 or 3p levels are very closely spaced.
This collection of closely spaced levels is called an energy band.*

(v) When the interatomic spacing r becomes equal to b but greater than a (i.e. r = b>a), the energy
gap between 35 and 3p levels completely disappears and the 8 N energy levels are continuously distributed.
In such a situation, it is not possible to distinguish between the electrons belonging to 35 and 3p subshells. We
can only say that 4 N levels are filled and 4 N levels are empty.
*There is a close relationship between the energy band structures of solids and their optical properties.
14/4 “Pn^iuUc^ ^ Fundamental Physics (XII) vL»im\i

(vi) When the interatomic spacing r becomes FIGURE 14.2

equal to a (Le. r^a, the actual spacing in the crystal), 4N States


then at absolute zero, the band of 4 N filled energy Empty
levels is separated from the band of 4 N unfilled energy Er
levels by an energy gap called energy band gap, which
is denoted by The lower completely filled band is
called valence band (V.B.) and the upper unfilled band Filled
I g

: 4N States
is called conduction band (C.B.). The positions of ● *

energy bands in a semiconductor at 0 K are shown in


Fig. 14.2. The lowest energy level in the conduction
band is shown as E^. and highest energy level in the valence band is shown as The separation between top
of valence band and bottom of conduction band is called energy band gap (Energy gap Eg).
Quantum states in silicon crystal having N atoms are shown in the following table

ww
Energy level Total states available Total states occupied

2N

Flo
1 s 2N

2 s 2N 2N

e
6N 6N
2p

rere
2N 2N

r FF
3 s
6N 2N
3p
uurr
A
DIFFERENCE BETWEEN VALENCE BAND, CONDUa iON BAND AND ENERGY BAND GAPfoor
1. Valence band. In the energy band diagram of semiconductors, the valence band is a lower band
kss
belonging to valence electrons of the given crystal. This band may be partially or completely filled with
Yoo
ooook

electrons. This band is never empty. In this band electrons are not capable of gaining energy from external
electric field. Therefore, the electrons in this band do not contribute to the electric current.
eBB

2. Conduction band. In the energy band diagram of semiconductors , the conduction band is an upper
band in which the electrons are not present at 0 K. At room temperature, this band is either empty or partially
uurr

filled with electrons. In this band, electrons can gain energy from external electric field and contribute to the
ad

electric current.
Yo

^.nt-rgj' band gap ('■■■ energy gap). In the energy band diagram, energy band gap is the separation
dY

between highest energy level of valence band and lowest energy level in conduction band. Electrons arc not
found in this band. This band is completely empty. The minimum energy required for shifting electrons from
Re
innd

valence band to conduction band is called energy band gap (Eg).


the electron from valence band to conduction band,
Fi

If X is the wavelength of radiation used in shifting


then energy band gap is E=hv = hc/X
g

where h is called Planck’s constant and c is the velocity of light. The energy band gap for different
materials is different.

~ DISTINCTION BETWEEN CONDUCTORS (OR METALS), SEMI-CONDUCTORS


.:.=D INS- lATORS ON THE BASIS OF THEIR ENERGY BANDS
Metals are good conductors of electricity, insulators are very poor conductors of electricity whereas the
semiconductors have the conductivity inbetween that of conductors and insulators.
Fermi energy is the maximum possible energy possessed by free electrons of a material at
absolute zero temperature (i.e, 0 K). The value offermi energy for different materials is different.
,ii levt semiconductor. It is that energy level in energy-band-diagram of semiconductor
for which the probability of occupancy (Le., the presence of maincurrent carriers electrons or
holes) becomes half.
SEMICONDUCTOR ELECTRONICS MATERIALS, DEVICES AND SIMPLE CIRCUITS 14/5

For the present discussion the fermi level may be defined as that energy level in energy-band-
diagram of semiconductor which corresponds to the centre of gravity of conduction electrons
and holes weighted according to their energies.
When the temperature of a semiconductor is raised to TK, the position of fermilevel is raised by kT from
its initial position at a given temperature, where k is Boltzmann constant.
Depending upon the relative positions of the valence band and the conduction band, the solids
can be classified into metals (conductors), insulators and semiconductors.
Metals (conductors). The energy band
diagram for a metal is .such that either the conduction
band is partially filled with electrons, Fig. 14.3(a) or
the conduction and valence band partly overlap each
other and there is no forbidden energy band gap in

ww
between. Fig. 143(h). In both the situations, it can

be considered that the metal has a single energy band


which is partly tilled and pardy empty.

FF loo
Many electrons from below the Fermi level,

ree
by acquiring a little more energy from any source.
can shift to the higher energy levels above the Fermi

reeF
level in the conducion band and behave as free
electrons. In this situadon, large number of electrons
are available for electrical conduction, hence the
oroFr
r ur
resistance of such a material is low or the
s ff
conductivity is high. Even if a small electric field is
applied across the metal, these free electrons start
k
YYouo
moving in a direction opposite to the direcdon of
koso

electric field. Due to it, a current begins to flow


through it and hence metal behaves its a conductor.
BBoo
r ee

Insulators. The energy band diagram of insulator is shown in Fig. 14.4. Here, the valence band is i:
completely filled, the conduction band is empty and energy gap is quite large (E^ > 3 eV). For example, in
case of diamond, the energy gap is of 6 eV. Since the valence band is completely filled as per Pauli’s exclusion
ad
ouur

principle, therefore the electrons are not free. Again due to large energy gap, no electron is able to go from the
Yo

valence band to the conduction band even if electric field is applied. Hence electrical conduction in these
materials is impossible and they behave as insulators.
d
Re
idnY
FFin

FIGURE 14.4
c

>.
■a
Q.

E XJ ^
Uj c CD ,.
o *
o ‘ ‘ Ci _
<D ^
C2 %
>,co
E’A
cn
C UJ
> r U1
■D O x>
O c c
S ® or
LL .CO CD
>

Semiconductors. The energy band diagram of a semiconductor is shown in Fig. 14.5. Here also, the
valence band is totally filled and the conduction band is empty but the energy gap between conduction band

I
14/6 p>uieU^'4. Fundamental Physics (XlI)CBani
and valence band is quite small, it is less than 3 eV. For example, the energy gap for germanium is of 0-72 eV
and for silicon it is of M eV. At zero kelvin temperature, electrons are not able to cross even this small
energy gap and hence the conduction band remains totally empty. Therefore, the semiconductor at zero
kelvin behaves as insulator. However at room temperature, some electrons in the valence band acquire
thermal energy greater than energy band gap less than 3 eV and jump over to the conduction band where they
are free to move under the influence of even a small electric field. As a result of it, the semiconductor acquires
small conductivity at room temperature. The resistance ofsemiconductor would not be as high as that ofinsulator.
Retain in Memory
1. The size of the energy gap between conduction band and valence band for different materials is
different.

2. The maximum energy which an electron can have at 0 K, is called fermi energy which is also

ww
different for different materials.

14.6. INTRINSIC SEMI-CONDUCTORS

FF loo
A pure semiconductor which is free of every impurity is called intrinsic semiconductor.

ree
The electrical conductivity of a pure semiconductor is totally governed by the number of electrons
and is called intrinsic conductivity.

reeF
excited from the valence band to the conduction band
Germanium and silicon are the important examples of intrinsic semiconductors which are widely used

oroFr
in electronic and transistor manufacturing. The electronic configuration of silicon and germanium
r ur
are as

Silicon (14), 1^", 2s~ 2p^ 3p~


follows :
s ff
Germanium (32), li'', 2P" 2/;^, 3s~ 3p^ 3J*®, 4p-
k
YYouo
Both the atoms have
FIGURE 14.6
koso

thus four valence electrons. I I 1 t I


/

The crystal structure of


BBoo

Gel
in two
germanium
r ee

dimensions is shown in Fig. I


I I
I I I
I
I
I
14.6(«). The four valence ● / /
Empty
ad
ouur

electrons of a germinium ,Ge ,Ge Conduction


Yo

Band
atom form four covalent I
/
> I \ I
/
I
Energy
bonds by sharing the I t I I I I
I
Gap
electrons of neighbouring .r
d

Gel Filled
Re
idnY

four germanium atoms. I


Valence
Each covalent bond shares I I I I I Band
FFin

two electrons one from


each atom. By forming e
such covalent bonds, each
Ge atom in the crystal behaves as if the outermost orbit of each atom is complete with eight electrons, having
no free electrons in the Ge-structure.
At temperature OK, for Ge-structure, the valence band is all full. The energy gap is 0-72 eV and the
conduction band is totally empty. Fig. 14.6(fe). Since no electron is available for conduction therefore, the
Ge-crystal at OK acts as electrical insulator. The conduction is possible if some of the electrons break away
from their covalent bonds and become free. The minimum energy required to break a covalent bond is 0-72 eV
for Ge and 1 ■ 1 eV for Si.
Even at room temperaUire, the thermal vibrations of the atomsprovide energy to the electrons m valence band
to enable some electrons to cross the energy gap andenter into the conduction band as free electrons, accounting
for some electrical conductivity of the semiconductor. Higher is the temperature, larger will be the number of
electrons crossing over to the conduction band, leaving behind equal number of holes in the valence band. In
intrinsic semiconductor at room temperature, fermi level is about half way in the energy gap.

I
SEMICONDUCTOR ELECTRONICS MATERIALS, DEVICES AND SIMPLE CIRCUITS 14/7

When an electron breaks away from a covalent bond, the empty place or vacancy left in the bond is
called a hole, shown by a hollow circle in the Fig. 14.6(a). When an external electric field is applied, these
free electrons and holes move in opposite directions and constitute a current flow through the germanium
crystal. The number offree electrons (in conduction band) and holes (in valence band) are exactly equal in
an intrinsic semiconducto.r

Thus in intrinsic semiconductor, n^. = n/, - n,


where n^, n,, are number density of electrons in conduction band and number density of holes in valence

w
band. «● is the number density of intrinsic carriers (electrons or holes) in a pure semiconductor.
It is very difficult to make an intrinsic semiconductor because of the difficulty in preparing extremely
pure material. r- © .;
Limitations of developing pure semiconductor ha.sed devices

e
1. In a pure semiconductor, at room temperature, the number of intrinsic charge carriers (electrons and

row
re
) is very small (» 10 m ). That is why the pure semiconductor is a low conductivity material.
2. In a pure semiconductor, the intrinsic charge carriers are always produced due to breakage of covalent

FFllo
eeF
bonds by virtue of thermal motion. Hence enough flexibility is not available to control their number in a
pure semiconductor.

u
3. In a pure semiconductor, the number of electrons (n^) is always equal to number of holes («,). It is

r
never possible in a pure semiconductor to have either large number of conduction electrons only or large

sFr
number of conduction holes only. That is why pure semiconductor is not of much use.

kro
It is important to note that
uor
(//) number of electrons is equal to number of holes.
offo
1. In an instrinsic semiconductor (/) there are two types of current carriers (/.e., free electrons and holes)
kos
Y
2. An intrinsic semiconductor is electrical neutral as a whole.
Yo
eerBB
oo

3. In a semiconductor the total current is due to the movement of both the free electrons and holes,
whereas in a metal conductor the current is due to flow of electrons only.
rY

14.7. FORMATION OF HOLES IN SEMI-CONDUCTORS


u

This can be understood in two ways. i


ou
ad
do

FIGURE 14.7
(/) From the energy band diagram of the
semiconductor, in the energy band diagram of the —
nY

C.B:- C.Br-
semiconductor, there is an energy gap of about 1 eV I
between the valence band and the conduction btmd. Fig.
nid


Re

14.7(a). At zero kelvin. the semiconductor behaves as


F
Fi

an insulator, as no electron from the valence band can


cross this energy gap and go to the conduction band. But
at higher temperature, some of the electrons gain energy ●V.Br ●V.B:—
due to thermal agitation and move from the valence band ■o—o-

to the conduction band. As a result of it, a vacancy is


created in the valence hand at a place where the electron
was present before moving to the conduction hand. Fig. e o
14.1(b). This vacancy is called a hole. Since
the absence of a negatively charged electron is equivalent to the presence of an equivalent amount of positive
charge, therefore.
a hole is considered as a seat ofpositive charge, having charge equal to that
of an electron.
The hole is considered as an active particle in the valence hand as an electron is in the conduction
band. The motion of the electrons in the conduction band and also the motion of holes in the valence hand
are responsible for the electrical conduction in .semiconductors .
14/8 'Pmtdee^'^ Fundamental Physics (XII)y^*j^

(h) b'rom the valence bond study of the semiconductor. Consider FIGURE 14.8

u semiconductor crystal of silicon or germanium under study. We know 1 1 I

that the Si or Ge have four valence electrons. The crystal structure of -A


Ge in two dimensions has been shown in Fig. 14.8. V ' Ge
‘t-
Eveiy atom ofGe tends to shaie one of its four valence electrons t I I I I I

with each of its four nearest neighbouring atoms, and also to take
\ I

share of one electron from each such neighbour. This pair of shared
electrons of two atoms of Ge is said to form a covalent bond or simply \ I

a valence bond. Thus, four valence electrons of a Ge atom form four I I I 1


I

covalent bonds by sharing the electrons of neighbouring four Ge atoms. Ge


Due to it, the Ge atoms in the structure are strongly held by covalent
bonds not in a plane as it may appear here but in space of tetrahedral 1 I I

ww
angles.
At low temperature, in a Ge crystal structure, the two shared electrons in a covalent bond can be
assumed to shuttle back-and-forth between the associated atoms, holding them together strongly.

Flo
When the temperature of Ge Is raised, the thermal energy ionises DO YOU KNOW ?
only a few atoms in the crystalline lattice. Due to it, few valence electrons

e
1. At room temperature, the

reree
con iribuiing to covalent bonds break and become free to move in whole
intrinsic current carriers
of the crystal lattice. While coming out of a covalent bond, the electron

r FF
concentration in a pure
leaves an empty space which is having positive charge equal to that of
the electron which is shown in Fig. 14.8 as an open circle. It is called a germanium is about lO’^ m“^
uurr
hole. An electron from a neighbouring atom can break away and can be
attracted by the missing electron (or hole), thus completing the covalent foor and in a pure silicon is about
10
10 .„-3.
m
ks s
bond there and creating a hole at another place. In our two dimensional 2. Germanium is widely used for
Yoo
example, we see that an electron from any of four neighbouring atoms making semiconductor
ooook

can come to complete the bond and hole can move to any of these devices because it has large
atoms. It is to be noted that breakage of each covalent bond produces current carrier concentration
eBB

one free electron and one hole in a crystalline lattice. as compared to silicon and
In order to creat the free electron and hole in a crystalline lattice a forbidden energy gap is less
uurr

than that of silicon. As a result


certain amount of ionisation energy would be involved. This
ad

of it, a very small external


ionisation energy is least for Ge, more for Si and highest for C.
Yo

Theoretically, it is found that the number of free electrons (n^) or holes energy will make germanium
dY

conductive as compared to
(n^,) produced as a result of an ionisation is given by silicon.
Re
innd

-E^tlkT
Fi

where «o a constant, k is a Boltzmann constant and T is the absolute temperature. For a given E^, as the
temperature T increases, increases.
14.8. ELECTRICAL CONDUCTION IN SEMI-CONDUCTORS
The electrical conduction in semiconductors is caused by the motion of the electrons in the
conduction band and also by the motion of the holes in the valence band.
When an electric field is applied across a semiconductor, the electrons in the conduction band travel in
a flow of electronic current (/^). At the
opposite direction to that of the applied electric field and consitute
same time , the holes in the valence band travel in the direction of the applied electric field and constitute a
hole current (/;,). It means there are two streams of current inside a semiconductor; namely the electronic
in the
current in . . conduction band and the hole current in the valence band. The effective current (/) in the
semiconductor is the sum of these two streams of current.
i.e.. i= hr
For a pure semiconductor at room temperature, the current strength is weak.
SEMICONDUCTOR ELECTRONICS MATERIALS, DEVICES ^ND SIMPLE CIRCUITS 14/9

14.9. DOPING

Doping 15 a process ofdeliberate addition ofa desirable impurity atoms to a pure semiconductor
to modify its properties in a controlled manner. The impurity atoms added are called dopants.

The impurity added may be = I part per million (ppm). In a doping process, it is required that
(/) the dopant atom should take the position of semiconductor atom in the lattice.

w
(/7) the presence of the dopant atom should not distort the crystal lattice.
(///) the size of the dopant atom should be almost the same as that of the crystal atom,
(/v) the concentration of dopant atoms should not be large (not more than 1 % of the crystal atoms).
It IS to be noted that the doping of a semiconductor increases its electrical conductivity to a great extent

e
Methods of doping. Doping is achieved in many ways; for this, we can

row
re
I. add the impurity atoms in the melt of the semiconductor , or

2. heat the crystalline semiconductor in an atmosphere containing dopant atoms or molecules so that the
latter diffuse into the semiconductor,

eeF
Fllo
or

3. implant dopant atoms or molecules by bombarding the semiconductor with them.

Fu
14.10. EXTRINSIC SEMI-CONDUCTOR

srr
roF
A doped semiconductor or a semi-conductor with suitable impurity atom added to it, is called

k
extrinsic semiconductor.
uor
Extrinsic semi conductors are of two types :
(/) «-type semiconductors
ofof
kos
(ii) />type semiconductors
Y
Yo
(0 n-type semiconductor. When a pure semiconductor of silicon (Si) or germanium (Ge) in which each
eerBB
oo

Si or Ge atom has four valence electrons, is doped with a controlled amount of pentavalem atoms, say arsenic
rY

or phosphorous or antimony or bismuth, which have five valence electrons, the impurity atom will replace the
Si or Ge atom as shown in Fig. 14.9(a). The four of the five valence electrons of the impurity atoms will form
u

covalent bonds by sharing the electrons with the adjoining four atoms of silicon, while the fifth electron is
very loosely bound with the parent impurity atom and is comparatively free to move. It is so because the force
ou
o
ad
d

of atti action between electron and nucleus of donor atom becomes very small due to high value of dielectric
nY

constant of Si semiconductor. The binding energy of electron becomes 0-045 eV for silicon n-tvoe
semiconductor and 0-01 eV for f— -
FIGURE 14.9
germanium n-type semiconductor.
nid
Re

Thus, each impurity atom added j I


F

I I t
/
Fi

donates one free electron to the crystal Tsi


structure. These impurity atoms which
donate free electrons for conduction t I I 1 e
I
I
'f- C.B.
= 0.045eV
r
I
1 I I I 1 I
for Si
are called donor atoms. Since the ●_/ k I
7 Donor
conduction of electricity is due to the Si vSi 1
Energy
I ■>
motion of electrons i.e. negative
r I
I I I I I I State
charges or n-type carriers, therefore,
1 I t I I
/ /

the resulting semiconductor is called /


Si;
C* o 0
V.B.
5 S'

donor-type or n-type semiconductor. / I

On giving up their fifth electron, the t



I I I

donor atoms become positively


charged. However, the matter remains e o
electrically neutral as a whole. The
extra electron of the donor atom orbits around the donor nucleus, in a hydrogen like manner. It has been
tound that 0-045 eV energy is required to remove this electron from the impurity atom of silicon semiconductor
and make it a free electron.
14/10 ‘Pnatdecfi'^ Fundamental Physics (XII)EZ3M1

At room temperature, some of the covalent bonds may get ruptured, producing thereby free electrons
and an equal number of holes in the n-type semiconductor. But overall, the total number of holes in «-type
semiconductor is relatively small than the electrons. Hence in n-tspe semiconductor, electrons are malority
carriers and holes are minority carriers.
Energy band diagram of «-type semiconductor is shown in Fig. 14.9(/7). For a silicon semiconductor
with impurity atoms of arsenic or phosphorous, the energies of the free electrons are slightly less than the
energies of the free electrons in the lowest energy level of conduction band. As a result of it, these electrons
occupy discrete energy levels (called donor energy levels) between the valence band and conduction band
and the lowest donor electron energy level lies at 0-01 eV for Ge and 0-045 eV for Si below the bottom of the
conduction band.
When we add pentavalent impurity in a pure semiconductor of Ge or Si, the Fermi level in energy

ww
gap shifts very close to conduction hand. If doping is very large, the Fenni level may move into the conduction
band.
It is to be noted that this energy is comparable to the thermal energy of electron at room temperature
the electrons from donor levels to conduction

Flo
Q.()3 eYy Thus, a very small energy supplied can excite
band. Due to it. the conductivity of semiconductor is remarkably improved.

e
ree
Hi) p-type semiconductor. FIGURE 14.10

Fr
When a pure semiconductor of

rF
Germanium {Ge) or Silicon {Si), I t i I 1 I

V
I ;
uurr
-4 -
I

in which each atom has four

for
(Ge,- Ge iGe
C.B.
valence electrons is doped with I ●1 I I r ● ●

a controlled amount of trivalent I 1 I I I


1 t 1
s ' Hole' /
Acceptor
kks
atoms say gallium, or Indium {In)
● I

Ge; ●Ge, / Energy


Yo
t>r Boron (B) or Aluminium (A/)
oooo

State
/ /

which have three valence I I I I I 1


I

● 5** J
I
1 I I I
I* .01 to.045eV
clecuons, the impurity atom will
eB

I / /
●jm--

replace the Ge or Si atom as ;Gei Ge' {Ge; V.B.

shown in Fig. 14.10(a). The three / It'- 7-r


ur

I I I I I I

valence electrons of the impurity


ad

atom will form covalent bonds by e o


YYo

sharing the elelctrons of the


adjoining three atoms of Ge, while there will be one incomplete covalent bond with a neighbouring Ge-atom,
dd

due to the deficiency of an electron. This deficiency is completed by taking an electron from one of the
Re

Ge-Gc bonds, thus completing the In-Ge bond. This makes Indium ionised (negatively charged) and creates
in

a *hole\ An eleclron-movingfrom a Ge-Ge bond to fill a hole, leaves a hole behind. That is how, holes move
F

in the semi-conductor structure. The trivalent atoms are called acceptor atoms and the conduction of electricity
occurs due to motion of holes, i.e., positive charges or p-type carriers. That is why the resulting semiconductor
is called acceptor type or p-type semiconductor.
Also, at ordinary temperature, some'of the covalent bonds may get ruptured, releasing equal number of
holes and electrons. Therefore, the total number of electrons is relatively small as compared to the number of
the holes in the p-tvpe semi-conductor. Hence in the p-type semiconductor, electrons are minority carriers
and holes are majority carriers.
Energy band diagram ofp-type semiconductoris shown in Fig. 14.10(/?). For a Ge or Si semiconductor,
the doping of impurity atoms of Indium or boron having valence three, produce some allowed energy levels
which are situated in the energy gap slightly above the valence band. These levels are called acceptor energy
levels.
When we add a trivalent impurity in a pure semiconductor of Ge or Si, the Fermi level in energy
gap shifts very close to valence band. If doping is very large, the Fermi level may move into the valence
band.
SEMICONDUCTOR ELECTRONICS MATERIALS, DEVICES AND SIMPLE CIRCUITS 14/11

At room temperature, due to thermal energy, the electrons from


DO YOU KNOW ?
the valence band are easily transferred to the acceptor level* until these
levels are filled. This produces a large number of holes in the valence 1. In pure germanium semicon
band and thereby the valence band becomes a hole conducting band. ductor, the fermi level is about
When an external electric field is applied to a p-type semiconductor, half way in the energy gap.
these holes will act as carriers of current. Due to it, the p-type 2. With rise in temperature, the
semiconductor shows its electrical conductivity much improved than Fermi level moves towards tlie
what it was for pure semiconductor. centre of the energy gap, for
both types y?-type and n-type of
Retain in Memory semiconductors.

1. In a doped semiconductor, the number density of electrons 3. n-type semiconductor is better


and holes are not equal. than the p-type semiconductor
as electrons have more mobility

w
Under thermal equilibrium, the product of the free negative
than holes. For a given voltage
and positive concentrations is a constant quanatity,
applied, conduction current in
independent of the amount of donor and acceptor impurity

Flo
n-type semiconductor is more
doping. This relationship is known as mass-action law and than that of />-type
is given by semiconductor.

ee
where n^ are the number density of electrons and holes 4. If the light of energy greater

Fr
respectively and n,- is number density of intrinsic carriers than the energy gap is incident
{i.e. electrons or holes) in a pure semiconductor. on an intrinsic semiconductor,
2. In n-type semiconductor, the number density of electrons
for
the electrons from the valence
ur
is nearly equal to the number density of donor atoms band cross over to conduction

and is very large as compared to number density of holes. band. Due to it electrons and
ks
Hence n^~ N^>> n,j holes pairs are created. Due to
Yo
increase in the number of
3. In p-type semiconductor, the number density of holes is
oo

current earners, the


nearly equal to the number density of acceptor atoms
eB

conductivity of semiconductor
and is very large as compared to number density of electrons.
Hence increases. This property of
semiconductor is called
r

4. The number density of intrisnic current carrier («,) of a photoconductivity.


ou
ad

semiconductor varies with temperature TK, according to


Y

relation ~EJ{2kT)
8
ni=nQ€
nd

where «q a constant, k is the Boltzmann constant and


Re

is the energy gap of the given


semiconductor.
Fi

5. Both /?-type and p-type semiconductors are neutral.


6. The fermilevel in n-type semiconductor lies in the forbidden energy gap near the conduction
band.

7. The fermilevel in p-type semiconductor lies in the forbidden energy gap near the valence band.

Sample Problem D A silicon specimen is made into ap-type semiconductor by doping, on an


average, one indium atom per 5 x 10^ silicon atoms. If the number density of atoms in the silicon
specimen is 5 x 10^* atoms/m^ ; find the number of acceptor atoms in silicon per cubic centimetre.
Sol. The doping of one indium atom in silicon semiconductor will produce one acceptor atom in
p-type semiconductor. Since one indium atom has been dopped per 5x10^ silicon atoms, so number density
of acceptor atoms in silicon

*Since the effective gap is reduced to about 0 045 eV for Si and 0-01 eV for Ge.
14/12 ^fuuUe^’4. Fundamental Physics (XII)B&I9D
28
no.density of Si atoms 5xl0
- 10^^ atoms/m^
doping fraction of impurity atoms SxlO”^
= 10^* X 10“^ atoms/cm-* = 1 x 10*^ atoms/cm^
Sample Problem The number densities of electrons and holes in pure silicon at 27"C are
equal and its value is 2*0 x 10^^ m“^. On doping with indium, the hole density increases to 4*5 x 10^ m“^,
find the electron density in doped silicon.
-3 .
Sol. Here ; n, = 2-0x 10'® m' 3 = 4-5 X 10^2 m

As n
n.
f _ (2-0x1Q'®)2 = 8-89 X 10’m-'^
n
h
4-5x1Q22

w
14.11. DISTINCTION BETWEEN INTRINSIC AND EXTRINSIC SEMI-CONDUCTORS

Flo
INTRINSIC SEMICONDUCTOR EXTRINSIC SEMICONDUCTOR

e
1. It is pure semiconductingmaterial and no 1. It is prepared by doping a small quantity of impurity

rree
impurity atoms are added to it. atoms to the pure semiconducting material.

r FF
2. Examples are crystalline forms of pure silicon 2. Examples are silicon and germanium crystals
and germanium. with impurity atoms of arsenic, antimony, phos
uurr
3. The number of free electrons in conduction band
and the number of holes in valence band are
3. for
phorous etc. or indium, boron, aluminium etc.
The number of free electrons and holes are never
equal. There is excess of electrons in n-type
kss
exactly equal and very small indeed. semiconductors and excess of holes in p-type
ooook
Yo
semiconductors.

4. Its electrical conductivity is low. 4. Its electrical conductivity is high.


eB

5. Its electrical conductivity is a function of 5. Its electrical conductivity depends upon the
temperature alone. temperature as well as on the quantity of impurity
urr

atoms doped in the structure.


ad
Yo

14.12. DISTINCTION BETWEEN n-TYPE


SEMI-CONDUCTORS AND /^-TYPE SEMI-CONDUCTORS
dY
Re

p-TYPE SEMICONDUCTOR
innd

n-TYPE SEMICONDUCTOR
Fi

1. It is an extrin.sic semiconductor which is obtained 1. It is an extrinsic semiconductor which is obtained


by doping the impurity atoms of Vth group of by doping the impurity atoms of III group of
periodic table to the pure germanium or silicon periodic table to the pure germanium or silicon
semiconductor. semiconductor.

2. The impurity atoms added, provide extra 2. The impurity atoms added, create vacancies of
electrons in the structure, and are called donor electrons {i.e. holes) in the structure and are
atoms. called acceptor atoms.
3. The electrons are majority carriers and holes are 3. The holes are majority carriers and electrons are
minority carriers. minority carriers.
4. The electron density (n^) is much greater than 4. The hole density {iif) is much greater than the
the hole density (»;,), i.e., » rtfj. electron density {nj), i.e., Hfj » n^.
5. The donor energy level is close to the conduction 5. The acceptor energy level is close to valence
band and far away from valence band. band and is far away from conduction band.
6. The fermi energy level lies in between the donor 6. The fermi energy level lies in between the
energy level and conduction band. acceptor energy level and valence band.
SEMICONDUCTOR ELECTRONICS MATERIALS, DEVICES AND SIMPLE CIRCUITS 14/13

14.13. ELECTRICAL CONDUCTIVITY OF EXTRINSIC SEMI-CONDUCTORS


Consider a block of semiconductor of length / and area of cross-section A, having electron density
and hole density Let V be the potential difference applied across the ends of the semiconductor, Fig.
14.11. The magnitude of the electric field applied is
FIGURE14.11
E = - ...(1) 1
I
+

Due to the electric field, both electrons and holes in this V


semiconductor move in mutually opposite direction.s with drift
velocities and V/, and contribute current and //, in the same cr* <r*
direction in the semiconductor. 3 >

Total current, I = Ig+ Ij, ...(2)

ww
Since electrons in the conduction band and holes in the valence band are moving randomly like electrons
in metals, therefore.
l^ = iigAev and
I,i = n^AeVh

Floo
From (2), I=n^Aev^ + ni^AeVf,^eA {n^ + n,, v^) ...(3)

ee
or
4 ...(4)

eer
A

FrF
I
Current density,

oor r
ur r
Let R be the resistance of the semiconductor and p be its resistivity, then
s ff
p = RA/l ...(5)
sk
YYoou
oooko

Dividing (1) and (5) we get, ...(6)


p ~ RA/l ~ RA~ A
eBB

E
From (4) and (6) we have - ^e(n^v^ + n,^Vi,)
uurr

We know that mobility of electron is defined as the drift velocity per unit electric field. If there is
ad

no
Yo

electric field applied, drift velocity is zero.


.. V V,
mobility of electrons, = — or £ and mobility of holes, P/, = — or
dY

= p^, £
Re
ind
FFin

E 1
~ = e [n^ . p, + fif,. p/,1 £ or
- =e(n^p^+«;,P;,) ...(7)

But electrical conductivity (c) is the reciprocal of resistivity (p), therefore.

1
Electrical conductivity. (T = - =e(«.^p^+rt/,p/,) ...(8)

1
For intrinsic semiconductor, = Uj c = - = en,(li^+iX,)
The relations (7) and (8) show that the conductivity and resistivity of a .semiconductor depend upon the
electron and hole number densities and their mobilities. As and increase with rise in temperature,
therefore, conductivity of semiconductor increases with rise in temperature and resistivity decreases with rise
in temperature. At room temperature c for Ge is greater than that for Si because number density of charge
carriers is more in Ge than in Si.
14/14 ‘Pn<ide€fr '4. Fundamental Physics (XII) PZSTim

Retain in Memory
The current density in a semiconductor is given by
/
y =- n,, Vf^) = e (n^ E + nf^\x^ E)
Conductivity of intrinsic semiconductor. In case of intrinsic semiconductor (silicon or
germanium). H
where is the number density of intrinsic carriers ii.e., electrons or holes).

oww
Thus the conductivity of intrinsic semi-conductor is
0,. = e («; + n,- 4/,) = en^ + jx,,)
Conductivity of «-type semiconductor. In this case » rif^ and rig * N^, where Nj is the
number density of donor atoms. Therefore conductivity of n type semiconductor is

e
re
Conductivity of p-type semiconductor. In this case H/j » rig and = N^j, where is the

FFrllo
number density of acceptor atoms. Therefore, conductivity of p-type semiconductor is

reF
e
Intrinsic concentration. The number density of intrinsic current carriers («,) of a semiconductor
uoru
varies with temperature T, according to relation

osFr
-EJlkT
s
n.

fkfor
where, /Iq = constant, independent of temperature, k = Boltzmann constant,
E„ = energy gap at 0 K
okso
o

The energy gap. The forbidden energy gap E^ in a semiconductor is a function of temperature. It
Y
Yo
has been found that
oo
BB

For germanium. (7) = 0-785 - 2-23 x 10"^T


For silicon. Eg (7) = 1-21 -3-60 X 10-'^T.
Y
r ree

At room temp. (300 K), for Ge, E, = 0-72 eV and for Si, £ = 1 ● 1 eV.
ouu

ft ft

The mobility of charge carriers varies as 7^'” over a temperature range of 100 to 400 K.
ad

’p-m
Ydo

i.e. OC

In case of germanium, m = 1-66 for electron and 2-33 for hole.


In case of silicon, m - 2-5 for electron and 2-7 for hole.
nidn
Re

Sample Problem The densities of electrons and holes in an extrinsic semiconductor are
F
Fi

7*5 X 10^^ cm“^ and 4*5 x 10^^ cm"^ respectively. The mobilities of electrons and holes are 23 x 10^ cm^/V-s
and 10^ cm-/V-s respectively. What is the type of semiconductor ? Find the resistivity of this
semiconductor.
Sol. Here, = 7-5 x 10^^ cm“^ ; = 4-5 x 10*^ cni"^ ;

\ig= 23 X 10^ cm-/V-s ; )X/, = 10- cm-/V-s ;


As, iig > Hfj so the extrinsic semiconductor is of N-type.
1
Now,

19
1-6x10 [7-5xI0'-^x23xl0-V4-5xl0'2xl02]
= 3*62 Q—cm
SEMICONDUCTOR ELECTRONICS MATERIALS, DEVICES AND SIMPLE CIRCUITS 14/15

14.14. EFFECT OF TEMPERATURE ON THE MOBILITY


AND CONDUCTIVITY OF ELECTRONS AND HOLES

With ±e increase in temperature, the mobility of electrons and holes in a semiconductor actually decreases,
like the decrease in mobility of electrons in metals. But, there is a large increase in the charge carrier
concentration due to more breakage of covalent bonds with the increasing temperature. It is so large that
conductivityincreases with increase in temperature and the decrease in mobility has no influence.
According to the equation of the drift velocity, v = e E x/m,
so V oc E ...(9)
But V can not be increased indefinitely by increasing E and the relation (9) does not hold good at high
values of E. It is so because, at high temperature, the increase in drift velocity of free electrons will bring
more collisions and hence average time between two successive collisions (i.c., x) starts decreasing. As a
result of it, the drift velocity saturates at thermalvelocity and becomes almost independent of electric field

ww
at higher values of E.
The exact value of E where drift velocity saturates depends on the nature of semiconductor, doping and
other defects in the semiconducting crystal.

Flo
In intrinsic semiconductor the fraction if) of the nurnber of electrons raised from valence band to
conductionband at temperature TK is given by ^-Eg/2 kT

e
rere
where Eg is the value of energy band gap. The above relation shows that as T increases/also increases. It

r FF
means with the increase in temperature the number of electrons in conduction band increases. Due to it, the
conductivityof semiconductorincreases with increase in temperature.
uurr
Retain in Memory
With the irse in temperature
for
kss
(/) the resistance of a metallic conductor increases
ooook
Yo

{ii) the conductivity of a metallic conductor decreases


eBB

{Hi) the resistance of a semiconductor decreases


(/v) the conductivity of a semiconductor increases.
urr

14.15. P-N JUNCTION


ad
Yo

When a p-type semiconductor crystal is brought into close contact with an n-type semiconductor
dY

crystal, the resulting arrangement is called a p-n junction or junction diode.


Re
innd

Formation of p-n junction


To make a p-n Junction, the n-type and p-type silicon crystals are cut into thin slices called wafers. If
Fi

on a wafer of n-type silicon, an aluminium film is placed and heated to a high temperature, say 580°C,
aluminium diffuses into silicon. In this way, a p-type semiconductor is formed on an n-type semiconductor. A
junction formed with such a formation of p-region on n- region is called p-n junction.
Another way to make a p-n junction is by diffusion of phosphorous into a p-type semiconductor.
The wafer of p-n junction formed, is cut into small pieces. Each piece is enclosed in a casing with
electric connections coming out from p and n regions.
Depletion region and Barrier electric Held in p-n junction
Two important processes occur during the formation of a p-n junction ; diffusion and drift. We know
that in an n-type semiconductor, electrons are majority carriers and holes are minority carriers. In p-type
semiconductorholes are majority carriers and electronsare minority carriers. When p-n junction is formed,
due to difference in concentration of charge carriers in the two regions of p-n junction, the electrons from
n-region diffuse through the junction into p-region and holes from p-region diffuse into n-region. The motion
of charge carriers, due to difference in their concentration in the two regions of a p-n junction, gives rise to
diffusion current across the junction.
14/16 ‘Pn.exdeep. 'a Fundamental Physics (XII) tTOWTl
When an electron diffuses from n-region to p-region of p-n junction, it leaves behind an ionised donor
atom in n-region, having positive charge which is immobile as it is bonded to the surrounding atoms. As
dii'fusion of electrons continues from n-region to /^-region of p-n junction, more positively charged donor
atoms are created in /j-region resulting a layer of positive charge {i.e., a positive space charge region) near
the junction in /i-region.
Similarly, when a hole diffuses from />region to n- region of p-n junction, it leaves behind an ionised
acceptor atom in /^-region having negative charge which is immobile. As the diffusion of holes continues
from /?-region to n-region of p-n junction, more negatively charged acceptor atoms are created in p-region
resulting a layer of negative charge (/>., a negative space charge region) near the junction in /^-region.
FIGURE 14.12
The space-charge regions on
both the sides of p-n junction Hole /
Vb \
I
which has immobile ions and Electron

ww
is devoid of any charge carrier o oiee ® el- o ● ●
will form a region called
depletion region or depletion o o ● ol®® © ®1 ^ ● o

FF loo
layer, Fig. 14.12(a).
o ● o
l©0 ©©;!●
oI ● ● o
i©0 ©@|'

ree
It is like a no man land on a border. Vb
I I
The thickness of this depletion region is O O OI0 0 ©©iO^« ● ●
1 i

reeF
of the order of one-tenth of a micrometer. \
Due to positive space charge region
Electron Depletion Layer Hole o

oroFr
r ur
on /i-side of junction and negative space e
charge region on p-side of junction, an
s ff
electric field is set up acorss the junction from positive charge towards
negative charge, as if a fictitious battery is connected across the junction DO YOU KNOW ?
k
YYouo
with its positive terminal to n-region and negative terminal to /7-region.
koso

● The diffusion of charge


This electric field sets a barrier at the junction which opposes further carriers takes place when
BBoo

diffusion of majority charge carriers into opposite regions. The physical there is a difference in the
r ee

distance from one side of the barrier to the other is known as the width
concentration of the charge
of the barrier. The difference of potential from one side cf the barrier carriers in the two regions.
to other side is known as the height of the barrier. Due to this electric
ad
ouur

● The conduction of charge


field developed at the junction, an electron on /7-side of the junction
Yo

carriers takes place when


moves to n-side and a hole on /i-side of junction moves to /;-side of there is a difference of
junction. The motion of these charge carriers due to electric field is
d

potential between two regions


Re

called drift. As a result of it, a drift current starts, which is opposite in


idnY

direction to the diffusion current. connected through a


conducting material.
FFin

In the beginning, the dijfusion current is large but drift current is


small. As the diffusion process continues, the space charge regions across ● The process of crossing of the
the junction extend. As a result of it, the strength of electric field across depletion region by electrons
and holes is known as
the junction increases and thereby drift current increases. This process
continues until the diffusion current becomes equal to the drift current. injection.
When this stage is reached, the movement of majority charge carriers
across the junction stops. Now the p-n junction is said to be in equilibrium state and there is no current across
the p-n junction. At this stage, the potential barrier across the p-n junction has maximum value V^. It means
the barrier voltage Vg stops the diffusion of majority charge carriers from p to n region across the junction
and vice versa.

Fig. \4.12(b) represents the potential distribution near the junction. This potential acts as a bairier,
hence known as potential barrier.
At room temperature of 300 K, is about 0-3 V for Ge and 0-7 V for Si. Infact, the value of Vp
depends on doping impurity, nature of .semiconductor and temperature but is independent of the design of
Junction diode. The value of Vq increases with rise in temperature for Ge and Si.
SEMICONDUCTOR ELECTRONICS MATERIALS, DEVICES AND SIMPLE CIRCUITS 14/17

Due to presence of potential barrier across the junction, an electron from /i-region requires the
energy eV^ to cross the junction. An equal amount of energy is required to move a hole, from p-region to
/i-region across the p-n junction.
The width of the depletion layer and the magnitude of the barrier potential depend on the nature
of semiconductor and doping concentration on the two sides of p-n junction. If the doping concentration in n-
type and p-type semiconductor forming p-n junction is small, the diffusing electrons and holes across the
junction can move to quite large distances before suffering a collision with another hole or electron to be
recombined. Due to it, the width of p-n junction is large and junction field is small. On the other hand if the
doping concentration in n-lype and p-type semiconductor forming p-n junction is large, the width of p-n
junction would be small and junction field would be large. It means the p-n junction will show different
behaviour by changing the doping levels on both the sides. If the width of the depletion region is about
10“^ m, the barrier electric field E for silicon p-n junction is

ww
0-7

d 10"^
= 7 X 10^ Vm ^ which is very high.

Retain in Memory

Flo
1. The p-n junction can be considered to be equivalent to

ee
FIGURE 14.13
a capacitor with p and «-regions acting as the plates of

rere
a capacitor and depletion region as the dielectric Hole o-

rFF
medium. Electron

2. Symbolically, a p-n junction is shown in Fig. 14.13. Anode Cathode


uurr
o- ■o
The direction of the arrow is from the p to the n- side. The
p-side is known as anode and n-side is known as cathode. foor
3. We can not measure the potential barrier ofp-n junction by putting a sensitive voltmeter across
ks s
its terminals because there are no free electrons or holes in the depletion layer and in the absence
Yoo
oook

of forward biasing, the depletion layer offers infinite resistance.


4. The contact potential or potential barrier at the p-n junction is given by
eBB

kT
B —
e
logg^ 2
uurr

n\
ad

where k is Boltzmann constant, T is the temperature in kelvin, N^, Nj^ are the number density of
Yo

acceptor atoms and donor atoms respectively and n, is the number density of intrinsic carriers
ii.e. electrons or holes).
dY
Re
innd

14.16. BIASING OF THE P-N JUNCTION


FFi

There are two methods of biasing the p-n junction,


(i) Forward biasing.
A p-n Junction is said to be forward biased if the positive terminal of the external battery B is
connected to p-side and the negative terminal to the n-side of p-n junction. Fig. 14.14(a).
The circuit diagram for foi^ward biasing of p-« junction is shown in Fig. l4A4(h).
In forward biasing, the applied voltage V of battery B mostly drops across the depletion region and the
voltage drop across the p-side and n-side of the p-n junction is negligible small. It is due to the fact that the
resistance of depletion region is very high as it has no free charge carriers. In forward biasing the
forward voltage opposes the potential barrier Vg. As a result of it, the potential barrier height is reduced and
width of depletion layer decreases. The effective height of the potential barrier or effective barrier potential
in forward biasing is VO, shown by solid curve in Fig. 14.14(c).
The majority carriers, electrons in the /i-region are repelled by the negative potential due to battery B
and move towards the p-/t junction. Similarly, the majority carriers, holes in the p-region are repelled by the
positive potential due to battery B, towards the junction. The positive potential of p-region attracts the electrons
14/18 ‘P'uuUe^ 4 Fundamental Physics (XII) LV»^WII

/■K FIGURE 14.14

P t Vb \ n Potential Barrier
1

o o ● !0 ©!
I
o

o <>-*● ● o-^ -0 0! o /

o o o-i^o
!e ©! o
/

le 'I
(Vb-V)
● Ch>0 o
© ©1

w
/
p n
/!
Depletion Layer
+ //
!

B B G

e
O

row
re
O
from the n-region and negative potential of /t-region attracts the holes from the /?-region. Due to it, the

eeF
Fllo
diffusion of majority carriers takes place across the junction. On crossing the junction, the number of the
electrons and holes will combine with each other. For each electron hole combination, a covalent bond in the

Fu
/)-region, near the positive terminal of the battery is broken and the liberated electron enters the positive
terminal of the battery B through lead wires. This action results in a new hole, which under the force of

srr
roF
applied voltage moves towards the p-n junction. At the other end, the electrons from the negative terminal of
the battery enter the n-region to replace the electrons lost due to the combination with the holes at the junction.

k
uor
Thus, an electric current will flow due to migration of majority carriers across the p-n junction ; which is
ofof
called forward current. The magnitude of this forward current is usually in milliampere (mA). Since the
small increase in forward voltage shows the large increase in forwitrd current, hence the resistance of p-n
kos
Y
junction is low to the flow of current when forward biased,
Yo
eerBB
oo

(ii) Reverse biasing.


rY

A p-n junction is said to be reverse biased if the positive terminal of the external battery B is
connected to n-side and the negative terminal to p-side of the p-n junction, Fig. 14.15(a).
u

The circuit diagram for reverse biasing of p-n junction is shown in Fig. 14.15(/7). In reverse biasing, the
ou
o
ad
d

applied voltage V of battery B, mostly drops across depletion region of p-n junction and its direction of
voltage is same as that of potential barrier. Due to it, the reverse bias voltage supports the potential barrier. As
nY

a result of it, tlie barrier height increases and width of depletion region increases. The effective barrier height
or barrier potential under reverse bias is (Vg + V), shown by solid curve in Fig. 14.15(c).
nid
Re
F

FIGURE 14.15
Fi

' V/ \
/
Vs \
P I n
T
-► O ●4-0 eee Potential Barrier

O'4-O o eee eee —► o

eee ®e® ■4-0


I o● ●4-0o

O

●-►O
I r —

©0© ®e® y
Vb + V ' /
Vb
I h

►1
I
O ■40 ©00 ®®©

P n
h- /j
/!

Depletion Layer - -y
/

e
B
■f
B
\ \ \
O
O
SEMICONDUCTOR ELECTRONICS MATERIALS, DEVICES AND SIMPLE CIRCUITS 14/19

In reverse biasing, there is no conduction across the junction due to majority earners. However, a few
minority carriers (holes in n-section and electrons in p-section) of p-n junction diode cross the junction after
being accelerated by high reverse bias voltage. They constitute a current that flows in the opposite direction.
This is called reverse current or leakage current. Since the lai'ge increase in reverse voltage shows small
increase
in reverse current, hence the resistance of p-n junction is high to the flow of current when
reverse biased. The reverse current is not limited by the magnitude of the applied voltage but is limited due
to concentration of minority carriers on either side of the junction.

w
It is essential to note that the potential barrier opposes the forward current and supports the reverse
cunent. The voltage of external battery for forward biasing is low (about 1-5 V) and for reverse biasing is
high (10 to 25 V).

Retain in MemoryJ

e
row
re
I. In germanium diode, the ratio of reverse to forward resistance is 40000 : 1, while for silicon, this
ratio is 10^ : 1.
2. \np-n junction, there is a diffusion of majority carriers across the junction in forward biasing and

eeF
ullo
drifting of charge carriers in reverse biasing.

FF
3. If the forward current flowing in/7-n junction is more than the rated value of the p-n semiconductor,
then the p-n junction will be damaged or destroyed due to over heating.

srr
roF
14.17. CHARACTERISTICSOF A P-N JUNCTIONDIODE

k
There are two types of characteristics of a p-n junction diode :
uor
ofof
(/) Forward characteristics (//) Reverse characteristics
(/) Forward characteristics.
kos
Y
These are the graphical relations between forward bias voltage applied to p-n junction and the
Yo
eerBB
oo

forward current through the p-n junction.


rY

We connect the external battery B with potential dividing arrangement, to the p-n junction as shown in
Fig. 14.16(a), so that p-n junction is forward biased.
u

For the given low forward bias


ou

voltage (noted from voltmeter V), note


o
ad
d

the corresponding forward current


nY

(from milliameterniA), which is due


to migration of majority carriers across
nid

mA
the p-n junction. Go on increasing the
Re
F

various values of forward bias voltage


Fi

and note the corresponding forward


currents. On plotting a graph between
forward bias voltage and forward
current, we get the curve OAB. This is A/VVWVW
the forward characteristics of the type
shown in Fig. 14.16(/?). It is found that
beyond forward voltage F = called
knee voltage (which is 0.3 V for Ge B
and 0.6 to 0.7 V for Si) the conductivity
is very high. It is at this value of e
farward biasing voltage for a p-n
junction, that the potential barrier is overcome and beyond it, the current increases rapidly with increase in
forward voltage. If forward voltage is increased beyond a certain safe value {i.e., rated value for the given p-
n junction), an extremely large current will be produced which may destroy the p-n junction diode due to
overheating. Germanium p-n junction can tolerate a temperature around 100"C and silicon p-n junction can
tolerate a temperature upto 170°C.
14/20 « Fundamental Physics (XII) twin

/ .. t t Voltage. It is that forward voltage beyond which the DO YOU KNOW ?


current through the junction starts increasing rapidly with ● The barrier eleciric field for
voltage, showing the linear variation. But below the knee
silicon p-n junction is very
voltage the variation is non-linear.
high (=7x10^ V/m).
(H) Reverse characteri.stics. ● The drifting of electrons from
These are the graphical relations between the reverse bias n to p side and drifting of
holes from p to n side of p-n
voltage applied to the p-n Junction and the reverse current
junction will make the
across the p-n junction.
conventional current from p to
We connect the external battery B with potential dividing n side across

arrangement top-n junction as shown in Fig. 14.17(a), so that p-/i junction p-n junction.

ww
is reverse biased. ● The height of potential barrier
FIGURE 14.17 in p-n junction diode is
proportional to temperature in
kelvin.

Flo
● An ideal junction diode acts

e
Reverse Bias (V) as a perfect conductor when it

ree
is forward biased and a perfect
-8. -6,1 -4.e-2d

Fr
1 0 $ insulator when it is reverse

rF
(0
B 2
3
cn
biased.
uurr F
o

5 O
● An ideal junction diode acts a

for
c

closed switch when forward


■VvVvVvW 10
<D
3
biased and open switch when
s
kks
●15 reverse biased. It means an
Yo
ideal junction diode acts like
oooo

H H H H
an automatic switch.
B
eB

o o
For the given reverse bias voltage (noted from the voltmeter V) applied to the p-n junction, note the
ur

reverse current (from the micro-ammeter |i A), which is due to migration of minority carriers across the p-n
ad

junction. Go on increasing the reverse bias voltage and note the corresponding reverse current. On plotting a
YYo

graph between reverse bias and reverse current, we get the reverse characteristics as shown in Fig. \A.\l{b).
From the characteristic curve we note that in reverse biasing of p-n junction diode, the reverse current is very
dd

small («\i A) and is voltage independent upto certain reverse bias voltage, known as breakdown voltage. It
Re

is called reverse saturation current. If the reverse bias voltage is equal to OB (i.e., breakdown voltage), the
in

reverse current through the p-n junction will increase abruptly. If this current exceeds the rated value of p-n
F

junction (specified by the manufacturer), the p-n junction will get damaged.
In general, the simple p-n junction diodes are never used beyond the reverse saturation current region,
otherwise they get burnt out due to heavy currents.
eV
The relation for the current / in the junction diode is given by / - /q exp. kT

where k is Boltzmann constant, k= 1 -38 x 10“^^ J mol“ ' K" * ; /q reverse saturation current.
In forward bias; V is positive, and low. Then > > 1, then forward current,
e I AV kT
AI /
AV kT ^
or
kT Al el
f
At room temperature 27°C (= 300 K), the resistance of p-n junction is
AV^ kT (l-38xlO~^^)x3QO 26x10"^
R
(l-6xl0"’^)x/f
ac
AI el I
f f
SEMICONDUCTOR ELECTRONICS MATERIALS, DEVICES AND SIMPLE CIRCUITS 14/21

In reverse bias', Vis negative and high, so < I, then reverse current. =
Notes : 1. It is clear from the V-I characteristics of p-n junction diode that p-n junction allows the
current
to pass only when it is forward biased. So when alternating voltage is applied across the junction
diode, cuiTent flows only during that part of the cycle when it is forward biased. Due to this reason, the p- n
junction diode can be used as a rectifier.
2. In a p-n junction diode change in temperature due to heating, affects the overall V-I characteristics of
p-n junction.
Sample Problem In a p-n junction diode the reverse saturation current is 10“^ A at 27®C.
Find the forward current for a voltage of 0-2 V. Given exp. {7*62) = 2038-6, k = 1-4 x 10-^^ JK”*.
Sol. The current in the junction diode is given by / = /q ij
where. Iq - reverse saturation current = 10"^ A ; T = 21 + 213 = 300 K.
l-6xi0"'^x0-2

ww
14xl0“^^x300
I = \e -1] = 10-^ ^ 10^5 [2038-6 - 1]
= 2037-6 X 10-5 ^ 2-0376 x lO'^ = 2-04 x 10‘2 A

Flo
14.18. DYNAMIC RESISTANCE OR A.C. RESISTANCE OF THE JUNCTION DIODE

e e
Dynamic resistance or ax. resistance ofjunction diode or incremental resistance is deifned as

reer
the ratio of a small change in voltage AV applied across the p-n junction to a small change in

rFF
junction current AI i.e.
uur r
In Fig. 14.16(/?),
AV _ DE
AI ~ CE In Fig. l4Al{b), ffoor
AV
AI
DE
CF
sks
In the forward characteristics of p-n junction diode, beyond the knee point, there is almost a linear
YYoo
region of the characteristics, showing that is independent of V above the knee point. The value of Rj in
ooko

forward bias beyond knee point is low (about few ohms) and is quite high (about mega ohm) in reverse bias.
eBB

Sample Problem The V-I characteristics of silicon diode is shown in Fig. 14.5(b) and 14.6(b).
Calculate the diode resistance in (a) forward bias at V = + 0*9 V (b) reverse bias V = - 3-0 V.
uurr

Sol. In forward characteristic for voltage V = -^0-9 V


ad

AV=DE= 1-0-0-80 = 0-2 V; A / = C£ = 10- 6 = 4 mA


Yo

.●. Forward resistance, R = AV _ 0-2V = 5on


/
dY

AI 4 mA
Re

In reverse characteristics for voltage V = -3V ;AV = -2-(-4) = 2V; A/=3-2=l|iA


innd
FFi

.'. Reverse resistance, R, =


AI IpA

14.19. P-N JUNCTION DIODE AS A RECTIFIER

Rectifier is a device which is used for converting alternating current/voltage into direct
current/voltage.

The process of converting altematuig current/voltage into direct cuixent/voltage is called rectification.
A p-n junction can be used as a rectifier in two ways :
(a) Half wave rectifier
(/?) Full wave rectifier
(a) n-n Junction diode as a half wave rectifier.
This rectifier converts one half a.c. into d.c.
Principle. Its working is based on the fact that the resistance of p-n junction becomes low when forward
biased and becomes high when reverse biased.
14/22 p>icuUzp.'^ Fundamental Physics (Xlljtitsiau
Circuit diagram. A.C. voltage to be rectified is connected to the primary ?2 of a stepdown transformer.
5] $2 is the secondary coil of the same transformer. 5] is connected to the portion p of the p-n junction. $2 is
connected to the portion n through load resistance R. Output is taken across the load resistance R. Fig.
14.18(a).

ooww
e
ree
rFl
Fre
rrF
Working. During the positive half cycle of the input A.C., suppose Pj is negative and P2 positive.
On account of mutual induction, 5, becomes positive, $2 becomes negative. The p-n junction is forward
ouur
sffoo
biased. The resistance of p-n junction becomes low. The forward current flows in the direction shown by
arrow heads. Thus, we get output across-load.
okks
During the negative half cycle of the input A.C., Pi is positive and P2 is negative. On account of
Yo
ooo

mutual induction, becomes negative and ^2 is positive. The p-n junction is reverse biased. It offers high
eBB

resistance and hence there is no flow of current and thus no output across load. The process is repeated. In the
output, we have current corresponding to one half of the wave, the other half is missing. The output voltage
is of the type shown in Fig. 14.18(/j).
uurr

That is why the process is called half wave rectification. It is not of much use. The output signal is
ad
YYo

available in bursts and not continuously.


Important notes for half wave rectifier
dd

1. The output obtained from p-n junction as a half wave rectifier is unidirectional, intermittent and
Re
iinn

pulsating d.c.
2. The efficiency of half wave rectifier is small.
F

3. The output d.c. voltage = mean load current x load resistance,

t.e.. V -1. xR, =—xP,


dc
dc L ^ L

where Iq is the maximum value of secondary half wave current.


4. The frequency of output voltage/current is equal to frequency of input supply voltage/cunent.
(b) p-n junction diode as full wave rectifier.
This rectifier converts both the halves of a.c. into d.c.
Circuit diagram. For full wave rectification, we have to use two p-n junctions diodes D^ and D2. The
arrangement is shown in Fig. 14.19.
Working. During the positive half cycle of the input A.C., the junction diode D, is forward biased as
shown in Fig. 14.19(a), and the />/; junction diode £>2 is reverse biased. The forward current flows on account
of majority carriers of junction diode D| in the direction shown.
SEMICONDUCTOR ELECTRONICS MATERIALS, DEVICES AND SIMPLE CIRCUITS 14/23

During the negative half cycle of input A.C.,


the junction diode £)j is reverse biased, and the
Junction diode D2 is forward biased, Fig. \4.\9{b).
The forward current flow.s on account of majority
carriers of Junction diode Z>2. We observe that during
both the halves, current through R flows in the same
direction. The input and output waveforms are shown
in Fig. 14.19(c). The output signal voltage is
unidirectional having ripple contents, i.e., d.c.
components and a.c. components of voltage. It can
be made d.c. voltage by filtering through a filter

ooww
circuit, before it can be put to any use.
Important notes for full wave rectifier
1. The output obtained from p-n Junction as a
full wave rectifier is unidirectional continuous

e
with ripple contents, which is not perfectly d.c.

re
voltage/current.

rFFl
ree
2. The efficiency of full wave rectifier is high.

F
3. The output d.c. voltage = mean load current

rF
X load resistance

fsfoor
ouur
21
0
I.e., V = /,dc xR,L =
dc
71
kosk
This value is twice of the output voltage
Yo
obtained in half wave rectifier.
oo
Y

4. The frequency of output voltage/current is


BB

twice the frequency of the input supply voltage/


rre

current.

Filter Circuits
ouu
Y

Filter circuits are used to smoothen the


ad

fluctuating or pulsating voltage obtained from the


dY

rectifier. In fact filler circuit is a device which is used


to filter out the a.c. components from the output of
innd

rectifier.
Re

The commonly used two simple filter circuits are discussed below :
Fi
F

1. Inductor Filter. If consists of an inductor L of


high inductanceconnected in series with a load resistance FIGURE 14.20

as shown in Fig. 14.20(a). L


Here an inductor of self inductance L offers a 7}
TOutput of ^
reactance to the current flowing through it, which is Input Filtered d.c.
a.c. rectifier output
given by 0
1
= (oL = 2nv L
For d.c., V = 0, therefore = 0 O
Output Voltage
For a.c. v = finite, therefore, = finite A

As a result of it, the a.c. component of output of the


rectifier is obstructed by inductor and d.c. component of o
output of the rectifier will pass through inductor. Due to it,
a smooth d.c. voltage appears across the load resistance >
0
Fig. U.20{b). Time
14/24 Fundamental Physics (XII) twin

2. Capacitor Filter. It consists of a capacitor C of high capacitance connected in parallel with its load
resistance as shown in Fig. 14.21(a).
Here the capacitor of capacitance C offers a reactance to the current flowing through it, which is given
by
FIGURE 14.21
1
c 73 i
coC 27tvC
Input Output of Filtered d.c.
=tc ^Rl
o

1 a.c. rectifier output


For d.c., V = 0, therefore = eo
(infinity) for <D
t 5
0

a.c. V = finite, therefore X^ is small but finite. As a result of o


it. the a.c. component of output of the rectifier is by passed Output Voltage
or filtered out and the d.c. component of output of the

ww
rectifier is obstructed which appears across the load
o
resistance /?^, The output voltage will be as shown by thick
solid curve in Fig. 14.21(/>), which is almost constant.

Flo
Output d.c. voltage. o Time
Such filter is widely used in power supplies

ee
rere
value of a.c. component I E (I r.m.s.

rFF
a.c. — a.c. _
-1
Ripple factor of a rectifier =
value of d.c. component /
d.c.
E
d.c J.c.
uurr
Ripple factor (r) of a full wave rectifier using capacitor filter is given by, r = foor I
ks s
Where is the load resistance, C is the capacitance and v is the frequency of alternating voltage to be
Yoo
oook

rectified.

It can be shown that ripple factor for half wave rectifier is 1.21 and for full wave rectifier is 0.48.
eBB

output d.c. power X100%


Efficiency of rectifier, T) = -
input a.c. power
uurr

For half wave rectifier, the value of rectification effeciency is 40-6% and for full wave rectifier is 8T2%.
ad
Yo

Retain in Memory
dY

1. In half wave rectifier


Re

is r.m.s. primary voltage, then maximum primary voltage, Vpm = V2V


innd

If Vrnis nm
FFi

Maximum secondary voltage, ,V0 = V =V


sm pm n
P

Maximum secondary current, Iq = V(R, + /?^)


I
_ 0
rms value of current, I ms
2

I /
_ 0 .
d.c. value of current, / d.c. Output d.c. voltage = / d.c.
n n

Output r.m.s. voltage = V(/2


d.c. output power
Rectification efficiency = xl00%
a.c. input power

Power efficiency = d.c. output power xl00%


a.c. input power for half cycle
SEMICONDUCTOR ELECTRONICS MATERIALS, DEVICES ^ND SIMPLE CIRCUITS 14/25

2. In full wave recUHer


n n

Maximum secondary voltage, V n


= ^ Vx —^ n
p p

110 _
I
"0
Full wave rectified current, le., mean load current, / d.c.
.
; / rms
K
21.0
Output d.c. voltage = /^^. x = XR^.
n

Form factor = -
rms

^dc
The value of form factor for half wave rectifier is 1 -57 and for full wave rectifier is M1.

w
DO YOU KNOW ?

Flo
1. A junction diode when used as a rectifier merely converts a.c. into d.c. but its input and output circuits are
not separate.

e
rree
2. A junction diode cannot work as an amplifier, i.e., it cannot increase the amplitude of signal current,
voltage or power.

r FF
uurr
14.20. SPECIAL PURPOSE P-N JUNCTION DIODES
for
Some devices which are basically junction diodes are developed for different applications. The important
kss
devices are as follow.s :
ooook

(a) Zenerdiode (/?) Photodiode (c) Light emitting diode (LED) (r/) Solar cell,
Yo

(a) ZENER DIODE


FIGURE 14.22
{>
eB

It was invented by C-Zener. It is design spiecially


o
to operate in the reverse breakdown voltage region
urr

continuously without being damaged.


ad

Zener diode is fabricated by heavily doping both


Yo

p-side and n-side of thep-n junction. As a result of it, +


dY

the depletion region of Zener diode is very thin


(< 10“^ m) and the electric field set up across the +
Re
innd

Vs^ H I
junction is extremely high.
Fi

O o
For example if a reverse voltage of 5 V is applied
across the/?-/i junction, the electric field set up across Forward Current (mA)
the p-n junction is,

-dV _ 5V
£ = = 5x lO^V/m. /
dx 10"^ m

This electric field is called ionized field. The Vz


symbol for zener diode is shown in Fig. 14.22(a). Reverse Forward
Voltage (V) - Current (I) Characteristics of Voltage (V) Voltage (V)
Zener diode

A circuit diagram to study the characteristics of o


a zener diode is shown in Fig. I4.22(/>) and 22(c). T Reverse
The V-I characteristics of zener diode is shown
Current (pA)
in Fig. \4.21{d).
14/26 'Pnadcc^ '4, Fundamental Physics (XII)

From V-I characteristics of zener diode we note that in reverse biased zener diode, when V= V. (i.e.,
zener voltage) a sudden large current flows in the circuit and the zener voltage V. remains constant for large
increase in reverse current. This feature of zener diode that the voltage drop across it is independent of
current flowing through it, has been used in making the zener diode as a d.c. voltage regulator or voltage
.stablizer.
the reverse break down voltage {or zener voltage)
Cause of sudden Increase in tlie reverse current at
When the reverse voltage across the/?■« junction increases, the electric field across the junction increases.
When reverse voltage becomes break down voltage, the electric field across the junction becomes very
high. This electric field will pull the minority carriers electrons from the parent atoms in the /7-side of
junction and accelerate them to cross-over the junction to reach the n-side of junction. This is responsible
for a sudden increase in the reverse current. This method of emission of electrons is called internal field

ww
emission.

Zener diode is available having zener voltage of 24 V to 200 V. Their power rating (i.e., maximum
power dissipation = zener breakdown voltage (V.)x maximum zener current (/.), vary from 150 mW to SOW.
For a proper working of a zener diode in any circuit, it is essential that

Flo
e
(fl) the zener diode must be reverse-biased.

ere
(b) the zener diode must have voltage greater than zener break down voltage (V^).

FFr
(c) the zener diode is to be used in a circuit where the current is less than the maximum zener current (Z^)
limited by power rating of the given zener diode.
uurr
orr
Zener diode as Voltage Regulator
sfo
One of the most important use of zener diode is in making the constant voltage power supply. The
potential barrier of zener diode lies between 0-1 V to 0-3 V approximately. Its working is based on the fact
kks
that in reverse breakdown (zener) region, a very small change in voltage across the zener diode produces a
Yo
very large change in current through the circuit but the voltage across the zener remains constant.
oooo

The circuit detail is shown in Fig. 14.23(a). Here


eB

the zener diode is joined in reverse bias to the


FIGURE 14.23
fluctuating d.c. input voltage through a resistance R of R

suitable value, depending upon the zener voltage and


urr

power rating of zener diode used. The constant output


ad

-t- +
N
YYo

voltage is tiiken across a load resistance connected Fluctuating Constant


Rl
in parallel with zener diode. D.c. Input Voltage Output Voltage

Working. When the input d.c. voltage across


dd
Re

zener diode increases beyond a certain limit {i.e., zener


inn

break-down voltage), the current through the circuit


e
F

rises, causing an increase in the voltage drop across


the resistor R without any change in the voltage across ^04
zener diode. As a result of it, the voltage across the Regulated
zener diode remains constant, even though current output voltage
through zener diode changes. Vz
When the input d.c. voltage across zener diode o
decreases, the current through the circuit goes down,
causing a decrease in the voltage across the resistor R
without any change in the voltage across zener diode. ●►V,-
As a result of it, the voltage across the zener diode Vb
remains constant. Therefore zener diode acts as a
voltage regulator.
A graph showing the variation of output voltage (V^) versus input voltage (V,) for a zener diode is shown
in Fig. \4.23{b). From graph we note that, the output voltage remains constant (= Vp after the reverse break
down voltage V^.
SEMICONDUCTOR ELECTRONICS MATERIALS, DEVICES AND SIMPLE CIRCUITS 14/27

Important points 1. In order to have a good load regulations, the value of series resistance R should be
such that the current through the zener diode is much larger than the load current. For this, we select, the
zener current five times the load current.

2. If two zener diodes each of breakdown voltage V are connected in series, then the effective breakdown
voltage of series combination of the diodes is 2 V.
(b) PHOTODIODE

Photodiode is a p-n junction diode which is an optoelectronic device in which current carriers
are generated by photons through photo excitation, Le., photoconduction by light.

A photodiode is fabricated with a transparent window to allow light to fall on it.


A photodiode is a special p~n junction diode made of photosensitive semiconducting material, whose
function is controlled by the light allowed to fall on it. In fact photodiode is an electrical device used to detect

ww
and convert light into an energy signal through the use of photodetector. It is operated under reverse bias
below the breakdown voltage. The conductivity of />-« junction photodiode is modulated by the absorption of
incident light in or near the depletion layer which exists at the p-n junction. The conductivity, of the

Flo
p-n junction of photodiode, increases with the increase in intensity of light falling on it.
Symbolically a photodiode is shown in Fig. 14.24(a). Fig. 14.24(Z?) shows an experimental arrangement

e
for the study of voltage (V)-current (I) characteristics of a photodiode in which the photodiode is reverse

rere
biased.

r FF
FIGURE 14.24
Light
uurr
for
ahiA

Reverse Voltage O
♦ volt
kss
Dark Current
ooook
Yo
o?
l2—-
Ma
eBB

'3 S 5
3 cn
p-side n-side CD

I3 > I2 > li
urr

> ’ pA
e
ad

o o
Yo
dY

When the photodiode is reverse biased with a voltage less than its breakdown voltage and no light is
incident on its junction, the reverse current is extremely small (almost negligible). This current is called dark
Re
innd

current.
Fi

When visible light of energy greater than forbidden energy gap {Le. hv > EJ is incident on a reverse
o

biased p-« junction phot*»diode, additional electron-hole pairs are created in the depletion layer (or near the
junction). These charge carriers will be separated by the junction field and made to flow across the junction,
creating a reverse current across the junction. The value of reverse saturation current increases with the
increase in the intensity of incident light as shown in Fig. 14.24(c), which is the V-/characteristics of photodiode.
It is found that the reverse saturation current through the photodiode varies almost linearly with the
light flux or light intensity.
Photodiodes are used for following purposes :
1. In photodetection for optical signals.
2. In demodulation for optical signals.
3. In light operated switches, Le., In switching the light on and off.
4. In optical communication equipments.
5. In logic circuits that require stability and high speed.
6. In reading of computers, punched cards and tapes, etc.
14/28 'P^Adce^'A Fundamental Physics (XII)BS29D

Retain in Memory
The photodiodes are used in reverse bias condition because the change in reverse current
through the photodiodedue to change in light flux or light intensity can be measured easily as the
reverse saturation current is directly proportional to the light flux or light intensity. But it is not so
when photodiode is forward biased.

(c) LIGHT EMITTING DIODE (LED).

Light emitting diode is a photoelectronic device which con verts electrical energy into light energy,
underforwardbias condition.

It is a heavily doped p-n junction diode which under forward bias emits spontaneous radiation.
The diode is covered with a transparent cover so that the emitted light may come out.

ww
Symbolically LED is shown in Fig. \4.25(a). The construction and circuit used for working with LED
is shown in Fig. \425{b).

Flo
FIGURE 14.25

e
1

eree
P P R

FFr
oorr
uur r
B
o
sf
♦V
sk
Yoo
o
oook
eBB

In an LED, the upper layer of p-type semiconductor is deposited by diffusion on n-type layer of
semiconductor. The metallised contacts are provided for applying the forward bias voltage to the junction
diode from battery B through a resistance R which controls the brightness of light emitted.
uurr
ad

Working of LED is discussed below :


Yo

When a p-n junction is forward biased, the size of depletion layer decreases. The movement of majority
carrier takes place across the junction. The electrons move from n-side to p-side through the junction and
dY

holes move from p-side to n-side through the junction. As a result of it, the concentration of the minority
Re
innd

carriers increases rapidly on the two sides of the junction boundary. These excess minority carriers on either
side of the junction boundary recombine with majority carriers there. In each recombination of electron and
FFi

hole the electron will fall from higher energy state to ground state. As a result of it, a photon is released whose
energy is nearly equal to the energy gap E^. If X is the wavelength of photon emitted, then
he he
E or X =
« X

For a p-n junction of Ge and Si, the larger percentage of this energy released is mainly transferred into
thermal energy of the vibrating lattice. As a result of it, no light is emitted. However in some other p-n
junction semiconductor diodes made of materials like gallium arsenide (GaAs),gallium phosphide (GaP) and
gallium-arsenidephosphide (GaAsP) a greater percentage of energy released is in the form of viside light.
Due to it, the junction becomes a light source, i.e., a light emitting diode (LED). The colour of light emitting
diode depends upon the type of material used in making the semiconductor diodes as given below:
1. Gallium-arsenide(GaAs) — infrared radiation. 2. Gallium-phosphide (GaP) — red or green light.
3. Gallium-arsenide-phosphide (GaAsP) — red or yellow light.
LEDs emit no light when reverse-biased.Rather the LEDs will be destroyed when reverse biased.
SEMICONDUCTOR ELECTRONICS MATERIALS, DEVICES AND SIMPLE CIRCUITS 14/29

When the forward current of the junction diode is small, the intensity of light emitting diode is small. As
the forward current increases, intensity of light also increases and reaches a maximum value. Beyond this
stage if forward current is further increased, the Intensity of light starts decreasing. During working LEDs are
biased such that the light emitting efficiency is maximum.
Note that: (0 The frequency of light emitted by an LED is related to the band gap of the semiconductor
used in LED, i.e., a type of material used in making the LEDs.
(ii) The intensity of light emitted by LED depends upon the doping level of the semiconductor used.
Important considerations required while fabricating a p-n junction to be used as a light emitting
diode (LED) are as follows :
(/) The reverse breakdown voltages of LEDs are generally low, around 5 V. That is why a care must be
taken while fabricating a p-u junction diode so that the high reverse voltages do not appear across LEDs.
(;7) There is a very little resistance to limit the current in LEDs. Therefore, a resistor must be used in

ww
series with LED to avoid any damage to it.
The voltage (V) - current (I) characteristics of LED is simitar to that of junction diode. For LEDs,
the threshold voltages are much higher and slighly different for different colours; Fig. 14.25(c).

Flo
USES

e
1. In Burglar-alarm systems, infrared LEDs are used.

rere
2. In calculators and digital watches, LEDs are used for numeric displays.

r FF
3. In the field of optical communication, where high radiance GaAs diode are matched into the optical
fibre cable.
uurr
foor
4. In computers LEDs are used in optical mouses for the computers. They are also used in computer
memories and in optical communication.
kss
5. In picture phones and video displays. LEDs are used in image sensing circuits for picture phones
Yoo
ooook

and video displays.


6. In traffic light. LEDs are used to illuminate the traffic lights.
eBB

7. In remote control. LEDs are used to transmit information to the electronic gadgets.

Advantages of LED^s over incande.scent lamp are as follows :


uurr

1. LED has less power and low operational voltage.


ad
Yo

2. LED has fast action and requires no warm up time.


dY

3. LED is cheap and easy to handle.


4. LED can be used for variety of uses ; e.g. in burglar alarm system, in the field of optical communication,
Re
innd

in digital watches etc.


Fi

Retain in Memory
The semiconductor used for fabrication of visible LEDs must have a band gap of 1 -6 eV because
the wavelength of visible light is from 4000 A to 7800 A, having energy 3 eV to 1-6 eV.

{(i) SOL.AR CELL. Solar cell is basically a solar energy converter. It is a special p-n junction device
which converts solar energy into electric energy. A solar cell is shown symbolically in Fig. 14.26(n) and in
construction along with circuit in Fig. 14.26(/>).
A solar cell consists of a silicon or gallium-arsenide p-n junction diode packed in a can with glass
window on top. The upper layer is of p-type semiconductor. It is very thin so that the incident light photons
may easily reach the p-n junction. On the top face of p-layer, the metal finger electrodes are prepared in
order to have enough spacing between the fingers for the light to reach the p-n junction through />-layer.
When photons of light (of energy hv > E^) fall at the junction, electron-hole pairs are generated in the
depletion layer (or near the junction). The electrons and holes produced move in opposite directions due to
junction field. The photo generated electrons move towards n-side of p-n junction. The photo generated holes
14/30 '4. Fundamental Physics (XII) PTSTim
move towards p-side of p-n junction. They will be collected at the two sides of the junction, giving irse to a
photo voltage between the top and bottom metal electrodes. The top metal contact acts as positive electrode
and bottom metal contact acts as negative electrode. When an external load is connected across metal electrodes
a photo current flows.
The electric current (/) in the circuit increases linearly with the increase in the intensity of sunlight.
Fig. 14.26(c)
FIGURE 14.26

Light Metal Finger


Electrodes
Glass
aI
X ■0 +

ww
p Vqc
0 ♦V
A

Floo
N

6-
^ ^SC
Intensity—► B

e
e

eere
Metal Contact
o o o
■O-

FFr
The /-V characteristics of a solar cell is shown in Fig. 14.26(c0. which is lying in fourth quadrant of

oorr
uur r
not draw current but supplies the same to the load. In
the coordinate axes. It is so because solar cell does s ff
graph point A represents open circuit voltage and point B represents short circuit current.
USES
sk
YYoo
1. Solar cells are used for charging storage batteries in day time, DO YOU KNOW ?
ooko

which can supply the power during night times. 1. Solar cells are prepared from
eBB

2. The solar cells are also used in artificial satellite to operate the elemental semiconductors like

various electrical instruments kept inside the satellite. Si (Eg = M eV), CuInS2
3. They are used for generating electrical energy in cooking food
uurr

(E^ = 1-04 eV), Ga As (E^ =


ad

and pumping water. 1-43 eV),CdTe(£ =1-45 eV),


Yo

4. Solar cells are used in calculators, wrist watches and light etc.

meters (in photography). 2. A solar cell can supply current


dY

5. Solar cells are used to produce electric power in remote areas, only if the incident light of
Re

energy hv is greater than the


nind

where power from the electric power supply is not available.


energy gap (£^) of the junction
FFi

6. Solar cells are used to power traffic signs. diode.


7. Solar cells are used in remote radiotelephones.

Retain in Memory
● A p-n junction can act as a junction laser provided the opposite faces of the p-n junction crystal
are flat and parallel so that the light produced due to electron-hole recombination in and around
the depletion region can be reflected back and forth within the crystal.
● The junction lasers are used in optical communication systems.

14.21. JUNCTION TRANSISTOR

A junction transistor is obtained by growing a thin layer of one type semiconductor in between two
thick layers of other similar type semiconductor. Thus, a junction transi.stor is a semiconductor device having
two junctions and three terminals.
The two types of junction transistors are p-n-p junction transistor and n-p-n junction transistor.
SEMICONDUCTOR ELECTRONICS MATERIALS, DEVICES AND SIMPLE CIRCUITS 14/31

A junction transistor is obtained by growing a thin layer of n-type semiconductor in between two
relatively thick layers ofp type semiconductor. A n-p-n junction transistor is obtained by growing a thin layer
of p-type semiconductor in between two relatively thick layers of n type semiconductor.
A brief description of three layers of a transistor is given below :
Emitter (E). It is the left hand side thick layer of moderate size of the transistor which is heavily doped.
Base (B). It is a central thin layer of transistor which is lightly doped.
Collector (C). It is the irght hand side thick layer of large size than that of emitter of the transistor ;

w
which is moderately doped.
The function of emitter is to emit the majority carriers. Function of collector is to collect the majority
carriers. Base provides the proper interaction between the emitter and the collector.
Symbolically, the two types of

e
transistors are represented in Fig.

row
re
14.27. The direction of arrowhead
indicates the direction of flow of
positive charge or conventional

eeF
Fllo
current.

Fu
In case of p-n-p transistor, Fig.
14.27(a), the arrowhead is inwards,

srr
because majority carriers are holes.

roF
In case of n-p-n transistor. Fig.

k
14.27(/?), the arrowhead is outwards,
uor
because
electrons.
majority carriers are ofof
kos
Y
A junction transistor is so named because it is a device which can transfer the reresistance by
Yo
eerBB
oo

interchanging the biasing across the junctions.


rY

The transistor is a current driven device, in which the emitter current controls the collector current.
u

Retain in Memory
ou
o
ad
d

l. The emitter is heavily doped and of moderate size.


nY

2. The collector is moderately doped and of large size.


3. The base is lightly doped and of small size.
nid
Re

14.22. TRANSISTOR ACTION OR WORKING OF JUNCTION TRANSISTOR


F
Fi

FIGURE 14.28
(a) p-n-p Transistor. The
pnp
experimental arrangement is shown in P n P
E C
Fig. 14.28. The emitter base junction
is forward biased. It means the positive B
pole of emitter base battery is E B c
connectedto emitter, and its negative -le Ic
. k u ”Ib
'Tc
pole to the base. Collector base ' ' lb —(le—Ic)
junction is reverse biased, i.e., the le Ic
negative pole of the collector base I—♦ < ■ < h H
I, Ic
battery Wqq is connected to collector Vee Vcc Vee Vcc
and its positive pole to the base.
o o
The resistance of emitter base junction is very low. So the voltage of (i.e., is quite small (= 1 -5 volt).
The resistance of collector base junction is very liigh. So Uie voltage of (i.e. V(^g) is quite large (* 45 volt).
Holes which are majority carriers in emitter (p-type semiconductor) are repelled towards base by positive
potential on emitter due to battery resulting in emitter current 1^. The base being thin and lightly doped
14/32 4 Fundamental Physics (XII) iWII

(/Kype semiconductor) has low number density of electrons. When holes enter the base region, then
only a few holes (says 5%) get neutralised by the electron-hole combination, resulting in base current
(= 5% = 0 05 /g). The remaining 95% holes pass over to the collector on account of high negative
potential of collector due to battery resulting in collector current (= 95% 1^ = 0-95 7^).
As one hole reaches the collector, it is neutralised by the flow of one electron from the negative terminal
of the battery to collector through connecting wire. At the same time a covalent bond is broken in the
emitter, the electron goes to the positive terminal of the battery through connecting wire and hole produced
begins to move towards base. Then one electron flows from negative terminal of battery to positive
terminal of battery When the hole coming from emitter combines with the electron in base, the deficiency
of electron in base is compensated by the flow of electron from negative terminal of battery to the base
through connecting wire. Thus, the current in p-n-p transistor is carried by holes and at the same time their

ooww
concentration is maintained. But in the external circuit the current is due to flow of electrons. The direction of
conventional current in the various arms of the circuit has been shown by arrow heads in the Fig. 14.28(£>).
Thus, in this case. 7, = /^ + /,.
In the base and flow in opposite directions.
ib) ii-p- Va.’sistoi. In this case also, the emitter base junction is forwimd biased, i.e., The positive

e
ree
pole of emitter base batteiy is connected to base and its negative pole to emitter.

rFl
The resistance of emitter base junction is very low. So the voltage of V^f (i.e., is quite small

Fre
(« I -5 V). The collector base junction is reverse biased, i.e., the positive pole of the collector base battery

rrF
is connected to collector and negative pole to base. The resistance of this junction is very high. So the
voltage of (l e. V(^q) is quite large (= 45 V).
ouur
negative potential of sffoo
Electrons which are majority carriers in emitter (/i-type semiconductor) are repelled towards base by
on emitter, resulting emitter current 7^. The base being thin and lightly doped
(/7-type semiconductor) has low number density of holes. When electrons enter the base region, then only
okks
a few holes (say 5%) get neutralised by the electron-hole combination, resulting in base current
Yo
oo

7^ (= 5% 7^ = 0 05 7g). The remaining 95% electrons pass over to the collector, on account of high positive
Y
BB

potential of collector due to battery V(^q, resulting in collector current (= 95% 7^ = 0-95 7^.
As one electron reaches the
rre

FIGURE 14.29
Emitter Collector
collector, it flows to the positive Junction Junction
terminal of battery through
ouu

n non
n
YY
ad

connecting wire. At the same lime, E C


one electron flows from negative
terminal of to positive terminal
dd

of V££ and one electron flows from E B C


Re
iinn

negative tenninal of V££ to emitter. ' ’ le -Ib C 4 i

When the electron coming from ^ ‘ Ib -(le~Ic) i k Ic


F

emitter combines with the hole in


base, the deficiency of hole in base >-
is compensated by the breakage of O Vee 1, Ic
covalent bond there. The electron so
Vcc O Vee 1
C Vcc

released Hows to the positive terminal of battery V££, through connecting wire. Thus, in n-p-n transistor, the
current is earned inside the transistor as well as in external circuit by the electrons. The direction of conventional
current in the various arms of the circuit has been shown by arrow heads in Fig \4.29(b). Thus, in this case.

In the base, 7^ and 7^. flow in opposite directions.


Sample Problem In a n-p-n transistor circuit the collector current is 9 mA. If 90% of the
electrons emitted reach the collector, find the emitter current and base current.
10
Sol. Here, 7=9 mA = =’/ or / =9x—= 10niA
100 10 ^ 9
So = IO-9-lmA
SEMICONDUCTOR ELECTRONICS MATERIALS, DEVICES AND SIMPLE CIRCUITS 14/33

14.23. MODES OF STUDY OF JUNCTION TRANSISTORS

A transistor can be studied with any one of its three terminals grounded which would serve as a link for
both, the input and output voltages. Thus, there are three external circuit connections for transistors as shown
in Fig. 14.30.
Fig. 14.30(n) represents the common base configuration (C6). Fig. 14.30(Z?) represents common emitter
configuration (C£) and Fig. 14.30(c) represents common collector configuration (CC) of npn transistor.

w
e
row
re
FFllo
eeF
u
r
sFr
kro
uor
offo
kos
Y
Yo
eerBB
oo
rY

14.24. COMMON BASE TRANSISTOR CHARACTERISTICS


u

The graphical representation of the variations among the various current and voltage variables of a
transistor are called the transistor characteristics. The common base transistor characteristics are of
ou
ad
do

two types :
nY

(/) Emitter or Input characteristics.

A graphical relation between the emitter voltage and the emitter current when collector voltage
nid
Re

is kept constant is called the emitter or input characteristics of the transistor.


F
Fi

(ii) Collector or Output characteristics.


A graphical relation which shows the variation of collector voltage and collector current when
emitter current is kept constant is called collector or output characteristics of the transistor.

To study the characteristics of common base


transistor using p-n-p transistor, we complete the FIGURE 14.31
circuit as shown in Fig. 14.31. Here emitter base
I
circuit is biased in forward direction with battery 3. le pnp Ic

■a
and collector base circuit is biased in the Veet
reverse direction with battery The emitter +
B !^VcC
voltage and emitter current can be studied by V,EB V,CB
voltmeter and ammeter respectively, whereas ' rib +
le Ic
the collector voltage and collector current by ●4 4

voltmeter and ammeter f. respectively. The


various currents obey the condition /^ = /^ + 7^.
14/34 ‘P’uidee^ 4 Fundamental Physics (XII) t»75TWn
I. Emitter or Input Characteristics
To get input characteristics, apply a suitable constant voltage on collector and by applying the various
values of emitter voltage, note the corresponding values of emitter current. Repeat the experiment for the
various constant collector voltages. Plot a graph between emitter voltage and emitter current, we get the
curves of the type shown in Fig. 14.32, called input characteristics of common base transistor.
Conclusions from the graphs : FIGURE 14.32
Ig (mA)
(/) For a given collector voltage, the emitter current
increases rapidly with increasing values of emitter base 10 --
VcBi--2V

A
●n T —

voltage. It means that input resistance is very small,

oww
(ii) For a higher negative collector voltage, the emitter 8

current irses more rapidly with the collector voltage.


c
Q

6 4-
In order to find input resistance of transistor
3
o

corresponding to emitter voltage 0-6 V, mark the point P <u


t! 4

ee
“"T--
on the input characteristic. Draw a tangent to the curve at s
UJ t
P. The reciprocal of the slope of the line AB will give us

FFrlo
2 4/i L L
I

r
input resistance R- of transistor. I

I 05 I 07

rF
>Veb (V)

ee
0
0-7-0-5 0-2V 0'2 0-4 0-6 0-8 1-0
Here, R.I = = 50Q
M~ BC 8-4 4 mA Emitter Base Voltage (V^g)

rF
ouru
II. Output or Collector Chaructcristics FIGURE 14.33
Ic (mA)

ffosor
To get the output or collector characteristics, adjust
os k
a suitable constant value of emitter current and by applying
the various values of collector voltages, note the t 20
I
I
I
ook
corresponding values of collector currents. Repeat the Ig = 20 mA
u +
Yo
I
Y
experimentfor the various constant emitter currents. Plot C 15
<i> T
^ 1 i
Ie = 15mA
Bo

t
a graph between collector voltage {Vcb) collector
I
reeB

I
^ 10
current (7^,). We get the curves of the type shown in Fig.
.3- I 1 3-
o I Ig = 10 mA
14.33 called output characteristics of common base o

i 5 J. L J L
ooY

transistor.
I Ig = 5 mA
uur

o
o I
I
ad

Conclusions from the graphs : o >VcB (V)


5V 10V 15V 20V 25V
(/) For a given value of emitter cunent, the collector
dY

current is not zero when collector voltage is zero. Collector Voltage (Vqb)
(//) For a given emitter current, there is a rapid increase in the collector current for an increase in low
nind
Re

negative collector voltage. This shows the region of low collectorresistance. The transistor is never
F
Fi

operated in this region.


{Hi) For a given emitter current, the collector current becomes saturated for a certain collector voltage
shown by horizontal line. Beyond this there is no change in collector current for a further increase in
negative collector voltage. This indicates a
FIGURE 14.34
region of high collector resistance. This mA
+
4
means that output resistance is very high.
5 ‘c
h B
14.25. COMMON EMITTER
npn
TRANSISTOR CHARACTERISTICS
RB E +
Fig. 14.34 shows the experimental set up for
V.
the study of characteristics of a transistor when V,CE CC

grounded emitter is kept as a common terminal.


Base is the input terminal and collector is the "le
output terminal as shown. The various currents
are marked keeping in view the condition Ib
4 ► ¥

h = h^h-
SEMICONDUCTOR ELECTRONICS MATERIALS, DEVICES AND SIMPLE CIRCUITS 14/35

1. Input characteristics
The input characteristics of the transistor is a graph which shows the variation of the base current
with base emitter voltage Vgg, keeping Vgg fixed. Their shape is shown in Fig. 14.35(a).

w
Flo
reeee
FFr
Conclusions from the graph

for
(0 The base current (/^) is small so long as Vgg is less than the barrier voltage.
ur
(ii) When Vgg is greater than the barrier voltage, the base current increases rapidly with the small
kkss
increase in Vgg, which is similar to the case of forward bias diode.
Yo
(Hi) For the given value of V^g, the value of decreases with the increase in collector-emitter voltage
oo
eB

(/v) The value of is much smaller than that in normal diode because more than 95% of the majority
charge carriers emitted from emitter go to the collector to form the collector current.
As long as the collector-emitter junction is reverse biased, the change in collector-emitter voltage appears
r
ou
ad

as the change in collector-base voltage and its effect on base current is negligible. Therefore, the input
characteristics of different values of Vgg have identical
YY

curves.
At a fixed point on the characterisites, we can find input dynamic resistance of transistor as given
below ;
ndd
Re

Input dynamic resistance of a transistor (/?,●) is defined as the ratio of change in base-emitter
Fi

voltage (A to the resulting change in the base current (A //,) at constant collector-emitter
voltage (Vgg), Le.,
AV
BE
R. =

is constant

The value of /?■ for common-emitter transistor circuit changes continuously. It can be any value from
few hundreds to a few thousand ohms.

To find the input resistance corresponding to base emitter voltage Vgg = 0-8 V, mark the point P on the
proper input characteristic. Draw a tangent AB at P. The reciprocal of slope of AB of input characteristic
will give us the input resistance of transistor (/?,).
AV BC (0-9-0-7) V
BE
Here. R. = = 5oon
' A/^ AC (600-200)pA
14/36 Fundamental Physics fXinpgTWTI

2. Output characteristics
The output characteristics of the transistor is a graph which shows the variation of collector current
f. with collector emitter voltage V(^£, keeping fixed. The shape of output characteristic curves of npn
transistor is shown in Fig. 14.35(b).
Conclusions from the graph
1. For a given value of base current, when the collector-emitter voltage ( increases from 0 to 0-5 V,
the collector current (/^) increases rapidly. This value of upto which 1^ increases rapidly is called knee
voltage.
2. When collector-emitter voltage becomes greater than the base-emitter voltage (Vg£) then the
collector-base junction is reverse biased. Now the collector current increa.ses linearly but very slowly with
the increase in for the given value of Ify.

ww
3. For a given value of the value of is larger for larger value of /^.
There are three regions of output characteristics of a transistor with common-emitter configuration.
1. Saturation region. It is the shaded region towards the left of the line OA which lies close to the zero

Floo
voltage axis where all the curves coincide. Here line OA is called saturation line. In this region Vgg.
Due to it, both the junctions of transistor are forward biased. As a result of it, collector current does not

ee
depend on base current /^.

eer
2. Cut off region. It is a shaded region below the curve /g = 0 (i.e., below the line OB). In this region

FFr
both the junctions are reverse biased and 1^ = 0 as well as /g = 0. The cut off for the collector current is not

oor r
uur r
obtained simply by making Ig - 0. In order to get cut-off, the emitter junction has to be made slightly reverse
biased in addition to Ig = 0. The transistor can work as a switch as it can be turned rapidly from cut-off
s ff
state for which /^ = 0 to on-state for which is maximum.
sk
YYoo
3. .Active region. It is a non-shaded region of the output characteristic in between the curves OA and
oooko

OB. In this region the emitter-base junction is forward biased and collector junction is reversed biased. In
this region for a given value of /g, increase with the increase in ^CE which shows that the transistor is in
eBB

active state and it can be used as an amplifier. FIGURE 14.36

3. Transfer Characteristics
uurr

Transfer characteristic is a graph which shows the variation of collector


ad
Yo

current l^ with the base current when collector-emitter voltage is


kept constant.
dY

Transfer characteristic of a transistor shown in Fig. 14.36, is almost a


Re

straight line.
ind
FFin

We can find the output resistance of transistor as given below. 1b(pA)

Output dynamics resistance of transistor (Rq) is defined as the ratio of change in collector-
emitter voltage (A V^e) change in collector current (A If) when base current ly is kept
constant, i.e..
AV
CE

Jl^ is constant

To find the output resistance for a given base current at a given collector voltage, mai k a point on the proper
output characteristic. Draw a tangent on this chiu-acteristic at that point. The reciprocal of slope of this tangent
will give us output resistance. The output resistance of a transistor is very high (of the order of 50 to 100 kQ).
HYBRID PARAMETERS (h-PARAMETERS)
The performance of a transistor can be judged with respect to its /i-piu'ameters. These /j-parameters of a
common-emitter transistor are as follows: [in /i-parameiers the first subscript indicates the nature of parameter
and second subscript indicators the type of circuit].
SEMICONDUCTOR ELECTRONICS MATERIALS, DEVICES AND SIMPLE CIRCUITS 14/37

(i) Input impedance

It is the ratio of small change in the base-emitter voltage (A to the corresponding change
in base current (A /^) when the collector-emitter voltage is kept constant, i.e.,
AV,be
Ihle
The unit of /t..
le
is ohm.
isconsiam
CE

(li) Reverse voltage ratio

It is the ratio of small change in base-emitter voltage (A to the corresponding change in


the collector emitter voltage (A Vcfr), keeping the base current (7^) constant, i.e.,

ww
AV,be
h re has no unit and no dimensions.
AV
CE - isconslani

Flo
(iii) Forward current ratio

e
It is the ratio of small change in the collector current (A /^) to the corresponding change in the

reree
base current (A 7^) at constant collector-emitter voltage i-e.,

r FF
A1
c
uurr
L A I,bjw
CE
isconslani
foor
has no unit and no dimensions.
ks s
(iv) Output admittance
Yoo
ooook

It is the ratio of small change in the collector current (A 7^) to the corresponding small change
in the collector-emitter voltage (A at constant base current (7^), i.e.,
eBB

h
oe
'AIcJ\ The unit of/t^^ is siemen.
uurr

isconslani
ad
Yo

Sample Problem In a transistor when base-emitter voltage is changed from 0*6 V to 0-8 V,
dY

the change in base current is 500 \iA to 700 |^A and the change in collector current is 10 mA to 40 mA,
Re

when the collector-emitter voltage is kept constant at 2 V. Find (c) input impendance (b) forward
innd

current ratio :
FFi

Sol. Here, AV^^ = 0-8 - 0-6 = 0-2 V


A//, = 700 - 500 = 200 |^A = 200 x K)-'"’ A
Alg = 40 - 10 = 30 mA = 30 x 10“^ A
AV
be 0-2
(a) Input impedance (hjg) = h:le = io^n
N.CEis
200x10'^
constant

A/
30x10-3 A
{b) Forward current ratio (/i^) = A/
h
fe - = 150
^ kCEis 200x10-^
constant

14.26. TRANSISTOR AS A SWITCH

Switch is a device for the on or off of current in the circuit.


14/38 T^adee^ Fundamental Physics (XII) W«TWH
To understand the operation of the transistor as a switch, we use npn transistor with common emitter
transistor circuit as shown in Fig. 14.37.
FIGURE 14.37
Ic
c r
Rb 1b B
\npil Rc
[
o
- >
E ir O)
0 ^CE 3 CD

oww
o>

^BE >
c o
> Vbb "1e
^cc
«
±
●b Ic

e
re
Using Kirchhoff’s voltage law ; for the input circuit we have

FFrllo
- VBB ^I^Rb+VBE = 0 or VBB = IbRb + V BE ...(10)

eF
In case of Si transistor VBE is 0-6 V to 0-7 V.

e
For the output circuit, we have
ouru
r
osrF
V CE = ,VCC “ ...(11)
“ ^cc + + ^CE = 0 or

We shall assume VBB as the d.c. input voltage Vj and V’^£ the output voltage Vq. Then
from (10),
from (II),
= ^Rb'^ ^be
0 = ^cc~^c^c
ffor
k ...(12)
...(13)
kso
Now we shall try to understand how \/q changes as V, increases from zero onwards.
ooo
Yo
Y
In case of Si-transistor, so long as V,- < 0-6 V, there is no collector current (i.e. = 0). Now the
BB

transistor will be in cut off state. In this situation, from (13), Vq = ^cc-
When V,- > 0*6 V but less than nearly UO V, there will be some collector current Iq From (13), we
Y
r ree

note that as increases, the value of output voltage Vq decreases. With the increase of V,- beyond 0-6 V,
ouu

increases almost linearly and so Vq decreases linearly till V,- becomes nearly 1-0 V. In this situation the
ad
Ydo

transistor is in active state.

When V,- ^ 1-0 V, the variation of V,- and Vq non linear, because with the increa.se in V,-, Vq is found to
decrease towards zero but never becomes zero. In this situation the collector current becomes maximum
nidn

and the transistor is in saturation state. If we plot a graph between Vq and V,-, we get a curve as shown in Fig.
Re

14.38. It is called transfer characteristic curve of a base biased transistor in CE configuration.


F
Fi

From above we note that as long as V,- is low (Le., V, < 0-6 V), Vq is high (= V^^-) and is nearly equal
to zero. The transistor is not working, i.e., transistor is in cut off state, i.e., the transistor is said to be in
switched off state. If V,- is high (i.e.. V,. > 1 -0 V), Vq is low and is nearly equal to zero. In this case, is nearly
equal to maximum or saturated. Now the transistor is fully conducting, i.e., the transistor is said to be in
switched on state. FIGURE 14.38
Q
Cut off
If we define low and high input states as below and
Region Active
above certain voltage levels corresponding to cut off and
saturation states of transistor, then we can say that low input *JRegt_o^*
state switches the transistor ojf and a high input state,
switches it on. This indicates that a transistor acts as a Saturation Region
switch.

Important point. The switching circuits are made in


such a way that the transistor does not remain in active
state. ►V;1
o
0'6 V 10V
SEMICONDUCTOR ELECTRONICS MATERIALS, DEVICES AND SIMPLE CIRCUITS 14/39

14.27. CONCEPT OF AN AMPLIFIER

An amplifier is a device which is used for increasing the amplitude of variation of alternating
voltage or current or power.

The amplifier thus produces an


FIGURE 14.39
enlarged version of the input signal.
Power Supply
The general concept of
amplification i.s represented in Fig. 1 Out Pul Voltage

oww
14.39. There are two input terminals
for the signal to be amplified and two % Amplifier : C\ or

Input
output terminals for connecting the Voltage ° |
load; and a means of supplying power Common Base Common
to the amplifier. Circuit Emitter Circuit

e
re
14.28. TRANSISTOR AS COMMON BASE AMPLIFIER

FFrllo
(c) Amplifier circuit using an n-p-n transi.stor.

rF
ee
The circuit details, using an n-p-n transistor as a common base amplifier are shown in Fig. 14.40. Here,
base is common to both the input and the output circuits. The input (emitter base) circuit is forward biased by
ouru
sor rF
using a low voltage battery ^EE- As a result, the resistance of input circuit is small. The output (collector
base) circuit is reverse biased by using a high voltage battery Due to it, the resistance of output circuit
is large. Rf is a load resistance connected in collector circuit. The low input a.c.voltage signal is applied
kffo
across emitter base circuit and the amplified a.c. voltage signal (i.e., output) is obtained as the change in
collector voltage {i.e., AV^^^). In circuit diagram, arrows represent the direction of conventional current or
os
hole current which is opposite to the direction of electronic current.
ook
Yo
Y
Bo

14.40
reeB

Cl Ic C2
HI -4
ouY

J t
ur

>
ad

Re
Yo

Rl 4
d

^7 V EE
V CE

i
nidn

Ic
Re

Input
A.c. Signal
X
F
Fi

X.Vcc
le
> T.

When no a.c. signal voltage is applied to the input ciruit but emitter base circuit is closed then let us
consider that /^, //, and be the emitter current, base current and collector current respectively. According to
Kirchhoff’s first law

...(14)
Let us consider 5% of the emitter current appears as base current due to electron hole combination
in base and 95% of the emitter current flows as a collector current, i.e.. If, = 5% of 1^ = 0-05 and
1^ = 95% of/^ = 0-95 /^. Due to collector current 7^., voltage drop across = If is collector voltage
(i.e., potential difference between collector and base) then
^CC = ^CB + L'
or
^CB ~ ^CC ~ ...(15)
14/40 Fundamental Physics (XII)EEIHn

When the input signal voltage is fed to the emitter base circuit, it will change the emitter base voltage
and hence the emitter cunent; which inturn will change the collector current. Due to it, the collector voltage
VcB will vary in accordance with relation (15). This variation in collector voltage appears as an amplified
output (Vq)-
Phase relationship between input and output voltages.
(/) When the positive half cycle of input a.c. signal voltage comes, it opposes the forward biasing of the
emitter base circuit. Due to it, the emitter cuirent decreases and consequently the collector current decreases.
As a result of it, the collector voltage increases [from relation (6)]. Since the collector is connected to the
positive terminal of battery, therefore the increase in collector voltage means the collector will become
more positive. This indicates that during positive half cycle of input a.c. signal voltage, the output signal
voltage at the collector also varies through a positive half cycle.
(if) When negative half cycle of input a.c. signal voltage comes, it supports the forwards biasing of

w
emitter base circuit. Due to it, the emitter current increases and consequently the collector current increases.
As a result of it. the collector voltage V(^b decreases [from relation (15)], i.e., the collector becomes less
positive. This indicates that during negative half cycle of input a.c. signal voltage, the output signal voltage

Flo
at the collector also varies through the negative half cycle.
Thus in common base amplifier circuit, the input signal voltage and the output collector voltage

ee
are in the same phase, as shown in Fig. 14.40.

Fr
(6) Amplifier circuit using p-n-p Tran.sistor.
The circuit details, using a p-n-p transistor as a common base amplifier are shown in Fig. 14.30. The

or
basic theory of this circuit is the same as in case of n-p-n transistor circuit. In this case also, output voltage
ur
signal obtained across collector is in phase with the input voltage signal as explained below :
sf
ok
Yo
Bo
re
ou
ad
Y
nd
Re
Fi

When the positive half cycle of input a.c. signal voltage comes, it supports the forward biasing of the
emitter-base circuit. Due to it, the emitter current increases and consequently the collector current increases.
From (6), we note that as increases, the collector voltage '^CB decreases. Since the collector is connected to
the negative terminal of ^CC battery, therefore, the decrease in collector voltage means the collector will
become less negative, i.e., shows positive variation. This indicates that during positive half cycle of input
a.c. signal voltage, the output signal voltage at the collector also varies through the positive half cycle.
Similarly, it can be shown that during negative half cycle of input a.c. signal voltage, the output
signal voltage at the collector also varies through the negative half cycle. Thus, in common base transistor
amplifier circuit the input signal voltage and the output collector voltage are in the same phase.
VARIOUS GAINS IN COMMON BASE AMPLIFIER
(1) Current gain or current transfer ratio, its value depends on whether the current flowing is d.c. or
a.c. in nature.
SEMICONDUCTOR ELECTRONICS MATERIALS, DEVICES ^ND SIMPLE CIRCUITS 14/41

(a) d.c. Current gain. It is defined as the ratio of collector curent (/^) to the emitter current
{Ig). It is denoted by Thus,

^dc ..(16)
The value of d.c. current gain is always less than 1. Its maximum value is 0-98.
(b) a.c. Current gain. It is defined as the ratio of change in collector current (A /^) to the
corresponding change in emitter current (A 1^) at constant collector base voltage. It is denoted
by ,aa.c.'
AI
a
a.c.
AI
~ , when V(-ij is constant ...(17)

w
(2) a.c. Voltage gain. It is defined as the ratio of change in output voltage, Le., collector voltage
change in input voltage, i.e., emitter-base voltage (A ). It is denoted by A

Flo
AV
CB

e
AK ...(18)

rree
I

Let Rq - the output resistance of the circuit, R^- = the input resistance of the circuit of a transistor,

r FF
0
uurr
Then
=
for
^ X resistance gain ...(19)
X

'' A'. /?..I

where Rq/R,' is called resistance gain.


kss
For example, if/?Q = 1000/?. and a = 0-95, then Ay= 0-95 x 1000 = 950.
ooook
Yo
a.c.

This is the order of voltage gain.


eB

From (19), Voltage gain = current gain x resistance gain.


(3) a.c. Power gain. It is defined as the ratio of change in output power to the change in input
urr

power, i.e.,
ad
Yo

a.c. power gam =


change in output power _ A ^A ...(20)
dY

changein input power AV.xAl^


Re
innd

AI

AV. AI
- =A^xa a.c. ...(21)
Fi

‘ ; V ^ j

AIxR,
c X) AI c AI
Ro
From (20), a.c. power gain = c
X
= ^ X resistance gain ...(22)
AI xR. AI AI R.
e i e
e i

As current gain < 1, power gain is slightly less than the voltage gain.
From (21), Power gain = voltage gain x current gain
Retain in Memory
● the whole working and gains of p-n-p common base transistor amplifier are exactly the same as
explained in the n-p-n transistor amplifier.
● the common base amplifier is preferred when radio frequency signals are amplified.

Sample Problem Q In a common basc circuit, the current gain is 0*96. If the base current is
60 |i A, find the emitter current and collector current.
14/42 ‘PxaxUe^ 4 Fundamental Physics (XII) kviWd

Sol. a
dc

60 60
= 1500 n A = 1‘5 mA
1-a 1-0-96 0-04
dc

h = ^e~h= 1500 - 60 = 1440 |i A = 1-44 mA


Sample Problem B For a common base amplifier, if the values of resistance gain and voltage

w
gain are 3000 and 2800 respectively, find the current gain and power gain of this amplifier,
voltage gain 2800
Sol. Current gain, a = = 0-93
resistance gain 3000

roow
e
(voltage gain)^ (2800)2
Power gain =

re
= 2613-3
resistance gain 3000

reF
uFFll
14.29. TRANSISTOR AS COMMON EMITTER AMPLIFIER

e
(a) Amplifier circuit using n-p-n transistor
The circuit details using an n-p-n transistor as a common emitter amplifier are shown in Fig. 14.42.

sFr
Here emitter is common to both the input and the output circuits. The input (emitter base) circuit is forward

foro
biased with battery and the output (collector-emitter) circuit is reversed biased with battery Due to
uor
fk
this, the resistance of input circuit is low and that of output circuit is high. is a load resistance connected
in collector circuit. The low input a.c. voltage signal is applied across base-emitter circuit and the amplified
okso
a.c. voltage signal {i.e., output) is obtained as the change in collector voltage (AV^£). In circuit diagram
Y
arrows represent the direction of conventional current or hole current, which is opposite to the direction of
Yo
oo
BB

electronic current.

When no a.c. signal voltage is applied to the input circuit but emitter base circuit is closed, let us
consider, that /^, //, and be the emitter current, base current and collector current respectively. Then according
rYree

to Kirchhoff’s first law


ouu

...(23)
ad
Ydo
nidn
Re
F
Fi

Let us consider 5% of the emitter current appears as base current due to electron hole combination in
base and 95% of emitter current flows as a collectorcurrent then, = 5% of = 0-05 Ig and = 95% of
= 0-95 Ig. Due to collector current /^, voltage drop across /?£ = 1^ R^. If is collector voltage
(i.e., potential difference between collector and emitter) then
V CC - ^C£ ^IcRl or
^CE = ^CC ~ h ...(24)
When the input signal voltage is fed to the emitter base circuit, it will change the emitter base voltage
and hence to the emitter current, which inturn will change the collector current. Due to it, the collector
voltage V(-jT will vary in accordance with relation (24). These variations in collector voltage appear as
amplified output (Vq).
SEMICONDUCTOR ELECTRONICS MATERIALS, DEVICES AND SIMPLE CIRCUITS 14/43

Phase relationship between input and output voltages


(0 When the positive half cycle of input a.c. signal voltage comes, it supports the forward biasing of the
emitter-base circuit. Due to it, the emitter current increases and consequently the collector current increases.
As a result of it, the collector voltage decreases [from relation (24)]. Since the collector is connected to
the positive terminal of Vqq battery, therefore decrease in collector voltage means the collector will become
less positive, which means negative w.r. to initial value. This indicates that during positive half cycle of input
a.c. signal voltage, the output signal voltage at the collector varies through a negative half cycle,
(ii) When negative half cycle of input a.c. signal voltage comes, it opposes the forward biasing of
emitter-base circuit, due to it the emitter current
decreases and hence collector current decreases; consequently
the collector voltage increases [from relation (24)]. i.e., the collector becomes more positive. This indicates
that during the negative half cycle of input a.c. signal voltage, the output signal voltage varies through
positive half cycle.

ww
Thus in a common emitter amplifier circuit, the input signal voltage and the output collector
voltage are in opposite phase i.e., 180” out of phase, as shown in Fig. 14.42.

FF loo
Phase relationship between input and output voltages can also be explained as discussed below ;
Let /?, be the input resistance of emitter-base circuit. When an input signal voltage V- is applied to the

ree
emitter-base circuit, let Alg, A/^ be the change in base current and collector current respectively. Then
V:I = A/o
B X R:t

rFee
AI
C
The a.c. current gain of transistor is, [3 or
- Pflc ^ ^B

F
ac
A/ B

oor r
rur
From (24), V.CE = KCC~^C^L s ff
As change in the base current changes the collector current but not so
M^CE=^-^C^L = -^hc^B)^L
k
YYoou
Output voltage. ^0 = ~ ~ Pac ^B
ookos

V,
-Kc^B^L
BBo

Voltage gain of common emitter amplifier Ay = —


V.
= ac
A/g/?. R-
re

I I

Here, -ve sign shows that the output voltage is 180” out of phase with the input voltage,
ouur
ad

(b) Amplifier circuit showing p-n-p transistor


Yo

The circuit details using a p-n-p transistor as a common emitter amplifier are shown in Fig. 14.43. The
basic theory of this circuit is the same as in case of n-p-n transistor circuit. In this case also, the output signal
voltage obtained across collector is 180” out of phase with the input voltage signal as explained below :
dY
Re
idn

When the positive half cycle of input a.c. signal voltage comes, it opposes the forward biasing of emitter
FFin

base circuit. Due to it, the emitter current decreases and hence collector current decreases; consequently the
collector voltage V^£ increases according to relation (24). Since the collector is connected to the negative
terminal of battery, therefore, the increase in collector voltage means, the collector will become more
negative. This indicates that during positive half cycle of input a.c. signal voltage, the output signal voltage
at the collector varies through negative half cycle, i.e., 180 ” out of phase.
FIGURE 14.43

Ic C2
C
Ci
R/ B ^ J
pnp I Rl
I

E
V;cE ’'Ic
V BB
- > le
Input 1

A.C. Signal 1
Vcc I

4^
Ib Ic
14/44 ‘Pfuzdee^'4. Fundamental Physics (XlllQsMD
Similarly it can be shown that during negative half cycle of input a.c. signal voltage, the output signal
voltage at the collector varies through the positive half cycle, i.e., 180° out of phase. Thus, in common
emitter transistor amplifier circuit, the input signal voltage and the output collector voltage are 180°
out of phase.
VARIOUS GAINS OF COMMON EMITTER AMPLIFIER
(1) Current gain or Common emitter current Amplification factor. It is of two types :
(i) d.c. Current gain. It is defined as the ratio of the collector current (/^.) to the base current
(/^) at a given collector-emitter voltage. It is denoted by p*-

c h
P
<e-‘c

ww
a
^dc ~ (V a = I,/I,)
I-a

FF loo
0-95
If a = 0-95, Prfc “ = 19.

ree
1-0-95

(ii) a.c. current gain. It is defined as ratio of change in collector current (A I^) to the

reeF
corresponding change in base current (A /^) at constant collector-emitter voltage. It is denoted

oroFr
by Kc.
r ur
s ff
A/
c
p Its value lies between 15 to 50, for a transistor.
k
a.c.
YYouo
kos o

Note that the value of current amplification factor (P^^.) can be obtained from the slope of collector
current versus plot, using different values of Ig in output characteristics of transistor.
BBoo

From Fig. 1435(b); = 3 0 V, when changes from 50 pA to 100 pA, changes from 3 mA to
r ee

5 mA.
ad

(5-3)mA
ouur

P = 40
Yo

a.c.
(100-50)pA

(2) TVans-conductance (g„). It is defined as the ratio of change in collector current (A 1^) to the
d
Re

change in base emitter voltage (A V,-). Therefore


dinY
FFin

AV
P a.c. R. =
I

gm or gm I
R.
Av;. A/, Av;. i

Its unit is Or ' or siemen (denoted by 5). It depends on the geometry, doping levels and biasing of the
1

transistor.

(3) a.c. voltage gain (Ay). It is defined as the ratio of the change in output voltage (AV^e )
the change in input voltage (AV,-), i.e.,

= Pa.c.X^ =P
AV,CE
Ay = X resistance gain. ...(25)
AV;. AI,XR.
a.c.
1

where Rq and /?,● are the output and input resistances of the circuit.
Here p„,, > > «« c ’ but resistance gain (/?(//?,●) is less than that in case of common base transistor
amplifier, hence the a.c. voltage gain in common emitter amplifier is greater as compared to that of common
base transistor amplifier.
SEMICONDUCTOR ELECTRONICS MATERIALS, DEVICES AND SIMPLE CIRCUITS 14/45

P a.c.
From (25), A/ = R.
I

Actually, Ay = -g,„x/?Q, Here negative sign indicates phase reversal of output.


(4) a.c. power gain. It is defined as the ratio of the change in output power to the change in
input power Le.

w
change in output power (A/^) A/“ AL
a.c. power gain = ^x — = P:a.c. xresistancegain
change in input power (AfJ) a/2 r.I

e
= ^a.c. X (P X resistance gain) = p XAy a.c. a.c.

Since P a.c.

re
,, therefore, extremely high power gain is possible in common emitter amplifier as

row
> a
a.c

compared to that in common base amplifier.


(5) Frequency Response Characteristic of an .Amplifier

eeF
ullo
It is the graphical study of the variation of the voltage gain (Ay;) of an amplifier with frequency (v) used

FF
for amplification. A typical frequency response characteristic of an amplifier is shown in Fig. 14.44.
From the graph we note that the voltage gain is low for low and high frequency range and high for mid

srr
frequency range.

kroF
FIGURE 14.44 FIGURE 14.45
Av
uor
Low
frequency
Mid
frequency
ofof
High
frequency
VvWV
kos
Y
►H H R;I
Yo
range range range
eerBB

V.-i*
oo

T
A >Vo
rY

I
I
u

■►V
ou
d
o
ad

(6) For operational or OP-inverting amplifier. Fig. 14.45, voltage gain is


nY

-R
/
^^=77
nid
Re

1 I
F
Fi

Here -ve sign shows that output voltage is out of phase with input voltage.
Relation between a and p
In a transistor,

or ^ = ^ + 1 or I
- = -+ 1 =
J+P
— = a and — = p
I

I
c
I
c a p P I
c
DO YOU KNOW ?
● The common-emitter
P P ac amplifier is preferred over
or a = also a
1+p
ac
1+P ac
common base amplifier
because the current gain, the
1 I 1 -a
Also
a
voltage gain and the power
or
P =
p a a 1-a gain in common emitter
transistor amplifier is greater
a
than that of common base
P ac
ac
Also
I-a ac transistor amplifier.
14/46 Fundamental Physics (XII) PTSmu

Sample Problem Q From the Output characteristics shown in Fig. 14.35(A), calculate the
values of current amplification factor of the transistor when Vc£ = 2V.
Sol. From the given characteristics, corresponding to ^CE~ 2-0 V, note the collector currents for two
base currents (say 50 |i A and 200 p A). The corresponding values of are 3-0 mA and 9-0 mA respectively.
Then A /^ = 9 - 3 = 6 mA ; A 7^. = 200 - 50 = 150 p A
_ A _ 6 mA
Current amplification factor, P ac
= 40
A/^ 150pA
Sample Problem The current gain for common emitter amplifier is 59. If the emitter current
is 6*0 mA, find the base current and collector current.
Sol. Here, = 59 ; 7^ = 6-0 mA ; 7^ = ? and 7^ = ?

ww
P or r - _ 6-0 = 0-1 mA
1 + 59

Flo
/^ = /^-/^:=60-0-l =5-9 mA

e
reree
1430. FEED BACK AMPLIFIER AND TRANSISTOR OSCILLATE

r FF
A process in which a portion of the output of an FIGURE 14.46
amplifier is fed back to the input is termed as feed
uurr
back and such amplifier is known as feed back
amplifier. foor
Transistor as
an Amplifier
Output
ks s
A process is said to be a positive feed back if
Yoo
the portion of the output is fed back in phase to the
ooook

,, ,.

input and a process is said to be a negative feed back


if the portion of the output is fed back out of phase |
eBB

Feed back
with the input. ^ circuit

Fig. 14.46 shows the block diagram of a feed


uurr

back amplifier in which the positive feed back is given


ad

to the input.
Yo

The overall gain (Aj0 of an amplifier with positive feed back is _ .4.. ■ and with negative feed
1-AP
dY

A
back is, A
^ i\ + Ap)
Re
innd

Where P is the feed back factor and A is the voltage gain without feed back. Here, A^ is called loop
Fi

gain. If AP = 1, then for positive feed back Aj^, = <x>, i.e., gain becomes infinite, which states that there is
output without any input. It means, the output is self sustained. In this situation the amplifier becomes oscillator.
The condition Ap = 1 is known as Barkhausen Criterion for Oscillations.

14.31. TRANSISTOR AS AN OSCILLATOR


An oscillator is a device of generating an alternating voltage of high frequency and of constant amplitude.
The energy required for this purpose is taken from the dc source. It means oscillator is a device for converting
energy of dc source into an alternating voltage of high frequency.
Tank circuit is a basic oscillatory circuit in which inductance L is connected in parallel to capacitance C.
1
The frequency of oscillations setup in the tank circuit is given by
2tz4w
the L-C circuir shows damped oscillations whose
Due to internal resistance of the inductance coil etc,
amplitude of oscillations goes on decreasing with time. The undamped oscillations can be produced by using
transistor as an oscillator, in which positive feed back is used to the transistor amplifier. The same can be
achieved by having inductive coupling of amplifier with L-C network.
SEMICONDUCTOR ELECTRONICS MATERIALS, DEVICES AND SIMPLE CIRCUITS 14/47

:!;!! of transistor an oscillator. FIGURE 14.47


SI
It is shown in Fig. 14.47. Here, L-C circuit is inserted O)—.-
3 ;d
in emitter base circuit of npn transistor which is O 4)

forward biased with battery V^q. The collector emitter


o
&
circuit is reverse biased with battery Vcc- ^
fl,
Q- D) t
Z3 (C ' a
is inserted in collector emitter circuit. It is coupled o
● ^
c
c
Vcc
O npn
with L in such a way that if increasing magnetic fiux rz
it-’ E
is linked with L, it will support the forward bias of Output L2§ l:L C
emitter base circuit and if decreasing magnetic flux
is linked with L, it will oppose the forward bias of K
emitter base circuit. Vbb

If we close the key K, the collector attains positive potential,due to it, there will be a weak
collector current which will start rising with time due to the inductance Lj. As a result of it, an increasing

w
magnetic flux is linked with Lj and hence with L. Due to mutual induction, an e.m.f. is induced in L which will
charge the upper plate of capacitor with positive charge, consequently there will be support to the forward

Flo
biasing of emitter base circuit. This results in an increase in the emitter current and hence an increase in the
collector current. Due to it, more increasing magnetic flux is linked with and hence with L. As a result of

e
it, more e.m.f. is induced in L, charging the upper plate of capacitor with more positive charge and hence

rree
providing more support to the forward biasing of emitter-base circuit, which results further increase in the

r FF
emitter current and hence in the collector current. The process continues till the collector current becomes
maximum or saturated.
uurr
for
Now the mutual induction stops playing its part. The capacitor C gets discharged through inductance L.
As a result of it, the support to the forward biasing of emitter-base circuit is withdrawn, thereby the emitter
kss
current decreases and hence collector current also decreases. Due to it, a decreasing magnetic flux is linked
with Ly and hence with L. Due to mutual induction, an e.m.f. is induced in L which will charge the lower plate
ooook
Yo
of capacitor C with positive charge and upper plate of capacitor with negative charge, consquently there will
be opposition to the forward biasing of emitter base circuit. This results in further decrease in emitter current
eB

and hence in collector current. This process continues till the collector current becomes zero. Now again
mutual induction stops playing its part. The condenser gets discharged through inductance L. As a result of it,
urr

the opposition to the forward biasing of emitter-base circuit is withdrawn, thereby the emitter current increases
ad

and hence collector cuixent also increases and the process repeats. Thus, the collector current oscillates
Yo

between a maximum and zero value.


dY

We know that in common emitter circuit, a signal voltage applied to the emitter base circuit suffers a
phase change of 180® in the collector emitter circuit. By coupling Lj, with L, we bring about a further phase
Re
innd

change of 180°. Thus, the energy which is fed to the L-C circuit is in proper phase at proper timings and hence
Fi

we get undamped oscillations in L-C circuit of frequency : v = 1 / 2 7t VlC


Therefore we get the carrier waves from the oscillator.
In this oscillator, the energy is being supplied by the battery to the L-C circuit at proper time and in
proper phase, therefore the battery gets consumed in oscillator. It means in oscillator D.C., is converted into
high frequency A.C.
1432. ADVAU HSOF5EM! CNDUCT . DEVICES
1. Semiconductor devices are much smaller in size and weight as compared to vaccum tubes.
2. Semiconductor devices are not to be heated for emission of electrons. They start operating instantly.
This saves a lot of electric power.
3. A transistorised equipment does not get heated, while operating. Therefore, no cooling arrangement is
required.
4. The semi-conductor devices are more rugged than the vacuum tubes. They can withstand rough handling.
14/48 ‘P'tadee^ Fundamental Physics (XII)
5. Semi-conductor devices have much longer life as compared to the life of vacuum tubes.
6. Semi-conductor devices are cheaper than vaccum tube devices.
7. Semi-conductor devices are low power devices.

14.33. DISADVANTAGES OF SEMI-CONDUCTOR DEVICES

1. Semi-conductor devices are very sensitive to changes of temperature whereas the vacuum tubes are
less sensitive.

2. It is difficult to produce semi-conductor devices with exactly identical characteristics.


3. The noise level in semi-conductordevices is higher than that of vacuum tubes.

oww
4. Semi-conductor devices cannot handle as much power as vacuum tubes.
In modem times, .semi-conductor devices are almost replacing vacuum lubes on account of their merits.

14.34. DECIMAL NUMBER SYSTEM

ee
The decimal number system uses ten digits ; 0, 1, 2, 3, 4, 5, 6, 7, 8 and 9. The base of the decimal

FFrlo
number system is 10. Each digit represents a discrete state. Any number can be represented in the decimal

r
number system. Every digit in a given number has a place value. For example, a number 647 can be written

rF
ee
as 647 = 6 X 10“ 4 X 10' + 7 X 10“ = 600 + 40-1-7
Thus digit 6 is in hundred’s place, 4 is in ten’s place and 7 is in the unit’s place. Obviously, the base of

rF
ouru
the decimal number system is 10.

14.35. BINARY NUMBER SYSTEM


ffosor
os k
The binary number system uses only two digits 0 and 1. Each binary digit is called a hit. The base of
the binary number system is 2, instead of ten which is the base in the decimal number system.
ook
Yo
Y
14.36. CONVERSION OF DECIMAL NUMBER SYSTEM INTO BINARY NUMBER SYSTEM
Bo
reeB

We go on dividing the given number (in the decimal system) by 2, until the quotient is zero. The remainders
obtained in the succesive divisions, taken in the reverse order give the binary representation of that number,
ooY

For example :
uur

2 2
(a) Binary representaion of ‘2’
ad

2 1-0
The remainders in reverse order are 1 and 0.
dY

0-1
.●. Binary representation of 2 is (10)2
nind
Re

2 3
(/») Binary representation of 3
F
Fi

2 1-1
The remainders in reverse order are 1, 1
0-1
.●. Binary representation of 3 is (11)2
2 10
(c) Binary representation of 10 2 5-0
The remainders in reverse order are 1,0,1.0
2 2-1
.●. Binary representation of 10 is (1010)2 2 1-0
0-1

2 23
2 11-1
(d) Binary representation of 23
2 5-1
The remainders in the reverse order are 1.0,1,1 and 1
2 2-1
Binary representation of 23 is (10111)2 and so on.
2 1-0
0-1
SEWnCONDUCTOR ELECTRONICS MATERIALS, DEVICES AND SIMPLE CIRCUITS 14/49

Note that: In Binary Representation of any number, the first bit is the most signiifcant bit (MSB) and
the last bit is the least significant bit (LSB). For example, in the binary representation of 23 :
MSB ^ 1 0 1 1 1 LSB

Symbolically, we write (23) jq = (10111)2


14.37. CONVERSION OF BINARY NUMBER SYSTEM INTO DECIMAL NUMBER SYSTEM
As the base in the binary number system is 2 therefore, we multiply least significant bit (LSB) by 2" the
next bit by 2^, next bit by 2^ and so on till the most significant bit (MSB) is reached, and add all these
products of multiplication. We shall get the decimal number of given binary number.
Sample Problem □ Convert binary number 10111 into decimal number.
Solution. 0 0 1 1 1)2= 1 x2°+ 1 x24 I x2^ + 0x2^+ I x2“*= 1 +2+4 + 0+ !6 = (23) 10

ww
T T
MSB LSB

Sample Problem Q Convert binary number 1010 into decimal

Flo
number.

e
Solution. (1 0 1 0)2 = 0x2® +1 X 2*+ 0x2^+1 x2^ = 0 + 2 + 0 + 8 = (10) 10

eree
T T

FFr
MSB LSB
uurr
TECHNIQUE OF FINDING THE BINARY EQUIVALENT OF A FRACTIONAL DECIMAL

orr
NUMBER

sfo
Multiply continuously the given fractional decimal number by 2. Each time record a carry in the
integer position (i.e. number before decimal). The process is to be continued till we get enough bits of
kks
Yoo
binary equivalentor zero. The carries in the forward order would give us the decimal binary equivalent.
oooo

Sample Problem a Determine the binary equivalent of (0*25) 10*


eBB

Solution. 0-25 x 2 = 0-50, with a carry of 0


0-50 x 2 = 100, with a carry of 1
urr

So the binary equivalent of (0-25)jq = (*01)2


ad
YYo

Sample Problem Determine the binary equivalent of (6*25) 10*


Solution. Here, 6-25 = 6 + 0-25
dd

We determine separately, the binary equivalent of 6 and 0-25. It is found that


Re
inn

(6)io = (liO)2
F

and
(0-25),0 = (.01)2
(6-25),0 = (110-01)2
TECHNIQUE OF FINDING THE FRACTIONAL DECIMAL NUMBER FROM DECIMAL
BINARY NUMBER

Multiply the successive bits after decimal with 2 2 2 and so on, till we reach to the last bit.
Then the addition of all these multiplications will give us the fractional decimal number of decimal binary
number.

Sample Problem
Find the fractional decimal number of (0*101)2.
Solution. (O-101 )2 = 1 X 2 * + O x 2"^ + 1 x 2“^
1 1
=-
2
+ 0 + -= - =(0*62S)jo.
14/50 'Pxtzdee^'A Fundamental Physics (XII)EZsI9I]

14.38. ADDITION OR SUBTRACTION OF BINARY NUMBERS

We can add or subtract binary numbers as discussed below.


Binai' ; ion. We can use the mathematical technique of '■carry' in the addition of binary numbers.
Keeping this technique in mind, we can easily understand the followings.
0 + 0 = 0

0+1 = 1

1+0=1
1 + 1 = 10 [i.e., 0 with carry of 1]

Sample Problem □ Let us add (110010)2 and (111101)2.

ww
Solution. 1 10 0 10
11110 1

110 1111

Flo
In the first column extreme right, the binary sum of 0 and 1 gives l,In the second column the binary sum

e
of 1 and 0 gives 1. In the third or fourth column, the binary sum of 0 and 1 gives 1. In fifth column, the binary

rere
sum of 1 and 1 gives 0 with carry of 1 at sixth column.

r FF
In sixth column, the binary sum of 1 and 1 gives 0 with carry of 1 at seventh column, but carry of I of
fifth column and 0 of sixth column will make the binary sum as 1 at sixth column, i.e.,
uurr
(110010)2 + (11101)2 = (1101111)2
for
kss
Sample Problem 0 Let US add (10101)2 and (1011)2
ooook
Yo
Solution. 10 10 1
10 11
eBB

10 0 0 0 0 I.e., (10101)2 + (10^ 1)2 = (100000)2


Binary Subtraction. For binary subtraction we make use of the followings :
urr
ad

0-0 = 0
Yo

1-0=1
dY

1-1 = 0
10-1 = 1
Re
innd

Sample Example S Let us find the difference of (111101)2 and (110010)2.


Fi

Solution. 11110 1

110 0 10

0 0 10 11

In the first column (from extreme right), the subtraction of binary 0 from binary 1 gives 1. In the second
column, the subtraction of binary 1 from binary 0 is not possible, so we have a carry of 1 from third column
for second column, leaving binary 0 at third column and making 10 at second column. Then subtraction of
binary 1 from 10 gives 1 at second column. In the third column, the subtraction of binary 0 from binary 0 (left
after carry of l)is 0. In the fourth column, the subtraction of binary 0 from binary 1 gives l.In the fifth and
sixth column, the subtraction of binary 1 from binary 1 gives 0 at each place. Thus
(111101)2 - (110010)2 = (001011)2
SEMICONDUCTOR ELECTRONICS MATERIALS, DEVICES AND SIMPLE CIRCUITS 14/51

14.39. ANALOGUE SIGNAL AND DIGITAL SIGNALS

‘ Signal. A continuous time


FIGURE 14.48
varying current or voltage signal is called
analogue signal. V

Fig. 14.48(a) represents a typical sinusoidal


1(5V)
analogue voltage signal. The electronic circuit
which gives out any type of analogue signal is O
called analogue electronic circuit. t t

Digital Signal. The signals which have two e o


levels of current or voltages (represented
by 0 & 1) are called digital signals.

ww
A digital circuit is an electronic circuit in which the current and voltage signals have only two
levels (either on or off). Fig. 14.48(i>) represents a typical digital signal in which the voltage at any lime can

Flo
be either zero or 5 V. The digital circuits make use of binai7 number system, according to which 0 and 5 V
values are denoted by the levels 0 and 1. Thus in digital circuits only two levels (represented by 0 and 1) of

e
the input and output voltages are possible.

rere
r FF
14.4f Ik GATE
uurr
foor
A digital circuit which either allows a signal to pass through or stops it, is called a gate. Such
gate allows the signal to pass through only when some logical conditions are satisfied. Hence
they are called ■ ■-;ic gates.
kss
Yoo
ooook

The logic gates are the building blocks of a digital system. Each logic gate follows a certain logical
relationship between input and output voltages.
eBB

The basic logic gates are of three types :


1. OR gate 2. AND gate and 3. NOT gate.
uurr

Each basic logic gate is represented by a symbol and its function is defined either by a truth table or by
ad

a Boolean expression.
Yo

In digital circuits, we mostly use junction diodes and transistors as swtiches to change from one voltage
dY

level to another. In the analysis of the digital circuits, we find out whether a voltage is low or high. In digital
circuits low and high voltages are often represented by levels 0 and 1 respectively,
Re
innd

ri-uth table. It is a table that shows all possible input combinations and the corresponding output
Fi

combinations for a logic gate. It is also called a table of combinations.


Boulcan expre.s.siors. George Boole invented a kind of algebra which deals with logical statements that
have only two values, namely either a true or a false value. The logical statements are called Boolean variables.
The true value of a Boolean variable is denoted by 1 and the false value by 0. It is to be noted that the
symbols 1 and 0 have nothing to do with numerical values 1 and 0. In electronic circuits, the symbols 1 and 0
are used to represent the active and the passive states (i.e. on or ojQ^states) of a circuit component, say a diode
or a transistor or a switch etc.

14.41. BASIC GATES

Basic gates are of three types :


(a) OR gate (b) AND gate (c) NOT gale
(a) OR gak' is a device that combines A with B to give y as the result.

The OR gate has two or more inputs and one output.


14/52 4- Fundamental Physics (XII) P75TWT1

The logic symbol of OR gate with ^4 and B inputs and y output is FIGURE 14.49
shown in Fig. 14.49.
In Boolean algebra, addition symbol (+) is referred to as OR. A

The Boolean expression.


B
A + B ~y, indicates thaty equals A OR B.
{h) AND gate is a device that combines A with B to give y as the
FIGURE 14.50
result. The AND gate has two or more inputs and one output. The logic
symbol of AND gate with A and B inputs and y output is shown in Fig. A
14.50.

The multiplication sign (either x or . ) is referred to as AND in B

Boolean algebra. The Boolean expression.

ww
FIGURE 14.51
A . B = y, indicates y equals A AND B.
(c) NOT gate is a device which inverts the inputs. If input is 1,
the output is not 1, but is zero and if input is 0, the output is not zero but A#-
is 1. The NOT gate is one input and one output device and its logic

Flo
symbol is as shown in Fig. 14.51.

e
The bar symbol (-) is referred to as NOT in Boolean algebra. The Boolean expression.

eree
A = y, indicates y equals NOT A.

FFr
The bar symbol over A means we change A to the alternative digit. Since, in binary system, 1 and 0 are
uurr
the only two digits, therefore, 1 = 0 and 0 = 1

orr
Thus, NOT operation is also called negation or inversion.

14.42. SOME USEFUL LAWS OF BOOLEAN ALGEBRA


sfo
kks
Yoo
oooo

(a) Commutative laws : (i)A + B = B+A ; (//) A.B = B.A.


(b) Associative laws : (/) A + (B + O = (A + fi) + C; (/OA. (B.O. = (A.B).C.
eBB

(c) Distributive laws : (0 A. (B + Q = A . B + A . C; (//) (A + B). (A + C) = A + B.C.

(d) Absorption laws: {/)A+A.B = A; (ii)A.(A+ A)=A;


urr
ad

(«0 A.(A + B) = A.B.


YYo

(e) Double complement function : A = A; (A + B) = A + B ; (A.B) = A.B


dd

(f) Boolean identities :


Re
inn

(0 A.(A + B) = A.B {//) A + A.B — A + B


F

(Hi) A + B.C = (A + B). (A + O (/v)(A +B). (A + 0= A.C + A.B


(g) De-Morgan’s Theorem. It states that the complement of the whole sum is equal to the product of
individual complements and vice versa Le. (/) A-^ B A.B (ii) A.B = A + B

Proof. Thi above two identities can be explained with the help of truth table as given below :
1 2 3 4 5 6 7 8 9 10

A B A + B A+B A B A.B A.B A.B A+B

0 0 0 1 1 1 I 0 1 1
0 1 1 0 1 0 0 0 1
1 0 1 0 0 1 0 0 1
1 1 1 0 0 0 0 1 0 0
SEMICONDUCTOR ELECTRONICS MATERIALS, DEVICES AND SIMPLE CIRCUITS 14/53

In truth table the columns 4 and 7 are identical, hence the theorem stands proved, i.e., A + B - A.B
The columns 9 and 10 are identical, hence the theorem stands proved, i.e., A.B = A+ B

De Morgan’s Theorem also states that


(/70 A^B = A.B = A.B iiv) A.B = A+B = A+B

These can also be proved by using truth table.


Proof for Boolean Identity / (ii) i.e. A-¥ A.B= A + B
Taking L.H.S. and applying distributive law, we have
A + A.5=(A + A).{A + 5) = 1 . (A + 5) = A + S(proved)
(h) Basic OR and AND gates relations

ww
OR AND

(i) A + 0 = A (0 A.0 = 0
(ii) A + i = I {ii) A . 1 = A

Flo
(m)A+A = A (iiOA .A=A
(iv) A.A = 0

ee
(/v) A + A = 1

rere
r FF
14.43. OR GATE

OR gate is a device that has two or more inputs and one output.
uurr
In Boolean algebra, the term
OR is represented by plus (+) and
foor FIGURE 14.52
ks s
t N
V.
\ N
Yoo
Boolean expression ; A + = y, I s
A B Y
oook

indicates y equals A OR B. In
\
A o- \ 0 0 0
practice, an OR gate can be realised
eBB

D c \
1
i ■)—oY 0 1 1
by the electronic circuit, making li J o

use of two ideal p ● n junction X-5V BO


►I >R / 1 0 1
uurr

diodes Dj and D2 as shown in Fig. °2



f 1 1 1
ad

I
14.52(a). Here negative terminal of I y
Yo

y
ow
the battery is grounded and
corresponds to the 0 level, and the e o
dY

positive terminal of the battery


Re
innd

{i.e., voltage 5 V in the present case) corresponds to level 1. The output y is voltage at C w.r.t. earth. The
operation of OR gate can be understood by the following four cases.
FFi

(0 When both A and B are connected to earth {i.e., A = 0 and B = 0) both the diodes do not conduct and
therefore no voltage develops across resistance R. The voltage at C is zero w.r.t. earth. Hence the output Y is
0 (in level).
{ii) When A is connected to earth and B is connected to positive terminal of battery 5 V {i.e., A = 0 and
B = 1), the junction diode Dj does not conduct while D2 conducts being forward biased. Since diode D2 is
ideal, no voltage drop takes place across D-). Now a voltage drop of 5 V takes place across R with C at + 5 V
w.r.t. earth. Therefore, the outf vvill be 1 (in level).
{Hi) When A is connected ,0 positive terminal of battery 5 V and B is earthed {i.e.. A = 1 and B = 0), the
junction diode Dj will conduct being forward biased and junction diode D2 will not conduct. Since diode Oj
is ideal, no voltage drop takes place across D[. Now a voltage drop of 5 V takes place across R, with C at
+ 5 V w.r.t. earth. Therefore, the output y will be 1 (in level).
{iv) When A and B arv connected to positive terminal of battery 5 V {i.e., A = 1 and B = 1), both the
diodes being forward biased will conduct. Since the diodes are ideal and connected in parallel, the voltage
drop across R cannot exceed 5 V, with C at + 5 V w.r.t. earth. Hence, the output y will be 1 (in level).
14/54 T^neieUe^ 4 Fundamental Physics (XII) PCTWn
Thus the operation of OR gale is based on the following rule. The output of an OR gate assumes 1 (in
level) if one or more inputs assume 1 (in level). The truth table of OR gate which combines the inputs A and
B to give output y is shown in Fig. 14.52(b).
analogue drc ● OK
FIGURE 14.52
The analogue electric circuit having function similar
to the OR gate is shown in Fig. 14.52(c). In this arrange
ment, off (or open) corresponds to 0 and on (or closed)
corresponds to 1. The inputs are introduced through the
Bulb
switch A and B. The lighting of the bulb is the output. Here
we find that the bulb glows (i.e., output is 1) when either y

switch A is closed or B is closed or both the switches are


Output
closed. The bulb remains off (i.e., output is 0) only when

ww
both the switches A and B are open (i.e., A = 0, B = 0) n
o
in

FF loo
AND gale is a device which has two or more inputs and one output.

ree
In Boolean algebra, the term AND is represented by dot (.) and the Boolean expression A.B = y implies
that y equals A AND B.

rFee
FIGURE 14.53
The AND gate can be realised

F
oor r
by the electronic circuit using two
rur
A B Y
A 0-
ideal p-n junction diodes D^ and D2
s ff
D \ 0 0 0
as shown in Fig. 14.53(a). The
1
V
c 1 0 1 0
resistance R is connected to the
k
«Y
YYoou
1 0 0
positive terminal of a 5 V battery I
ookos

=c- 5V
^2 /

permanently. The output y is the BO-


-^5V
/
/ 1 1 1
BBo

voltage at C w.r.l. earth. The


operation of AND gate can be
re

0 e o
understood by the following four
ouur
ad

cases.
Yo

(/) When both A and B are connected to earth (i.e. A = 0 and 5 = 0) both the diodes Dj and £>2 get
foPA'ard biased and hence conduct. The diodes being ideal, no voltage drop takes place across either diode.
Therefore a voltage drop of 5 V takes place across R. with C at zero potential w..rt. earth. Thus the output y
dY
Re
idn

(which is the voltage at O is 0 (in level).


FFin

(ii) When A is earthed and B is connected to positive terminal of battery 5 V (/. e. A = 0 and 5=1), the
diode D^ will conduct while D2 will not conduct. Since diode Dj is ideal, no voltage drop takes place across
D^. Therefore a voltage drop of 5 V takes place across R, having £) at -i- 5 V and C at zero volt w..rt. earth.
Now the output y is 0 (in level).
(Hi) When A is connected to positive terminal of battery 5 V and 5 is earthed (i.e. A = 1 and 5 = 0), the
diode D2 will conduct while Oj will not conduct. Since £>2 is ideal, no voltage drop takes place across £>2.
Therefore, a voltage drop of 5 V takes place across R, having terminal Dat + 5N and C at zero volt w.r.t.
earth. Now the output y is 0 (in level).
(iv) When A and 5 both are connected to positive terminal of battery 5 V (i.e. A = 1 and 5=1), none of
the diodes will conduct. There will be no current through R. Now potential at C is equal to potential at D,
which is equal to + 5 V .w.rt. earth. Hence the output y is 1 (in level).
Thus, the operation of AND gate is based on the following rule : The output of an AND gate as.sumes
1 (in level) only if all the inputs assume 1 (in level).
The truth table of AND gate which combines the inputs A and 5 to give output y is shown in Fig. I4.53(/?).
SEMICONDUCTOR ELECTRONICS MATERIALS, DEVICES AND SIMPLE CIRCUITS 14/55

vnalogue circinC of AND gate FIGURE 14.53

The analogue electric citcuit having function similar to the %A %B


AND gate is shown in Fig. 14.53(c). In this arrangement off (or
open) corresponds to 0 and on (or closed) corresponds to 1. The
Bulb
inputs are introduced through the switches A and B. The
lighting of the bulb is the output. Here we find that the bulb y

glows {i.e., output is 1) only when both the switches A and B are Output
closed. The bulb remains off {i.e., output is 0) when either switch
A or switch B or both are open.
Sample Problem What is the Output
o
wave form (y) of the (i) OR gate and (ii) AND FIGURE 14.54
gate for the following inputs A and B as shown

ww
tl t2 t3 U t5 *6
in Fig. 14.54. (NCERT Solved Example)

Sol. The Boolean expression for

Flo
OR gate is A + B=y A

AND gate is A.B^y

e
Input

reree
From the given wave forms, we have the B
following results for OR gate and AND gate.

r FF
Time interval Input values Output value y for
uurr
OR
gate
AND
gate
foor FIGURE 14.55
ks s
ti t2 t3 U t5 *6
Yoo
At r< r 1 A = 0, B = 0 0 0
ooook

For fj to t2 A= l.B = 0 0
eBB

For ^2 to /3 A = 1,B= 1 1 1
e
For ij to ^4 A = 0, B = I 1 0

For /4 to A = 0. B = 0 0 0
O
uurr

For to A= 1,B = 0 1 0
ad
Yo

For t > rg A = 0, B = 1 1 0
dY

The output wave forms for OR gate and AND gate are shown in Fig. 14.55(a) and (b) respectively.
Re
innd

14.45. NOT GATE


FFi

NOT gate is a device which has only one input and one output.

In Boolean algebra, the term NOT is represented by bar symbol (-) and the Boolean expression A = y
indicates thaty equals NOT A.
A NOT gate cannot be realised by using diodes. However it can be realised by making use of a transistor.
An electronic circuit of a NOT gale using n-p-n transistor is shown in Fig. 14.56(a). The base B of the
transistor is connected to the input A through a resistance and the emitter E is earthed. The collector is
connected to 5 V battery. The output y is the voltage at C w.r.t. earth.
The resistors Rf, and are so chosen that if emitter-base junction is unbiased {i.e.. base is at level zero),
the transistor is in cut off mode and if emitter-base junction is forward biased by 5 V {i.e., base is at level 1),
the transistor is in saturation state.

The operation of the NOT gate can be understood by the following two cases.
(0 When A is earthed {i.e., A = 0) the base of the transistor also get earthed. Now, base emitter junction
is not forward biased but base-collector junction is reverse biased. As the emitter current is zero, the base
14/56 ‘PnauUe^ <1 Fundamental Physics (XII) LWll
current is also zero and hence the FIGURE 14.56
S

collector current will also be zero.


N

Under these conditions the transistor s


s
N

is said to be in cut off mode and


s
s
N

voltage at C will be + 5 V w.r.r. earth


s

I"5V's
due to battery in collector circuit. r
X.
V

Hence the output y is 1 (in level).


s
R Rc s
s

(//) When A is connected to ■oY A Y


positive terminal of battery 5 V (i.e., A A 0 1


1 ■o o- ✓

= 1), the base-emitter junction gets y

1 0
forward biased. There will be emitter .T.5V
current, base current and collector

ww
current. The values of resistors and y
y
o
are so adjusted that in this
y

0
y
o
arrangement a large collector current
y
y

Flo
flows. In this situation, the transistor
is said to have gone to saturation state. The voltage drop across due to forward biasing of emitter is just

ee
equal to 5 V, which is equal and opposite to the potential drop across R^ due to battery in collector circuit.

eer
Hence, the voltage at C is zero volt. Therefore, the output y is 0 (in level).

FFr
Thus the operation of NOT gate is based on the following rule : The output of a NOT gate is 1 (in

oorr
level) if input is 0 (in level) and vice versa.
uur r
The truth table of NOT gate is shown in Fig. 14.56(/?). FIGURE 14.56
s ff
Analogue circuit of NOT gate
sk
YYoo
The analogue electric circuit having function similar Bulb
ooko

to the NOT gate is shown in Fig. 14.56(c). In this y


eBB

arrangement, off (or open) corresponds to 0 and on (or A


closed) corresponds to 1. The input is introduced through Output
the switch A. The lighting of the bulb is output. Here, we
uurr

f \
find that the bulb glows (i.e., output is 1) only when switch
ad

o
A is open or off (i.e., A = 0) and bulb does not glow (i.e.,
Yo

output is 0) when switch A is closed (i.e., A = 1).


dY

Sample Problem What is the output waveform/ of


Re

FIGURE 14.57(a)
the NOT gate for the input signal A as shown in Fig. 14.57(a)?
innd
FFi

Sol. The Boolean expression for NOT gate is, A - y. From k*-
O
the given wave form, we have the following results for NOT gate h fa f4 fs fe

Time interval Input value Output value

For ? < / 1 A = 0 y=l


FIGURE 14.57(b)
For to /2 A= 1 y=0
For ^2 to A= 1 y=0
YA---4-
For ^3 to /4 A = 1 y=0 O fz fa f4 f5 fe
For to A = 0 y=l
For to fg A = I y =0
For t > /g A=0 y=I

The output wave form is a.s shown in Fig. 14.57(6).


SEMICONDUCTOR ELECTRONICS MATERIALS, DEVICES AND SIMPLE CIRCUITS 14/57

DO YOU KNOW ?

(0 A + 1 = 1 (»)A1 =A (Hi) A + A = 1 (iv) A ● A = 0 (v) A + A = 4 (v/)A-A=A

{vii) A-¥B = A- B (viii) A - B = A +B (ijc) A-VB = A + B (Jc) A-B = A-B

w
14.46. COMBINATION OF GATES

Various combinations of three basic gates {OR, AND and NOT) provide us large number of complicated
digital circuits, which are also known as ‘gates’. Few combinations of the basic logic gates, using their
symbols are discussed below.

e
roow
(1) NAND gate. If we connect the output /of AND gate to the input of a NOT gate, shown in Fig.

re
1 4.58(a), the gate so obtained is called NAND gate. The logic symbol of NAND gate is shown in Fig. 14.58(/?).
FIGURE 14.58

F
A

uFFll
Y' Y
0

ree
B

sFr
kro
A

>
uor
A« i*

B offo B«—K
■T—1—AAA/V
O
kos
Y
Yo
ereBB

A B Y' Y
oo

A B Y
5V^
0 0 0 1
0 0 1
uurY

1 0 0 1
1 0 1
0 1 0 1
0 1 1 V
1 1 1 0 AND Gate NOT Gate
1 1 0
ad

o o o
doo
nY

In Boolean expression, the VAAD gale is expressed as, y = A.B, and is being read as ‘A AND 5 negated’.
nid
Re

The truth table of NAND gate can be obtained by logically using the truth table of AA® and NOT gates as
F
Fi

shown in Fig. 14.58(c) and its final truth table is shown in Fig. 14.58(J)
Circuit for NAND gate is as shown in Fig. 14.58(e).
(2) NOR gate. If we connect the output y'of an FIGURE 14.59
OR gale to the input of the NOT gate as shown in Fig.
Y
14.59(a), the gate so obtained is called the NOR gate. Y'

In Boolean expression, the NOR gate is expressed |—i e


as, y = A + 5 and is being read as ‘A OR B negated’.
The logic symbol of NOR gate is shown in Fig. 14.59(/>).
The truth table of NOR gate can be obtained by ■oY O
Bo
logically using the truth table of OR and NOT gates as
shown in Fig. 14.60(a) and {b).
Circuit for NOR gate is as shown in Fig. 14.60(c).
14/58 T^'teuUefr'A Fundamental Physics (XH)EEDBD

FIGURE 14.60

A B Y' Y A B Y
A«— = A+B
0 0 0 1 0 0 1
-vwv
1 0 1 0 1 0 0 B—W-
Ri.
0 1 1 0 0 1 0

1 1 1 0 1 1 0

o o OR Gate
V
NOT Gate

ww
Retain in Memory
NAND gate and NOR gate are the universal gates as all other gates can be obtained with their
help.

Flo
ee
(3) EX OR gate. It can be obtained by using OR, AND and NOT gates as shown in Fig. 14.61(a). The

rere
operation EXOR checks for the exclusivity in the value of the two signals A and B. It means if A and B are

rFF
not identical (i.e. if A = 0 and 5 = 1 or vice versa), the outputy = 1, and if both are identical, then the output
y = 0. This operation is also called exclusive OR gate, designated EXOR. The Boolean expression of this
uurr
operation is given by

y = A.B + A .B = A®B
foor
ks s
Yoo
The logic symbol and the truth table for EXOR gate is shown in Fig. 14.61(Z>) and (c).
oook

Note that when EXOR gate is worked with more than two signals, then the output y = 0, for even
eBB

number of signals being 1 or all signals being 0, and give y = 1 for odd number of signals being 1.
FIGURE 14.61
uurr

Y' A B Y
ad

0 0 0
Yo

Ao- 0 1 1
●oY
■oY
dY

1 0 1
Bo-
1 1 0
Re
innd

o o
FFi

(4) EXNOR gate. If EXOR gate is combined with NOT FIGURE 14.62
A B Y
gate we get EXNOR gate. The EXNOR operation is the negation
0 0 1
of the EXOR observation. The boolean expression for EXNOR A 0 1 0
gate is
1 0 0
B
y=A®B 1 1 1

The symbol and the truth table for EXNOR gate is shown
in Fig. 14.62(a) and (b).
O o

14.47. INTEGRATED CIRCUIT

Integrated circuit is that circuit in which the circuit components such as resistors, capacitors,
diodes and transistors etc. are automatically parts of a small semiconductor chip.
SEMICONDUCTOR ELECTRONICS MATERIALS, DEVICES AND SIMPLE CIRCUITS 14/59

It means an integrated circuit consists of number of circuit components (e.g., resistors, capacitors
inductors, diodes, transistors, logic gates etc.) and their inter-connections in single small package to perform
a complete electronic function. The various components of the integrated circuit are formed and connected
within a small chip of semiconducting material.

ww
Flo
e
rere
r FF
uurr
foor
kss
Yoo
ooook
eBB

Fig. 14.63(a) shows a thin slice of single crystal of silicon about 0 05 cm thick called silicon wafer. The
uurr

wafers may have diameter ranging from 2-5 cm to 10 cm. These wafers are u.sed to make devices and integrated
ad
Yo

circuits.

Fig. 14.63(/)) shows a small portion of the silicon wafer of dimensions 50 mil x 50 mil. (where mil
dY

stands for milli inch and 1 mil = 2-54 x cm). This small portion of the wafer is called a silicon chip.
Re
innd

The area required for a resistor, diode and a transistor is of dimensions (12 mil x 2 mil), (4-5 mil x 3 mil) and
(6-5 mil X 4 mil) respectively, and have been shown on silicon chip. The various circuit components such as
Fi

resistors, capacitors, inductors, diodes, transistors and logic gates etc. can be grown over such one silicon
chip. These components are connected internally to produce a desired circuit.
Fig. 14.63(c) shows the mounting of silicon chip into a casing, which has pins connected internally to
the integrated circuit and other ends of the pins are brought out of the casing for external connections.
Classification of Integrated Circuits
The integrated circuits are classified into following categories depending on the number of components
fabricated on a chip.
1. Small scale integration circuits (SSI). These integrated circuits have circuit components < 10.
2. Medium scale integration circuits (MSI). These integrated circuits have circuit components < 100.
3. Large scale integration circuits (LSI). The.se integrated circuits have circuit components < 1000.
4. Very large scale integration circuits (VLSI). These integrated circuits have circuit components > 1000.
The technology used for the fabrication of IC is quite complicated but is becoming cheaper due to large
scale industrial production.
14/60 “PniicCce^ 'a Fundamental Physics (XII) kviwii

The following processes are involved in the fabrication of integrated circuits :


(1) Epitaxial growth. This process is used to have a layer of n-type or p-type on silicon chip as and
when desired.

(2) Oxidation. This process is used to have a layer of insulating Si02 on silicon chip which is able to
separate one region of silicon chip from the other.
(3) Photolithograph. By this process, the different regions of silicon chip are selected photographically
where the different components to be used in integrated circuits can be installed.
(4) Diffusion of different impurities. This process is used to obtain different device structures on
silicon chip.
(5) Metallisation. This process involves the deposition of metal films which may interconnect different
components installed on chip to obtain the required circuit.

loow w
14.48. ADVANTAGES OF INTEGRATED CIRCUITS
OVER CONVENTIONAL ELECTRONIC CIRCUITS
1. They are highly reliable due to lesser number of connections.
2. They require very small space, as the various circuit elements are fabricated in a single chip of

ee
semiconducting material.

Fr
3. They have lesser weight due to very small circuits.
4. Their total cost is low.
r FF
rree
5. They require low power to operate.
fofr oF
u
6. They have greater ability to operate at extreme values of temperatures.
ks
14.49. LIMITATIONS OF INTEGRATED CIRCUITS OVER CONVENTIONAL CIRCUITS
YYouro
soo

1. If any component in an integrated circuit (IC) goes out of order, the whole IC has to be replaced by
BBook

new one.
r ee

2. It is not possible to produce high power (greater than 10 watt) integrated circuits.
3. In an integrated circuit, it is not possible to fabricate inductors and transformers on the surface of
ouru
ad

single semiconductor chip. These components are connected from outside to the semiconductor chip.
Yo

14.50. USES OF INTEGRATED CIRCUITS

The integrated circuits are widely used in making the television, radio, video cassette recorders and
d
Re
iYn

computers.
Large scale availability of computers in the market has been made possible by the IC technology.
FFind

Curiosity Question
f Q. Transistors are used as amplifiers in almost every electronic instrument. But in computers,
several transistors are connected together to perform logic operations or to add numbers
together. Is each transistor in computer working as amplifier ?
Ans. No ; In computers, small currents in the base-emitter circuits can turn on or off large currents in
the collector circuits. Due to it, these transistors in computers act as fast switches and not as
amplifiers.
1
SEMICONDUCTOR ELECTRONICS MATERIALS, DEVICES AND SIMPLE CIRCUITS 14/61

rw
TYPE I. INTRINSIC AND In a pure semiconductor; no. den.sity of electron
EXTRINSIC OF SEMICONDUCTOR is equal to no. density of hole. Therefore number of
holes in the given sample = 12 x 10^^
Formulae used. Example A semiconductor has equal

e
electron and hole concentration of 2 x 10^ m“^.
{i)E^ = hc/X (») nf On doping with a certain impurity, the hole

r
(Hi) Electric current, 1 = eA (n^ U;,) concentration increases to 4 x 10^® m”^. (/) What

o
(iv) Electrical conductivity. type of semiconductor is obtained on doping ?

llou
F
{//) Calculate the new electron hole concentration

w
!
of the semiconductor, {in) How does the energy
gap vary with doping ? (CBSF 2001)

FF
Units used. Eg is in joule, h in J-s ; X in metre

s
Solution. (/) Since on doping a semiconductor,
n
e' in m"^. I is in ampere, A is m-, or Vf^ in the hole concentration = 4 x lO’^ m"^) becomes
ms
-1
uro
, a in Sm“’, p in Q m, e in coulomb, p and P;, greater than electron concentration {n^ = 2 x 10^ m"^),

ok
rere
are in m^ V"' s"’.
so p*type semiconductor is formed on doping.
Ui) Here, Hj = 2 x 10^ m
-3 .

roF
Example Q The mean free path of = 10'*^ m ^ \ n = 9

fof
conduction electrons in copper is about 4 x 1(H m.
o
Y
(2x10^)-
ooY
Find the electric field which can give, on an
average, 2 eV energy to a conduction electron in a
n
= 10^ m-3
4xlO’0
B
n
ks
p
block of copper.
Yo
Solution. Here, mean free path, uii) Energy gap decreases with doping due
r

(/ = 4x 10“^ m; to formation of acceptor energy levels just above the


eB
e

valence band in the energy gap region.


ru

Electric field, E=1 \


d

Energy of electron = 2 eV Example Q Suppose a pure Si-crystal has


o

Force on electron in electric field = eE


5 X 10^* atoms m“^. It is doped by 1 ppm concen
nou
ad

The work done on electron when it moves tration of pentavalent As. Calculate the number of
iY

through distance d = e Ed. This work done is equal electrons and holes. Given that ri; - 1-5 x 10^^ m"^.
to the energy transferred to the electron, so (NCfc. ● rnpie)
nd
F

eEd=2&V
Re

1
Solution. 1 ppm = 1 part per million =
£=^= 2V
10^'
Fi

or = 5 X 10^ V/m
d 4x10"® m So no. of pentavalent As atoms doped in given Si
Example @ In a pure semiconductor, the 5x10^®
number of conduction electron is 6 x 10^^ per cubic crystal, -
10^ = 5 X 10^- m As one penta
metre. How many holes are there in a sample of valent atom donates one free electron to the crystal
size 1 cm X 1 cm X 2 mm ? structure, so no. of free electrons in the given Si
Solution. Number density of electron, crystal, = 5 X 10^^ m ^
«^ = 6x 10’^ m"^ ;
Volume of sample, V = 1 cm x I cm x 2 mm
n.
f _ (1-5x10'^)^
Number of holes, Pj, = —
= 10"^ m x 10"“ m X 2 x 10“^ m
n 5x10^2
2 X 10"^ m^ = 4*5 X 10^ m"^
No. of electrons in the given sample Exam
pie 0 Suppose the energy liberated
= «^x V=6 X 10*^ X 2x 10" in the recombination of a hole-electron pair is
= 12 X 10'-. converted into electromagnetic radiation. If the
14/62 ^>utdeep-’4- Fundamental Physics (X11)BSI9D
maximum wavelength emitted is 660 nm, what is 1
or p = - 0-508 ohm-m
the band width ? (Use = 6*6 x J-s) 1-968
Solution. Here, X = 660 nm - 660 x 10“^ m
Bandwidth is equal to the energy corresponding
to wavelength X-, which is given by I Formulae used.
TYPE II. p-n JUNCTIONS

he
(6-6xl0~^^)x(3xl0^) j AV
X 660x10"^ (/) Dynamic resistance, =
M

(6-6x10-34)x(3x10^) eV = 1-875 eV (//) Wattage of diode = voltage drop x current


(660x10-^)x(1-6x10-*^) (m) The relation for the current / in the junction

ww
diode is
Example A semiconductor is made
eVIkT
I = lQ(e -1)
extrinsic by adding trivalent impurity. The
resulting semiconductor should be of (iv) In p-n junction as a half wave rectifier

Flo
conductivity 12 mho m“^ Mobility of holes is (a) Maximum primary voltage, vpm = V2 V

e
400 cm^ V~^ s”^. Neglecting electron contribution

eree
find the number density of accetor atoms. Charge where Vrms is .rm.s. primary voltage.

FFr
on an electron = 1-6 x 10“^® C. (b) Maximum secondary voltage,
uurr
Solution. Here, o = 12 mho m"', n

orr
V^=V....=V x-^
P/, = 400cm“ V-* s"' 0 sm pm

= 400x 10^ V-i s~K sfo


(c) Maximum secondary current, /q =
n
P
V.
0
kks
As o = eW^'i,;,
Yoo
oooo

a 12 (d) Half wave rectified current, i.e., mean load


or N^ = current
eBB

(l-6xl0"*^)x(400xl0^)
Uc. = y^ Irr„s = I(/2
= 1-875 X 10^‘ m-3
urr

Example B A semiconductor is known to (e) Output d.c. voltage = ^ x n


ad

have an electron concentration of 5 x 10^^ cm“^


YYo

(/) Output .rm.s. voltage = Vq/2


and a hole concentration 8 x 10*^ cm“^ (a) Is the
(v) In p-n junction as a full wave rectifier
semi-conductor n-type or p-type ? (b) What is
dd

(fl) Maximum secondary voltage,


the resistivity of the sample, if the electron mobility
Re
inn

is 23,000 cm^ V“* s“* and hole mobility is 100 cm^ V = V x^ = Viv, "5
x—^
V”^ s~^ ? Take charge on electron, c = 1-6 x 10“^^ C.
F

.im pm nns
n n
p p
Solution. Here,
(b) Full wave rectified current, i.e., mean load
= 5x 10^-cm-^ = 5x lO’^m -3 .
21 I
19 -3 . 0
n,, = 8x 10'3
-3
cm= 8 X 10 m current, / 2. and 1 nns
dc
V2
[i^ = 23000 cm- V-’ s"’= 2-3 V's"* ;
n

p/, = 100 cm2 v“i S-’ = 0-01 m2 V"* s"‘. 21


0
(c) Out put d.c. voltage = xRl
(fl) Since the semiconductor has greater hole TZ

density, hence it is p-type


Example!^ The following table provides
(/7)Now, - =e(ii^p.^ + n,,[l,,) the set of values V and I obtained for a given diode.
Assuming the characteristics to be nearly
= 1-6 X 10-'^ [5 X 10‘*^x2-3-f 8 X lO’^^xO-OI] linear, over this range, calculate the forward and
= 1-6 X 10"‘9 [1-15 X 10‘^ + 0-08 X 10'^] reverse bias resistance of the given diode.
= 1-968 [CBSE 2007 (C)l
SEMICONDUCTOR ELECTRONICS MATERIALS, DEVICES AND SIMPLE CIRCUITS 14/63

V I (ii) If now germanium diode is connected to


12 V in reverse poJarity, find new values of Vq
Forward 2-0 V 60 mA and I.
biasing 2-4 V 80 mA
Solution. (/) As germanium diode conducts at
Reverse OV OpA lower voltage (0-3 V) than silicon diode which
biasing -2 V 0*25 \lA conducts at 0-7 V, so the current will prefer to pass
through germanium diode. Then
Solution. In forward biasing : Output voltage, Vq = 12 - 0-3 - 11*7 V
AV=24-2-0 = 04 V :
V 11-7
A/ = 80 - 60 = 20 mA = 20 X 10“^ A and current, / = —
0
= 2-34x 10-3 A
Forward resistance of diode. R 5x103
AV 04 - 2*34 mA
= 20Q

w
(ii) When germanium diode is reverse biased, it
"

A/ 20x10-3
will offer very large resistance to the current flowing
In reverse biasing : AV =-2-0 = -2V ;
through it. Now the current will flow through the
A/ = -2-5-0 = -2-5 pA = -2-5 x A

Flo
silicon diode. Then
Reverse resistance of diode,
output voltage, Vq = 12 - 0-7 = 11*3 V

reee
-2
^rb = = 8 X 10^’ .Q V 11-3
-2-5x10-^ and current, I = —
0 _
= 2-26x 10-3 A

FFr
R 5x1Q3
Example E] A p-n junction when forward - 2*26 mA
biased has a drop of 0*6 V which is assumed to be
uurr
independent of current The current in excess of
10 mA through the diode produces a large Joule for
Example DD Find the total current in the
circuits shown in Fig. 14.65(a), (/»), (c) and (d).
kkss
heating effect which bums the diode. If we want to Each diode used is ideal
use 1.5 V battery to forward bias the diode, what
Yo
oooo

FIGURE 14.65
should be the value of resistor used in series with the
diode so that the maximum current does not exceed Di D2
eB

6 mA ?
Solution. Here, voltage drop across p-n O
rr

3V ion
junction, = 0.6 V, E.M.F. of battery used = 1.5 V h
ou
ad

Let R be the maximum value of resistance R


YY

D1 D2
used in sereis with the p-n junction so that the
maximum current is 6 mA. Using Ohm’s law, we get,
nndd

IR + 0.6 = 1.5 or IR = 1.5 - 0.6 = 0.9 o


Re

3V ion
0-9 0-9
Fi

or R = = i5on
/ 6x10-3 Di
Exam
(!) Calculate the value of
□2
output voltage Vq and current I if silicon diode {>!
and germanium diode conduct at 0*7 V and 0*3 V o
3V ion
respectively. Fig. 14.64. ●'VW

Di
Ge FIGURE 14.64
i>l
1 I □2
Si
o
3 V ion

S
5kn ^ Vo
Solution. In Fig. 14.65(a); both the diodes D 1
=-12 V
and £>2 are forward biased. Their effective resistance
T in circent is zero. Current in circuit is
14/64 Fundamental Physics fXIT1 PZgTWn

= 0-3 A Example EB A full wave rectifier uses two


10
diodes, the internal resistance of each diode may
In Fig. 14.65(/>); diode Dy is forward bias and be assumed constant at 25 Q. The transformer
diode D-y is reverse biased. Resistance due to D2 will r.m.s. secondary voltage from centre tap to each
be infinity to the current fiowing through it. So current end of the secondary is 50 V and load resistance is
3-0 975 n. Find (i) the mean load current (ii) the rms
in the circuit. I = = 0 value of load current.
104- CC

Solution. Here, r=25Q.\R = 975 il;


In Fig. 14.65(c) ; Both diodes Dy and D-> are .rm.s.
= 50 V
forward biased, their effective resistance in the circuit
is zero. So current in the circuit is So, Fy = V2 F. r.m.s.
= V2x50 = 70-7V
Max. load current
30

w
/ = = 0-3 A F, 70-7
^0 _
10 /
0
= 70-7x10-3 A
r+R (254-975)
In Fig. 14.65(d) ; Diode Dy is forward biased
and diode Dt is reversed biased. No current will flow Mean load current,

Flo
through D2 as its resistance becomes infinity being 21
0 _ 2x70-7x10-3
reverse biased. The resistance to current by Dy is zero. I = 45x10*3 A

ee
m
71 22/7
The current in the circuit is due to Dy only, which is

Fr
given by (ii) Root mean square value of load current,
3-0
/
0 70-7x10-3
I = = 0*3 A I = 50xl0-3mA

for 2r~ .s
ur
r.m.s.
10

Example [Q In the Fig. 14.66, find out the Example The V-I characteristics of a
ks
current passing through and zener diode. silicon diode is shown in the Fig. 14.67. Calculate
Yo
the resistance of the diode at (a) Iq = 15 mA and
oo

FIGURE 14.66
(&)Fo = -10V. (CBSE 2015)
eB

I II
* (NCERT Solved Example)
R=80D I2’' FIGURE 14.67
J
l(mA)
ur

10V
svE ;>Ri_ = ioon
ad
Yo

Silicon
20
/
IM). board 2010) /
d

15
Re

Solution. Here, F. = 5 V,
in

10
Voltage drop across R,
F

F^ = input voltage -F. = 10-5 = 5V


F 5V -10V
R _
Current through R. I — 0.7 0.8
V(V)
R 80 a IpA
= 6-25 X 10-3 ^
Voltage drop across = F. = 5 V. Solution. In forward characteristics of p-n
junction, at / = 15 mA, the diode characteristics is a
Cunent through R^ is, = 5-= 3x10-3 A straight line between / = 10 mA to / = 20 mA.
100
(a) From the forward characteristic curve,
Since, / = 72^4-/,. So at / = 10 mA, F= 0-7 V and at / = 20 mA,
Current through zener diode is, F = 0-8 V. Using ohm’s law
/, = / - /^ = 6-25 X 10-3 - 5 X 10-3 R
AF ^ (0-8-Q-7)V _ 0-lV = ioa
= 1*25x10-3 A Jb
A/~(20-10)mA 10mA
SEMICONDUCTOR ELECTRONICS MATERIALS, DEVICES AND SIMPLE CIRCUITS 14/65

ib) From the reverse characteristic curve,


at V = -10V,/ = -l^A I TYPE ill. TRANSISTORS

R
V _ -lOV = io^n Formulae used. (/) }g = lfy +
I ~^T|Ia
rh ~
I Al
(«) and ot uc
Example!]^ In Zener regulated power e
A/

supply a Zener diode with = 6*0 V is used for A/


regulation. The load current is to be 4*0 mA and
the unregulated input is 10*0 V. What should be
(»') >?dc = ^ "
the value of series resistance R ?
(iv) For common base amplifier
(NCERT Solved Example)
AV
Solution. In order to have a good load voltage gain, \ - £- = « ai x-^

w
AV. /?-
regulations, the value of series resistance R should
be such that the current through the Zener diode is (v) For common emitter amplifier
much larger than the load current. For this, we select

Flo
the Zener current five times the load current, i.e., AV^. R.
voltage gain, AV = ^ac
P x-^^

/, = 5 X 4 = 20 mA. R:

reeee
The total current through series resistance (vi) Power gain = voltage gain x current gain

FFr
/? = /. + /^ = 20 + 4 = 24 mA.
The voltage drop across R= 10 0 - 6 0 = 4-0 V Example m The current gain in common

for
emitter amplifier is 59. If the emitter current is
4-OV
ur
Therefore, R = = 167fl
24 mA 6*0 mA, find, (/) base current (ii) collector current.
kkss
(CBSE 2003)
Example [0 A 6 V zener diode along with
Solution. Here, (3 = 59, Ig = 6-0 mA
Yo
a series resistance R is connected across a 12 V
oo

I
supply. Calculate the minimum value of the =Lzl^=L^-1
eB

'

resistance R required, if the maximum zener


current is 30 mA. 6
59 = —-lor- = 60 or /. = —
- = 0'lmA
Solution. Refer to Fig. 14.68, V,- = 12 V,
r

h * 60
ou
ad

/i1 - 30 mA
/^ = /^-/^ = 60-01 =5-9 mA
YY

FIGURE 14.66
Example (0 The potential difference
ndd

I R
Re

V across the collector of a transistor, used in common


emitter mode is 1*5 V, with the collector resistance
Fi

I1
of 3 kQ. Find (i) the emitter current and (//) the
V,- Rl V.
>● base current, if the d.c. gain of the transistor is
50. [CBSE 2003 (C)l
Solution. Here, VCE = 1-5 V,/?c=3 X 10^ a ;
When zener current is maximum, then load P = 50.
V i-5
current is minimum {i.e., /●? = 0) - _ 'CE _
= 0-5x10-^ A
Maximum current, i = l\ + /2 = 30 + 0 R(j 3x10^
= 30 mA = 30x 10“3 A
Voltage drop across zener diode = 6 V
I _
B
_ 0-5xIQ-^ = 0-01 X lO""-^ A
P 50
Since / is maximum, so minimum value of R is
= O’Ol inA
V. -V 12-6
z
R =
I 30x10-3
= 200t2 = hi + ^ 1^"'^ + 0-5x10-'"'
= 0-51 X iO-^ A = 0-51 niA
14/66 U Fundamental Physics (XII) L^AIWU

Example m For a common emitter tran- 0-25 X 2 = 0-50, with a carry 0


sistor amplifier, the audio signal voltage across the 0-50 X 2 = 1-00, with a carry 1
collector resistance of 2 kQ is 2 V. Suppose the Binary equivalent of (61-25)]q
current amplification factor of the transistor is = (111101-01)2
100, hud the input signal voltage and base current,
if the base resistance is 1 k£2. (CBSE 2016,2017)
Example^ What is the decimal number
of binary number (111001-01)2 ’
Solution. Here, = 2 x 10^ ; Vq = 2 V,
Solution. (111001 01)2 = (111001)2 + (0-01)2
p„^.= 100, Rj= 10^ .Q

oww
V.
(111001)2= 1 X 20 + 0x2' + 0x2^
A = — = 3 X + 1 X 2^ + I X 2^ + 1 X 2^
Voltage gain,
V YI <^ac R.
I = 1+ 0 + 0 + 8+16 + 32

2x10^ = (57)|0

e
= 100x = 200
(0-01)2 = 0 X 2-' + 1 X 2-2

re
= 0 + 0-25
V,

FFrllo
or V. = '0 _ — = 0-01 V
= (0-25) 10

reF
200 200
(111001-01)2 = (57),o-H(0-25),o

e
K 001
Base current. = 10"'' A = (57-25)10
uoru R: 10^

osFr
Example^ In a silicon transistor, a
Exam
pie ^ What is the binary additon
of (101010)2 and (OlOlODj ?
fkfor
change of 7-89 mA in the emitter current produces
Solution. 10 10 10
a change of 7-8 mA in the collector current. What
okso
change in the base current is necessary to produce 0 10 10 1
an equivalent change in the collector current ?
Y
Yo
111111
oo

Solution. Here, A /^ = 7-89 mA,


BB

A /^ = 7-8 mA, A 4 = ? Le. (101010)2 + (010101)2 = (151111)2


A/^ _ 7-8
Y
r ree

As aa.c. = 0-9886 Example Subtract (10101)2 from


ouu

A 7-89
(111001)2.
ad
Ydo

a 0-9886 Solution. 1110 0 1


a.c.
Now P a.c.
= 86-72
10 10 1
1-a 1-0-9886
a.c.
nidn

10 0 10 0
As P,.,, = AVA7,
Re

A/^ _ 7-8 = 89-94 X 10-3 Le. (111001)2 - (10101)2 = (100100)2


F

A/,=
Fi

P ~ 86-72

I
a.c.

TYPE IV. DECIMAL NUMBER I TYPE V. LOGIC GATES

SYSTEM AND BINARY SYSTEM (0 for OR gate ; A+B=y


(/i) for AND gate ; A.B=y
Example m What is the binary equivalent (in) for NOT gate ; A= y
of (61-25)io ?
(iv) for NOR gate ; A+B= y
Solution. 2 61
(v) for NAND gate ; A-B = y
2 30-1
2 15-0
(vi) for EXOR gate ; A-B + A-B = y
2 7-1
2 3-1 Example @3 You are given the two circuits
2 1-1 as shown in Fig. 14.69 (a) and (b). Show that circuit
0-1
SEMICONDUCTOR ELECTRONICS MATERIALS, DEVICES AND SIMPLE CIRCUITS 14/67

(a) acts as OR gate while the circuit (b) acts as


AND gate. (CBSE 2011, 2013, 2014 (C)]
A B x=A y=^B A+B y=A + B

FIGURE 14.69 0 0 1 1 1 0

I 0 0 1 1 0

0 1 1 0 1 0
1 1 0 0 0 1
A
As output is 1 only if both the inputs are 1. So,
the circuit works as AND gate. The logic symbol of

oww
B ●- the AND gate is shown in Fig. 14.71.
FIGURE 14.71
Solution. In Fig. 14.69(ci), the first gate is a
A
NOR gale and second gate is NOT gate.
>

e
B

FFrlo
The Boolean expression of this combination of

re
gates is
Example Identify which basic gate,

ree
F
y' ^ A + B ^ A -B OR, AND and NOT is represented by the circuits
in the dotted line boxes 1, 2 and 3 Fig. 14.72.

rF
y = y = A.B = A + B = A + B
FIGURE 14.72

fsoor
ouur
i.e.,y = A + B. It is a Boolean Expression of OR 1 2
gate. Thus, circuit {a) acts as OR gate.
skf I
I
I
I
“I 3

In Fig. \A.69{b), the output of two NOT gates T


u
I
ooko
I I

Q
I
is made input to a NOR gate.
Yo
_j r 1 0
\
Y
r
The Boolean expression of the combination of
O
B I
Bo

o-
B
reB

gates IS I

y= A + B = A.B = A.B
uur

Solution. Crt) In dotted line box 1 is a NOT gate.


oY

i.e., y = A-B. It is a Boolean Expression of AND


ad

Explanation. Its circuit is as shown in Fig. 14.73.


gate. Thus, circuit (i>) acts as AND gate.
dY

Example ^ You are given a circuit below.


FIGURE 14.73

>
Write its truth table. Hence, identify the logic
innd

A*A = A
Re

O O
operation carried out by this circuit. Draw the A Y
Fi
F

logic symbol of the gate it corresponds.


FIGURE 14.70
Here y = A.A = A + A = A = NOT A
A

X
ih) In dotted line box 2 is an OR gate.
Explanation. Its circuit is as shown in Fig.
Z
II y 14.74.
III
B
FIGURE 14.74

>
(CBSE 2«H) A
A o
A.B = A+B
Solution. Here, gate I is NOT gate, gate II is
NOT gate and gate III is NOR gate. Therefore
> ●O
Y

x=A, y = B and z=A + B =A ■ B = AB


Truth table of the circuit is given below :
BO-
> B
14/68 ‘Pxixdec^ A Fundamental Physics (XII)

Here, y = A.B=^A-^B = AORB A.B.C^-A.B.C-^A.B.C -A.{B + C)


(From De-Morgan’s Theorem) Solution. Taking L.H.S. of the given identity,
we have
(c) In dotted line box 3 is AND gate.
A.B.C + A.B.C-^ A.B.C
Explanation. Its circuit is as shown in Fig.
14.75. = A.{B.C + ~B.C + B£)
FIGURE 14.75 = A.\B.(C + C) + B.C]
A

>Y = (A.B).(A.B)
= A.\B.{\) + B.C] = A.[B -J- B.C]
>
O- A.B
o- 0 = A.B +A.B
B = 2(A.B) (V C + C = l)
= A.B
= A.(B + Q (V B + 'b.C = B+C)

w
Wliich is the right hand side of Uie given identity.
Here, y = (A.B).(A. B) = A.B+A.B
Applying de Morgon’s theorem
I I

Flo
TYPE VI. TYPICAL EXAMPLES
(0 C + D = C.D (/O C.D = C + D
Example 0j] If resistivity of pure silicon is

ee
y=A.B + A.B = A.(B + B)^A.B
3000 Q m, and the electron and hole mobilities

Fr
= A ANDfi. (v B + B = BORB = B)
are 0.12 m^ V”^ s"' and 0.045 V“^ s -I
Similarly, we can explain the values of y for
other values of A and B in truth table.
respectively, determine

for
ur
(a) the resistivity of a specimen of the
Example ^ Write the truth table for the material when 10^^ atoms of phosphorous are
circuit shown in Fig. 14.76. added per m^.
ks
Yo
FIGURE 14.76 (b) the resistivity of the specimen if further
oo

A V^ V2
2 X 10^^ boron atoms per m^ arc also added.
Given charge on electron = 1*6 x 10“*^ C.
eB

B
III
Solution. Here, p = 3000 m,
= 0.12 m- V-’ S-', p;, = 0.045 V' s”'
ur
ad

Solution. Here gate 1 is OR gate, gate II is NOT The resistivity of pure Si is given by
Yo

gate and gate III is AND gate. 1 1 1


.-. Output of gate I is >’l = (A + B) P = -
a
The output of gate II is,
d
Re
in

^2 = ->j = (A + S)= A-5 1


F

or ih =
The output of gate III is y = y^B = {A - B)- B I

The truth table for the circuit is shown below.


1

A B A B y2=(A-B) y = (A-B)-B 1.6 X Ur ‘ ^ X 3000 (0.12 + 0.045)


0 0 1 1 1 1 = 1 26x 10 16 ni
_-3

0 1 1 0 0 0 (f?) When lO’^ atoms of phosphorous {i.e. donor


1 0 0 1 0 0 atoms of valence five) are added per m^, the
1 1 0 0 0 0
semiconductor becomes n-type semiconductor. Then
19
1.26 X 10^^)
Thus output is 1 when both the inputs are zero. 1
This truth table is similar to NOR gate. Resistivity, P =
e/i p
Example m Let A, B and C be any three 1
logic variables, prove the following Boolean = 5.21 Q m
1.6xl0'‘^xl0'^x0.l2
identity.
SEMICONDUCTOR ELECTRONICS MATERIALS, DEVICES AND SIMPLE CIRCUITS 14/69

ih) When 2 X lO’^ atoms of boron (/.e. acceptor = 1.6 X 10-'^ X ( 0.45 X 10*^x0.135
atoms of valence three) are added per the + 5x 1022x0.048)
semiconductor becomes p-typ e semi-conductor. Then
= (9.72x 10-” +384) Sm"*
nf, - ~N^-N^j=2x 10*^ = 384 Sm-‘
Since hence iijj = ( V «, = n,) 1 1
Resistivity, p = —
Resistivity, p = o ” 384
= 0.0026 Q m

Electrical conductivity of pure Si-crystal


1.6xl0-'*^xl0‘*^x0.045 a = e «, (p^ + p;,)

oww
= 13.89 a m
= 1.6 X I0“'‘^x 1.5 X !0*^(0.!35 +0.048)
= 0.4392 X 10--^Sm-'
Example m Predict the effect on the
electrical properties of a .silicon crystal at room 1
Resistivity, p = —

e
temperature if every millionth silicon atom is o 0.4392x10“^

FFrlo
re
replaced by an atom of indium. Given, = 2272 n m
concentration of silicon atoms = 5 x lO^^ m“^,

ree
Thus we see that the conductivity of silicon

F
intrinsic carrier concentration = 1*5 x 10*^ m"^ ;
p^ = 0.135 m2/Vs and Py, = 0.048 m2/Vs. doped with indium (384 S m”*) becomes much greater
than its intrinsic conductivity (0.4392 x 10“^ S m“*)

rF
Solution. Here, concentration of Si atom
and the resistivity (= 0.0026 fl m) has
= 5 X 102^ m“^

fsoor
become much smaller than the intrinsic resistivity
ouur
Doping of indium is 1 atom in 10^ atoms of Si. skf (2272 a m).
Indium has valence three. Each doped indium atom
Example A potential barrier of 0*4 V
creates one hole in Si-crystal and hence acts as
ooko
exists across a p-n junction, (a) If the depletion region
Yo
acceptor atom.
is 4*0 X 10“ ^ m wide, what is the intensity of the
Y
Concentration of acceptor atoms,
Bo

electric field in this region ? If an electron with speed


reB

n,,=^5x iO-^x 10-^ m'^ 4*0 X 10® m/s approaches the p-n junction from the
= 5 X 1Q22 n-side, find the speed with which it will enter the
p-side.
uur

Intrinsic carrier concentration,


oY
ad

1.5 X 10'^’m“^ Solution. («) The elctric field.


dY

Hole concentration is increased by an amount V 0-4


£■ = - = 10 X 10^ V/m
5x1Q22 4-0x10“^
innd

= 3.33x10^
Re

n. 1.5x10*^
(/;) Let VI be the speed of electron when it enters
the depletion layer and V2 is the speed when it comes
Fi
F

New electron concentration, out of the depletion layer. According to principle of


conservation of energy
n
_ (1.5xl0‘^)2 = 0.45 X 10*0 m“3 /C.E. before entering the depletion layer = gain
5x1Q22 in .PE. + K.E. after crossing the depletion layer
Electron concentration has been reduced by an so. — mv7 = exV + —
1 2
2 * 2
mv2
«■
amount = —L =
(1.5x10*^) = 3 't3vin6
n, 0.45x10"^ or - X 9-1 X lO"-"** X (4 X 10®)2
2
It means the hole concentration has been
increased over its intrinsic concentration by the same = 1-6 X 10“*Ox04+ - X 9-1 X 10 -31 X t»2
amount with which the electron concentration has 2
been decreased.
or 0-728 X 10-*-‘* = 0-64 x IQ-*^ + 4-55
The conductivity of doped silicon is given by
X 10-"''*
<T=e(/i^P,. + «/,P/,)
14/70 ‘P'uidcefr’4' Fundamental Physics (XII)B&m

or 0-088 X 10"^^ = 4-55 x 10“^' ^2 n xe


(98/100)«^x<? 98
and /
On solving = 1*39 X 10^ m/s / t

Current transfer ratio.


Example SS In an npn transistor 10 10
I- 98
electrons enter the emitter in 10~ ^ s. If 2% of the a = — = 0-98
electrons are lost in the base, find the current / 100
transfer ratio and the current amplification factor.
Current amplification factor,
n xe
Solution. I = a 0-98
t P = = 49
\-a 1-0-98

ww
z^ir zsi g'.lamc®;

Floo
. Energy band of a solid. The large number of energy levels confined in a small region of energy range of a

ree
given solid, constitute what is known as energy biuids.
In some solids, there is an energy gap in between the energy bands. This energy gap is called forbidden

rFee
energy gap. The energy band above the forbidden energy gap is called conduction band and the energy band

F
oor r
rur
below the forbidden energy gap is called valence band.
The conductivity of a solid depends upon the number of electrons present in the conduction band and
s ff
number of holes present in valence band.
osk
' Classification of metals, insulators and semi-conductors on the basis of energy bands
YYoou
oook

(a) In metals. The conduction band and valence band partly overlap each other and there is no forbidden
eBB

energy gap.
(b) In insulators, the conduction band is empty and valence band is completely filled and forbidden energy
gap is quite large > 3 eV. No electron from valence band can cross over to conduction band at room
uur r

temperature, even if electric field is applied. Hence there is no conductivity of the insulators,
ad

(c) In semiconductors, the conduction band is empty and valence band is totally filled at OK but the
Yo

forbidden energy gap between conduction band and valence band is quite small, which is about 1 eV. No
electron from valence band can cross over to conduction band at OK. Therefore, the semiconductor behaves
dY

as insulator at OK. At room temperature, some electrons in the valence band acquire thermal energy, greater
Re
idn

than energy gap of 1 eV and jump over to the conduction band where they are free to move under the
FFin

influence of even a small electric field. Due to it, the semiconductor acquires small conductivity at room
temperature.
^ Hole. It is a seal of positive charge which is produced when an electron breaks away from a covalent bond
in a semiconductor. Hole has a positive charge equal to that of electron. Mobility of hole is smaller than that
of electron.

I Intrinsic semiconductor. A pure semiconductor which is free from every impurity is called intrinsic
semiconductor. Germanium (Ge) and Silicon (Si) are the important examples of intrinsic semiconductors.
In intrinsic semiconductor, n, = = n,-.
where ;i^, /i;, are the number density of electrons in conduction band anJ the number density of holes in
valence band, /i, is the number density of intrinsic carriers (electrons or holes) in a pure semi- conductor.
5. Doping. It is a process of deliberate addition of a desirable impurity to a pure semiconductor to modify its
properties in a controlled manner. The impurity atoms added are called dop(mts. The impurity added may
be - 1 in 10*'^ atoms.
6. Extrinsic semiconductor. A doped semiconductor or a semiconductor with suitable impurity atoms added
to it is called extrinsic semiconductor. Extrinsic semiconductor tire of two types :
SEMICONDUCTOR ELECTRONICS MATERIALS, DEVICES AND SIMPLE CIRCUITS 14/71

(i) n-type semiconductor. When a pure semiconductor of Ge or Si is doped with a controlled amount of
pentavalent atoms, say arsenic or phosphorous or antimony or bismuth, we get n-type semiconductor or
donor type semi-conductor.
It is called n-type semiconductor because the conduction of electricity in such semiconductor is due to
motion of electrons, i.e., negative charges, or n-type carriers. It is called donor type, because the doped
impurity atom donates one free electron to semiconductor for conduction. In n-type semiconductor electrons
are majority carriers and holes are minority carriers.
(ii) P‘type semiconductor. When a pure semi-conductor of Ge or Si is doped with a controlled amount of
trivalent atoms, say indium or Boron or aluminium, we get p-type semiconductor or acceptor type
semiconductor. It is called p-type because the conduction of electricity in such semiconductors is due to
motion of holes, i.e., positive charges. It is called acceptor type semi-conductor because the doped impurity
atom creates a hole in semiconductor which accepts the electron, resulting conduction in p-type semi

w
conductor.

In p-type semiconductor, holes are majority carriers and electrons are minority carriers.
7. Electrical conductivity of semiconductor (a) is the reciprocal of its resistivity (p) and is given by

Flo
1
a = - = e(/i^p^+/i;,p^)

e
reee
where are the number density of electrons and holes and P;, are the mobility of electrons and holes

Fr
in the given semiconductor.
8. Effect of temperature on conductivity of semi conductor. With the increase in temperature, more number

for
ur
of covalent bonds are broken, resulting large increase in current carrier concentration (i.e. electrons and
holes). Due to it the conductivity of semiconductor increases.
ks
Mobility of electrons and of holes in a semiconductor decreases with increase in temperature,
Yo
9. p-n junction. When ap-type crystal is brought into close contact with n-type crystal, the resulting arrangement
oo

is called p-n junction or junction diode.


eB

10. Depletion region. It is a layer created around the p-n junction which is devoid of free charge carriers and
has immobile ions.
ur

11. Forward biasing of p-n junction. When positive terminal of external battery is connected to p-side and
ad

negative to n-side of p-n junction, the p-n junction is said to be forward biased. In forward biasing, the
Yo

conduction across p-n junction takes place due to migration of majority carriers (i.e. electrons from n-side
to p-side and holes from p-side to n-side). The size of the depletion region decreases. The resistance of the
p-n junction becomes low.
d
Re
in

12. Reverse biasing of p-n junction. A p-n junction is said to be reverse biased if the positive terminal of the
external battery is connected to n-side and the negative terminal to p-side of p-n junction. In reverse biasing,
F

the conduction across the p-n junction does not take place due to majority carriers but takes place due to
minority carriers if the voltage of external battery is large. The size of the depletion region increases. The
resistance of the p-n junction becomes high in reverse biasing.
13. Dynamic resistance or a.c. resistance of junction diode is defined as the ratio of a small change in
voltage (A V) applied across the p-n junction to a small change in current AI i.e.. Rg = A V/AI
14. Characteristics of p-n junction diode. The graphical relations between forward bias voltage and forward
current are called forward characteristics. The graphical relations between reverse bias voltage and reverse
current is called reverse characteristics of p-n junction.
15. Knee voltage. It is the forward voltage beyond which the current through the junction starts to increase
rapidly with linear variation. But below the knee voltage, the characteristic curve is non linear.
The knee voltage for Si is 0.7 V and for Ge is 0.3 V.
16. Junction-Transistor. It is a semiconductor devise which is obtained by growing a thin layer of one type
semiconductor in between two thick layers of other similar type semiconductor, i.e., a junction transistor is
having two junctions and three terminals.
14/72 "pKeutte^'^- Fundamental Physics (XIOEHHD
If central thin layer is of p-type and outer thick layers are of 7?-type semiconductor, we get n-p-n transistor.
If central thin layer is of /i-type and outer thick layers are of p-type semiconductor, we get p-n-p transistor.
The thin layer of junction transistor is said to form the base (B). One of the thick layers serves as emitter (E)
and the other thick layer serves as collector (O.
The function of emitter is to emit the majority carriers. Function of collector is to collect the majority
carriers and base provides the proper interaction between the emitter and the collector.
Symbolically, npn transistor has been shown in Fig. 14.77(n) and pnp transistor in Fig. In these
figures, the arrow head indicates the direction of flow of positive charge between emitter and base.
FIGURE 14.77

w
Flo
ee
A junction transistor is a transference of resistance, which can be achieved by interchanging the biasing

Fr
across the junction triode, hence the name a junction transistor is given,
In the operation of a transistor
where is the emitter current, is the base current and is the collector current.

for
ur
17. In common ba.se transistor Amplifier : The output signal is in phase with input signal.
ks
(/) D.C. current gain ). It is defined as the ratio of collector current (/^.) to the emitter current (/^),
Yo
I.e.. ^d.c. ~
oo

(//■) A.C. Current gain (ot^ <.). It is defined as the ratio of change in collector current (A /^) to the change in
eB

emitter current (A /^) at constant collector voltage, i.e., a


(I.e.

(Hi) A.C. Voltage gain (A^/). It is defined as the ratio of change in output voltage (A to the change in
input voltage (A V',), i.e..
ur
ad

AV ALxR R
Yo

CB _
where R,/Rj is called resistance gain.
C o
● 0
= a X —
a.c.

AV I AI^xR. R:I
d

(iV) A.c. Power gain. It is defined as the ratio of change in output power to the change in input power i.e.
Re
in

AV^r^XAl AV \ ( A/
CB c _ CB c
F

a.c. power gam = X


AVxA/
t e
AVI
/ V

(Al^f X
R o ')
= OCTa.c.. X resistance gam
(A/^)2 /?.
I

18. In common Emitter Transistor amplifier : The output signal is 180“ out of phase with input signal.
(/) d.c. current gain ). It is defined as the ratio of the collector current (/^) to the base current (/^),
I.e.,

(ii) a.c. current gain ^). It is defined as the ratio of the change in collector current (A /^.) to the change
in base current (A /^) at constant collector voltage, i.e.,
P..c.=A/,/A/,
(Hi) IVansconductance (g„,). It is defined as the ratio of change in collector current (A /^,) to the change in
input base-emitter voltage (A V,), i.e..
SEMICONDUCTOR ELECTRONICS MATERIALS, DEVICES AND SIMPLE CIRCUITS 14/73

gm . Its unit is siemen or *


AV.

(iv) A.C. Voltage gain (Ay). It is defined as the ratio of the change in output voltage (A to the change
in input voltage (A V,), i.e..
AV

AV(

Here negative sign shows the phase reversal of output,


(v) A.C. Power gain. It is defined as the ratio of the change in output power to the change in input power,

a.c. power gain = change in oulput power ^ ^ ^


AV.CE
i.e..

change in input power A/^ AV-

ww
19. Analogue signal. A continuous time varying current or voltage signal is called analogue signal.
20. Analogue circuit. An electronic circuit which gives out any type of analogue signal is called analogue

Floo
circuit.

e
21. Digital signal. A signal which has two levels of voltage (represented by 0 and 1) is called digital signal.

eere
22. Logic gate. A digital circuit which either allows a signal to pass through or stops it, is called a gate. Such
gate allows the signal to pass through only when some logical conditions are satisfied. Hence they are

FFr
called logic gates.

oorr
uur r
23. Truth table. It is a table that shows all possible input combinations and the corresponding output
s ff
combinations for a logic gate. It is also called a table of combinations.
24. Basic logic gates are of three types
sk
YYoo
(/) OR gate is a device having two or more inputs and one output Fig. 14.78.
ooko

In Boolean algebra, the term OR is represented by plus (+) and Boolean expression for OR gate is
eBB

A + B = f, indicates Y equals A OR B.
FIGURE 14.78
FIGURE 14.79
uurr

Symbol of OR gate
ad

Symbol of AND gate


Yo


y y
dY

B*.
e
Re

e
nind

Truth table of OR gate Truth table of AND gate


FFi

A B Y A B Y

0 0 0 0 0 0

0 1 1 0 1 0
1

: : ©
1 0 1

1 1 1 e 1

(i7) AND gate. It is a device having two or more inputs and one output Fig. 14.79.
In Boolena algebra, the term AND is represented by dot (●) and the Boolean expression A.B = Y, indicates Y
equals A AND B.

(in) NOT gate. It is device which has one input and one output. Fig. 14.80.

In Boolean algebra, the term NOT is represented by bar symbol (-) and the Boolean expression A = Y ,
indicates that Y equals NOT A.
14/74 'a Fundamental Physics (XII)

FIGURE 14.80

Symbol of NOT gate


Truth table of NOT gate
y A B
A*- -●
0 1 ©
e 1 0

25. NAND gate. It is obtained when the output of AND gate is made as the input of a NOT gate. Boolean
expressionfor NAND gate is Y = A.B , Fig. 14.81.

w
FIGURE 14.81

Symbol
A*- y

Flo

eeee
o Truth table

Fr
A B Y

0 0 1

for
ur
1 0 1

0 1 1
o 1 1 0
ks
Yo
oo

26. NOR gate. It is obtained when the output of OR gate is made as the input of a NOT gate. Boolean expression
eB

for NOR gate is Y =A-\- B , Fig. 14.82.


FIGURE 14.82
ur

Symbol
ad
Yo

A» y
©
B ●-
z e
d
Re
in

Truth table
A B Y
F

0 0 1

1 0 0

0 1 0
o
1 1 0
SEMICONDUCTOR ELECTRONICS MATERIALS, DEVICES AND SIMPLE CIRCUITS 14/75

QUESTIONS

Based on NOERT Book

(C) (EPcX^PsiX^Pce
1. Multiple Choice Questions
(d) (E^)c<(E,)s,<(E^)g,
1. In Conductor, Semiconductor and Insulator, the 8. In an unbiased p-n junction, holes ditfuse from the
forbidden energy gap are £|, £2 ^3 /^-region to ?z-region because
respectively. Which one is correct (a) free electrons in the ?i-region attract them.
(a) £j < £2 < £3 (b) £^ > £2 = £3 (b) they move across the Junction by the potential
(c) £1 = £2 < £3 (d) £[ > £2 > £3 difference.

ww
2. Silicon is doped with which of the following to (c) hole concentration in p-region is more as
obtain P type semiconductor compared to n-region.
(a) Phosphorus (b) Gallium (i/) All the above.
(c) Germanium (d) Bismuth 9. When a forward bias is applied to a p-n Junction,

Flo
3. What happens to resistance of an intrinsic It

e
ree
semiconductor when heated (a) raises the potential barrier.
(a) increases (b) remains constant ib) reduces the majority carrier current to zero,

Fr
rF
(c) decreases (d) decreases linearly (c) lowers the potential barrier,
uurr
4. A semiconductor has an electron concentration (t/) none of the above.

for
of 6 X 10^^ per m^ and hole concentration of 10. In a p-type silicon, which of the following
8-5 X 10^ per m^. Then it is statement is true :
s
(a) N type semiconductor
kks
(a) Electrons are majority carriers and trivalent
Yo
(b) P type semiconductor
oooo

atoms are the dopants.


(c) intrinsic semiconductor (h) Electrons are minority carriers and peniavalenl
eB

(d) conductor atoms are the dopants,


5. What type of doping is used in Zener diode (c) Holes are minority carries and pentavalent
atoms are the dopants.
ur

(a) light (b) moderate


ad

(c) heavy (d) no doping id) Holes are majority carries and trivalent atoms
YYo

are the dopants.


6. In an u-type silicon, which of the following
statement is true ; 11. The intrinsic semiconductor becomes an insulator
dd

at
(a) Electrons are majority carriers and trivalent
Re
in

atoms are the dopants. (<7) 0°C (b) -100°C


(c) 300 K (d) OK.
F

{b) Electrons are minority carriers and pentavalent


atoms are the dopants, 12. In the forward bias arrangement of a W-junction
diode
(c) Holes are minority carries and pentavalent
atoms are the dopants. (a) The A^-end is connected to the positive terminal
(d) Holes are majority carries and trivalent atoms of the battery
are the dopants. (b) The £*end is connected to the positive terminal
7. Carbon, silicon and germanium have four valence of the battery
electrons each. These are characterized by valence (c) The direction of current is from 7V-end to
and conduction bands separated by energy band £-end in the diode

gap respectively equal to (£^)c, (£g)si and (£g)cc- (d) The 7*-end is connected to the negative terminal
Which of the following statements is true ? of battery
(fl) (£psi<(^Pce<(^Pc 13. In a £iV-junction diode
(a) The current in the reverse biased condition is
{b) (£pc<(^Poe<(£^Psi generally very small - |M
14/76 ^ Fundamental Physics (XII) P2STMT1
(b) The current in the reverse biased condition is 20. What is the current in the circuit shown in Fig.
small but the forward biased current is I4(Q).2
independent of the bias voltage
FIGURE 14(Q),2
(r) The reverse biased current is strongly
dependent on the applied bias voltage -4 V 300 a - 1 V

(d) The forward biased current is very small in


comparison to reverse biased current
14. A /^-type semiconductor can be obtained by (a) 10”- A (b) 10-3 A
adding (c) 1 A (d) zero
(a) Arsenic to pure silicon 21. The energy gap between conduction band and
valence band is of the order of 0 07 V. it is a/an
(b) Gallium to pure silicon

oww
(a) insulator (b) conductor
(c) Antimony to pure germanium
(c) semiconductor (d) alloy
{d) Phosphorous to pure germanium
15. Electrical conductivity of a semiconductor 22. In the forward bias/?-n junction diode, the potential
barrier in the depletion region is of the form
(a) Decreases with the irse in its temperature

e
(b) Increases with the irse in its temperature

ree
FIGURE 14(Q).3

rFl
(c) Does not change with the irse in its temperature Potential

Fre
4 Barrier ,. Potential
(d) First increases and then decreases with the rise Barrier

rrF
in its temperature
16. The cut-in voltage for silicon diode is
ouur
approximately
(a) 0-2 V {b) 0-6 V
sffoo
P )i P n
okks
(c) M V id) 14 V
Yo
(a) (b)
17. The depletion layer in the P-A/junction region is
ooo

caused by
eBB

(a) Drift of holes


A Potential ^ Potential
Barrier Barrier
(b) Diffusion of charge carriers
uurr

(c) Migration of impurity ions


ad

P n P n
YYo

(d) Drift of electrons


18. Which is reverse biased diode
dd

FIGURE 14(Q).1 (c) id)


Re
iinn
F

o 23. Refer to Fig. 14(Q).4, the current through the ideal


diode is
o o -20 V
-■-sv
-10 V FIGURE 14(Q).4

2 V 6 V
100 a
—W
0 15V 0 10V
ilO V
-5 V

19. If a full wave rectifier circuit is operating from


(a) (1/25) A (b) (3/50)A
50 Hz mains, the fundamental frequency in the
ripple will be (c) (1/50)A (d) Zero
(a) 50 Hz {b) 70-7 Hz 24. A sinusoidal voltage of peak value 200 volt is
connected to a diode and resistor R in the circuit.
(c) 100 Hz (d) 25 Hz
Fig. 14(Q).5, so that half wave rectification occurs.
SEMICONDUCTOR ELECTRONICS MATERIALS, DEVICES AND SIMPLE CIRCUITS 14/77

If the forward resistance of the diode is negligible are 5 and 0-5 respectively. If the input
compared to /?, the r.m.s. voltage (in volt) across R voltage is 0 02 V. the output voltage is
is approximately (fl) 0-06 V (h) 0 I2 V

FIGURE 14(Q).5 (c) 1-2 V Ui) 12 V


30. In a npn transistor lO'^ electrons enter the emitter
in 10“^ s. 4% of the electrons are lost in the base.
The current transfer ratio will be

Eq = 200 V R (n) 0-98 (h) 0-97


(c) 0-96 Ul) 0-94

31. In the Boolean algebra A.B equals

(a) 200 (h) 100 (a) A+B (*) A + B

ww
200 {c)A-B (d) A.B
(c) (d) 280
32. Following diagram performs the logic function of

Flo
25. Name the type of diodes whose V-I characteristics
FIGURE 14(Q).7
are shown in Fig. I4(Q).6(/) and (//),

e
A

rere
FIGURE 14(Q).6 V
B

r FF
Al 14
(a) OR gate (b) NOR gate
uurr
V
for
(c) AND gate {(I) NAND gate
33. In the given Boolean expression :
kss
Q y = A.B+ B .A A = 1,B = 1, then y will be
ooook
Yo

(a)0 ib) 1
0) 00
eBB

(c) II (d) 10
(a) (0 solar cell (»7) zener diode 34. What is the output y of the gate circuit shown in
Fig.l4(Q).8 ?
urr

(b) (0 zener diode (n) solar cell


ad

(c) {/) photo diode (ii) zener diode FIGURE 14(Q).8


Yo

{d) (0 zener diode (n) photo diode


}
A
dY

V
26. If the base and collector of a transistor are in
B«—[>
Re

forward bias, then it cannot be used as


innd

(a) a switch (b) an amplifier (a) A.B (6) A.B


Fi

(c) an oscillator (d) all the above


27. The current gain in common base transistor is 0-95. (c) A.B id) A.B
The current gain of the same transistor in common 35. Which of the following is not true in Boolean
emitter mode is
algebra ?
ia) 95 ib)45
ia) (1+1).1=0 (/?) (1 + 1).1=0
(c) 38 id) 19
28. In an n-p-n transistor the collector current is 24 (c) (0+ 1).I =0 id) (0 + 0).l = 0
mA. If 80% of electrons reach collector, its base 36. In computers several transistors are connected
current (in mA) is together to perform logic operations or to add
(a) 36 (h) 26 number together. Each transistor in operation is
(c) 16 id) 6 working as
29. In common emitter amplifier, the current gain is (a) an amplifier (b) a rectifier
60. The collector resistance and input resistance (c) a switch (d) an oscillator
14/78 Fundamental Physics fXIIlrosTI

II. Assertion-Reason TVpe Questions Reason. Drifting of minority charge carriers takes
place across the p-n junction in reverse biasing.
Direction. In each of the following questions two
statements are given, one labelled Assertion (A) 44. Assertion. An ideal junction diode can act like an
automatic switch.
and the other labelled Reason (R). Select the
correct answer to these questions from the codes Reason. An ideal junction diode can act as open
(a), (6), (c) and (d) as given below : switch when forward biased and close switch when
reverse biased.
(a) Both A and R are true, and R is the correct
explanation of A. 45. Assertion. A junction diode cannot work as an
(b) Both A and R are true, but R is not the correct amplifier.
explanation of A. Reason. A junction diode can be used as a rectifier.
(c) A is true but R is false. 46. Assertion. The photodiodes are used in reverse bias
condition.

w
(^0 A is false and R is also false.
Reason. The reverse saturation current in
37. Assertion. Diamond behaves as an insulator.
photodiodes is directly proportional to the light flux

Flo
Reason. There is a large energy gap of value 5-51 or light intensity but not so when photodiode is
eV in the energy band diagram of diamond. forward biased.

reeee
38. Assertion. The maximum energy which an electron 47. Assertion. Transistor is a current operating device.
in a material can have at 0 K, is called fermi energy.

FFr
Reason. The change in collector current is related
Reason. Fermi energy for different materials is with the change in base voltage.

for
same.
48. Assertion. Transistor does work in railway
ur
39. Assertion. A hole created farther below the top of carriage.
valence band has higher energy than created at the
kkss
Reason. The railway carriage does not work as an
top of valence band. electric screen.
Yo
oo

Reason. To create a hole, less energy is required 49. Assertion. The emitter and collector of transistor
to remove an electron which is farther below from
eB

can be interchanged.
the top of valence band.
Reason. The emitter and collector of a transistor
40. Assertion, n-type semiconductor is better for use are thick, of similar type and nearly equally doped.
r

than p-rype semiconductor.


ou
ad

50. Assertion. The output of two input NOR gate when


Reason. The electrons have more mobility than
YY

fed as input to a NOT gate, the combination will


holes.
work as AND gate.
ndd

41. Assertion. With the rise in temperature, the


Re

Reason. The Boolean expression for the above


resistance of semiconductor increases.
combination of gates is y = A + B = A.B ■
Fi

Reason. With rise in temperature, there will be


more frequent collisions of current carriers with 51. A.ssertion. NAND gate is called digital building
ions or atoms of the semiconductor. block.

42. Assertion. In the absence of biasing of the p-n Reason. The NAND gate can produce all the three
junction, the depletion layer offers infinite basic gates, whose combination can provide the
resistance. various complicated digital circuits.
Reason. There are no free electrons or holes in the 52. Assertion. The electrical conductivity of a
depletion layer of p-n junction. semiconductor increases on doping.
43. Assertion. There is a diffusion of majority Reason. Doping always increases the number of
electrons in the semiconductor.
charge carriers across the p-n junction in forward
biasing. (CBSE Sample Paper 2022-23)
SEMICONDUCTOR ELECTRONICS MATERIALS, DEVICES AND SIMPLE CIRCUITS 14/79

ANSWERS

I. Multiple Choice Questions

1. (a) 2. (/?) 3. (c) 4. (a) 5. (c) 6. (c) 7. (c) 8. (c) 9. (c) 10. (d)
11. (f/) 12. (b) 13. (a) 14. (b) 15. (b) 16. (b) 17. (h) 18. (/?) 19.(c) W.(d)
21. (b) 22. (b) 23. (d) 24. (b) 25. (6) 26. (/;) 27. (d) 28. (c/) 29. (J) 30. (c)

31. (b) 32. (c) 33. («) 34. {b) 35.{a) 36.(c)

II. Assertion-Reason Type Questions


37. (a) 38. (c) 39. (c) 40. (ci) 41. id) 42. (a) 43. ib) 44.(c) 45. (Z?) 46. (a)

47.(c) 48. (d) 49. id) 50. (d) 51. (a) 52. (c)

ww
HiNTS/EXPLANATiONS For Difficult Questions

FF loo
I. Multiple Choice Questions It varies symmetrically linear, having p-side

ree
negative and /j-side positive. Thus option (b) is
3. The value of temperaturecoefficient of resistance true.
of a smiconductor is negative.

reFe
23. Here p-/? junction is reverse biased, so its effective
5. Due to heavy doping in the semiconductors of tlie resistance is infinity. So current through the diode,

oroFr
rur
zener diode the depletion layer becomes thin and
the electric field within the layer is intensified.
s ff / =
6-2
= 0
11. At OK temperature, semiconductor behaves as an OO
k
YYouo
insulator, because at very low temperature electrons 24. The output .rm.s. voltage in a half wave rectifier
okso

cannot cross over the energy gap from the valence circuit is.
band to conduction band.
BBoo

13. In reverse biased p-n junction, the potential barrier ^0 _ ^0 = 200 = 100V
r ee

V
rms
2 2 2
across the junction increases. Due to it, the
thickness of the depletion layer increases, so the 25. The characteristic curve (/) is of zener diode and
ad
ouur

current in p-n junction becomes very small. curve (») is of solar cell.
Yo

14. A p-type semiconductor can be obtained by doping 26. If the base and collector of a transistor are forward
a pure semiconductor of silicon with travalent biased then transistor will not work as an amplifier.
d
idnY
Re

atoms, i.e.. Gallium. 27. Here, a = = 0-95 ;


15. With rise in temperature, the electrical resistivity
FFin

/ I
of semiconductor decreases and its electrical
|3 = ^-
[. I -I I-1 /I
conductivity increases. b e e c e

18. Because p-side of p-fi junction is more negative


0-95
than n-side of p-n junction. cc
= 19
I -a 1-0-95
20. Here p-n junction is reverse biased, hence provides
very high resistance. Due to it current in the circuit
becomes zero. 28. Here, = 24 mA.
80
I or I _ 100^ c
100 ^ 80
21. The energy gap between conduction band and
valence band is 0-07 eV, which is very small. 100
/ X 24 = 30mA
Therefore, the electrons can go very easily from 80
valence band to conduction band after getting a
little energy. Hence, the material is a conductor.
= /,, - /^ = 30 - 24 = 6 mA
22. When p-n junction is forward biased with voltage
29. Here. (3 = 60, Rq = 5 kil. /f,- = 0-5 k£2,
V. the potential across the p-n junction is (V^ - V). V,- = 0-02V, V(, = ?
14/80 7^>t6uUep. Fundamental Physics (XII) VOL.ll

38. Here Assertion is true but Reason is false as fermi


As, A =-y. xp or Vq =
V. R. R. xpxv;. energy for different materials is different.
39. Here Assertion is true but Reason is false, since
higher energy is to be spent to create a hole farther
V = —x 60x002 = 12 V
0 “ below the top of valence band.
0-5
40. Both Assertion and Reason are true and the
30. No. of electrons reaching the collector Reason is the correct explanation of Assertion.
96 41. With the rise in temperature, there will be decrease
n xlO‘0 =0-96x10^^^
^ 100 in resistance of semiconductor as temperature
coefficient of resistance of semiconductor is

Emitter current, / e = %xe negative. Therefore, with the rise in temperature,


there will be less frequent collisions of current

ww
t

carriers with the ions or atoms of semiconductor.


„ n^xe Thus both Assertion and Reason are false.
collector current, I =
42. Both Assertion and Reason are true and Reason

Flo
.'. Current transfer ratio, is the correct explanation of Assertion.

e
43. Both Assertion and Reason are true but Reason

reree
/
a = -^ _ _ 0-96x10*0 = 0-96 is not the correct explanation of Assertion.

r FF
% 10*0
44. Here Assertion is true but Reason is false,
because an ideal junction when forward biased
uurr
31. In Boolean algebra A.B = A + R.
32. The Boolean expression of the given gate is
foor
offers low resistance, hence acts as close switch
and when reverse biased, offers high resistance,
ks s
hence acts as open switch.
Yoo
y = A.B - A.B ● Thus AND gate is formed.
ooook

45. Both Assertion and Reason are true and Reason


is not the correct explanation of Assertion.
33. y = l.l+l.l =1.0 + 1.0 = 0 + 0 = 0
eBB

46. Both Assertion and Reason are true and Reason

34. The input to AND gate will be A and R. So the is the correct explanation of Assertion.
uurr

47. Here Assertion is true but Reason is false as the


output isy =A.B.
ad

change in collector current is related with the base


Yo

current and not to the change in base voltage.


35. The option (a) is not true because (1 + 1) = 1 and
dY

48. Both Assertion and Reason are false as transistor


does not work in railway carriage since railway
Re

(1 + 1 ).l = 1.1 = 1 and not 0. All other options are


innd

true. carriage works as an electric screen.


FFi

49. Both Assertion and Reason are false. We cannot


36. In computers, small currents in the base-emitter
interchange emitter and collector as their sizes are
circuits can turn on or off large currents in the
collector circuits. Due to it, these transistors act as
different and they are doped to different levels.
fast switches and not as amplirier or oscillator or 50. Both Assertion and Reason are false as Boolean
rectifier. expression for the given combination of gates will

II. Assertion-Reason lype Questions be y = A + B = A + B, /.e., an OR gate is formed.


51. Both Assertion and Reason are true and Reason
37. Diamond is electrically neutral. In ordinary
conditions, no electron can go from valence band is the correct explanation of Assertion.
to conduction band in diamond due to large energy 52. Here Assertion is true but Reason is false because
gap between valence band apd conduction band. on doping a semiconductor with an element of
Hence diamond behaves as an insulator. Therefore, valency three, no. of holes in the semiconductor
both Assertion and Reason are true and Reason will increase without the increase in number of
is the correct explanation of Assertion. electrons.
SEMICONDUCTOR ELECTRONICS MATERIALS, DEVICES AND SIMPLE CIRCUITS 14/81

P R O B L E.I\/I<S

1. Energy band of Solids electrons in energy slates of 2 s, 6 N electrons


in energy slates of 2 p. These are completely
1. The metallic conductors are opaque. Why ? filled energy states. Whereas only N out of the
Sol. This is because the free electrons of metallic 2 N available energy states of 3 s are filled and
conductor absorb all the light energy incident the remaining N states arc empty. These ideas
on metallic conductor and no amount of light arc shown in table.
can pass through metallic conductor.

oww
Energy Total states Total states
2. Out of the ionic, covalent and metallic and
van der Waal’s solids, which will be widely
level available occupied
used to produce a conductor, semiconductor 1 s IN 2N
and insulator ?
2s IN IN

e
Sol. Metallic solids are used to produce good

rFFlo
2p 6N 6N

re
conductors. Covalent solids are generally used
3s 2N I N
to produce semiconductors and the ionic and

ree
F
van der Waal’s solids are used to produce
insulators. II. «-type and p-type semiconductors

rF
3. C, Si and Ge have same lattice structure. Why
5. An //-type semiconductor has a large number
is C insulator while Si and Ge intrinsic

fsfoor
ouur
of electrons but still it is electrically neutnial.
semiconductor ?
Explain. (CBSK 2008, Hr. Board 2010)
kosk
(NCERT Solved Example) Sol. An n-type semiconductor is formed by doping
Sol. We know that C, Si and Ge are four valence pure germanium or silicon crystal with suitable
Yo
oo

atoms. Their four valence electrons are present impurity atoms of valence five. As the impurity
Y
BB

in the second, third and fourth orbit of the atoms take the positions of Ge atoms in
respective atoms. The energy gap of Ge is 0-72 germanium crystal, its four electrons form
rre

eV, of Si is 1-1 eV and of C is 5-54 eV. The covalent bonds by sharing electrons with the
energy required to take out an electron from neighbouring four atoms of germanium whereas
oYuu

these atoms (i.e., ionisation energy E^) will be the fifth electron is left free. Since the atom on
ad

least for Ge, more for Si and maximum for C. the whole is electrically neutral, the n-type
dY

Due to which, the number of free electrons for semiconductor is also neutral.
conduction will be significant in Ge and Si but 6. A p-type semiconductor has a large number
innd
Re

will be negligibly small for C. That is why C is of holes but still it is electrically neutral.
an insulator while Si and Ge arc intrinsic
Explain.
Fi
F

semiconductor.
Sol. A p-type semiconductor is formed by doping
4. Discuss the quantum states of sodium crystal pure germanium or silicon crystal with suitable
having N atoms. impurity atoms of valence three. As the impurity
Sol. For sodium, the atomic number is 11. The atoms take the positions of germanium atoms
electronic configuration of sodium is 1 2s^2p^ in germanium crystal its three electrons form
3j'. It means in a sodium atom there are two covalent bonds by sharing electrons with the
electrons in energy level Is, two electrons in neighbouring three atoms of germanium
energy level 2s, 6 electrons in energy level 2p whereas the fourth covalent bond is left
and one electron in energy level 3^, though there incomplete for want of one electron. This creates
can be two electrons in this energy level. a hole. Since the atom on the whole is electrical
A sodium atom has 11 electrons. A sodium neutral, the p-type semiconductor is also neutral.
crystal having N atoms the total number of 7. A doped semiconductor has impurity levels
electrons will be 11 M Out of these electrons, 40 meV below the conduction band, (a) Is the
2 N electrons are in energy states of \ s, 2 N material /i-type or p-type ? (b) In a thermal
14/82 4 Fundamental Physics (XII)

collision, an amount kT of energy is given to


FIGURE 14(Q).9 +5V
the extra electron loosely bound to the +10V Q
impurity ion and this electron is just able to ^ R
jump into the conduction band. Calculate the
temperature T. Given k - 8‘62 x 10"^ eV/K.
+5V R
Sol. (a) Since energy gap between impurity levels
and conduction band = 40 meV, which is much
o o
smaller than energy gap of pure semiconductor
between valence band and conduction band
(-“ 1 eV), hence the impurity levels arc of donor -10V -12 V
o—
levels and the impurity added has valence five. R I
Thus, the doped semiconductor is of «-type. 6
R ‘j.

o o

ww
(b) As per question,
kT - 40 meV
-5V h

y. _ 40meV

FF loo
or

k
R

ree
40xl0"^eV
= 464K o
8-62xI0"^eV/K -10V

rFee
8. A hole created farther below the top of
The equivalent circuits of above cases are shown

F
valence band has higher energy than created

oor r
rur
at the top of valence band. Explain. in Fig. 14(Q).10(a), (b), (c), (d) and (e)
s ff
Sol. When an electron is removed from the filled From these circuits we note that
valence band to the bottom of the conduction (a) Reverse biased
k
YYoou
band, a hole is created in the valence band. It is (b) Forward Biased
ookos

very clear that to create a hole which is farther


(c) Reverse biased
below from the top of valence band more energy
BBo

(d) Forward biased


is required to remove an electron. It shows that
re

the hole created farther below the top of valence (e) Forward biased.
band has higher energy,
ouur
ad

FIGURE 14(Q).10
Yo

III. p’li Junctions + +

R R +
10V
9. The resistance of p-n junction is low when 5V
5V T
dY

forward biased and is high when reverse


Re

B
idn

biased. Explain. (CBSE 2020)


FFin

Sol. A small increase in forward voltage across p-n o o


junction shows large increase in forward current. +
-%Wv ‘W\Ar-i
Hence the resistance (= voltage/current) of p-n 10V R R
7V
junction is low when forward biased.
A large increase in reverse voltage across p-n
junction shows small increase in reverse current.
o (D
Hence the resistance of p-n junction is high
when reverse biased.

10. In the following Fig. 14(Q).9(a), (A), (c), (d) and


(c), which of the diode.s are forward biased and 1-
10 V.
which are reverse biased and why ? O
Sol. To decide the biasing of the junction diode we
draw the equivalent circuits of various cases and 11. A p-n junction is fabricated from a
note the effective potentials at the ends A and B semiconductor with band gap of 2*8 eV. Can
of junction diode. it detect a wavelength of 600 nm ?
SEMICONDUCTOR ELECTRONICS MATERIALS, DEVICES AND SIMPLE CIRCUITS 14/83

he 6-6x10-3'^x3x108 14. A zener diode has a contact potential of 0*8 V


Sol. Energy, £ = — eV
in the absence of biasing. It undergoes zener
600x10-9 x1-6x10-19
= 2-06 eV < 2-8 eV
breakdownfor an electric field of 10^’ Vm“*
at the depletion region ot'p-n junction. If the
As £ < £^, so p-n junction cannot detect the width of the depletion region is 2-4 ^ m, what
radiation of given wavelength.
should be the reverse biased potential for the
12. What are the readings of ammeters Aj and zener breakdown to occur ?
^2 shown in Fig. 14(Q).ll ? Neglect the Sol. Here, ihe breakdown eleciric field of ihe zener
resistances of the ammeters, when the p-n diode, £= 10^ Vm"'.
junction used is ideal one.
The width of the depletion region,
FIGURE 14(Q).11 d=2-4x 10-6 m

5n .-. VBreukilown = Exd- I06x2-4x 1Q-6


-^1

ww
= 2-4V

15. How reverse current suddenly increases at


2V
I the breakdown voltage in case of zener
diode ? {CBSE 2008 u I)

Flo
Sol. We know that the reverse current through the

e
●VvA^’V junction diode is due to flow of minority carriers

rere
{i.e., flow of electronsfrom/j to /t-side and holes

r FF
Sol. Here the p-n junction is reversed biased with from n to /?-side of p-n junction diode). As the
battery of emf 2 V. Hence resistance of p-n reverse bias voltage across the junction is
increased, the electric field at the junction
uurr
junction becomes infinite. No current flows
through ammeter Aj. So the reading of ammeter
Aj is zero.
foor
becomes significant. When the reverse bias
voltage becomes equal to Zener voltage
kss
(i.e., V = Vj), then the electric field strength
The current through ammeter A2
Yoo
across the junction becomes quite high. This
ooook

= - = 0*4A
electric field across the junction is sufficient to
pull valence electrons from the host atoms on
eBB

5
13. Why are Si and GaAs are preferred materials the p-side and accelerate them towards n-side.
The movement of these electrons across the
for solar cells ? (NCERT Solved Example)
uurr

junction accounts for high current which is


Sol. The energy for the maximum intensity of the
ad

observed at the break down reverse voltage.


solar radiation is nearly 1 -5 eV. In order to have
Yo

16. What do the acronyms LASER and LED


photo excitation the energy of radiation (hv) stand for ? Name the factor which determines
dY

must be greater than energy band gap (£^). (i) frequency and (H) intensitv of light emitted
Therefore, the semiconductor with energy band
Re

fCBSE 2005 (01


innd

by LED.
gap equals to or less than 1-5 eV or lower and
with higher absorption coefficient, is likely to Sol. LASER stands for light amplification by
Fi

stimulated emission of Radiation. LED stands


give better solar conversion efficiency. The
energy band gap for Si is about M eV, while for light emitting diode.
for GaAs, it is about 1-53 eV. The GaAs is better (0 The frequency of light emitted by an LED is
inspite of its higher band gap than Si because it related to the band gap of the semiconductor
absorbs relatively more energy from the incident used in LED, i.e., a type of material used in
making the LED.
solar radiations being of relatively higher
absorption coefficient. We do not use materials (//) The intensity of light emitted by LED
having band gap more than 1 -5 eV because for depends upon the doping level of the
semiconductor used.
such materials most of the solar radiations of
energy more than 1 -5 eV are absorbed on the 17. In V-/ characteristics of a p-n junction diode,
top layer of the solar cell and do not reach in or why is the current under reverse bias almost
near the depletion region. Due to it, only a small independent of the applied potential iipto a
portion of higher solar energy is used for critical voltage ? (CnSi. 2013)

photoconversion whereas the significant part of Sol. When a p-n junction is reverse biased, a very
the solar energy is of no use. small current (of the order of few |oA) flows due

t
14/84 ‘Pn4t<Ue^'A Fundamental Physics (XII)GZsI9Il

to drifting of minority charge carriers whose 22. In only one of the circuits given below, the
number density remains constant upto the lamp L lights. Which circuit is it ? Give reason
for your answer. [CBSE 2001 (C)l
critical voltage. As a result of it, the current
under a reverse bias is almost independent of FIGURE 14(Q).12
the applied potential upto critical voltage.
IV. Transistors L
R. R
18. In a transistor, base is made thin and doped 6V 6 V

with little impurity atoms. Why ?


(CBSE 2002, CBSE Sample Paper 2013)
e o
Sol. The base region in a transistor is made very thin
so that there is a better conduction of majority

ww
cturiers from emitter to collector through base. Sol. The lamp L lights up in the circuit shown in
Due to it, the base current is quite weak, the Fig. 14(Q). 12{a) because in this circuit, the input
collector current is nearly equal to the emitter circuit (emitter-base circuit) is forward biased
current. As a result of it, the transistor can give and output circuit (collector-emitter circuit) is

Flo
good power gain and voltage gain. reverse biased.

e
The base region in a transistor is doped lightly The lamp L does not light up in the circuit shown

eree
so that the number density of majority carriers in Fig. 14(Q).12{/j) because in this case, the
(electrons inp-n-p and holes in n-p-n transistors) input circuit (emitter-base circuit) is reverse

FFr
is low. When emitter is forward biased, the biased and output circuit (collector-emitter
majority carriers move from emitter to collector

oorr
uur r
circuit) is forward biased.
through base. Since base is thin and lightly 23. If the base region of a transistor is made
sf
doped, only a small (about 5%) electron-hole large, as compared to a usual transistor, how
combination will take place giving weak base does it affect (i) the collector current and
sk
Yoo
current and remaining majority carriers will be
(«) current gain of this transistor.
oook

collected by collector giving collector current (CBSE 2000)


nearly equal to emitter current.
eBB

Sol. If the base region of a transistor is made large,


19. What will happen if both, emitter and
collector of a transistor are reversed biased ? then most of the majority charge carriers coming
from emitter would get neutralised in the base
uurr

Sol. In this situation, no current will flow through


by the electron-hole combination. As a result
ad

transistor because there is no conduction due to


Yo

of it, only few majority carriers will be reaching


majority carriers across theemitter-ba.se junction
the collector. Due to it (0 the collector current
or collector-base junction.
dY

will reduce and hence (//) the current gain will


20. What will happen if emitter as well as
Re

reduce.
collector in a transistor are forward biased ?
innd

24. In a radio receiver, the short-wave and


Sol. In this situation, the majority charge carriers will
FFi

medium-wave stations are tuned by using


flow in the emitter-base circuit and also in the
collector-base circuit. In this case, the working
the same capacitor but coils of different
inductance. In which case the value of
of transistor will be equivalent to two p-n
inductance is more ? Explain.
junction diodes with a common base. It means
Sol. The value of inductance is more while tuning a
the puipose of transistor will be defeated. radio receiver at medium wave station than that
21. A trans'stor is current operated device. at short wave station. We know that the short
Explain. (Hr. Board 2011) wave station emit radiowaves of short
Sol. In a transistor, emitter current = base current
wavelength (k^) and medium wave stations emit
+ collector current. It means emitter current radiowaves of longer wavelength (A.,„), i.e.,
controls the collector current and base current.
For a given emitter current, collector current is Let v^, be the frequencies of short wave and
controlled by base current. Therefore, the medium wave signals.
change in collector current is related with the
Then v^ = c/X^ and v,„ = c/?i m
base current and not to the base voltage change.
That is why, the transistor is a current operating or
m
...(i)
device. V k
m

1
SEMICONDUCTOR ELECTRONICS MATERIALS, DEVICES AND SIMPLE CIRCUITS 14/85

If Ly be the self inductance of the short wave 28. Discuss how the AND gate is realised from
and medium wave coils respectively and C be the NAND gate.
the capacitance of capacitor used, then Sol. If the output y^of NAND gate is connected to
1 I the input of NOT gate (made from NAND gate
V and Vm
by joining two inputs), as shown in Fig.
2k^ 14(Q).l5(fl), then we get back an AND gate.
●>
Lm
V
s ^7 ^ m
FIGURE 14(Q),15
or
...(ii)
V L v2 A

>-i
m s m
I II
From (/) and (//), ^ O
B
Lm Xm 2
->] (AsX^>X^) >L, A B Y' Y

ww
L X -
s
s 0 0 1 0

25. How will you test in a simple way whether a


O 10 1 0
transistor is spoiled or in working order ? 0 1 1

FF loo
0
(Pb. Board 2002) 1 1 0 1
Sol. A transistor has two junctions; one junction

ree
between emitter and base and other between The output of gate / is, v' = A.B
base and collector. A junction of transistor has
low resistance when forward biased and high The output of gate II is,

reFe
resistance when reverse biased. In a spoiled

oroFr
rur
transistor, the resistance of the junction is found y = /./ = {A.B ).A.B = {A.B ) + {A.B )
to be low when forward biased or reverse
s ff = {A.B) + {A.B) = {A.B)
biased, which can be checked using AVO-meter. which is the Boolean expression for AND gate.
k
YYouo
V. Gates Thus the combination works as AND gate. The
okso

truth table of the combinations is shown in Fig.


26. Why are the NAND (or NOR) gates called as 14(Q).I5(^), which is the truth table of AND gate.
BBoo

digital building block ? 29. Discuss how the OR gate Is realised from the
r ee

Sol. The repeated use of NAND (or NOR) gales can NAND gate. (CBSE 2011)
produce all the three basic gates i.e. OR, AND Sol. If the input terminals of NAND gate are joined
ad
ouur

and NOT gates ; whose various combinations together we get NOT gate. Now if the inputs A
Yo

provide us large number of complicated digital and B are inverted by using two NOT gates
circuits. Hence in digital circuits, NAND (or (obtained from NAND gates) and their outputs
NOR) gates are called a digital building block.
d

are jointly fed to the NAND gate as shown in


Re
idnY

27. Discuss how a NOT gate is realised using Fig. 14(Q). 16(«). we get the arrangement which
NAND gate. works as OR gate.
FFin

Sol. If the two inpuLs of the NAND gate are joined to FIGURE 14(Q).16
make one input as shown in Fig. i4(Q).13. then

>
the NAND gale functions as a NOT gate. As, A o- I
A

y—●
y = A.A =A + A =A
o III
In the truth table of NAND gate, if we use y

= /4, we obtain the truth table a« ‘^Iven in Fig.


14(Q).I4, which is the tmih fable ol NOT gate.
BO- II
> B

FIGURE 14(Q).13 FIGURE 14(Q).14 A B A B Y = A.B

A A B=A Y 0 0 11 0
o-
—oy 0 0 1 O 0 110 1
B
1 1 0 10 0 1 1

110 0 1

I
14/86 ‘Pnadetfr ^ Fundamental Physics (Xll)iagiaiJ
Explanation. The output of gate I is,
FIGURE 14(Q).18
= A.A =A+A = A
>'l
The output of gate II is,
= B.B

The output of gate III is,

= A .B = A + B — A + B

As y = A + ^, is the Boolean expression for OR


0 0 1 1 0
gate, thus the combination works as OR gate.
©10 0 1 0

ww
The truth table of the arrangement is shown in 0 1 1 0 0
Fig. 14(Q).16(/;). 1 1 0 0 1

30. Discuss how the OR gate is realised from the

FF loo
NOR gate.
The output of gate I is.
Sol. If the output y'of NOR gale is used as the input

ree
of NOT gate (made from NOR gale by joining v,=A
two inputs), as shown in Fig. 14(Q). 17(a). we

rFee
get back an OR gate. The output of gate II is.

F
oor r
rur
FIGURE 14(Q).17 s ff >’2
The output of gate III is
Ao- y
O ■oy
k
I
y = Yl + y., = A+B — A.B - A.B
YYoou
Bo-
ookos

which is the Boolean expression for AND gate.


BBo

A B y' y
The truth table of the arrangement will be as
0
re

1
^ 0 0 shown in Fig. 14(Q).18(/?), which is similar to
O 1 0 0 1
truth table of AND gate (by reading the
0 1 0 1
ouur
ad

1 1 0 1 columns, A, B and y only).


Yo

32. Identify the logic gates marked P and Q in


the circuit Fig. 14(Q).19. Write the truth table
The output of 1 is, y = A + B - A.B for this combination. (CBSE 2010, 2014)
dY
Re
idn

(From De Morgan’s theorem)


FIGURE 14(Q).19
FFin

The output of II is,


A X'
P
y = y' = A.B = A+ B = A + B B Q X

which is the Boolean expression for OR gate.


The truth table of the combination will be as
shown in Fig. 14(Q).17(/?), which is the truth Sol. Logic gate P is NAND gate and Q is OR gate.
table of OR gate. The Boolean expression for NAND gate is
31. Discuss how the AND gate is realised from
X' = A^ = A + B
the NOR gate.
Sol. If the input terminals of NOR gate are joined The Boolean expression for the combination of
together we get NOT gate. Now if the inputs A gates is
and B are inverted by using two NOT gates X = X'+B = {A + B) + B
(obtained from NOR gates) and their outputs
are jointly fed to the NOR gate as shown in Fig. = A + (B + B) = A + \
14(Q).18(a), we get the arrangement which [●.● B + B = 1]
works as AND gate.

1
SEMICONDUCTOR ELECTRONICS MATERIALS, DEVICES AND SIMPLE CIRCUITS 14/87

The truth table for the combination of gates is Sol. In the, given circuit we have used OR gate and
shown below.
AND gate. Let / be the output of the OR gate.
A B Therefore A + B —y'. The AND gate receives A
A B A +B X -(A+ B) + B
and y'as input. The output of this gate is y.
0 0 1 1 Therefore, y = A.y'= A.(A + B). The following
0 1 0 table («) shows all the combinations of A and B
1 0 0 1 1 related to v.
1
1 1 0 0 0 1 A B y' = A +B y = A.fA + B)
It means output is always 1 whatever may be 0 0 0 0
the values of inputs A and B.
0 1 1 0
33. Sketch the output y from a NOR gate having 1

ww
0 1
inputs A and B as given in Fig. 14{Q).20(a) 1 I 1
and {b).

{a)
FIGURE 14(Q),20

Flo
{h) A B

e
/1 t2 U ^5 ^6

eree
0 0 0
0 1 0

FFr
A--
(a) 1 0
uurr 1

orr
B -- -~{b)
Thus the table {b) is the truth table of the given

●(c)
sfo
electronic circuit.

35. The output of an OR gate is connected to both


kks
Yoo
I

the inputs of a NOR gate. Draw the logic


oooo

circuit of this combination and write the truth


eBB

table.
Sol. For NOR gate the Boolean expression is
Sol. The circuit is shown in Fig. 14(Q).22.
A + S = V
urr

For the given wave form, we have the following FIGURE 14(Q).22
ad

values for A. B and y.


YYo

For time t < rj, A= 1.j9= I hence y = 0


For lime r, to i2. A = 0, fi = 0 hence y = 1 y
dd

For lime ^2 to ^3, A = 0, S = I hence y = 0


Re
inn

For time to A = l,fi = 0 hence y = 0 Here, y'= (A + B)


For time to A= l.e= 1 hence y = 0
F

and
For time ^5 to A = 0, 5 = 0 hence y = 1 y = (A + 5) + (A+5)
For time t > A = 0. 5 = 1 hence y = 0
The wave form for output y is as shown in Fig. = (A + 5).(A+S)
14(Q).20(c)
34. Construct the truth table for the function y = (A + 5)
of A and B represented in Fig. 14(Q).21. Truth table of the given logic circuit is given
(CBSE 2014) below, which is for NOR gate.
FIGURE 14(Q).21 A B r Y

0 0 0 1
A 0 1 1 0
y 1 0 1 0
B 1 1 1 0
14/88 Fundamental Physics (XII)RRHID

● Very Short Answer

\
● Short Answer
● Long Answer

VERY SHORT ANSWER QUESTIONS Cart ying 1 mark

Ans. EG-^> EG-,>


I. Energy band of Solids 10. Name the charge carriers in the following at

w
1. According to energy band diagram, what room temperature : (0 Conductor («) Intrin
makes a substance (a) conductor sic semiconductor iiii) Insulator.
{b) insulator ? Ans. (0 Electrons (//) Electrons and holes (///) None.
Ans. (a) Overlapping of conduction band and valence 11. How does the forbidden energy gap of an

e
band {b) A large energy gap (more than 3 eV) intrinsic semiconductor vary with the

roow
between conduction band and valence band.

re
increase in temperature ?
2. What is fermi level and fermi energy ? Ans. The energy gap of an intrinsic semiconductor
Ans. In an energy band, the highest energy level does not change with the increase in

reF
uFFll
occupied by electron at is called fermi level temperature.
and its energy is called fermi-energy.

e
3. Where does the fermi-level of intrinsic II. n-type and p-type semiconductors

sFr
semiconductor lie ? 12. Why is doping of intrinsic semiconductors
Ans. The fermi-level of intrinsic semiconductor lies

oro
necessary ? (CBSE 2022)

k
mid way between its valence and conduction Ans. An intrinsic semiconductor is a pure
uor
gap of intrinsic semi-conductor.
off
bands i.e., in the middle of forbidden energy semiconductor which is free from impurities and
is of limited conductivity. In order to increase
kos
4. What is doping ? the conductivity of intrinsic semiconductor,
Y
Yo
Ans. Doping is a process of delibrate addition of a some desirable impurities are doped in pure
reeBB
oo

desirable impurity in a pure semiconductor to semiconductor. That is why doping of intrinsic


modify its properties in a controlled manner. semiconductor necessary.
uurY

5. Why doping is done in semiconductor ? 13. How does the energy gap of an intrinsic
Ans. The doping is done in semiconductor to increase semiconductor vary, when doped with a
the number of mobile electrons/holes and hence pentavalent impurity ?
ad

to increase the conductivity of semiconductor. Ans. When an intrinsic semiconductoris doped with
doo
nY

6. The forbidden energy gap of germanium is the impurity atoms of valence five like As, P or
0-72 eV. What do you understand by it ? Sb, some additional energy levels are produced,
situated in the energy gap slightly below the
nid

Ans. It states that if an energy of 0-72 eV is given to


Re

conduction band which are called donor energy


an electron in the valence band of germanium
F

levels. Due to it. energy gap in semiconductor


Fi

it will jump to the conduction band, crossing


decreases.
an energy gap of 0-72 eV.
7. Why diamond behaves like an insulator ? 14. A specimen of silicon is to be made P-type
semiconductor. For this one atom of indium,
Ans. In the energy band diagram of diamond, there
is a large energy gap = 5-54 eV. Due to it. no
on an average, is doped in 5 x 10^ silicon
atoms. If the number density of silicon is
electron can go from valence band to conduction 5 X 10^ atoms/m^, then find the number of
band, in ordinary conditions.
acceptor atoms per cm^.
8. Why do Ge and Si are semiconductors ?
Ans. No density of silicon = 5 x 10^* atoms/m^
Ans. In the energy band diagram of Ge and Si. the = 5 X 10^^ atoms/cm'-*.
energy gap is 0.72 eV and 1.1 eV respectively
between conduction band and valence band. As No. of acceptor atom/cm^ = 5 x 10^^/(5 x 10^)
= lO^S/cm^
a result of it, they behave as semiconductor.
9. The forbidden energy band gap in conduc 15. A semiconductor has equal electron and hole
tors, semiconductors and insulators are £G|, concentration of 6 x lO"* m“^. On doping with
EGi and EG^ respectively. How are they a certain impurity, electron concentration
related ?
SEMICONDUCTOR ELECTRONICS MATERIALS, DEVICES AND SIMPLE CIRCUITS 14/89

increases to8 x 10*- Identify the type of


semiconductor. IV. p-n junction.s
(CBSE 2004)
Ans. Since in doped semiconductor the electron con- 23. Can a slab of p-type semiconductor be
cenlration is more than the hole concentration physically joined to another «-type
(i.e.. n^, > /ly,) so resulting semiconductor is semiconductor slab to form p-n Junction ?
n-type semiconductor. Justify your answer. (CBSE 2020)
16. Give the ratio of number of holes and the Ans. Any slab, what so ever flat it may be, will have
number of conduction electrons In a (i) pure roughness which is much larger than the
semiconductor («) «-type semiconductor and interatomic crystal spacing. As a result of it, a
(Hi) p-type semiconductor. continuous contact of the atomic junction will
Ans. (/) n,/n^ = I (//) n,/n^ < 1 (iii) ii/Jitg > 1 not be possible. The junction will behave as
17. What is the location of donor energy levels discontinuous for the flowing charge carriers.
in the energy band diagram of «-type 24. What happens when a forward bias is applied

w
semiconductor ? to ap-/i junction ? (CBSE 2015)

Ans. In the energy band diagram of n-type Ans. When a forward bias is applied to a p-n junction,
semiconductor, the donor energy levels lies in the size of the depletion layer decreases. The

Flo
the energy gap between valence band and resistance of the junction becomes low. The
conduction band. The lowest donor energy level movement of the majority carriers takes place

ee
lies at 0-01 eV for Ge and 0-045 eV for Si below across the junction, resulting current, known as

Fr
the bottom of conduction band. forward current which increases rapidly with
18. What is the location of acceptor energy levels increase in forward voltage.
in the energy band diagram of p-type 25. Can we measure the potential difference of a

for
ur
semiconductor ? p-n junction by putting a sensitive voltmeter
across its terminals ?
Ans. In the energy band diagram of p-iype silicon
semiconductor, the acceptor energy levels lie in Ans. No, because the voltmeter to be used to measure
ks
energy gap slightly above the valence band. The potential difference across the p-n junction must
Yo
have a very high resistance as compared to
oo

highest acceptor energy levels lies at 0-045 eV


above the lop of valence band. junction resistance, which is nearly infinite, if
eB

not biased. Apart from it, there are no free charge


III. Conductivity of semiconductors carriers in the depletion region of p-n junction.
26. In the depletion region of an unbiased p-n
ur

19. What factors determine the electrical


junction diode, what are the charge carriers ?
ad

conductivity of metal ?
Yo

Ans. In the depletion region of an unbiased p-n


Ans. The electrical conductivity of a metal depends junction diode, there are only fixed ions but no
upon the number of free electrons and their drift
free charge carriers.
velocity through the metal on applying the field.
nd

27. What is the direction of diffusion current in


Re

20. What are the safe limits of temperature for a junction diode ?
germanium and silicon ?
Fi

Ans. 80°C to 200°C. Ans. In junction diode, the direction of diffusion


21. Mention some properties of semiconductors. current is from P-region to A/-region.
28. If the forward bias on p-n junction is
Ans. A semiconductor has (a) covalent bonding increased from zero to 0 05 V, then no current
(b) crystalline nature (c) negative temperature in the circuit flows. What is the contact
coefficient. The conductivity of a semiconductor potential of junction diode ?
increases with the addition of impurities of Ans. 0-05 V.
material of valence three or five.
22. Why is the conductivity of //-type semi 29. Why are photodiodes used preferably in
reverse bias condition ? (CBSE 2019)
conductor greater than that of the p-type
semiconductor even when both of these have Ans. In reverse bias condition of photodiode, the
the same level of doping ? [CBSE 2005 (C)] change in saturation reverse current is directly
Ans. It is because, under a given electric field, the proportional to the change in the incident light
flux or light intensity, which can be measured
mobility of electron is higher than that of hole.
accurately. It is not so when photodiode is
forward biased.
14/90 Fundamental Physics (XII)Eaiadl

30. Explain briefly how a photodiode operates. 37. An ideal junction diode acts as a switch.
[CBSE 2018 (C)] Explain.
Ans. Photodiode is a p-n junction diode which is an Ans. An ideal junction diode acts as a closed switch
when forward biased and open switch when
optoelectronic device in which current carriers reverse biased. It means an ideal junction diode
are generated by photons through photon acts like an automatic switch.
excitation, i.e., photoconduction by light. The
operation of a photodiode depends on the inner 38. Name the junction diode whose I - V
characteristics are drawn below :
photoelectric effect.
(CBSE 2017)
31. Three photodiodes Dj, and £>3 are made
of semiconductors having band gaps of 2*5
eV, 2 eV and 3 eV respectively. Which of them FIGURE 14(Q).23
will not be able to detect light of wavelength U
600 nm ? (CBSE 2019)

ww
Ans. Energy of light,
he 1242 nm eV
£ = — = 2-07 eV. ■►V

Floo
X 600 nm

As this value of energy (= 2-07 eV) is less than

ree
band gaps of 2-5 eV and 3 eV, so photodiodes
D, and £>3 will not be able to detect the light of

rFee
given wavelength.

F
32. Differentiate between the threshold voltage Ans. Solar cell.

oor r
rur
and the break down voltage for a diode. 39. Write the property of a junction diode which
s ff
(CBSE 2022) makes it suitable for rectification of a.c.
Ans. In forward biasing of p-n junction, the voltage voltage. (CBSE 2022)
k
at which the current starts to increase rapidly is Ans. p-n junction is used for rectification of a.c.
YYoou
ookos

known as threshold voltage or knee voltage. voltage because p-n junction conducts in
In reverse biasing of a p-n junction, the voltage forward biasing and does not conduct in
BBo

at which p-n junction of a diode breaks down reverse biasing. It means, p-n junction diode
re

with sudden rise in current is known as break allows the current through it in one direction
down voltage. when worked with a.c. voltage, hence acts as
ouur
ad

33. What is zener breakdown ? a rectifier.


Yo

Ans. When a very large reverse voltage is applied 40. State the level of doping and biasing condition
across a p-n junction, so that there is a sudden used in light emitting diode (LED).
large reverse current flowing through p-n
dY

Ans. LED is a photoelectronic device which converts


Re

junction. Then the effect is called zener electrical energy into light energy under forward
idn

breakdown.
FFin

bias condition.
34. Zener diode is used in reverse bias. When its
LED is a heavily doped p-n junction diode
reverse bias is increased, how does the which works under forward biasing and emits
thickness of the depletion layer change ?
spontaneous radiation.
(CBSE Sample Paper 2020)
41. Write any two advantages of LED over the
Ans. Increases.
conventional low power lamps.
35. From the information of energy band gaps
of diodes, how do you decide which can be
Ans. The advantages of LED over low power lamp
are as follows ;
light emitting diodes ?
(CBSE Sample Paper 20201 1. LED has less power and low operation
voltage.
Ans. Diodes with band gap energy in the visible
spectrum range can function as LED. 2. LED has fast action and requires no warm up
time.
36. What is an ideal junction diode ?
Ans. An ideal junction diode is one which acts as a 3. LED is cheap and easy to handle.
perfect conductor when forward biased and
perfect insulator when reverse biased.
SEMICONDUCTOR ELECTRONICS MATERIALS, DEVICES AND SIMPLE CIRCUITS 14/91

V. Transistors From (0, L = hL + i or 1 = -I + l


‘c a p
42. Why is the transistor called a junction
‘ 1-1= 1-a
a
transistor ? (HP Board 2004) or or P =
Ans. A junction transistor is so called because it is a P a a 1-a

device which can transfer the resistance by 50. State the relation for the voltage gain in terms
interchanging the biasing across the junctions. of trans-conductance, using transistor as an
43. In a transistor, emitter is always forward amplifier.
biased. Why ? Ans. Voltage gain = - trans-conductance x output
Ans. The emitter is always forward biased to enable resistance.
the majority carriers to cross the emitter-base 51. What type of feed back is required in an
junction, so that current flows through the oscillator ?
transistor.

ww
Ans. Positive feed back.
44. Is transistor a current controlled or
52. What is the condition for the state of
temperature controlled device ? saturation of a transistor ?
Ans. Transistor is a current controlled device.
Ans. The transistor is in saturation state if

Flo
45. What is the phase relationship in the output > /^. In this condition, during the positive

e
and input voltages in the common base half cycle of a.c. signal, the current increa.ses

e
transistor amplifier. resulting an increase in /^, which is so large that

rere
rFF
Ans. Output voltage is in phase with the input signal V^E becomes less than Vqe. Due to it, the
voltage. transistor goes into saturation state and no
uur r
46. What is the phase relationship betwen longer works as an amplifier.
collector and base voltages in common-
emitter configuration. state ?
foor
53. How is a transistor biased to be in active
(CBSE 2014)
sks
Ans. The output collector voltage is 180° out of phase Ans. Transistor is said to be in active state when its
Yoo
with respect to input signal voltage applied to emitter-base junction is suitably forward biased
ooko

base.
and base-collector junction is suitably reverse
eBB

47. Define transconductance of a transistor. On biased.

what factors does it depend ? 54. Under what condition a transistor works as
Ans. It depends on the geometry, doping levels and an open switch ?
uurr

biasing of the transistor. It is defined as the ratio Ans. A transistor will work as an open switch in the
ad

of the change in collector current to the change cut off state i.e., when both the emitter and
Yo

in base-emitter voltage. collector are reversed biased.

48. Can two p-n junction diodes placed back


dY

55. In a transistor, the area of base-collector


to back work as a p-n-p transistor ? junction is made larger than the area of the
Re
innd

(IM). Board 2003) emitter-base junction. Why ?


Ans. During the working of a transistor, most of the
FFi

Ans. No, by joining two p-n junction diodes back


to back, the n-region which will form the base heat is produced at the base collector junction.
The area of this junction is made large so that
will be too thick and will not serve the purpo.se
of base in a transistor. the heat may be dissipated to surroundings.
56. Three amplifier circuits are connected in
49. Derive a relation between current gain of series. The voltage gain of each is 5. What is
common base amplifier and current gain the final voltage amplification ?
of common emitter amplifier. Ans. Final voltage amplification = 5x5x5 = 125.
Ans. For a junction transistor, we know 57. In a transistor, doping level in base is increa
= + Ic ...(0 sed slightly. How' will it affect (0 collector
Current gain for common base amplifier, current and («) base current ? (CBSE 2011)
Ans. (0 Collector current will decrease, as more of
Current gain for common emitter amplifier. the majority carriers going from emitter to
collector will get neutralised in base by electron
hole combination. Due to it (i7) the base current
will increase.
14/92 ^●taxUeft. ^ Fundamental Physics (XII)
58. How does the voltage gain vary with the
FIGURE 14(Q).25
frequency of the input signals for a transistor
amplifier ?
Ans. For a transistor amplifier, the voltage gain is low ■●y
at low and high frequencies and constant high A

at mid frequencies.
66. Draw the logic symbol of the gate given
VI. Gates below :

59. State the rule used in the operation of OR FIGURE 14(Q).26


gate.
A
Ans. The output of an OR gate assumes 1 in (level)
B
if one or more inputs assume 1 (in level).

ww
60. State the rule used in the operation of AND
gate. Ans. The Boolean expression for the given gate is
Ans. The output of an AND gate assumes 1 (in level)

Flo
if all the inputs assume 1 (in level). y=(A + B) = A + B
61. State the rule used in the operation of NOT It means OR gate is formed.

e
Logic symbol of OR gate is shown in Fig.

rere
gate.
Ans. The output of a NOT gate is 1 (in level) if input 14(Q).27.

r FF
is 0 (in level) and vice versa.
FIGURE 14(Q).27
62. What is the difference between analogue and
uurr
digital circuits ?
Ans. Analogue circuit deals with any type of
for A

B
kss
electronic pulses whereas digital circuit deals
ooook

with square type voltage variation. 67. What is integrated circuit ?


Yo

63. Name the logic gate realised using p-n Ans. Integrated circuit is that circuit in which the
eBB

junction diodes in the given Fig. 14(Q).24. circuit components such as resistors, capacitors,
Give its logic symbol. diodes and transistors, etc., are automatically
parts of a small semiconductor chip.
urr

FIGURE 14(Q).24 68. Write the full forms of the terms (/) MSI and
ad

(h) LSI used for different types of integrated


Yo

A
y circuits. [CBSE 2004 (C)l
dY

B
Ans. (0 MSI stands for Medium Scale Integration
Re

circuits.
innd

fR
(ii) LSI stands for Large Scale Integration circuits.
Fi

69. Write the full form of the terms (i) SSI and
(ii) VLSI used for different types of
integrated circuits. [CBSE 2004 (C)]
Ans. (0 SSI stands for Small Scale Integration circuits.
Ans. OR gate ;y =A + B. Its logic symbol is shown
(I'O VLSI stands for Very Large Scale Integration
in Fig. 14.49.
circuits.
64. What is the output y for a gate, when
70. Name the category of the integrated circuit
y = A^'i which is utilizing the circuit components
(0 < 100 and (ii) ^ 1000.
Ans. y = A B = A B - represents AND gate. Ans. (/) The integrated circuit utilizing the circuit
65. How 2-input NAND gate can be converted components < 100 is called Medium scale
into a NOT gate ? integration circuit (MSI).
Ans. When two inputs of NAND gate are joined, so (ii) The integrated circuit utilizing the circuit
that it has one input, it works as a NOT gate. components < 1000 is called Large scale
Symbolically it is shown in Fig. 14(Q).25. integration circuit (LSI).
SEMICONDUCTOR ELECTRONICS MATERIALS, DEVICES AND SIMPLE CIRCUITS 14/93

SHORT ANSWER QUESTIONS Carrying 2 marks

I. Energy band of Solids 5. In a pure semiconductor crystal of Si, if


antimony is added then what type of extrinsic
1. Distinguish between electrons and holes. semiconductor is obtained. Draw the energy’
Ans. (/) Electron is a negatively charged particle band diagram of this extrinsic semiconductor
having charge = 1-6 x I(T*^ C. Hole is a seat so formed. (CBSE Sample Paper 2022-23)
having positive charge which is produced when Ans. When antimony is added to a pure silicon
an electron breaks away from a covalent bond semiconductor, then /i-type extrinsic
in a semiconductor. Hole is having the same semiconductor will be obtained, since antimony
charge as that of electron, {ii) Energy of a hole (Sb) has valence five. For energy band diagram
is high as compared to that of electron.(m) The refer to Fig. 14.9 (h).
mobility of a hole is smaller than that of electron. 6. Distinguish between intrinsic and extrinsic

ww
2. Why is the semiconductor damaged by a semi- conductors on the ba.sis of energy band
strong current ? diagram.
(Pb. Board 2004, HP Board 2010) (CBSE (D), 2015, Jharkhand Board 2011)

Flo
Ans. When a strong current is passed through a Ans. The energy band diagrams of intrinsic
semiconductor,it heats up the semiconductor. semiconductor and extrinsic semiconductor

ee
Due to it, the large number of covalent bonds have been shown in Fig. 14.6(/>) and \4.9{b) or

rere
break up in semiconductor, resulting large 14.10(i?).

r FF
number of charge carriers. As a result of it, the From these energy band diagrams we note that
material starts behaving as a conductor. At this (/) The energy gap is of I ● 1 eV for intrinsic
uurr
stage, the semiconductor losses the property of
low conduction, hence it is said to be damaged.
3. Define a hole. State its characteristics.
foor
semiconductor and less than 1-1 eV (due to
donor energy state or acceptor energy state) for
extrinsic semiconductor. (//) The conductivity
ks s
Ans. Hole is a seat of positive charge which is of extrinsic semiconductor is more than intrinsic
Yoo
oook

produced when an electron breaks away from semiconductor because of smaller energy gap
a covalent bond in a semiconductor. for extrinsic semiconductor. (///) The resistivity
eBB

of intrinsic semiconductor is more than extrinsic


Characteristics of a hole
semiconductor as resistivity is reciprocal of
1. Hole carries a unit positive charge. conductivity.
uurr

2. It has the same magnitude of charge as that 7. The energy gap of silicon is 1*14 eV. Find the
ad

of electron.
Yo

maximum wavelength at which silicon starts


3. Energy of a hole is high as compared to energy absorption.
dY

that of electron. Ans. Here, £ = \-\A eV = 1-14 x 1-6 x 10"*^ J.


Re

4. The mobility of hole is smaller than that of


innd

electron. he 6-62xl0-^'*x3xl0^
X = — = 10,888A".
FFi

5. In external electric field holes move in a £ l-6xlO''^xM4


direction opposite to that of electron. 8. The ratio of number density of free electrons
to holes for two dilTerent materials, A and
n. «-type and p-type semiconductors are (/) equal to one and (ii) less than one
respectively. Name the type of semiconductor
4. Which semiconductor has more mobility : to which A and B belong. Draw energy level
p-type or w-type ? Explain. diagram for A and B.
Ans. The mobility of electrons in n-type Ans. (/) The material A is a pure semiconductor and
semiconductor is more than the mobility of (//) the material B i,s p-type semiconductor. For
holes in p-type semiconductor. Since the n-type their energy level diagrams, refer to Figs. \4.6(b)
semiconductor has electrons as majority carriers and 14.10{^).
and holes as minority carriers, whereas the 9. Distinguish between intrinsic semiconductor
p-iype semiconductor has holes as majority and p-type semiconductor. Give rea.son, why
carriers and electrons as minority carriers, a p-type semiconductor crystal is electrically
therefore mobility of «-type is more than that
neutral, although tif, » ?
of p-type.
14/94 “P^cicCeep. Fundamental Physics (XII)
Ans. In a pure semiconductors (Ge or Si) called Ans. («) An intrinsic semiconductor is one which is
intrinsic semiconductor the electrically extremely pure with a purity level of 99-9999%.
conductivity is related by the electrons thermally It is very difficult to achieve this much purity in
excited from the valence band to the conduction a semiconductor.
band. There are equal number densities of free (b) Electrical conductivity of an intrinsic
electrons and holes in conduction band and semiconductor at a given temperature depends
valence band of intrinsic semiconductor. on (0 the number densities of intrinsic charge
When a pure semiconductor of Ge or Si is doped carriers present in it and («') the width of the
with impurity atoms of valence three (like B or forbidden energy gap between conduction band
Al), some additional energy levels are created and valence band,

just above the upper energy level of valence (c) The conductivity of a semiconductor
band. Due to it, band gap of p-iype increases exponentially with temperature.
semiconductor becomes smaller than intrinsic
12. How does the conductivity and resistivity

w
semiconductor. Also in p-type semiconductor, change with rise of temperature in case of
the number density of holes is more than that of intrinsic semiconductors ?
electrons.
Ans. When the temperature of the semiconductor is

Flo
A p-lype semiconductor is obtained when a raised, more covalent bonds are broken in the
trivalent atoms (B or Al) are doped in pure given semiconductor. Due to it, there is a large
semiconductor of Ge or Si. Here each doped

ee
increase in the charge carrier concentration in
trivalent atom shares its three valence electrons semiconductor. The fraction (/) of the number

Fr
with the three atoms of Ge or Si and form bonds. of electrons raised from valence band to
While the fourth bond remains unbounded. Due
conduction band at temperature TK in intrinsic
to it, a hole is created. Since the impurity atoms semiconductor is given by

for
ur
and atoms of semiconductor are electrically
f -E^/2kT
neutral, hence p-type semiconductor is also
neutral. where is the value of forbidden energy gap.
ks
Yo
If means, if T increases,/also increases, i.e.,
oo

III. Conductivity of semiconductors with the increase in temperature, the number of


electrons in conduction band increases. This
eB

10. Carbon and silicon are known to have similar leads to increase in conductivity of intrinsic
lattice structures. However, the four bonding semiconductor. As re.sistivity is the reciprocal
electrons of carbon are present in second of electrical conductivity, hence resistivity of
r
ou

orbit while those of silicon are present in its


ad

pure semiconductor decreases with increase in


third orbit. How does this difference result temperature.
Y

in a difference in their electrical


13. The diagram Fig. 14(Q).28 shows a piece of
conductivities ? (CBSE Sample Paper 2008) pure semiconductor S in series with a
nd
Re

Ans. We know that in an atom binding energy of variable resistor R, and a source of constant
electron is less in higher orbit and more in lower voltage V. Would you increase or decrease the
Fi

orbit. Therefore, the energy required to remove value of R to keep the reading of ammeter
an electron from silicon atom is much smaller (A) constant, when semi-conductor S is
than that in case of carbon atom. That is why, heated ? Give reason.
the number of free electrons for conduction in
FIGURE 14(Q).28
silicon is quite significant but very small for
carbon. As a result of it, the conductivity of
silicon is much higher than that of carbon. V S

11. (a) Is it difficult to make an intrinsic


semiconductor ?
(b) Name two factors on which the electrical
conductivity of intrinsic semiconductor at a
given temperature depends,
(c) How does the conductivity of a
semiconductor change with the rise in its
temperature ? (CBSE Sample Paper 2022-23)
SEMICONDUCTOR ELECTRONICS MATERIALS, DEVICES AND SIMPLE CIRCUITS 14/95

Ans. When the pure semiconductor S is healed, its Ans. Depletion region is a region created around the
resistance decreases. Due to it, the current in p~n juction which is devoid of free charge
the circuit increases. To keep the reading of carriers and has immobile ions.
ammeter (A) constant, the value of R has to be Regarding the formation of depletion region,
increased. refer to Art. 14.15.

17. Why does the width of depletion layer of a


IV. p-n Junctions p-n junction increase in reverse biasing ?
14. The graph of potential barrier versus width Ans. When p-n junction is reversed biased, the
of depletion region for an unbiased diode is positive terminal of the external battery is
shown in graph (A). In comparison to (A), connected to n-side of p-n junction and its
graph (B) and (C) arc obtained after biasing negative terminal to p-side of p-n junction. The
the diode in different ways. Identify the type positive terminal of the external battery attracts
of biasing in (B) and (C) and justify your the majority carrier electrons from the n-region

ww
answer.
(CBSK Sample Ihiptr 2(12 and its negative terminal attracts the majority
carrier holes from p-region. Due to it, the
FIGURE 14(Q).29 majority charge carriers move away from the

Flo
fV{x) AV(.r) fV(.r) junction. This increases the width of the
depletion layer.

ee
18. Can we take one slab of p-type semiconductor

rere
/ and physically join it to another n-type

r FF
L X semiconductor to get p-n junction ?
(A) (B) (C) (NCEPT Solved Example)
uurr
Ans. (B) Reverse biased. When p-n junction diode foor
Ans. No, Any slab of p-type or n-type semiconductor,
howsoever flat its surface may be, will have
roughness on its surface much larger than the
ks s
is reversed biased, then height of the potential
interatomic crystal spacing (== 2 to 3 A). When
Yoo
barrier increases as the external battery supports
ooook

the potential barrier. such slabs of p-type and n-type semiconductors


are brought physically incontact, a continuous
(C) is forward biased. When p-n junction is
eBB

contact at the atomic level will not be possible


forward biased, then height of potential barrier
at the junction. Due to it, the junction will
decreases as the battery opposes the potential
behave as a discontinuity for the flowing charge
uurr

barrier,
carrier.
ad

15. Briefly explain how a potential barrier is set


Yo

19. For the circuit shown in Fig. 14(Q).30(a),


up across a p-n junction as a result of
find the current flowing through the 1 12
diffusion and drift of the charge carriers.
dY

resistor. Assume that the two diode are idea)


(CBSE 2022, 2020)
Re

diodes. [CBSE 2013 (C)l


innd

Ans. When a p-type semiconductor is brought into a


close contact with n-type semiconductor, then
FFi

Dl 2Q FIGURE 14(Q).30
p-n junction is formed. Now, diffusion of
D> ■V'VV
majority charge carriers {i.e., electrons from n-
side to p-side and holes from p-side to n-side) □2
2Q 2Cl
takes place across the junction. Also there will
<
be drifting of minority charge carriers {i.e.,
electrons from p-side to n-side and holes from 6 V 6 V
1 .Q 1 n ,
n-side to p-side) lakes place across the junction. 1 A:' AAAi—
As a result of it, there will be accumulation of
negative charges in the p-region and positive o o
charges in the n-region, resulting a potential
difference across the junction, which is called Ans. Here, diode D^ is forward biased, it offers zero
potential barrier. resistance. Diode Do is reverse biased, if offers
16. What is meant by depletion region in a infinite resistance.
junction diode ? How is this region formed ? The equivalent circuit of the given circuit is
(CBSE 2014, 20111 shown in Fig. 14(Q).30(/?)
14/96 ‘P^euiee^ 4 Fundamental Physics (XII)
proportional to the light Hux or light intensity.
Current through I Q resistor, I = ~—j- = 2 A But it is not so when photodiode is forward
biased.
20. The current in the forward bias is known to
be more (~ mA) than the current in the As the reverse saturation current through a
reverse bias (~ p A). What is the reason then photodiode increases with the increase in light
to operate the photodiodes in reverse bias ? intensity or light flux, so I^> l^> l2> /j.
(NCERT Solved Example) 22. (a) Draw the circuit diagram of an iltumi-
Ans. In case of n-type semiconductor, let n be the nated photodiode and its I-V characteristics.
majority carrier (i.e., electrons) density and p (b) How can a photodiode be used to measure
be the minority carrier (holes) density. Where the light intensity. (CBSE 2022)
n > p. On illumination of semiconductor, there Ans. (r<) For circuit diagram of an illuminated
will be production of equal number of electrons photodiode and its I-V characteristics refer to
and holes. Let A n. A p be the increase in Fig. 14.24 (b) and (c) respectively.

ww
majority carrier density and minority carrier (h) From the I-V characteristics, we note that
density due to illumination of semiconductor, value of reverse saturation current increases with
where A n = A p. Hence, fractional change in the increa.se in intensity of incident light, it is

Flo
majority carrier (= A nJn), fractional change in also found that the reverse saturation current
minority carrier (= A p/p). Since n > > p, so through the photodiode varies almost linearly

e
An

reree
with the intensity of incident light. Thus
n
P knowing the reverse saturation current, we can

r FF
it means, due to photo-effects the fractional determine the intensity of the incident light.
change due to minority carriers dominates. As 23. Draw V-7 characteristics of a p-n junction
uurr
a result of it, the fractional change in the reverse
bias current is more easily measurable than the
fractional change in the forward bias current. It
foor
diode. Why does the reverse current show a
sudden increase at the critical voltage ?
ks s
Name any semiconductor device which
is due to this reason, photodiodes are preferably
Yoo
operates under the reverse bias in the
ooook

used in the reverse bias condition for measuring breakdown region. (CBSE 2013)
light intensity. Ans. For V-/characteristics of ap-w junction, see Fig.
eBB

21. Why is a photodide operated in reverse bias HA6ib) and Fig. I4.n{b).
mode ? Fig. 14(Q).31 shows reverse bias When p-n junction is operated under a reverse
current, under different illuminating
uurr

bias condition a very small reverse current


intensities /j I2, It, and for a given
ad

(about few pA) flows due to drifting of minority


photodiode. Arrange the intensities Ip A, /j
Yo

charge carriers, which is independent of the


and 74 in decreasing order of wavelength, applied potential upto a critical voltage.
dY

(('BSh 2020. ●npk Pupi-r 2011)


When the reverse voltage across the p-n junction
Re
innd

FIGURE 14(Q).31
reaches a critical voltage, the reverse cunent
suddenly rises to a large value. It is due to
FFi

I(mA)
sudden increase in the number of minority
charge carriers, resulting breakdown of the
diode. If the p-n junction is lightly doped, the
avalanche breakdown occurs due to ionisation
by collision. If the p-n junction is heavily doped,
I the zener breakdown occurs at even low voltages
h- due to field emission.
I3 A semiconductor device which operates under
I4 the reverse bias in the breakdown region is the
zener diode.

24. Consider the half wave rectifier circuit Fig.


Ans. The photodiodes are used in reverse bias I4(Q).32. Assume the diode to be a silicon
condition because the change in reverse current diode with a threshold voltage of 0*7 V. Draw
through the photodiode due to change in light
the output if the input is a sine wave with an
flux or light intensity can be measured easily, amplitude of 2 V.
as the reverse saturation current is directly
SEMICONDUCTOR ELECTRONICS MATERIALS, DEVICES AND SIMPLE CIRCUITS 14/97

will be as shown in Fig. 14(Q).34 («) and (h).


FIGURE 14(Q).32

■ yj

0 u A
^■7
r
Vo=2V
0 Out put

Ans. Since the diode threshold voltage is 0-7 V, the


diode will not conduct till the input voltage
reaches from 0 to 0-7 V. It will conduct from
0-7 V to 2 V input voltage and again till the
input voltage falls down to 0-7 V. Thus, the input
and output wave form for the half wave rectifier

ww
will be as shown in the Fig. 14(Q).33(i) and (ii)
respectively.

Floo
27. The potential barrier of silicon at 30‘'C is

ee
0-7 eV. What is its value at 130"C ?

eer
Ans. The height of potential barrier in p-n junction
is directly proportional to temperature in kelvin,

FFr
i.e., Vg cc T. So

oorr
uur r
V.
130 _ 130 + 273 403
s ff
y. 30 + 273 303
30
sk
YYoo
403 0-7x403
ooko

^130 “^30^ 303 303


= 0-93eV
eBB

28. Explain the term depletion region and


potential barrier for a p- n junction.
uurr

Ans. Deplection region is a region created around


ad

25. In half wave rectifiction, if the input freq the /7-n junction which is devoid of free charge
Yo

uency is 50 Hz, what is the output frequency ? carriers and has immobile ions. It is created due
What is the output frequency of a full wave to diffusion of majority carriers across the
dY

rectifier for the same frequency ? junction when p-n junction is formed.
Re

Ans. Since the output voltage obtained in half wave Potential barrier is a potential difference or
nind

rectification is once in one cycle of input junction voltage developed across the junction
FFi

voltage, hence the output irpple frequency after due to migration of majority carriers across it
half wave rectification = 50 Hz. In a full wave when p-n junction is formed. It opposes the
rectification, we get the output voltage twice in further migration of majority carriers across the
the same direction for one cycle of input voltage. p-n junction due to this junction voltage, it
Hence the output ripple frequency after full appears as if a fictitious battery is connected
wave recitification = 100 Hz. across the p-n junction, with its positive terminal
26. Consider a silicon diode with a threshold to n-region and negative terminal to p-region
voltage of - (F7V, used as a half wave rectifier. of p-n junction. The value of potential barrier
Draw the output if the input is a sine wave is 0-3 volt for germanium and 0-7 volt for silicon
form of amplitude 2V and circuit is on in semiconductor diode.
reverse position. 29. Give any two difference between a half wave
Ans. Since the junction diode in half wave rectifier and a full wave rectifier.
rectification circuit is on in reversed position, Ans. (0 In half wave rectifier, the output voltage is
hence it will be forward biased during negative unidirection intermittent and varying voltage.
half cycle of input sine wave. Thus, the input In full wave rectifier, the output voltage is
and output waveform for this half wave rectifier unidirectional varying voltage.
14/98 ‘Pxeideejh'A Fundamental Physics (XII)CQ29D

(i’O In half wave rectifier, the frequency of output 35. {a) In Fig. 14(Q).35, is (i) the emitter, and
signal is same as that of input signal. (h) the collector, forward or reverse biased ?
In full wave rectifier, the frequency of output [CBSE 2001 (C)l
signal is double than that of input signal.
30. Name the p-n junction diode which emit FIGURE 14(Q).35
spontaneous radiation when forward biased. +1 V
How do we choose the semiconductor, to be p
used in those diodes, if the emitted radiations
c
is to be in the visible region ?
Ans. Light emitting diodes {LED) emit spontaneous B

radiation when forward biased. For visible VqO

ooww
LED's phosphor doped Ga As and GaP are the
E
suitable semiconductors.
31. Name the semiconductor device that can be o

used to regulate an irregulated dc power -2 V

.supply. With the help of l-V characteristics

e
of this device, explain Its working. {b) Under what condition does the transistor

ree
rFl
(CBSE 2011) with CE configuration act as an amplifier ?

Fre
Ans. To regulate an irregulated dc power supply, the (CBSE 2014)

rrF
semiconductor device used is zener diode as Ans. (a) The transistor used is p-n-p transistor. Here
voltage regulator. For its I-Vcharacteri.stics and emitter is at - 2 V and collector at + 1 V, both
working, refer to Art. 14.20(a).
ouur
sffoo
belong to p-type semiconductor, hence
32. A student wants to use two p-n junction (0 emitter is reverse biased and (if) collector is
diodes to convert alternating current into forward biased.
okks
direct current. Draw the labelled circuit (b) For a transistor to act as an amplifier, it must
Yo
diagram she should use and explain how it have the operating point close to the centre of
ooo
Y
works. (CBSE 2018) its active region.
BB

Ans. Two p-n junction diodes are used to convert 36. TVansistor does not work in railway carriage.
rr e

a.c. into d.c. as full wave rectifier. For circuit Why ?


diagram and working refer to Art. \A.\9{b). Ans. The railway carriage works as an electric screen.
ouu

33. Zener diode has higher dopant density as The electric field inside the carriage is zero and
YY
ad

compared to ordinary p-n junction diode. any change from outside in electric field cannot
How does it aHect (i) the width of depletion enter the carriage. Hence the electromagnetic
dd

layer and (») the junction field ? signals do not find their entry in the railway
carriage. Due to it, the transistor does not work
Re
iinn

Ans. (i) Due to higher dopant density in the Zener


diode, the width of the depletion layer decreases. in railway carriage.
F

(I'O Due to higher dopant density in the Zener 37. Why transistor can not be used as rectifier ?
diode, the junction field (£) increases, since the Ans. In order to use a transistor as a rectifier, either
width {d) of depletion layer decreases its emitter-base segment or the collector-base
[v £= segment is to be used. Since base is thin one
and lightly doped, either of the two segments
V. Transistors
does not work properly as p-n junction diode,
34.
which is required for a rectifier. That is why, a
Can we interchange emitter and collector of transistor can not be used as a rectifier,
a transistor ? Explain.
Ans.
38. In the working of a transisor, emitter-base
No, we cannot interchange emitter and collector junction is forward biased while collector-
of a transistor due to following reasons : base junction is reverse biased. Why ?
(z) In a transistor emitter is heavily doped and (CBSE 2004 (C)l
collector is comparatively lightly doped,
Ans. The emitter will send the majority carriers
(n) In a transistor, the contact area of collector-
(electrons or holes) towards base only, when it
base junction is larger than that of emitter-base is forward biased. The collector will collect the
junction.
majority carriers only when it is reversed biased.
SEMICONDUCTOR ELECTRONICS MATERIALS, DEVICES AND SIMPLE CIRCUITS 14/99

39. What is the function of base region of a 44. What do you understand by truth table and
transistor ? Why is this region made thin and Boolean expression ?
slightly doped ? (CBSE 2015)
Ans. TYuth table. It is a table that shows all possible
Ans. In a transistor, the base region brings an input combinations and the corresponding
interaction between emitter and collector. The
output combinations for a given logic gate. It is
base is kept thin and slightly doped so that, when also called a table of combinations.
emitter of transistor is forward biased, the less
number of electron-hole combination takes Boolean expression. It is an expression
place in base region and majority of charge invented by George Boole which deals with
carriers going from emitter towards base, pass logical combination of inputs and outputs for a
over to collector. As a result of it, the collector given logic gate.
current becomes quite good in comparison to The Boolean expression for OR gate is

ww
base current.
A-¥ B = y, indicates that y equals A OR B. Here
40. Differentiate between three segments of a
transistor on the basis of their size and level
symbol plus (+) is referred to as OR.
of doping. (CBSE 2014) The Boolean expression for AND gate is ;

Flo
Ans. Emitter is of moderate size and heavily doped. A.B = y, indicates that y equals A AND B. Here

e
Base is very thin and lightly doped, collector is symbol dot (●) is referred to as AND.

ree
moderately doped and of larger in size.
The Boolean expression for NOT gate is, A = y,

Fr
41. Write the functions of three segments of a

rF
transistor. (CBSE (D) 2019, 2016) indicates that y equals NOT A or A negated. Here
uurr
Ans. The function of emitter is to emit the majority symbol bar over A means we change A to the

for
carriers. Function of collector is to collect the
alternative digit I. e. I =0 and 0 =1
majority caniers coming from emitter through
s
base. Base provides the proper interaction 45. Give Boolean expression and TVuth table of
kks
between the emitter and the collector. (Uttarakhand Board 2012)
Yo
NOR gate.
oooo

VI. Gates
Ans. The Boolean expression for NOR gate is
eB

A+B = y, indicates that y equals A OR B


42. Explain analogue signals and digital signals.
(MP Board 2011) negated. For truth table of NOR gate refer to
ur

Fig. 14.59(b).
ad

Ans. Analogue Signals. A continuous time varying


YYo

current or voltage singal is called analogue 46. Give Boolean expression, circuit symbol and
Truth table for NAND gate. (CBSE 2018)
signal. Fig. 14.48(a) represents a typical
sinusoidal analogue volage signal. Ans. The Boolean expression for NAND gate is
dd
Re

Digital Signal. A signal which has two levels


in

A.B = y, indicates that y equals A AND B,


of voltages (represented by 0 & 1) is called
F

digital signal. Fig. 14.48(/?) represents a typical negated. For truth table of NAND gate, refer to
degital signal in which the voltage at any time Fig. 14.5B(d). Art. 14.46(1).
can be either zero or 5 V.
47. Identify the logic gates marked X, Y in Fig.
43. What do you understand by logic gate ? Why 14(Q).36(a). Write down the output aty, when
is it so called ? State the basic types of logic A = 1,5 = 1 and A = 0, S = 0.
gates.
FIGURE 14(Q).36
Ans. Logic gate. A digital circuit or electronic circuit
which either allows a signal to pass through it A
or stops it, is called a gate. A gate which allows B y'
the signal to pass through it when some logical O
conditions are satisified is called a logic gate. It A B y'(=A ● B) y = y
means, for a logic gate there is a certain logical
1 1 0 1
relationship between input and output voltages. o
0 0 1 0
The basic logic gates are of three types : 1. OR
gate 2. AND gate and 3. NOT gate
14/100 ^ Fundamental Physics (XII) WWTI1
Ans. Here the logic gate marked as X is NAND gate The Boolean expression for this gate is
and marked as X is NOT gate.
X=A-¥B
The Boolean expression for this combination
The truth table for the given values of A and B
of gate is y = y' = A.B of the above circuit is shown below.

Therefore, the output at y, will be as given in


Fig. 14(Q).36(/;). A A B y=A
48. Identify the logic gate marked P and Q in the 0 1 0
given logic circuit Fig. 14(Q).37(a). Write 0 1 1
down the output at X for the inputs
(i) A = 0, B = 0 and (m) A = \,B =\. 50. Draw the output waveform at X, using the
given inputs A and B for the logic circuit
FIGURE 14(Q).37
shown in Fig. 14{Q).39. Also identify the logic

ww
A operation performed by this circuit.
X
B (CBSK 2011)

Flo
FIGURE 14(Q).39

A y X

e
A B y’(=A+B) X = y’B

rree
0 0 1 0 B

r FF
1 1 0 0
Ans. The Boolean expression for the circuit is
uurr
ICBSE 2003 (C)]
Ans. Here, P gate is NOR gate and Q gate is AND
gate. The Boolean expression for the output at
for
y = A+B and X =A + B = A+B
Output waveform atX, is shown in Fig. 14(Q).40.
kss
Xis FIGURE 14(Q).40
X =y'B=(A + B)-B
ooook
Yo
The Truth table is as shown in Fig. 14(Q).45(fe).
49. (i) Identify the logic gates marked P and Q in
eBB

A
the given logic circuit Fig. 14(Q).38.
FIGURE 14(Q).38
urr
ad

A
^>o B
Yo

Q
dY

B
Re

X
innd

(h) Write down the output at X for the inputs


A = 0, B = 0 and A = 1, B = 1. (CBSE 2010)
Fi

Ans. Logic gate P is NOT gate and logic gate Q is h h ^3 4 4 4 4


OR gate.
It means this logic circuit will work as OR gate.

SHORT ANSWER QUESTIONS Carrying 3 marks

1. Draw the energy band diagrams for conductors, ductivity of a solid is determined by the presence
semiconductors and insulators. Which band of majority charge carriers in the given solid or
determines the electrical conductivity of a the valence band and conduction band are how
much closest to the fermi level of the solid.
solid ? How is the electrical conductivity of a
semiconductor affected with rise in its Electrical conductivity of semiconductors
temperature ? Explain. (CBSE 2022) increases with increase in temperature as more
Ans. For energy band diagrams of conductors, electrons can jump from valence band to
conduction band.
semiconductors and insulator, refer to Figs. 14.3,
2. What are the Instrinsic semiconductors ?
14.5 and 14.4 respectively. The electrical con-
Explain how do they work ? [Art. 14.6]
SEMICONDUCTOR ELECTRONICS MATERIALS, DEVICES AND SIMPLE CIRCUITS 14/101

3. What do you understand by term ‘holes’ in a 16. (a) Explain with the help of suitable diagram.
semiconductor ? Discuss how they move under the two processes which occur during the
the influence of an electric field ? [Art. 14.7] formations of p-n junction diode. Hence define
4. Discuss electrical conduction in
the terms (/) depletion region and {ii) potential
semiconductors. [Art. 14.8] barrier. [CBSE 2017 (C)] [Art. 14.16 (i)]
5. What is doping ? State the methods of doping. (b) Draw a circuit diagram of a p-n junction
[Art. 14.9] diode under forward bias and explain its
6. How is /i-type semiconductor fonned ? Name working. [CBSE 2017 (C)] [Art. 14.16 (/)]
the majority charge carriers in it. Draw the 17. Draw a circuit diagram of a full-wave rectifier.
energy band diagram of an /j-type Explain its working principle. Draw the input/
semiconductor. [Art. 14.10 (i)] output wave forms indicating clearly the
7. Draw the energy band diagram of a p-type functions of the two diodes used.

semiconductor. Deduce an expression for the (CBSE 2019, 2017, 2008 ; MP Board 2011, 2012 ;

ww
conductivity of a p-type semiconductor. HP Board 2011, 2012 ; Karnataka 2012 ;
[Arts. 14.10 (ii) and 14.13] Chliatisgurli Board 2011, 2012)
8. Draw energy band diagram ofp and n type semi [Art. 14.19(b)]

Flo
conductors. Also write two differences between 18. Suggest an idea to convert a full wave bridge
p and n type semiconductors. rectifier to a half wave reclificr by changing the

ee
(CBSE Sample Paper 2020) [Art. 14.10] connecting wire/s. Draw the diagram and ex

rere
9. Discuss the effect of temperature on the mobility plain your answer. (CBSE Sample Paper 2020)

rFF
and conductivity of electrons and holes. Ans. Circuit diagram for full wave rectifier is shown
[Art. 14.14] in Fig. 14.19. The full wave rectifier can be con
uurr
verted into half wave rectifier if we change the
10. What do you understand by electrical
conductivity of a semiconductor ? Find a foor
connection of resistance R from point C to point
B. Now no currrent flows through diode D-,.
ks s
relation for the same. [Art. 14.13]
19. With what consideration in view, a photodiode
Yoo
11. What is p-n junction ? How is a p-n junction
oook

is fabricated ? Stale its working with a help of a


made ? How potential barrier is caused in it.
suitable diagram.
eBB

[Art. 14.15]
Even though the current in the forward bias is
12. (a) Explain the formation of depletion layer and known to be more than in the reverse bias, yet
potential barrier in a p-n junction. the photodiode works in reverse bias. What is
uurr

(CBSE 2016, 2010) [Art. 14.15] the reason ? (CBSE (D), 2015) [Art. 14.20(b)]
ad

(b) Draw the circuit diagram of a half wave


Yo

20. (a) In the following diagram, which bulb out of


rectifier and explain its working. and will glow and why ?
dY

(CBSE 2022, 2016) [Art. 14.17(a)]


FIGURE 14(Q).41
Re

13. (a) Explain with the help of a diagram, how


innd

D1 □2
depletion region and potential barrier are formed
FFi

in a junction diode. (CBSE 2016)


(b) If a small voltage is applied to a p-n junction
diode how will the barrier potential be affected
+ 9 V
when it is (/) forward biased and (ii) reverse B1 B2
biased ? (CBSE 2015) [Art. 14.15, 14.16]
14. Using the necessary circuit diagram, show how
the V-I characteristics of a p-n junction are ob
tained in (/) forward biasing (//) Reverse biasing.
(b) Draw a diagram of an illuminated p-n
How are these characteristics made use of in
junction solar cell
rectification ? (CBSE 2014) [Art. 14.19.3]
(c) Explain briefly the three proces.ses due to
15. Explain briefly with the help of necessary which generation of takes place in a solar
diagrams, the forward and the reverse biasing cell. (CBSE 2017)
of a p-n junction diode. Also draw their
characteristic curves in the two cases.
Ans. (a) Here, is forward biased and Do is reverse
biased, .so bulb B^ will glow.
(CBSE 2017) [Art. 14.17]
(b) Fig. 14.26(a). (c) Art. 14.2()(d).
14/102 ‘a Fundamental Physics (XII) cfviwii

21. How is a zener diode fabricated ? What cause,


the setting up of high electric field even for small electric field as f = —
V
^ = 5 X 10’ V/m.
d 10"’
reverse bias voltage across the diode ?
Describe with the help of a circuit diagram, the 28. Briefly explain how e.m.f. is generated in a solar
working of zener diode as a voltage regular. cell. Draw its l-W characteristics.

(CBSE 2»15) [Art. 14.20] (CBSE 2022) [Art. 14.20(d)]


22. A zener diode is fabricated by heavily doping 29. Write the functions of the three segments of a
both p- and n-sides of the junction. Explain transistor. (CBSE 2019)
why ? Briefly explain the use of zener diode as Ans. The three segments of a transistor are :
a d.c. voltage regular with the help of a circuit
(a) Emitter (E). It is a left hand side thick layer
diagram. BSE 2016, 2012) [Art. 14. 20(a)]
of the transistor which is heavily doped. It
23. Draw the circuit diagram of an illuminated provides the majority carriers when suitably

w
photodiode in reverse bias ? How is photodiode biased.
used to measure light intensity ? iC'BSE 2010)
{b) Base (B). It is a central thin layer of the

Flo
[Art. 14.20(b)] transistor, which is lightly doped. It provides
24. What is light emitting diode (LED) ? Mention interaction between emitter and collector,

ee
two important advantages of LEDs over con (c) Collector (C). It is a right hand side thick
ventional lamps. (CBSE 2010) [Art. 14.20(c)]

Fr
layer of the transistor which is moderately
25. Explain with the help of a circuit diagram the doped. It collects the majority carriers coming
working of a photodiode. Write briefly how it from emitter through base.

for
ur
is used to detect the optical signals. 30. Discuss the action of a (/) p-n-p junction
tCBSE 2017, 2013) [Art. 14.20(b)] transistor and (n) n-p-n junction transistor.
ks
26. Mention the important considerations required [Art. 14.21]
Yo
while fabricating ap-n junction diode to be used Or
oo

as Light Emitting Diode (LED). What should


Describe the principle, construction and
eB

be the order of the band gap of an LED if it is


working of a transistor.
required to emit light in the visible region.
(MP Board 2011) [Art. 14.22]
I CBSE 2013) [Art. 14.23(c)]
r

31. Explain working principle of transistor. Draw


ou
ad

27. Give reasons for the following :


circuit diagram for a common base n-p-n
(/) High reverse voltage do not appear across a
Y

transistor with input and output charactristic


LED.
curves. (Utturnkhand Board 2012)
nd

(//) Sunlight is not always required for the


Re

[Arts. 14.22 and 14.24]


working of a solar cell.
32. Sketch the three modes of study of n-p-n
Fi

{Hi) The electric field, of the junction of a zener junction transistor. [Art. 14.23]
diode, is very high even for a small reverse bias
33. Describe briefly with the help of a circuit
voltage of about 5 V. (CBSE 2016)
diagram, how the flow of currrent carriers in a
Ans. (/) LED is a heavily doped p-n junction, when p-n-p transistor is regulated with emitter-base
a reverse voltage is applied across LED, the junction forward biased and base-collector
minority carriers increase near the junction and junction reverse biased.
current starts flowing. Due to it, the high reverse
(CBSE 2012) [Art. 14.24]
voltage do not appear across the junction.
34. Draw the circuit diagram of the ‘set up’ used to
(//) It is so because the energy produced by soUu-
study the characteristics of a n-p-n transistor
cell can be stored in batteries for later use, when
in its common emitter configuration. Sketch the
sunlight is not available.
typical (0 Input characteristics and {ii) Output
{Hi) Since depletion width of the junction of characteristics for this transistor configuration.
zener diode is very small (< 10~’m), even a (CBSE 2016, 2017 (D). 2019)
small voltage say 5 V can set up a very high
[Art. 14.25]
SEMICONDUCTOR ELECTRONICS MATERIALS, DEVICES AND SIMPLE CIRCUITS 14/103

35. Explain that a transistor can be used as a switch. 42. Give the logic symbol, truth table and Boolean
(Hr. Board 2012) expression for OR gate. How is it realised in
practice ? (Raj. Board 20111 [Art. 14.43]
[Art. 14.26]
36. Discuss briefly the concept of an amplifier. 43. Give the logic symbol, truth table and Boolean
expression for AND gate ? How is it obtained
[Art. 14.27]
in practice ?
37. Drawing a labelled circuit diagram, explain the
(HP Board 2011, CBSK Sample Paper)
working principle of a common emitter
[Art. 14.44]
amplifier using p-n-p transistor.
[Art. 14.29(b)] (HP Board 2013) 44. How is NOT gate realised ? Explain.
[Art. 14.45]
38. Draw a circuit diagram of a transistor amplifier
in CE configuration. Define the terms (/) Input 45. What is logic gate ? Distinguish between AND

ww
resistance and (ii) Current amplification factor. and OR gates. i.lharkhund Board 2011)

How are these determined using input and output [Art. 14.40]
characteristics ? (CBSE 2018, 2015 (D))
46. What are main logic gates ? How many types

Flo
[Art. 14.25] are they ? Draw their symbols and truth tables.

ee
39. Explain the working of common base p- n- p IMP Board 2012)

transistor as an amplifier and determine its

rere
[Arts. 14.40, 14.41]

r FF
voltage gain, current gain and power gain.
47. Give the logic symbol. Boolean expression and
(Hr. Board 2011) [Art. 14.28(b)]
truth table of NAND gate. (HI Hoard 2011)
uurr
40. Give a circuit diagram of a common emitter
amplifier using an n-p-n transistor. Draw the foor [Art. 14.46(1)]
48. Discuss an integrated circuit. Explain the mean
ks s
input and output waveforms of the signal. Write ings of SSI, MSI, LSI and VLSI. [Art. 14.47]
Yoo
the expression for its voltage gain and current 49. What is integrated circuit ? Discuss the advan
oook

gain. [CBSE 2018 (C), 2014, 2009] tages of integrated circuits over conventional
electronic circuit. ( Tamil N; :lu Board 2012)
eBB

[Art. 14.29(a)]
[Arts. 14.47 and 14.48]
41. Explain the advantages and disadvantages of
50. Discuss the various processes involved in the
semiconducting devices compared to vacuum
uurr

fabrication of integrated circuits. Give the


ad

tube. [Arts. 14.31 and 14.32]


limitations of integrated circuits over
Yo

conventional circuits. [Arts. 14.47 and 14.49]


dY
Re

LONG ANSWER QUESTIONS Carrying 5 or more marks


innd

1. Explain formation of energy bands in solids. semiconductors with the help of their energy
FFi

Distinguish between conductors, extrinsic and band diagram. [Art. 14.10]


intrinsic semiconductors and insulators on the 4. Distinguish between (/) Intrinsic and Extrinsic
basis of band theory. [Arts. 14.3 & 14.5] semiconductor and [Art. 14.11]
2. Draw the “Energy bands” diagram for a (/) pure (;7) n-type semiconductor and p- type semi
semiconductor (//) insulator. How does the conductor. [Art. 14.12]
energy band, for a pure semiconductor, get 5. Distinguish between /i-type and p-type semi-con
affected when this semiconductor is doped with ductors on the basis of energy band diagrams.
{a) an acceptor impurity (b) donor impurity ? Explain the process of conduction in both type
Hence discuss why the ‘holes* and the of materials. [Art. 14. 10 & 14.12]
‘electrons’ respectively, become the ‘majority 6. Explain the use of/>« junction diode a.s a rectifier.
charge carriers’ in these two cases. Draw the circuit diagram of a full wave rectifier.
[CBSE (D), 2016 (C)] Explain its working giving its input and output
[Refer to Arts 14.5 & 14.10] wave forms. (Hr. Board 20111 [Art 14.19(b)]
3. What is an extrinsic semiconductor ? Discuss 7. Explain (/) forward biasing (ii) reverse biasing of
the working of the various types of extrinsic a P-N junction diode. With the help of a circuit
14/104 'a Fundamental Physics (XII) kV»lWll
diagram, explain the use of this device as a half 13. Explain briefly the use of a junction transistor
wave rectifier. [Arts. 14.16 and 14.19(a)] as an oscillator. (Hr. Board 2011) [Art. 14.31]
8. What is a p-n junction diode ? Define the term 14. What is the function of emitter, base and
'dynamic re.sistance’ for the junction diode. collector in a transistor ? Explain with the help
With the help of a circuit diagram explain the of a circuit diagi m, the use of n-p- n transistor
working of a p-n junction diode as a full wave as an amplifier in common emitter
rectifer. [Arts. 14.15, 14.18, 14.19(b)] configuration. [Art. 14.21 & 14.29]
9. How is zener diode fabricated so as to make it a 15. {a) Draw the circuit arrangement for studying
special purpose diode ? Draw /-V characteristics the input and the output characteristic of an
of zener diode and explain the significance of n-p-n transistor in CE configuration. With the
breakdown voltage. help of these characteristics define (() input
Explain briefiy. with the help of a circuit diagram, resistance (ii) cunent amplification factor.
how a p-n junction diode works as a half wave (CBSE 2010) [Art. 14.25]
rectifier. (CBSE 2009) [Arts. 14.20 & 14.19(a)] (b) Describe briefly with the help of a circuit

w
10. Draw the symbolic representation of (/) p-n-p, diagram how an n-p-n transistor is used to
(ii) n - p - n transistor. Why is the base region produce self .sustained oscillations.
of transistor thin and lightly doped ? With proper (CBSE 2012, 2010) [Art. 14.31]

Flo
circuit diagram, show the biasing of a p-n-p 16. What do you understand by decimal number
transistor in common base configuration. system and binary number system. Discuss

reeee
Explain the movement of charge carriers conversion of decimal number system into
through different parts of the transistor in such binary number system with illustrations.

FFr
a configuraton and show that /^, = [Art. 14.34, 14.35, 14.36]
(CBSE 2007) [Arts. 14.21 & 14.22]
17. What do you understand by logic gate ? Why is

for
11. Discuss common base amplifier using n-p-n it so called ? Discuss the various types of basic
ur
transistor, calculate its current gain, voltage gain gates. [Art 14.40 & 14.41]
and power gain. [Art. 14.28]
18. Describe (/) NAND gate, (//) NOR gate and (Hi)
kkss
12. Discuss common emitter amplifer, using n-p-n XOR gate. [Art 14.46]
Yo
transistor. Find its current gain, voltage gain and
oo

power gain. (Hr. Board 2012, CBSE 2009)


eB

[Art. 14.29]
r

CASE-BASED VERY SHORT/SHORT QUESTIONS


ou
ad

CASE 1. When a p-n junction is formed, a thin


YY

As a result of it, the conduction of majority charge


layer is created at the junction of /?-type and n-type carriers takes place across the junction. The current so
semiconductor which is devoid of free charge carriers
ndd

set up is forward current which increases rapidly in


Re

but has immobile ions. It is called depletion layer. A forward biasing.


potential barrier is developed at the junction which
Fi

2. Reverse biasing of p-n junction. If n-side of


prevents the diffusion of majority charge carriers across /7-n junction is connected to positive terminal of external
the junction. When no external voltage is applied across battery and /;-side of p-n junction to negative terminal
the p-n junction, the resistance of depletion layer of external battery, then the p-n junction is said to be
becomes infinite. In order to have conduction across
reverse biased.
the junction a biasing of p-n junction is to be done which
is in two ways. In reverse biasing, the width of depletion layer
increases. The value of potential barrier also increases.
1. Forward biasing o(p-n junction. If /j-side of
As a result of it, no conduction takes place across the
/j-n junction is connected to positive terminal of external
junction due to majority charge carriers. However if
battery and n-sidc oi p-n junction is connected to
the voltage of external battery used for reverse biasing
negative terminal of external battery, then the p-n is high, then conduction across the p-n junction takes
junction is said to be forward biased.
place due to minority carriers. The current so set up is
In forward biasing, the width of depletion layer called reverse current, which is quite weak.
decreases. The value of potential barrier also decreases.
SEMICONDUCTOR ELECTRONICS MATERIALS, DEVICES AND SIMPLE CIRCUITS 14/105

Read the above paragraph carefully and emit the majority carriers. Function of collector is to
answer the following very short and short collect the majority carriers. Base provides the proper
answer questions : interaction between emitter and the collector.

1. Can we measure the potential difference of a p-n A junction transistor is so named because it is a
junction by putting a sensitive voltmeter across device which can transfer the resistance by
its junction, when p-n junction is not biased ? interchanging the biasing across the junction.
2. What is the direction of diffusion current in a p-n Read the above paragraph carefully and
junction ? answer the following very short and short
3. Why does the width of depletion layer of a p-n answer questions :
junction increase in reverse biasing ? 5. Can two p-n junction diodes placed back to back
work as p-n-p transistor ?
4. What are the readings of Ammeters Aj and
6. In the given Fig. 14(Q).43, the emitter E and the

ww
shown in Fig. I4(Q).42 ? Neglect the resistance
of ammeter, when p-n junction u.sed is ideal one. collector C are forward or reverse biased. Explain.

FIGURE 14(Q).42 FIGURE 14(Q).43


+1 V

Flo
5Q

e
C

ree
OV B

Fr
2V

rF
uurr
E

for
5Q
s -2 V
kks
CASE 2. A junction transistor is obtained by
Yo
oooo

growing a thin layer of one type semiconductor in 7. What will happen if emitter be reverse biased and
collector be forward biased in a transistor ?
between two thick layers of other similar type
eB

semiconductor. Thus a junction transistor is a 8. In one of the circuits given below the lamp lights.
semiconductor device having two junctions and three Fig. 14(Q).44. Which circuit is it ? Explain.
ur

terminals. There are two types of junction transistors.


ad

FIGURE 14(Q).44
(0 P-/I-/J junction transistor and (//) n-/?-n junction
YYo

transistor. L L

A /7-n-p junction transistor is obtained by growing


dd

R
a thin layer of n-type semiconductor in between two s
Re

<. 6V 6V
in

relatively thick layers of p-typc semiconductor. A


n-p-n junction transistor is obtained by growing a thin
F

T T

layer of p-type .semiconductor in between two relatively


thick layers of n-type semiconductor.
The three layers of transistor are
(/) Emitter (E). It is the left hand side thick layer (a) (b)
of transistor of moderate size and heavily doped.
(h) Base (B). It is a central thin layer of transistor, CASE 3. LED is a heavily doped P-N junction
which is lightly doped. which under forward bias emits spontaneous radiation.
(Hi) Collector (C). It is the right hand side thick When it is forward biased, due to recombination of holes
layer of transistor of large size than that of emitter and and electrons at the junction, energy is released in the
is moderately doped. form of photons. In the case of Si and Ge diode, the
During the working of a transistor, the emitter- energy released in recombination lies in the infrared
base junction is forward biased and collector-base region. LEDs that can emit red, yellow, orange, green
junction is reverse biased. The function of emitter is to and blue light are commercially available. The
14/106 Fundamental Physics (XH)EEHD
semiconductor used for fabrication of visible LEDs must 11. A student connects the blue coloured LED as

atleast have a band gap of 1-8 eV. The compound shown in the Fig. 14(Q).45. The LED did not
semiconductor Gallium Arsenide - Phosphide is used glow when switch 5 is closed. Explain why ?
for making LEDs of different colours.
FIGURE 14(Q).45
(CBSE Sample Paper 2022-23) 1.5 V
S
Read the above paragraph carefully and
< >
answer the following very short and short
answer questions :
R
9. Why are LEDs made of compound semicon
ductor and not of elemental semiconductor ? LED

10. What should be the order of band gap of an LED,


if it is required to emit light in the visible 12. State the important considerations required

w
region ? while fabricating a p-n junction diode to be used
as a light emitting diode (LED).

Flo
ANSWERS

e
1. No, because the voltmeter to be used to measure to p-type semiconductor, hence emitter is reverse

rree
the potential difference across the p-n junction biased and collector is forward biased.

r FF
must have a very high resistance, which is nearly 7. In a transistor if emitter is reverse biased and
infinite, if not biased. Apart from it, there are no collector is forward biased, then emitter will work
uurr
for
free charge carriers in the depletion region of p-n as collector and collector will work as emitter. It
junction. means the transistor will work but with lower
2. In junction diode, the direction of diffusion amplification as in actual transistor the size of
kss
current is from p-region to n-region of p-n emitter is smaller than that of collector but doping
ooook
Yo
junction. of emitter is more than that of collector.
3. When a p-n junction is reverse biased, the positive
8. A lamp will glow if the transistor works properly.
eB

terminal of the external battery is connected to It will be so if emitter-base circuit is forward


/i-side of p-n junction and its negative terminal biased and collector-emitter circuit is reverse
to p-side of p-n junction. The positive terminal
rr

biased. It is so in circuit (a). So the bulb of circuit


ou

of external battery attracts the majority carrier


ad

(a) will glow.


electrons from n-region and the negative terminal
YY

of battery attracts the majority carrier holes from 9. It is so because the band gap in the elemental
/7-region. Due to it, the majority charge carriers conductor has a value that can detect the light of
nndd
Re

move away from the junction, thereby increasing wavelength which lies in the infrared region.
the width of the depletion layer. 10. l-8eVto3eV.
Fi

4. Here p-n junction is reverse biased with battery


11. In the given circuit, LED is reverse biased that is
of emf 2 volt. Hence resistance of p-n junction why it is not glowing.
becomes infinite. No current flows through
ammeter A,. So, the reading of ammeter Aj is 12. The following important considerations are
zero. required while fabricating a p-n junction to be
used as a light emitting diode (LED).
2
1. The reverse break down voltage of LEDs are
The current through ammeter A2 = j = 0*4 A generally low, around 5 V. That is why a care must
5. No, by joining two p-n junction diodes back to be taken while fabricating a p-n junction diode
back to form a transistor, we get a thick central so that the high reverse voltages do not appear
across LEDs.
region of n-type semiconductor, which will not
serve the purpose of base in a transistor. 2. There is a very little resistance to limit the
6. The transistor used is p-n-p transistor. Here emitter current in LEDs. Therefore a resistor must be used

is at - 2 V and collector is at + 1 V, both belong in series with LED to avoid any damage to it.
SEMICONDUCTOR ELECTRONICS MATERIALS, DEVICES AND SIMPLE CIRCUITS 14/107

CASE-BASED MCQs AND ASSERTION-REASON QUESTIONS

CASE 1. In the energy band diagram of a Choose the correct option out of the four options
semiconductor, there is a forbidden energy gap (of about given below :
1 eV) between valence band and conduction band. Its (a) Both A and R are true and R is correct
value is 0-75 eV for Ge and 1-12 eV for Si. In semi
explanation of A.
conductor, the conduction is due to free electrons and
(b) Both A and R are true and R is not a correct
holes. At absolute temperature OK, the conduction band
explanation of A.
of a semiconductor is completely empty and the
semiconductor behaves as an ideal insulator. When
(c) A is true, but R is false.

temperature of a semiconductor is raised, some of the (d) A is false, and R is also false.
covalent bonds are broken. Due to it, the electrons so 3. Assertion, n-type semiconductor is negatively

ww
created in the valence band move into conduction band charged.
by gaining thermal energy. As a result of it, the electrons Reason, n-type semiconductor has negatively
deficiencies are created in the valence band which are charged carriers only.

Flo
called holes.
4. Assertion. The conductivity of a semiconductor
In a pure semiconductor, the mobility of electrons increases with rise in temperature.

ee
is more than that of holes. In a pure semiconductor, the Reason. With the rise in temperature of a semi

rere
number density of free electrons (rig) in conduction band conductor, more number of electrons go from

rFF
is equal to the number density of free holes (n^j) in the valence band to conduction band.
valence band, i.e.,
uurr
The conductivity of a pure semiconductor is very
low. Its conductivity can be increased by doping certain
foor
C.ASE 2. When a /?-type semiconductor is brought
in close contact with n-type semiconductor, we get p-n
junction or junction diode. When a p-n junction is
ks s
impurity elements. A doped semiconductor is called
Yoo
formed, then due to difference in concentration of
oook

extrinsic semiconductor, which are of two types ; majority current carriers on two sides of junction,
(0 rt-type semiconductor and (ii) p-iygt semiconductor. diffusion of majority cuirent carriers takes place from
eBB

In M-type semiconductor, electrons are majority one side of p-n junction to another side. The process
carriers and holes are minority carriers. In p-type continues till a thin layer is created at the junction called
semiconductor, holes are majority carriers and electrons
uurr

depletion layer, which is devoid of any current carrier


ad

are minority carriers. Both n-type and /?-type but has immobile ions. Then the diffusion process stops.
Yo

semiconductorsare electrically neutral. The thickness of depletion layer is of the order of


Based on the above paragraph, answer lO”^ cm. When the depletion layer is sufficiently built
dY

questions no. 1 to 4 : up, it prevents the diffusion of majority current carriers


Re
innd

across the junction, creating a barrier at the junction


1. In intrinsic semiconductor, at room temperature,
number of free electrons and holes are called potential barrier (V^). The value of potential
FFi

barrier is 0-3 V for Ge, p-n junction and 0-7 V for Si,
(a) equal (b) zero
p-n junction. A p-n junction can be forward biased and
(c) unequal (d) infinite reverse biased.

2. The mobility of electrons is greater than that of Based on the above paragraph, answer
holes in an intrinsic semiconductor. This is due to questions no. 5 to 8 :
(a) their negative charge 5. In a p-n junction semiconductor, the barrier
(b) more mass potential opposes diffusion of
(c) more value of energy as they are in conduction (a) free electrons from n-region only
band (b) holes from ./^-region only
(d) all the above (c) majority carriers from both the regions
For Question No. 3 and 4, we have given two (d) minority carriers from both the regions
.statements each, one labelled as Assertion (A) and 6. The potential barrier in the depletion layer of p-n
other labelled as Reason (R). junction is due to
14/108 ‘P'uutce^'4- Fundamental Physics (XII)

(a) ions (b) electrons 7. Assertion. In p-n junction, the barrier potential
(c) holes (</) forbidden band opposes the diffusion of majority current carriers
For Question No. 7 and 8, we have given two
from both the regions.
statements each, one labelled as Assertion (A) and Reason. The value of potential barrier for silicon
other labelled as Reason (R). p-n junction is 0-3 V.
Choose the correct option out of the four options 8. Assertion. In forward biasing of p-n junction, the
given below : direction of current in the junction is from n-side
(a) Both A and R are true and R is the correct
to p-side of junction.
explanation of A. Reason. Current in the junction diode is due to
(b) Both A and R are true and R is not the correct
flow of majority current carriers electrons from
n-side to p-side of p-n junction.
explanation of A.
(c) A is true, but R is false.

ww
(d) A is false, and R is also false.

Floo
ANSWERS

l.(fl) 2.(c) X{d) 4. (a) 5.(c) 6. (a) 7. (c) 8. (J)

ree
rFee
HINTS/EXPLANATiONS For Difficult Questions

F
oor r
1. In intrinsic semiconductor, at room temperature, 6. The potential barrier is caused in depletion layer
rur
there will be equal number of electrons and holes. due to presence of immobile ions on the faces
s ff
2. Mobility of electrons is greater than that of holes of depletion layer and absence of any mobile
in an intrinsic semiconductor because the free current carrier.
osk
YYoou
electrons have more energy in conduction band 7. Here Assertion is true but Reason is false
oook

than the holes in valence band. because the value of potential barrier for silicon
eBB

3. Both Assertion and Reason are false as n-type p-n junction is 0-7 V.
semiconductor is electrically neutral and in it, 8. Both Assertion and Reason are false as the

the electrons are majority carriers and holes are direction of current in the junction diode when
uur r

minority carriers. forward biased, is the direction of flow of


ad

positive charge, i.e., due to diffusion of majorily


Yo

4. Both Assertion and Reason are true and


Reason is the correct explanation of Assertion. carriers holes, from p-side to «-side of p-n
5. In a p-n junction, the barrier potential opposes junction.
dY
Re

the diffusion of majority current carriers from


idn

both the regions.


FFin

TYPE I. INTRINSIC AND


2. In a pure semiconductor, the number of
EXTRINSIC OF SEMICONDUCTOR conduction electrons is 6 x 10 per cubic metre.
Find the total number of current carriers
1. A semiconductor has equal electron and hole
(electrons and holes) in a same semiconductor
concentration of 6 x lO^/m^. On doping with
certain impurity, electron concentration increases
of size 1 cm X 1 cm X 1 mm. [Ans. 12 x 10*^]
to 9 X 10’3/m3. (/) Identify the new semi 3. The electrical conductivity of a semiconductor
conductor obtained after doping, (ii) Calculate increases when electromagnetic radiation of
the new hole concentration, (in) How does the wavelength shorter than 2480 nm is incident on
energy gap vary with doping ? (CBSE 2005) it. Find the band gap of the semiconductor. Given
h = 6-63 X 10-3** Js 1 ev= 1-6 x IQ-’^J.
[Ans. (0 71-type (ii) 4 x 10**/m3
(iii) Energy gap decreases with doping] (HT) [Ans. 0-5 eV]
SEMICONDUCTOR ELECTRONICS MATERIALS, DEVICES AND SIMPLE CIRCUITS 14/109

4. A doped semiconductor has impurity levels


20 meV below the conduction band. Is the
material n-type or p-type ? What is the
wavelength of light so that the electron of
impurity level is just able to jump into conduction
band ?

Use h = 6-63 X lO-"''* Js, c = 3 x 10*^ m/s.


[Ans. /I'type, 6-22 x 10“^ m]
5. Determine the number density of donor atoms
which have to be added to an intrinsic
germanium semiconductor to produce an n-

w
type semiconductor of conductivity 5 cm"
1
, given that the mobility of electron in o-type
germanium is 3900 cm^ V"' s“'. Neglect the

Flo
contribution of holes to conductivity.

e
[Ans. 8-012 X lO'^ cm"^] ▼Ir(tiA)

rree
6. On doping germanium with donor atoms of 10. A p-n junction is fabricated from a semiconductor

r FF
density 10^^ cm"^, find its conductivity if with band gap of 3-0 eV. Can it detect a wave
mobility of electrons is 3800 cm^/V-s and intrin length of (/) 600 nm (//) 400 nm ?
uurr
sic carrier concentration is 2-5 x 10*^ cni“^. Also
find the ratio of conductivity of doped germanium for
Given, h = 6-6 x lO"^*^ Js. [Ans. {/) no («) yes]
11, A potential barrier of 0-7 V exists across a p-n
kss
and pure germanium. junction (a) If the depletion layer is 7-0 x 10
-7
ooook

[Ans. 60-8 S cm"* ; 4 x 10^ : 1] m thick, what is the intensity of the electric filed
Yo
in this region ? (h) If an electron is approaching
7. The energy of a photon of sodium light the p-n junction from the n-side with a speed
eB

wavelength 5890 A equals the energy gap of a 6 0 X 10^ ms"', determine the speed with which
semiconducting material. Find (a) the minimum it enters the p-side of p-n junction.
urr

energy E required to create a hole-electron combi [Ans. (a) 1-0 x 10^ V/m (b) 3-4 x 10^ ms"*]
ad

nation. (b) the value of E/kT at a temperature of


Yo

12. A sinusoidal voltage of rms value 200 volt is


ITC, where k = 8-62 x 10"^ eV/K, h = 6-63 x connected to a diode and a resistor R in the circuit
dY

10"34 Js. [Ans. 2-1 eV, 81-2] as shown in Fig. 14(Q).47, so that half wave
rectification occurs. If the diode is ideal, what is
Re
innd

8. Calculate the number of states per cubic metre


the rms voltage across /? ? If resistor R is replaced
of sodium in 3s band. The density of sodium is
Fi

by capacitor of capacitance C, find the potential


1013 kg/m^. How many of them are empty ? difference across C.
[Ans. 5-3 X 10^ ; 2-65 x 10^] FIGURE 14(Q).47

TYPE II. p-n JUNCTIONS


R, ' Out put
9. Tlie V-/characteristics of a silicon diode is given
i
in Fig. 14(Q).46. Calculate the diode resistance in
(fl) forward bias at V = + 2 V and V = + 1 V and [Ans. lOoViv ; 200V2V]
{b) reverse bias V = - 1 V and V = - 2 V.
(CBSE (F), 2004)
13. Find the current through the resistance R in Fig.
14(Q).48 if {a) /? = 10 £2 and {b) R = 2QC1.
[Ans. (a) 20 £2,40 £2 (A) 8 x 10^ £2,8 x 10® £2]
14/110 <t Fundamental Physics (XII) ky/«j^ii
19. The current gain for common emitter amplifier
FIGURE 14(Q).48
is 69. If the emitter current is 7-0 mA, find
ion
4V (0 base current and (ii) collector current.
h
[Ans. (/) 0*1 mA (ii) 6*9 mA]
20. Output characteristics of an n-p-n transistor in
6V
R CE configuration is shown in the Fig. 14(Q).50.
1 W"--/sr Determine

in
AAAV

oww
[Ans. (a) 0*50 A (b) 0*33 A]
14. A silicon p-n junction diode whose knee voltage
is 0-7 V is connected to a battery with supply
voltage 3 V and 100 mA in forward biasing. If a
resistor R is used in series of the circuit, then a

e
FFr lo
current of 20 mA passes through the diode. Find

re
the wattage of the resistor and of the diode.

ree
[Ans. 4*6 X 10"2 W, 1*4 x 10'^ W]

F
15. The applied input a.c. to a half wave rectifier is

rF
120 watt. The d.c. output is 50 watt. What is the
rectification effeciency and power effeciency ?

fsfoor
(0 dynamic output resistance (ii) dc current gain
ouur
[Ans. 41*67% ; 83*33%]
and (Hi) ac current gain at an opererating point
16. Find the average value of d.c. voltage that can
kosk
be obtained from the half wave rectifier of Fig. = lOV, when Ig = 30pA. (CBSE 2013)

14(Q).49, assuming that the diode is ideal one. [Ans. (i) 20 kQ ; (ii) 116*67 ; (Hi) 120]
Yo
oo

21. The input resistance of a silicon transistor is


Y
FIGURE 14(Q).49
BB

10 :1
665 Q. Its base current is changed by 15 pA
t>f which results in the change in collector current
rre

o
by 2 mA. This transistor is used as a common
200 V .. o
Rl Output
emitter amplifier with a load resistance of 5 kfl.
o
oYuu

50 Hz’J o
o
j Calculate (a) current gain, ; (b) trans
ad

conductance and (c) voltage gain of the


dY

amplifier. (CHSE 2001 (C)]


[Ans. 9*0 V]
[Ans. (fl) 133*3 (b) 0*2 (c) 1000]
innd
Re

22. A change of 0-2 mA in the base current causes a


fYPIv HI. TRANSISTORS
Fi
F

change of 5 mA in the collector current for a


17. The following data was obtained for a given common emitter amplifier, (i) Find the a.c. current
transistor. gain of the transistor (ii) If the input resistance is
V 10-0 V 10-0 V
2 kfl and its voltage gain is 75. Calculate the
CE
load resistance used in the circuit.
V 0-82 V 0-72 V
BE (CBSE 2004) [Ans. (i) 25 (ii) 6 kQ]
4.. I 80pA 30 |M
B
TYPE I\. DECIMAL NUMBER
For this data, calculate the input resistance of the
SYSTEM AND BINARY SYSTEM
given transistor. [CBSE (D). 2016 (C)]
[Ans. 2 X 10^ n]
23. Find the decimal number of binary number
18. In a silicon transistor, a change of 8-0 mA in the [Ans. 61*25]
(111101-01)2
emitter current produces a change of 7-9 mA in
the collector current. What change in the base 24. Find the binary (/) addition (//) subtraction of the
current is necessary to produce an equivalent following set of numbers ; lOlOlO and 010101.
change in collector current ? [Ans. 0*1 mA]
[Ans. (/) (111111)2 (010101)2]
SEMICONDUCTOR ELECTRONICS MATERIALS, DEVICES AND SIMPLE CIRCUITS 14/111

TYPE V. LOGIC GATES 27. Find the output y of the logic gate whose symbol
is shown in Fig. 14(Q).52(a) and give a truth table
25. The truth tables shown below in (a) and (/?) for the same.
belong to which gates ?
A B Y FIGURE 14(Q).52

(a) 0 0 0 AO-
0 ] 0 BO- ■oY ©
I 0 0 Co-
I 1 1
A B C y
0 0 0 0
(b) A B Y
1 0 0 0

ww
0 0 1 0 1 0 0
0 0 1 0 o
1 0 1 1 0 0
0 1 1 0 1 1 0
1 1 1 1
1 1 0

Flo
e
Kiij. Board 2012)

eree
[Ans. Truth table is given in Fig. 14(Q).52(/»)]
[Ans. (fl) AND gate {b) NAND gate]

FFr
26. In the following circuit, if the input wave form 28. Find output yin the following circuit Fig. 14{Q).53.
is as shown in the figure, what will be the wave

oorr
uur r
form (i) across R in Fig. 14(Q).51(a) and FIGURE 14(Q).53

(li) across the diode in Fig. 14(Q).51(£»). Assume A


sf
o
A
that the diode is ideal.
II -o
sk
I
Yoo
FIGURE 14(Q).51
1>B
oook
eBB

[Ans. 0]
5V
29. Name the 2-input logic gate, whose truth table is
uurr

0- R > given here. If this logic gate is connected to a


ad

NOT gate, what will be the output when


Yo

-5V (a) o
(0 A = 1, R = 1 and (n) A = 0, R =1 ?
dY

5V A B Output y
Re
innd

Ans.
0i.--L(0 to 5V) 0 0 1
FFi

0 1 1
O 1 0 1
o W'/vVv 1 1 0
R

5V [Ans. NAND gate

0- A B Out put y output of Not gate


1 1 0 1
-5V (b) o-
0 1 1 0 ]
-5V
Ans.

30. Identify the logic gales marked X and Y in


OV J L {OV to -5V2 Fig. 14(Q).54. Writedown the output at y, when
A= l,iB= I andA = 0, F= 1.
o

f
14/112 ‘PmuUcfr’4. Fundamental Physics fXinPZSTWn

FIGURE 14(Q).54 36. In a full wave junction diode rectifier, the input
a.c. has r.m.s. value of 20 V. The transformer used
is a step up transformer having primary and
A secondary turns ratio 1 ; 2. Calculate the d.c. and
Y
X a.c. voltage in the rectified output.
B y'
[Ans. 36 V, 27-I4 V]
37. In a centre trap full wave rectifier, the load
(CBSE 2014)
resistance = I kQ. Each diode has a forward
[Ans. X is NAND gate y is OR gate ;
bias dynamic resistance 20 Q. The voltage across
A B y
half the secondary winding is llO sin lOO k t.
Find (i) the peak value of current (ii) the dc value
1 1 0 1 of current {Hi) the rms value of current.
0 1 1 1 ] [Ans. (0 108 niA («) 68-7 mA (i7i) 76-38 niA]
38. Construct a proper combination of 3 NOT and

ww
one AND gates in order to get the output of
TYPE VI. TYPICAL PROBLEMS
y =A .B .C , from three inputs A, B and C.

Floo
31. The impurity levels of a doped semiconductor
are 30 meV below the conduction band.
FIGURE 14(Q).55

ree
Determine whether the semiconductor is n-type
or p-type. At the room temperature thermal A o-
collisons occur as a result of which the extra A

rFee
electron loosely bound to the impurity ion gets Y=A.B.C

F
B
an amount of energy k T [where k is Boltzmann B o- ●o

oor r
rur
constant = 8-62 x 10“^ eV/K] and hence this
s ff
electron can jump into conduction band. What is C
Co-
the value of T ? [Ans. «-type ; 348 K]
osk
YYoou
32. Find the current produced at room temperature
oook

in a pure germanium plate of area 2 x 10“^ m^ [Ans. Fig. 14(Q).55]


and of thickness 1-2 x 10"^ m when a potential
eBB

39. Identify the gate represented by the block diagram


of 5 V is applied across the faces. Concentration of Fig. 14(Q).56. Write its boolean expression,
of carriers in germanium at room temperature is the truth table and name of the gate, it works.
uur r

1-6 X 10*^ per cubic metre. The mobility of


ad

electrons and holes are 0-4 m^ V“* s“* and FIGURE 14(Q).56
Yo

0-2 m^ V"' s“' respectively. How much heat is


generated in the plate in 100 second. A
o-

[Ans. 1-28 X lir’^A, 6-4 x 10-‘*J]


dY

■o
Re

33. A full wave rectifier uses two diodes, the internal o-


idn

B
resistance of each diode may be assumed constant
FFin

at 30 n. The transformer r.m.s. secondary voltage


from centre tap to each end of the secondary is [Ans. y = A + B ; NOR gate]
50 V and load resistance is 970 C2. Find (/) the
mean load current (//) the rms value of load Truth Table for the combined gates
current. [Ans. (0 45 mA (ii) 50 niAJ
34. A half wave rectifier is used to supply 50 V d.c. A B A+B A + B
to a resistance load of 800 12. Diode has a
resistance of 200 12. Calculate maximum a.c. 0 0 0
voltage required. [Ans. 196-43 V] 0 0
35. A transistor is connected in common emitter
0 1 0
configuration. The collector supply is 8 V and
the voltage drop across a resistor of 80012 in the 1 1 0

collector circuit is 0-5 V. If the current gain factor


(a) is 0-96, find the base current. 40. Write the truth table for the circuit shown in

[Ans. 0-026 niA] Fig. 14(Q).57. Name the gate so formed.

I
SEMICONDUCTOR ELECTRONICS MATERIALS, DEVICES AND SIMPLE CIRCUITS 14/113

FIGURE 14(Q).57
A
I
B

III

II

oww
[Ans. NOR gate. Truth table see in hint]

ee
For Difficult Questions

FFrlo
r
1. Here, «, = 6 x 10^ m ^= 9 x 10 12 .„-3
m 5. Here, <j = 5 Q * cm * ; = 3900 cm2 y * s *,

rF
ee
n
nf (6 X 10^)2 = 4xl0‘* m-3
h~ n..
9x10*2 We know, a = —
ouru - = en p

rF
e

P
As, after doping, > n/j, so the new
semiconductoris n-type. Energy gap decreases 5

ffosor
G
or n
with doping. ^ ep^ (1-6x10-*5)x3900
os k
2. Total no. of current carriers in a sample = no. of = 8012x 10 *® cm 3
electrons + no. of holes in that sample.
ook
Yo
Since one donor atom, provides one free electron
Y
In a pure semiconductor, no. of electrons = no.
to the germanium semiconductor, therefore, the
Bo

of holes = no. density electron x volume of


density of donor atoms = 8-012 x 10*® cm"®.
reeB

sample
= (6 X 10*5) X (10-2 ^ jo-2x 10-3) = 6 X io'2 6. G = n e Pg = 10*2x 1-6 x 10-'5 x 3800
oouY

Total number of current carriers = 60-8 S cm-*


ur

= 6x 10*2+ 6 X 10*2 = 12 X 10*2


ad

17
_^d 10
= 4x10® : 1
13
dY

he a.
I
n-
I
2-5x10
3. Band gap, E^ = h\ = — 7. (a) Energy required to create a hole-electron
nidn

combination is equal to energy E of photon which


Re

(6-63x10"34)x(3x10^)
= 8 02 X 10-2*3 J can push an electron from valence band to
F

2480x10-5
Fi

conduction band.

8-02x10-20 he 6-63x10-34x3x10^
« O-SeV
1-6x10"*5 Thus, £ =
A 5890x10-*0x1-6xI0-*5
4. Here, £ = 20 meV = 20 x lQ-3 x 1-6 x 10-*5 J = 2-1 eV
= 20x 1-6 X 10-22 J
(b) Here T= 2TC = 273 -t- 27 = 300 K
When electron from impurity levels, due to E 2-1
excitation, jumps to conduction band, increases = 81-2
the number density of electrons in conduction kT 8-62x10-3x300
band as compared to number density of holes in 8. Mass of one cubic metre of sodium
valence band. Thus the material is n-type. = 1 X 1013 = 1013 kg.
he _ (6-63x10"34)x(3x10^) Number of sodium atoms in the given sodium,
X
max -22
£ 20x1-6x10 6-023 x1Q26
N = X1013 =2-65 X 1028
= 6-22 x 10-® m
23

t 1
14/114 7h<ideeft^ ^ Fundamental Physics (XII) VOL.II

Total number of states in 3s band -IN


0-7
= 2 X 2-65 X 1()28 layer, £=— = - = 1*0x10® V/m
d 70x10-
= 5-30 X 102»
(b) Here, Uj = 6-0 x 10^
-1 .
ms
; Let ^2 be the
Number of states occupied in 3s band = N velocity with which electron enters the p-region.
Number of empty states in 3s band Then
=2N-N=N
= 2*65 X -x91x10-3*x(60x105)2
2
9. (a) Forward bias diode resistance is
AV = (1-6x10-^9)x0-7 + -x91x10-3‘
2
xu?2
R
f AI
where AV and A/ are the small change in the 1-64 X 10"*^ =M2xlO“‘^ + 91x10"^* ^22
2

loow w
voltage and current surrounding the desired
values. From graph On solving, V2 = 3*4 x 10^ ms~^
12. The output rms voltage in half wave rectifier
AtVy= + 2V, AV=2-2 V-1-8 V = 04V circuit across R is.

You might also like